Sei sulla pagina 1di 1216

G.R. No.

L-23145 November 29, 1968 to paraphrase Frankfurter, arising out of a specific problem, addressed to the
attainment of specific ends by the use of specific remedies, with full and
TESTATE ESTATE OF IDONAH SLADE PERKINS, deceased. RENATO D. ample support from legal doctrines of weight and significance.
TAYAG, ancillary administrator-appellee,
vs. The facts will explain why. As set forth in the brief of appellant Benguet
BENGUET CONSOLIDATED, INC., oppositor-appellant. Consolidated, Inc., Idonah Slade Perkins, who died on March 27, 1960 in
New York City, left among others, two stock certificates covering 33,002
Cirilo F. Asperillo, Jr., for ancillary administrator-appellee. shares of appellant, the certificates being in the possession of the County
Ross, Salcedo, Del Rosario, Bito and Misa for oppositor-appellant. Trust Company of New York, which as noted, is the domiciliary administrator
of the estate of the deceased. 2 Then came this portion of the appellant's
FERNANDO, J.: brief: "On August 12, 1960, Prospero Sanidad instituted ancillary
administration proceedings in the Court of First Instance of Manila; Lazaro A.
Marquez was appointed ancillary administrator, and on January 22, 1963, he
Confronted by an obstinate and adamant refusal of the domiciliary
was substituted by the appellee Renato D. Tayag. A dispute arose between
administrator, the County Trust Company of New York, United States of
the domiciary administrator in New York and the ancillary administrator in the
America, of the estate of the deceased Idonah Slade Perkins, who died in
Philippines as to which of them was entitled to the possession of the stock
New York City on March 27, 1960, to surrender to the ancillary administrator
certificates in question. On January 27, 1964, the Court of First Instance of
in the Philippines the stock certificates owned by her in a Philippine
Manila ordered the domiciliary administrator, County Trust Company, to
corporation, Benguet Consolidated, Inc., to satisfy the legitimate claims of
"produce and deposit" them with the ancillary administrator or with the Clerk
local creditors, the lower court, then presided by the Honorable Arsenio
of Court. The domiciliary administrator did not comply with the order, and on
Santos, now retired, issued on May 18, 1964, an order of this tenor: "After
February 11, 1964, the ancillary administrator petitioned the court to "issue
considering the motion of the ancillary administrator, dated February 11,
an order declaring the certificate or certificates of stocks covering the 33,002
1964, as well as the opposition filed by the Benguet Consolidated, Inc., the
shares issued in the name of Idonah Slade Perkins by Benguet
Court hereby (1) considers as lost for all purposes in connection with the
Consolidated, Inc., be declared [or] considered as lost." 3
administration and liquidation of the Philippine estate of Idonah Slade
Perkins the stock certificates covering the 33,002 shares of stock standing in
her name in the books of the Benguet Consolidated, Inc., (2) orders said It is to be noted further that appellant Benguet Consolidated, Inc. admits that
certificates cancelled, and (3) directs said corporation to issue new "it is immaterial" as far as it is concerned as to "who is entitled to the
certificates in lieu thereof, the same to be delivered by said corporation to possession of the stock certificates in question; appellant opposed the
either the incumbent ancillary administrator or to the Probate Division of this petition of the ancillary administrator because the said stock certificates are
Court."1 in existence, they are today in the possession of the domiciliary
administrator, the County Trust Company, in New York, U.S.A...." 4
From such an order, an appeal was taken to this Court not by the domiciliary
administrator, the County Trust Company of New York, but by the Philippine It is its view, therefore, that under the circumstances, the stock certificates
corporation, the Benguet Consolidated, Inc. The appeal cannot possibly cannot be declared or considered as lost. Moreover, it would allege that there
prosper. The challenged order represents a response and expresses a policy,

1
was a failure to observe certain requirements of its by-laws before new stock 1. Appellant Benguet Consolidated, Inc. did not dispute the power of the
certificates could be issued. Hence, its appeal. appellee ancillary administrator to gain control and possession of all assets
of the decedent within the jurisdiction of the Philippines. Nor could it. Such a
As was made clear at the outset of this opinion, the appeal lacks merit. The power is inherent in his duty to settle her estate and satisfy the claims of local
challenged order constitutes an emphatic affirmation of judicial authority creditors.5 As Justice Tuason speaking for this Court made clear, it is a
sought to be emasculated by the wilful conduct of the domiciliary "general rule universally recognized" that administration, whether principal or
administrator in refusing to accord obedience to a court decree. How, then, ancillary, certainly "extends to the assets of a decedent found within the state
can this order be stigmatized as illegal? or country where it was granted," the corollary being "that an administrator
appointed in one state or country has no power over property in another state
As is true of many problems confronting the judiciary, such a response was or country."6
called for by the realities of the situation. What cannot be ignored is that
conduct bordering on wilful defiance, if it had not actually reached it, cannot It is to be noted that the scope of the power of the ancillary administrator
without undue loss of judicial prestige, be condoned or tolerated. For the law was, in an earlier case, set forth by Justice Malcolm. Thus: "It is often
is not so lacking in flexibility and resourcefulness as to preclude such a necessary to have more than one administration of an estate. When a person
solution, the more so as deeper reflection would make clear its being dies intestate owning property in the country of his domicile as well as in a
buttressed by indisputable principles and supported by the strongest policy foreign country, administration is had in both countries. That which is granted
considerations. in the jurisdiction of decedent's last domicile is termed the principal
administration, while any other administration is termed the ancillary
It can truly be said then that the result arrived at upheld and vindicated the administration. The reason for the latter is because a grant of administration
honor of the judiciary no less than that of the country. Through this does not ex proprio vigore have any effect beyond the limits of the country in
challenged order, there is thus dispelled the atmosphere of contingent which it is granted. Hence, an administrator appointed in a foreign state has
frustration brought about by the persistence of the domiciliary administrator no authority in the [Philippines]. The ancillary administration is proper,
to hold on to the stock certificates after it had, as admitted, voluntarily whenever a person dies, leaving in a country other than that of his last
submitted itself to the jurisdiction of the lower court by entering its domicile, property to be administered in the nature of assets of the deceased
appearance through counsel on June 27, 1963, and filing a petition for relief liable for his individual debts or to be distributed among his heirs." 7
from a previous order of March 15, 1963.
It would follow then that the authority of the probate court to require that
Thus did the lower court, in the order now on appeal, impart vitality and ancillary administrator's right to "the stock certificates covering the 33,002
effectiveness to what was decreed. For without it, what it had been decided shares ... standing in her name in the books of [appellant] Benguet
would be set at naught and nullified. Unless such a blatant disregard by the Consolidated, Inc...." be respected is equally beyond question. For appellant
domiciliary administrator, with residence abroad, of what was previously is a Philippine corporation owing full allegiance and subject to the
ordained by a court order could be thus remedied, it would have entailed, unrestricted jurisdiction of local courts. Its shares of stock cannot therefore
insofar as this matter was concerned, not a partial but a well-nigh complete be considered in any wise as immune from lawful court orders.
paralysis of judicial authority.
Our holding in Wells Fargo Bank and Union v. Collector of Internal
Revenue8 finds application. "In the instant case, the actual situs of the shares
of stock is in the Philippines, the corporation being domiciled [here]." To the

2
force of the above undeniable proposition, not even appellant is insensible. It of truth may be lacking in such a conclusion arrived at. It is to be
does not dispute it. Nor could it successfully do so even if it were so minded. remembered however, again to borrow from Frankfurter, "that fictions which
the law may rely upon in the pursuit of legitimate ends have played an
2. In the face of such incontrovertible doctrines that argue in a rather important part in its development."11
conclusive fashion for the legality of the challenged order, how does
appellant, Benguet Consolidated, Inc. propose to carry the extremely heavy Speaking of the common law in its earlier period, Cardozo could state fictions
burden of persuasion of precisely demonstrating the contrary? It would "were devices to advance the ends of justice, [even if] clumsy and at times
assign as the basic error allegedly committed by the lower court its offensive."12 Some of them have persisted even to the present, that eminent
"considering as lost the stock certificates covering 33,002 shares of Benguet jurist, noting "the quasi contract, the adopted child, the constructive trust, all
belonging to the deceased Idonah Slade Perkins, ..." 9 More specifically, of flourishing vitality, to attest the empire of "as if" today." 13 He likewise noted
appellant would stress that the "lower court could not "consider as lost" the "a class of fictions of another order, the fiction which is a working tool of
stock certificates in question when, as a matter of fact, his Honor the trial thought, but which at times hides itself from view till reflection and analysis
Judge knew, and does know, and it is admitted by the appellee, that the said have brought it to the light."14
stock certificates are in existence and are today in the possession of the
domiciliary administrator in New York."10 What cannot be disputed, therefore, is the at times indispensable role that
fictions as such played in the law. There should be then on the part of the
There may be an element of fiction in the above view of the lower court. That appellant a further refinement in the catholicity of its condemnation of such
certainly does not suffice to call for the reversal of the appealed order. Since judicial technique. If ever an occasion did call for the employment of a legal
there is a refusal, persistently adhered to by the domiciliary administrator in fiction to put an end to the anomalous situation of a valid judicial order being
New York, to deliver the shares of stocks of appellant corporation owned by disregarded with apparent impunity, this is it. What is thus most obvious is
the decedent to the ancillary administrator in the Philippines, there was that this particular alleged error does not carry persuasion.
nothing unreasonable or arbitrary in considering them as lost and requiring
the appellant to issue new certificates in lieu thereof. Thereby, the task 3. Appellant Benguet Consolidated, Inc. would seek to bolster the above
incumbent under the law on the ancillary administrator could be discharged contention by its invoking one of the provisions of its by-laws which would set
and his responsibility fulfilled. forth the procedure to be followed in case of a lost, stolen or destroyed stock
certificate; it would stress that in the event of a contest or the pendency of an
Any other view would result in the compliance to a valid judicial order being action regarding ownership of such certificate or certificates of stock
made to depend on the uncontrolled discretion of the party or entity, in this allegedly lost, stolen or destroyed, the issuance of a new certificate or
case domiciled abroad, which thus far has shown the utmost persistence in certificates would await the "final decision by [a] court regarding the
refusing to yield obedience. Certainly, appellant would not be heard to ownership [thereof]."15
contend in all seriousness that a judicial decree could be treated as a mere
scrap of paper, the court issuing it being powerless to remedy its flagrant Such reliance is misplaced. In the first place, there is no such occasion to
disregard. apply such by-law. It is admitted that the foreign domiciliary administrator did
not appeal from the order now in question. Moreover, there is likewise the
It may be admitted of course that such alleged loss as found by the lower express admission of appellant that as far as it is concerned, "it is
court did not correspond exactly with the facts. To be more blunt, the quality immaterial ... who is entitled to the possession of the stock certificates ..."

3
Even if such were not the case, it would be a legal absurdity to impart to such person distinct and separate from its individual stockholders.... It owes its
a provision conclusiveness and finality. Assuming that a contrariety exists existence to law. It is an artificial person created by law for certain specific
between the above by-law and the command of a court decree, the latter is purposes, the extent of whose existence, powers and liberties is fixed by its
to be followed. charter."19 Dean Pound's terse summary, a juristic person, resulting from an
association of human beings granted legal personality by the state, puts the
It is understandable, as Cardozo pointed out, that the Constitution overrides matter neatly.20
a statute, to which, however, the judiciary must yield deference, when
appropriately invoked and deemed applicable. It would be most highly There is thus a rejection of Gierke's genossenchaft theory, the basic theme of
unorthodox, however, if a corporate by-law would be accorded such a high which to quote from Friedmann, "is the reality of the group as a social and
estate in the jural order that a court must not only take note of it but yield to legal entity, independent of state recognition and concession." 21 A corporation
its alleged controlling force. as known to Philippine jurisprudence is a creature without any existence until
it has received the imprimatur of the state according to law. It is logically
The fear of appellant of a contingent liability with which it could be saddled inconceivable therefore that it will have rights and privileges of a higher
unless the appealed order be set aside for its inconsistency with one of its priority than that of its creator. More than that, it cannot legitimately refuse to
by-laws does not impress us. Its obedience to a lawful court order certainly yield obedience to acts of its state organs, certainly not excluding the
constitutes a valid defense, assuming that such apprehension of a possible judiciary, whenever called upon to do so.
court action against it could possibly materialize. Thus far, nothing in the
circumstances as they have developed gives substance to such a fear. As a matter of fact, a corporation once it comes into being, following
Gossamer possibilities of a future prejudice to appellant do not suffice to American law still of persuasive authority in our jurisdiction, comes more
nullify the lawful exercise of judicial authority. often within the ken of the judiciary than the other two coordinate branches. It
institutes the appropriate court action to enforce its right. Correlatively, it is
4. What is more the view adopted by appellant Benguet Consolidated, Inc. is not immune from judicial control in those instances, where a duty under the
fraught with implications at war with the basic postulates of corporate theory. law as ascertained in an appropriate legal proceeding is cast upon it.

We start with the undeniable premise that, "a corporation is an artificial being To assert that it can choose which court order to follow and which to
created by operation of law...."16 It owes its life to the state, its birth being disregard is to confer upon it not autonomy which may be conceded but
purely dependent on its will. As Berle so aptly stated: "Classically, a license which cannot be tolerated. It is to argue that it may, when so minded,
corporation was conceived as an artificial person, owing its existence through overrule the state, the source of its very existence; it is to contend that what
creation by a sovereign power." 17As a matter of fact, the statutory language any of its governmental organs may lawfully require could be ignored at will.
employed owes much to Chief Justice Marshall, who in the Dartmouth So extravagant a claim cannot possibly merit approval.
College decision defined a corporation precisely as "an artificial being,
invisible, intangible, and existing only in contemplation of law." 18 5. One last point. In Viloria v. Administrator of Veterans Affairs, 22 it was shown
that in a guardianship proceedings then pending in a lower court, the United
The well-known authority Fletcher could summarize the matter thus: "A States Veterans Administration filed a motion for the refund of a certain sum
corporation is not in fact and in reality a person, but the law treats it as of money paid to the minor under guardianship, alleging that the lower court
though it were a person by process of fiction, or by regarding it as an artificial had previously granted its petition to consider the deceased father as not

4
entitled to guerilla benefits according to a determination arrived at by its main deplorable consequences attendant on appellant prevailing attest to the
office in the United States. The motion was denied. In seeking a necessity of negative response from us. That is what appellant will get.
reconsideration of such order, the Administrator relied on an American
federal statute making his decisions "final and conclusive on all questions of That is all then that this case presents. It is obvious why the appeal cannot
law or fact" precluding any other American official to examine the matter succeed. It is always easy to conjure extreme and even oppressive
anew, "except a judge or judges of the United States possibilities. That is not decisive. It does not settle the issue. What carries
court."23 Reconsideration was denied, and the Administrator appealed. weight and conviction is the result arrived at, the just solution obtained,
grounded in the soundest of legal doctrines and distinguished by its
In an opinion by Justice J.B.L. Reyes, we sustained the lower court. Thus: correspondence with what a sense of realism requires. For through the
"We are of the opinion that the appeal should be rejected. The provisions of appealed order, the imperative requirement of justice according to law is
the U.S. Code, invoked by the appellant, make the decisions of the U.S. satisfied and national dignity and honor maintained.
Veterans' Administrator final and conclusive when made on claims property
submitted to him for resolution; but they are not applicable to the present WHEREFORE, the appealed order of the Honorable Arsenio Santos, the
case, where the Administrator is not acting as a judge but as a litigant. There Judge of the Court of First Instance, dated May 18, 1964, is affirmed. With
is a great difference between actions against the Administrator (which must costs against oppositor-appelant Benguet Consolidated, Inc.
be filed strictly in accordance with the conditions that are imposed by the
Veterans' Act, including the exclusive review by United States courts), and Makalintal, Zaldivar and Capistrano, JJ., concur.
those actions where the Veterans' Administrator seeks a remedy from our Concepcion, C.J., Reyes, J.B.L., Dizon, Sanchez and Castro, JJ., concur in
courts and submits to their jurisdiction by filing actions therein. Our attention the result.
has not been called to any law or treaty that would make the findings of the
Veterans' Administrator, in actions where he is a party, conclusive on our
courts. That, in effect, would deprive our tribunals of judicial discretion and
render them mere subordinate instrumentalities of the Veterans'
Administrator."

It is bad enough as the Viloria decision made patent for our judiciary to
accept as final and conclusive, determinations made by foreign governmental
agencies. It is infinitely worse if through the absence of any coercive power
by our courts over juridical persons within our jurisdiction, the force and
effectivity of their orders could be made to depend on the whim or caprice of
alien entities. It is difficult to imagine of a situation more offensive to the
dignity of the bench or the honor of the country.

Yet that would be the effect, even if unintended, of the proposition to which
appellant Benguet Consolidated seems to be firmly committed as shown by
its failure to accept the validity of the order complained of; it seeks its
reversal. Certainly we must at all pains see to it that it does not succeed. The

5
ANG PUE & COMPANY, ET AL., plaintiffs-appellants,
vs.
SECRETARY OF COMMERCE AND INDUSTRY, defendant-appellee.

Felicisimo E. Escaran for plaintiffs-appellants.


Office of the Solicitor General for defendant-appellee.

DIZON, J.:

Action for declaratory relief filed in the Court of First Instance of Iloilo by Ang
Pue & Company, Ang Pue and Tan Siong against the Secretary of Commerce
and Industry to secure judgment "declaring that plaintiffs could extend for five
years the term of the partnership pursuant to the provisions of plaintiffs'
Amendment to the Article of Co-partnership."

The answer filed by the defendant alleged, in substance, that the extension
for another five years of the term of the plaintiffs' partnership would be in
violation of the provisions of Republic Act No. 1180.

It appears that on May 1, 1953, Ang Pue and Tan Siong, both Chinese
citizens, organized the partnership Ang Pue & Company for a term of five
years from May 1, 1953, extendible by their mutual consent. The purpose of
the partnership was "to maintain the business of general merchandising,
buying and selling at wholesale and retail, particularly of lumber, hardware
and other construction materials for commerce, either native or foreign." The
corresponding articles of partnership (Exhibit B) were registered in the Office
of the Securities & Exchange Commission on June 16, 1953.

On June 19, 1954 Republic Act No. 1180 was enacted to regulate the retail
business. It provided, among other things, that, after its enactment, a
partnership not wholly formed by Filipinos could continue to engage in the
retail business until the expiration of its term.

On April 15, 1958 prior to the expiration of the five-year term of the
G.R. No. L-17295 July 30, 1962
partnership Ang Pue & Company, but after the enactment of the Republic Act
1180, the partners already mentioned amended the original articles of part

6
ownership (Exhibit B) so as to extend the term of life of the partnership to Bengzon, C.J., Padilla, Labrador, Concepcion, Barrera, Paredes, Regala and
another five years. When the amended articles were presented for Makalintal, JJ., concur.
registration in the Office of the Securities & Exchange Commission on April Bautista Angelo and Reyes, J.B.L., JJ., took no part.
16, 1958, registration was refused upon the ground that the extension was in
violation of the aforesaid Act.

From the decision of the lower court dismissing the action, with costs, the
plaintiffs interposed this appeal.

The question before us is too clear to require an extended discussion. To


organize a corporation or a partnership that could claim a juridical personality
of its own and transact business as such, is not a matter of absolute right but
a privilege which may be enjoyed only under such terms as the State may
deem necessary to impose. That the State, through Congress, and in the
manner provided by law, had the right to enact Republic Act No. 1180 and to
provide therein that only Filipinos and concerns wholly owned by Filipinos
may engage in the retail business can not be seriously disputed. That this
provision was clearly intended to apply to partnership already existing at the
time of the enactment of the law is clearly showing by its provision giving
them the right to continue engaging in their retail business until the expiration
of their term or life.

To argue that because the original articles of partnership provided that the
partners could extend the term of the partnership, the provisions of Republic
Act 1180 cannot be adversely affect appellants herein, is to erroneously
assume that the aforesaid provision constitute a property right of which the
partners can not be deprived without due process or without their consent.
The agreement contain therein must be deemed subject to the law existing at
the time when the partners came to agree regarding the extension. In the
present case, as already stated, when the partners amended the articles of
partnership, the provisions of Republic Act 1180 were already in force, and
there can be not the slightest doubt that the right claimed by appellants to
extend the original term of their partnership to another five years would be in
violation of the clear intent and purpose of the law aforesaid.
[G.R. Nos. 84132-33 : December 10, 1990.]
WHEREFORE, the judgment appealed from is affirmed, with costs.

7
192 SCRA 257 expressed purpose of salvaging this and the other Agrix companies that the
aforementioned decree was issued by President Marcos.
NATIONAL DEVELOPMENT COMPANY AND NEW AGRIX, INC.,
Petitioners, vs. PHILIPPINE VETERANS BANK, THE EX-OFFICIO Pursuant thereto, the private respondent filed a claim with the AGRIX Claims
SHERIFF and GODOFREDO QUILING, in his capacity as Deputy Sheriff Committee for the payment of its loan credit. In the meantime, the New Agrix,
of Calamba, Laguna, Respondents. Inc. and the National Development Company, petitioners herein, invoking
Sec. 4 (1) of the decree, filed a petition with the Regional Trial Court of
Calamba, Laguna, for the cancellation of the mortgage lien in favor of the
private respondent. For its part, the private respondent took steps to
DECISION extrajudicially foreclose the mortgage, prompting the petitioners to file a
second case with the same court to stop the foreclosure. The two cases were
consolidated.

After the submission by the parties of their respective pleadings, the trial
CRUZ, J.:
court rendered the impugned decision. Judge Francisco Ma. Guerrero
annulled not only the challenged provision, viz., Sec. 4 (1), but the entire
Pres. Decree No. 1717 on the grounds that: (1) the presidential exercise of
legislative power was a violation of the principle of separation of powers; (2)
This case involves the constitutionality of a presidential decree which, like all the law impaired the obligation of contracts; and (3) the decree violated the
other issuances of President Marcos during his regime, was at that time equal protection clause. The motion for reconsideration of this decision
regarded as sacrosanct. It is only now, in a freer atmosphere, that his acts having been denied, the present petition was filed.: rd
are being tested by the touchstone of the fundamental law that even then
was supposed to limit presidential action.: rd The petition was originally assigned to the Third Division of this Court but
because of the constitutional questions involved it was transferred to the
The particular enactment in question is Pres. Decree No. 1717, which Court en banc. On August 30, 1988, the Court granted the petitioner's prayer
ordered the rehabilitation of the Agrix Group of Companies to be for a temporary restraining order and instructed the respondents to cease
administered mainly by the National Development Company. The law and desist from conducting a public auction sale of the lands in question.
outlined the procedure for filing claims against the Agrix companies and After the Solicitor General and the private respondent had filed their
created a Claims Committee to process these claims. Especially relevant to comments and the petitioners their reply, the Court gave due course to the
this case, and noted at the outset, is Sec. 4(1) thereof providing that "all petition and ordered the parties to file simultaneous memoranda. Upon
mortgages and other liens presently attaching to any of the assets of the compliance by the parties, the case was deemed submitted.
dissolved corporations are hereby extinguished."
The petitioners contend that the private respondent is now estopped from
Earlier, the Agrix Marketing, Inc. (AGRIX) had executed in favor of private contesting the validity of the decree. In support of this contention, it cites the
respondent Philippine Veterans Bank a real estate mortgage dated July 7, recent case of Mendoza v. Agrix Marketing, Inc., 1 where the constitutionality
1978, over three (3) parcels of land situated in Los Baos, Laguna. During of Pres. Decree No. 1717 was also raised but not resolved. The Court, after
the existence of the mortgage, AGRIX went bankrupt. It was for the

8
noting that the petitioners had already filed their claims with the AGRIX claim, which was kept pending for more than seven years for alleged lack of
Claims Committee created by the decree, had simply dismissed the petition supporting papers. Significantly, the validity of that claim was not questioned
on the ground of estoppel. by the petitioner when it sought to restrain the extrajudicial foreclosure of the
mortgage by the private respondent. The petitioner limited itself to the
The petitioners stress that in the case at bar the private respondent also argument that the private respondent was estopped from questioning the
invoked the provisions of Pres. Decree No. 1717 by filing a claim with the decree because of its earlier compliance with its provisions.
AGRIX Claims Committee. Failing to get results, it sought to foreclose the
real estate mortgage executed by AGRIX in its favor, which had been Independently of these observations, there is the consideration that an
extinguished by the decree. It was only when the petitioners challenged the affront to the Constitution cannot be allowed to continue existing simply
foreclosure on the basis of Sec. 4 (1) of the decree, that the private because of procedural inhibitions that exalt form over substance.
respondent attacked the validity of the provision. At that stage, however,
consistent with Mendoza, the private respondent was already estopped from The Court is especially disturbed by Section 4(1) of the decree, quoted
questioning the constitutionality of the decree. above, extinguishing all mortgages and other liens attaching to the assets of
AGRIX. It also notes, with equal concern, the restriction in Subsection (ii)
The Court does not agree that the principle of estoppel is applicable. thereof that all "unsecured obligations shall not bear interest" and in
Subsection (iii) that "all accrued interests, penalties or charges as of date
It is not denied that the private respondent did file a claim with the AGRIX hereof pertaining to the obligations, whether secured or unsecured, shall not
Claims Committee pursuant to this decree. It must be noted, however, that be recognized."
this was done in 1980, when President Marcos was the absolute ruler of this
country and his decrees were the absolute law. Any judicial challenge to These provisions must be read with the Bill of Rights, where it is clearly
them would have been futile, not to say foolhardy. The private respondent, no provided in Section 1 that "no person shall be deprived of life, liberty or
less than the rest of the nation, was aware of that reality and knew it had no property without due course of law nor shall any person be denied the equal
choice under the circumstances but to conform.: nad protection of the law" and in Section 10 that "no law impairing the obligation
of contracts shall be passed."
It is true that there were a few venturesome souls who dared to question the
dictator's decisions before the courts of justice then. The record will show, In defending the decree, the petitioners argue that property rights, like all
however, that not a single act or issuance of President Marcos was ever rights, are subject to regulation under the police power for the promotion of
declared unconstitutional, not even by the highest court, as long as he was in the common welfare. The contention is that this inherent power of the state
power. To rule now that the private respondent is estopped for having abided may be exercised at any time for this purpose so long as the taking of the
with the decree instead of boldly assailing it is to close our eyes to a cynical property right, even if based on contract, is done with due process of law.
fact of life during that repressive time.
This argument is an over-simplification of the problem before us. The police
This case must be distinguished from Mendoza, where the petitioners, after power is not a panacea for all constitutional maladies. Neither does its mere
filing their claims with the AGRIX Claims Committee, received in settlement invocation conjure an instant and automatic justification for every act of the
thereof shares of stock valued at P40,000.00 without protest or reservation. government depriving a person of his life, liberty or property.
The herein private respondent has not been paid a single centavo on its

9
A legislative act based on the police power requires the concurrence of a A mortgage lien is a property right derived from contract and so comes under
lawful subject and a lawful method. In more familiar words, a) the interests of the protection of the Bill of Rights. So do interests on loans, as well as
the public generally, as distinguished from those of a particular class, should penalties and charges, which are also vested rights once they accrue.
justify the interference of the state; and b) the means employed are Private property cannot simply be taken by law from one person and given to
reasonably necessary for the accomplishment of the purpose and not unduly another without compensation and any known public purpose. This is plain
oppressive upon individuals. 2 arbitrariness and is not permitted under the Constitution.

Applying these criteria to the case at bar, the Court finds first of all that the And not only is there arbitrary taking, there is discrimination as well. In
interests of the public are not sufficiently involved to warrant the interference extinguishing the mortgage and other liens, the decree lumps the secured
of the government with the private contracts of AGRIX. The decree speaks creditors with the unsecured creditors and places them on the same level in
vaguely of the "public, particularly the small investors," who would be the prosecution of their respective claims. In this respect, all of them are
prejudiced if the corporation were not to be assisted. However, the record considered unsecured creditors. The only concession given to the secured
does not state how many there are of such investors, and who they are, and creditors is that their loans are allowed to earn interest from the date of the
why they are being preferred to the private respondent and other creditors of decree, but that still does not justify the cancellation of the interests earned
AGRIX with vested property rights.:-cralaw before that date. Such interests, whether due to the secured or the
unsecured creditors, are all extinguished by the decree. Even assuming such
The public interest supposedly involved is not identified or explained. It has cancellation to be valid, we still cannot see why all kinds of creditors,
not been shown that by the creation of the New Agrix, Inc. and the extinction regardless of security, are treated alike.
of the property rights of the creditors of AGRIX, the interests of the public as
a whole, as distinguished from those of a particular class, would be promoted Under the equal protection clause, all persons or things similarly situated
or protected. The indispensable link to the welfare of the greater number has must be treated alike, both in the privileges conferred and the obligations
not been established. On the contrary, it would appear that the decree was imposed. Conversely, all persons or things differently situated should be
issued only to favor a special group of investors who, for reasons not given, treated differently. In the case at bar, persons differently situated are similarly
have been preferred to the legitimate creditors of AGRIX. treated, in disregard of the principle that there should be equality only among
equals.- nad
Assuming there is a valid public interest involved, the Court still finds that the
means employed to rehabilitate AGRIX fall far short of the requirement that One may also well wonder why AGRIX was singled out for government help,
they shall not be unduly oppressive. The oppressiveness is patent on the among other corporations where the stockholders or investors were also
face of the decree. The right to property in all mortgages, liens, interests, swindled. It is not clear why other companies entitled to similar concern were
penalties and charges owing to the creditors of AGRIX is arbitrarily not similarly treated. And surely, the stockholders of the private respondent,
destroyed. No consideration is paid for the extinction of the mortgage rights. whose mortgage lien had been cancelled and legitimate claims to accrued
The accrued interests and other charges are simply rejected by the decree. interests rejected, were no less deserving of protection, which they did not
The right to property is dissolved by legislative fiat without regard to the get. The decree operated, to use the words of a celebrated case, 3 "with an
private interest violated and, worse, in favor of another private interest. evil eye and an uneven hand."

10
On top of all this, New Agrix, Inc. was created by special decree take precedence over all other claims, including those of the government. In
notwithstanding the provision of Article XIV, Section 4 of the 1973 arriving at this ruling, the Court recognized the mortgage lien as a property
Constitution, then in force, that: right protected by the due process and contract clauses notwithstanding the
argument that the amendment in Section 110 of the Labor Code was a
SEC. 4. The Batasang Pambansa shall not, except by general law, provide proper exercise of the police power.: nad
for the formation, organization, or regulation of private corporations, unless
such corporations are owned or controlled by the Government or any The Court reaffirms and applies that ruling in the case at bar.
subdivision or instrumentality thereof. 4
Our finding, in sum, is that Pres. Decree No. 1717 is an invalid exercise of
The new corporation is neither owned nor controlled by the government. The the police power, not being in conformity with the traditional requirements of
National Development Corporation was merely required to extend a loan of a lawful subject and a lawful method. The extinction of the mortgage and
not more than P10,000,000.00 to New Agrix, Inc. Pending payment thereof, other liens and of the interest and other charges pertaining to the legitimate
NDC would undertake the management of the corporation, but with the creditors of AGRIX constitutes taking without due process of law, and this is
obligation of making periodic reports to the Agrix board of directors. After compounded by the reduction of the secured creditors to the category of
payment of the loan, the said board can then appoint its own management. unsecured creditors in violation of the equal protection clause. Moreover, the
The stocks of the new corporation are to be issued to the old investors and new corporation, being neither owned nor controlled by the Government,
stockholders of AGRIX upon proof of their claims against the abolished should have been created only by general and not special law. And insofar as
corporation. They shall then be the owners of the new corporation. New the decree also interferes with purely private agreements without any
Agrix, Inc. is entirely private and so should have been organized under the demonstrated connection with the public interest, there is likewise an
Corporation Law in accordance with the above-cited constitutional provision. impairment of the obligation of the contract.

The Court also feels that the decree impairs the obligation of the contract With the above pronouncements, we feel there is no more need to rule on
between AGRIX and the private respondent without justification. While it is the authority of President Marcos to promulgate Pres. Decree No. 1717
true that the police power is superior to the impairment clause, the principle under Amendment No. 6 of the 1973 Constitution. Even if he had such
will apply only where the contract is so related to the public welfare that it will authority, the decree must fall just the same because of its violation of the Bill
be considered congenitally susceptible to change by the legislature in the of Rights.
interest of the greater number. 5 Most present-day contracts are of that
nature. But as already observed, the contracts of loan and mortgage WHEREFORE, the petition is DISMISSED. Pres. Decree No. 1717 is
executed by AGRIX are purely private transactions and have not been shown declared UNCONSTITUTIONAL. The temporary restraining order dated
to be affected with public interest. There was therefore no warrant to amend August 30, 1988, is LIFTED. Costs against the petitioners.- nad
their provisions and deprive the private respondent of its vested property
rights. SO ORDERED.

It is worth noting that only recently in the case of the Development Bank of Fernan (C.J.), Narvasa, Gutierrez, Jr., Paras, Gancayco Padilla, Bidin,
the Philippines v. NLRC, 6 we sustained the preference in payment of a Sarmiento, Grio-Aquino, Medialdea and Regalado, JJ., concur.
mortgage creditor as against the argument that the claims of laborers should

11
Melencio-Herrera, J., In the result. In Dumlao v. COMELEC, 95 SCRA
392 (1980), a portion of the second paragraph of section 4 of Batas
Pambansa Blg. 52 was declared null and void for being
unconstitutional.

Feliciano, J., is on leave.

12
13
b) Interest at 12% per annum 5,706.14
G.R. No. L-19891 July 31, 1964

J.R.S. BUSINESS CORPORATION, J.R. DA SILVA and A.J.


BELTRAN, petitioners, c) Liquidated damages at 7% per annum 3,330.58
vs.
IMPERIAL INSURANCE, INC., MACARIO M. OFILADA, Sheriff of Manila
and
HON. AGUSTIN MONTESA, Judge of the Court of First Instance of d) Costs of suit 135.60
Manila, respondents.

Felipe N. Aurea for petitioners.


Taada, Teehankee and Carreon for respondent Imperial Insurance, Inc. e) Attorney's fees 2,000.00

PAREDES, J.:
2) WHEREAS, the DEFENDANTS bind themselves, jointly and
Petitioner J. R. Da Silva, is the President of the J.R.S. Business Corporation, severally, and hereby promise to pay their aforementioned obligation
an establishment duly franchised by the Congress of the Philippines, to to the PLAINTIFF at its business address at 301-305 Banquero St.,
conduct a messenger and delivery express service. On July 12, 1961, the (Ground Floor), Regina Building, Escolta, Manila, within sixty (60)
respondent Imperial Insurance, Inc., presented with the CFI of Manila a days from March 16, 1962 or on or before May 14, 1962;
complaint (Civ. Case No. 47520), for sum of money against the petitioner
corporation. After the defendants therein have submitted their Answer, the 3) WHEREAS, in the event the DEFENDANTS FAIL to pay in full the
parties entered into a Compromise Agreement, assisted by their respective total amount of PESOS SIXTY ONE THOUSAND ONE HUNDRED
counsels, the pertinent portions of which recite: SEVENTY TWO & 32/100 (P61,172.32), Philippine Currency, for any
reason whatsoever, on May 14, 1962, the PLAINTIFF shall be
1) WHEREAS, the DEFENDANTS admit and confess their joint and entitled, as a matter of right, to move for the execution of the
solidary indebtedness to the PLAINTIFF in the full sum of PESOS decision to be rendered in the above-entitled case by this Honorable
SIXTY ONE THOUSAND ONE HUNDRED SEVENTY-TWO & Court based on this COMPROMISE AGREEMENT.
32/100 (P61,172.32), Philippine Currency, itemized as follows:
On March 17, 1962, the lower court rendered judgment embodying the
contents of the said compromise agreement, the dispositive portion of which
a) Principal P50,000.00reads

WHEREFORE, the Court hereby approves the above-quoted


compromise agreement and renders judgment in accordance

14
therewith, enjoining the parties to comply faithfully and strictly with within the next ten (10) days, and they would be able to discharge the
the terms and conditions thereof, without special pronouncement as judgment debt. Respondents opposed the said motion and on June 21, 1962,
to costs. the lower court denied the motion for postponement of the auction sale.

Wherefore, the parties respectfully pray that the foregoing stipulation In the sale which was conducted in the premises of the JRS Business
of facts be admitted and approved by this Honorable Court, without Corporation at 1341 Perez St., Paco, Manila, all the properties of said
prejudice to the parties adducing other evidence to prove their case corporation contained in the Notices of Sale dated May 26, 1962, and June
not covered by this stipulation of facts. 1wph1.t 2, 1962 (the latter notice being for the whole capital stocks of the defendant,
JRS Business Corporation, the business name, right of operation, the whole
On May 15, 1962, one day after the date fixed in the compromise agreement, assets, furnitures and equipments, the total liabilities and Net Worth, books
within which the judgment debt would be paid, but was not, respondent of accounts, etc., etc.), were bought by respondent Imperial Insurance, Inc.,
Imperial Insurance Inc., filed a "Motion for the Insurance of a Writ of for P10,000.00, which was the highest bid offered. Immediately after the sale,
Execution". On May 23, 1962, a Writ of Execution was issued by respondent respondent Insurance Company took possession of the proper ties and
Sheriff of Manila and on May 26, 1962, Notices of Sale were sent out for the started running the affairs and operating the business of the JRS Business
auction of the personal properties of the petitioner J.R.S. Business Corporation. Hence, the present appeal.
Corporation. On June 2, 1962, a Notice of Sale of the "whole capital stocks of
the defendants JRS Business Corporation, the business name, right of It would seem that the matters which need determination are (1) whether the
operation, the whole assets, furnitures and equipments, the total liabilities, respondent Judge acted without or in excess of his jurisdiction or with grave
and Net Worth, books of accounts, etc., etc." of the petitioner corporation abuse of discretion in promulgating the Order of June 21, 1962, denying the
was, handed down. On June 9, the petitioner, thru counsel, presented an motion for postponement of the scheduled sale at public auction, of the
"Urgent Petition for Postponement of Auction Sale and for Release of Levy properties of petitioner; and (2) whether the business name or trade name,
on the Business Name and Right to Operate of Defendant JRS Business franchise (right to operate) and capital stocks of the petitioner are properties
Corporation", stating that petitioners were busy negotiating for a loan with or property rights which could be the subject of levy, execution and sale.
which to pay the judgment debt; that the judgment was for money only and,
therefore, plaintiff (respondent Insurance Company) was not authorized to The respondent Court's act of postponing the scheduled sale was within the
take over and appropriate for its own use, the business name of the discretion of respondent Judge, the exercise of which, one way or the other,
defendants; that the right to operate under the franchise, was not did not constitute grave abuse of discretion and/or excess of jurisdiction.
transferable and could not be considered a personal or immovable, property, There was a decision rendered and the corresponding writ of execution was
subject to levy and sale. On June 10, 1962, a Supplemental Motion for issued. Respondent Judge had jurisdiction over the matter and erroneous
Release of Execution, was filed by counsel of petitioner JRS Business conclusions of law or fact, if any, committed in the exercise of such
Corporation, claiming that the capital stocks thereof, could not be levied upon jurisdiction are merely errors of judgment, not correctible by certiorari (Villa
and sold under execution. Under date of June 20, 1962, petitioner's counsel Rey Transit v. Bello, et al., L-18957, April 23, 1963, and cases cited therein.)
presented a pleading captioned "Very Urgent Motion for Postponement of
Public Auction Sale and for Ruling on Motion for Release of Levy on The corporation law, on forced sale of franchises, provides
the Business Name, Right to Operate and Capital Stocks of JRS Business
Corporation". The auction sale was set for June 21, 1962. In said motion,
petitioners alleged that the loan they had applied for, was to be secured

15
Any franchise granted to a corporation to collect tolls or to occupy, 160; Adams v. Yazon & M. V. R. Co., 24 So. 200, 317, 28 So.
enjoy, or use public property or any portion of the public domain or 956, 77 Miss. 253, 60 L.R.A. 33 et seq.
any right of way over public property or the public domain, and any
rights and privileges acquired under such franchise may be levied The primary franchise of a corporation that is, the right to exist as
upon and sold under execution, together with the property necessary such, is vested "in the individuals who compose the corporation and
for the enjoyment, the exercise of the powers, and the receipt of the not in the corporation itself" (14 C.J. pp. 160, 161; Adams v.
proceeds of such franchise or right of way, in the same manner and Railroad, supra; 2 Fletcher's Cyclopedia Corp. Secs. 1153, 1158; 3
with like effect as any other property to satisfy any judgment against Thompson on Corporations 2d Ed.] Secs. 2863, 2864), and cannot
the corporation: Provided, That the sale of the franchise or right of be conveyed in the absence of a legislative authority so to do (14A
way and the property necessary for the enjoyment, the exercise of CJ. 543, 577; 1 Fletcher's Cyc. Corp. Sec. 1224; Memphis & L.R.R.
the powers, and the receipt of the proceeds of said franchise or right Co. v. Berry 5 S. Ct. 299, 112 U.S. 609, 28 L.E.d. 837; Vicksburg
of way is especially decreed and ordered in the judgment: And Waterworks Co. v. Vicksburg, 26 S. Ct. 660, 202 U.S. 453, 50 L.E.d.
provided, further, That the sale shall not become effective until 1102, 6 Ann. Cas. 253; Arthur v. Commercial & Railroad Bank, 9
confirmed by the court after due notice. (Sec. 56, Corporation Law.) Smedes & M. 394, 48 Am. Dec. 719), but the specify or secondary
franchises of a corporation are vested in the corporation and may
In the case of Gulf Refining Co. v. Cleveland Trust Co., 108 So., 158, it was ordinarily be conveyed or mortgaged under a general power granted
held to a corporation to dispose of its property (Adams v. Railroad, supra;
14A C.J. 542, 557; 3 Thompson on Corp. [2nd Ed.] Sec.
The first question then for decision is the meaning of the word 2909), except such special or secondary franchises as are charged
"franchise" in the statute. with a public use (2 Fletcher's Cyc. Corp. see. 1225; 14A C.J. 544; 3
Thompson on Corp. [2d Ed.] sec. 2908; Arthur v. Commercial & R.R.
"A franchise is a special privilege conferred by governmental Bank, supra; McAllister v. Plant, 54 Miss. 106).
authority, and which does not belong to citizens of the
country generally as a matter of common right. ... Its The right to operate a messenger and express delivery service, by virtue of a
meaning depends more or less upon the connection in which legislative enactment, is admittedly a secondary franchise (R.A. No. 3260,
the word is employed and the property and corporation to entitled "An Act granting the JRS Business Corporation a franchise to
which it is applied. It may have different significations. conduct a messenger and express service)" and, as such, under our
corporation law, is subject to levy and sale on execution together and
"For practical purposes, franchises, so far as relating to including all the property necessary for the enjoyment thereof. The law,
corporations, are divisible into (1) corporate or general however, indicates the procedure under which the same (secondary
franchises; and (2) special or secondary franchises. The franchise and the properties necessary for its enjoyment) may be sold under
former is the franchise to exist as a corporation, while the execution. Said franchise can be sold under execution, when such sale is
latter are certain rights and privileges conferred upon especially decreed and ordered in the judgment and it becomes effective
existing corporations, such as the right to use the streets of a only when the sale is confirmed by the Court after due notice (Sec. 56, Corp.
municipality to lay pipes or tracks, erect poles or string Law). The compromise agreement and the judgment based thereon, do not
wires." 2 Fletcher's Cyclopedia Corp. See. 1148; 14 C.J. p. contain any special decree or order making the franchise answerable for the
judgment debt. The same thing may be stated with respect to petitioner's

16
trade name or business name and its capital stock. Incidentally, the trade
name or business name corresponds to the initials of the President of the
petitioner corporation and there can be no serious dispute regarding the fact
that a trade name or business name and capital stock are necessarily
included in the enjoyment of the franchise. Like that of a franchise, the law
mandates, that property necessary for the enjoyment of said franchise, can
only be sold to satisfy a judgment debt if the decision especially so provides.
As We have stated heretofore, no such directive appears in the decision.
Moreover, a trade name or business name cannot be sold separately from
the franchise, and the capital stock of the petitioner corporation or any other
corporation, for the matter, represents the interest and is the property of
stockholders in the corporation, who can only be deprived thereof in the
manner provided by law (Therbee v. Baker, 35 N.E. Eq. [8 Stew.] 501, 505; In
re Wells' Estate, 144 N.W. 174, 177, Wis. 294, cited in 6 Words and Phrases,
109).

It, therefore, results that the inclusion of the franchise, the trade name and/or
business name and the capital stock of the petitioner corporation, in the sale
of the properties of the JRS Business Corporation, has no justification. The
sale of the properties of petitioner corporation is set aside, in so far as it
authorizes the levy and sale of its franchise, trade name and capital stocks.
Without pronouncement as to costs.

Bengzon, C.J., Padilla, Bautista Angelo, Concepcion, Reyes, J.B.L., Regala


and Makalintal, JJ., concur.

17
G.R. No. L-4935 May 28, 1954

J. M. TUASON & CO., INC., represented by it Managing PARTNER,


GREGORIA ARANETA, INC., plaintiff-appellee,
vs.
QUIRINO BOLAOS, defendant-appellant.

Araneta and Araneta for appellee.


Jose A. Buendia for appellant.

REYES, J.:

This is an action originally brought in the Court of First Instance of Rizal,


Quezon City Branch, to recover possesion of registered land situated in
barrio Tatalon, Quezon City.

Plaintiff's complaint was amended three times with respect to the extent and
description of the land sought to be recovered. The original complaint
described the land as a portion of a lot registered in plaintiff's name under
Transfer Certificate of Title No. 37686 of the land record of Rizal Province
and as containing an area of 13 hectares more or less. But the complaint
was amended by reducing the area of 6 hectares, more or less, after the
defendant had indicated the plaintiff's surveyors the portion of land claimed
and occupied by him. The second amendment became necessary and was
allowed following the testimony of plaintiff's surveyors that a portion of the
area was embraced in another certificate of title, which was plaintiff's
Transfer Certificate of Title No. 37677. And still later, in the course of trial,
after defendant's surveyor and witness, Quirino Feria, had testified that the
area occupied and claimed by defendant was about 13 hectares, as shown in
his Exhibit 1, plaintiff again, with the leave of court, amended its complaint to
make its allegations conform to the evidence.

Defendant, in his answer, sets up prescription and title in himself thru "open,
continuous, exclusive and public and notorious possession (of land in

18
dispute) under claim of ownership, adverse to the entire world by defendant VIII. The trial court erred in not ordering the plaintiff to reconvey the
and his predecessor in interest" from "time in-memorial". The answer further land in litigation to the defendant.
alleges that registration of the land in dispute was obtained by plaintiff or its
predecessors in interest thru "fraud or error and without knowledge (of) or As to the first assigned error, there is nothing to the contention that the
interest either personal or thru publication to defendant and/or predecessors present action is not brought by the real party in interest, that is, by J. M.
in interest." The answer therefore prays that the complaint be dismissed with Tuason and Co., Inc. What the Rules of Court require is that an action be
costs and plaintiff required to reconvey the land to defendant or pay its value. brought in the name of, but not necessarily by, the real party in interest.
(Section 2, Rule 2.) In fact the practice is for an attorney-at-law to bring the
After trial, the lower court rendered judgment for plaintiff, declaring defendant action, that is to file the complaint, in the name of the plaintiff. That practice
to be without any right to the land in question and ordering him to restore appears to have been followed in this case, since the complaint is signed by
possession thereof to plaintiff and to pay the latter a monthly rent of P132.62 the law firm of Araneta and Araneta, "counsel for plaintiff" and commences
from January, 1940, until he vacates the land, and also to pay the costs. with the statement "comes now plaintiff, through its undersigned counsel." It
is true that the complaint also states that the plaintiff is "represented herein
Appealing directly to this court because of the value of the property involved, by its Managing Partner Gregorio Araneta, Inc.", another corporation, but
defendant makes the following assignment or errors: there is nothing against one corporation being represented by another
person, natural or juridical, in a suit in court. The contention that Gregorio
I. The trial court erred in not dismissing the case on the ground that Araneta, Inc. can not act as managing partner for plaintiff on the theory that it
the case was not brought by the real property in interest. is illegal for two corporations to enter into a partnership is without merit, for
the true rule is that "though a corporation has no power to enter into a
partnership, it may nevertheless enter into a joint venture with another where
II. The trial court erred in admitting the third amended complaint.
the nature of that venture is in line with the business authorized by its
charter." (Wyoming-Indiana Oil Gas Co. vs. Weston, 80 A. L. R., 1043, citing
III. The trial court erred in denying defendant's motion to strike. 2 Fletcher Cyc. of Corp., 1082.) There is nothing in the record to indicate that
the venture in which plaintiff is represented by Gregorio Araneta, Inc. as "its
IV. The trial court erred in including in its decision land not involved in managing partner" is not in line with the corporate business of either of them.
the litigation.
Errors II, III, and IV, referring to the admission of the third amended
V. The trial court erred in holding that the land in dispute is covered complaint, may be answered by mere reference to section 4 of Rule 17,
by transfer certificates of Title Nos. 37686 and 37677. Rules of Court, which sanctions such amendment. It reads:

Vl. The trial court erred in not finding that the defendant is the true Sec. 4. Amendment to conform to evidence. When issues not
and lawful owner of the land. raised by the pleadings are tried by express or implied consent of the
parties, they shall be treated in all respects, as if they had been
VII. The trial court erred in finding that the defendant is liable to pay raised in the pleadings. Such amendment of the pleadings as may
the plaintiff the amount of P132.62 monthly from January, 1940, until be necessary to cause them to conform to the evidence and to raise
he vacates the premises. these issues may be made upon motion of any party at my time,
even of the trial of these issues. If evidence is objected to at the trial

19
on the ground that it is not within the issues made by the pleadings, title No. 37677 of the land records of the same province, both lots having
the court may allow the pleadings to be amended and shall be so been originally registered on July 8, 1914 under original certificate of title No.
freely when the presentation of the merits of the action will be 735. The identity of the lots was established by the testimony of Antonio
subserved thereby and the objecting party fails to satisfy the court Manahan and Magno Faustino, witnesses for plaintiff, and the identity of the
that the admission of such evidence would prejudice him in portion thereof claimed by defendant was established by the testimony of his
maintaining his action or defense upon the merits. The court may own witness, Quirico Feria. The combined testimony of these three
grant a continuance to enable the objecting party to meet such witnesses clearly shows that the portion claimed by defendant is made up of
evidence. a part of lot 4-B-3-C and major on portion of lot 4-B-4, and is well within the
area covered by the two transfer certificates of title already mentioned. This
Under this provision amendment is not even necessary for the purpose of fact also appears admitted in defendant's answer to the third amended
rendering judgment on issues proved though not alleged. Thus, commenting complaint.
on the provision, Chief Justice Moran says in this Rules of Court:
As the land in dispute is covered by plaintiff's Torrens certificate of title and
Under this section, American courts have, under the New Federal was registered in 1914, the decree of registration can no longer be impugned
Rules of Civil Procedure, ruled that where the facts shown entitled on the ground of fraud, error or lack of notice to defendant, as more than one
plaintiff to relief other than that asked for, no amendment to the year has already elapsed from the issuance and entry of the decree. Neither
complaint is necessary, especially where defendant has himself court the decree be collaterally attacked by any person claiming title to, or
raised the point on which recovery is based, and that the appellate interest in, the land prior to the registration proceedings. (Sorogon vs.
court treat the pleadings as amended to conform to the evidence, Makalintal,1 45 Off. Gaz., 3819.) Nor could title to that land in derogation of
although the pleadings were not actually amended. (I Moran, Rules that of plaintiff, the registered owner, be acquired by prescription or adverse
of Court, 1952 ed., 389-390.) possession. (Section 46, Act No. 496.) Adverse, notorious and continuous
possession under claim of ownership for the period fixed by law is ineffective
Our conclusion therefore is that specification of error II, III, and IV are without against a Torrens title. (Valiente vs. Judge of CFI of Tarlac,2 etc., 45 Off. Gaz.,
merit.. Supp. 9, p. 43.) And it is likewise settled that the right to secure possession
under a decree of registration does not prescribed. (Francisco vs. Cruz, 43
Off. Gaz., 5105, 5109-5110.) A recent decision of this Court on this point is
Let us now pass on the errors V and VI. Admitting, though his attorney, at the
that rendered in the case of Jose Alcantara et al., vs. Mariano et al., 92 Phil.,
early stage of the trial, that the land in dispute "is that described or
796. This disposes of the alleged errors V and VI.
represented in Exhibit A and in Exhibit B enclosed in red pencil with the name
Quirino Bolaos," defendant later changed his lawyer and also his theory and
tried to prove that the land in dispute was not covered by plaintiff's certificate As to error VII, it is claimed that `there was no evidence to sustain the finding
of title. The evidence, however, is against defendant, for it clearly establishes that defendant should be sentenced to pay plaintiff P132.62 monthly from
that plaintiff is the registered owner of lot No. 4-B-3-C, situate in barrio January, 1940, until he vacates the premises.' But it appears from the record
Tatalon, Quezon City, with an area of 5,297,429.3 square meters, more or that that reasonable compensation for the use and occupation of the
less, covered by transfer certificate of title No. 37686 of the land records of premises, as stipulated at the hearing was P10 a month for each hectare and
Rizal province, and of lot No. 4-B-4, situated in the same barrio, having an that the area occupied by defendant was 13.2619 hectares. The total rent to
area of 74,789 square meters, more or less, covered by transfer certificate of be paid for the area occupied should therefore be P132.62 a month. It is
appears from the testimony of J. A. Araneta and witness Emigdio Tanjuatco

20
that as early as 1939 an action of ejectment had already been filed against
defendant. And it cannot be supposed that defendant has been paying rents,
for he has been asserting all along that the premises in question 'have
always been since time immemorial in open, continuous, exclusive and
public and notorious possession and under claim of ownership adverse to the
entire world by defendant and his predecessors in interest.' This assignment
of error is thus clearly without merit.

Error No. VIII is but a consequence of the other errors alleged and needs for
further consideration.

During the pendency of this case in this Court appellant, thru other counsel,
has filed a motion to dismiss alleging that there is pending before the Court
of First Instance of Rizal another action between the same parties and for the
same cause and seeking to sustain that allegation with a copy of the
complaint filed in said action. But an examination of that complaint reveals
that appellant's allegation is not correct, for the pretended identity of parties
and cause of action in the two suits does not appear. That other case is one
for recovery of ownership, while the present one is for recovery of
possession. And while appellant claims that he is also involved in that order
action because it is a class suit, the complaint does not show that such is
really the case. On the contrary, it appears that the action seeks relief for
each individual plaintiff and not relief for and on behalf of others. The motion
for dismissal is clearly without merit.

Wherefore, the judgment appealed from is affirmed, with costs against the
plaintiff.

Paras, C.J., Pablo, Bengzon, Montemayor, Jugo, Bautista Angelo, Labrador,


and Concepcion, JJ., concur.

21
22
Sycip, Salazar, Hernandez & Gatmaitan for Luciano E. Salazar.

GUTIERREZ, JR., J.:

G.R. No. 75875 December 15, 1989 These consolidated petitions seek the review of the amended decision of the
Court of Appeals in CA-G.R. SP Nos. 05604 and 05617 which set aside the
WOLRGANG AURBACH, JOHN GRIFFIN, DAVID P. WHITTINGHAM and earlier decision dated June 5, 1986, of the then Intermediate Appellate Court
CHARLES CHAMSAY, petitioners, and directed that in all subsequent elections for directors of Sanitary Wares
vs. Manufacturing Corporation (Saniwares), American Standard Inc. (ASI)
SANITARY WARES MANUFACTURING CORPORATOIN, ERNESTO V. cannot nominate more than three (3) directors; that the Filipino stockholders
LAGDAMEO, ERNESTO R. LAGDAMEO, JR., ENRIQUE R. LAGDAMEO, shall not interfere in ASI's choice of its three (3) nominees; that, on the other
GEORGE F. LEE, RAUL A. BONCAN, BALDWIN YOUNG and AVELINO V. hand, the Filipino stockholders can nominate only six (6) candidates and in
CRUZ, respondents. the event they cannot agree on the six (6) nominees, they shall vote only
among themselves to determine who the six (6) nominees will be, with
G.R. No. 75951 December 15, 1989 cumulative voting to be allowed but without interference from ASI.

SANITARY WARES MANUFACTURING CORPORATION, ERNESTO R. The antecedent facts can be summarized as follows:
LAGDAMEO, ENRIQUE B. LAGDAMEO, GEORGE FL .EE RAUL A.
BONCAN, BALDWIN YOUNG and AVELINO V. CRUX, petitioners, In 1961, Saniwares, a domestic corporation was incorporated for the primary
vs. purpose of manufacturing and marketing sanitary wares. One of the
THE COURT OF APPEALS, WOLFGANG AURBACH, JOHN GRIFFIN, incorporators, Mr. Baldwin Young went abroad to look for foreign partners,
DAVID P. WHITTINGHAM, CHARLES CHAMSAY and LUCIANO European or American who could help in its expansion plans. On August 15,
SALAZAR, respondents. 1962, ASI, a foreign corporation domiciled in Delaware, United States
entered into an Agreement with Saniwares and some Filipino investors
G.R. Nos. 75975-76 December 15, 1989 whereby ASI and the Filipino investors agreed to participate in the ownership
of an enterprise which would engage primarily in the business of
manufacturing in the Philippines and selling here and abroad vitreous china
LUCIANO E. SALAZAR, petitioner,
and sanitary wares. The parties agreed that the business operations in the
vs.
Philippines shall be carried on by an incorporated enterprise and that the
SANITARY WARES MANUFACTURING CORPORATION, ERNESTO V.
name of the corporation shall initially be "Sanitary Wares Manufacturing
LAGDAMEO, ERNESTO R. LAGDAMEO, JR., ENRIQUE R. LAGDAMEO,
Corporation."
GEORGE F. LEE, RAUL A. BONCAN, BALDWIN YOUNG, AVELINO V.
CRUZ and the COURT OF APPEALS, respondents.
The Agreement has the following provisions relevant to the issues in these
cases on the nomination and election of the directors of the corporation:
Belo, Abiera & Associates for petitioners in 75875.

23
3. Articles of Incorporation their desire to expand the export operations of the company to which ASI
objected as it apparently had other subsidiaries of joint joint venture groups
(a) The Articles of Incorporation of the Corporation shall be in the countries where Philippine exports were contemplated. On March 8,
substantially in the form annexed hereto as Exhibit A and, 1983, the annual stockholders' meeting was held. The meeting was presided
insofar as permitted under Philippine law, shall specifically by Baldwin Young. The minutes were taken by the Secretary, Avelino Cruz.
provide for After disposing of the preliminary items in the agenda, the stockholders then
proceeded to the election of the members of the board of directors. The ASI
(1) Cumulative voting for directors: group nominated three persons namely; Wolfgang Aurbach, John Griffin and
David P. Whittingham. The Philippine investors nominated six, namely;
Ernesto Lagdameo, Sr., Raul A. Boncan, Ernesto R. Lagdameo, Jr., George
xxx xxx xxx
F. Lee, and Baldwin Young. Mr. Eduardo R, Ceniza then nominated Mr.
Luciano E. Salazar, who in turn nominated Mr. Charles Chamsay. The
5. Management chairman, Baldwin Young ruled the last two nominations out of order on the
basis of section 5 (a) of the Agreement, the consistent practice of the parties
(a) The management of the Corporation shall be vested in a during the past annual stockholders' meetings to nominate only nine persons
Board of Directors, which shall consist of nine individuals. As as nominees for the nine-member board of directors, and the legal advice of
long as American-Standard shall own at least 30% of the Saniwares' legal counsel. The following events then, transpired:
outstanding stock of the Corporation, three of the nine
directors shall be designated by American-Standard, and the ... There were protests against the action of the Chairman
other six shall be designated by the other stockholders of the and heated arguments ensued. An appeal was made by the
Corporation. (pp. 51 & 53, Rollo of 75875) ASI representative to the body of stockholders present that a
vote be taken on the ruling of the Chairman. The Chairman,
At the request of ASI, the agreement contained provisions designed to Baldwin Young, declared the appeal out of order and no vote
protect it as a minority group, including the grant of veto powers over a on the ruling was taken. The Chairman then instructed the
number of corporate acts and the right to designate certain officers, such as Corporate Secretary to cast all the votes present and
a member of the Executive Committee whose vote was required for represented by proxy equally for the 6 nominees of the
important corporate transactions. Philippine Investors and the 3 nominees of ASI, thus
effectively excluding the 2 additional persons nominated,
Later, the 30% capital stock of ASI was increased to 40%. The corporation namely, Luciano E. Salazar and Charles Chamsay. The ASI
was also registered with the Board of Investments for availment of incentives representative, Mr. Jaqua protested the decision of the
with the condition that at least 60% of the capital stock of the corporation Chairman and announced that all votes accruing to ASI
shall be owned by Philippine nationals. shares, a total of 1,329,695 (p. 27, Rollo, AC-G.R. SP No.
05617) were being cumulatively voted for the three ASI
The joint enterprise thus entered into by the Filipino investors and the nominees and Charles Chamsay, and instructed the
American corporation prospered. Unfortunately, with the business successes, Secretary to so vote. Luciano E. Salazar and other proxy
there came a deterioration of the initially harmonious relations between the holders announced that all the votes owned by and or
two groups. According to the Filipino group, a basic disagreement was due to represented by them 467,197 shares (p. 27, Rollo, AC-G.R.

24
SP No. 05617) were being voted cumulatively in favor of positions of directors and that the body decided not to break
Luciano E. Salazar. The Chairman, Baldwin Young, the tie. (pp. 37-39, Rollo of 75975-76)
nevertheless instructed the Secretary to cast all votes
equally in favor of the three ASI nominees, namely, Wolfgang These incidents triggered off the filing of separate petitions by the parties
Aurbach, John Griffin and David Whittingham and the six with the Securities and Exchange Commission (SEC). The first petition filed
originally nominated by Rogelio Vinluan, namely, Ernesto was for preliminary injunction by Saniwares, Emesto V. Lagdameo, Baldwin
Lagdameo, Sr., Raul Boncan, Ernesto Lagdameo, Jr., Young, Raul A. Bonean Ernesto R. Lagdameo, Jr., Enrique Lagdameo and
Enrique Lagdameo, George F. Lee, and Baldwin Young. The George F. Lee against Luciano Salazar and Charles Chamsay. The case was
Secretary then certified for the election of the following denominated as SEC Case No. 2417. The second petition was for quo
Wolfgang Aurbach, John Griffin, David Whittingham Ernesto warranto and application for receivership by Wolfgang Aurbach, John Griffin,
Lagdameo, Sr., Ernesto Lagdameo, Jr., Enrique Lagdameo, David Whittingham, Luciano E. Salazar and Charles Chamsay against the
George F. Lee, Raul A. Boncan, Baldwin Young. The group of Young and Lagdameo (petitioners in SEC Case No. 2417) and
representative of ASI then moved to recess the meeting Avelino F. Cruz. The case was docketed as SEC Case No. 2718. Both sets of
which was duly seconded. There was also a motion to parties except for Avelino Cruz claimed to be the legitimate directors of the
adjourn (p. 28, Rollo, AC-G.R. SP No. 05617). This motion to corporation.
adjourn was accepted by the Chairman, Baldwin Young, who
announced that the motion was carried and declared the The two petitions were consolidated and tried jointly by a hearing officer who
meeting adjourned. Protests against the adjournment were rendered a decision upholding the election of the Lagdameo Group and
registered and having been ignored, Mr. Jaqua the ASI dismissing the quo warranto petition of Salazar and Chamsay. The ASI
representative, stated that the meeting was not adjourned Group and Salazar appealed the decision to the SEC en banc which affirmed
but only recessed and that the meeting would be the hearing officer's decision.
reconvened in the next room. The Chairman then threatened
to have the stockholders who did not agree to the decision of
The SEC decision led to the filing of two separate appeals with the
the Chairman on the casting of votes bodily thrown out. The
Intermediate Appellate Court by Wolfgang Aurbach, John Griffin, David
ASI Group, Luciano E. Salazar and other stockholders,
Whittingham and Charles Chamsay (docketed as AC-G.R. SP No. 05604)
allegedly representing 53 or 54% of the shares of
and by Luciano E. Salazar (docketed as AC-G.R. SP No. 05617). The
Saniwares, decided to continue the meeting at the elevator
petitions were consolidated and the appellate court in its decision ordered the
lobby of the American Standard Building. The continued
remand of the case to the Securities and Exchange Commission with the
meeting was presided by Luciano E. Salazar, while Andres
directive that a new stockholders' meeting of Saniwares be ordered
Gatmaitan acted as Secretary. On the basis of the
convoked as soon as possible, under the supervision of the Commission.
cumulative votes cast earlier in the meeting, the ASI Group
nominated its four nominees; Wolfgang Aurbach, John
Griffin, David Whittingham and Charles Chamsay. Luciano E. Upon a motion for reconsideration filed by the appellees Lagdameo Group)
Salazar voted for himself, thus the said five directors were the appellate court (Court of Appeals) rendered the questioned amended
certified as elected directors by the Acting Secretary, Andres decision. Petitioners Wolfgang Aurbach, John Griffin, David P. Whittingham
Gatmaitan, with the explanation that there was a tie among and Charles Chamsay in G.R. No. 75875 assign the following errors:
the other six (6) nominees for the four (4) remaining

25
I. THE COURT OF APPEALS, IN EFFECT, UPHELD THE I
ALLEGED ELECTION OF PRIVATE RESPONDENTS AS
MEMBERS OF THE BOARD OF DIRECTORS OF THE AMENDED DECISION OF THE RESPONDENT
SANIWARES WHEN IN FACT THERE WAS NO ELECTION COURT, WHILE RECOGNIZING THAT THE
AT ALL. STOCKHOLDERS OF SANIWARES ARE DIVIDED INTO
TWO BLOCKS, FAILS TO FULLY ENFORCE THE BASIC
II. THE COURT OF APPEALS PROHIBITS THE INTENT OF THE AGREEMENT AND THE LAW.
STOCKHOLDERS FROM EXERCISING THEIR FULL
VOTING RIGHTS REPRESENTED BY THE NUMBER OF II
SHARES IN SANIWARES, THUS DEPRIVING
PETITIONERS AND THE CORPORATION THEY THE AMENDED DECISION DOES NOT CATEGORICALLY
REPRESENT OF THEIR PROPERTY RIGHTS WITHOUT RULE THAT PRIVATE PETITIONERS HEREIN WERE THE
DUE PROCESS OF LAW. DULY ELECTED DIRECTORS DURING THE 8 MARCH
1983 ANNUAL STOCKHOLDERS MEETING OF
III. THE COURT OF APPEALS IMPOSES CONDITIONS SANTWARES. (P. 24, Rollo-75951)
AND READS PROVISIONS INTO THE AGREEMENT OF
THE PARTIES WHICH WERE NOT THERE, WHICH The issues raised in the petitions are interrelated, hence, they are discussed
ACTION IT CANNOT LEGALLY DO. (p. 17, Rollo-75875) jointly.

Petitioner Luciano E. Salazar in G.R. Nos. 75975-76 assails the amended The main issue hinges on who were the duly elected directors of Saniwares
decision on the following grounds: for the year 1983 during its annual stockholders' meeting held on March 8,
1983. To answer this question the following factors should be determined: (1)
11.1. ThatAmendedDecisionwouldsanctiontheCA'sdisregard the nature of the business established by the parties whether it was a joint
of binding contractual agreements entered into by venture or a corporation and (2) whether or not the ASI Group may vote their
stockholders and the replacement of the conditions of such additional 10% equity during elections of Saniwares' board of directors.
agreements with terms never contemplated by the
stockholders but merely dictated by the CA . The rule is that whether the parties to a particular contract have thereby
established among themselves a joint venture or some other relation
11.2. The Amended decision would likewise sanction the depends upon their actual intention which is determined in accordance with
deprivation of the property rights of stockholders without due the rules governing the interpretation and construction of contracts. (Terminal
process of law in order that a favored group of stockholders Shares, Inc. v. Chicago, B. and Q.R. Co. (DC MO) 65 F Supp 678; Universal
may be illegally benefitted and guaranteed a continuing Sales Corp. v. California Press Mfg. Co. 20 Cal. 2nd 751, 128 P 2nd 668)
monopoly of the control of a corporation. (pp. 14-15, Rollo-
75975-76) The ASI Group and petitioner Salazar (G.R. Nos. 75975-76) contend that the
actual intention of the parties should be viewed strictly on the "Agreement"
On the other hand, the petitioners in G.R. No. 75951 contend that:

26
dated August 15,1962 wherein it is clearly stated that the parties' intention Contrary to ASI Group's stand, the Lagdameo and Young Group pleaded in
was to form a corporation and not a joint venture. their Reply and Answer to Counterclaim in SEC Case No. 2417 that the
Agreement failed to express the true intent of the parties, to wit:
They specifically mention number 16 under Miscellaneous Provisions which
states: xxx xxx xxx

xxx xxx xxx 4. While certain provisions of the Agreement would make it
appear that the parties thereto disclaim being partners or
c) nothing herein contained shall be construed to constitute joint venturers such disclaimer is directed at third parties and
any of the parties hereto partners or joint venturers in is not inconsistent with, and does not preclude, the existence
respect of any transaction hereunder. (At P. 66, Rollo-GR of two distinct groups of stockholders in Saniwares one of
No. 75875) which (the Philippine Investors) shall constitute the majority,
and the other ASI shall constitute the minority stockholder. In
They object to the admission of other evidence which tends to show that the any event, the evident intention of the Philippine Investors
parties' agreement was to establish a joint venture presented by the and ASI in entering into the Agreement is to enter into ajoint
Lagdameo and Young Group on the ground that it contravenes the parol venture enterprise, and if some words in the Agreement
evidence rule under section 7, Rule 130 of the Revised Rules of Court. appear to be contrary to the evident intention of the parties,
According to them, the Lagdameo and Young Group never pleaded in their the latter shall prevail over the former (Art. 1370, New Civil
pleading that the "Agreement" failed to express the true intent of the parties. Code). The various stipulations of a contract shall be
interpreted together attributing to the doubtful ones that
sense which may result from all of them taken jointly (Art.
The parol evidence Rule under Rule 130 provides:
1374, New Civil Code). Moreover, in order to judge the
intention of the contracting parties, their contemporaneous
Evidence of written agreements-When the terms of an and subsequent acts shall be principally considered. (Art.
agreement have been reduced to writing, it is to be 1371, New Civil Code). (Part I, Original Records, SEC Case
considered as containing all such terms, and therefore, there No. 2417)
can be, between the parties and their successors in interest,
no evidence of the terms of the agreement other than the
It has been ruled:
contents of the writing, except in the following cases:

In an action at law, where there is evidence tending to prove


(a) Where a mistake or imperfection of the writing, or its
that the parties joined their efforts in furtherance of an
failure to express the true intent and agreement of the
enterprise for their joint profit, the question whether they
parties or the validity of the agreement is put in issue by the
intended by their agreement to create a joint adventure, or to
pleadings.
assume some other relation is a question of fact for the jury.
(Binder v. Kessler v 200 App. Div. 40,192 N Y S 653; Pyroa
(b) When there is an intrinsic ambiguity in the writing.

27
v. Brownfield (Tex. Civ. A.) 238 SW 725; Hoge v. George, 27 agreed to provide technology and know-how to Saniwares
Wyo, 423, 200 P 96 33 C.J. p. 871) and the latter paid royalties for the same. (At p. 2).

In the instant cases, our examination of important provisions of the xxx xxx xxx
Agreement as well as the testimonial evidence presented by the Lagdameo
and Young Group shows that the parties agreed to establish a joint venture It is pertinent to note that the provisions of the Agreement
and not a corporation. The history of the organization of Saniwares and the requiring a 7 out of 9 votes of the board of directors for
unusual arrangements which govern its policy making body are all consistent certain actions, in effect gave ASI (which designates 3
with a joint venture and not with an ordinary corporation. As stated by the directors under the Agreement) an effective veto power.
SEC: Furthermore, the grant to ASI of the right to designate certain
officers of the corporation; the super-majority voting
According to the unrebutted testimony of Mr. Baldwin Young, requirements for amendments of the articles and by-laws;
he negotiated the Agreement with ASI in behalf of the and most significantly to the issues of tms case, the
Philippine nationals. He testified that ASI agreed to accept provision that ASI shall designate 3 out of the 9 directors and
the role of minority vis-a-vis the Philippine National group of the other stockholders shall designate the other 6, clearly
investors, on the condition that the Agreement should indicate that there are two distinct groups in Saniwares,
contain provisions to protect ASI as the minority. namely ASI, which owns 40% of the capital stock and the
Philippine National stockholders who own the balance of
An examination of the Agreement shows that certain 60%, and that 2) ASI is given certain protections as the
provisions were included to protect the interests of ASI as minority stockholder.
the minority. For example, the vote of 7 out of 9 directors is
required in certain enumerated corporate acts [Sec. 3 (b) (ii) Premises considered, we believe that under the Agreement
(a) of the Agreement]. ASI is contractually entitled to there are two groups of stockholders who established a
designate a member of the Executive Committee and the corporation with provisions for a special contractual
vote of this member is required for certain transactions [Sec. relationship between the parties, i.e., ASI and the other
3 (b) (i)]. stockholders. (pp. 4-5)

The Agreement also requires a 75% super-majority vote for Section 5 (a) of the agreement uses the word "designated" and not
the amendment of the articles and by-laws of Saniwares "nominated" or "elected" in the selection of the nine directors on a six to three
[Sec. 3 (a) (iv) and (b) (iii)]. ASI is also given the right to ratio. Each group is assured of a fixed number of directors in the board.
designate the president and plant manager [Sec. 5 (6)]. The
Agreement further provides that the sales policy of Moreover, ASI in its communications referred to the enterprise as joint
Saniwares shall be that which is normally followed by ASI venture. Baldwin Young also testified that Section 16(c) of the Agreement
[Sec. 13 (a)] and that Saniwares should not export that "Nothing herein contained shall be construed to constitute any of the
"Standard" products otherwise than through ASI's Export parties hereto partners or joint venturers in respect of any transaction
Marketing Services [Sec. 13 (6)]. Under the Agreement, ASI

28
hereunder" was merely to obviate the possibility of the enterprise being determined in accordance with a procedure agreed upon by
treated as partnership for tax purposes and liabilities to third parties. them.

Quite often, Filipino entrepreneurs in their desire to develop the industrial and Appellants contend that the above provision is included in
manufacturing capacities of a local firm are constrained to seek the the Corporation Code's chapter on close corporations and
technology and marketing assistance of huge multinational corporations of Saniwares cannot be a close corporation because it has 95
the developed world. Arrangements are formalized where a foreign group stockholders. Firstly, although Saniwares had 95
becomes a minority owner of a firm in exchange for its manufacturing stockholders at the time of the disputed stockholders
expertise, use of its brand names, and other such assistance. However, there meeting, these 95 stockholders are not separate from each
is always a danger from such arrangements. The foreign group may, from the other but are divisible into groups representing a single
start, intend to establish its own sole or monopolistic operations and merely Identifiable interest. For example, ASI, its nominees and
uses the joint venture arrangement to gain a foothold or test the Philippine lawyers count for 13 of the 95 stockholders. The
waters, so to speak. Or the covetousness may come later. As the Philippine YoungYutivo family count for another 13 stockholders, the
firm enlarges its operations and becomes profitable, the foreign group Chamsay family for 8 stockholders, the Santos family for 9
undermines the local majority ownership and actively tries to completely or stockholders, the Dy family for 7 stockholders, etc. If the
predominantly take over the entire company. This undermining of joint members of one family and/or business or interest group are
ventures is not consistent with fair dealing to say the least. To the extent that considered as one (which, it is respectfully submitted, they
such subversive actions can be lawfully prevented, the courts should extend should be for purposes of determining how closely held
protection especially in industries where constitutional and legal Saniwares is there were as of 8 March 1983, practically only
requirements reserve controlling ownership to Filipino citizens. 17 stockholders of Saniwares. (Please refer to discussion in
pp. 5 to 6 of appellees' Rejoinder Memorandum dated 11
The Lagdameo Group stated in their appellees' brief in the Court of Appeal December 1984 and Annex "A" thereof).

In fact, the Philippine Corporation Code itself recognizes the Secondly, even assuming that Saniwares is technically not a
right of stockholders to enter into agreements regarding the close corporation because it has more than 20 stockholders,
exercise of their voting rights. the undeniable fact is that it is a close-held corporation.
Surely, appellants cannot honestly claim that Saniwares is a
Sec. 100. Agreements by stockholders.- public issue or a widely held corporation.

xxx xxx xxx In the United States, many courts have taken a realistic
approach to joint venture corporations and have not rigidly
applied principles of corporation law designed primarily for
2. An agreement between two or more stockholders, if in
public issue corporations. These courts have indicated that
writing and signed by the parties thereto, may provide that in
express arrangements between corporate joint ventures
exercising any voting rights, the shares held by them shall
should be construed with less emphasis on the ordinary
be voted as therein provided, or as they may agree, or as
rules of law usually applied to corporate entities and with
more consideration given to the nature of the agreement

29
between the joint venturers (Please see Wabash Ry v. Paragraph 2 refers to pooling and voting agreements in
American Refrigerator Transit Co., 7 F 2d 335; Chicago, M & particular. Does this provision necessarily imply that these
St. P. Ry v. Des Moines Union Ry; 254 Ass'n. 247 US. 490'; agreements can be valid only in close corporations as
Seaboard Airline Ry v. Atlantic Coast Line Ry; 240 N.C. defined by the Code? Suppose that a corporation has twenty
495,.82 S.E. 2d 771; Deboy v. Harris, 207 Md., 212,113 A 2d five stockholders, and therefore cannot qualify as a close
903; Hathway v. Porter Royalty Pool, Inc., 296 Mich. 90, 90, corporation under section 96, can some of them enter into
295 N.W. 571; Beardsley v. Beardsley, 138 U.S. 262; "The an agreement to vote as a unit in the election of directors? It
Legal Status of Joint Venture Corporations", 11 Vand Law is submitted that there is no reason for denying stockholders
Rev. p. 680,1958). These American cases dealt with legal of corporations other than close ones the right to enter into
questions as to the extent to which the requirements arising not voting or pooling agreements to protect their interests, as
from the corporate form of joint venture corporations should long as they do not intend to commit any wrong, or fraud on
control, and the courts ruled that substantial justice lay with the other stockholders not parties to the agreement. Of
those litigants who relied on the joint venture agreement course, voting or pooling agreements are perhaps more
rather than the litigants who relied on the orthodox principles useful and more often resorted to in close corporations. But
of corporation law. they may also be found necessary even in widely held
corporations. Moreover, since the Code limits the legal
As correctly held by the SEC Hearing Officer: meaning of close corporations to those which comply with
the requisites laid down by section 96, it is entirely possible
It is said that participants in a joint venture, in organizing the that a corporation which is in fact a close corporation will not
joint venture deviate from the traditional pattern of come within the definition. In such case, its stockholders
corporation management. A noted authority has pointed out should not be precluded from entering into contracts like
that just as in close corporations, shareholders' agreements voting agreements if these are otherwise valid. (Campos &
in joint venture corporations often contain provisions which Lopez-Campos, op cit, p. 405)
do one or more of the following: (1) require greater than
majority vote for shareholder and director action; (2) give In short, even assuming that sec. 5(a) of the Agreement
certain shareholders or groups of shareholders power to relating to the designation or nomination of directors restricts
select a specified number of directors; (3) give to the the right of the Agreement's signatories to vote for directors,
shareholders control over the selection and retention of such contractual provision, as correctly held by the SEC, is
employees; and (4) set up a procedure for the settlement of valid and binding upon the signatories thereto, which include
disputes by arbitration (See I O' Neal, Close Corporations, appellants. (Rollo No. 75951, pp. 90-94)
1971 ed., Section 1.06a, pp. 15-16) (Decision of SEC
Hearing Officer, P. 16) In regard to the question as to whether or not the ASI group may vote their
additional equity during elections of Saniwares' board of directors, the Court
Thirdly paragraph 2 of Sec. 100 of the Corporation Code of Appeals correctly stated:
does not necessarily imply that agreements regarding the
exercise of voting rights are allowed only in close As in other joint venture companies, the extent of ASI's
corporations. As Campos and Lopez-Campos explain: participation in the management of the corporation is spelled

30
out in the Agreement. Section 5(a) hereof says that three of In our decision sought to be reconsidered, we opted to
the nine directors shall be designated by ASI and the uphold the second over the first. Upon further reflection, we
remaining six by the other stockholders, i.e., the Filipino feel that the proper and just solution to give due
stockholders. This allocation of board seats is obviously in consideration to both factors suggests itself quite clearly.
consonance with the minority position of ASI. This Court should recognize and uphold the division of the
stockholders into two groups, and at the same time uphold
Having entered into a well-defined contractual relationship, it the right of the stockholders within each group to cumulative
is imperative that the parties should honor and adhere to voting in the process of determining who the group's
their respective rights and obligations thereunder. Appellants nominees would be. In practical terms, as suggested by
seem to contend that any allocation of board seats, even in appellant Luciano E. Salazar himself, this means that if the
joint venture corporations, are null and void to the extent that Filipino stockholders cannot agree who their six nominees
such may interfere with the stockholder's rights to cumulative will be, a vote would have to be taken among the Filipino
voting as provided in Section 24 of the Corporation Code. stockholders only. During this voting, each Filipino
This Court should not be prepared to hold that any stockholder can cumulate his votes. ASI, however, should
agreement which curtails in any way cumulative voting not be allowed to interfere in the voting within the Filipino
should be struck down, even if such agreement has been group. Otherwise, ASI would be able to designate more than
freely entered into by experienced businessmen and do not the three directors it is allowed to designate under the
prejudice those who are not parties thereto. It may well be Agreement, and may even be able to get a majority of the
that it would be more cogent to hold, as the Securities and board seats, a result which is clearly contrary to the
Exchange Commission has held in the decision appealed contractual intent of the parties.
from, that cumulative voting rights may be voluntarily waived
by stockholders who enter into special relationships with Such a ruling will give effect to both the allocation of the
each other to pursue and implement specific purposes, as in board seats and the stockholder's right to cumulative voting.
joint venture relationships between foreign and local Moreover, this ruling will also give due consideration to the
stockholders, so long as such agreements do not adversely issue raised by the appellees on possible violation or
affect third parties. circumvention of the Anti-Dummy Law (Com. Act No. 108, as
amended) and the nationalization requirements of the
In any event, it is believed that we are not here called upon Constitution and the laws if ASI is allowed to nominate more
to make a general rule on this question. Rather, all that than three directors. (Rollo-75875, pp. 38-39)
needs to be done is to give life and effect to the particular
contractual rights and obligations which the parties have The ASI Group and petitioner Salazar, now reiterate their theory that the ASI
assumed for themselves. Group has the right to vote their additional equity pursuant to Section 24 of
the Corporation Code which gives the stockholders of a corporation the right
On the one hand, the clearly established minority position of to cumulate their votes in electing directors. Petitioner Salazar adds that this
ASI and the contractual allocation of board seats Cannot be right if granted to the ASI Group would not necessarily mean a violation of
disregarded. On the other hand, the rights of the the Anti-Dummy Act (Commonwealth Act 108, as amended). He cites section
stockholders to cumulative voting should also be protected. 2-a thereof which provides:

31
And provided finally that the election of aliens as members of distinction between these two business forms, and has held
the board of directors or governing body of corporations or that although a corporation cannot enter into a partnership
associations engaging in partially nationalized activities shall contract, it may however engage in a joint venture with
be allowed in proportion to their allowable participation or others. (At p. 12, Tuazon v. Bolanos, 95 Phil. 906 [1954])
share in the capital of such entities. (amendments introduced (Campos and Lopez-Campos Comments, Notes and
by Presidential Decree 715, section 1, promulgated May 28, Selected Cases, Corporation Code 1981)
1975)
Moreover, the usual rules as regards the construction and operations of
The ASI Group's argument is correct within the context of Section 24 of the contracts generally apply to a contract of joint venture. (O' Hara v. Harman 14
Corporation Code. The point of query, however, is whether or not that App. Dev. (167) 43 NYS 556).
provision is applicable to a joint venture with clearly defined agreements:
Bearing these principles in mind, the correct view would be that the
The legal concept of ajoint venture is of common law origin. resolution of the question of whether or not the ASI Group may vote their
It has no precise legal definition but it has been generally additional equity lies in the agreement of the parties.
understood to mean an organization formed for some
temporary purpose. (Gates v. Megargel, 266 Fed. 811 Necessarily, the appellate court was correct in upholding the agreement of
[1920]) It is in fact hardly distinguishable from the the parties as regards the allocation of director seats under Section 5 (a) of
partnership, since their elements are similar community of the "Agreement," and the right of each group of stockholders to cumulative
interest in the business, sharing of profits and losses, and a voting in the process of determining who the group's nominees would be
mutual right of control. Blackner v. Mc Dermott, 176 F. 2d. under Section 3 (a) (1) of the "Agreement." As pointed out by SEC, Section 5
498, [1949]; Carboneau v. Peterson, 95 P. 2d., 1043 [1939]; (a) of the Agreement relates to the manner of nominating the members of the
Buckley v. Chadwick, 45 Cal. 2d. 183, 288 P. 2d. 12 289 P. board of directors while Section 3 (a) (1) relates to the manner of voting for
2d. 242 [1955]). The main distinction cited by most opinions these nominees.
in common law jurisdictions is that the partnership
contemplates a general business with some degree of This is the proper interpretation of the Agreement of the parties as regards
continuity, while the joint venture is formed for the execution the election of members of the board of directors.
of a single transaction, and is thus of a temporary nature.
(Tufts v. Mann 116 Cal. App. 170, 2 P. 2d. 500 [1931];
To allow the ASI Group to vote their additional equity to help elect even a
Harmon v. Martin, 395 111. 595, 71 NE 2d. 74 [1947]; Gates
Filipino director who would be beholden to them would obliterate their
v. Megargel 266 Fed. 811 [1920]). This observation is not
minority status as agreed upon by the parties. As aptly stated by the
entirely accurate in this jurisdiction, since under the Civil
appellate court:
Code, a partnership may be particular or universal, and a
particular partnership may have for its object a specific
undertaking. (Art. 1783, Civil Code). It would seem therefore ... ASI, however, should not be allowed to interfere in the
that under Philippine law, a joint venture is a form of voting within the Filipino group. Otherwise, ASI would be
partnership and should thus be governed by the law of able to designate more than the three directors it is allowed
partnerships. The Supreme Court has however recognized a to designate under the Agreement, and may even be able to

32
get a majority of the board seats, a result which is clearly On the other hand, the Lagdameo and Young Group (petitioners in G.R. No.
contrary to the contractual intent of the parties. 75951) object to a cumulative voting during the election of the board of
directors of the enterprise as ruled by the appellate court and submits that
Such a ruling will give effect to both the allocation of the the six (6) directors allotted the Filipino stockholders should be selected by
board seats and the stockholder's right to cumulative voting. consensus pursuant to section 5 (a) of the Agreement which uses the word
Moreover, this ruling will also give due consideration to the "designate" meaning "nominate, delegate or appoint."
issue raised by the appellees on possible violation or
circumvention of the Anti-Dummy Law (Com. Act No. 108, as They also stress the possibility that the ASI Group might take control of the
amended) and the nationalization requirements of the enterprise if the Filipino stockholders are allowed to select their nominees
Constitution and the laws if ASI is allowed to nominate more separately and not as a common slot determined by the majority of their
than three directors. (At p. 39, Rollo, 75875) group.

Equally important as the consideration of the contractual intent of the parties Section 5 (a) of the Agreement which uses the word designates in the
is the consideration as regards the possible domination by the foreign allocation of board directors should not be interpreted in isolation. This
investors of the enterprise in violation of the nationalization requirements should be construed in relation to section 3 (a) (1) of the Agreement. As we
enshrined in the Constitution and circumvention of the Anti-Dummy Act. In stated earlier, section 3(a) (1) relates to the manner of voting for these
this regard, petitioner Salazar's position is that the Anti-Dummy Act allows nominees which is cumulative voting while section 5(a) relates to the manner
the ASI group to elect board directors in proportion to their share in the of nominating the members of the board of directors. The petitioners in G.R.
capital of the entity. It is to be noted, however, that the same law also limits No. 75951 agreed to this procedure, hence, they cannot now impugn its
the election of aliens as members of the board of directors in proportion to legality.
their allowance participation of said entity. In the instant case, the foreign
Group ASI was limited to designate three directors. This is the allowable The insinuation that the ASI Group may be able to control the enterprise
participation of the ASI Group. Hence, in future dealings, this limitation of six under the cumulative voting procedure cannot, however, be ignored. The
to three board seats should always be maintained as long as the joint validity of the cumulative voting procedure is dependent on the directors thus
venture agreement exists considering that in limiting 3 board seats in the 9- elected being genuine members of the Filipino group, not voters whose
man board of directors there are provisions already agreed upon and interest is to increase the ASI share in the management of Saniwares. The
embodied in the parties' Agreement to protect the interests arising from the joint venture character of the enterprise must always be taken into account,
minority status of the foreign investors. so long as the company exists under its original agreement. Cumulative
voting may not be used as a device to enable ASI to achieve stealthily or
With these findings, we the decisions of the SEC Hearing Officer and SEC indirectly what they cannot accomplish openly. There are substantial
which were impliedly affirmed by the appellate court declaring Messrs. safeguards in the Agreement which are intended to preserve the majority
Wolfgang Aurbach, John Griffin, David P Whittingham, Emesto V. Lagdameo, status of the Filipino investors as well as to maintain the minority status of the
Baldwin young, Raul A. Boncan, Emesto V. Lagdameo, Jr., Enrique foreign investors group as earlier discussed. They should be maintained.
Lagdameo, and George F. Lee as the duly elected directors of Saniwares at
the March 8,1983 annual stockholders' meeting. WHEREFORE, the petitions in G.R. Nos. 75975-76 and G.R. No. 75875 are
DISMISSED and the petition in G.R. No. 75951 is partly GRANTED. The

33
amended decision of the Court of Appeals is MODIFIED in that Messrs.
Wolfgang Aurbach John Griffin, David Whittingham Emesto V. Lagdameo,
Baldwin Young, Raul A. Boncan, Ernesto R. Lagdameo, Jr., Enrique
Lagdameo, and George F. Lee are declared as the duly elected directors of
Saniwares at the March 8,1983 annual stockholders' meeting. In all other
respects, the questioned decision is AFFIRMED. Costs against the
petitioners in G.R. Nos. 75975-76 and G.R. No. 75875.

SO ORDERED.

Fernan, C.J., (Chairman), Bidin and Cortes, JJ., concur.

Feliciano, J., took no part.

34
understandable, and interpretative American jurisprudence is found in
abundance, yet the issue submitted is not lightly to be resolved. The
question, flatly presented, is, whether Act. No. 2761 of the Philippine
Legislature is valid or, more directly stated, whether the Government of
the Philippine Islands, through its Legislature, can deny the registry of
vessels in its coastwise trade to corporations having alien stockholders.

FACTS.

Smith, Bell & Co., (Ltd.), is a corporation organized and existing under the
laws of the Philippine Islands. A majority of its stockholders are British
subjects. It is the owner of a motor vessel known as the Bato built for it in the
Philippine Islands in 1916, of more than fifteen tons gross The Bato was
brought to Cebu in the present year for the purpose of transporting plaintiff's
merchandise between ports in the Islands. Application was made at Cebu,
the home port of the vessel, to the Collector of Customs for a certificate of
Philippine registry. The Collector refused to issue the certificate, giving as his
G.R. No. 15574 September 17, 1919 reason that all the stockholders of Smith, Bell & Co., Ltd., were not citizens
either of the United States or of the Philippine Islands. The instant action is
SMITH, BELL & COMPANY (LTD.), petitioner, the result.
vs.
JOAQUIN NATIVIDAD, Collector of Customs of the port of LAW.
Cebu, respondent.
The Act of Congress of April 29, 1908, repealing the Shipping Act of April 30,
Ross and Lawrence for petitioner. 1906 but reenacting a portion of section 3 of this Law, and still in force,
Attorney-General Paredes for respondent. provides in its section 1:

MALCOLM, J.: That until Congress shall have authorized the registry as vessels of
the United States of vessels owned in the Philippine Islands, the
A writ of mandamus is prayed for by Smith, Bell & Co. (Ltd.), against Joaquin Government of the Philippine Islands is hereby authorized to adopt,
Natividad, Collector of Customs of the port of Cebu, Philippine Islands, to from time to time, and enforce regulations governing the
compel him to issue a certificate of Philippine registry to the petitioner for its transportation of merchandise and passengers between ports or
motor vessel Bato. The Attorney-General, acting as counsel for respondent, places in the Philippine Archipelago. (35 Stat. at L., 70; Section
demurs to the petition on the general ground that it does not state facts 3912, U. S. Comp Stat. [1916]; 7 Pub. Laws, 364.)
sufficient to constitute a cause of action. While the facts are thus admitted,
and while, moreover, the pertinent provisions of law are clear and

35
The Act of Congress of August 29, 1916, commonly known as the Jones coinage laws of the Philippines become a law until it has been
Law, still in force, provides in section 3, (first paragraph, first sentence), 6, 7, approved by the President of the United States: Provided
8, 10, and 31, as follows. further, That the President shall approve or disapprove any act
mentioned in the foregoing proviso within six months from and after
SEC. 3. That no law shall be enacted in said Islands which shall its enactment and submission for his approval, and if not
deprive any person of life, liberty, or property without due process of disapproved within such time it shall become a law the same as if it
law, or deny to any person therein the equal protection of the laws. . . had been specifically approved.
.
SEC. 31. That all laws or parts of laws applicable to the Philippines
SEC. 6. That the laws now in force in the Philippines shall continue in not in conflict with any of the provisions of this Act are hereby
force and effect, except as altered, amended, or modified herein, continued in force and effect." (39 Stat at L., 546.)
until altered, amended, or repealed by the legislative authority herein
provided or by Act of Congress of the United States. On February 23, 1918, the Philippine Legislature enacted Act No. 2761. The
first section of this law amended section 1172 of the Administrative Code to
SEC. 7. That the legislative authority herein provided shall have read as follows:
power, when not inconsistent with this Act, by due enactment to
amend, alter modify, or repeal any law, civil or criminal, continued in SEC. 1172. Certificate of Philippine register. Upon registration of a
force by this Act as it may from time to time see fit vessel of domestic ownership, and of more than fifteen tons gross, a
certificate of Philippine register shall be issued for it. If the vessel is
This power shall specifically extend with the limitation herein of domestic ownership and of fifteen tons gross or less, the taking of
provided as to the tariff to all laws relating to revenue provided as to the certificate of Philippine register shall be optional with the owner.
the tariff to all laws relating to revenue and taxation in effect in the
Philippines. "Domestic ownership," as used in this section, means ownership
vested in some one or more of the following classes of persons: (a)
SEC. 8. That general legislative power, except as otherwise herein Citizens or native inhabitants of the Philippine Islands; (b) citizens of
provided, is hereby granted to the Philippine Legislature, authorized the United States residing in the Philippine Islands; (c) any
by this Act. corporation or company composed wholly of citizens of the
Philippine Islands or of the United States or of both, created under
SEC. 10. That while this Act provides that the Philippine government the laws of the United States, or of any State thereof, or of thereof, or
shall have the authority to enact a tariff law the trade relations the managing agent or master of the vessel resides in the Philippine
between the islands and the United States shall continue to be Islands
governed exclusively by laws of the Congress of the United
States: Provided, That tariff acts or acts amendatory to the tariff of Any vessel of more than fifteen gross tons which on February eighth,
the Philippine Islands shall not become law until they shall receive nineteen hundred and eighteen, had a certificate of Philippine
the approval of the President of the United States, nor shall any act register under existing law, shall likewise be deemed a vessel of
of the Philippine Legislature affecting immigration or the currency or domestic ownership so long as there shall not be any change in the

36
ownership thereof nor any transfer of stock of the companies or whole or in part whether the Government of the Philippine Islands, through
corporations owning such vessel to person not included under the its Legislature, can deny the registry of vessel in its coastwise trade to
last preceding paragraph. corporations having alien stockholders .

Sections 2 and 3 of Act No. 2761 amended sections 1176 and 1202 of the OPINION.
Administrative Code to read as follows:
1. Considered from a positive standpoint, there can exist no measure of
SEC. 1176. Investigation into character of vessel. No application doubt as to the power of the Philippine Legislature to enact Act No. 2761.
for a certificate of Philippine register shall be approved until the The Act of Congress of April 29, 1908, with its specific delegation of authority
collector of customs is satisfied from an inspection of the vessel that to the Government of the Philippine Islands to regulate the transportation of
it is engaged or destined to be engaged in legitimate trade and that it merchandise and passengers between ports or places therein, the liberal
is of domestic ownership as such ownership is defined in section construction given to the provisions of the Philippine Bill, the Act of Congress
eleven hundred and seventy-two of this Code. of July 1, 1902, by the courts, and the grant by the Act of Congress of August
29, 1916, of general legislative power to the Philippine Legislature, are
The collector of customs may at any time inspect a vessel or certainly superabundant authority for such a law. While the Act of the local
examine its owner, master, crew, or passengers in order to ascertain legislature may in a way be inconsistent with the Act of Congress regulating
whether the vessel is engaged in legitimate trade and is entitled to the coasting trade of the Continental United States, yet the general rule that
have or retain the certificate of Philippine register. only such laws of the United States have force in the Philippines as are
expressly extended thereto, and the abnegation of power by Congress in
SEC. 1202. Limiting number of foreign officers and engineers on favor of the Philippine Islands would leave no starting point for convincing
board vessels. No Philippine vessel operating in the coastwise argument. As a matter of fact, counsel for petitioner does not assail
trade or on the high seas shall be permitted to have on board more legislative action from this direction (See U. S. vs. Bull [1910], 15 Phil., 7;
than one master or one mate and one engineer who are not citizens Sinnot vs. Davenport [1859] 22 How., 227.)
of the United States or of the Philippine Islands, even if they hold
licenses under section one thousand one hundred and ninety-nine 2. It is from the negative, prohibitory standpoint that counsel argues against
hereof. No other person who is not a citizen of the United States or the constitutionality of Act No. 2761. The first paragraph of the Philippine Bill
of the Philippine Islands shall be an officer or a member of the crew of Rights of the Philippine Bill, repeated again in the first paragraph of the
of such vessel. Any such vessel which fails to comply with the terms Philippine Bill of Rights as set forth in the Jones Law, provides "That no law
of this section shall be required to pay an additional tonnage tax of shall be enacted in said Islands which shall deprive any person of life, liberty,
fifty centavos per net ton per month during the continuance of said or property without due process of law, or deny to any person therein the
failure. equal protection of the laws." Counsel says that Act No. 2761 denies to
Smith, Bell & Co., Ltd., the equal protection of the laws because it, in effect,
ISSUES. prohibits the corporation from owning vessels, and because classification of
corporations based on the citizenship of one or more of their stockholders is
capricious, and that Act No. 2761 deprives the corporation of its properly
Predicated on these facts and provisions of law, the issues as above stated
without due process of law because by the passage of the law company was
recur, namely, whether Act No 2761 of the Philippine Legislature is valid in

37
automatically deprived of every beneficial attribute of ownership in Fraser vs. McConway & Torley Co. [1897], 82 Fed , 257; Juniata Limestone
the Bato and left with the naked title to a boat it could not use . Co. vs. Fagley [1898], 187 Penn., 193, all relating to the employment of
aliens by private corporations.)
The guaranties extended by the Congress of the United States to the
Philippine Islands have been used in the same sense as like provisions A literal application of general principles to the facts before us would, of
found in the United States Constitution. While the "due process of law and course, cause the inevitable deduction that Act No. 2761 is unconstitutional
equal protection of the laws" clause of the Philippine Bill of Rights is couched by reason of its denial to a corporation, some of whole members are
in slightly different words than the corresponding clause of the Fourteenth foreigners, of the equal protection of the laws. Like all beneficient
Amendment to the United States Constitution, the first should be interpreted propositions, deeper research discloses provisos. Examples of a denial of
and given the same force and effect as the latter. (Kepner vs. U.S. [1904], rights to aliens notwithstanding the provisions of the Fourteenth Amendment
195 U. S., 100; Sierra vs. Mortiga [1907], 204 U. S.,.470; U. S. vs. Bull could be cited. (Tragesser vs. Gray [1890], 73 Md., 250, licenses to sell
[1910], 15 Phil., 7.) The meaning of the Fourteenth Amendment has been spirituous liquors denied to persons not citizens of the United States;
announced in classic decisions of the United States Supreme Court. Even at Commonwealth vs. Hana [1907], 195 Mass , 262, excluding aliens from the
the expense of restating what is so well known, these basic principles must right to peddle; Patsone vs. Commonwealth of Pennsylvania [1914], 232 U.
again be set down in order to serve as the basis of this decision. S. , 138, prohibiting the killing of any wild bird or animal by any unnaturalized
foreign-born resident; Ex parte Gilleti [1915], 70 Fla., 442, discriminating in
The guaranties of the Fourteenth Amendment and so of the first paragraph of favor of citizens with reference to the taking for private use of the common
the Philippine Bill of Rights, are universal in their application to all person property in fish and oysters found in the public waters of the State;
within the territorial jurisdiction, without regard to any differences of race, Heim vs. McCall [1915], 239 U. S.,.175, and Crane vs. New York [1915], 239
color, or nationality. The word "person" includes aliens. (Yick Wo vs. Hopkins U. S., 195, limiting employment on public works by, or for, the State or a
[1886], 118 U. S., 356; Truax vs. Raich [1915], 239 U. S., 33.) Private municipality to citizens of the United States.)
corporations, likewise, are "persons" within the scope of the guaranties in so
far as their property is concerned. (Santa Clara County vs. Southern Pac. R. One of the exceptions to the general rule, most persistent and far reaching in
R. Co. [1886], 118.U. S., 394; Pembina Mining Co. vs. Pennsylvania influence is, that neither the Fourteenth Amendment to the United States
[1888],.125 U. S., 181 Covington & L. Turnpike Road Co. vs. Sandford Constitution, broad and comprehensive as it is, nor any other amendment,
[1896], 164 U. S., 578.) Classification with the end in view of providing "was designed to interfere with the power of the State, sometimes termed its
diversity of treatment may be made among corporations, but must be based `police power,' to prescribe regulations to promote the health, peace, morals,
upon some reasonable ground and not be a mere arbitrary selection (Gulf, education, and good order of the people, and legislate so as to increase the
Colorado & Santa Fe Railway Co. vs. Ellis [1897],.165 U. S., 150.) Examples industries of the State, develop its resources and add to its wealth and
of laws held unconstitutional because of unlawful discrimination against prosperity. From the very necessities of society, legislation of a special
aliens could be cited. Generally, these decisions relate to statutes which had character, having these objects in view, must often be had in certain
attempted arbitrarily to forbid aliens to engage in ordinary kinds of business districts." (Barbier vs. Connolly [1884], 113 U.S., 27; New Orleans Gas
to earn their living. (State vs. Montgomery [1900], 94 Maine, 192, peddling Co. vs. Lousiana Light Co. [1885], 115 U.S., 650.) This is the same police
but see. Commonwealth vs. Hana [1907], 195 Mass., 262; Templar vs. Board power which the United States Supreme Court say "extends to so dealing
of Examiners of Barbers [1902], 131 Mich., 254, barbers; Yick with the conditions which exist in the state as to bring out of them the
Wo vs. Hopkins [1886], 118 U. S.,.356, discrimination against Chinese; greatest welfare in of its people." (Bacon vs. Walker [1907], 204 U.S., 311.)
Truax vs. Raich [1915], 239 U. S., 33; In re Parrott [1880], 1 Fed , 481; For quite similar reasons, none of the provision of the Philippine Organic Law

38
could could have had the effect of denying to the Government of the undesirable foreigners. The Government has thus assumed to act for the all-
Philippine Islands, acting through its Legislature, the right to exercise that sufficient and primitive reason of the benefit and protection of its own citizens
most essential, insistent, and illimitable of powers, the sovereign police and of the self-preservation and integrity of its dominion. (In re Patterson
power, in the promotion of the general welfare and the public interest. (U. [1902], 1 Phil., 93; Forbes vs. Chuoco, Tiaco and Crossfield [1910], 16 Phil.,
S. vs. Toribio [1910], 15 Phil., 85; Churchill and Tait vs. Rafferty [1915], 32 534;.228 U.S., 549; In re McCulloch Dick [1918], 38 Phil., 41.) Boats owned
Phil., 580; Rubi vs. Provincial Board of Mindoro [1919], 39 Phil., 660.) by foreigners, particularly by such solid and reputable firms as the instant
Another notable exception permits of the regulation or distribution of the claimant, might indeed traverse the waters of the Philippines for ages without
public domain or the common property or resources of the people of the doing any particular harm. Again, some evilminded foreigner might very
State, so that use may be limited to its citizens. (Ex parte Gilleti [1915], 70 easily take advantage of such lavish hospitality to chart Philippine waters, to
Fla., 442; McCready vs. Virginia [1876], 94 U. S., 391; obtain valuable information for unfriendly foreign powers, to stir up
Patsone vs. Commonwealth of Pennsylvania [1914], 232U. S., 138.) Still insurrection, or to prejudice Filipino or American commerce. Moreover, under
another exception permits of the limitation of employment in the construction the Spanish portion of Philippine law, the waters within the domestic
of public works by, or for, the State or a municipality to citizens of the United jurisdiction are deemed part of the national domain, open to public use.
States or of the State. (Atkin vs. Kansas [1903],191 U. S., 207; (Book II, Tit. IV, Ch. I, Civil Code; Spanish Law of Waters of August 3, 1866,
Heim vs. McCall [1915], 239 U.S., 175; Crane vs. New York [1915], 239 U. arts 1, 2, 3.) Common carriers which in the Philippines as in the United
S., 195.) Even as to classification, it is admitted that a State may classify with States and other countries are, as Lord Hale said, "affected with a public
reference to the evil to be prevented; the question is a practical one, interest," can only be permitted to use these public waters as a privilege and
dependent upon experience. (Patsone vs. Commonwealth of Pennsylvania under such conditions as to the representatives of the people may seem
[1914], 232 U. S., 138.) wise. (See De Villata vs. Stanley [1915], 32 Phil., 541.)

To justify that portion of Act no. 2761 which permits corporations or In Patsone vs. Commonwealth of Pennsylvania ([1913], 232 U.S., 138), a
companies to obtain a certificate of Philippine registry only on condition that case herein before mentioned, Justice Holmes delivering the opinion of the
they be composed wholly of citizens of the Philippine Islands or of the United United States Supreme Court said:
States or both, as not infringing Philippine Organic Law, it must be done
under some one of the exceptions here mentioned This must be done, This statute makes it unlawful for any unnaturalized foreign-born
moreover, having particularly in mind what is so often of controlling effect in resident to kill any wild bird or animal except in defense of person or
this jurisdiction our local experience and our peculiar local conditions. property, and `to that end' makes it unlawful for such foreign-born
person to own or be possessed of a shotgun or rifle; with a penalty of
To recall a few facts in geography, within the confines of Philippine $25 and a forfeiture of the gun or guns. The plaintiff in error was
jurisdictional limits are found more than three thousand islands. Literally, and found guilty and was sentenced to pay the abovementioned fine. The
absolutely, steamship lines are, for an Insular territory thus situated, the judgment was affirmed on successive appeals. (231 Pa., 46; 79 Atl.,
arteries of commerce. If one be severed, the life-blood of the nation is lost. If 928.) He brings the case to this court on the ground that the statute
on the other hand these arteries are protected, then the security of the is contrary to the 14th Amendment and also is in contravention of the
country and the promotion of the general welfare is sustained. Time and treaty between the United States and Italy, to which latter country the
again, with such conditions confronting it, has the executive branch of the plaintiff in error belongs .
Government of the Philippine Islands, always later with the sanction of the
judicial branch, taken a firm stand with reference to the presence of

39
Under the 14th Amendment the objection is twofold; unjustifiably corporations such as Smith, Bell &. Co. Ltd., the right to register vessels in
depriving the alien of property, and discrimination against such aliens the Philippines coastwise trade, does not belong to that vicious species of
as a class. But the former really depends upon the latter, since it class legislation which must always be condemned, but does fall within
hardly can be disputed that if the lawful object, the protection of wild authorized exceptions, notably, within the purview of the police power, and so
life (Geer vs. Connecticut, 161 U.S., 519; 40 L. ed., 793; 16 Sup. Ct. does not offend against the constitutional provision.
Rep., 600), warrants the discrimination, the, means adopted for
making it effective also might be adopted. . . . This opinion might well be brought to a close at this point. It occurs to us,
however, that the legislative history of the United States and the Philippine
The discrimination undoubtedly presents a more difficult question. Islands, and, probably, the legislative history of other countries, if we were to
But we start with reference to the evil to be prevented, and that if the take the time to search it out, might disclose similar attempts at restriction on
class discriminated against is or reasonably might be considered to the right to enter the coastwise trade, and might thus furnish valuable aid by
define those from whom the evil mainly is to be feared, it properly which to ascertain and, if possible, effectuate legislative intention.
may be picked out. A lack of abstract symmetry does not matter. The
question is a practical one, dependent upon experience. . . . 3. The power to regulate commerce, expressly delegated to the
Congress by the Constitution, includes the power to nationalize ships
The question therefore narrows itself to whether this court can say built and owned in the United States by registries and enrollments,
that the legislature of Pennsylvania was not warranted in assuming and the recording of the muniments of title of American vessels. The
as its premise for the law that resident unnaturalized aliens were the Congress "may encourage or it may entirely prohibit such commerce,
peculiar source of the evil that it desired to prevent. and it may regulate in any way it may see fit between these two
(Barrett vs. Indiana,. 229 U.S., 26, 29; 57 L. ed., 1050, 1052; 33 Sup. extremes." (U.S. vs. Craig [1886], 28 Fed., 795; Gibbons vs. Ogden
Ct. Rep., 692.) [1824], 9 Wheat., 1; The Passenger Cases [1849], 7 How., 283.)

Obviously the question, so stated, is one of local experience, on Acting within the purview of such power, the first Congress of the United
which this court ought to be very slow to declare that the state States had not been long convened before it enacted on September 1, 1789,
legislature was wrong in its facts (Adams vs. Milwaukee, 228 U.S., "An Act for Registering and Clearing Vessels, Regulating the Coasting Trade,
572, 583; 57 L. ed., 971,.977; 33 Sup. Ct. Rep., 610.) If we might and for other purposes." Section 1 of this law provided that for any ship or
trust popular speech in some states it was right; but it is enough that vessel to obtain the benefits of American registry, it must belong wholly to a
this court has no such knowledge of local conditions as to be able to citizen or citizens of the United States "and no other." (1 Stat. at L., 55.) That
say that it was manifestly wrong. . . . Act was shortly after repealed, but the same idea was carried into the Acts of
Congress of December 31, 1792 and February 18, 1793. (1 Stat. at L., 287,
Judgment affirmed. 305.).Section 4 of the Act of 1792 provided that in order to obtain the registry
of any vessel, an oath shall be taken and subscribed by the owner, or by one
We are inclined to the view that while Smith, Bell & Co. Ltd., a corporation of the owners thereof, before the officer authorized to make such registry,
having alien stockholders, is entitled to the protection afforded by the due- declaring, "that there is no subject or citizen of any foreign prince or state,
process of law and equal protection of the laws clause of the Philippine Bill of directly or indirectly, by way of trust, confidence, or otherwise, interested in
Rights, nevertheless, Act No. 2761 of the Philippine Legislature, in denying to such vessel, or in the profits or issues thereof." Section 32 of the Act of 1793
even went so far as to say "that if any licensed ship or vessel shall be

40
transferred to any person who is not at the time of such transfer a citizen of of the Philippine Islands (Adm. Code of 1916, sec. 1345; Adm. Code of 1917,
and resident within the United States, ... every such vessel with her tackle, sec. 1172). And now Act No. 2761 has returned to the restrictive idea of the
apparel, and furniture, and the cargo found on board her, shall be forefeited." original Customs Administrative Act which in turn was merely a reflection of
In case of alienation to a foreigner, Chief Justice Marshall said that all the the statutory language of the first American Congress.
privileges of an American bottom were ipso facto forfeited. (U.S. vs. Willings
and Francis [1807], 4 Cranch, 48.) Even as late as 1873, the Attorney- Provisions such as those in Act No. 2761, which deny to foreigners the right
General of the United States was of the opinion that under the provisions of to a certificate of Philippine registry, are thus found not to be as radical as a
the Act of December 31, 1792, no vessel in which a foreigner is directly or first reading would make them appear.
indirectly interested can lawfully be registered as a vessel of the United.
States. (14 Op. Atty.-Gen. [U.S.], 340.) Without any subterfuge, the apparent purpose of the Philippine Legislature is
seen to be to enact an anti-alien shipping act. The ultimate purpose of the
These laws continued in force without contest, although possibly the Act of Legislature is to encourage Philippine ship-building. This, without doubt, has,
March 3, 1825, may have affected them, until amended by the Act of May 28, likewise, been the intention of the United States Congress in passing
1896 (29 Stat. at L., 188) which extended the privileges of registry from navigation or tariff laws on different occasions. The object of such a law, the
vessels wholly owned by a citizen or citizens of the United States to United States Supreme Court once said, was to encourage American trade,
corporations created under the laws of any of the states thereof. The law, as navigation, and ship-building by giving American ship-owners exclusive
amended, made possible the deduction that a vessel belonging to a domestic privileges. (Old Dominion Steamship Co. vs. Virginia [1905], 198 U.S., 299;
corporation was entitled to registry or enrollment even though some stock of Kent's Commentaries, Vol. 3, p. 139.)
the company be owned by aliens. The right of ownership of stock in a
corporation was thereafter distinct from the right to hold the property by the In the concurring opinion of Justice Johnson in Gibbons vs. Ogden ([1824], 9
corporation (Humphreys vs. McKissock [1890], 140 U.S., 304; Wheat., 1) is found the following:
Queen vs. Arnaud [1846], 9 Q. B., 806; 29 Op. Atty.-Gen. [U.S.],188.)
Licensing acts, in fact, in legislation, are universally restraining acts;
On American occupation of the Philippines, the new government found a as, for example, acts licensing gaming houses, retailers of spirituous
substantive law in operation in the Islands with a civil law history which it liquors, etc. The act, in this instance, is distinctly of that character,
wisely continued in force Article fifteen of the Spanish Code of Commerce and forms part of an extensive system, the object of which is to
permitted any foreigner to engage in Philippine trade if he had legal capacity encourage American shipping, and place them on an equal footing
to do so under the laws of his nation. When the Philippine Commission came with the shipping of other nations. Almost every commercial nation
to enact the Customs Administrative Act (No. 355) in 1902, it returned to the reserves to its own subjects a monopoly of its coasting trade; and a
old American policy of limiting the protection and flag of the United States to countervailing privilege in favor of American shipping is
vessels owned by citizens of the United States or by native inhabitants of the contemplated, in the whole legislation of the United States on this
Philippine Islands (Sec. 117.) Two years later, the same body reverted to the subject. It is not to give the vessel an American character, that the
existing Congressional law by permitting certification to be issued to a citizen license is granted; that effect has been correctly attributed to the act
of the United States or to a corporation or company created under the laws of of her enrollment. But it is to confer on her American privileges, as
the United States or of any state thereof or of the Philippine Islands (Act No. contradistinguished from foreign; and to preserve the. Government
1235, sec. 3.) The two administration codes repeated the same provisions from fraud by foreigners, in surreptitiously intruding themselves into
with the necessary amplification of inclusion of citizens or native inhabitants

41
the American commercial marine, as well as frauds upon the purposes of obtaining a certificate of Philippine registry in the coastwise trade
revenue in the trade coastwise, that this whole system is projected. to citizens of the Philippine Islands, and to citizens of the United States, does
not violate the provisions of paragraph 1 of section 3 of the Act of Congress
The United States Congress in assuming its grave responsibility of legislating of August 29, 1916 No treaty right relied upon Act No. 2761 of the Philippine
wisely for a new country did so imbued with a spirit of Americanism. Legislature is held valid and constitutional .
Domestic navigation and trade, it decreed, could only be carried on by
citizens of the United States. If the representatives of the American people The petition for a writ of mandamus is denied, with costs against the
acted in this patriotic manner to advance the national policy, and if their petitioner. So ordered.
action was accepted without protest in the courts, who can say that they did
not enact such beneficial laws under the all-pervading police power, with the Arellano, C.J., Torres, Johnson, Araullo, Street, Avancea and Moir,
prime motive of safeguarding the country and of promoting its prosperity? JJ., concur.
Quite similarly, the Philippine Legislature made up entirely of Filipinos,
representing the mandate of the Filipino people and the guardian of their
rights, acting under practically autonomous powers, and imbued with a
strong sense of Philippinism, has desired for these Islands safety from
foreign interlopers, the use of the common property exclusively by its citizens
and the citizens of the United States, and protection for the common good of
the people. Who can say, therefore, especially can a court, that with all the
facts and circumstances affecting the Filipino people before it, the Philippine
Legislature has erred in the enactment of Act No. 2761?

Surely, the members of the judiciary are not expected to live apart from
active life, in monastic seclusion amidst dusty tomes and ancient records,
but, as keen spectators of passing events and alive to the dictates of the
general the national welfare, can incline the scales of their decisions in
favor of that solution which will most effectively promote the public policy. All
the presumption is in favor of the constitutionally of the law and without good
and strong reasons, courts should not attempt to nullify the action of the
Legislature. "In construing a statute enacted by the Philippine Commission
(Legislature), we deem it our duty not to give it a construction which would be
repugnant to an Act of Congress, if the language of the statute is fairly
susceptible of another construction not in conflict with the higher law." (In
re Guaria [1913], 24. Phil., 36; U.S. vs. Ten Yu [1912], 24 Phil., 1.) That is
the true construction which will best carry legislative intention into effect.
G.R. No. L-19550 June 19, 1967
With full consciousness of the importance of the question, we nevertheless
are clearly of the opinion that the limitation of domestic ownership for

42
HARRY S. STONEHILL, ROBERT P. BROOKS, JOHN J. BROOKS and as "the subject of the offense; stolen or embezzled and proceeds or fruits of
KARL BECK, petitioners, the offense," or "used or intended to be used as the means of committing the
vs. offense," which is described in the applications adverted to above as
HON. JOSE W. DIOKNO, in his capacity as SECRETARY OF JUSTICE; "violation of Central Bank Laws, Tariff and Customs Laws, Internal Revenue
JOSE LUKBAN, in his capacity as Acting Director, National Bureau of (Code) and the Revised Penal Code."
Investigation; SPECIAL PROSECUTORS PEDRO D. CENZON, EFREN I.
PLANA and MANUEL VILLAREAL, JR. and ASST. FISCAL MANASES G. Alleging that the aforementioned search warrants are null and void, as
REYES; JUDGE AMADO ROAN, Municipal Court of Manila; JUDGE contravening the Constitution and the Rules of Court because, inter alia:
ROMAN CANSINO, Municipal Court of Manila; JUDGE HERMOGENES (1) they do not describe with particularity the documents, books and things to
CALUAG, Court of First Instance of Rizal-Quezon City Branch, and be seized; (2) cash money, not mentioned in the warrants, were actually
JUDGE DAMIAN JIMENEZ, Municipal Court of Quezon City, respondents. seized; (3) the warrants were issued to fish evidence against the
aforementioned petitioners in deportation cases filed against them; (4) the
Paredes, Poblador, Cruz and Nazareno and Meer, Meer and Meer and Juan searches and seizures were made in an illegal manner; and (5) the
T. David for petitioners. documents, papers and cash money seized were not delivered to the courts
Office of the Solicitor General Arturo A. Alafriz, Assistant Solicitor General that issued the warrants, to be disposed of in accordance with law on
Pacifico P. de Castro, Assistant Solicitor General Frine C. Zaballero, Solicitor March 20, 1962, said petitioners filed with the Supreme Court this original
Camilo D. Quiason and Solicitor C. Padua for respondents. action for certiorari, prohibition, mandamus and injunction, and prayed that,
pending final disposition of the present case, a writ of preliminary injunction
CONCEPCION, C.J.: be issued restraining Respondents-Prosecutors, their agents and /or
representatives from using the effects seized as aforementioned or any
Upon application of the officers of the government named on the margin 1 copies thereof, in the deportation cases already adverted to, and that, in due
hereinafter referred to as Respondents-Prosecutors several judges 2 course, thereafter, decision be rendered quashing the contested search
hereinafter referred to as Respondents-Judges issued, on different warrants and declaring the same null and void, and commanding the
dates,3 a total of 42 search warrants against petitioners herein 4 and/or the respondents, their agents or representatives to return to petitioners herein, in
corporations of which they were officers,5 directed to the any peace officer, to accordance with Section 3, Rule 67, of the Rules of Court, the documents,
search the persons above-named and/or the premises of their offices, papers, things and cash moneys seized or confiscated under the search
warehouses and/or residences, and to seize and take possession of the warrants in question.
following personal property to wit:
In their answer, respondents-prosecutors alleged, 6 (1) that the contested
Books of accounts, financial records, vouchers, correspondence, search warrants are valid and have been issued in accordance with law; (2)
receipts, ledgers, journals, portfolios, credit journals, typewriters, and that the defects of said warrants, if any, were cured by petitioners' consent;
other documents and/or papers showing all business transactions and (3) that, in any event, the effects seized are admissible in evidence
including disbursements receipts, balance sheets and profit and loss against herein petitioners, regardless of the alleged illegality of the
statements and Bobbins (cigarette wrappers). aforementioned searches and seizures.

On March 22, 1962, this Court issued the writ of preliminary injunction prayed
for in the petition. However, by resolution dated June 29, 1962, the writ was

43
partially lifted or dissolved, insofar as the papers, documents and things seized or the privacy of whose homes had not been disturbed; nor
seized from the offices of the corporations above mentioned are concerned; could they claim for themselves the benefits of the Fourth
but, the injunction was maintained as regards the papers, documents and Amendment, when its violation, if any, was with reference to the
things found and seized in the residences of petitioners herein. 7 rights of another. Remus vs. United States (C.C.A.)291 F. 501, 511. It
follows, therefore, that the question of the admissibility of the
Thus, the documents, papers, and things seized under the alleged authority evidence based on an alleged unlawful search and seizure
of the warrants in question may be split into two (2) major groups, namely: does not extend to the personal defendants but
(a) those found and seized in the offices of the aforementioned corporations, embraces only the corporation whose property was taken. . . . (A
and (b) those found and seized in the residences of petitioners herein. Guckenheimer & Bros. Co. vs. United States, [1925] 3 F. 2d. 786,
789, Emphasis supplied.)
As regards the first group, we hold that petitioners herein have no cause of
action to assail the legality of the contested warrants and of the seizures With respect to the documents, papers and things seized in the residences of
made in pursuance thereof, for the simple reason that said corporations have petitioners herein, the aforementioned resolution of June 29, 1962, lifted the
their respective personalities, separate and distinct from the personality of writ of preliminary injunction previously issued by this Court, 12 thereby, in
herein petitioners, regardless of the amount of shares of stock or of the effect, restraining herein Respondents-Prosecutors from using them in
interest of each of them in said corporations, and whatever the offices they evidence against petitioners herein.
hold therein may be.8 Indeed, it is well settled that the legality of a seizure
can be contested only by the party whose rights have been impaired In connection with said documents, papers and things, two (2) important
thereby,9 and that the objection to an unlawful search and seizure is purely questions need be settled, namely: (1) whether the search warrants in
personal and cannot be availed of by third parties. 10 Consequently, question, and the searches and seizures made under the authority thereof,
petitioners herein may not validly object to the use in evidence against them are valid or not, and (2) if the answer to the preceding question is in the
of the documents, papers and things seized from the offices and premises of negative, whether said documents, papers and things may be used in
the corporations adverted to above, since the right to object to the admission evidence against petitioners herein.1wph1.t
of said papers in evidence belongs exclusively to the corporations, to whom
the seized effects belong, and may not be invoked by the corporate officers Petitioners maintain that the aforementioned search warrants are in the
in proceedings against them in their individual capacity. 11 Indeed, it has been nature of general warrants and that accordingly, the seizures effected upon
held: the authority there of are null and void. In this connection, the
Constitution 13 provides:
. . . that the Government's action in gaining possession of papers
belonging to the corporation did not relate to nor did it affect The right of the people to be secure in their persons, houses, papers,
the personal defendants. If these papers were unlawfully seized and and effects against unreasonable searches and seizures shall not be
thereby the constitutional rights of or any one were invaded, they violated, and no warrants shall issue but upon probable cause, to be
were the rights of the corporation and not the rights of the other determined by the judge after examination under oath or affirmation
defendants. Next, it is clear that a question of the lawfulness of a of the complainant and the witnesses he may produce, and
seizure can be raised only by one whose rights have been invaded. particularly describing the place to be searched, and the persons or
Certainly, such a seizure, if unlawful, could not affect the things to be seized.
constitutional rights of defendants whose property had not been

44
Two points must be stressed in connection with this constitutional mandate, probable cause in connection with one specific offense." Not satisfied with
namely: (1) that no warrant shall issue but upon probable cause, to be this qualification, the Court added thereto a paragraph, directing that "no
determined by the judge in the manner set forth in said provision; and (2) that search warrant shall issue for more than one specific offense."
the warrant shall particularly describe the things to be seized.
The grave violation of the Constitution made in the application for the
None of these requirements has been complied with in the contested contested search warrants was compounded by the description therein made
warrants. Indeed, the same were issued upon applications stating that the of the effects to be searched for and seized, to wit:
natural and juridical person therein named had committed a "violation of
Central Ban Laws, Tariff and Customs Laws, Internal Revenue (Code) and Books of accounts, financial records, vouchers, journals,
Revised Penal Code." In other words, no specific offense had been alleged in correspondence, receipts, ledgers, portfolios, credit journals,
said applications. The averments thereof with respect to the offense typewriters, and other documents and/or papers showing all
committed were abstract. As a consequence, it was impossible for the judges business transactions including disbursement receipts, balance
who issued the warrants to have found the existence of probable cause, for sheets and related profit and loss statements.
the same presupposes the introduction of competent proof that the party
against whom it is sought has performed particular acts, or Thus, the warrants authorized the search for and seizure of records
committed specific omissions, violating a given provision of our criminal laws. pertaining to all business transactions of petitioners herein, regardless of
As a matter of fact, the applications involved in this case do not allege any whether the transactions were legal or illegal. The warrants sanctioned the
specific acts performed by herein petitioners. It would be the legal heresy, of seizure of all records of the petitioners and the aforementioned corporations,
the highest order, to convict anybody of a "violation of Central Bank Laws, whatever their nature, thus openly contravening the explicit command of our
Tariff and Customs Laws, Internal Revenue (Code) and Revised Penal Bill of Rights that the things to be seized be particularly described as
Code," as alleged in the aforementioned applications without reference well as tending to defeat its major objective: the elimination
to any determinate provision of said laws or of general warrants.

To uphold the validity of the warrants in question would be to wipe out Relying upon Moncado vs. People's Court (80 Phil. 1), Respondents-
completely one of the most fundamental rights guaranteed in our Prosecutors maintain that, even if the searches and seizures under
Constitution, for it would place the sanctity of the domicile and the privacy of consideration were unconstitutional, the documents, papers and things thus
communication and correspondence at the mercy of the whims caprice or seized are admissible in evidence against petitioners herein. Upon mature
passion of peace officers. This is precisely the evil sought to be remedied by deliberation, however, we are unanimously of the opinion that the position
the constitutional provision above quoted to outlaw the so-called general taken in the Moncado case must be abandoned. Said position was in line
warrants. It is not difficult to imagine what would happen, in times of keen with the American common law rule, that the criminal should not be allowed
political strife, when the party in power feels that the minority is likely to wrest to go free merely "because the constable has blundered," 16 upon the theory
it, even though by legal means. that the constitutional prohibition against unreasonable searches and
seizures is protected by means other than the exclusion of evidence
Such is the seriousness of the irregularities committed in connection with the unlawfully obtained, 17 such as the common-law action for damages against
disputed search warrants, that this Court deemed it fit to amend Section 3 of the searching officer, against the party who procured the issuance of the
Rule 122 of the former Rules of Court 14 by providing in its counterpart, under search warrant and against those assisting in the execution of an illegal
the Revised Rules of Court 15 that "a search warrant shall not issue but upon

45
search, their criminal punishment, resistance, without liability to an unlawful . . . Today we once again examine the Wolf's constitutional
seizure, and such other legal remedies as may be provided by other laws. documentation of the right of privacy free from unreasonable state
intrusion, and after its dozen years on our books, are led by it to
However, most common law jurisdictions have already given up this close the only courtroom door remaining open to evidence secured
approach and eventually adopted the exclusionary rule, realizing that this by official lawlessness in flagrant abuse of that basic right, reserved
is the only practical means of enforcing the constitutional injunction against to all persons as a specific guarantee against that very same
unreasonable searches and seizures. In the language of Judge Learned unlawful conduct. We hold that all evidence obtained by searches
Hand: and seizures in violation of the Constitution is, by that same authority,
inadmissible in a State.
As we understand it, the reason for the exclusion of evidence
competent as such, which has been unlawfully acquired, is that Since the Fourth Amendment's right of privacy has been declared
exclusion is the only practical way of enforcing the constitutional enforceable against the States through the Due Process Clause of
privilege. In earlier times the action of trespass against the offending the Fourteenth, it is enforceable against them by the same sanction
official may have been protection enough; but that is true no longer. of exclusion as it used against the Federal Government. Were it
Only in case the prosecution which itself controls the seizing officials, otherwise, then just as without the Weeks rule the assurance against
knows that it cannot profit by their wrong will that wrong be unreasonable federal searches and seizures would be "a form of
repressed.18 words," valueless and underserving of mention in a perpetual charter
of inestimable human liberties, so too, without that rule the freedom
In fact, over thirty (30) years before, the Federal Supreme Court had already from state invasions of privacy would be so ephemeral and so neatly
declared: severed from its conceptual nexus with the freedom from all brutish
means of coercing evidence as not to permit this Court's high regard
as a freedom "implicit in the concept of ordered liberty." At the time
If letters and private documents can thus be seized and held and
that the Court held in Wolf that the amendment was applicable to the
used in evidence against a citizen accused of an offense, the
States through the Due Process Clause, the cases of this Court as
protection of the 4th Amendment, declaring his rights to be secure
we have seen, had steadfastly held that as to federal officers the
against such searches and seizures, is of no value, and, so far as
Fourth Amendment included the exclusion of the evidence seized in
those thus placed are concerned, might as well be stricken from the
violation of its provisions. Even Wolf "stoutly adhered" to that
Constitution. The efforts of the courts and their officials to bring the
proposition. The right to when conceded operatively enforceable
guilty to punishment, praiseworthy as they are, are not to be aided
against the States, was not susceptible of destruction by avulsion of
by the sacrifice of those great principles established by years of
the sanction upon which its protection and enjoyment had always
endeavor and suffering which have resulted in their embodiment in
been deemed dependent under the Boyd, Weeks and Silverthorne
the fundamental law of the land.19
Cases. Therefore, in extending the substantive protections of due
process to all constitutionally unreasonable searches state or
This view was, not only reiterated, but, also, broadened in subsequent federal it was logically and constitutionally necessarily that the
decisions on the same Federal Court. 20After reviewing previous decisions exclusion doctrine an essential part of the right to privacy be
thereon, said Court held, in Mapp vs. Ohio (supra.): also insisted upon as an essential ingredient of the right newly
recognized by the Wolf Case. In short, the admission of the new

46
constitutional Right by Wolf could not tolerate denial of its most commission of a crime. But, then, this fishing expedition is indicative of the
important constitutional privilege, namely, the exclusion of the absence of evidence to establish a probable cause.
evidence which an accused had been forced to give by reason of the
unlawful seizure. To hold otherwise is to grant the right but in reality Moreover, the theory that the criminal prosecution of those who secure an
to withhold its privilege and enjoyment. Only last year the Court itself illegal search warrant and/or make unreasonable searches or seizures would
recognized that the purpose of the exclusionary rule to "is to deter suffice to protect the constitutional guarantee under consideration, overlooks
to compel respect for the constitutional guaranty in the only the fact that violations thereof are, in general, committed By agents of the
effectively available way by removing the incentive to disregard party in power, for, certainly, those belonging to the minority could not
it" . . . . possibly abuse a power they do not have. Regardless of the handicap under
which the minority usually but, understandably finds itself in
The ignoble shortcut to conviction left open to the State tends to prosecuting agents of the majority, one must not lose sight of the fact that the
destroy the entire system of constitutional restraints on which the psychological and moral effect of the possibility 21 of securing their conviction,
liberties of the people rest. Having once recognized that the right to is watered down by the pardoning power of the party for whose benefit the
privacy embodied in the Fourth Amendment is enforceable against illegality had been committed.
the States, and that the right to be secure against rude invasions of
privacy by state officers is, therefore constitutional in origin, we can In their Motion for Reconsideration and Amendment of the Resolution of this
no longer permit that right to remain an empty promise. Because it is Court dated June 29, 1962, petitioners allege that Rooms Nos. 81 and 91 of
enforceable in the same manner and to like effect as other basic Carmen Apartments, House No. 2008, Dewey Boulevard, House No. 1436,
rights secured by its Due Process Clause, we can no longer permit it Colorado Street, and Room No. 304 of the Army-Navy Club, should be
to be revocable at the whim of any police officer who, in the name of included among the premises considered in said Resolution as residences of
law enforcement itself, chooses to suspend its enjoyment. Our herein petitioners, Harry S. Stonehill, Robert P. Brook, John J. Brooks and
decision, founded on reason and truth, gives to the individual no Karl Beck, respectively, and that, furthermore, the records, papers and other
more than that which the Constitution guarantees him to the police effects seized in the offices of the corporations above referred to include
officer no less than that to which honest law enforcement is entitled, personal belongings of said petitioners and other effects under their
and, to the courts, that judicial integrity so necessary in the true exclusive possession and control, for the exclusion of which they have a
administration of justice. (emphasis ours.) standing under the latest rulings of the federal courts of federal courts of the
United States. 22
Indeed, the non-exclusionary rule is contrary, not only to the letter, but also,
to the spirit of the constitutional injunction against unreasonable searches We note, however, that petitioners' theory, regarding their alleged possession
and seizures. To be sure, if the applicant for a search warrant has competent of and control over the aforementioned records, papers and effects, and the
evidence to establish probable cause of the commission of a given crime by alleged "personal" nature thereof, has Been Advanced, notin their petition or
the party against whom the warrant is intended, then there is no reason why amended petition herein, but in the Motion for Reconsideration and
the applicant should not comply with the requirements of the fundamental Amendment of the Resolution of June 29, 1962. In other words, said theory
law. Upon the other hand, if he has no such competent evidence, then it would appear to be readjustment of that followed in said petitions, to suit the
is not possible for the Judge to find that there is probable cause, and, hence, approach intimated in the Resolution sought to be reconsidered and
no justification for the issuance of the warrant. The only possible explanation amended. Then, too, some of the affidavits or copies of alleged affidavits
(not justification) for its issuance is the necessity of fishing evidence of the attached to said motion for reconsideration, or submitted in support thereof,

47
contain either inconsistent allegations, or allegations inconsistent with the
theory now advanced by petitioners herein.

Upon the other hand, we are not satisfied that the allegations of said petitions
said motion for reconsideration, and the contents of the aforementioned
affidavits and other papers submitted in support of said motion, have
sufficiently established the facts or conditions contemplated in the cases
relied upon by the petitioners; to warrant application of the views therein
expressed, should we agree thereto. At any rate, we do not deem it
necessary to express our opinion thereon, it being best to leave the matter
open for determination in appropriate cases in the future.

We hold, therefore, that the doctrine adopted in the Moncado case must be,
as it is hereby, abandoned; that the warrants for the search of three (3)
residences of herein petitioners, as specified in the Resolution of June 29,
1962, are null and void; that the searches and seizures therein made are
illegal; that the writ of preliminary injunction heretofore issued, in connection
with the documents, papers and other effects thus seized in said residences
of herein petitioners is hereby made permanent; that the writs prayed for are
granted, insofar as the documents, papers and other effects so seized in the
aforementioned residences are concerned; that the aforementioned motion
for Reconsideration and Amendment should be, as it is hereby, denied; and
that the petition herein is dismissed and the writs prayed for denied, as
regards the documents, papers and other effects seized in the twenty-nine
(29) places, offices and other premises enumerated in the same Resolution,
without special pronouncement as to costs.

It is so ordered.

Reyes, J.B.L., Dizon, Makalintal, Bengzon, J.P., Zaldivar and Sanchez, JJ.,
concur.

48
[G.R. No. L-32409. February 27, 1971.]

BACHE & CO. (PHIL.), INC. and FREDERICK E.


SEGGERMAN, Petitioners, v. HON. JUDGE VIVENCIO M. RUIZ, MISAEL
P. VERA, in his capacity as Commissioner of Internal Revenue,
ARTURO LOGRONIO, RODOLFO DE LEON, GAVINO VELASQUEZ,
MIMIR DELLOSA, NICANOR ALCORDO, JOHN DOE, JOHN DOE, JOHN
DOE, and JOHN DOE, Respondents.

49
San Juan, Africa, Gonzales & San Agustin, for Petitioners. Revenue, wrote a letter addressed to respondent Judge Vivencio M. Ruiz
requesting the issuance of a search warrant against petitioners for violation
Solicitor General Felix Q. Antonio, Assistant Solicitor General Crispin of Section 46(a) of the National Internal Revenue Code, in relation to all other
V . Bautista, Solicitor Pedro A. Ramirez and Special Attorney Jaime M. pertinent provisions thereof, particularly Sections 53, 72, 73, 208 and 209,
Maza for Respondents. and authorizing Revenue Examiner Rodolfo de Leon, one of herein
respondents, to make and file the application for search warrant which was
attached to the letter.

In the afternoon of the following day, February 25, 1970, respondent De Leon
DECISION and his witness, respondent Arturo Logronio, went to the Court of First
Instance of Rizal. They brought with them the following papers: respondent
Veras aforesaid letter-request; an application for search warrant already
filled up but still unsigned by respondent De Leon; an affidavit of respondent
Logronio subscribed before respondent De Leon; a deposition in printed form
of respondent Logronio already accomplished and signed by him but not yet
VILLAMOR, J.: subscribed; and a search warrant already accomplished but still unsigned by
respondent Judge.

At that time respondent Judge was hearing a certain case; so, by means of a
note, he instructed his Deputy Clerk of Court to take the depositions of
This is an original action of certiorari, prohibition and mandamus, with prayer respondents De Leon and Logronio. After the session had adjourned,
for a writ of preliminary mandatory and prohibitory injunction. In their petition respondent Judge was informed that the depositions had already been taken.
Bache & Co. (Phil.), Inc., a corporation duly organized and existing under the The stenographer, upon request of respondent Judge, read to him her
laws of the Philippines, and its President, Frederick E. Seggerman, pray this stenographic notes; and thereafter, respondent Judge asked respondent
Court to declare null and void Search Warrant No. 2-M-70 issued by Logronio to take the oath and warned him that if his deposition was found to
respondent Judge on February 25, 1970; to order respondents to desist from be false and without legal basis, he could be charged for perjury. Respondent
enforcing the same and/or keeping the documents, papers and effects seized Judge signed respondent de Leons application for search warrant and
by virtue thereof, as well as from enforcing the tax assessments on petitioner respondent Logronios deposition, Search Warrant No. 2-M-70 was then sign
corporation alleged by petitioners to have been made on the basis of the said by respondent Judge and accordingly issued.
documents, papers and effects, and to order the return of the latter to
petitioners. We gave due course to the petition but did not issue the writ of Three days later, or on February 28, 1970, which was a Saturday, the BIR
preliminary injunction prayed for therein. agents served the search warrant petitioners at the offices of petitioner
corporation on Ayala Avenue, Makati, Rizal. Petitioners lawyers protested
The pertinent facts of this case, as gathered from record, are as the search on the ground that no formal complaint or transcript of testimony
follows:chanrob1es virtual 1aw library was attached to the warrant. The agents nevertheless proceeded with their
search which yielded six boxes of documents.
On February 24, 1970, respondent Misael P. Vera, Commissioner of Internal

50
On March 3, 1970, petitioners filed a petition with the Court of First Instance "No search warrant shall issue for more than one specific offense.
of Rizal praying that the search warrant be quashed, dissolved or recalled,
that preliminary prohibitory and mandatory writs of injunction be issued, that "SEC. 4. Examination of the applicant. The judge or justice of the peace
the search warrant be declared null and void, and that the respondents be must, before issuing the warrant, personally examine on oath or affirmation
ordered to pay petitioners, jointly and severally, damages and attorneys fees. the complainant and any witnesses he may produce and take their
On March 18, 1970, the respondents, thru the Solicitor General, filed an depositions in writing, and attach them to the record, in addition to any
answer to the petition. After hearing, the court, presided over by respondent affidavits presented to him." (Rule 126, Revised Rules of Court.)
Judge, issued on July 29, 1970, an order dismissing the petition for
dissolution of the search warrant. In the meantime, or on April 16, 1970, the The examination of the complainant and the witnesses he may produce,
Bureau of Internal Revenue made tax assessments on petitioner corporation required by Art. III, Sec. 1, par. 3, of the Constitution, and by Secs. 3 and 4,
in the total sum of P2,594,729.97, partly, if not entirely, based on the Rule 126 of the Revised Rules of Court, should be conducted by the judge
documents thus seized. Petitioners came to this Court. himself and not by others. The phrase "which shall be determined by the
judge after examination under oath or affirmation of the complainant and the
The petition should be granted for the following reasons:chanrob1es virtual witnesses he may produce," appearing in the said constitutional provision,
1aw library was introduced by Delegate Francisco as an amendment to the draft
submitted by the Sub-Committee of Seven. The following discussion in the
1. Respondent Judge failed to personally examine the complainant and his Constitutional Convention (Laurel, Proceedings of the Philippine
witness. Constitutional Convention, Vol. III, pp. 755-757) is
enlightening:jgc:chanrobles.com.ph
The pertinent provisions of the Constitution of the Philippines and of the
Revised Rules of Court are:jgc:chanrobles.com.ph "SR. ORENSE. Vamos a dejar compaero los piropos y vamos al grano.

"(3) The right of the people to be secure in their persons, houses, papers and En los casos de una necesidad de actuar inmediatamente para que no se
effects against unreasonable searches and seizures shall not be violated, frusten los fines de la justicia mediante el registro inmediato y la incautacion
and no warrants shall issue but upon probable cause, to be determined by del cuerpo del delito, no cree Su Seoria que causaria cierta demora el
the judge after examination under oath or affirmation of the complainant and procedimiento apuntado en su enmienda en tal forma que podria frustrar los
the witnesses he may produce, and particularly describing the place to be fines de la justicia o si Su Seoria encuentra un remedio para esto casos con
searched, and the persons or things to be seized." (Art. III, Sec. 1, el fin de compaginar los fines de la justicia con los derechos del individuo en
Constitution.) su persona, bienes etcetera, etcetera.

"SEC. 3. Requisites for issuing search warrant. A search warrant shall not "SR. FRANCISCO. No puedo ver en la practica el caso hipottico que Su
issue but upon probable cause in connection with one specific offense to be Seoria pregunta por la siguiente razon: el que solicita un mandamiento de
determined by the judge or justice of the peace after examination under oath registro tiene que hacerlo por escrito y ese escrito no aparecer en la Mesa
or affirmation of the complainant and the witnesses he may produce, and del Juez sin que alguien vaya el juez a presentar ese escrito o peticion de
particularly describing the place to be searched and the persons or things to sucuestro. Esa persona que presenta el registro puede ser el mismo
be seized. denunciante o alguna persona que solicita dicho mandamiento de registro.
Ahora toda la enmienda en esos casos consiste en que haya peticion de

51
registro y el juez no se atendra solamente a sea peticion sino que el juez In the case at bar, no personal examination at all was conducted by
examiner a ese denunciante y si tiene testigos tambin examiner a los respondent Judge of the complainant (respondent De Leon) and his witness
testigos. (respondent Logronio). While it is true that the complainants application for
search warrant and the witness printed-form deposition were subscribed and
"SR. ORENSE. No cree Su Seoria que el tomar le declaracion de ese sworn to before respondent Judge, the latter did not ask either of the two any
denunciante por escrito siempre requeriria algun tiempo?. question the answer to which could possibly be the basis for determining
whether or not there was probable cause against herein petitioners. Indeed,
"SR. FRANCISCO. Seria cuestio de un par de horas, pero por otro lado the participants seem to have attached so little significance to the matter that
minimizamos en todo lo posible las vejaciones injustas con la expedicion notes of the proceedings before respondent Judge were not even taken. At
arbitraria de los mandamientos de registro. Creo que entre dos males this juncture it may be well to recall the salient facts. The transcript of
debemos escoger. el menor. stenographic notes (pp. 61-76, April 1, 1970, Annex J-2 of the Petition) taken
at the hearing of this case in the court below shows that per instruction of
x x x respondent Judge, Mr. Eleodoro V. Gonzales, Special Deputy Clerk of Court,
took the depositions of the complainant and his witness, and that
stenographic notes thereof were taken by Mrs. Gaspar. At that time
respondent Judge was at the sala hearing a case. After respondent Judge
"MR. LAUREL. . . . The reason why we are in favor of this amendment is was through with the hearing, Deputy Clerk Gonzales, stenographer Gaspar,
because we are incorporating in our constitution something of a fundamental complainant De Leon and witness Logronio went to respondent Judges
character. Now, before a judge could issue a search warrant, he must be chamber and informed the Judge that they had finished the depositions.
under the obligation to examine personally under oath the complainant and if Respondent Judge then requested the stenographer to read to him her
he has any witness, the witnesses that he may produce . . ."cralaw virtua1aw stenographic notes. Special Deputy Clerk Gonzales testified as
library follows:jgc:chanrobles.com.ph

The implementing rule in the Revised Rules of Court, Sec. 4, Rule 126, is "A And after finishing reading the stenographic notes, the Honorable Judge
more emphatic and candid, for it requires the judge, before issuing a search requested or instructed them, requested Mr. Logronio to raise his hand and
warrant, to "personally examine on oath or affirmation the complainant and warned him if his deposition will be found to be false and without legal basis,
any witnesses he may produce . . ."cralaw virtua1aw library he can be charged criminally for perjury. The Honorable Court told Mr.
Logronio whether he affirms the facts contained in his deposition and the
Personal examination by the judge of the complainant and his witnesses is affidavit executed before Mr. Rodolfo de Leon.
necessary to enable him to determine the existence or non-existence of a
probable cause, pursuant to Art. III, Sec. 1, par. 3, of the Constitution, and "Q And thereafter?
Sec. 3, Rule 126 of the Revised Rules of Court, both of which prohibit the
issuance of warrants except "upon probable cause." The determination of "A And thereafter, he signed the deposition of Mr. Logronio.
whether or not a probable cause exists calls for the exercise of judgment
after a judicial appraisal of facts and should not be allowed to be delegated in "Q Who is this he?
the absence of any rule to the contrary.
"A The Honorable Judge.

52
provisions of the Tax Code referred to above. Thus we find the
"Q The deposition or the affidavit? following:chanrob1es virtual 1aw library

"A The affidavit, Your Honor."cralaw virtua1aw library Sec. 46(a) requires the filing of income tax returns by corporations.

Thereafter, respondent Judge signed the search warrant. Sec. 53 requires the withholding of income taxes at source.

The participation of respondent Judge in the proceedings which led to the Sec. 72 imposes surcharges for failure to render income tax returns and for
issuance of Search Warrant No. 2-M-70 was thus limited to listening to the rendering false and fraudulent returns.
stenographers readings of her notes, to a few words of warning against the
commission of perjury, and to administering the oath to the complainant and Sec. 73 provides the penalty for failure to pay the income tax, to make a
his witness. This cannot be consider a personal examination. If there was an return or to supply the information required under the Tax Code.
examination at all of the complainant and his witness, it was the one
conducted by the Deputy Clerk of Court. But, as stated, the Constitution and Sec. 208 penalizes" [a]ny person who distills, rectifies, repacks, compounds,
the rules require a personal examination by the judge. It was precisely on or manufactures any article subject to a specific tax, without having paid the
account of the intention of the delegates to the Constitutional Convention to privilege tax therefore, or who aids or abets in the conduct of illicit distilling,
make it a duty of the issuing judge to personally examine the complainant rectifying, compounding, or illicit manufacture of any article subject to specific
and his witnesses that the question of how much time would be consumed by tax . . .," and provides that in the case of a corporation, partnership, or
the judge in examining them came up before the Convention, as can be seen association, the official and/or employee who caused the violation shall be
from the record of the proceedings quoted above. The reading of the responsible.
stenographic notes to respondent Judge did not constitute sufficient
compliance with the constitutional mandate and the rule; for by that manner Sec. 209 penalizes the failure to make a return of receipts, sales, business,
respondent Judge did not have the opportunity to observe the demeanor of or gross value of output removed, or to pay the tax due thereon.
the complainant and his witness, and to propound initial and follow-up
questions which the judicial mind, on account of its training, was in the best The search warrant in question was issued for at least four distinct offenses
position to conceive. These were important in arriving at a sound inference under the Tax Code. The first is the violation of Sec. 46(a), Sec. 72 and Sec.
on the all-important question of whether or not there was probable cause. 73 (the filing of income tax returns), which are interrelated. The second is the
violation of Sec. 53 (withholding of income taxes at source). The third is the
2. The search warrant was issued for more than one specific offense. violation of Sec. 208 (unlawful pursuit of business or occupation); and the
fourth is the violation of Sec. 209 (failure to make a return of receipts, sales,
Search Warrant No. 2-M-70 was issued for" [v]iolation of Sec. 46(a) of the business or gross value of output actually removed or to pay the tax due
National Internal Revenue Code in relation to all other pertinent provisions thereon). Even in their classification the six above-mentioned provisions are
thereof particularly Secs. 53, 72, 73, 208 and 209." The question is: Was the embraced in two different titles: Secs. 46(a), 53, 72 and 73 are under Title II
said search warrant issued "in connection with one specific offense," as (Income Tax); while Secs. 208 and 209 are under Title V (Privilege Tax on
required by Sec. 3, Rule 126? Business and Occupation).

To arrive at the correct answer it is essential to examine closely the Respondents argue that Stonehill, Et. Al. v. Diokno, Et Al., L-19550, June 19,

53
1967 (20 SCRA 383), is not applicable, because there the search warrants In Stonehill, this Court, speaking thru Mr. Chief Justice Roberto Concepcion,
were issued for "violation of Central Bank Laws, Internal Revenue (Code) said:jgc:chanrobles.com.ph
and Revised Penal Code;" whereas, here Search Warrant No 2-M-70 was
issued for violation of only one code, i.e., the National Internal Revenue "The grave violation of the Constitution made in the application for the
Code. The distinction more apparent than real, because it was precisely on contested search warrants was compounded by the description therein made
account of the Stonehill incident, which occurred sometime before the of the effects to be searched for and seized, to wit:chanrob1es virtual 1aw
present Rules of Court took effect on January 1, 1964, that this Court library
amended the former rule by inserting therein the phrase "in connection with
one specific offense," and adding the sentence "No search warrant shall Books of accounts, financial records, vouchers, journals, correspondence,
issue for more than one specific offense," in what is now Sec. 3, Rule 126. receipts, ledgers, portfolios, credit journals, typewriters, and other documents
Thus we said in Stonehill:jgc:chanrobles.com.ph and/or paper showing all business transactions including disbursement
receipts, balance sheets and related profit and loss statements.
"Such is the seriousness of the irregularities committed in connection with the
disputed search warrants, that this Court deemed it fit to amend Section 3 of "Thus, the warrants authorized the search for and seizure of records
Rule 122 of the former Rules of Court that a search warrant shall not issue pertaining to all business transactions of petitioners herein, regardless of
but upon probable cause in connection with one specific offense. Not whether the transactions were legal or illegal. The warrants sanctioned the
satisfied with this qualification, the Court added thereto a paragraph, seizure of all records of the petitioners and the aforementioned corporations,
directing that no search warrant shall issue for more than one specific whatever their nature, thus openly contravening the explicit command of our
offense." Bill of Rights that the things to be seized be particularly described as
well as tending to defeat its major objective: the elimination of general
3. The search warrant does not particularly describe the things to be seized. warrants."cralaw virtua1aw library

The documents, papers and effects sought to be seized are described in While the term "all business transactions" does not appear in Search Warrant
Search Warrant No. 2-M-70 in this manner:jgc:chanrobles.com.ph No. 2-M-70, the said warrant nevertheless tends to defeat the major objective
of the Bill of Rights, i.e., the elimination of general warrants, for the language
"Unregistered and private books of accounts (ledgers, journals, columnars, used therein is so all-embracing as to include all conceivable records of
receipts and disbursements books, customers ledgers); receipts for petitioner corporation, which, if seized, could possibly render its business
payments received; certificates of stocks and securities; contracts, inoperative.
promissory notes and deeds of sale; telex and coded messages; business
communications, accounting and business records; checks and check stubs; In Uy Kheytin, Et. Al. v. Villareal, etc., Et Al., 42 Phil. 886, 896, this Court had
records of bank deposits and withdrawals; and records of foreign occasion to explain the purpose of the requirement that the warrant should
remittances, covering the years 1966 to 1970."cralaw virtua1aw library particularly describe the place to be searched and the things to be seized, to
wit:jgc:chanrobles.com.ph
The description does not meet the requirement in Art III, Sec. 1, of the
Constitution, and of Sec. 3, Rule 126 of the Revised Rules of Court, that the ". . . Both the Jones Law (sec. 3) and General Orders No. 58 (sec. 97)
warrant should particularly describe the things to be seized. specifically require that a search warrant should particularly describe the
place to be searched and the things to be seized. The evident purpose and

54
intent of this requirement is to limit the things to be seized to those, and only place, the rule requiring the filing of a motion for reconsideration before an
those, particularly described in the search warrant to leave the officers of application for a writ of certiorari can be entertained was never intended to be
the law with no discretion regarding what articles they shall seize, to the end applied without considering the circumstances. (Matutina v. Buslon, Et Al.,
that unreasonable searches and seizures may not be made, that abuses 109 Phil., 140.) In the case at bar time is of the essence in view of the tax
may not be committed. That this is the correct interpretation of this assessments sought to be enforced by respondent officers of the Bureau of
constitutional provision is borne out by American authorities."cralaw Internal Revenue against petitioner corporation, On account of which
virtua1aw library immediate and more direct action becomes necessary. (Matute v. Court of
Appeals, Et Al., 26 SCRA 768.) Lastly, the rule does not apply where, as in
The purpose as thus explained could, surely and effectively, be defeated this case, the deprivation of petitioners fundamental right to due process
under the search warrant issued in this case. taints the proceeding against them in the court below not only with irregularity
but also with nullity. (Matute v. Court of Appeals, Et Al., supra.)
A search warrant may be said to particularly describe the things to be seized
when the description therein is as specific as the circumstances will ordinarily It is next contended by respondents that a corporation is not entitled to
allow (People v. Rubio; 57 Phil. 384); or when the description expresses a protection against unreasonable search and seizures. Again, we find no merit
conclusion of fact not of law by which the warrant officer may be guided in the contention.
in making the search and seizure (idem., dissent of Abad Santos, J.,); or
when the things described are limited to those which bear direct relation to "Although, for the reasons above stated, we are of the opinion that an officer
the offense for which the warrant is being issued (Sec. 2, Rule 126, Revised of a corporation which is charged with a violation of a statute of the state of
Rules of Court). The herein search warrant does not conform to any of the its creation, or of an act of Congress passed in the exercise of its
foregoing tests. If the articles desired to be seized have any direct relation to constitutional powers, cannot refuse to produce the books and papers of
an offense committed, the applicant must necessarily have some evidence, such corporation, we do not wish to be understood as holding that a
other than those articles, to prove the said offense; and the articles subject of corporation is not entitled to immunity, under the 4th Amendment, against
search and seizure should come in handy merely to strengthen such unreasonable searches and seizures. A corporation is, after all, but an
evidence. In this event, the description contained in the herein disputed association of individuals under an assumed name and with a distinct legal
warrant should have mentioned, at least, the dates, amounts, persons, and entity. In organizing itself as a collective body it waives no constitutional
other pertinent data regarding the receipts of payments, certificates of stocks immunities appropriate to such body. Its property cannot be taken without
and securities, contracts, promissory notes, deeds of sale, messages and compensation. It can only be proceeded against by due process of law, and
communications, checks, bank deposits and withdrawals, records of foreign is protected, under the 14th Amendment, against unlawful discrimination . . ."
remittances, among others, enumerated in the warrant. (Hale v. Henkel, 201 U.S. 43, 50 L. ed. 652.)

Respondents contend that certiorari does not lie because petitioners failed to "In Linn v. United States, 163 C.C.A. 470, 251 Fed. 476, 480, it was thought
file a motion for reconsideration of respondent Judges order of July 29, that a different rule applied to a corporation, the ground that it was not
1970. The contention is without merit. In the first place, when the questions privileged from producing its books and papers. But the rights of a
raised before this Court are the same as those which were squarely raised in corporation against unlawful search and seizure are to be protected even if
and passed upon by the court below, the filing of a motion for reconsideration the same result might have been achieved in a lawful way." (Silverthorne
in said court before certiorari can be instituted in this Court is no longer a Lumber Company, Et. Al. v. United States of America, 251 U.S. 385, 64 L. ed.
prerequisite. (Pajo, etc., Et. Al. v. Ago, Et Al., 108 Phil., 905). In the second 319.)

55
PREMISES CONSIDERED, the petition is granted. Accordingly, Search
In Stonehill, Et. Al. v. Diokno, Et Al., supra, this Court impliedly recognized Warrant No. 2-M-70 issued by respondent Judge is declared null and void;
the right of a corporation to object against unreasonable searches and respondents are permanently enjoined from enforcing the said search
seizures, thus:jgc:chanrobles.com.ph warrant; the documents, papers and effects seized thereunder are ordered to
be returned to petitioners; and respondent officials the Bureau of Internal
"As regards the first group, we hold that petitioners herein have no cause of Revenue and their representatives are permanently enjoined from enforcing
action to assail the legality of the contested warrants and of the seizures the assessments mentioned in Annex "G" of the present petition, as well as
made in pursuance thereof, for the simple reason that said corporations have other assessments based on the documents, papers and effects seized
their respective personalities, separate and distinct from the personality of under the search warrant herein nullified, and from using the same against
herein petitioners, regardless of the amount of shares of stock or the interest petitioners in any criminal or other proceeding. No pronouncement as to
of each of them in said corporations, whatever, the offices they hold therein costs.
may be. Indeed, it is well settled that the legality of a seizure can be
contested only by the party whose rights have been impaired thereby, and Concepcion, C.J., Dizon, Makalintal, Zaldivar, Fernando, Teehankee and
that the objection to an unlawful search and seizure is purely personal and Makasiar, JJ., concur.
cannot be availed of by third parties. Consequently, petitioners herein may
not validly object to the use in evidence against them of the documents, Reyes, J.B.L., J., concurs with Mr. Justice Barredo.
papers and things seized from the offices and premises of the corporations
adverted to above, since the right to object to the admission of said papers in Castro, J., concurs in the result.
evidence belongs exclusively to the corporations, to whom the seized effects
belong, and may not be invoked by the corporate officers in proceedings
against them in their individual capacity . . ."cralaw virtua1aw library

In the Stonehill case only the officers of the various corporations in whose
offices documents, papers and effects were searched and seized were the
petitioners. In the case at bar, the corporation to whom the seized documents
belong, and whose rights have thereby been impaired, is itself a petitioner.
On that score, petitioner corporation here stands on a different footing from
the corporations in Stonehill.

The tax assessments referred to earlier in this opinion were, if not entirely
as claimed by petitioners at least partly as in effect admitted by
respondents based on the documents seized by virtue of Search Warrant
No. 2-M-70. Furthermore, the fact that the assessments were made some
one and one-half months after the search and seizure on February 25, 1970,
is a strong indication that the documents thus seized served as basis for the
assessments. Those assessments should therefore not be enforced.

56
57
G.R. No. 75885 May 27, 1987

BATAAN SHIPYARD & ENGINEERING CO., INC. (BASECO), petitioner,


vs.
PRESIDENTIAL COMMISSION ON GOOD GOVERNMENT, CHAIRMAN
JOVITO SALONGA, COMMISSIONER MARY CONCEPCION BAUTISTA,
COMMISSIONER RAMON DIAZ, COMMISSIONER RAUL R. DAZA,
COMMISSIONER QUINTIN S. DOROMAL, CAPT. JORGE B. SIACUNCO,
et al., respondents.

Apostol, Bernas, Gumaru, Ona and Associates for petitioner.

Vicente G. Sison for intervenor A.T. Abesamis.

NARVASA, J.:

Challenged in this special civil action of certiorari and prohibition by a private


corporation known as the Bataan Shipyard and Engineering Co., Inc. are: (1)
Executive Orders Numbered 1 and 2, promulgated by President Corazon C.
Aquino on February 28, 1986 and March 12, 1986, respectively, and (2) the
sequestration, takeover, and other orders issued, and acts done, in

58
accordance with said executive orders by the Presidential Commission on 8. Bay Transport
Good Government and/or its Commissioners and agents, affecting said
corporation. 9. And all affiliate companies of Alfredo
"Bejo" Romualdez
1. The Sequestration, Takeover, and Other Orders Complained of
You are hereby ordered:
a. The Basic Sequestration Order
1. To implement this sequestration order with a minimum
The sequestration order which, in the view of the petitioner corporation, disruption of these companies' business activities.
initiated all its misery was issued on April 14, 1986 by Commissioner Mary
Concepcion Bautista. It was addressed to three of the agents of the 2. To ensure the continuity of these companies as going
Commission, hereafter simply referred to as PCGG. It reads as follows: concerns, the care and maintenance of these assets until
such time that the Office of the President through the
RE: SEQUESTRATION ORDER Commission on Good Government should decide otherwise.

By virtue of the powers vested in the Presidential 3. To report to the Commission on Good Government
Commission on Good Government, by authority of the periodically.
President of the Philippines, you are hereby directed to
sequester the following companies. Further, you are authorized to request for Military/Security
Support from the Military/Police authorities, and such other
1. Bataan Shipyard and Engineering Co., acts essential to the achievement of this sequestration
Inc. (Engineering Island Shipyard and order. 1
Mariveles Shipyard)
b. Order for Production of Documents
2. Baseco Quarry
On the strength of the above sequestration order, Mr. Jose M. Balde, acting
3. Philippine Jai-Alai Corporation for the PCGG, addressed a letter dated April 18, 1986 to the President and
other officers of petitioner firm, reiterating an earlier request for the
4. Fidelity Management Co., Inc. production of certain documents, to wit:

5. Romson Realty, Inc. 1. Stock Transfer Book

6. Trident Management Co. 2. Legal documents, such as:

7. New Trident Management 2.1. Articles of Incorporation

59
2.2. By-Laws 9. Complete list of depository banks for all funds with the
authorized signatories for withdrawals thereof.
2.3. Minutes of the Annual Stockholders
2
Meeting from 1973 to 1986 10. Schedule of company investments and placements.

2.4. Minutes of the Regular and Special The letter closed with the warning that if the documents were not submitted
Meetings of the Board of Directors from within five days, the officers would be cited for "contempt in pursuance with
1973 to 1986 Presidential Executive Order Nos. 1 and 2."

2.5. Minutes of the Executive Committee c. Orders Re Engineer Island


Meetings from 1973 to 1986
(1) Termination of Contract for Security
2.6. Existing contracts with Services
suppliers/contractors/others.
A third order assailed by petitioner corporation, hereafter referred to simply
3. Yearly list of stockholders with their corresponding as BASECO, is that issued on April 21, 1986 by a Capt. Flordelino B. Zabala,
share/stockholdings from 1973 to 1986 duly certified by the a member of the task force assigned to carry out the basic sequestration
Corporate Secretary. order. He sent a letter to BASECO's Vice-President for Finance, 3 terminating
the contract for security services within the Engineer Island compound
4. Audited Financial Statements such as Balance Sheet, between BASECO and "Anchor and FAIRWAYS" and "other civilian security
Profit & Loss and others from 1973 to December 31, 1985. agencies," CAPCOM military personnel having already been assigned to the
area,
5. Monthly Financial Statements for the current year up to
March 31, 1986. (2) Change of Mode of Payment of Entry
Charges
6. Consolidated Cash Position Reports from January to April
15, 1986. On July 15, 1986, the same Capt. Zabala issued a Memorandum addressed
to "Truck Owners and Contractors," particularly a "Mr. Buddy Ondivilla
7. Inventory listings of assets up dated up to March 31, National Marine Corporation," advising of the amendment in part of their
1986. contracts with BASECO in the sense that the stipulated charges for use of
the BASECO road network were made payable "upon entry and not anymore
subject to monthly billing as was originally agreed upon." 4
8. Updated schedule of Accounts Receivable and Accounts
Payable.
d. Aborted Contract for Improvement of Wharf at Engineer
Island

60
On July 9, 1986, a PCGG fiscal agent, S. Berenguer, entered into a contract subject to specified guidelines and safeguards including audit and
in behalf of BASECO with Deltamarine Integrated Port Services, Inc., in verification. 8
virtue of which the latter undertook to introduce improvements costing
approximately P210,000.00 on the BASECO wharf at Engineer Island, g. The TAKEOVER Order
allegedly then in poor condition, avowedly to "optimize its utilization and in
return maximize the revenue which would flow into the government coffers," By letter dated July 14, 1986, Commissioner Ramon A. Diaz decreed the
in consideration of Deltamarine's being granted "priority in using the provisional takeover by the PCGG of BASECO, "the Philippine Dockyard
improved portion of the wharf ahead of anybody" and exemption "from the Corporation and all their affiliated companies." 9 Diaz invoked the provisions
payment of any charges for the use of wharf including the area where it may of Section 3 (c) of Executive Order No. 1, empowering the Commission
install its bagging equipments" "until the improvement remains in a condition
suitable for port operations." 5 It seems however that this contract was never
* * To provisionally takeover in the public interest or to
consummated. Capt. Jorge B. Siacunco, "Head- (PCGG) BASECO
prevent its disposal or dissipation, business enterprises and
Management Team," advised Deltamarine by letter dated July 30, 1986 that
properties taken over by the government of the Marcos
"the new management is not in a position to honor the said contract" and
Administration or by entities or persons close to former
thus "whatever improvements * * (may be introduced) shall be deemed
President Marcos, until the transactions leading to such
unauthorized * * and shall be at * * (Deltamarine's) own risk." 6
acquisition by the latter can be disposed of by the
appropriate authorities.
e. Order for Operation of Sesiman Rock Quarry, Mariveles,
Bataan
A management team was designated to implement the order, headed by
Capt. Siacunco, and was given the following powers:
By Order dated June 20, 1986, Commissioner Mary Bautista first directed a
PCGG agent, Mayor Melba O. Buenaventura, "to plan and implement
1. Conducts all aspects of operation of the subject
progress towards maximizing the continuous operation of the BASECO
companies;
Sesiman Rock Quarry * * by conventional methods;" but afterwards,
Commissioner Bautista, in representation of the PCGG, authorized another
party, A.T. Abesamis, to operate the quarry, located at Mariveles, Bataan, an 2. Installs key officers, hires and terminates personnel as
agreement to this effect having been executed by them on September 17, necessary;
1986. 7
3. Enters into contracts related to management and
f. Order to Dispose of Scrap, etc. operation of the companies;

By another Order of Commissioner Bautista, this time dated June 26, 1986, 4. Ensures that the assets of the companies are not
Mayor Buenaventura was also "authorized to clean and beautify the dissipated and used effectively and efficiently; revenues are
Company's compound," and in this connection, to dispose of or sell "metal duly accounted for; and disburses funds only as may be
scraps" and other materials, equipment and machineries no longer usable, necessary;

61
5. Does actions including among others, seeking of military the same ruler opted to promulgate the Freedom Constitution on March 25,
support as may be necessary, that will ensure compliance to 1986 wherein under Section I of the same, Article IV (Bill of Rights) of the
this order; 1973 Constitution was adopted providing, among others, that "No person
shall be deprived of life, liberty and property without due process of law."
6. Holds itself fully accountable to the Presidential (Const., Art. I V, Sec. 1)." 12
Commission on Good Government on all aspects related to
this take-over order. It declares that its objection to the constitutionality of the Executive Orders
"as well as the Sequestration Order * * and Takeover Order * * issued
h. Termination of Services of BASECO purportedly under the authority of said Executive Orders, rests on four
Officers fundamental considerations: First, no notice and hearing was accorded * * (it)
before its properties and business were taken over; Second, the PCGG is not
Thereafter, Capt. Siacunco, sent letters to Hilario M. Ruiz, Manuel S. a court, but a purely investigative agency and therefore not competent to act
Mendoza, Moises M. Valdez, Gilberto Pasimanero, and Benito R. Cuesta I, as prosecutor and judge in the same cause; Third, there is nothing in the
advising of the termination of their services by the PCGG. 10 issuances which envisions any proceeding, process or remedy by which
petitioner may expeditiously challenge the validity of the takeover after the
same has been effected; and Fourthly, being directed against specified
2. Petitioner's Plea and Postulates
persons, and in disregard of the constitutional presumption of innocence and
general rules and procedures, they constitute a Bill of Attainder." 13
It is the foregoing specific orders and acts of the PCGG and its members and
agents which, to repeat, petitioner BASECO would have this Court nullify.
b. Re Order to Produce Documents
More particularly, BASECO prays that this Court-

It argues that the order to produce corporate records from 1973 to 1986,
1) declare unconstitutional and void Executive Orders Numbered 1 and 2;
which it has apparently already complied with, was issued without court
authority and infringed its constitutional right against self-incrimination, and
2) annul the sequestration order dated April- 14, 1986, and all other orders unreasonable search and seizure. 14
subsequently issued and acts done on the basis thereof, inclusive of the
takeover order of July 14, 1986 and the termination of the services of the
c. Re PCGG's Exercise of Right of Ownership and
BASECO executives. 11
Management

a. Re Executive Orders No. 1 and 2, and the Sequestration


BASECO further contends that the PCGG had unduly interfered with its right
and Takeover Orders
of dominion and management of its business affairs by

While BASECO concedes that "sequestration without resorting to judicial


1) terminating its contract for security services with Fairways & Anchor,
action, might be made within the context of Executive Orders Nos. 1 and 2
without the consent and against the will of the contracting parties; and
before March 25, 1986 when the Freedom Constitution was promulgated,
amending the mode of payment of entry fees stipulated in its Lease Contract
under the principle that the law promulgated by the ruler under a
revolutionary regime is the law of the land, it ceased to be acceptable when

62
with National Stevedoring & Lighterage Corporation, these acts being in governing these remedies. It is needful that these misconceptions and
violation of the non-impairment clause of the constitution; 15 doubts be dispelled so that uninformed and useless debates about them may
be avoided, and arguments tainted b sophistry or intellectual dishonesty be
2) allowing PCGG Agent Silverio Berenguer to enter into an "anomalous quickly exposed and discarded. Towards this end, this opinion will essay an
contract" with Deltamarine Integrated Port Services, Inc., giving the latter free exposition of the law on the matter. In the process many of the objections
use of BASECO premises; 16 raised by BASECO will be dealt with.

3) authorizing PCGG Agent, Mayor Melba Buenaventura, to manage and 4. The Governing Law
operate its rock quarry at Sesiman, Mariveles; 17
a. Proclamation No. 3
4) authorizing the same mayor to sell or dispose of its metal scrap,
equipment, machinery and other materials; 18 The impugned executive orders are avowedly meant to carry out the explicit
command of the Provisional Constitution, ordained by Proclamation No.
5) authorizing the takeover of BASECO, Philippine Dockyard Corporation, 3, 23 that the President-in the exercise of legislative power which she was
and all their affiliated companies; authorized to continue to wield "(until a legislature is elected and convened
under a new Constitution" "shall give priority to measures to achieve the
6) terminating the services of BASECO executives: President Hilario M. Ruiz; mandate of the people," among others to (r)ecover ill-gotten properties
EVP Manuel S. Mendoza; GM Moises M. Valdez; Finance Mgr. Gilberto amassed by the leaders and supporters of the previous regime and protect
Pasimanero; Legal Dept. Mgr. Benito R. Cuesta I; 19 the interest of the people through orders of sequestration or freezing of
assets or accounts." 24
20
7) planning to elect its own Board of Directors;
b. Executive Order No. 1
8) allowing willingly or unwillingly its personnel to take, steal, carry away from
petitioner's premises at Mariveles * * rolls of cable wires, worth P600,000.00 Executive Order No. 1 stresses the "urgent need to recover all ill-gotten
on May 11, 1986; 21 wealth," and postulates that "vast resources of the government have been
amassed by former President Ferdinand E. Marcos, his immediate family,
relatives, and close associates both here and abroad." 25 Upon these
9) allowing "indiscriminate diggings" at Engineer Island to retrieve gold bars
premises, the Presidential Commission on Good Government was
supposed to have been buried therein. 22
created, 26 "charged with the task of assisting the President in regard to
(certain specified) matters," among which was precisely-
3. Doubts, Misconceptions regarding Sequestration, Freeze and Takeover
Orders
* * The recovery of all in-gotten wealth accumulated by
former President Ferdinand E. Marcos, his immediate family,
Many misconceptions and much doubt about the matter of sequestration, relatives, subordinates and close associates, whether
takeover and freeze orders have been engendered by misapprehension, or located in the Philippines or abroad, including the takeover
incomplete comprehension if not indeed downright ignorance of the law or sequestration of all business enterprises and entities

63
owned or controlled by them, during his administration, regulations as may be necessary to carry out the purposes of * * (its
directly or through nominees, by taking undue advantage of creation). 30
their public office and/or using their powers, authority,
influence, connections or relationship. 27 c. Executive Order No. 2

In relation to the takeover or sequestration that it was authorized to Executive Order No. 2 gives additional and more specific data and directions
undertake in the fulfillment of its mission, the PCGG was granted "power and respecting "the recovery of ill-gotten properties amassed by the leaders and
authority" to do the following particular acts, to wit: supporters of the previous regime." It declares that:

1. To sequester or place or cause to be placed under its 1) * * the Government of the Philippines is in possession of
control or possession any building or office wherein any ill- evidence showing that there are assets and properties
gotten wealth or properties may be found, and any records purportedly pertaining to former Ferdinand E. Marcos, and/or
pertaining thereto, in order to prevent their destruction, his wife Mrs. Imelda Romualdez Marcos, their close
concealment or disappearance which would frustrate or relatives, subordinates, business associates, dummies,
hamper the investigation or otherwise prevent the agents or nominees which had been or were acquired by
Commission from accomplishing its task. them directly or indirectly, through or as a result of the
improper or illegal use of funds or properties owned by the
2. To provisionally take over in the public interest or to government of the Philippines or any of its branches,
prevent the disposal or dissipation, business enterprises and instrumentalities, enterprises, banks or financial institutions,
properties taken over by the government of the Marcos or by taking undue advantage of their office, authority,
Administration or by entities or persons close to former influence, connections or relationship, resulting in their
President Marcos, until the transactions leading to such unjust enrichment and causing grave damage and prejudice
acquisition by the latter can be disposed of by the to the Filipino people and the Republic of the Philippines:"
appropriate authorities. and

3. To enjoin or restrain any actual or threatened commission 2) * * said assets and properties are in the form of bank
of acts by any person or entity that may render moot and accounts, deposits, trust accounts, shares of stocks,
academic, or frustrate or otherwise make ineffectual the buildings, shopping centers, condominiums, mansions,
efforts of the Commission to carry out its task under this residences, estates, and other kinds of real and personal
order. 28 properties in the Philippines and in various countries of the
world." 31
So that it might ascertain the facts germane to its objectives, it was granted
power to conduct investigations; require submission of evidence by Upon these premises, the President-
subpoenae ad testificandum and duces tecum; administer oaths; punish for
contempt. 29 It was given power also to promulgate such rules and 1) froze "all assets and properties in the Philippines in which
former President Marcos and/or his wife, Mrs. Imelda

64
Romualdez Marcos, their close relatives, subordinates, A third executive order is relevant: Executive Order No. 14, 33 by which the
business associates, dummies, agents, or nominees have PCGG is empowered, "with the assistance of the Office of the Solicitor
any interest or participation; General and other government agencies, * * to file and prosecute all cases
investigated by it * * as may be warranted by its findings." 34 All such cases,
2) prohibited former President Ferdinand Marcos and/or his whether civil or criminal, are to be filed "with the Sandiganbayanwhich shall
wife * *, their close relatives, subordinates, business have exclusive and original jurisdiction thereof." 35 Executive Order No. 14
associates, duties, agents, or nominees from transferring, also pertinently provides that civil suits for restitution, reparation of damages,
conveying, encumbering, concealing or dissipating said or indemnification for consequential damages, forfeiture proceedings
assets or properties in the Philippines and abroad, pending provided for under Republic Act No. 1379, or any other civil actions under the
the outcome of appropriate proceedings in the Philippines to Civil Code or other existing laws, in connection with * * (said Executive
determine whether any such assets or properties were Orders Numbered 1 and 2) may be filed separately from and proceed
acquired by them through or as a result of improper or illegal independently of any criminal proceedings and may be proved by a
use of or the conversion of funds belonging to the preponderance of evidence;" and that, moreover, the "technical rules of
Government of the Philippines or any of its branches, procedure and evidence shall not be strictly applied to* * (said)civil cases." 36
instrumentalities, enterprises, banks or financial institutions,
or by taking undue advantage of their official position, 5. Contemplated Situations
authority, relationship, connection or influence to unjustly
enrich themselves at the expense and to the grave damage The situations envisaged and sought to be governed are self-evident, these
and prejudice of the Filipino people and the Republic of the being:
Philippines;
1) that "(i)ll-gotten properties (were) amassed by the leaders
3) prohibited "any person from transferring, conveying, and supporters of the previous regime"; 37
encumbering or otherwise depleting or concealing such
assets and properties or from assisting or taking part in their a) more particularly, that ill-gotten wealth (was) accumulated
transfer, encumbrance, concealment or dissipation under by former President Ferdinand E. Marcos, his immediate
pain of such penalties as are prescribed by law;" and family, relatives, subordinates and close associates, * *
located in the Philippines or abroad, * * (and) business
4) required "all persons in the Philippines holding such enterprises and entities (came to be) owned or controlled by
assets or properties, whether located in the Philippines or them, during * * (the Marcos) administration, directly or
abroad, in their names as nominees, agents or trustees, to through nominees, by taking undue advantage of their public
make full disclosure of the same to the Commission on Good office and/or using their powers, authority, influence,
Government within thirty (30) days from publication of * (the) Connections or relationship; 38
Executive Order, * *. 32
b) otherwise stated, that "there are assets and properties
d. Executive Order No. 14 purportedly pertaining to former President Ferdinand E.
Marcos, and/or his wife Mrs. Imelda Romualdez Marcos,

65
their close relatives, subordinates, business associates, property and free enterprise which are deemed pillars of a free society such
dummies, agents or nominees which had been or were as ours, and to which all members of that society may without exception lay
acquired by them directly or indirectly, through or as a result claim.
of the improper or illegal use of funds or properties owned by
the Government of the Philippines or any of its branches, * * Democracy, as a way of life enshrined in the Constitution,
instrumentalities, enterprises, banks or financial institutions, embraces as its necessary components freedom of
or by taking undue advantage of their office, authority, conscience, freedom of expression, and freedom in the
influence, connections or relationship, resulting in their pursuit of happiness. Along with these freedoms are
unjust enrichment and causing grave damage and prejudice included economic freedom and freedom of enterprise within
to the Filipino people and the Republic of the Philippines"; 39 reasonable bounds and under proper control. * * Evincing
much concern for the protection of property, the Constitution
c) that "said assets and properties are in the form of bank distinctly recognizes the preferred position which real estate
accounts. deposits, trust. accounts, shares of stocks, has occupied in law for ages. Property is bound up with
buildings, shopping centers, condominiums, mansions, every aspect of social life in a democracy as democracy is
residences, estates, and other kinds of real and personal conceived in the Constitution. The Constitution realizes the
properties in the Philippines and in various countries of the indispensable role which property, owned in reasonable
world;" 40 and quantities and used legitimately, plays in the stimulation to
economic effort and the formation and growth of a solid
2) that certain "business enterprises and properties (were) social middle class that is said to be the bulwark of
taken over by the government of the Marcos Administration democracy and the backbone of every progressive and
or by entities or persons close to former President Marcos. 41 happy country. 42

6. Government's Right and Duty to Recover All Ill-gotten Wealth a. Need of Evidentiary Substantiation in Proper Suit

There can be no debate about the validity and eminent propriety of the Consequently, the factual premises of the Executive Orders cannot simply be
Government's plan "to recover all ill-gotten wealth." assumed. They will have to be duly established by adequate proof in each
case, in a proper judicial proceeding, so that the recovery of the ill-gotten
Neither can there be any debate about the proposition that assuming the wealth may be validly and properly adjudged and consummated; although
above described factual premises of the Executive Orders and Proclamation there are some who maintain that the fact-that an immense fortune, and "vast
No. 3 to be true, to be demonstrable by competent evidence, the recovery resources of the government have been amassed by former President
from Marcos, his family and his dominions of the assets and properties Ferdinand E. Marcos, his immediate family, relatives, and close associates
involved, is not only a right but a duty on the part of Government. both here and abroad," and they have resorted to all sorts of clever schemes
and manipulations to disguise and hide their illicit acquisitions-is within the
realm of judicial notice, being of so extensive notoriety as to dispense with
But however plain and valid that right and duty may be, still a balance must
proof thereof, Be this as it may, the requirement of evidentiary substantiation
be sought with the equally compelling necessity that a proper respect be
has been expressly acknowledged, and the procedure to be followed
accorded and adequate protection assured, the fundamental rights of private
explicitly laid down, in Executive Order No. 14.

66
b. Need of Provisional Measures to Collect and Conserve unjust enrichment of the ostensible owner and grave damage and prejudice
Assets Pending Suits to the State. 44 And this, too, is the sense in which the term is commonly
understood in other jurisdictions. 45
Nor may it be gainsaid that pending the institution of the suits for the
recovery of such "ill-gotten wealth" as the evidence at hand may reveal, there b. "Freeze Order"
is an obvious and imperative need for preliminary, provisional measures to
prevent the concealment, disappearance, destruction, dissipation, or loss of A "freeze order" prohibits the person having possession or control of property
the assets and properties subject of the suits, or to restrain or foil acts that alleged to constitute "ill-gotten wealth" "from transferring, conveying,
may render moot and academic, or effectively hamper, delay, or negate encumbering or otherwise depleting or concealing such property, or from
efforts to recover the same. assisting or taking part in its transfer, encumbrance, concealment, or
dissipation." 46 In other words, it commands the possessor to hold the
7. Provisional Remedies Prescribed by Law property and conserve it subject to the orders and disposition of the authority
decreeing such freezing. In this sense, it is akin to a garnishment by which
To answer this need, the law has prescribed three (3) provisional remedies. the possessor or ostensible owner of property is enjoined not to deliver,
These are: (1) sequestration; (2) freeze orders; and (3) provisional takeover. transfer, or otherwise dispose of any effects or credits in his possession or
control, and thus becomes in a sense an involuntary depositary thereof. 47
Sequestration and freezing are remedies applicable generally to unearthed
instances of "ill-gotten wealth." The remedy of "provisional takeover" is c. Provisional Takeover
peculiar to cases where "business enterprises and properties (were) taken
over by the government of the Marcos Administration or by entities or In providing for the remedy of "provisional takeover," the law acknowledges
persons close to former President Marcos." 43 the apparent distinction between "ill gotten" "business enterprises and
entities" (going concerns, businesses in actual operation), generally, as to
a. Sequestration which the remedy of sequestration applies, it being necessarily inferred that
the remedy entails no interference, or the least possible interference with the
By the clear terms of the law, the power of the PCGG to sequester actual management and operations thereof; and "business enterprises which
property claimed to be "ill-gotten" means to place or cause to be placed were taken over by the government government of the Marcos
under its possession or control said property, or any building or office wherein Administration or by entities or persons close to him," in particular, as to
any such property and any records pertaining thereto may be found, which a "provisional takeover" is authorized, "in the public interest or to
including "business enterprises and entities,"-for the purpose of preventing prevent disposal or dissipation of the enterprises." 48 Such a "provisional
the destruction, concealment or dissipation of, and otherwise conserving and takeover" imports something more than sequestration or freezing, more than
preserving, the same-until it can be determined, through appropriate judicial the placing of the business under physical possession and control, albeit
proceedings, whether the property was in truth will- gotten," i.e., acquired without or with the least possible interference with the management and
through or as a result of improper or illegal use of or the conversion of funds carrying on of the business itself. In a "provisional takeover," what is taken
belonging to the Government or any of its branches, instrumentalities, into custody is not only the physical assets of the business enterprise or
enterprises, banks or financial institutions, or by taking undue advantage of entity, but the business operation as well. It is in fine the assumption of
official position, authority relationship, connection or influence, resulting in control not only over things, but over operations or on- going activities. But,

67
to repeat, such a "provisional takeover" is allowed only as regards "business There is thus no cause for the apprehension voiced by BASECO 50 that
enterprises * * taken over by the government of the Marcos Administration or sequestration, freezing or provisional takeover is designed to be an end in
by entities or persons close to former President Marcos." itself, that it is the device through which persons may be deprived of their
property branded as "ill-gotten," that it is intended to bring about a
d. No Divestment of Title Over Property Seized permanent, rather than a passing, transitional state of affairs. That this is not
so is quite explicitly declared by the governing rules.
It may perhaps be well at this point to stress once again the provisional,
contingent character of the remedies just described. Indeed the law plainly Be this as it may, the 1987 Constitution should allay any lingering fears about
qualifies the remedy of take-over by the adjective, "provisional." These the duration of these provisional remedies. Section 26 of its Transitory
remedies may be resorted to only for a particular exigency: to prevent in the Provisions, 51 lays down the relevant rule in plain terms, apart from extending
public interest the disappearance or dissipation of property or business, and ratification or confirmation (although not really necessary) to the institution by
conserve it pending adjudgment in appropriate proceedings of the primary presidential fiat of the remedy of sequestration and freeze orders:
issue of whether or not the acquisition of title or other right thereto by the
apparent owner was attended by some vitiating anomaly. None of the SEC. 26. The authority to issue sequestration or freeze
remedies is meant to deprive the owner or possessor of his title or any right orders under Proclamation No. 3 dated March 25, 1986 in
to the property sequestered, frozen or taken over and vest it in the relation to the recovery of ill-gotten wealth shag remain
sequestering agency, the Government or other person. This can be done operative for not more than eighteen months after the
only for the causes and by the processes laid down by law. ratification of this Constitution. However, in the national
interest, as certified by the President, the Congress may
That this is the sense in which the power to sequester, freeze or provisionally extend said period.
take over is to be understood and exercised, the language of the executive
orders in question leaves no doubt. Executive Order No. 1 declares that the A sequestration or freeze order shall be issued only upon
sequestration of property the acquisition of which is suspect shall last "until showing of a prima facie case. The order and the list of the
the transactions leading to such acquisition * * can be disposed of by the sequestered or frozen properties shall forthwith be registered
appropriate authorities." 49 Executive Order No. 2 declares that the assets or with the proper court. For orders issued before the
properties therein mentioned shall remain frozen "pending the outcome of ratification of this Constitution, the corresponding judicial
appropriate proceedings in the Philippines to determine whether any such action or proceeding shall be filed within six months from its
assets or properties were acquired" by illegal means. Executive Order No. 14 ratification. For those issued after such ratification, the
makes clear that judicial proceedings are essential for the resolution of the judicial action or proceeding shall be commenced within six
basic issue of whether or not particular assets are "ill-gotten," and resultant months from the issuance thereof.
recovery thereof by the Government is warranted.
The sequestration or freeze order is deemed automatically
e. State of Seizure Not To Be Indefinitely Maintained; The lifted if no judicial action or proceeding is commenced as
Constitutional Command herein provided. 52

f. Kinship to Attachment Receivership

68
As thus described, sequestration, freezing and provisional takeover are akin takeover, given its fundamental character of temporariness or conditionality;
to the provisional remedy of preliminary attachment, or receivership. 53 By and taking account specially of the constitutionally expressed "mandate of
attachment, a sheriff seizes property of a defendant in a civil suit so that it the people to recover ill-gotten properties amassed by the leaders and
may stand as security for the satisfaction of any judgment that may be supporters of the previous regime and protect the interest of the
obtained, and not disposed of, or dissipated, or lost intentionally or otherwise, people;" 59 as well as the obvious need to avoid alerting suspected
pending the action. 54 By receivership, property, real or personal, which is possessors of "ill-gotten wealth" and thereby cause that disappearance or
subject of litigation, is placed in the possession and control of a receiver loss of property precisely sought to be prevented, and the fact, just as self-
appointed by the Court, who shall conserve it pending final determination of evident, that "any transfer, disposition, concealment or disappearance of said
the title or right of possession over it. 55 All these remedies sequestration, assets and properties would frustrate, obstruct or hamper the efforts of the
freezing, provisional, takeover, attachment and receivership are Government" at the just recovery thereof. 60
provisional, temporary, designed for-particular exigencies, attended by no
character of permanency or finality, and always subject to the control of the 8. Requisites for Validity
issuing court or agency.
What is indispensable is that, again as in the case of attachment and
g. Remedies, Non-Judicial receivership, there exist a prima facie factual foundation, at least, for the
sequestration, freeze or takeover order, and adequate and fair opportunity to
Parenthetically, that writs of sequestration or freeze or takeover orders are contest it and endeavor to cause its negation or nullification. 61
not issued by a court is of no moment. The Solicitor General draws attention
to the writ of distraint and levy which since 1936 the Commissioner of Both are assured under the executive orders in question and the rules and
Internal Revenue has been by law authorized to issue against property of a regulations promulgated by the PCGG.
delinquent taxpayer. 56 BASECO itself declares that it has not manifested "a
rigid insistence on sequestration as a purely judicial remedy * * (as it feels) a. Prima Facie Evidence as Basis for Orders
that the law should not be ossified to a point that makes it insensitive to
change." What it insists on, what it pronounces to be its "unyielding position,
Executive Order No. 14 enjoins that there be "due regard to the requirements
is that any change in procedure, or the institution of a new one, should
of fairness and due process." 62Executive Order No. 2 declares that with
conform to due process and the other prescriptions of the Bill of Rights of the
respect to claims on allegedly "ill-gotten" assets and properties, "it is the
Constitution." 57 It is, to be sure, a proposition on which there can be no
position of the new democratic government that President Marcos * * (and
disagreement.
other parties affected) be afforded fair opportunity to contest these claims
before appropriate Philippine authorities." 63 Section 7 of the Commission's
h. Orders May Issue Ex Parte Rules and Regulations provides that sequestration or freeze (and takeover)
orders issue upon the authority of at least two commissioners, based on
Like the remedy of preliminary attachment and receivership, as well as the affirmation or complaint of an interested party, or motu proprio when the
delivery of personal property in replevinsuits, sequestration and provisional Commission has reasonable grounds to believe that the issuance thereof is
takeover writs may issue ex parte. 58 And as in preliminary attachment, warranted. 64 A similar requirement is now found in Section 26, Art. XVIII of
receivership, and delivery of personality, no objection of any significance may the 1987 Constitution, which requires that a "sequestration or freeze order
be raised to the ex parte issuance of an order of sequestration, freezing or shall be issued only upon showing of a prima facie case." 65

69
b. Opportunity to Contest approbation and sanction. As already mentioned, the Provisional or
"Freedom" Constitution recognizes the power and duty of the President to
And Sections 5 and 6 of the same Rules and Regulations lay down the enact "measures to achieve the mandate of the people to * * * (recover ill-
procedure by which a party may seek to set aside a writ of sequestration or gotten properties amassed by the leaders and supporters of the previous
freeze order, viz: regime and protect the interest of the people through orders of sequestration
or freezing of assets or accounts." And as also already adverted to, Section
SECTION 5. Who may contend.-The person against whom a 26, Article XVIII of the 1987 Constitution 67 treats of, and ratifies the "authority
writ of sequestration or freeze or hold order is directed may to issue sequestration or freeze orders under Proclamation No. 3 dated
request the lifting thereof in writing, either personally or March 25, 1986."
through counsel within five (5) days from receipt of the writ
or order, or in the case of a hold order, from date of The institution of these provisional remedies is also premised upon the
knowledge thereof. State's inherent police power, regarded, as t lie power of promoting the public
welfare by restraining and regulating the use of liberty and property," 68 and
SECTION 6. Procedure for review of writ or order.-After due as "the most essential, insistent and illimitable of powers * * in the promotion
hearing or motu proprio for good cause shown, the of general welfare and the public interest," 69and said to be co-extensive with
Commission may lift the writ or order unconditionally or self-protection and * * not inaptly termed (also) the'law of overruling
subject to such conditions as it may deem necessary, taking necessity." " 70
into consideration the evidence and the circumstance of the
case. The resolution of the commission may be appealed by 10. PCGG not a "Judge"; General Functions
the party concerned to the Office of the President of the
Philippines within fifteen (15) days from receipt thereof. It should also by now be reasonably evident from what has thus far been said
that the PCGG is not, and was never intended to act as, a judge. Its general
Parenthetically, even if the requirement for a prima facie showing of "ill- function is to conduct investigations in order to collect evidenceestablishing
gotten wealth" were not expressly imposed by some rule or regulation as a instances of "ill-gotten wealth;" issue sequestration, and such orders as may
condition to warrant the sequestration or freezing of property contemplated in be warranted by the evidence thus collected and as may be necessary to
the executive orders in question, it would nevertheless be exigible in this preserve and conserve the assets of which it takes custody and control and
jurisdiction in which the Rule of Law prevails and official acts which are prevent their disappearance, loss or dissipation; and eventually file and
devoid of rational basis in fact or law, or are whimsical and capricious, are prosecute in the proper court of competent jurisdiction all cases investigated
condemned and struck down. 66 by it as may be warranted by its findings. It does not try and decide, or hear
and determine, or adjudicate with any character of finality or compulsion,
9. Constitutional Sanction of Remedies cases involving the essential issue of whether or not property should be
forfeited and transferred to the State because "ill-gotten" within the meaning
of the Constitution and the executive orders. This function is reserved to the
If any doubt should still persist in the face of the foregoing considerations as
designated court, in this case, the Sandiganbayan. 71 There can therefore be
to the validity and propriety of sequestration, freeze and takeover orders, it
no serious regard accorded to the accusation, leveled by BASECO, 72 that
should be dispelled by the fact that these particular remedies and the
the PCGG plays the perfidious role of prosecutor and judge at the same time.
authority of the PCGG to issue them have received constitutional

70
11. Facts Preclude Grant of Relief to Petitioner BASECO's Stock and Transfer Book. 75 Their names and the number of
shares respectively held by them are as follows:
Upon these premises and reasoned conclusions, and upon the facts
disclosed by the record, hereafter to be discussed, the petition cannot
succeed. The writs of certiorari and prohibition prayed for will not be issued.
1. Jose A. Rojas 1,248 shares
The facts show that the corporation known as BASECO was owned or
controlled by President Marcos "during his administration, through nominees,
by taking undue advantage of his public office and/or using his powers,
authority, or influence, " and that it was by and through the same means, that 2. Severino G. de 1,248 shares
BASECO had taken over the business and/or assets of the National Shipyard la Cruz
and Engineering Co., Inc., and other government-owned or controlled
entities.

12. Organization and Stock Distribution of BASECO 3. Emilio T. Yap 2,508 shares

BASECO describes itself in its petition as "a shiprepair and shipbuilding


company * * incorporated as a domestic private corporation * * (on Aug. 30,
1972) by a consortium of Filipino shipowners and shipping executives. Its 4. Jose Fernandez 1,248 shares
main office is at Engineer Island, Port Area, Manila, where its Engineer Island
Shipyard is housed, and its main shipyard is located at Mariveles
Bataan." 73 Its Articles of Incorporation disclose that its authorized capital
stock is P60,000,000.00 divided into 60,000 shares, of which 12,000 shares 5. Jose Francisco 128 shares
with a value of P12,000,000.00 have been subscribed, and on said
subscription, the aggregate sum of P3,035,000.00 has been paid by the
incorporators. 74 The same articles Identify the incorporators, numbering
fifteen (15), as follows: (1) Jose A. Rojas, (2) Anthony P. Lee, (3) Eduardo T. 6. Manuel S. 96 shares
Marcelo, (4) Jose P. Fernandez, (5) Generoso Tanseco, (6) Emilio T. Yap, (7) Mendoza
Antonio M. Ezpeleta, (8) Zacarias Amante, (9) Severino de la Cruz, (10) Jose
Francisco, (11) Dioscoro Papa, (12) Octavio Posadas, (13) Manuel S.
Mendoza, (14) Magiliw Torres, and (15) Rodolfo Torres.
7. Anthony P. Lee 1,248 shares
By 1986, however, of these fifteen (15) incorporators, six (6) had ceased to
be stockholders, namely: (1) Generoso Tanseco, (2) Antonio Ezpeleta, (3)
Zacarias Amante, (4) Octavio Posadas, (5) Magiliw Torres, and (6) Rodolfo
Torres. As of this year, 1986, there were twenty (20) stockholders listed in

71
8. Hilario M. Ruiz 32 shares Drydock shares

9. Constante L. 8 shares 16. Manuel Jacela 1 share


Farias

17. Jonathan G. Lu 1 share


10. Fidelity 65,882
Management, Inc. shares

18. Jose J. 1 share


Tanchanco
11. Trident 7,412 shares
Management

19. Dioscoro Papa 128 shares

12. United Phil. 1,240 shares


Lines
20. Edward T. 4 shares
Marcelo

13. Renato M. 8 shares


Tanseco
TOTAL 218,819
shares.

14. Fidel Ventura 8 shares

13 Acquisition of NASSCO by BASECO

15. Metro Bay 136,370

72
Barely six months after its incorporation, BASECO acquired from National P10,047,940.00 of which, as set out in the document of sale, P2,000.000.00
Shipyard & Steel Corporation, or NASSCO, a government-owned or was paid upon its execution, and the balance stipulated to be payable in
controlled corporation, the latter's shipyard at Mariveles, Bataan, known as installments. 78
the Bataan National Shipyard (BNS), and except for NASSCO's Engineer
Island Shops and certain equipment of the BNS, consigned for future 16. Acquisition of Other Assets of NASSCO; Intervention of Marcos
negotiation all its structures, buildings, shops, quarters, houses, plants,
equipment and facilities, in stock or in transit. This it did in virtue of a Some nine months afterwards, or on July 15, 1975, to be precise, BASECO,
"Contract of Purchase and Sale with Chattel Mortgage" executed on again with the intervention of President Marcos, acquired ownership of the
February 13, 1973. The price was P52,000,000.00. As partial payment rest of the assets of NASSCO which had not been included in the first two (2)
thereof, BASECO delivered to NASSCO a cash bond of P11,400,000.00, purchase documents. This was accomplished by a deed entitled "Contract of
convertible into cash within twenty-four (24) hours from completion of the Purchase and Sale," 79which, like the Memorandum of Agreement dated
inventory undertaken pursuant to the contract. The balance of October 9, 1973 supra also bore at the upper right-hand corner of its first
P41,600,000.00, with interest at seven percent (7%) per annum, page, the handwritten notation of President Marcos reading, "APPROVED,
compounded semi-annually, was stipulated to be paid in equal semi-annual July 29, 1973," and underneath it, his usual full signature. Transferred to
installments over a term of nine (9) years, payment to commence after a BASECO were NASSCO's "ownership and all its titles, rights and interests
grace period of two (2) years from date of turnover of the shipyard to over all equipment and facilities including structures, buildings, shops,
BASECO. 76 quarters, houses, plants and expendable or semi-expendable assets, located
at the Engineer Island, known as the Engineer Island Shops, including all the
14. Subsequent Reduction of Price; Intervention of Marcos equipment of the Bataan National Shipyards (BNS) which were excluded
from the sale of NBS to BASECO but retained by BASECO and all other
Unaccountably, the price of P52,000,000.00 was reduced by more than one- selected equipment and machineries of NASSCO at J. Panganiban Smelting
half, to P24,311,550.00, about eight (8) months later. A document to this Plant." In the same deed, NASSCO committed itself to cooperate with
effect was executed on October 9, 1973, entitled "Memorandum Agreement," BASECO for the acquisition from the National Government or other
and was signed for NASSCO by Arturo Pacificador, as Presiding Officer of appropriate Government entity of Engineer Island. Consideration for the sale
the Board of Directors, and David R. Ines, as General Manager. 77 This was set at P5,000,000.00; a down payment of P1,000,000.00 appears to
agreement bore, at the top right corner of the first page, the word have been made, and the balance was stipulated to be paid at 7% interest
"APPROVED" in the handwriting of President Marcos, followed by his usual per annum in equal semi annual installments over a term of nine (9) years, to
full signature. The document recited that a down payment of P5,862,310.00 commence after a grace period of two (2) years. Mr. Arturo Pacificador again
had been made by BASECO, and the balance of P19,449,240.00 was signed for NASSCO, together with the general manager, Mr. David R. Ines.
payable in equal semi-annual installments over nine (9) years after a grace
period of two (2) years, with interest at 7% per annum. 17. Loans Obtained

15. Acquisition of 300 Hectares from Export Processing Zone Authority It further appears that on May 27, 1975 BASECO obtained a loan from the
NDC, taken from "the last available Japanese war damage fund of
On October 1, 1974, BASECO acquired three hundred (300) hectares of land $19,000,000.00," to pay for "Japanese made heavy equipment (brand
in Mariveles from the Export Processing Zone Authority for the price of new)." 80On September 3, 1975, it got another loan also from the NDC in the

73
amount of P30,000,000.00 (id.). And on January 28, 1976, it got still another Capt. A.T. Romualdez' report to the President was submitted eleven (11)
loan, this time from the GSIS, in the sum of P12,400,000.00. 81 The claim has days later. It opened with the following caption:
been made that not a single centavo has been paid on these loans. 82
MEMORANDUM:
18. Reports to President Marcos
FOR : The President
In September, 1977, two (2) reports were submitted to President Marcos
regarding BASECO. The first was contained in a letter dated September 5, SUBJECT: An Evaluation and Re-assessment of a
1977 of Hilario M. Ruiz, BASECO president. 83 The second was embodied in Performance of a Mission
a confidential memorandum dated September 16, 1977 of Capt. A.T.
Romualdez. 84 They further disclose the fine hand of Marcos in the affairs of FROM: Capt. A.T. Romualdez.
BASECO, and that of a Romualdez, a relative by affinity.
Like Ruiz, Romualdez wrote that BASECO faced great difficulties in meeting
a. BASECO President's Report its loan obligations due chiefly to the fact that "orders to build ships as
expected * * did not materialize."
In his letter of September 5, 1977, BASECO President Ruiz reported to
Marcos that there had been "no orders or demands for ship construction" for He advised that five stockholders had "waived and/or assigned their holdings
some time and expressed the fear that if that state of affairs persisted, inblank," these being: (1) Jose A. Rojas, (2) Severino de la Cruz, (3) Rodolfo
BASECO would not be able to pay its debts to the Government, which at the Torres, (4) Magiliw Torres, and (5) Anthony P. Lee. Pointing out that "Mr.
time stood at the not inconsiderable amount of P165,854,000.00. 85 He Magiliw Torres * * is already dead and Mr. Jose A. Rojas had a major heart
suggested that, to "save the situation," there be a "spin-off (of their) attack," he made the following quite revealing, and it may be added, quite
shipbuilding activities which shall be handled exclusively by an entirely new cynical and indurate recommendation, to wit:
corporation to be created;" and towards this end, he informed Marcos that
BASECO was
* * (that) their replacements (be effected) so we can register
their names in the stock book prior to the implementation
* * inviting NDC and LUSTEVECO to participate by of your instructions to pass a board resolution to legalize the
converting the NDC shipbuilding loan to BASECO amounting transfers under SEC regulations;
to P341.165M and assuming and converting a portion of
BASECO's shipbuilding loans from REPACOM amounting to
2. By getting their replacements, the families cannot
P52.2M or a total of P83.365M as NDC's equity contribution
question us later on; and
in the new corporation. LUSTEVECO will participate by
absorbing and converting a portion of the REPACOM loan of 87
Bay Shipyard and Drydock, Inc., amounting to P32.538M. 86 3. We will owe no further favors from them.

b. Romualdez' Report He also transmitted to Marcos, together with the report, the following
documents: 88

74
1. Stock certificates indorsed and assigned in blank with Capt. Romualdez also recommended that BASECO's loans be restructured
assignments and waivers; 89 "until such period when BASECO will have enough orders for ships in order
for the company to meet loan obligations," and that
2. The articles of incorporation, the amended articles, and
the by-laws of BASECO; An LOI may be issued to government agencies using floating
equipment, that a linkage scheme be applied to a certain
3. Deed of Sales, wherein NASSCO sold to BASECO four percent of BASECO's net profit as part of BASECO's
(4) parcels of land in "Engineer Island", Port Area, Manila; amortization payments to make it justifiable for you, Sir. 91

4. Transfer Certificate of Title No. 124822 in the name of It is noteworthy that Capt. A.T. Romualdez does not appear to be a
BASECO, covering "Engineer Island"; stockholder or officer of BASECO, yet he has presented a report on
BASECO to President Marcos, and his report demonstrates intimate
5. Contract dated October 9, 1973, between NASSCO and familiarity with the firm's affairs and problems.
BASECO re-structure and equipment at Mariveles, Bataan;
19. Marcos' Response to Reports
6. Contract dated July 16, 1975, between NASSCO and
BASECO re-structure and equipment at Engineer Island, President Marcos lost no time in acting on his subordinates'
Port Area Manila; recommendations, particularly as regards the "spin-off" and the "linkage
scheme" relative to "BASECO's amortization payments."
7. Contract dated October 1, 1974, between EPZA and
BASECO re 300 hectares of land at Mariveles, Bataan; a. Instructions re "Spin-Off"

8. List of BASECO's fixed assets; Under date of September 28, 1977, he addressed a Memorandum to
Secretary Geronimo Velasco of the Philippine National Oil Company and
9. Loan Agreement dated September 3, 1975, BASECO's Chairman Constante Farias of the National Development Company,
loan from NDC of P30,000,000.00; directing them "to participate in the formation of a new corporation resulting
from the spin-off of the shipbuilding component of BASECO along the
following guidelines:
10. BASECO-REPACOM Agreement dated May 27, 1975;

a. Equity participation of government shall be through


11. GSIS loan to BASECO dated January 28, 1976 of
LUSTEVECO and NDC in the amount of P115,903,000
P12,400,000.00 for the housing facilities for BASECO's rank-
consisting of the following obligations of BASECO which are
and-file employees. 90
hereby authorized to be converted to equity of the said new
corporation, to wit:

75
1. NDC P83,865,000 (P31.165M loan & BASECO, it was made to pay BASECO instead the amount
P52.2M Reparation) of P18.285M); 2) the shipbuilding equipment procured from
reparations through EPZA, now in the possession of
2. LUSTEVECO P32,538,000 (Reparation) BASECO and BSDI (Bay Shipyard & Drydocking, Inc.) be
transferred to LUSTEVECO through PNOC; and 3) the
b. Equity participation of government shall be in the form of shipbuilding equipment (thus) transferred be invested by
non- voting shares. LUSTEVECO, acting through PNOC and NDC, as the
government's equity participation in a shipbuilding
corporation to be established in partnership with the private
For immediate compliance. 92
sector.

Mr. Marcos' guidelines were promptly complied with by his subordinates.


xxx xxx xxx
Twenty-two (22) days after receiving their president's memorandum, Messrs.
Hilario M. Ruiz, Constante L. Farias and Geronimo Z. Velasco, in
representation of their respective corporations, executed a PRE- And so, through a simple letter of instruction and
INCORPORATION AGREEMENT dated October 20, 1977. 93 In it, they memorandum, BASECO's loan obligation to NDC and
undertook to form a shipbuilding corporation to be known as "PHIL-ASIA REPACOM * * in the total amount of P83.365M and BSD's
SHIPBUILDING CORPORATION," to bring to realization their president's REPACOM loan of P32.438M were wiped out and converted
instructions. It would seem that the new corporation ultimately formed was into non-voting preferred shares. 95
actually named "Philippine Dockyard Corporation (PDC)." 94
20. Evidence of Marcos'
b. Letter of Instructions No. 670
Ownership of BASECO
Mr. Marcos did not forget Capt. Romualdez' recommendation for a letter of
instructions. On February 14, 1978, he issued Letter of Instructions No. 670 It cannot therefore be gainsaid that, in the context of the proceedings at bar,
addressed to the Reparations Commission REPACOM the Philippine the actuality of the control by President Marcos of BASECO has been
National Oil Company (PNOC), the Luzon Stevedoring Company sufficiently shown.
(LUSTEVECO), and the National Development Company (NDC). What is
commanded therein is summarized by the Solicitor General, with pithy and Other evidence submitted to the Court by the Solicitor General proves that
not inaccurate observations as to the effects thereof (in italics), as follows: President Marcos not only exercised control over BASECO, but also that
he actually owns well nigh one hundred percent of its outstanding stock.
* * 1) the shipbuilding equipment procured by BASECO
through reparations be transferred to NDC subject to It will be recalled that according to petitioner- itself, as of April 23, 1986, there
reimbursement by NDC to BASECO (of) the amount of s were 218,819 shares of stock outstanding, ostensibly owned by twenty (20)
allegedly representing the handling and incidental expenses stockholders. 96 Four of these twenty are juridical persons: (1) Metro Bay
incurred by BASECO in the installation of said Drydock, recorded as holding 136,370 shares; (2) Fidelity Management,
equipment (so instead of NDC getting paid on its loan to Inc., 65,882 shares; (3) Trident Management,7,412 shares; and (4) United

76
Phil. Lines, 1,240 shares. The first three corporations, among themselves, subsequent recorded statements as a mere gesture of defiance rather than a
own an aggregate of 209,664 shares of BASECO stock, or 95.82% of the verifiable factual declaration.
outstanding stock.
By resolution dated September 25, 1986, this Court granted BASECO's
Now, the Solicitor General has drawn the Court's attention to the intriguing counsel a period of 10 days "to SUBMIT, as undertaken by him, * * the
circumstance that found in Malacanang shortly after the sudden flight of certificates of stock issued to the stockholders of * * BASECO as of April 23,
President Marcos, were certificates corresponding to more than ninety-five 1986, as listed in Annex 'P' of the petition.' 101 Counsel thereafter moved for
percent (95%) of all the outstanding shares of stock of BASECO, endorsed in extension; and in his motion dated October 2, 1986, he declared inter
blank, together with deeds of assignment of practically all the outstanding alia that "said certificates of stock are in the possession of third parties,
shares of stock of the three (3) corporations above mentioned (which among whom being the respondents themselves * * and petitioner is still
hold 95.82% of all BASECO stock), signed by the owners thereof although endeavoring to secure copies thereof from them." 102 On the same day he
not notarized. 97 filed another motion praying that he be allowed "to secure copies of the
Certificates of Stock in the name of Metro Bay Drydock, Inc., and of all other
More specifically, found in Malacanang (and now in the custody of the Certificates, of Stock of petitioner's stockholders in possession of
PCGG) were: respondents." 103

1) the deeds of assignment of all 600 outstanding shares of In a Manifestation dated October 10, 1986,, 104 the Solicitor General not
Fidelity Management Inc. which supposedly owns as unreasonably argued that counsel's aforestated motion to secure copies of
aforesaid 65,882 shares of BASECO stock; the stock certificates "confirms the fact that stockholders of petitioner
corporation are not in possession of * * (their) certificates of stock," and the
2) the deeds of assignment of 2,499,995 of the 2,500,000 reason, according to him, was "that 95% of said shares * * have been
outstanding shares of Metro Bay Drydock Corporation endorsed in blank and found in Malacaang after the former President and
which allegedly owns 136,370 shares of BASECO stock; his family fled the country." To this manifestation BASECO's counsel replied
on November 5, 1986, as already mentioned, Stubbornly insisting that the
firm's stockholders had not really assigned their stock. 105
3) the deeds of assignment of 800 outstanding shares of
Trident Management Co., Inc. which allegedly owns 7,412
shares of BASECO stock, assigned in blank; 98 and In view of the parties' conflicting declarations, this Court resolved on
November 27, 1986 among other things "to require * * the petitioner * *
to deposit upon proper receipt with Clerk of Court Juanito Ranjo the originals
4) stock certificates corresponding to 207,725 out of the
of the stock certificates alleged to be in its possession or accessible to it,
218,819 outstanding shares of BASECO stock; that is, all but
mentioned and described in Annex 'P' of its petition, (and other pleadings) * *
5 % all endorsed in blank. 99
within ten (10) days from notice." 106 In a motion filed on December 5,
1986, 107 BASECO's counsel made the statement, quite surprising in the
While the petitioner's counsel was quick to dispute this asserted fact, premises, that "it will negotiate with the owners (of the BASECO stock in
assuring this Court that the BASECO stockholders were still in possession of question) to allow petitioner to borrow from them, if available, the certificates
their respective stock certificates and had "never endorsed * * them in blank referred to" but that "it needs a more sufficient time therefor" (sic). BASECO's
or to anyone else," 100 that denial is exposed by his own prior and counsel however eventually had to confess inability to produce the originals

77
of the stock certificates, putting up the feeble excuse that while he had provisional takeover of the corporation in the public interest, in accordance
"requested the stockholders to allow * * (him) to borrow said certificates, * * with the terms of Executive Orders No. 1 and 2, pending the filing of the
some of * * (them) claimed that they had delivered the certificates to third requisite actions with the Sandiganbayan to cause divestment of title thereto
parties by way of pledge and/or to secure performance of obligations, while from Marcos, and its adjudication in favor of the Republic pursuant to
others allegedly have entrusted them to third parties in view of last national Executive Order No. 14.
emergency." 108 He has conveniently omitted, nor has he offered to give the
details of the transactions adverted to by him, or to explain why he had not As already earlier stated, this Court agrees that this assessment of the facts
impressed on the supposed stockholders the primordial importance of is correct; accordingly, it sustains the acts of sequestration and takeover by
convincing this Court of their present custody of the originals of the stock, or the PCGG as being in accord with the law, and, in view of what has thus far
if he had done so, why the stockholders are unwilling to agree to some sort been set out in this opinion, pronounces to be without merit the theory that
of arrangement so that the originals of their certificates might at the very least said acts, and the executive orders pursuant to which they were done, are
be exhibited to the Court. Under the circumstances, the Court can only fatally defective in not according to the parties affected prior notice and
conclude that he could not get the originals from the stockholders for the hearing, or an adequate remedy to impugn, set aside or otherwise obtain
simple reason that, as the Solicitor General maintains, said stockholders in relief therefrom, or that the PCGG had acted as prosecutor and judge at the
truth no longer have them in their possession, these having already been same time.
assigned in blank to then President Marcos.
22. Executive Orders Not a Bill of Attainder
21. Facts Justify Issuance of Sequestration and Takeover Orders
Neither will this Court sustain the theory that the executive orders in question
In the light of the affirmative showing by the Government that, prima facie at are a bill of attainder. 110 "A bill of attainder is a legislative act which inflicts
least, the stockholders and directors of BASECO as of April, 1986 109 were punishment without judicial trial." 111 "Its essence is the substitution of a
mere "dummies," nominees or alter egos of President Marcos; at any rate, legislative for a judicial determination of guilt." 112
that they are no longer owners of any shares of stock in the corporation, the
conclusion cannot be avoided that said stockholders and directors have no In the first place, nothing in the executive orders can be reasonably
basis and no standing whatever to cause the filing and prosecution of the construed as a determination or declaration of guilt. On the contrary, the
instant proceeding; and to grant relief to BASECO, as prayed for in the executive orders, inclusive of Executive Order No. 14, make it perfectly clear
petition, would in effect be to restore the assets, properties and business that any judgment of guilt in the amassing or acquisition of "ill-gotten wealth"
sequestered and taken over by the PCGG to persons who are "dummies," is to be handed down by a judicial tribunal, in this case,
nominees or alter egos of the former president. the Sandiganbayan, upon complaint filed and prosecuted by the PCGG. In
the second place, no punishment is inflicted by the executive orders, as the
From the standpoint of the PCGG, the facts herein stated at some length do merest glance at their provisions will immediately make apparent. In no
indeed show that the private corporation known as BASECO was "owned or sense, therefore, may the executive orders be regarded as a bill of attainder.
controlled by former President Ferdinand E. Marcos * * during his
administration, * * through nominees, by taking advantage of * * (his) public 23. No Violation of Right against Self-Incrimination and Unreasonable
office and/or using * * (his) powers, authority, influence * *," and that Searches and Seizures
NASSCO and other property of the government had been taken over by
BASECO; and the situation justified the sequestration as well as the

78
BASECO also contends that its right against self incrimination and * * The corporation is a creature of the state. It is presumed
unreasonable searches and seizures had been transgressed by the Order of to be incorporated for the benefit of the public. It received
April 18, 1986 which required it "to produce corporate records from 1973 to certain special privileges and franchises, and holds them
1986 under pain of contempt of the Commission if it fails to do so." The order subject to the laws of the state and the limitations of its
was issued upon the authority of Section 3 (e) of Executive Order No. 1, charter. Its powers are limited by law. It can make no
treating of the PCGG's power to "issue subpoenas requiring * * the contract not authorized by its charter. Its rights to act as a
production of such books, papers, contracts, records, statements of accounts corporation are only preserved to it so long as it obeys the
and other documents as may be material to the investigation conducted by laws of its creation. There is a reserve right in the legislature
the Commission, " and paragraph (3), Executive Order No. 2 dealing with its to investigate its contracts and find out whether it has
power to "require all persons in the Philippines holding * * (alleged "ill- exceeded its powers. It would be a strange anomaly to hold
gotten") assets or properties, whether located in the Philippines or abroad, in that a state, having chartered a corporation to make use of
their names as nominees, agents or trustees, to make full disclosure of the certain franchises, could not, in the exercise of sovereignty,
same * *." The contention lacks merit. inquire how these franchises had been employed, and
whether they had been abused, and demand the production
It is elementary that the right against self-incrimination has no application to of the corporate books and papers for that purpose. The
juridical persons. defense amounts to this, that an officer of the corporation
which is charged with a criminal violation of the statute may
While an individual may lawfully refuse to answer plead the criminality of such corporation as a refusal to
incriminating questions unless protected by an immunity produce its books. To state this proposition is to answer
statute, it does not follow that a corporation, vested with it. While an individual may lawfully refuse to answer
special privileges and franchises, may refuse to show its incriminating questions unless protected by an immunity
hand when charged with an abuse ofsuchprivileges * * 113 statute, it does not follow that a corporation, vested with
special privileges and franchises may refuse to show its
hand when charged with an abuse of such
Relevant jurisprudence is also cited by the Solicitor General. 114
privileges. (Wilson v. United States, 55 Law Ed., 771, 780
[emphasis, the Solicitor General's])
* * corporations are not entitled to all of the constitutional
protections which private individuals have. * * They are not
At any rate, Executive Order No. 14-A, amending Section 4 of Executive
at all within the privilege against self-incrimination, although
Order No. 14 assures protection to individuals required to produce evidence
this court more than once has said that the privilege runs
before the PCGG against any possible violation of his right against self-
very closely with the 4th Amendment's Search and Seizure
incrimination. It gives them immunity from prosecution on the basis of
provisions. It is also settled that an officer of the company
testimony or information he is compelled to present. As amended, said
cannot refuse to produce its records in its possession upon
Section 4 now provides that
the plea that they will either incriminate him or may
incriminate it." (Oklahoma Press Publishing Co. v. Walling,
327 U.S. 186; emphasis, the Solicitor General's). xxx xxx xxx

79
The witness may not refuse to comply with the order on the Equally evident is that the resort to the provisional remedies in question
basis of his privilege against self-incrimination; but no should entail the least possible interference with business operations or
testimony or other information compelled under the order (or activities so that, in the event that the accusation of the business enterprise
any information directly or indirectly derived from such being "ill gotten" be not proven, it may be returned to its rightful owner as far
testimony, or other information) may be used against the as possible in the same condition as it was at the time of sequestration.
witness in any criminal case, except a prosecution for
perjury, giving a false statement, or otherwise failing to b. PCGG Has Only Powers of Administration
comply with the order.
The PCGG may thus exercise only powers of administration over the
The constitutional safeguard against unreasonable searches and seizures property or business sequestered or provisionally taken over, much like a
finds no application to the case at bar either. There has been no search court-appointed receiver, 115 such as to bring and defend actions in its own
undertaken by any agent or representative of the PCGG, and of course no name; receive rents; collect debts due; pay outstanding debts; and generally
seizure on the occasion thereof. do such other acts and things as may be necessary to fulfill its mission as
conservator and administrator. In this context, it may in addition enjoin or
24. Scope and Extent of Powers of the PCGG restrain any actual or threatened commission of acts by any person or entity
that may render moot and academic, or frustrate or otherwise make
One other question remains to be disposed of, that respecting the scope and ineffectual its efforts to carry out its task; punish for direct or indirect
extent of the powers that may be wielded by the PCGG with regard to the contempt in accordance with the Rules of Court; and seek and secure the
properties or businesses placed under sequestration or provisionally taken assistance of any office, agency or instrumentality of the government. 116 In
over. Obviously, it is not a question to which an answer can be easily given, the case of sequestered businesses generally (i.e., going concerns,
much less one which will suffice for every conceivable situation. businesses in current operation), as in the case of sequestered objects, its
essential role, as already discussed, is that of conservator, caretaker,
a. PCGG May Not Exercise Acts of Ownership "watchdog" or overseer. It is not that of manager, or innovator, much less an
owner.
One thing is certain, and should be stated at the outset: the PCGG cannot
exercise acts of dominion over property sequestered, frozen or provisionally c. Powers over Business Enterprises Taken Over by Marcos
taken over. AS already earlier stressed with no little insistence, the act of or Entities or Persons Close to him; Limitations Thereon
sequestration; freezing or provisional takeover of property does not import or
bring about a divestment of title over said property; does not make the PCGG Now, in the special instance of a business enterprise shown by evidence to
the owner thereof. In relation to the property sequestered, frozen or have been "taken over by the government of the Marcos Administration or by
provisionally taken over, the PCGG is a conservator, not an entities or persons close to former President Marcos," 117 the PCGG is
owner. Therefore, it can not perform acts of strict ownership; and this is given power and authority, as already adverted to, to "provisionally take (it)
specially true in the situations contemplated by the sequestration rules over in the public interest or to prevent * * (its) disposal or dissipation;" and
where, unlike cases of receivership, for example, no court exercises effective since the term is obviously employed in reference to going concerns, or
supervision or can upon due application and hearing, grant authority for the business enterprises in operation, something more than mere physical
performance of acts of dominion. custody is connoted; the PCGG may in this case exercise some measure of

80
control in the operation, running, or management of the business itself. But constitute the controlling or a substantial part of the corporate voting power.
even in this special situation, the intrusion into management should be The stock is not to be voted to replace directors, or revise the articles or by-
restricted to the minimum degree necessary to accomplish the legislative will, laws, or otherwise bring about substantial changes in policy, program or
which is "to prevent the disposal or dissipation" of the business enterprise. practice of the corporation except for demonstrably weighty and defensible
There should be no hasty, indiscriminate, unreasoned replacement or grounds, and always in the context of the stated purposes of sequestration or
substitution of management officials or change of policies, particularly in provisional takeover, i.e., to prevent the dispersion or undue disposal of the
respect of viable establishments. In fact, such a replacement or substitution corporate assets. Directors are not to be voted out simply because the power
should be avoided if at all possible, and undertaken only when justified by to do so exists. Substitution of directors is not to be done without reason or
demonstrably tenable grounds and in line with the stated objectives of the rhyme, should indeed be shunned if at an possible, and undertaken only
PCGG. And it goes without saying that where replacement of management when essential to prevent disappearance or wastage of corporate property,
officers may be called for, the greatest prudence, circumspection, care and and always under such circumstances as assure that the replacements are
attention - should accompany that undertaking to the end that truly truly possessed of competence, experience and probity.
competent, experienced and honest managers may be recruited. There
should be no role to be played in this area by rank amateurs, no matter how In the case at bar, there was adequate justification to vote the incumbent
wen meaning. The road to hell, it has been said, is paved with good directors out of office and elect others in their stead because the evidence
intentions. The business is not to be experimented or played around with, not showed prima facie that the former were just tools of President Marcos and
run into the ground, not driven to bankruptcy, not fleeced, not ruined. Sight were no longer owners of any stock in the firm, if they ever were at all. This is
should never be lost sight of the ultimate objective of the whole exercise, why, in its Resolution of October 28, 1986; 118 this Court declared that
which is to turn over the business to the Republic, once judicially established
to be "ill-gotten." Reason dictates that it is only under these conditions and Petitioner has failed to make out a case of grave abuse or
circumstances that the supervision, administration and control of business excess of jurisdiction in respondents' calling and holding of a
enterprises provisionally taken over may legitimately be exercised. stockholders' meeting for the election of directors as
authorized by the Memorandum of the President * * (to the
d. Voting of Sequestered Stock; Conditions Therefor PCGG) dated June 26, 1986, particularly, where as in this
case, the government can, through its designated directors,
So, too, it is within the parameters of these conditions and circumstances properly exercise control and management over what appear
that the PCGG may properly exercise the prerogative to vote sequestered to be properties and assets owned and belonging to the
stock of corporations, granted to it by the President of the Philippines through government itself and over which the persons who appear in
a Memorandum dated June 26, 1986. That Memorandum authorizes the this case on behalf of BASECO have failed to show any right
PCGG, "pending the outcome of proceedings to determine the ownership of * or even any shareholding in said corporation.
* (sequestered) shares of stock," "to vote such shares of stock as it may
have sequestered in corporations at all stockholders' meetings called for the It must however be emphasized that the conduct of the PCGG nominees in
election of directors, declaration of dividends, amendment of the Articles of the BASECO Board in the management of the company's affairs should
Incorporation, etc." The Memorandum should be construed in such a manner henceforth be guided and governed by the norms herein laid down. They
as to be consistent with, and not contradictory of the Executive Orders earlier should never for a moment allow themselves to forget that they are
promulgated on the same matter. There should be no exercise of the right to conservators, not owners of the business; they are fiduciaries, trustees, of
vote simply because the right exists, or because the stocks sequestered

81
whom the highest degree of diligence and rectitude is, in the premises,
required.

25. No Sufficient Showing of Other Irregularities

As to the other irregularities complained of by BASECO, i.e., the cancellation


or revision, and the execution of certain contracts, inclusive of the termination
of the employment of some of its executives, 119 this Court cannot, in the
present state of the evidence on record, pass upon them. It is not necessary
to do so. The issues arising therefrom may and will be left for initial
determination in the appropriate action. But the Court will state that absent
any showing of any important cause therefor, it will not normally substitute its
judgment for that of the PCGG in these individual transactions. It is clear
however, that as things now stand, the petitioner cannot be said to have
established the correctness of its submission that the acts of the PCGG in
question were done without or in excess of its powers, or with grave abuse of
discretion.

WHEREFORE, the petition is dismissed. The temporary restraining order


issued on October 14, 1986 is lifted. G.R. No. L-22619 December 2, 1924

Yap, Fernan, Paras, Gancayco and Sarmiento, JJ., concur. NATIONAL COAL COMPANY, plaintiff-appellee,
vs.
THE COLLECTOR OF INTERNAL REVENUE, defendant-appellant.

Attorney-General Villa-Real for appellant.


Perfecto J. Salas Rodriguez for appellee.

JOHNSON, J.:

This action was brought in the Court of First Instance of the City of Manila on
the 17th day of July, 1923, for the purpose of recovering the sum of
P12,044.68, alleged to have been paid under protest by the plaintiff company
to the defendant, as specific tax on 24,089.3 tons of coal. Said company is a

82
corporation created by Act No. 2705 of the Philippine Legislature for the The question confronting us in this appeal is whether the plaintiff is subject to
purpose of developing the coal industry in the Philippine Islands and is the taxes under section 15 of Act No. 2719, or to the specific taxes under
actually engaged in coal mining on reserved lands belonging to the section 1496 of the Administrative Code.
Government. It claimed exemption from taxes under the provision of sections
14 and 15 of Act No. 2719, and prayed for a judgment ordering the defendant The plaintiff corporation was created on the 10th day of March, 1917, by Act
to refund to the plaintiff said sum of P12,044.68, with legal interest from the No. 2705, for the purpose of developing the coal industry in the Philippine
date of the presentation of the complaint, and costs against the defendant. Island, in harmony with the general plan of the Government to encourage the
development of the natural resources of the country, and to provided facilities
The defendant answered denying generally and specifically all the material therefor. By said Act, the company was granted the general powers of a
allegations of the complaint, except the legal existence and personality of the corporation "and such other powers as may be necessary to enable it to
plaintiff. As a special defense, the defendant alleged (a) that the sum of prosecute the business of developing coal deposits in the Philippine Island
P12,044.68 was paid by the plaintiff without protests, and (b) that said sum and of mining, extracting, transporting and selling the coal contained in said
was due and owing from the plaintiff to the Government of the Philippine deposits." (Sec. 2, Act No. 2705.) By the same law (Act No. 2705) the
Islands under the provisions of section 1496 of the Administrative Code and Government of the Philippine Islands is made the majority stockholder,
prayed that the complaint be dismissed, with costs against the plaintiff. evidently in order to insure proper government supervision and control, and
thus to place the Government in a position to render all possible
Upon the issue thus presented, the case was brought on for trial. After a encouragement, assistance and help in the prosecution and furtherance of
consideration of the evidence adduced by both parties, the Honorable Pedro the company's business.
Conception, judge, held that the words "lands owned by any person, etc.," in
section 15 of Act No. 2719 should be understood to mean "lands held in On May 14, 1917, two months after the passage of Act No. 2705, creating
lease or usufruct," in harmony with the other provision of said Act; that the the National Coal Company, the Philippine Legislature passed Act No. 2719
coal lands possessed by the plaintiff, belonging to the Government, fell within "to provide for the leasing and development of coal lands in the Philippine
the provisions of section 15 of Act No. 2719; and that a tax of P0.04 per ton Islands." On October 18, 1917, upon petition of the National Coal Company,
of 1,016 kilos on each ton of coal extracted therefrom, as provided in said the Governor-General, by Proclamation No. 39, withdrew "from settlement,
section, was the only tax which should be collected from the plaintiff; and entry, sale or other disposition, all coal-bearing public lands within the
sentenced the defendant to refund to the plaintiff the sum of P11,081.11 Province of Zamboanga, Department of Mindanao and Sulu, and the Island
which is the difference between the amount collected under section 1496 of of Polillo, Province of Tayabas." Almost immediately after the issuance of
the Administrative Code and the amount which should have been collected said proclamation the National Coal Company took possession of the coal
under the provisions of said section 15 of Act No. 2719. From that sentence lands within the said reservation, with an area of about 400 hectares, without
the defendant appealed, and now makes the following assignments of error: any further formality, contract or lease. Of the 30,000 shares of stock issued
by the company, the Government of the Philippine Islands is the owner of
I. The court below erred in holding that section 15 of Act No. 2719 does not 29,809 shares, that is, of 99 1/3 per centum of the whole capital stock.
refer to coal lands owned by persons and corporations.
If we understand the theory of the plaintiff-appellee, it is, that it claims to be
II. The court below erred in holding that the plaintiff was not subject to the tax the owner of the land from which it has mined the coal in question and is
prescribed in section 1496 of the Administrative Code. therefore subject to the provisions of section 15 of Act No. 2719 and not to
the provisions of the section 1496 of the Administrative Code. That

83
contention of the plaintiff leads us to an examination of the evidence upon said Act (No. 2705). No provisions of Act No. 2705 are found to be
the question of the ownership of the land from which the coal in question was inconsistent with the provisions of the Corporation Law. As a private
mined. Was the plaintiff the owner of the land from which the coal in question corporation, it has no greater rights, powers or privileges than any other
was mined? If the evidence shows the affirmative, then the judgment should corporation which might be organized for the same purpose under the
be affirmed. If the evidence shows that the land does not belong to the Corporation Law, and certainly it was not the intention of the Legislature to
plaintiff, then the judgment should be reversed, unless the plaintiff's rights fall give it a preference or right or privilege over other legitimate private
under section 3 of said Act. corporations in the mining of coal. While it is true that said proclamation No.
39 withdrew "from settlement, entry, sale, or other disposition of coal-bearing
The only witness presented by the plaintiff upon the question of the public lands within the Province of Zamboanga . . . and the Island of Polillo,"
ownership of the land in question was Mr. Dalmacio Costas, who stated that it made no provision for the occupation and operation by the plaintiff, to the
he was a member of the board of directors of the plaintiff corporation; that the exclusion of other persons or corporations who might, under proper
plaintiff corporation took possession of the land in question by virtue of the permission, enter upon the operate coal mines.
proclamation of the Governor-General, known as Proclamation No. 39 of the
year 1917; that no document had been issued in favor of the plaintiff On the 14th day of May, 1917, and before the issuance of said proclamation,
corporation; that said corporation had received no permission from the the Legislature of the Philippine Island in "an Act for the leasing and
Secretary of Agriculture and Natural Resources; that it took possession of development of coal lands in the Philippine Islands" (Act No. 2719), made
said lands covering an area of about 400 hectares, from which the coal in liberal provision. Section 1 of said Act provides: "Coal-bearing lands of the
question was mined, solely, by virtue of said proclamation (Exhibit B, No. 39). public domain in the Philippine Island shall not be disposed of in any manner
except as provided in this Act," thereby giving a clear indication that no "coal-
Said proclamation (Exhibit B) was issued by Francis Burton Harrison, then bearing lands of the public domain" had been disposed of by virtue of said
Governor-General, on the 18th day of October, 1917, and provided: proclamation.
"Pursuant to the provision of section 71 of Act No. 926, I hereby withdraw
from settlement, entry, sale, or other disposition, all coal-bearing public lands Neither is there any provision in Act No. 2705 creating the National Coal
within the Province of Zamboanga, Department of Mindanao and Sulu, and Company, nor in the amendments thereof found in Act No. 2822, which
the Island of Polillo, Province of Tayabas." It will be noted that said authorizes the National Coal Company to enter upon any of the reserved
proclamation only provided that all coal-bearing public lands within said coal lands without first having obtained permission from the Secretary of
province and island should be withdrawn from settlement, entry, sale, or Agriculture and Natural Resources.lawphi1.net
other disposition. There is nothing in said proclamation which authorizes the
plaintiff or any other person to enter upon said reversations and to mine coal, The following propositions are fully sustained by the facts and the law:
and no provision of law has been called to our attention, by virtue of which
the plaintiff was entitled to enter upon any of the lands so reserved by said (1) The National Coal Company is an ordinary private corporation organized
proclamation without first obtaining permission therefor. under Act No. 2705, and has no greater powers nor privileges than the
ordinary private corporation, except those mentioned, perhaps, in section 10
The plaintiff is a private corporation. The mere fact that the Government of Act No. 2719, and they do not change the situation here.
happens to the majority stockholder does not make it a public corporation.
Act No. 2705, as amended by Act No. 2822, makes it subject to all of the
provisions of the Corporation Law, in so far as they are not inconsistent with

84
(2) It mined on public lands between the month of July, 1920, and the months Resources of "unreserved, unappropriated coal-bearing public lands," and
of March, 1922, 24,089.3 tons of coal. the obligation to the Government which shall be imposed by said Secretary
upon the lessee.lawphi1.net
(3) Upon demand of the Collector of Internal Revenue it paid a tax of P0.50 a
ton, as taxes under the provisions of article 1946 of the Administrative Code Third. The internal revenue duty and tax which must be paid upon coal-
on the 15th day of December, 1922. bearing lands owned by any person, firm, association or corporation.

(4) It is admitted that it is neither the owner nor the lessee of the lands upon To repeat, it will be noted, first, that Act No. 2719 provides an internal
which said coal was mined. revenue duty and tax upon unreserved, unappropriated coal-bearing public
lands which may be leased by the Secretary of Agriculture and Natural
(5) The proclamation of Francis Burton Harrison, Governor-General, of the Resources; and, second, that said Act (No. 2719) provides an internal
18th day of October, 1917, by authority of section 1 of Act No. 926, revenue duty and tax imposed upon any person, firm, association or
withdrawing from settlement, entry, sale, or other dispositon all coal-bearing corporation, who may be the owner of "coal-bearing lands." A reading of said
public lands within the Province of Zamboanga and the Island of Polillo, was Act clearly shows that the tax imposed thereby is imposed upon two classes
not a reservation for the benefit of the National Coal Company, but for any of persons only lessees and owners.
person or corporation of the Philippine Islands or of the United States.
The lower court had some trouble in determining what was the correct
(6) That the National Coal Company entered upon said land and mined said interpretation of section 15 of said Act, by reason of what he believed to be
coal, so far as the record shows, without any lease or other authority from some difference in the interpretation of the language used in Spanish and
either the Secretary of Agriculture and Natural Resources or any person English. While there is some ground for confusion in the use of the language
having the power to grant a leave or authority. in Spanish and English, we are persuaded, considering all the provisions of
said Act, that said section 15 has reference only to persons, firms,
From all of the foregoing facts we find that the issue is well defined between associations or corporations which had already, prior to the existence of said
the plaintiff and the defendant. The plaintiff contends that it was liable only to Act, become the owners of coal lands. Section 15 cannot certainty refer to
pay the internal revenue and other fees and taxes provided for under section "holders or lessees of coal lands' for the reason that practically all of the
15 of Act No. 2719; while the defendant contends, under the facts of record, other provisions of said Act has reference to lessees or holders. If section 15
the plaintiff is obliged to pay the internal revenue duty provided for in section means that the persons, firms, associations, or corporation mentioned
1496 of the Administrative Code. That being the issue, an examination of the therein are holders or lessees of coal lands only, it is difficult to understand
provisions of Act No. 2719 becomes necessary. why the internal revenue duty and tax in said section was made different
from the obligations mentioned in section 3 of said Act, imposed upon
lessees or holders.
An examination of said Act (No. 2719) discloses the following facts important
for consideration here:
From all of the foregoing, it seems to be made plain that the plaintiff is neither
a lessee nor an owner of coal-bearing lands, and is, therefore, not subject to
First. All "coal-bearing lands of the public domain in the Philippine Islands
any other provisions of Act No. 2719. But, is the plaintiff subject to the
shall not be disposed of in any manner except as provided in this Act."
provisions of section 1496 of the Administrative Code?
Second. Provisions for leasing by the Secretary of Agriculture and Natural

85
Section 1496 of the Administrative Code provides that "on all coal and coke
there shall be collected, per metric ton, fifty centavos." Said section (1496) is
a part of article, 6 which provides for specific taxes. Said article provides for a
specific internal revenue tax upon all things manufactured or produced in the
Philippine Islands for domestic sale or consumption, and upon things
imported from the United States or foreign countries. It having been
demonstrated that the plaintiff has produced coal in the Philippine Islands
and is not a lessee or owner of the land from which the coal was produced,
we are clearly of the opinion, and so hold, that it is subject to pay the internal
revenue tax under the provisions of section 1496 of the Administrative Code,
and is not subject to the payment of the internal revenue tax under section 15
of Act No. 2719, nor to any other provisions of said Act.

Therefore, the judgment appealed from is hereby revoked, and the defendant
is hereby relieved from all responsibility under the complaint. And, without
any finding as to costs, it is so ordered.

Street, Malcolm, Avancea, Villamor, Ostrand and Romualdez, JJ., concur.

PHILIPPINE SOCIETY FOR G.R. No. 169752

THE PREVENTION OF

CRUELTY TO ANIMALS,

Petitioners, Members:

PUNO, C.J.

86
QUISUMBING, VELASCO, JR.,

NACHURA, and
YNARES-SANTIAGO,
REYES, JJ.

SANDOVAL-GUTIERREZ,
COMMISSION ON AUDIT,

CARPIO, DIR. RODULFO J. ARIESGA

(in his official capacity as Director


AUSTRIA-MARTINEZ,
of the Commission on Audit), MS.

CORONA, MERLE M. VALENTIN and MS.

SUSAN GUARDIAN (in their official


- versus - CARPIO-MORALES,
capacities as Team Leader and Team

Member, respectively, of the audit


AZCUNA,

Team of the Commission on Audit), Promulgated:

TINGA,

Respondents. September 25, 2007

CHICO-NAZARIO,

x-------------------------------------------------------
----x
GARCIA,
DECISION

87
For the purpose of enhancing its powers in promoting animal welfare and
enforcing laws for the protection of animals, the petitioner was initially
AUSTRIA-MARTINEZ, J.: imbued under its charter with the power to apprehend violators of animal
welfare laws. In addition, the petitioner was to share one-half (1/2) of the
fines imposed and collected through its efforts for violations of the laws
related thereto. As originally worded, Sections 4 and 5 of Act No. 1285
provide:
Before the Court is a special civil action for Certiorari and Prohibition under
Rule 65 of the Rules of Court, in relation to Section 2 of Rule 64, filed by the
petitioner assailing Office Order No. 2005-021[1] dated September 14, 2005
issued by the respondents which constituted the audit team, as well as its
September 23, 2005 Letter[2] informing the petitioner that respondents audit SEC. 4. The said society is authorized to appoint not
team shall conduct an audit survey on the petitioner for a detailed audit of its to exceed five agents in the City of Manila, and not to exceed
accounts, operations, and financial transactions. No temporary restraining two in each of the provinces of the Philippine Islands who
order was issued. shall have all the power and authority of a police officer to
make arrests for violation of the laws enacted for the
prevention of cruelty to animals and the protection of
animals, and to serve any process in connection with the
execution of such laws; and in addition thereto, all the police
The petitioner was incorporated as a juridical entity over one hundred years
force of the Philippine Islands, wherever organized, shall, as
ago by virtue of Act No. 1285, enacted on January 19, 1905, by the
occasion requires, assist said society, its members or
Philippine Commission. The petitioner, at the time it was created, was
agents, in the enforcement of all such laws.
composed of animal aficionados and animal propagandists. The objects of
the petitioner, as stated in Section 2 of its charter, shall be to enforce laws
relating to cruelty inflicted upon animals or the protection of animals in the
Philippine Islands, and generally, to do and perform all things which may tend
in any way to alleviate the suffering of animals and promote their welfare. [3] SEC. 5. One-half of all the fines imposed and
collected through the efforts of said society, its members or
its agents, for violations of the laws enacted for the
prevention of cruelty to animals and for their protection, shall
belong to said society and shall be used to promote its
At the time of the enactment of Act No. 1285, the original Corporation Law,
objects.
Act No. 1459, was not yet in existence. Act No. 1285 antedated both the
Corporation Law and the constitution of the Securities and Exchange
Commission. Important to note is that the nature of the petitioner as a
corporate entity is distinguished from the sociedad anonimasunder the
Spanish Code of Commerce. (emphasis supplied)

88
Subsequently, however, the power to make arrests as well as the privilege to
retain a portion of the fines collected for violation of animal-related laws were
recalled by virtue of Commonwealth Act (C.A.) No. 148, [4] which reads, in its Sec. 3. This Act shall take effect upon its approval.
entirety, thus:

Approved, November 8, 1936. (Emphasis supplied)


Be it enacted by the National Assembly of the Philippines:

Section 1. Section four of Act Numbered Twelve hundred


and eighty-five as amended by Act Numbered Thirty five Immediately thereafter, then President Manuel L. Quezon issued Executive
hundred and forty-eight, is hereby further amended so as to Order (E.O.) No. 63 dated November 12, 1936, portions of which provide:
read as follows:

Whereas, during the first regular session of the National


Sec. 4. The said society is authorized to Assembly, Commonwealth Act Numbered One Hundred
appoint not to exceed ten agents in the City Forty Eight was enacted depriving the agents of the Society
of Manila, and not to exceed one in each for the Prevention of Cruelty to Animals of their power to
municipality of the Philippines who shall arrest persons who have violated the laws prohibiting cruelty
have the authority to denounce to regular to animals thereby correcting a serious defect in one of the
peace officers any violation of the laws laws existing in our statute books.
enacted for the prevention of cruelty to
animals and the protection of animals and to
cooperate with said peace officers in the
prosecution of transgressors of such laws.
xxxx

Sec. 2. The full amount of the fines collected for violation of


the laws against cruelty to animals and for the protection of Whereas, the cruel treatment of animals is an offense
animals, shall accrue to the general fund of the against the State, penalized under our statutes, which the
Municipality where the offense was committed. Government is duty bound to enforce;

89
subsidiaries; and (d) such non-governmental entities
receiving subsidy or equity, directly or indirectly, from or
Now, therefore, I, Manuel L. Quezon, President of the through the government, which are required by law or the
Philippines, pursuant to the authority conferred upon me by granting institution to submit to such audit as a condition of
the Constitution, hereby decree, order, and direct the subsidy or equity.However, where the internal control system
Commissioner of Public Safety, the Provost Marshal General of the audited agencies is inadequate, the Commission may
as head of the Constabulary Division of the Philippine Army, adopt such measures, including temporary or special pre-
every Mayor of a chartered city, and every municipal audit, as are necessary and appropriate to correct the
president to detail and organize special members of the deficiencies. It shall keep the general accounts of the
police force, local, national, and the Constabulary to watch, Government, and for such period as may be provided by law,
capture, and prosecute offenders against the laws enacted preserve the vouchers and other supporting papers
to prevent cruelty to animals. (Emphasis supplied) pertaining thereto. (Emphasis supplied)

On December 1, 2003, an audit team from respondent Commission on Audit Petitioner explained thus:
(COA) visited the office of the petitioner to conduct an audit survey pursuant
to COA Office Order No. 2003-051 dated November 18, 2003[5] addressed to
the petitioner. The petitioner demurred on the ground that it was a private
entity not under the jurisdiction of COA, citing Section 2(1) of Article IX of the a. Although the petitioner was created by special legislation, this
Constitution which specifies the general jurisdiction of the COA, viz: necessarily came about because in January 1905 there was as
yet neither a Corporation Law or any other general law under
which it may be organized and incorporated, nor a Securities and
Exchange Commission which would have passed upon its
Section 1. General Jurisdiction. The Commission on Audit organization and incorporation.
shall have the power, authority, and duty to examine, audit,
and settle all accounts pertaining to the revenue and
receipts of, and expenditures or uses of funds and property,
owned or held in trust by, or pertaining to the Government, b. That Executive Order No. 63, issued during the Commonwealth
or any of its subdivisions, agencies, or instrumentalities, period, effectively deprived the petitioner of its power to make
including government-owned and controlled corporations arrests, and that the petitioner lost its operational funding,
with original charters, and on a post-audit basis: (a) underscore the fact that it exercises no governmental function. In
constitutional bodies, commissions and officers that have fine, the government itself, by its overt acts, confirmed petitioners
been granted fiscal autonomy under the Constitution; (b) status as a private juridical entity.
autonomous state colleges and universities; (c) other
government-owned or controlled corporations and their

90
The COA General Counsel issued a Memorandum [6] dated May 6, 2004,
asserting that the petitioner was subject to its audit authority. In a letter
dated May 17, 2004,[7]respondent COA informed the petitioner of the result of
the evaluation, furnishing it with a copy of said Memorandum dated May 6,
2004 of the General Counsel. Hence, herein Petition on the following grounds:

A.

Petitioner thereafter filed with the respondent COA a Request for Re-
evaluation dated May 19, 2004,[8] insisting that it was a private domestic
corporation.
RESPONDENT COMMISSION ON AUDIT COMMITTED
GRAVE ABUSE OF DISCRETION AMOUNTING TO LACK
OR EXCESS OF JURISDICTION WHEN IT RULED THAT
PETITIONER IS SUBJECT TO ITS AUDIT AUTHORITY.
Acting on the said request, the General Counsel of respondent COA, in a
Memorandum dated July 13, 2004,[9] affirmed her earlier opinion that the
petitioner was a government entity that was subject to the audit jurisdiction of
respondent COA. In a letter dated September 14, 2004, the respondent COA
B.
informed the petitioner of the result of the re-evaluation, maintaining its
position that the petitioner was subject to its audit jurisdiction, and requested
an initial conference with the respondents.

PETITIONER IS ENTITLED TO THE RELIEF SOUGHT,


THERE BEING NO APPEAL, NOR ANY PLAIN, SPEEDY
AND ADEQUATE REMEDY IN THE ORDINARY COURSE
In a Memorandum dated September 16, 2004, Director Delfin Aguilar
OF LAW AVAILABLE TO IT.[10]
reported to COA Assistant Commissioner Juanito Espino, Corporate
Government Sector, that the audit survey was not conducted due to the
refusal of the petitioner because the latter maintained that it was a private The essential question before this Court is whether the petitioner qualifies as
corporation. a government agency that may be subject to audit by respondent COA.

Petitioner received on September 27, 2005 the subject COA Office Order Petitioner argues: first, even though it was created by special legislation in
2005-021 dated September 14, 2005 and the COA Letter dated September 1905 as there was no general law then existing under which it may be
23, 2005. organized or incorporated, it exercises no governmental functions because
these have been revoked by C.A. No. 148 and E.O. No. 63; second, nowhere

91
in its charter is it indicated that it is a public corporation, unlike, for instance, auditing power; second, the petitioner exercises sovereign powers, that is, it
C.A. No. 111 which created the Boy Scouts of the Philippines, defined its is tasked to enforce the laws for the protection and welfare of animals which
powers and purposes, and specifically stated that it was An Act to Create a ultimately redound to the public good and welfare, and, therefore, it is
Public Corporation in which, even as amended by Presidential Decree No. deemed to be a government instrumentality as defined under the
460, the law still adverted to the Boy Scouts of the Philippines as a public Administrative Code of 1987, the purpose of which is connected with the
corporation, all of which are not obtaining in the charter of the administration of government, as purportedly affirmed by American
petitioner; third, if it were a government body, there would have been no jurisprudence; third, by virtue of Section 23, [11] Title II, Book III of the same
need for the State to grant it tax exemptions under Republic Act No. 1178, Code, the Office of the President exercises supervision or control over the
and the fact that it was so exempted strengthens its position that it is a petitioner; fourth, under the same Code, the requirement under its special
private institution; fourth, the employees of the petitioner are registered and charter for the petitioner to render a report to the Civil Governor, whose
covered by the Social Security System at the latters initiative and not through functions have been inherited by the Office of the President, clearly reflects
the Government Service Insurance System, which should have been the the nature of the petitioner as a government instrumentality; fifth, despite the
case had the employees been considered government employees; fifth, the passage of the Corporation Code, the law creating the petitioner had not
petitioner does not receive any form of financial assistance from the been abolished, nor had it been re-incorporated under any general
government, since C.A. No. 148, amending Section 5 of Act No. 1285, states corporation law; and finally, sixth, Republic Act No. 8485, otherwise known as
that the full amount of the fines, collected for violation of the laws against the Animal Welfare Act of 1998, designates the petitioner as a member of its
cruelty to animals and for the protection of animals, shall accrue to the Committee on Animal Welfare which is attached to the Department of
general fund of the Municipality where the offense was committed; sixth, C.A. Agriculture.
No. 148 effectively deprived the petitioner of its powers to make arrests and
serve processes as these functions were placed in the hands of the police
force; seventh, no government appointee or representative sits on the board
of trustees of the petitioner; eighth, a reading of the provisions of its charter In view of the phrase One-half of all the fines imposed and collected through
(Act No. 1285) fails to show that any act or decision of the petitioner is the efforts of said society, the Court, in a Resolution dated January 30, 2007,
subject to the approval of or control by any government agency, except to the required the Office of the Solicitor General (OSG) and the parties to
extent that it is governed by the law on private corporations in general; and comment on: a) petitioner's authority to impose fines and the validity of the
finally, ninth, the Committee on Animal Welfare, under the Animal Welfare Act provisions of Act No. 1285 and Commonwealth Act No. 148 considering that
of 1998, includes members from both the private and the public sectors. there are no standard measures provided for in the aforecited laws as to the
manner of implementation, the specific violations of the law, the person/s
authorized to impose fine and in what amount; and, b) the effect of the 1935
and 1987 Constitutions on whether petitioner continues to exist or should
The respondents contend that since the petitioner is a body politic created by organize as a private corporation under the Corporation Code, B.P. Blg. 68
virtue of a special legislation and endowed with a governmental purpose, as amended.
then, indubitably, the COA may audit the financial activities of the
latter. Respondents in effect divide their contentions into six strains: first, the
test to determine whether an entity is a government corporation lies in the
manner of its creation, and, since the petitioner was created by virtue of a
special charter, it is thus a government corporation subject to respondents

92
Petitioner and the OSG filed their respective Comments. Respondents filed a
Manifestation stating that since they were being represented by the OSG
which filed its Comment, they opted to dispense with the filing of a separate First, the Court agrees with the petitioner that the charter test cannot be
one and adopt for the purpose that of the OSG. applied.

The petitioner avers that it does not have the authority to impose fines for Essentially, the charter test as it stands today provides:
violation of animal welfare laws; it only enjoyed the privilege of sharing in the
fines imposed and collected from its efforts in the enforcement of animal
welfare laws; such privilege, however, was subsequently abolished by C.A.
No. 148; that it continues to exist as a private corporation since it was
[T]he test to determine whether a corporation is government
created by the Philippine Commission before the effectivity of the
owned or controlled, or private in nature is simple. Is it
Corporation law, Act No. 1459; and the 1935 and 1987 Constitutions.
created by its own charter for the exercise of a public
function, or by incorporation under the general corporation
law? Those with special charters are government
corporations subject to its provisions, and its employees are
The OSG submits that Act No. 1285 and its amendatory laws did not give under the jurisdiction of the Civil Service Commission, and
petitioner the authority to impose fines for violation of laws [12] relating to the are compulsory members of the Government Service
prevention of cruelty to animals and the protection of animals; that even prior Insurance System. xxx (Emphasis supplied)[13]
to the amendment of Act No. 1285, petitioner was only entitled to share in the
fines imposed; C.A. No. 148 abolished that privilege to share in the fines
collected; that petitioner is a public corporation and has continued to exist
since Act No. 1285; petitioner was not repealed by the 1935 and 1987
The petitioner is correct in stating that the charter test is predicated, at best,
Constitutions which contain transitory provisions maintaining all laws issued
on the legal regime established by the 1935 Constitution, Section 7, Article
not inconsistent therewith until amended, modified or repealed.
XIII, which states:

The petition is impressed with merit.


Sec. 7. The National Assembly shall not, except by general
law, provide for the formation, organization, or regulation of
private corporations, unless such corporations are owned or
controlled by the Government or any subdivision or
The arguments of the parties, interlaced as they are, can be disposed of in instrumentality thereof.[14]
five points.

93
retroactive effect, unless the contrary is provided. [16] All statutes are to be
construed as having only a prospective operation, unless the purpose and
The foregoing proscription has been carried over to the 1973 and the 1987 intention of the legislature to give them a retrospective effect is expressly
Constitutions. Section 16 of Article XII of the present Constitution provides: declared or is necessarily implied from the language used. In case of doubt,
the doubt must be resolved against the retrospective effect. [17]

Sec. 16. The Congress shall not, except by general


law, provide for the formation, organization, or regulation of There are a few exceptions. Statutes can be given retroactive effect in the
private corporations. Government-owned or controlled following cases: (1) when the law itself so expressly provides; (2) in case of
corporations may be created or established by special remedial statutes; (3) in case of curative statutes; (4) in case of laws
charters in the interest of the common good and subject to interpreting others; and (5) in case of laws creating new rights. [18] None of the
the test of economic viability. exceptions is present in the instant case.

Section 16 is essentially a re-enactment of Section 7 of Article XVI of the The general principle of prospectivity of the law likewise applies to Act No.
1935 Constitution and Section 4 of Article XIV of the 1973 Constitution. 1459, otherwise known as the Corporation Law, which had been enacted by
virtue of the plenary powers of the Philippine Commission on March 1, 1906,
a little over a year after January 19, 1905, the time the petitioner emerged as
a juridical entity. Even the Corporation Law respects the rights and powers of
juridical entities organized beforehand, viz:
During the formulation of the 1935 Constitution, the Committee on
Franchises recommended the foregoing proscription to prevent the pressure
of special interests upon the lawmaking body in the creation of corporations
or in the regulation of the same. To permit the lawmaking body by special law
to provide for the organization, formation, or regulation of private SEC. 75. Any corporation or sociedad anonima formed,
corporations would be in effect to offer to it the temptation in many cases to organized, and existing under the
favor certain groups, to the prejudice of others or to the prejudice of the laws of the Philippine Islands and lawfully transacting busine
interests of the country.[15] ss in the Philippine Islands on the date of the passage of this
Act, shall be subject to the provisions hereof so far as such
provisions may be applicable and shall
be entitled at its option either to continue business as such
corporation or to reform and organize under and by virtue of
And since the underpinnings of the charter test had been introduced by the
the provisions of this Act, transferring all corporate interests
1935 Constitution and not earlier, it follows that the test cannot apply to the
to the new corporation which, if a stock corporation, is
petitioner, which was incorporated by virtue of Act No. 1285, enacted
authorized to issue its shares of stock at par to the
on January 19, 1905. Settled is the rule that laws in general have no

94
stockholders or members of the old corporation according to
their interests. (Emphasis supplied).
Time and again the Court must caution even the most brilliant scholars of the
law and all constitutional historians on the danger of imposing legal concepts
of a later date on facts of an earlier date.[20]
As pointed out by the OSG, both the 1935 and 1987 Constitutions contain
transitory provisions maintaining all laws issued not inconsistent therewith
until amended, modified or repealed.[19]
The amendments introduced by C.A. No. 148 made it clear that the petitioner
In a legal regime where the charter test doctrine cannot be applied, the mere was a private corporation and not an agency of the government. This was
fact that a corporation has been created by virtue of a special law does not evident in Executive Order No. 63, issued by then President of the
necessarily qualify it as a public corporation. Philippines Manuel L. Quezon, declaring that the revocation of the powers of
the petitioner to appoint agents with powers of arrest corrected a serious
defect in one of the laws existing in the statute books.

What then is the nature of the petitioner as a corporate entity? What legal
regime governs its rights, powers, and duties?
As a curative statute, and based on the doctrines so far discussed, C.A. No.
148 has to be given retroactive effect, thereby freeing all doubt as to which
class of corporations the petitioner belongs, that is, it is a quasi-public
As stated, at the time the petitioner was formed, the applicable law was the corporation, a kind of private domestic corporation, which the Court will
Philippine Bill of 1902, and, emphatically, as also stated above, no further elaborate on under the fourth point.
proscription similar to the charter test can be found therein.

Second, a reading of petitioners charter shows that it is not subject to control


The textual foundation of the charter test, which placed a limitation on the or supervision by any agency of the State, unlike government-owned and
power of the legislature, first appeared in the 1935 Constitution. However, the -controlled corporations.No government representative sits on the board of
petitioner was incorporated in 1905 by virtue of Act No. 1258, a law trustees of the petitioner. Like all private corporations, the successors of its
antedating the Corporation Law (Act No. 1459) by a year, and the 1935 members are determined voluntarily and solely by the petitioner in
Constitution, by thirty years. There being neither a general law on the accordance with its by-laws, and may exercise those powers generally
formation and organization of private corporations nor a restriction on the accorded to private corporations, such as the powers to hold property, to sue
legislature to create private corporations by direct legislation, the Philippine and be sued, to use a common seal, and so forth. It may adopt by-laws for its
Commission at that moment in history was well within its powers in 1905 to internal operations: the petitioner shall be managed or operated by its
constitute the petitioner as a private juridical entity. officers in accordance with its by-laws in force. The pertinent provisions of
the charter provide:

95
Section 1. Anna L. Ide, Kate S. Wright, John L. Sec. 3. The said society shall be operated under the
Chamberlain, William F. Tucker, Mary S. direction of its officers, in accordance with its by-laws in
Fergusson, Amasa S. Crossfield, Spencer Cosby, Sealy force, and this charter.
B. Rossiter, Richard P. Strong, Jose Robles Lahesa,
Josefina R. de Luzuriaga, and such other persons as may be
associated with them in conformity with this act, and their
successors, are hereby constituted and created a body xxxx
politic and corporate at law, under the name and style of The
Philippines Society for the Prevention of Cruelty to Animals.

Sec. 6. The principal office of the society shall be


kept in the city of Manila, and the society shall have full
As incorporated by this Act, said society shall have power to locate and establish branch offices of the society
the power to add to its organization such and as many wherever it may deem advisable in the Philippine Islands,
members as it desires, to provide for and choose such such branch offices to be under the supervision and control
officers as it may deem advisable, of the principal office.
and in such manner as it may wish, and to remove members
as it shall provide.

Third. The employees of the petitioner are registered and covered by the
Social Security System at the latters initiative, and not through the
It shall have the right to sue and be sued, to use a Government Service Insurance System, which should be the case if the
common seal, to employees are considered government employees. This is another indication
receive legacies and donations, to conduct social enterprises of petitioners nature as a private entity. Section 1 of Republic Act No. 1161,
for the purpose of obtaining funds, to levy dues upon as amended by Republic Act No. 8282, otherwise known as the Social
itsmembers and provide for their collection to hold real and Security Act of 1997, defines the employer:
personal estate such as may be necessary for the
accomplishment of the purposes of the society, and to adopt
such by-laws for its government as may not be inconsistent
with law or this charter.
Employer Any person, natural or juridical, domestic
or foreign, who carries on in the Philippines any trade,
business, industry, undertaking or activity of any kind and
uses the services of another person who is under his orders
xxxx as regards the employment, except the Government and any

96
of its political subdivisions, branches or instrumentalities, public benefit. Private schools and universities are likewise private
including corporations owned or controlled by the corporations; and yet, they are rendering public service. Private hospitals and
Government: Provided, That a self-employed person shall be wards are charged with heavy social responsibilities. More so with all
both employee and employer at the same time. (Emphasis common carriers. On the other hand, there may exist a public corporation
supplied) even if it is endowed with gifts or donations from private individuals.

Fourth. The respondents contend that the petitioner is a body politic because The true criterion, therefore, to determine whether a corporation is public or
its primary purpose is to secure the protection and welfare of animals which, private is found in the totality of the relation of the corporation to the State. If
in turn, redounds to the public good. the corporation is created by the State as the latters own agency or
instrumentality to help it in carrying out its governmental functions, then that
corporation is considered public; otherwise, it is private. Applying the above
test, provinces, chartered cities, and barangays can best exemplify public
This argument, is, at best, specious. The fact that a certain juridical entity is corporations. They are created by the State as its own device and agency for
impressed with public interest does not, by that circumstance alone, make the accomplishment of parts of its own public works.[25]
the entity a public corporation, inasmuch as a corporation may be private
although its charter contains provisions of a public character, incorporated
solely for the public good. This class of corporations may be considered
quasi-public corporations, which are private corporations that render public It is clear that the amendments introduced by C.A. No. 148 revoked the
service, supply public wants,[21] or pursue other eleemosynary powers of the petitioner to arrest offenders of animal welfare laws and the
objectives. While purposely organized for the gain or benefit of its members, power to serve processes in connection therewith.
they are required by law to discharge functions for the public
benefit. Examples of these corporations are utility,[22] railroad, warehouse,
telegraph, telephone, water supply corporations and transportation
companies.[23] It must be stressed that a quasi-public corporation is a Fifth. The respondents argue that since the charter of the petitioner requires
species of private corporations, but the qualifying factor is the type of the latter to render periodic reports to the Civil Governor, whose functions
service the former renders to the public: if it performs a public service, then it have been inherited by the President, the petitioner is, therefore, a
becomes a quasi-public corporation.[24] government instrumentality.

Authorities are of the view that the purpose alone of the corporation cannot This contention is inconclusive. By virtue of the fiction that all corporations
be taken as a safe guide, for the fact is that almost all corporations are owe their very existence and powers to the State, the reportorial requirement
nowadays created to promote the interest, good, or convenience of the is applicable to all corporations of whatever nature, whether they are public,
public. A bank, for example, is a private corporation; yet, it is created for a quasi-public, or private corporationsas creatures of the State, there is a

97
reserved right in the legislature to investigate the activities of a corporation to hand when charged with an abuse of such privileges.
determine whether it acted within its powers. In other words, the reportorial (Wilson v. United States, 55 Law Ed., 771, 780.)[27]
requirement is the principal means by which the State may see to it that its
creature acted according to the powers and functions conferred upon
it. These principles were extensively discussed in Bataan Shipyard &
Engineering Co., Inc. v. Presidential Commission on Good Government. WHEREFORE, the petition is GRANTED. Petitioner is DECLARED a private
[26]
Here, the Court, in holding that the subject corporation could not invoke domestic corporation subject to the jurisdiction of the Securities and
the right against self-incrimination whenever the State demanded the Exchange Commission. The respondents are ENJOINED from investigating,
production of its corporate books and papers, extensively discussed the examining and auditing the petitioner's fiscal and financial affairs.
purpose of reportorial requirements, viz:

SO ORDERED.
x x x The corporation is a creature of the state. It is
presumed to be incorporated for the benefit of the public. It
received certain special privileges and franchises, and holds
them subject to the laws of the state and the limitations of its
charter. Its powers are limited by law. It can make no
contract not authorized by its charter. Its rights to act as a
corporation are only preserved to it so long as it obeys the
laws of its creation. There is a reserve[d] right in the
legislature to investigate its contracts and find out whether it
has exceeded its powers. It would be a strange anomaly to
hold that a state, having chartered a corporation to make
use of certain franchises, could not, in the exercise of
sovereignty, inquire how these franchises had been G.R. No. 95237-38 September 13, 1991
employed, and whether they had been abused, and demand
the production of the corporate books and papers for that
DAVAO CITY WATER DISTRICT, CAGAYAN DE ORO CITY WATER
purpose. The defense amounts to this, that an officer of the
DISTRICT, METRO CEBU WATER DISTRICT, ZAMBOANGA CITY WATER
corporation which is charged with a criminal violation of the
DISTRICT, LEYTE METRO WATER DISTRICT, BUTUAN CITY WATER
statute may plead the criminality of such corporation as a
DISTRICT, CAMARINES NORTE WATER DISTRICT, LAGUNA WATER
refusal to produce its books. To state this proposition is to
DISTRICT, DUMAGUETE CITY WATER DISTRICT, LA UNION WATER
answer it. While an individual may lawfully refuse to answer
DISTRICT, BAYBAY WATER DISTRICT, METRO LINGAYEN WATER
incriminating questions unless protected by an immunity
DISTRICT, URDANETA WATER DISTRICT, COTABATO CITY WATER
statute, it does not follow that a corporation vested with
DISTRICT, MARAWI WATER DISTRICT, TAGUM WATER DISTRICT,
special privileges and franchises may refuse to show its
DIGOS WATER DISTRICT, BISLIG WATER DISTRICT, and MECAUAYAN

98
WATER DISTRICT,petitioners, The respondents, on the other hand, are the Civil Service Commission (CSC)
vs. and the Commission on Audit (COA), both government agencies and
CIVIL SERVICE COMMISSION, and COMMISSION ON represented in this case by the Solicitor General.
AUDIT, respondents.
On April 17, 1989, this Court ruled in the case of Tanjay Water District v.
Rodolfo S. De Jesus for petitioners. Gabaton, et al. (G.R. No. 63742, 172 SCRA 253):

Evalyn H. Itaas-Fetalino, Rogelio C. Limare and Daisy B. Garcia-Tingzon for Significantly, Article IX (B), Section 2(1) of the 1987
CSC. Constitution provides that the Civil Service embraces all
branches, subdivisions, instrumentalities, and agencies of
the government, including government-owned and controlled
corporations with original charters. Inasmuch as PD No. 198,
MEDIALDEA, J.:p as amended, is the original charter of the petitioner, Tanjay
Water District, and respondent Tarlac Water District and all
water districts in the country, they come under the coverage
Whether or not the Local Water Districts formed and created pursuant to the
of the Civil Service Law, rules and regulations. (Sec. 35, Art.
provisions of Presidential Decree No. 198, as amended, are government-
VIII and Sec. 37, Art. IX of PD No. 807).
owned or controlled corporations with original charter falling under the Civil
Service Law and/or covered by the visitorial power of the Commission on
Audit is the issue which the petitioners entreat this Court, en banc, to shed As an offshoot of the immediately cited ruling, the CSC. issued Resolution
light on. No. 90-575, the dispositive portion of which reads:

Petitioners are among the more than five hundred (500) water districts NOW THEREFORE, in view of all the foregoing, the
existing throughout the country formed pursuant to the provisions of Commission resolved, as it hereby resolves to rule that Local
Presidential Decree No. 198, as amended by Presidential Decrees Nos. 768 Water Districts, being quasi-public corporations created by
and 1479, otherwise known as the "Provincial Water Utilities Act of 1973." law to perform public services and supply public wants, the
matter of hiring and firing of its officers and employees
should be governed by the Civil Service Law, rules and
Presidential Decree No. 198 was issued by the then President Ferdinand E.
regulations. Henceforth, all appointments of personnel of the
Marcos by virtue of his legislative power under Proclamation No. 1081. It
different local water districts in the country shall be submitted
authorized the different local legislative bodies to form and create their
to the Commission for appropriate action. (Rollo. p. 22).
respective water districts through a resolution they will pass subject to the
guidelines, rules and regulations therein laid down. The decree further
created and formed the "Local Water Utilities Administration" (LWUA), a However, on May 16, 1990, in G.R. No. 85760, entitled "Metro Iloilo Water
national agency attached to the National Economic and Development District v. National Labor Relations Commission, et al.," the Third Division of
Authority (NEDA), and granted with regulatory power necessary to optimize this Court ruled in a minute resolution:
public service from water utilities operations.
xxx xxx xxx

99
Considering that PD 198 is a general legislation empowering the extent of the amount of such subsidies, pursuant to the provision of the
and/or authorizing government agencies and entities to Government Auditing Code of the Phils.
create water districts, said PD 198 cannot be considered as
the charter itself creating the Water District. Public It is to be observed that just like the question of whether the employees of
respondent NLRC did not commit any grave abuse of the water districts falls under the coverage of the Civil Service Law, the
discretion in holding that the operative act, that created the conflict between the water districts and the COA is also dependent on the
Metro Iloilo Water District was the resolution of the final determination of whether or not water districts are government-owned or
Sangguniang Panglunsod of Iloilo City. Hence, the controlled corporations with original charter. The reason behind this is Sec.
employees of Water Districts are not covered by Civil 2(1), Article IX-D of the 1987 constitution which reads:
Service Laws as the latter do (sic) not have original charters.
Sec. 2(1) The Commission on Audit shall have the power,
In adherence to the just cited ruling, the CSC suspended the implementation authority, and duty to examine, audit, and settle all accounts
of Resolution No. 90-575 by issuing Resolution No. 90-770 which reads: pertaining to the revenue and receipts of, and expenditures
or uses of funds and property, owned or held in trust by, or
xxx xxx xxx pertaining to the Government, or any of its subdivisions,
agencies or instrumentalities, including government-owned
NOW, THEREFORE, in view of all the foregoing, the or controlled corporations with original charters, and on a
Commission resolved to rule, as it hereby rules, that the post audit basis. (emphasis supplied)
implementation of CSC. Resolution No. 575 dated June 27,
1990 be deferred in the meantime pending clarification from Petitioners' main argument is that they are private corporations without
the Supreme Court are regards its conflicting decisions in original charter, hence they are outside the jurisdiction of respondents CSC
the cases of Tanjay Water District v. Gabaton and Metro and COA. Reliance is made on the Metro Iloilo case which declared
Iloilo Water District v. National Labor Relations petitioners as quasi-public corporations created by virtue of PD 198, a
Commission. (p. 26, Rollo) general legislation which cannot be considered as the charter itself creating
the water districts. Holding on to this ruling, petitioners contend that they are
In the meanwhile, there exists a divergence of opinions between COA on one private corporations which are only regarded as quasi-public or semi-public
hand, and the (LWUA), on the other hand, with respect to the authority of because they serve public interest and convenience and that since PD 198 is
COA to audit the different water districts. a general legislation, the operative act which created a water district is not
the said decree but the resolution of the sanggunian concerned.
COA opined that the audit of the water districts is simply an act of
discharging the visitorial power vested in them by law (letter of COA to LWUA After a fair consideration of the parties' arguments coupled with a careful
dated August 13, 1985, pp. 29-30, Rollo). study of the applicable laws as well as the constitutional provisions involved,
We rule against the petitioners and reiterate Our ruling in Tanjay case
On the other hand, LWUA maintained that only those water districts with declaring water districts government-owned or controlled corporations with
subsidies from the government fall within the COA's jurisdiction and only to original charter.

100
As early as Baguio Water District v. Trajano, et al., (G.R. No. 65428, The Labor Arbiter failed to take into accout the provisions of
February 20, 1984, 127 SCRA 730), We already ruled that a water district is Presidential Decree No. 1479, which went into effect on 11
a corporation created pursuant to a special law P.D. No. 198, as June 1978, P.D. No. 1479, wiped away Section 25 of PD 198
amended, and as such its officers and employees are covered by the Civil quoted above, and Section 26 of PD 198 was renumbered
Service Law. as Section 25 in the following manner:

In another case (Hagonoy Water District v. NLRC, G.R. No. 81490, August Section 26 of the same decree PD 198 is hereby amended
31, 1988, 165 SCRA 272), We ruled once again that local water districts are to read as Section 25 as follows:
quasi-public corporations whose employees belong to the Civil Service. The
Court's pronoucement in this case, as extensively quoted in Section 25. Authorization. The district may exercise all
the Tanjay case, supra, partly reads: the powers which are expressly granted by this Title or which
are necessarily implied from or incidental to the powers and
"The only question here is whether or not local water districts purposes herein stated. For the purpose of carrying out the
are governmkent owned or controlled corporations whose objectives of this Act, a district is hereby granted the power
employees are subject to the provisions of the Civil Service of eminent domain, the exercise thereof shall, however, be
Law. The Labor Arbiter asserted jurisdiction over the alleged subject to review by the Administration.
illegal dismissal of private respondent Villanueva by relying
on Section 25 of Presidential decree No. 198, known as the Thus, Section 25 of PD 198 exempting the employees of
Provincial Water Utilities Act of 1973" which went onto effect water districts from the application of the Civil Service Law
in 25 May 1973, and which provides as follows: was removed from the statute books:

Exemption from Civil Service. The district xxx xxx xxx


and its employees, being engaged in a
proprietary function, are hereby exempt from We grant the petition for the following reasons:
the provisions of the Civil Service Law.
Collective Bargaining shall be available only
1. Section 25 of PD No. 198 was repealed by Section 3 of
to personnel below supervisory
PD No. 1479; Section 26 of PD No. 198 was amended ro
levels: Provided, however, That the total of
read as Sec. 25 by Sec. 4 of PD No. 1479. The amendatory
all salaries, wages emoluments, benefits or
decree took effect on June 11, 1978.
other compensation paid to all employees in
any month shall not exceed fifty percent
(50%) of average net monthy revenue. Said xxx xxx xxx
net revenue representing income from water
sales and sewerage service charges, less 3. The BWD is a corporation created pursuant to a special
pro-rata share of debt service and expenses law PD No. 198, as amended. As such its officers and
for fuel or energy for pumping during the employees are part of the Civil Service (Sec. 1, Art. XII-B,
preceding fiscal year. [1973] Constitution; PD No. 868).

101
Ascertained from a consideration of the whole statute, PD 198 is a special code, but on the contrary, they were created pursuant to a special law and
law applicable only to the different water districts created pursuant thereto. In are governed primarily by its provision.
all its essential terms, it is obvious that it pertains to a special purpose which
is intended to meet a particular set of conditions and cirmcumstances. The No consideration may thus be given to petitioners' contention that the
fact that said decree generally applies to all water districts throughout the operative act which created the water districts are the resolutions of the
country does not change the fact that PD 198 is a special law. Accordingly, respective local sanggunians and that consequently, PD 198, as amended,
this Court's resolution in Metro Iloilo case declaring PD 198 as a general cannot be considered as their charter.
legislation is hereby abandoned.
It is to be noted that PD 198, as amended is the source of authorization and
By "government-owned or controlled corporation with original charter," We power to form and maintain a district. Section 6 of said decree provides:
mean government owned or controlled corporation created by a special law
and not under the Corporation Code of the Philippines. Thus, in the case Sec. 6. Formation of District. This Act is the source of
of Lumanta v. NLRC (G.R. No. 82819, February 8, 1989, 170 SCRA 79, 82), authorization and power to form and maintain a district.
We held: Once formed, a district is subject to the provisions of this Act
and not under the jurisdiction of any political
The Court, in National Service Corporation (NASECO) v. subdivision, . . . .
National Labor Relations Commission, G.R. No 69870,
promulgated on 29 November 1988, quoting extensively Moreover, it must be observed that PD 198, contains all the essential terms
from the deliberations of 1986 Constitutional Commission in necessary to constitute a charter creating a juridical person. For example,
respect of the intent and meaning of the new phrase "with Section 6(a) provides for the name that will be used by a water district, thus:
original character," in effect held that government-owned
and controlled corporations with original charter refer to
Sec. 6. . . . To form a district, the legislative body of any city,
corporations chartered by special law as distinguished from
municipality or province shall enact a resolution containing
corporations organized under our general incorporation
the following:
statute the Corporations Code. In NASECO, the company
involved had been organized under the general incorporation
statute and was a sbusidiary of the National Investment a) The name of the local water district, which shall include
Development Corporation (NIDC) which in turn was a the name of the city, municipality, or province, or region
subsidiary of the Philippine National Bank, a bank chartered thereof, served by said system, followed by the words "Water
by a special statute. Thus, government-owned or controlled District."
corporations like NASECO are effectively, excluded from the
scope of the Civil Service. (emphasis supplied) It also prescribes for the numbers and qualifications of the members of the
Board of Directors:
From the foregoing pronouncement, it is clear that what has been excluded
from the coverage of the CSC are those corporations created pursuant to the Sec. 8. Number and Qualification. The Board of Directors
Corporation Code. Significantly, petitioners are not created under the said of a district shall be composed of five citizens of the
Philippines who are of voting age and residents within the

102
district. One member shall be a representative of civic- fails to make his appointments on or before December 15,
oriented service clubs, one member of representative of selection shall be made from said list of nominees by
professional associations, one member a representative of majority vote of the seated directors of the district
business, commercial or financial organizations, one constituting a quorum. Initial nominations for all five seats of
member a representative of educational institutions and one the board shall be solicited by the legislative body or bodies
member a representative of women's organization. No public at the time of adoption of the resolution forming the district.
official shall serve as director. Provided, however, that if the Thirty days thereafter, a list of nominees shall be submitted
district has availed of the financial assistance of the to the provincial governor in the event the resolution forming
Administration, the Administration may appoint any of its the district is by a provincial board, or the mayor of the city or
personnel to sit in the board of directors with all the rights municipality in the event the resolution forming the adoption
and privileges appertaining to a regular member for such of the district is by the city or municipal board of councilors,
period as the indebtedness remains unpaid in which case who shall select the initial directors therefrom within 15 days
the board shall be composed of six members; (as amended after receipt of such nominations;
by PDs Nos. 768 and 1479).
their terms of office:
the manner of their appointment and nominations;
Sec. 11. Term of Office. Of the five initial directors of each
Sec. 9. Appointment. Board members shall be appointed newly formed district, two shall be appointed for a maximum
by the appointing authority. Said appointments shall be made term of two years, two for a maximum term of four years, and
from a list of nominees, if any, submitted pursuant to Section one for a maximum term of six years. Terms of office of all
10. If no nominations are submitted, the appointing authority directors in a given district shall be such that the term of at
shall appoint any qualified person of the category to the least one director, but not more then two, shall expire on
vacant position; December 31 of each even-numbered year. Regular terms of
office after the initial terms shall be for six years commencing
Sec.10. Nominations. On or before October 1 of each on January 1 of odd-numbered years. Directors may be
even numbered year, the secretary of the district shall removed for cause only, subject to review and approval of
contact each known organization, association, or institution the Administration; (as amended by PD 768).
being represented by the director whose term will expire on
December 31 and solicit nominations from these the manner of filling up vacancies:
organizations to fill the position for the ensuing term. One
nomination may be submitted in writing by each such Sec. 12. Vacancies. In the event of a vacancy in the
organization to the Secretary of the district on or before board of directors occurring more than six months before
November 1 of such year: This list of nominees shall be expiration of any director's term, the remaining directors shall
transmitted by the Secretary of the district to the office of the within 30 days, serve notice to or request the secretary of the
appointing authority on or before November 15 of such year district for nominations and within 30 days, thereafter a list of
and he shall make his appointment from the list submitted on nominees shall be submitted to the appointing authority for
or before December 15. In the event the appointing authority

103
his appointment of a replacement director from the list of final creation of a district, this Court is of the opinion that said resolution
nominees. In the absence of such nominations, the cannot be considered as its charter, the same being intended only to
appointing authority shall make such appointment. If within implement the provisions of said decree. In passing a resolution forming a
30 days after submission to him of a list of nominees the water district, the local sanggunian is entrusted with no authority or discretion
appointing authority fails to make an appointment, the to grant a charter for the creation of a private corporation. It is merely given
vacancy shall be filled from such list by a majority vote of the the authority for the formation of a water district, on a local option basis, to be
remaining members of the Board of Directors constituting a exercised under and in pursuance of PD 198.
quorum. Vacancies occurring within the last six months of an
unexpired term shall also be filled by the Board in the above More than the aforequoted provisions, what is of important interest in the
manner. The director thus appointed shall serve the case at bar is Section 3, par. (b) of the same decree which reads:
unexpired term only; (as amended by PD 768).
Sec. 3(b). Appointing authority. The person empowered to
and the compensation and personal liability of the members of the Board of appoint the members of the Board of Directors of a local
Directors: water district, depending upon the geographic coverage and
population make-up of the particular district. In the event that
Sec. 13. Compensation. Each director shall receive a per more than seventy-five percent of the total active water
diem, to be determined by the board, for each meeting of the service connections of a local water districts are within the
board actually attended by him, but no director shag receive boundary of any city or municipality, the appointing authority
per diems in any given month in excess of the equivalent of shall be the mayor of that city or municipality, as the case
the total per diems of four meetings in any given month. No may be; otherwise, the appointing authority shall be the
director shall receive other compensation for services to the governor of the province within which the district is
district. located: Provided, That if the existing waterworks system in
the city or municipality established as a water district under
Any per diem in excess of P50.00 shall be subject to this Decree is operated and managed by the province, initial
approval of the Administration (as amended by PD 768). appointment shall be extended by the governor of the
province. Subsequent appointments shall be as specified
Sec. 14. Personal Liability. No director may be held to be herein.
personally liable for any action of the district.
If portions of more than one province are included within the
Noteworthy, the above quoted provisions of PD 198, as amended, are similar boundary of the district, and the appointing authority is to be
to those which are actually contained in other corporate charters. The the governors then the power to appoint shall rotate between
conclusion is inescapable that the said decree is in truth and in fact the the governors involved with the initial appointments made by
charter of the different water districts for it clearly defines the latter's primary the governor in whose province the greatest number of
purpose and its basic organizational set-up. In other words, PD 198, as service connections exists (as amended by PD 768).
amended, is the very law which gives a water district juridical personality.
While it is true that a resolution of a local sanggunian is still necessary for the

104
The above-quoted section definitely sets to naught petitioners' contention
that they are private corporations. It is clear therefrom that the power to
appoint the members who will comprise the Board of Directors belongs to the
local executives of the local subdivision units where such districts are
located. In contrast, the members of the Board of Directors or trustees of a
private corporation are elected from among the members and stockholders
thereof. It would not be amiss to emphasize at this point that a private
corporation is created for the private purpose, benefit, aim and end of its
members or stockholders. Necessarily, said members or stockholders should
be given a free hand to choose those who will compose the governing body
of their corporation. But this is not the case here and this clearly indicates
that petitioners are definitely not private corporations.

The foregoing disquisition notwithstanding, We are, however, not unaware of


the serious repercussion this may bring to the thousands of water districts'
employees throughout the country who stand to be affected because they do
not have the necessary civil service eligibilities. As these employees are
equally protected by the constitutional guarantee to security of tenure, We
find it necessary to rule for the protection of such right which cannot be
impaired by a subsequent ruling of this Court. Thus, those employees who
have already acquired their permanent employment status at the time of the
promulgation of this decision cannot be removed by the mere reason that
they lack the necessary civil service eligibilities.

ACCORDINGLY, the petition is hereby DISMISSED. Petitioners are declared


"government-owned or controlled corporations with original charter" which fall
under the jurisdiction of the public respondents CSC and COA.

SO ORDERED.

Fernan, C.J., Narvasa, Melencio-Herrera, Cruz, Paras, Padilla, Grio-


Aquino, Regalado and Davide, Jr., JJ., concur.

Gutierrez, Jr., Feliciano and Sarmiento, JJ., are on leave.

105
106
G.R. No. L-6776 May 21, 1955

THE REGISTER OF DEEDS OF RIZAL, petitioner-appellee,


vs.
UNG SIU SI TEMPLE, respondent-appellant.

Alejo F. Candido for appellant.


Office of the Solicitor General Querube C. Makalintal and Solicitor Felix V.
Makasiar for appellee.

REYES, J.B.L., J.:

The Register of Deeds for the province of Rizal refused to accept for record a
deed of donation executed in due form on January 22, 1953, by Jesus Dy, a
Filipino citizen, conveying a parcel of residential land, in Caloocan, Rizal,
known as lot No. 2, block 48-D, PSD-4212, G.L.R.O. Record No. 11267, in
favor of the unregistered religious organization "Ung Siu Si Temple",
operating through three trustees all of Chinese nationality. The donation was
duly accepted by Yu Juan, of Chinese nationality, founder and deaconess of
the Temple, acting in representation and in behalf of the latter and its
trustees.

The refusal of the Registrar was elevated en Consultato the IVth Branch of
the Court of First Instance of Manila. On March 14, 1953, the Court upheld
the action of the Rizal Register of Deeds, saying:

The question raised by the Register of Deeds in the above


transcribed consulta is whether a deed of donation of a parcel of land
executed in favor of a religious organization whose founder, trustees
and administrator are Chinese citizens should be registered or not.

It appearing from the record of the Consulta that UNG SIU SI


TEMPLE is a religious organization whose deaconess, founder,
trustees and administrator are all Chinese citizens, this Court is of
the opinion and so hold that in view of the provisions of the sections
1 and 5 of Article XIII of the Constitution of the Philippines limiting the

107
acquisition of land in the Philippines to its citizens, or to corporations associations qualified to acquire or hold lands of the public domain in
or associations at least sixty per centum of the capital stock of which the Philippines,
is owned by such citizens adopted after the enactment of said Act
No. 271, and the decision of the Supreme Court in the case of the Constitution makes no exception in favor of religious associations.
Krivenko vs. the Register of Deeds of Manila, the deed of donation in Neither is there any such saving found in sections 1 and 2 of Article XIII,
question should not be admitted for admitted for registration. (Printed restricting the acquisition of public agricultural lands and other natural
Rec. App. pp 17-18). resources to "corporations or associations at least sixty per centum of the
capital of which is owned by such citizens" (of the Philippines).
Not satisfied with the ruling of the Court of First Instance, counsel for the
donee Uy Siu Si Temple has appealed to this Court, claiming: (1) that the The fact that the appellant religious organization has no capital stock does
acquisition of the land in question, for religious purposes, is authorized and not suffice to escape the Constitutional inhibition, since it is admitted that its
permitted by Act No. 271 of the old Philippine Commission, providing as members are of foreign nationality. The purpose of the sixty per centum
follows: requirement is obviously to ensure that corporations or associations allowed
to acquire agricultural land or to exploit natural resources shall be controlled
SECTION 1. It shall be lawful for all religious associations, of by Filipinos; and the spirit of the Constitution demands that in the absence of
whatever sort or denomination, whether incorporated in the capital stock, the controlling membership should be composed of Filipino
Philippine Islands or in the name of other country, or not incorporated citizens.
at all, to hold land in the Philippine Islands upon which to build
churches, parsonages, or educational or charitable institutions. To permit religious associations controlled by non-Filipinos to acquire
agricultural lands would be to drive the opening wedge to revive alien
SEC. 2. Such religious institutions, if not incorporated, shall hold the religious land holdings in this country. We can not ignore the historical fact
land in the name of three Trustees for the use of such that complaints against land holdings of that kind were among the factors
associations; . . .. (Printed Rec. App. p. 5.) that sparked the revolution of 1896.

and (2) that the refusal of the Register of Deeds violates the freedom of As to the complaint that the disqualification under article XIII is violative of the
religion clause of our Constitution [Art. III, Sec. 1(7)]. freedom of religion guaranteed by Article III of the Constitution, we are by no
means convinced (nor has it been shown) that land tenure is indispensable
We are of the opinion that the Court below has correctly held that in view of to the free exercise and enjoyment of religious profession or worship; or that
the absolute terms of section 5, Title XIII, of the Constitution, the provisions one may not worship the Deity according to the dictates of his own
of Act No. 271 of the old Philippine Commission must be deemed repealed conscience unless upon land held in fee simple.
since the Constitution was enacted, in so far as incompatible therewith. In
providing that, The resolution appealed from is affirmed, with costs against appellant.

Save in cases of hereditary succession, no private agricultural land Pablo, Acting C.J., Bengzon, Montemayor, Reyes, A., Bautista Angelo,
shall be transferred or assigned except to individuals, corporations or Labrador, and Concepcion, JJ., concur.

108
109
110
The essential facts are not in dispute. On November 4,1946, the Pacific
Airways Corporation registered its articles of incorporation with the Securities
and Exchanged Commission. The article were prepared and the registration
was effected by the accused, who was in fact the organizer of the
corporation. The article stated that the primary purpose of the corporation
was to carry on the business of a common carrier by air, land or water; that
its capital stock was P1,000,000, represented by 9,000 preferred and
G.R. No. L-6055 June 12, 1953
100,000 common shares, each preferred share being of the par value of
p100 and entitled to 1/3 vote and each common share, of the par value of P1
THE PEOPLE OF THE PHILIPPINES, plaintiff-appellee, and entitled to one vote; that the amount capital stock actually subscribed
vs. was P200,000, and the names of the subscribers were Arsenio Baylon, Eruin
WILLIAM H. QUASHA, defendant-appellant. E. Shannahan, Albert W. Onstott, James O'Bannon, Denzel J. Cavin, and
William H. Quasha, the first being a Filipino and the other five all Americans;
Jose P. Laurel for appellant and William H. Quasha in his own behalf. that Baylon's subscription was for 1,145 preferred shares, of the total value of
Office of the Solicitor General Juan R. Liwag and Assistant Solicitor General P114,500, and for 6,500 common shares, of the total par value of P6,500,
Francisco Carreon for appellee. while the aggregate subscriptions of the American subscribers were for 200
preferred shares, of the total par value of P20,000, and 59,000 common
REYES, J.: shares, of the total par value of P59,000; and that Baylon and the American
subscribers had already paid 25 per cent of their respective subscriptions.
William H. Quasha, a member of the Philippine bar, was charged in the Court Ostensibly the owner of, or subscriber to, 60.005 per cent of the subscribed
of First Instance of Manila with the crime of falsification of a public and capital stock of the corporation, Baylon nevertheless did not have the
commercial document in that, having been entrusted with the preparation controlling vote because of the difference in voting power between the
and registration of the article of incorporation of the Pacific Airways preferred shares and the common shares. Still, with the capital structure as it
Corporation, a domestic corporation organized for the purpose of engaging in was, the article of incorporation were accepted for registration and a
business as a common carrier, he caused it to appear in said article of certificate of incorporation was issued by the Securities and Exchange
incorporation that one Arsenio Baylon, a Filipino citizen, had subscribed to Commission.
and was the owner of 60.005 per cent of the subscribed capital stock of the
corporation when in reality, as the accused well knew, such was not the case, There is no question that Baylon actually subscribed to 60.005 per cent of
the truth being that the owner of the portion of the capital stock subscribed to the subscribed capital stock of the corporation. But it is admitted that the
by Baylon and the money paid thereon were American citizen whose name money paid on his subscription did not belong to him but to the Americans
did not appear in the article of incorporation, and that the purpose for making subscribers to the corporate stock. In explanation, the accused testified,
this false statement was to circumvent the constitutional mandate that no without contradiction, that in the process of organization Baylon was made a
corporation shall be authorize to operate as a public utility in the Philippines trustee for the American incorporators, and that the reason for making Baylon
unless 60 per cent of its capital stock is owned by Filipinos. such trustee was as follows:

Found guilty after trial and sentenced to a term of imprisonment and a fine,
the accused has appealed to this Court.

111
Q. According to this article of incorporation Arsenio Baylon xxx xxx xxx
subscribed to 1,135 preferred shares with a total value of P1,135. Do
you know how that came to be? 4. Making untruthful statements in a narration of facts.

A. Yes. ART. 172. Falsification by private individuals and use of falsified


documents. The penalty of prision correccional in its medium and
The people who were desirous of forming the corporation, whose names are maximum period and a fine of not more than 5,000 pesos shall be
listed on page 7 of this certified copy came to my house, Messrs. imposed upon:
Shannahan, Onstott, O'Bannon, Caven, Perry and Anastasakas one evening.
There was considerable difficulty to get them all together at one time xxx xxx xxx
because they were pilots. They had difficulty in deciding what their respective
share holdings would be. Onstott had invested a certain amount of money in 1. Any private individual who shall commit any of the falsifications
airplane surplus property and they had obtained a considerable amount of enumerated in the next preceding article in any public or official
money on those planes and as I recall they were desirous of getting a document or letter of exchange or any other kind of commercial
corporation formed right away. And they wanted to have their respective document.
shares holdings resolved at a latter date. They stated that they could get
together that they feel that they had no time to settle their respective share
Commenting on the above provision, Justice Albert, in his well-known work
holdings. We discussed the matter and finally it was decided that the best
on the Revised Penal Code ( new edition, pp. 407-408), observes, on the
way to handle the things was not to put the shares in the name of anyone of
authority of U.S. vs. Reyes, (1 Phil., 341), that the perversion of truth in the
the interested parties and to have someone act as trustee for their respective
narration of facts must be made with the wrongful intent of injuring a third
shares holdings. So we looked around for a trustee. And he said "There are a
person; and on the authority of U.S. vs. Lopez (15 Phil., 515), the same
lot of people whom I trust." He said, "Is there someone around whom we
author further maintains that even if such wrongful intent is proven, still the
could get right away?" I said, "There is Arsenio. He was my boy during the
untruthful statement will not constitute the crime of falsification if there is no
liberation and he cared for me when i was sick and i said i consider him my
legal obligation on the part of the narrator to disclose the truth. Wrongful
friend." I said. They all knew Arsenio. He is a very kind man and that was
intent to injure a third person and obligation on the part of the narrator to
what was done. That is how it came about.
disclose the truth are thus essential to a conviction for a crime of falsification
under the above article of the Revised Penal Code.
Defendant is accused under article 172 paragraph 1, in connection with
article 171, paragraph 4, of the Revised Penal Code, which read:
Now, as we see it, the falsification imputed in the accused in the present
case consists in not disclosing in the articles of incorporation that Baylon was
ART. 171. Falsification by public officer, employee, or notary or a mere trustee ( or dummy as the prosecution chooses to call him) of his
ecclesiastic minister. The penalty of prision mayor and a fine not American co-incorporators, thus giving the impression that Baylon was the
to exceed 5,000 pesos shall be imposed upon any public officer, owner of the shares subscribed to by him which, as above stated, amount to
employee, or notary who, taking advantage of his official position, 60.005 per cent of the sub-scribed capital stock. This, in the opinion of the
shall falsify a document by committing any of the following acts: trial court, is a malicious perversion of the truth made with the wrongful intent
circumventing section 8, Article XIV of the Constitution, which provides that "

112
no franchise, certificate, or any other form of authorization for the operation It is urged, however, that the formation of the corporation with 60 per cent of
of a public utility shall be granted except to citizens of the Philippines or to its subscribed capital stock appearing in the name of Baylon was an
corporation or other entities organized under the law of the Philippines, indispensable preparatory step to the subversion of the constitutional
sixty per centum of the capital of which is owned by citizens of the prohibition and the laws implementing the policy expressed therein. This view
Philippines . . . ." Plausible though it may appear at first glance, this opinion is not correct. For a corporation to be entitled to operate a public utility it is
loses validity once it is noted that it is predicated on the erroneous not necessary that it be organized with 60 per cent of its capital owned by
assumption that the constitutional provision just quoted was meant to prohibit Filipinos from the start. A corporation formed with capital that is entirely alien
the mere formation of a public utility corporation without 60 per cent of its may subsequently change the nationality of its capital through transfer of
capital being owned by the Filipinos, a mistaken belief which has induced the shares to Filipino citizens. conversely, a corporation originally formed with
lower court to that the accused was under obligation to disclose the whole Filipino capital may subsequently change the national status of said capital
truth about the nationality of the subscribed capital stock of the corporation through transfer of shares to foreigners. What need is there then for a
by revealing that Baylon was a mere trustee or dummy of his American co- corporation that intends to operate a public utility to have, at the time of its
incorporators, and that in not making such disclosure defendant's intention formation, 60 per cent of its capital owned by Filipinos alone? That condition
was to circumvent the Constitution to the detriment of the public interests. may anytime be attained thru the necessary transfer of stocks. The moment
Contrary to the lower court's assumption, the Constitution does not prohibit for determining whether a corporation is entitled to operate as a public utility
the mere formation of a public utility corporation without the required is when it applies for a franchise, certificate, or any other form of
formation of Filipino capital. What it does prohibit is the granting of a authorization for that purpose. And that can be done after the corporation has
franchise or other form of authorization for the operation of a public utility to already come into being and not while it is still being formed. And at that
a corporation already in existence but without the requisite proportion of moment, the corporation must show that it has complied not only with the
Filipino capital. This is obvious from the context, for the constitutional requirement of the Constitution as to the nationality of its capital, but also
provision in question qualifies the terms " franchise", "certificate", or "any with the requirements of the Civil Aviation Law if it is a common carrier by air,
other form of authorization" with the phrase "for the operation of a public the Revised Administrative Code if it is a common carrier by water, and the
utility," thereby making it clear that the franchise meant is not the "primary Public Service Law if it is a common carrier by land or other kind of public
franchise" that invest a body of men with corporate existence but the service.
"secondary franchise" or the privilege to operate as a public utility after the
corporation has already come into being. Equally untenable is the suggestion that defendant should at least be held
guilty of an "impossible crime" under article 59 of the Revised Penal Code. It
If the Constitution does not prohibit the mere formation of a public utility not being possible to suppose that defendant had intended to commit a crime
corporation with the alien capital, then how can the accused be charged with for the simple reason that the alleged constitutional prohibition which he is
having wrongfully intended to circumvent that fundamental law by not charged for having tried to circumvent does not exist, conviction under that
revealing in the articles of incorporation that Baylon was a mere trustee of his article is out of the question.
American co-incorporation and that for that reason the subscribed capital
stock of the corporation was wholly American? For the mere formation of the The foregoing consideration can not but lead to the conclusion that the
corporation such revelation was not essential, and the Corporation Law does defendant can not be held guilty of the crime charged. The majority of the
not require it. Defendant was, therefore, under no obligation to make it. In the court, however, are also of the opinion that, even supposing that the act
absence of such obligation and of the allege wrongful intent, defendant imputed to the defendant constituted falsification at the time it was
cannot be legally convicted of the crime with which he is charged. perpetrated, still with the approval of the Party Amendment to the

113
Constitution in March, 1947, which placed Americans on the same footing as
Filipino citizens with respect to the right to operate public utilities in the
Philippines, thus doing away with the prohibition in section 8, Article XIV of
the Constitution in so far as American citizens are concerned, the said act
has ceased to be an offense within the meaning of the law, so that defendant
can no longer be held criminally liable therefor.

In view of the foregoing, the judgment appealed from is reversed and the
defendant William H. Quasha acquitted, with costs de oficio.

Paras, C.J., Pablo, Bengzon, Padilla, Tuason, Jugo, Bautista Angelo, and
Labrador, JJ., concur.

114
Ramirez and Ortigas for petitioner.
Ewald Huenefeld for respondent.

PARAS, C.J.:

On October 1, 1941, the respondent corporation, Christern Huenefeld, & Co.,


Inc., after payment of corresponding premium, obtained from the petitioner
,Filipinas Cia. de Seguros, fire policy No. 29333 in the sum of P1000,000,
covering merchandise contained in a building located at No. 711 Roman
Street, Binondo Manila. On February 27, 1942, or during the Japanese
military occupation, the building and insured merchandise were burned. In
due time the respondent submitted to the petitioner its claim under the policy.
The salvage goods were sold at public auction and, after deducting their
value, the total loss suffered by the respondent was fixed at P92,650. The
petitioner refused to pay the claim on the ground that the policy in favor of
the respondent had ceased to be in force on the date the United States
declared war against Germany, the respondent Corporation (though
organized under and by virtue of the laws of the Philippines) being controlled
by the German subjects and the petitioner being a company under American
jurisdiction when said policy was issued on October 1, 1941. The petitioner,
however, in pursuance of the order of the Director of Bureau of Financing,
Philippine Executive Commission, dated April 9, 1943, paid to the respondent
the sum of P92,650 on April 19, 1943.

The present action was filed on August 6, 1946, in the Court of First Instance
of Manila for the purpose of recovering from the respondent the sum of
P92,650 above mentioned. The theory of the petitioner is that the insured
merchandise were burned up after the policy issued in 1941 in favor of the
respondent corporation has ceased to be effective because of the outbreak
of the war between the United States and Germany on December 10, 1941,
G.R. No. L-2294 May 25, 1951 and that the payment made by the petitioner to the respondent corporation
during the Japanese military occupation was under pressure. After trial, the
FILIPINAS COMPAIA DE SEGUROS, petitioner, Court of First Instance of Manila dismissed the action without pronouncement
vs. as to costs. Upon appeal to the Court of Appeals, the judgment of the Court
CHRISTERN, HUENEFELD and CO., INC., respondent. of First Instance of Manila was affirmed, with costs. The case is now before
us on appeal by certiorari from the decision of the Court of Appeals.

115
The Court of Appeals overruled the contention of the petitioner that the World War II revived the problem again. It was known that German
respondent corporation became an enemy when the United States declared and other enemy interests were cloaked by domestic corporation
war against Germany, relying on English and American cases which held that structure. It was not only by legal ownership of shares that a material
a corporation is a citizen of the country or state by and under the laws of influence could be exercised on the management of the corporation
which it was created or organized. It rejected the theory that nationality of but also by long term loans and other factual situations. For that
private corporation is determine by the character or citizenship of its reason, legislation on enemy property enacted in various countries
controlling stockholders. during World War II adopted by statutory provisions to the control
test and determined, to various degrees, the incidents of control.
There is no question that majority of the stockholders of the respondent Court decisions were rendered on the basis of such newly enacted
corporation were German subjects. This being so, we have to rule that said statutory provisions in determining enemy character of domestic
respondent became an enemy corporation upon the outbreak of the war corporation.
between the United States and Germany. The English and American cases
relied upon by the Court of Appeals have lost their force in view of the latest The United States did not, in the amendments of the Trading with the
decision of the Supreme Court of the United States in Clark vs. Uebersee Enemy Act during the last war, include as did other legislations the
Finanz Korporation, decided on December 8, 1947, 92 Law. Ed. Advance applications of the control test and again, as in World War I, courts
Opinions, No. 4, pp. 148-153, in which the controls test has been adopted. In refused to apply this concept whereby the enemy character of an
"Enemy Corporation" by Martin Domke, a paper presented to the Second American or neutral-registered corporation is determined by the
International Conference of the Legal Profession held at the Hague enemy nationality of the controlling stockholders.
(Netherlands) in August. 1948 the following enlightening passages appear:
Measures of blocking foreign funds, the so called freezing
Since World War I, the determination of enemy nationality of regulations, and other administrative practice in the treatment of
corporations has been discussion in many countries, belligerent and foreign-owned property in the United States allowed to large degree
neutral. A corporation was subject to enemy legislation when it was the determination of enemy interest in domestic corporations and
controlled by enemies, namely managed under the influence of thus the application of the control test. Court decisions sanctioned
individuals or corporations, themselves considered as enemies. It such administrative practice enacted under the First War Powers Act
was the English courts which first the Daimler case applied this new of 1941, and more recently, on December 8, 1947, the Supreme
concept of "piercing the corporate veil," which was adopted by the Court of the United States definitely approved of the control theory. In
peace of Treaties of 1919 and the Mixed Arbitral established after the Clark vs. Uebersee Finanz Korporation, A. G., dealing with a Swiss
First World War. corporation allegedly controlled by German interest, the Court: "The
property of all foreign interest was placed within the reach of the
The United States of America did not adopt the control test during vesting power (of the Alien Property Custodian) not to appropriate
the First World War. Courts refused to recognized the concept friendly or neutral assets but to reach enemy interest which
whereby American-registered corporations could be considered as masqueraded under those innocent fronts. . . . The power of seizure
enemies and thus subject to domestic legislation and administrative and vesting was extended to all property of any foreign country or
measures regarding enemy property. national so that no innocent appearing device could become a Trojan
horse."

116
It becomes unnecessary, therefore, to dwell at length on the authorities cited In the case of an ordinary fire policy, which grants insurance only
in support of the appealed decision. However, we may add that, in Haw Pia from year, or for some other specified term it is plain that when the
vs. China Banking Corporation,* 45 Off Gaz., (Supp. 9) 299, we already held parties become alien enemies, the contractual tie is broken and the
that China Banking Corporation came within the meaning of the word contractual rights of the parties, so far as not vested. lost. (Vance,
"enemy" as used in the Trading with the Enemy Acts of civilized countries not the Law on Insurance, Sec. 44, p. 112.)
only because it was incorporated under the laws of an enemy country but
because it was controlled by enemies. The respondent having become an enemy corporation on December 10,
1941, the insurance policy issued in its favor on October 1, 1941, by the
The Philippine Insurance Law (Act No. 2427, as amended,) in section 8, petitioner (a Philippine corporation) had ceased to be valid and enforcible,
provides that "anyone except a public enemy may be insured." It stands to and since the insured goods were burned after December 10, 1941, and
reason that an insurance policy ceases to be allowable as soon as an during the war, the respondent was not entitled to any indemnity under said
insured becomes a public enemy. policy from the petitioner. However, elementary rules of justice (in the
absence of specific provision in the Insurance Law) require that the premium
Effect of war, generally. All intercourse between citizens of paid by the respondent for the period covered by its policy from December
belligerent powers which is inconsistent with a state of war is 11, 1941, should be returned by the petitioner.
prohibited by the law of nations. Such prohibition includes all
negotiations, commerce, or trading with the enemy; all acts which will The Court of Appeals, in deciding the case, stated that the main issue hinges
increase, or tend to increase, its income or resources; all acts of on the question of whether the policy in question became null and void upon
voluntary submission to it; or receiving its protection; also all acts the declaration of war between the United States and Germany on December
concerning the transmission of money or goods; and all contracts 10, 1941, and its judgment in favor of the respondent corporation was
relating thereto are thereby nullified. It further prohibits insurance predicated on its conclusion that the policy did not cease to be in force. The
upon trade with or by the enemy, upon the life or lives of aliens Court of Appeals necessarily assumed that, even if the payment by the
engaged in service with the enemy; this for the reason that the petitioner to the respondent was involuntary, its action is not tenable in view
subjects of one country cannot be permitted to lend their assistance of the ruling on the validity of the policy. As a matter of fact, the Court of
to protect by insurance the commerce or property of belligerent, alien Appeals held that "any intimidation resorted to by the appellee was not unjust
subjects, or to do anything detrimental too their country's interest. but the exercise of its lawful right to claim for and received the payment of
The purpose of war is to cripple the power and exhaust the the insurance policy," and that the ruling of the Bureau of Financing to the
resources of the enemy, and it is inconsistent that one country effect that "the appellee was entitled to payment from the appellant was, well
should destroy its enemy's property and repay in insurance the value founded." Factually, there can be no doubt that the Director of the Bureau of
of what has been so destroyed, or that it should in such manner Financing, in ordering the petitioner to pay the claim of the respondent,
increase the resources of the enemy, or render it aid, and the merely obeyed the instruction of the Japanese Military Administration, as
commencement of war determines, for like reasons, all trading may be seen from the following: "In view of the findings and conclusion of
intercourse with the enemy, which prior thereto may have been this office contained in its decision on Administrative Case dated February 9,
lawful. All individuals therefore, who compose the belligerent powers, 1943 copy of which was sent to your office and the concurrence therein of
exist, as to each other, in a state of utter exclusion, and are public the Financial Department of the Japanese Military Administration,
enemies. (6 Couch, Cyc. of Ins. Law, pp. 5352-5353.) and following the instruction of said authority, you are hereby ordered to pay
the claim of Messrs. Christern, Huenefeld & Co., Inc. The payment of said

117
claim, however, should be made by means of crossed check." (Emphasis
supplied.)

It results that the petitioner is entitled to recover what paid to the respondent
under the circumstances on this case. However, the petitioner will be entitled
to recover only the equivalent, in actual Philippines currency of P92,650 paid
on April 19, 1943, in accordance with the rate fixed in the Ballantyne scale.

Wherefore, the appealed decision is hereby reversed and the respondent


corporation is ordered to pay to the petitioner the sum of P77,208.33,
Philippine currency, less the amount of the premium, in Philippine currency,
that should be returned by the petitioner for the unexpired term of the policy
in question, beginning December 11, 1941. Without costs. So ordered.

Feria, Pablo, Bengzon, Tuason, Montemayor, Jugo and Bautista Angelo,


JJ., concur.

118
G.R. No. L-8451 December 20, 1957

THE ROMAN CATHOLIC APOSTOLIC ADMINISTRATOR OF DAVAO,


INC., petitioner,
vs.
THE LAND REGISTRATION COMMISSION and THE REGISTER OF
DEEDS OF DAVAO CITY, respondents.

Teodoro Padilla, for petitioner.


Office of the Solicitor General Ambrosio Padilla, Assistant Solicitor General
Jose G. Bautista and Troadio T. Quianzon, Jr., for respondents.

119
As the Register of Deeds entertained some doubts as to the registerability if
the document, the matter was referred to the Land Registration
FELIX, J.: Commissioner en consulta for resolution in accordance with section 4 of
Republic Act No. 1151. Proper hearing on the matter was conducted by the
This is a petition for mandamus filed by the Roman Catholic Apostolic Commissioner and after the petitioner corporation had filed its memorandum,
Administrator of Davao seeking the reversal of a resolution by the Land a resolution was rendered on September 21, 1954, holding that in view of the
Registration Commissioner in L.R.C. Consulta No. 14. The facts of the case provisions of Section 1 and 5 of Article XIII of the Philippine Constitution, the
are as follows: vendee was not qualified to acquire private lands in the Philippines in the
absence of proof that at least 60 per centum of the capital, property, or
assets of the Roman Catholic Apostolic Administrator of Davao, Inc., was
On October 4, 1954, Mateo L. Rodis, a Filipino citizen and resident of the
actually owned or controlled by Filipino citizens, there being no question that
City of Davao, executed a deed of sale of a parcel of land located in the
the present incumbent of the corporation sole was a Canadian citizen. It was
same city covered by Transfer Certificate No. 2263, in favor of the Roman
also the opinion of the Land Registration Commissioner that section 159 of
Catholic Apostolic Administrator of Davao Inc., s corporation sole organized
the corporation Law relied upon by the vendee was rendered operative by
and existing in accordance with Philippine Laws, with Msgr. Clovis Thibault, a
the aforementioned provisions of the Constitution with respect to real estate,
Canadian citizen, as actual incumbent. When the deed of sale was presented
unless the precise condition set therein that at least 60 per cent of its
to Register of Deeds of Davao for registration, the latter.
capital is owned by Filipino citizens be present, and, therefore, ordered
the Registered Deeds of Davao to deny registration of the deed of sale in the
having in mind a previous resolution of the Fourth Branch of the absence of proof of compliance with such condition.
Court of First Instance of Manila wherein the Carmelite Nuns of
Davao were made to prepare an affidavit to the effect that 60 per
After the motion to reconsider said resolution was denied, an action
cent of the members of their corporation were Filipino citizens when
for mandamus was instituted with this Court by said corporation sole, alleging
they sought to register in favor of their congregation of deed of
that under the Corporation Law as well as the settled jurisprudence on the
donation of a parcel of land
matter, the deed of sale executed by Mateo L. Rodis in favor of petitioner is
actually a deed of sale in favor of the Catholic Church which is qualified to
required said corporation sole to submit a similar affidavit declaring that 60 acquire private agricultural lands for the establishment and maintenance of
per cent of the members thereof were Filipino citizens. places of worship, and prayed that judgment be rendered reserving and
setting aside the resolution of the Land Registration Commissioner in
The vendee in the letter dated June 28, 1954, expressed willingness to question. In its resolution of November 15, 1954, this Court gave due course
submit an affidavit, both not in the same tenor as that made the Progress of to this petition providing that the procedure prescribed for appeals from the
the Carmelite Nuns because the two cases were not similar, for whereas the Public Service Commission of the Securities and Exchange Commissions
congregation of the Carmelite Nuns had five incorporators, the corporation (Rule 43), be followed.
sole has only one; that according to their articles of incorporation, the
organization of the Carmelite Nuns became the owner of properties donated Section 5 of Article XIII of the Philippine Constitution reads as follows:
to it, whereas the case at bar, the totality of the Catholic population of Davao
would become the owner of the property bought to be registered.
SEC. 5. Save in cases of hereditary succession, no private
agricultural land shall be transferred or assigned except to

120
individuals, corporations, or associations qualified to acquire or hold And elaborating on the composition of the Catholic Church in the Philippines,
lands of the public domain in the Philippines. petitioner explained that as a religious society or organization, it is made up
of 2 elements or divisions the clergy or religious members and the faithful
Section 1 of the same Article also provides the following: or lay members. The 1948 figures of the Bureau of Census showed that
there were 277,551 Catholics in Davao and aliens residing therein numbered
SECTION 1. All agricultural, timber, and mineral lands of the public domain, 3,465. Ever granting that all these foreigners are Catholics, petitioner
water, minerals, coal, petroleum, and other mineral oils, all forces of potential contends that Filipino citizens form more than 80 per cent of the entire
energy, and other natural resources of the Philippines belong to the State, Catholics population of that area. As to its clergy and religious composition,
and their disposition, exploitation, development, or utilization shall be limited counsel for petitioner presented the Catholic Directory of the Philippines for
to cititzens of the Philippines, or to corporations or associations at least sixty 1954 (Annex A) which revealed that as of that year, Filipino clergy and
per centum of the capital of which is owned by such citizens, SUBJECT TO women novices comprise already 60.5 per cent of the group. It was,
ANY EXISTING RIGHT, grant, lease, or concession AT THE TIME OF THE therefore, allowed that the constitutional requirement was fully met and
INAUGURATION OF THE GOVERNMENT ESTABLISHED UNDER satisfied.
CONSTITUTION. Natural resources, with the exception of public agricultural
land, shall not be alienated, and no license, concession, or leases for the Respondents, on the other hand, averred that although it might be true that
exploitation, development, or utilization of any of the natural resources shall petitioner is not the owner of the land purchased, yet he has control over the
be granted for a period exceeding twenty-five years, renewable for another same, with full power to administer, take possession of, alienate, transfer,
twenty-five years, except as to water rights for irrigation, water supply, encumber, sell or dispose of any or all lands and their improvements
fisheries, or industrial uses other than the development of water power, in registered in the name of the corporation sole and can collect, receive,
which cases other than the development and limit of the grant. demand or sue for all money or values of any kind that may be kind that may
become due or owing to said corporation, and vested with authority to enter
In virtue of the foregoing mandates of the Constitution, who are considered into agreements with any persons, concerns or entities in connection with
"qualified" to acquire and hold agricultural lands in the Philippines? What is said real properties, or in other words, actually exercising all rights of
the effect of these constitutional prohibition of the right of a religious ownership over the properties. It was their stand that the theory that
corporation recognized by our Corporation Law and registered as properties registered in the name of the corporation sole are held in true for
a corporation sole, to possess, acquire and register real estates in its name the benefit of the Catholic population of a place, as of Davao in the case at
when the Head, Manager, Administrator or actual incumbent is an alien? bar should be sustained because a conglomeration of persons cannot just be
pointed out as the cestui que trust or recipient of the benefits from the
property allegedly administered in their behalf. Neither can it be said that the
Petitioner consistently maintained that a corporation sole, irrespective of the
mass of people referred to as such beneficiary exercise ant right of
citizenship of its incumbent, is not prohibited or disqualified to acquire and
ownership over the same. This set-up, respondents argued, falls short of a
hold real properties. The Corporation Law and the Canon Law are explicit in
trust. The respondents instead tried to prove that in reality, the beneficiary of
their provisions that a corporation sole or "ordinary" is not the owner of the of
ecclesiastical properties are not members or faithful of the church but
the properties that he may acquire but merely the administrator thereof. The
someone else, by quoting a portion a portion of the ought of fidelity
Canon Law also specified that church temporalities are owned by the
subscribed by a bishop upon his elevation to the episcopacy wherein he
Catholic Church as a "moral person" or by the diocess as minor "moral
promises to render to the Pontificial Father or his successors an account of
persons" with the ordinary or bishop as administrator.
his pastoral office and of all things appertaining to the state of this church.

121
Respondents likewise advanced the opinion that in construing the SEC. 155. In order to become a corporation sole the bishop, chief
constitutional provision calling for 60 per cent of Filipino citizenship, the priest, or presiding elder of any religious denomination, society or
criterion of the properties or assets thereof. church must file with the Securities and Exchange Commissioner
articles of incorporation setting forth the following facts:
In solving the problem thus submitted to our consideration, We can say the
following: A corporation sole is a special form of corporation usually xxx xxx xxx.
associated with the clergy. Conceived and introduced into the common law
by sheer necessity, this legal creation which was referred to as "that unhappy (3) That as such bishop, chief priest, or presiding elder he is charged
freak of English law" was designed to facilitate the exercise of the functions with the administration of the temporalities and the management of
of ownership carried on by the clerics for and on behalf of the church which the estates and properties of his religious denomination, society, or
was regarded as the property owner (See I Couvier's Law Dictionary, p. 682- church within its territorial jurisdiction, describing it;
683).
xxx xxx xxx.
A corporation sole consists of one person only, and his successors (who will
always be one at a time), in some particular station, who are incorporated by (As amended by Commonwealth Act No. 287).
law in order to give them some legal capacities and advantages, particularly
that of perpetuity, which in their natural persons they could not have had. In
SEC. 157. From and after the filing with the Securities and Exchange
this sense, the king is a sole corporation; so is a bishop, or dens, distinct
Commissioner of the said articles of incorporation, which verified by
from their several chapters (Reid vs. Barry, 93 Fla. 849, 112 So. 846).
affidavit or affirmation as aforesaid and accompanied by the copy of
the commission, certificate of election, or letters of appointment of
The provisions of our Corporation law on religious corporations are the bishop, chief priest, or presiding elder, duly certified as
illuminating and sustain the stand of petitioner. Section 154 thereof provides: prescribed in the section immediately preceding such the bishop,
chief priest, or presiding elder, as the case may be, shall become a
SEC. 154. For the administration of the temporalities of any corporation sole and all temporalities, estates, and properties the
religious denomination, society or church and the management of religious denomination, society, or church therefore administered or
the estates and the properties thereof, it shall be lawful for the managed by him as such bishop, chief priest, or presiding elder,
bishop, chief priest, or presiding either of any such religious shall be held in trust by him as a corporation sole, for the use,
denomination, society or church to become a corporation sole, purpose, behalf, and sole benefit of his religious denomination,
unless inconsistent wit the rules, regulations or discipline of his society, or church, including hospitals, schools, colleges, orphan,
religious denomination, society or church or forbidden by competent asylums, parsonages, and cemeteries thereof. For the filing of such
authority thereof. articles of incorporation, the Securities and Exchange Commissioner
shall collect twenty-five pesos. (As amended by Commonwealth Act.
See also the pertinent provisions of the succeeding sections of the same No. 287); and.
Corporation Law copied hereunder:
SEC. 163. The right to administer all temporalities and all property
held or owned by a religious order or society, or by the diocese,

122
synod, or district organization of any religious denomination or properties do not pass to the administrators, who are the owners of church
church shall, on its incorporation, pass to the corporation and shall properties?.
be held in trust for the use, purpose behalf, and benefit of the
religious society, or order so incorporated or of the church of which Bouscaren and Elis, S.J., authorities on cannon law, on their treatise
the diocese, or district organization is an organized and constituent comment:
part.
In matters regarding property belonging to the Universal Church and
The Cannon Law contains similar provisions regarding the duties of the to the Apostolic See, the Supreme Pontiff exercises his office of
corporation sole or ordinary as administrator of the church properties, as supreme administrator through the Roman Curia; in matters
follows: regarding other church property, through the administrators of the
individual moral persons in the Church according to that norms, laid
Al Ordinario local pertenence vigilar diligentemente sobre down in the Code of Cannon Law. This does not mean, however,
la administracion de todos los bienes eclesiasticos que se hallan en that the Roman Pontiff is the owner of all the church property; but
su territorio y no estuvieren sustraidos de su jurisdiccion, salvs las merely that he is the supreme guardian (Bouscaren and Ellis,
prescriciones legitimas que le concedan mas aamplios derechos. Cannon Law, A Text and Commentary, p. 764).

Teniendo en cuenta los derechos y las legitimas costumbres y and this Court, citing Campes y Pulido, Legislacion y Jurisprudencia
circunstancias, procuraran los Ordinarios regular todo lo Canonica, ruled in the case of Trinidad vs. Roman Catholic Archbishop of
concerniente a la administracion de los bienes eclesciasticos, dando Manila, 63 Phil. 881, that:
las oportunas instucciones particularles dentro del narco del derecho
comun. (Title XXVIII, Codigo de Derecho Canonico, Lib. III, Canon The second question to be decided is in whom the ownership of the
1519).1 properties constituting the endowment of the ecclesiastical or
collative chaplaincies is vested.
That leaves no room for doubt that the bishops or archbishops, as the case
may be, as corporation's sole are merely administrators of the church Canonists entertain different opinions as to the persons in whom the
properties that come to their possession, in which they hold in trust for the ownership of the ecclesiastical properties is vested, with respect to
church. It can also be said that while it is true that church properties could be which we shall, for our purpose, confine ourselves to stating with
administered by a natural persons, problems regarding succession to said Donoso that, while many doctors cited by Fagnano believe that it
properties can not be avoided to rise upon his death. Through this legal resides in the Roman Pontiff as Head of the Universal Church, it is
fiction, however, church properties acquired by the incumbent of a more probable that ownership, strictly speaking, does not reside in
corporation sole pass, by operation of law, upon his death not his personal the latter, and, consequently, ecclesiastical properties are owned by
heirs but to his successor in office. It could be seen, therefore, that a the churches, institutions and canonically established private
corporation sole is created not only to administer the temporalities of the corporations to which said properties have been donated.
church or religious society where he belongs but also to hold and transmit
the same to his successor in said office. If the ownership or title to the Considering that nowhere can We find any provision conferring ownership of
church properties on the Pope although he appears to be the supreme

123
administrator or guardian of his flock, nor on the corporation sole or heads of Vatican, yet it cannot be said that there is a merger of personalities resultant
dioceses as they are admittedly mere administrators of said properties, therein. Neither can it be said that the political and civil rights of the faithful,
ownership of these temporalities logically fall and develop upon the church, inherent or acquired under the laws of their country, are affected by that
diocese or congregation acquiring the same. Although this question of relationship with the Pope. The fact that the Roman Catholic Church in
ownership of ecclesiastical properties has off and on been mentioned in almost every country springs from that society that saw its beginning in
several decisions of the Court yet in no instance was the subject of Europe and the fact that the clergy of this faith derive their authorities and
citizenship of this religious society been passed upon. receive orders from the Holy See do not give or bestow the citizenship of the
Pope upon these branches. Citizenship is a political right which cannot be
We are not unaware of the opinion expressed by the late Justice Perfecto in acquired by a sort of "radiation". We have to realize that although there is a
his dissent in the case of Agustines vs. Court of First Instance of Bulacan, 80 fraternity among all the catholic countries and the dioceses therein all over
Phil. 565, to the effect that "the Roman Catholic Archbishop of Manila is only the globe, the universality that the word "catholic" implies, merely
a branch of a universal church by the Pope, with permanent residence in characterize their faith, a uniformity in the practice and the interpretation of
Rome, Italy". There is no question that the Roman Catholic Church existing their dogma and in the exercise of their belief, but certainly they are separate
in the Philippines is a tributary and part of the international religious and independent from one another in jurisdiction, governed by different laws
organization, for the word "Roman" clearly expresses its unity with and under which they are incorporated, and entirely independent on the others in
recognizes the authority of the Pope in Rome. However, lest We become the management and ownership of their temporalities. To allow theory that
hasty in drawing conclusions, We have to analyze and take note of the the Roman Catholic Churches all over the world follow the citizenship of their
nature of the government established in the Vatican City, of which it was said: Supreme Head, the Pontifical Father, would lead to the absurdity of finding
the citizens of a country who embrace the Catholic faith and become
GOVERNMENT. In the Roman Catholic Church supreme authority members of that religious society, likewise citizens of the Vatican or of Italy.
and jurisdiction over clergy and laity alike as held by the pope who And this is more so if We consider that the Pope himself may be an Italian or
(since the Middle Ages) is elected by the cardinals assembled in national of any other country of the world. The same thing be said with
conclave, and holds office until his death or legitimate abdication. . . regard to the nationality or citizenship of the corporation sole created under
While the pope is obviously independent of the laws made, and the the laws of the Philippines, which is not altered by the change of citizenship
officials appointed, by himself or his predecessors, he usually of the incumbent bishops or head of said corporation sole.
exercises his administrative authority according to the code of canon
law and through the congregations, tribunals and offices of the Curia We must therefore, declare that although a branch of the Universal Roman
Romana. In their respective territories (called generally dioceses) Catholic Apostolic Church, every Roman Catholic Church in different
and over their respective subjects, the patriarchs, metropolitans or countries, if it exercises its mission and is lawfully incorporated in accordance
archbishops and bishops exercise a jurisdiction which is called with the laws of the country where it is located, is considered an entity or
ordinary (as attached by law to an office given to a person. . . person with all the rights and privileges granted to such artificial being under
(Collier's Encyclopedia, Vol. 17, p. 93). the laws of that country, separate and distinct from the personality of the
Roman Pontiff or the Holy See, without prejudice to its religious relations with
While it is true and We have to concede that in the profession of their faith, the latter which are governed by the Canon Law or their rules and
the Roman Pontiff is the supreme head; that in the religious matters, in the regulations.
exercise of their belief, the Catholic congregation of the faithful throughout
the world seeks the guidance and direction of their Spiritual Father in the

124
We certainly are conscious of the fact that whatever conclusion We may the new dioceses, one of them being petitioner herein, the Roman Catholic
draw on this matter will have a far reaching influence, nor can We overlook Apostolic Administrator of Davao, Inc., which was registered with the
the pages of history that arouse indignation and criticisms against church Securities and Exchange Commission on September 12, 1950, and
landholdings. This nurtured feeling that snowbailed into a strong nationalistic succeeded in the administrative for all the "temporalities" of the Roman
sentiment manifested itself when the provisions on natural to be embodied in Catholic Church existing in Davao.
the Philippine Constitution were framed, but all that has been said on this
regard referred more particularly to landholdings of religious corporations According to our Corporation Law, Public Act No. 1549, approved April 1,
known as "Friar Estates" which have already bee acquired by our 1906, a corporation sole.
government, and not to properties held by corporations sole which, We
repeat, are properties held in trust for the benefit of the faithful residing within is organized and composed of a single individual, the head of any
its territorial jurisdiction. Though that same feeling probably precipitated and religious society or church, for the ADMINISTRATION of the
influenced to a large extent the doctrine laid down in the celebrated Krivenco temporalities of such society or church. By "temporalities" is meant
decision, We have to take this matter in the light of legal provisions and estate and properties not used exclusively for religious worship. The
jurisprudence actually obtaining, irrespective of sentiments. successor in office of such religious head or chief priest incorporated
as a corporation sole shall become the corporation sole on
The question now left for our determination is whether the Universal Roman ascension to office, and shall be permitted to transact business as
Catholic Apostolic Church in the Philippines, or better still, the corporation such on filing with the Securities and Exchange Commission a copy
sole named the Roman Catholic Apostolic Administrator of Davao, Inc., is of his commission, certificate of election or letter of appointment duly
qualified to acquire private agricultural lands in the Philippines pursuant to certified by any notary public or clerk of court of record (Guevara's
the provisions of Article XIII of the Constitution. The Philippine Corporation Law, p. 223).

We see from sections 1 and 5 of said Article quoted before, that only persons The Corporation Law also contains the following provisions:
or corporations qualified to acquire hold lands of the public domain in the
Philippines may acquire or be assigned and hold private agricultural lands. SECTION 159. Any corporation sole may purchase and hold real
Consequently, the decisive factor in the present controversy hinges on the estate and personal; property for its church, charitable, benevolent,
proposition or whether or not the petitioner in this case can acquire or educational purposes, and may receive bequests or gifts of such
agricultural lands of the public domain. purposes. Such corporation may mortgage or sell real property held
by it upon obtaining an order for that purpose from the Court of First
From the data secured from the Securities and Exchange Commission, We Instance of the province in which the property is situated; but before
find that the Roman Catholic Bishop of Zamboanga was incorporated (as a making the order proof must be made to the satisfaction of the Court
corporation sole) in September, 1912, principally to administer its that notice of the application for leave to mortgage or sell has been
temporalities and manage its properties. Probably due to the ravages of the given by publication or otherwise in such manner and for such time
last war, its articles of incorporation were reconstructed in the Securities and as said Court or the Judge thereof may have directed, and that it is
Exchange Commission on April 8, 1948. At first, this corporation sole to the interest of the corporation that leave to mortgage or sell must
administered all the temporalities of the church existing or located in the be made by petition, duly verified by the bishop, chief priest, or
island of Mindanao. Later on, however, new dioceses were formed and new presiding elder acting as corporation sole, and may be opposed by
corporations sole were created to correspond with the territorial jurisdiction of

125
any member of the religious denomination, society or church for which it was created, independently of the nationality of its incumbent
represented by the corporation sole: Provided, however, That in unique and single member and head, the bishop of the dioceses. It can be
cases where the rules, regulations, and discipline of the religious also maintained without fear of being gainsaid that the Roman Catholic
denomination, society or church concerned represented by such Apostolic Church in the Philippines has no nationality and that the framers of
corporation sole regulate the methods of acquiring, holding, selling the Constitution, as will be hereunder explained, did not have in mind the
and mortgaging real estate and personal property, such rules, religious corporations sole when they provided that 60 per centum of the
regulations, and discipline shall control and the intervention of the capital thereof be owned by Filipino citizens.
Courts shall not be necessary.
There could be no controversy as to the fact that a duly registered
It can, therefore, be noticed that the power of a corporation sole to corporation sole is an artificial being having the right of succession and the
purchase real property, like the power exercised in the case at bar, it is not power, attributes, and properties expressly authorized by law or incident to its
restricted although the power to sell or mortgage sometimes is, depending existence (section 1, Corporation Law). In outlining the general powers of a
upon the rules, regulations, and discipline of the church concerned corporation. Public Act. No. 1459 provides among others:
represented by said corporation sole. If corporations sole can purchase and
sell real estate for its church, charitable, benevolent, or educational SEC. 13. Every corporation has the power:
purposes, can they register said real properties? As provided by law, lands
held in trust for specific purposes me be subject of registration (section 69, (5) To purchase, hold, convey, sell, lease, lot, mortgage, encumber,
Act 496), and the capacity of a corporation sole, like petitioner herein, to and otherwise deal with such real and personal property as the
register lands belonging to it is acknowledged, and title thereto may be purpose for which the corporation was formed may permit, and the
issued in its name (Bishop of Nueva Segovia vs. Insular Government, 26 transaction of the lawful business of the corporation may reasonably
Phil. 300-1913). Indeed it is absurd that while the corporations sole that and necessarily require, unless otherwise prescribed in this Act: . . .
might be in need of acquiring lands for the erection of temples where the
faithful can pray, or schools and cemeteries which they are expressly
In implementation of the same and specially made applicable to a form of
authorized by law to acquire in connection with the propagation of the Roman
corporation recognized by the same law, Section 159 aforequoted expressly
Catholic Apostolic faith or in furtherance of their freedom of religion they
allowed the corporation sole to purchase and hold real as well as personal
could not register said properties in their name. As professor Javier J.
properties necessary for the promotion of the objects for which said
Nepomuceno very well says "Man in his search for the immortal and
corporation sole is created. Respondent Land Registration Commissioner,
imponderable, has, even before the dawn of recorded history, erected
however, maintained that since the Philippine Constitution is a later
temples to the Unknown God, and there is no doubt that he will continue to
enactment than public Act No. 1459, the provisions of Section 159 in
do so for all time to come, as long as he continues 'imploring the aid of
amplification of Section 13 thereof, as regard real properties, should be
Divine Providence'" (Nepomuceno's Corporation Sole, VI Ateneo Law
considered repealed by the former.
Journal, No. 1, p. 41, September, 1956). Under the circumstances of this
case, We might safely state that even before the establishment of the
Philippine Commonwealth and of the Republic of the Philippines every There is a reason to believe that when the specific provision of the
corporation sole then organized and registered had by express provision of Constitution invoked by respondent Commissioner was under consideration,
law the necessary power and qualification to purchase in its name private the framers of the same did not have in mind or overlooked this particular
lands located in the territory in which it exercised its functions or ministry and form of corporation. It is undeniable that the naturalization and conservation

126
of our national resources was one of the dominating objectives of the already prepared and capable of doing so. But that is not the case of
Convention and in drafting the present Article XII of the Constitution, the corporations sole in the Philippines, for, We repeat, they are mere
delegates were goaded by the desire (1) to insure their conservation for administrators of the "temporalities" or properties titled in their name and for
Filipino posterity; (2) to serve as an instrument of national defense, helping the benefit of the members of their respective religion composed of an
prevent the extension into the country of foreign control through peaceful overwhelming majority of Filipinos. No mention nor allusion whatsoever is
economic penetration; and (3) to prevent making the Philippines a source of made in the Constitution as to the prohibition against or the liability of the
international conflicts with the consequent danger to its internal security and Roman Catholic Church in the Philippines to acquire and hold agricultural
independence (See The Framing of the Philippine Constitution by Professor lands. Although there were some discussions on landholdings, they were
Jose M. Aruego, a Delegate to the Constitutional Convention, Vol. II. P. 592- mostly confined in the inclusion of the provision allowing the Government to
604). In the same book Delegate Aruego, explaining the reason behind the break big landed estates to put an end to absentee landlordism.
first consideration, wrote:
But let us suppose, for the sake of argument, that the above referred to
At the time of the framing of Philippine Constitution, Filipino capital inhibitory clause of Section 1 of Article XIII of the constitution does have
had been to be rather shy. Filipinos hesitated s a general rule to bearing on the petitioner's case; even so the clause requiring that at least 60
invest a considerable sum of their capital for the development, per centum of the capital of the corporation be owned by Filipinos is
exploitation and utilization of the natural resources of the country. subordinated to the petitioner's aforesaid right already existing at the time of
They had not as yet been so used to corporate as the peoples of the the inauguration of the Commonwealth and the Republic of the Philippines.
west. This general apathy, the delegates knew, would mean the In the language of Mr. Justice Jose P. Laurel (a delegate to the Constitutional
retardation of the development of the natural resources, unless Convention), in his concurring opinion of the case of Gold Creek mining
foreign capital would be encouraged to come and help in that Corporation, petitioner vs. Eulogio Rodriguez, Secretary of Agriculture and
development. They knew that the naturalization of the natural Commerce, and Quirico Abadilla, Director of the Bureau of Mines,
resources would certainly not encourage theINVESTMENT OF respondent, 66 Phil. 259:
FOREIGN CAPITAL into them. But there was a general feeling in the
Convention that it was better to have such a development retarded The saving clause in the section involved of the Constitution was
or even postpone together until such time when the Filipinos would originally embodied in the report submitted by the Committee on
be ready and willing to undertake it rather than permit the natural Naturalization and Preservation of Land and Other Natural
resources to be placed under the ownership or control of foreigners Resources to the Constitutional Convention on September 17, 1954.
in order that they might be immediately be developed, with the It was later inserted in the first draft of the Constitution as section 13
Filipinos of the future serving not as owners but utmost as tenants or of Article XIII thereof, and finally incorporated as we find it now. Slight
workers under foreign masters. By all means, the delegates have been the changes undergone by the proviso from the time
believed, the natural resources should be conserved for Filipino when it comes out of the committee until it was finally adopted. When
posterity. first submitted and as inserted to the first draft of the Constitution it
reads: 'subject to any right, grant, lease, or concession existing in
It could be distilled from the foregoing that the farmers of the Constitution respect thereto on the date of the adoption of the Constitution'. As
intended said provisions as barrier for foreigners or corporations financed by finally adopted, the proviso reads: 'subject to any existing right, grant,
such foreigners to acquire, exploit and develop our natural resources, saving lease, or concession at the time of the inauguration of the
these undeveloped wealth for our people to clear and enrich when they are Government established under this Constitution'. This recognition is

127
not mere graciousness but springs form the just character of the character of the government established. The farmers of the Constitution
government established. The framers of the Constitution were not were not obscured by the rhetoric of democracy or swayed to hostility by an
obscured by the rhetoric of democracy or swayed to hostility by an intense spirit of nationalism. They well knew that conservation of our natural
intense spirit of nationalism. They well knew that conservation of our resources did not mean destruction or annihilation of ACQUIRED
natural resources did not mean destruction or annihilation of PROPERTY RIGHTS".
acquired property rights. Withal, they erected a government neither
episodic nor stationary but well-nigh conservative in the protection of But respondents' counsel may argue that the preexisting right of acquisition
property rights. This notwithstanding nationalistic and socialistic traits of public or private lands by a corporation which does not fulfill this 60 per
discoverable upon even a sudden dip into a variety of the provisions cent requisite, refers to purchases of the Constitution and not to later
embodied in the instrument. transactions. This argument would imply that even assuming that petitioner
had at the time of the enactment of the Constitution the right to purchase real
The writer of this decision wishes to state at this juncture that during the property or right could not be exercised after the effectivity of our
deliberation of this case he submitted to the consideration of the Court the Constitution, because said power or right of corporations sole, like the herein
question that may be termed the "vested right saving clause" contained in petitioner, conferred in virtue of the aforequoted provisions of the Corporation
Section 1, Article XII of the Constitution, but some of the members of this Law, could no longer be exercised in view of the requisite therein prescribed
Court either did not agree with the theory of the writer, or were not ready to that at least 60 per centum of the capital of the corporation had to be Filipino.
take a definite stand on the particular point I am now to discuss deferring our It has been shown before that: (1) the corporation sole, unlike the ordinary
ruling on such debatable question for a better occasion, inasmuch as the corporations which are formed by no less than 5 incorporators, is composed
determination thereof is not absolutely necessary for the solution of the of only one persons, usually the head or bishop of the diocese, a unit which
problem involved in this case. In his desire to face the issues squarely, the is not subject to expansion for the purpose of determining any percentage
writer will endeavor, at least as a disgression, to explain and develop his whatsoever; (2) the corporation sole is only the administrator and not the
theory, not as a lucubration of the Court, but of his own, for he deems it owner of the temporalities located in the territory comprised by said
better and convenient to go over the cycle of reasons that are linked to one corporation sole; (3) such temporalities are administered for and on behalf of
another and that step by step lead Us to conclude as We do in the dispositive the faithful residing in the diocese or territory of the corporation sole; and (4)
part of this decision. the latter, as such, has no nationality and the citizenship of the incumbent
Ordinary has nothing to do with the operation, management or administration
It will be noticed that Section 1 of Article XIII of the Constitution provides, of the corporation sole, nor effects the citizenship of the faithful connected
among other things, that "all agricultural lands of the public domain and their with their respective dioceses or corporation sole.
disposition shall be limited to citizens of the Philippines or to corporations at
least 60 per centum of the capital of which is owned by such citizens, In view of these peculiarities of the corporation sole, it would seem obvious
SUBJECT TO ANY EXISTING RIGHT AT THE TIME OF THE that when the specific provision of the Constitution invoked by respondent
INAUGURATION OF THE GOVERNMENT ESTABLISHED UNDER THIS Commissioner (section 1, Art. XIII), was under consideration, the framers of
CONSTITUTION." the same did not have in mind or overlooked this particular form of
corporation. If this were so, as the facts and circumstances already indicated
As recounted by Mr. Justice Laurel in the aforementioned case of Gold Creek tend to prove it to be so, then the inescapable conclusion would be that this
Mining Corporation vs. Rodriguez et al., 66 Phil. 259, "this recognition (in the requirement of at least 60 per cent of Filipino capital was never intended to
clause already quoted), is not mere graciousness but springs from the just

128
apply to corporations sole, and the existence or not a vested right becomes The fact that the appellant religious organization has no capital stock
unquestionably immaterial. does not suffice to escape the Constitutional inhibition, since it is
admitted that its members are of foreign nationality. The purpose of
But let us assumed that the questioned proviso is material. yet We might say the sixty per centum requirement is obviously to ensure that
that a reading of said Section 1 will show that it does not refer to any actual corporation or associations allowed to acquire agricultural land or to
acquisition of land up to the right, qualification or power to acquire and hold exploit natural resources shall be controlled by Filipinos; and the
private real property. The population of the Philippines, Catholic to a high spirit of the Constitution demands that in the absence of capital
percentage, is ever increasing. In the practice of religion of their faithful the stock, the controlling membership should be composed of Filipino
corporation sole may be in need of more temples where to pray, more citizens.
schools where the children of the congregation could be taught in the
principles of their religion, more hospitals where their sick could be treated, In that case respondent-appellant Ung Siu Si Temple was not a corporation
more hallow or consecrated grounds or cemeteries where Catholics could be sole but a corporation aggregate, i.e., an unregistered organization operating
buried, many more than those actually existing at the time of the enactment through 3 trustees, all of Chinese nationality, and that is why this Court laid
of our Constitution. This being the case, could it be logically maintained that down the doctrine just quoted. With regard to petitioner, which likewise is a
because the corporation sole which, by express provision of law, has the non-stock corporation, the case is different, because it is a registered
power to hold and acquire real estate and personal property of its churches, corporation sole, evidently of no nationality and registered mainly to
charitable benevolent, or educational purposes (section 159, Corporation administer the temporalities and manage the properties belonging to the
Law) it has to stop its growth and restrain its necessities just because the faithful of said church residing in Davao. But even if we were to go over the
corporation sole is a non-stock corporation composed of only one person record to inquire into the composing membership to determine whether the
who in his unity does not admit of any percentage, especially when that citizenship requirement is satisfied or not, we would find undeniable proof
person is not the owner but merely an administrator of the temporalities of that the members of the Roman Catholic Apostolic faith within the territory of
the corporation sole? The writer leaves the answer to whoever may read and Davao are predominantly Filipino citizens. As indicated before, petitioner has
consider this portion of the decision. presented evidence to establish that the clergy and lay members of this
religion fully covers the percentage of Filipino citizens required by the
Anyway, as stated before, this question is not a decisive factor in disposing Constitution. These facts are not controverted by respondents and our
the case, for even if We were to disregard such saving clause of the conclusion in this point is sensibly obvious.
Constitution, which reads: subject to any existing right, grant, etc., at the
same time of the inauguration of the Government established under this Dissenting OpinionDiscussed. After having developed our theory in the
Constitution, yet We would have, under the evidence on record, sufficient case and arrived at the findings and conclusions already expressed in this
grounds to uphold petitioner's contention on this matter. decision. We now deem it proper to analyze and delve into the basic
foundation on which the dissenting opinion stands up. Being aware of the
In this case of the Register of Deeds of Rizal vs. Ung Sui Si Temple, 2 G.R. transcendental and far-reaching effects that Our ruling on the matter might
No. L-6776, promulgated May 21, 1955, wherein this question was have, this case was thoroughly considered from all points of view, the Court
considered from a different angle, this Court through Mr. Justice J.B.L. sparing no effort to solve the delicate problems involved herein.
Reyes, said:
At the deliberations had to attain this end, two ways were open to a prompt
dispatch of the case: (1) the reversal of the doctrine We laid down in the

129
celebrated Krivenko case by excluding urban lots and properties from the the temporalities of the Roman Catholic Church in the Philippines. In the third
group of the term "private agricultural lands" use in this section 5, Article XIII place, every corporation, be it aggregate or sole, is only entitled to purchase,
of the Constitution; and (2) by driving Our reasons to a point that might convey, sell, lease, let, mortgage, encumber and otherwise deal with real
indirectly cause the appointment of Filipino bishops or Ordinary to head the properties when it is pursuant to or in consonance with the purposes for
corporations sole created to administer the temporalities of the Roman which the corporation was formed, and when the transactions of the lawful
Catholic Church in the Philippines. With regard to the first way, a great business of the corporation reasonably and necessarily require such dealing
majority of the members of this Court were not yet prepared nor agreeable to section 13-(5) of the Corporation Law, Public Act No. 1459 and
follow that course, for reasons that are obvious. As to the second way, it considering these provisions in conjunction with Section 159 of the same law
seems to be misleading because the nationality of the head of a diocese which provides that a corporation sole may only "purchase and hold real
constituted as a corporation sole has no material bearing on the functions of estate and personal properties for its church, charitable, benevolent or
the latter, which are limited to the administration of the temporalities of the educational purposes", the above mentioned fear of revitalization of religious
Roman Catholic Apostolic Church in the Philippines. landholdings in the Philippines is absolutely dispelled. The fact that the law
thus expressly authorizes the corporations sole to receive bequests or gifts
Upon going over the grounds on which the dissenting opinion is based, it of real properties (which were the main source that the friars had to acquire
may be noticed that its author lingered on the outskirts of the issues, thus their big haciendas during the Spanish regime), is a clear indication that the
throwing the main points in controversy out of focus. Of course We fully requisite that bequests or gifts of real estate be for charitable, benevolent, or
agree, as stated by Professor Aruego, that the framers of our Constitution educational purposes, was, in the opinion of the legislators, considered
had at heart to insure the conservation of the natural resources of Our sufficient and adequate protection against the revitalization of religious
motherland of Filipino posterity; to serve them as an instrument of national landholdings.
defense, helping prevent the extension into the country of foreign
control through peaceful economic penetration; and to prevent making the Finally, and as previously stated, We have reason to believe that when the
Philippines a source of international conflicts with the consequent danger to Delegates to the Constitutional Convention drafted and approved Article XIII
its internal security and independence. But all these precautions adopted by of the Constitution they do not have in mind the corporation sole. We come to
the Delegates to Our Constitutional Assembly could have not been intended this finding because the Constitutional Assembly, composed as it was by a
for or directed against cases like the one at bar. The emphasis and great number of eminent lawyers and jurists, was like any other legislative
wonderings on the statement that once the capacity of a corporation sole to body empowered to enact either the Constitution of the country or any public
acquire private agricultural lands is admitted there will be no limit to the areas statute, presumed to know the conditions existing as to particular subject
that it may hold and that this will pave the way for the "revival or revitalization matter when it enacted a statute (Board of Commerce of Orange Country vs.
of religious landholdings that proved so troublesome in our past", cannot Bain, 92 S.E. 176; N. C. 377).
even furnish the "penumbra" of a threat to the future of the Filipino people. In
the first place, the right of Filipino citizens, including those of foreign Immemorial customs are presumed to have been always in the mind
extraction, and Philippine corporations, to acquire private lands is not subject of the Legislature in enacting legislation. (In re Kruger's Estate, 121
to any restriction or limit as to quantity or area, and We certainly do not see A. 109; 277 P. 326).
any wrong in that. The right of Filipino citizens and corporations to acquire
public agricultural lands is already limited by law. In the second place, The Legislative is presumed to have a knowledge of the state of the
corporations sole cannot be considered as aliens because they have no law on the subjects upon which it legislates. (Clover Valley Land and
nationality at all. Corporations sole are, under the law, mere administrators of Stock Co. vs. Lamb et al., 187, p. 723,726.)

130
The Court in construing a statute, will assume that the legislature who according to the Constitution must be a Filipino (sections 1 and
acted with full knowledge of the prior legislation on the subject and 5, Article XIII).
its construction by the courts. (Johns vs. Town of Sheridan, 89 N. E.
899, 44 Ind. App. 620.). 5. That section 159 of the Corporation Law expressly authorized the
corporation sole to purchase and holdreal estate for its church,
The Legislature is presumed to have been familiar with the subject charitable, benevolent or educational purposes, and to receive
with which it was dealing . . . . (Landers vs. Commonwealth, 101 S. bequests or giftsfor such purposes;
E. 778, 781.).
6. That in approving our Magna Carta the Delegates to the
The Legislature is presumed to know principles of statutory Constitutional Convention, almost all of whom were Roman
construction. (People vs. Lowell, 230 N. W. 202, 250 Mich. 349, Catholics, could not have intended to curtail the propagation of the
followed in P. vs. Woodworth, 230 N.W. 211, 250 Mich. 436.). Roman Catholic faith or the expansion of the activities of their
church, knowing pretty well that with the growth of our population
It is not to be presumed that a provision was inserted in a more places of worship, more schools where our youth could be
constitution or statute without reason, or that a result was intended taught and trained; more hallow grounds where to bury our dead
inconsistent with the judgment of men of common sense guided by would be needed in the course of time.
reason" (Mitchell vs. Lawden, 123 N.E. 566, 288 Ill. 326.) See City of
Decatur vs. German, 142 N. E. 252, 310 Ill. 591, and may other Long before the enactment of our Constitution the law authorized the
authorities that can be cited in support hereof. corporations sole even to receive bequests or gifts of real estates and this
Court could not, without any clear and specific provision of the Constitution,
Consequently, the Constitutional Assembly must have known: declare that any real property donated, let as say this year, could no longer
be registered in the name of the corporation sole to which it was conveyed.
1. That a corporation sole is organized by and composed of a single That would be an absurdity that should not receive our sanction on the
individual, the head of any religious society or church operating pretext that corporations sole which have no nationality and are non-stock
within the zone, area or jurisdiction covered by said corporation sole corporations composed of only one person in the capacity of administrator,
(Article 155, Public Act No. 1459); have to establish first that at least sixty per centum of their capital belong to
Filipino citizens. The new Civil Code even provides:
2. That a corporation sole is a non-stock corporation;
ART. 10. In case of doubt in the interpretation or application of
laws, it is presumed that the lawmaking body intended right and
3. That the Ordinary ( the corporation sole proper) does not own the
justice to prevail.
temporalities which he merely administers;

Moreover, under the laws of the Philippines, the administrator of the


4. That under the law the nationality of said Ordinary or of any
properties of a Filipino can acquire, in the name of the latter, private lands
administrator has absolutely no bearing on the nationality of the
without any limitation whatsoever, and that is so because the properties thus
person desiring to acquire real property in the Philippines by
acquired are not for and would not belong to the administrator but to the
purchase or other lawful means other than by hereditary succession,

131
Filipino whom he represents. But the dissenting Justice inquires: If the subjects himself to the jurisdiction of the Philippine courts of justice and these
Ordinary is only the administrator, for whom does he administer? And who tribunals can thus entertain grievances arising out of or with respect to the
can alter or overrule his acts? We will forthwith proceed to answer these temporalities of the church which came into the possession of the corporation
questions. The corporations sole by reason of their peculiar constitution and sole as administrator. It may be alleged that the courts cannot intervene as to
form of operation have no designed owner of its temporalities, although by the matters of doctrine or teachings of the Roman Catholic Church. That is
the terms of the law it can be safely implied that the Ordinary holds them in correct, but the courts may step in, at the instance of the faithful for whom the
trust for the benefit of the Roman Catholic faithful to their respective locality temporalities are being held in trust, to check undue exercise by the
or diocese. Borrowing the very words of the law, We may say that the corporation sole of its power as administrator to insure that they are used for
temporalities of every corporation sole are held in trust for the use, purpose, the purpose or purposes for which the corporation sole was created.
behalf and benefit of the religious society, or order so incorporated or of the
church to which the diocese, synod, or district organization is an organized American authorities have these to say:
and constituent part (section 163 of the Corporation Law).
It has been held that the courts have jurisdiction over an action
In connection with the powers of the Ordinary over the temporalities of the brought by persons claiming to be members of a church, who allege
corporation sole, let us see now what is the meaning and scope of the word a wrongful and fraudulent diversion of the church property to uses
"control". According to the Merriam-Webster's New International Dictionary, foreign to the purposes of the church, since no ecclesiastical
2nd ed., p. 580, on of the acceptations of the word "control" is: question is involved and equity will protect from wrongful diversion of
the property (Hendryx vs. Peoples United Church, 42 Wash. 336, 4
4. To exercise restraining or directing influence over; to dominate; L.R.A. n.s. 1154).
regulate; hence, to hold from action; to curb; subject; also, Obs. to
overpower. The courts of the State have no general jurisdiction and control over
the officers of such corporations in respect to the performance of
SYN: restrain, rule, govern, guide, direct; check, subdue. their official duties; but as in respect to the property which they hold
for the corporation, they stand in position of TRUSTEES and the
It is true that under section 159 of the Corporation Law, the intervention of courts may exercise the same supervision as in other cases of
the courts is not necessary, to mortgage or sell real property held by the trust (Ramsey vs. Hicks, 174 Ind. 428, 91 N.E. 344, 92 N.E. 164, 30
corporation sole where the rules, regulations and discipline of the religious L.R.A. n.s. 665; Hendryx vs. Peoples United Church, supra.).
denomination, society or church concerned presented by such corporation
sole regulates the methods of acquiring, holding, selling and mortgaging real Courts of the state do not interfere with the administration of church
estate, and that the Roman Catholic faithful residing in the jurisdiction of the rules or discipline unless civil rights become involved and which must
corporation sole has no say either in the manner of acquiring or of selling real be protected (Morris St., Baptist Church vs. Dart, 67 S.C. 338, 45
property. It may be also admitted that the faithful of the diocese cannot S.E. 753, and others). (All cited in Vol. II, Cooley's Constitutional
govern or overrule the acts of the Ordinary, but all this does not mean that Limitations, p. 960-964.).
the latter can administer the temporalities of the corporation sole without
check or restraint. We must not forget that when a corporation sole is If the Constitutional Assembly was aware of all the facts above enumerated
incorporated under Philippine laws, the head and only member thereof and of the provisions of law relative to existing conditions as to management

132
and operation of corporations sole in the Philippines, and if, on the other formulation. The courts have had to define those policies and have
hand, almost all of the Delegates thereto embraced the Roman Catholic faith, often drawn on natural law and natural rights theories in doing so.
can it be imagined even for an instant that when Article XIII of the The interpretation of constitutions tends to respond to changing
Constitution was approved the framers thereof intended to prevent or curtail conceptions of political and social values. The extent to which these
from then on the acquisition sole, either by purchase or donation, of real extraneous aids affect the judicial construction of constitutions
properties that they might need for the propagation of the faith and for there cannot be formulated in precise rules, but their influence cannot be
religious and Christian activities such as the moral education of the youth, ignored in describing the essentials of the process (Rottschaeffer on
the care, attention and treatment of the sick and the burial of the dead of the Constitutional Law, 1939 ed., p. 18-19).
Roman Catholic faithful residing in the jurisdiction of the respective
corporations sole? The mere indulgence in said thought would impress upon There are times that when even the literal expression of legislation
Us a feeling of apprehension and absurdity. And that is precisely the leit may be inconsistent with the general objectives of policy behind it,
motiv that permeates the whole fabric of the dissenting opinion. and on the basis of equity or spirit of the statute the courts rationalize
a restricted meaning of the latter. A restricted interpretation is usually
It seems from the foregoing that the main problem We are confronted with in applied where the effect of literal interpretation will make for injustice
this appeal, hinges around the necessity of a proper and adequate and absurdity or, in the words of one court, the language must be so
interpretation of sections 1 and 5 of Article XIII of the Constitution. Let Us unreasonable 'as to shock general common sense'. (Vol. 3,
then be guided by the principles of statutory construction laid down by the Sutherland on Statutory Construction, 3rd ed., 150.).
authorities on the matter:
A constitution is not intended to be a limitation on the development of
The most important single factor in determining the intention of the a country nor an obstruction to its progress and foreign relations
people from whom the constitution emanated is the language in (Moscow Fire Ins. Co. of Moscow, Russia vs. Bank of New York and
which it is expressed. The words employed are to be taken in their Trust Co., 294 N. Y. S.648; 56 N.E. 2d. 745, 293 N.Y. 749).
natural sense, except that legal or technical terms are to be given
their technical meaning. The imperfections of language as a vehicle Although the meaning or principles of a constitution remain fixed and
for conveying meanings result in ambiguities that must be resolved unchanged from the time of its adoption, a constitution must be
by result to extraneous aids for discovering the intent of the framers. construed as if intended to stand for a great length of time, and it is
Among the more important of these are a consideration of the history progressive and not static. Accordingly, it should not receive too
of the times when the provision was adopted and of the purposes narrow or literal an interpretation but rather the meaning given it
aimed at in its adoption. The debates of constitutional convention, should be applied in such manner as to meet new or changed
contemporaneous construction, and practical construction by the conditions as they arise (U.S. vs. Lassic, 313 U.S. 299, 85 L. Ed.,
legislative and executive departments, especially if long continued, 1368).
may be resorted to resolve, but not to create,
ambiguities. . . . Consideration of the consequences flowing from Effect should be given to the purpose indicated by a fair
alternative constructions of doubtful provisions constitutes an interpretation of the language used and that construction which
important interpretative device. . . . The purposes of many of the effectuates, rather than that which destroys a plain intent or purpose
broadly phrased constitutional limitations were the promotion of
policies that do not lend themselves to definite and specific

133
of a constitutional provision, is not only favored but will be adopted thing; it must have become 'fixed and established'" (Balboa vs. Farrales, 51
(State ex rel. Randolph Country vs. Walden, 206 S.W. 2d 979). Phil. 498). But the case at bar has to be considered as an exception to the
rule because among the rights granted by section 159 of the Corporation
It is quite generally held that in arriving at the intent and purpose the Law was the right to receive bequests or gifts of real properties for charitable,
construction should be broad or liberal or equitable, as the better benevolent and educational purposes. And this right to receive such
method of ascertaining that intent, rather than technical (Great bequests or gifts (which implies donations in futuro), is not a mere potentiality
Southern Life Ins. Co. vs. City of Austin, 243 S.W. 778). that could be impaired without any specific provision in the Constitution to
that effect, especially when the impairment would disturbingly affect the
All these authorities uphold our conviction that the framers of the Constitution propagation of the religious faith of the immense majority of the Filipino
had not in mind the corporations sole, nor intended to apply them the people and the curtailment of the activities of their Church. That is why the
provisions of section 1 and 5 of said Article XIII when they passed and writer gave us a basis of his contention what Professor Aruego said in his
approved the same. And if it were so as We think it is, herein petitioner, the book "The Framing of the Philippine Constitution" and the enlightening
Roman Catholic Apostolic Administrator of Davao, Inc., could not be deprived opinion of Mr. Justice Jose P. Laurel, another Delegate to the Constitutional
of the right to acquire by purchase or donation real properties for charitable, Convention, in his concurring opinion in the case of Goldcreek Mining Co. vs.
benevolent and educational purposes, nor of the right to register the same in Eulogio Rodriguez et al., 66 Phil. 259. Anyway the majority of the Court did
its name with the Register of Deeds of Davao, an indispensable requisite not deem necessary to pass upon said "vested right saving clause" for the
prescribed by the Land Registration Act for lands covered by the Torrens final determination of this case.
system.
JUDGMENT
We leave as the last theme for discussion the much debated question above
referred to as "the vested right saving clause" contained in section 1, Article Wherefore, the resolution of the respondent Land Registration Commission
XIII of the Constitution. The dissenting Justice hurls upon the personal of September 21, 1954, holding that in view of the provisions of sections 1
opinion expressed on the matter by the writer of the decision the most and 5 of Article XIII of the Philippine Constitution the vendee (petitioner) is
pointed darts of his severe criticism. We think, however, that this strong not qualified to acquire lands in the Philippines in the absence of proof that at
dissent should have been spared, because as clearly indicated before, some least 60 per centum of the capital, properties or assets of the Roman
members of this Court either did not agree with the theory of the writer or Catholic Apostolic Administrator of Davao, Inc. is actually owned or controlled
were not ready to take a definite stand on that particular point, so that there by Filipino citizens, and denying the registration of the deed of sale in the
being no majority opinion thereon there was no need of any dissension absence of proof of compliance with such requisite, is hereby reversed.
therefrom. But as the criticism has been made the writer deems it necessary Consequently, the respondent Register of Deeds of the City of Davao is
to say a few words of explanation. ordered to register the deed of sale executed by Mateo L. Rodis in favor of
the Roman Catholic Apostolic Administrator of Davao, Inc., which is the
The writer fully agrees with the dissenting Justice that ordinarily "a capacity subject of the present litigation. No pronouncement is made as to costs. It is
to acquire (property) in futuro, is not in itself a vested or existing property so ordered.
right that the Constitution protects from impairment. For a property right to be
vested (or acquired) there must be a transition from the potential or Bautista Angelo and Endencia, JJ., concur.
contingent to the actual, and the proprietary interest must have attached to a

134
Paras, C.J., and Bengzon, J., concur in the result.

135
136
G.R. No. L-55289 June 29, 1982 its predecessors-in-interest had possessed the land for more than thirty
years. It invoked section 48(b) of the Public Land Law, which provides:
REPUBLIC OF THE PHILIPPINES, represented by the Director of
Lands, petitioner-appellant, Chapter VIII.Judicial confirmation of imperfect or
vs. incomplete titles.
JUDGE CANDIDO P. VILLANUEVA, of the Court of First Instance of
Bulacan, Malolos Branch VII, and IGLESIA NI CRISTO, as a corporation xxx xxx xxx
sole, represented by ERAO G. MANALO, as Executive
Minister,respondents-appellees. SEC. 48. The following-described citizens of the Philippines,
occupying lands of the public domain or claiming to own any
such lands or an interest therein, but whose titles have not
been perfected or completed, may apply to the Court of First
AQUINO, J.: Instance of the province where the land is located for
confirmation of their claims and the issuance of a certificate
Like L-49623, Manila Electric Company vs. Judge Castro-Bartolome, this of title therefore, under the Land Register Act, to wit:
case involves the prohibition in section 11, Article XIV of the Constitution that
"no private corporation or association may hold alienable lands of the public xxx xxx xxx
domain except by lease not to exceed one thousand hectares in area".
(b) Those who by themselves or through their predecessors-
Lots Nos. 568 and 569, located at Barrio Dampol, Plaridel, Bulacan, with an in-interest have been in open, continuous, exclusive, and
area of 313 square meters and an assessed value of P1,350 were acquired notorious possession and occupation of agricultural lands of
by the Iglesia Ni Cristo on January 9, 1953 from Andres Perez in exchange the public domain, under a bona fide claim of acquisition of
for a lot with an area of 247 square meters owned by the said church (Exh. ownership, for at least thirty years immediately preceding the
D). filing of the application for confirmation of title except when
prevented by war or force majeure. These shall be
The said lots were already possessed by Perez in 1933. They are not conclusively presumed to have performed all the conditions
included in any military reservation. They are inside an area which was essential to a Government grant and shall be entitled to a
certified as alienable or disposable by the Bureau of Forestry in 1927. The certificate of title under the provisions of this chapter." (As
lots are planted to santol and mango trees and banana plants. A chapel amended by Republic Act No. 1942, approved on June 22,
exists on the said land. The land had been declared for realty tax purposes. 1957.)
Realty taxes had been paid therefor (Exh. N).
The Republic of the Philippines, through the Direct/r of Lands, opposed the
On September 13, 1977, the Iglesia Ni Cristo, a corporation sole, duly application on the grounds that applicant, as a private corporation, is
existing under Philippine laws, filed with the Court of First Instance of disqualified to hold alienable lands of the public domain, that the land applied
Bulacan an application for the registration of the two lots. It alleged that it and for is public land not susceptible of private appropriation and that the
applicant and its predecessors-in-interest have not been in the open,

137
continuous, exclusive and notorious possession of the land since June 12, since time immemorial, for such possession would justify the presumption
1945. that the land had never been part of the public domain or that it had been a
private property even before the Spanish conquest. "
After hearing, the trial court ordered the registration of the two lots, as
described in Plan Ap-04-001344 (Exh. E), in the name of the Iglesia Ni In Uy Un vs. Perez, 71 Phil. 508, it was noted that the right of an occupant of
Cristo, a corporation sole, represented by Executive Minister Erao G. public agricultural land to obtain a confirmation of his title under section 48(b)
Manalo, with office at the corner of Central and Don Mariano Marcos of the Public Land Law is a "derecho dominical incoativo"and that before the
Avenues, Quezon City, From that decision, the Republic of the Philippines issuance of the certificate of title the occupant is not in the juridical sense the
appealed to this Court under Republic Act No. 5440. The appeal should be true owner of the land since it still pertains to the State.
sustained.
The lower court's judgment is reversed and set aside. The application for
As correctly contended by the Solicitor General, the Iglesia Ni Cristo, as a registration of the Iglesia Ni Cristo is dismissed with costs against said
corporation sole or a juridical person, is disqualified to acquire or hold applicant.
alienable lands of the public domain, like the two lots in question, because of
the constitutional prohibition already mentioned and because the said church SO ORDERED.
is not entitled to avail itself of the benefits of section 48(b) which applies only
to Filipino citizens or natural persons. A corporation sole (an "unhappy freak Barredo, Makasiar, Guerrero, Melencio-Herrera, Escolin, Vasquez, Relova
of English law") has no nationality (Roman Catholic Apostolic Adm. of Davao, and Gutierrez, Jr., JJ., concur.
Inc. vs. Land Registration Commission, 102 Phil. 596. See Register of Deeds
vs. Ung Siu Si Temple, 97 Phil. 58 and sec. 49 of the Public Land Law).
Concepcion, Jr., J., is on leave.

The contention in the comments of the Iglesia Ni Cristo (its lawyer did not file
Plana, J., took no part.
any brief) that the two lots are private lands, following the rule laid down in
Susi vs. Razon and Director of Lands, 48 Phil. 424, is not correct. What was
considered private land in the Susi case was a parcel of land possessed by a
Filipino citizen since time immemorial, as in Cario vs. Insular
Government, 212 U.S. 449, 53 L. ed. 594, 41 Phil. 935 and 7 Phil. 132. The
lots sought to be registered in this case do not fall within that category. They
are still public lands. A land registration proceeding under section 48(b)
"presupposes that the land is public" (Mindanao vs. Director of Lands, L-
19535, July 10, 1967, 20 SCRA 641, 644).

As held in Oh Cho vs. Director of Lands, 75 Phil. 890, "all lands that were not
acquired from the Government, either by purchase or by grant, belong to the
public domain. An exception to the rule would be any land that should have
been in the possession of an occupant and of his predecessors-in-interest

138
139
Office of the Solicitor General for petitioner.
V. Jaime and L. E. Petilla for respondent.

PAREDES, J.:

This is a petition to review the decision of the Court of Tax Appeals, reversing
the decision of the Collector of Internal Revenue, assessing against and
demanding from the "Club Filipino, Inc. de Cebu", the sum of P12,068.84 as
fixed and percentage taxes, surcharge and compromise penalty, allegedly
due from it as a keeper of bar and restaurant.

As found by the Court of Tax Appeals, the "Club Filipino, Inc. de Cebu,"
(Club, for short), is a civic corporation organized under the laws of the
Philippines with an original authorized capital stock of P22,000.00, which was
subsequently increased to P200,000.00, among others, to it "proporcionar,
operar, y mantener un campo de golf, tenis, gimnesio (gymnasiums), juego
de bolos (bowling alleys), mesas de billar y pool, y toda clase de juegos no
prohibidos por leyes generales y ordenanzas generales; y desarollar y
cultivar deportes de toda clase y denominacion cualquiera para el recreo y
entrenamiento saludable de sus miembros y accionistas" (sec. 2, Escritura
de Incorporacion del Club Filipino, Inc. Exh. A). Neither in the articles or by-
laws is there a provision relative to dividends and their distribution, although
it is covenanted that upon its dissolution, the Club's remaining assets, after
paying debts, shall be donated to a charitable Philippine Institution in Cebu
(Art. 27, Estatutos del Club, Exh. A-a.).

The Club owns and operates a club house, a bowling alley, a golf course (on
a lot leased from the government), and a bar-restaurant where it sells wines
and liquors, soft drinks, meals and short orders to its members and their
guests. The bar-restaurant was a necessary incident to the operation of the
G.R. No. L-12719 May 31, 1962 club and its golf-course. The club is operated mainly with funds derived from
membership fees and dues. Whatever profits it had, were used to defray its
THE COLLECTOR OF INTERNAL REVENUE, petitioner, overhead expenses and to improve its golf-course. In 1951. as a result of a
vs. capital surplus, arising from the re-valuation of its real properties, the value or
THE CLUB FILIPINO, INC. DE CEBU, respondent. price of which increased, the Club declared stock dividends; but no actual
cash dividends were distributed to the stockholders. In 1952, a BIR agent

140
discovered that the Club has never paid percentage tax on the gross receipts 2. Whether it is liable for the payment of the sum of P500.00 as compromise
of its bar and restaurant, although it secured B-4, B-9(a) and B-7 licenses. In penalty.
a letter dated December 22, 1852, the Collector of Internal Revenue
assessed against and demanded from the Club, the following sums: Section 182, of the Tax Code states, "Unless otherwise provided, every
person engaging in a business on which the percentage tax is imposed shall
pay in full a fixed annual tax of ten pesos for each calendar year or fraction
thereof in which such person shall engage in said business." Section 183
As percentage tax on its gross receipts
provides in general that "the percentage taxes on business shall be payable
during the tax years 1946 to 1951 P9,599.07
at the end of each calendar quarter in the amount lawfully due on the
business transacted during each quarter; etc." And section 191, same Tax
Code, provides "Percentage tax . . . Keepers of restaurants, refreshment
parlors and other eating places shall pay a tax three per centum, and
Surcharge therein 2,399.77
keepers of bar and cafes where wines or liquors are served five per
centum of their gross receipts . . .". It has been held that the liability for fixed
and percentage taxes, as provided by these sections, does not ipso
As fixed tax for the years 1946 to 1952 70.00 factoattach by mere reason of the operation of a bar and restaurant. For the
liability to attach, the operator thereof must be engaged in the business as a
barkeeper and restaurateur. The plain and ordinary meaning of business is
restricted to activities or affairs where profit is the purpose or livelihood is the
Compromise penalty 500.00 motive, and the term business when used without qualification, should be
construed in its plain and ordinary meaning, restricted to activities for profitor
livelihood (The Coll. of Int. Rev. v. Manila Lodge No. 761 of the BPOE [Manila
Elks Club] & Court of Tax Appeals, G.R. No. L-11176, June 29, 1959, giving
The Club wrote the Collector, requesting for the cancellation of the full definitions of the word "business"; Coll. of Int. Rev. v. Sweeney, et al.
assessment. The request having been denied, the Club filed the instant [International Club of Iloilo, Inc.], G.R. No. L-12178, Aug. 21, 1959, the facts
petition for review. of which are similar to the ones at bar; Manila Polo Club v. B. L. Meer, etc.,
No. L-10854, Jan. 27, 1960).
The dominant issues involved in this case are twofold:
Having found as a fact that the Club was organized to develop and cultivate
1. Whether the respondent Club is liable for the payment of the sum of sports of all class and denomination, for the healthful recreation and
12,068.84, as fixed and percentage taxes and surcharges prescribed in entertainment of its stockholders and members; that upon its dissolution, its
sections 182, 183 and 191 of the Tax Code, under which the assessment remaining assets, after paying debts, shall be donated to a charitable
was made, in connection with the operation of its bar and restaurant, during Philippine Institution in Cebu; that it is operated mainly with funds derived
the periods mentioned above; and from membership fees and dues; that the Club's bar and restaurant catered
only to its members and their guests; that there was in fact no cash dividend
distribution to its stockholders and that whatever was derived on retail from
its bar and restaurant was used to defray its overall overhead expenses and

141
to improve its golf-course (cost-plus-expenses-basis), it stands to reason that A tax is a burden, and, as such, it should not be deemed imposed upon
the Club is not engaged in the business of an operator of bar and restaurant fraternal, civic, non-profit, nonstock organizations, unless the intent to the
(same authorities, cited above). contrary is manifest and patent" (Collector v. BPOE Elks Club, et al., supra),
which is not the case in the present appeal.
It is conceded that the Club derived profit from the operation of its bar and
restaurant, but such fact does not necessarily convert it into a profit-making Having arrived at the conclusion that respondent Club is not engaged in the
enterprise. The bar and restaurant are necessary adjuncts of the Club to business as an operator of a bar and restaurant, and therefore, not liable for
foster its purposes and the profits derived therefrom are necessarily fixed and percentage taxes, it follows that it is not liable for any penalty, much
incidental to the primary object of developing and cultivating sports for the less of a compromise penalty.
healthful recreation and entertainment of the stockholders and members.
That a Club makes some profit, does not make it a profit-making Club. As WHEREFORE, the decision appealed from is affirmed without costs.
has been remarked a club should always strive, whenever possible, to have
surplus (Jesus Sacred Heart College v. Collector of Int. Rev., G.R. No. L- Padilla, Bautista Angelo, Labrador, Concepcion, Reyes, J.B.L., Barrera and
6807, May 24, 1954; Collector of Int. Rev. v. Sinco Educational Corp., G.R. Dizon, JJ., concur.
No. L-9276, Oct. 23, 1956).1wph1.t Bengzon, C.J., is on leave.

It is claimed that unlike the two cases just cited (supra), which are non-stock,
the appellee Club is a stock corporation. This is unmeritorious. The facts that
the capital stock of the respondent Club is divided into shares, does not
detract from the finding of the trial court that it is not engaged in the business
of operator of bar and restaurant. What is determinative of whether or not the
Club is engaged in such business is its object or purpose, as stated in its
articles and by-laws. It is a familiar rule that the actual purpose is not
controlled by the corporate form or by the commercial aspect of the business
prosecuted, but may be shown by extrinsic evidence, including the by-laws
and the method of operation. From the extrinsic evidence adduced, the Tax
Court concluded that the Club is not engaged in the business as a barkeeper
and restaurateur.

Moreover, for a stock corporation to exist, two requisites must be complied


with, to wit: (1) a capital stock divided into shares and (2) an authority to
distribute to the holders of such shares, dividends or allotments of the
surplus profits on the basis of the shares held (sec. 3, Act No. 1459). In the
case at bar, nowhere in its articles of incorporation or by-laws could be found
an authority for the distribution of its dividends or surplus profits. Strictly
speaking, it cannot, therefore, be considered a stock corporation, within the
contemplation of the corporation law.

142
Virgilio E. Dulay for petitioners.

Torres, Tobias, Azura & Jocson for private respondents.

NOCON, J.:

This is a petition for review on certiorari to annul and set aside the
decision 1 of the Court of Appeals affirming the decision 2 of the Regional Trial
Court of Pasay, Branch 114 Civil Cases Nos. 8198-P, and 2880-P, the
dispositive portion of which reads, as follows:

Wherefore, in view of all the foregoing considerations, in this


Court hereby renders judgment, as follows:

In Civil Case No. 2880-P, the petition filed by Manuel R.


Dulay Enterprises, Inc. and Virgilio E. Dulay for annulment or
declaration of nullity of the decision of the Metropolitan Trial
Court, Branch 46, Pasay City, in its Civil Case No. 38-81
entitled "Edgardo D. Pabalan, et al., vs. Spouses Florentino
Manalastas, et al.," is dismissed for lack of merits;

In Civil Case No. 8278-P, the complaint filed by Manuel R.


Dulay Enterprises, Inc. for cancellation of title of Manuel A.
Torres, Jr. (TCT No. 24799 of the Register of Deeds of
Pasay City) and reconveyance, is dismissed for lack or
G.R. No. 91889 August 27, 1993 merit, and,

MANUEL R. DULAY ENTERPRISES, INC., VIRGILIO E. DULAY AND In Civil Case No. 8198-P, defendants Manuel R. Dulay
NEPOMUCENO REDOVAN, petitioners, Enterprises, Inc. and Virgilio E. Dulay are ordered to
vs. surrender and deliver possession of the parcel of land,
THE HONORABLE COURT OF APPEALS, EDGARDO D. PABALAN, together with all the improvements thereon, described in
MANUEL A. TORRES, JR., MARIA THERESA V. VELOSO AND Transfer Certificate of Title No. 24799 of the Register of
CASTRENSE C. VELOSO, respondents. Deeds of Pasay City, in favor of therein plaintiffs Manuel A.
Torres, Jr. as owner and Edgardo D. Pabalan as real estate

143
administrator of said Manuel A. Torres, Jr.; to account for and On December 23, 1976, Manuel Dulay by virtue of Board Resolution
return to said plaintiffs the rentals from dwelling unit No. 8-A No 18 6 of petitioner corporation sold the subject property to private
of the apartment building (Dulay Apartment) from June 1980 respondents spouses Maria Theresa and Castrense Veloso in the amount of
up to the present, to indemnify plaintiffs, jointly and severally, P300,000.00 as evidenced by the Deed of Absolute Sale. 7 Thereafter, TCT
expenses of litigation in the amount of P4,000.00 and No. 17880 was cancelled and TCT No. 23225 was issued to private
attorney's fees in the sum of P6,000.00, for all the three (3) respondent Maria Theresa Veloso. 8 Subsequently, Manuel Dulay and private
cases. Co-defendant Nepomuceno Redovan is ordered to respondents spouses Veloso executed a Memorandum to the Deed of
pay the current and subsequent rentals on the premises Absolute Sale of December 23, 1976 9 dated December 9, 1977 giving
leased by him to plaintiffs. Manuel Dulay within (2) years or until December 9, 1979 to repurchase the
subject property for P200,000.00 which was, however, not annotated either in
The counterclaim of defendants Virgilio E. Dulay and Manuel TCT No. 17880 or TCT No. 23225.
R. Dulay Enterprises, Inc. and N. Redovan, dismissed for
lack of merit. With costs against the three (3) aforenamed On December 24, 1976, private respondent Maria Veloso, without the
defendants. 3 knowledge of Manuel Dulay, mortgaged the subject property to private
respondent Manuel A. Torres for a loan of P250,000.00 which was duly
The facts as found by the trial court are as follows: annotated as Entry No. 68139 in TCT No. 23225. 10

Petitioner Manuel R. Dulay Enterprises, Inc, a domestic corporation with the Upon the failure of private respondent Maria Veloso to pay private
following as members of its Board of Directors: Manuel R. Dulay with 19,960 respondent Torres, the subject property was sold on April 5, 1978 to private
shares and designated as president, treasurer and general manager, Atty. respondent Torres as the highest bidder in an extrajudicial foreclosure sale
Virgilio E. Dulay with 10 shares and designated as vice-president; Linda E. as evidenced by the Certificate of Sheriff's Sale 11 issued on April 20, 1978.
Dulay with 10 shares; Celia Dulay-Mendoza with 10 shares; and Atty. Plaridel
C. Jose with 10 shares and designated as secretary, owned a property On July 20, 1978, private respondent Maria Veloso executed a Deed of
covered by TCT No. 17880 4 and known as Dulay Apartment consisting of Absolute Assignment of the Right to Redeem 12 in favor of Manuel Dulay
sixteen (16) apartment units on a six hundred eighty-nine (689) square assigning her right to repurchase the subject property from private
meters lot, more or less, located at Seventh Street (now Buendia Extension) respondent Torres as a result of the extra sale held on April 25, 1978.
and F.B. Harrison Street, Pasay City.
As neither private respondent Maria Veloso nor her assignee Manuel Dulay
Petitioner corporation through its president, Manuel Dulay, obtained various was able to redeem the subject property within the one year statutory period
loans for the construction of its hotel project, Dulay Continental Hotel (now for redemption, private respondent Torres filed an Affidavit of Consolidation of
Frederick Hotel). It even had to borrow money from petitioner Virgilio Dulay Ownership 13 with the Registry of Deeds of Pasay City and TCT No.
to be able to continue the hotel project. As a result of said loan, petitioner 24799 14 was subsequently issued to private respondent Manuel Torres on
Virgilio Dulay occupied one of the unit apartments of the subject property April 23, 1979.
since property since 1973 while at the same time managing the Dulay
Apartment at his shareholdings in the corporation was subsequently On October 1, 1979, private respondent Torres filed a petition for the
increased by his father. 5 issuance of a writ of possession against private respondents spouses Veloso

144
and Manuel Dulay in LRC Case No. 1742-P. However, when petitioner 3. Ordering the defendants to pay attorney's fees in the sum
Virgilio Dulay was never authorized by the petitioner corporation to sell or of P2,000.00 and P1,000.00 as other expenses of litigation
mortgage the subject property, the trial court ordered private respondent and for them to pay the costs of the suit. 15
Torres to implead petitioner corporation as an indispensable party but the
latter moved for the dismissal of his petition which was granted in an Order Thereafter or on May 17, 1985, petitioner corporation and Virgilio Dulay filed
dated April 8, 1980. an action against the presiding judge of the Metropolitan Trial Court of Pasay
City, private respondents Pabalan and Torres for the annulment of said
On June 20, 1980, private respondent Torres and Edgardo Pabalan, real decision with the Regional Trial Court of Pasay in Civil Case No. 2880-P.
estate administrator of Torres, filed an action against petitioner corporation,
Virgilio Dulay and Nepomuceno Redovan, a tenant of Dulay Apartment Unit Thereafter, the three (3) cases were jointly tried and the trial court rendered a
No. 8-A for the recovery of possession, sum of money and damages with decision in favor of private respondents.
preliminary injunction in Civil Case, No. 8198-P with the then Court of First
Instance of Rizal. Not satisfied with said decision, petitioners appealed to the Court of Appeals
which rendered a decision on October 23, 1989, the dispositive portion of
On July 21, 1980, petitioner corporation filed an action against private which reads, as follows:
respondents spouses Veloso and Torres for the cancellation of the Certificate
of Sheriff's Sale and TCT No. 24799 in Civil Case No. 8278-P with the then PREMISES CONSIDERED, the decision being appealed
Court of First Instance of Rizal. should be as it is hereby AFFIRMED in full. 16

On January 29, 1981, private respondents Pabalan and Torres filed an action On November 8, 1989, petitioners filed a Motion for Reconsideration which
against spouses Florentino and Elvira Manalastas, a tenant of Dulay was denied on January 26, 1990.
Apartment Unit No. 7-B, with petitioner corporation as intervenor for
ejectment in Civil Case No. 38-81 with the Metropolitan Trial Court of Pasay
Hence, this petition.
City which rendered a decision on April 25, 1985, dispositive portion of which
reads, as follows:
During the pendency of this petition, private respondent Torres died on April
3, 1991 as shown in his death certificate 17 and named Torres-Pabalan Realty
Wherefore, judgment is hereby rendered in favor of the
& Development Corporation as his heir in his holographic will 18 dated
plaintiff (herein private respondents) and against the
October 31, 1986.
defendants:

Petitioners contend that the respondent court had acted with grave abuse of
1. Ordering the defendants and all persons claiming
discretion when it applied the doctrine of piercing the veil of corporate entity
possession under them to vacate the premises.
in the instant case considering that the sale of the subject property between
private respondents spouses Veloso and Manuel Dulay has no binding effect
2. Ordering the defendants to pay the rents in the sum of on petitioner corporation as Board Resolution No. 18 which authorized the
P500.000 a month from May, 1979 until they shall have sale of the subject property was resolved without the approval of all the
vacated the premises with interest at the legal rate;

145
members of the board of directors and said Board Resolution was prepared proper call or notice in a close corporation is deemed ratified by the absent
by a person not designated by the corporation to be its secretary. director unless the latter promptly files his written objection with the secretary
of the corporation after having knowledge of the meeting which, in his case,
We do not agree. petitioner Virgilio Dulay failed to do.

Section 101 of the Corporation Code of the Philippines provides: It is relevant to note that although a corporation is an entity which has a
personality distinct and separate from its individual stockholders or
Sec. 101. When board meeting is unnecessary or improperly members, 19 the veil of corporate fiction may be pierced when it is used to
held. Unless the by-laws provide otherwise, any action by defeat public convenience justify wrong, protect fraud or defend crime. 20 The
the directors of a close corporation without a meeting shall privilege of being treated as an entity distinct and separate from its
nevertheless be deemed valid if: stockholder or members is therefore confined to its legitimate uses and is
subject to certain limitations to prevent the commission of fraud or other
illegal or unfair act. When the corporation is used merely as an alter ego or
1. Before or after such action is taken, written consent
business conduit of a person, the law will regard the corporation as the act of
thereto is signed by all the directors, or
that person. 21 The Supreme Court had repeatedly disregarded the separate
personality of the corporation where the corporate entity was used to annul a
2. All the stockholders have actual or implied knowledge of valid contract executed by one of its members.
the action and make no prompt objection thereto in writing;
or
Petitioners' claim that the sale of the subject property by its president,
Manuel Dulay, to private respondents spouses Veloso is null and void as the
3. The directors are accustomed to take informal action with alleged Board Resolution No. 18 was passed without the knowledge and
the express or implied acquiese of all the stockholders, or consent of the other members of the board of directors cannot be sustained.
As correctly pointed out by the respondent Court of Appeals:
4. All the directors have express or implied knowledge of the
action in question and none of them makes prompt objection Appellant Virgilio E. Dulay's protestations of complete
thereto in writing. innocence to the effect that he never participated nor was
even aware of any meeting or resolution authorizing the
If a directors' meeting is held without call or notice, an action mortgage or sale of the subject premises (see par. 8,
taken therein within the corporate powers is deemed ratified affidavit of Virgilio E. Dulay, dated May 31, 1984, p. 14, Exh.
by a director who failed to attend, unless he promptly files "21") is difficult to believe. On the contrary, he is very much
his written objection with the secretary of the corporation privy to the transactions involved. To begin with, he is a
after having knowledge thereof. incorporator and one of the board of directors designated at
the time of the organization of Manuel R. Dulay Enterprise,
In the instant case, petitioner corporation is classified as a close corporation Inc. In ordinary parlance, the said entity is loosely referred to
and consequently a board resolution authorizing the sale or mortgage of the as a "family corporation". The nomenclature, if imprecise,
subject property is not necessary to bind the corporation for the action of its however, fairly reflects the cohesiveness of a group and the
president. At any rate, corporate action taken at a board meeting without parochial instincts of the individual members of such an

146
aggrupation of which Manuel R. Dulay Enterprises, Inc. is Moreover, the appellate courts will not disturb the findings of the trial judge
typical: four-fifths of its incorporators being close relatives unless he has plainly overlooked certain facts of substance and value that, if
namely, three (3) children and their father whose name considered, might affect the result of the case, 25 which is not present in the
identifies their corporation (Articles of Incorporation of instant case.
Manuel R. Dulay Enterprises, Inc. Exh. "31-A"). 22
Petitioners' contention that private respondent Torres never acquired
Besides, the fact that petitioner Virgilio Dulay on June 24, 1975 executed an ownership over the subject property since the latter was never in actual
affidavit 23 that he was a signatory witness to the execution of the post-dated possession of the subject property nor was the property ever delivered to him
Deed of Absolute Sale of the subject property in favor of private respondent is also without merit.
Torres indicates that he was aware of the transaction executed between his
father and private respondents and had, therefore, adequate knowledge Paragraph 1, Article 1498 of the New Civil Code provides:
about the sale of the subject property to private respondents.
When the sale is made through a public instrument, the
Consequently, petitioner corporation is liable for the act of Manuel Dulay and execution thereof shall be equivalent to the delivery of the
the sale of the subject property to private respondents by Manuel Dulay is thing which is the object of the contract, if from the deed the
valid and binding. As stated by the trial court: contrary do not appear or cannot clearly be inferred.

. . . the sale between Manuel R. Dulay Enterprises, Inc. and Under the aforementioned article, the mere execution of the deed of sale in a
the spouses Maria Theresa V. Veloso and Castrense C. public document is equivalent to the delivery of the property. Likewise, this
Veloso, was a corporate act of the former and not a personal Court had held that:
transaction of Manuel R. Dulay. This is so because Manuel
R. Dulay was not only president and treasurer but also the It is settled that the buyer in a foreclosure sale becomes the
general manager of the corporation. The corporation was a absolute owner of the property purchased if it is not
closed family corporation and the only non-relative in the redeemed during the period of one year after the registration
board of directors was Atty. Plaridel C. Jose who appeared of the sale. As such, he is entitled to the possession of the
on paper as the secretary. There is no denying the fact, said property and can demand it at any time following the
however, that Maria Socorro R. Dulay at times acted as consolidation of ownership in his name and the issuance to
secretary. . . ., the Court can not lose sight of the fact that the him of a new transfer certificate of title. The buyer can in fact
Manuel R. Dulay Enterprises, Inc. is a closed family demand possession of the land even during the redemption
corporation where the incorporators and directors belong to period except that he has to post a bond in accordance with
one single family. It cannot be concealed that Manuel R. Section 7 of Act No. 3133 as amended. No such bond is
Dulay as president, treasurer and general manager almost required after the redemption period if the property is not
had absolute control over the business and affairs of the redeemed. Possession of the land then becomes an
corporation. 24 absolute right of the purchaser as confirmed owner. 26

147
Therefore, prior physical delivery or possession is not legally required since
the execution of the Deed of Sale in deemed equivalent to delivery.

Finally, we hold that the respondent appellate court did not err in denying
petitioner's motion for reconsideration despite the fact that private
respondents failed to submit their comment to said motion as required by the
respondent appellate court from resolving petitioners' motion for
reconsideration without the comment of the private respondent which was
required merely to aid the court in the disposition of the motion. The courts
are as much interested as the parties in the early disposition of cases before
them. To require otherwise would unnecessarily clog the courts' dockets.

WHEREFORE, the petition is DENIED and the decision appealed from is


hereby AFFIRMED.

SO ORDERED.

Narvasa, C.J., Padilla and Regalado, JJ., concur.

Puno, J., took no part.

148
defendant corporation by J. Amado Araneta, and asking that the corporation
be dissolved; that J. Amado Araneta be declared personally accountable for
the amounts of the unauthorized and fraudulent disbursements and
disposition of assets made by him, and that he be required to account for
said assets, and that pending trial and disposition of the case on its merits a
receiver be appointed to take possession of the books, records and assets of
the defendant corporation preparatory to its dissolution and liquidation and
distribution of the assets. Over the strong objection of the defendants, the
trial court presided by respondent Judge Jose Teodoro, granted the petition
for the appointment of a receiver and designated Mr. Alfredo Yulo as such
receiver with a bond of P50,000. Failing to secure a reconsideration of the
order appointing a receiver, the defendants in said case, Financing
Corporation of the Philippines and J. Amado Araneta, as petitioners, have
filed the present petition for certiorari with preliminary injunction to revoke
and set aside the order. Acting upon that part of the petition asking for a writ
G.R. No. L-4900 August 31, 1953 of preliminary injunction, a majority of the court granted the same upon the
filing of a bond by the petitioners in the sum of P50,000.
FINANCING CORPORATION OF THE PHILIPPINES and J. AMADO
ARANETA, petitioners, The main contention of the petitioners in opposing the appointment of a
vs. receiver in this case is that said appointment is merely an auxiliary remedy;
HON. JOSE TEODORO, Judge of the Court of First Instance of Negros that the principal remedy sought by the respondents in the action in Negros
Occidental, Branch II, and ENCARNACION LIZARES VDA. DE Occidental was the dissolution of the Financing Corporation of the
PANLILIO, respondents. Philippines; that according to the law a suit for the dissolution of a
corporation can be brought and maintained only by the State through its legal
Vicente Hilado for petitioners. counsel, and that respondents, much less the minority stockholders of said
Antonio Barredo for respondents. corporation, have no right or personality to maintain the action for dissolution,
and that inasmuch as said action cannot be maintained legally by the
respondents, then the auxiliary remedy for the appointment of a receiver has
MONTEMAYOR, J.:
no basis.

In civil case No. 1924 of the Court of First Instance of Negros Occidental,
True it is that the general rule is that the minority stockholders of a
Asuncion Lopez Vda. de Lizares, Encarnacion Lizares Vda. de Panlilio and
corporation cannot sue and demand its dissolution. However, there are cases
Efigenia Vda. de Paredes, in their own behalf and in behalf of the other
that hold that even minority stockholders may ask for dissolution, this, under
minority stockholders of the Financing Corporation of the Philippines, filed a
the theory that such minority members, if unable to obtain redress and
complaint against the said corporation and J. Amado Araneta, its president
protection of their rights within the corporation, must not and should not be
and general manager, claiming among other things alleged gross
left without redress and remedy. This was what probably prompted this Court
mismanagement and fraudulent conduct of the corporate affairs of the
to state in the case of Hall, et al. vs. Judge Piccio,* G.R. No. L-2598 (47 Off.

149
Gaz. No. 12 Supp., p. 200) that even the existence of a de jure corporation Considering plaintiffs' complaint and verified motion for appointment
may be terminated in a private suit for its dissolution by the stockholders of a receiver together, as they have been treated jointly in the
without the intervention of the State. It was therein further held that although opposition of the defendants, the grounds of the prayer for
there might be some room for argument on the right of minority stockholders receivership may be briefly stated to be: (1) imminent danger of
to ask for dissolution,-that question does not affect the court's jurisdiction insolvency; (2) fraud and mismanagement, such as, particularly, (a)
over the case, and that the remedy by the party dissatisfied was to appeal wrongful and unauthorized diversion from corporate purposes and
from the decision of the trial court. We repeat that although as a rule, minority use for personal benefit of defendant Araneta, for the benefit of the
stockholders of a corporation may not ask for its dissolution in a private suit, corporations under his control and of which he is majority
and that such action should be brought by the Government through its legal stockholder and/or for the benefit of his relatives, personal friends
officer in a quo warranto case, at their instance and request, there might be and the political organization to which he is affiliated of approximately
exceptional cases wherein the intervention of the State, for one reason or over one and a half million pesos of the funds of the defendant
another, cannot be obtained, as when the State is not interested because the corporation in the form of uncollected allowances and loans, either
complaint is strictly a matter between the stockholders and does not involve, without or with uncollected interest, and either unsecured or
in the opinion of the legal officer of the Government, any of the acts or insufficiently secured, and sometimes with a securities appearing in
omissions warranting quo warranto proceedings, in which minority favor of defendant Araneta as if the funds advanced or loaned were
stockholders are entitled to have such dissolution. When such action or his own; (b) unauthorized and profitless pledging of securities owned
private suit is brought by them, the trial court had jurisdiction and may or may by defendant corporation to secure obligations amounting to
not grant the prayer, depending upon the facts and circumstances attending P588,645.34 of another corporation controlled by defendant Araneta;
it. The trial court's decision is of course subject to review by the appellate (c) unauthorized and profitless using of the name of the defendant
tribunal. Having such jurisdiction, the appointment of a receiver pendente corporation in the shipping of sugar belonging to other corporations
lite is left to the sound discretion of the trial court. As was said in the case controlled by defendant Araneta to the benefit of said corporations in
of Angeles vs. Santos (64 Phil., 697), the action having been properly the amount of at least P104,343.36; (d) refusal by defendant Araneta
brought and the trial court having entertained the same, it was within the to endorse to the defendant corporation shares of stock and other
power of said court upon proper showing to appoint a receiverpendente securities belonging to it but which are still in his name; (e) negligent
lite for the corporation; that although the appointment of a receiver upon failure to endorse other shares of stock belonging to defendant
application of the minority stockholders is a power to be exercised with great corporation but still in the names of the respective vendors; and (f)
caution, nevertheless, it should be exercised necessary in order not to illegal and unauthorized transfer and deposit in the United States of
entirely ignore and disregard the rights of said minority stockholders, America of 6,426,281 shares of the Atok-Big Wedge Mining
especially when said minority stockholders are unable to obtain redress and Company; (3) violations of the corporation law and the by-laws of the
protection of their rights within the corporation itself. corporation such as (a) refusal to allow minority stockholders to
examine the books and records of the corporation; (b) failure to call
In that civil case No. 1924 of Negros Occidental court, allegations of and hold stockholders' and directors' meetings; (c) virtual disregard
mismanagement and misconduct by its President and Manager were made, and ignoring of the board of directors by defendant Araneta who has
specially in connection with the petition for the appointment of a receiver. in been and is conducting the affairs of the corporation under his
order to have an idea of the seriousness of said allegations, we reproduce a absolute control and for his personal benefit and for the benefit of the
pertinent portion of the order of respondent Judge Teodoro dated June 23, corporations controlled by him, to the prejudice and in disregard of
1951, subject of these certiorari proceedings: the rights of the plaintiffs and other minority stockholders; and (d)

150
irregularity in the keeping and (e) errors and omissions in the books
and failure of the same to reflect the real and actual transactions of
the defendant corporations; (4) failure to achieve the fundamental
purpose of the corporation; (5) if administration, possession and
control of the affairs, books, etc. of defendant corporation are left in
the hands of the defendant Araneta and the present corporate
officials, under his power and influence, the remaining assets of the
corporation are in danger of being further dissipated, wasted or lost
and of becoming ultimately unavailable for distribution among its
stockholders; and (6) the best means to protect and preserve the
assets of defendant corporation is the appointment of a receiver.

In conclusion, we hold that the trial court through respondent Judge Teodoro
had jurisdiction and properly entertained the original case; that he also had
jurisdiction to appoint a receiver pendente lite, and considering the
allegations made in connection with the petition for the appointment of a
receiver, he neither exceeded his jurisdiction nor abused his discretion in
appointing a receiver. The petition for certiorari is hereby denied, with costs.
The writ of preliminary injunction heretofore issued is hereby ordered
dissolved.

Paras, C.J., Pablo, Bengzon, Padilla, Tuason, Reyes, Jugo, Bautista Angelo, G.R. No. L-43350 December 23, 1937
and Labrador, JJ., concur.
CAGAYAN FISHING DEVELOPMENT CO., INC., plaintiff-appellant,
vs.
TEODORO SANDIKO, defendant-appellee.

Arsenio P. Dizon for appellant.


Sumulong, Lavides and Sumulong for appellee.

LAUREL, J.:

This is an appeal from a judgment of the Court of First Instance of Manila


absolving the defendant from the plaintiff's complaint.

151
Manuel Tabora is the registered owner of four parcels of land situated in the The defendant having failed to pay the sum stated in the promissory note,
barrio of Linao, town of Aparri, Province of Cagayan, as evidenced by plaintiff, on January 25, 1934, brought this action in the Court of First
transfer certificate of title No. 217 of the land records of Cagayan, a copy of Instance of Manila praying that judgment be rendered against the defendant
which is in evidence as Exhibit 1. To guarantee the payment of a loan in the for the sum of P25,300, with interest at legal rate from the date of the filing of
sum of P8,000, Manuel Tabora, on August 14, 1929, executed in favor of the the complaint, and the costs of the suits. After trial, the court below, on
Philippine National Bank a first mortgage on the four parcels of land above- December 18, 1934, rendered judgment absolving the defendant, with costs
mentioned. A second mortgage in favor of the same bank was in April of against the plaintiff. Plaintiff presented a motion for new trial on January 14,
1930 executed by Tabora over the same lands to guarantee the payment of 1935, which motion was denied by the trial court on January 19 of the same
another loan amounting to P7,000. A third mortgage on the same lands was year. After due exception and notice, plaintiff has appealed to this court and
executed on April 16, 1930 in favor of Severina Buzon to whom Tabora was makes an assignment of various errors.
indebted in the sum of P2,9000. These mortgages were registered and
annotations thereof appear at the back of transfer certificate of title No. 217. In dismissing the complaint against the defendant, the court below, reached
the conclusion that Exhibit B is invalid because of vice in consent and
On May 31, 1930, Tabora executed a public document entitled "Escritura de repugnancy to law. While we do not agree with this conclusion, we have
Transpaso de Propiedad Inmueble" (Exhibit A) by virtue of which the four however voted to affirm the judgment appealed from the reasons which we
parcels of land owned by him was sold to the plaintiff company, said to under shall presently state.
process of incorporation, in consideration of one peso (P1) subject to the
mortgages in favor of the Philippine National Bank and Severina Buzon and, The transfer made by Tabora to the Cagayan fishing Development Co., Inc.,
to the condition that the certificate of title to said lands shall not be plaintiff herein, was affected on May 31, 1930 (Exhibit A) and the actual
transferred to the name of the plaintiff company until the latter has fully and incorporation of said company was affected later on October 22, 1930
completely paid Tabora's indebtedness to the Philippine National Bank. (Exhibit 2). In other words, the transfer was made almost five months before
the incorporation of the company. Unquestionably, a duly organized
The plaintiff company filed its article incorporation with the Bureau of corporation has the power to purchase and hold such real property as the
Commerce and Industry on October 22, 1930 (Exhibit 2). A year later, on purposes for which such corporation was formed may permit and for this
October 28, 1931, the board of directors of said company adopted a purpose may enter into such contracts as may be necessary (sec. 13, pars. 5
resolution (Exhibit G) authorizing its president, Jose Ventura, to sell the four and 9, and sec. 14, Act No. 1459). But before a corporation may be said to
parcels of lands in question to Teodoro Sandiko for P42,000. Exhibits B, C be lawfully organized, many things have to be done. Among other things, the
and D were thereafter made and executed. Exhibit B is a deed of sale law requires the filing of articles of incorporation (secs. 6 et seq., Act. No.
executed before a notary public by the terms of which the plaintiff sold ceded 1459). Although there is a presumption that all the requirements of law have
and transferred to the defendant all its right, titles, and interest in and to the been complied with (sec. 334, par. 31 Code of Civil Procedure), in the case
four parcels of land described in transfer certificate in turn obligated himself before us it can not be denied that the plaintiff was not yet incorporated when
to shoulder the three mortgages hereinbefore referred to. Exhibit C is a it entered into a contract of sale, Exhibit A. The contract itself referred to the
promisory note for P25,300. drawn by the defendant in favor of the plaintiff, plaintiff as "una sociedad en vias de incorporacion." It was not even a de
payable after one year from the date thereof. Exhibit D is a deed of mortgage facto corporation at the time. Not being in legal existence then, it did not
executed before a notary public in accordance with which the four parcels of possess juridical capacity to enter into the contract.
land were given a security for the payment of the promissory note, Exhibit C.
All these three instrument were dated February 15, 1932.

152
Corporations are creatures of the law, and can only come into 78 N. E., 128; Holyoke Envelope Co., vs. U. S. Envelope Co., 182 Mass.,
existence in the manner prescribed by law. As has already been 171; 65 N. E., 54.) It should be observed that Manuel Tabora was the
stated, general law authorizing the formation of corporations are registered owner of the four parcels of land, which he succeeded in
general offers to any persons who may bring themselves within their mortgaging to the Philippine National Bank so that he might have the
provisions; and if conditions precedent are prescribed in the statute, necessary funds with which to convert and develop them into fishery. He
or certain acts are required to be done, they are terms of the offer, appeared to have met with financial reverses. He formed a corporation
and must be complied with substantially before legal corporate composed of himself, his wife, and a few others. From the articles of
existence can be acquired. (14 C. J., sec. 111, p. 118.) incorporation, Exhibit 2, it appears that out of the P48,700, amount of capital
stock subscribed, P45,000 was subscribed by Manuel Tabora himself and
That a corporation should have a full and complete organization and P500 by his wife, Rufina Q. de Tabora; and out of the P43,300, amount paid
existence as an entity before it can enter into any kind of a contract on subscription, P42,100 is made to appear as paid by Tabora and P200 by
or transact any business, would seem to be self evident. . . . A his wife. Both Tabora and His wife were directors and the latter was treasurer
corporation, until organized, has no being, franchises or faculties. as well. In fact, to this day, the lands remain inscribed in Tabora's name. The
Nor do those engaged in bringing it into being have any power to defendant always regarded Tabora as the owner of the lands. He dealt with
bind it by contract, unless so authorized by the charter there is not a Tabora directly. Jose Ventura, president of the plaintiff corporation, intervened
corporation nor does it possess franchise or faculties for it or others only to sign the contract, Exhibit B, in behalf of the plaintiff. Even the
to exercise, until it acquires a complete existence. (Gent vs. Philippine National Bank, mortgagee of the four parcels of land, always
Manufacturers and Merchant's Mutual Insurance Company, 107 Ill., treated Tabora as the owner of the same. (SeeExhibits E and F.) Two civil
652, 658.) suits (Nos. 1931 and 38641) were brought against Tabora in the Court of
First Instance of Manila and in both cases a writ of attachment against the
Boiled down to its naked reality, the contract here (Exhibit A) was entered four parcels of land was issued. The Philippine National Bank threatened to
into not between Manuel Tabora and a non-existent corporation but between foreclose its mortgages. Tabora approached the defendant Sandiko and
the Manuel Tabora as owner of the four parcels of lands on the one hand and succeeded in the making him sign Exhibits B, C, and D and in making him,
the same Manuel Tabora, his wife and others, as mere promoters of a among other things, assume the payment of Tabora's indebtedness to the
corporations on the other hand. For reasons that are self-evident, these Philippine National Bank. The promisory note, Exhibit C, was made payable
promoters could not have acted as agent for a projected corporation since to the plaintiff company so that it may not attached by Tabora's creditors, two
that which no legal existence could have no agent. A corporation, until of whom had obtained writs of attachment against the four parcels of land.
organized, has no life and therefore no faculties. It is, as it were, a child in
ventre sa mere. This is not saying that under no circumstances may the acts If the plaintiff corporation could not and did not acquire the four parcels of
of promoters of a corporation be ratified by the corporation if and when land here involved, it follows that it did not possess any resultant right to
subsequently organized. There are, of course, exceptions (Fletcher Cyc. of dispose of them by sale to the defendant, Teodoro Sandiko.
Corps., permanent edition, 1931, vol. I, secs. 207 et seq.), but under the
peculiar facts and circumstances of the present case we decline to extend Some of the members of this court are also of the opinion that the transfer
the doctrine of ratification which would result in the commission of injustice or from Manuel Tabora to the Cagayan Fishing Development Company, Inc.,
fraud to the candid and unwary.(Massachusetts rule, Abbott vs. Hapgood, which transfer is evidenced by Exhibit A, was subject to a condition
150 Mass., 248; 22 N. E. 907, 908; 5 L. R. A., 586; 15 Am. St. Rep., 193; precedent (condicion suspensiva), namely, the payment of the mortgage debt
citing English cases; Koppel vs. Massachusetts Brick Co., 192 Mass., 223; of said Tabora to the Philippine National Bank, and that this condition not

153
having been complied with by the Cagayan Fishing Development Company, RIZAL LIGHT & ICE CO., INC., petitioner,
Inc., the transfer was ineffective. (Art. 1114, Civil Code; Wise & Co. vs. Kelly vs.
and Lim, 37 Phil., 696; Manresa, vol. 8, p. 141.) However, having arrived at THE PUBLIC SERVICE COMMISSION and MORONG ELECTRIC CO.,
the conclusion that the transfer by Manuel Tabora to the Cagayan Fishing INC., respondents.
Development Company, Inc. was null because at the time it was affected the
corporation was non-existent, we deem it unnecessary to discuss this Amado A. Amador, Jr. for petitioner.
point.lawphil.net Atilano C. Bautista and Pompeyo F. Olivas for respondents.

The decision of the lower court is accordingly affirmed, with costs against the
appellant. So Ordered.
ZALDIVAR, J.:
Villa-Real, Abad Santos, Imperial, Diaz and Concepcion, JJ., concur.
These two cases, being interrelated, are decided together.

Case G.R. No. L-20993 is a petition of the Rizal Light & Ice Co., Inc. to
review and set aside the orders of respondent Public Service
Commission, 1 dated August 20, 1962, and February 15, 1963, in PSC Case
No. 39716, cancelling and revoking the certificate of public convenience and
necessity and forfeiting the franchise of said petitioner. In the same petition,
the petitioner prayed for the issuance of a writ of preliminary injunction ex
parte suspending the effectivity of said orders and/or enjoining respondents
Commission and/or Municipality of Morong, Rizal, from enforcing in any way
the cancellation and revocation of petitioner's franchise and certificate of
public convenience during the pendency of this appeal. By resolution of
March 12, 1963, this Court denied the petition for injunction, for lack of merit.

G.R. No. L-20993 September 28, 1968


Case G. R. L-21221 is likewise a petition of the Rizal Light & Ice Co., Inc. to
review and set aside the decision of the Commission dated March 13, 1963
RIZAL LIGHT & ICE CO., INC., petitioner, in PSC Case No. 62-5143 granting a certificate of public convenience and
vs. necessity to respondent Morong Electric Co., Inc. 2 to operate an electric light,
THE MUNICIPALITY OF MORONG, RIZAL and THE PUBLIC SERVICE heat and power service in the municipality of Morong, Rizal. In the petition
COMMISSION, respondents. Rizal Light & Ice Co., Inc. also prayed for the issuance of a writ of preliminary
injunction ex parte suspending the effectivity of said decision. Per resolution
---------------------------- of this Court, dated May 6, 1963, said petition for injunction was denied.

G.R. No. L-21221 September 28, 1968 The facts, as they appear in the records of both cases, are as follows:

154
Petitioner Rizal Light & Ice Co., Inc. is a domestic corporation with business of public convenience and to forfeit its franchise on the ground, among other
address at Morong, Rizal. On August 15, 1949, it was granted by the things, that it failed to comply with the conditions of said certificate and
Commission a certificate of public convenience and necessity for the franchise. Said petition was set for hearing jointly with the order to show
installation, operation and maintenance of an electric light, heat and power cause. The hearings had been postponed several times.
service in the municipality of Morong, Rizal.
Meanwhile, inspections had been made of petitioner's electric plant and
In an order dated December 19, 1956, the Commission required the installations by the engineers of the Commission, as follows: April 15, 1958
petitioner to appear before it on February 18, 1957 to show cause why it by Engineer Antonio M. Alli; September 18, 1959, July 12-13, 1960, and June
should not be penalized for violation of the conditions of its certificate of 21-24, 1961, by Engineer Meliton S. Martinez. The inspection on June 21-24,
public convenience and the regulations of the Commission, and for failure to 1961 was made upon the request of the petitioner who manifested during the
comply with the directives to raise its service voltage and maintain them hearing on December 15, 1960 that improvements have been made on its
within the limits prescribed in the Revised Order No. 1 of the Commission, service since the inspection on July 12-13, 1960, and that, on the basis of the
and to acquire and install a kilowattmeter to indcate the load in kilowatts at inspection report to be submitted, it would agree to the submission of the
any particular time of the generating unit. 3 case for decision without further hearing.

For failure of the petitioner to appear at the hearing on February 18, 1957, When the case was called for hearing on July 5, 1961, petitioner failed to
the Commission ordered the cancellation and revocation of petitioner's appear. Respondent municipality was then allowed to present its
certificate of public convenience and necessity and the forfeiture of its documentary evidence, and thereafter the case was submitted for decision.
franchise. Petitioner moved for reconsideration of said order on the ground
that its manager, Juan D. Francisco, was not aware of said hearing. On July 7, 1961, petitioner filed a motion to reopen the case upon the ground
Respondent municipality opposed the motion alleging that petitioner has not that it had not been furnished with a copy of the report of the June 21-24,
rendered efficient and satisfactory service and has not complied with the 1961 inspection for it to reply as previously agreed. In an order dated August
requirements of the Commission for the improvement of its service. The 25, 1961, petitioner was granted a period of ten (10) days within which to
motion was set for hearing and Mr. Pedro S. Talavera, Chief, Industrial submit its written reply to said inspection report, on condition that should it
Division of the Commission, was authorized to conduct the hearing for the fail to do so within the said period the case would be considered submitted
reception of the evidence of the parties. 4 for decision. Petitioner failed to file the reply. In consonance with the order of
August 25, 1961, therefore, the Commission proceeded to decide the case.
Finding that the failure of the petitioner to appear at the hearing set for On July 29, 1962 petitioner's electric plant was burned.
February 18, 1957 the sole basis of the revocation of petitioner's
certificate was really due to the illness of its manager, Juan D. Francisco, In its decision, dated August 20, 1962, the Commission, on the basis of the
the Commission set aside its order of revocation. Respondent municipality inspection reports of its aforenamed engineers, found that the petitioner had
moved for reconsideration of this order of reinstatement of the certificate, but failed to comply with the directives contained in its letters dated May 21,
the motion was denied. 1954 and September 4, 1954, and had violated the conditions of its
certificate of public convenience as well as the rules and regulations of the
In a petition dated June 25, 1958, filed in the same case, respondent Commission. The Commission concluded that the petitioner "cannot render
municipality formally asked the Commission to revoke petitioner's certificate the efficient, adequate and satisfactory electric service required by its

155
certificate and that it is against public interest to allow it to continue its there was an absence of electric service in the municipality of Morong and
operation." Accordingly, it ordered the cancellation and revocation of that applicant Morong Electric, a Filipino-owned corporation duly organized
petitioner's certificate of public convenience and the forfeiture of its franchise. and existing under the laws of the Philippines, has the financial capacity to
maintain said service. These circumstances, considered together with the
On September 18, 1962, petitioner moved for reconsideration of the decision, denial of the motion for reconsideration filed by petitioner in Case No. 39715
alleging that before its electric plant was burned on July 29, 1962, its service on February, 15, 1963, such that as far as the Commission was concerned
was greatly improved and that it had still existing investment which the the certificate of the petitioner was already declared revoked and cancelled,
Commission should protect. But eight days before said motion for the Commission approved the application of Morong Electric and ordered the
reconsideration was filed, or on September 10, 1962, Morong Electric, having issuance in its favor of the corresponding certificate of public convenience
been granted a municipal franchise on May 6, 1962 by respondent and necessity.1awphl.nt
municipality to install, operate and maintain an electric heat, light and power
service in said municipality approved by the Provincial Board of Rizal on On March 8, 1963, petitioner filed with this Court a petition to review the
August 31, 1962 filed with the Commission an application for a certificate decision in Case No. 39715 (now G. R. No. L-20993). Then on April 26,
of public convenience and necessity for said service. Said application was 1963, petitioner also filed a petition to review the decision in Case No. 62-
entitled "Morong Electric Co., Inc., Applicant", and docketed as Case No. 62- 5143 (now G. R. No. L-21221).
5143.
In questioning the decision of the Commission in Case No. 39715, petitioner
Petitioner opposed in writing the application of Morong Electric, alleging contends: (1) that the Commission acted without or in excess of its
among other things, that it is a holder of a certificate of public convenience to jurisdiction when it delegated the hearing of the case and the reception of
operate an electric light, heat and power service in the same municipality of evidence to Mr. Pedro S. Talavera who is not allowed by law to hear the
Morong, Rizal, and that the approval of said application would not promote same; (2) that the cancellation of petitioner's certificate of public convenience
public convenience, but would only cause ruinous and wasteful competition. was unwarranted because no sufficient evidence was adduced against the
Although the opposition is dated October 6, 1962, it was actually received by petitioner and that petitioner was not able to present evidence in its defense;
the Commission on November 8, 1962, or twenty four days after the order of (3) that the Commission failed to give protection to petitioner's investment;
general default was issued in open court when the application was first called and (4) that the Commission erred in imposing the extreme penalty of
for hearing on October 15, 1962. On November 12, 1962, however, the revocation of the certificate.
petitioner filed a motion to lift said order of default. But before said motion
could be resolved, petitioner filed another motion, dated January 4, 1963, this In questioning the decision in Case No. 62-5143, petitioner contends: (1) that
time asking for the dismissal of the application upon the ground that applicant the Commission erred in denying petitioner's motion to dismiss and
Morong Electric had no legal personality when it filed its application on proceeding with the hearing of the application of the Morong Electric; (2) that
September 10, 1962, because its certificate of incorporation was issued by the Commission erred in granting Morong Electric a certificate of public
the Securities and Exchange Commission only on October 17, 1962. This convenience and necessity since it is not financially capable to render the
motion to dismiss was denied by the Commission in a formal order issued on service; (3) that the Commission erred when it made findings of facts that are
January 17, 1963 on the premise that applicant Morong Electric was a de not supported by the evidence adduced by the parties at the trial; and (4) that
facto corporation. Consequently, the case was heard on the merits and both the Commission erred when it did not give to petitioner protection to its
parties presented their respective evidence. On the basis of the evidence investment a reiteration of the third assignment of error in the other
adduced, the Commission, in its decision dated March 13, 1963, found that case.1awphl.nt

156
We shall now discuss the appeals in these two cases separately. without or in excess of jurisdiction in so authorizing Mr. Talavera should be
dismissed. 9
G.R. No. L-20993
2. Anent the second assigned error, the gist of petitioner's contention is that
1. Under the first assignment of error, petitioner contends that while Mr. the evidence consisting of inspection reports upon which the
Pedro S. Talavera, who conducted the hearings of the case below, is a Commission based its decision is insufficient and untrustworthy in that (1) the
division chief, he is not a lawyer. As such, under Section 32 of authors of said reports had not been put to test by way of cross-examination;
Commonwealth Act No. 146, as amended, the Commission should not have (2) the reports constitute only one side of the picture as petitioner was not
delegated to him the authority to conduct the hearings for the reception of able to present evidence in its defense; (3) judicial notice was not taken of
evidence of the parties. the testimony of Mr. Harry B. Bernardino, former mayor of respondent
municipality, in PSC Case No. 625143 (the other case, G. R. No. L-21221) to
We find that, really, Mr. Talavera is not a lawyer. 5 Under the second the effect that the petitioner had improved its service before its electric power
paragraph of Section 32 of Commonwealth Act No. 146, as amended, 6 the plant was burned on July 29, 1962 which testimony contradicts the
Commission can only authorize a division chief to hear and investigate a inspection reports; and (4) the Commission acted both as prosecutor and
case filed before it if he is a lawyer. However, the petitioner is raising this judge passing judgment over the very same evidence presented by it as
question for the first time in this appeal. The record discloses that petitioner prosecutor a situation "not conducive to the arrival at just and equitable
never made any objection to the authority of Mr. Talavera to hear the case decisions."
and to receive the evidence of the parties. On the contrary, we find that
petitioner had appeared and submitted evidence at the hearings conducted Settled is the rule that in reviewing the decision of the Public Service
by Mr. Talavera, particularly the hearings relative to the motion for Commission this Court is not required to examine the proof de novo and
reconsideration of the order of February 18, 1957 cancelling and revoking its determine for itself whether or not the preponderance of evidence really
certificate. We also find that, through counsel, petitioner had entered into justifies the decision. The only function of this Court is to determine whether
agreements with Mr. Talavera, as hearing officer, and the counsel for or not there is evidence before the Commission upon which its decision
respondent municipality, regarding procedure in order to abbreviate the might reasonably be based. This Court will not substitute its discretion for
proceedings. 7 It is only after the decision in the case turned out to be adverse that of the Commission on questions of fact and will not interfere in the
to it that petitioner questioned the proceedings held before Mr. Talavera. latter's decision unless it clearly appears that there is no evidence to support
it. 10 Inasmuch as the only function of this Court in reviewing the decision of
This Court in several cases has ruled that objection to the delegation of the Commission is to determine whether there is sufficient evidence before
authority to hear a case filed before the Commission and to receive the the Commission upon which its decision can reasonably be based, as it is
evidence in connection therewith is a procedural, not a jurisdictional point, not required to examine the proof de novo, the evidence that should be made
and is waived by failure to interpose timely the objection and the case had the basis of this Court's determination should be only those presented in this
been decided by the Commission. 8 Since petitioner has never raised any case before the Commission. What then was the evidence presented before
objection to the authority of Mr. Talavera before the Commission, it should be the Commission and made the basis of its decision subject of the present
deemed to have waived such procedural defect, and consonant with the appeal? As stated earlier, the Commission based its decision on the
precedents on the matter, petitioner's claim that the Commission acted inspection reports submitted by its engineers who conducted the inspection
of petitioner's electric service upon orders of the Commission. 11 Said
inspection reports specify in detail the deficiencies incurred, and violations

157
committed, by the petitioner resulting in the inadequacy of its service. We by us for the reason that he is a technical man and he knows well as he has
consider that said reports are sufficient to serve reasonably as bases of the done a good job and I think our proposition would expedite the matter. We
decision in question. It should be emphasized, in this connection that said sincerely believe that the inspection report will be the best evidence to decide
reports, are not mere documentary proofs presented for the consideration of this matter.
the Commission, but are the results of the Commission's own observations
and investigations which it can rightfully take into consideration, 12 particularly xxx xxx xxx
in this case where the petitioner had not presented any evidence in its
defense, and speaking of petitioner's failure to present evidence, as well as ATTY. LUQUE:
its failure to cross-examine the authors of the inspection reports, petitioner
should not complain because it had waived not only its right to cross-
... This is a very important matter and to show the good faith of respondent in
examine but also its right to present evidence. Quoted hereunder are the
this case we will not even cross-examine the engineer when he makes a new
pertinent portions of the transcripts of the proceedings where the petitioner,
report. We will agree to the findings and, your honor please, considering as
through counsel, manifested in clear language said waiver and its decision to
we have manifested before that Engineer Martinez is an experienced
abide by the last inspection report of Engineer Martinez:
engineer of this Commission and the points reported by Engineer Martinez
on the situation of the plant now will prevent the necessity of having a
Proceedings of December 15, 1960 hearing, of us bringing new evidence and complainant bringing new
evidence. ... .
COMMISSION:
xxx xxx xxx
It appears at the last hearing of this case on September 23, 1960, that an
engineer of this Commission has been ordered to make an inspection of all COMMISSION (to Atty. Luque):
electric services in the province of Rizal and on that date the engineer of this
Commission is still undertaking that inspection and it appears that the said
Q Does the Commission understand from the counsel for
engineer had actually made that inspection on July 12 and 13, 1960. The
applicant that if the motion is granted he will submit this order to
engineer has submitted his report on November 18, 1960 which is attached
show cause for decision without any further hearing and the decision
to the records of this case.
will be based on the report of the engineer of this Commission?

ATTY. LUQUE (Councel for Petitioner):


A We respectfully reply in this manner that we be allowed or
be given an opportunity just to read the report and 99%, we will
... (W)e respectfully state that while the report is, as I see it attached to the agree that the report will be the basis of that decision. We just want
records, clear and very thorough, it was made sometime July of this year and to find out the contents of the report, however, we request that we be
I understand from the respondent that there is some improvement since this furnished with a copy of the report before the hearing so that we will
report was made ... we respectfully request that an up-to-date inspection be just make a manifestation that we will agree.
made ... . An inspector of this Commission can be sent to the plant and
considering that the engineer of this Commission, Engineer Meliton Martinez,
COMMISSION (to Atty. Luque):
is very acquainted to the points involved we pray that his report will be used

158
Q In order to prevent the delay of the disposition of this case COMMISSION:
the Commission will allow counsel for the applicant to submit his
written reply to the report that the engineer of this Commission. Will To give applicant a chance to have a day in court the Commission grants the
he submit this case without further hearing upon the receipt of that request of applicant that it be given 10 days within which to submit a written
written reply? reply on the report of the engineer of the Commission who inspected the
electric service, in the municipality of Morong, Rizal, and after the submission
A Yes, your honor. of the said written reply within 10 days from today this case will be
considered submitted for decision.
Proceedings of August 25, 1961
The above-quoted manifestation of counsel for the petitioner, specifically the
ATTY. LUQUE (Counsel for petitioner): statement referring to the inspection report of Engineer Martinez as the "best
evidence to decide this matter," can serve as an argument against
In order to avoid any delay in the consideration of this case we are petitioner's claim that the Commision should have taken into consideration
respectfully move (sic) that instead of our witnesses testifying under oath that the testimony of Mr. Bernardino. But the primary reasons why the
we will submit a written reply under oath together with the memorandum Commission could not have taken judicial cognizance of said testimony are:
within fifteen (15) days and we will furnish a copy and upon our submission first, it is not a proper subject of judicial notice, as it is not a "known" fact
of said written reply under oath and memorandum we consider this case that is, well established and authoritatively settled, without qualification and
submitted. This suggestion is to abbreviate the necessity of presenting contention; 13 second, it was given in a subsequent and distinct case after the
witnesses here which may prolong the resolution of this case. petitioner's motion for reconsideration was heard by the Commission en
banc and submitted for decision, 14 and third, it was not brought to the
attention of the Commission in this case through an appropriate pleading. 15
ATTY. OLIVAS (Counsel for respondent municipality):

Regarding the contention of petitioner that the Commission had acted both
I object on the ground that there is no resolution by this Commission on the
as prosecutor and judge, it should be considered that there are two matters
action to reopen the case and second this case has been closed.
that had to be decided in this case, namely, the order to show cause dated
December 19, 1956, and the petition or complaint by respondent municipality
ATTY. LUQUE: dated June 25, 1958. Both matters were heard jointly, and the record shows
that respondent municipality had been allowed to present its evidence to
With regard to the testimony on the ground for opposition we respectfully substantiate its complaint. It can not be said, therefore, that in this case the
submit to this Commission our motion to submit a written reply together with Commission had acted as prosecutor and judge. But even assuming, for the
a memorandum. Also as stated to expedite the case and to avoid further sake of argument, that there was a commingling of the prosecuting and
hearing we will just submit our written reply. According to our records we are investigating functions, this exercise of dual function is authorized by Section
furnished with a copy of the report of July 17, 1961. We submit your honor. 17(a) of Commonwealth Act No. 146, as amended, under which the
Commission has power "to investigate, upon its own initiative or upon
xxx xxx xxx complaint in writing, any matter concerning any public service as regards
matters under its jurisdiction; to, require any public service to furnish safe,
adequate, and proper service as the public interest may require and warrant;

159
to enforce compliance with any standard, rule, regulation, order or other to render adequate, sufficient and satisfactory service and had violated the
requirement of this Act or of the Commission ... ." Thus, in the case important conditions of its certificate as well as the directives and the rules
of Collector of Internal Revenue vs. Estate of F. P. Buan, L-11438, July 31, and regulations of the Commission, the rule cannot apply. To apply that rule
1958, this Court held that the power of the Commission to cancel and revoke unqualifiedly is to encourage violation or disregard of the terms and
a certificate of public convenience and necessity may be exercised by it even conditions of the certificate and the Commission's directives and regulations,
without a formal charge filed by any interested party, with the only limitation and would close the door to other applicants who could establish, operate
that the holder of the certificate should be given his day in court. and provide adequate, efficient and satisfactory service for the benefit and
convenience of the inhabitants. It should be emphasized that the paramount
It may not be amiss to add that when prosecuting and investigating duties consideration should always be the public interest and public convenience.
are delegated by statute to an administrative body, as in the case of the The duty of the Commission to protect investment of a public utility operator
Public Service Commission, said body may take steps it believes appropriate refers only to operators of good standing those who comply with the laws,
for the proper exercise of said duties, particularly in the manner of informing rules and regulations and not to operators who are unconcerned with the
itself whether there is probable violation of the law and/or its rules and public interest and whose investments have failed or deteriorated because of
regulations. It may initiate an investigation, file a complaint, and then try the their own fault. 18
charge as preferred. So long as the respondent is given a day in court, there
can be no denial of due process, and objections to said procedure cannot be 4. The last assignment of error assails the propriety of the penalty imposed
sustained. by the Commission on the petitioner that is, the revocation of the
certificate and the forfeiture of the franchise. Petitioner contends that the
3. In its third assignment of error, petitioner invokes the "protection-of- imposition of a fine would have been sufficient, as had been done by the
investment rule" enunciated by this Court in Batangas Transportation Co. vs. Commission in cases of a similar nature.
Orlanes 16 in this wise:
It should be observed that Section 16(n) of Commonwealth Act No. 146, as
The Government having taken over the control and supervision of all amended, confers upon the Commission ample power and discretion to order
public utilities, so long as an operator under a prior license complies the cancellation and revocation of any certificate of public convenience
with the terms and conditions of his license and reasonable rules and issued to an operator who has violated, or has willfully and contumaciously
regulations for its operation and meets the reasonable demands of refused to comply with, any order, rule or regulation of the Commission or
the public, it is the duty of the Commission to protect rather than to any provision of law. What matters is that there is evidence to support the
destroy his investment by the granting of the second license to action of the Commission. In the instant case, as shown by the evidence, the
another person for the same thing over the same route of travel. The contumacious refusal of the petitioner since 1954 to comply with the
granting of such a license does not serve its convenience or promote directives, rules and regulations of the Commission, its violation of the
the interests of the public. conditions of its certificate and its incapability to comply with its commitment
as shown by its inadequate service, were the circumstances that warranted
The above-quoted rule, however, is not absolute, for nobody has exclusive the action of the Commission in not merely imposing a fine but in revoking
right to secure a franchise or a certificate of public convenience. 17 Where, as altogether petitioner's certificate. To allow petitioner to continue its operation
in the present case, it has been shown by ample evidence that the petitioner, would be to sacrifice public interest and convenience in favor of private
despite ample time and opportunity given to it by the Commission, had failed interest.

160
A grant of a certificate of public convenience confers no property Coming now to the other case, let it be stated at the outset that before any
rights but is a mere license or privilege, and such privilege is forfeited certificate may be granted, authorizing the operation of a public service, three
when the grantee fails to comply with his commitments behind which requisites must be complied with, namely: (1) the applicant must be a citizen
lies the paramount interest of the public, for public necessity cannot of the Philippines or of the United States, or a corporation or co-partnership,
be made to wait, nor sacrificed for private convenience. (Collector of association or joint-stock company constituted and organized under the laws
Internal Revenue v. Estate of F. P. Buan, et al., L-11438 and of the Philippines, sixty per centum at least of the stock or paid-up capital of
Santiago Sambrano, et al. v. PSC, et al., L-11439 & L-11542-46, July which belongs entirely to citizens of the Philippines or of the United
31, 1958) States; 19 (2) the applicant must be financially capable of undertaking the
proposed service and meeting the responsibilities incident to its
(T)he Public Service Commission, ... has the power to specify and operation; 20 and (3) the applicant must prove that the operation of the public
define the terms and conditions upon which the public utility shall be service proposed and the authorization to do business will promote the public
operated, and to make reasonable rules and regulations for its interest in a proper and suitable manner. 21
operation and the compensation which the utility shall receive for its
services to the public, and for any failure to comply with such rules As stated earlier, in the decision appealed from, the Commission found that
and regulations or the violation of any of the terms and conditions for Morong Electric is a corporation duly organized and existing under the laws
which the license was granted, the Commission has ample power to of the Philippines, the stockholders of which are Filipino citizens, that it is
enforce the provisions of the license or even to revoke it, for any financially capable of operating an electric light, heat and power service, and
failure or neglect to comply with any of its terms and provisions. that at the time the decision was rendered there was absence of electric
(Batangas Trans. Co. v. Orlanes, 52 Phil. 455, 460; emphasis service in Morong, Rizal. While the petitioner does not dispute the need of an
supplied) electric service in Morong, Rizal, 22 it claims, in effect, that Morong Electric
should not have been granted the certificate of public convenience and
Presumably, the petitioner has in mind Section 21 of Commonwealth Act No. necessity because (1) it did not have a corporate personality at the time it
146, as amended, which provides that a public utility operator violating or was granted a franchise and when it applied for said certificate; (2) it is not
failing to comply with the terms and conditions of any certificate, or any financially capable of undertaking an electric service, and (3) petitioner was
orders, decisions or regulations of the Commission, shall be subject to a fine rendering efficient service before its electric plant was burned, and therefore,
and that the Commission is authorized and empowered to impose such fine, being a prior operator its investment should be protected and no new party
after due notice and hearing. It should be noted, however, that the last should be granted a franchise and certificate of public convenience and
sentence of said section states that the remedy provided therein "shall not be necessity to operate an electric service in the same locality.
a bar to, or affect any other remedy provided in this Act but shall be
cumulative and additional to such remedy or remedies." In other words, the 1. The bulk of petitioner's arguments assailing the personality of Morong
imposition of a fine may only be one of the remedies which the Commission Electric dwells on the proposition that since a franchise is a contract, 23 at
may resort to, in its discretion. But that remedy is not exclusive of, or has least two competent parties are necessary to the execution thereof, and
preference over, the other remedies. And this Court will not substitute its parties are not competent except when they are in being. Hence, it is
discretion for that of the Commission, as long as there is evidence to support contended that until a corporation has come into being, in this jurisdiction, by
the exercise of that discretion by the Commission. the issuance of a certificate of incorporation by the Securities and Exchange
Commission (SEC) it cannot enter into any contract as a corporation. The
G. R. No. L-21221 certificate of incorporation of the Morong Electric was issued by the SEC on

161
October 17, 1962, so only from that date, not before, did it acquire juridical While a franchise cannot take effect until the grantee corporation is
personality and legal existence. Petitioner concludes that the franchise organized, the franchise may, nevertheless, be applied for before the
granted to Morong Electric on May 6, 1962 when it was not yet in esse is null company is fully organized.
and void and cannot be the subject of the Commission's consideration. On
the other hand, Morong Electric argues, and to which argument the A grant of a street franchise is valid although the corporation is not
Commission agrees, that it was a de factocorporation at the time the created until afterwards. (Fletcher, Cyclopedia Corp. Permanent
franchise was granted and, as such, it was not incapacitated to enter into any Edition, Rev. Vol. 6-A, Sec. 2881)
contract or to apply for and accept a franchise. Not having been
incapacitated, Morong Electric maintains that the franchise granted to it is And Thompson gives the reason for the rule:
valid and the approval or disapproval thereof can be properly determined by
the Commission.
(I)n the matter of the secondary franchise the authorities are
numerous in support of the proposition that an ordinance granting a
Petitioner's contention that Morong Electric did not yet have a legal privilege to a corporation is not void because the beneficiary of the
personality on May 6, 1962 when a municipal franchise was granted to it is ordinance is not fully organized at the time of the introduction of the
correct. The juridical personality and legal existence of Morong Electric ordinance. It is enough that organization is complete prior to the
began only on October 17, 1962 when its certificate of incorporation was passage and acceptance of the ordinance. The reason is that a
issued by the SEC. 24 Before that date, or pending the issuance of said privilege of this character is a mere license to the corporation until it
certificate of incorporation, the incorporators cannot be considered as de accepts the grant and complies with its terms and conditions.
facto corporation. 25 But the fact that Morong Electric had no corporate (Thompson on Corporations, Vol. 4, 3rd Ed., Sec. 2929) 26
existence on the day the franchise was granted in its name does not render
the franchise invalid, because later Morong Electric obtained its certificate of
The incorporation of Morong Electric on October 17, 1962 and its acceptance
incorporation and then accepted the franchise in accordance with the terms
of the franchise as shown by its action in prosecuting the application filed
and conditions thereof. This view is sustained by eminent American
with the Commission for the approval of said franchise, not only perfected a
authorities. Thus, McQuiuin says:
contract between the respondent municipality and Morong Electric but also
cured the deficiency pointed out by the petitioner in the application of Morong
The fact that a company is not completely incorporated at the time EIectric. Thus, the Commission did not err in denying petitioner's motion to
the grant is made to it by a municipality to use the streets does not, dismiss said application and in proceeding to hear the same. The efficacy of
in most jurisdictions, affect the validity of the grant. But such grant the franchise, however, arose only upon its approval by the Commission on
cannot take effect until the corporation is organized. And in Illinois it March 13, 1963. The reason is that
has been decided that the ordinance granting the franchise may be
presented before the corporation grantee is fully organized, where
Under Act No. 667, as amended by Act No. 1022, a municipal council
the organization is completed before the passage and acceptance.
has the power to grant electric franchises, subject to the approval of
(McQuillin, Municipal Corporations, 3rd Ed., Vol. 12, Chap. 34, Sec.
the provincial board and the President. However, under Section
34.21)
16(b) of Commonwealth Act No. 146, as amended, the Public
Service Commission is empowered "to approve, subject to
Fletcher says: constitutional limitations any franchise or privilege granted under the

162
provisions of Act No. 667, as amended by Act No. 1022, by any of cases, this Court has said it will not disturb unless patently unsupported by
political subdivision of the Philippines when, in the judgment of the evidence. An examination of the record of this case readily shows that the
Commission, such franchise or privilege will properly conserve the testimony of Mr. Ingal and the documents he presented to establish the
public interests and the Commission shall in so approving impose financial capability of Morong Electric provide reasonable grounds for the
such conditions as to construction, equipment, maintenance, service, above finding of the Commission.
or operation as the public interests and convenience may reasonably
require, and to issue certificates of public convenience and necessity It is now a very well-settled rule in this jurisdiction that the findings
when such is required or provided by any law or franchise." Thus, the and conclusions of fact made by the Public Service Commission,
efficacy of a municipal electric franchise arises, therefore, only after after weighing the evidence adduced by the parties in a public
the approval of the Public Service Commission. (Almendras vs. service case, will not be disturbed by the Supreme Court unless
Ramos, 90 Phil. 231) . those findings and conclusions appear not to be reasonably
supported by evidence. (La Mallorca and Pampanga Bus Co. vs.
The conclusion herein reached regarding the validity of the franchise granted Mercado, L-19120, November 29, 1965)
to Morong Electric is not incompatible with the holding of this Court
in Cagayan Fishing Development Co., Inc. vs. Teodoro Sandiko 27upon which For purposes of appeal, what is decisive is that said testimonial
the petitioner leans heavily in support of its position. In said case this Court evidence provides reasonable support for the Public Service
held that a corporation should have a full and complete organization and Commission's findings of financial capacity on the part of applicants,
existence as an entity before it can enter into any kind of a contract or rendering such findings beyond our power to disturb. (Del Pilar
transact any business. It should be pointed out, however, that this Court did Transit vs. Silva, L-21547, July 15, 1966)
not say in that case that the rule is absolute or that under no circumstances
may the acts of promoters of a corporation be ratified or accepted by the It may be worthwhile to mention in this connection that per inspection report
corporation if and when subsequently organized. Of course, there are dated January 20, 1964 29 of Mr. Meliton Martinez of the Commission, who
exceptions. It will be noted that American courts generally hold that a inspected the electric service of Morong on January 15-16, 1964, Morong
contract made by the promoters of a corporation on its behalf may be Electric "is serving electric service to the entire area covered by its approved
adopted, accepted or ratified by the corporation when organized. 28 plan and has constructed its line in accordance with the plans and
specifications approved by the Commission." By reason thereof, it was
2. The validity of the franchise and the corporate personality of Morong recommended that the requests of Morong Electric (1) for the withdrawal of
Electric to accept the same having been shown, the next question to be its deposit in the amount of P1,000.00 with the Treasurer of the Philippines,
resolved is whether said company has the financial qualification to operate and (2) for the approval of Resolution No. 160 of the Municipal Council of
an electric light, heat and power service. Petitioner challenges the financial Morong, Rizal, exempting the operator from making the additional P9,000.00
capability of Morong Electric, by pointing out the inconsistencies in the deposit mentioned in its petition, dated September 16, 1963, be granted. This
testimony of Mr. Jose P. Ingal, president of said company, regarding its report removes any doubt as to the financial capability of Morong Electric to
assets and the amount of its initial investment for the electric plant. In this operate and maintain an electric light, heat and power service.
connection it should be stated that on the basis of the evidence presented on
the matter, the Commission has found the Morong Electric to be "financially 3. With the financial qualification of Morong Electric beyond doubt, the
qualified to install, maintain and operate the proposed electric light, heat and remaining question to be resolved is whether, or not, the findings of fact of
power service." This is essentially a factual determination which, in a number

163
the Commission regarding petitioner's service are supported by evidence. It Co., Inc. vs. Mercado, L-19120, November 29, 1965 citing
is the contention of the petitioner that the Commission made some findings of Pangasinan Trans. Co., Inc. vs. Dela Cruz, 96 Phil. 278)
fact prejudicial to its position but which do not find support from the evidence
presented in this case. Specifically, petitioner refers to the statements or For that matter, petitioner's pretension that it has a prior right to the operation
findings that its service had "turned from bad to worse," that it miserably of an electric service in Morong, Rizal, is not tenable; and its plea for
failed to comply with the oft-repeated promises to bring about the needed protection of its investment, as in the previous case, cannot be entertained.
improvement, that its equipment is unserviceable, and that it has no longer
any plant site and, therefore, has discredited itself. Petitioner further states WHEREFORE, the two decisions of the Public Service Commission,
that such statements are not only devoid of evidentiary support but contrary appealed from, should be, as they are hereby affirmed, with costs in the two
to the testimony of its witness, Mr. Harry Bernardino, who testified that cases against petitioner Rizal Light & Ice Co., Inc. It is so ordered.
petitioner was rendering efficient and satisfactory service before its electric
plant was burned on July 29, 1962.
Concepcion, C.J., Reyes, J.B.L., Dizon, Makalintal, Sanchez, Castro,
Angeles and Fernando, JJ., concur.
On the face of the decision appealed from, it is obvious that the Commission
in describing the kind of service petitioner was rendering before its certificate
was ordered revoked and cancelled, took judicial notice of the records of the
previous case (PSC Case No. 39715) where the quality of petitioner's service
had been squarely put in issue. It will be noted that the findings of the
Commission were made notwithstanding the fact that the aforementioned
testimony of Mr. Bernardino had been emphasized and pointed out in
petitioner's Memorandum to the Commission. 30 The implication is simple:
that as between the testimony of Mr. Bernardino and the inspection reports of
the engineers of the Commission, which served as the basis of the
revocation order, the Commission gave credence to the latter. Naturally,
whatever conclusion or finding of fact that the Commission arrived at
regarding the quality of petitioner's service are not borne out by the evidence
presented in this case but by evidence in the previous case. 31 In this
connection, we repeat, the conclusion, arrived at by the Commission after
weighing the conflicting evidence in the two related cases, is a conclusion of
fact which this Court will not disturb.

And it has been held time and again that where the Commission has
reached a conclusion of fact after weighing the conflicting evidence,
that conclusion must be respected, and the Supreme Court will not
interfere unless it clearly appears that there is no evidence to support
the decision of the Commission. (La Mallorca and Pampanga Bus

164
165
CRUZ, J.:

We gave limited due course to this petition on the question of the solidary
liability of the petitioners with their co-defendants in the lower court 1 because
of the challenge to the following paragraph in the dispositive portion of the
decision of the respondent court: *

1. Defendants are hereby ordered to jointly and severally pay the


plaintiff the amount of P50,000.00 for the preparation of the project
study and his technical services that led to the organization of the
defendant corporation, plus P10,000.00 attorney's fees; 2

The petitioners claim that this order has no support in fact and law because
they had no contract whatsoever with the private respondent regarding the
above-mentioned services. Their position is that as mere subsequent
investors in the corporation that was later created, they should not be held
solidarily liable with the Filipinas Orient Airways, a separate juridical entity,
and with Barretto and Garcia, their co-defendants in the lower court, ** who
were the ones who requested the said services from the private
respondent. 3

We are not concerned here with the petitioners' co-defendants, who have not
appealed the decision of the respondent court and may, for this reason, be
presumed to have accepted the same. For purposes of resolving this case
before us, it is not necessary to determine whether it is the promoters of the
proposed corporation, or the corporation itself after its organization, that shall
be responsible for the expenses incurred in connection with such
organization.
G.R. No. L-48627

The only question we have to decide now is whether or not the petitioners
FERMIN Z. CARAM, JR. and ROSA O. DE CARAM, petitioners
themselves are also and personallyliable for such expenses and, if so, to
vs.
what extent.
THE HONORABLE COURT OF APPEALS and ALBERTO V.
ARELLANO, respondents.
The reasons for the said order are given by the respondent court in its
decision in this wise:

166
As to the 4th assigned error we hold that as to the remuneration due corporation. By the same token, We find no reason why the plaintiff
the plaintiff for the preparation of the project study and the pre- should not be similarly compensated not only for having actively
organizational services in the amount of P50,000.00, not only the participated in the preparation of the project study for several months
defendant corporation but the other defendants including defendants and its subsequent revision but also in his having been involved in
Caram should be jointly and severally liable for this amount. As we the pre-organization of the defendant corporation, in the preparation
above related it was upon the request of defendants Barretto and of the franchise, in inviting the interest of the financiers and in the
Garcia that plaintiff handled the preparation of the project study training and screening of personnel. We agree that for these special
which project study was presented to defendant Caram so the latter services of the plaintiff the amount of P50,000.00 as compensation is
was convinced to invest in the proposed airlines. The project study reasonable. 5
was revised for purposes of presentation to financiers and the banks.
It was on the basis of this study that defendant corporation was The above finding bolsters the conclusion that the petitioners were not
actually organized and rendered operational. Defendants Garcia and involved in the initial stages of the organization of the airline, which were
Caram, and Barretto became members of the Board and/or officers being directed by Barretto as the main promoter. It was he who was putting
of defendant corporation. Thus, not only the defendant corporation all the pieces together, so to speak. The petitioners were merely among the
but all the other defendants who were involved in the preparatory financiers whose interest was to be invited and who were in fact persuaded,
stages of the incorporation, who caused the preparation and/or on the strength of the project study, to invest in the proposed airline.
benefited from the project study and the technical services of plaintiff
must be liable. 4 Significantly, there was no showing that the Filipinas Orient Airways was a
fictitious corporation and did not have a separate juridical personality, to
It would appear from the above justification that the petitioners were not justify making the petitioners, as principal stockholders thereof, responsible
really involved in the initial steps that finally led to the incorporation of the for its obligations. As a bona fide corporation, the Filipinas Orient Airways
Filipinas Orient Airways. Elsewhere in the decision, Barretto was described should alone be liable for its corporate acts as duly authorized by its officers
as "the moving spirit." The finding of the respondent court is that the project and directors.
study was undertaken by the private respondent at the request of Barretto
and Garcia who, upon its completion, presented it to the petitioners to induce In the light of these circumstances, we hold that the petitioners cannot be
them to invest in the proposed airline. The study could have been presented held personally liable for the compensation claimed by the private
to other prospective investors. At any rate, the airline was eventually respondent for the services performed by him in the organization of the
organized on the basis of the project study with the petitioners as major corporation. To repeat, the petitioners did not contract such services. It was
stockholders and, together with Barretto and Garcia, as principal officers. only the results of such services that Barretto and Garcia presented to them
and which persuaded them to invest in the proposed airline. The most that
The following portion of the decision in question is also worth considering: can be said is that they benefited from such services, but that surely is no
justification to hold them personally liable therefor. Otherwise, all the other
... Since defendant Barretto was the moving spirit in the pre- stockholders of the corporation, including those who came in later, and
organization work of defendant corporation based on his experience regardless of the amount of their share holdings, would be equally and
and expertise, hence he was logically compensated in the amount of personally liable also with the petitioners for the claims of the private
P200,000.00 shares of stock not as industrial partner but more for respondent.
his technical services that brought to fruition the defendant

167
The petition is rather hazy and seems to be flawed by an ambiguous
ambivalence. Our impression is that it is opposed to the imposition of solidary
responsibility upon the Carams but seems to be willing, in a vague,
unexpressed offer of compromise, to accept joint liability. While it is true that
it does here and there disclaim total liability, the thrust of the petition seems
to be against the imposition of solidary liability only rather than against any
liability at all, which is what it should have categorically argued.

Categorically, the Court holds that the petitioners are not liable at all, jointly
or jointly and severally, under the first paragraph of the dispositive portion of
the challenged decision. So holding, we find it unnecessary to examine at
this time the rules on solidary obligations, which the parties-needlessly, as it
turns out have belabored unto death.

WHEREFORE, the petition is granted. The petitioners are declared not liable
under the challenged decision, which is hereby modified accordingly. It is so
ordered.

Yap (Chairman), Narvasa, Melencio-Herrera, Feliciano and Sarmiento, JJ.,


concur.
Gancayco, J., took no part.

168
G.R. No. L-5003 June 27, 1953

NAZARIO TRILLANA, administrator-appellee,


vs.
QUEZON COLLEGE, INC., claimant-appellant.

Singson, Barnes, Yap and Blanco for appellant.


Delgado, Flores & Macapagal for appellee.

PARAS, J.:

Damasa Crisostomo sent the following letter to the Board of Trustees of the
Quezon College:

June 1, 1948

The BOARD OF TRUSTEES


Quezon College
Manila

Gentlemen:

Please enter my subscription to dalawang daan (200) shares of your


capital stock with a par value of P100 each. Enclosed you will find
(Babayaran kong lahat pagkatapos na ako ay makapag-pahuli ng
isda) pesos as my initial payment and the balance payable in
accordance with law and the rules and regulations of the Quezon
College. I hereby agree to shoulder the expenses connected with
said shares of stock. I further submit myself to all lawful demands,
decisions or directives of the Board of Trustees of the Quezon
College and all its duly constituted officers or authorities (ang nasa
itaas ay binasa at ipinaliwanag sa akin sa wikang tagalog na aking
nalalaman).

169
subscription for 200 shares be entered) not only did not enclose any initial
payment but stated that "babayaran kong lahat pagkatapos na ako ay
Very respectfully, makapagpahuli ng isda." There is nothing in the record to show that the
Quezon College, Inc. accepted the term of payment suggested by Damasa
(Sgd.) DAMASA CRISOSTOMO Crisostomo, or that if there was any acceptance the same came to her
Signature of subscriber knowledge during her lifetime. As the application of Damasa Crisostomo is
obviously at variance with the terms evidenced in the form letter issued by
the Quezon College, Inc., there was absolute necessity on the part of the
College to express its agreement to Damasa's offer in order to bind the latter.
Nilagdaan sa aming harapan:
Conversely, said acceptance was essential, because it would be unfair to
immediately obligate the Quezon College, Inc. under Damasa's promise to
JOSE CRISOSTOMO pay the price of the subscription after she had caused fish to be caught. In
EDUARDO CRISOSTOMO other words, the relation between Damasa Crisostomo and the Quezon
College, Inc. had only thus reached the preliminary stage whereby the latter
Damasa Crisostomo died on October 26, 1948. As no payment appears to offered its stock for subscription on the terms stated in the form letter, and
have been made on the subscription mentioned in the foregoing letter, the Damasa applied for subscription fixing her own plan of payment, a
Quezon College, Inc. presented a claim before the Court of First Instance of relation, in the absence as in the present case of acceptance by the Quezon
Bulacan in her testate proceeding, for the collection of the sum of P20,000, College, Inc. of the counter offer of Damasa Crisostomo, that had not ripened
representing the value of the subscription to the capital stock of the Quezon into an enforceable contract.
College, Inc. This claim was opposed by the administrator of the estate, and
the Court of First Instance of Bulacan, after hearing issued an order Indeed, the need for express acceptance on the part of the Quezon College,
dismissing the claim of the Quezon College, Inc. on the ground that the Inc. becomes the more imperative, in view of the proposal of Damasa
subscription in question was neither registered in nor authorized by the Crisostomo to pay the value of the subscription after she has harvested fish,
Securities and Exchange Commission. From this order the Quezon College, a condition obviously dependent upon her sole will and, therefore, facultative
Inc. has appealed. in nature, rendering the obligation void, under article 1115 of the old Civil
Code which provides as follows: "If the fulfillment of the condition should
It is not necessary for us to discuss at length appellant's various assignments depend upon the exclusive will of the debtor, the conditional obligation shall
of error relating to the propriety of the ground relief upon by the trial court, be void. If it should depend upon chance, or upon the will of a third person,
since, as pointed out in the brief for the administrator and appellee, there are the obligation shall produce all its effects in accordance with the provisions of
other decisive considerations which, though not touched by the lower court, this code." It cannot be argued that the condition solely is void, because it
amply sustained the appealed order. would have served to create the obligation to pay, unlike a case, exemplified
by Osmea vs. Rama (14 Phil., 99), wherein only the potestative condition
It appears that the application sent by Damasa Crisostomo to the Quezon was held void because it referred merely to the fulfillment of an already
College, Inc. was written on a general form indicating that an applicant will existing indebtedness.
enclose an amount as initial payment and will pay the balance in accordance
with law and the regulations of the College. On the other hand, in the letter In the case of Taylor vs. Uy Tieng Piao, et al. (43 Phil., 873, 879), this Court
actually sent by Damasa Crisostomo, the latter (who requested that her already held that "a condition, facultative as to the debtor, is obnoxious to the

170
first sentence contained in article 1115 and renders the whole obligation
void."

Wherefore, the appealed order is affirmed, and it is so ordered with costs


against appellant.

Tuason, Padilla and Reyes, JJ., concur in the result.

171
172
G.R. Nos. L-48195 and 48196 May 1, 1942 Municipality of Silang, Province of Cavite, Philippine Islands, party of
the Second Part, hereinafter called the seller,
SOFRONIO T. BAYLA, ET AL., petitioners,
vs. WITNESSETH:
SILANG TRAFFIC CO., INC., respondent.
SILANG TRAFFIC CO., petitioner, vs. SOFRONIO BAYLA, ET That the subscriber promises to pay personally or by his duly
AL., respondents. authorized agent to the seller at the Municipality of Silang, Province
of Cavite, Philippine Islands, the sum of one thousand five hundred
E. A. Beltran for petitioners. pesos (P1,500), Philippine currency, as purchase price of FIFTEEN
Conrado V. Sanchez, Melchor C. Benitez, and Enrique M. Fernando for (15) shares of capital stock, said purchase price to be paid as
respondent. follows, to wit: five (5%) per cent upon the execution of the contract,
the receipt whereof is hereby acknowledged and confessed, and the
OZAETA, J.: remainder in installments of five per cent, payable within the first
month of each and every quarter thereafter, commencing on the 1st
Petitioners in G.R. No. 48195 instituted this action in the Court of First day of July, 1935, with interest on deferred payments at the rate of
Instance of Cavite against the respondent Silang Traffic Co., Inc. (cross- SIX (6%) per cent per annum until paid.
petitioner in G.R. No. 48196), to recover certain sums of money which they
had paid severally to the corporation on account of shares of stock they That the said subscriber further agrees that if he fails to pay any of
individually agreed to take and pay for under certain specified terms and said installment when due, or to perform any of the aforesaid
conditions, of which the following referring to the petitioner Josefa Naval, is conditions, or if said shares shall be attached or levied upon by
typical: creditors of the said subscriber, then the said shares are to revert to
the seller and the payments already made are to be forfeited in favor
AGREEMENT FOR INSTALLMENT SALE OF SHARES IN THE of said seller, and the latter may then take possession, without
"SILANG TRAFFIC COMPANY, INC.," resorting to court proceedings.

The said seller upon receiving full payment, at the time and manner
hereinbefore specified, agrees to execute and deliver to said
Silang, Cavite, P. I. subscriber, or to his heirs and assigns, the certificate of title of said
shares, free and clear of all encumbrances.

In testimony whereof, the parties have hereunto set their hands in


THIS AGREEMENT, made and entered into between Mrs. Josefa
the Municipality of Silang, Province of Cavite, Philippine Islands, this
Naval, of legal age, married and resident of the Municipality of
30th day of March, 1935.
Silang, Province of Cavite, Philippine Islands, party of the First Part,
hereinafter called the subscriber, and the "Silang Traffic Company,
Inc.," a corporation duly organized and existing by virtue of and
under the laws of the Philippine Islands, with its principal office in the

173
(Sgd.) JOSEFA NAVAL Toledo................
SILANG TRAFFIC COMPANY, INC.
Subscriber

By (Sgd.) LINO GOMEZ Petitioners' action for the recovery of the sums above mentioned is based on
President. a resolution by the board of directors of the respondent corporation on
August 1, 1937, of the following tenor:

A mocion sel Sr. Marcos Caparas y secundado por el Sr. Alejandro


(Exhibit 1. Notarial acknowledgment omitted.) Bayla, que para el bien de la corporacion y la pronta terminacion del
asunto civil No. 3125 titulado "Vicente F. Villanueva et al. vs. Lino
The agreements signed by the other petitioners were of the same date Gomez et al.," en el Juzgado de Primera Instancia de Cavite, donde
(March 30, 1935) and in identical terms as the foregoing except as to the se gasto y se gastara no poca cantidad de la Corporacion, se
number of shares and the corresponding purchase price. The petitioners resolvio y se aprobo por la Junta Directiva los siguientes:
agreed to purchase the following number of shares and, up to April 30, 1937,
had paid the following sums on account thereof: (a) Que se dejara sin efecto lo aprobado por la Junta Directiva el 3
de marzo, 1935, art. 11, sec. 162, sobre las cobranzas que se haran
por el Secretario Tesorero de la Corporacion a los accionistas que
habian tomado o suscrito nuevas acciones y que se permitia a estos
Sofronio T. 8 shares P360 pagar 20% del valor de las acciones suscritas en un ao, con interes
Bayla....... de 6% y el pago o jornal que se hara por trimestre.

(b) Se dejara sin efecto, en vista de que aun no esta pagado todo el
valor de las 123 acciones, tomadas de las acciones no expedidas
Venancio 8 shares 375 (unissued stock) de la Corporacion y que fueron suscritas por los
Toledo........ siguienes:

Josefa 15 shares 675 Lino 10 Acciones


Naval.............. Gomez.....................

Paz 15 shares 675 Venancio 8 Acciones

174
Toledo............. Sofronio 8 Acciones
Bayla.................

Melchor P. 17 Acciones
Benitez........ Dionisio 3 Acciones
Dungca.............

Isaias 14 Acciones
y devolver a las personas arriba descritas toda la cantidad que estas habian
Videa.................
pagado por las 123 acciones.

(c) Que se dejara sin efecto lo aprobado por la Junta Directiva el 3


marzo, 1935, art. V. sec. 165, sobre el cambio o trueque de las 31
Esteban 10 Acciones
acciones del Treasury Stock, contra las 32 acciones del Sr.
Velasco............
Numeriano Aldaba, en la corporacion Northern Luzon Transportation
Co. y que se devuelva al Sr. Numeriano Aldaba las 32 acciones
mencionadas despues que el haya devuelto el certificado de las 31
acciones de la Silang Traffic Co., Inc.
Numeriano S. 15 Acciones
Aldaba.... (d) Permitir al Tesorero de la Corporacion para que devuelva a las
personas arriba indicadas, las cantidades pagadas por las 123
acciones. (Exhibit A-1.)

Inocencio 8 Acciones The respondent corporation set up the following defenses: (1) That the
Cruz................. above-quoted resolution is not applicable to the petitioners Sofronio T. Bayla,
Josefa Naval, and Paz Toledo because on the date thereof "their subscribed
shares of stock had already automatically reverted to the defendant, and the
installments paid by them had already been forfeited"; and (2) that said
Josefa 15 Acciones resolution of August 1, 1937, was revoked and cancelled by a subsequent
Naval .................. resolution of the board of directors of the defendant corporation dated August
22, 1937.

175
The trial court absolved the defendant from the complaint and declared Whether a particular contract is a subscription or a sale of stock is a matter of
canceled (forfeited) in favor of the defendant the shares of stock in question. construction and depends upon its terms and the intention of the parties (4
It held that the resolution of August 1, 1937, was null and void, citing Velasco Fletcher, Cyclopedia of Corporation [permanent edition], 29, cited in Salmon,
vs. Poizat (37 Phil., 802), wherein this Court held that "a corporation has no Dexter & Co. vs. Unson (47 Phil. 649, 652). In the Unson case just cited, this
legal capacity to release an original subscriber to its capital stock from the Court held that a subscription to stock in an existing corporation is, as
obligation to pay for shares; and any agreement to this effect is invalid" between the subscriber and the corporation, simply a contract of purchase
Plaintiffs below appealed to the Court of Appeals, which modified of the trial and sale.
court as follows:
It seems clear from the terms of the contracts in question that they are
That part of the judgment dismissing plaintiff's complaint is affirmed, contracts of sale and not of subscription. The lower courts erred in
but that part thereof declaring their subscription canceled is overlooking the distinction between subscription and purchase "A
reversed. Defendant is directed to grant plaintiffs 30 days after final subscription, properly speaking, is the mutual agreement of the subscribers
judgment within which to pay the arrears on their subscription. to take and pay for the stock of a corporation, while a purchase is an
Without pronouncement as to costs. independent agreement between the individual and the corporation to buy
shares of stock from it at stipulated price." (18 C. J. S., 760.) In some
Both parties appealed to this Court by petition and cross-petition particulars the rules governing subscriptions and sales of shares are
for certiorari. Petitioners insist that they have the right to recover the amounts different. For instance, the provisions of our Corporation Law regarding calls
involved under the resolution of August 1, 1937, while the respondent and for unpaid subscription and assessment of stock (sections 37-50) do not
cross-petitioner on its part contends that said amounts have been apply to a purchase of stock. Likewise the rule that corporation has no legal
automatically forfeited and the shares of stock have reverted to the capacity to release an original subscriber to its capital stock from the
corporation under the agreement hereinabove quoted. obligation to pay for his shares, is inapplicable to a contract of purchase of
shares.
The parties litigant, the trial court, and the Court of Appeals have interpreted
or considered the said agreement as a contract of subscription to the capital The next question to determine is whether under the contract between the
stock of the respondent corporation. It should be noted, however, that said parties the failure of the purchaser to pay any of the quarterly installments on
agreement is entitled "Agreement for Installment Sale of Shares in the Silang the purchase price automatically gave rise to the forfeiture of the amounts
Traffic Company, Inc.,"; that while the purchaser is designated as already paid and the reversion of the shares to the corporation. The contract
"subscriber," the corporation is described as "seller"; that the agreement was provides for interest of the rate of six per centum per annum on deferred
entered into on March 30, 1935, long after the incorporation and organization payments. It is also provides that if the purchaser fails to pay any of said
of the corporation, which took place in 1927; and that the price of the stock installments when due, the said shares are to revert to the seller and the
was payable in quarterly installments spread over a period of five years. It payments already made are to be forfeited in favor of said seller. The
also appears that in civil case No. 3125 of the Court of First Instance of respondent corporation contends that when the petitioners failed to pay the
Cavite mentioned in the resolution of August 1, 1937, the right of the installment which fell due on or before July 31, 1937, forfeiture automatically
corporation to sell the shares of stock to the person named in said resolution took place, that is to say, without the necessity of any demand from the
(including herein petitioners) was impugned by the plaintiffs in said case, who corporation, and that therefore the resolution of August 1, 1937, authorizing
claimed a preferred right to buy said shares. the refund of the installments already paid was inapplicable to the petitioners,
who had already lost any and all rights under said contract. The contention

176
is, we think, untenable. The provision regarding interest on deferred Wherefore, the judgment of the court of appeals is hereby reversed and
payments would not have been inserted if it had been the intention of the another judgment will be entered against the defendant Silang Traffic Co.,
parties to provide for automatic forfeiture and cancelation of the contract. Inc., ordering it to pay to the plaintiffs Sofronio T. Bayla, Venancio Toledo,
Moreover, the contract did not expressly provide that the failure of the Josefa Naval, and Paz Toledo, the sums of P360, P375, P675, and P675,
purchaser to pay any installment would give rise to forfeiture and cancelation respectively, with legal interest on each of said sums from May 28, 1938, the
without the necessity of any demand from the seller; and under article 1100 date of the filing of the complaint, until the date of payment, and with costs in
of the Civil Code persons obliged to deliver or do something are not in default the three instances. So ordered.
until the moment the creditor demands of them judicially or extrajudicially the
fulfillment of their obligation, unless (1) the obligation or the law expressly Yulo, C.J., Moran, Paras and Bocobo, JJ., concur.
provides that demand shall not be necessary in order that default may arise,
(2) by reason of the nature and circumstances of the obligation it shall
appear that the designation of the time at which that thing was to be
delivered or the service rendered was the principal inducement to the
creation of the obligation.

Is the resolution of August 1, 1937, valid? The contract in question being one
of purchase and not subscription as we have heretofore pointed out, we see
no legal impediment to its rescission by agreement of the parties. According
to the resolution of August 1, 1937, the recission was made for the good of
the corporation and in order to terminate the then pending civil case involving
the validity of the sale of the shares in question among others. To that
rescission the herein petitioners apparently agreed, as shown by their
demand for the refund of the amounts they had paid as provided in said
resolution. It appears from the record that said civil case was subsequently
dismissed, and that the purchasers of shares of stock, other than the herein
petitioners, who were mentioned in said resolution were able to benefit by G.R. No. L-11528 March 15, 1918
said resolution. It would be an unjust discrimination to deny the same benefit
to the herein petitioners.
MIGUEL VELASCO, assignee of The Philippine Chemical Product Co.
(Ltd.), plaintiff-appellant,
We may add that there is no intimation in this case that the corporation was vs.
insolvent, or that the right of any creditor of the same was in any way JEAN M. POIZAT, defendant-appellee.
prejudiced by the rescission.
Vicente Rodriguez for appellant.
The attempted revocation of said rescission by the resolution of August 22, A. J. Burke for appellee.
1937, was invalid, it not having been agreed to by the petitioners.
STREET, J.:

177
From the amended complaint filed in this cause upon February 5, 1915, it provided that, in case he should refuse to make such payment, the
appears that the plaintiff, as assignee in insolvency of "The Philippine management of the corporation should be authorized to undertake judicial
Chemical Product Company" (Ltd.) is seeking to recover of the defendant, proceedings against him. When notification of this resolution reached Poizat
Jean M. Poizat, the sum of P1,500, upon a subscription made by him to the through the mail it evoked from him a manifestation of surprise and pain,
corporate stock of said company. It appears that the corporation in question which found expression in a letter written by him in reply, dated July 27,
was originally organized by several residents of the city of Manila, where the 1914, and addressed to Velasco, as treasurer and administrator. In this letter
company had its principal place of business, with a capital of P50,000, Poizat states that he had been given to understand by some member of the
divided into 500 shares. The defendant subscribed for 20 shares of the stock board of directors that he was to be relieved from his subscription upon the
of the company, an paid in upon his subscription the sum of P500, the par terms conceded to Infante; and he added:
value of 5 shares . The action was brought to recover the amount subscribed
upon the remaining shares. My desire to be relieved from the payment of the remaining 75 per
cent arises from the poor opinion which I entertain of the business
It appears that the defendant was a stock holder in the company from the and the faint hope of ever recovering any money invested. In
inception of the enterprise, and for sometime acted as its treasurer and consequence, I prefer to lose the whole of the 25 per cent I have
manager. While serving in this capacity he called in and collected all already paid rather than to continue investing more money in what I
subscriptions to the capital stock of the company, except the aforesaid 15 consider to be ruinous proposition.
shares subscribed by himself and another 15 shares owned by Jose R.
Infante. Within a short while the unfavorable opinion entertained by Poizat as to the
prospect of the company was found to be fully justified, as the company soon
Upon July 13, 1914, a meeting of the board of directors of the company was went into voluntary insolvency, Velasco being named as the assignee. He
held at which a majority of the stock was presented. Up[on this occasion two qualified at once by giving bond, and was duly inducted into the office of
resolutions, important to be here noted, were adopted. The first was a assignee upon November 25, 1914, by virtue of a formal transfer executed
proposal that the directors, or shareholders, of the company should make by the clerk in pursuance of section 32 of Act No. 1956.
good by new subscriptions, in proportion to their respective holdings, 15
shares which had been surrendered by Infante. It seems that this The answer of the defendant consisted of a general denial and a so-called
shareholder had already paid 25 per cent of his subscription upon 20 shares, special defense, consisting of a concatenation of statements more
leaving 15 shares unpaid for, and an understanding had been reached by appropriate for a demurrer than as material for a special defense. The
him and the management by which he was to be released from the obligation principal contention is that the call made by the board of directors of the
of his subscription, it being understood that what he had already paid should company on July 13, 1914 , was not made pursuant to the requirements of
not be refunded. Accordingly the directors present at this meeting subscribed sections 37 and 38 of the Corporation Law (Act No. 1459), and in particular
P1,200 toward taking up his shares, leaving a deficiency of P300 to be that the action was instituted before the expiration of the 30 days specified in
recovered by voluntary subscriptions from stockholders not present at the section 38.
meeting.
At the hearing of the Court of First Instance, judgment was rendered in favor
The other proposition was o the effect that Juan [Jean] M. Poizat, who was of the defendant, and the complaint was dismissed. From this action the
absent, should be required to pay the amount of his subscription upon the 15 plaintiff has appealed.
shares for which he was still indebted to the company. The resolution further

178
We think that Poizat is liable upon this subscription. A stock subscription is a conditions should be determined with reference to the rules governing
contract between the corporation on one side, and the subscriber on the contract liability in general; and where it appears as in this case that a
other, and courts will enforce it for or against either. It is a rule, accepted by matured stock subscription is unpaid, none of the provisions contained in
the Supreme Court of the United States, that a subscription for shares of section 38 to 48, inclusive, of Act No. 1459 can be permitted to obstruct or
stock does not require an express promise to pay the amount subscribed, as impede the action to recover thereon. By virtue of the first subsection of
the law implies a promise to pay on the part of the subscriber. (7 Ruling Case section 36 of the Insolvency Law (Act No. 1956) the assignee of the insolvent
Law, sec. 191.) Section 36 of the Corporation Law clearly recognizes that a corporation succeeds to all the corporate rights of action vested in the
stock subscription is subsisting liability from the time the subscription is corporation prior to its insolvency; and the assignee therefore has the same
made, since it requires the subscriber to pay interest quarterly from that date freedom with respect to suing upon the stock subscription as the directors
unless he is relieved from such liability by the by-laws of the corporation. The themselves would have had under section 49 above cited.
subscriber is as much bound to pay the amount of the share subscribed by
him as he would be to pay any other debt, and the right of the company to But there is another reason why the present plaintiff must prevail in this case,
demand payment is no less incontestable. even supposing that the failure of the directors to comply with the
requirements of the provisions of sections 38 to 48, inclusive, of Act No. 1459
The provisions of the Corporation Law (Act No. 1459) given recognition of might have been an obstacle to a recovery by the corporation itself. That
two remedies for the enforcement of stock subscriptions. The first and most reason is this: When insolvency supervenes upon a corporation and the court
special remedy given by the statute consists in permitting the corporation to assumes jurisdiction to wind up, all unpaid stock subscriptions become
put up the unpaid stock for sale and dispose of it for the account of the payable on demand, and are at once recoverable in an action instituted by
delinquent subscriber. In this case the provisions of section 38 to 48, the assignee or receiver appointed by the court. This rule apparently had
inclusive , of the Corporation Law are applicable and must be followed. The origin in a recognition of the principle that a court of equity, having jurisdiction
other remedy is by action in court, concerning which we find in section 49 the of the insolvency proceedings, could, if necessary, make the call itself, in its
following provision: capacity as successor to the powers exercised by the board of directors of
the defunct company. Later a further rule gained recognition to the effect that
Nothing in this Act shall prevent the directors from collecting, by the receiver or assignee, in an action instituted by proper authority, could
action in any court of proper jurisdiction, the amount due on any himself proceed to collect the subscription without the necessity of any prior
unpaid subscription, together with accrued interest and costs and call whatever. This conclusion is well supported by reference to the following
expenses incurred. authorities:

It is generally accepted doctrine that the statutory right to sell the subscriber's . . . a court of equity may enforce payment of the stock subscriptions,
stock is merely a remedy in addition to that which proceeds by action in although there have been no calls for them by the company.
court; and it has been held that the ordinary legal remedy by action exists (Hatch vs. Dana, 101 U. S., 205.)
even though no express mention thereof is made in the statute.
(Instone vs. Frankfort Bridge Co., 2 Bibb [Ky.], 576; 5 Am. Dec., 638.) It is again insisted that the plaintiffs cannot recover because the suit
was not preceded by a call or assessment against no right of action
No attempt is made in the Corporation Law to define the precise conditions accrues. In a suit by a solvent going corporation to collect a
under which an action may be maintained upon a stock subscription, as such subscription, and in certain suits provided by the statute this would
be true; but it is now quite well settled that when the corporation

179
becomes insolvent, with proceedings instituted by creditors to wind the board transcended its powers, and he no doubt still remained liable on
up and distribute its assets, no call or assessment is necessary such of his shares as were not taken up and paid for by other persons.
before the institution of suits to collect unpaid balances on
subscription. (Ross-Meehan Shoe F. Co. vs. Southern Malleable Iron The general doctrine is that the corporation has no legal capacity to
Co., 72 Fed., 957, 960; see also Henry vs. Vermillion etc. R. R. Co., release an original subscriber to its capital stock from the obligation
17 Ohio, 187, and Thompson on Corporations 2d ed., vol. 3, sec. of paying for his shares, in whole or in part, . . . (10 Cyc., 450.)
2697.)
The suggestion contained in Poizat's letter of July 27, 1914, to the effect that
It evidently cannot be permitted that a subscriber should escape from his he understood that he was to be relieved upon the same terms as Infante is,
lawful obligation by reason of the failure of the officers of the corporation to for the same reason, of no merit as matter of defense, even if an agreement
perform their duty in making a call; and when the original model of making to that effect had been duly proved.
the call becomes impracticable, the obligation must be treated as due upon
demand. If the corporation must be treated still an active entity, and this From what has been said it is manifest that the defendant is liable for P1,500,
action should be dismissed for irregularity in the making of the call, other the amount of his subscription upon the unpaid shares. Under section 36 of
steps could be taken by the board to cure the defect and another action the Corporation Law he is also liable for interest at the lawful rate from the
could be brought; but where the company is being wound up, no such date of his subscription, unless relieved from this liability by the by-laws of
procedure would be practicable. The better doctrine is that when insolvency the company. These by-laws have not been introduced in evidence and there
supervenes all unpaid subscriptions become at once due and enforceable. is no proof showing the exact date upon which the subscription was made,
though it is alleged in the original complaint that the company was organized
The printed bill of exceptions in this cause does not contain the original upon March 23, 1914. This allegation is not admitted in the agreed statement
complaint, nor does it state who was plaintiff therein or the date when the of facts. The defendant, however, inferentially admits in his letter of July 27,
action was instituted. It may, however, be gathered from the papers 1914, that his subscription had been made prior to July 13, 1914. It resulted
transmitted to this court that the action was originally instituted in the name of that in our opinion he should be held liable for interest from that date.
the Philippine Chemical Product Co. (Ltd.), prior to its insolvency, and that
later the assignee was substituted as plaintiff and then filed the amended The judgment of the lower court is therefore reversed, and judgment will be
complaint, with the permission of the court. Now, if we concede that no right rendered in favor of the plaintiff and against the defendant for the sum of one
of action existed when the original complaint was filed, a right of action thousand five hundred pesos (P1,500), with interest from July 13, 1014, and
certainly existed when the assignee filed his amended complaint; and as the costs of both instances. So ordered.
bill of exceptions fails to show that any exception was taken to the action of
the court in allowing the amended complaint to be filed, no objection would
Arellano, C.J., Torres, Johnson, Carson, Araullo, Malcolm, Avancea and
be here entertained on the ground that the action was prematurely brought.
Fisher, JJ., concur.

The circumstance that the board of directors in their meeting of July 13,
1914, resolved to release Infante from his obligation upon a subscription for
15 shares is no wise prejudicial to the right of the corporation or its assignee
to recover from Poizat upon a subscription made by him. In releasing Infante

180
G.R. Nos. L-24177-85 June 29, 1968

PHILIPPINE NATIONAL BANK, plaintiff-appellee,


vs.

181
BITULOK SAWMILL, INC., DINGALAN LUMBER CO., INC., SIERRA command, considering the persuasiveness of the plea that defendants-
MADRE LUMBER CO., INC., NASIPIT LUMBER CO., INC., appellees would "not have subscribed to [the] capital stock" of the Philippine
WOODWORKS, INC., GONZALO PUYAT, TOMAS B. MORATO, FINDLAY Lumber Distributing Agency "were it not for the assurance of the [then]
MILLAR LUMBER CO., INC., ET AL., INSULAR LUMBER CO., ANAKAN President of the Republic of the Philippines that the Government would back
LUMBER CO., AND CANTILAN LUMBER CO., INC., defendants-appellees. [it] up by investing P9.00 for every peso" 4 subscribed, a condition which was
not fulfilled, such commitment not having been complied with, be justified?
Tomas Besa, Simplicio N. Angeles and Jose B. Galang for plaintiff-appellee. The answer must be in the negative.
Bausa, Ampil and Suarez for defendant-appellant Woodworks, Inc.
Pacifico de Ocampo for defendant-appellant Anakan Lumber Co. It cannot be otherwise even if an element of unfairness and injustice could be
Ross, Selph, Salcedo, Del Rosario, Bito and Misa for defendant-appellant predicated, as the lower court, in a rather sympathetic mood, did find in the
Insular Lumber Co. plaintiff bank, as creditor, compelling defendant lumber producers under the
Garin, Boquiren and Tamesis for defendant-appellant Nasipit Lumber Co., above circumstances to pay the balance of their subscriptions. For a plain
Inc. and statutory command, if applicable, must be respected. The rule of law
Feria, Manglapus and Associates for defendant-appellant Gonzalo Puyat. cannot be satisfied with anything less. The appeal must be sustained.
Sycip, Salazar and Associates for defendant-appellant Cantilan Lumber Co.,
Inc. In these various suits decided jointly, the Philippine National Bank, as
Ozaeta, Gibbs and Ozaeta for defendant-appellant Findlay Millar Lumber creditor, and therefore the real party in interest, was allowed by the lower
Co., Inc. court to substitute the receiver of the Philippine Lumber Distributing Agency
Dominador Alafriz for defendant-appellant Bitulok Sawmill, Inc. in these respective actions for the recovery from defendant lumber producers
De la Costa and De la Costa for defendant-appellant Tomas B. Morato. the balance of their stock subscriptions. The amount sought to be collected
from defendants-appellees Bitulok Sawmill, Inc., Dingalan Lumber Co., Inc.,
FERNANDO, J.: and Sierra Madre Lumber Co., Inc., is P5,000.00, defendants-appellees
having made a partial payment of P15,000.00 of their total subscription worth
In the face of a statutory norm, which, as interpreted in a uniform line of P20,000.00; from defendant-appellee Nasipit Lumber Co., Inc., the sum of
decisions by this Court, speaks unequivocally and is free from doubt, the P10,000.00, defendant-appellee having made a partial payment of
lower court with full recognition that the case for the plaintiff creditor, P10,000.00 of its total subscription worth P20,000.00; from defendant-
Philippine National Bank, "is meritorious strictly from the legal appellee Woodworks, Inc., the sum of P10,886.00, defendant-appellee
standpoint" 1 but apparently unable to "close its eyes to the equity of the having made a partial payment of P9,114.00 of its total subscription worth
case" 2 dismissed nine (9) cases filed by it, seeking "to recover from the P20,000.00; from defendant-appellee Gonzalo Puyat the sum of P10,000.00,
defendant lumber producers [Bitulok Sawmill, Inc.; Dingalan Lumber Co., defendant-appellee having made a partial payment of P10,000.00 of his total
Inc., Sierra Madre Lumber Co., Inc.; Nasipit Lumber Co., Inc.; Woodworks, subscription worth P20,000.00; from defendant-appellee Tomas Morato the
Inc.; Gonzalo Puyat; Tomas B. Morato; Findlay Millar Lumber Co., Inc.; sum of P10,000.00, defendant-appellee having made a partial payment of
Insular Lumber Co., Inc.; Anakan Lumber Co., Inc.; and Cantilan Lumber Co., P10,000.00 of his total subscription worth P20,000.00; from defendant-
Inc.] the balance of their stock subscriptions to the Philippine Lumber appellee Findlay Millar Lumber Co., Inc., the sum of P10,000.00, defendant-
Distributing Agency, Inc." 3 In essence then, the crucial question posed by this appellee having made a partial payment of P10,000.00 of its total
appeal from such a decision of the lower court is adherence to the rule of subscription worth P20,000.00; from defendant-appellee Insular Lumber Co.,
law. Otherwise stated, would non-compliance with a plain statutory Inc., the sum of P5,000.00, defendant-appellee having made a partial

182
payment of P15,000.00 of its total subscription worth P20,000.00; from President Roxas instructed the Hon. Emilio Abello, then Executive Secretary
defendant-appellee Anakan Lumber Co., Inc., the sum of P15,000.00, and Chairman of the Board of Directors of the Philippine National Bank, for
defendant-appellee having made a partial payment of P5,000.00 of its total the latter to grant said agency an overdraft in the original sum of
subscription worth P20,000.00; and from defendant-appellee Cantilan P250,000.00 which was later increased to P350,000.00, which was approved
Lumber Co., Inc., the sum of P7,500.00, defendant-appellee having made a by said Board of Directors of the Philippine National Bank on July 28, 1947,
partial payment of P2,500.00 of its total subscription worth P10,000.00, plus payable on or before April 30, 1958, with interest at the rate of 6% per
interest at the legal rate from the filing of the suits and the costs of the suits annum, and secured by the chattel mortgages on the stock of lumber of said
in all the nine (9) cases. agency." 7 The Philippine Government did not invest the P9.00 for every peso
coming from defendant lumber producers. The loan extended to the
The Philippine Lumber Distributing Agency, Inc., according to the lower court, Philippine Lumber Distributing Agency by the Philippine National Bank was
"was organized sometime in the early part of 1947 upon the initiative and not paid. Hence, these suits.
insistence of the late President Manuel Roxas of the Republic of the
Philippines who for the purpose, had called several conferences between For the lower court, the above facts sufficed for their dismissal. To its mind "it
him and the subscribers and organizers of the Philippine Lumber Distributing is grossly unfair and unjust for the plaintiff bank now to compel the lumber
Agency, Inc." 5 The purpose was praiseworthy, to insure a steady supply of producers to pay the balance of their subscriptions .... Indeed, when the late
lumber, which could be sold at reasonable prices to enable the war sufferers President Roxas made representations to the plaintiff bank, thru the Hon.
to rehabilitate their devastated homes. The decision continues: "He Emilio Abello, who was then the Executive Secretary and Chairman of its
convinced the lumber producers to form a lumber cooperative and to pool Board of Directors, to grant said overdraft to the agency, it was the only way
their sources together in order to wrest, particularly, the retail trade from by which President Roxas could make good his commitment that the
aliens who were acting as middlemen in the distribution of lumber. At the Government would invest in said agency to the extent already mentioned
beginning, the lumber producers were reluctant to organize the cooperative because, according to said late President Roxas, the legislature had not
agency as they believed that it would not be easy to eliminate from the retail appropriated any amount for such counterpart. Consequently, viewing from
trade the alien middlemen who had been in this business from time all considerations of equity in the case, the Court finds that plaintiff bank
immemorial, but because the late President Roxas made it clear that such a should not collect any more from the defendants the balance of their
cooperative agency would not be successful without a substantial working subscriptions to the capital stock of the Philippine Lumber Distributing
capital which the lumber producers could not entirely shoulder, and as an Agency, Inc." 8
inducement he promised and agreed to finance the agency by making the
Government invest P9.00 by way of counterpart for every peso that the Even with the case for defendant lumber producers being put forth in its
members would invest therein,...." 6 strongest possible light in the appealed decision, the plaintiff creditor, the
Philippine National Bank, should have been the prevailing party. On the law
This was the assurance relied upon according to the decision, which stated as it stands, the judgment reached by the lower court cannot be sustained.
that the amount thus contributed by such lumber producers was not enough The appeal, as earlier made clear, possesses merit.
for the operation of its business especially having in mind the primary
purpose of putting an end to alien domination in the retail trade of lumber In Philippine Trust Co. v. Rivera, 9 citing the leading case of Velasco v.
products. Nor was there any appropriation by the legislature of the Poizat, 10 this Court held: "It is established doctrine that subscriptions to the
counterpart fund to be put up by the Government, namely, P9.00 for every capital of a corporation constitute a fund to which creditors have a right to
peso invested by defendant lumber producers. Accordingly, "the late look for satisfaction of their claims and that the assignee in insolvency can

183
maintain an action upon any unpaid stock subscription in order to realize What it decrees must be followed; what it commands must be obeyed. It
assets for the payment of its debt.... A corporation has no power to release must be respected, the wishes of the President, to the contrary
an original subscriber to its capital stock from the obligation of paying for his notwithstanding, even if impelled by the most worthy of motives and the most
shares, without a valuable consideration for such release; and as against persuasive equitable considerations. To repeat, such is not the case here.
creditors a reduction of the capital stock can take place only in the manner For at no time did President Roxas ever give defendant lumber producers to
and under the conditions prescribed by the statute or the charter or the understand that the failure of the Government for any reason to put up the
articles of incorporation. Moreover, strict compliance with the statutory counterpart fund could terminate their statutory liability.
regulations is necessary...." The Poizat doctrine found acceptance in later
cases. 11One of the latest cases, Lingayen Gulf Electric Power v. Such is not the law. Unfortunately, the lower court was of a different mind.
Baltazar, 12 Speaks to this effect: "In the case of Velasco v. Poizat, 13 the That is not to pay homage to the rule of law. Its decision then, one it is to be
corporation involved was insolvent, in which case all unpaid stock repeated influenced by what it considered to be the "equity of the case", is
subscriptions become payable on demand and are immediately recoverable not legally impeccable.
in an action instituted by the assignee."
WHEREFORE, the decision of the lower court is reversed and the cases
It would be unwarranted to ascribe to the late President Roxas the view that remanded to the lower court for judgment according to law, with full
the payment of the stock subscriptions, as thus required by law, could be consideration of the legal defenses raised by defendants-appellees, Bitulok
condoned in the event that the counterpart fund to be invested by the Sawmill, Inc.; Dingalan Lumber Co., Inc.; Sierra Madre Lumber Co., Inc.;
Government would not be available. Even if such were the case, however, Nasipit Lumber Co., Inc.; Woodworks, Inc.; Gonzalo Puyat; Tomas B. Morato;
and such a promise were in fact made, to further the laudable purpose to Findlay Millar Lumber Co., Inc.; Anakan Lumber Co., Inc.; and Cantilan
which the proposed corporation would be devoted and the possibility that the Lumber Co., Inc. No pronouncement as to costs.
lumber producers would lose money in the process, still the plain and specific
wording of the applicable legal provision as interpreted by this Court must be Concepcion, C.J., Reyes, J.B.L., Dizon, Makalintal, Zaldivar, Sanchez and
controlling. It is a well-settled principle that with all the vast powers lodged in Angeles, JJ., concur.
the Executive, he is still devoid of the prerogative of suspending the Castro, J., took no part.
operation of any statute or any of its terms.

The emphatic and categorical language of an American decision cited by the


late Justice Laurel, in People v. Vera, 14 comes to mind: "By the twentieth
article of the declaration of rights in the constitution of this commonwealth, it
is declared that the power of suspending the laws, or the execution of the
laws, ought never to be exercised but by the legislature, or by authority
derived from it, to be exercised in such particular cases only as the
legislature shall expressly provide for...." Nor could it be otherwise
considering that the Constitution specifically enjoins the President to see to it
that all laws be faithfully executed. 15 There may be a discretion as to what a
particular legal provision requires; there can be none whatsoever as to the
enforcement and application thereof once its meaning has been ascertained.

184
185
six telegraph wires of the Government, or whether it is only required to
furnish poles with crosspieces sufficient to carry four wires only.

It is admitted that the present poles and crosspieces between said


municipalities are sufficient to carry four telegraph wires and that they do now
carry four telegraph wires, by virtue of an agreement between the
respondents and the Bureau of the Posts of the Philippine Government. It is
admitted that the poles and not sufficient to carry six telegraph wires.

The petitioner relies upon the provisions of section 84 of act No. 1459. Act
No. 1459 is the General Corporation Law and was adopted by the United
States Philippine Commission on March 1, 1906. (Vol. 5, Pub. Laws, pp. 224-
268.) Section 84 of the said Act provides:
G.R. No. L-30646 January 30, 1929
The railroad corporation shall establish along the whole length of the
THE GOVERNMENT OF THE PHILIPPINE ISLANDS, petitioner, road a telegraph line for the use of the railroad. The posts of this line
vs. may be used for Government wires and shall be of sufficient length
THE MANILA RAILROAD COMPANY and JOSE PAEZ as Manager of said and strength and equipped with sufficient crosspiece to carry the
Company, respondents. number of wires which the Government may consider necessary for
the public service. The establishment, protection, and maintenance
Attorney-General Jaranilla for petitioner. of the wires and stations necessary for the public service shall be at
Jose Abreu for respondents. the cost of the Government. (Vol. 5, P. L., p. 247.)

JOHNSON, J.: The plaintiff contends that under said section 84 the defendant company is
required to erect and maintain posts for its telegraph wires, of sufficient
This is a petition in the Supreme Court of the extraordinary legal writ length and strength, and equipped with sufficient crosspieces to carry the
of mandamus presented by the Government of the Philippine Islands, number of wires which the Government may consider necessary for the
praying that the writ be issued to compel the Manila Railroad Company and public service, and that six wires are now necessary for the public service.
Jose Paez, as its manager, to provide and equip the telegraph poles of said
company between the municipality of Paniqui, Province of Tarlac, and the The respondents answered by a general and special defense. In their special
Municipality of San Fernando, Province of La Union, with crosspieces for six defense they contend that section 84 of Act No. 1459 has been repealed by
telegraph wires belonging to the Government, which, it is alleged, are section 1, paragraph 8 of Act No. 1510 of the United States Philippine
necessary for public service between said municipalities. Commission (vol. 5, P. L., pp. 350-358), and that under the provisions of said
Act No. 1510 the Government is entitled to place on the poles of the
The only question raised by the petition is whether the dependant company company four wires only. Act No. 1510 is the charter of the Manila Railroad
is required to provide and equip its telegraph poles with crosspieces to carry

186
Company. It was adopted by the United States Philippine Commission on said charter or contract it would be seen that there is no indication that the
July 7, 1906. Section 1, paragraph 8, of said Act No. 1510 provides: Government intended to impose upon said company any other conditions as
obligations not expressly found in said charter or contract. If that is true, then
8. The grantee (the Manila Railroad Company) shall have the right to certainly the Government cannot impose upon said company any conditions
construct and operate telegraph, telephone, and electrical or obligations found in any general law, which does not expressly modify said
transmission lines over said railways for the use of the railways and contract.
their business, and also, with the approval of the Secretary of War,
for public service and commercial purposes but these latter Section 84 of the Corporation Law (Act No. 1459) was intended to apply to all
privileges shall be subject to the following provisions: railways in the Philippine Islands which did not have a special charter
contract. Act No. 1510 applies only to the Manila Railroad Company, one of
In the construction of telegraph or telephone lines along the right of the respondents, and being a special charter of said company, its adoption
way the grantee (the Manila Railroad Company) shall erect and had the effect of superseding the provisions of the general Corporation Law
maintain poles with sufficient space thereon to permit the Philippine which are applicable to railraods in general. The special charter (Act No.
Government, at the expense of said Government, to place, operate, 1510) had the effect of superseding the general Corporation Law upon all
and maintain four wires for telegraph, telephone, and electrical matters covered by said special charter. Said Act, inasmuch as it contained a
transmission for any Government purposes between the termini of special provision relating to the erection of telegraph and telephone poles,
the lines of railways main or branch; and the Philippine Government and the number of wires which the Government might place thereon,
reserves to itself the right to construct, maintain, and operate superseded the general law upon that question.
telegraph, telephone, or electrical transmission lines over the right of
way of said railways for commercial military, or government Act No. 1510 is a special charter of the respondent company. It constitutes a
purposes, without unreasonably interfering with the construction, contract between the respondent company and the state; and the state and
maintenance, and operation by the grantee of its railways, telegraph, the grantee of a charter are equally bound by its provisions. For the state to
telephone, and electrical transmission lines. impose an obligation or a duty upon the respondent company, which is not
expressly provided for in the charter (Act No. 1510), would amount to a
To answer the question above stated, it becomes necessary to determine violation of said contract between the state and the respondent company.
whether section 84 of Act No. 1459 is applicable to the Manila Railroad The provisions of Act No. 1459 relating to the number of wires which the
Company, or whether the manila Railroad Company is governed by section Government may place upon the poles of the company are different and
1, paragraph 8, of Act No. 1510. As has been said, Act No. 1459 is a general more enerous than the provisions of the charter upon the same question.
law applicable to corporations generally, while Act No. 1510 is the charter of Therefore, to allow the plaintiff to require of the respondent company a
the Manila Railroad Company and constitute a contract between it and the compliance with said section 84 of Act No. 1459, would be to require of the
Government. respondent company and the performance of an obligation which is not
imposed upon it by its charter. The charter of a corporation is a contract
Inasmuch as Act No. 1510 is the charter of Manila Railroad Company and between three parties: (a) it is a contract between the state and the
constitute a contract between it and the Governmemnt, it would seem that corporation to which the charter is granted; (b) it is a contact between the
the company is governd by its contract and not by the provisions of any stockholders and the state and (c) it is also a contract between the
general law upon questions covered by said contract. From a reading of the corporation and its stockholders. (Cook on Corporations, vol. 2, sec. 494 and
cases cited.)

187
The question is not whether Act No. 1510 repealed Act No. 1459; but
whether, after the adoption of Act No. 1510, the respondents are obliged to
comply with the special provision above mentioned, contained in Act No.
1459. We must answer that question in the native. Both laws are still in force,
unless otherwise repealed. Act No. 1510 is applicable to respondents upon
the question before us, while Act No. 1459 is not applicable.

The petitioner, in view of all the foregoing facts and the law applicable
thereto, has not shown itself entitled to the remedy prayed for. The prayer of
the petition must, therefore, be denied. And without any finding as to costs, it
is so ordered.

Street, Malcolm, Villamor, Ostrand, Johns, Romualdez and Villa-Real, JJ.,


concur.

188
(2) There exist valid reasons for refusing to register the transfer of the subject
of stock, namely:

(a) a pending controversy over the ownership of the


certificates of stock with the Regional Trial Court;

(b) claims that the Deeds of Assignment covering the subject


certificates of stock were fictitious and antedated; and
G.R. No. 96674 June 26, 1992
(c) claims on a resultant possible deprivation of inheritance
RURAL BANK OF SALINAS, INC., MANUEL SALUD, LUZVIMINDA TRIAS share in relation with a conflicting claim over the subject
and FRANCISCO TRIAS, petitioners, certificates of stock.

vs. The facts are not disputed.

COURT OF APPEALS*, SECURITIES AND EXCHANGE COMMISSION, On June 10, 1979, Clemente G. Guerrero, President of the Rural Bank of
MELANIA A. GUERRERO, LUZ ANDICO, WILHEMINA G. ROSALES, Salinas, Inc., executed a Special Power of Attorney in favor of his wife,
FRANCISCO M. GUERRERO, JR., and FRANCISCO GUERRERO , private respondent Melania Guerrero, giving and granting the latter full power
SR.,respondents. and authority to sell or otherwise dispose of and/or mortgage 473 shares of
stock of the Bank registered in his name (represented by the Bank's stock
certificates nos. 26, 49 and 65), to execute the proper documents therefor,
and to receive and sign receipts for the dispositions.

PARAS, J.:
On February 27, 1980, and pursuant to said Special Power of Attorney,
private respondent Melania Guerrero, as Attorney-in-Fact, executed a Deed
The basic controversy in this case is whether or not the respondent court of Assignment for 472 shares out of the 473 shares, in favor of private
erred in sustaining the Securities and Exchange Commission when it respondents Luz Andico (457 shares), Wilhelmina Rosales (10 shares) and
compelled by Mandamus the Rural Bank of Salinas to register in its stock Francisco Guerrero, Jr. (5 shares).
and transfer book the transfer of 473 shares of stock to private respondents.
Petitioners maintain that the Petition for Mandamus should have been denied
Almost four months later, or two (2) days before the death of Clemente
upon the following grounds.
Guerrero on June 24, 1980, private respondent Melania Guerrero, pursuant
to the same Special Power of Attorney, executed a Deed of Assignmentfor
(1) Mandamus cannot be a remedy cognizable by the Securities and the remaining one (1) share of stock in favor of private respondent Francisco
Exchange Commission when the purpose is to register certificates of stock in Guerrero, Sr.
the names of claimants who are not yet stockholders of a corporation:

189
Subsequently, private respondent Melania Guerrero presented to petitioner hearing officer denied the Motion for Intervention for lack of merit. On appeal,
Rural Bank of Salinas the two (2) Deeds of Assignment for registration with a the SEC En Banc affirmed the decision of the hearing officer.
request for the transfer in the Bank's stock and transfer book of the 473
shares of stock so assigned, the cancellation of stock certificates in the name Intervenor Guerrero filed a complaint before the then Court of First Instance
of Clemente G. Guerrero, and the issuance of new stock certificates covering of Rizal, Quezon City Branch, against private respondents for the annulment
the transferred shares of stocks in the name of the new owners thereof. of the Deeds of Assignment, docketed as Civil Case No. Q-32050.
However, petitioner Bank denied the request of respondent Melania Petitioners, on the other hand, filed a Motion to Dismiss and/or to Suspend
Guerrero. Hearing of SEC Case No. 1979 until after the question of whether the subject
Deeds of Assignment are fictitious, void or simulated is resolved in Civil Case
On December 5, 1980, private respondent Melania Guerrero filed with the No. Q-32050. The SEC Hearing Officer denied said motion.
Securities and Exchange Commission" (SEC) an action
for mandamus against petitioners Rural Bank of Salinas, its President and On December 10, 1984, the SEC Hearing Officer rendered a Decision
Corporate Secretary. The case was docketed as SEC Case No. 1979. granting the writ of Mandamus prayed for by the private respondents and
directing petitioners to cancel stock certificates nos. 26, 49 and 65 of the
Petitioners filed their Answer with counterclaim on December 19, 1980 Bank, all in the name of Clemente G. Guerrero, and to issue new certificates
alleging the upon the death of Clemente G. Guerrero, his 473 shares of stock in the names of private respondents, except Melania Guerrero. The
became the property of his estate, and his property and that of his widow dispositive, portion of the decision reads:
should first be settled and liquidated in accordance with law before any
distribution can be effected so that petitioners may not be a party to any WHEREFORE, judgment is hereby rendered in favor of the
scheme to evade payment of estate or inheritance tax and in order to avoid petitioners and against the respondents, directing the latter,
liability to any third persons or creditors of the late Clemente G. Guerrero. particularly the corporate secretary of respondent Rural
Bank of Salinas, Inc., to register in the latter's Stock and
On January 29, 1981, a motion for intervention was filed by Maripol Transfer Book the transfer of 473 shares of stock of
Guerrero, a legally adopted daughter of the late Clemente G. Guerrero and respondent Bank and to cancel Stock Certificates Nos. 26,
private respondent Melania Guerrero, who stated therein that on November 45 and 65 and issue new Stock Certificates covering the
26, 1980 (almost two weeks before the filing of the petition for Mandamus) a transferred shares in favor of petitioners, as follows:
Petition for the administration of the estate of the late Clemente G. Guerrero
had been filed with the Regional Trial Court, Pasig, Branch XI, docketed as 1. Luz Andico 457 shares
Special Proceedings No. 9400. Maripol Guerrero further claimed that the
Deeds of Assignment for the subject shares of stock are fictitious and 2. Wilhelmina Rosales 10 shares
antedated; that said conveyances are donations since the considerations
therefor are below the book value of the shares, the assignees/private
3. Francisco Guerrero, Jr. 5 shares
respondents being close relatives of private respondent Melania Guerrero;
and that the transfer of the shares in question to assignees/private
respondents, other than private respondent Melania Guerrero, would deprive 4. Francisco Guerrero, Sr. 1 share
her (Maripol Guerrero) of her rightful share in the inheritance. The SEC

190
and to pay to the above-named petitioners, the dividends for In the case of Fleisher vs. Botica Nolasco, 47 Phil. 583, the Court
said shares corresponding to the years 1981, 1982, 1983 interpreted Sec. 63 in his wise:
and 1984 without interest.
Said Section (Sec. 35 of Act 1459 [now Sec. 63 of the
No pronouncement as to costs. Corporation Code]) contemplates no restriction as to whom
the stocks may be transferred. It does not suggest that any
SO ORDERED. (p. 88, Rollo) discrimination may be created by the corporation in favor of,
or against a certain purchaser. The owner of shares, as
On appeal, the SEC En Banc affirmed the decision of the Hearing Officer. owner of personal property, is at liberty, under said section to
Petitioner filed a petition for review with the Court of Appeals but said Court dispose them in favor of whomever he pleases, without
likewise affirmed the decision of the SEC. limitation in this respect, than the general provisions of
law. . . .
We rule in favor of the respondents.
The only limitation imposed by Section 63 of the Corporation Code is
when the corporation holds any unpaid claim against the shares
Section 5 (b) of P.D. No. 902-A grants to the SEC the original and exclusive
intended to be transferred, which is absent here.
jurisdiction to hear and decide cases involving intracorporate controversies.
An intracorporate controversy has been defined as one which arises between
a stockholder and the corporation. There is no distinction, qualification, nor A corporation, either by its board, its by-laws, or the act of its officers, cannot
any exception whatsoever (Rivera vs. Florendo, 144 SCRA 643 [1986]). The create restrictions in stock transfers, because:
case at bar involves shares of stock, their registration, cancellation and
issuances thereof by petitioner Rural Bank of Salinas. It is therefore within . . . Restrictions in the traffic of stock must have their source
the power of respondent SEC to adjudicate. in legislative enactment, as the corporation itself cannot
create such impediment. By-laws are intended merely for the
Respondent SEC correctly ruled in favor of the registering of the shares of protection of the corporation, and prescribe regulation, not
stock in question in private respondent's names. Such ruling finds support restriction; they are always subject to the charter of the
under Section 63 of the Corporation Code, to wit: corporation. The corporation, in the absence of such power,
cannot ordinarily inquire into or pass upon the legality of the
transactions by which its stock passes from one person to
Sec. 63. . . . Shares of stock so issued are personal property
another, nor can it question the consideration upon which a
and may be transferred by delivery of the certificate or
sale is based. . . . (Tomson on Corporation Sec.
certificates indorsed by the owner or his attorney-in-fact or
4137, citedin Fleisher vs. Nolasco, Supra).
other person legally authorized to make the transfer. No
transfer, however, shall be valid, except as between the
parties, until the transfer is recorded in the books of the The right of a transferee/assignee to have stocks transferred to his name is
corporation . . . an inherent right flowing from his ownership of the stocks. Thus:

191
Whenever a corporation refuses to transfer and register
stock in cases like the present, mandamuswill lie to compel
the officers of the corporation to transfer said stock in the
books of the corporation" (26, Cyc. 347, Hyer vs. Bryan, 19
Phil. 138; Fleisher vs. Botica Nolasco, 47 Phil. 583, 594).

The corporation's obligation to register is ministerial.

In transferring stock, the secretary of a corporation acts in


purely ministerial capacity, and does not try to decide the
question of ownership. (Fletcher, Sec. 5528, page 434).

The duty of the corporation to transfer is a ministerial one


and if it refuses to make such transaction without good
cause, it may be compelled to do so by mandamus. (See.
5518, 12 Fletcher 394)

For the petitioner Rural Bank of Salinas to refuse registration of the


transferred shares in its stock and transfer book, which duty is ministerial on
its part, is to render nugatory and ineffectual the spirit and intent of Section
63 of the Corporation Code. Thus, respondent Court of Appeals did not err in
upholding the Decision of respondent SEC affirming the Decision of its
Hearing Officer directing the registration of the 473 shares in the stock and
transfer book in the names of private respondents. At all events, the
registration is without prejudice to the proceedings in court to determine the
validity of the Deeds of Assignment of the shares of stock in
question.WHEREFORE, the petition is DISMISSED for lack of merit. SO
ORDERED.

192
G.R. No. 41570 September 6, 1934

RED LINE TRANSPORTATION CO., petitioner-appellant,


vs.
RURAL TRANSIT CO., LTD., respondent-appellee.

L. D. Lockwood for appellant.


Ohnick and Opisso for appellee.

BUTTE, J.:

193
This case is before us on a petition for review of an order of the Public corporation, Rural Transit Company, Ltd. Said motion for reconsideration was
Service Commission entered December 21, 1932, granting to the Rural set down for hearing on March 24, 1933. On March 23, 1933, the Rural
Transit Company, Ltd., a certificate of public convenience to operate a Transit Company, Ltd., the applicant, filed a motion for postponement. This
transportation service between Ilagan in the Province of Isabela and motion was verified by M. Olsen who swears "that he was the secretary of
Tuguegarao in the Province of Cagayan, and additional trips in its existing the Rural Transit Company, Ltd., in the above entitled case." Upon the
express service between Manila Tuguegarao. hearing of the motion for reconsideration, the commission admitted without
objection the following documents filed in said case No. 42343 in the Court of
On June 4, 1932, the Rural Transit Company, Ltd., a Philippine corporation, First Instance of Manila for the dissolution of the Rural Transit Company, Ltd.
filed with the Public Company Service Commission an application in which it the petition for dissolution dated July 6, 1932, the decision of the said Court
is stated in substance that it is the holder of a certificate or public of First Instance of Manila, dated February 28, 1933, decreeing the
convenience to operate a passenger bus service between Manila and dissolution of the Rural Transit Company, Ltd.
Tuguegarao; that it is the only operator of direct service between said points
and the present authorized schedule of only one trip daily is not sufficient; At the trial of this case before the Public Service Commission an issue was
that it will be also to the public convenience to grant the applicant a certificate raised as to who was the real party in interest making the application,
for a new service between Tuguegarao and Ilagan. whether the Rural Transit Company, Ltd., as appeared on the face of the
application, or the Bachrach Motor Company, Inc., using name of the Rural
On July 22, 1932, the appellant, Red Line Transportation Company, filed an Transit Company, Ltd., as a trade name. The evidence given by the
opposition to the said application alleging in substance that as to the service applicant's secretary, Olsen, is certainly very dubious and confusing, as may
between Tuguegarao and Ilagan, the oppositor already holds a certificate of be seen from the following:
public convenience and is rendering adequate and satisfactory service; that
the granting of the application of the Rural Transit Company, Ltd., would not Q. Will you please answer the question whether it is the
serve public convenience but would constitute a ruinous competition for the Bachrach Motor Company operating under the trade name of the
oppositor over said route. Rural Transit Company, Limited, or whether it is the Rural Transit
Company, Limited in its own name this application was filed?
After testimony was taken, the commission, on December 21, 1932,
approved the application of the Rural Transit Company, Ltd., and ordered A. The Bachrach Motor Company is the principal stockholder.
that the certificate of public convenience applied for be "issued to the
applicant Rural Transit Company, Ltd.," with the condition, among others, that Q. Please answer the question.
"all the other terms and conditions of the various certificates of public
convenience of the herein applicant and herein incorporated are made a part ESPELETA. Objecion porque la pregunta ya ha sido contestada.
hereof."
JUEZ. Puede contestar.
On January 14, 1933, the oppositor Red Line Transportation Company filed a
motion for rehearing and reconsideration in which it called the commission's
A. I do not know what the legal construction or relationship
attention to the fact that there was pending in the Court of First Instance of
existing between the two.
Manila case N. 42343, an application for the voluntary dissolution of the

194
JUDGE. I do not know what is in your mind by not telling the real JUDGE. My question was, when you filed this application you
applicant in this case? evidently made it for the operator?

A. It is the Rural Transit Company, Ltd. A. Yes, sir.

JUDGE. As an entity by itself and not by the Bachrach Motor JUDGE. Who was that operator you had in mind?
Company?
A. According to the status of the ownership of the certificates
A. I do not know. I have not given that phase of the matter of the former Rural Transit Company, the operator was the operator
much thought, as in previous occassion had not necessitated. authorized in case No. 23217 to whom all of the assets of the former
Rural Transit Company were sold.
JUDGE. In filing this application, you filed it for the operator on that
line? Is it not! JUDGE. Bachrach Motor Company?

A. Yes, sir. A. All actions have been prosecuted in the name of the Rural
Transit Company, Ltd.
JUDGE. Who is that operator?
JUDGE. You mean the Bachrach Motor Company, Inc., doing
A. The Rural Transit Company, Ltd. business under the name of the Rural Transit Company, Ltd.?

JUDGE. By itself, or as a commercial name of the Bachrach Motor A. Yes, sir.


Company?
LOCKWOOD. I move that this case be dismissed, your Honor, on
A. I cannot say. the ground that this application was made in the name of one party
but the real owner is another party.
ESPELETA. The Rural Transit Company, Ltd., is a corporation duly
established in accordance with the laws of the Philippine Islands. ESPELETA. We object to that petition.

JUDGE. According to the records of this commission the Bachrach JUDGE. I will have that in mind when I decide the case. If I agree
Motor Company is the owner of the certificates and the Rural Transit with you everything would be finished.
Company, Ltd., is operating without any certificate.
The Bachrach Motor Company, Inc., entered no appearance and ostensibly
JUDGE. If you filed this application for the Rural Transit Company, took no part in the hearing of the application of the Rural Transit Company,
Ltd., and afterwards it is found out that the Rural Transit Company, Ltd. It may be a matter of some surprise that the commission did not on its
Ltd., is not an operator, everything will be turned down. own motion order the amendment of the application by substituting the

195
Bachrach Motor Company, Inc., as the applicant. However, the hearing The order of the commission of November 26, 1932, authorizing the
proceeded on the application as filed and the decision of December 2, 1932, Bachrach Motor Co., Incorporated, to assume the name of the Rural Transit
was rendered in favor of the Rural Transit Company, Ltd., and the certificate Co., Ltd. likewise in corporated, as its trade name being void, and accepting
ordered to be issued in its name, in the face of the evidence that the said the order of December 21, 1932, at its face as granting a certificate of public
corporation was not the real party in interest. In its said decision, the convenience to the applicant Rural Transit Co., Ltd., the said order last
commission undertook to meet the objection by referring to its resolution of mentioned is set aside and vacated on the ground that the Rural Transit
November 26, 1932, entered in another case. This resolution in case No. Company, Ltd., is not the real party in interest and its application was
23217 concludes as follows: fictitious.

Premises considered we hereby authorize the Bachrach Motor Co., In view of the dissolution of the Rural Transit Company, Ltd. by judicial
Inc., to continue using the name of "Rural Transit Co., Ltd.," as its decree of February 28, 1933, we do not see how we can assess costs
trade name in all the applications, motions or other petitions to be against said respondent, Rural Transit Company, Ltd.
filed in this commission in connection with said business and that this
authority is given retroactive effect as of the date, of filing of the Malcolm, Villa-Real, Imperial and Goddard, JJ., concur.
application in this case, to wit, April 29, 1930.

We know of no law that empowers the Public Service Commission or any


court in this jurisdiction to authorize one corporation to assume the name of
another corporation as a trade name. Both the Rural Transit Company, Ltd.,
and the Bachrach Motor Co., Inc., are Philippine corporations and the very
law of their creation and continued existence requires each to adopt and
certify a distinctive name. The incorporators "constitute a body politic and
corporate under the name stated in the certificate." (Section 11, Act No.
1459, as amended.) A corporation has the power "of succession by its
corporate name." (Section 13, ibid.) The name of a corporation is therefore
essential to its existence. It cannot change its name except in the manner
provided by the statute. By that name alone is it authorized to transact
business. The law gives a corporation no express or implied authority to
assume another name that is unappropriated: still less that of another
corporation, which is expressly set apart for it and protected by the law. If any
corporation could assume at pleasure as an unregistered trade name the
name of another corporation, this practice would result in confusion and open
the door to frauds and evasions and difficulties of administration and
supervision. The policy of the law expressed in our corporation statute and
the Code of Commerce is clearly against such a practice. (Cf. Scarsdale
Pub. Co. Colonial Press vs. Carter, 116 New York Supplement, 731; Svenska
Nat. F. i. C. vs. Swedish Nat. Assn., 205 Illinois [Appellate Courts], 428, 434.)

196
197
G.R. No. L-26370 July 31, 1970

PHILIPPINE FIRST INSURANCE COMPANY, INC., plaintiff-appellant,


vs.
MARIA CARMEN HARTIGAN, CGH, and O. ENGKEE, defendants-
appellees.

198
Bausa, Ampil & Suarez for plaintiff-appellant. Banking Corporation payable within 30 days after the date of
the promissory note with the usual banking interest; that the
Nicasio E. Martin for defendants-appellees. plaintiff agreed to act as such co-maker of the promissory
note upon the application of the defendant Maria Carmen
Hartigan, CGH, who together with Antonio F. Chua and
Chang Ka Fu, signed an indemnity agreement in favor of the
plaintiff, undertaking jointly and severally, to pay the plaintiff
BARREDO, J.:
damages, losses or expenses of whatever kind or nature,
including attorney's fees and legal costs, which the plaintiff
Appeal from the decision dated 6 October 1962 of the Court of First Instance may sustain as a result of the execution by the plaintiff and
of Manila dismissing the action in its Civil Case No. 48925 brought by co-maker of Maria Carmen Hartigan, CGH, of the promissory
the herein plaintiff-appellant Philippine First Insurance Co., Inc. to the Court note above-referred to; that as a result of the execution of
of Appeals which could, upon finding that the said appeal raises purely the promissory note by the plaintiff and Maria Carmen
questions of law, declared itself without jurisdiction to entertain the same and, Hartigan, CGH, the China Banking Corporation delivered to
in its resolution dated 15 July 1966, certified the records thereof to this Court the defendant Maria Carmen Hartigan, CGH, the sum of
for proper determination. P5,000.00 which said defendant failed to pay in full, such
that on August 31, 1961 the same was. renewed and as of
The antecedent facts are set forth in the pertinent portions of the resolution of November 27, 1961 there was due on account of the
the Court of Appeals referred to as follows: promissory note the sum of P4,559.50 including interest. The
complaint ends with a prayer for judgment against the
According to the complaint, plaintiff was originally organized defendants, jointly and severally, for the sum of P4,559.50
as an insurance corporation under the name of 'The Yek with interest at the rate of 12% per annum from November
Tong Lin Fire and Marine Insurance Co., Ltd.' The articles of 23, 1961 plus P911.90 by way of attorney's fees and costs.
incorporation originally presented before the Security and
Exchange Commissioner and acknowledged before Notary Although O. Engkee was made as party defendant in the
Public Mr. E. D. Ignacio on June 1, 1953 state that the name caption of the complaint, his name is not mentioned in the
of the corporation was 'The Yek Tong Lin Fire and Marine body of said complaint. However, his name Appears in the
Insurance Co., Ltd.' On May 26, 1961 the articles of Annex A attached to the complaint which is the counter
incorporation were amended pursuant to a certificate of the indemnity agreement supposed to have been signed
Board of Directors dated March 8, 1961 changing the name according to the complaint by Maria Carmen Hartigan, CGH,
of the corporation to 'Philippine First Insurance Co., Inc.'. Antonio F. Chua and Chang Ka Fu.

The complaint alleges that the plaintiff Philippine First In their answer the defendants deny the allegation that the
Insurance Co., Inc., doing business under the name of 'The plaintiff formerly conducted business under the name and
Yek Tong Lin Fire and Marine Insurance Co., Lt.' signed as style of 'The Yek Tong Lin Fire and Marine Insurance Co.,
co-maker together with defendant Maria Carmen Hartigan, Ltd.' They admit the execution of the indemnity agreement
CGH, a promissory note for P5,000.00 in favor of the China but they claim that they signed said agreement in favor of

199
the Yek Tong Lin Fire and Marine Insurance Co., Ltd.' and defendants and the Yek Tong Lin Fire & Marine Insurance
not in favor of the plaintiff. They likewise admit that they Co., Ltd. (Annex A to plaintiff's complaint ); and
failed to pay the promissory note when it fell due but they
allege that since their obligation with the China Banking (b) Whether or not a suit for indemnity or reimbursement
Corporation based on the promissory note still subsists, the may under said indemnity agreement prosper without plaintiff
surety who co-signed the promissory note is not entitled to having yet paid the amount due under said promissory note.
collect the value thereof from the defendants otherwise they
will be liable for double amount of their obligation, there In the first place, the change of name of the Yek Tong Lin
being no allegation that the surety has paid the obligation to Fire & Marine Insurance Co., Ltd. to the Philippines First
the creditor. Insurance Co., Inc. is of dubious validity. Such change of
name in effect dissolved the original corporation by a
By way of special defense, defendants claim that there is no process of dissolution not authorized by our corporation law
privity of contract between the plaintiff and the defendants (see Secs. 62 and 67, inclusive, of our Corporation Law).
and consequently, the plaintiff has no cause of action against Moreover, said change of name, amounting to a dissolution
them, considering that the complaint does not allege that the of the Yek Tong Lin Fire & Marine Insurance Co., Ltd., does
plaintiff and the 'Yek Tong Lin Fire and Marine Insurance Co., not appear to have been effected with the written note or
Ltd.' are one and the same or that the plaintiff has acquired assent of stockholders representing at least two-thirds of the
the rights of the latter. The parties after the admission of subscribed capital stock of the corporation, a voting
Exhibit A which is the amended articles of incorporation and proportion required not only for the dissolution of a
Exhibit 1 which is a demand letter dated August 16, 1962 corporation but also for any amendment of its articles of
signed by the manager of the loans and discount department incorporation (Secs. 18 and 62, Corporation Law).
of the China Banking Corporation showing that the Furthermore, such change of corporate name appears to be
promissory note up to said date in the sum of P4,500.00 was against public policy and may be effected only by express
still unpaid, submitted the case for decision based on the authority of law (Red Line Transportation Co. v. Rural Transit
pleadings. Co., Ltd., 60 Phil. 549, 555; Cincinnati Cooperage Co., Ltd.
vs. Vate, 26 SW 538, 539; Pilsen Brewing Co. vs. Wallace,
Under date of 6 October 1962, the Court of First Instance of Manila rendered 125 NE 714), but there is nothing in our corporation law
the decision appealed. It dismissed the action with costs against the plaintiff authorizing the change of corporate name in this jurisdiction.
Philippine First Insurance Co., Inc., reasoning as follows:
In the second place, assuming that the change of name of
... With these undisputed facts in mind, the parties correctly the Yek Tong Lin Fire & Marine Insurance Co. Ltd., to
concluded that the issues for resolution by this Court are as Philippines pine First Insurance Co., Inc., as accomplished
follows: on March 8, 1961, is valid, that would mean that the original
corporation, the Yek Tong Lin Fire & Marine Insurance Co.,
(a) Whether or not the plaintiff is the real party in interest that Ltd., became dissolved and of no further existence since
may validly sue on the indemnity agreement signed by the March 8, 1961, so that on May 15, 1961, the date the
indemnity agreement, Annex A, was executed, said original

200
corporation bad no more power to enter into any agreement In due time, the Philippine First Insurance Company, Inc. moved for
with the defendants, and the agreement entered into by it reconsideration of the decision aforesaid, but said motion was denied on
was ineffective for lack of capacity of said dissolved December 3, 1962 in an order worded thus:
corporation to enter into said agreement. At any rate, even if
we hold that said change of name is valid, the fact remains The motion for reconsideration, dated November 8, 1962,
that there is no evidence showing that the new entity, the raises no new issue that we failed to consider in rendering
Philippine First Insurance Co., Inc. has with the consent of our decision of October 6, 1962. However, it gives us an
the original parties, assumed the obligations or was opportunity to amplify our decision as regards the question
assigned the rights of action in the original corporation, the of change of name of a corporation in this jurisdiction.
Yek Tong Lin Fire & Marine Insurance Co., Ltd. In other
words, there is no evidence of conventional subrogation of We find nothing in our Corporation Law authorizing a change
the Plaintiffs in the rights of the Yek Tong Lin Fire & Marine of name of a corporation organized pursuant to its
Insurance Co., Ltd. under said indemnity agreement (Arts. provisions. Sec. 18 of the Corporation Law authorizes, in our
1300, 1301, New Civil Code). without such subrogation opinion, amendment to the Articles of Incorporation of a
assignment of rights, the herein plaintiff has no cause of corporation only as to matters other than its corporate name.
action against the defendants, and is, therefore, not the right Once a corporation is organized in this jurisdiction by the
party in interest as plaintiff. execution and registration of its Articles of Incorporation, it
shall continue to exist under its corporate name for the
Last, but not least, assuming that the said change of name lifetime of its corporate existence fixed in its Articles of
was legal and operated to dissolve the original corporation, Incorporation, unless sooner legally dissolved (Sec. 11,
the dissolved corporation, must pursuant to Sec. 77 of our Corp. Law). Significantly, change of name is not one of the
corporation law, be deemed as continuing as a body methods of dissolution of corporations expressly authorized
corporate for three (3) years from March 8, 1961 for the by our Corporation Law. Also significant is the fact that the
purpose of prosecuting and defending suits. It is, therefore, power to change its corporate name is not one of the general
the Yek Tong Lin Fire & Marine Insurance Co., Ltd. that is the powers conferred on corporations in this jurisdiction (Sec.
proper party to sue the defendants under said indemnity 13, Corp. Law). The enumeration of corporate powers made
agreement up to March 8, 1964. in our Corporation Law implies the exclusion of all others
(Thomas v. West Jersey R. Co., 101 U.S. 71, 25 L. ed. 950).
Having arrived at the foregoing conclusions, this Court need It is obvious, in this connection, that change of name is not
not squarely pass upon issue (b) formulated above. one of the powers necessary to the exercise of the powers
conferred on corporations by said Sec. 13 (see Sec. 14,
WHEREFORE, plaintiff's action is hereby dismissed, with Corp. Law).
costs against the plaintiff.
To rule that Sec. 18 of our Corporation Law authorizes the
change of name of a corporation by amendment of its
Articles of Incorporation is to indulge in judicial legislation.
We have examined the cases cited in Volume 13 of

201
American Jurisprudence in support of the proposition that We find no justification, legal, moral, or practical, for
the general power to alter or amend the charter of a adhering to the view taken by the Securities and Exchange
corporation necessarily includes the power to alter the name Commissioner that the name of a corporation in the
of a corporation, and find no justification for said conclusion Philippines may be changed by mere amendment of its
arrived at by the editors of American Jurisprudence. On the Articles of Incorporation as to its corporate name. A change
contrary, the annotations in favor of plaintiff's view appear to of corporate name would serve no useful purpose, but on the
have been based on decisions in cases where the statute contrary would most probably cause confusion. Only a
itself expressly authorizes change of corporate name by dubious purpose could inspire a change of a corporate.
amendment of its Articles of Incorporation. The correct rule in name which, unlike a natural person's name, was chosen by
harmony with the provisions of our Corporation Law is well the incorporators themselves; and our Courts should not
expressed in an English case as follows: lend their assistance to the accomplishment of dubious
purposes.
After a company has been completely
register without defect or omission, so as to WHEREFORE, we hereby deny plaintiff's motion for
be incorporated by the name set forth in the reconsideration, dated November 8, 1962, for lack of merit.
deed of settlement, such incorporated
company has not the power to change its In this appeal appellant contends that
name ... Although the King by his
prerogative might incorporate by a new I
name, and the newly named corporation
might retain former rights, and sometimes its
THE TRIAL COURT ERRED IN HOLDING THAT IN THIS
former name also, ... it never appears to be
JURISDICTION, THERE IS NOTHING IN OUR
such an act as the corporation could do by
CORPORATION LAW AUTHORIZING THE CHANGE OF
itself, but required the same power as
CORPORATE NAME;
created the corporation. (Reg. v. Registrar of
Joint Stock Cos 10 Q.B. 839, 59 E.C.L.
839). II

The contrary view appears to represent the minority doctrine, THE TRIAL COURT ERRED IN DECLARING THAT A
judging from the annotations on decided cases on the CHANGE OF CORPORATE NAME APPEARS TO BE
matter. AGAINST PUBLIC POLICY;

The movant invokes as persuasive precedent the action of III


the Securities Commissioner in tacitly approving the
Amended, Articles of Incorporation on May 26, 1961. We THE TRIAL COURT ERRED IN HOLDING THAT A CHANGE
regret that we cannot in good conscience lend approval to OF CORPORATE NAME HAS THE LEGAL EFFECT OF
this action of the Securities and Exchange Commissioner. DISSOLVING THE ORIGINAL CORPORATION:

202
IV Sec. 18. Any corporation may for legitimate corporate
purpose or purposes, amend its articles of incorporation by a
THE TRIAL COURT ERRED IN HOLDING THAT THE majority vote of its board of directors or trustees and the vote
CHANGE OF NAME OF THE YEK TONG LIN FIRE & or written assent of two-thirds of its members, if it be a
MARINE INSURANCE CO., LTD. IS OF DUBIOUS nonstock corporation or, if it be a stock corporation, by the
VALIDITY; vote or written assent of the stockholders representing at
least two-thirds of the subscribed capital stock of the
V corporation Provided, however, That if such amendment to
the articles of incorporation should consist in extending the
corporate existence or in any change in the rights of holders
THE TRIAL COURT ERRED IN HOLDING THAT THE
of shares of any class, or would authorize shares with
APPELLANT HEREIN IS NOT THE RIGHT PARTY
preferences in any respect superior to those of outstanding
INTEREST TO SUE DEFENDANTS-APPELLEES;
shares of any class, or would restrict the rights of any
stockholder, then any stockholder who did not vote for such
IV corporate action may, within forty days after the date upon
which such action was authorized, object thereto in writing
THE TRIAL COURT FINALLY ERRED IN DISMISSING THE and demand Payment for his shares. If, after such a demand
COMPLAINT. by a stockholder, the corporation and the stockholder cannot
agree upon the value of his share or shares at the time such
Appellant's Position is correct; all the above assignments of error are well corporate action was authorized, such values all be
taken. The whole case, however, revolves around only one question. May a ascertained by three disinterested persons, one of whom
Philippine corporation change its name and still retain its original personality shall be named by the stockholder, another by the
and individuality as such? corporation, and the third by the two thus chosen. The
findings of the appraisers shall be final, and if their award is
The answer is not difficult to find. True, under Section 6 of the Corporation not paid by the corporation within thirty days after it is made,
Law, the first thing required to be stated in the Articles of Incorporation of any it may be recovered in an action by the stockholder against
corn corporation is its name, but it is only one among many matters equally if the corporation. Upon payment by the corporation to the
not more important, that must be stated therein. Thus, it is also required, for stockholder of the agreed or awarded price of his share or
example, to state the number and names of and residences of the shares, the stockholder shall forthwith transfer and assign
incorporators and the residence or location of the principal office of the the share or shares held by him as directed by the
corporation, its term of existence, the amount of its capital stock and the corporation: Provided, however, That their own shares of
number of shares into which it is divided, etc., etc. stock purchased or otherwise acquired by banks, trust
companies, and insurance companies, should be disposed
On the other hand, Section 18 explicitly permits the articles of incorporation of within six months after acquiring title thereto.
to be amended thus:
Unless and until such amendment to the articles of
incorporation shall have been abandoned or the action

203
rescinded, the stockholder making such demand in writing The Securities and, Exchange Commissioner shall be entitled to collect and
shall cease to be a stockholder and shall have no rights with receive the sum of ten pesos for filing said copy of the amended articles of
respect to such shares, except the right to receive payment incorporation. Provided, however, That when the amendment consists in
therefor as aforesaid. extending the term of corporate existence, the Securities and Exchange
Commissioner shall be entitled to collect and receive for the filing of its
A stockholder shall not be entitled to payment for his shares amended articles of incorporation the same fees collectible under existing
under the provisions of this section unless the value of the law for the filing of articles of incorporation. The Securities & Exchange
corporate assets which would remain after such payment Commissioner shall not hereafter file any amendment to the articles of
would be at least equal to the aggregate amount of its debts incorporation of any bank, banking institution, or building and loan
and liabilities and the aggregate par value and/or issued association unless accompanied by a certificate of the Monetary Board (of
value of the remaining subscribed capital stock. the Central Bank) to the effect that such amendment is in accordance with
law. (As further amended by Act No. 3610, Sec. 2 and Sec. 9. R.A. No. 337
A copy of the articles of incorporation as amended, duly and R.A. No. 3531.)
certified to be correct by the president and the secretary of
the corporation and a majority of the board of directors or It can be gleaned at once that this section does not only authorize
trustees, shall be filed with the Securities and Exchange corporations to amend their charter; it also lays down the procedure for such
Commissioner, who shall attach the same to the original amendment; and, what is more relevant to the present discussion, it contains
articles of incorporation, on file in his office. From the time of provisos restricting the power to amend when it comes to the term of their
filing such copy of the amended articles of incorporation, the existence and the increase or decrease of the capital stock. There is no
corporation shall have the same powers and it and the prohibition therein against the change of name. The inference is clear that
members and stockholders thereof shall thereafter be such a change is allowed, for if the legislature had intended to enjoin
subject to the same liabilities as if such amendment had corporations from changing names, it would have expressly stated so in this
been embraced in the original articles of section or in any other provision of the law.
incorporation: Provided, however, That should the
amendment consist in extending the corporate life, the No doubt, "(the) name (of a corporation) is peculiarly important as necessary
extension shall not exceed 50 years in any one to the very existence of a corporation. The general rule as to corporations is
instance. Provided, further, That the original articles and that each corporation shall have a name by which it is to sue and be sued
amended articles together shall contain all provisions and do all legal acts. The name of a corporation in this respect designates
required by law to be set out in the articles of the corporation in the same manner as the name of an individual designates
incorporation: And provided, further, That nothing in this the person." 1 Since an individual has the right to change his name under
section shall be construed to authorize any corporation to certain conditions, there is no compelling reason why a corporation may not
increase or diminish its capital stock or so as to effect any enjoy the same right. There is nothing sacrosanct in a name when it comes
rights or actions which accrued to others between the time of to artificial beings. The sentimental considerations which individuals attach to
filing the original articles of incorporation and the filing of the their names are not present in corporations and partnerships. Of course, as
amended articles. in the case of an individual, such change may not be made exclusively. by
the corporation's own act. It has to follow the procedure prescribed by law for

204
the purpose; and this is what is important and indispensably prescribed necessary implication, they authorize corporations new
strict adherence to such procedure. names and prescribe the mode of procedure for that
purpose. The same steps must be taken under some
Local well known corporation law commentators are unanimous in the view statutes to effect a change in a corporate name, as when
that a corporation may change its name by merely amending its charter in any other amendment of the corporate charter is sought ....
the manner prescribed by law. 2 American authorities which have persuasive When the general law thus deals with the subject, a
force here in this regard because our corporation law is of American origin, corporation can change its name only in the manner
the same being a sort of codification of American corporate law, 3 are of the provided. (6 Fletcher, Cyclopedia of the Law of Private
same opinion. Corporations, 1968 Revised Volume, pp. 212-213.)
(Emphasis supplied)
A general power to alter or amend the charter of a
corporation necessarily includes the power to alter the name The learned trial judge held that the above-quoted proposition are not
of the corporation. Ft. Pitt Bldg., etc., Assoc. v. Model Plan supported by the weight of authority because they are based on decisions in
Bldg., etc., Assoc., 159 Pa. St. 308, 28 Atl. 215; In re Fidelity cases where the statutes expressly authorize change of corporate name by
Mut. Aid Assoc., 12 W.N.C. (Pa.) 271; Excelsior Oil Co., 3 amendment of the articles of incorporation. We have carefully examined
Pa. Co. Ct. 184; Wetherill Steel Casting Co., 5 Pa. Co. Ct. these authorities and We are satisfied of their relevance. Even Lord Denman
337. who has been quoted by His Honor from In Reg. v. Registrar of Joint Stock
Cos. 10, Q.B., 59 E.C.L. maintains merely that the change of its name never
xxx xxx xxx appears to be such an act as the corporation could do for itself, but
required ;the same Power as created a corporation." What seems to have
been overlooked, therefore, is that the procedure prescribes by Section 18 of
Under the General Laws of Rhode Island, c 176, sec. 7,
our Corporation Law for the amendment of corporate charters is practically
relating to an increase of the capital stock of a corporation, it
identical with that for the incorporation itself of a corporation.
is provided that 'such agreement may be amended in any
other particular, excepting as provided in the following
section', which relates to a decrease of the capital stock This In the appealed order of dismissal, the trial court, made the observation that,
section has been held to authorize a change in the name of according to this Court in Red Line Transportation Co. v. Rural Transit Co.,
a corporation. Armington v. Palmer, 21 R.I. 109, 42 Atl. 308, Ltd., 60 Phil, 549, 555, change of name of a corporation is against public
43, L.R.A. 95, 79 Am. St. Rep. 786. (Vol. 19, American and policy. We must clarify that such is not the import of Our said decision. What
English Annotated Cases, p. 1239.) this Court held in that case is simply that:

Fletcher, a standard authority on American an corporation law also says: We know of no law that empowers the Public Service
Commission or any court in this jurisdiction to authorize one
corporation to assume the name of another corporation as a
Statutes are to be found in the various jurisdictions dealing
trade name. Both the Rural Transit Company, Ltd., and the
with the matter of change in corporate names. Such statutes
Bachrach Motor Co., Inc., are Philippine corporations and
have been subjected to judicial construction and have, in the
the very law of their creation and continued existence
main, been upheld as constitutional. In direct terms or by
requires each to adopt and certify a distinctive name. The

205
incorporators 'constitute a body politic and corporate under name it should sue and be sued in its new name .... (13 Am.
the name stated in the certificate.' (Section 11, Act No. 1459, Jur. 276-277, citing cases.)
as amended.) A corporation has the power 'of succession by
its corporate name.' (Section 13, ibid.) The name of a A mere change in the name of a corporation, either by the
corporation is therefore essential to its existence. It cannot legislature or by the corporators or stockholders under
change its name except in the manner provided by the legislative authority, does not, generally speaking, affect the
statute. By that name alone is it authorized to transact identity of the corporation, nor in any way affect the rights,
business. The law gives a corporation no express or implied privileges, or obligations previously acquired or incurred by
authority to assume another name that is unappropriated; it. Indeed, it has been said that a change of name by a
still less that of another corporation, which is expressly set corporation has no more effect upon the identity of the
apart for it and protected by the law. If any corporation could corporation than a change of name by a natural person has
assume at pleasure as an unregistered trade name the upon the identity of such person. The corporation, upon such
name of another corporation, this practice would result in change in its name, is in no sense a new corporation, nor the
confusion and open the door to frauds and evasions and successor of the original one, but remains and continues to
difficulties of administration and supervision. The policy of be the original corporation. It is the same corporation with a
the law as expressed our corporation statute and the Code different name, and its character is in no respect changed. ...
of Commerce is clearly against such a practice. (Cf. (6 Fletcher, Cyclopedia of the Law of Private Corporations,
Scarsdale Pub. Co. Colonial Press vs. Carter, 116 New 224-225, citing cases.)
York Supplement, 731; Svenska Nat. F. i. C. vs. Swedish
Nat. Assn., 205 Illinois [Appellate Courts], 428, 434.) The change in the name of a corporation has no more effect
upon its identity as a corporation than a change of name of a
In other words, what We have held to be contrary to public policy is the use natural person has upon his identity. It does not affect the
by one corporation of the name of another corporation as its trade name. We rights of the corporation, or lessen or add to its obligations.
are certain no one will disagree that such an act can only "result in confusion
and open the door to frauds and evasions and difficulties of administration England. Doe v. Norton, 11 M. & W. 913, 7 Jur. 751, 12 L.
and supervision." Surely, the Red Line case was not one of change of name. J. Exch. 418.

Neither can We share the posture of His Honor that the change of name of a United States. Metropolitan Nat. Bank v. Claggett, 141
corporation results in its dissolution. There is unanimity of authorities to the U.S. 520, 12 S. Ct. 60, 35 U.S. (L. ed.) 841.
contrary.
Alabama. Lomb v. Pioneer Sav., etc., Co., 106 Ala. 591,
An authorized change in the name of a corporation has no 17 So. 670; North Birmingham Lumber Co. v. Sims, 157 Ala.
more effect upon its identity as a corporation than a change 595, 48 So. 84.
of name of a natural person has upon his identity. It does not
affect the rights of the corporation or lessen or add to its
Connecticut. Trinity Church v. Hall, 22 Com. 125.
obligations. After a corporation has effected a change in its

206
Illinois. Mt. Palatine Academy v. Kleinschnitz 28 III, The fact that the corporation by its old name makes a format
133; St. Louis etc. R. Co. v. Miller, 43 Ill. 199; Reading v. transfer of its property to the corporation by its new name
Wedder, 66 III. 80. does not of itself show that the change in name has affected
a change in the identity of the corporation. Palfrey v.
Indiana. Rosenthal v. Madison etc., Plank Road Co., 10 Association for Relief, etc., 110 La. 452, 34 So. 600. The fact
Ind. 358. that a corporation organized as a state bank afterwards
becomes a national bank by complying with the provisions of
Kentucky. Cahill v. Bigger, 8 B. Mon. 211; Wilhite v. the National Banking Act, and changes its name accordingly,
Convent of Good Shepherd, 177 Ky. 251, 78 S. W. 138. has no effect on its right to sue upon obligations or liabilities
incurred to it by its former name. Michigan Ins. Bank v.
Eldred 143 U.S. 293, 12 S. Ct. 450, 36 U.S. (L. ed.) 162.
Maryland. Phinney v. Sheppard & Enoch Pratt Hospital,
88 Md. 633, 42 Atl. 58, writ of error dismissed, 177 U.S. 170,
20 S. Ct. 573, 44 U.S. (L. ed.) 720. A deed of land to a church by a particular name has been
held not to be affected by the fact that the church afterwards
took a different name. Cahill v. Bigger, 8 B. Mon (ky) 211.
Missouri. Dean v. La Motte Lead Co., 59 Mo. 523.

A change in the name of a corporation is not a divestiture of


Nebraska. Carlon v. City Sav. Bank, 82 Neb. 582, 188 N.
title or such a change as requires a regular transfer of title to
W. 334. New York First Soc of M.E. Church v. Brownell, 5
property, whether real or personal, from the corporation
Hun 464.
under one name to the same corporation under another
name. McCloskey v. Doherty, 97 Ky. 300, 30 S. W. 649. (19
Pennsylvania. Com. v. Pittsburgh, 41 Pa. St. 278. American and English Annotated Cases 1242-1243.)

South Carolina. South Carolina Mut Ins. Co. v. Price 67 As was very aptly said in Pacific Bank v. De Ro 37 Cal. 538,
S.C. 207, 45 S.E. 173. "The changing of the name of a corporation is no more the
creation of a corporation than the changing of the name of a
Virginia. Wilson v. Chesapeake etc., R. Co., 21 Gratt natural person is the begetting of a natural person. The act,
654; Wright-Caesar Tobacco Co. v. Hoen,105 Va. 327, 54 in both cases, would seem to be what the language which
S.E. 309. we use to designate it imports a change of name, and not
a change of being.
Washington. King v. Ilwaco R. etc., Co., 1 Wash. 127. 23
Pac. 924. Having arrived at the above conclusion, We have agree with appellant's pose
that the lower court also erred in holding that it is not the right party in interest
Wisconsin. Racine Country Bank v. Ayers, 12 Wis. 512. to sue defendants-appellees. 4 As correctly pointed out by appellant, the
approval by the stockholders of the amendment of its articles of incorporation
changing the name "The Yek Tong Lin Fire & Marine Insurance Co., Ltd." to

207
"Philippine First Insurance Co., Inc." on March 8, 1961, did not automatically Nowhere in the record is it seriously pretended that the indebtedness sued
change the name of said corporation on that date. To be effective, Section 18 upon has already been paid. If appellees entertained any fear that they might
of the Corporation Law, earlier quoted, requires that "a copy of the articles of again be made liable to Yek Tong Lin Fire & Marine Insurance Co. Ltd., or to
incorporation as amended, duly certified to be correct by the president and someone else in its behalf, a cursory examination of the records of the
the secretary of the corporation and a majority of the board of directors or Securities & Exchange Commission would have sufficed to clear up the fact
trustees, shall be filed with the Securities & Exchange Commissioner", and it that Yek Tong Lin had just changed its name but it had not ceased to be their
is only from the time of such filing, that "the corporation shall have the same creditor. Everyone should realize that when the time of the courts is utilized
powers and it and the members and stockholders thereof shall thereafter be for cases which do not involve substantial questions and the claim of one of
subject to the same liabilities as if such amendment had been embraced in the parties, therein is based on pure technicality that can at most delay only
the original articles of incorporation." It goes without saying then that the ultimate outcome necessarily adverse to such party because it has no
appellant rightly acted in its old name when on May 15, 1961, it entered into real cause on the merits, grave injustice is committed to numberless litigants
the indemnity agreement, Annex A, with the defendant-appellees; for only whose meritorious cases cannot be given all the needed time by the courts.
after the filing of the amended articles of incorporation with the Securities & We address this appeal once more to all members of the bar, in particular,
Exchange Commission on May 26, 1961, did appellant legally acquire its since it is their bounden duty to the profession and to our country and people
new name; and it was perfectly right for it to file the present case In that new at large to help ease as fast as possible the clogged dockets of the courts.
name on December 6, 1961. Such is, but the logical effect of the change of Let us not wait until the people resort to other means to secure speedy, just
name of the corporation upon its actions. and inexpensive determination of their cases.

Actions brought by a corporation after it has changed its WHEREFORE, judgment of the lower court is reversed, and this case is
name should be brought under the new name although for remanded to the trial court for further proceedings consistent herewith With
the enforcement of rights existing at the time the change was costs against appellees.
made. Lomb v. Pioneer Sav., etc., Co., 106 Ala. 591, 17 So.
670: Newlan v. Lombard University, 62 III. 195; Thomas v. Concepcion, C.J., Reyes, J.B.L., Dizon, Makalintal, Zaldivar, Castro,
Visitor of Frederick County School, 7 Gill & J (Md.) Fernando, Teehankee and Villamor, JJ., concur.
388; Delaware, etc., R. Co. v. Trick, 23 N. J. L.
321; Northumberland Country Bank v. Eyer, 60 Pa. St.
436; Wilson v. Chesapeake etc., R. Co., 21 Gratt (Va.) 654.

The change in the name of the corporation does not affect its
right to bring an action on a note given to the corporation
under its former name. Cumberland College v. Ish, 22. Cal.
641; Northwestern College v. Schwagler, 37 Ia. 577. (19
American and English Annotated Cases 1243.)

In consequence, We hold that the lower court erred in dismissing appellant's


complaint. We take this opportunity, however, to express the Court's feeling
that it is apparent that appellee's position is more technical than otherwise.

208
G.R. No. L-28351 July 28, 1977

UNIVERSAL MILLS CORPORATION, petitioner,


vs.
UNIVERSAL TEXTILE MILLS, INC., respondent.

Emigdio G. Tanjuatco for petitioner.

Picazo, Santayana, Reyes, Tayao & Alfonso for respondent.

BARREDO, J.:

Appeal from the order of the Securities and Exchange Commission in S.E.C.
Case No. 1079, entitled In the Matter of the Universal Textile Mills, Inc. vs.
Universal Mills Corporation, a petition to have appellant change its corporate
name on the ground that such name is "confusingly and deceptively similar"
to that of appellee, which petition the Commission granted.

209
According to the order, "the Universal Textile Mills, Inc. was organ on From the facts proved and the jurisprudence on the matter, it
December 29, 1953, as a textile manufacturing firm for which it was issued a appears necessary under the circumstances to enjoin the
certificate of registration on January 8, 1954. The Universal Mills respondent Universal Mills Corporation from further using its
Corporation, on the other hand, was registered in this Commission on present corporate name. Judging from what has already
October 27, 1954, under its original name, Universal Hosiery Mills happened, confusion is not only apparent, but possible. It
Corporation, having as its primary purpose the "manufacture and production does not matter that the instance of confusion between the
of hosieries and wearing apparel of all kinds." On May 24, 1963, it filed an two corporate names was occasioned only by a fire or an
amendment to its articles of incorporation changing its name to Universal extraordinary occurrence. It is precisely the duty of this
Mills Corporation, its present name, for which this Commission issued the Commission to prevent such confusion at all times and
certificate of approval on June 10, 1963. under all circumstances not only for the purpose of
protecting the corporations involved but more so for the
The immediate cause of this present complaint, however, was the occurrence protection of the public.
of a fire which gutted respondent's spinning mills in Pasig, Rizal. Petitioner
alleged that as a result of this fire and because of the similarity of In today's modern business life where people go by
respondent's name to that of herein complainant, the news items appearing tradenames and corporate images, the corporate name
in the various metropolitan newspapers carrying reports on the fire created becomes the more important. This Commission cannot close
uncertainty and confusion among its bankers, friends, stockholders and its eyes to the fact that usually it is the sound of all the other
customers prompting petitioner to make announcements, clarifying the real words composing the names of business corporations that
Identity of the corporation whose property was burned. Petitioner presented sticks to the mind of those who deal with them. The word
documentary and testimonial evidence in support of this allegation. "textile" in Universal Textile Mills, Inc.' can not possibly
assure the exclusion of all other entities with similar names
On the other hand, respondent's position is that the names from the mind of the public especially so, if the business they
of the two corporations are not similar and even if there be are engaged in are the same, like in the instant case.
some similarity, it is not confusing or deceptive; that the only
reason that respondent changed its name was because it This Commission further takes cognizance of the fact that
expanded its business to include the manufacture of fabrics when respondent filed the amendment changing its name to
of all kinds; and that the word 'textile' in petitioner's name is Universal Mills Corporation, it correspondingly filed a written
dominant and prominent enough to distinguish the two. It undertaking dated June 5, 1963 and signed by its President,
further argues that petitioner failed to present evidence of Mr. Mariano Cokiat, promising to change its name in the
confusion or deception in the ordinary course of business; event that there is another person, firm or entity who has
that the only supposed confusion proved by complainant obtained a prior right to the use of such name or one similar
arose out of an extraordinary occurrence a disastrous fire. to it. That promise is still binding upon the corporation and its
(pp. 16-&17, Record.) responsible officers. (pp. 17-18, Record.)

Upon these premises, the Commission held: It is obvious that the matter at issue is within the competence of the
Securities and Exchange Commission to resolve in the first instance in the
exercise of the jurisdiction it used to possess under Commonwealth Act 287

210
as amended by Republic Act 1055 to administer the application and WHEREFORE, with the reservation already mentioned, the appealed
enforcement of all laws affecting domestic corporations and associations, decision is affirmed. Costs against petitioners.
reserving to the courts only conflicts of judicial nature, and, of course, the
Supreme Court's authority to review the Commissions actuations in Fernando (Chairman), Antonio, Aquino, Concepcion Jr. and Santos, JJ.,
appropriate instances involving possible denial of due process and grave concur.
abuse of discretion. Thus, in the case at bar, there being no claim of denial of
any constitutional right, all that We are called upon to determine is whether or
not the order of the Commission enjoining petitioner to its corporate name
constitutes, in the light of the circumstances found by the Commission, a
grave abuse of discretion.

We believe it is not. Indeed, it cannot be said that the impugned order is


arbitrary and capricious. Clearly, it has rational basis. The corporate names in
question are not Identical, but they are indisputably so similar that even
under the test of "reasonable care and observation as the public generally
are capable of using and may be expected to exercise" invoked by appellant,
We are apprehensive confusion will usually arise, considering that under the
second amendment of its articles of incorporation on August 14, 1964,
appellant included among its primary purposes the "manufacturing, dyeing,
finishing and selling of fabrics of all kinds" in which respondent had been
engaged for more than a decade ahead of petitioner. Factually, the
Commission found existence of such confusion, and there is evidence to
support its conclusion. Since respondent is not claiming damages in this
proceeding, it is, of course, immaterial whether or not appellant has acted in
good faith, but We cannot perceive why of all names, it had to choose a
name already being used by another firm engaged in practically the same
business for more than a decade enjoying well earned patronage and
goodwill, when there are so many other appropriate names it could possibly
adopt without arousing any suspicion as to its motive and, more importantly,
any degree of confusion in the mind of the public which could mislead even
its own customers, existing or prospective. Premises considered, there is no
warrant for our interference.

As this is purely a case of injunction, and considering the time that has
elapsed since the facts complained of took place, this decision should not be
deemed as foreclosing any further remedy which appellee may have for the
protection of its interests.

211
G.R. No. L-15429 December 1, 1919

UY SIULIONG, MARIANO LIMJAP, GACU UNG JIENG, EDILBERTO


CALIXTO and UY CHO YEE, petitioners,
vs.
THE DIRECTOR OF COMMERCE AND INDUSTRY, respondent.

Kincaid and Perkins for petitioners.


Attorney-General Paredes for respondent.

JOHNSON, J.:

212
The purpose of this action is to obtain the writ of mandamus to require the (c) The purchase and sale of bills of exchange, bonds, stocks, or
respondent to file and register, upon the payment of the lawful fee, articles of "participaciones de sociedades mercantiles e industriales [joint account of
incorporation, and to issue to the petitioners as the incorporators of a certain mercantile and industrial associations]," and of all classes of mercantile
corporation to be known as "Siuliong y Compaia, Inc.," a certificate under documents; "comisiones [commissions];" "consignaciones [consignments];"
the seal of the office of said respondent, certifying that the articles of
incorporation have been duly filed and registered in his office in accordance (d) To act as agents for life, marine and fire insurance
with the law. companies; lawphi1.net

To the petition the respondent demurred and the cause was finally submitted (e) To purchase and sell boats of all classes "y fletamento de los mismos
upon the petition and demurrer. [and charterage of same];" and

The important facts necessary for the solution of the question presented, (f) To purchase and sell industrial and mercantile establishments.
which are found in the petition, may be stated as follows:
While the articles of incorporation of "Siuliong y Cia., Inc." states that its
1. That prior to the presentation of the petition the petitioners had been purpose is to acquire and continue the business, with some of its objects or
associated together as partners, which partnership was known as "mercantil purposes, of Siuliong & Co., it will be found upon an examination of the
regular colectiva, under the style and firm name of "Siuliong y Cia.;" purposes enumerated in the proposed articles of incorporation of "Siuliong y
Cia., Inc.," that some of the purposes of the original partnership of "Siuliong y
2. That the petitioners herein, who had theretofore been members of said Cia." have been omitted. For example, the articles of partnership of "Siuliong
partnership of "Siuliong y Cia.," desired to dissolve said partnership and to y Cia." gave said company the authority to purchase and sell all classes "de
form a corporation composed of the same persons as incorporators, to be fincas rusticas y urbanas [of rural and city real estate]" as well as the right to
known as "Siulong y Compaia, Incorporada;" act as agents for the establishment of any other business which it might
esteem convenient for the interests of "la compaia [the company]." (Exhibit
3. That the purpose of said corporation, "Siuliong y Cia., Inc.," is (a) to C).
acquire the business of the partnership theretofore known as Siuliong & Co.,
and (b) to continue said business with some of its objects or purposes; The respondent in his argument in support of the demurrer contends (a) that
the proposed articles of incorporation presented for file and registry permitted
4. That an examination of the articles of incorporation of the said "Siuliong y the petitioners to engage in a business which had for its end more than one
Compaia, Incorporada" (Exhibit A) shows that it is to be organized for the purpose; (b) that it permitted the petitioners to engage in the banking
following purposes: business, and (c) to deal in real estate, in violation of the Act of Congress of
July 1, 1902.
(a) The purchase and sale, importation and exportation, of the products of
the country as well as of foreign countries; The petitioners, in reply to said argument of the respondent, while insisting
that said proposed articles of incorporation do not permit it to enter into the
(b) To discount promissory notes, bills of exchange, and other negotiable banking business nor to engage in the purchase and sale of real estate in
instruments; violation of said Act of Congress, expressly renounced in open court their

213
right to engage in such business under their articles of incorporation, even organized under the laws of the Philippine Islands for mercantile purposes,
though said articles might be interpreted in a way to authorize them to so to and to engage in such incidental business as may be
do. That renouncement on the part of the petitioners eliminates from the necessary and advisable to give effect to, and aid in, the successful
purposes of said proposed corporation (of "Siuliong y Cia., Inc.") any right to operation and conduct of the principal business.1awphi1.net
engage in the banking business as such, or in the purchase and sale of real
estate. In the present case we are fully persuaded that all of the power and authority
included in the articles of incorporation of "Siuliong y Cia., Inc.," enumerated
We come now to the consideration of the principal question raised by the above in paragraph 4 (Exhibit A) are only incidental to the principal
respondent, to wit: that the proposed articles of incorporation of "Siuliong y purpose of said proposed incorporation, to wit: "mercantile business." The
Cia., Inc.," permits it to engage in a business with more than one purpose. purchase and sale, importation and exportation of the products of the
country, as well as of foreign countries, might make it necessary to purchase
If upon an examination of the articles of incorporation we find that its purpose and discount promissory notes, bills of exchange, bonds, negotiable
is to engage in a business with but one principal purpose, then that instruments, stock, and interest in other mercantile and industrial
contention of the respondent will have been answered and it will be associations. It might also become important and advisable for the
unnecessary to discuss at length the question whether or not a corporation successful operation of the corporation to act as agent for insurance
organized for commercial purposes in the Philippine Islands can be companies as well as to buy, sell and equip boats and to buy and sell other
organized for more than one purpose. establishments, and industrial and mercantile businesses.

The attorney for the respondent, at the time of the argument, admitted in While we have arrived at the conclusion that the proposed articles of
open court that corporations in the Philippine Islands might be organized for incorporation do not authorize the petitioners to engage in a business with
both the "importation and exportation" of merchandise and that there might more than one purpose, we do not mean to be understood as having decided
be no relation between the kind of merchandise imported with the class of that corporations under the laws of the Philippine Islands may not engage in
merchandise exported. a business with more than one purpose. Such an interpretation might work a
great injustice to corporations organized under the Philippine laws. Such an
Referring again to be proposed articles of incorporation, a copy of which is interpretation would give foreign corporations, which are permitted to be
united with the original petition and marked Exhibit A, it will be seen that the registered under the laws here and which may be organized for more than
only purpose of said corporation are those enumerated in subparagraphs (a), one purpose, a great advantage over domestic corporations. We do not
(b), (c), (d), (e) and ( f ) of paragraph 4 above. While said articles of believe that it was the intention of the legislature to give foreign corporations
incorporation are somewhat loosely drawn, it is clear from a reading of the such an advantage over domestic corporations.
same that the principal purpose of said corporation is to engage in
a mercantile business, with the power to do and perform the particular acts Considering the particular purposes and objects of the proposed articles of
enumerated in said subparagraphs above referred to. incorporation which are specially enumerated above, we are of the opinion
that it contains nothing which violates in the slightest degree any of the
Without discussing or deciding at this time whether a corporation organized provisions of the laws of the Philippine Islands, and the petitioners are,
under the laws of the Philippine Islands may be organized for more than one therefore, entitled to have such articles of
purpose, we are of the opinion and so decide that a corporation may be incorporation filed and registered as prayed for by them and to have issued
to them a certificate under the seal of the office of the respondent, setting

214
forth that such articles of incorporation have been duly filed in his office.
(Sec. 11, Act No. 1459.)

Therefore, the petition prayed for is hereby granted, and without any finding
as to costs, it is so ordered.

Arellano, C.J., Torres and Avancea, JJ., concur.

G.R. No. L-22238 February 18, 1967

215
CLAVECILLIA RADIO SYSTEM, petitioner-appellant, its rejoinder. Thereafter, the City Judge, on September 18, 1963,
vs. denied the motion to dismiss for lack of merit and set the case for
HON. AGUSTIN ANTILLON, as City Judge of the Municipal Court of hearing.1wph1.t
Cagayan de Oro City
and NEW CAGAYAN GROCERY, respondents-appellees. Hence, the Clavecilla Radio System filed a petition for prohibition with
preliminary injunction with the Court of First Instance praying that the City
B. C. Padua for petitioner and appellant. Judge, Honorable Agustin Antillon, be enjoined from further proceeding with
Pablo S. Reyes for respondents and appellees. the case on the ground of improper venue. The respondents filed a motion to
dismiss the petition but this was opposed by the petitioner. Later, the motion
REGALA, J.: was submitted for resolution on the pleadings.

This is an appeal from an order of the Court of First Instance of Misamis In dismissing the case, the lower court held that the Clavecilla Radio System
Oriental dismissing the petition of the Clavecilla Radio System to prohibit the may be sued either in Manila where it has its principal office or in Cagayan
City Judge of Cagayan de Oro from taking cognizance of Civil Case No. de Oro City where it may be served, as in fact it was served, with summons
1048 for damages. through the Manager of its branch office in said city. In other words, the court
upheld the authority of the city court to take cognizance of the
It appears that on June 22, 1963, the New Cagayan Grocery filed a case.1wph1.t
complaint against the Clavecilla Radio System alleging, in effect, that on
March 12, 1963, the following message, addressed to the former, was filed at In appealing, the Clavecilla Radio System contends that the suit against it
the latter's Bacolod Branch Office for transmittal thru its branch office at should be filed in Manila where it holds its principal office.
Cagayan de Oro:
It is clear that the case for damages filed with the city court is based upon tort
NECAGRO CAGAYAN DE ORO (CLAVECILLA) and not upon a written contract. Section 1 of Rule 4 of the New Rules of
Court, governing venue of actions in inferior courts, provides in its paragraph
REURTEL WASHED NOT AVAILABLE REFINED TWENTY FIFTY IF (b) (3) that when "the action is not upon a written contract, then in the
AGREEABLE SHALL SHIP LATER REPLY POHANG municipality where the defendant or any of the defendants resides or may be
served with summons." (Emphasis supplied)
The Cagayan de Oro branch office having received the said
message omitted, in delivering the same to the New Cagayan Settled is the principle in corporation law that the residence of a corporation
Grocery, the word "NOT" between the words "WASHED" and is the place where its principal office is established. Since it is not disputed
"AVAILABLE," thus changing entirely the contents and purport of the that the Clavecilla Radio System has its principal office in Manila, it follows
same and causing the said addressee to suffer damages. After that the suit against it may properly be filed in the City of Manila.
service of summons, the Clavecilla Radio System filed a motion to
dismiss the complaint on the grounds that it states no cause of action The appellee maintain, however, that with the filing of the action in Cagayan
and that the venue is improperly laid. The New Cagayan Grocery de Oro City, venue was properly laid on the principle that the appellant may
interposed an opposition to which the Clavecilla Radio System filed also be served with summons in that city where it maintains a branch office.

216
This Court has already held in the case of Cohen vs. Benguet Commercial
Co., Ltd., 34 Phil. 526; that the term "may be served with summons" does not
apply when the defendant resides in the Philippines for, in such case, he may
be sued only in the municipality of his residence, regardless of the place
where he may be found and served with summons. As any other corporation,
the Clavecilla Radio System maintains a residence which is Manila in this
case, and a person can have only one residence at a time (See Alcantara vs.
Secretary of the Interior, 61 Phil. 459; Evangelists vs. Santos, 86 Phil. 387).
The fact that it maintains branch offices in some parts of the country does not
mean that it can be sued in any of these places. To allow an action to be
instituted in any place where a corporate entity has its branch offices would
create confusion and work untold inconvenience to the corporation.

It is important to remember, as was stated by this Court in Evangelista vs.


Santos, et al., supra, that the laying of the venue of an action is not left to
plaintiff's caprice because the matter is regulated by the Rules of Court.
Applying the provision of the Rules of Court, the venue in this case was
improperly laid.

The order appealed from is therefore reversed, but without prejudice to the
filing of the action in Which the venue shall be laid properly. With costs
against the respondents-appellees.

Concepcion, C.J., Reyes, J.B.L., Dizon, Makalintal, Bengzon, J.P., Zaldivar,


Sanchez and Castro, JJ., concur.

217
218
G.R. No. L-23606 July 29, 1968

ALHAMBRA CIGAR & CIGARETTE MANUFACTURING COMPANY,


INC., petitioner,
vs.
SECURITIES & EXCHANGE COMMISSION, respondent.

Gamboa and Gamboa for petitioner.


Office of the Solicitor General for respondent.

SANCHEZ, J.:

To the question May a corporation extend its life by amendment of its


articles of incorporation effected during the three-year statutory period for
liquidation when its original term of existence had already expired? the
answer of the Securities and Exchange Commissioner was in the negative.
Offshoot is this appeal.

That problem emerged out of the following controlling facts:

Petitioner Alhambra Cigar and Cigarette Manufacturing Company, Inc.


(hereinafter referred to simply as Alhambra) was duly incorporated under
Philippine laws on January 15, 1912. By its corporate articles it was to exist
for fifty (50) years from incorporation. Its term of existence expired on
January 15, 1962. On that date, it ceased transacting business, entered into
a state of liquidation.

Thereafter, a new corporation. Alhambra Industries, Inc. was formed to


carry on the business of Alhambra.

On May 1, 1962, Alhambra's stockholders, by resolution named Angel S.


Gamboa trustee to take charge of its liquidation.

219
On June 20, 1963 within Alhambra's three-year statutory period for On September 8, 1964, SEC, after a conference hearing, issued an order
liquidation - Republic Act 3531 was enacted into law. It amended Section 18 denying the reconsideration sought.
of the Corporation Law; it empowered domestic private corporations to
extend their corporate life beyond the period fixed by the articles of Alhambra now invokes the jurisdiction of this Court to overturn the conclusion
incorporation for a term not to exceed fifty years in any one instance. below.1
Previous to Republic Act 3531, the maximum non-extendible term of such
corporations was fifty years. 1. Alhambra relies on Republic Act 3531, which amended Section 18 of the
Corporation Law. Well it is to take note of the old and the new statutes as
On July 15, 1963, at a special meeting, Alhambra's board of directors they are framed. Section 18, prior to and after its modification by Republic
resolved to amend paragraph "Fourth" of its articles of incorporation to Act 3531, covers the subject of amendment of the articles of incorporation of
extend its corporate life for an additional fifty years, or a total of 100 years private corporations. A provision thereof which remains unaltered is that a
from its incorporation. corporation may amend its articles of incorporation "by a majority vote of its
board of directors or trustees and ... by the vote or written assent of the
On August 26, 1963, Alhambra's stockholders, representing more than two- stockholders representing at least two-thirds of the subscribed capital
thirds of its subscribed capital stock, voted to approve the foregoing stock ... "
resolution. The "Fourth" paragraph of Alhambra's articles of incorporation
was thus altered to read: But prior to amendment by Republic Act 3531, an explicit prohibition existed
in Section 18, thus:
FOURTH. That the term for which said corporation is to exist is fifty
(50) years from and after the date of incorporation, and for an ... Provided, however, That the life of said corporation shall not be
additional period of fifty (50) years thereafter. extended by said amendment beyond the time fixed in the original
articles: ...
On October 28, 1963, Alhambra's articles of incorporation as so amended
certified correct by its president and secretary and a majority of its board of This was displaced by Republic Act 3531 which enfranchises all private
directors, were filed with respondent Securities and Exchange Commission corporations to extend their corporate existence. Thus incorporated into the
(SEC). structure of Section 18 are the following:

On November 18, 1963, SEC, however, returned said amended articles of ... Provided, however, That should the amendment consist in
incorporation to Alhambra's counsel with the ruling that Republic Act 3531 extending the corporate life, the extension shall not exceed fifty
"which took effect only on June 20, 1963, cannot be availed of by the said years in any one instance: Provided, further, That the original
corporation, for the reason that its term of existence had already expired articles, and amended articles together shall contain all provisions
when the said law took effect in short, said law has no retroactive effect." required by law to be set out in the articles of incorporation: ...

On December 3, 1963, Alhambra's counsel sought reconsideration of SEC's As we look in retrospect at the facts, we find these: From July 15 to October
ruling aforesaid, refiled the amended articles of incorporation. 28, 1963, when Alhambra made its attempt to extend its corporate existence,
its original term of fifty years had already expired (January 15, 1962); it was

220
in the midst of the three-year grace period statutorily fixed in Section 77 of 2. Republic Act 3531, amending Section 18 of the Corporation Law, is silent,
the Corporation Law, thus: . it is true, as to when such act of extension may be made. But even with a
superficial knowledge of corporate principles, it does not take much effort to
SEC. 77. Every corporation whose charter expires by its own reach a correct conclusion. For, implicit in Section 77 heretofore quoted is
limitation or is annulled by forfeiture or otherwise, or whose corporate that the privilege given to prolong corporate life under the amendment must
existence for other purposes is terminated in any other manner, shall be exercised before the expiry of the term fixed in the articles of
nevertheless be continued as a body corporate for three years after incorporation.
the time when it would have been so dissolved, for the purpose of
prosecuting and defending suits by or against it and of enabling it Silence of the law on the matter is not hard to understand. Specificity is not
gradually to settle and close its affairs, to dispose of and convey its really necessary. The authority to prolong corporate life was inserted by
property and to divide its capital stock, but not for the purpose of Republic Act 3531 into a section of the law that deals with the power of a
continuing the business for which it was established. 2 corporation to amend its articles of incorporation. (For, the manner of
prolongation is through an amendment of the articles.) And it should be
Plain from the language of the provision is its meaning: continuance of a clearly evident that under Section 77 no corporation in a state of liquidation
"dissolved" corporation as a body corporate for three years has for its can act in any way, much less amend its articles, "for the purpose of
purpose the final closure of its affairs, and no other; the corporation is continuing the business for which it was established".
specifically enjoined from "continuing the business for which it was
established". The liquidation of the corporation's affairs set forth in Section 77 All these dilute Alhambra's position that it could revivify its corporate life
became necessary precisely because its life had ended. For this reason simply because when it attempted to do so, Alhambra was still in the process
alone, the corporate existence and juridical personality of that corporation to of liquidation. It is surely impermissible for us to stretch the law that merely
do business may no longer be extended. empowers a corporation to act in liquidation to inject therein the power to
extend its corporate existence.
Worth bearing in mind, at this juncture, is the basic development of
corporation law. 3. Not that we are alone in this view. Fletcher has written: "Since the privilege
of extension is purely statutory, all of the statutory conditions precedent must
The common law rule, at the beginning, was rigid and inflexible in that upon be complied with in order that the extension may be effectuated. And,
its dissolution, a corporation became legally dead for all purposes. Statutory generally these conditions must be complied with, and the steps necessary
authorizations had to be provided for its continuance after dissolution "for to effect the extension must be taken, during the life of the corporation, and
limited and specified purposes incident to complete liquidation of its before the expiration of the term of existence as original fixed by its charter
affairs".3 Thus, the moment a corporation's right to exist as an "artificial or the general law, since, as a rule, the corporation is ipso facto dissolved as
person" ceases, its corporate powers are terminated "just as the powers of a soon as that time expires. So where the extension is by amendment of the
natural person to take part in mundane affairs cease to exist upon his articles of incorporation, the amendment must be adopted before that time.
death".4 There is nothing left but to conduct, as it were, the settlement of the And, similarly, the filing and recording of a certificate of extension after that
estate of a deceased juridical person. time cannot relate back to the date of the passage of a resolution by the
stockholders in favor of the extension so as to save the life of the
corporation. The contrary is true, however, and the doctrine of relation will
apply, where the delay is due to the neglect of the officer with whom the

221
certificate is required to be filed, or to a wrongful refusal on his part to receive True it is, that the Alabama Supreme Court has stated in one case. 8 that a
it. And statutes in some states specifically provide that a renewal may be had corporation empowered by statute to renew its corporate existence may do
within a specified time before or after the time fixed for the termination of the so even after the expiration of its corporate life, provided renewal is taken
corporate existence".5 advantage of within the extended statutory period for purposes of liquidation.
That ruling, however, is inherently weak as persuasive authority for the
The logic of this position is well expressed in a foursquare case decided by situation at bar for at least two reasons: First. That case was a suit for
the Court of Appeals of Kentucky.6There, pronouncement was made as mandamus to compel a former corporate officer to turn over books and
follows: records that came into his possession and control by virtue of his office. It
was there held that such officer was obliged to surrender his books and
... But section 561 (section 2147) provides that, when any records even if the corporation had already expired. The holding on the
corporation expires by the terms of its articles of incorporation, it may continued existence of the corporation was a mere dictum. Second.
be thereafter continued to act for the purpose of closing up its Alabama's law is different. Corporations in that state were authorized not only
business, but for no other purpose. The corporate life of the Home to extend but also to renew their corporate existence.That very case defined
Building Association expired on May 3, 1905. After that date, by the the word "renew" as follows; "To make new again; to restore to freshness; to
mandate of the statute, it could continue to act for the purpose of make new spiritually; to regenerate; to begin again; to recommence; to
closing up its business, but for no other purpose. The proposed resume; to restore to existence, to revive; to re-establish; to recreate; to
amendment was not made until January 16, 1908, or nearly three replace; to grant or obtain an extension of Webster's New International Dict.;
years after the corporation expired by the terms of the articles of 34 Cyc. 1330; Carter v. Brooklyn Life Ins. Co., 110 N.Y. 15, 21, 22, 17 N.E.
incorporation. When the corporate life of the corporation was ended, 396; 54 C.J. 379. Sec".9
there was nothing to extend. Here it was proposed nearly three years
after the corporate life of the association had expired to revivify the On this point, we again draw from Fletcher: "There is a broad distinction
dead body, and to make that relate back some two years and eight between the extension of a charter and the grant of a new one. To renew a
months. In other words, the association for two years and eight charter is to revive a charter which has expired, or, in other words, "to give a
months had only existed for the purpose of winding up its business, new existence to one which has been forfeited, or which has lost its vitality by
and, after this length of time, it was proposed to revivify it and make lapse of time". To "extend" a charter is "to increase the time for the existence
it a live corporation for the two years and eight months daring which of one which would otherwise reach its limit at an earlier period". 10Nowhere in
it had not been such. our statute Section 18, Corporation Law, as amended by Republic Act
3531 do we find the word "renew" in reference to the authority given to
The law gives a certain length of time for the filing of records in this corporations to protract their lives. Our law limits itself to extensionof
court, and provides that the time may be extended by the court, but corporate existence. And, as so understood, extension may be
under this provision it has uniformly been held that when the time made only before the term provided in the corporate charter expires.
was expired, there is nothing to extend, and that the appeal must be
dismissed... So, when the articles of a corporation have expired, it is Alhambra draws attention to another case 11 which declares that until the end
too late to adopt an amendment extending the life of a corporation; of the extended period for liquidation, a dissolved corporation "does not
for, the corporation having expired, this is in effect to create a new become an extinguished entity". But this statement was obviously lifted out of
corporation ..."7 context. That case dissected the question whether or not suits can be
commenced by or against a corporation within its liquidation period. Which

222
was answered in the affirmative. For, the corporation still exists for the Republic Act 3531. Since the phrase "on or before", etc., was omitted in
settlement of its affairs. Republic Act 3531, which contains no similar limitation, it follows, according
to Alhambra, that it is not necessary to extend corporate existence on or
People, ex rel. vs. Green,12 also invoked by Alhambra, is as unavailing. before the expiration of its original term.
There, although the corporation amended its articles to extend its existence
at a time when it had no legal authority yet, it adopted the amended articles That Republic Act 3531 stands mute as to when extention of corporate
later on when it had the power to extend its life and during its original term existence may be made, assumes no relevance. We have already said, in
when it could amend its articles. the face of a familiar precept, that a defunct corporation is bereft of any legal
faculty not otherwise expressly sanctioned by law.
The foregoing notwithstanding, Alhambra falls back on the contention that its
case is arguably within the purview of the law. It says that before cessation of Illuminating here is the explanatory note of H.B. 1774, later Republic Act
its corporate life, it could not have extended the same, for the simple reason 3531 now in dispute. Its first paragraph states that "Republic Act No. 1932
that Republic Act 3531 had not then become law. It must be remembered allows the automatic extension of the corporate existence of domestic life
that Republic Act 3531 took effect on June 20, 1963, while the original term insurance corporations upon amendment of their articles of incorporation on
of Alhambra's existence expired before that date on January 15, 1962. or before the expiration of the terms fixed by said articles". The succeeding
The mischief that flows from this theory is at once apparent. It would certainly lines are decisive: "This is a good law, a sane and sound one. There appears
open the gates for all defunct corporations whose charters have expired to be no valid reason why it should not be made to apply to other private
even long before Republic Act 3531 came into being to resuscitate their corporations.13
corporate existence.
The situation here presented is not one where the law under consideration is
4. Alhambra brings into argument Republic Act 1932, which amends Section ambiguous, where courts have to put in harness extrinsic aids such as a look
196 of the Insurance Act, now reading as follows: 1wph1.t at another statute to disentangle doubts. It is an elementary rule in legal
hermeneutics that where the terms of the law are clear, no statutory
SEC. 196. Any provision of law to the contrary notwithstanding, every construction may be permitted. Upon the basic conceptual scheme under
domestic life insurance corporation, formed for a limited period under which corporations operate, and with Section 77 of the Corporation Law
the provisions of its articles of incorporation, may extend its particularly in mind, we find no vagueness in Section 18, as amended by
corporate existence for a period not exceeding fifty years in any one Republic Act 3531. As we view it, by directing attention to Republic Act 1932,
instance by amendment to its articles of incorporation on or before Alhambra would seek to create obscurity in the law; and, with that, ask of us
the expiration of the term so fixed in said articles ... a ruling that such obscurity be explained. This, we dare say, cannot be done.

To be observed is that the foregoing statute unlike Republic Act 3531 The pari materia rule of statutory construction, in fact, commands that
expressly authorizes domestic insurance corporations to extend their statutes must be harmonized with each other. 14 So harmonizing, the
corporate existence "on or before the expiration of the term" fixed in their conclusion is clear that Section 18 of the Corporation Law, as amended by
articles of incorporation. Republic Act 1932 was approved on June 22, 1957, Republic Act 3531 in reference to extensions of corporate existence, is to be
long before the passage of Republic Act 3531 in 1963. Congress, Alhambra read in the same light as Republic Act 1932. Which means that domestic
points out, must have been aware of Republic Act 1932 when it passed corporations in general, as with domestic insurance companies, can extend

223
corporate existence only on or before the expiration of the term fixed in their
charters.

5. Alhambra pleads for munificence in interpretation, one which brushes


technicalities aside. Bases for this posture are that Republic Act 3531 is a
remedial statute, and that extension of corporate life is beneficial to the
economy.

Alhambra's stance does not induce assent. Expansive construction is


possible only when there is something to expand. At the time of the passage
of Republic Act 3531, Alhambra's corporate life had already expired. It had
overstepped the limits of its limited existence. No life there is to prolong.

Besides, a new corporation Alhambra Industries, Inc., with but slight


change in stockholdings15 has already been established. Its purpose is to
carry on, and it actually does carry on, 16 the business of the dissolved entity.
The beneficial-effects argument is off the mark.

The way the whole case shapes up then, the only possible drawbacks of
Alhambra might be that, instead of the new corporation (Alhambra Industries,
Inc.) being written off, the old one (Alhambra Cigar & Cigarette
Manufacturing Company, Inc.) has to be wound up; and that the old
corporate name cannot be retained fully in its exact form. 17 What is important
though is that the word Alhambra, the name that counts [it has goodwill],
remains.

FOR THE REASONS GIVEN, the ruling of the Securities and Exchange
Commission of November 18, 1963, and its order of September 8, 1964, both
here under review, are hereby affirmed.

Costs against petitioner Alhambra Cigar & Cigarette Manufacturing


Company, Inc. So ordered.

Concepcion, C.J., Reyes, J.B.L., Dizon, Makalintal, Zaldivar, Castro, Angeles


and Fernando, JJ., concur.

224
years. In 1906, the governing Philippine Commission enacted Act 1459,
commonly known as the Corporation Law, establishing in the islands the
American type of juridical entities known as corporation, to take effect on
April 1, 1906. Of its enactment, this Court said in its decision in Harden vs.
Benguet Consolidated Mining Co., 58 Phil., 141, at pp. 145-146, and
147:chanroblesvirtuallawlibrary

When the Philippine Islands passed to the sovereignty of the United States,
the attention of the Philippine Commission was early drawn to the fact there
is no entity in Spanish law exactly corresponding to the motion of the
corporation in English and American law; chan roblesvirtualawlibraryand in
the Philippine Bill, approved July 1, 1906, the Congress of the United States
inserted certain provisions, under the head of Franchises, which were
[G.R. No. L-7231. March 28, 1956.] intended to control the lawmaking power in the Philippine Islands in the
matter of granting of franchises, privileges and concessions. These
BENGUET CONSOLIDATED MINING CO., Petitioner, vs. MARIANO provisions are found in sections 74 and 75 of the Act. The provisions of
PINEDA, in his capacity as Securities and Exchange section 74 have been superseded by section 28 of the Act of Congress of
Commissioner, Respondent. CONSOLIDATED MINES, INC., Intervenor. August 29, 1916, but in section 75 there is a provision referring to mining
corporations, which still remains the law, as amended. This provision, in its
original form, reads as follows:chanroblesvirtuallawlibrary cralaw it shall be
unlawful for any member of a corporation engaged in agriculture or mining
DECISION and for any corporation organized for any purpose except irrigation to be in
any wise interested in any other corporation engaged in agriculture or in
mining.
REYES, J. B. L., J.:

Under the guidance of this and certain other provisions thus enacted by
Appeal under Rule 43 from a decision of the Securities and Exchange
Congress, the Philippine Commission entered upon the enactment of a
Commissioner, denying the right of a sociedad anonima to extend its
general law authorizing the creation of corporations in the Philippine Islands.
corporate existence by amendment of its original articles of association, or
This rather elaborate piece of legislation is embodied in what is called our
alternatively, to reform and continue existing under the Corporation Law (Act
Corporation Law (Act No. 1459 of the Philippine Commission). The evident
1459) beyond the original period.
purpose of the commission was to introduce the American corporation into
the Philippine Islands as the standard commercial entity and to hasten the
The Petitioner, the Benguet Consolidated Mining Co. (hereafter termed day when the sociedad anonima of the Spanish law would be obsolete. That
Benguet for short), was organized on June 24,1903, as a sociedad statute is a sort of codification of American corporate law.
anonima regulated by Articles 151 et seq., of the Spanish Code of
Commerce of 1886, then in force in the Philippines. The articles of
As it was the intention of our lawmakers to stimulate the introduction of the
association expressly provided that it was organized for a term of fifty (50)
American corporation into the Philippine law in the place of the sociedad

225
anonima, it was necessary to make certain adjustment resulting from the inconsistent with this Act, are hereby repealed with the exception of Act
continued co-existence, for a time, of the two forms of commercial entities. Numbered fifty-two, entitled An Act providing for examinations of banking
Accordingly, in section 75 of the Corporation Law, a provision is found institutions in the Philippines, and for reports by their officers, as amended,
making the sociedad anonima subject to the provisions of the Corporation and Act Numbered Six hundred sixty-seven, entitled An Act prescribing the
Law so far as such provisions may be applicable and giving to the method of applying to governments of municipalities, except the city of
sociedades anonimas previously created in the Islands the option to continue Manila and of provinces for franchises to contract and operate street railway,
business as such or to reform and organize under the provisions of the electric light and power and telephone lines, the conditions upon which the
Corporation Law. Again, in section 191 of the Corporation Law, the Code of same may be granted, certain powers of the grantee of said franchises, and
Commerce is repealed in so far as it relates to sociedades anonimas. The of grantees of similar franchises under special Act of the Commission, and
purpose of the commission in repealing this part of the Code of Commerce for other purposes. Provided, however, That nothing in this Act contained
was to compel commercial entities thereafter organized to incorporate under shall be deemed to repeal the existing law relating to those classes of
the Corporation Law, unless they should prefer to adopt some form or other associations which are termed sociedades colectivas, and sociedades de
of the partnership. To this provision was added another to the effect that cuentas en participacion, as to which association the existing law shall be
existing sociedades anonimas, which elected to continue their business as deemed to be still in force; chan roblesvirtualawlibraryAnd provided, further,
such, instead of reforming and reorganizing under the Corporation Law, That existing corporations or sociedades anonimas, lawfully organized as
should continue to be governed by the laws that were in force prior to the such, which elect to continue their business as such sociedades anonimas
passage of this Act in relation to their organization and method of transacting instead of reforming and reorganizing under and by virtue of the provisions of
business and to the rights of members thereof as between themselves, but this Act, shall continue to be governed by the laws that were in force prior to
their relations to the public and public officials shall be governed by the the passage of this Act in relation to their organization and method of
provisions of this Act. transacting business and to the rights of members thereof as between
themselves, but their relations to the public and public officials shall be
Specifically, the two sections of Act No. 1459 referring to sociedades governed by the provisions of this Act.
anonimas then already existing, provide as
follows:chanroblesvirtuallawlibrary As the expiration of its original 50 year term of existence approached, the
Board of Directors of Benguet adopted in 1946 a resolution to extend its life
SEC. 75. Any corporation or a sociedad anonima formed, organized, and for another 50 years from July 3, 1946 and submitted it for registration to
existing under the laws of the Philippines on the date of the passage of this the Respondent Securities and Exchange Commissioner. Upon advice of the
Act, shall be subject to the provisions hereof so far as such provisions may Secretary of Justice (Op. No. 45, Ser. 1917) that such extension was
be applicable and shall be entitled at its option either to continue business as contrary to law, the registration was denied. The matter was dropped,
such corporation or to reform and organize under and by virtue of the allegedly because the stockholders of Benguet did not approve of the
provisions of this Act, transferring all corporate interests to the new Directors action.
corporation which, if a stock corporation, is authorized to issue its shares of
stock at par to the stockholders or members of the old corporation according Some six years later in 1953, the shareholders of Benguet adopted a
to their interests. resolution empowering the Director to effectuate the extension of the
Companys business life for not less than 20 and not more than 50 years,
SEC. 191. The Code of Commerce, in so far as it relates to corporation or and this by either (1) an amendment to the Articles of Association or Charter
sociedades anonimas, and all other Acts or parts of Acts in conflict or of this Company or (2) by reforming and reorganizing the Company as a

226
Philippine Corporation, or (3) by both or (4) by any other means. reformation or reorganization as a corporation in accordance with section 75
Accordingly, the Board of Directors on May 27, 1953, adopted a resolution to of the Philippine Corporation Law.
the following effect
Relying mainly upon the adverse opinion of the Secretary of Justice (Op. No.
Be It 180, s. 1953), the Securities and Exchange Commissioner denied the
registration and ruled:chanroblesvirtuallawlibrary
Resolved, that the Company be reformed, reorganized and organized under
the provisions of section 75 and other provisions of the Philippine (1) That the Benguet, as sociedad anonima, had no right to extend the
Corporation Law as a Philippine corporation with a corporate life and original term of corporate existence stated in its Articles of Association, by
corporate powers as set forth in the Articles of Incorporation attached hereto subsequent amendment thereof adopted after enactment of the Corporation
as Schedule I and made a part hereof by this reference; chan Law (Act No. 1459); chan roblesvirtualawlibraryand
roblesvirtualawlibraryand
(2) That Benguet, by its conduct, had chosen to continue as sociedad
Be It anonima, under section 75 of Act No. 1459, and could no longer exercise the
option to reform into a corporation, specially since it would indirectly produce
FURTHER RESOLVED, that any five or more of the following shareholders the effect of extending its life.
of the Company be and they hereby are authorized as instructed to act for
and in behalf of the share holders of the Company and of the Company as This ruling is the subject of the present appeal.
Incorporators in the reformation, reorganization and organization of the
Company under and in accordance with the provisions aforesaid of said Petitioner Benguet contends:chanroblesvirtuallawlibrary
Philippine Corporation Law, and in such capacity, they are hereby authorized
and instructed to execute the aforesaid Articles of Incorporation attached to (1) That the proviso of section 18 of the Corporation Law to the effect
these Minutes as Schedule I hereof, with such amendments, deletion and
additions thereto as any five or more of those so acting shall deem
that the life of said corporation shall not be extended by amendment beyond
necessary, proper, advisable or convenient to effect prompt registration of
the time fixed in the original articles.
said Articles under Philippine Law; chan roblesvirtualawlibraryand five or
more of said Incorporators are hereby further authorized and directed to do
all things necessary, proper, advisable or convenient to effect such does not apply to sociedades anonimas already in existence at the passage
registration. of the law, like Petitioner herein;

In pursuance of such resolution, Benguet submitted in June, 1953, to the (2) That to apply the said restriction imposed by section 18 of the
Securities and Exchange Commissioner, for alternative registration, two Corporation Law to sociedades anonimas already functioning when the said
documents:chanroblesvirtuallawlibrary (1) Certification as to the Modification law was enacted would be in violation of constitutional inhibitions;
of (the articles of association of) the Benguet Consolidated Mining Company,
extending the term of its existence to another fifty years from June 15, (3) That even assuming that said restriction was applicable to it, Benguet
1953; chan roblesvirtualawlibraryand (2) articles of incorporation, covering its could still exercise the option of reforming and reorganizing under section 75

227
of the Corporation Law, thereby prolonging its corporate existence, since the The term of existence of association (partnership or sociedad anonima) is
law is silent as to the time when such option may be exercised or availed of. coterminous with their possession of an independent legal personality,
distinct from that of their component members. When the period expires, the
The first issue arises because the Code of Commerce of 1886 under which sociedad anonima loses the power to deal and enter into further legal
Benguet was organized, contains no prohibition (to extend the period of relations with other persons; chan roblesvirtualawlibraryit is no longer
corporate existence), equivalent to that set forth in section 18 of the possible for it to acquire new rights or incur new obligations, have only as
Corporation Law. Neither does it expressly authorize the extension. But the may be required by the process of liquidating and winding up its affairs. By
text of Article 223, reading:chanroblesvirtuallawlibrary the same token, its officers and agents can no longer represent it after the
expiration of the life term prescribed, save for settling its business.
ART. 223. After the termination of the period for which commercial Necessarily, therefore, third persons or strangers have an interest in knowing
associations are constituted, it shall not be understood as extended by the the duration of the juridical personality of the sociedad anonima, since the
implied or presumed will of the members; chan roblesvirtualawlibraryand if latter cannot be dealt with after that period; chan
the members desire to continue in association, they shall draw up new roblesvirtualawlibrarywherefore its prolongation or cessation is a matter
articles, subject to all the formalities prescribed for their creation as provided directly involving the companys relations to the public at large.
in Article 119. (Code of Commerce.)
On the importance of the term of existence set in the articles of association of
would seem to imply that the period of existence of the sociedad anonimas commercial companies under the Spanish Code of Commerce, D. Lorenzo
(or of any other commercial association for that matter) may be extended if Benito y Endar, professor of mercantile law in the Universidad Central de
the partners or members so agree before the expiration of the original period. Madrid, has this to say:chanroblesvirtuallawlibrary

While the Code of Commerce, in so far as sociedades anonimas are La duracion de la Sociedad. La necesidad de consignar este requisito en
concerned, was repealed by Act No 1459, Benguet claims that article 223 is el contrato social tiene un valor analogo al que dijimos tenia el mismo al
still operative in its favor under the last proviso of section 191 of the tratar de las compaias colectivas, aun cuando respecto de las anonimas no
Corporation law (ante, p. 4 to the effect that existing sociedades anonimas haya de tenerse en cuenta para nada lo que dijimos entonces acerca de la
would continue to be governed by the law in force before Act 1459, trascendencia que ello tiene para los socios; chan
roblesvirtualawlibraryporque no existiendo en las anonimas la serie de
responsibilidades de caracter personal que afectan a los socios colectivos,
in relation to their organization and method of transacting business and to
es claro que la duracion de la sociedad importa conocerla a los socios y los
the rights of members among themselves, but their relations to the public and
terceros, porque ella marca al limite natural del desenvolvimiento de la
public officials shall be governed by the provisions of this Act.
empresa constituida y el comienzo de la liquidacion de la sociedad. (3
Benito, Derecho Mercantil, 292-293.)
Benguet contends that the period of corporate life relates to its organization
and the rights of its members inter se, and not to its relations to the public or
Interesa, pues, la fijacion de la vida de la compaia, desenvolviendose con
public officials.
normalidad y regularidad, tanto a los asociados como a los terceros. A
aquellos, porque su libertad economica, en cierto modo limitada por la
We find this contention untenable. existencia del contrato de compaia, se recobra despues de realizada, mas
o menos cumplidamente, la finalidad comun perseguida; chan

228
roblesvirtualawlibraryy a los terceros, porque les advierte el momento en such other steps as are necessary to endow the legal entity with the capacity
que, extinguida la compaia, no cabe y a la creacion con ella de nuevas to transact the legitimate business for which it was created. Waltson vs.
relaciones juridicas, de que nazcan reciprocamente derechos y obligaciones, Oliver, 30 P. 172, 173, 49 Kan. 107, 33 Am. St. Rep. 355; chan
sino solo la liquidacion de los negocios hasta entonces convenidos, sin otra roblesvirtualawlibraryTopeka Bridge Co. vs. Cummings, 3 Kan. 55, 77; chan
excepcion que la que luego mas adelante habremos de sealar. (3 Benito, roblesvirtualawlibraryHunt vs. Kansas & M. Bridge Co., 11 Kan. 412,
Derecho Mercantil, p. 245.) 439; chan roblesvirtualawlibraryAspen Water & Light Co., vs. City of Aspen,
37 P. 728, 730, 6 Colo. App. 12; chan roblesvirtualawlibraryNemaha Coal &
The State and its officers also have an obvious interest in the term of life of Mining Co., vs. Settle 38 P. 483, 484, 54 Kan. 424.
associations, since the conferment of juridical capacity upon them during
such period is a privilege that is derived from statute. It is obvious that no Under a statute providing that, until articles of incorporation should be
agreement between associates can result in giving rise to a new and distinct recorded, the corporation should transact no business except its own
personality, possessing independent rights and obligations, unless the law organization, it is held that the term organization means simply the process
itself shall decree such result. And the State is naturally interested that this of forming and arranging into suitable disposition the parties who are to act
privilege be enjoyed only under the conditions and not beyond the period that together in, and defining the objects of, the compound body, and that this
it sees fit to grant; chan roblesvirtualawlibraryand, particularly, that it be not process, even when complete in all its parts, does not confer a franchise
abused in fraud and to the detriment of other parties; chan either valid or defective, but, on the contrary, it is only the act of the
roblesvirtualawlibraryand for this reason it has been ruled that the limitation individuals, and something else must be done to secure the corporate
(of corporate existence) to a definite period is an exercise of control in the franchise. Abbott vs. Omaha Smelting & Refining Co. 4 Neb. 416, 421. (30
interest of the public (Smith vs. Eastwood Wire Manufacturing Co., 43 Atl. Words and Phrases, p. 282.)
568).
It is apparent from the foregoing definitions that the term organization
We cannot assent to the thesis of Benguet that its period of corporate relates merely to the systematization and orderly arrangement of the internal
existence has relation to its organization. The latter term is defined in and managerial affairs and organs of the Petitioner Benguet, and has nothing
Websters International Dictionary as:chanroblesvirtuallawlibrary to do with the prorogation of its corporate life.

The executive structure of a business; chan roblesvirtualawlibrarythe From the double fact that the duration of its corporate life (and juridical
personnel of management, with its several duties and places in personality) has evident connection with the Petitioners relations to the
administration; chan roblesvirtualawlibrarythe various persons who conduct a public, and that it bears none to the Petitioners organization and method of
business, considered as a unit. transacting business, we derive the conclusion that the prohibition contained
in section 18 of the Corporation Law (Act No. 1459) against extension of
The legal definitions of the term organization are concordant with that given corporate life by amendment of the original articles was designed and
above:chanroblesvirtuallawlibrary intended to apply to compaias anonimas that, like Petitioner Benguet,
were already existing at the passage of said law. This conclusion is
Organize or organization, as used in reference to corporations, has a well- reinforced by the avowed policy of the law to hasten the day when
understood meaning, which is the election of officers, providing for the compaias anonimas would be extinct, and replace them with the American
subscription and payment of the capital stock, the adoption of by-laws, and type of corporation (Harden vs. Benguet Consolidated Mining Co., supra), for
the indefinite prorogation of the corporation life of sociedades anonimas

229
would maintain the unnecessary duality of organizational types instead of in 1906 was merely a possibility in futuro, a contingency that did not fulfill the
reducing them to a single one; chan roblesvirtualawlibraryand what is more, it requirements of a vested right entitled to constitutional protection, defined by
would confer upon these sociedades anonimas, whose obsolescence was this Court in Balboa vs. Farrales, 51 Phil., 498, 502, as
sought, the advantageous privilege of perpetual existence that the new follows:chanroblesvirtuallawlibrary
corporation could not possess.
Vested right is some right or interest in the property which has become fixed
Of course, the retroactive application of the limitations on the terms of and established, and is no longer open to doubt or controversy,
corporate existence could not be made in violation of constitutional inhibitions
specially those securing equal protection of the laws and prohibiting A vested right is defined to be an immediate fixed right of present or future
impairment of the obligation of contracts. It needs no argument to show that if enjoyment, and rights are vested in contradistinction to being expectant or
Act No. 1459 allowed existing compaias anonimas to be governed by the contingent (Pearsall vs. Great Northern R. Co., 161 U. S. 646, 40 L. Ed.
old law in respect to their organization, methods of transacting business and 838).
the rights of the members among themselves, it was precisely in deference
to the vested rights already acquired by the entity and its members at the In Corpus Juris Secundum we find:chanroblesvirtuallawlibrary
time the Corporation Law was enacted. But we do not agree
with Petitioner Benguet (and here lies the second issue in this appeal) that
Rights are vested when the right to enjoyment, present or prospective, has
the possibility to extend its corporate life under the Code of Commerce
become the property of some particular person or persons as a present
constituted a right already vested when Act No. 1459 was adopted. At that
interest. The right must be absolute, complete, and unconditional,
time, Benguets existence was well within the 50 years period set in its
independent of a contingency, and a mere expectancy of future benefit, or a
articles of association; chan roblesvirtualawlibraryand its members had not
contingent interest in property founded on anticipated continuance of existing
entered into any agreement that such period should be extended. It is safe to
laws, does not constitute a vested right. So, inchoate rights which have not
say that none of the members of Benguet anticipated in 1906 any need to
been acted on are not vested. (16 C.J. S. 214-215.)
reach an agreement to increase the term of its corporate life, barely three
years after it had started. The prorogation was purely speculative; chan
roblesvirtualawlibrarya mere possibility that could not be taken for granted. It Since there was no agreement as yet to extend the period of Benguets
was as yet conditional, depending upon the ultimate decision of the members corporate existence (beyond the original 50 years) when the Corporation Law
and directors. They might agree to extend Benguets existence beyond the was adopted in 1906, neither Benguet nor its members had any actual or
original 50 years; chan roblesvirtualawlibraryor again they might not. It must vested right to such extension at that time. Therefore, when the Corporation
be remembered that in 1906, the success of Benguet in its mining ventures Law, by section 18, forbade extensions of corporate life, neither Benguet nor
was by no means so certain as to warrant continuation of its operations its members were deprived of any actual or fixed right constitutionally
beyond the 50 years set in its articles. The records of this Court show that protected.
Benguet ran into financial difficulties in the early part of its existence, to the
extent that, as late as 1913, ten years after it was found, 301,100 shares of To hold, as Petitioner Benguet asks, that the legislative power could not
its capital stock (with a par value of $1 per share) were being offered for sale deprive Benguet or its members of the possibility to enter at some indefinite
at 25 centavos per share in order to raise the sum of P75,000 that was future time into an agreement to extend Benguets corporate life, solely
needed to rehabilitate the company (Hanlon vs. Hausermann and Beam, 40 because such agreements were authorized by the Code of Commerce,
Phil., 796). Certainly the prolongation of the corporate existence of Benguet would be tantamount to saying that the said Code was irrepealable on that

230
point. It is a well settled rule that no person has a vested interest in any rule amendment of its articles of association, without objection from the officers of
of law entitling him to insist that it shall remain unchanged for his benefit. the defunct Bureau of Commerce and Industry, then in charge of the
(New York C. R. Co. vs. White, 61 L. Ed (U.S.) 667; chan enforcement of the Corporation Laws, although the exact question was never
roblesvirtualawlibraryMondou vs. New York N. H. & H. R. Co., 56 L. Ed. raised then. Be that as it may, it is a well established rule in this jurisdiction
327; chan roblesvirtualawlibraryRainey vs. U. S., 58 L. Ed. 617; chan that the government is never estopped by mistake or error on the part of its
roblesvirtualawlibraryLilly Co. vs. Saunders, 125 ALR. 1308; chan agents (Pineda vs. Court of First Instance of Tayabas, 52 Phil., 803, 807),
roblesvirtualawlibraryShea vs. Olson, 111 ALR. 998). and that estopped cannot give validity to an act that is prohibited by law or is
against public policy (Eugenio vs. Perdido, (97 Phil., 41, May 19, 1955; chan
There can be no vested right in the continued existence of a statute or rule roblesvirtualawlibrary19 Am. Jur. 802); chan roblesvirtualawlibraryso that
of the common law which precludes its change or repeal, nor in any omission the Respondent, Securities and Exchange Commissioner, was not bound by
to legislate on a particular matter or subject. Any right conferred by statute the rulings of his predecessor if they be inconsistent with law. Much less
may be taken away by statute before it has become vested, but after a right could erroneous decisions of executive officers bind this Court and induce it
has vested, repeal of the statute or ordinance which created the right does to sanction an unwarranted interpretation or application of legal principles.
not and cannot affect much right. (16 C.J. S. 222-223.)
We now turn to the third and last issue of this appeal, concerning the
It is a general rule of constitutional law that a person has no vested right in exercise of the option granted by section 75 of the Corporation Law to every
statutory privileges and exemptions (Brearly School vs. Ward, 201 NY. 358, sociedad anonima formed, organized and existing under the laws of the
40 LRA NS. 1215; chan roblesvirtualawlibraryalso, Cooley, Constitutional Philippines on the date of the passage of this Act to either continue business
Limitations, 7th ed., p. 546). as such sociedad anonima or to reform and organize under the provisions of
the Corporation Law. Petitioner-Appellant Benguet contends that as the law
It is not amiss to recall here that after Act No. 1459 the Legislature found it does not determine the period within which such option may be exercised,
advisable to impress further restrictions upon the power of corporations to Benguet may exercise it at any time during its corporate existence; chan
deal in public lands, or to hold real estate beyond a maximum area; chan roblesvirtualawlibraryand that in fact on June 22, 1953, it chose to reform
roblesvirtualawlibraryand to prohibit any corporation from endeavouring to itself into a corporation for a period of 50 years from that date, filing the
control or hold more than 15 per cent of the voting stock of an agricultural or corresponding papers and by-laws with the Respondent Commissioner of
mining corporation (Act No. 3518). These prohibitions are so closely Securities and Exchange registration; chan roblesvirtualawlibrarybut the
integrated with our public policy that Commonwealth Act No. 219 sought to latter refused to accept them as belatedly made.
extend such restrictions to associations of all kinds. It would be subversive of
that policy to enable Benguet to prolong its peculiar status of sociedad The Petitioners argument proceeds from the unexpressed assumption that
anonimas, and enable it to cast doubt and uncertainty on whether it is, or not, Benguet, as sociedad anonima, had not exercised the option given by
subject to those restrictions on corporate power, as it once endeavoured to section 75 of the Corporation Law until 1953. This we find to be incorrect.
do in the previous case of Harden vs. Benguet Mining Corp. 58 Phil., 149. Under that section, by continuing to do business as sociedad anonima,
Benguet in fact rejected the alternative to reform as a corporation under Act
Stress has been laid upon the fact that the Compaia Maritima (like Benguet, No. 1459. It will be noted from the text of section 75 (quoted earlier in this
a sociedad anonima established before the enactment of the Corporation opinion) that no special act or manifestation is required by the law from the
Law) has been twice permitted to extend its corporate existence by existing sociedades anonimas that prefer to remain and continue as such. It
is when they choose to reform and organize under the Corporation Law that

231
they must, in the words of the section, transfer all corporate interests to the the Corporation Law to show or hint that a sociedad anonima may make
new corporation. Hence if they do not so transfer, the sociedades anonimas more than one choice thereunder, since only one option is provided for.
affected are to be understood to have elected the alternative to continue
business as such corporation (sociedad anonima) 2 While no express period of time is fixed by the law within which sociedades
anonimas may elect under section 75 of Act No. 1459 either to reform or to
The election of Benguet to remain a sociedad anonima after the enactment retain their status quo, there are powerful reasons to conclude that the
of the Corporation Law is evidence, not only by its failure, from 1906 to 1953, legislature intended such choice to be made within a reasonable time from
to adopt the alternative to transfer its corporate interests to a new the effectivity of the Act. To enable a sociedad anonima to choose
corporation, as required by section 75; chan roblesvirtualawlibraryit also reformation when its stipulated period of existence is nearly ended, would be
appears from positive acts. Thus around 1933, Benguet claimed and to allow it to enjoy a term of existence far longer than that granted to
defended in court its acquisition of shares of the capital stock of the Balatoc corporations organized under the Corporation Law; chan
Mining Company, on the ground that as a sociedad anonima it (Benguet) was roblesvirtualawlibraryin Benguets case, 50 years as sociedad anonima, and
not a corporation within the purview of the laws prohibiting a mining another 50 years as an American type of corporation under Act 1459; chan
corporation from becoming interested in another mining corporation (Harden roblesvirtualawlibrarya result incompatible with the avowed purpose of the
vs. Benguet Mining Corp., 58 Phil., p. 149). Even in the present proceedings, Act to hasten the disappearance of the sociedades anonimas. Moreover,
Benguet has urged its right to amend its original articles of association as such belated election, if permitted, would enable sociedades anonimas to
sociedad anonima and extend its life as such under the provisions of the reap the full advantage of both types of organization. Finally, it would permit
Spanish Code of Commerce. Such appeals to privileges as sociedad sociedades anonimas to prolong their corporate existence indirectly by
anonima under the Code of 1886 necessarily imply that Benguet has belated reformation into corporations under Act No. 1459, when they could
rejected the alternative of reforming under the Corporation Law. not do so directly by amending their articles of association.
As Respondent Commissioners order, now under appeal, has stated
Much stress is laid upon allegedly improper motives on the part of the
A sociedad anonima could not claim the benefit of both, but must have to intervenor, Consolidated Mines, Inc., in supporting the orders appealed from,
choose one and discard the other. If it elected to become a corporation it on the ground that intervenor seeks to terminate Benguets operating
could not continue as a sociedad anonima; chan roblesvirtualawlibraryand if contract and appropriate the profits that are the result of Benguets efforts in
it choose to remain as a sociedad anonima, it could not become a developing the mines of the intervenor. Suffice it to say that whatever such
corporation. motives should be, they are wholly irrelevant to the issues in this appeal, that
exclusively concern the legal soundness of the order of
Having thus made its choice, Benguet may not now go back and seek to the Respondent Securities and Exchange Commissioner rejecting the claims
change its position and adopt the reformation that it had formerly repudiated. of the Benguet Consolidated Mining Company to extend its corporate life.
The election of one of several alternatives is irrevocable once made (as now
expressly recognized in article 940 of the new Civil Code of the Neither are we impressed by the prophesies of economic chaos that would
Philippines):chanroblesvirtuallawlibrary such rule is inherent in the nature of allegedly ensure with the cessation of Benguets activities. If its mining
the choice, its purpose being to clarify and render definite the rights of the properties are really susceptible of profitable operation, inexorable economic
one exercising the option, so that other persons may act in consequence. laws will ensure their exploitation; chan roblesvirtualawlibraryif, on the other
While successive choices may be provided there is nothing in section 75 of hand, they can no longer be worked at a profit, then catastrophe becomes
inevitable, whether or not Petitioner Benguet retains corporate existence.

232
Sustaining the opinions of the Respondent Securities and Exchange
Commissioner and of the Secretary of Justice, we rule
that:chanroblesvirtuallawlibrary

(1) The prohibition contained in section 18 of Act No. 1459, against


extending the period of corporate existence by amendment of the original
articles, was intended to apply, and does apply, to sociedades anonimas
already formed, organized and existing at the time of the effectivity of the
Corporation Law (Act No. 1459) in 1906;

(2) The statutory prohibition is valid and impairs no vested rights or


constitutional inhibition where no agreement to extend the original period of
corporate life was perfected before the enactment of the Corporation Law;

(3) A sociedad anonima, existing before the Corporation Law, that continues
to do business as such for a reasonable time after its enactments, is deemed
to have made its election and may not subsequently claim to reform into a
corporation under section 75 of Act No. 1459.

In view of the foregoing, the order appealed from is affirmed. Costs


against Petitioner-AppellantBenguet Consolidated Mining Company.

Padilla, Montemayor, Reyes, A. Labrador, Concepcion and


Endencia, JJ., concur.

233
G.R. No. 9321 September 24, 1914

NORBERTO ASUNCION, ET AL., petitioners-appellants,


vs.
MANUEL DE YRIARTE, respondent-appellee.

Modesto Reyes for appellants.


Attorney-General Villamor for appellee.

MORELAND, J.:

This is an action to obtain a writ of mandamus to compel the chief of the


division of achieves of the Executive Bureau to file a certain articles of
incorporation.

The chief of the division of archives, the respondent, refused to file the
articles of incorporation, hereinafter referred to, upon the ground that the
object of the corporation, as stated in the articles, was not lawful and that, in
pursuance of section 6 of Act No. 1459, they were not registerable.

234
The proposed incorporators began an action in the Court of First Instance of articles of incorporation presented foe registration are perfect in form, the
the city of Manila to compel the chief of the division of archives to receive division of archives must accept and register them and issue the
and register said articles of incorporation and to do any and all acts corresponding certificate of incorporation no matter what the purpose of the
necessary for the complete incorporation of the persons named in the corporation may be as expressed in the articles. We do not believe it was
articles. The court below found in favor of the defendant and refused to order intended that the division of archives should issue a certificate of
the registration of the articles mentioned, maintaining ad holding that the incorporation to, and thereby put the seal of approval of the Government
defendant, under the Corporation Law, had authority to determine both the upon, a corporation which was organized for base of immoral purposes. That
sufficiency of the form of the articles and the legality of the object of the such corporation might later, if it sought to carry out such purposes, be
proposed corporation. This appeal is taken from that judgment. dissolved, or its officials imprisoned or itself heavily fined furnished no reason
why it should have been created in the first instance. It seems to us to be not
The first question that arises is whether or not the chief of the division of only the right but the duty of the divisions of archives to determine the
archives has authority, under the Corporation for registration, to decide not lawfulness of the objects and purposes of the corporation before it issues a
only as to the sufficiency of the form of the articles, but also as to the certificate of incorporation.
lawfulness of the purpose of the proposed corporation.
It having determined that the division of archives, through its officials, has
It is strongly urged on the part of the appellants that the duties of the authority to determine not only the sufficiency as to form of the articles of
defendant are purely ministerial and that he has no authority to pass upon incorporation offered for registration, but also the lawfulness of the purposes
the lawfulness of the object for which the incorporators propose to organize. of leads us to the determination of the question whether or not the chief of
No authorities are cited to support this proposition and we are of the opinion the division of archives, who is the representative thereof and clothed by it
that it is not sound. with authority to deal subject to mandamus in the performance of his duties.

Section 6 of the Corporation Law reads in part as follows: We are of the opinion that he may be mandamused if he act in violation of
law or if he refuses, unduly, to comply with the law. While we have held that
Five or more persons, not exceeding fifteen, a majority of whom are defendant has power to pass upon the lawfulness of the purposes of the
residents of the Philippine Islands, may form a private corporation for proposed corporation and that he may, in the fulfillment of his duties,
any lawful purpose by filing with the division of archives, patents, determine the question of law whether or not those purposes are lawful and
copyrights, and trademarks if the Executive Bureau articles of embraced within that class concerning which the law permits corporations to
incorporation duly executed and acknowledged before a notary be formed, that does not necessarily mean, as we have already intimated,
public, . . . . that his duties are not ministerial. On the contrary, there is no incompatibility
in holding, as we do hold, that his duties are ministerial and that he has no
authority to exercise discretion in receiving and registering articles of
Simply because the duties of an official happens to be ministerial, it does not
incorporation. He may exercise judgment that is, the judicial function in
necessarily follow that he may not, in the administration of his office,
the determination of the question of law referred to, but he may not use
determine questions of law. We are of the opinion that it is the duty of the
discretion. The question whether or not the objects of a proposed corporation
division of archives, when articles of incorporation are presented for
are lawful is one that can be decided one way only. If he err in the
registration, to determine whether the objects of the corporation as
determination of that question and refuse to file articles which should be filed
expressed in the articles are lawful. We do not believe that, simply because
under the law, the decision is subject to review and correction and, upon

235
proper showing, he will be ordered to file the articles. This is the same kind of The municipality of Pasig as recognized by law contains within its limits
determination which a court makes when it decides a case upon the merits, several barrios or small settlements, like Pulo or San Miguel, which have no
the court makes when it decides a case upon the merits. When a case is local government of their own but are governed by the municipality of Pasig
presented to a court upon the merits, the court can decide only one way and through its municipal president and council. The president and members of
be right. As a matter of law, there is only one way and be right. As a matter of the municipal council are elected by a general vote of the municipality, the
law, there is only one course to pursue. In a case where the court or other qualified electors of all the barrios having the right to participate.
official has discretion in the resolution of a question, then, within certain
limitations, he may decide the question either way and still be right. The municipality of Pasig is a municipal corporation organized by law. It has
Discretion, it may be said generally, is a faculty conferred upon a court or the control of all property of the municipality. The various barrios of the
other official by which he may decide a question either way and still be right. municipality have no right to own or hold property, they not being recognized
The power conferred upon the division of archives with respect to the as legal entities by any law. The residents of the barrios participate in the
registration of articles of incorporation is not of that character. It is of the advantages which accrue to the municipality from public property and receive
same character as the determination of a lawsuit by a court upon the merits. all the benefits incident to residence in a municipality organized by law. If
It can be decided only one way correctly. there is any public property situated in the barrio of Pulo or San Miguel not
belonging to the general government or the province, it belongs to the
If, therefore, the defendant erred in determining the question presented when municipality of Pasig and the sole authority to manage and administer the
the articles were offered for registration, then that error will be corrected by same resides in that municipality. Until the present laws upon the subject are
this court in this action and he will be compelled to register the articles as charged no other entity can be the owner of such property or control or
offered. If, however, he did not commit an error, but decided that question administer it.
correctly, then, of course, his action will be affirmed to the extent that we will
deny the relief prayed for. The object of the proposed corporation, as appears from the articles offered
for registration, is to make of the barrio of Pulo or San Miguel a corporation
The next question leads us to the determination of whether or not the which will become the owner of and have the right to control and administer
purposes of the corporation as stated in the articles of incorporation are any property belonging to the municipality of Pasig found within the limits of
lawful within the meaning of the Corporation Law. that barrio. This clearly cannot be permitted. Otherwise municipalities as now
established by law could be deprived of the property which they now own
The purpose of the incorporation as stated in the articles is: "That the object and administer. Each barrio of the municipality would become under the
of the corporation is (a) to organize and regulate the management, scheme proposed, a separate corporation, would take over the ownership,
disposition, administration and control which the barrio of Pulo or San Miguel administration, and control of that portion of the municipal territory within its
or its inhabitants or residents have over the common property of said limits. This would disrupt, in a sense, the municipalities of the Islands by
residents or inhabitants or property belonging to the whole barrio as such; dividing them into a series of smaller municipalities entirely independent of
and (b) to use the natural products of the said property for institutions, the original municipality.
foundations, and charitable works of common utility and advantage to the
barrio or its inhabitants." What the law does not permit cannot be obtained by indirection. The object
of the proposed corporation is clearly repugnant to the provisions of the
Municipal Code and the governments of municipalities as they have been
organized thereunder. (Act No. 82, Philippine Commission.)

236
The judgment appealed from is affirmed, with costs against appellants. G.R. No. 117188 August 7, 1997

Arellano, C.J., Torres, Johnson, Carson and Araullo, JJ., concur. LOYOLA GRAND VILLAS HOMEOWNERS (SOUTH) ASSOCIATION,
INC., petitioner,
vs.
HON. COURT OF APPEALS, HOME INSURANCE AND GUARANTY
CORPORATION, EMDEN ENCARNACION and HORATIO
AYCARDO, respondents.

ROMERO, J.:

May the failure of a corporation to file its by-laws within one month from the
date of its incorporation, as mandated by Section 46 of the Corporation
Code, result in its automatic dissolution?

This is the issue raised in this petition for review on certiorari of the
Decision 1 of the Court of Appeals affirming the decision of the Home
Insurance and Guaranty Corporation (HIGC). This quasi-judicial body
recognized Loyola Grand Villas Homeowners Association (LGVHA) as the
sole homeowners' association in Loyola Grand Villas, a duly registered
subdivision in Quezon City and Marikina City that was owned and developed
by Solid Homes, Inc. It revoked the certificates of registration issued to
Loyola Grand Villas homeowners (North) Association Incorporated (the North
Association for brevity) and Loyola Grand Villas Homeowners (South)
Association Incorporated (the South Association).

LGVHAI was organized on February 8, 1983 as the association of


homeowners and residents of the Loyola Grand Villas. It was registered with
the Home Financing Corporation, the predecessor of herein respondent
HIGC, as the sole homeowners' organization in the said subdivision under
Certificate of Registration No. 04-197. It was organized by the developer of
the subdivision and its first president was Victorio V. Soliven, himself the
owner of the developer. For unknown reasons, however, LGVHAI did not file
its corporate by-laws.

237
Sometime in 1988, the officers of the LGVHAI tried to register its by-laws. WHEREFORE, judgment is hereby rendered recognizing the Loyola
They failed to do so. 2 To the officers' consternation, they discovered that Grand Villas Homeowners Association, Inc., under Certificate of
there were two other organizations within the subdivision the North Registration No. 04-197 as the duly registered and existing
Association and the South Association. According to private respondents, a homeowners association for Loyola Grand Villas homeowners, and
non-resident and Soliven himself, respectively headed these associations. declaring the Certificates of Registration of Loyola Grand Villas
They also discovered that these associations had five (5) registered Homeowners (North) Association, Inc. and Loyola Grand Villas
homeowners each who were also the incorporators, directors and officers Homeowners (South) Association, Inc. as hereby revoked or
thereof. None of the members of the LGVHAI was listed as member of the cancelled; that the receivership be terminated and the Receiver is
North Association while three (3) members of LGVHAI were listed as hereby ordered to render an accounting and turn-over to Loyola
members of the South Association. 3 The North Association was registered Grand Villas Homeowners Association, Inc., all assets and records of
with the HIGC on February 13, 1989 under Certificate of Registration No. 04- the Association now under his custody and possession.
1160 covering Phases West II, East III, West III and East IV. It submitted its
by-laws on December 20, 1988. The South Association appealed to the Appeals Board of the HIGC. In its
Resolution of September 8, 1993, the Board 4 dismissed the appeal for lack
In July, 1989, when Soliven inquired about the status of LGVHAI, Atty. of merit.
Joaquin A. Bautista, the head of the legal department of the HIGC, informed
him that LGVHAI had been automatically dissolved for two reasons. First, it Rebuffed, the South Association in turn appealed to the Court of Appeals,
did not submit its by-laws within the period required by the Corporation Code raising two issues. First, whether or not LGVHAI's failure to file its by-laws
and, second, there was non-user of corporate charter because HIGC had not within the period prescribed by Section 46 of the Corporation Code resulted
received any report on the association's activities. Apparently, this in the automatic dissolution of LGVHAI. Second, whether or not two
information resulted in the registration of the South Association with the homeowners' associations may be authorized by the HIGC in one "sprawling
HIGC on July 27, 1989 covering Phases West I, East I and East II. It filed its subdivision." However, in the Decision of August 23, 1994 being assailed
by-laws on July 26, 1989. here, the Court of Appeals affirmed the Resolution of the HIGC Appeals
Board.
These developments prompted the officers of the LGVHAI to lodge a
complaint with the HIGC. They questioned the revocation of LGVHAI's In resolving the first issue, the Court of Appeals held that under the
certificate of registration without due notice and hearing and concomitantly Corporation Code, a private corporation commences to have corporate
prayed for the cancellation of the certificates of registration of the North and existence and juridical personality from the date the Securities and Exchange
South Associations by reason of the earlier issuance of a certificate of Commission (SEC) issues a certificate of incorporation under its official seal.
registration in favor of LGVHAI. The requirement for the filing of by-laws under Section 46 of the Corporation
Code within one month from official notice of the issuance of the certificate of
On January 26, 1993, after due notice and hearing, private respondents incorporation presupposes that it is already incorporated, although it may file
obtained a favorable ruling from HIGC Hearing Officer Danilo C. Javier who its by-laws with its articles of incorporation. Elucidating on the effect of a
disposed of HIGC Case No. RRM-5-89 as follows: delayed filing of by-laws, the Court of Appeals said:

238
We also find nothing in the provisions cited by the petitioner, i.e., importantly, the South Association did not dispute the fact that LGVHAI had
Section 46 and 22, Corporation Code, or in any other provision of the been organized and that, thereafter, it transacted business within the period
Code and other laws which provide or at least imply that failure to file prescribed by law.
the by-laws results in an automatic dissolution of the corporation.
While Section 46, in prescribing that by-laws must be adopted within On the second issue, the Court of Appeals reiterated its previous ruling 5 that
the period prescribed therein, may be interpreted as a mandatory the HIGC has the authority to order the holding of a referendum to determine
provision, particularly because of the use of the word "must," its which of two contending associations should represent the entire community,
meaning cannot be stretched to support the argument that automatic village or subdivision.
dissolution results from non-compliance.
Undaunted, the South Association filed the instant petition for review
We realize that Section 46 or other provisions of the Corporation on certiorari. It elevates as sole issue for resolution the first issue it had
Code are silent on the result of the failure to adopt and file the by- raised before the Court of Appeals, i.e., whether or not the LGVHAI's failure
laws within the required period. Thus, Section 46 and other related to file its by-laws within the period prescribed by Section 46 of the
provisions of the Corporation Code are to be construed with Section Corporation Code had the effect of automatically dissolving the said
6 (1) of P.D. 902-A. This section empowers the SEC to suspend or corporation.
revoke certificates of registration on the grounds listed therein.
Among the grounds stated is the failure to file by-laws (see also II Petitioner contends that, since Section 46 uses the word "must" with respect
Campos: The Corporation Code, 1990 ed., pp. 124-125). Such to the filing of by-laws, noncompliance therewith would result in "self-
suspension or revocation, the same section provides, should be extinction" either due to non-occurrence of a suspensive condition or the
made upon proper notice and hearing. Although P.D. 902-A refers to occurrence of a resolutory condition "under the hypothesis that (by) the
the SEC, the same principles and procedures apply to the public issuance of the certificate of registration alone the corporate personality is
respondent HIGC as it exercises its power to revoke or suspend the deemed already formed." It asserts that the Corporation Code provides for a
certificates of registration or homeowners association. (Section 2 [a], "gradation of violations of requirements." Hence, Section 22 mandates that
E.O. 535, series 1979, transferred the powers and authorities of the the corporation must be formally organized and should commence
SEC over homeowners associations to the HIGC.) transaction within two years from date of incorporation. Otherwise, the
corporation would be deemed dissolved. On the other hand, if the
We also do not agree with the petitioner's interpretation that Section corporation commences operations but becomes continuously inoperative for
46, Corporation Code prevails over Section 6, P.D. 902-A and that five years, then it may be suspended or its corporate franchise revoked.
the latter is invalid because it contravenes the former. There is no
basis for such interpretation considering that these two provisions Petitioner concedes that Section 46 and the other provisions of the
are not inconsistent with each other. They are, in fact, Corporation Code do not provide for sanctions for non-filing of the by-laws.
complementary to each other so that one cannot be considered as However, it insists that no sanction need be provided "because the
invalidating the other. mandatory nature of the provision is so clear that there can be no doubt
about its being an essential attribute of corporate birth." To petitioner, its
The Court of Appeals added that, as there was no showing that the submission is buttressed by the facts that the period for compliance is
registration of LGVHAI had been validly revoked, it continued to be the duly "spelled out distinctly;" that the certification of the SEC/HIGC must show that
registered homeowners' association in the Loyola Grand Villas. More

239
the by-laws are not inconsistent with the Code, and that a copy of the by- personality of a corporation like the LGVHAI. This is so because Section 9 of
laws "has to be attached to the articles of incorporation." Moreover, no the Corporation Code provides that the corporate existence and juridical
sanction is provided for because "in the first place, no corporate identity has personality of a corporation begins from the date the SEC issues a certificate
been completed." Petitioner asserts that "non-provision for remedy or of incorporation under its official seal. Consequently, even if the by-laws have
sanction is itself the tacit proclamation that non-compliance is fatal and no not yet been filed, a corporation may be considered a de facto corporation.
corporate existence had yet evolved," and therefore, there was "no need to To emphasize the fact the LGVHAI was registered as the sole homeowners'
proclaim its demise." 6 In a bid to convince the Court of its arguments, association in the Loyola Grand Villas, private respondents point out that
petitioner stresses that: membership in the LGVHAI was an "unconditional restriction in the deeds of
sale signed by lot buyers."
. . . the word MUST is used in Sec. 46 in its universal literal meaning
and corollary human implication its compulsion is integrated in its In its reply to private respondents' comment on the petition, petitioner
very essence MUST is always enforceable by the inevitable reiterates its argument that the word " must" in Section 46 of the Corporation
consequence that is, "OR ELSE". The use of the word MUST in Code is mandatory. It adds that, before the ruling in Chung Ka Bio
Sec. 46 is no exception it means file the by-laws within one month v.Intermediate Appellate Court could be applied to this case, this Court must
after notice of issuance of certificate of registration OR ELSE. first resolve the issue of whether or not the provisions of P.D. No. 902-A
The OR ELSE, though not specified, is inextricably a part of MUST . prescribing the rules and regulations to implement the Corporation Code can
Do this or if you do not you are "Kaput". The importance of the by- "rise above and change" the substantive provisions of the Code.
laws to corporate existence compels such meaning for as decreed
the by-laws is "the government" of the corporation. Indeed, how can The pertinent provision of the Corporation Code that is the focal point of
the corporation do any lawful act as such without by-laws. Surely, no controversy in this case states:
law is indeed to create chaos. 7
Sec. 46. Adoption of by-laws. Every corporation formed under this
Petitioner asserts that P.D. No. 902-A cannot exceed the scope and power of Code, must within one (1) month after receipt of official notice of the
the Corporation Code which itself does not provide sanctions for non-filing of issuance of its certificate of incorporation by the Securities and
by-laws. For the petitioner, it is "not proper to assess the true meaning of Exchange Commission, adopt a code of by-laws for its government
Sec. 46 . . . on an unauthorized provision on such matter contained in the not inconsistent with this Code. For the adoption of by-laws by the
said decree." corporation, the affirmative vote of the stockholders representing at
least a majority of the outstanding capital stock, or of at least a
In their comment on the petition, private respondents counter that the majority of the members, in the case of non-stock corporations, shall
requirement of adoption of by-laws is not mandatory. They point to P.D. No. be necessary. The by-laws shall be signed by the stockholders or
902-A as having resolved the issue of whether said requirement is mandatory members voting for them and shall be kept in the principal office of
or merely directory. Citing Chung Ka Bio v. Intermediate Appellate the corporation, subject to the stockholders or members voting for
Court, 8 private respondents contend that Section 6(I) of that decree provides them and shall be kept in the principal office of the corporation,
that non-filing of by-laws is only a ground for suspension or revocation of the subject to inspection of the stockholders or members during office
certificate of registration of corporations and, therefore, it may not result in hours; and a copy thereof, shall be filed with the Securities and
automatic dissolution of the corporation. Moreover, the adoption and filing of Exchange Commission which shall be attached to the original
by-laws is a condition subsequent which does not affect the corporate articles of incorporation.

240
Notwithstanding the provisions of the preceding paragraph, by-laws On page 34, referring to the adoption of by-laws, are we made to
may be adopted and filed prior to incorporation; in such case, such understand here, Mr. Speaker, that by-laws must immediately be
by-laws shall be approved and signed by all the incorporators and filed within one month after the issuance? In other words, would this
submitted to the Securities and Exchange Commission, together with be mandatory or directory in character?
the articles of incorporation.
MR. MENDOZA. This is mandatory.
In all cases, by-laws shall be effective only upon the issuance by the
Securities and Exchange Commission of a certification that the by- MR. FUENTEBELLA. It being mandatory, Mr. Speaker, what would
laws are not inconsistent with this Code. be the effect of the failure of the corporation to file these by-laws
within one month?
The Securities and Exchange Commission shall not accept for filing
the by-laws or any amendment thereto of any bank, banking MR. MENDOZA. There is a provision in the latter part of the Code
institution, building and loan association, trust company, insurance which identifies and describes the consequences of violations of any
company, public utility, educational institution or other special provision of this Code. One such consequences is the dissolution of
corporations governed by special laws, unless accompanied by a the corporation for its inability, or perhaps, incurring certain penalties.
certificate of the appropriate government agency to the effect that
such by-laws or amendments are in accordance with law. MR. FUENTEBELLA. But it will not automatically amount to a
dissolution of the corporation by merely failing to file the by-laws
As correctly postulated by the petitioner, interpretation of this provision of law within one month. Supposing the corporation was late, say, five days,
begins with the determination of the meaning and import of the word "must" what would be the mandatory penalty?
in this section Ordinarily, the word "must" connotes an imperative act or
operates to impose a duty which may be enforced. 9 It is synonymous with MR. MENDOZA. I do not think it will necessarily result in the
"ought" which connotes compulsion or mandatoriness. 10 However, the word automatic or ipso facto dissolution of the corporation. Perhaps, as in
"must" in a statute, like "shall," is not always imperative. It may be consistent the case, as you suggested, in the case of El Hogar Filipino where
with an exercise of discretion. In this jurisdiction, the tendency has been to a quo warranto action is brought, one takes into account the gravity
interpret "shall" as the context or a reasonable construction of the statute in of the violation committed. If the by-laws were late the filing of the
which it is used demands or requires. 11 This is equally true as regards the by-laws were late by, perhaps, a day or two, I would suppose that
word "must." Thus, if the languages of a statute considered as a whole and might be a tolerable delay, but if they are delayed over a period of
with due regard to its nature and object reveals that the legislature intended months as is happening now because of the absence of a clear
to use the words "shall" and "must" to be directory, they should be given that requirement that by-laws must be completed within a specified
meaning. 12 period of time, the corporation must suffer certain consequences. 13

In this respect, the following portions of the deliberations of the Batasang This exchange of views demonstrates clearly that automatic corporate
Pambansa No. 68 are illuminating: dissolution for failure to file the by-laws on time was never the intention of the
legislature. Moreover, even without resorting to the records of deliberations of
MR. FUENTEBELLA. Thank you, Mr. Speaker.

241
the Batasang Pambansa, the law itself provides the answer to the issue functions of by-laws. Viewed in this light, the adoption of by-laws is a
propounded by petitioner. matter of practical, if not one of legal, necessity. Moreover, the
peculiar circumstances attending the formation of a corporation may
Taken as a whole and under the principle that the best interpreter of a statute impose the obligation to adopt certain by-laws, as in the case of a
is the statute itself (optima statuli interpretatix est ipsum statutum), 14 Section close corporation organized for specific purposes. And the statute or
46 aforequoted reveals the legislative intent to attach a directory, and not general laws from which the corporation derives its corporate
mandatory, meaning for the word "must" in the first sentence thereof. Note existence may expressly require it to make and adopt by-laws and
should be taken of the second paragraph of the law which allows the filing of specify to some extent what they shall contain and the manner of
the by-laws even prior to incorporation. This provision in the same section of their adoption. The mere fact, however, of the existence of power in
the Code rules out mandatory compliance with the requirement of filing the the corporation to adopt by-laws does not ordinarily and of necessity
by-laws "within one (1) month after receipt of official notice of the issuance of make the exercise of such power essential to its corporate life, or to
its certificate of incorporation by the Securities and Exchange Commission." the validity of any of its acts. 17
It necessarily follows that failure to file the by-laws within that period does not
imply the "demise" of the corporation. By-laws may be necessary for the Although the Corporation Code requires the filing of by-laws, it does not
"government" of the corporation but these are subordinate to the articles of expressly provide for the consequences of the non-filing of the same within
incorporation as well as to the Corporation Code and related the period provided for in Section 46. However, such omission has been
statutes. 15 There are in fact cases where by-laws are unnecessary to rectified by Presidential Decree No. 902-A, the pertinent provisions on the
corporate existence or to the valid exercise of corporate powers, thus: jurisdiction of the SEC of which state:

In the absence of charter or statutory provisions to the contrary, by- Sec. 6. In order to effectively exercise such jurisdiction, the
laws are not necessary either to the existence of a corporation or to Commission shall possess the following powers:
the valid exercise of the powers conferred upon it, certainly in all
cases where the charter sufficiently provides for the government of xxx xxx xxx
the body; and even where the governing statute in express terms
confers upon the corporation the power to adopt by-laws, the failure (1) To suspend, or revoke, after proper notice and hearing, the
to exercise the power will be ascribed to mere nonaction which will franchise or certificate of registration of corporations, partnerships or
not render void any acts of the corporation which would otherwise be associations, upon any of the grounds provided by law, including the
valid. 16 (Emphasis supplied.) following:

As Fletcher aptly puts it: xxx xxx xxx

It has been said that the by-laws of a corporation are the rule of its 5. Failure to file by-laws within the required period;
life, and that until by-laws have been adopted the corporation may
not be able to act for the purposes of its creation, and that the first
xxx xxx xxx
and most important duty of the members is to adopt them. This
would seem to follow as a matter of principle from the office and

242
In the exercise of the foregoing authority and jurisdiction of the As the "rules and regulations or private laws enacted by the corporation to
Commission or by a Commissioner or by such other bodies, boards, regulate, govern and control its own actions, affairs and concerns and its
committees and/or any officer as may be created or designated by stockholders or members and directors and officers with relation thereto and
the Commission for the purpose. The decision, ruling or order of any among themselves in their relation to it," 19 by-laws are indispensable to
such Commissioner, bodies, boards, committees and/or officer may corporations in this jurisdiction. These may not be essential to corporate birth
be appealed to the Commission sitting en banc within thirty (30) days but certainly, these are required by law for an orderly governance and
after receipt by the appellant of notice of such decision, ruling or management of corporations. Nonetheless, failure to file them within the
order. The Commission shall promulgate rules of procedures to period required by law by no means tolls the automatic dissolution of a
govern the proceedings, hearings and appeals of cases falling with corporation.
its jurisdiction.
In this regard, private respondents are correct in relying on the
The aggrieved party may appeal the order, decision or ruling of the pronouncements of this Court in Chung Ka Bio v.Intermediate Appellate
Commission sitting en banc to the Supreme Court by petition for Court, 20 as follows:
review in accordance with the pertinent provisions of the Rules of
Court. . . . . Moreover, failure to file the by-laws does not automatically
operate to dissolve a corporation but is now considered only a
Even under the foregoing express grant of power and authority, there can be ground for such dissolution.
no automatic corporate dissolutionsimply because the incorporators failed to
abide by the required filing of by-laws embodied in Section 46 of the Section 19 of the Corporation Law, part of which is now Section 22 of
Corporation Code. There is no outright "demise" of corporate existence. the Corporation Code, provided that the powers of the corporation
Proper notice and hearing are cardinal components of due process in any would cease if it did not formally organize and commence the
democratic institution, agency or society. In other words, the incorporators transaction of its business or the continuation of its works within two
must be given the chance to explain their neglect or omission and remedy years from date of its incorporation. Section 20, which has been
the same. reproduced with some modifications in Section 46 of the Corporation
Code, expressly declared that "every corporation formed under this
That the failure to file by-laws is not provided for by the Corporation Code but Act, must within one month after the filing of the articles of
in another law is of no moment. P.D. No. 902-A, which took effect incorporation with the Securities and Exchange Commission, adopt a
immediately after its promulgation on March 11, 1976, is very much apposite code of by-laws." Whether this provision should be given mandatory
to the Code. Accordingly, the provisions abovequoted supply the law or only directory effect remained a controversial question until it
governing the situation in the case at bar, inasmuch as the Corporation Code became academic with the adoption of PD 902-A. Under this decree,
and P.D. No. 902-A are statutes in pari materia. Interpretare et concordare it is now clear that the failure to file by-laws within the required period
legibus est optimus interpretandi. Every statute must be so construed and is only a ground for suspension or revocation of the certificate of
harmonized with other statutes as to form a uniform system of registration of corporations.
jurisprudence. 18
Non-filing of the by-laws will not result in automatic dissolution of the
corporation. Under Section 6(I) of PD 902-A, the SEC is empowered

243
to "suspend or revoke, after proper notice and hearing, the franchise SO ORDERED.
or certificate of registration of a corporation" on the ground inter
alia of "failure to file by-laws within the required period." It is clear Regalado, Puno and Mendoza, JJ., concur.
from this provision that there must first of all be a hearing to
determine the existence of the ground, and secondly, assuming such Torres, Jr., J., is on leave.
finding, the penalty is not necessarily revocation but may be only
suspension of the charter. In fact, under the rules and regulations of
the SEC, failure to file the by-laws on time may be penalized merely
with the imposition of an administrative fine without affecting the
corporate existence of the erring firm.

It should be stressed in this connection that substantial compliance


with conditions subsequent will suffice to perfect corporate
personality. Organization and commencement of transaction of
corporate business are but conditions subsequent and not
prerequisites for acquisition of corporate personality. The adoption
and filing of by-laws is also a condition subsequent. Under Section
19 of the Corporation Code, a Corporation commences its corporate
existence and juridical personality and is deemed incorporated from
the date the Securities and Exchange Commission issues certificate
of incorporation under its official seal. This may be done even before
the filing of the by-laws, which under Section 46 of the Corporation
Code, must be adopted "within one month after receipt of official
notice of the issuance of its certificate of incorporation." 21

That the corporation involved herein is under the supervision of the HIGC
does not alter the result of this case. The HIGC has taken over the
specialized functions of the former Home Financing Corporation by virtue of
Executive Order No. 90 dated December 17, 1989. 22 With respect to
homeowners associations, the HIGC shall "exercise all the powers,
authorities and responsibilities that are vested on the Securities and
Exchange Commission . . . , the provision of Act 1459, as amended by P.D.
902-A, to the contrary notwithstanding." 23

WHEREFORE, the instant petition for review on certiorari is hereby DENIED


and the questioned Decision of the Court of Appeals AFFIRMED. This
Decision is immediately executory. Costs against petitioner.

244
[G.R. No. 121466. August 15, 1997]

PMI COLLEGES, petitioner, vs. THE NATIONAL LABOR RELATIONS


COMMISSION and ALEJANDRO GALVAN, respondents.

DECISION

245
ROMERO, J.: the period covering October 1991 to September 1992; (2) shipyard and plant
visits and on-the-job training of Classes 41 and 42 for the period covering
Subject of the instant petition for certiorari under Rule 65 of the Rules of October 1991 to September 1992 on board M/V Sweet Glory vessel; and (3)
Court is the resolution[1] of public respondent National Labor Relations as Acting Director of Seaman Training Course for 3-1/2 months.
Commission[2] rendered on August 4, 1995, affirming in toto the December 7,
1994 decision[3] of Labor Arbiter Pablo C. Espiritu declaring petitioner PMI In support of the abovementioned claims, private respondent submitted
Colleges liable to pay private respondent Alejandro Galvan P405,000.00 in documentary evidence which were annexed to his complaint, such as the
unpaid wages and P40,532.00 as attorneys fees. detailed load and schedule of classes with number of class hours and rate
per hour (Annex A); PMI Colleges Basic Seaman Training Course (Annex B);
A chronicle of the pertinent events on record leading to the filing of the the aforementioned letter-request for payment of salaries by the Acting
instant petition is as follows: Director of PMI Colleges (Annex C); unpaid load of private respondent
(Annex D); and vouchers prepared by the accounting department of
On July 7, 1991, petitioner, an educational institution offering courses on petitioner but whose amounts indicated therein were actually never paid to
basic seamans training and other marine-related courses, hired private private respondent (Exhibit E).
respondent as contractual instructor with an agreement that the latter shall
be paid at an hourly rate of P30.00 to P50.00, depending on the description Private respondents claims, as expected, were resisted by petitioner. It
of load subjects and on the schedule for teaching the same. Pursuant to this alleged that classes in the courses offered which complainant claimed to
engagement, private respondent then organized classes in marine have remained unpaid were not held or conducted in the school premises of
engineering. PMI Colleges. Only private respondent, it was argued, knew whether classes
were indeed conducted. In the same vein, petitioner maintained that it
Initially, private respondent and other instructors were compensated for exercised no appropriate and proper supervision of the said classes which
services rendered during the first three periods of the abovementioned activities allegedly violated certain rules and regulations of the Department of
contract. However, for reasons unknown to private respondent, he stopped Education, Culture and Sports (DECS). Furthermore, the claims, according to
receiving payment for the succeeding rendition of services. This claim of non- petitioner, were all exaggerated and that, at any rate, private respondent
payment was embodied in a letter dated March 3, 1992, written by petitioners abandoned his work at the time he should have commenced the same.
Acting Director, Casimiro A. Aguinaldo, addressed to its President, Atty.
Santiago Pastor, calling attention to and appealing for the early approval and In reply, private respondent belied petitioners allegations contending,
release of the salaries of its instructors including that of private respondent. It among others, that he conducted lectures within the premises of petitioners
appeared further in said letter that the salary of private respondent rented space located at 5th Floor, Manufacturers Bldg., Sta. Cruz, Manila;
corresponding to the shipyard and plant visits and the ongoing on-the-job that his students duly enrolled with the Registrars Office of petitioner; that
training of Class 41 on board MV Sweet Glory of Sweet Lines, Inc. was not shipyard and plant visits were conducted at Fort San Felipe, Cavite Naval
yet included. This request of the Acting Director apparently went Base; that petitioner was fully aware of said shipyard and plant visits
unheeded. Repeated demands having likewise failed, private respondent because it even wrote a letter for that purpose; and that basic seaman
was soon constrained to file a complaint[4] before the National Capital Region courses 41 and 42 were sanctioned by the DECS as shown by the records of
Arbitration Branch on September 14, 1993 seeking payment for salaries the Registrars Office.
earned from the following: (1) basic seaman course Classes 41 and 42 for

246
Later in the proceedings below, petitioner manifested that Mr. Tomas G. IV. Whether the NLRC findings in its questioned resolution have
Cloma, Jr., a member of the petitioners Board of Trustees wrote a letter [5] to sound legal and factual support.
the Chairman of the Board on May 23, 1994, clarifying the case of private
respondent and stating therein, inter alia, that under petitioners by-laws only We see no compelling reason to grant petitioners plea; the same must,
the Chairman is authorized to sign any contract and that private respondent, therefore, be dismissed.
in any event, failed to submit documents on the alleged shipyard and plant
visits in Cavite Naval Base. At once, a mere perusal of the issues raised by petitioner already invites
dismissal for demonstrated ignorance and disregard of settled rules
Attempts at amicable settlement having failed, the parties were required on certiorari. Except perhaps for the third issue, the rest glaringly call for a re-
to submit their respective position papers. Thereafter, on June 16, 1994, the examination, evaluation and appreciation of the weight and sufficiency of
Labor Arbiter issued an order declaring the case submitted for decision on factual evidence presented before the Labor Arbiter. This, of course, the
the basis of the position papers which the parties filed. Petitioner, however, Court cannot do in the exercise of its certiorari jurisdiction without
vigorously opposed this order insisting that there should be a formal trial on transgressing the well-defined limits thereof. The corrective power of the
the merits in view of the important factual issues raised. In another order Court in this regard is confined only to jurisdictional issues and a
dated July 22, 1994, the Labor Arbiter impliedly denied petitioners opposition, determination of whether there is such grave abuse of discretion amounting
reiterating that the case was already submitted for decision. Hence, a to lack or excess of jurisdiction on the part of a tribunal or agency. So
decision was subsequently rendered by the Labor Arbiter on December 7, unyielding and consistent are the decisional rules thereon that it is indeed
1994 finding for the private respondent. On appeal, the NLRC affirmed the surprising why petitioners counsel failed to accord them the observance they
same in toto in its decision of August 4, 1995. deserve.

Aggrieved, petitioner now pleads for the Court to resolve the following Thus, in San Miguel Foods, Inc. Cebu B-Meg Feed Plant v. Hon.
issues in its favor, to wit: Bienvenido Laguesma,[6] we were emphatic in declaring that:

I. Whether the money claims of private respondent representing This Court is definitely not the proper venue to consider this matter for it
salaries/wages as contractual instructor for class instruction, is not a trier of facts. x x x Certiorari is a remedy narrow in its scope and
on-the-job training and shipboard and plant visits have valid inflexible in character. It is not a general utility tool in the legal
legal and factual bases; workshop. Factual issues are not a proper subject for certiorari, as the power
of the Supreme Court to review labor cases is limited to the issue of
II. Whether claims for salaries/wages for services relative to on-the- jurisdiction and grave abuse of discretion. x x x (Emphasis supplied).
job training and shipboard and plant visits by instructors,
assuming the same were really conducted, have valid bases; Of the same tenor was our disquisition in Ilocos Sur Electric
Cooperative, Inc. v. NLRC[7] where we made plain that:
III. Whether the petitioner was denied its right to procedural due
process; and In certiorari proceedings under Rule 65 of the Rules of Court, judicial review
by this Court does not go so far as to evaluate the sufficiency of evidence
upon which the Labor Arbiter and the NLRC based their determinations, the

247
inquiry being limited essentially to whether or not said public respondents In any event, granting that we may have to delve into the facts and
had acted without or in excess of its jurisdiction or with grave abuse of evidence of the parties, we still find no puissant justification for us to adjudge
discretion. (Emphasis supplied). both the Labor Arbiters and NLRCs appreciation of such evidence as
indicative of any grave abuse of discretion.
To be sure, this does not mean that the Court would disregard
altogether the evidence presented. We merely declare that the extent of First. Petitioner places so much emphasis on its argument that private
review of evidence we ordinarily provide in other cases is different when it is respondent did not produce a copy of the contract pursuant to which he
a special civil action of certiorari. The latter commands us to merely rendered services. This argument is, of course, puerile. The absence of such
determine whether there is basis established on record to support the copy does not in any manner negate the existence of a contract of
findings of a tribunal and such findings meet the required quantum of proof, employment since (C)ontracts shall be obligatory, in whatever form they have
which in this instance, is substantial evidence. Our deference to the expertise been entered into, provided all the essential requisites for their validity are
acquired by quasi-judicial agencies and the limited scope granted to us in the present.[9] The only exception to this rule is when the law requires that a
exercise of certiorari jurisdiction restrain us from going so far as to probe into contract be in some form in order that it may be valid or enforceable, or that a
the correctness of a tribunals evaluation of evidence, unless there is palpable contract be proved in a certain way. However, there is no requirement under
mistake and complete disregard thereof in which case certiorari would be the law that the contract of employment of the kind entered into by petitioner
proper. In plain terms, in certiorari proceedings, we are concerned with mere with private respondent should be in any particular form. While it may have
errors of jurisdiction and not errors of judgment. Thus: been desirable for private respondent to have produced a copy of his
contract if one really exists, but the absence thereof, in any case, does not
The rule is settled that the original and exclusive jurisdiction of this Court to militate against his claims inasmuch as:
review a decision of respondent NLRC (or Executive Labor Arbiter as in this
case) in a petition for certiorari under Rule 65 does not normally include an No particular form of evidence is required to prove the existence of an
inquiry into the correctness of its evaluation of the evidence. Errors of employer-employee relationship. Any competent and relevant evidence to
judgment, as distinguished from errors of jurisdiction, are not within the prove the relationship may be admitted. For, if only documentary evidence
province of a special civil action for certiorari, which is merely confined to would be required to show that relationship, no scheming employer would
issues of jurisdiction or grave abuse of discretion. It is thus incumbent upon ever be brought before the bar of justice, as no employer would wish to come
petitioner to satisfactorily establish that respondent Commission or executive out with any trace of the illegality he has authored considering that it should
labor arbiter acted capriciously and whimsically in total disregard of evidence take much weightier proof to invalidate a written instrument. x x x [10]
material to or even decisive of the controversy, in order that the extraordinary
writ of certiorari will lie. By grave abuse of discretion is meant such capricious At any rate, the vouchers prepared by petitioners own accounting
and whimsical exercise of judgment as is equivalent to lack of jurisdiction, department and the letter-request of its Acting Director asking for payment of
and it must be shown that the discretion was exercised arbitrarily or private respondents services suffice to support a reasonable conclusion that
despotically. For certiorari to lie there must be capricious, arbitrary and private respondent was employed with petitioner. How else could one explain
whimsical exercise of power, the very antithesis of the judicial prerogative in the fact that private respondent was supposed to be paid the amounts
accordance with centuries of both civil law and common law traditions. [8] mentioned in those documents if he were not employed? Petitioners
evidence is wanting in this respect while private respondent affirmatively
The Court entertains no doubt that the foregoing doctrines apply with stated that the same arose out of his employment with petitioner. As between
equal force in the case at bar. the two, the latter is weightier inasmuch as we accord affirmative testimony

248
greater value than a negative one. For the foregoing reasons, we find it Third. As regards the amounts demanded by private respondent, we
difficult to agree with petitioners assertion that the absence of a copy of the can only rely upon the evidence presented which, in this case, consists of the
alleged contract should nullify private respondents claims. computation of private respondent as well as the findings of both the Labor
Arbiter and the NLRC. Petitioner, it must be stressed, presented no
Neither can we concede that such contract would be invalid just satisfactory proof to the contrary. Absent such proof, we are constrained to
because the signatory thereon was not the Chairman of the Board which rely upon private respondents otherwise straightforward explanation of his
allegedly violated petitioners by-laws. Since by-laws operate merely as claims.
internal rules among the stockholders, they cannot affect or prejudice third
persons who deal with the corporation, unless they have knowledge of the Fourth. The absence of a formal hearing or trial before the Labor Arbiter
same.[11]No proof appears on record that private respondent ever knew is no cause for petitioner to impute grave abuse of discretion. Whether to
anything about the provisions of said by-laws. In fact, petitioner itself merely conduct one or not depends on the sole discretion of the Labor Arbiter, taking
asserts the same without even bothering to attach a copy or excerpt thereof into account the position papers and supporting documents submitted by the
to show that there is such a provision. How can it now expect the Labor parties on every issue presented. If the Labor Arbiter, in his judgment, is
Arbiter and the NLRC to believe it? That this allegation has never been confident that he can rely on the documents before him, he cannot be faulted
denied by private respondent does not necessarily signify admission of its for not conducting a formal trial anymore, unless it would appear that, in view
existence because technicalities of law and procedure and the rules of the particular circumstances of a case, the documents, without more, are
obtaining in the courts of law do not strictly apply to proceedings of this really insufficient.
nature.
As applied to the instant case, we can understand why the Labor Arbiter
Second. Petitioner bewails the fact that both the Labor Arbiter and the has opted not to proceed to trial, considering that private respondent, through
NLRC accorded due weight to the documents prepared by private annexes to his position paper, has adequately established that, first of all, he
respondent since they are said to be self-serving. Self-serving evidence is was an employee of petitioner; second, the nature and character of his
not to be literally taken as evidence that serves ones selfish interest. [12] The services, and finally, the amounts due him in consideration of his
fact alone that most of the documents submitted in evidence by private services. Petitioner, it should be reiterated, failed to controvert them. Actually,
respondent were prepared by him does not make them self-serving since it offered only four documents later in the course of the proceedings. It has
they have been offered in the proceedings before the Labor Arbiter and that only itself to blame if it did not attach its supporting evidence with its position
ample opportunity was given to petitioner to rebut their veracity and paper. It cannot now insist that there be a trial to give it an opportunity to
authenticity. Petitioner, however, opted to merely deny them which denial, ventilate what it should have done earlier. Section 3, Rule V of the New
ironically, is actually what is considered self-serving evidence [13] and, Rules of Procedure of the NLRC is very clear on the matter:
therefore, deserves scant consideration. In any event, any denial made by
petitioner cannot stand against the affirmative and fairly detailed manner by Section 3. x x x
which private respondent supported his claims, such as the places where he
conducted his classes, on-the-job training and shipyard and plant visits; the These verified position papers x x x shall be accompanied by all supporting
rate he applied and the duration of said rendition of services; the fact that he documents including the affidavits of their respective witnesses which shall
was indeed engaged as a contractual instructor by petitioner; and that part of take the place of the latters direct testimony. The parties shall thereafter not
his services was not yet remunerated. These evidence, to reiterate, have be allowed to allege facts, or present evidence to prove facts, not referred to
never been effectively refuted by petitioner.

249
and any cause or causes of action not included in the complaint or position what it perceives to be complex factual issues. We rule that the Labor Arbiter
papers, affidavits and other documents. x x x (Emphasis supplied). and the NLRC were not remiss in their duty to afford petitioner due
process. The essence of due process is merely that a party be afforded a
Thus, given the mandate of said rule, petitioner should have foreseen reasonable opportunity to be heard and to submit any evidence he may have
that the Labor Arbiter, in view of the non-litigious nature of the proceedings in support of his defense.[18]
before it, might not proceed at all to trial. Petitioner cannot now be heard to
complain of lack of due process. The following is apropos: WHEREFORE, in view of the foregoing, the instant petition is hereby
DISMISSED for lack of merit while the resolution of the National Labor
The petitioners should not have assumed that after they submitted their Relations Commission dated August 4, 1995 is hereby AFFIRMED.
position papers, the Labor Arbiter would call for a formal trial or hearing. The
holding of a trial is discretionary on the Labor Arbiter, it is not a matter of right SO ORDERED.
of the parties, especially in this case, where the private respondents had
already presented their documentary evidence. Regalado, (Chairman), Puno, and Mendoza, JJ., concur.

xxx Torres, Jr., J., on leave.

The petitioners did ask in their position paper for a hearing to thresh out
some factual matters pertinent to their case. However, they had no right or
reason to assume that their request would be granted. The petitioners should
have attached to their position paper all the documents that would prove their
claim in case it was decided that no hearing should be conducted or was
necessary. In fact, the rules require that position papers shall be
accompanied by all supporting documents, including affidavits of witnesses
in lieu of their direct testimony.[14]

It must be noted that adequate opportunity was given to petitioner in the


presentation of its evidence, such as when the Labor Arbiter granted
petitioners Manifestation and Motion[15]dated July 22, 1994 allowing it to
submit four more documents. This opportunity notwithstanding, petitioner still
failed to fully proffer all its evidence which might help the Labor Arbiter in
resolving the issues. What it desired instead, as stated in its petition, [16] was
to require presentation of witnesses buttressed by relevant documents in
support thereof. But this is precisely the opportunity given to petitioner when
the Labor Arbiter granted its Motion and Manifestation. It should have
presented the documents it was proposing to submit. The affidavits of its
witnesses would have sufficed in lieu of their direct testimony[17] to clarify

250
Bank of the Philippines (DBP) on January 3, 1962 in consideration of
the amount of P935,000.00. This mortgage was foreclosed. In the
foreclosure sale under Act No. 3135 held on October 25, 1974, the
said properties were awarded to Rosita Pea as highest bidder. A
certificate of sale was issued in her favor by the Senior Deputy
Sheriff of Pampanga, Edgardo A. Zabat, upon payment of the sum of
P128,000.00 to the Office of the Provincial Sheriff (Exh. 23). The
certificate of sale was registered on October 29, 1974 (Exh. G).

On November 19, 1974, the board of directors of PAMBUSCO,


through three (3) out of its five (5) directors, resolved to assign its
G.R. No. 91478 February 7, 1991 right of redemption over the aforesaid lots and authorized one of its
members, Atty. Joaquin Briones "to execute and sign a Deed of
ROSITA PEA petitioner, Assignment for and in behalf of PAMBUSCO in favor of any
vs. interested party . . ." (Exh. 24). Consequently, on March 18, 1975,
THE COURT OF APPEALS, SPOUSES RISING T. YAP and CATALINA Briones executed a Deed of Assignment of PAMBUSCO's
YAP, PAMPANGA BUS CO., INC., JESUS DOMINGO, JOAQUIN redemption right over the subject lots in favor of Marcelino Enriquez
BRIONES, SALVADOR BERNARDEZ, MARCELINO ENRIQUEZ and (Exh. 25). The latter then redeemed the said properties and a
EDGARDO A. ZABAT, respondents. certificate of redemption dated August 15, 1975 was issued in his
favor by Sheriff Zabat upon payment of the sum of one hundred forty
Cesar L. Villanueva for petitioner. thousand, four hundred seventy four pesos P140,474.00) to the
Martin N. Roque for private respondents. Office of the Provincial Sheriff of Pampanga (Exh. 26).

A day after the aforesaid certificate was issued, Enriquez executed a


deed of absolute sale of the subject properties in favor of plaintiffs-
appellants, the spouses Rising T. Yap and Catalina Lugue, for the
GANCAYCO, J.: sum of P140,000.00 (Exh. F).

The validity of the redemption of a foreclosed real property is the center of On August 18, 1975, a levy on attachment in favor of Capitol Allied
this controversy. Trading was entered as an additional encumbrance on TCT Nos.
4314, 4315 and 4316 and a Notice of a pending consulta was also
The facts as found by the respondent court are not disputed. annotated on the same titles concerning the Allied Trading case
entitled Dante Gutierrez, et al. vs. PAMBUSCO (Civil Case No. 4310)
in which the registrability of the aforesaid lots in the name of the
A reading of the records shows that [Pampanga Bus Co.]
spouses Yap was sought to be resolved (Exh. 20-F). The certificate
PAMBUSCO, original owners of the lots in question under TCT Nos.
of sale issued by the Sheriff in favor of defendant Pea, the
4314, 4315 and 4316, mortgaged the same to the Development
resolution of the PAMBUSCO's board of directors assigning its

251
redemption rights to any interested party, the deed of assignment (c) Deed of Sale dated August 16, 1975 executed by defendant
PAMBUSCO executed in favor of Marcelino B. Enriquez, the Marcelino Enriquez in favor of defendant Rising Yap. (Original
certificate of redemption issued by the Sheriff in favor of Enriquez as Record, p. 244)
well as the deed of absolute sale of the subject lots executed by
Enriquez in favor of the plaintiffs-appellants were all annotated on On November 17, 1975, the Land Registration Commission opined
the same certificates of title likewise on August 18, 1975. Also, on the under LRC Resolution No. 1029 that "the levy on attachment in favor
same date, the Office of the Provincial Sheriff of San Fernando, of Capitol Allied Trading (represented by Dante Gutierrez) should be
Pampanga informed defendant-appellee by registered mail "that the carried over on the new title that would be issued in the name of
properties under TCT Nos. 4314, 4315 and 4316 . . . . were all Rising Yap in the event that he is able to present the owner's
redeemed by Mr. Marcelino B. Enriquez on August 15,1975 . . . ;" duplicates of the certificates of title herein involved" (Exh. G).
and that she may now get her money at the Sheriffs Office (Exh. J
and J-1). Meanwhile, defendant Pea, through counsel, wrote the Sheriff
asking for the execution of a deed of final sale in her favor on the
On September 8, 1975, Pea wrote the Sheriff notifying him that the ground that "the one (1) year period of redemption has long elapsed
redemption was not valid as it was made under a void deed of without any valid redemption having been exercised;" hence she "will
assignment. She then requested the recall of the said redemption now refuse to receive the redemption money . . . (Exh. 28).
and a restraint on any registration or transaction regarding the lots in
question (Exh. 27). On Dec. 30, 1977, plaintiff Yap wrote defendant Pea asking
payment of back rentals in the amount of P42,750.00 "for the use
On Sept. 10, 1975, the CFI Branch III, Pampanga in the and occupancy of the land and house located at Sta. Lucia, San
aforementioned Civil Case No. 4310, entitled Dante Gutierrez, et al. Fernando, Pampanga," and informing her of an increase in monthly
vs. PAMBUSCO, et al., ordered the Register of Deeds of rental to P2,000; otherwise, to vacate the premises or face an
Pampanga . . . to desist from registering or noting in his registry of eviction cum collection suit (Exh. D).
property . . . any of the following documents under contract, until
further orders: In the meantime, the subject lots, formerly under TCT Nos. 4314,
4315 and 4316 were registered on June 16, 1978 in the name of the
(a) Deed of Assignment dated March 18, 1975 executed by the spouses Yap under TCT Nos. 148983-R, 148984-R and 148985-R,
defendant Pampanga Bus Company in virtue of a resolution of its with an annotation of a levy on attachment in favor of Capitol Allied
Board of Directors in favor of defendant Marcelino Enriquez; Trading. The LRC Resolution No. 1029 allowing the conditioned
registration of the subject lots in the name of the spouses Yap was
(b) A Certificate of Redemption issued by defendant Deputy Sheriff also annotated on TCT No. 4315 on June 16, 1978 and the notice of
Edgardo Zabat in favor of defendant Marcelino Enriquez dated a pending consulta noted thereon on August 18, 1975 was cancelled
August 15, 1975; on the same date.

252
No Trial on the merits was held concerning Civil Case No. 4310. In Enriquez who likewise could not have become [the] owner of the
an order dated February 17, 1983, the case was dismissed without properties in question by redeeming the same on August 18, 1975
prejudice. (Exh. 26) under an alleged[ly] void deed of assignment executed in
his favor on March 18, 1975 by the original owners of the land in
Despite the foregoing, defendant-appellee Pea remained in question, the PAMBUSCO. The defense was that since the deed of
possession of the lots in question hence, the spouses Yap were assignment executed by PAMBUSCO in favor of Enriquez was
prompted to file the instant case. 1 void ab initio for being an ultra vires act of its board of directors and,
for being without any valuable consideration, it could not have had
The antecedents of the present petition are as follows: any legal effect; hence, all the acts which flowed from it and all the
rights and obligations which derived from the aforesaid void deed are
likewise void and without any legal effect.
Plaintiffs-appellants, the spouses Rising T. Yap and Catalina Lugue,
are the registered owners of the lots in question under Transfer
Certificate of Title (TCT) Nos. 148983-R, 148984-R, 148985-R. In the Further, it was alleged in the same Answer that plaintiffs are buyers
complaint filed on December 15, 1978, appellants sought to recover in bad faith because they have caused the titles of the subject
possession over the subject lands from defendants Rosita Pea and properties with the Register of Deeds to be issued in their names
Washington Distillery on the ground that being registered owners, despite an order from the then CFI, Br. III, Pampanga in Civil Case
they have to enforce their right to possession against defendants No. 4310, entitled Dante Gutierrez, et al. vs. Pampanga Bus
who have been allegedly in unlawful possession thereof since Company, Inc., et al., to desist from registering or noting in his
October 1974 "when the previous owners assigned (their) right to registry of property . . . any of the above-mentioned documents
collect rentals . . . in favor of plaintiffs" (Record, p. 5). The amount under contest, until further orders. (Record, p. 11).
claimed as damages is pegged on the total amount of unpaid rentals
from October 1974 (as taken from the allegations in the complaint) For its part, defendant Washington Distillery stated that it has never
up to December 1978 at a monthly rate of P1,500.00 'and the further occupied the subject lots hence they should not have been
sum of P2,000.00 a month from January 1979 until the defendants impleaded in the complaint.
finally vacate the . . . premises in question with interest at the legal
rate (Record, p. 61). The defendants, therefore, prayed that the complaint be dismissed;
that the deed of assignment executed in favor of Marcelino Enriquez,
In their answer, defendants Rosita Pea and Washington Distillery the certificate of redemption issued by the Provincial Sheriff also in
denied the material allegations of the complaint and by way of an favor of Marcelino Enriquez, and the deed of sale of these parcels of
affirmative and special defense asserted that Pea is now the land executed by Marcelino Enriquez in favor of the plaintiffs herein
legitimate owner of the subject lands for having purchased the same be all declared null and void; and further, that TCT Nos. 148983-R,
in a foreclosure proceeding instituted by the DBP . . . against 148984-R and 148985-R, covering these parcels issued in the
PAMBUSCO . . . and no valid redemption having been effected plaintiffs name be cancelled and, in lieu thereof, corresponding
within the period provided by law. It was contended that plaintiffs certificates of title over these same parcels be issued in the name of
could not have acquired ownership over the subject properties under defendant Rosita Pea.
a deed of absolute sale executed in their favor by one Marcelino B.

253
Thereafter, the defendants with prior leave of court filed a third-party (e) TCT Nos. 148983-R, 148984-R and 148985-R of the
complaint third-party defendants PAMBUSCO, Jesus Domingo, Register of Deeds of Pampanga in the name of the plaintiffs
Joaquin Briones, Salvador Bernardez (as members of the Board of also covering these parcels.
Directors of PAMBUSCO), Marcelino Enriquez, and Deputy Sheriff
Edgardo Zabat of Pampanga. All these third-party defendants, how Third-party defendant Edgardo Zabat, in his capacity as
ever, were declared as in default for failure to file their answer, Deputy Sheriff of Pampanga is directed to execute in favor of
except Edgardo Zabat who did file his answer but failed to appear at defendant Rosita Pea the corresponding certificate of final
the pre-trial. sale involving the parcels bought by her in the auction sale
of October 25, 1974 for which a certificate of sale had been
After trial, a decision was rendered by the court in favor of the issued to her.
defendants-appellees, to wit:
Finally, the third-party defendants herein except Deputy
WHEREFORE, and in view of all the foregoing, judgment is Sheriff Edgardo Zabat are hereby ordered to pay the
hereby rendered dismissing the complaint filed by the defendants/third party plaintiffs, jointly and severally, the
plaintiffs against the defendants and declaring as null and amount of P10,000.00 as attorney's fees plus costs. 2
void the following:
Thus, an appeal from said judgment of the trial court was interposed by
(a) The resolution of the Board of Directors of PAMBUSCO private respondents to the Court of Appeals wherein in due course a decision
approved on November 19, 1974 assigning the was rendered on June 20, 1989, the dispositive part of which reads as
PAMBUSCO's right of redemption concerning the parcels follows:
involved herein
WHEREFORE, premises considered, the judgment of the trial court
(b) The deed of assignment dated March 18, 1975 executed on appeal is REVERSED. Defendant-appellee Pea is hereby
in favor of Marcelino Enriquez pursuant to the resolution ordered to vacate the lands in question and pay the plaintiffs-
referred to in the preceding paragraph; appellants the accrued rentals from October, 1974 in the amount of
P1,500.00 per month up to December, 1978 and the amount of
(c) The certificate of redemption dated August 15, 1975 P2,000.00 per month thereafter, until appellee finally vacate (sic) the
issued by Deputy Sheriff Edgardo Zabat in favor of Marcelino premises with interest at the legal rate.
Enriquez concerning these parcels;
SO ORDERED.3
(d) The deed of absolute sale dated August 15, 1975
executed by Marcelino Enriquez in favor of the plaintiffs A motion for reconsideration filed by the appellee was denied in a resolution
concerning the same parcels and dated December 27, 1989.

Hence, this petition for review on certiorari of said decision and resolution of
the appellate court predicated on the following assigned errors:

254
First Assignment of Error COMPLY WITH THE FORMALITIES MANDATORILY REQUIRED
UNDER THE LAW FOR DONATIONS.
THE RESPONDENT COURT OF APPEALS ERRED IN HOLDING
THAT THE TRIAL COURT HAD NO JURISDICTION TO RULE ON Sixth Assignment of Error
THE VALIDITY OF THE QUESTIONED RESOLUTION AND
TRANSFERS. THE RESPONDENT COURT OF APPEALS ERRED IN HOLDING
THAT RESPONDENTS YAP ARE PURCHASERS IN GOOD FAITH
Second Assignment of Error AND IN FURTHER HOLDING THAT IT WAS TOO LATE FOR
PETITIONER TO INTERPOSE THE ISSUE THAT RESPONDENTS
THE RESPONDENT COURT OF APPEALS ERRED IN HOLDING YAP WERE PURCHASERS IN BAD FAITH.
THAT PETITIONER HAS NO LEGAL STANDING TO ASSAIL THE
VALIDITY OF THE QUESTIONED RESOLUTION AND THE SERIES Seventh Assignment of Error
OF SUCCEEDING TRANSACTIONS LEADING TO THE
REGISTRATION OF THE SUBJECT PROPERTIES IN FAVOR OF THE RESPONDENT COURT OF APPEALS ERRED IN
THE RESPONDENTS YAP. REVERSING THE DECISION OF THE TRIAL COURT. 4

Third Assignment of Error The petition is impressed with merit.

THE RESPONDENT COURT OF APPEALS ERRED IN HOLDING First, the preliminary issues.
THAT THE RESOLUTION OF RESPONDENT PAMBUSCO,
ADOPTED ON 19 NOVEMBER 1974, ASSIGNING ITS RIGHT OF The respondent court ruled that the trial court has no jurisdiction to annul the
REDEMPTION IS NOT VOID OR AT THE VERY LEAST LEGALLY board resolution as the matter falls within the jurisdiction of the Securities
DEFECTIVE. and Exchange Commission (SEC) and that petitioner did not have the proper
standing to have the same declared null and void.
Fourth Assignment of Error
In Philex Mining Corporation vs. Reyes, 5
THE RESPONDENT COURT OF APPEALS ERRED IN HOLDING
THAT THE DEED OF ASSIGNMENT, DATED 8 MARCH 1975, IN this Court held that it is the fact of relationship between the parties that
FAVOR OF RESPONDENT ENRIQUEZ IS NOT VOID OR AT THE determines the proper and exclusive jurisdiction of the SEC to hear and
VERY LEAST VOIDABLE OR RESCISSIBLE. decide intra-corporate disputes; that unless the controversy has arisen
between and among stockholders of the corporation, or between the
Fifth Assignment of Error stockholders and the officers of the corporation, then the case is not within
the jurisdiction of the SEC. Where the issue involves a party who is neither a
THE RESPONDENT COURT OF APPEALS ERRED IN NOT stockholder or officer of the corporation, the same is not within the jurisdiction
HOLDING THAT THE QUESTIONED DEED OF ASSIGNMENT, of the SEC.
DATED 8 MARCH 1975, WAS VOID AB INITIO FOR FAILING TO

255
In Union Glass & Container Corporation vs. Securities and Exchange a result of the deed of sale entered into by and between defendant
Commission, 6 this Court defined the relationships which are covered within PHHC and defendant Melisenda L. Santos. We believe that the
"intra-corporate disputes" under Presidential Decree No. 902-A, as amended, plaintiff should be given a chance to present evidence to establish
as follows: that she suffered detriment and that she is entitled to relief.
(Emphasis supplied.)
Otherwise stated, in order that the SEC can take cognizance of a
case, the controversy must pertain to any of the following There can be no question in this case that the questioned resolution and
relationships (a) between the corporation, partnership or association series of transactions resulting in the registration of the properties in the
and the public; (b) between the corporation, partnership or name of respondent Yap spouses adversely affected the rights of petitioner
association and its stockholders, partners, members, or officers; (c) to the said properties. Consequently, petitioner has the legal standing to
between the corporation, partnership or association and the state in question the validity of said resolution and transactions.
so far as its franchise, permit or license to operate is concerned; and
(d) among the stockholders, partners or associates themselves. As to the question of validity of the board resolution of respondent
PAMBUSCO adopted on November 19, 1974, Section 4, Article III of the
In this case, neither petitioner nor respondents Yap spouses are stockholders amended by-laws of respondent PAMBUSCO, provides as follows:
or officers of PAMBUSCO. Consequently, the issue of the validity of the
series of transactions resulting in the subject properties being registered in Sec. 4. Notices of regular and special meetings of the Board of
the names of respondents Yap may be resolved only by the regular courts. Directors shall be mailed to each Director not less than five days
before any such meeting, and notices of special meeting shall state
Respondent court held that petitioner being a stranger to the questioned the purpose or purposes thereof Notices of regular meetings shall be
resolution and series of succeeding transactions has no legal standing to sent by the Secretary and notices of special meetings by the
question their validity. President or Directors issuing the call. No failure or irregularity of
notice of meeting shall invalidate any regular meeting or proceeding
In Teves vs. People's Homesite and Housing Corporation, 7 this Court held: thereat; Provided a quorum of the Board is present, nor of any
special meeting; Provided at least four Directors are present.
We note however, in reading the complaint that the plaintiff is (Emphasis supplied.) 8
seeking the declaration of the nullity of the deed of sale, not as a
party in the deed, or because she is obliged principally or subsidiarily The trial court in finding the resolution void held as follows:
under the deed, but because she has an interest that is affected by
the deed. This Court has held that a person who is not a party On the other hand, this Court finds merit in the position taken by the
obliged principally or subsidiarily in a contract may exercise an defendants that the questioned resolution should be declared invalid
action for nullity of the contract if he is prejudiced in his rights with it having been approved in a meeting attended by only 3 of the 5
respect to one of the contracting parties, and can show the detriment members of the Board of Directors of PAMBUSCO which attendance
which would positively result to him from the contract in which he is short of the number required by the by-laws of the corporation.
had no intervention, Indeed, in the case now before Us, the
complaint alleges facts which show that plaintiff suffered detriment as xxx xxx xxx

256
In the meeting of November 19, 1974 when the questioned Moreover, there is no categorical declaration in the by-laws that a
resolution was approved, the three members of the Board of failure to comply with the attendance requirement in a special
Directors of PAMBUSCO who were present were Jesus Domingo, meeting should make all the acts of the board therein null and
Joaquin Briones, and Salvador Bernardez The remaining 2 others, void ab initio. A cursory reading of the subject provision, as
namely: Judge Pio Marcos and Alfredo Mamuyac were both absent aforequoted, would show that its framers only intended to make
therefrom. voidable a board meeting held without the necessary compliance
with the attendance requirement in the by-laws. Just the use of the
As it becomes clear that the resolution approved on November 19, word "invalidate" already denotes a legal imputation of validity to the
1974 is null and void it having been approved by only 3 of the questioned board meeting absent its invalidation in the proceedings
members of the Board of Directors who were the only ones present prescribed by the corporation's by-laws and/or the general
at the said meeting, the deed of assignment subsequently executed incorporation law. More significantly, it should be noted that even if
in favor of Marcelino Enriquez pursuant to this resolution also the subject special meeting is itself declared void, it does not follow
becomes null and void. . . . 9 that the acts of the board therein are ipso facto void and without any
legal effect. Without the declaration of nullity of the subject board
However, the respondent court overturning said legal conclusions of the trial proceedings, its validity should be maintained and the acts borne out
court made the following disquisition: of it should be presumed valid. Considering that the subject special
board meeting has not been declared void in a proper proceeding,
nor even in the trial by the court below, there is no reason why the
It should be noted that the provision in Section 4, Article III of
acts of the board in the said special meeting should be treated as
PAMBUSCO's amended by-laws would apply only in case of a failure
void AB. initio. . . . 10
to notify the members of the board of directors on the holding of a
special meeting, . . . .
The Court disagrees.
In the instant case, however, there was no proof whatsoever, either
by way of documentary or testimonial evidence, that there was such The by-laws of a corporation are its own private laws which substantially
a failure or irregularity of notice as to make the aforecited provision have the same effect as the laws of the corporation. They are in effect,
apply. There was not even such an allegation in the Answer that written, into the charter. In this sense they become part of the fundamental
should have necessitated a proof thereof. The fact alone that only law of the corporation with which the corporation and its directors and officers
three (3) out of five (5) members of the board of directors attended must comply. 11
the subject special meeting, was not enough to declare the aforesaid
proceeding void ab initio, much less the board resolution borne out of Apparently, only three (3) out of five (5) members of the board of directors of
it, when there was no proof of irregularity nor failure of notice and respondent PAMBUSCO convened on November 19, 1974 by virtue of a
when the defense made in the Answer did not touch upon the said prior notice of a special meeting. There was no quorum to validly transact
failure of attendance. Therefore, the judgment declaring the nullity of business since, under Section 4 of the amended by-laws hereinabove
the subject board resolution must be set aside for lack of proof. reproduced, at least four (4) members must be present to constitute a
quorum in a special meeting of the board of directors of respondent
PAMBUSCO.

257
Under Section 25 of the Corporation Code of the Philippines, the articles of It is also undisputed that at the time of the passage of the questioned
incorporation or by-laws of the corporation may fix a greater number than the resolution, respondent PAMBUSCO was insolvent and its only remaining
majority of the number of board members to constitute the quorum necessary asset was its right of redemption over the subject properties. Since the
for the valid transaction of business. Any number less than the number disposition of said redemption right of respondent PAMBUSCO by virtue of
provided in the articles or by-laws therein cannot constitute a quorum and the questioned resolution was not approved by the required number of
any act therein would not bind the corporation; all that the attending directors stockholders under the law, the said resolution, as well as the subsequent
could do is to adjourn. 12 assignment executed on March 8, 1975 assigning to respondent Enriquez
the said right of redemption, should be struck down as null and void.
Moreover, the records show that respondent PAMBUSCO ceased to operate
as of November 15, 1949 as evidenced by a letter of the SEC to said Respondent court, in upholding the questioned deed of assignment, which
corporation dated April 17, 1980. 13 Being a dormant corporation for several appears to be without any consideration at all, held that the consideration
years, it was highly irregular, if not anomalous, for a group of three (3) thereof is the liberality of the respondent PAMBUSCO in favor of its former
individuals representing themselves to be the directors of respondent corporate officer, respondent Enriquez, for services rendered. Assuming this
PAMBUSCO to pass a resolution disposing of the only remaining asset of the to be so, then as correctly argued by petitioner, it is not just an ordinary deed
corporation in favor of a former corporate officer. of assignment, but is in fact a donation. Under Article 725 of the Civil Code,
in order to be valid, such a donation must be made in a public document and
As a matter of fact, the three (3) alleged directors who attended the special the acceptance must be made in the same or in a separate instrument. In the
meeting on November 19, 1974 were not listed as directors of respondent latter case, the donor shall be notified of the acceptance in an authentic form
PAMBUSCO in the latest general information sheet of respondent and such step must be noted in both instruments. 16
PAMBUSCO filed with the SEC dated 18 March 1951. 14 Similarly, the latest
list of stockholders of respondent PAMBUSCO on file with the SEC does not Non-compliance with this requirement renders the donation null and
show that the said alleged directors were among the stockholders of void. 17 Since undeniably the deed of assignment dated March 8, 1975 in
respondent PAMBUSCO. 15 question, 18 shows that there was no acceptance of the donation in the same
and in a separate document, the said deed of assignment is thus void ab
Under Section 30 of the then applicable Corporation Law, only persons who initio and of no force and effect.
own at least one (1) share in their own right may qualify to be directors of a
corporation. Further, under Section 28 1/2 of the said law, the sale or WHEREFORE, the petition is GRANTED. The questioned decision of the
disposition of an and/or substantially all properties of the corporation respondent Court of Appeals dated June 20, 1989 and its resolution dated
requires, in addition to a proper board resolution, the affirmative votes of the December 27, 1989 are hereby REVERSED AND SET ASIDE and another
stockholders holding at least two-thirds (2/3) of the voting power in the judgment is hereby rendered AFFIRMING in toto the decision of the trial
corporation in a meeting duly called for that purpose. No doubt, the court.
questioned resolution was not confirmed at a subsequent stockholders
meeting duly called for the purpose by the affirmative votes of the SO ORDERED.
stockholders holding at least two-thirds (2/3) of the voting power in the
corporation. The same requirement is found in Section 40 of the present Narvasa, Cruz, Grio-Aquino and Medialdea, JJ., concur.
Corporation Code.

258
G.R. No. 101699 March 13, 1996

BENJAMIN A. SANTOS, petitioner,


vs.
NATIONAL LABOR RELATIONS COMMISSION, HON. LABOR ARBITER
FRUCTUOSO T. AURELLANO and MELVIN D. MILLENA, respondents.

VITUG, J.:p

In a petition for certiorari under Rule 65 of the Rules of Court, petitioner


Benjamin A. Santos, former President of the Mana Mining and Development
Corporation ("MMDC"), questions the resolution of the National Labor
Relations Commission ("NLRC") affirming the decision of Labor Arbiter
Fructuoso T. Aurellano who, having held illegal the termination of
employment of private respondent Melvin D. Millena, has ordered petitioner
MMDC, as well as its president (herein petitioner) and the executive vice-
president in their personal capacities, to pay Millena his monetary claims.

259
Private respondent, on 01 October 1985, was hired to be the project Rodillano E. Velasquez. Private respondent requested that he be reimbursed
accountant for MMDC's mining operations in Gatbo, Bacon, Sorsogon. On 12 the "advances" he had made for the company and be paid his "accrued
August 1986, private respondent sent to Mr. Gil Abao, the MMDC corporate salaries/claims. 3
treasurer, a memorandum calling the latter's attention to the failure of the
company to comply with the withholding tax requirements of, and to make the The claim was not heeded; on 20 October 1986, private respondent filed with
corresponding monthly remittances to, the Bureau of Internal Revenue the NLRC Regional Arbitration, Branch No. V, in Legazpi City, a complaint for
("BIR") on account of delayed payments of accrued salaries to the company's illegal dismissal, unpaid salaries, 13th month pay, overtime pay, separation
laborers and employees. 1 pay and incentive leave pay against MMDC and its two top officials, namely,
herein petitioner Benjamin A. Santos (the President) and Rodillano A.
In a letter, dated 08 September 1986, Abao advised private respondent Velasquez (the executive vice-president). in his complaint-affidavit (position
thusly: paper), submitted on 27 October 1986, Millena alleged, among other things,
that his dismissal was merely an offshoot of his letter of 12 August 1986 to
Regarding Gatbo operations, as you also are aware, the Abao about the company's inability to pay its workers and to remit
rainy season is now upon us and the peace and order withholding taxes to the BIR. 4
condition in Sorsogon has deteriorated. It is therefore, the
board's decision that it would be useless for us to continue A copy of the notice and summons was served on therein respondents
operations, especially if we will always be in the "hole," so to (MMDC, Santos and Velasquez) on 29 October 1986. 5 At the initial hearing
speak. Our first funds receipts will be used to pay all our on 14 November 1986 before the Labor Arbiter, only the complainant,
debts. We will stop production until the advent of the dry Millena, appeared; however, Atty. Romeo Perez, in representation of the
season, and until the insurgency problem clears. We will respondents, requested by telegram that the hearing be reset to 01
undertake only necessary maintenance and repair work and December 1986. Although the request was granted by the Labor Arbiter,
will keep our overhead down to the minimum manageable private respondent was allowed, nevertheless, to present his evidence ex
level. Until we resume full-scale operations, we will not need parte at that initial hearing.
a project accountant as there will be very little paper work at
the site, which can be easily handled at Makati. The scheduled 01st December 1986 hearing was itself later reset to 19
December 1986. On 05 December 1986, the NLRC in Legazpi City again
We appreciate the work you have done for Mana and we will received a telegram from Atty. Perez asking for fifteen (15) days within which
not hesitate to take you back when we resume work at to submit the respondents' position paper. On 19 December 1986, Atty. Perez
Gatbo. However it would be unfair to you if we kept you in sent yet another telegram seeking a further postponement of the hearing and
the payroll and deprive you of the opportunity to earn more, asking for a period until 15 January 1987 within which to submit the position
during this period of Mana's crisis. 2 paper.

Private respondent expressed "shock" over the termination of his On 15 January 1987, Atty. Perez advised the NLRC in Legazpi City that the
employment. He complained that he would not have resigned from the Sycip, position paper had finally been transmitted through the mail and that he was
Gorres & Velayo accounting firm, where he was already a senior staff auditor, submitting the case for resolution without further hearing. The position paper
had it not been for the assurance of a "continuous job" by MMDC's Engr. was received by the Legazpi City NLRC office on 19 January 1987.

260
Complainant Millena filed, on 26 February 1987, his rejoinder to the position A writ of execution correspondingly issued; however, it was returned
paper. unsatisfied for the failure of the sheriff to locate the offices of the corporation
in the address indicated. Another writ of execution and an order of
On 27 July 1988, Labor Arbiter Fructuoso T. Aurellano, finding no valid cause garnishment was thereupon served on petitioner at his residence.
for terminating complainant's employment, ruled, citing this Court's
pronouncement in Construction & Development Corporation of the Contending that he had been denied due process, petitioner filed a motion
Philippines vs. Leogardo, Jr. 6 that a partial closure of an establishment due for reconsideration of the NLRC's resolution along with a prayer for the
to losses was a retrenchment measure that rendered the employer liable for quashal of the writ of execution and order of garnishment. He averred that he
unpaid salaries and other monetary claims. The Labor Arbiter adjudged had never received any notice, summons or even a copy of the complaint;
hence, he said, the Labor Arbiter at no time had acquired jurisdiction over
WHEREFORE, the respondents are hereby ordered to pay him.
the petitioner the amount of P37,132.25 corresponding to the
latter's unpaid salaries and advances; P5,400.00 for On 16 August 1991, the NLRC 11 dismissed the motion for reconsideration.
petitioner's 13th month pay; P3,340.95 as service incentive Citing Section 2, Rule 13, 12 and Section 13, Rule 14, 13 of the Rules of Court,
leave pay; and P5,400.00 as separation pay. The it ruled that the Regional Arbitration office had not, in fact, been remiss in the
respondents are further ordered to pay the petitioner 10% of observance of the legal processes for acquiring jurisdiction over the case and
the monetary awards as attorney's fees. over the persons of the respondents therein. The NLRC was also convinced
that Atty. Perez had been the authorized counsel of MMDC and its two most
All other claims are dismissed for lack of sufficient evidence. ranking officers.

SO ORDERED. 7 In holding petitioner personally liable for private respondent's claim, the
NLRC cited Article 289 14 of the Labor Code and the ruling in A.C. Ransom
Alleging abuse of discretion by the Labor Arbiter, the company and its co- Labor Union-CCLU vs. NLRC 15 to the effect that "(t)he responsible officer of
respondents filed a "motion for reconsideration and/or appeal. 8 The an employer corporation (could) be held personally, not to say even
motion/appeal was forthwith indorsed to the Executive Director of the NLRC criminally, liable for non-payment of backwages," and that
in Manila. of Gudez vs. NLRC 16 which amplified that "where the employer corporation
(was) no longer existing and unable to satisfy the judgment in favor of the
employee, the officer should be liable for acting on behalf of the corporation.
In a resolution, dated 04 September 1989, the NLRC 9 affirmed the decision
of the Labor Arbiter. It held that the reasons relied upon by MMDC and its co-
respondents in the dismissal of Millena, i.e., the rainy season, deteriorating In the instant petition for certiorari, petitioner Santos reiterates that he should
peace and order situation and little paperwork, were "not causes mentioned not have been adjudged personally liable by public respondents, the latter
under Article 282 of the Labor Code of the Philippines" and that Millena, not having validly acquired jurisdiction over his person whether by personal
being a regular employee, was "shielded by the tenurial clause mandated service of summons or by substituted service under Rule 19 of the Rules of
under the law. 10 Court.

261
Petitioner's contention is unacceptable. The fact that Atty. Romeo B. Perez arise when a corporation is used to evade a just and due obligation or to
has been able to timely ask for a deferment of the initial hearing on 14 justify a wrong, 26 to shield or perpetrate fraud, 27 to carry out similar other
November 1986, coupled with his subsequent active participation in the unjustifable aims or intentions, or as a subterfuge to commit injustice and so
proceedings, should disprove the supposed want of service of legal process. circumvent the law. 28 In Tramat Mercantile, Inc., vs. Court of Appeals, 29 the
Although as a rule, modes of service of summons are strictly followed in Court has collated the settled instances when, without necessarily piercing
order that the court may acquire jurisdiction over the person of a the veil of corporate fiction, personal civil liability can also be said to lawfully
defendant, 17such procedural modes, however, are liberally construed attach to a corporate director, trustee or officer; to wit: When
in quasi-judicial proceedings, substantial compliance with the same being
considered adequate. 18 Moreover, jurisdiction over the person of the (1) He assents (a) to a patently unlawful act of the
defendant in civil cases is acquired not only by service of summons but also corporation, or (b) for bad faith or gross negligence in
by voluntary appearance in court and submission to its directing its affairs, or (c) for conflict of interest, resulting in
authority. 19 "Appearance" by a legal advocate is such "voluntary submission damages to the corporation, its stockholders or other
to a court's jurisdiction." 20 It may be made not only by actual physical persons;
appearance but likewise by the submission of pleadings in compliance with
the order of the court or tribunal. (2) He consents to the issuance of watered stocks or who,
having knowledge thereof, does not forthwith file with the
To say that petitioner did not authorize Atty. Perez to represent him in the corporate secretary his written objection thereto;
case 21 is to unduly tax credulity. Like the Solicitor General, the Court likewise
considers it unlikely that Atty. Perez would have been so irresponsible as to (3) He agrees to hold himself personally and solidarily liable
represent petitioner if he were not, in fact, authorized. 22 Atty. Perez is an with the corporation; or
officer of the court, and he must be presumed to have acted with due
propriety. The employment of a counsel or the authority to employ an
(4) He is made, by a specific provision of law, to personally
attorney, it might be pointed out, need not be proved in writing; such fact
answer for his corporate action.
could be inferred from circumstantial evidence. 23 Petitioner was not just an
ordinary official of the MMDC; he was the President of the company.
The case of petitioner is way off these exceptional instances. It is not
even shown that petitioner has had a direct hand in the dismissal of
Petitioner, in any event, argues that public respondents have gravely abused
private respondent enough to attribute to him (petitioner) a patently
their discretion "in finding petitioner solidarily liable with MMDC even (in) the
unlawful act while acting for the corporation. Neither can Article
absence of bad faith and malice on his part." 24 There is merit in this plea.
289 30 of the Labor Code be applied since this law specifically refers
only to the imposition of penalties under the Code. It is undisputed
A corporation is a juridical entity with legal personality separate and distinct that the termination of petitioner's employment has, instead, been
from those acting for and in its behalf and, in general, from the people due, collectively, to the need for a further mitigation of losses, the
comprising it. The rule is that obligations incurred by the corporation, acting onset of the rainy season, the insurgency problem in Sorsogon and
through its directors, officers and employees, are its sole liabilities. the lack of funds to further support the mining operation in Gatbo.
Nevertheless, being a mere fiction of law, peculiar situations or valid grounds
can exist to warrant, albeit done sparingly, the disregard of its independent
being and the lifting of the corporate veil. 25 As a rule, this situation might

262
It is true, there were various cases when corporate officers were themselves being owner of one-half (1/2) interest of said corporation,
held by the Court to be personally accountable for the payment of wages and and his alleged arbitrary dismissal of private respondents.
money claims to its employees. In A.C. Ransom Labor Union-CCLU
vs. NLRC, 31 for instance, the Court ruled that under the Minimum Wage Law, Petitioner Sunio was impleaded in the Complaint in his
the responsible officer of an employer corporation could be held personally capacity as General Manager of petitioner corporation. There
liable for nonpayment of backwages for "(i)f the policy of the law were appears to be no evidence on record that he acted
otherwise, the corporation employer (would) have devious ways for evading maliciously or in bad faith in terminating the services of
payment of back wages." In the absence of a clear identification of the officer private respondents. His act, therefore, was within the scope
directly responsible for failure to pay the backwages, the Court considered of his authority and was a corporate act.
the President of the corporation as such officer. The case was cited in Chua
vs. NLRC 32 in holding personally liable the vice-president of the company, It is basic that a corporation is invested by law with a
being the highest and most ranking official of the corporation next to the personality separate and distinct from those of the persons
President who was dismissed, for the latter's claim for unpaid wages. composing it as well as from that of any other legal entity to
which it may be related. Mere ownership by a single
A review of the above exceptional cases would readily disclose the stockholder or by another corporation of all or nearly all of
attendance of facts and circumstances that could rightly sanction personal the capital stock of a corporation is not of itself sufficient
liability an the part of the company officer. In A.C. Ransom, the corporate ground for disregarding the separate corporate personality.
entity was a family corporation and execution against it could not be Petitioner Sunio, therefore, should not have been made
implemented because of the disposition posthaste of its leviable assets personally answerable for the payment of private
evidently in order to evade its just and due obligations. The doctrine of respondents' back salaries.
"piercing the veil of corporate fiction" was thus clearly appropriate. Chua
likewise involved another family corporation, and this time the conflict was The Court, to be sure, did appear to have deviated somewhat in Gudez
between two brothers occupying the highest ranking positions in the vs. NLRC; 34 however, it should be clear from our recent pronouncement
company. There were incontrovertible facts which pointed to extreme in Mam Realty Development Corporation and Manuel Centeno
personal animosity that resulted, evidently in bad faith, in the easing out from vs. NLRC 35 that the Sunio doctrine still prevails.
the company of one of the brothers by the other.
WHEREFORE, the instant petition for certiorari is given DUE COURSE and
The basic rule is still that which can be deduced from the Court's the decision of the Labor Arbiter, affirmed by the NLRC, is hereby MODIFIED
pronouncement in Sunio vs. National Labor Relations Commission; 33 thus: insofar as it holds herein petitioner Benjamin Santos personally liable with
Mana Mining and Development Corporation, which portion of the questioned
We come now to the personal liability of petitioner, Sunio, judgment is now SET ASIDE. In all other respects, the questioned decision
who was made jointly and severally responsible with remains unaffected. No costs.
petitioner company and CIPI for the payment of the
backwages of private respondents. This is reversible error. SO ORDERED.
The Assistant Regional Director's Decision failed to disclose
the reason why he was made personally liable.
Padilla, Bellosillo, Kapunan and Hermosisima, Jr., JJ., concur.
Respondents, however, alleged as grounds thereof, his the

263
264
G.R. No. L-18216 October 30, 1962

STOCKHOLDERS OF F. GUANZON AND SONS, INC., petitioners-


appellants,
vs.
REGISTER OF DEEDS OF MANILA, respondent-appellee.

Ramon C. Fernando for petitioners-appellants.


Office of the Solicitor General for respondent-appellee.

BAUTISTA ANGELO, J.:

On September 19, 1960, the five stockholders of the F. Guanzon and Sons,
Inc. executed a certificate of liquidation of the assets of the corporation
reciting, among other things, that by virtue of a resolution of the stockholders
adopted on September 17, 1960, dissolving the corporation, they have
distributed among themselves in proportion to their shareholdings, as
liquidating dividends, the assets of said corporation, including real properties
located in Manila.

265
The certificate of liquidation, when presented to the Register of Deeds of The Commissioner of Land Registration, however, entertained a different
Manila, was denied registration on seven grounds, of which the following opinion. He concurred in the view expressed by the register of deed to the
were disputed by the stockholders: effect that the certificate of liquidation in question, though it involves a
distribution of the corporation's assets, in the last analysis represents a
3. The number of parcels not certified to in the acknowledgment; transfer of said assets from the corporation to the stockholders. Hence, in
substance it is a transfer or conveyance.
5. P430.50 Reg. fees need be paid;
We agree with the opinion of these two officials. A corporation is a juridical
6. P940.45 documentary stamps need be attached to the document; person distinct from the members composing it. Properties registered in the
name of the corporation are owned by it as an entity separate and distinct
from its members. While shares of stock constitute personal property they do
7. The judgment of the Court approving the dissolution and directing
not represent property of the corporation. The corporation has property of its
the disposition of the assets of the corporation need be presented
own which consists chiefly of real estate (Nelson v. Owen, 113 Ala., 372, 21
(Rules of Court, Rule 104, Sec. 3).
So. 75; Morrow v. Gould, 145 Iowa 1, 123 N.W. 743). A share of stock only
typifies an aliquot part of the corporation's property, or the right to share in its
Deciding the consulta elevated by the stockholders, the Commissioner of proceeds to that extent when distributed according to law and equity (Hall &
Land Registration overruled ground No. 7 and sustained requirements Nos. Faley v. Alabama Terminal, 173 Ala 398, 56 So., 235), but its holder is not the
3, 5 and 6. owner of any part of the capital of the corporation (Bradley v. Bauder 36 Ohio
St., 28). Nor is he entitled to the possession of any definite portion of its
The stockholders interposed the present appeal. property or assets (Gottfried v. Miller, 104 U.S., 521; Jones v. Davis, 35 Ohio
St., 474). The stockholder is not a co-owner or tenant in common of the
As correctly stated by the Commissioner of Land Registration, the propriety corporate property (Halton v. Hohnston, 166 Ala 317, 51 So 992).
or impropriety of the three grounds on which the denial of the registration of
the certificate of liquidation was predicated hinges on whether or not that On the basis of the foregoing authorities, it is clear that the act of liquidation
certificate merely involves a distribution of the corporation's assets or should made by the stockholders of the F. Guanzon and Sons, Inc. of the latter's
be considered a transfer or conveyance. assets is not and cannot be considered a partition of community property, but
rather a transfer or conveyance of the title of its assets to the individual
Appellants contend that the certificate of liquidation is not a conveyance or stockholders. Indeed, since the purpose of the liquidation, as well as the
transfer but merely a distribution of the assets of the corporation which has distribution of the assets of the corporation, is to transfer their title from the
ceased to exist for having been dissolved. This is apparent in the minutes for corporation to the stockholders in proportion to their shareholdings, and
dissolution attached to the document. Not being a conveyance the certificate this is in effect the purpose which they seek to obtain from the Register of
need not contain a statement of the number of parcel of land involved in the Deeds of Manila, that transfer cannot be effected without the
distribution in the acknowledgment appearing therein. Hence the amount of corresponding deed of conveyance from the corporation to the stockholders.
documentary stamps to be affixed thereon should only be P0.30 and not It is, therefore, fair and logical to consider the certificate of liquidation as one
P940.45, as required by the register of deeds. Neither is it correct to require in the nature of a transfer or conveyance.
appellants to pay the amount of P430.50 as registration fee.

266
WHEREFORE, we affirm the resolution appealed from, with costs against 2. The trial court erred in not holding that the interest on bonds and
appellants. other indebtedness of the plaintiff corporation, paid by it outside of
the Philippine Islands to corporations not residing therein, were not,
Labrador, Concepcion, Reyes, J.B.L., Paredes, Dizon, Regala and on the part of the recipients thereof, income from Philippine sources,
Makalintal, JJ., concur. and hence not subject to Philippine income tax.
Barrera, J., took no part.
The facts, as stated by the appellant and as accepted by the appellee, may
be summarized as follows: The plaintiff is a corporation organized under the
laws of the Philippine Islands. It operates a gas plant in the City of Manila
and furnishes gas service to the people of the metropolis and surrounding
municipalities by virtue of a franchise granted to it by the Philippine
Government. Associated with the plaintiff are the Islands Gas and Electric
G.R. No. L-42780 January 17, 1936
Company domiciled in New York, United States, and the General Finance
Company domiciled in Zurich, Switzerland. Neither of these last mentioned
MANILA GAS CORPORATION, plaintiff-appellant, corporations is resident in the Philippines.
vs.
THE COLLECTOR OF INTERNAL REVENUE, defendant-appellee.
For the years 1930, 1931, and 1932, dividends in the sum of P1,348,847.50
were paid by the plaintiff to the Islands Gas and Electric Company in the
DeWitt, Perkins and Ponce Enrile for appellant. capacity of stockholders upon which withholding income taxes were paid to
Office of the Solicitor-General Hilado for appellee. the defendant totalling P40,460.03 For the same years interest on bonds in
the sum of P411,600 was paid by the plaintiff to the Islands Gas and Electric
MALCOLM, J.: Company upon which withholding income taxes were paid to the defendant
totalling P12,348. Finally for the stated time period, interest on other
This is an action brought by the Manila Gas Corporation against the Collector indebtedness in the sum of P131,644,90 was paid by the plaintiff to the
of Internal Revenue for the recovery of P56,757.37, which the plaintiff was Islands Gas and Electric Company and the General Finance Company
required by the defendant to deduct and withhold from the various sums paid respectively upon which withholding income taxes were paid to the defendant
it to foreign corporations as dividends and interest on bonds and other totalling P3,949.34.
indebtedness and which the plaintiff paid under protest. On the trial court
dismissing the complaint, with costs, the plaintiff appealed assigning as the Some uncertainty existing regarding the place of payment, we will not go into
principal errors alleged to have been committed the following: this factor of the case at this point, except to remark that the bonds and other
tokens of indebtedness are not to be found in the record. However, Exhibits
1. The trial court erred in holding that the dividends paid by the E, F, and G, certified correct by the Treasurer of the Manila Gas Corporation,
plaintiff corporation were subject to income tax in the hands of its purport to prove that the place of payment was the United States and
stockholders, because to impose the tax thereon would be to impose Switzerland.
a tax on the plaintiff, in violation of the terms of its franchise, and
would, moreover, be oppressive and inequitable.

267
The appeal naturally divides into two subjects, one covered by the first income tax, the exemption clause in the charter of the corporation
assigned error, and the other by the second assigned error. We shall discuss notwithstanding.
these subjects and errors in order.
For the foreign reasons, we are led to sustain the decision of the trial
1. Appellant first contends that the dividends paid by it to its court and to overrule appellant's first assigned error.
stockholders, the Islands Gas and Electric Company , were not
subject to tax because to impose a tax thereon would be to do so on 2. In support of its second assignment of error, appellant contends
the plaintiff corporation, in violation of the terms of its franchise and that, as the Islands Gas and Electric Company and the General
would, moreover, be oppressive and inequitable. This argument is Finance Company are domiciled in the United States and
predicated on the constitutional provision that no law impairing the Switzerland respectively, and as the interest on the bonds and other
obligation of contracts shall be enacted. The particular portion of the indebtedness earned by said corporations has been paid in their
franchise which is invoked provides: respective domiciles, this is not income from Philippine sources
within the meaning of the Philippine Income Tax Law. Citing sections
The grantee shall annually on the fifth day of January of 10 (a) and 13 (e) of Act No. 2833, the Income Tax Law, appellant
each year pay to the City of Manila and the municipalities in asserts that their applicability has been squarely determined by
the Province of Rizal in which gas is sold, two and one half decisions of this court in the cases of Manila Railroad Co. vs.
per centum of the gross receipts within said city and Collector of Internal Revenue (No. 31196, promulgated December 2,
municipalities, respectively, during the preceding year. Said 1929, nor reported), and Philippine Railway Co. vs. Posadas (No.
payment shall be in lieu of all taxes, Insular, provincial and 38766, promulgated October 30, 1933 [58 Phil., 968]) wherein it was
municipal, except taxes on the real estate, buildings, plant, held that interest paid to non-resident individuals or corporations is
machinery, and other personal property belonging to the not income from Philippine sources, and hence not subject to the
grantee. Philippine Income Tax. The Solicitor-General answers with the
observation that the cited decisions interpreted the Income Tax Law
The trial judge was of the opinion that the instant case was governed before it was amended by Act No. 3761 to cover the interest on
by our previous decision in the case of Philippine Telephone and bonds and other obligations or securities paid "within or without the
Telegraph Co., vs. Collector of Internal Revenue ([1933], 58 Phil. Philippine Islands." Appellant rebuts this argument by "assuming, for
639). In this view we concur. It is true that the tax exemption the sake of the argument, that by the amendment introduced to
provision relating to the Manila Gas Corporation hereinbefore quoted section 13 of Act No. 2833 by Act No. 3761 the Legislature intended
differs in phraseology from the tax exemption provision to be found in the interest from Philippine sources and so is subject to tax," but with
the franchise of the Telephone and Telegraph Company, but the ratio the necessary sequel that the amendatory statute is invalid and
decidendi of the two cases is substantially the same. As there held unconstitutional as being the power of the Legislature to enact.
and as now confirmed, a corporation has a personality distinct from
that of its stockholders, enabling the taxing power to reach the latter Taking first under observation that last point, it is to be observed that neither
when they receive dividends from the corporation. It must be in the pleadings, the decision of the trial court, nor the assignment of errors,
considered as settled in this jurisdiction that dividends of a domestic was the question of the validity of Act No. 3761 raised. Under such
corporation, which are paid and delivered in cash to foreign circumstances, and no jurisdictional issue being involved, we do not feel that
corporations as stockholders, are subject to the payment in the it is the duty of the court to pass on the constitutional question, and

268
accordingly will refrain from doing so. (Cadwaller-Gibson Lumber Co. vs. Del Pushing to one side that portion of Act No. 3761 which permits taxation of
Rosario [1913], 26 Phil., 192; Macondray and Co. vs. Benito and Ocampo, P. interest on bonds and other indebtedness paid without the Philippine Islands,
137, ante; State vs. Burke [1912], 175 Ala., 561.) the question is if the income was derived from sources within the Philippine
Islands.
As to the applicability of the local cases cited and of the Porto Rican case of
Domenech vs. United Porto Rican Sugar co. ([1932], 62 F. [2d], 552), we In the judgment of the majority of the court, the question should be answered
need only observe that these cases announced good law, but that each he in the affirmative. The Manila Gas Corporation operates its business entirely
must be decided on its particular facts. In other words, in the opinion of the within the Philippines. Its earnings, therefore come from local sources. The
majority of the court, the facts at bar and the facts in those cases can be place of material delivery of the interest to the foreign corporations paid out
clearly differentiated. Also, in the case at bar there is some uncertainty of the revenue of the domestic corporation is of no particular moment. The
concerning the place of payment, which under one view could be considered place of payment even if conceded to be outside of tho country cannot alter
the Philippines and under another view the United States and Switzerland, the fact that the income was derived from the Philippines. The word "source"
but which cannot be definitely determined without the necessary conveys only one idea, that of origin, and the origin of the income was the
documentary evidence before, us. Philippines.

The approved doctrine is that no state may tax anything not within its In synthesis, therefore, we hold that conditions have not been provided which
jurisdiction without violating the due process clause of the constitution. The justify the court in passing on the constitutional question suggested; that the
taxing power of a state does not extend beyond its territorial limits, but within facts while somewhat obscure differ from the facts to be found in the cases
such it may tax persons, property, income, or business. If an interest in relied upon, and that the Collector of Internal Revenue was justified in
property is taxed, the situs of either the property or interest must be found withholding income taxes on interest on bonds and other indebtedness paid
within the state. If an income is taxed, the recipient thereof must have a to non-resident corporations because this income was received from sources
domicile within the state or the property or business out of which the income within the Philippine Islands as authorized by the Income Tax Law. For the
issues must be situated within the state so that the income may be said to foregoing reasons, the second assigned error will be overruled.
have a situs therein. Personal property may be separated from its owner, and
he may be taxed on its account at the place where the property is although it Before concluding, it is but fair to state that the writer's opinion on the first
is not the place of his own domicile and even though he is not a citizen or subject and the first assigned error herein discussed is accurately set forth,
resident of the state which imposes the tax. But debts owing by corporations but that his opinion on the second subject and the second assigned error is
are obligations of the debtors, and only possess value in the hands of the not accurately reflected, because on this last division his views coincide with
creditors. (Farmers Loan Co. vs. Minnesota [1930], 280 U.S., 204; Union those of the appellant. However, in the interest of the prompt disposition of
Refrigerator Transit Co. vs. Kentucky [1905], 199 U.S., 194 State Tax on this case, the decision has been written up in accordance with instructions
Foreign held Bonds [1873, 15 Wall., 300; Bick vs. Beach [1907], 206 U. S., received from the court.
392; State ex rel. Manitowoc Gas Co. vs. Wig. Tax Comm. [1915], 161 Wis.,
111; United States Revenue Act of 1932, sec. 143.) Judgment affirmed, with the cost of this instance assessed against the
appellant.
These views concerning situs for taxation purposes apply as well to an
organized, unincorporated territory or to a Commonwealth having the status Hull, Vickers, Imperial, Butte, and Recto, JJ., concur.
of the Philippines.

269
270
G.R. No. 58168 December 19, 1989

CONCEPCION MAGSAYSAY-LABRADOR, SOLEDAD MAGSAYSAY-


CABRERA, LUISA MAGSAYSAY-CORPUZ, assisted be her husband, Dr.
Jose Corpuz, FELICIDAD P. MAGSAYSAY, and MERCEDES
MAGSAYSAY-DIAZ, petitioners,
vs.
THE COURT OF APPEALS and ADELAIDA RODRIGUEZ-MAGSAYSAY,
Special Administratrix of the Estate of the late Genaro F.
Magsaysay respondents.

FERNAN, C.J.:

In this petition for review on certiorari, petitioners seek to reverse and set
aside [1] the decision of the Court of Appeals dated July l3, 1981, 1 affirming
that of the Court of First Instance of Zambales and Olongapo City which
denied petitioners' motion to intervene in an annulment suit filed by herein

271
private respondent, and [2] its resolution dated September 7, 1981, denying On March 7, 1979, herein petitioners, sisters of the late senator, filed a
their motion for reconsideration. motion for intervention on the ground that on June 20, 1978, their brother
conveyed to them one-half (1/2 ) of his shareholdings in SUBIC or a total of
Petitioners are raising a purely legal question; whether or not respondent 416,566.6 shares and as assignees of around 41 % of the total outstanding
Court of Appeals correctly denied their motion for intervention. shares of such stocks of SUBIC, they have a substantial and legal interest in
the subject matter of litigation and that they have a legal interest in the
The facts are not controverted. success of the suit with respect to SUBIC.

On February 9, 1979, Adelaida Rodriguez-Magsaysay, widow and special On July 26, 1979, the court denied the motion for intervention, and ruled that
administratix of the estate of the late Senator Genaro Magsaysay, brought petitioners have no legal interest whatsoever in the matter in litigation and
before the then Court of First Instance of Olongapo an action against Artemio their being alleged assignees or transferees of certain shares in SUBIC
Panganiban, Subic Land Corporation (SUBIC), Filipinas Manufacturer's Bank cannot legally entitle them to intervene because SUBIC has a personality
(FILMANBANK) and the Register of Deeds of Zambales. In her complaint, separate and distinct from its stockholders.
she alleged that in 1958, she and her husband acquired, thru conjugal funds,
a parcel of land with improvements, known as "Pequena Island", covered by On appeal, respondent Court of Appeals found no factual or legal justification
TCT No. 3258; that after the death of her husband, she discovered [a] an to disturb the findings of the lower court. The appellate court further stated
annotation at the back of TCT No. 3258 that "the land was acquired by her that whatever claims the petitioners have against the late Senator or against
husband from his separate capital;" [b] the registration of a Deed of SUBIC for that matter can be ventilated in a separate proceeding, such that
Assignment dated June 25, 1976 purportedly executed by the late Senator in with the denial of the motion for intervention, they are not left without any
favor of SUBIC, as a result of which TCT No. 3258 was cancelled and TCT remedy or judicial relief under existing law.
No. 22431 issued in the name of SUBIC; and [c] the registration of Deed of
Mortgage dated April 28, 1977 in the amount of P 2,700,000.00 executed by Petitioners' motion for reconsideration was denied. Hence, the instant
SUBIC in favor of FILMANBANK; that the foregoing acts were void and done recourse.
in an attempt to defraud the conjugal partnership considering that the land is
conjugal, her marital consent to the annotation on TCT No. 3258 was not Petitioners anchor their right to intervene on the purported assignment made
obtained, the change made by the Register of Deeds of the titleholders was by the late Senator of a certain portion of his shareholdings to them as
effected without the approval of the Commissioner of Land Registration and evidenced by a Deed of Sale dated June 20, 1978. 2 Such transfer,
that the late Senator did not execute the purported Deed of Assignment or petitioners posit, clothes them with an interest, protected by law, in the matter
his consent thereto, if obtained, was secured by mistake, violence and of litigation.
intimidation. She further alleged that the assignment in favor of SUBIC was
without consideration and consequently null and void. She prayed that the Invoking the principle enunciated in the case of PNB v. Phil. Veg. Oil Co., 49
Deed of Assignment and the Deed of Mortgage be annulled and that the Phil. 857,862 & 853 (1927), 3petitioners strongly argue that their ownership of
Register of Deeds be ordered to cancel TCT No. 22431 and to issue a new 41.66% of the entire outstanding capital stock of SUBIC entitles them to a
title in her favor. significant vote in the corporate affairs; that they are affected by the action of
the widow of their late brother for it concerns the only tangible asset of the

272
corporation and that it appears that they are more vitally interested in the Here, the interest, if it exists at all, of petitioners-movants is indirect,
outcome of the case than SUBIC. contingent, remote, conjectural, consequential and collateral. At the very
least, their interest is purely inchoate, or in sheer expectancy of a right in the
Viewed in the light of Section 2, Rule 12 of the Revised Rules of Court, this management of the corporation and to share in the profits thereof and in the
Court affirms the respondent court's holding that petitioners herein have no properties and assets thereof on dissolution, after payment of the corporate
legal interest in the subject matter in litigation so as to entitle them to debts and obligations.
intervene in the proceedings below. In the case of Batama Farmers'
Cooperative Marketing Association, Inc. v. Rosal, 4 we held: "As clearly While a share of stock represents a proportionate or aliquot interest in the
stated in Section 2 of Rule 12 of the Rules of Court, to be permitted to property of the corporation, it does not vest the owner thereof with any legal
intervene in a pending action, the party must have a legal interest in the right or title to any of the property, his interest in the corporate property being
matter in litigation, or in the success of either of the parties or an interest equitable or beneficial in nature. Shareholders are in no legal sense the
against both, or he must be so situated as to be adversely affected by a owners of corporate property, which is owned by the corporation as a distinct
distribution or other disposition of the property in the custody of the court or legal person. 8
an officer thereof ."
Petitioners further contend that the availability of other remedies, as declared
To allow intervention, [a] it must be shown that the movant has legal interest by the Court of appeals, is totally immaterial to the availability of the remedy
in the matter in litigation, or otherwise qualified; and [b] consideration must of intervention.
be given as to whether the adjudication of the rights of the original parties
may be delayed or prejudiced, or whether the intervenor's rights may be We cannot give credit to such averment. As earlier stated, that the movant's
protected in a separate proceeding or not. Both requirements must concur as interest may be protected in a separate proceeding is a factor to be
the first is not more important than the second. 5 considered in allowing or disallowing a motion for intervention. It is significant
to note at this juncture that as per records, there are four pending cases
The interest which entitles a person to intervene in a suit between other involving the parties herein, enumerated as follows: [1] Special Proceedings
parties must be in the matter in litigation and of such direct and immediate No. 122122 before the CFI of Manila, Branch XXII, entitled "Concepcion
character that the intervenor will either gain or lose by the direct legal Magsaysay-Labrador, et al. v. Subic Land Corp., et al.", involving the validity
operation and effect of the judgment. Otherwise, if persons not parties of the of the transfer by the late Genaro Magsaysay of one-half of his shareholdings
action could be allowed to intervene, proceedings will become unnecessarily in Subic Land Corporation; [2] Civil Case No. 2577-0 before the CFI of
complicated, expensive and interminable. And this is not the policy of the Zambales, Branch III, "Adelaida Rodriguez-Magsaysay v. Panganiban, etc.;
law. 6 Concepcion Labrador, et al. Intervenors", seeking to annul the purported
Deed of Assignment in favor of SUBIC and its annotation at the back of TCT
The words "an interest in the subject" mean a direct interest in the cause of No. 3258 in the name of respondent's deceased husband; [3] SEC Case No.
action as pleaded, and which would put the intervenor in a legal position to 001770, filed by respondent praying, among other things that she be
litigate a fact alleged in the complaint, without the establishment of which declared in her capacity as the surviving spouse and administratrix of the
plaintiff could not recover. 7 estate of Genaro Magsaysay as the sole subscriber and stockholder of
SUBIC. There, petitioners, by motion, sought to intervene. Their motion to
reconsider the denial of their motion to intervene was granted; [4] SP No. Q-
26739 before the CFI of Rizal, Branch IV, petitioners herein filing a contingent

273
claim pursuant to Section 5, Rule 86, Revised Rules of Court. 9 Petitioners' Feliciano, J., is on leave.
interests are no doubt amply protected in these cases.

Neither do we lend credence to petitioners' argument that they are more


interested in the outcome of the case than the corporation-assignee, owing to
the fact that the latter is willing to compromise with widow-respondent and
since a compromise involves the giving of reciprocal concessions, the only
conceivable concession the corporation may give is a total or partial
relinquishment of the corporate assets. 10

Such claim all the more bolsters the contingent nature of petitioners' interest
in the subject of litigation.

The factual findings of the trial court are clear on this point. The petitioners
cannot claim the right to intervene on the strength of the transfer of shares
allegedly executed by the late Senator. The corporation did not keep books
and records. 11 Perforce, no transfer was ever recorded, much less effected
as to prejudice third parties. The transfer must be registered in the books of
the corporation to affect third persons. The law on corporations is explicit.
Section 63 of the Corporation Code provides, thus: "No transfer, however,
shall be valid, except as between the parties, until the transfer is recorded in
the books of the corporation showing the names of the parties to the
transaction, the date of the transfer, the number of the certificate or
certificates and the number of shares transferred."

And even assuming arguendo that there was a valid transfer, petitioners are
nonetheless barred from intervening inasmuch as their rights can be
ventilated and amply protected in another proceeding.

WHEREFORE, the instant petition is hereby DENIED. Costs against


petitioners.

SO ORDERED.

Gutierrez, Jr., Bidin and Corte's, JJ., concur.

274
275
G.R. No. 82797 February 27, 1991

GOOD EARTH EMPORIUM INC., and LIM KA PING, petitioners,


vs.
HONORABLE COURT OF APPEALS and ROCES-REYES REALTY
INC., respondents.

A.E. Dacanay for petitioners.


Antonio Quintos Law Office for private respondent.

PARAS, J.:

This is a petition for review on certiorari of the December 29, 1987


decision * of the Court of Appeals in CA-G.R. No. 11960 entitled "ROCES-
REYES REALTY, INC. vs. HONORABLE JUDGE REGIONAL TRIAL COURT
OF MANILA, BRANCH 44, GOOD EARTH EMPORIUM, INC. and LIM KA

276
PING" reversing the decision of respondent Judge ** of the Regional Trial On May 16, 1984, Roces filed a motion for execution which was opposed by
Court of Manila, Branch 44 in Civil Case No. 85-30484, which reversed the GEE on May 28, 1984 simultaneous with the latter's filing of a Notice of
resolution of the Metropolitan Trial Court Of Manila, Branch 28 in Civil Case Appeal (Rollo, p. 112, Ibid.). On June 13, 1984, the trial court resolved such
No. 09639, *** denying herein petitioners' motion to quash the alias writ of motion ruling:
execution issued against them.
After considering the motion for the issuance of a writ of execution
As gathered from the records, the antecedent facts of this case, are as filed by counsel for the plaintiff (herein respondents) and the
follows: opposition filed in relation thereto and finding that the defendant
failed to file the necessary supersedeas bond, this court resolved to
A Lease Contract, dated October 16, 1981, was entered into by and between grant the same for being meritorious. (Rollo, p. 112)
ROCES-REYES REALTY, INC., as lessor, and GOOD EARTH EMPORIUM,
INC., as lessee, for a term of three years beginning November 1, 1981 and On June 14, 1984, a writ of execution was issued by the lower court.
ending October 31, 1984 at a monthly rental of P65,000.00 (Rollo, p. 32; Meanwhile, the appeal was assigned to the Regional Trial Court (Manila)
Annex "C" of Petition). The building which was the subject of the contract of Branch XLVI. However, on August 15, 1984, GEE thru counsel filed with the
lease is a five-storey building located at the corner of Rizal Avenue and Regional Trial Court of Manila, a motion to withdraw appeal citing as reason
Bustos Street in Sta. Cruz, Manila. that they are satisfied with the decision of the Metropolitan Trial Court of
Manila, Branch XXVIII, which said court granted in its Order of August 27,
From March 1983, up to the time the complaint was filed, the lessee had 1984 and the records were remanded to the trial court (Rollo, p. 32; CA
defaulted in the payment of rentals, as a consequence of which, private Decision). Upon an ex-parte Motion of ROCES, the trial court issued
respondent ROCES-REYES REALTY, INC., (hereinafter designated as an Alias Writ of Execution dated February 25, 1985 (Rollo, p. 104; Annex "D"
ROCES for brevity) filed on October 14, 1984, an ejectment case (Unlawful of Petitioner's Memorandum), which was implemented on February 27, 1985.
Detainer) against herein petitioners, GOOD EARTH EMPORIUM, INC. and GEE thru counsel filed a motion to quash the writ of execution and notice of
LIM KA PING, hereinafter designated as GEE, (Rollo, p. 21; Annex "B" of the levy and an urgent Ex-parte Supplemental Motion for the issuance of a
Petition). After the latter had tendered their responsive pleading, the lower restraining order, on March 7, and 20, 1985, respectively. On March 21,
court (MTC, Manila) on motion of Roces rendered judgment on the pleadings 1985, the lower court issued a restraining order to the sheriff to hold the
dated April 17, 1984, the dispositive portion of which states: execution of the judgment pending hearing on the motion to quash the writ of
execution (Rollo, p. 22; RTC Decision). While said motion was pending
Judgment is hereby rendered ordering defendants (herein resolution, GEE filed a Petition for Relief from judgment before another court,
petitioners) and all persons claiming title under him to vacate the Regional Trial Court of Manila, Branch IX, which petition was docketed as
premises and surrender the same to the plaintiffs (herein Civil Case No. 80-30019, but the petition was dismissed and the injunctive
respondents); ordering the defendants to pay the plaintiffs the rental writ issued in connection therewith set aside. Both parties appealed to the
of P65,000.00 a month beginning March 1983 up to the time Court of Appeals; GEE on the order of dismissal and Roces on denial of his
defendants actually vacate the premises and deliver possession to motion for indemnity, both docketed as CA-G.R. No. 15873-CV. Going back
the plaintiff; to pay attorney's fees in the amount of P5,000.00 and to to the original case, the Metropolitan Trial Court after hearing and disposing
pay the costs of this suit. (Rollo, p. 111; Memorandum of some other incidents, promulgated the questioned Resolution, dated April 8,
Respondents) 1985, the dispositive portion of which reads as follows:

277
Premises considered, the motion to quash the writ is hereby denied The main issue in this case is whether or not there was full satisfaction of the
for lack of merit. judgment debt in favor of respondent corporation which would justify the
quashing of the Writ of Execution.
The restraining orders issued on March 11 and 23, 1985 are hereby
recalled, lifted and set aside. (Rollo, p. 20, MTC Decision) A careful study of the common exhibits (Exhibits 1/A and 2/B) shows that
nowhere in any of said exhibits was there any writing alluding to or referring
GEE appealed and by coincidence. was raffled to the same Court, RTC to any settlement between the parties of petitioners' judgment obligation
Branch IX. Roces moved to dismiss the appeal but the Court denied the (Rollo, pp. 45-48).
motion. On certiorari, the Court of Appeals dismissed Roces' petition and
remanded the case to the RTC. Meantime, Branch IX became vacant and Moreover, there is no indication in the receipt, Exhibit "1", that it was in
the case was re-raffled to Branch XLIV. payment, full or partial, of the judgment obligation. Likewise, there is no
indication in the pacto de retro sale which was drawn in favor of Jesus
On April 6, 1987, the Regional Trial Court of Manila, finding that the amount Marcos Roces and Marcos V. Roces and not the respondent corporation, that
of P1 million evidenced by Exhibit "I" and another P1 million evidenced by the obligation embodied therein had something to do with petitioners'
the pacto de retro sale instrument (Exhibit "2") were in full satisfaction of the judgment obligation with respondent corporation.
judgment obligation, reversed the decision of the Municipal Trial Court, the
dispositive portion of which reads: Finding that the common exhibit, Exhibit 1/A had been signed by persons
other than judgment creditors (Roces-Reyes Realty, Inc.) coupled with the
Premises considered, judgment is hereby rendered reversing the fact that said exhibit was not even alleged by GEE and Lim Ka Ping in their
Resolution appealed from quashing the writ of execution and original motion to quash the alias writ of execution (Rollo, p. 37) but
ordering the cancellation of the notice of levy and declaring the produced only during the hearing (Ibid.) which production resulted in
judgment debt as having been fully paid and/or Liquidated. (Rollo, p. petitioners having to claim belatedly that there was an "overpayment" of
29). about half a million pesos (Rollo, pp. 25-27) and remarking on the utter
absence of any writing in Exhibits "1/A" and "2/B" to indicate payment of the
On further appeal, the Court of Appeals reversed the decision of the Regional judgment debt, respondent Appellate Court correctly concluded that there
Trial Court and reinstated the Resolution of the Metropolitan Trial Court of was in fact nopayment of the judgment debt. As aptly observed by the said
Manila, the dispositive portion of which is as follows: court:

WHEREFORE, the judgment appealed from is hereby REVERSED What immediately catches one's attention is the total absence of any
and the Resolution dated April 8, 1985, of the Metropolitan Trial writing alluding to or referring to any settlement between the parties
Court of Manila Branch XXXIII is hereby REINSTATED. No of private respondents' (petitioners') judgment obligation. In moving
pronouncement as to costs. (Rollo, p. 40). for the dismissal of the appeal Lim Ka Ping who was then assisted
by counsel simply stated that defendants (herein petitioners) are
satisfied with the decision of the Metropolitan Trial Court (Records of
GEE's Motion for Reconsideration of April 5, 1988 was denied (Rollo, p. 43).
CA, p. 54).
Hence, this petition.

278
Notably, in private respondents' (petitioners') Motion to Quash the does not make one's property also of the corporation, and vice-versa, for
Writ of Execution and Notice of Levy dated March 7, 1985, there is they are separate entities (Traders Royal Bank v. CA-G.R. No. 78412,
absolutely no reference to the alleged payment of one million pesos September 26, 1989; Cruz v. Dalisay, 152 SCRA 482). Shareowners are in
as evidenced by Exhibit 1 dated September 20, 1984. As pointed out no legal sense the owners of corporate property (or credits) which is owned
by petitioner (respondent corporation) this was brought out by Linda by the corporation as a distinct legal person (Concepcion Magsaysay-
Panutat, Manager of Good Earth only in the course of the latter's Labrador v. CA-G.R. No. 58168, December 19, 1989). As a consequence of
testimony. (Rollo, p. 37) the separate juridical personality of a corporation, the corporate debt or credit
is not the debt or credit of the stockholder, nor is the stockholder's debt or
Article 1240 of the Civil Code of the Philippines provides that: credit that of the corporation (Prof. Jose Nolledo's "The Corporation Code of
the Philippines, p. 5, 1988 Edition, citing Professor Ballantine).
Payment shall be made to the person in whose favor the obligation
has been constituted, or his successor in interest, or any person The absence of a note to evidence the loan is explained by Jesus Marcos
authorized to receive it. Roces who testified that the IOU was subsequently delivered to private
respondents (Rollo, pp. 97-98). Contrary to the Regional Trial Court's
In the case at bar, the supposed payments were not made to Roces-Reyes premise that it was incumbent upon respondent corporation to prove that the
Realty, Inc. or to its successor in interest nor is there positive evidence that amount was delivered to the Roces brothers in the payment of the loan in the
the payment was made to a person authorized to receive it. No such proof latter's favor, the delivery of the amount to and the receipt thereof by the
was submitted but merely inferred by the Regional Trial Court (Rollo, p. 25) Roces brothers in their names raises the presumption that the said amount
from Marcos Roces having signed the Lease Contract as President which was due to them.1wphi1 There is a disputable presumption that money paid
was witnessed by Jesus Marcos Roces. The latter, however, was no longer by one to the other was due to the latter (Sec. 5(f) Rule 131, Rules of Court).
President or even an officer of Roces-Reyes Realty, Inc. at the time he It is for GEE and Lim Ka Ping to prove otherwise. In other words, it is for the
received the money (Exhibit "1") and signed the sale with pacto de latter to prove that the payments made were for the satisfaction of their
retro (Exhibit "2"). He, in fact, denied being in possession of authority to judgment debt and not vice versa.
receive payment for the respondent corporation nor does the receipt show
that he signed in the same capacity as he did in the Lease Contract at a time The fact that at the time payment was made to the two Roces brothers, GEE
when he was President for respondent corporation (Rollo, p. 20, MTC was also indebted to respondent corporation for a larger amount, is not
decision). supportive of the Regional Trial Court's conclusions that the payment was in
favor of the latter, especially in the case at bar where the amount was not
On the other hand, Jesus Marcos Roces testified that the amount of P1 receipted for by respondent corporation and there is absolutely no indication
million evidenced by the receipt (Exhibit "1") is the payment for a loan in the receipt from which it can be reasonably inferred, that said payment
extended by him and Marcos Roces in favor of Lim Ka Ping. The assertion is was in satisfaction of the judgment debt. Likewise, no such inference can be
home by the receipt itself whereby they acknowledged payment of the loan in made from the execution of the pacto de retro sale which was not made in
their names and in no other capacity. favor of respondent corporation but in favor of the two Roces brothers in their
individual capacities without any reference to the judgment obligation in favor
of respondent corporation.
A corporation has a personality distinct and separate from its individual
stockholders or members. Being an officer or stockholder of a corporation

279
In addition, the totality of the amount covered by the receipt (Exhibit "1/A") payment of the judgment obligation as claimed by petitioner. The burden of
and that of the sale with pacto de retro(Exhibit "2/B") all in the sum of P2 evidence resting on the petitioners to establish the facts upon which their
million, far exceeds petitioners' judgment obligation in favor of respondent action is premised has not been satisfactorily discharged and therefore, they
corporation in the sum of P1,560,000.00 by P440,000.00, which militates have to bear the consequences.
against the claim of petitioner that the aforesaid amount (P2M) was in full
payment of the judgment obligation. PREMISES CONSIDERED, the petition is hereby DENIED and the Decision
of the Respondent court is hereby AFFIRMED, reinstating the April 8, 1985
Petitioners' explanation that the excess is interest and advance rentals for an Resolution of the Metropolitan Trial Court of Manila.
extension of the lease contract (Rollo, pp. 25-28) is belied by the absence of
any interest awarded in the case and of any agreement as to the extension of SO ORDERED.
the lease nor was there any such pretense in the Motion to Quash
the Alias Writ of Execution. Melencio-Herrera, Padilla, Sarmiento and Regalado, JJ., concur.

Petitioners' averments that the respondent court had gravely abused its
discretion in arriving at the assailed factual findings as contrary to the
evidence and applicable decisions of this Honorable Court are therefore,
patently unfounded. Respondent court was correct in stating that it "cannot
go beyond what appears in the documents submitted by petitioners
themselves (Exhibits "1" and "2") in the absence of clear and convincing
evidence" that would support its claim that the judgment obligation has
indeed been fully satisfied which would warrant the quashal of the Alias Writ
of Execution.
G.R. No. L-30188 October 2, 1928
It has been an established rule that when the existence of a debt is fully
established by the evidence (which has been done in this case), the burden FELIPE TAYKO, EDUARDO BUENO, BAUTISTA TAYKO, BERNARDO
of proving that it has been extinguished by payment devolves upon the SOLDE and VICENTE ELUM, petitioners,
debtor who offers such a defense to the claim of the plaintiff creditor (herein vs.
respondent corporation) (Chua Chienco v. Vargas, 11 Phil. 219; Ramos v. NICOLAS CAPISTRANO, acting as Judge of First Instance of Oriental
Ledesma, 12 Phil. 656; Pinon v. De Osorio, 30 Phil. 365). For indeed, it is Negros. ALFREDO B. CACNIO, as Provincial Fiscal of Oriental Negros,
well-entrenched in Our jurisprudence that each party in a case must prove and JUAN GADIANI, respondents.
his own affirmative allegations by the degree of evidence required by law
(Stronghold Insurance Co. v. CA, G.R. No. 83376, May 29,1989; Tai Tong Abad Santos, Camus and Delgado and Teopisto Guingona for petitioners.
Chuache & Co. v. Insurance Commission, 158 SCRA 366). Araneta and Zaragoza for respondents.
The respondent Judge in his own behalf.
The appellate court cannot, therefore, be said to have gravely abused its
discretion in finding lack of convincing and reliable evidence to establish

280
OSTRAND, J.: provincial fiscal refused to file criminal charges against the petitioners for
violation of the election law for lack of sufficient evidence to sustain the
This is a petition for a writ of prohibition enjoining the respondent judge from same; that said respondent is neither a judge de jure nor de facto, but that,
making cognizance of certain civil and criminal election cases in which the notwithstanding this fact, he continues to hold the office of judge of the Court
petitioners are parties. of First Instance of Oriental Negros and pretends to be duly qualified and
acting judge of the said province; and that he has tried, and continues to try,
The petitioners allege that the respondent judge, previous to this date, was to act as such judge and that there is reasonable ground to believe that he
appointed judge of the Court of First Instance of Oriental Negros, to hold will take cognizance of the cases in question unless he be restrained by
office during good behavior and until he should reach the age of 65 years; order of this court; that in acting as a duly qualified judge notwithstanding the
that he now has reached that age and, therefore, under the provisions of facts alleged in the fifth, sixth, and seventh paragraphs hereof, the
section 148 of the Administrative Code as amended, is disqualified from respondent judge acted and is about to act without and in excess of
acting as a judge of the Court of First Instance. The petitioners further allege jurisdiction and also after the loss of jurisdiction.
that in view of the many election protests and criminal cases for violation of
the election law filed in the Court of First Instance of Oriental Negros arising To this petition the respondents demur on the ground that the facts stated in
in the Court of First Instance of Oriental Negros arising from the last election that (1) none of the facts alleged in the petition divest the respondent judge
of June 5, 1928, the Honorable Sixto de la Costa was duly designated and of his jurisdiction to take cognizance of the cases referred to in the complaint,
acted as auxiliary judge of the Province of Oriental Negros; that between the and (2) even admitting as true, for the sake of this demurrer, the facts alleged
auxiliary judge and the respondent judge herein there was an understanding, in paragraph 7 of the petition, the respondent judge is still a de facto judge
and the assignment of the said auxiliary judge was made with this and his title to the office and his jurisdiction to hear the cases referred to in
understanding, that the said auxiliary judge so designated would hear and the petition cannot be questioned by prohibition, as this writ, even when
take cognizance of all election protests and criminal actions then pending or directed against persons acting as judges, cannot be treated as a substitute
to filed arising from the said last general election, and that the respondent for quo warranto, or be rightfully called upon to perform any of the functions
Honorable Nicolas Capistrano would try and hear the ordinary cases pending of that writ.
in the said court, but, notwithstanding this understanding or agreement, the
respondent judge tried and is still trying to take cognizance of the election The ground upon which the petition rests may be reduced to three
protests an criminal actions in said court; that the respondent judge declared propositions. (1) That the assignment of the Auxiliary Judge, Sixto de la
in open court that he will try the criminal cases herein mentioned for the Costa, to Dumaguete was made with the understanding that the he was to
reason that the auxiliary judge refused to try the same on the ground that the hear and take cognizance of all election contests and criminal causes for
preliminary investigations were held before him, when, in truth and in fact, violation of the election law and that the respondent judge was to take
the said auxiliary judge did not make the statement imputed to him and was cognizance of the ordinary cases and that there was an understanding
and is still willing to try the election protests and criminal cases for violation of between them that this arrangement was to be followed.
the election law pending in the court of the Province of Oriental Negros; that
the respondent Honorable Nicolas Capistrano, in spite of the fact that he was (2) That the respondent judge took great interest and an active part
holding and is now pretending to hold the office of judge of the Court of First in the filing of the criminal charges against the petitioners herein to
Instance of Oriental Negros, took great interest and active part in the filing of the unjustifiable extent of appointing a deputy fiscal who filed the
criminal charges against the petitioners herein to the unjustifiable extent of proper informations when the regular provincial fiscal refused to file
appointing a deputy fiscal, who then filed the proper informations, when the them for lack of sufficient evidence.

281
(3) That the respondent judge is already over 65 years of age and of law and not an allegation of facts upon which such a
has, therefore, automatically ceased as judge of the Court of First conclusion can be formed and may, therefore, be
Instance of Oriental Negros and that he is neither a judge de disregarded. It follows that in appointing an acting fiscal, the
jure nor de facto. respondent judge was well within his jurisdiction.

(a) But little need be said as to the first proposition. A writ of (c) The third ground upon which the petition is based is the
prohibition to a judge of an interior court will only lie in cases most important and merits some consideration. It is well
where he acts without or in excess of his jurisdiction (section settled that the title to the office of a judge, whether de
226, Code of Civil Procedure), and it is obvious that a mere jure or de facto, can only be determined in a proceeding in
"understanding" as to the distribution of cases for trial did not the nature of quo warranto and cannot be tested by
deprive the respondent judge of the jurisdiction conferred prohibition. But counsel for the petitioners maintains that the
upon him by law. It may be noted that it is not alleged that respondent judge is neither a judge de jure nor de facto and
another judge had taken cognizance of the cases in question that, therefore, prohibition will lie. In this, counsel is
or that they had been definitely assigned to trial before such undoubtedly mistaken.
other judge.
The respondent judge has been duly appointed to the office of Judge of the
(b) The second proposition is equally untenable.1awph! Court of First Instance of Oriental Negros, but section 148 of the
l.net That the respondent judge took great interest and an Administrative Code, as amended, provides that "Judges of the Court of First
active part in the filing of the criminal charges against the Instance and auxiliary judges shall be appointed to serve until they shall
petitioners to the extent of appointing a deputy fiscal when reach the age of sixty-five years." In view of this provision and assuming, as
the regular provincial fiscal refused to file the proper we must, that the allegations of the petition are true, it is evident that the
informations, did not disqualify him from trying the case in respondent is no longer a judge de jure, but we do not think that it can be
question. Section 1679 of the Administrative Code provides successfully disputed that he is still a judge de facto.
that "when a provincial fiscal shall be disqualified by
personal interest to act in a particular case or when for any Briefly defined, a de facto judge is one who exercises the duties of a judicial
reason he shall be unable, or shall fail, to discharge any of office under color of an appointment or election thereto (Brown vs. O'Connell,
the duties of his position, the judge of the Court of First 36 Conn., 432). He differs, on the one hand, from a mere usurper who
Instance of the province shall appoint an acting provincial undertakes to act officially without any color of right, and on the other hand,
fiscal, . . . ." (Emphasis ours.) from a judge de jure who is in all respects legally appointed and qualified and
whose term of office has not expired (State vs. Carroll, 38 Conn., 449; Denny
The determination of the question as to whether the fiscal vs. Matton, 2 Allen [Mass.], 361; Van Slyke vs. Farmers' Mut. Fire Ins. Co.,
has failed to discharge his duty in the prosecution of a crime 39 Wis., 390).
must necessarily, to a large extent, lie within the sound
discretion of the presiding judge, and there is no allegation in Apart from any constitutional or statutory regulation on the subject
the petition that such discretion was abused in the present there seems to be a general rule of law that an incumbent of an
instance. It is true that it is stated that the appointment of the office will hold over after the conclusion of his term until the elction
acting fiscal was "unjustifiable," but that is only a conclusion

282
and qualification of a successor (22 R. C. L., pp. 554-5). When a be questioned in a suit before him is generally recognized in the case of a
judge in good faith remains in office after his title has ended, he is special judge, and it is held that a party to an action before a special judge
a de facto officer (Sheehan's Case, 122 Mass., 445). may question his title to the office of a judge on the proceedings before him,
and that the judgment will be reversed on appeal, where proper exceptions
Applying the principles stated to the facts set forth in the petition before us, are taken, if the person assuming to act as special judge is not a judge de
we cannot escape the conclusion that, on the assumption that said facts are jure. The title of a de facto officer cannot be indirectly questioned in a
true, the respondent judge must be considered a judge de facto. His term of proceeding to obtain a writ of prohibition to prevent him from doing an official
office may have expired, but his successor has not been appointed, and as act nor in a suit to enjoin the collection of a judgment rendered by him.
good faith is presumed, he must be regarded as holding over in good faith. Having at least colorable right to the office his title can be determined only in
The contention of counsel for the petitioners that the auxiliary judge present a quo warranto proceeding or information in the nature of a quo warranto at
in the district must be considered the regular judge seems obviously suit of the sovereign." (15 R. C. L., pp. 519-521.)
erroneous.
The demurrer to the petition is sustained, and inasmuch as it is evident that
In these circumstances the remedy prayed for cannot be granted. "The the weakness of the petition cannot be cured by amendment the present
rightful authority of a judge, in the full exercise of his public judicial function, proceedings are hereby dismissed with the costs against the petitioners
cannot be questioned by any merely private suitor, nor by any other, jointly and severally. The preliminary injunction hereinbefore issued is
excepting in the form especially provided by law. A judge de facto assumes dissolved. So ordered.
the exercise of a part of the prerogative of sovereignty, and the legality of that
assumption is open to the attack of the sovereign power alone. Accordingly, it Avancea, C. J., Johnson, Street, Malcolm, Villamor, Romualdez, and Villa-
is a well established principle, dating from the earliest period and repeatedly Real, JJ., concur.
confirmed by an unbroken current of decisions, that the official acts of a de
facto judge are just as valid for all purposes as those of a de jure judge, so
far as the public or third persons who are interested therein are concerned.
The rule is the same in civil criminal cases. The principle is one founded in
policy and convenience, for the right of no one claiming a title or interest
under or through the proceedings of an officer having an apparent authority
to act would be safe, if it were necessary in every case to examine the
legality of the title of such officer up to its original source, and the title or
interest of such person were held to be invalidated by some accidental defect
or flaw in the appointment, election or qualification of such officer, or in the
rights of those from whom his appointment or election emanated; nor could
the supremacy of the laws be maintained, or their execution enforced, if the
acts of the judge having a colorable, but not a legal title, were to be deemed
invalid. As in the case of judges of courts of record, the acts of a justice de
facto cannot be called in question in any suit to which he is not a party. The
official acts of a de facto justice cannot b attacked collaterally. An exception
to the general rule that the title of a person assuming to act as judge cannot

283
284
deposit for the goods being handled by the former; and that because plaintiff
was separated from the service in October, 1959, he sought to recover the
sum of P5,300.00 representing the P100.00 monthly deductions from his
salary; P4,770.00 corresponding to his 10% commissions that were withheld,
G.R. No. L-21114 November 28, 1967 and P1,500.00 as separation pay, or the total sum of P11,570.00. These
three items were respectively the subject matter of the first, second and third
causes of action of the complaint.
FEDERICO FERNANDEZ, plaintiff-appellant,
vs.
P. CUERVA and CO., defendant-appellee. On January 2, 1963, defendant filed a motion to dismiss the complaint upon
the grounds that the actions had prescribed and that the court had no
jurisdiction over the case. The court below, after allowing the parties to
Gerardo P. Moreno, Jr. for plaintiff-appellant.
submit their respective memorandum on the questions of prescription and
N.O. Bueno for defendant-appellee.
jurisdiction, dismissed the case, in an order issued on January 29, 1963,
holding that because the claim of plaintiff in the first two causes of action
ZALDIVAR, J.: amounting to P10,070.00 represented the sum total of unauthorized
deductions from his salaries and withheld commissions, under Section 10,
This is an appeal from the order of the Court of First Instance of Manila, paragraph (f) of Republic Act No. 602, otherwise known as the Minimum
dated January 29, 1963, in its Civil Case No. 52946, dismissing the Wage Law, the action to recover the same was already barred under Section
complaint upon the ground that the action in the first two causes of action 17 of said Act inasmuch as it was not brought within three years from the
had prescribed and that it had no jurisdiction over the third cause of action. time the right of action accrued; and that because the remaining claim of
plaintiff was limited to his separation pay amounting only to P1,500.00, the
It appears that plaintiff Federico Fernandez was employed as salesman by action to collect the same was not within the original jurisdiction of the court.
defendant P. Cuerva & Co. from March, 1949 to October, 1959. After his
separation from the service, plaintiff filed a claim, on July 26, 1960, before On February 1, 1963, plaintiff moved to reconsider the above-mentioned
Regional Office No. 4 of the Department of Labor,1 docketed as L. S. Case order, advancing as his main argument the fact that his having filed a similar
No. 2940, to recover unpaid salaries and commissions, and separation pay. claim with Regional Office No. 4 of the Department of Labor had suspended
the running of the prescriptive period insofar as his claim for refund of
During the pendency of said case, or on December 17, 1962, plaintiff again unauthorized deductions and withheld commissions was concerned which
instituted a similar complaint against the same defendant with the Court of were the subject matters of the first and second causes of action that were
First Instance of Manila (Civil Case No. 52946) alleging, among others, that dismissed by the court. The defendant filed an opposition to the motion for
he was employed by defendant company as salesman in March, 1949 with a reconsideration. In an order dated February 15, 1963, the court denied
salary of P200.00 per month that beginning June, 1955 until the termination plaintiff's motion for reconsideration. Hence this appeal by the plaintiff direct
of his services in October, 1959, his salary was increased to, P300.00 to this Court on purely questions of law.
monthly and was given, in addition, a commission of 10% on his sales; that
the increase of P100.00 a month and the 10% commission were not actually We are in accord with the court a quo that the law applicable to the case at
received by him as there was a verbal understanding between him and bar is Republic Act 602 because the bond or deposit sought to be recovered
defendant company that the same would be retained by the latter as bond or by appellant was actually the sum total of the unauthorized deductions from

285
his salaries and withheld commissions under Section 10 thereof. Under Defendant answers the question in the negative. While defendant does not
Section 17 of said law, "any action . . . to enforce any cause of action under question the applicability to the case at bar of Article 1155 of the Civil Code,
this Act may be commenced within three years after the cause of action which provides that the "prescription of actions is interrupted when they are
accrued, and every such action shall be forever barred unless commenced filed before the Court," nevertheless, it contends that inasmuch as plaintiff's
within three years after the cause of action accrued." Since a right of action claim was lodged with the regional office of the Department of Labor, which is
accrues only from the moment the right to commence the action comes into not a court, the same could not be considered a judicial demand that would
existence, and prescription begins to run from that time, 2 the question to be suspend the running of the prescriptive period.
resolved is: When did the right of action of plaintiff accrue?
We do not agree with defendant. It is true that the claim filed by plaintiff with
To answer the foregoing query, it is meet to recall that while the amounts the regional office of the Department of Labor is not a judicial demand in the
withheld by defendant were actually deductions from plaintiff's salaries and same sense of the term "judicial demand" because the same was not
unpaid commissions, they were, however, constituted as a bond or a deposit instituted in a court of justice. Judicial notice, however, should be taken that
to answer for any liability that he might incur in connection with the goods on December 10, 1956, Reorganization Plan No. 20-A was promulgated
handled by him. The bond and/or deposit was thus answerable for pursuant to Republic Act 997, and under Section 25 of said reorganization
merchandise entrusted to plaintiff during the period of his employment with plan each regional office of the Department of Labor was vested with original
defendant. It was, therefore, not feasible for plaintiff to demand every month and exclusive jurisdiction over all cases affecting all money claims arising
or every payday, or during the period of his employment with the company from violations of labor standards on working conditions such as unpaid
the return or refund of those amounts withheld as contended by defendant, wages, underpayment, overtime and separation pay, etc., to the exclusion of
because the undertaking for which the bond or deposit was constituted was courts.3Consequently, when plaintiff wanted to enforce his claim after his
still subsisting. And so the right of plaintiff to commence an action for the dismissal from the service in October, 1959, he had no choice but to file the
return or refund of the amounts representing such bond or deposit would same with Regional Office No. 4 of the Department of Labor which was the
accrue only when the same was no longer needed, and the time when it was agency then empowered to take cognizance of the claim. He could not
no longer needed only came in October 1959 when plaintiff was separated institute the action to recover his claim in the court of justice because of the
from the service. Having ceased to be employed by the defendant, the bond provisions of Reorganization Plan No. 20-A. At least it may be said that on
put up by plaintiff thereby became unnecessary or useless. July 26, 1960, when plaintiff filed his claim with Regional Office. No. 4 of the
Department of Labor, he acted in accordance with the procedure that was
It would seem, however, that even if We count from October, 1959 in then prescribed under authority of law. Under the circumstances, We believe
computing the prescriptive period, plaintiff's action to recover the amount that the filing by plaintiff of his claim before the regional office of the
held by defendant as bond is already barred because more than three years Department of Labor had the attributes of a judicial demand. And We say this
had elapsed by the time plaintiff instituted the present case in the court below because under the provisions of Section 25 of Reorganization Plan No. 20-A
on December 17, 1962. The record, however, shows that on July 26, 1960, each regional office of the Department of Labor was invested with
plaintiff filed a similar claim against the defendant with Regional Office No. 4 jurisdiction, similar to that of a court, to receive, determine, and adjudicate
of the Department of Labor. claims arising out of employer-employee relations as specified in said
section. We quote Section 25 of Reorganization Plan No. 20-A:
At this juncture, the question posed is: Did the filing by plaintiff of that claim
with the regional office of the Department of Labor suspend the running of Each Regional Office shall have original and exclusive jurisdiction
the period of prescription? over all cases affecting all money claims arising from violations of

286
labor standards on working conditions, including but not restrictive to: [t]here are several instances wherein courts, out of equity, have
unpaid wages, underpayment, overtime, separation pay, and relaxed its operation (cf. note in Cooley's Constitutional Limitations
maternity leave of employees/laborers and unpaid wages, overtime, 8th ed., p. 383 and Notes 53 A.L.R., 273) or qualified its effects
separation pay, vacation pay, and payment for medical services of "since the actual existence of a statute prior to such declaration is an
domestic held. (Emphasis supplied) operative fact, and may have consequences which cannot justly be
ignored" (Chicot County vs. Baster, 308 U.S., 371) and a realistic
It can be gathered from a reading of the above-quoted Section 25 of approach is eroding the general doctrine (Warring vs. Colpoys 136
Reorganization Plan No. 20-A that some sort of judicial powers was Am. Law Rep., 1025, 1030).
conferred upon the regional offices of the Department of Labor over money
claims mentioned in said section. Certainly, it can be considered that filing a We believe that it is only fair and just that the foregoing doctrine should be
money claim before a regional office of the Department of Labor pursuant to applied in favor of the plaintiff in the present case.
Section 25 of Reorganization Plan No. 20-A is like filing a complaint in court
to enforce said money claim. We believe that the filing of a claim before an We have noted in the record that it was precisely because Section 25 of
administrative agency which is vested with authority to decide said claim Reorganization Plan No. 20-A was declared unconstitutional by this Court on
would produce the effect of a judicial demand for the purpose of interrupting June 30, 1961 that the plaintiff, without awaiting the action of Regional Office
the running of the period of prescription. The purpose of the law on No. 4 of the Department of Labor on the claim that he filed on July 26, 1960,
prescription and the statute of limitations is to protect the person who is instituted his action in the present case in the court below on December 17,
diligent and vigilant in asserting his right, and conversely to punish the 1962. The move of plaintiff was precisely intended to protect his right of
person who sleeps on his right. 4 Indeed, it cannot be said that in the case action from the adverse effect of the decision of this Court. The Regional
before Us the plaintiff had slept on his right, because shortly after he was Office No. 4 of the Department of Labor dismissed plaintiff's claim on January
separated from the service by the defendant he filed his claim before the 16, 1963 upon the ground that it had no more jurisdiction to pass upon the
agency of the government that was at the time clothed with exclusive claim as a result of the ruling of this Court in the Corominas case.
authority to pass upon his claim.
Considering that from October, 1959 when plaintiff was separated from the
We have taken note of the fact that on June 30, 1961, Section 25 of service up to July 26, 1960 when he filed his claim with Regional Office No. 4
Reorganization Plan No. 20-A had been declared unconstitutional by this of the Department of Labor only eight months had elapsed, and that since
Court in the case of Corominas, et al. v. The Labor Standards Commission, July 26, 1960 until the filing of the complaint in the court below on December
et al., supra. It appears, however, that the plaintiff had filed his claim before 17, 1962 the running of prescriptive period was deemed interrupted, it is
Regional Office No. 4 of the Department of Labor on July 26, 1960, or about clear that plaintiff's action to enforce his claim was not yet barred by the
one year before said Section 25 had been declared unconstitutional. The statute of limitations when he filed his complaint in the court below. Plaintiff's
circumstance that Section 25 of Reorganization Plan No. 20-A had been action may be considered as brought before the court still within the period of
declared unconstitutional should not be counted against the defendant in the three years from the time his right of action accrued in accordance with the
present case. In the case of Manila Motor Co., Inc. v. Flores, 99 Phil., 738, provisions of Section 17 of Republic Act 602 (Minimum Wage Law). Only
this Court upheld the right of a party under the Moratorium Law which had about nine months of the three-year period provided in Section 17 of
accrued in his favor before said law was declared unconstitutional by this Republic Act 602 may be considered as having lapsed when plaintiff
Court in the case of Rutter v. Esteban, 93 Phil., 68. This Court, in its decision commenced his action in the court below. And considering further that the
in the Manila Motor case, quoted the following doctrine: amount sought to be recovered in the complaint is more than P10,000.00, it

287
follows that the court a quo has the exclusive and original jurisdiction to
entertain the action of the plaintiff. The lower court, therefore, erred when it
dismissed plaintiff's complaint.

WHEREFORE, the order appealed from is set aside, and this case is
remanded to the court below for further proceedings, with costs against the
defendant-appellee. It is so ordered.

Dizon, Makalintal, Bengzon, J.P., Sanchez, Castro and Angeles, JJ., concur.
Concepcion, C.J., and Reyes, J.B.L., J., took no part.

288
(2) Immediately after the execution of said articles of incorporation, the
corporation proceeded to do business with the adoption of by-laws and the
election of its officers.

(3) On December 2, 1947, the said articles of incorporation were filed in the
office of the Securities and Exchange Commissioner, for the issuance of the
corresponding certificate of incorporation.

G.R. No. L-2598 June 29, 1950 (4) On March 22, 1948, pending action on the articles of incorporation by the
aforesaid governmental office, the respondents Fred Brown, Emma Brown,
Hipolita D. Chapman and Ceferino S. Abella filed before the Court of First
C. ARNOLD HALL and BRADLEY P. HALL, petitioners,
Instance of Leyte the civil case numbered 381, entitled "Fred Brown et al. vs.
vs.
Arnold C. Hall et al.", alleging among other things that the Far Eastern
EDMUNDO S. PICCIO, Judge of the Court of First Instance of Leyte,
Lumber and Commercial Co. was an unregistered partnership; that they
FRED BROWN, EMMA BROWN, HIPOLITA CAPUCIONG, in his capacity
wished to have it dissolved because of bitter dissension among the
as receiver of the Far Eastern Lumber and Commercial Co.,
members, mismanagement and fraud by the managers and heavy financial
Inc.,respondents.
losses.

Claro M. Recto for petitioners.


(5) The defendants in the suit, namely, C. Arnold Hall and Bradley P. Hall,
Ramon Diokno and Jose W. Diokno for respondents.
filed a motion to dismiss, contesting the court's jurisdiction and the sufficiently
of the cause of action.
BENGZON, J.:
(6) After hearing the parties, the Hon. Edmund S. Piccio ordered the
This is petition to set aside all the proceedings had in civil case No. 381 of dissolution of the company; and at the request of plaintiffs, appointed of the
the Court of First Instance of Leyte and to enjoin the respondent judge from properties thereof, upon the filing of a P20,000 bond.
further acting upon the same.
(7) The defendants therein (petitioners herein) offered to file a counter-bond
Facts: (1) on May 28, 1947, the petitioners C. Arnold Hall and Bradley P. Hall, for the discharge of the receiver, but the respondent judge refused to accept
and the respondents Fred Brown, Emma Brown, Hipolita D. Chapman and the offer and to discharge the receiver. Whereupon, the present special civil
Ceferino S. Abella, signed and acknowledged in Leyte, the article of action was instituted in this court. It is based upon two main propositions, to
incorporation of the Far Eastern Lumber and Commercial Co., Inc., wit:
organized to engage in a general lumber business to carry on as general
contractors, operators and managers, etc. Attached to the article was an
(a) The court had no jurisdiction in civil case No. 381 to decree the
affidavit of the treasurer stating that 23,428 shares of stock had been
dissolution of the company, because it being a de facto corporation,
subscribed and fully paid with certain properties transferred to the
dissolution thereof may only be ordered in a quo warranto proceeding
corporation described in a list appended thereto.
instituted in accordance with section 19 of the Corporation Law.

289
(b) Inasmuch as respondents Fred Brown and Emma Brown had signed the being. The immunity if collateral attack is granted to corporations
article of incorporation but only a partnership. "claiming in good faith to be a corporation under this act." Such a
claim is compatible with the existence of errors and irregularities; but
Discussion: The second proposition may at once be dismissed. All the parties not with a total or substantial disregard of the law. Unless there has
are informed that the Securities and Exchange Commission has not, so far, been an evident attempt to comply with the law the claim to be a
issued the corresponding certificate of incorporation. All of them know, or corporation "under this act" could not be made "in good faith." (Fisher
sought to know, that the personality of a corporation begins to exist only from on the Philippine Law of Stock Corporations, p. 75. See
the moment such certificate is issued not before (sec. 11, Corporation also Humphreys vs. Drew, 59 Fla., 295; 52 So., 362.)
Law). The complaining associates have not represented to the others that
they were incorporated any more than the latter had made similar Second, this is not a suit in which the corporation is a party. This is a litigation
representations to them. And as nobody was led to believe anything to his between stockholders of the alleged corporation, for the purpose of obtaining
prejudice and damage, the principle of estoppel does not apply. Obviously its dissolution. Even the existence of a de jure corporation may be terminated
this is not an instance requiring the enforcement of contracts with the in a private suit for its dissolution between stockholders, without the
corporation through the rule of estoppel. intervention of the state.

The first proposition above stated is premised on the theory that, inasmuch There might be room for argument on the right of minority stockholders to
as the Far Eastern Lumber and Commercial Co., is a de facto corporation, sue for dissolution;1 but that question does not affect the court's jurisdiction,
section 19 of the Corporation Law applies, and therefore the court had not and is a matter for decision by the judge, subject to review on appeal. Whkch
jurisdiction to take cognizance of said civil case number 381. Section 19 brings us to one principal reason why this petition may not prosper, namely:
reads as follows: the petitioners have their remedy by appealing the order of dissolution at the
proper time.
. . . The due incorporation of any corporations claiming in good faith
to be a corporation under this Act and its right to exercise corporate There is a secondary issue in connection with the appointment of a receiver.
powers shall not be inquired into collaterally in any private suit to But it must be admitted that receivership is proper in proceedings for
which the corporation may be a party, but such inquiry may be had at dissolution of a company or corporation, and it was no error to reject the
the suit of the Insular Government on information of the Attorney- counter-bond, the court having declared the dissolution. As to the amount of
General. the bond to be demanded of the receiver, much depends upon the discretion
of the trial court, which in this instance we do not believe has been clearly
There are least two reasons why this section does not govern the situation. abused.
Not having obtained the certificate of incorporation, the Far Eastern Lumber
and Commercial Co. even its stockholders may not probably claim "in Judgment: The petition will, therefore, be dismissed, with costs. The
good faith" to be a corporation. preliminary injunction heretofore issued will be dissolved.

Under our statue it is to be noted (Corporation Law, sec. 11) that it is Ozaeta, Pablo, Tuason, Montemayor, and Reyes, JJ., concur.
the issuance of a certificate of incorporation by the Director of the
Bureau of Commerce and Industry which calls a corporation into

290
291
Charles C. De Selms for appellant.
Gibbs & McDonough and Roman Ozaeta for appellee.

OSTRAND, J.:

This action is brought to recover the sum of P24,736.47, the balance due on
the following promissory note:

P37,757.22

MANILA, P. I., Nov. 28, 1921.

MANILA, P. I., Nov. 28, 1921.

On demand, after date we promise to pay to the Asia Banking


Corporation, or order, the sum of thirty-seven thousand seven
hundred fifty-seven and 22/100 pesos at their office in Manila, for
value received, together with interest at the rate of ten per cent per
annum.

No. ________ Due __________

THE STANDARD PRODUCTS CO., INC.


By (Sgd.) GEORGE H. SEAVER
G.R. No. 22106 September 11, 1924

ASIA BANKING CORPORATION, plaintiff-appellee,


vs. By President
STANDARD PRODUCTS, CO., INC., defendant-appellant.

292
The court below rendered judgment in favor of the plaintiff for the sum
demanded in the complaint, with interest on the sum of P24,147.34 from
November 1, 1923, at the rate of 10 per cent per annum, and the costs. From
this judgment the defendant appeals to this court.
G.R. No. L-11442 May 23, 1958
At the trial of the case the plaintiff failed to prove affirmatively the corporate
existence of the parties and the appellant insists that under these MANUELA T. VDA. DE SALVATIERRA, petitioner,
circumstances the court erred in finding that the parties were corporations vs.
with juridical personality and assigns same as reversible error. HON. LORENZO C. GARLITOS, in his capacity as Judge of the Court of
First Instance of Leyte, Branch II, and SEGUNDINO
There is no merit whatever in the appellant's contention. The general rule is REFUERZO, respondents.
that in the absence of fraud a person who has contracted or otherwise dealt
with an association in such a way as to recognize and in effect admit its legal Jimenez, Tantuico, Jr. and Tolete for petitioner.
existence as a corporate body is thereby estopped to deny its corporate Francisco Astilla for respondent Segundino Refuerzo.
existence in any action leading out of or involving such contract or dealing,
unless its existence is attacked for cause which have arisen since making the
FELIX, J.:
contract or other dealing relied on as an estoppel and this applies to foreign
as well as to domestic corporations. (14 C. J., 227; Chinese Chamber of
Commerce vs. Pua Te Ching, 14 Phil., 222.) This is a petition for certiorari filed by Manuela T. Vda. de Salvatierra seeking
to nullify the order of the Court of First Instance of Leyte in Civil Case No.
1912, dated March 21, 1956, relieving Segundino Refuerzo of liability for the
The defendant having recognized the corporate existence of the plaintiff by
contract entered into between the former and the Philippine Fibers Producers
making a promissory note in its favor and making partial payments on the
Co., Inc., of which Refuerzo is the president. The facts of the case are as
same is therefore estopped to deny said plaintiff's corporate existence. It is,
follows:
of course, also estopped from denying its own corporate existence. Under
these circumstances it was unnecessary for the plaintiff to present other
evidence of the corporate existence of either of the parties. It may be noted Manuela T. Vda. de Salvatierra appeared to be the owner of a parcel of land
that there is no evidence showing circumstances taking the case out of the located at Maghobas, Poblacion, Burauen, Teyte. On March 7, 1954, said
rules stated. landholder entered into a contract of lease with the Philippine Fibers
Producers Co., Inc., allegedly a corporation "duly organized and existing
under the laws of the Philippines, domiciled at Burauen, Leyte, Philippines,
The judgment appealed from is affirmed, with the costs against the appellant.
and with business address therein, represented in this instance by Mr.
So ordered.
Segundino Q. Refuerzo, the President". It was provided in said contract,
among other things, that the lifetime of the lease would be for a period of 10
Street, Malcolm, Avancea, Villamor and Romualdez, JJ., concur. years; that the land would be planted to kenaf, ramie or other crops suitable
to the soil; that the lessor would be entitled to 30 per cent of the net income
accruing from the harvest of any, crop without being responsible for the cost
of production thereof; and that after every harvest, the lessee was bound to

293
declare at the earliest possible time the income derived therefrom and to January 31, 1956, defendant Segundino Refuerzo filed a motion claiming
deliver the corresponding share due the lessor. that the decision rendered in said Civil Case No. 1912 was null and void with
respect to him, there being no allegation in the complaint pointing to his
Apparently, the aforementioned obligations imposed on the alleged personal liability and thus prayed that an order be issued limiting such liability
corporation were not complied with because on April 5, 1955, Alanuela T. to defendant corporation. Over plaintiff's opposition, the Court a quo granted
Vda, de Salvatierra filed with the Court of First Instance of Leyte a complaint the same and ordered the Provincial Sheriff of Leyte to release all properties
against the Philippine Fibers Producers Co., Inc., and Segundino Q. belonging to the movant that might have already been attached, after finding
Refuerzo, for accounting, rescission and damages (Civil Case No. 1912). that the evidence on record made no mention or referred to any fact which
She averred that sometime in April, 1954, defendants planted kenaf on 3 might hold movant personally liable therein. As plaintiff's petition for relief
hectares of the leased property which crop was, at the time of the from said order was denied, Manuela T. Vda. de Salvatierra instituted the
commencement of the action, already harvested, processed and sold by instant action asserting that the trial Judge in issuing the order complained
defendants; that notwithstanding that fact, defendants refused to render an of, acted with grave abuse of discretion and prayed that same be declared a
accounting of the income derived therefrom and to deliver the lessor's share; nullity.
that the estimated gross income was P4,500, and the deductible expenses
amounted to P1,000; that as defendants' refusal to undertake such task was From the foregoing narration of facts, it is clear that the order sought to be
in violation of the terms of the covenant entered into between the plaintiff and nullified was issued by tile respondent Judge upon motion of defendant
defendant corporation, a rescission was but proper. Refuerzo, obviously pursuant to Rule 38 of the Rules of Court. Section 3 of
said Rule, however, in providing for the period within which such a motion
As defendants apparently failed to file their answer to the complaint, of which may be filed, prescribes that:
they were allegedly notified, the Court declared them in default and
proceeded to receive plaintiff's evidence. On June 8, 1955, the lower Court SEC. 3. WHEN PETITION FILED; CONTENTS AND
rendered judgment granting plaintiff's prayer, and required defendants to VERIFICATION. A petition provided for in either of the preceding
render a complete accounting of the harvest of the land subject of the sections of this rule must be verified, filed within sixty days after the
proceeding within 15 days from receipt of the decision and to deliver 30 per petitioner learns of the judgment, order, or other proceeding to be set
cent of the net income realized from the last harvest to plaintiff, with legal aside, and not more than six months after such judgment or order
interest from the date defendants received payment for said crop. It was was entered, or such proceeding was taken; and must be must be
further provide that upon defendants' failure to abide by the said requirement, accompanied with affidavit showing the fraud, accident, mistake, or
the gross income would be fixed at P4,200 or a net income of P3,200 after excusable negligence relied upon, and the facts constituting the
deducting the expenses for production, 30 per cent of which or P960 was petitioner is good and substantial cause of action or defense, as the
held to be due the plaintiff pursuant to the aforementioned contract of lease, case may be, which he may prove if his petition be granted". (Rule
which was declared rescinded. 38)

No appeal therefrom having been perfected within the reglementary period, The aforequoted provision treats of 2 periods, i.e., 60 days after petitioner
the Court, upon motion of plaintiff, issued a writ of execution, in virtue of learns of the judgment, and not more than 6 months after the judgment or
which the Provincial Sheriff of Leyte caused the attachment of 3 parcels of order was rendered, both of which must be satisfied. As the decision in the
land registered in the name of Segundino Refuerzo. No property of the case at bar was under date of June 8, 1955, whereas the motion filed by
Philippine Fibers Producers Co., Inc., was found available for attachment. On respondent Refuerzo was dated January 31, 1956, or after the lapse of 7

294
months and 23 days, the filing of the aforementioned motion was clearly inescapable conclusion that plaintiff Manuela T. Vda. de Salvatierra was
made beyond the prescriptive period provided for by the rules. The remedy really made to believe that such corporation was duly organized in
allowed by Rule 38 to a party adversely affected by a decision or order is accordance with law.
certainly an alert of grace or benevolence intended to afford said litigant a
penultimate opportunity to protect his interest. Considering the nature of such There can be no question that a corporation with registered has a juridical
relief and the purpose behind it, the periods fixed by said rule are non- personality separate and distinct from its component members or
extendible and never interrupted; nor could it be subjected to any condition or stockholders and officers such that a corporation cannot be held liable for the
contingency because it is of itself devised to meet a condition or contingency personal indebtedness of a stockholder even if he should be its president
(Palomares vs. Jimenez,* G.R. No. L-4513, January 31, 1952). On this score (Walter A. Smith Co. vs. Ford, SC-G.R. No. 42420) and conversely, a
alone, therefore, the petition for a writ of certiorari filed herein may be stockholder or member cannot be held personally liable for any financial
granted. However, taking note of the question presented by the motion for obligation be, the corporation in excess of his unpaid subscription. But this
relief involved herein, We deem it wise to delve in and pass upon the merit of rule is understood to refer merely to registered corporations and cannot be
the same. made applicable to the liability of members of an unincorporated association.
The reason behind this doctrine is obvious-since an organization which
Refuerzo, in praying for his exoneration from any liability resulting from the before the law is non-existent has no personality and would be incompetent
non-fulfillment of the obligation imposed on defendant Philippine Fibers to act and appropriate for itself the powers and attribute of a corporation as
Producers Co., Inc., interposed the defense that the complaint filed with the provided by law; it cannot create agents or confer authority on another to act
lower court contained no allegation which would hold him liable personally, in its behalf; thus, those who act or purport to act as its representatives or
for while it was stated therein that he was a signatory to the lease contract, agents do so without authority and at their own risk. And as it is an
he did so in his capacity as president of the corporation. And this allegation elementary principle of law that a person who acts as an agent without
was found by the Court a quo to be supported by the records. Plaintiff on the authority or without a principal is himself regarded as the principal,
other hand tried to refute this averment by contending that her failure to possessed of all the rights and subject to all the liabilities of a principal, a
specify defendant's personal liability was due to the fact that all the time she person acting or purporting to act on behalf of a corporation which has no
was under the impression that the Philippine Fibers Producers Co., Inc., valid existence assumes such privileges and obligations and comes
represented by Refuerzo was a duly registered corporation as appearing in personally liable for contracts entered into or for other acts performed as
the contract, but a subsequent inquiry from the Securities and Exchange such, agent (Fay vs. Noble, 7 Cushing [Mass.] 188. Cited in II Tolentino's
Commission yielded otherwise. While as a general rule a person who has Commercial Laws of the Philippines, Fifth Ed., P. 689-690). Considering that
contracted or dealt with an association in such a way as to recognize its defendant Refuerzo, as president of the unregistered corporation Philippine
existence as a corporate body is estopped from denying the same in an Fibers Producers Co., Inc., was the moving spirit behind the consummation
action arising out of such transaction or dealing, (Asia Banking Corporation of the lease agreement by acting as its representative, his liability cannot be
vs. Standard Products Co., 46 Phil., 114; Compania Agricola de Ultramar vs. limited or restricted that imposed upon corporate shareholders. In acting on
Reyes, 4 Phil., 1; Ohta Development Co.; vs. Steamship Pompey, 49 Phil., behalf of a corporation which he knew to be unregistered, he assumed the
117), yet this doctrine may not be held to be applicable where fraud takes a risk of reaping the consequential damages or resultant rights, if any, arising
part in the said transaction. In the instant case, on plaintiff's charge that she out of such transaction.
was unaware of the fact that the Philippine Fibers Producers Co., Inc., had
no juridical personality, defendant Refuerzo gave no confirmation or denial Wherefore, the order of the lower Court of March 21, 1956, amending its
and the circumstances surrounding the execution of the contract lead to the previous decision on this matter and ordering the Provincial Sheriff of Leyte

295
to release any and all properties of movant therein which might have been
attached in the execution of such judgment, is hereby set aside and nullified
as if it had never been issued. With costs against respondent Segundino
Refuerzo. It is so ordered.

Paras, C.J., Bengzon, Montemayor, Reyes, A., Bautista Angelo, Labrador,


Concepcion, Reyes, J.B.L., and Endencia, JJ., concur.

G.R. No. L-19118 January 30, 1965

MARIANO A. ALBERT, plaintiff-appellant,


vs.
UNIVERSITY PUBLISHING CO., INC., defendant-appellee.

Uy & Artiaga and Antonio M. Molina for plaintiff-appellant.


Aruego, Mamaril & Associates for defendant-appellees.

BENGZON, J.P., J.:

No less than three times have the parties here appealed to this Court.

In Albert vs. University Publishing Co., Inc., L-9300, April 18, 1958, we found
plaintiff entitled to damages (for breach of contract) but reduced the amount
from P23,000.00 to P15,000.00.

Then in Albert vs. University Publishing Co., Inc., L-15275, October 24, 1960,
we held that the judgment for P15,000.00 which had become final and
executory, should be executed to its full amount, since in fixing it, payment
already made had been considered.

296
Now we are asked whether the judgment may be executed against Jose M. As aforesaid, we reduced the amount of damages to P15,000.00, to be
Aruego, supposed President of University Publishing Co., Inc., as the real executed in full. Thereafter, on July 22, 1961, the court a quo ordered
defendant. issuance of an execution writ against University Publishing Co., Inc. Plaintiff,
however, on August 10, 1961, petitioned for a writ of execution against Jose
Fifteen years ago, on September 24, 1949, Mariano A. Albert sued University M. Aruego, as the real defendant, stating, "plaintiff's counsel and the Sheriff
Publishing Co., Inc. Plaintiff alleged inter alia that defendant was a of Manila discovered that there is no such entity as University Publishing
corporation duly organized and existing under the laws of the Philippines; Co., Inc." Plaintiff annexed to his petition a certification from the securities
that on July 19, 1948, defendant, through Jose M. Aruego, its President, and Exchange Commission dated July 31, 1961, attesting: "The records of
entered into a contract with plaintifif; that defendant had thereby agreed to this Commission do not show the registration of UNIVERSITY PUBLISHING
pay plaintiff P30,000.00 for the exclusive right to publish his revised CO., INC., either as a corporation or partnership." "University Publishing Co.,
Commentaries on the Revised Penal Code and for his share in previous Inc." countered by filing, through counsel (Jose M. Aruego's own law firm), a
sales of the book's first edition; that defendant had undertaken to pay in eight "manifestation" stating that "Jose M. Aruego is not a party to this case," and
quarterly installments of P3,750.00 starting July 15, 1948; that per contract that, therefore, plaintiff's petition should be denied.
failure to pay one installment would render the rest due; and that defendant
had failed to pay the second installment. Parenthetically, it is not hard to decipher why "University Publishing Co.,
Inc.," through counsel, would not want Jose M. Aruego to be considered a
Defendant admitted plaintiff's allegation of defendant's corporate existence; party to the present case: should a separate action be now instituted against
admitted the execution and terms of the contract dated July 19, 1948; but Jose M. Aruego, the plaintiff will have to reckon with the statute of limitations.
alleged that it was plaintiff who breached their contract by failing to deliver his
manuscript. Furthermore, defendant counterclaimed for The court a quo denied the petition by order of September 9, 1961, and from
damages.1wph1.t this, plaintiff has appealed.

Plaintiff died before trial and Justo R. Albert, his estate's administrator, was The fact of non-registration of University Publishing Co., Inc. in the Securities
substituted for him. and Exchange Commission has not been disputed. Defendant would only
raise the point that "University Publishing Co., Inc.," and not Jose M. Aruego,
The Court of First Instance of Manila, after trial, rendered decision on April is the party defendant; thereby assuming that "University Publishing Co.,
26, 1954, stating in the dispositive portion Inc." is an existing corporation with an independent juridical personality.
Precisely, however, on account of the non-registration it cannot be
IN VIEW OF ALL THE FOREGOING, the Court renders judgment in considered a corporation, not even a corporation de facto (Hall vs. Piccio, 86
favor of the plaintiff and against the defendant the University Phil. 603). It has therefore no personality separate from Jose M. Aruego; it
Publishing Co., Inc., ordering the defendant to pay the administrator cannot be sued independently.
Justo R. Albert, the sum of P23,000.00 with legal [rate] of interest
from the date of the filing of this complaint until the whole amount The corporation-by-estoppel doctrine has not been invoked. At any rate, the
shall have been fully paid. The defendant shall also pay the costs. same is inapplicable here. Aruego represented a non-existent entity and
The counterclaim of the defendant is hereby dismissed for lack of induced not only the plaintiff but even the court to believe in such
evidence. representation. He signed the contract as "President" of "University

297
Publishing Co., Inc.," stating that this was "a corporation duly organized and 47 Phil. 23, 32)." (Sicat vs. Reyes, L-11023, Dec. 14, 1956.) And it may not
existing under the laws of the Philippines," and obviously misled plaintiff be amiss to mention here also that the "due process" clause of the
(Mariano A. Albert) into believing the same. One who has induced another to Constitution is designed to secure justice as a living reality; not to sacrifice it
act upon his wilful misrepresentation that a corporation was duly organized by paying undue homage to formality. For substance must prevail over form.
and existing under the law, cannot thereafter set up against his victim the It may now be trite, but none the less apt, to quote what long ago we said
principle of corporation by estoppel (Salvatiera vs. Garlitos, 56 O.G. 3069). in Alonso vs. Villamor, 16 Phil. 315, 321-322:

"University Publishing Co., Inc." purported to come to court, answering the A litigation is not a game of technicalities in which one, more deeply
complaint and litigating upon the merits. But as stated, "University Publishing schooled and skilled in the subtle art of movement and position,
Co., Inc." has no independent personality; it is just a name. Jose M. Aruego entraps and destroys the other. It is, rather, a contest in which each
was, in reality, the one who answered and litigated, through his own law firm contending party fully and fairly lays before the court the facts in
as counsel. He was in fact, if not, in name, the defendant. issue and then, brushing side as wholly trivial and indecisive all
imperfections of form and technicalities of procedure, asks that
Even with regard to corporations duly organized and existing under the law, Justice be done upon the merits. Lawsuits, unlike duels, are not to
we have in many a case pierced the veil of corporate fiction to administer the be won by a rapier's thrust. Technicality, when it deserts its proper
ends of justice. * And in Salvatiera vs. Garlitos, supra, p. 3073, we ruled: "A office as an aid to justice and becomes its great hindrance and chief
person acting or purporting to act on behalf of a corporation which has no enemy, deserves scant consideration from courts. There should be
valid existence assumes such privileges and obligations and becomes no vested rights in technicalities.
personally liable for contracts entered into or for other acts performed as
such agent." Had Jose M. Aruego been named as party defendant instead of, The evidence is patently clear that Jose M. Aruego, acting as representative
or together with, "University Publishing Co., Inc.," there would be no room for of a non-existent principal, was the real party to the contract sued upon; that
debate as to his personal liability. Since he was not so named, the matters of he was the one who reaped the benefits resulting from it, so much so that
"day in court" and "due process" have arisen. partial payments of the consideration were made by him; that he violated its
terms, thereby precipitating the suit in question; and that in the litigation he
In this connection, it must be realized that parties to a suit are "persons who was the real defendant. Perforce, in line with the ends of justice,
have a right to control the proceedings, to make defense, to adduce and responsibility under the judgment falls on him.
cross-examine witnesses, and to appeal from a decision" (67 C.J.S. 887)
and Aruego was, in reality, the person who had and exercised these rights. We need hardly state that should there be persons who under the law are
Clearly, then, Aruego had his day in court as the real defendant; and due liable to Aruego for reimbursement or contribution with respect to the
process of law has been substantially observed. payment he makes under the judgment in question, he may, of course,
proceed against them through proper remedial measures.
By "due process of law" we mean " "a law which hears before it condemns;
which proceeds upon inquiry, and renders judgment only after trial. ... ." (4 PREMISES CONSIDERED, the order appealed from is hereby set aside and
Wheaton, U.S. 518, 581.)"; or, as this Court has said, " "Due process of law" the case remanded ordering the lower court to hold supplementary
contemplates notice and opportunity to be heard before judgment is proceedings for the purpose of carrying the judgment into effect against
rendered, affecting one's person or property" (Lopez vs. Director of Lands, University Publishing Co., Inc. and/or Jose M. Aruego. So ordered.

298
Bengzon, C.J., Concepcion, Reyes, J.B.L., Barrera, Paredes, Dizon, Regala,
Makalintal and Zaldivar, JJ., concur.
Bautista Angelo, J., took no part. G.R. No. 136448 November 3, 1999

LIM TONG LIM, petitioner,


vs.
PHILIPPINE FISHING GEAR INDUSTRIES, INC., respondent.

PANGANIBAN, J.:

A partnership may be deemed to exist among parties who agree to borrow


money to pursue a business and to divide the profits or losses that may arise
therefrom, even if it is shown that they have not contributed any capital of
their own to a "common fund." Their contribution may be in the form of credit
or industry, not necessarily cash or fixed assets. Being partner, they are all
liable for debts incurred by or on behalf of the partnership. The liability for a
contract entered into on behalf of an unincorporated association or ostensible
corporation may lie in a person who may not have directly transacted on its
behalf, but reaped benefits from that contract.

The Case

In the Petition for Review on Certiorari before us, Lim Tong Lim assails the
November 26, 1998 Decision of the Court of Appeals in CA-GR CV
41477, 1 which disposed as follows:

WHEREFORE, [there being] no reversible error in the


appealed decision, the same is hereby affirmed. 2

The decretal portion of the Quezon City Regional Trial Court (RTC) ruling,
which was affirmed by the CA, reads as follows:

WHEREFORE, the Court rules:

299
1. That plaintiff is entitled to the writ of preliminary c. P50,000.00 as and for attorney's fees,
attachment issued by this Court on September 20, 1990; plus P8,500.00 representing P500.00 per
appearance in court;
2. That defendants are jointly liable to plaintiff for the
following amounts, subject to the modifications as d. P65,000.00 representing P5,000.00
hereinafter made by reason of the special and unique facts monthly rental for storage charges on the
and circumstances and the proceedings that transpired nets counted from September 20, 1990
during the trial of this case; (date of attachment) to September 12, 1991
(date of auction sale);
a. P532,045.00 representing [the] unpaid
purchase price of the fishing nets covered e. Cost of suit.
by the Agreement plus P68,000.00
representing the unpaid price of the floats With respect to the joint liability of defendants for the
not covered by said Agreement; principal obligation or for the unpaid price of nets
and floats in the amount of P532,045.00 and
b. 12% interest per annum counted from P68,000.00, respectively, or for the total amount
date of plaintiff's invoices and computed on P600,045.00, this Court noted that these items were
their respective amounts as follows: attached to guarantee any judgment that may be
rendered in favor of the plaintiff but, upon agreement
i. Accrued interest of of the parties, and, to avoid further deterioration of
P73,221.00 on Invoice No. the nets during the pendency of this case, it was
14407 for P385,377.80 ordered sold at public auction for not less than
dated February 9, 1990; P900,000.00 for which the plaintiff was the sole and
winning bidder. The proceeds of the sale paid for by
ii. Accrued interest for plaintiff was deposited in court. In effect, the amount
P27,904.02 on Invoice No. of P900,000.00 replaced the attached property as a
14413 for P146,868.00 guaranty for any judgment that plaintiff may be able
dated February 13, 1990; to secure in this case with the ownership and
possession of the nets and floats awarded and
delivered by the sheriff to plaintiff as the highest
iii. Accrued interest of
bidder in the public auction sale. It has also been
P12,920.00 on Invoice No.
noted that ownership of the nets [was] retained by
14426 for P68,000.00 dated
the plaintiff until full payment [was] made as
February 19, 1990;
stipulated in the invoices; hence, in effect, the
plaintiff attached its own properties. It [was] for this
reason also that this Court earlier ordered the
attachment bond filed by plaintiff to guaranty

300
damages to defendants to be cancelled and for the The buyers, however, failed to pay for the fishing nets and the floats; hence,
P900,000.00 cash bidded and paid for by plaintiff to private respondents filed a collection suit against Chua, Yao and Petitioner
serve as its bond in favor of defendants. Lim Tong Lim with a prayer for a writ of preliminary attachment. The suit was
brought against the three in their capacities as general partners, on the
From the foregoing, it would appear therefore that allegation that "Ocean Quest Fishing Corporation" was a nonexistent
whatever judgment the plaintiff may be entitled to in corporation as shown by a Certification from the Securities and Exchange
this case will have to be satisfied from the amount of Commission. 5 On September 20, 1990, the lower court issued a Writ of
P900,000.00 as this amount replaced the attached Preliminary Attachment, which the sheriff enforced by attaching the fishing
nets and floats. Considering, however, that the total nets on board F/B Lourdes which was then docked at the Fisheries Port,
judgment obligation as computed above would Navotas, Metro Manila.
amount to only P840,216.92, it would be inequitable,
unfair and unjust to award the excess to the Instead of answering the Complaint, Chua filed a Manifestation admitting his
defendants who are not entitled to damages and liability and requesting a reasonable time within which to pay. He also turned
who did not put up a single centavo to raise the over to respondent some of the nets which were in his possession. Peter Yao
amount of P900,000.00 aside from the fact that they filed an Answer, after which he was deemed to have waived his right to
are not the owners of the nets and floats. For this cross-examine witnesses and to present evidence on his behalf, because of
reason, the defendants are hereby relieved from any his failure to appear in subsequent hearings. Lim Tong Lim, on the other
and all liabilities arising from the monetary judgment hand, filed an Answer with Counterclaim and Crossclaim and moved for the
obligation enumerated above and for plaintiff to lifting of the Writ of Attachment. 6 The trial court maintained the Writ, and
retain possession and ownership of the nets and upon motion of private respondent, ordered the sale of the fishing nets at a
floats and for the reimbursement of the P900,000.00 public auction. Philippine Fishing Gear Industries won the bidding and
deposited by it with the Clerk of Court. deposited with the said court the sales proceeds of P900,000. 7

SO ORDERED. 3 On November 18, 1992, the trial court rendered its Decision, ruling that
Philippine Fishing Gear Industries was entitled to the Writ of Attachment and
The Facts that Chua, Yao and Lim, as general partners, were jointly liable to pay
respondent. 8
On behalf of "Ocean Quest Fishing Corporation," Antonio Chua and Peter
Yao entered into a Contract dated February 7, 1990, for the purchase of The trial court ruled that a partnership among Lim, Chua and Yao existed
fishing nets of various sizes from the Philippine Fishing Gear Industries, Inc. based (1) on the testimonies of the witnesses presented and (2) on a
(herein respondent). They claimed that they were engaged in a business Compromise Agreement executed by the three 9 in Civil Case No. 1492-MN
venture with Petitioner Lim Tong Lim, who however was not a signatory to which Chua and Yao had brought against Lim in the RTC of Malabon, Branch
the agreement. The total price of the nets amounted to P532,045. Four 72, for (a) a declaration of nullity of commercial documents; (b) a reformation
hundred pieces of floats worth P68,000 were also sold to the Corporation. 4 of contracts; (c) a declaration of ownership of fishing boats; (d) an injunction
and (e) damages. 10 The Compromise Agreement provided:

301
a) That the parties plaintiffs & Lim Tong Lim specific undertaking, that is for commercial fishing . . . .
agree to have the four (4) vessels sold in the Oviously, the ultimate undertaking of the defendants was to
amount of P5,750,000.00 including the divide the profits among themselves which is what a
fishing net. This P5,750,000.00 shall be partnership essentially is . . . . By a contract of partnership,
applied as full payment for P3,250,000.00 in two or more persons bind themselves to contribute money,
favor of JL Holdings Corporation and/or Lim property or industry to a common fund with the intention of
Tong Lim; dividing the profits among themselves (Article 1767, New
Civil Code). 13
b) If the four (4) vessel[s] and the fishing net
14
will be sold at a higher price than Hence, petitioner brought this recourse before this Court.
P5,750,000.00 whatever will be the excess
will be divided into 3: 1/3 Lim Tong Lim; 1/3 The Issues
Antonio Chua; 1/3 Peter Yao;
In his Petition and Memorandum, Lim asks this Court to reverse the assailed
c) If the proceeds of the sale the vessels will Decision on the following grounds:
be less than P5,750,000.00 whatever the
deficiency shall be shouldered and paid to I THE COURT OF APPEALS ERRED IN HOLDING, BASED
JL Holding Corporation by 1/3 Lim Tong Lim; ON A COMPROMISE AGREEMENT THAT CHUA, YAO AND
1/3 Antonio Chua; 1/3 Peter Yao. 11 PETITIONER LIM ENTERED INTO IN A SEPARATE CASE,
THAT A PARTNERSHIP AGREEMENT EXISTED AMONG
The trial court noted that the Compromise Agreement was silent as to the THEM.
nature of their obligations, but that joint liability could be presumed from the
equal distribution of the profit and loss. 21 II SINCE IT WAS ONLY CHUA WHO REPRESENTED THAT
HE WAS ACTING FOR OCEAN QUEST FISHING
Lim appealed to the Court of Appeals (CA) which, as already stated, affirmed CORPORATION WHEN HE BOUGHT THE NETS FROM
the RTC. PHILIPPINE FISHING, THE COURT OF APPEALS WAS
UNJUSTIFIED IN IMPUTING LIABILITY TO PETITIONER
Ruling of the Court of Appeals LIM AS WELL.

In affirming the trial court, the CA held that petitioner was a partner of Chua III THE TRIAL COURT IMPROPERLY ORDERED THE
and Yao in a fishing business and may thus be held liable as a such for the SEIZURE AND ATTACHMENT OF PETITIONER LIM'S
fishing nets and floats purchased by and for the use of the partnership. The GOODS.
appellate court ruled:
In determining whether petitioner may be held liable for the fishing nets and
The evidence establishes that all the defendants including floats from respondent, the Court must resolve this key issue: whether by
herein appellant Lim Tong Lim undertook a partnership for a

302
their acts, Lim, Chua and Yao could be deemed to have entered into a Specifically, both lower courts ruled that a partnership among the three
partnership. existed based on the following factual findings: 15

This Court's Ruling (1) That Petitioner Lim Tong Lim requested Peter Yao who
was engaged in commercial fishing to join him, while Antonio
The Petition is devoid of merit. Chua was already Yao's partner;

First and Second Issues: (2) That after convening for a few times, Lim, Chua, and Yao
verbally agreed to acquire two fishing boats, the FB
Existence of a Partnership Lourdes and the FB Nelson for the sum of P3.35 million;

and Petitioner's Liability (3) That they borrowed P3.25 million from Jesus Lim, brother
of Petitioner Lim Tong Lim, to finance the venture.
In arguing that he should not be held liable for the equipment purchased from
respondent, petitioner controverts the CA finding that a partnership existed (4) That they bought the boats from CMF Fishing
between him, Peter Yao and Antonio Chua. He asserts that the CA based its Corporation, which executed a Deed of Sale over these two
finding on the Compromise Agreement alone. Furthermore, he disclaims any (2) boats in favor of Petitioner Lim Tong Lim only to serve as
direct participation in the purchase of the nets, alleging that the negotiations security for the loan extended by Jesus Lim;
were conducted by Chua and Yao only, and that he has not even met the
representatives of the respondent company. Petitioner further argues that he (5) That Lim, Chua and Yao agreed that the refurbishing, re-
was a lessor, not a partner, of Chua and Yao, for the "Contract of Lease " equipping, repairing, dry docking and other expenses for the
dated February 1, 1990, showed that he had merely leased to the two the boats would be shouldered by Chua and Yao;
main asset of the purported partnership the fishing boat F/B Lourdes. The
lease was for six months, with a monthly rental of P37,500 plus 25 percent of (6) That because of the "unavailability of funds," Jesus Lim
the gross catch of the boat. again extended a loan to the partnership in the amount of P1
million secured by a check, because of which, Yao and Chua
We are not persuaded by the arguments of petitioner. The facts as found by entrusted the ownership papers of two other boats,
the two lower courts clearly showed that there existed a partnership among Chua's FB Lady Anne Mel and Yao's FB Tracy to Lim Tong
Chua, Yao and him, pursuant to Article 1767 of the Civil Code which Lim.
provides:
(7) That in pursuance of the business agreement, Peter Yao
Art. 1767 By the contract of partnership, two or more and Antonio Chua bought nets from Respondent Philippine
persons bind themselves to contribute money, property, or Fishing Gear, in behalf of "Ocean Quest Fishing
industry to a common fund, with the intention of dividing the Corporation," their purported business name.
profits among themselves.

303
(8) That subsequently, Civil Case No. 1492-MN was filed in We stress that under Rule 45, a petition for review like the present case
the Malabon RTC, Branch 72 by Antonio Chua and Peter should involve only questions of law. Thus, the foregoing factual findings of
Yao against Lim Tong Lim for (a) declaration of nullity of the RTC and the CA are binding on this Court, absent any cogent proof that
commercial documents; (b) reformation of contracts; (c) the present action is embraced by one of the exceptions to the rule. 16 In
declaration of ownership of fishing boats; (4) injunction; and assailing the factual findings of the two lower courts, petitioner effectively
(e) damages. goes beyond the bounds of a petition for review under Rule 45.

(9) That the case was amicably settled through a Compromise Agreement
Compromise Agreement executed between the parties-
litigants the terms of which are already enumerated above. Not the Sole Basis of Partnership

From the factual findings of both lower courts, it is clear that Chua, Yao and Petitioner argues that the appellate court's sole basis for assuming the
Lim had decided to engage in a fishing business, which they started by existence of a partnership was the Compromise Agreement. He also claims
buying boats worth P3.35 million, financed by a loan secured from Jesus Lim that the settlement was entered into only to end the dispute among them, but
who was petitioner's brother. In their Compromise Agreement, they not to adjudicate their preexisting rights and obligations. His arguments are
subsequently revealed their intention to pay the loan with the proceeds of the baseless. The Agreement was but an embodiment of the relationship extant
sale of the boats, and to divide equally among them the excess or loss. among the parties prior to its execution.
These boats, the purchase and the repair of which were financed with
borrowed money, fell under the term "common fund" under Article 1767. The A proper adjudication of claimants' rights mandates that courts must review
contribution to such fund need not be cash or fixed assets; it could be an and thoroughly appraise all relevant facts. Both lower courts have done so
intangible like credit or industry. That the parties agreed that any loss or profit and have found, correctly, a preexisting partnership among the parties. In
from the sale and operation of the boats would be divided equally among implying that the lower courts have decided on the basis of one piece of
them also shows that they had indeed formed a partnership. document alone, petitioner fails to appreciate that the CA and the RTC
delved into the history of the document and explored all the possible
Moreover, it is clear that the partnership extended not only to the purchase of consequential combinations in harmony with law, logic and fairness. Verily,
the boat, but also to that of the nets and the floats. The fishing nets and the the two lower courts' factual findings mentioned above nullified petitioner's
floats, both essential to fishing, were obviously acquired in furtherance of argument that the existence of a partnership was based only on the
their business. It would have been inconceivable for Lim to involve himself so Compromise Agreement.
much in buying the boat but not in the acquisition of the aforesaid equipment,
without which the business could not have proceeded. Petitioner Was a Partner,

Given the preceding facts, it is clear that there was, among petitioner, Chua Not a Lessor
and Yao, a partnership engaged in the fishing business. They purchased the
boats, which constituted the main assets of the partnership, and they agreed
We are not convinced by petitioner's argument that he was merely the lessor
that the proceeds from the sales and operations thereof would be divided
of the boats to Chua and Yao, not a partner in the fishing venture. His
among them.
argument allegedly finds support in the Contract of Lease and the registration

304
papers showing that he was the owner of the boats, including F/B corporation is sued on any transaction entered by it as a
Lourdes where the nets were found. corporation or on any tort committed by it as such, it shall not
be allowed to use as a defense its lack of corporate
His allegation defies logic. In effect, he would like this Court to believe that he personality.
consented to the sale of his own boats to pay a debt of Chua and Yao, with
the excess of the proceeds to be divided among the three of them. No lessor One who assumes an obligation to an ostensible corporation
would do what petitioner did. Indeed, his consent to the sale proved that as such, cannot resist performance thereof on the ground
there was a preexisting partnership among all three. that there was in fact no corporation.

Verily, as found by the lower courts, petitioner entered into a business Thus, even if the ostensible corporate entity is proven to be legally
agreement with Chua and Yao, in which debts were undertaken in order to nonexistent, a party may be estopped from denying its corporate existence.
finance the acquisition and the upgrading of the vessels which would be used "The reason behind this doctrine is obvious an unincorporated association
in their fishing business. The sale of the boats, as well as the division among has no personality and would be incompetent to act and appropriate for itself
the three of the balance remaining after the payment of their loans, proves the power and attributes of a corporation as provided by law; it cannot create
beyond cavil that F/B Lourdes, though registered in his name, was not his agents or confer authority on another to act in its behalf; thus, those who act
own property but an asset of the partnership. It is not uncommon to register or purport to act as its representatives or agents do so without authority and
the properties acquired from a loan in the name of the person the lender at their own risk. And as it is an elementary principle of law that a person who
trusts, who in this case is the petitioner himself. After all, he is the brother of acts as an agent without authority or without a principal is himself regarded
the creditor, Jesus Lim. as the principal, possessed of all the right and subject to all the liabilities of a
principal, a person acting or purporting to act on behalf of a corporation
We stress that it is unreasonable indeed, it is absurd for petitioner to which has no valid existence assumes such privileges and obligations and
sell his property to pay a debt he did not incur, if the relationship among the becomes personally liable for contracts entered into or for other acts
three of them was merely that of lessor-lessee, instead of partners. performed as such agent. 17

Corporation by Estoppel The doctrine of corporation by estoppel may apply to the alleged corporation
and to a third party. In the first instance, an unincorporated association,
Petitioner argues that under the doctrine of corporation by estoppel, liability which represented itself to be a corporation, will be estopped from denying its
can be imputed only to Chua and Yao, and not to him. Again, we disagree. corporate capacity in a suit against it by a third person who relied in good
faith on such representation. It cannot allege lack of personality to be sued to
evade its responsibility for a contract it entered into and by virtue of which it
Sec. 21 of the Corporation Code of the Philippines provides:
received advantages and benefits.

Sec. 21. Corporation by estoppel. All persons who


On the other hand, a third party who, knowing an association to be
assume to act as a corporation knowing it to be without
unincorporated, nonetheless treated it as a corporation and received benefits
authority to do so shall be liable as general partners for all
from it, may be barred from denying its corporate existence in a suit brought
debts, liabilities and damages incurred or arising as a result
against the alleged corporation. In such case, all those who benefited from
thereof: Provided however, That when any such ostensible

305
the transaction made by the ostensible corporation, despite knowledge of its as wholly trivial and indecisive all imperfections of form and
legal defects, may be held liable for contracts they impliedly assented to or technicalities of procedure, asks that justice be done upon
took advantage of. the merits. Lawsuits, unlike duels, are not to be won by a
rapier's thrust. Technicality, when it deserts its proper office
There is no dispute that the respondent, Philippine Fishing Gear Industries, is as an aid to justice and becomes its great hindrance and
entitled to be paid for the nets it sold. The only question here is whether chief enemy, deserves scant consideration from courts.
petitioner should be held jointly 18 liable with Chua and Yao. Petitioner There should be no vested rights in technicalities.
contests such liability, insisting that only those who dealt in the name of the
ostensible corporation should be held liable. Since his name does not appear Third Issue:
on any of the contracts and since he never directly transacted with the
respondent corporation, ergo, he cannot be held liable. Validity of Attachment

Unquestionably, petitioner benefited from the use of the nets found Finally, petitioner claims that the Writ of Attachment was improperly issued
inside F/B Lourdes, the boat which has earlier been proven to be an asset of against the nets. We agree with the Court of Appeals that this issue is now
the partnership. He in fact questions the attachment of the nets, because the moot and academic. As previously discussed, F/B Lourdes was an asset of
Writ has effectively stopped his use of the fishing vessel. the partnership and that it was placed in the name of petitioner, only to
assure payment of the debt he and his partners owed. The nets and the
It is difficult to disagree with the RTC and the CA that Lim, Chua and Yao floats were specifically manufactured and tailor-made according to their own
decided to form a corporation. Although it was never legally formed for design, and were bought and used in the fishing venture they agreed upon.
unknown reasons, this fact alone does not preclude the liabilities of the three Hence, the issuance of the Writ to assure the payment of the price stipulated
as contracting parties in representation of it. Clearly, under the law on in the invoices is proper. Besides, by specific agreement, ownership of the
estoppel, those acting on behalf of a corporation and those benefited by it, nets remained with Respondent Philippine Fishing Gear, until full payment
knowing it to be without valid existence, are held liable as general partners. thereof.

Technically, it is true that petitioner did not directly act on behalf of the WHEREFORE, the Petition is DENIED and the assailed Decision
corporation. However, having reaped the benefits of the contract entered into AFFIRMED. Costs against petitioner.
by persons with whom he previously had an existing relationship, he is
deemed to be part of said association and is covered by the scope of the SO ORDERED.
doctrine of corporation by estoppel. We reiterate the ruling of the Court
in Alonso v. Villamor: 19 Melo, Purisima and Gonzaga-Reyes, JJ., concur.

A litigation is not a game of technicalities in which one, more Vitug, J., pls. see concurring opinion.
deeply schooled and skilled in the subtle art of movement
and position, entraps and destroys the other. It is, rather, a
contest in which each contending party fully and fairly lays
before the court the facts in issue and then, brushing aside

306
307
REGALADO, J.:

This is a petition to review the decision of respondent Court of Appeals,


dated August 3, 1989, in CA-GR CV No. 15412, entitled "Buenaflor M.
Castillo Umali, et al. vs. Philippine Machinery Parts Manufacturing Co., Inc.,
et al.," 1the dispositive portion whereof provides:

WHEREFORE, viewed in the light of the entire record, the


judgment appealed from must be, as it is hereby
REVERSED. In lieu thereof, a judgment is hereby rendered-

1) Dismissing the complaint, with cost against plaintiffs;

2) Ordering plaintiffs-appellees to vacate the subject


properties; and

3) Ordering plaintiffs-appellees to pay upon defendants'


counterclaims:

G.R. No. 89561 September 13, 1990


a) To defendant-appellant PM Parts: (i)
damages consisting of the value of the fruits
BUENAFLOR C. UMALI, MAURICIA M. VDA. DE CASTILLO, VICTORIA M. in the subject parcels of land of which they
CASTILLO, BERTILLA C. RADA, MARIETTA C. ABAEZ, LEOVINA C. were deprived in the sum of P26,000.00 and
JALBUENA and SANTIAGO M. RIVERA, petitioners, (ii) attorney's fees of P15,000.00
vs.
COURT OF APPEALS, BORMAHECO, INC. and PHILIPPINE MACHINERY
b) To defendant-appellant Bormaheco: (i)
PARTS MANUFACTURING CO., INC., respondents.
expenses of litigation in the amount of
P5,000.00 and (ii) attorney's fees of
Edmundo T. Zepeda for petitioners. P15,000.00.

Martin M. De Guzman for respondent BORMAHECO, Inc. SO ORDERED.

Renato J. Robles for P.M. Parts Manufacturing Co., Inc.

308
The original complaint for annulment of title filed in the court a quo by herein c) That the above-enumerated four (4)
petitioners included as party defendants the Philippine Machinery Parts parcels of land were the subject of the Deed
Manufacturing Co., Inc. (PM Parts), Insurance Corporation of the Philippines of Extra-Judicial Partition executed by the
(ICP), Bormaheco, Inc., (Bormaheco) and Santiago M. Rivera (Rivera). A heirs of Felipe Castillo (per Exhibit D) and by
Second Amended Complaint was filed, this time impleading Santiago M. virtue thereof the titles thereto has (sic) been
Rivera as party plaintiff. cancelled and in lieu thereof, new titles in
the name of Mauricia Meer Vda. de Castillo
During the pre-trial conference, the parties entered into the following and of her children, namely: Buenaflor,
stipulation of facts: Bertilla, Victoria, Marietta and Leovina, all
surnamed Castillo has (sic) been issued,
As between all parties: Plaintiff Buenaflor M. namely: TCT No. T-12113 (Exhibit E ); TCT
Castillo is the judicial administratrix of the No. T-13113 (Exhibit F); TCT No. T-13116
estate of Felipe Castillo in Special (Exhibit G ) and TCT No. T13117 (Exhibit H )
Proceeding No. 4053, pending before
Branch IX, CFI of Quezon (per Exhibit A) d) That mentioned parcels of land were
which intestate proceedings was instituted submitted as guaranty in the Agreement of
by Mauricia Meer Vda. de Castillo, the Counter-Guaranty with Chattel-Real Estate
previous administratrix of the said Mortgage executed on 24 October 1970
proceedings prior to 1970 (per exhibits A-1 between Insurance Corporation of the
and A-2) which case was filed in Court way Philippines and Slobec Realty Corporation
back in 1964; represented by Santiago Rivera (Exhibit 1);

b) The four (4) parcels of land described in e) That based on the Certificate of Sale
paragraph 3 of the Complaint were originally issued by the Sheriff of the Province of
covered by TCT No. T-42104 and Tax Dec. Quezon in favor of Insurance Corporation of
No. 14134 with assessed value of the Philippines it was able to transfer to itself
P3,100.00; TCT No. T 32227 and Tax Dec. the titles over the lots in question, namely:
No. 14132, with assessed value of TCT No. T-23705 (Exhibit M), TCT No. T
P5,130,00; TCT No. T-31762 and Tax Dec. 23706 (Exhibit N ), TCT No. T-23707
No. 14135, with assessed value of (Exhibit 0) and TCT No. T 23708 (Exhibit P);
P6,150.00; and TCT No. T-42103 with Tax
Dec. No. 14133, with assessed value of f) That on 10 April 1975, the Insurance
P3,580.00 (per Exhibits A-2 and B, B-1 to B- Corporation of the Philippines sold to PM
3 C, C-1 -to C3 Parts the immovables in question (per
Exhibit 6 for PM Parts) and by reason
thereof, succeeded in transferring unto itself
the titles over the lots in dispute, namely: per

309
TCT No. T-24846 (Exhibit Q ), per TCT No. j) That the Surety Bond No. 14010 issued by
T-24847 (Exhibit R ), TCT No. T-24848 co-defendant ICP was likewise secured by
(Exhibit), TCT No. T-24849 (Exhibit T ); an Agreement with Counter-Guaranty with
Real Estate Mortgage executed by Slobec
g) On 26 August l976, Mauricia Meer Vda. Realty & Development, Inc., Mauricia
de Castillo' genther letter to Modesto N. Castillo Meer, Buenaflor Castillo, Bertilla
Cervantes stating that she and her children Castillo, Victoria Castillo, Marietta Castillo
refused to comply with his demands (Exhibit and Leovina Castillo, as mortgagors in favor
V-2); of ICP which document was executed and
ratified before notary public Alberto R.
h) That from at least the months of October, Navoa of the City of Manila on October
November and December 1970 and January 24,1970;
1971, Modesto N. Cervantes was the Vice-
President of Bormaheco, Inc. later President k) That the property mortgaged consisted of
thereof, and also he is one of the Board of four (4) parcels of land situated in Lucena
Directors of PM Parts; on the other hand, City and covered by TCT Nos. T-13114,
Atty. Martin M. De Guzman was the legal T13115,
counsel of Bormaheco, Inc., later Executive T-13116 and T-13117 of the Register of
Vice-President thereof, and who also is the Deeds of Lucena City;
legal counsel of Insurance Corporation of
the Philippines and PM Parts; that Modesto l) That the tractor sold by defendant
N. Cervantes served later on as President of Bormaheco, Inc. to Slobec Realty &
PM Parts, and that Atty. de Guzman was Development, Inc. was delivered to
retained by Insurance Corporation of the Bormaheco, Inc. on or about October
Philippines specifically for foreclosure 2,1973, by Mr. Menandro Umali for purposes
purposes only; of repair;

i) Defendant Bormaheco, Inc. on November m) That in August 1976, PM Parts notified


25, 1970 sold to Slobec Realty and Mrs. Mauricia Meer about its ownership and
Development, Inc., represented by Santiago the assignment of Mr. Petronilo Roque as
Rivera, President, one (1) unit Caterpillar caretaker of the subject property;
Tractor D-7 with Serial No. 281114
evidenced by a contract marked Exhibit J n) That plaintiff and other heirs are harvest
and Exhibit I for Bormaheco, Inc.; fruits of the property (daranghita) which is
worth no less than Pl,000.00 per harvest.

310
As between plaintiffs and them that those two documents were
defendant Bormaheco, Inc executed on 11 December 1970. The
certified xerox copy of the notarial register of
o) That on 25 November 1970, at Makati, Notary Public Guillermo Aragones issued by
Rizal, Same Rivera, in representation of the the Bureau of Records Management is
Slobec Realty & Development Corporation hereto submitted as Exhibit BB That said
executed in favor of Bormaheco, Inc., chattel mortgage was superseded by
represented by its Vice-President Modesto another document dated January 23, 1971;
N. Cervantes a Chattel Mortgage concerning
one unit model CAT D7 Caterpillar Crawler r) That on 23 January 1971, Slobec Realty
Tractor as described therein as security for Development Corporation, represented by
the payment in favor of the mortgagee of the Santiago Rivera, received from Bormaheco,
amount of P180,000.00 (per Exhibit K) that Inc. one (1) tractor Caterpillar Model D-7
Id document was superseded by another pursuant to Invoice No. 33234 (Exhibits 9
chattel mortgage dated January 23, 1971 and 9-A, Bormaheco, Inc.) and delivery
(Exhibit 15); receipt No. 10368 (per Exhibits 10 and 10-A
for Bormaheco, Inc
p) On 18 December 1970, at Makati, Rizal,
the Bormaheco, Inc., represented by its s) That on 28 September 1973, Atty. Martin
Vice-President Modesto Cervantes and M. de Guzman, as counsel of Insurance
Slobec Realty Corporation represented by Corporation of the Philippines purchased at
Santiago Rivera executed the sales public auction for said corporation the four
agreement concerning the sale of one (1) (4) parcels of land subject of tills case (per
unit Model CAT D7 Caterpillar Crawler Exhibit L), and which document was
Tractor as described therein for the amount presented to the Register of Deeds on 1
of P230,000.00 (per Exhibit J) which October 1973;
document was superseded by the Sales
Agreement dated January 23,1971 (Exhibit t) Although it appears that the realties in
16); issue has (sic) been sold by Insurance
Corporation of the Philippines in favor of PM
q) Although it appears on the document Parts on 1 0 April 1975, Modesto N.
entitled Chattel Mortgage (per Exhibit K) that Cervantes, formerly Vice- President and
it was executed on 25 November 1970, and now President of Bormaheco, Inc., sent his
in the document entitled Sales Agreement letter dated 9 August 1976 to Mauricia Meer
(per Exhibit J) that it was executed on 18 Vda. de Castillo (Exhibit V), demanding that
December 1970, it appears in the notarial she and her children should vacate the
register of the notary public who notarized premises;

311
u) That the Caterpillar Crawler Tractor Model evidenced by the contract marked Exhibit '16'. As shown by
CAT D-7 which was received by Slobec the contract, the price was P230,000.00 of which P50,000.00
Realty Development Corporation was was to constitute a down payment, and the balance of
actually reconditioned and repainted. " 2 P180,000.00 payable in eighteen monthly installments. On
the same date, Slobec, through Rivera, executed in favor of
We cull the following antecedents from the decision of respondent Court of Bormaheco a Chattel Mortgage (Exh. K, p. 29, Record) over
Appeals: the said equipment as security for the payment of the
aforesaid balance of P180,000.00. As further security of the
Plaintiff Santiago Rivera is the nephew of plaintiff Mauricia aforementioned unpaid balance, Slobec obtained from
Meer Vda. de Castillo. The Castillo family are the owners of Insurance Corporation of the Phil. a Surety Bond, with ICP
a parcel of land located in Lucena City which was given as (Insurance Corporation of the Phil.) as surety and Slobec as
security for a loan from the Development Bank of the principal, in favor of Bormaheco, as borne out by Exhibit '8'
Philippines. For their failure to pay the amortization, (p. 111, Record). The aforesaid surety bond was in turn
foreclosure of the said property was about to be initiated. secured by an Agreement of Counter-Guaranty with Real
This problem was made known to Santiago Rivera, who Estate Mortgage (Exhibit I, p. 24, Record) executed by
proposed to them the conversion into subdivision of the four Rivera as president of Slobec and Mauricia Meer Vda. de
(4) parcels of land adjacent to the mortgaged property to Castillo, Buenaflor Castillo Umali, Bertilla Castillo-Rada,
raise the necessary fund. The Idea was accepted by the Victoria Castillo, Marietta Castillo and Leovina Castillo
Castillo family and to carry out the project, a Memorandum Jalbuena, as mortgagors and Insurance Corporation of the
of Agreement (Exh. U p. 127, Record) was executed by and Philippines (ICP) as mortgagee. In this agreement, ICP
between Slobec Realty and Development, Inc., represented guaranteed the obligation of Slobec with Bormaheco in the
by its President Santiago Rivera and the Castillo family. In amount of P180,000.00. In giving the bond, ICP required that
this agreement, Santiago Rivera obliged himself to pay the the Castillos mortgage to them the properties in question,
Castillo family the sum of P70,000.00 immediately after the namely, four parcels of land covered by TCTs in the name of
execution of the agreement and to pay the additional amount the aforementioned mortgagors, namely TCT Nos. 13114,
of P400,000.00 after the property has been converted into a 13115, 13116 and 13117 all of the Register of Deeds for
subdivision. Rivera, armed with the agreement, Exhibit U , Lucena City.
approached Mr. Modesto Cervantes, President of defendant
Bormaheco, and proposed to purchase from Bormaheco two On the occasion of the execution on January 23, 1971, of
(2) tractors Model D-7 and D-8 Subsequently, a Sales the Sales Agreement Exhibit '16', Slobec, represented by
Agreement was executed on December 28,1970 (Exh. J, p. Rivera received from Bormaheco the subject matter of the
22, Record). said Sales Agreement, namely, the aforementioned tractor
Caterpillar Model D-7 as evidenced by Invoice No. 33234
On January 23, 1971, Bormaheco, Inc. and Slobec Realty (Exhs. 9 and 9-A, p. 112, Record) and Delivery Receipt No.
and Development, Inc., represented by its President, 10368 (Exhs. 10 and 10-A, p. 113). This tractor was known
Santiago Rivera, executed a Sales Agreement over one unit by Rivera to be a reconditioned and repainted one
of Caterpillar Tractor D-7 with Serial No. 281114, as [Stipulation of Facts, Pre-trial Order, par. (u)].

312
Meanwhile, for violation of the terms and conditions of the Instance of Quezon and docketed thereat as Civil Case No.
Counter-Guaranty Agreement (Exh. 1), the properties of the 8085. Thereafter, they filed an Amended Complaint on
Castillos were foreclosed by ICP As the highest bidder with a January 10, 1980 (p. 444, Record). On July 20, 1983,
bid of P285,212.00, a Certificate of Sale was issued by the plaintiffs filed their Second Amended Complaint, impleading
Provincial Sheriff of Lucena City and Transfer Certificates of Santiago M. Rivera as a party plaintiff (p. 706, Record). They
Title over the subject parcels of land were issued by the contended that all the aforementioned transactions starting
Register of Deeds of Lucena City in favor of ICP namely, with the Agreement of Counter-Guaranty with Real Estate
TCT Nos. T-23705, T 23706, T-23707 and T-23708 (Exhs. M Mortgage (Exh. I), Certificate of Sale (Exh. L) and the Deeds
to P, pp. 38-45). The mortgagors had one (1) year from the of Authority to Sell, Sale and the Affidavit of Consolidation of
date of the registration of the certificate of sale, that is, until Ownership (Annexes F, G, H, I) as well as the Deed of Sale
October 1, 1974, to redeem the property, but they failed to (Annexes J, K, L and M) are void for being entered into in
do so. Consequently, ICP consolidated its ownership over fraud and without the consent and approval of the Court of
the subject parcels of land through the requisite affidavit of First Instance of Quezon, (Branch IX) before whom the
consolidation of ownership dated October 29, 1974, as administration proceedings has been pending. Plaintiffs pray
shown in Exh. '22'(p. 138, Rec.). Pursuant thereto, a Deed of that the four (4) parcels of land subject hereof be declared
Sale of Real Estate covering the subject properties was as owned by the estate of the late Felipe Castillo and that all
issued in favor of ICP (Exh. 23, p. 139, Rec.). Transfer Certificates of Title Nos. 13114,13115,13116,13117,
23705, 23706, 23707, 23708, 24846, 24847, 24848 and
On April 10, 1975, Insurance Corporation of the Phil. ICP 24849 as well as those appearing as encumbrances at the
sold to Phil. Machinery Parts Manufacturing Co. (PM Parts) back of the certificates of title mentioned be declared as a
the four (4) parcels of land and by virtue of said conveyance, nullity and defendants to pay damages and attorney's fees
PM Parts transferred unto itself the titles over the lots in (pp. 71071 1, Record).
dispute so that said parcels of land are now covered by TCT
Nos. T-24846, T-24847, T-24848 and T-24849 (Exhs. Q-T, In their amended answer, the defendants controverted the
pp. 46-49, Rec.). complaint and alleged, by way of affirmative and special
defenses that the complaint did not state facts sufficient to
Thereafter, PM Parts, through its President, Mr. Modesto state a cause of action against defendants; that plaintiffs are
Cervantes, sent a letter dated August 9,1976 addressed to not entitled to the reliefs demanded; that plaintiffs are
plaintiff Mrs. Mauricia Meer Castillo requesting her and her estopped or precluded from asserting the matters set forth in
children to vacate the subject property, who (Mrs. Castillo) in the Complaint; that plaintiffs are guilty of laches in not
turn sent her reply expressing her refusal to comply with his asserting their alleged right in due time; that defendant PM
demands. Parts is an innocent purchaser for value and relied on the
face of the title before it bought the subject property (p. 744,
On September 29, 1976, the heirs of the late Felipe Castillo, Record). 3
particularly plaintiff Buenaflor M. Castillo Umali as the
appointed administratrix of the properties in question filed an After trial, the court a quo rendered judgment, with the
action for annulment of title before the then Court of First following decretal portion:

313
WHEREFORE, judgment is hereby rendered in favor of the name of Philippine Machinery Parts Manufacturing Co., Inc.,
plaintiffs and against the defendants, declaring the following are similarly declared null and void, and the Register of
documents: Deeds of Lucena City is hereby directed to issue, in lieu
thereof, transfer certificates of title in the names of the
Agreement of Counter-Guaranty with plaintiffs, except Santiago Rivera.
Chattel-Real Estate Mortgage dated October
24,1970 (Exhibit 1); Orders the defendants jointly and severally to pay the
plaintiffs moral damages in the sum of P10,000.00,
Sales Agreement dated December 28, 1970 exemplary damages in the amount of P5,000.00, and actual
(Exhibit J) litigation expenses in the sum of P6,500.00.

Chattel Mortgage dated November 25, 1970 Defendants are likewise ordered to pay the plaintiffs, jointly
(Exhibit K) and severally, the sum of P10,000.00 for and as attomey's
fees. With costs against the defendants.
Sales Agreement dated January 23, 1971
(Exhibit 16); SO ORDERED. 4

Chattel Mortgage dated January 23, 1971 As earlier stated, respondent court reversed the aforequoted decision of the
(Exhibit 17); trial court and rendered the judgment subject of this petition-

Certificate of Sale dated September 28, Petitioners contend that respondent Court of Appeals erred:
1973 executed by the Provincial Sheriff of
Quezon in favor of Insurance Corporation of 1. In holding and finding that the actions entered into
the Philippines (Exhibit L); between petitioner Rivera with Cervantes are all fair and
regular and therefore binding between the parties thereto;
null and void for being fictitious, spurious and without
consideration. Consequently, Transfer Certificates of Title 2. In reversing the decision of the lower court, not only based
Nos. T 23705, T-23706, T23707 and T-23708 (Exhibits M, N, on erroneous conclusions of facts, erroneous presumptions
O and P) issued in the name of Insurance Corporation of the not supported by the evidence on record but also, holding
Philippines, are likewise null and void. valid and binding the supposed payment by ICP of its
obligation to Bormaheco, despite the fact that the surety
The sale by Insurance Corporation of the- Philippines in bond issued it had already expired when it opted to foreclose
favor of defendant Philippine Machinery Parts Manufacturing extrajudically the mortgage executed by the petitioners;
Co., Inc., over Id four (4) parcels of land and Transfer
Certificates of Title Nos. T 24846, T-24847, T-24848 and T- 3. In aside the finding of the lower court that there was
24849 subsequently issued by virtue of said sale in the necessity to pierce the veil of corporate existence; and

314
4. In reversing the decision of the lower court of affirming the declaration of nullity of the contract. Equity and fair play dictates that one
same 5 who commits a breach of his contract may not seek refuge under the
protective mantle of the law.
I. Petitioners aver that the transactions entered into between Santiago M.
Rivera, as President of Slobec Realty and Development Company (Slobec) The evidence of record, on an overall calibration, does not convince us of the
and Mode Cervantes, as Vice-President of Bormaheco, such as the Sales validity of petitioners' contention that the contracts entered into by the parties
Agreement, 6 Chattel Mortgage 7 and the Agreement of Counter-Guaranty are either absolutely simulated or downright fraudulent.
with Chattel/Real Estate Mortgage, 8 are all fraudulent and simulated and
should, therefore, be declared nun and void. Such allegation is premised There is absolute simulation, which renders the contract null and void, when
primarily on the fact that contrary to the stipulations agreed upon in the Sales the parties do not intend to be bound at all by the same. 9 The basic
Agreement (Exhibit J), Rivera never made any advance payment, in the characteristic of this type of simulation of contract is the fact that the
alleged amount of P50,000.00, to Bormaheco; that the tractor was received apparent contract is not really desired or intended to either produce legal
by Rivera only on January 23, 1971 and not in 1970 as stated in the Chattel effects or in any way alter the juridical situation of the parties. The
Mortgage (Exhibit K); and that when the Agreement of Counter-Guaranty with subsequent act of Rivera in receiving and making use of the tractor subject
Chattel/Real Estate Mortgage was executed on October 24, 1970, to secure matter of the Sales Agreement and Chattel Mortgage, and the simultaneous
the obligation of ICP under its surety bond, the Sales Agreement and Chattel issuance of a surety bond in favor of Bormaheco, concomitant with the
Mortgage had not as yet been executed, aside from the fact that it was execution of the Agreement of Counter-Guaranty with Chattel/Real Estate
Bormaheco, and not Rivera, which paid the premium for the surety bond Mortgage, conduce to the conclusion that petitioners had every intention to
issued by ICP be bound by these contracts. The occurrence of these series of transactions
between petitioners and private respondents is a strong indication that the
At the outset, it will be noted that petitioners submission under the first parties actually intended, or at least expected, to exact fulfillment of their
assigned error hinges purely on questions of fact. Respondent Court of respective obligations from one another.
Appeals made several findings to the effect that the questioned documents
are valid and binding upon the parties, that there was no fraud employed by Neither will an allegation of fraud prosper in this case where petitioners failed
private respondents in the execution thereof, and that, contrary to petitioners' to show that they were induced to enter into a contract through the insidious
allegation, the evidence on record reveals that petitioners had every intention words and machinations of private respondents without which the former
to be bound by their undertakings in the various transactions had with private would not have executed such contract. To set aside a document solemnly
respondents. It is a general rule in this jurisdiction that findings of fact of said executed and voluntarily delivered, the proof of fraud must be clear and
appellate court are final and conclusive and, thus, binding on this Court in the convincing. 10 We are not persuaded that such quantum of proof exists in the
absence of sufficient and convincing proof, inter alia, that the former acted case at bar.
with grave abuse of discretion. Under the circumstances, we find no
compelling reason to deviate from this long-standing jurisprudential The fact that it was Bormaheco which paid the premium for the surety bond
pronouncement. issued by ICP does not per se affect the validity of the bond. Petitioners
themselves admit in their present petition that Rivera executed a Deed of
In addition, the alleged failure of Rivera to pay the consideration agreed upon Sale with Right of Repurchase of his car in favor of Bormaheco and agreed
in the Sales Agreement, which clearly constitutes a breach of the contract, that a part of the proceeds thereof shall be used to pay the premium for the
cannot be availed of by the guilty party to justify and support an action for the

315
bond. 11 In effect, Bormaheco accepted the payment of the premium as an private respondents in the foreclosure of the mortgaged properties, this fact
agent of ICP The execution of the deed of sale with a right of repurchase in alone is not, under the circumstances, sufficient to justify the piercing of the
favor of Bormaheco under such circumstances sufficiently establishes the corporate fiction, since petitioners do not intend to hold the officers and/or
fact that Rivera recognized Bormaheco as an agent of ICP Such payment to members of respondent corporations personally liable therefor. Petitioners
the agent of ICP is, therefore, binding on Rivera. He is now estopped from are merely seeking the declaration of the nullity of the foreclosure sale, which
questioning the validity of the suretyship contract. relief may be obtained without having to disregard the aforesaid corporate
fiction attaching to respondent corporations. Secondly, petitioners failed to
II. Under the doctrine of piercing the veil of corporate entity, when valid establish by clear and convincing evidence that private respondents were
grounds therefore exist, the legal fiction that a corporation is an entity with a purposely formed and operated, and thereafter transacted with petitioners,
juridical personality separate and distinct from its members or stockholders with the sole intention of defrauding the latter.
may be disregarded. In such cases, the corporation will be considered as a
mere association of persons. The members or stockholders of the The mere fact, therefore, that the businesses of two or more corporations are
corporation will be considered as the corporation, that is, liability will attach interrelated is not a justification for disregarding their separate
directly to the officers and stockholders. 12 The doctrine applies when the personalities, 16 absent sufficient showing that the corporate entity was
corporate fiction is used to defeat public convenience, justify wrong, protect purposely used as a shield to defraud creditors and third persons of their
fraud, or defend crime, 13 or when it is made as a shield to confuse the rights.
legitimate issues 14 or where a corporation is the mere alter ego or business
conduit of a person, or where the corporation is so organized and controlled III. The main issue for resolution is whether there was a valid foreclosure of
and its affairs are so conducted as to make it merely an instrumentality, the mortgaged properties by ICP Petitioners argue that the foreclosure
agency, conduit or adjunct of another corporation. 15 proceedings should be declared null and void for two reasons, viz.: (1) no
written notice was furnished by Bormaheco to ICP anent the failure of Slobec
In the case at bar, petitioners seek to pierce the V621 Of corporate entity of in paying its obligation with the former, plus the fact that no receipt was
Bormaheco, ICP and PM Parts, alleging that these corporations employed presented to show the amount allegedly paid by ICP to Bormaheco; and (b)
fraud in causing the foreclosure and subsequent sale of the real properties at the time of the foreclosure of the mortgage, the liability of ICP under the
belonging to petitioners While we do not discount the possibility of the surety bond had already expired.
existence of fraud in the foreclosure proceeding, neither are we inclined to
apply the doctrine invoked by petitioners in granting the relief sought. It is our Respondent court, in finding for the validity of the foreclosure sale, declared:
considered opinion that piercing the veil of corporate entity is not the proper
remedy in order that the foreclosure proceeding may be declared a nullity Now to the question of whether or not the foreclosure by the
under the circumstances obtaining in the legal case at bar. ICP of the real estate mortgage was in the exercise of a legal
right, We agree with the appellants that the foreclosure
In the first place, the legal corporate entity is disregarded only if it is sought to proceedings instituted by the ICP was in the exercise of a
hold the officers and stockholders directly liable for a corporate debt or legal right. First, ICP has in its favor the legal presumption
obligation. In the instant case, petitioners do not seek to impose a claim that it had indemnified Bormaheco by reason of Slobec's
against the individual members of the three corporations involved; on the default in the payment of its obligation under the Sales
contrary, it is these corporations which desire to enforce an alleged right Agreement, especially because Bormaheco consented to
against petitioners. Assuming that petitioners were indeed defrauded by

316
ICPs foreclosure of the mortgage. This presumption is in presented in writing to the Corporation within THIRTY (30)
consonance with pars. R and Q Section 5, Rule 5, * New DAYS from the expiration of this BOND, and that the obligee
Rules of Court which provides that it is disputably presumed hereby waives his right to bring claim or file any action
that private transactions have been fair and regular. likewise, against Surety and after the termination of one (1) year from
it is disputably presumed that the ordinary course of the time his cause of action accrues. 18
business has been followed: Second, ICP had the right to
proceed at once to the foreclosure of the mortgage as The surety bond was dated October 24, 1970. However, an
mandated by the provisions of Art. 2071 Civil Code for these annotation on the upper part thereof states: "NOTE: EFFECTIVITY
further reasons: Slobec, the principal debtor, was admittedly DATE OF THIS BOND SHALL BE ON JANUARY 22, 1971." 19
insolvent; Slobec's obligation becomes demandable by
reason of the expiration of the period of payment; and its On the other hand, the Sales Agreement dated January 23, 1971 provides
authorization to foreclose the mortgage upon Slobec's that the balance of P180,000.00 shall be payable in eighteen (18) monthly
default, which resulted in the accrual of ICPS liability to installments. 20 The Promissory Note executed by Slobec on even date in
Bormaheco. Third, the Agreement of Counter-Guaranty with favor of Bormaheco further provides that the obligation shall be payable on or
Real Estate Mortgage (Exh. 1) expressly grants to ICP the before February 23, 1971 up to July 23, 1972, and that non-payment of any
right to foreclose the real estate mortgage in the event of of the installments when due shall make the entire obligation immediately
'non-payment or non-liquidation of the entire indebtedness or due and demandable. 21
fraction thereof upon maturity as stipulated in the contract'.
This is a valid and binding stipulation in the absence of
It is basic that liability on a bond is contractual in nature and is ordinarily
showing that it is contrary to law, morals, good customs,
restricted to the obligation expressly assumed therein. We have repeatedly
public order or public policy. (Art. 1306, New Civil Code). 17
held that the extent of a surety's liability is determined only by the clause of
the contract of suretyship as well as the conditions stated in the bond. It
1. Petitioners asseverate that there was no notice of default issued by cannot be extended by implication beyond the terms the contract. 22
Bormaheco to ICP which would have entitled Bormaheco to demand
payment from ICP under the suretyship contract.
Fundamental likewise is the rule that, except where required by the
provisions of the contract, a demand or notice of default is not required to fix
Surety Bond No. B-1401 0 which was issued by ICP in favor of Bormaheco, the surety's liability. 23 Hence, where the contract of suretyship stipulates that
wherein ICP and Slobec undertook to guarantee the payment of the balance notice of the principal's default be given to the surety, generally the failure to
of P180,000.00 payable in eighteen (18) monthly installments on one unit of comply with the condition will prevent recovery from the surety. There are
Model CAT D-7 Caterpillar Crawler Tractor, pertinently provides in part as certain instances, however, when failure to comply with the condition will not
follows: extinguish the surety's liability, such as a failure to give notice of slight
defaults, which are waived by the obligee; or on mere suspicion of possible
1. The liability of INSURANCE CORPORATION OF THE default; or where, if a default exists, there is excuse or provision in the
PHILIPPINES, under this BOND will expire Twelve (I 2) suretyship contract exempting the surety for liability therefor, or where the
months from date hereof. Furthermore, it is hereby agreed surety already has knowledge or is chargeable with knowledge of the
and understood that the INSURANCE CORPORATION OF default. 24
THE PHILIPPINES will not be liable for any claim not

317
In the case at bar, the suretyship contract expressly provides that ICP shag was to expire on January 22, 1972, twelve (1 2) months from its effectivity
not be liable for any claim not filed in writing within thirty (30) days from the date, whereas Slobec's installment payment was to end on July 23, 1972.
expiration of the bond. In its decision dated May 25 1987, the court a Therefore, while ICP guaranteed the payment by Slobec of the balance of
quocategorically stated that '(n)o evidence was presented to show that P180,000.00, such guaranty was valid only for and within twelve (1 2)
Bormaheco demanded payment from ICP nor was there any action taken by months from the date of effectivity of the surety bond, or until January 22,
Bormaheco on the bond posted by ICP to guarantee the payment of plaintiffs 1972. Thereafter, from January 23, 1972 up to July 23, 1972, the liability of
obligation. There is nothing in the records of the proceedings to show that Slobec became an unsecured obligation. The default of Slobec during this
ICP indemnified Bormaheco for the failure of the plaintiffs to pay their period cannot be a valid basis for the exercise of the right to foreclose by ICP
obligation. " 25 The failure, therefore, of Bormaheco to notify ICP in writing since its surety contract had already been terminated. Besides, the liability of
about Slobec's supposed default released ICP from liability under its surety ICP was extinguished when Bormaheco failed to file a written claim against it
bond. Consequently, ICP could not validly foreclose that real estate mortgage within thirty (30) days from the expiration of the surety bond. Consequently,
executed by petitioners in its favor since it never incurred any liability under the foreclosure of the mortgage, after the expiration of the surety bond under
the surety bond. It cannot claim exemption from the required written notice which ICP as surety has not incurred any liability, should be declared null and
since its case does not fall under any of the exceptions hereinbefore void.
enumerated.
3. Lastly, it has been held that where The guarantor holds property of the
Furthermore, the allegation of ICP that it has paid Bormaheco is not principal as collateral surety for his personal indemnity, to which he may
supported by any documentary evidence. Section 1, Rule 131 of the Rules of resort only after payment by himself, until he has paid something as such
Court provides that the burden of evidence lies with the party who asserts an guarantor neither he nor the creditor can resort to such collaterals. 30
affirmative allegation. Since ICP failed to duly prove the fact of payment, the
disputable presumption that private transactions have been fair and regular, The Agreement of Counter-Guaranty with Chattel/Real Estate Mortgage
as erroneously relied upon by respondent Court of Appeals, finds no states that it is being issued for and in consideration of the obligations
application to the case at bar. assumed by the Mortgagee-Surety Company under the terms and conditions
of ICP Bond No. 14010 in behalf of Slobec Realty Development Corporation
2. The liability of a surety is measured by the terms of his contract, and, while and in favor of Bormaheco, Inc. 31 There is no doubt that said Agreement of
he is liable to the full extent thereof, such liability is strictly limited to that Counter-Guaranty is issued for the personal indemnity of ICP Considering
assumed by its terms. 26 While ordinarily the termination of a surety's liability that the fact of payment by ICP has never been established, it follows,
is governed by the provisions of the contract of suretyship, where the pursuant to the doctrine above adverted to, that ICP cannot foreclose on the
obligation of a surety is, under the terms of the bond, to terminate at a subject properties,
specified time, his obligation cannot be enlarged by an unauthorized
extension thereof. 27 This is an exception to the general rule that the IV. Private respondent PM Parts posits that it is a buyer in good faith and,
obligation of the surety continues for the same period as that of the principal therefore, it acquired a valid title over the subject properties. The submission
debtor. 28 is without merit and the conclusion is specious

It is possible that the period of suretyship may be shorter than that of the We have stated earlier that the doctrine of piercing the veil of corporate
principal obligation, as where the principal debtor is required to make fiction is not applicable in this case. However, its inapplicability has no
payment by installments. 29 In the case at bar, the surety bond issued by ICP

318
bearing on the good faith or bad faith of private respondent PM Parts. It must SO ORDERED.
be noted that Modesto N. Cervantes served as Vice-President of Bormaheco
and, later, as President of PM Parts. On this fact alone, it cannot be said that Melencio-Herrera (Chairman), Paras and Padilla, JJ., concur.
PM Parts had no knowledge of the aforesaid several transactions executed
between Bormaheco and petitioners. In addition, Atty. Martin de Guzman, Sarmiento, J., is on leave.
who is the Executive Vice-President of Bormaheco, was also the legal
counsel of ICP and PM Parts. These facts were admitted without qualification
in the stipulation of facts submitted by the parties before the trial court.
Hence, the defense of good faith may not be resorted to by private
respondent PM Parts which is charged with knowledge of the true relations
existing between Bormaheco, ICP and herein petitioners. Accordingly, the
transfer certificates of title issued in its name, as well as the certificate of
sale, must be declared null and void since they cannot be considered
altogether free of the taint of bad faith.

WHEREFORE, the decision of respondent Court of Appeals is hereby


REVERSED and SET ASIDE, and judgment is hereby rendered declaring the
following as null and void: (1) Certificate of Sale, dated September 28,1973,
executed by the Provincial Sheriff of Quezon in favor of the Insurance
Corporation of the Philippines; (2) Transfer Certificates of Title Nos. T-23705,
T-23706, T-23707 and T-23708 issued in the name of the Insurance
Corporation of the Philippines; (3) the sale by Insurance Corporation of the
Philippines in favor of Philippine Machinery Parts Manufacturing Co., Inc. of
the four (4) parcels of land covered by the aforesaid certificates of title; and
(4) Transfer Certificates of Title Nos. T-24846, T-24847, T-24848 and T24849
subsequently issued by virtue of said sale in the name of the latter
corporation.

The Register of Deeds of Lucena City is hereby directed to cancel Transfer


Certificates of Title Nos. T-24846, T-24847, T24848 and T-24849 in the name
of Philippine Machinery Parts Manufacturing Co., Inc. and to issue in lieu
thereof the corresponding transfer certificates of title in the name of herein
petitioners, except Santiago Rivera.

The foregoing dispositions are without prejudice to such other and proper
legal remedies as may be available to respondent Bormaheco, Inc. against
herein petitioners.

319
G.R. No. L-47673 October 10, 1946 licensed to do business in the Philippines, owned nine hundred and
ninety-five (995) shares out of the total capital stock of the plaintiff
KOPPEL (PHILIPPINES), INC., plaintiff-appellant, from the year 1928 up to and including the year 1936, and the
vs. remaining five (5) shares only were and are owned one each by
ALFREDO L. YATCO, Collector of Internal Revenue, defendant-appellee. officers of the plaintiff corporation.

Padilla, Carlos and Fernando for appellant. II. That plaintiff, at all times material to this case, was and now is duly
Office of the Solicitor General Ozaeta, First Assistant Solicitor General licensed to engage in business as a merchant and commercial
Reyes and. broker in the Philippines; and was and is the holder of the
Office of the Solicitor General Reyes and Solicitor Caizanes for appellee. corresponding merchant's and commercial broker's privilege tax
receipts.

III. That the defendant Collector of Internal revenue is now Mr.


HILADO, J.: Bibiano L. Meer in lieu of Mr. Alfredo L. Yatco.

This is an appeal by Koppel (Philippines), Inc., from the judgment of the IV. That during the period from January 1, 1929, up to and including
Court of First Instance of Manila in civil case No. 51218 of said court December 31, 1932, plaintiff transacted business in the Philippines in
dismissing said corporation's complaint for the recovery of the sum of the following manner, with the exception of the transactions which
P64,122.51 which it had paid under protest to the Collector of Internal are described in paragraphs V and VI of this stipulation:
Revenue on October 30, 1936, as merchant sales tax. The main facts of the
case were stipulated in the court below as follows: When a local buyer was interested in the purchase of railway
materials, machinery, and supplies, it asked for price quotations from
AGREED STATEMENT OF FACTS plaintiff. Atypical form of such request is attached hereto and made a
part hereof as Exhibit A. (Exhibit A represents typical transactions
arising from written requests for quotations, while Exhibits B to G,
Now come the plaintiff by attorney Eulogio P. Revilla and the
inclusive, are typical transactions arising from verbal requests for
defendant by the Solicitor General and undersigned Assistant
quotation.) Plaintiff then cabled for the quotation desired for Koppel
Attorney of the Bureau of Justice and, with leave of this Honorable
Industrial Car and Equipment Company. A sample of the pertinent
Court, hereby respectfully stipulated and agree to the following facts,
cable is hereto attached and made a part hereof as Exhibit B. Koppel
to wit:
Industrial Car and Equipment Company answered by cable quoting
its cost price, usually A. C. I. F. Manila cost price, which was later
I. That plaintiff is a corporation duly organized and existing under and followed by a letter of confirmation. A sample of the said cable
by virtue of the laws of the Philippines, with principal office therein at quotation and of the letter of confirmation are hereto attached and
the City of Manila, the capital stock of which is divided into thousand made a part hereof as Exhibits C and C-1. Plaintiff, however, quoted
(1,000) shares of P100 each. The Koppel Industrial Car and by Koppel Industrial Car and Equipment Company. Copy of the
Equipment company, a corporation organized and existing under the plaintiff's letter to purchaser is hereto attached and made a part
laws of the State of Pennsylvania, United States of America, and not

320
hereof as Exhibit D. On the basis of these quotations, orders were documents or copies thereof. A sample of said drafts is hereto
placed by the local purchasers, copies of which orders are hereto attached as Exhibit G. Plaintiff received by way of compensation a
attached as Exhibits E and E-1. percentage of the profits realized on the above transactions as fixed
in paragraph 6 of the plaintiff's contract with Koppel Industrial Car
A cable was then sent to Koppel Industrial Car and Equipment and Equipment Company, which contract is hereto attached as
company giving instructions to ship the merchandise to Manila Exhibit H, and suffered its corresponding share in the losses
forwarding the customer's order. Sample of said cable is hereto resulting from some of the transactions.
attached as Exhibit F. The bills of lading were usually made to "order"
and indorsed in blank with notation to the effect that the buyer be That the total gross sales from January 1, 1929, up to and including
notified of the shipment of the goods covered in the bills of lading; December 31, 1932, effected in the foregoing manner and under the
commercial invoices were issued by Koppel Industrial Car and above specified conditions, amount to P3, 596,438.84.
Equipment Company in the names of the purchasers and certificates
of insurance were likewise issued in their names, or in the name of V. That when a local sugar central was interested in the purchase of
Koppel Industrial Car and Equipment Company but indorsed in blank railway materials, machinery and supplies, it secured quotations
and attached to drafts drawn by Koppel Industrial Car and from, and placed the corresponding orders with, the plaintiff in
Equipment Company on the purchasers, which were forwarded substantially the same manner as outlined in paragraph IV of this
through foreign banks to local banks. Samples of the bills of lading stipulation, with the only difference that the purchase orders which
are hereto attached as Exhibits F-1, I-1, I-2 and I-3. Bills of ladings, were agreed to by the central and the plaintiff are similar to the
Exhibits I-1, I-2 and I-3, may equally have been employed, but said sample hereto attached and made a part hereof as Exhibit I. Typical
Exhibits I-1, I-2 and I-3 have no connection with the transaction samples of the bills of lading covering the herein transaction are
covered by Exhibits B to G, inclusive. The purchasers secured the hereto attached and made a part hereto as Exhibits I-1, I-2 and I-3.
shipping papers by arrangement with the banks, and thereupon The value of the sales carried out in the manner mentioned in this
received and cleared the shipments. If the merchandise were of paragraph is P133,964.98.
European origin, and if there was not sufficient time to forward the
documents necessary for clearance, through foreign banks to local VI. That sometime in February, 1929, Miguel J. Ossorio, of Manila,
banks, to the purchasers, the Koppel Industrial Car and Equipment Philippines, placed an option with Koppel Industrial Car and
company did, in many cases, send the documents directly from Equipment Company, through plaintiff, to purchase within three
Europe to plaintiff with instructions to turn these documents over to months a pair of Atlas-Diesel Marine Engines. Koppel Industrial Car
the purchasers. In many cases, where sales was effected on the and Equipment Company purchased said Diesel Engines in
basis of C. I. F. Manila, duty paid, plaintiff advanced the sums Stockholm, Sweden, for $16,508.32. The suppliers drew a draft for
required for the payment of the duty, and these sums, so advanced, the amount of $16,508.32 on the Koppel Industrial Car and
were in every case reimbursed to plaintiff by Koppel Industrial Car Equipment Company, which paid the amount covered by the draft.
and Equipment Company. The price were payable by drafts agreed Later, Miguel J. Ossorio definitely called the deal off, and as Koppel
upon in each case and drawn by Koppel Industrial Car and Industrial Car and Equipment Company could not ship to or draw on
Equipment Company on respective purchasers through local banks, said Mr. Miguel J. Ossorio, it in turn drew another draft on plaintiff for
and payments were made to the banks by the purchasers on the same amount at six months sight, with the understanding that
presentation and delivery to them of the above-mentioned shipping Koppel Industrial Car and Equipment Company would reimburse

321
plaintiff when said engines were disposed of. Plaintiff honored the refuses, notwithstanding demands by plaintiff, to return to the plaintiff
draft and debited the said sum of $16,508.32 to merchandise said sum of P64,122.51 or any part thereof.
account. The engines were left stored at Stockholm, Sweden. On
April 1, 1930, a new local buyer, Mr. Cesar Barrios, of Iloilo, xxx xxx xxx
Philippines, was found and the same engines were sold to him for
$21,000 (P42,000) C. I. F. Hongkong. The engines were shipped to That the parties hereby reserve the right to present
Hongkong and a draft for $21,000 was drawn by Koppel Industrial additional evidence in support of their respective
Car and Equipment Company on Mr. Cesar Barrios. After the draft contentions.
was fully paid by Mr. Barrios, Koppel Industrial Car and Equipment
Company reimbursed plaintiff with cost price of $16,508.32 and
Manila, Philippines, December 26, 1939
credited it with $1,152.95 as its share of the profit on the transaction.
Exhibits J and J-1 are herewith attached and made integral parts of
this stipulation with particular reference to paragraph VI hereof. (Sgd.) ROMAN OZAETA
Solicitor General
VII. That plaintiff's share in the profits realized out of these
transactions described in paragraphs IV, V and VI hereof totaling (Sgd.) ANTONIO CAIZARES
P3,772,403.82, amounts to P132,201.30; and that plaintiff within the Assistant Attorney
time provided by law returned the aforesaid amount P132,201.30 for
the purpose of the commercial broker's 4 per cent tax and paid (Sgd.) E. P. REVILLA
thereon the sum P5,288.05 as such tax. Attorney for the Plaintiff
3rd Floor, Perez Samanillo Bldg., Manila
VIII. That defendant demanded of the plaintiff the sum of P64,122.51
as the merchants' sales tax of 1% per cent on the amount of Both parties adduced some oral evidence in clarification of or
P3,772,403.82, representing the total gross value of the sales addition to their agreed statement of facts. A preponderance of
mentioned in paragraphs IV, V and VI hereof, including the 25 per evidence has established, besides the facts thus stipulated, the
cent surcharge for the late payment of the said tax, which tax and following:
surcharge were determined after the amount of P5,288.05
mentioned in paragraph VI hereof was deducted. (a) The shares of stock of plaintiff corporation were and are
all owned by Koppel Industries Car and Equipment
IX. That plaintiff, on October 30, 1936, paid under protest said sum of Company of Pennsylvania, U. S. A., exceptive which were
P64,122.51 in order to avoid further penalties, levy and distraint necessary to qualify the Board of Directors of said plaintiff
proceedings. corporation;

X. That defendant, on November 10, 1936, overruled plaintiff's (b) In the transactions involved herein the plaintiff
protest, and defendant has failed and refused and still fails and corporation acted as the representative of Koppel Industrial
Car and Equipment Company only, and not as the agent of
both the latter company and the respective local purchasers

322
plaintiff's principal witness, A.H. Bishop, its resident Vice- buyers and made delivery in the Philippines without the
President, in his testimony invariably referred to Koppel necessity of cabling its principal in America either for price
Industrial Car and Equipment Co. as "our principal" 9 t. s. n., quotations or confirmation or rejection of that agreed upon
pp. 10, 11, 12, 19, 75), except that at the bottom of page 10 between it and the buyer (t.s.n., pp. 39-43);
to the top of page 11, the witness stated that they had
"several principal" abroad but that "our principal abroad was, (h) Whenever the deliveries made by Koppel Industrial Car
for the years in question, Koppel Industrial Car and and Equipment Company were incomplete or insufficient to
Equipment Company," and on page 68, he testified that what fill the local buyer's orders, plaintiff used to make good the
he actually said was ". . . but our principal abroad" and not deficiencies by deliveries from its own local stock, but in
"our principal abroad" as to which it is very significant that such cases it charged its principal only the actual cost of the
neither this witness nor any other gave the name of even a merchandise thus delivered by it from its stock and in such
single other principal abroad of the plaintiff corporation; transactions plaintiff did not realize any profit (t.s.n., pp. 53-
54);
(c) The plaintiff corporation bore alone incidental expenses
as, for instance, cable expenses-not only those of its own (i) The contract of sale involved herein were all perfected in
cables but also those of its "principal" (t.s.n., pp. 52, 53); the Philippines.

(d) the plaintiff's "share in the profits" realized from the Those described in paragraph IV of the agreed statement of facts
transactions in which it intervened was left virtually in the went through the following process: (1) "When a local buyer was
hands of Koppel Industrial Car and Equipment Company interested in the purchase of railway materials, machinery, and
(t.s.n., p. 51); supplies, it asked for price quotations from plaintiff"; (2) "Plaintiff then
cabled for the quotation desired from Koppel Industrial Car and
(e) Where drafts were not paid by the purchasers, the local Equipment Company"; (3) "Plaintiff, however, quoted to the
banks were instructed not to protest them but to refer them purchaser a selling price above the figures quoted by Koppel
to plaintiff which was fully empowered by Koppel Industrial Industrial Car and Equipment Company"; (4) "On the basis of these
Car and Equipment company to instruct the banks with quotations, orders were placed by the local purchasers . . ."
regards to disposition of the drafts and documents (t.s.n., p.
50; Exhibit G);lawphil.net Those described in paragraph V of said agreed statement of facts
were transacted "in substantially the same manner as outlined in
(f) Where the goods were European origin, consular paragraph IV."
invoices, bill of lading, and, in general, the documents
necessary for clearance were sent directly to plaintiff (t.s.n., As to the single transaction described in paragraph VI of the same
p. 14); agreed statement of facts, discarding the Ossorio option which
anyway was called off, "On April 1, 1930, a new local buyer, Mr.
(g) If the plaintiff had in stock the merchandise desired by Cesar Barrios, of Iloilo, Philippines, was found and the same engines
local buyers, it immediately filled the orders of such local

323
were sold to him for $21,000(P42,000) C.I.F. Hongkong." (Emphasis 6. The court a quo erred in not holding that appellant acted as a
supplied.). commercial broker in the sole transaction covered under paragraph
VI of the agreed statement of facts;
(j) Exhibit H contains the following paragraph:
7. the court a quo erred in dismissing appellant's complaint.
It is clearly understood that the intent of this contract is that the
broker shall perform only the functions of a broker as set forth above, The lower court found and held that Koppel (Philippines), Inc. is a mere
and shall not take possession of any of the materials or equipment dummy or brach ("hechura") of Koppel industrial Car and Equipment
applying to said orders or perform any acts or duties outside the Company. The lower court did not deny legal personality to Koppel
scope of a broker; and in no sense shall this contract be construed (Philippines), Inc. for any and all purposes, but in effect its conclusion was
as granting to the broker the power to represent the principal as its that, in the transactions involved herein, the public interest and convenience
agent or to make commitments on its behalf. would be defeated and what would amount to a tax evasion perpetrated,
unless resort is had to the doctrine of "disregard of the corporate fiction."
The Court of First Instance held for the defendant and dismissed plaintiff's
complaint with costs to it. I. In its first assignment of error appellant submits that the trial court erred in
not holding that it is a domestic corporation distinct and separate from and
Upon this appeal, seven errors are assigned to said judgment as follows:. not a mere branch of Koppel Industrial Car and Equipment Company. It
contends that its corporate existence as Philippine corporation can not be
1. That the court a quo erred in not holding that appellant is a collaterally attacked and that the Government is estopped from so doing. As
domestic corporation distinct and separate from, and not a mere stated above, the lower court did not deny legal personality to appellant for
branch of Koppel Industrial Car and Equipment Co.; any and all purposes, but held in effect that in the transaction involved in this
case the public interest and convenience would be defeated and what would
amount to a tax evasion perpetrated, unless resort is had to the doctrine of
2. the court a quo erred in ignoring the ruling of the Secretary of
"disregard of the corporate fiction." In other words, in looking through the
Finance, dated January 31, 1931, Exhibit M;
corporate form to the ultimate person or corporation behind that form, in the
particular transactions which were involved in the case submitted to its
3. the court a quo erred in not holding that a character of a broker is determination and judgment, the court did so in order to prevent the
determined by the nature of the transaction and not by the basis or contravention of the local internal revenue laws, and the perpetration of what
measure of his compensation; would amount to a tax evasion, inasmuch as it considered and in our
opinion, correctly that appellant Koppel (Philippines), Inc. was a mere
4. The court a quo erred in not holding that appellant acted as a branch or agency or dummy ("hechura") of Koppel Industrial Car and
commercial broker in the transactions covered under paragraph VI of Equipment Co. The court did not hold that the corporate personality of
the agreed statement of facts; Koppel (Philippines), Inc., would also be disregarded in other cases or for
other purposes. It would have had no power to so hold. The courts' action in
5. The court a quo erred in not holding that appellant acted as a this regard must be confined to the transactions involved in the case at bar
commercial broker in the transactions covered under paragraph v of "for the purpose of adjudging the rights and liabilities of the parties in the
the agreed statement of facts;

324
case. They have no jurisdiction to do more." (1 Flethcer, Cyclopedia of Manifestly, the principle is the same whether the "person" be natural or
Corporation, Permanent ed., p. 124, section 41.) artificial.

A leading and much cited case puts it as follows: A very numerous and growing class of cases wherein the corporate
entity is disregarded is that (it is so organized and controlled, and its
If any general rule can be laid down, in the present state of authority, affairs are so conducted, as to make it merely an instrumentality,
it is that a corporation will be looked upon as a legal entity as a agency, conduit or adjunct of another corporation)." (1 Fletcher,
general rule, and until sufficient reason to the contrary appears; but, Cyclopedia of Corporation, Permanent ed., pp. 154, 155.)
when the notion of legal entity is used to defeat public convinience,
justify wrong, protect fraud, or defend crime, the law will regard the While we recognize the legal principle that a corporation does not
corporation as an association of persons. (1 Fletcher Cyclopedia of lose its entity by the ownership of the bulk or even the whole of its
Corporation [Permanent Edition], pp. 135, 136; United States vs. stock, by another corporation (Monongahela Co. vs. Pittsburg Co.,
Milwaukee Refrigeration Transit Co., 142 Fed., 247, 255, per 196 Pa., 25; 46 Atl., 99; 79 Am. St. Rep., 685) yet it is equally well
Sanborn, J.) settled and ignore corporate forms." (Colonial Trust Co. vs. Montello
Brick Works, 172 Fed., 310.)
In his second special defense appellee alleges "that the plaintiff was and is in
fact a branch or subsidiary of Koppel Industrial Car and Equipment Co., a Where it appears that two business enterprises are owned,
Pennsylvania corporation not licensed to do business in the Philippines but conducted and controlled by the same parties, both law and equity
actually doing business here through the plaintiff; that the said foreign will, when necessary to protect the rights of third persons, disregard
corporation holds 995 of the 1,000 shares of the plaintiff's capital stock, the the legal fiction that two corporations are distinct entities, and treat
remaining five shares being held by the officers of the plaintiff herein in order them as identical. (Abney vs. Belmont Country Club Properties, Inc.,
to permit the incorporation thereof and to enable its aforesaid officers to act 279 Pac., 829.)
as directors of the plaintiff corporation; and that plaintiff was organized as a
Philippine corporation for the purpose of evading the payment by its parent . . . the legal fiction of distinct corporate existence will be disregarded
foreign corporation of merchants' sales tax on the transactions involved in in a case where a corporation is so organized and controlled and its
this case and others of similar nature." affairs are so conducted, as to make it merely an instrumentality or
adjunct of another corporation. (Hanter vs. Baker Motor Vehicle Co.,
By most courts the entity is normally regarded but is disregarded to 190 Fed., 665.)
prevent injustice, or the distortion or hiding of the truth, or to let in a
just defense. (1 Fletcher, Cyclopedia of Corporation, Permanent In United States vs. Lehigh Valley R. Co. 9220 U.S., 257; 55 Law. ed., 458,
Edition, pp. 139,140; emphasis supplied.) 464), the Supreme Court of the United States disregarded the artificial
personality of the subsidiary coal company in order to avoid that the parent
Another rule is that, when the corporation is the mere alter ego, or corporation, the Lehigh Valley R. Co., should be able, through the fiction of
business conduit of a person, it may de disregarded." (1 Fletcher, that personality, to evade the prohibition of the Hepburn Act against the
Cyclopedia of Corporation, Permanent Edition, p. 136.) transportation by railroad companies of the articles and commodities
described therein.

325
Chief Justice White, speaking for the court, said: a bona fide separate corporation, we can not conceive how this arrangement
could have been adopted, for if there was any factor in its business as to
. . . Coming to discharge this duty it follows, in view of the express which it would in that case naturally have been opposed to being thus
prohibitions of the commodities clause, it must be held that while the controlled, it must have been precisely the amount of profit which it could
right of a railroad company as a stockholder to use its stock endeavor and hope to earn. No group of businessmen could be expected to
ownership for the purpose of a bona fide separate administration of organize a mercantile corporation the ultimate end of which could only be
the affairs of a corporation in which it has a stock interest may not be profit if the amount of that profit were to be subjected to such a unilateral
denied, the use of such stock ownership in substance for the control of another corporation, unless indeed the former has previously been
purpose of destroying the entity of a producing, etc., corporation, and designed by the incorporators to serve as a mere subsidiary, branch or
commingling its affairs in administration with the affairs of the railroad agency of the latter. Evidently, Koppel Industrial Car and Equipment
company, so as to make the two corporations virtually one, brings the Company made us of its ownership of the overwhelming majority 99.5%
railroad company so voluntarily acting as to such producing, etc., of the capital stock of the local corporation to control the operations of the
corporation within the prohibitions of the commodities clause. In latter to such an extent that it had the final say even as to how much should
other words, that by operation and effect of the commodities clause be allotted to said local entity in the so-called sharing in the profits. We can
there is duty cast upon a railroad company proposing to carry in not overlook the fact that in the practical working of corporate organizations
interstate commerce the product of a producing, etc., corporation in of the class to which these two entities belong, the holder or holders of the
which it has a stock interest, not to abuse such power so as virtually controlling part of the capital stock of the corporation, particularly where the
to do by indirection that which the commodities clause prohibits, a control is determined by the virtual ownership of the totality of the shares,
duty which plainly would be violated by the unnecessary dominate not only the selection of the Board of Directors but, more often than
commingling of the affairs of the producing company with its own, so not, also the action of that Board. Applying this to the instant case, we can
as to cause them to be one and inseparable. not conceive how the Philippine corporation could effectively go against the
policies, decisions, and desires of the American corporation with regards to
Corrobarative authorities can be cited in support of the same proposition, the scheme which was devised through the instrumentality of the contract
which we deem unnecessary to mention here. Exhibit H, as well as all the other details of the system which was adopted in
order to avoid paying the 1 per cent merchants sales tax. Neither can we
conceive how the Philippine corporation could avoid following the directions
From the facts hereinabove stated, as established by a preponderance of the
of the American corporation held 99.5 per cent of the capital stock of the
evidence , particularly those narrated in paragraph (a), (b), (c), (d), (e),(f), (h),
Philippine corporation. In the present instance, we note that Koppel
(i), and (j) after the agreed statement of facts, we find that, in so far as the
(Philippines), Inc., was represented in the Philippines by its "resident Vice-
sales involved herein are concerned, Koppel (Philippines), Inc., and Koppel
President." This fact necessarily leads to the inference that the corporation
Industrial Car and Equipment company are to all intents and purposes one
had at least a Vice-President, and presumably also a President, who were
and the same; or, to use another mode of expression, that, as regards those
not resident in the Philippines but in America, where the parent corporation is
transactions, the former corporation is a mere branch, subsidiary or agency
domiciled. If Koppel (Philippines), Inc., had been intended to operate as a
of the latter. To our mind, this is conclusively borne out by the fact, among
regular domestic corporation in the Philippines, where it was formed, the
others, that the amount of he so-called "share in the profits" of Koppel
record and the evidence do not disclose any reason why all its officers
(Philippines), Inc., was ultimately left to the sole, unbridled control of Koppel
should not reside and perform their functions in the Philippines.
Industrial Car and Equipment Company. If, in their relations with each other,
Koppel (Philippines), Inc., was considered and intended to function as

326
Other facts appearing from the evidence, and presently to be stated, branch, or agency of the American parent entity. Only upon this basis can it
strengthen our conclusion, because they can only be explained if the local be comprehended why it seems not to matter at all how much profit would be
entity is considered as a mere subsidiary, branch or agency of the parent allocated to plaintiff, or even that no profit at all be so allocated to it, at any
organization. Plaintiff charged the parent corporation no more than actual given time or after any given period.
cost without profit whatsoever for merchandise allegedly of its own to
complete deficiencies of shipments made by said parent corporation (t.s.n., As already stated above, under the evidence the sales in the Philippines of
pp. 53, 54) a fact which could not conceivably have been the case if the railway materials, machinery and supplies imported here by Koppel
plaintiff had acted in such transactions as an entirely independent entity Industrial Car and Equipment Company could have been as conviniently and
doing business for profit, of course with the American concern. There efficiently transacted and handled if not more so had said corporation
has been no attempt even to explain, if the latter situation really obtained, merely established a branch or agency in the Philippines and obtained
why these two corporations should have thus departed from the ordinary license to do business locally; and if it had done so and said sales had been
course of business. Plaintiff was charged by the American corporation with effected by such branch or agency, there seems to be no dispute that the 1
the cost even of the latter's cable quotations from ought that appears from per cent merchants' sales tax then in force would have been collectible. So
the evidence, this can only be comprehended by considering plaintiff as such far as we can discover, there would be only one, but very important,
a subsidiary, branch or agency of the parent entity, in which case it would be difference between the two schemes a difference in tax liability on the
perfectly understandable that for convenient accounting purposes and the ground that the sales were made through another and distinct corporation, as
easy determination of the profits or losses of the parent corporation's alleged broker, when we have seen that this latter corporation is virtually
Philippines should be charged against the Philippine office and set off against owned by the former, or that they practically one and the same, is to sanction
its receipts, thus separating the accounts of said branch from those which the a circumvention of our tax laws, and permit a tax evasion of no mean
central organization might have in other countries. The reference to plaintiff proportions and the consequent commission of a grave injustice to the
by local banks, under a standing instruction of the parent corporation, of Government. Not only this; it would allow the taxpayer to do by
unpaid drafts drawn on Philippine customers by said parent corporation, indirection what the tax laws prohibited to be done directly (non-payment of
whenever said customers dishonored the drafts, and the fact that the legitimate taxes), paraphrasing the United States Supreme Court in United
American corporation had previously advised said banks that plaintiff in those States vs. Lehigh Valley R. Co., supra.
cases was "fully empowered to instruct (the banks) with regard to the
disposition of the drafts and documents" (t.s.n., p. 50), in the absence of any The act of one corporation crediting or debiting the other for certain items,
other satisfactory explanation naturally give rise to the inference that plaintiff expenses or even merchandise sold or disposed of, is perfectly compatible
was a subsidiary, branch or agency of the American concern, rather than an with the idea of the domestic entity being or acting as a mere branch, agency
independent corporation acting as a broker. For, without such positive or subsidiary of the parent organization. Such operations were called for any
explanation, this delegation of power is indicative of the relations between way by the exigencies or convenience of the entire business. Indeed,
central and branch offices of the same business enterprise, with the latter accounting operation such as these are invitable, and have to be effected in
acting under instructions already given by the former. Far from disclosing a the ordinary course of business enterprise extends its trade to another land
real separation between the two entities, particularly in regard to the through a branch office, or through another scheme amounting to the same
transactions in question, the evidence reveals such commongling and thing.
interlacing of their activities as to render even incomprehensible certain
accounting operations between them, except upon the basis that the
If plaintiff were to act as broker in the Philippines for any other corporation,
Philippine corporation was to all intents and purposes a mere subsidiary,
entity or person, distinct from Koppel Industrial Car and Equipment company,

327
an entirely different question will arise, which, however, we are not called the Philippines. In those cases where no such price quotations from the
upon, nor in a position, to decide. American corporation were needed, of course, the sales effected in those
cases described in paragraph V of the agreed statement of facts were, as
As stated above, Exhibit H contains to the following paragraph: expressed therein, transacted "in substantially the same manner as outlined
in paragraph VI." Even the single transaction described in paragraph VI of
It is clearly understood that the intent of this contract is that the the agreed statement of facts was also perfected in the Philippines, because
broker shall perform only the functions of a broker as set forth above, the contracting parties were here and the consent of each was given here.
and shall not take possession of any of the materials or equipment While it is true that when the contract was thus perfected in the Philippines
applying to said orders or perform any acts or duties outside the the pair of Atlas-Diesel Marine Engines were in Sweden and the agreement
scope of a broker; and in no sense shall this contract be construed was to deliver them C.I.F. Hongkong, the contract of sale being consensual
as granting to the broker the power to represent the principal as its perfected by mere consent (Civil Code, article 1445; 10 Manresa, 4th
agent or to make commitments on its behalf. ed., p. 11), the location of the property and the place of delivery did not
matter in the question of where the agreement was perfected.
The foregoing paragraph, construed in the light of other facts noted
elsewhere in this decision, betrays, we think a deliberate intent, through the In said paragraph VI, we read the following, as indicating where the contract
medium of a scheme devised with great care, to avoid the payment of was perfected, considering beforehand that one party, Koppel
precisely the 1 per cent merchants' sales tax in force in the Philippines (Philippines),Inc., which in contemplation of law, as to that transaction, was
before, at the time of, and after, the making of the said contract Exhibit H. If the same Koppel Industrial Car Equipment Co., was in the Philippines:
this were to be allowed, the payment of a tax, which directly could not have
been avoided, could be evaded by indirection, consideration being had of the . . . on April 1, 1930, a new local buyer Mr. Cesar Barrios, of Iloilo,
aforementioned peculiar relations between the said American and local Philippines, was found and the same engines were sold to him for
corporations. Such evasion, involving as it would, a violation of the former $21,000 (P42,000) C.I.F. Hongkong . . . (Emphasis supplied.)
Internal Revenue Law, would even fall within the penal sanction of section
2741 of the Revised Administrative Code. Which only goes to show the Under the revenue law in force when the sales in question took place, the
illegality of the whole scheme. We are not here concerned with the merchants' sales tax attached upon the happening of the respective sales of
impossibility of collecting the merchants' sales tax, as a mere incidental the "commodities, goods, wares, and merchandise" involved, and we are
consequence of transactions legal in themselves and innocent in their clearly of opinion that such "sales" took place upon the perfection of the
purpose. We are dealing with a scheme the primary, not to say the sole, corresponding contracts. If such perfection took place in the Philippines, the
object of which the evasion of the payment of such tax. It is this aim of the merchants' sales tax then in force here attached to the transactions.
scheme that makes it illegal.
Even if we should consider that the Philippine buyers in the cases covered by
We have said above that the contracts of sale involved herein were all paragraph IV and V of the agreed statement of facts, contracted with Koppel
perfected in the Philippines. From the facts stipulated in paragraph IV of the Industrial Car and Equipment company, we will arrive at the same final result.
agreed statement of facts, it clearly appears that the Philippine purchasers It can not be denied in that case that said American corporation contracted
had to wait for Koppel Industrial Car and Equipment Company to through Koppel (Philippines), Inc., which was in the Philippines. The real
communicate its cost prices to Koppel (Philippines), Inc., were perfected in transaction in each case of sale, in final effect, began with an offer of sale

328
from the seller, said American corporation, through its agent, the local But this court, in Philippine Trust Company and Smith, Bell and Co. vs.
corporation, of the railway materials, machinery, and supplies at the prices Mitchell(59 Phil., 30, 36), said:
quoted, and perfected or completed by the acceptance of that offer by the
local buyers when the latter, accepting those prices, placed their orders. The . . . The rule of stare decisis is entitled to respect. Stability in the law,
offer could not correctly be said to have been made by the local buyers when particularly in the business field, is desirable. But idolatrous
they asked for price quotations, for they could not rationally be taken to have reverence for precedent, simply as precedent, no longer rules. More
bound themselves to buy before knowing the prices. And even if we should important than anything else is that court should be right. . . .
take into consideration the fact that the american corporation contracted, at
least partly, through correspondence, according to article 54 of the Code of III. In the view we take of the case, and after the disposition made above of
Commerce, the respective contracts were completed from the time of the the first assignment of error, it becomes unnecessary to make any specific
acceptance by the local buyers, which happened in the Philippines. ruling on the third, fourth, fifth, sixth, and seventh assignments of error, all of
which are necessarily disposed of adversely to appellant's contention.
Contracts executed through correspondence shall be completed
from the time an answer is made accepting the proposition or the Wherefore, he judgment appealed from is affirmed, with costs of both
conditions by which the latter may be modified." (Code of instances against appellant. So ordered.
Commerce, article 54; emphasis supplied.)
Moran, C.J., Paras, Feria, Pablo, Bengzon, Briones, and Tuason, JJ.,
A contract is as a rule considered as entered into at the place where concur.
the place it is performed. So where delivery is regarded as made at
the place of delivery." (13 C. J., 580-81, section 581.)

(In the consensual contract of sale delivery is not needed for its
perfection.)

II. Appellant's second assignment of error can be summarily disposed of. It is


clear that the ruling of the Secretary of Finance, Exhibit M, was not binding
upon the trial court, much less upon this tribunal, since the duty and power of
interpreting the laws is primarily a function of the judiciary. (Ortua vs. Singson
Encarnacion, 59 Phil., 440, 444.) Plaintiff cannot be excused from abiding by
this legal principle, nor can it properly be heard to say that it relied on the
Secretary's ruling and that, therefore, the courts should not now apply an
interpretation at variance therewith. The rule of stare decisis is undoubtedly
entitled to more respect in the construction of statutes than the
interpretations given by officers of the administrative branches of the
government, even those entrusted with the administration of particular laws.

329
G.R. No. L-13119 September 22, 1959

RICARDO TANTONGCO, Petitioner, vs. KAISAHAN NG MGA


MANGGAGAWA SA LA CAMPAN (KKM) AND THE HONORABLE COURT
OF INDUSTRIAL RELATIONS,Respondents.

Ernesto C. Estrella for petition for petitioner.


Carlos E. Santiago for respondent Union.
Pedro M. Ligaya for respondent CIR.

MONTEMAYOR, J.: chanrobles virtual law library

This is a petition for certiorari and prohibition with prayer for issuance
of a writ of preliminary injunction to prohibit respondent Court of Industrial
Relations from proceeding with the hearing of the contempt proceedings for
which petitioner Ricardo Tantongco was cited to appear the present his
evidence. The contempt proceedings which petitioner seeks to stop are
based on the order of the Court of Industrial Relations, dated September 30,
1957, which reads as follows:

It appearing that the Order of this Court, in the above-entitled case,


dated February 18, 1957 (folios 134-166), has become final and executory
and the respondents have failed to comply with the same, the said

330
respondents, namely, the La Campana Starch and Coffee Factory or its to entertain and decide the case. The motion was denied by the Court of
manager or the person who has charge of the management, and the Industrial Relations, which said:
administrator of the Estate of Ramon Tantongco are hereby ordered to
comply with said order, within five days from receipt hereof, particularly the . . . There was only management for the business of gawgaw and
following, to wit: coffee with whom the laborers are dealing regarding their work. Hence, the
filing of action against the La Campana Starch and Coffee Factory is proper
(a) To reinstate the persons named in the said Order of February 18, and justified.
1957;chanrobles virtual law library
The order of denial was appealed to this Tribunal
(b) To deposit the amount of P65,534.01 with this Court. through certiorari under G.R. No. L-5677. In disposing of the case, we held:

With respect to possible back wages from August 28, 1957 as As to the first ground, petitioners obviously do not question the fact that
mentioned in the petition for contempt of August 30, 1957, the same shall the number of employees of the La Campana Gaugau Packing involved in
first be determined.chanroblesvirtualawlibrary chanrobles virtual law library the case is more than the jurisdictional number (31) required by law, but they
contend that the industrial court has no jurisdiction to try case against La
Failure to comply with this Order shall be directly dealt with Campana Coffee Factory Co. Inc. because the latter has allegedly only 14
accordingly. laborers and only five of these are members of respondent Kaisahan. This
contention loses force when it is noted that, as found by the industrial court -
It would appear that petitioner Ricardo Tantongco failed to comply with and this finding is conclusive upon us - La Campana Gaugau Packing and La
said order and so, as already stated, he was cited to appear and to adduce Campana Coffee Factory Co. Inc., are operating under one single
evidence on his behalf to show why he should not be punished for indirect management, that is, one business though with two trade names. True, the
contempt.chanroblesvirtualawlibrary chanrobles virtual law library coffee factory is a corporation , and, by legal fiction, an entity existing
separate and part from the persons composing it, that is, Tan Tong and his
family. But is settled this fiction of law, which has been introduced as a matter
The facts in this case may be briefly narrated thus: Sometime in June,
of convenience and to subserve the ends of justice cannot be invoke to
1951, members of the Kaisahan ng mga Manggagawa sa La Campana, a
further an end subversive of that
labor union to which were affiliated workers in the La Campana Starch
purpose.chanroblesvirtualawlibrary chanrobles virtual law library
Factory and La Campana Coffee Factory, two separate entities but under the
one management, presented demands for higher wages, and more privileges
and benefits in connection with their work. When the management failed and ... The attempt to make the two factories appear as two separate
refused to grant the demands, the Department of Labor intervened; but business, when in reality, they are but one is but a device to defeat the ends
failing to settle the controversy, it certified the dispute to the Court of of the law (the Act governing capital and labor relations) and should not be
Industrial Relations on July 17, 1951, where it was docketed as Case No. permitted to prevail. (La Campana Coffee Factory, et al., vs. Kaisahan ng
584-V. On the theory that the laborers presenting the demands were only the mga Manggagawa, etc. et al., 93 Phil., 160; 49 Off. Gaz., [6] 2300.)
ones working in the coffee factory, said company filed through the
management a motion to dismiss claiming that inasmuch as there were only Upon the return of the case to the Court of Industrial Relations, the
14 of them in said factory, the Court of Industrial Relations had no jurisdiction latter proceeded with the hearing. In the meantime incidental cases involving

331
the same parties came up and were filed before the Court of Industrial Rule 3, Section 21, and Rule 88, Section 1 of the Rules of Court. On August
Relations in the following cases: 23, 1956, the Court of Industrial Relations denied the motion to dismiss and
proceed to hear the incidental cases against the La Campana
Case No. 584-V(1) - petition for contempt against the La Campana entities.chanroblesvirtualawlibrary chanrobles virtual law library
Starch and Coffee Factory for having employed 21 new laborers in violation
of the order of July 21, 1951, filed on July 25, 1951;chanrobles virtual law On June 12, 1956, a partial decision was rendered in the main case
library No. 584-V, which partial decision was elevated to us and is still pending
appeal. On February 18, 1957, the Court of Industrial Relations issued an
Case No. 584-V(2) - petition of La Campana for authority for authority order in incidental Cases No. 584-V(1), V(2), V(5) and V(6), directing the
to dismiss Loreto Bernabe, filed on July 25, 19651;chanrobles virtual law "management of the respondent company and or the administrator of the
library Estate of Ramon Tantongco", to reinstate the dismissed laborers mentioned
therein with back wages. This order of February 18, 1957, as well as the
Case No. 584-V(3) - petition of Union to reinstate Bonifacio Calderon order directing the inclusion of the administrator of the estate of Ramon
with backpay, filed on August 3, 1951;chanrobles virtual law library Tantongco as additional respondent in the incidental cases, and the order
denying the petition of the administrator to dismiss said incidental cases were
appealed to this tribunal though certiorari. The appeal, however, was
Case No. 584-V(5) - petition of Union to reinstate Marcelo Estrada and
summarily dismissed by this Court in its resolution of June 12, 1957, as
Exequiel Rapiz with back pay and to punish officials of the company for
follows:
contempt, filed on February 13, 1952; and chanrobles virtual law library

This Court, deliberating upon the allegations of the petition filed in case
Case No. 584-V(6) - petition of union for reinstatement of Ibardolaza
l-12355 (La Campana Starch Coffee Factory et al. vs. Kaisahan ng Mga
and seven other member-laborers and to punish the officers of the company
Manggagawa sa la Campana, KKM, et al) for review, on certiorari of the
for contempt, filed on July 15, 1953.
decision of the Court of Industrial Relations referred to therein, and finding
that there is no merit in the petition, RESOLVE TO DISMISS the same.
These five cases were heard jointly. In the meantime Ramon
Tantongco supposed to be the owner and manager of the La Campana
The CIR order of February 18, ,1957, in the incidental cases Nos. 584-
Starch Factory and the person in charge of the La Campana Coffee Factory
V to V(6), having become final and executory , the laborers involved reported
died on May 16, 1956. On motion of the labor union, the Court of Industrial
for work on August 28, 1957, but they were not admitted by the management.
Relations order the inclusion as party respondent of the administrator of the
Consequently, the union filed a petition dated August 30, 1957, to hold
estate of Ramon Tantongco who was Ricardo
respondents in said cases for contempt. After hearing the CIR issued the
Tantongco.chanroblesvirtualawlibrary chanrobles virtual law library
order of September 30, 1957, subject of this petition, ordering "the La
Campana Starch and Coffee Factory or its manager or the person who has
Ricardo Tantongco, as administrator, under a special appearance filed charge of its management and the administrator of the estate of Ramon
a motion to dismiss all the cases including the main case, that is to say, Tantongco" to "reinstate the persons named in the order of February 18,
Cases No. 584-(V) to 584-V(6), on the ground that said cases involved 1957" and "to deposit the amount of P65,534.01." For refusal or failure to
claims for sums of money and consequently should be filed before the comply with said order, petitioner Ricardo Tantongco was required to appear
probate court having jurisdiction over the estate, pursuant to the provisions of before the attorney of the CIR in contempt proceedings. Petitioner now seeks

332
to prohibit the CIR from proceeding with the trial for contempt and to enjoin defeat the jurisdiction of the CIR. We there found that although there were
respondent CIR from enforcing its order of September 30, ostensibly two separate companies or entities, they were managed by the
1957.chanroblesvirtualawlibrary chanrobles virtual law library same person or persons and the workers in both were used interchangeably
so that in order to determine whether or not the CIR had jurisdiction, the
Petitioner contends that upon the death of Ramon Tantongco, the number of workers in both entitles, not in only one, was to be considered.
claims of the laborers should have been dismissed and that said claims However, we still believe that although the family of Ramon Tantongco was
should have been filed with the probate court having jurisdiction over the practically the owner of both the coffee factory and the starch factory,
administration proceedings of the estate of Ramon Tantongco, pursuant to nevertheless these entities are separate from the personality of Ramon. The
the provisions of Rule 3, Section 21 of the Rules of Court and that the failure coffee factory is a stock corporation and the shares are owned not only by
to file claims with the administrator forever barred said claims as provided in Ramon but also by others, such as petitioner Ricardo who not only is a
Rule 87, Section 5 of the Rules of Court, especially after the assets of the stockholder and director and treasurer but also the management of the same
estate had been distributed among the heirs, and petitioner had ceased to be Furthermore, petitioner is now estopped from claiming that the two entities in
the administrator of the estate. As already stated this same question was question and Ramon are one. Thus in Annex 3-CIR (par. 1 thereof) which is a
raised by petitioner in G.R. No. L-12355, entitled "La Campana Starch and complaint for injunction filed by La Campana Food Products, et al and La
Coffee Factory and Ricardo Tantongco, etc. vs. Kaisahan ng mga Campana Starch Packing against the consolidated Labor Organization of the
Manggagawa sa La Campana (KKM)," which, as already stated, was Philippines, in civil Case No. P-25482 in the Court of First Instance of Rizal,
summarily dismissed by this Court in a resolution dated June 12, 1957. petitioner admitted the existence and operation of said entities; in Annex 4-
Consequently, said question may not again be raised in the present case. CIR where petitioner appeared as General Manager representing the two
Furthermore, it may be recalled that both in the main case in the incidental entities in its agreement with the La Campana Workers Union to resolve the
cases No. 584-V to 584-V(6), Ramon Tantongco was never a party. The party dispute between the two entities and the laborers in case Nos. 1072-V and
there was the La Campana Starch and Coffee Factory by which name it was 1371-ULP, the existence of the two entities appears to have been admitted;
sought to designate the two entities La Campana Starch Packing and the La and in Annex 5-A-CIR, an answer to the complaint of La Campana Workers
Campana Coffee Factory. Naturally, the claims contained in said cases were Union in case No. 1471-ULP (Annex 5-CIR), petitioner admitted the
not the claims contemplated by law to be submitted before the administrator. allegation that said two factories were in existence and doing business with
In other words the death of Ramon Tantongco did not deprive the CIR of its petitioner as manager of the same.chanroblesvirtualawlibrary chanrobles
jurisdiction over the cases aforementioned. Moreover, the money claims of virtual law library
the laborers were merely incidental to their demands for reinstatement for
having been unjustly dismissed, and for better working In relation to the order of the CIR requiring petitioner to appear in the
conditions.chanroblesvirtualawlibrary chanrobles virtual law library contempt proceedings instituted against him, petitioner contends that after he
ceased to be the administrator of the estate of Ramon Tantongco, he may
Petitioner, however, contends that in G.R. No. L-5677, we "pierced the not now be compelled to comply with the order of the court. In answer, it is
veil of corporate existence", and held that the La Campana Starch and enough to bear in mind the jurisdiction and authority of the CIR as to
Coffee Factory and its owner, Ramon Tantongco, were one; so that with the compliance with and violations of its orders under section 6, Commonwealth
death of Ramon, the La Campana entities ceased to exist, resulting in the Act No. 143, which we quote below:
loss of jurisdiction of the CIR to enforce its order against said entities. The
reason we applied the so-called "piercing the veil of corporate existence" in . . . The Court or any Judge thereof shall have furthermore, all the
G.R. No. L-5677 was to avoid the technicality therein advanced in order to inherent powers of a court of justice provided in paragraph 5 of Rule 124 of

333
the Supreme Court, as well as the power to punish direct and indirect
contempt as provided in Rule 64 of the same Court, under the same
procedure and penalties provided
therein.chanroblesvirtualawlibrary chanrobles virtual law library

Any violation of any order, award, or decision of the Court of Industrial


Relations shall, after such order, award or decision has become final,
conclusive, and executory, constitute contempt of court: . .
.chanroblesvirtualawlibrary chanrobles virtual law library
G.R. No. 110358 November 9, 1994

In case the employer (or landlord) committing any such violation or


QUINTIN ROBLEDO, MARIO SINLAO, LEONARDO SAAVEDRA,
contempt is an association or corporation, the manager or the person who
VICENTE SECAPURI, DANIEL AUSTRIA, ET AL., petitioners,
has the charge of the management of the business of the association or
vs.
corporation and the officers of directors thereof who have ordered or
THE NATIONAL LABOR RELATIONS COMMISSION, BACANI SECURITY
authorized the violation of contempt shall be liable. . . .
AND ALLIED SERVICES CO., INC., AND BACANI SECURITY AND
PROTECTIVE AGENCY AND/OR ALICIA BACANI, respondents.
In conclusion, we find and hold that the La Campana Starch and Food
Products Company which stands for the La Campana Starch and Coffee
Benjamin C. Pineda for petitioners.
Factory are entities distinct from the personality of Ramon Tantongco; that
after the death of Ramon these two entities continued to exist and to operate
under the management of petitioner and that consequently he is the proper Villanueva, Ebora & Caa for private respondents.
person and official to which the orders of the CIR are addressed and who is
in duty bound to comply with the same. We further find that the CIR acted
with in its jurisdiction in issuing its order of September 30, 1957 and in
requiring petitioner to appear to give his evidence if any in relation with the MENDOZA, J.:
contempt proceedings instituted against
him.chanroblesvirtualawlibrary chanrobles virtual law library This is a petition for review of the decision of the First Division 1 of the
National Labor Relations Commission, setting aside the decision of the Labor
In view of the foregoing, the petition for certiorari is .hereby denied and Arbiter which held private respondents jointly and severally liable to the
the writ of preliminary injunction dissolved, with costs. petitioners for overtime and legal holiday pay.

Paras, C.J., Bengzon, Padilla, Bautista Angelo, Labrador, Concepcion, The facts of this case are as follows:
Endencia and Barrera, JJ., concur.
Petitioners were former employees of Bacani Security and Protective Agency
(BSPA, for brevity). They were employed as security guards at different times
during the period 1969 to December 1989 when BSPA ceased to operate.

334
BSPA was a single proprietorship owned, managed and operated by the late On March 1, 1992, the Labor Arbiter rendered a decision upholding the right
Felipe Bacani. It was registered with the Bureau of Trade and Industry as a of the petitioners. The dispositive portion of his decision reads:
business name in 1957. Upon its expiration, the registration was renewed on
July 1, 1987 for a term of five (5) years ending 1992. CONFORMABLY WITH THE FOREGOING, the judgment is
hereby rendered finding complainants entitled to their money
On December 31, 1989, Felipe Bacani retired the business name and BSPA claims as herein above computed and to be paid by all the
ceased to operate effective on that day. At that time, respondent Alicia respondents herein in solidum except BSPA which has
Bacani, daughter of Felipe Bacani, was BSPA's Executive Directress. already been retired from business.

On January 15, 1990 Felipe Bacani died. An intestate proceeding was Respondents are further ordered to pay attorney's fees
instituted for the settlement of his estate before the Regional Trial Court, equivalent to five (5) percent of the awarded money claims.
National Capital Region, Branch 155, Pasig, Metro Manila.
All other claims are hereby dismissed for lack of merit.
Earlier, on October 26, 1989, respondent Bacani Security and Allied Services
Co., Inc. (BASEC, for brevity) had been organized and registered as a SO ORDERED.
corporation with the Securities and Exchange Commission. The following
were the incorporators with their respective shareholdings: On appeal the National Labor Relations Commission reversed. In a decision
dated March 30, 1993, the NLRC's First Division declared the Labor Arbiter
ALICIA BACANI 25,250 shares without jurisdiction and instead suggested that petitioners file their claims
LYDIA BACANI 25,250 shares with the Regional Trial Court, Branch 155, Pasig, Metro Manila, where an
AMADO P. ELEDA 25,250 shares intestate proceeding for the settlement of Bacani's estate was pending.
VICTORIA B. AURIGUE 25,250 shares Petitioners moved for a reconsideration but their motion was denied for lack
FELIPE BACANI 20,000 shares of merit. Hence this petition for review.

The primary purpose of the corporation was to "engage in the business of No appeal lies to review decisions of the NLRC. Nonetheless the petition in
providing security" to persons and entities. This was the same line of this case was treated as a special civil action of certiorari to determine
business that BSPA was engaged in. Most of the petitioners, after losing their whether the NLRC did not commit a grave abuse of its discretion in reversing
jobs in BSPA, were employed in BASEC. the Labor Arbiter's decision.

On July 5, 1990, some of the petitioners filed a complaint with The issues in this case are two fold: first, whether Bacani Security and Allied
the Department of Labor and Employment, National Capital Region, for Services Co. Inc. (BASEC) and Alicia Bacani can be held liable for claims of
underpayment of wages and nonpayment of overtime pay, legal holiday pay, petitioners against Bacani Security and Protective Agency (BSPA)
separation pay and/or retirement/resignation benefits, and for the return of and, second, if the claims were the personal liability of the late Felipe Bacani,
their cash bond which they posted with BSPA. Made respondents were BSPA as owner of BSPA, whether the Labor Arbiter had jurisdiction to decide the
and BASEC. Petitioners were subsequently joined by the rest of the claims.
petitioners herein who filed supplementary complaints.

335
Petitioners contend that public respondent erred in setting aside the Labor or that a corporation is the mere alter ego or business conduit of a person or
Arbiter's judgment on the ground that BASEC is the same entity as BSPA the where the corporation is so organized and controlled and its affairs are so
latter being owned and controlled by one and the same family, namely the conducted as to make it merely an instrumentality, agency, conduit or adjunct
Bacani family. For this reason they urge that the corporate fiction should be of another corporation. 5 It is apparent, therefore, that the doctrine has no
disregarded and BASEC should be held liable for the obligations of the application to this case where the purpose is not to hold the individual
defunct BSPA. stockholders liable for the obligations of the corporation but, on the contrary,
to hold the corporation liable for the obligations of a stockholder or
We find the petition to be without merit. stockholders. Piercing the veil of corporate entity means looking through the
corporate form to the individual stockholders composing it. Here there is no
As correctly found by the NLRC, BASEC is an entity separate and distinct reason to pierce the veil of corporate entity because there is no question that
from that of BSPA. BSPA is a single proprietorship owned and operated by petitioners' claims, assuming them to be valid, are the personal liability of the
Felipe Bacani. Hence its debts and obligations were the personal obligations late Felipe Bacani. It is immaterial that he was also a stockholder of BASEC.
of its owner. Petitioners' claim which are based on these debts and personal
obligations, did not survive the death of Felipe Bacani on January 15, 1990 Indeed, the doctrine is stood on its head when what is sought is to make a
and should have been filed instead in the intestate proceedings involving his corporation liable for the obligations of a stockholder. But there are several
estate. reasons why BASEC is not liable for the personal obligations of Felipe
Bacani. For one, BASEC came into existence before BSPA was retired as a
Indeed, the rule is settled that unless expressly assumed labor contracts are business concern. BASEC was incorporated on October 26, 1989 and its
not enforceable against the transferee of an enterprise. The reason for this is license to operate was released on May 28, 1990, while BSPA ceased to
that labor contracts are in personam. 2 Consequently, it has been held that operate on December 31, 1989. Before, BSPA was retired, BASEC was
claims for backwages earned from the former employer cannot be filed already existing. It is, therefore, not true that BASEC is a mere continuity of
against the new owners of an enterprise. 3Nor is the new operator of a BSPA.
business liable for claims for retirement pay of employees. 4
Second, Felipe Bacani was only one of the five (5) incorporators of BASEC.
Petitioners claim, however, that BSPA was intentionally retired in order to He owned the least number of shares in BASEC, which included among its
allow expansion of its business and even perhaps an increase in its incorporators persons who are not members of his family. That his wife Lydia
capitalization for credit purpose. According to them, the Bacani family merely and daughter Alicia were also incorporators of the same company is not
continued the operation of BSPA by creating BASEC in order to avoid the sufficient to warrant the conclusion that they hold their shares in his behalf.
obligations of the former. Petitioners anchor their claim on the fact that Felipe
Bacani, after having ceased to operate BSPA, became an incorporator of Third, there is no evidence to show that the assets of BSPA were transferred
BASEC together with his wife and daughter. Petitioners urge piercing the veil to BASEC. If BASEC was a mere continuation of BSPA, all or at least a
of corporate entity in order to hold BASEC liable for BSPA's obligations. substantial part of the latter's assets should have found their way to BASEC.

The doctrine of piercing the veil of corporate entity is used whenever a court Neither can respondent Alicia Bacani be held liable for BSPA's obligations.
finds that the corporate fiction is being used to defeat public convenience, Although she was Executive Directress of BSPA, she was merely an
justify wrong, protect fraud, or defend crime, or to confuse legitimate issues, employee of the BSPA, which was a single proprietorship.

336
Now, the claims of petitioners are actually money claims against the estate of
Felipe Bacani. They must be filed against his estate in accordance with Sec.
5 of Rule 86 which provides in part:

Sec. 5. Claims which must be filed under the notice. If not


filed, barred; exceptions. All claims for money against the
decedent, arising from contract, express or implied, whether
the same be due, not due, or contingent, all claims for
funeral expenses and expenses for the last sickness of the
decedent, and judgment for money against the decedent,
must be filed within the time limited in the notice; otherwise
they are barred forever, except that they may be set forth as
counterclaims in any action that the executor or
administrator may bring against the claimants . . .

The rationale for the rule is that upon the death of the defendant, a testate or
intestate proceeding shall be instituted in the proper court wherein all his
creditors must appear and file their claims which shall be paid proportionately
out of the property left by the deceased. The objective is to avoid duplicity of
procedure. Hence the ordinary actions must be taken out from the ordinary
courts. 6 Under Art. 110 of the Labor Code, money claims of laborers enjoy
preference over claims of other creditors in case of bankruptcy or liquidation
of the employer's business.

WHEREFORE, the petition for certiorari is DISMISSED.

SO ORDERED.

Narvasa, C.J., Regalado and Puno, JJ., concur.

337
338
G.R. No. L-2886 August 22, 1952

GREGORIO ARANETA, INC., plaintiff-appellant,


vs.
PAZ TUASON DE PATERNO and JOSE VIDAL, defendants-appellants.

Araneta and Araneta for appellant.


Ramirez and Ortigas for defendants-appellants.
Perkins, Ponce Enrile and Contreras And La O and Feria for appellee.

TUASON, J.:

This is a three-cornered contest between the purchasers, the seller, and the
mortgagee of certain portions (approximately 40,703 square meters) of a big
block of residential land in the district of Santa Mesa, Manila. The plaintiff,
which is the purchaser, and the mortgagee elevated this appeal. Though not
an appellant, the seller and mortgagor has made assignments of error in her
brief, some to strengthen the judgment and others for the purpose of new
trial.

The case is extremely complicated and multiple issues were raised.

The salient facts in so far as they are not controverted are these. Paz Tuason
de Paterno is the registered owner of the aforesaid land, which was
subdivided into city lots. Most of these lots were occupied by lessees who
had contracts of lease which were to expire on December 31,1952, and
carried a stipulation to the effect that in the event the owner and lessor
should decide to sell the property the lessees were to be given priority over
other buyers if they should desire to buy their leaseholds, all things being
equal. Smaller lots were occupied by tenants without formal contract.

339
In 1940 and 1941 Paz Tuason obtained from Jose Vidal several loans Paz Tuason se obliga a entregar mediante un propio las cartasque
totalling P90,098 and constituted a first mortgage on the aforesaid property dirigira a este efecto a los arrendatarios, de conformidad con el
to secure the debt. In January and April, 1943, she obtained additional loans formulario adjunto, que se marca como Apendice A.
of P30,000 and P20,000 upon the same security. On each of the last-
mentioned occasions the previous contract of mortgage was renewed and Expirado el plazo arriba mencionado, Paz Tuason otorgara las
the amounts received were consolidated. In the first novated contract the escrituras correspondientes de venta a los arrendatarios que hayan
time of payment was fixed at two years and in the second and last at four decidido comprar sus respectivos lotes.
years. New conditions not relevant here were also incorporated into the new
contracts. 9. Los alquieres correspondientes a este ao se prorratearan entre
la vendedora y el comprador, correspondiendo al comprador los
There was, besides, a separate written agreement entitled "Penalidad del alquileres correspondientes a Noviembre y Diciembre de este ao y
Documento de Novacion de Esta Fecha" which, unlike the principal asimismo sera por cuenta del comprador el amillaramiento
contracts, was not registered. The tenor of this separate agreement, all correspondiente a dichos meses.
copies, of which were alleged to have been destroyed or lost, was in dispute
and became the subject of conflicting evidence. The lower court did not make 10. Paz Tuason, reconoce haver recibido en este acto de Gregorio
categorical findings on this point, however, and it will be our task to do so at Araneta, Inc., la suma de Ciento Noventa Mil Pesos (P190,000)como
the appropriate place in this decision. adelanto del precio de venta que Gregorio Araneta, Inc., tuviere que
pagar a Paz Tuason.
In 1943 Paz Tuason decided to sell the entire property for the net amount of
P400,000 and entered into negotiations with Gregorio Araneta, Inc. for this La cantidad que Paz Tuason recibe en este acto sera aplicadapor
purpose. The result of the negotiations was the execution on October 19, ella a saldar su deuda con Jose Vidal, los amillaramientos, sobre el
1943, of a contract called "Promesa de Compra y Venta" and identified as utilizado por Paz Tuason para otros fines.
Exhibit "1." This contract provided that subject to the preferred right of the
lessees and that of Jose Vidal as mortgagee, Paz Tuason would sell to
11. Una vez determinados los lotes que Paz Tuason podra vendera
Gregorio Araneta, Inc. and the latter would buy for the said amount of
Gregorio Araneta, Inc., Paz Tuason otorgara una escritura deventa
P400,000 the entire estate under these terms.
definitiva sobre dichos lotes a favor de Gregorio Araneta, Inc.

El precio sera pagado como sigue: un 40 por ciento juntamente con


Gregorio Araneta, Inc., pagara el precio de venta como sigue: 90 por
la carta de aceptacion del arrendatario, un 20 por ciento delprecio al
ciento del mismo al otorgarse la escritura de venta definitiva
otorgarse la escritura de compromiso de venta, y el remanente 40
descontandose de la cantidad que entonces se tenga que pagar de
por ciento al otorgarse la escritura de venta definitiva, la cual sera
adelanto de P190,000 que se entrega en virtud de esta escritura. El
otorgada despues de que se habiese canceladola hipoteca a favor
10 por ciento remanente se pagara a Paz Tuazon, una vez se haya
de Jose Vidal que pesa sobre dichos lotes. Lacomision del 5 por
cancelado la hipoteca que pesa actualmente sobre el terreno.
ciento que corresponde a Jose Araneta serapagada al otorgarse la
escritura de compromiso de venta.
No obstante la dispuesto en el parrafo 8, cualquier arrendatario que
decida comprar el lote que occupa con contrato de arrendamiento

340
podra optar por pedir el otorgamiento inmediato a su favor el acto de (P125,174.99), the Vendor acknowledges to have received by virtue
la escritura de venta definitiva pagando en el acto el 50 por ciento of the advance of One Hundred Ninety Thousand (P190,000) Pesos
del precio (ademas del 40 por ciento que debio incluir en su carta de made by the Vendee to the Vendor upon the execution of the
aceptacion) y el remanente de 10 por ciento inmediatemente aforesaid contract entitled "Promesa de Compra y Venta". The
despues de cancelarse la hipoteca que pesa sobre el terreno. balance of Sixty-Four Thousand Eight Hundred Twenty-five Pesos
and One centavo (P64,825.01) between the sum of P125,174.99,
12. Si la mencionada cantidad de P190,000 excediere del 90 por has been returned by the Vendor to the Vendee, which amount the
ciento de la cantidad que Gregorio Araneta, Inc., tuviere que vender Vendee acknowledges to have received by these presents;
a dicho comprador, el saldo sera pagado inmediatamente por Paz
Tuazon, tomandolo de las cantidades que reciba de los The aforesaid sum of P190,000 was delivered by the Vendee to the
arrendatarios como precio de venta. Vendor by virtue of four checks issued by the Vendee against the
Bank of the Philippine Islands, as follows:
In furtherance of this promise to buy and sell, letters were sent the lessees
giving them until August 31, 1943, an option to buy the lots they occupied at
the price and terms stated in said letters. Most of the tenants who held No. C-286445 in favor of Paz Tuason de P13,476.62
contracts of lease took advantage of the opportunity thus extended and after Paterno
making the stipulated payments were giving their deeds of conveyance.
These sales, as far as the record would show, have been respected by the
seller.
No. C-286444 in favor of the City Treasurer, 3,373.38
With the elimination of the lots sold or be sold to the tenants there remained Manila
unencumbered, except for the mortgage to Jose Vidal, Lots 1, 8-16 and 18
which have an aggregate area of 14,810.20 square meters; and on
December 2, 1943, Paz Tuason and Gregorio Araneta, Inc. executed with
regard to these lots an absolute deed of sale, the terms of which, except in No. C-286443 in favor of Jose Vidal 30,000.00
two respects, were similar to those of the sale to the lessees. This deed,
copy of which is attached to the plaintiff's complaint as Exhibit A, provided,
among other things, as follows: No. C-286442 in favor of Jose Vidal 143,150.00

The aforesaid lots are being sold by he Vendor to the Vendee


separately at the prices mentioned in paragraph (6) of the aforesaid
contract entitled "Promesa de Compra y Venta," making a total sum Total P190,000.00
of One Hundred Thirty-Nine Thousand Eighty-three pesos and
Thirty-two centavos (P139,083.32), ninety (90%) per cent of which
amount, i.e., the sum of One Hundred Twenty-five Thousand One The return of the sum of P64,825.01 was made by the Vendor to the
Hundred Seventy-four Pesos and Ninety-nine centavos Vendee in a liquidation which reads as follows:

341
Hemos recibido de Da. Paz Tuason de Paterno la
cantidad de Sesenta y Cuatro mil Ochocientos Veinticinco
Pesos y un centimo (P64,825.01) enconcepto de
devolucion que nos hace del excesode lo pagadoa ella
de P190,000.00 P68,563.21

Menos el 90% de P139,083.32, importe de los lotes que


vamos a comprar 125,174.99

Menos las comisiones de 5 % recibidas de


Exceso 64,825.01 Josefina de Pabalan P538.60

L.E. Dumas 1,084.43

Cheque BIF No. D-442988 de Simplicio del Rosario 21,984.20 Angela S. Tuason 1,621.94 3,244.97

Cheque PNB No. 177863-K de L.E. Dumas 21,688.60 P65,318.24

Cheque PNB No. 267682-K de Alfonso Sycip 20,000.00 Menos cheque BIF No. C-288642 a favor de
Da. Paz Tuason de Paterno que le entregamos
como exceso 493.23

Cheque PNB No. 83940 de Josefina de Pabalan 4,847.96

P64,825.01

Billetes recibidos de Alfonso Sycip 42.96

342
Manila, Noviembre 2, 1943 that this sale is being executed free from any option or right on the
part of the lessees to purchase the lots respectively leased by them.

It is therefore clearly understood that the Vendor will pay the existing
GREGORIO ARANETA, INCORPORATED mortgage on her property in favor of Jose Vidal.
Por;
(Fdo.) "JOSE ARANETA
The liquidation of the amounts respectively due between the Vendor
Presidente
and the Vendee in connection with the rents and real estate taxes as
stipulated in paragraph (9) of the contract entitled "Promesa de
Compara y Venta" will be adjusted between the parties in a separate
Recibido cheque No. C-288642 BIF-P493.23 document.

Should any of the aforesaid lessees of lots Nos. 2, 3, 4, 5, 6, 7, 9 and


17 fail to carry out their respective obligations under the option to
Por: purchase exercised by them so that the rights of the lessee to
(Fdo.) "M.J. GONZALEZ purchase the respective property leased by him is cancelled, the
Vendor shall be bound to sell the same to the herein Vendee,
Gregorio Araneta, Incorporated, in conformity with the terms and
conditions provided in the aforesaid contract of "Promesa de Compra
In view of the foregoing liquidation, the vendor acknowledges fully y Venta";
and unconditionally, having received the sum of P125,174.99 of the
present legal currency and hereby expressly declares that she will
The documentary stamps to be affixed to this deed will be for the
not hold the Vendee responsible for any loss that she might suffer
account of the Vendor while the expenses for the registration of this
due to the fact that two of the checks paid to her by the Vendee were
document will be for the account of the Vendee.
issued in favor of Jose Vidal and the latter has, up to the present
time, not yet collected the same.
The remaining area of the property of the Vendor subject to Transfer
Certificates of Title Nos. 60471 and 60472, are lots Nos. 2, 3, 4, 5, 6,
The ten (10%) per cent balance of the purchase price not yet paid in
7, 9, and 17, all of the Consolidation of lots Nos. 20 and 117 of plan
the total sum of P13,908.33 will be paid by the Vendee to the Vendor
II-4755, G.L.R.O. Record No. 7680.
when the existing mortgage over the property sold by the Vendor to
the Vendee is duly cancelled in the office of the Register of Deeds, or
sooner at the option of the Vendee. Before the execution of the above deed, that is, on October 20, 1943, the day
immediately following the signing of the agreement to buy and sell, Paz
Tuason had offered to Vidal the check for P143,150 mentioned in Exhibit A,
This Deed of Sale is executed by the Vendor free from all liens and
in full settlement of her mortgage obligation, but the mortgagee had refused
encumbrances, with the only exception of the existing lease
to receive that check or to cancel the mortgage, contending that by the
contracts on parcels Nos. 1, 10, 11, and 16, which lease contracts
will expire on December 31, 1953, with the understanding, however,

343
separate agreement before mentioned payment of the mortgage was not to the accrual of interest on the loans, Vidal's claim to attorney's fees, and the
be effected totally or partially before the end of four years from April, 1943. application of the debt moratorium law which the debtor now invokes. These
matters will be taken up in the discussion of the controversy between Paz
Because of this refusal of Vidal's Paz Tuason, through Atty. Alfonso Ponce Tuason and Jose Vidal.
Enrile, commenced an action against the mortgagee in October or the early
paret of November 1943. the record of that case was destroyed and no copy The principal bone of contention between Gregorio Araneta, Inc., and Paz
of the complaint was presented in evidence. Attached to the complaint or Tuason was the validity of the deed of sale of Exhibit A on which the suit was
deposited with the clerk of court by Attorney Ponce Enrile simultaneously predicated. The lower court's judgment was that this contract was invalid and
with the docketing of the suit were the check for P143,150 previously turned was so declared, "sin per juicio de que la demandada Paz Tuason de
down by Vidal, another certified check for P12,932.61, also drawn by Paterno pague a la entidad demandante todas las cantidades que habia
Gregorio Araneta, Inc., in favor of Vidal, and one ordinary check for P30,000 estado recibiendo de lareferida entidad demandante, en concepto de pago
issued by Paz Tuazon. These three checks were supposed to cover the de losterrenos, en moneda corriente, segun el cambio que debiaregir al
whole indebtedness to Vidal including the principal and interest up to that tiempo de otorgarse la escritura segun la escalade "Ballentine",
time and the penalty provided in the separate agreement. descontando, sin embargo, de dichas cantidades cualesquiera que la
demandante haya estadorecibiendo como alquileres de los terrenos
But the action against Vidal never came on for trial and the record and the supuestamentevendidos a ella." The court based its opinion that Exhibit 1.
checks were destroyed during the war operations in January or February, His Honor, Judge Sotero Rodas, agreedwith the defendant that under
1945; and neither was the case reconstituted afterward. This failure of the paragraph 8 of Exhibit 1 there was to be no absolute sale to Gregorio
suit for the cancellation of Vidal's mortgage, coupled with the destruction of Araneta, Inc., unless Vidal's mortgage was cancelled.
the checks tendered to the mortgagee, the nullification of the bank deposit on
which those checks had been drawn, and the tremendous rise of real estate In our opinion the trial court was in error in its interpretation of Exhibit 1. The
value following the termination of the war, gave occasion to the breaking off contemplated execution of an absolute deed of sale was not contingent on
the schemes outlined in Exhibits 1 and A; Paz Tuason after liberation the cancellation of Vidal's mortgage. What Exhibit 1 did provide (eleventh
repudiated them for the reasons to be hereafter set forth. The instant action paragraph) was that such deed of absolute sale should be executed "una vez
was the offshoot, begun by Gregorio Araneta, Inc. to compel Paz Tuason to determinado los lotes que Paz Tuason podra vender a Gregorio Araneta,
deliver to the plaintiff a clear title to the lots described in Exhibit A free from Inc." The lots which could be sold to Gregorio Araneta, Inc. were definitely
all liens and encumbrances, and a deed of cancellation of the mortgage to known by October 31, 1943, which was the expiry of the tenants' option to
Vidal. Vidal came into the case in virtue of a summon issued by order of the buy, and the lots included in the absolute of which the occupants' option to
court, and filed a cross-claim against Paz Tuazon to foreclose his mortgage. buy lapsed unconditionally. Such deed as Exhibit A was then in a condition to
be made.
It should be stated that the outset that all the parties are in agreement that
Vidal's loans are still outstanding. Paz Tuason's counsel concede that the Vidal's mortgage was not an obstacle to the sale. An amount had been set
tender of payment to Vidal was legally defective and did not operate to aside to take care of it, and the parties, it would appear, were confident that
discharge the mortgage, while the plaintiff is apparently uninterested in this the suit against the mortgagee would succeed. The only doubt in their minds
feature of the case considering the matter one largely between the mortgagor was in the amount to which Vidal was entitled. The failure of the court to try
and the mortgagee, although to a certain degree this notion is incorrect. At and decide that the case was not foreseen either.
any rate, the points of discord between Paz Tuason and Vidal concern only

344
This refutes, were think, the charge that there was undue rush on the part of granted the privilege to deduct as much as 40 per cent of the stipulated price
the plaintiff to push across the sale. The fact that simultaneously with Exhibit pending discharge of the mortgage, although his percentage was later
A similar deeds were given the lessees who had elected to buy their reduced to 10 as in the case of Gregorio Araneta, Inc. It has also been that
leaseholds, which comprise an area about twice as big as the lots described the validity of the sales to the tenants has not been contested; that these
in Exhibit A, and the further fact that the sale to the lessees have never been sales embraced in the aggregate 24,245.40 square meters for P260,916.68
questioned and the proceeds thereof have been received by the defendant, as compared to 14,811.20 square meters sold to Gregorio Araneta, Inc. for
should add to dispel any suspicion of bad faith on the part of the plaintiff. If P139,083.32; that the seller has already received from the tenant purchasers
anyone was in a hurry it could have been the defendant. The clear 90 per cent of the purchase money.
preponderance of the evidence that Paz Tuason was pressed for cash and
that the payment of the mortgage was only an incident, or a necessary There is good reason to believe that had Gregorio Araneta, Inc. not insisted
means to effectuate the sale. Otherwise she could have settled her mortgage on charging to the defendant the loss of the checks deposited with the court,
obligation merely by selling a portion of her estate, say, some of the lots the sale in question would have gone the smooth way of the sales to the
leased to tenants who, except two who were in concentration camps, were tenants. Thus Dindo Gonzales, defendant's son, declared:
only too anxious to buy and own the lots on which their houses were built.
P. Despues de haberse presentado esta demanda, recuerda usted
Whatever the terms of Exhibit 1, the plaintiff and the defendant were at haber tenido conversacion con Salvador Araneta acerca de este
perfect liberty to make a new agreement different from or even contrary to asunto?
the provisions of that document. The validity of the subsequent sale must of
necessity depend on what it said and not on the provisions of the promise to R. Si Seor.
buy and sell.
P. Usted fue quien se acerco al seor Salvador Araneta?
It is as possible proof or fraud that the discrepancies between the two
documents bear some attention. It was alleged that Attorneys Salvador
R. Si, seor.
Araneta and J. Antonio Araneta who the defendant said had been her
attorneys and had drawn Exhibit A, and not informed or had misinformed her
about its contents; that being English, she had not read the deed of sale; that P. Quiero usted decir al Honorable Juzgado que era lo que usted dijo
if she had not trusted the said attorneys she would not have been so foolish al seor Salvador Araneta?
as to affix her signature to a contract so one-sided.
R. No creo que es propio que yo diga, por tratarse de mi madre.
The evidence does not support the defendant. Except in two particulars,
Exhibit A was a substantial compliance with Exhibit 1 in furtherance of which P. En otras palabras, usted quiere decir que no quiere usted que se
Exhibit A was made. One departure was the proviso that 10 per cent of the vuelva decir o repetir ante este Honorable Juzgado lo que usted dijo
purchase price should be paid only after Vidal's mortgage should have been al seor Salvador Araneta, pues, se trata de su madre?
cancelled. This provisional deduction was not onerous or unusual. It was not
onerous or unusual that the vendee should withhold a relatively small portion R. No, seor.
of the purchase price before all the impediments to the final consummation of
the sale had been removed. The tenants who had bought their lots had been

345
P. Puede usted decirnos que quiso usted decir cuando que no In view of the foregoing liquidation, the Vendor acknowledges fully
quisiera decir? and unconditionally, having received the sum of P125,174.99 of the
present legal currency and hereby expressly declares that she will
R. Voy a decir lo que Salvador Araneta, yo me acerque a Don not hold the Vendee responsible for any loss that she might suffer
Salvador Araneta, y yo le dije que es una verguenza de que due to the fact that two of the checks paid to her by the Vendee were
nosotros, en la familia tengamos que ir a la Corte por este, y tambien used in favor of Jose Vidal and the latter has, up to the present time,
dije que mi madre de por si quiere vender el terreno a ellos, porque not yet collected the same.
mi madre quiere pagar al seor Vidal, y que es una verguenza,
siendo entre parientes, tener que venir por este; era lo que yo dije al It was argued that no person in his or her right senses would knowingly have
seor Salvador Araneta. agreed to a covenant so iniquitous and unreasonable.

xxx xxx xxx In the light of all the circumstances, it is difficult to believe that the defendant
was deceived into signing Exhibit A, in spite of the provision of which she and
P. No recuerda usted tambien dijo al seor Salvador Araneta que her son complaint. Intelligent and well educated who had been managing her
usted no comulgaba con ella (su madre) en este asunto? affairs, she had an able attorney who was assisting her in the suit against
Vidal, a case which was instituted precisely to carry into effect Exhibit A or
R. Si, Seor; porque yo creia que mi madre solamente queria anular Exhibit 1, and a son who is leading citizen and a business-man and knew the
esta venta, pero cuando me dijo el seor La O y sus abogados English language very well if she did not. Dindo Gonzalez took active part in,
que, encima de quitar la propiedad, todavia tendria ella que pagar al if he was not the initiator of the negotiations that led to the execution of
seor Vidal, este no veso claro. Exhibit 1, of which he was an attesting witness besides. If the defendant
signed Exhibit A without being apprised of its import, it can hardly be
conceived that she did not have her attorney or her son read it to her
xxx xxx xxx
afterward. The transaction involved the alienation of property then already
worth a fortune and now assessed by the defendant at several times higher.
P. Ahora bien; de tal suerte que, tal como nosotros desperendemos Doubts in defendant's veracity are enhanced by the fact that she denied or at
de su testimonio, tanto, usted como, su madre, esteban muy least pretended in her answer to be ignorant of the existence of Exhibit A,
conformes en la venta, es asi? and that only after she was confronted with the signed copy of the document
on the witness did she spring up the defense of fraud. It would look as if she
R. Si, seor. gambled on the chance that no signed copy of the deed had been saved
from the war. She could not have forgotten having signed so important a
The other stipulation embodied in Exhibit A which had no counterpart in document even if she had not understood some of its provisions.
Exhibit 1 was that by which Gregorio Araneta Inc. would hold Paz Tuason
liable for the lost checks and which, as stated, appeared to be at the root of From the unreasonableness and inequity of the aforequoted Exhibit A it is not
the whole trouble between the plaintiff and the defendant. to be presumed that the defendant did not understand it. It was highly
possible that she did not attach much importance to it, convinced that Vidal
The stipulation reads: could be forced to accept the checks and not foreseeing the fate that lay in
store for the case against the mortgagee.

346
Technical objections are made against the deed of sale. real estate business. The corporate entity was not used to circumvent the law
or perpetrate deception. There is no denying that Gregorio Araneta, Inc.
First of these is that Jose Araneta, since deceased, was defendant's agent entered into the contract for itself and for its benefit as a corporation. The
and at the same time the president of Gregorio Araneta, Inc. contract and the roles of the parties who participated therein were exactly as
they purported to be and were fully revealed to the seller. There is no
The trial court found that Jose Araneta was not Paz Tuason's agent or broker. pretense, nor is there reason to suppose, that if Paz Tuason had known Jose
This finding is contrary to the clear weight of the evidence, although the point Araneta to Gregorio Araneta, Inc's president, which she knew, she would not
would be irrelevant, if the court were right in its holding that Exhibit A was have gone ahead with the deal. From her point of view and from the point of
void on another ground, i.e., it was inconsistent with Exhibit 1. view of public interest, it would have made no difference, except for the
brokerage fee, whether Gregorio Araneta, Inc. or Jose Araneta was the
purchaser. Under these circumstances the result of the suggested disregard
Without taking into account defendant's Exhibit 7 and 8, which the court
of a technicality would be, not to stop the commission of deceit by the
rejected and which, in our opinion, should have been admitted, Exhibit 1 is
purchaser but to pave the way for the evasion of a legitimate and binding
decisive of the defendant's assertion. In paragraph 8 of Exhibit 1 Jose
commitment buy the seller. The principle invoked by the defendant is
Araneta was referred to as defendant's agent or broker "who acts in this
resorted to by the courts as a measure or protection against deceit and not to
transaction" and who as such was to receive a commission of 5 per cent,
open the door to deceit. "The courts," it has been said, "will not ignore the
although the commission was to be charged to the purchasers, while in
corporate entity in order to further the perpetration of a fraud." (18 C.J.S.
paragraph 13 the defendant promised, in consideration of Jose Araneta's
381.)
services rendered to her, to assign to him all her right, title and interest to and
in certain lots not embraced in the sales to Gregorio Araneta, Inc. or the
tenants. The corporate theory aside, and granting for the nonce that Jose Araneta and
Gregorio Araneta, Inc. were identical and that the acts of one where the acts
of the other, the relation between the defendant and Jose Araneta did not fall
However, the trial court hypothetically admitting the existence of the relation
within the purview of article 1459 of the Spanish Civil Code. 1
of principal and agent between Paz Tuason and Jose Araneta, pointed out
that not Jose Araneta but Gregorio Araneta, Inc. was the purchaser, and cited
the well-known distinction between the corporation and its stockholders. In Agency is defined in article 1709 in broad term, and we have not come
other words, the court opined that the sale to Gregorio Araneta, Inc. was not across any commentary or decision dealing directly with the precise meaning
a sale to Jose Araneta the agent or broker. of agency as employed in article 1459. But in the opinion of Manresa(10
Manresa 4th ed. 100), agent in the sense there used is one who accepts
another's representation to perform in his name certain acts of more or less
The defendant would have the court ignore this distinction and apply to this
transcendency, while Scaevola (Vol. 23, p. 403) says that the agent's in
case the other well-known principle which is thus stated in 18 C.J.S. 380:
capacity to buy his principal's property rests in the fact that the agent and the
"The courts, at law and in equity, will disregard the fiction of corporate entity
principal form one juridicial person. In this connection Scaevola observes that
apart from the members of the corporation when it is attempted to be used as
the fear that greed might get the better of the sentiments of loyalty and
a means of accomplishing a fraud or an illegal act.".
disinterestedness which should animate an administrator or agent, is the
reason underlying various classes of incapacity enumerated in article 1459.
It will at once be noted that this principle does not fit in with the facts of the And as American courts commenting on similar prohibition at common law
case at bar. Gregorio Araneta, Inc. had long been organized and engaged in

347
put it, the law does not trust human nature to resist the temptations likely to of sale to the latter, and charged the defendant the corresponding fees for all
arise of antogonism between the interest of the seller and the buyer. this work, did not themselves prove that they were the seller's attorneys.
These letters and documents were wrapped up with the contemplated sale in
So the ban of paragraph 2 of article 1459 connotes the idea of trust and which Gregorio Araneta, Inc. was interested, and could very well have been
confidence; and so where the relationship does not involve considerations of written by Attorneys Araneta and Araneta in furtherance of Gregorio Araneta's
good faith and integrity the prohibition should not and does not apply. To own interest. In collecting the fees from the defendant they did what any
come under the prohibition, the agent must be in a fiduciary with his principal. other buyer could have appropriately done since all such expenses normally
were to be defrayed by the seller.
Tested by this standard, Jose Araneta was not an agent within the meaning
of article 1459. By Exhibits 7 and 8 he was to be nothing more than a go- Granting that Attorney Araneta and Araneta were attorneys for the defendant,
between or middleman between the defendant and the purchaser, bringing yet they were not forbidden to buy the property in question. Attorneys are
them together to make the contract themselves. There was no confidence to only prohibited from buying their client's property which is the subject of
be betrayed. Jose Araneta was not authorize to make a binding contract for litigation. (Art. 1459, No. 5, Spanish Civil Code.) The questioned sale was
the defendant. He was not to sell and he did not sell the defendant's property. effected before the subject thereof became involved in the present action.
He was to look for a buyer and the owner herself was to make, and did There was already at the time of the sale a litigation over this property
make, the sale. He was not to fix the price of the sale because the price had between the defendant and Vidal, but Attys. Salvador Araneta and J. Antonio
been already fixed in his commission. He was not to make the terms of Araneta were not her attorneys in that case.
payment because these, too, were clearly specified in his commission. In
fine, Jose Araneta was left no power or discretion whatsoever, which he From the pronouncement that Exhibit A is valid, however, it does not follow
could abuse to his advantage and to the owner's prejudice. that the defendant should be held liable for the loss of the certified checks
attached to the complaint against Vidal or deposited with the court, or of the
Defendant's other ground for repudiating Exhibit A is that the law firm of funds against which they had been issued. The matter of who should bear
Araneta & Araneta who handled the preparation of that deed and this loss does not depend upon the validity of the sale but on the extent and
represented by Gregorio Araneta, Inc. were her attorneys also. On this point scope of the clause hereinbefore quoted as applied to the facts of the
the trial court's opinion is likewise against the defendant. present case.

Since attorney Ponce Enrile was the defendant's lawyer in the suit against The law and the evidence on this branch of the case revealed these facts, of
Vidal, it was not likely that she employed Atty. Salvador Araneta and J. some of which passing mention has already been made.
Antonio Araneta as her attorneys in her dealings with Gregorio Araneta, Inc.,
knowing, as she did, their identity with the buyer. If she had needed legal The aforesaid checks, one for P143,150 and one for P12,932.61, were
counsels, in this transaction it seems certain that she would have availed issued by Gregorio Araneta, Inc. and payable to Vidal, and were drawn
herself of the services of Mr. Ponce Enrile who was allegedly representing against the Bank of the Philippines with which Gregorio Araneta, Inc. had a
her in another case to pave the way for the sale. deposit in the certification stated that they were to be "void if not presented
for payment date of acceptance" office (Bank) within 90 days from date of
The fact that Attys. Salvador and Araneta and J. Antonio Araneta drew acceptance."
Exhibits 1 and A, undertook to write the letters to the tenants and the deeds

348
Under banking laws and practice, by the clarification" the funds represented But as to Gregorio Araneta and Paz Tuason, the conditions specified in the
by the check were transferred from the credit of the maker to that of the certification and the prevailing regulations of the Bank were the law of the
payee or holder, and, for all intents and purposes, the latter became the case. Not only this, but they were aware of and abided by those regulations
depositor of the drawee bank, with rights and duties of one such relation." and practice, as instanced by the fact that the parties presented testimony to
But the transfer of the corresponding funds from the credit of the depositor to prove those regulations and practice. And that Gregorio Araneta, Inc. knew
that of that of the payee had to be co-extensive with the life of the checks, that Vidal had not cashed the checks within 90 days is not, and could not
which in the case was 90 days. If the checks were not presented for payment successfully be denied.
within that period they became invalid and the funds were automatically
restored to the credit of the drawer though not as a current deposit but as In these circumstances, the stipulation in Exhibit A that the defendant or
special deposit. This is the consensus of the evidence for both parties which seller "shall not hold the vendee responsible for any loss of these checks"
does not materially differ on this proposition. was unconscionable, void and unenforceable in so far as the said stipulation
would stretch the defendant's liability for this checks beyond 90 days. It was
The checks were never collected and the account against which they were not in accord with law, equity or good conscience to hold a party responsible
drawn was not used or claimed by Gregorio Araneta, Inc.; and since that for something he or she had no access to and could not make use of but
account "was opened during the Japanese occupation and in Japanese which was under the absolute control and disposition of the other party. To
currency," the checks "became obsolete as the account subject thereto is make Paz Tuason responsible for those checks after they expired and when
considered null and void in accordance with Executive Order No. 49 of the they were absolutely useless would be like holding an obligor to answer for
President of the Philippines", according to the Bank. the loss or destruction of something which the obligee kept in its safe with no
power given the obligor to protect it or interfere with the obligee's possession.
Whether the Bank of the Philippines could lawfully limit the negotiability of
certified checks to a period less than the period provided by the Statute of To the extent that the contract Exhibit A would hold the vendor responsible for
Limitations does not seem material. The limitation imposed by the Bank as to those checks after they had lapsed, the said contract was without
time would adversely affect the payee, Jose Vidal, who is not trying to consideration. The checks having become obsolete, the benefit in exchange
recover on the instruments but on the contrary rejected them from the outset, for which the defendant had consented to be responsible for them had
insisting that the payment was premature. As far as Vidal was concerned, it vanished. The sole motivation on her part for the stipulation was the fact that
was of no importance whether the certification was or was not restricted. On by the checks the mortgage might or was to be released. After 90 days the
the other hand, neither the plaintiff nor the defendant now insists that Vidal defendant stood to gain absolutely nothing by them, which had become
should present, or should have presented, the checks for collection. They in veritable scraps of paper, while the ownership of the deposit had reverted to
fact agree that the offer of those checks to Vidal did not, for technical reason, the plaintiff which alone could withdraw and make use of it.
work to wipe out the mortgage.
What the plaintiff could and should have done if the disputed stipulation was
to be kept alive was to keep the funds accessible for the purpose of paying
the mortgage, by writing new checks either to Vidal or to the defendant, as
was done with the check for P30,000, or placing the deposit at the
defendant's disposal. The check for P30,000 intended for the penalty
previously had been issued in the name of Vidal and certified, too, but by
mutual agreement it was changed to an ordinary check payable to Paz

349
Tuason. Although that check was also deposited with the court and lost, its under the Ballantyne conversion table. As has been said, where the parties
loss undoubtedly was imputable to the defendant's account, and she did not do not see eye-to-eye was in regard to the mortgagee's claim to attorney's
seem to disown her liability for it. fees and interest from October, 1943, which was reached a considerable
amount. It was contended that, having offered to pay Vidal her debt in that
Let it be remembered that the idea of certifying the lost checks was all the month, the defendant was relieved thereafter from paying such interest.
plaintiff's. The plaintiff would not trust the defendant and studiously so
arranged matters that she could not by any possibility put a finger on the It is to be recalled that Paz Tuason deposited with the court three checks
money. For all the practical intents and purposes the plaintiff dealt directly which were intended to cover the principal and interest up to October, 1943,
with the mortgagee and excluded the defendant from meddling in the manner plus the penalty provided in the instrument "Penalidad del Documento de
of payment to Vidal. And let it also be kept in mind that Gregorio Araneta, Inc. Novacion de Esta Fecha." The mortgagor maintains that although these
was not a mere accommodator in writing these checks. It was as much checks may not have constituted a valid payment for the purpose of
interested in the cancellation of the mortgage as Paz Tuason. discharging the debt, yet they did for the purpose of stopping the running of
interest. The defendant draws attention to the following citations:
Coming down to Vidal's cross-claim Judge Rodas rendered no judgment
other than declaring that the mortgage remained intact and subsisting. The An offer in writing to pay a particular sum of money or to deliver a
amount to be paid Vidal was not named and the question whether interest written instrument or specific personal property is, if rejected,
and attorney's fees were due was not passed upon. The motion for equivalent to the actual production and tender of the money,
reconsideration of the decision by Vidal's attorney's praying that Paz Tuason instrument or property. (Sec. 24, Rule 123.)
be sentenced to pay the creditor P244,917.90 plus interest at the rate of 1
percent monthly from September 10, 1948 and that the mortgaged property It is not accord with either the letter or the spirit of the law to impose
be ordered sold in case of default within 90 days, and another motion by the upon the person affecting a redemption of property, in addition to 12
defendant seeking specification of the amount she had to pay the mortgagee per cent interest per annum up to the time of the offer to redeem, a
were summarily denied by Judge Potenciano Pecson, to whom the motions further payment of 6 per cent per annum from the date of the officer
were submitted, Judge Rodas by that time having been appointed to the to redeem. (Fabros vs. Villa Agustin, 18 Phil., 336.)
Court of Appeals.
A tender by the debtor of the amount of this debt, if made in the
All the facts and evidence on this subject are on the record, however, and we proper manner, will suspend the running of interest on the debt for
may just as well determine from these facts and evidence the amount to the time of such tender. (30 Am. Jur., 42.)
which the mortgagee is entitled, instead of remanding the case for new trial,
if only to avoid further delay if the disposition of this case.

It is obvious that Vidal had a right to judgment for his credit and to foreclose
the mortgage if the credit was not paid.

There is no dispute as to the amount of the principal and there is agreement


that the loans made in 1943, in Japanese war notes, should be computed

350
In the case of Fabrosa vs. Villa Agustin, supra, a parcel of land had been We are of the opinion that the court erred in excluding Vidal's statement.
sold on execution to one Tabliga. Within the period of redemption Fabros, to There is no reason to suspect that Vidal's attorney did not correctly read the
whom the land had been mortgaged by the execution debtor, had offered to paper to him. The reading was a contemporaneous incident of the writing
redeem the land from the execution creditor and purchaser at public auction. and the circumstances under which the document was read precluded every
The trial court ruled that the redemptioner was not obliged to pay the possibility of design, premeditation, or fabrication.
stipulated interest of 12 per cent after he offered to redeem the property;
nevertheless he was sentenced to pay 6 per cent interest from the date of Nevertheless, Vidal's testimony, like the testimony of Lucio M. Tiangco's, was
the offer. based on recollection which, with the lapse of time, was for from infallible. By
contrast, the testimony of Attorneys Ponce Enrile, Salvador Araneta, and J.
This court on appeal held that "there is no reason for this other (6 per cent) Antonio Araneta does not suffer from such weakness and is entitled to full
interest, which appears to be a penalty for delinquency while there was no faith and credit. The document was the subject of a close and concerted
delinquency." The court cited an earlier decision, Martinez vs. Campbell, 10 study on their part with the object of finding the rights and obligations of the
Phil., 626, where this doctrine was laid down: "When the right of redemption mortgagee and the mortgagor in the premises and mapping out the course to
is exercised within the term fixed by section 465 of the Code of Civil be pursued. And the results of their study and deliberation were translated
Procedure, and an offer is made of the amount due for the repurchase of the into concrete action and embodied in a letter which has been preserved. In
property to which said right refers, it is neither reasonable nor just that the line with the results of their study, action was instituted in court to compel
repurchaser should pay interest on the redemption money after the time acceptance by Vidal of the checks consigned with the complaint, and before
when he offered to repurchase and tendered the money therefor." the suit was commenced, and with the document before him, Atty. Ponce
Enrile, in behalf of his client, wrote Vidal demanding that he accept the
In the light of these decisions and law, the next query is; Did the mortgagor payment and execute a deed of cancellation of the mortgage. In his letter
have the right under the contract to pay the mortgage on October 20, 1943? Atty. Ponce Enrile reminded Vidal that the recital in the "Penalidad del
The answer to this question requires an inquiry into the provision of the Documento de Novacion de Esta Fecha" was "to the effect that should the
"Penalidad del Documento de Novacion de Esta Fecha." debtor wish to pay the debt before the expiration of the period the reinstated
(two years) such debtor would have to pay, in addition to interest due, the
Vidal introduced oral evidence to the effect that he reserved unto himself in penalty of P30,000 this is in addition to the penalty clause of 10 per cent
that agreement the right "to accept or refuse the total payment of the loan of the total amount due inserted in the document of mortgage of January 20,
outstanding . . ., if at the time of such offer of payment he considered it 1943."
advantageous to his interest." This was gist of Vidal's testimony and that of
Lucio M. Tiangco, one of Vidal's former attorneys who, as notary public, had Atty. Ponce Enrile's concept of the agreement, formed after mature and
authenticated the document. Vidal's above testimony was ordered stricken careful reading of it, jibes with the only possible reason for the insertion of the
out as hearsay, for Vidal was blind and, according to him, only had his other penalty provision. There was no reason for the penalty unless it was for
lawyer read the document to him. defendant's paying her debt before the end of the agreed period. It was to
Vidal's interest that the mortgage be not settled in the near future, first,
because his money was earning good interest and was guaranteed by a solid
security, and second, which was more important, he, in all probability, shared
the common belief that Japanese war notes were headed for a crash and

351
that four years thence, judging by the trends of the war, the hostilities would Vidal, however, is entitled to the penalty, a point which the debtor seems to a
be over. grant. The suspension of the running of the interest is premised on the thesis
that the debt was considered paid as of the date the offer to pay the principal
To say, as Vidal says, that the debtor could not pay the mortgage within four was made. It is precisely the mortgagor's contention that he was to pay said
years and, at the same time, that there would be penalty if she paid after that penalty if and when she paid the mortgage before the expiration of the four-
period, would be a contradiction. Moreover, adequate remedy was provided year period provided in the mortgage contract. This penalty was designed to
for failure to pay or after the expiration of the mortgage: increased rate or take the place of the interest which the creditor would be entitled to collect if
interest, foreclosure of the mortgage, and attorney's fees. the duration of the mortgage had not been cut short and from which interest
the debtor has been relieved. "In obligations with a penalty clause the penalty
It is therefore to be concluded that the defendant's offer to pay Vidal in shall substitute indemnity for damages and the payment of interest. . ." (Art.
October, 1943, was in accordance with the parties' contract and terminated 1152, Civil Code of Spain.).
the debtor's obligation to pay interest. The technical defects of the
consignation had to do with the discharge of the mortgage, which is To summarize, the following are our findings and decision:
conceded on all sides to be still in force because of the defects. But the
matter of the suspension of the running of interest on the loan stands of a The contract of sale Exhibit A was valid and enforceable, but the loss of the
different footing and is governed by different principles. These principles checks for P143,150 and P12,932.61 and invalidation of the corresponding
regard reality rather than technicality, substance rather than form. Good faith deposit is to be borne by the buyer. Gregorio Araneta, Inc. the value of these
of the offer or and ability to make good the offer should in simple justice checks as well as the several payments made by Paz Tuason to Gregorio
excuse the debtor from paying interest after the offer was rejected. A debtor Araneta, Inc. shall be deducted from the sum of P190,000 which the buyer
can not be considered delinquent who offered checks backed by sufficient advanced to the seller on the execution of Exhibit 1.
deposit or ready to pay cash if the creditor chose that means of payment.
Technical defects of the offer cannot be adduced to destroy its effects when The buyer shall be entitled to the rents on the land which was the subject of
the objection to accept the payment was based on entirely different grounds. the sale, rents which may have been collected by Paz Tuason after the date
If the creditor had told the debtor that he wanted cash or an ordinary check, of the sale.
which Vidal now seems to think Paz Tuason should have tendered, certainly
Vidal's wishes would have been fulfilled, gladly. Paz Tuason shall pay Jose Vidal the amount of the mortgage and the
stipulated interest up to October 20,1943, plus the penalty of P30,000,
The plain truth was that the mortgagee bent all his efforts to put off the provided that the loans obtained during the Japanese occupation shall be
payment, and thanks to the defects which he now, with obvious reduced according to the Ballantyne scale of payment, and provided that the
inconsistency, points out, the mortgage has not perished with the checks. date basis of the computation as to the penalty is the date of the filing of the
suit against Vidal.
Falling within the reasons for the stoppage of interest are attorney's fees. In
fact there is less merit in the claim for attorney's fees than in the claim for Paz Tuason shall pay the amount that shall have been found due under the
interest; for the creditor it was who by his refusal brought upon himself this contracts of mortgage within 90 days from the time the court's judgment upon
litigation, refusal which, as just shown, resulted greatly to his benefit. the liquidation shall have become final, otherwise the property mortgaged
shall be ordered sold provided by law.

352
Vidal's mortgage is superior to the purchaser's right under Exhibit A, which is
hereby declared subject to said mortgage. Should Gregorio Araneta, Inc. be
forced to pay the mortgage, it will be subrogated to the right of the
mortgagee.
G.R. No. L-15121 August 31, 1962
This case will be remanded to the court of origin with instruction to hold a
rehearing for the purpose of liquidation as herein provided. The court also GREGORIO PALACIO, in his own behalf and in behalf of his minor
shall hear and decide all other controversies relative to the liquidation which child,
may have been overlooked at this decision, in a manner not inconsistent with MARIO PALACIO, plaintiffs-appellants,
the above findings and judgment. vs.
FELY TRANSPORTATION COMPANY, defendant-appellee.
The mortgagor is not entitled to suspension of payment under the debt
moratorium law or orders. Among other reasons: the bulk of the debt was a Antonio A. Saba for plaintiffs-appellants.
pre-war obligation and the moratorium as to such obligations has been Mercado, Ver and Reyes for defendant-appellee.
abrogated unless the debtor has suffered war damages and has filed claim
for them; there is no allegation or proof that she has. In the second place, the
REGALA, J.:
debtor herself caused her creditor to be brought into the case which resulted
in the filing of the cross-claim to foreclose the mortgage. In the third place,
prompt settlement of the mortgage is necessary to the settlement of the This is an appeal by the plaintiffs from the decision of the Court of First
dispute and liquidation between Gregorio Araneta, Inc. and Paz Tuason. If for Instance of Manila which dismissed their complaint.
no other reason, Paz Tuason would do well to forego the benefits of the
moratorium law. Originally taken to the Court of Appeals, this appeal was certified to this
Court on the ground that it raises purely questions of law.
There shall be no special judgments as to costs of either instance.
The parties in this case adopt the following findings of fact of the lower court:

In their complaint filed with this Court on May 15, 1954, plaintiffs
allege, among other things, "that about December, 1952, the
defendant company hired Alfredo Carillo as driver of AC-787 (687) (a
registration for 1952) owned and operated by the said defendant
company; that on December 24, 1952, at about 11:30 a.m., while the
driver Alfonso (Alfredo) Carillo was driving AC-687 at Halcon Street,
Quezon City, wilfully, unlawfully and feloniously and in a negligent,
reckless and imprudent manner, run over a child Mario Palacio of the
herein plaintiff Gregorio Palacio; that on account of the aforesaid
injuries, Mario Palacio suffered a simple fracture of the right tenor
(sic), complete third, thereby hospitalizing him at the Philippine

353
Orthopedic Hospital from December 24, 1952, up to January 8, No. Q-1084 of the Court of First Instance of Quezon City, in which
1953, and continued to be treated for a period of five months both the civil and criminal cases were simultaneously tried by
thereafter; that the plaintiff Gregorio Palacio herein is a welder by agreement of the parties in said case. In the Counterclaim of the
occupation and owner of a small welding shop and because of the Answer, defendant alleges that in view of the filing of this complaint
injuries of his child he has abandoned his shop where he derives which is a clearly unfounded civil action merely to harass the
income of P10.00 a day for the support of his big family; that during defendant, it was compelled to engage the services of a lawyer for
the period that the plaintiff's (Gregorio Palacio's) child was in the an agreed amount of P500.00.
hospital and who said child was under treatment for five months in
order to meet the needs of his big family, he was forced to sell one During the trial, plaintiffs presented the transcript of the stenographic
air compressor (heavy duty) and one heavy duty electric drill, for a notes of the trial of the case of "People of the Philippines vs. Alfredo
sacrifice sale of P150.00 which could easily sell at P350.00; that as a Carillo, Criminal Case No. Q-1084," in the Court of First Instance of
consequence of the negligent and reckless act of the driver Alfredo Rizal, Quezon City (Branch IV), as Exhibit "A".1wph1.t
Carillo of the herein defendant company, the herein plaintiffs were
forced to litigate this case in Court for an agreed amount of P300.00 It appears from Exhibit "A" that Gregorio Palacio, one of the herein
for attorney's fee; that the herein plaintiffs have now incurred the plaintiffs, testified that Mario Palacio, the other plaintiff, is his son;
amount of P500.00 actual expenses for transportation, that as a result of the reckless driving of accused Alfredo Carillo, his
representation and similar expenses for gathering evidence and child Mario was injured and hospitalized from December 24, 1952, to
witnesses; and that because of the nature of the injuries of plaintiff January 8, 1953; that during all the time that his child was in the
Mario Palacio and the fear that the child might become a useless hospital, he watched him during the night and his wife during the
invalid, the herein plaintiff Gregorio Palacio has suffered moral day; that during that period of time he could not work as he slept
damages which could be conservatively estimated at P1,200.00. during the day; that before his child was injured, he used to earn
P10.00 a day on ordinary days and on Sundays from P20 to P50 a
On May 23, 1956, defendant Fely Transportation Co., filed a Motion Sunday; that to meet his expenses he had to sell his compressor and
to Dismiss on the grounds (1) that there is no cause of action against electric drill for P150 only; and that they could have been sold for
the defendant company, and (2) that the cause of action is barred by P300 at the lowest price.
prior judgment..
During the trial of the criminal case against the driver of the jeep in
In its Order, dated June 8, 1956, this Court deferred the the Court of First Instance of Quezon City (Criminal Case No. Q-
determination of the grounds alleged in the Motion to Dismiss until 1084) an attempt was unsuccessfully made by the prosecution to
the trial of this case. prove moral damages allegedly suffered by herein plaintiff Gregorio
Palacio. Likewise an attempt was made in vain by the private
On June 20, 1956, defendant filed its answer. By way of affirmative prosecutor in that case to prove the agreed attorney's fees between
defenses, it alleges (1) that complaint states no cause of action him and plaintiff Gregorio Palacio and the expenses allegedly
against defendant, and (2) that the sale and transfer of the jeep AC- incurred by the herein plaintiffs in connection with that case. During
687 by Isabelo Calingasan to the Fely Transportation was made on the trial of this case, plaintiff Gregorio Palacio testified substantially
December 24, 1955, long after the driver Alfredo Carillo of said jeep to the same facts.
had been convicted and had served his sentence in Criminal Case

354
The Court of First Instance of Quezon City in its decision in Criminal THE LOWER COURT ERRED IN HOLDING THAT THE CAUSE OF
Case No. 1084 (Exhibit "2") determined and thoroughly discussed ACTION OF THE PLAINTIFFS-APPELLANTS IS BARRED BY
the civil liability of the accused in that case. The dispositive part PRIOR JUDGMENT.
thereof reads as follows:
With respect to the first and second assignments of errors, plaintiffs contend
IN VIEW OF THE FOREGOING, the Court finds the accused Alfredo that the defendant corporate should be made subsidiarily liable for damages
Carillo y Damaso guilty beyond reasonable doubt of the crime in the criminal case because the sale to it of the jeep in question, after the
charged in the information and he is hereby sentenced to suffer conviction of Alfred Carillo in Criminal Case No. Q-1084 of the Court of First
imprisonment for a period of Two Months & One Day of Arresto Instance of Quezon City was merely an attempt on the part of Isabelo
Mayor; to indemnify the offended party, by way of consequential Calingasan its president and general manager, to evade his subsidiary civil
damages, in the sum of P500.00 which the Court deems reasonable; liability.
with subsidiary imprisonment in case of insolvency but not to exceed
/3 of the principal penalty imposed; and to pay the costs. The Court agrees with this contention of the plaintiffs. Isabelo Calingasan
and defendant Fely Transportation may be regarded as one and the same
On the basis of these facts, the lower court held action is barred by the person. It is evident that Isabelo Calingasan's main purpose in forming the
judgment in the criminal case and, that under Article 103 of the Revised corporation was to evade his subsidiary civil liability1 resulting from the
Penal Code, the person subsidiarily liable to pay damages is Isabel conviction of his driver, Alfredo Carillo. This conclusion is borne out by the
Calingasan, the employer, and not the defendant corporation. fact that the incorporators of the Fely Transportation are Isabelo Calingasan,
his wife, his son, Dr. Calingasan, and his two daughters. We believe that this
Against that decision the plaintiffs appealed, contending that: is one case where the defendant corporation should not be heard to say that
it has a personality separate and distinct from its members when to allow it to
THE LOWER COURT ERRED IN NOT SUSTAINING THAT THE do so would be to sanction the use of the fiction of corporate entity as a
DEFENDANT-APPELLEE IS SUBSIDIARILY LIABLE FOR shield to further an end subversive of justice. (La Campana Coffee Factory,
DAMAGES AS A RESULT OF CRIMINAL CASE NO. Q-1084 OF et al. v. Kaisahan ng mga Manggagawa, etc., et al., G.R. No. L-5677, May
THE COURT OF FIRST INSTANCE OF QUEZON CITY FOR THE 25, 1953) Furthermore, the failure of the defendant corporation to prove that
REASON THAT THE INCORPORATORS OF THE FELY it has other property than the jeep (AC-687) strengthens the conviction that
TRANSPORTATION COMPANY, THE DEFENDANT-APPELLEE its formation was for the purpose above indicated.
HEREIN, ARE ISABELO CALINGASAN HIMSELF, HIS SON AND
DAUGHTERS; And while it is true that Isabelo Calingasan is not a party in this case, yet, is
held in the case of Alonso v. Villamor, 16 Phil. 315, this Court can substitute
THE LOWER COURT ERRED IN NOT CONSIDERING THAT THE him in place of the defendant corporation as to the real party in interest. This
INTENTION OF ISABELO CALINGASAN IN INCORPORATING THE is so in order to avoid multiplicity of suits and thereby save the parties
FELY TRANSPORTATION COMPANY, THE DEFENDANT- unnecessary expenses and delay. (Sec. 2, Rule 17, Rules of Court; Cuyugan
APPELLEE HEREIN, WAS TO EVADE HIS CIVIL LIABILITY AS A v. Dizon. 79 Phil. 80; Quison v. Salud, 12 Phil. 109.)
RESULT OF THE CONVICTION OF HIS DRIVER OF VEHICLE AC-
687 THEN OWNED BY HIM:

355
Accordingly, defendants Fely Transportation and Isabelo Calingasan should
be held subsidiarily liable for P500.00 which Alfredo Carillo was ordered to
pay in the criminal case and which amount he could not pay on account of
insolvency.

We also sustain plaintiffs' third assignment of error and hold that the present
action is not barred by the judgment of the Court of First Instance of Quezon
City in the criminal case. While there seems to be some confusion on part of
the plaintiffs as to the theory on which the is based whether ex-delito or
quasi ex-delito (culpa aquiliana) We are convinced, from the discussion
prayer in the brief on appeal, that they are insisting the subsidiary civil liability
of the defendant. As a matter of fact, the record shows that plaintiffs merely
presented the transcript of the stenographic notes (Exhibit "A") taken at the
hearing of the criminal case, which Gregorio Palacio corroborated, in support
of their claim for damages. This rules out the defense of res judicata,
because such liability proceeds precisely from the judgment in the criminal
action, where the accused was found guilty and ordered to pay an indemnity
in the sum P500.00.

WHEREFORE, the decision of the lower court is hereby reversed and


defendants Fely Transportation and Isabelo Calingasan are ordered to pay,
jointly and severally, the plaintiffs the amount of P500.00 and the costs.

Bengzon, C.J., Padilla, Bautista Angelo, Labrador, Concepcion, Barrera,


Paredes, Dizon and Makalintal, concur.
Reyes, J.B.L., J., took no part.

356
357
Inc. and Alberto Onstott jointly and severally, to refund to private respondent,
Nazario Dumpit, the amount of P13,722.50 with 12% interest per annum, as
resolved by the National Housing Authority in its Resolution of July 10, 1979
in Case No. 2167, as well as the Resolution of October 28, 1980 denying
petitioners' Motion for Reconsideration of said Resolution of May 2, 1980, are
being assailed in this petition.

On March 28, 1965, petitioner Palay, Inc., through its President, Albert
Onstott executed in favor of private respondent, Nazario Dumpit, a Contract
to Sell a parcel of Land (Lot No. 8, Block IV) of the Crestview Heights
Subdivision in Antipolo, Rizal, with an area of 1,165 square meters, - covered
by TCT No. 90454, and owned by said corporation. The sale price was
P23,300.00 with 9% interest per annum, payable with a downpayment of
P4,660.00 and monthly installments of P246.42 until fully paid. Paragraph 6
of the contract provided for automatic extrajudicial rescission upon default in
payment of any monthly installment after the lapse of 90 days from the
expiration of the grace period of one month, without need of notice and with
forfeiture of all installments paid.

G.R. No. L-56076 September 21, 1983 Respondent Dumpit paid the downpayment and several installments
amounting to P13,722.50. The last payment was made on December 5, 1967
PALAY, INC. and ALBERT ONSTOTT, petitioner, for installments up to September 1967.
vs.
JACOBO C. CLAVE, Presidential Executive Assistant NATIONAL On May 10, 1973, or almost six (6) years later, private respondent wrote
HOUSING AUTHORITY and NAZARIO DUMPIT respondents. petitioner offering to update all his overdue accounts with interest, and
seeking its written consent to the assignment of his rights to a certain
Santos, Calcetas-Santos & Geronimo Law Office for petitioner. Lourdes Dizon. He followed this up with another letter dated June 20, 1973
reiterating the same request. Replying petitioners informed respondent that
his Contract to Sell had long been rescinded pursuant to paragraph 6 of the
Wilfredo E. Dizon for private respondent.
contract, and that the lot had already been resold.

Questioning the validity of the rescission of the contract, respondent filed a


letter complaint with the National Housing Authority (NHA) for reconveyance
MELENCIO-HERRERA, J.: with an altenative prayer for refund (Case No. 2167). In a Resolution, dated
July 10, 1979, the NHA, finding the rescission void in the absence of either
The Resolution, dated May 2, 1980, issued by Presidential Executive judicial or notarial demand, ordered Palay, Inc. and Alberto Onstott in his
Assistant Jacobo Clave in O.P. Case No. 1459, directing petitioners Palay,

358
capacity as President of the corporation, jointly and severally, to refund respondent Nazario M. Dumpit thereby denying substantial
immediately to Nazario Dumpit the amount of P13,722.50 with 12% interest justice to the petitioners, particularly petitioner Onstott
from the filing of the complaint on November 8, 1974. Petitioners' Motion for
Reconsideration of said Resolution was denied by the NHA in its Order dated We issued a Temporary Restraining Order on Feb 11, 1981 enjoining the
October 23, 1979. 1 enforcement of the questioned Resolutions and of the Writ of Execution that
had been issued on December 2, 1980. On October 28, 1981, we dismissed
On appeal to the Office of the President, upon the allegation that the NHA the petition but upon petitioners' motion, reconsidered the dismissal and gave
Resolution was contrary to law (O.P. Case No. 1459), respondent due course to the petition on March 15, 1982.
Presidential Executive Assistant, on May 2, 1980, affirmed the Resolution of
the NHA. Reconsideration sought by petitioners was denied for lack of merit. On the first issue, petitioners maintain that it was justified in cancelling the
Thus, the present petition wherein the following issues are raised: contract to sell without prior notice or demand upon respondent in view of
paragraph 6 thereof which provides-
I
6. That in case the BUYER falls to satisfy any monthly
Whether notice or demand is not mandatory under the installment or any other payments herein agreed upon, the
circumstances and, therefore, may be dispensed with by BUYER shall be granted a month of grace within which to
stipulation in a contract to sell. make the payment of the t in arrears together with the one
corresponding to the said month of grace. -It shall be
II understood, however, that should the month of grace herein
granted to the BUYER expire, without the payment &
Whether petitioners may be held liable for the refund of the corresponding to both months having been satisfied, an
installment payments made by respondent Nazario M. interest of ten (10%) per cent per annum shall be charged on
Dumpit. the amounts the BUYER should have paid; it is understood
further, that should a period of NINETY (90) DAYS elapse to
begin from the expiration of the month of grace hereinbefore
III
mentioned, and the BUYER shall not have paid all the
amounts that the BUYER should have paid with the
Whether the doctrine of piercing the veil of corporate fiction corresponding interest up to the date, the SELLER shall
has application to the case at bar. have the right to declare this contract cancelled and of no
effect without notice, and as a consequence thereof, the
IV SELLER may dispose of the lot/lots covered by this Contract
in favor of other persons, as if this contract had never been
Whether respondent Presidential Executive Assistant entered into. In case of such cancellation of this Contract, all
committed grave abuse of discretion in upholding the the amounts which may have been paid by the BUYER in
decision of respondent NHA holding petitioners solidarily accordance with the agreement, together with all the
liable for the refund of the installment payments made by improvements made on the premises, shall be considered as
rents paid for the use and occupation of the above

359
mentioned premises and for liquidated damages suffered by judgment before taking extrajudicial steps to protect its
virtue of the failure of the BUYER to fulfill his part of this interest. Otherwise, the party injured by the other's breach
agreement : and the BUYER hereby renounces his right to will have to passively sit and watch its damages accumulate
demand or reclaim the return of the same and further during the pendency of the suit until the final judgment of
obligates peacefully to vacate the premises and deliver the rescission is rendered when the law itself requires that he
same to the SELLER. should exercise due diligence to minimize its own damages
(Civil Code, Article 2203).
Well settled is the rule, as held in previous jurisprudence, 2 that judicial action
for the rescission of a contract is not necessary where the contract provides We see no conflict between this ruling and the previous
that it may be revoked and cancelled for violation of any of its terms and jurisprudence of this Court invoked by respondent declaring
conditions. However, even in the cited cases, there was at least a written that judicial action is necessary for the resolution of a
notice sent to the defaulter informing him of the rescission. As stressed reciprocal obligation (Ocejo Perez & Co., vs. International
in University of the Philippines vs. Walfrido de los Angeles 3 the act of a party Banking Corp., 37 Phil. 631; Republic vs. Hospital de San
in treating a contract as cancelled should be made known to the other. We Juan De Dios, et al., 84 Phil 820) since in every case where
quote the pertinent excerpt: the extrajudicial resolution is contested only the final award
of the court of competent jurisdiction can conclusively settle
Of course, it must be understood that the act of a party in whether the resolution was proper or not. It is in this sense
treating a contract as cancelled or resolved in account of that judicial action win be necessary, as without it, the
infractions by the other contracting party must be made extrajudicial resolution will remain contestable and subject to
known to the other and is always provisional being ever judicial invalidation unless attack thereon should become
subject to scrutiny and review by the proper court. If the barred by acquiescense, estoppel or prescription.
other party denies that rescission is justified it is free to
resort to judicial action in its own behalf, and bring the matter Fears have been expressed that a stipulation providing for a
to court. Then, should the court, after due hearing, decide unilateral rescission in case of breach of contract may render
that the resolution of the contract was not warranted, the nugatory the general rule requiring judicial action (v.
responsible party will be sentenced to damages; in the Footnote, Padilla Civil Law, Civil Code Anno., 1967 ed. Vol.
contrary case, the resolution will be affirmed, and the IV, page 140) but, as already observed, in case of abuse or
consequent indemnity awarded to the party prejudiced. error by the rescinder the other party is not barred from
questioning in court such abuse or error, the practical effect
In other words, the party who deems the contract violated of the stipulation being merely to transfer to the defaulter the
may consider it resolved or rescinded, and act accordingly, initiative of instituting suit, instead of the
without previous court action, but it proceeds at its own rescinder (Emphasis supplied).
risk. For it is only the final judgment of the corresponding
court that will conclusively and finally settle whether the Of similar import is the ruling in Nera vs. Vacante 4, reading:
action taken was or was not correct in law. But the law
definitely does not require that the contracting party who
believes itself injured must first file suit and wait for a

360
A stipulation entitling one party to take possession of the Sec. 3(b) ... the actual cancellation of the contract shall take
land and building if the other party violates the contract does place after thirty days from receipt by the buyer of the notice
not ex propio vigore confer upon the former the right to take of cancellation or the demand for rescission of the contract
possession thereof if objected to without judicial intervention by a notarial act and upon full payment of the cash surrender
and determination. value to the buyer. (Emphasis supplied).

This was reiterated in Zulueta vs. Mariano 5 where we held that extrajudicial The contention that private respondent had waived his right to be notified
rescission has legal effect where the other party does not oppose it. 6 Where under paragraph 6 of the contract is neither meritorious because it was a
it is objected to, a judicial determination of the issue is still necessary. contract of adhesion, a standard form of petitioner corporation, and private
respondent had no freedom to stipulate. A waiver must be certain and
In other words, resolution of reciprocal contracts may be made extrajudicially unequivocal, and intelligently made; such waiver follows only where liberty of
unless successfully impugned in Court. If the debtor impugns the declaration, choice has been fully accorded. 9 Moreover, it is a matter of public policy to
it shall be subject to judicial determination. 7 protect buyers of real estate on installment payments against onerous and
oppressive conditions. Waiver of notice is one such onerous and oppressive
In this case, private respondent has denied that rescission is justified and condition to buyers of real estate on installment payments.
has resorted to judicial action. It is now for the Court to determine whether
resolution of the contract by petitioners was warranted. Regarding the second issue on refund of the installment
payments made by private respondent. Article 1385 of the
We hold that resolution by petitioners of the contract was ineffective and Civil Code provides:
inoperative against private respondent for lack of notice of resolution, as held
in the U.P. vs. Angeles case, supra ART. 1385. Rescission creates the obligation to return the
things which were the object of the contract, together with
Petitioner relies on Torralba vs. De los Angeles 8 where it was held that "there their fruits, and the price with its interest; consequently, it can
was no contract to rescind in court because from the moment the petitioner be carried out only when he who demands rescission can
defaulted in the timely payment of the installments, the contract between the return whatever he may be obliged to restore.
parties was deemed ipso facto rescinded." However, it should be noted that
even in that case notice in writing was made to the vendee of the Neither sham rescission take place when the things which
cancellation and annulment of the contract although the contract entitled the are the object of the contract are legally in the possession of
seller to immediate repossessing of the land upon default by the buyer. third persons who did not act in bad faith.

The indispensability of notice of cancellation to the buyer was to be later In this case, indemnity for damages may be demanded from
underscored in Republic Act No. 6551 entitled "An Act to Provide Protection the person causing the loss.
to Buyers of Real Estate on Installment Payments." which took effect on
September 14, 1972, when it specifically provided: As a consequence of the resolution by petitioners, rights to the lot should be
restored to private respondent or the same should be replaced by another
acceptable lot. However, considering that the property had already been sold

361
to a third person and there is no evidence on record that other lots are still WHEREFORE, the questioned Resolution of respondent public official, dated
available, private respondent is entitled to the refund of installments paid plus May 2, 1980, is hereby modified. Petitioner Palay, Inc. is directed to refund to
interest at the legal rate of 12% computed from the date of the institution of respondent Nazario M. Dumpit the amount of P13,722.50, with interest at
the action. 10 It would be most inequitable if petitioners were to be allowed to twelve (12%) percent per annum from November 8, 1974, the date of the
retain private respondent's payments and at the same time appropriate the filing of the Complaint. The temporary Restraining Order heretofore issued is
proceeds of the second sale to another. hereby lifted.

We come now to the third and fourth issues regarding the personal liability of No costs.
petitioner Onstott who was made jointly and severally liable with petitioner
corporation for refund to private respondent of the total amount the latter had SO ORDERED.
paid to petitioner company. It is basic that a corporation is invested by law
with a personality separate and distinct from those of the persons composing
it as wen as from that of any other legal entity to which it may be
related. 11 As a general rule, a corporation may not be made to answer for
acts or liabilities of its stockholders or those of the legal entities to which it
may be connected and vice versa. However, the veil of corporate fiction may
be pierced when it is used as a shield to further an end subversive of
justice 12 ; or for purposes that could not have been intended by the law that
created it 13 ; or to defeat public convenience, justify wrong, protect fraud, or
defend crime. 14 ; or to perpetuate fraud or confuse legitimate issues 15 ; or to
circumvent the law or perpetuate deception 16 ; or as an alter ego, adjunct or
business conduit for the sole benefit of the stockholders. 17

We find no badges of fraud on petitioners' part. They had literally relied,


albeit mistakenly, on paragraph 6 (supra) of its contract with private
respondent when it rescinded the contract to sell extrajudicially and had sold
it to a third person.

In this case, petitioner Onstott was made liable because he was then the
President of the corporation and he a to be the controlling stockholder. No
sufficient proof exists on record that said petitioner used the corporation to
defraud private respondent. He cannot, therefore, be made personally liable
just because he "appears to be the controlling stockholder". Mere ownership
by a single stockholder or by another corporation is not of itself sufficient
ground for disregarding the separate corporate personality. 18 In this respect
then, a modification of the Resolution under review is called for.

362
363
provisions of Batas Pambansa Blg. 130. Complainants demanded
reinstatement with full backwages, living allowance, 13th month pay and
other benefits under existing laws and/or separation pay.

On October 21, 1987, PIF, through its General Manager, was notified about
the complaint and summons for the hearing set for November 6, 1987. The
hearing was re-set for November 27, 1987 for failure of respondents to
appear. On November 30, 1987 respondents (petitioners herein) moved for
the cancellation of the hearing scheduled on November 6, 1987 so that they
could engage a counsel to properly represent them preferably on November
17, 1987.
G.R. No. 89879 April 20, 1990
On December 10, 1987 both parties were directed to submit their respective
JAIME PABALAN AND EDUARDO LAGDAMEO, petitioners, position papers within ten (10) days. By mutual agreement the hearing was
vs. re-set on December 21, 1987 but on said date respondents and/or counsel
NATIONAL LABOR RELATIONS COMMISSION, LABOR ARBITER failed to appear. The hearing was re-set on January 14, 1988 on which date
AMBROSIO B. SISON, ELIZABETH RODEROS, ET AL., and THE respondents were given a deadline to submit their position paper.
SHERIFF OF THE NATIONAL LABOR RELATIONS
COMMISSION, respondents.
On January 4, 1988 complainants filed their position paper. On January 14,
1988 counsel for respondents moved that he be given until January 22, 1988
Sofronio A. Larcia and Conrado Abriol Padilla for petitioners. to file their position paper. The labor arbiter granted the motion. The PIF filed
Apolinario N. Lomabao, Jr. for private respondents. its position paper on January 22, 1988. The heating for February 17, 1988
was re-set to March 9, 1988 and on March 29, 1988 on which dates
respondents failed to appear.

On May 5, 1988, with leave of the labor arbiter, complainants filed their
GANCAYCO, J.: supplemental position paper impleading the petitioners as officers of the PIF
in the complaint for their illegal transfer to a new firm.
Once again the parameters of the liability of the officers of a corporation as to
unpaid wages and other claims of the employees of a corporation which has On July 13, 1988 a decision was rendered by the labor arbiter the dispositive
a separate and distinct personality are brought to fore in this case. part of which reads as follows:

On October 20, 1987, eighty-four (84) workers of the Philippine Inter- IN VIEW OF THE FOREGOING CONSIDERATION, respondent
Fashion, Inc. (PIF) filed a complaint against the latter for illegal transfer Philippine Inter-Fashion and its officers Mr. Jaime Pabalan and Mr.
simultaneous with illegal dismissal without justifiable cause and in violation of Eduardo Lagdameo are hereby ordered to:
the provision of the Labor Code on security of tenure as well as the

364
1. reinstate the sixty two (62) complainants to their former or THE ARBITER AND THE NLRC COMMITTED A GRAVE ABUSE OF
equivalent position without loss of seniority rights and privileges; DISCRETION IN ADJUDGING PETITIONERS HEREIN AS JOINTLY
AND SEVERALLY LIABLE WITH PHILIPPINE INTER-FASHION,
2. to pay, jointly and severally, their backwages and other benefits INC. TO PAY THE JUDGMENT DEBT.
from the time they were dismissed up to the time they are actually
reinstated, the computation to be based from the latest minimum On September 25, 1989 this Court dismissed the petition for insufficiency in
wage law at the time of their dismissal. (See attached Annex "A" of form and substance, having failed to comply with the Rules of Court and
complainants' position paper.) Administrative Circular No. 1-88 requiting the verification of the petition. A
motion for reconsideration filed by the petitioners of the said resolution was
SO ORDERED. 1 denied on October 16, 1989 for failure to raise any substantial arguments to
warrant a modification thereof. However, acting on an urgent motion to
Not satisfied therewith petitioners filed a motion for reconsideration in the include the motion for reconsideration of the resolution of September 25,
First Division of the public respondent, National Labor Relations Commission 1989 in the court's calendar which the Court granted, on November 30, 1989
(NLRC), which nevertheless, affirmed the appealed decision and dismissed the Court resolved to set aside said resolutions of September 25, 1989 and
the appeal for lack of merit in a resolution dated June 30, 1989. Petitioners October 16, 1989, and to require respondents to comment thereon within ten
were ordered to pay the appeal fee in accordance with law. (10) days from notice thereof. A temporary restraining order was issued
enjoining respondents from enforcing or implementing the questioned
decision of the labor arbiter affirmed by the NLRC upon a bond to be filed by
Hence the herein petition for certiorari with prayer for the issuance of a
petitioners in the amount of P100,000.00. However, on February 7, 1990 for
temporary restraining order wherein the petitioners raised the following
failure of petitioner to file the required bond despite extensions of time
issues:
granted them, the Court resolved to lift the temporary restraining order issued
on November 13, 1989.
A
Now to the merit of the petition.
THE ARBITER AND THE NLRC DID NOT ACQUIRE JURISDICTION
OVER THE PERSONS OF THE PETITIONERS AND, THEREFORE,
Petitioners do not question the merits of the decision insofar as PIF is
THE DECISION AND THE RESOLUTION, UNDER DISPUTE, ARE
concerned in this proceeding.1wphi1 The first two issues they raised are to
NULL AND VOID.
the effect that the public respondents never acquired jurisdiction over them
as they have not been served with summons and thus they were deprived
B due process.

THE DECISION AND THE NLRC RESOLUTION SUFFER FROM A The Court finds these grounds to be devoid of merit. As the record shows
LEGAL AND CONSTITUTIONAL INFIRMITY BECAUSE THEY while originally it was PIF which was impleaded as respondent before the
SANCTION A DEPRIVATION OF PETITIONERS' PROPERTIES labor arbiter, petitioners also appeared in their behalf through counsel.
WITHOUT DUE PROCESS OF LAW. Thereafter when the supplemental position paper was filed by complainants,
petitioners were impleaded as respondents to which they filed an opposition
C

365
inasmuch as they filed their own supplemental position papers. They were When the notion of legal entity is used as a means to perpetrate
therefore properly served with summons and they were not deprived of due fraud or an illegal act or as a vehicle for the evasion of an existing
process. obligation, the circumvention of statutes, and or (to) confuse
legitimate issues the veil which protects the corporation will be
Petitioners contend however that under the circumstances of the case as lifted. 5
officers of the corporation PIF they could not be jointly and severally held
liable with the corporation for its liability in this case. In this particular case complainants did not allege or show that petitioners, as
officers of the corporation deliberately and maliciously designed to evade the
The settled rule is that the corporation is vested by law with a personality financial obligation of the corporation to its employees, or used the transfer of
separate and distinct from the persons composing it, including its officers as the employees as a means to perpetrate an illegal act or as a vehicle for the
well as from that of any other legal entity to which it may be related. Thus, a evasion of existing obligations, the circumvention of statutes, or to confuse
company manager acting in good faith within the scope of his authority in the legitimate issues.
terminating the services of certain employees cannot be held personally
liable for damages. 2 Mere ownership by a single stockholder or by another Indeed, in the questioned resolution of the NLRC dated June 30, 1989 there
corporation of all or nearly all capital stocks of the corporation is not by itself is no finding as to why petitioners were being held jointly and severally liable
sufficient ground for disregarding the separate corporate personality. 3 for the liability and obligation of the corporation except as to invocation of the
ruling of this Court in A.C. Ransom Labor Union-CCLU vs. NLRC 6 in that the
As a general rule, officers of a corporation are not personally liable for their liability in the cases of illegal termination of employees extends not only to
official acts unless it is shown that they have exceeded their the corporation as a corporate entity but also to its responsible officers acting
authority. 4 However, the legal fiction that a corporation has a personality in the interest of the corporation or employer.
separate and distinct from stockholders and members may be disregarded
as follows: It must be noted, however, that A.C. Ransom Labor Union-CCLU
vs. NLRC the corporation was a family corporation and that during the strike
This finding does not ignore the legal fiction that a corporation has a the members of the family organized another corporation which was the
personality separate and distinct from its stockholders and members, Rosario Industrial Corporation to which all the assets of the A.C. Ransom
for, as this Court had held "where the incorporators and directors Corporation were transferred to continue its business which acts of such
belong to a single family, the corporation and its members can be officers and agents of A.C. Ransom Corporation were intended to avoid
considered as one in order to avoid its being used as an instrument payment of its obligations to its employees. In such case this Court
to commit injustice," or to further an end subversive of justice. In the considered the president of the corporation to be personally liable together
case of Claparols vs. CIR involving almost similar facts as in this with the corporation for the satisfaction of the claim of the employees. 7
case, it was also held that the shield of corporate fiction should be
pierced when it is deliberately and maliciously designed to evade Not one of the above circumstances has been shown to be present. Hence
financial obligations to employees. petitioners can not be held jointly and severally liable with the PIF corporation
under the questioned decision and resolution of the public respondent.
To the same effect . . . (are) this Court's rulings in still other cases:

366
WHEREFORE, the petition is GRANTED and the questioned resolution of
the public respondent dated June 30, 1989 is hereby modified by relieving
petitioners of any liability as officers of the PIF and holding that the liability
shall be solely that of Philippine Inter-Fashion, Inc. No costs.

SO ORDERED.

Narvasa, Cruz, Grio-Aquino and Medialdea, JJ., concur.

367
368
the judgment. Private respondent moved for the issuance of an alias writ
against the officers of Philsa. This motion was opposed by the officers, led by
petitioner, the president and general manager of the corporation.

On February 12, 1988, the POEA issued a resolution, the dispositive portion
of which read:
G.R. No. 85416 July 24, 1990
WHEREFORE, premises considered, let an alias writ of
FRANCISCO V. DEL ROSARIO, petitioner, Execution be issued and the handling sheriff is ordered to
vs. execute against the properties of Mr. Francisco V. del
NATIONAL LABOR RELATIONS COMMISSION and LEONARDO V. -Rosario and if insufficient, against the cash and/or surety
ATIENZA, respondents. bond of Bonding Company concerned for the full satisfaction
of the judgment awarded.
Jardeleza, Sobrevias, Diaz, Hayudini & Bodegon Law Offices for petitioner.
Petitioner appealed to the NLRC. On September 23, 1988, the NLRC
dismissed the appeal. On October 21, 1988, petitioner's motion for
Lourdes T. Pagayatan for private respondent.
reconsideration was denied.

Thus, this petition was filed on October 28, 1988, alleging that the NLRC
gravely abused its discretion. On November 10, 1988 the Court issued a
CORTES, J.: temporary restraining order enjoining the enforcement of the NLRC's
decision dated September 23, 1988 and resolution dated October 21, 1988.
In POEA Case No. 85-06-0394, the Philippine Overseas Employment The petition was given due course on June 14, 1989.
Administration (POEA) promulgated a decision on February 4, 1986
dismissing the complaint for money claims for lack of merit. The decision was After considering the undisputed facts and the arguments raised in the
appealed to the National Labor Relations Commission (NLRC), which on pleadings, the Court finds grave abuse of discretion on the part of the NLRC.
April 30, 1987 reversed the POEA decision and ordered Philsa Construction
and Trading Co., Inc. (the recruiter) and Arieb Enterprises (the foreign
The action of the NLRC affirming the issuance of an alias writ of execution
employer) to jointly and severally pay private respondent the peso equivalent
against petitioner, on the theory that the corporate personality of Philsa
of $16,039.00, as salary differentials, and $2,420.03, as vacation leave
should be disregarded, was founded primarily on the following findings of the
benefits. The case was elevated to the Supreme Court, but the petition was
POEA
dismissed on August 31, 1987 and entry of judgment was made on
September 24, 1987.
xxx xxx xxx
A writ of execution was issued by the POEA but it was returned unsatisfied
as Philsa was no longer operating and was financially incapable of satisfying 6. Per the certification issued by the Licensing Division of
this Office, it appears that Philsa Construction & Trading Co.,

369
Inc., with office address at 126 Pioneer St., Mandaluyong, successor-in-interest shall be considered as one and the liability of the
Metro Manila, represented by Mr. Francisco V. del Rosario, former shall attach to the latter [Koppel v. Yatco, supra; Liddell & Co. v.
President and General Manager, was formerly a registered Collector of Internal Revenue, G.R. No. L-9687, June 30, 1961, 2 SCRA
construction contractor whose authority was originally issued 632].
on July 21, 1978 but was already delisted from the list of
agencies/entities on August 15, 1986 for inactivity; But for the separate juridical personality of a corporation to be disregarded,
the wrongdoing must be clearly and convincingly established. It cannot be
7. Per another certification issued by the Licensing Division presumed.
of this Office, it also appears that another corporation, Philsa
International Placement & Services Corp., composed of In this regard we find the NLRC's decision wanting. The conclusion that
practically the same set of incorporators/stockholders, was Philsa allowed its license to expire so as to evade payment of private
registered as a licensed private employment agency whose respondent's claim is not supported by the facts. Philsa's corporate
license was issued on November 5, 1981, represented by personality therefore remains inviolable.
the same Mr. Francisco V. del Rosario as its President/
General Manager. Consider the following undisputed facts:

and an application of the ruling of the Court in A.C. Ransom Labor Union- (1) Private respondent filed his complaint with the POEA on
CCLU v. NLRC, G.R. No. 69494, June 10, 1986, 142 SCRA 269. June 4, 1985;

However, we find that the NLRC's reliance on the findings of the POEA and (2) The last renewal of Philsa's license expired on October
the ruling in A. C. Ransom is totally misplaced. 12, 1985;

1. Under the law a corporation is bestowed juridical personality, separate and (3) The POEA dismissed private respondent's complaint on
distinct from its stockholders [Civil Code, Art. 44; Corporation Code, sec. 2]. February 4, 1986;
But when the juridical personality of the corporation is used to defeat public
convenience, justify wrong, protect fraud or defend crime, the corporation
(4) Philsa was delisted for inactivity on August 15, 1986; *
shall be considered as a mere association of persons [Koppel (Phil.), Inc. v.
Yatco, 77 Phil. 496 (1946), citing 1 Fletcher, Cyclopedia of Corporations,
135-136; see also Palay, Inc. v. Clave, G.R. No. 56076, September 21, 1983, (5) The dismissal of the complaint was appealed to the
124 SCRA 638], and its responsible officers and/or stockholders shall be held NLRC and it was only on April 30, 1987 that the judgment
individually liable [Namarco v. Associated Finance Co., Inc., G.R. No. L- awarding differentials and benefits to private respondent was
20886, April 27, 1967, 19 SCRA 962]. For the same reasons, a corporation rendered.
shall be liable for the obligations of a stockholder [Palacio v. Fely
Transportation Company, G.R. No. L-15121, August 31, 1962, 5 SCRA 1011; Thus, at the time Philsa allowed its license to lapse in 1985 and even at the
Emilio Cano Enterprises, Inc. v. Court of Industrial Relations, G.R. No. L- time it was delisted in 1986, there was yet no judgment in favor of private
20502, February 26, 1965, 13 SCRA 290], or a corporation and its respondent. An intent to evade payment of his claims cannot therefore be
implied from the expiration of Philsa's license and its delisting.

370
Neither will the organization of Philsa International Placement and Services back wages to the 22 strikers, organized ROSARIO to
Corp. and its registration with the POEA as a private employment agency replace RANSOM, with the latter to be eventually phased out
imply fraud since it was organized and registered in 1981, several years if the 22 strikers win their case. RANSOM actually ceased
before private respondent filed his complaint with the POEA in 1985. The operations on May 1, 1973, after the December 19, 1972
creation of the second corporation could not therefore have been in Decision of the Court of Industrial Relations was
anticipation of private respondent's money claims and the consequent promulgated against RANSOM. [At p. 274.]
adverse judgment against Philsa
The distinguishing marks of fraud were therefore clearly apparent in A. C.
Likewise, substantial identity of the incorporators of the two corporations Ransom. A new corporation was created, owned by the same family,
does not necessarily imply fraud. engaging in the same business and operating in the same compound.

The circumstances of this case distinguish it from those in earlier decisions of Thus, considering that the non-payment of the workers was a continuing
the Court in labor cases where the veil of corporate fiction was pierced. situation, the Court adjudged its President, the "responsible officer" of the
corporation, personally liable for the backwages awarded, he being the chief
In La Campana Coffee Factory, Inc. v. Kaisahan ng Manggagawa sa La operation officer or "manager" who could be held criminally liable for
Campana (KKM) 93 Phil. 160 (1953), La Campana Coffee Factory, Inc. and violations of Republic Act No. 602 (the old Minimum Wage Law.)
La Campana Gaugau Packing were substantially owned by the same person.
They had one office, one management, and a single payroll for both In the case now before us, not only has there been a failure to establish
businesses. The laborers of the gaugaufactory and the coffee factory were fraud, but it has also not been shown that petitioner is the corporate officer
also interchangeable, i.e., the workers in one factory worked also in the other responsible for private respondent's predicament. It must be emphasized that
factory. the claim for differentials and benefits was actually directed against the
foreign employer. Philsa became liable only because of its undertaking to be
In Claparols v. Court of Industrial Relations, G.R. No. L-30822, July 31, 1975, jointly and severally bound with the foreign employer, an undertaking
65 SCRA 613, the Claparols Steel and Nail Plant, which was ordered to pay required by the rules of the POEA [Rule II, sec. 1(d) (3)], together with the
its workers backwages, ceased operations on June 30, 1957 and was filing of cash and surety bonds [Rule 11, sec. 4], in order to ensure that
succeeded on the next day, July 1, 1957 by the Claparols Steel Corporation. overseas workers shall find satisfaction for awards in their favor.
Both corporations were substantially owned and controlled by the same
person and there was no break or cessation in operations. Moreover, all the At this juncture, the Court finds it appropriate to point out that a judgment
assets of the steel and nail plant were transferred to the new corporation. against a recruiter should initially be enforced against the cash and surety
bonds filed with the POEA. As provided in the POEA Rules and Regulations
2. As earlier stated, we also find that, contrary to the NLRC'S holding, the
ruling in A. C. Ransom is inapplicable to this case. In A. C. Ransom, the
Court said: ... The bonds shall answer for all valid and legal claims
arising from violations of the conditions for the grant and use
... In the instant case, it would appear that RANSOM, in of the license or authority and contracts of employment. The
1969, foreseeing the possibility or probability of payment of bonds shall likewise guarantee compliance with the

371
provisions of the Labor Code and its implementing rules and
regulations relating to recruitment and placement, the rules
of the Administration and relevant issuances of the Ministry
and all liabilities which the Administration may impose. ...
[Rule II, see. 4.]

Quite evidently, these bonds do not answer for a single specific liability, but
for all sorts of liabilities of the recruiter to the worker and to the POEA.
Moreover, the obligations guaranteed by the bonds are continuing. Thus, the
bonds are subject to replenishment when they are garnished, and failure to
replenish shall cause the suspension or cancellation of the recruiter's license
[Rule II, sec. 19]. Furthermore, a cash bond shall be refunded to a recruiter
who surrenders his license only upon posting of a surety bond of similar
amount valid for three (3) years [Rule II, sec. 20]. All these, to ensure
recovery from the recruiter.

It is therefore surprising why the POEA ordered execution "against the


properties of Mr. Francisco V. del Rosario and if insufficient, against the cash
and/or surety bond of Bonding Company concerned for the till satisfaction of
the judgment awarded" in complete disregard of the scheme outlined in the
POEA Rules and Regulations. On this score alone, the NLRC should not
have affirmed the POEA.

WHEREFORE, the petition is GRANTED and the decision and resolution of


the NLRC, dated September 23, 1988 and October 21, 1988, respectively, in
POEA Case No. 85-06-0394 are SET ASIDE. The temporary restraining
order issued by the Court on November 10, 1988 is MADE PERMANENT.

SO ORDERED.

Fernan, C.J., Gutierrez, Jr., Feliciano and Bidin, JJ., concur.

372
G.R. No. L-23893 October 29, 1968

VILLA REY TRANSIT, INC., plaintiff-appellant,


vs.
EUSEBIO E. FERRER, PANGASINAN TRANSPORTATION CO., INC. and
PUBLIC SERVICE COMMISSION,defendants.
EUSEBIO E. FERRER and PANGASINAN TRANSPORTATION CO.,
INC., defendants-appellants.

373
PANGASINAN TRANSPORTATION CO., INC., third-party plaintiff-appellant, Barely three months thereafter, or on March 6, 1959: a corporation called
vs. Villa Rey Transit, Inc. (which shall be referred to hereafter as the
JOSE M. VILLARAMA, third-party defendant-appellee. Corporation) was organized with a capital stock of P500,000.00 divided into
5,000 shares of the par value of P100.00 each; P200,000.00 was the
Chuidian Law Office for plaintiff-appellant. subscribed stock; Natividad R. Villarama (wife of Jose M. Villarama) was one
Bengzon, Zarraga & Villegas for defendant-appellant / third-party plaintiff- of the incorporators, and she subscribed for P1,000.00; the balance of
appellant. P199,000.00 was subscribed by the brother and sister-in-law of Jose M.
Laurea & Pison for third-party defendant-appellee. Villarama; of the subscribed capital stock, P105,000.00 was paid to the
treasurer of the corporation, who was Natividad R. Villarama.
ANGELES, J.:
In less than a month after its registration with the Securities and Exchange
This is a tri-party appeal from the decision of the Court of First Instance of Commission (March 10, 1959), the Corporation, on April 7, 1959,
Manila, Civil Case No. 41845, declaring null and void the sheriff's sale bought five certificates of public convenience, forty-nine buses, tools and
of two certificates of public convenience in favor of defendant Eusebio E. equipment from one Valentin Fernando, for the sum of P249,000.00, of which
Ferrer and the subsequent sale thereof by the latter to defendant Pangasinan P100,000.00 was paid upon the signing of the contract; P50,000.00 was
Transportation Co., Inc.; declaring the plaintiff Villa Rey Transit, Inc., to be payable upon the final approval of the sale by the PSC; P49,500.00 one year
the lawful owner of the said certificates of public convenience; and ordering after the final approval of the sale; and the balance of P50,000.00 "shall be
the private defendants, jointly and severally, to pay to the plaintiff, the sum of paid by the BUYER to the different suppliers of the SELLER."
P5,000.00 as and for attorney's fees. The case against the PSC was
dismissed. The very same day that the aforementioned contract of sale was executed,
the parties thereto immediately applied with the PSC for its approval, with a
The rather ramified circumstances of the instant case can best be prayer for the issuance of a provisional authority in favor of the vendee
understood by a chronological narration of the essential facts, to wit: Corporation to operate the service therein involved. 1 On May 19, 1959, the
PSC granted the provisional permit prayed for, upon the condition that "it may
be modified or revoked by the Commission at any time, shall be subject to
Prior to 1959, Jose M. Villarama was an operator of a bus transportation,
whatever action that may be taken on the basic application and shall be valid
under the business name of Villa Rey Transit, pursuant to certificates of
only during the pendency of said application." Before the PSC could take
public convenience granted him by the Public Service Commission (PSC, for
final action on said application for approval of sale, however, the Sheriff of
short) in Cases Nos. 44213 and 104651, which authorized him to operate a
Manila, on July 7, 1959, levied on two of the five certificates of public
total of thirty-two (32) units on various routes or lines from Pangasinan to
convenience involved therein, namely, those issued under PSC cases Nos.
Manila, and vice-versa. On January 8, 1959, he sold the aforementioned two
59494 and 63780, pursuant to a writ of execution issued by the Court of First
certificates of public convenience to the Pangasinan Transportation
Instance of Pangasinan in Civil Case No. 13798, in favor of Eusebio Ferrer,
Company, Inc. (otherwise known as Pantranco), for P350,000.00 with the
plaintiff, judgment creditor, against Valentin Fernando, defendant, judgment
condition, among others, that the seller (Villarama) "shall not for a period of
debtor. The Sheriff made and entered the levy in the records of the PSC. On
10 years from the date of this sale, apply for any TPU service identical or
July 16, 1959, a public sale was conducted by the Sheriff of the
competing with the buyer."
said two certificates of public convenience. Ferrer was the highest bidder,
and a certificate of sale was issued in his name.

374
Thereafter, Ferrer sold the two certificates of public convenience to Pantranco, on its part, filed a third-party complaint against Jose M. Villarama,
Pantranco, and jointly submitted for approval their corresponding contract of alleging that Villarama and the Corporation, are one and the same; that
sale to the PSC.2 Pantranco therein prayed that it be authorized provisionally Villarama and/or the Corporation was disqualified from operating the two
to operate the service involved in the said two certificates. certificates in question by virtue of the aforementioned agreement between
said Villarama and Pantranco, which stipulated that Villarama "shall not for a
The applications for approval of sale, filed before the PSC, by Fernando and period of 10 years from the date of this sale, apply for any TPU service
the Corporation, Case No. 124057, and that of Ferrer and Pantranco, Case identical or competing with the buyer."
No. 126278, were scheduled for a joint hearing. In the meantime, to wit, on
July 22, 1959, the PSC issued an order disposing that during the pendency Upon the joinder of the issues in both the complaint and third-party
of the cases and before a final resolution on the aforesaid applications, the complaint, the case was tried, and thereafter decision was rendered in the
Pantranco shall be the one to operate provisionally the service under terms, as above stated.
the two certificates embraced in the contract between Ferrer and Pantranco.
The Corporation took issue with this particular ruling of the PSC and elevated As stated at the beginning, all the parties involved have appealed from the
the matter to the Supreme Court,3 which decreed, after deliberation, that until decision. They submitted a joint record on appeal.
the issue on the ownership of the disputed certificates shall have been finally
settled by the proper court, the Corporation should be the one to operate the Pantranco disputes the correctness of the decision insofar as it holds that
lines provisionally. Villa Rey Transit, Inc. (Corporation) is a distinct and separate entity from
Jose M. Villarama; that the restriction clause in the contract of January 8,
On November 4, 1959, the Corporation filed in the Court of First Instance of 1959 between Pantranco and Villarama is null and void; that the Sheriff's
Manila, a complaint for the annulment of the sheriff's sale of the sale of July 16, 1959, is likewise null and void; and the failure to award
aforesaid two certificates of public convenience (PSC Cases Nos. 59494 and damages in its favor and against Villarama.
63780) in favor of the defendant Ferrer, and the subsequent sale thereof by
the latter to Pantranco, against Ferrer, Pantranco and the PSC. The plaintiff Ferrer, for his part, challenges the decision insofar as it holds that the
Corporation prayed therein that all the orders of the PSC relative to the sheriff's sale is null and void; and the sale of the two certificates in question
parties' dispute over the said certificates be annulled. by Valentin Fernando to the Corporation, is valid. He also assails the award
of P5,000.00 as attorney's fees in favor of the Corporation, and the failure to
In separate answers, the defendants Ferrer and Pantranco averred that the award moral damages to him as prayed for in his counterclaim.
plaintiff Corporation had no valid title to the certificates in question because
the contract pursuant to which it acquired them from Fernando was subject to The Corporation, on the other hand, prays for a review of that portion of the
a suspensive condition the approval of the PSC which has not yet been decision awarding only P5,000.00 as attorney's fees, and insisting that it is
fulfilled, and, therefore, the Sheriff's levy and the consequent sale at public entitled to an award of P100,000.00 by way of exemplary damages.
auction of the certificates referred to, as well as the sale of the same by
Ferrer to Pantranco, were valid and regular, and vested unto Pantranco, a
After a careful study of the facts obtaining in the case, the vital issues to be
superior right thereto.
resolved are: (1) Does the stipulation between Villarama and Pantranco, as
contained in the deed of sale, that the former "SHALL NOT FOR A PERIOD
OF 10 YEARS FROM THE DATE OF THIS SALE, APPLY FOR ANY TPU

375
SERVICE IDENTICAL OR COMPETING WITH THE BUYER," apply to new Q. Doctor, your answer then is that since your money and your
lines only or does it include existing lines?; (2) Assuming that said stipulation wife's money are one money and you did not know when your wife
covers all kinds of lines, is such stipulation valid and enforceable?; (3) In the was paying debts with the incorporator's money?
affirmative, that said stipulation is valid, did it bind the Corporation?
A. Because sometimes she uses my money, and sometimes the
For convenience, We propose to discuss the foregoing issues by starting money given to her she gives to me and I deposit the money.
with the last proposition.
Q. Actually, aside from your wife, you were also the custodian of
The evidence has disclosed that Villarama, albeit was not an incorporator or some of the incorporators here, in the beginning?
stockholder of the Corporation, alleging that he did not become such,
because he did not have sufficient funds to invest, his wife, however, was an A. Not necessarily, they give to my wife and when my wife hands
incorporator with the least subscribed number of shares, and was elected to me I did not know it belonged to the incorporators.
treasurer of the Corporation. The finances of the Corporation which, under all
concepts in the law, are supposed to be under the control and administration Q. It supposes then your wife gives you some of the money
of the treasurer keeping them as trust fund for the Corporation, were, received by her in her capacity as treasurer of the corporation?
nonetheless, manipulated and disbursed as if they were the private funds of
Villarama, in such a way and extent that Villarama appeared to be the actual
A. Maybe.
owner-treasurer of the business without regard to the rights of the
stockholders. The following testimony of Villarama,4 together with the other
evidence on record, attests to that effect: Q. What did you do with the money, deposit in a regular account?

Q. Doctor, I want to go back again to the incorporation of the Villa A. Deposit in my account.
Rey Transit, Inc. You heard the testimony presented here by the
bank regarding the initial opening deposit of ONE HUNDRED FIVE Q. Of all the money given to your wife, she did not receive any
THOUSAND PESOS, of which amount Eighty-Five Thousand Pesos check?
was a check drawn by yourself personally. In the direct examination
you told the Court that the reason you drew a check for Eighty-Five A. I do not remember.
Thousand Pesos was because you and your wife, or your wife, had
spent the money of the stockholders given to her for incorporation. Q. Is it usual for you, Doctor, to be given Fifty Thousand Pesos
Will you please tell the Honorable Court if you knew at the time your without even asking what is this?
wife was spending the money to pay debts, you personally knew she
was spending the money of the incorporators? xxx xxx xxx

A. You know my money and my wife's money are one. We never JUDGE: Reform the question.
talk about those things.

376
Q. The subscription of your brother-in-law, Mr. Reyes, is Fifty- Ford trucks for the Corporation. Exhibits 20 and 21 disclose that the said
Two Thousand Pesos, did your wife give you Fifty-two Thousand purchases were paid by Philippine Bank of Commerce Checks Nos. 992618-
Pesos? B and 993621-B, respectively. These checks have been sufficiently
established by Fausto Abad, Assistant Accountant of Manila Trading &
A. I have testified before that sometimes my wife gives me Supply Co., from which the trucks were purchased 9 and Aristedes Solano, an
money and I do not know exactly for what. employee of the Philippine Bank of Commerce, 10 as having been drawn by
Villarama.
The evidence further shows that the initial cash capitalization of the
corporation of P105,000.00 was mostly financed by Villarama. Of the Exhibits 6 to 19 and Exh. 22, which are photostatic copies of ledger entries
P105,000.00 deposited in the First National City Bank of New York, and vouchers showing that Villarama had co-mingled his personal funds and
representing the initial paid-up capital of the Corporation, P85,000.00 was transactions with those made in the name of the Corporation, are very
covered by Villarama's personal check. The deposit slip for the said amount illuminating evidence. Villarama has assailed the admissibility of these
of P105,000.00 was admitted in evidence as Exh. 23, which shows on its exhibits, contending that no evidentiary value whatsoever should be given to
face that P20,000.00 was paid in cash and P85,000.00 thereof was covered them since "they were merely photostatic copies of the originals, the best
by Check No. F-50271 of the First National City Bank of New York. The evidence being the originals themselves." According to him, at the time
testimonies of Alfonso Sancho 5 and Joaquin Amansec,6 both employees of Pantranco offered the said exhibits, it was the most likely possessor of the
said bank, have proved that the drawer of the check was Jose Villarama originals thereof because they were stolen from the files of the Corporation
himself. and only Pantranco was able to produce the alleged photostat copies
thereof.
Another witness, Celso Rivera, accountant of the Corporation, testified that
while in the books of the corporation there appears an entry that the Section 5 of Rule 130 of the Rules of Court provides for the requisites for the
treasurer received P95,000.00 as second installment of the paid-in admissibility of secondary evidence when the original is in the custody of the
subscriptions, and, subsequently, also P100,000.00 as the first installment of adverse party, thus: (1) opponent's possession of the original; (2) reasonable
the offer for second subscriptions worth P200,000.00 from the original notice to opponent to produce the original; (3) satisfactory proof of its
subscribers, yet Villarama directed him (Rivera) to make vouchers liquidating existence; and (4) failure or refusal of opponent to produce the original in
the sums.7 Thus, it was made to appear that the P95,000.00 was delivered to court.11 Villarama has practically admitted the second and fourth
Villarama in payment for equipment purchased from him, and the requisites.12 As to the third, he admitted their previous existence in the files of
P100,000.00 was loaned as advances to the stockholders. The said the Corporation and also that he had seen some of them. 13 Regarding the
accountant, however, testified that he was not aware of any amount of first element, Villarama's theory is that since even at the time of the issuance
money that had actually passed hands among the parties involved, 8 and of the subpoena duces tecum, the originals were already missing, therefore,
actually the only money of the corporation was the P105,000.00 covered by the Corporation was no longer in possession of the same. However, it is not
the deposit slip Exh. 23, of which as mentioned above, P85,000.00 was paid necessary for a party seeking to introduce secondary evidence to show that
by Villarama's personal check. the original is in the actual possession of his adversary. It is enough that the
circumstances are such as to indicate that the writing is in his possession or
Further, the evidence shows that when the Corporation was in its initial under his control. Neither is it required that the party entitled to the custody of
months of operation, Villarama purchased and paid with his personal checks the instrument should, on being notified to produce it, admit having it in his
possession.14 Hence, secondary evidence is admissible where he denies

377
having it in his possession. The party calling for such evidence may introduce while he was still the owner of Villa Rey Transit, a single proprietorship." But
a copy thereof as in the case of loss. For, among the exceptions to the best with his admission that he had received P350,000.00 from Pantranco for the
evidence rule is "when the original has been lost, destroyed, or cannot be sale of the two certificates and one unit,24 it becomes difficult to accept
produced in court."15 The originals of the vouchers in question must be Villarama's explanation that he and his wife, after consultation, 25 spent the
deemed to have been lost, as even the Corporation admits such loss. money of their relatives (the stockholders) when they were supposed to have
Viewed upon this light, there can be no doubt as to the admissibility in their own money. Even if Pantranco paid the P350,000.00 in check to him, as
evidence of Exhibits 6 to 19 and 22. claimed, it could have been easy for Villarama to have deposited said check
in his account and issued his own check to pay his obligations. And there is
Taking account of the foregoing evidence, together with Celso Rivera's no evidence adduced that the said amount of P350,000.00 was all spent or
testimony,16 it would appear that: Villarama supplied the organization was insufficient to settle his prior obligations in his business, and in the light
expenses and the assets of the Corporation, such as trucks and of the stipulation in the deed of sale between Villarama and Pantranco that
equipment;17there was no actual payment by the original subscribers of the P50,000.00 of the selling price was earmarked for the payments of accounts
amounts of P95,000.00 and P100,000.00 as appearing in the due to his creditors, the excuse appears unbelievable.
books;18 Villarama made use of the money of the Corporation and deposited
them to his private accounts;19 and the Corporation paid his personal On his having paid for purchases by the Corporation of trucks from the
accounts.20 Manila Trading & Supply Co. with his personal checks, his reason was that
he was only sharing with the Corporation his credit with some companies.
Villarama himself admitted that he mingled the corporate funds with his own And his main reason for mingling his funds with that of the Corporation and
money.21 He also admitted that gasoline purchases of the Corporation were for the latter's paying his private bills is that it would be more convenient that
made in his name22 because "he had existing account with Stanvac which he kept the money to be used in paying the registration fees on time, and
was properly secured and he wanted the Corporation to benefit from the since he had loaned money to the Corporation, this would be set off by the
rebates that he received."23 latter's paying his bills. Villarama admitted, however, that the corporate funds
in his possession were not only for registration fees but for other important
The foregoing circumstances are strong persuasive evidence showing that obligations which were not specified.26
Villarama has been too much involved in the affairs of the Corporation to
altogether negative the claim that he was only a part-time general manager. Indeed, while Villarama was not the Treasurer of the Corporation but was,
They show beyond doubt that the Corporation is his alter ego. allegedly, only a part-time manager,27he admitted not only having held the
corporate money but that he advanced and lent funds for the Corporation,
It is significant that not a single one of the acts enumerated above as proof of and yet there was no Board Resolution allowing it. 28
Villarama's oneness with the Corporation has been denied by him. On the
contrary, he has admitted them with offered excuses. Villarama's explanation on the matter of his involvement with the corporate
affairs of the Corporation only renders more credible Pantranco's claim that
Villarama has admitted, for instance, having paid P85,000.00 of the initial his control over the corporation, especially in the management and
capital of the Corporation with the lame excuse that "his wife had requested disposition of its funds, was so extensive and intimate that it is impossible to
him to reimburse the amount entrusted to her by the incorporators and which segregate and identify which money belonged to whom. The interference of
she had used to pay the obligations of Dr. Villarama (her husband) incurred Villarama in the complex affairs of the corporation, and particularly its
finances, are much too inconsistent with the ends and purposes of the

378
Corporation law, which, precisely, seeks to separate personal responsibilities As We read the disputed clause, it is evident from the context thereof that the
from corporate undertakings. It is the very essence of incorporation that the intention of the parties was to eliminate the seller as a competitor of the
acts and conduct of the corporation be carried out in its own corporate name buyer for ten years along the lines of operation covered by the certificates of
because it has its own personality. public convenience subject of their transaction. The word "apply" as broadly
used has for frame of reference, a service by the seller on lines or routes that
The doctrine that a corporation is a legal entity distinct and separate from the would compete with the buyer along the routes acquired by the latter. In this
members and stockholders who compose it is recognized and respected in jurisdiction, prior authorization is needed before anyone can operate a TPU
all cases which are within reason and the law.29 When the fiction is urged as service,33whether the service consists in a new line or an old one acquired
a means of perpetrating a fraud or an illegal act or as a vehicle for the from a previous operator. The clear intention of the parties was to prevent the
evasion of an existing obligation, the circumvention of statutes, the seller from conducting any competitive line for 10 years since, anyway, he
achievement or perfection of a monopoly or generally the perpetration of has bound himself not to apply for authorization to operate along such lines
knavery or crime,30 the veil with which the law covers and isolates the for the duration of such period.34
corporation from the members or stockholders who compose it will be lifted
to allow for its consideration merely as an aggregation of individuals. If the prohibition is to be applied only to the acquisition of new certificates of
public convenience thru an application with the Public Service Commission,
Upon the foregoing considerations, We are of the opinion, and so hold, that this would, in effect, allow the seller just the same to compete with the buyer
the preponderance of evidence have shown that the Villa Rey Transit, Inc. is as long as his authority to operate is only acquired thru transfer or sale from
an alter ego of Jose M. Villarama, and that the restrictive clause in the a previous operator, thus defeating the intention of the parties. For what
contract entered into by the latter and Pantranco is also enforceable and would prevent the seller, under the circumstances, from having a
binding against the said Corporation. For the rule is that a seller or promisor representative or dummy apply in the latter's name and then later on
may not make use of a corporate entity as a means of evading the obligation transferring the same by sale to the seller? Since stipulations in a contract is
of his covenant.31 Where the Corporation is substantially the alter ego of the the law between the contracting parties,
covenantor to the restrictive agreement, it can be enjoined from competing
with the covenantee.32 Every person must, in the exercise of his rights and in the
performance of his duties, act with justice, give everyone his due,
The Corporation contends that even on the supposition that Villa Rey Transit, and observe honesty and good faith. (Art. 19, New Civil Code.)
Inc. and Villarama are one and the same, the restrictive clause in the
contract between Villarama and Pantranco does not include the purchase of We are not impressed of Villarama's contention that the re-wording of the two
existing lines but it only applies to application for the new lines. The clause in previous drafts of the contract of sale between Villarama and Pantranco is
dispute reads thus: significant in that as it now appears, the parties intended to effect the least
restriction. We are persuaded, after an examination of the supposed drafts,
(4) The SELLER shall not, for a period of ten (10) years from the that the scope of the final stipulation, while not as long and prolix as those in
date of this sale apply for any TPU service identical or competing the drafts, is just as broad and comprehensive. At most, it can be said that
with the BUYER. (Emphasis supplied) the re-wording was done merely for brevity and simplicity.

379
The evident intention behind the restriction was to eliminate the sellers as a The law concerning contracts which tend to restrain business or
competitor, and this must be, considering such factors as the good will 35 that trade has gone through a long series of changes from time to time
the seller had already gained from the riding public and his adeptness and with the changing condition of trade and commerce. With trifling
proficiency in the trade. On this matter, Corbin, an authority on Contracts has exceptions, said changes have been a continuous development of a
this to say.36 general rule. The early cases show plainly a disposition to avoid and
annul all contract which prohibited or restrained any one from using a
When one buys the business of another as a going concern, he lawful trade "at any time or at any place," as being against the
usually wishes to keep it going; he wishes to get the location, the benefit of the state. Later, however, the rule became well established
building, the stock in trade, and the customers. He wishes to step that if the restraint was limited to "a certain time" and within "a
into the seller's shoes and to enjoy the same business relations with certain place," such contracts were valid and not "against the benefit
other men. He is willing to pay much more if he can get the "good of the state." Later cases, and we think the rule is now well
will" of the business, meaning by this the good will of the customers, established, have held that a contract in restraint of trade is valid
that they may continue to tread the old footpath to his door and providing there is a limitation upon either time or place. A contract,
maintain with him the business relations enjoyed by the seller. however, which restrains a man from entering into business or trade
without either a limitation as to time or place, will be held invalid.
... In order to be well assured of this, he obtains and pays for the
seller's promise not to reopen business in competition with the The public welfare of course must always be considered and if it be
business sold. not involved and the restraint upon one party is not greater than
protection to the other requires, contracts like the one we are
As to whether or not such a stipulation in restraint of trade is valid, our discussing will be sustained. The general tendency, we believe, of
jurisprudence on the matter37says: modern authority, is to make the test whether the restraint is
reasonably necessary for the protection of the contracting parties. If
the contract is reasonably necessary to protect the interest of the
parties, it will be upheld. (Emphasis supplied.)

Analyzing the characteristics of the questioned stipulation, We find that


although it is in the nature of an agreement suppressing competition, it is,
however, merely ancillary or incidental to the main agreement which is that of
sale. The suppression or restraint is only partial or limited: first, in scope, it
refers only to application for TPU by the seller in competition with the lines
sold to the buyer; second, in duration, it is only for ten (10) years; and
third, with respect to situs or territory, the restraint is only along the lines
covered by the certificates sold. In view of these limitations, coupled with the
consideration of P350,000.00 for just two certificates of public convenience,
and considering, furthermore, that the disputed stipulation is only incidental
to a main agreement, the same is reasonable and it is not harmful nor
obnoxious to public service.38 It does not appear that the ultimate result of the

380
clause or stipulation would be to leave solely to Pantranco the right to misplaced. In the said Red Line case, the agreement therein sought to be
operate along the lines in question, thereby establishing monopoly or enforced was virtually a division of territory between two operators, each
predominance approximating thereto. We believe the main purpose of the company imposing upon itself an obligation not to operate in any territory
restraint was to protect for a limited time the business of the buyer. covered by the routes of the other. Restraints of this type, among common
carriers have always been covered by the general rule invalidating
Indeed, the evils of monopoly are farfetched here. There can be no danger of agreements in restraint of trade. 42
price controls or deterioration of the service because of the close supervision
of the Public Service Commission.39 This Court had stated long ago,40that Neither are the other cases relied upon by the plaintiff-appellee applicable to
"when one devotes his property to a use in which the public has an interest, the instant case. In Pampanga Bus Co., Inc. v. Enriquez,43the undertaking of
he virtually grants to the public an interest in that use and submits it to such the applicant therein not to apply for the lifting of restrictions imposed on his
public use under reasonable rules and regulations to be fixed by the Public certificates of public convenience was not an ancillary or incidental
Utility Commission." agreement. The restraint was the principal objective. On the other hand,
in Red Line Transportation Co., Inc. v. Gonzaga,44 the restraint there in
Regarding that aspect of the clause that it is merely ancillary or incidental to question not to ask for extension of the line, or trips, or increase of equipment
a lawful agreement, the underlying reason sustaining its validity is well was not an agreement between the parties but a condition imposed in the
explained in 36 Am. Jur. 537-539, to wit: certificate of public convenience itself.

... Numerous authorities hold that a covenant which is incidental to Upon the foregoing considerations, Our conclusion is that the stipulation
the sale and transfer of a trade or business, and which purports to prohibiting Villarama for a period of 10 years to "apply" for TPU service along
bind the seller not to engage in the same business in competition the lines covered by the certificates of public convenience sold by him to
with the purchaser, is lawful and enforceable. While such covenants Pantranco is valid and reasonable. Having arrived at this conclusion, and
are designed to prevent competition on the part of the seller, it is considering that the preponderance of the evidence have shown that Villa
ordinarily neither their purpose nor effect to stifle competition Rey Transit, Inc. is itself the alter ego of Villarama, We hold, as prayed for in
generally in the locality, nor to prevent it at all in a way or to an extent Pantranco's third party complaint, that the said Corporation should, until the
injurious to the public. The business in the hands of the purchaser is expiration of the 1-year period abovementioned, be enjoined from operating
carried on just as it was in the hands of the seller; the former merely the line subject of the prohibition.
takes the place of the latter; the commodities of the trade are as
open to the public as they were before; the same competition exists To avoid any misunderstanding, it is here to be emphasized that the 10-year
as existed before; there is the same employment furnished to others prohibition upon Villarama is not against his application for, or purchase of,
after as before; the profits of the business go as they did before to certificates of public convenience, but merely the operation of TPU along the
swell the sum of public wealth; the public has the same opportunities lines covered by the certificates sold by him to Pantranco. Consequently, the
of purchasing, if it is a mercantile business; and production is not sale between Fernando and the Corporation is valid, such that the rightful
lessened if it is a manufacturing plant. ownership of the disputed certificates still belongs to the plaintiff being the
prior purchaser in good faith and for value thereof. In view of the ancient rule
The reliance by the lower court on tile case of Red Line Transportation Co. v. of caveat emptorprevailing in this jurisdiction, what was acquired by Ferrer in
Bachrach41 and finding that the stipulation is illegal and void seems the sheriff's sale was only the right which Fernando, judgment debtor, had in
the certificates of public convenience on the day of the sale. 45

381
Accordingly, by the "Notice of Levy Upon Personalty" the Commissioner of Anent the question of damages allegedly suffered by the parties, each of the
Public Service was notified that "by virtue of an Order of Execution issued by appellants has its or his own version to allege.
the Court of First Instance of Pangasinan, the rights, interests, or
participation which the defendant, VALENTIN A. FERNANDO in the above Villa Rey Transit, Inc. claims that by virtue of the "tortious acts" of defendants
entitled case may have in the following realty/personalty is attached or levied (Pantranco and Ferrer) in acquiring the certificates of public convenience in
upon, to wit: The rights, interests and participation on the Certificates of question, despite constructive and actual knowledge on their part of a prior
Public Convenience issued to Valentin A. Fernando, in Cases Nos. 59494, sale executed by Fernando in favor of the said corporation, which
etc. ... Lines Manila to Lingayen, Dagupan, etc. vice versa." Such notice of necessitated the latter to file the action to annul the sheriff's sale to Ferrer
levy only shows that Ferrer, the vendee at auction of said certificates, merely and the subsequent transfer to Pantranco, it is entitled to collect actual and
stepped into the shoes of the judgment debtor. Of the same principle is the compensatory damages, and attorney's fees in the amount of P25,000.00.
provision of Article 1544 of the Civil Code, that "If the same thing should have The evidence on record, however, does not clearly show that said
been sold to different vendees, the ownership shall be transferred to the defendants acted in bad faith in their acquisition of the certificates in
person who may have first taken possession thereof in good faith, if it should question. They believed that because the bill of sale has yet to be approved
be movable property." by the Public Service Commission, the transaction was not a consummated
sale, and, therefore, the title to or ownership of the certificates was still with
There is no merit in Pantranco and Ferrer's theory that the sale of the the seller. The award by the lower court of attorney's fees of P5,000.00 in
certificates of public convenience in question, between the Corporation and favor of Villa Rey Transit, Inc. is, therefore, without basis and should be set
Fernando, was not consummated, it being only a conditional sale subject to aside.
the suspensive condition of its approval by the Public Service Commission.
While section 20(g) of the Public Service Act provides that "subject to Eusebio Ferrer's charge that by reason of the filing of the action to annul the
established limitation and exceptions and saving provisions to the contrary, it sheriff's sale, he had suffered and should be awarded moral, exemplary
shall be unlawful for any public service or for the owner, lessee or operator damages and attorney's fees, cannot be entertained, in view of the
thereof, without the approval and authorization of the Commission previously conclusion herein reached that the sale by Fernando to the Corporation was
had ... to sell, alienate, mortgage, encumber or lease its property, franchise, valid.
certificates, privileges, or rights or any part thereof, ...," the same section also
provides: Pantranco, on the other hand, justifies its claim for damages with the
allegation that when it purchased ViIlarama's business for P350,000.00, it
... Provided, however, That nothing herein contained shall be intended to build up the traffic along the lines covered by the certificates but it
construed to prevent the transaction from being negotiated or was rot afforded an opportunity to do so since barely three months had
completed before its approval or to prevent the sale, alienation, or elapsed when the contract was violated by Villarama operating along the
lease by any public service of any of its property in the ordinary same lines in the name of Villa Rey Transit, Inc. It is further claimed by
course of its business. Pantranco that the underhanded manner in which Villarama violated the
contract is pertinent in establishing punitive or moral damages. Its contention
It is clear, therefore, that the requisite approval of the PSC is not a condition as to the proper measure of damages is that it should be the purchase price
precedent for the validity and consummation of the sale. of P350,000.00 that it paid to Villarama. While We are fully in accord with
Pantranco's claim of entitlement to damages it suffered as a result of
Villarama's breach of his contract with it, the record does not sufficiently

382
supply the necessary evidentiary materials upon which to base the award
and there is need for further proceedings in the lower court to ascertain the
proper amount.

PREMISES CONSIDERED, the judgment appealed from is hereby modified


as follows:

1. The sale of the two certificates of public convenience in question by


Valentin Fernando to Villa Rey Transit, Inc. is declared preferred over that
made by the Sheriff at public auction of the aforesaid certificate of public
convenience in favor of Eusebio Ferrer;

2. Reversed, insofar as it dismisses the third-party complaint filed by


Pangasinan Transportation Co. against Jose M. Villarama, holding that Villa
Rey Transit, Inc. is an entity distinct and separate from the personality of
Jose M. Villarama, and insofar as it awards the sum of P5,000.00 as
attorney's fees in favor of Villa Rey Transit, Inc.;

3. The case is remanded to the trial court for the reception of evidence in
consonance with the above findings as regards the amount of damages
suffered by Pantranco; and

4. On equitable considerations, without costs. So ordered.

Concepcion, C. J., Reyes, J.B.L., Dizon, Makalintal, Castro and Fernando,


JJ., concur.
Sanchez and Capistrano, JJ., took no part.
Zaldivar, J., is on leave.

383
384
STATEMENT

For a number of years prior to the times alleged in the complaint, the plaintiff
was in the employ of the International Banking Corporation of Manila, and it
is conceded that he is a competent and experienced business man. July 31,
1916, C. D. Willits and I. L. Patterson were partners doing business in San
Francisco, California, under the name of Willits & Patterson. The plaintiff was
then in San Francisco, and as a result of negotiations the plaintiff and the
firm entered into a written contract, known in the record as Exhibit A, by
which the plaintiff was employed as the agent of the firm in the Philippine
Islands for certain purposes for the period of five years at a minimum salary
of $200 per month and travelling expenses. The plaintiff returned to Manila
and entered on the discharge of his duties under the contract. As a result of
plaintiff's employment and the world war conditions, the business of the firm
in the Philippines very rapidly increased and grew beyond the fondest hopes
of either party. A dispute arose between the plaintiff and the firm as to the
construction of Exhibit A as to the amount which plaintiff should receive for
his services. Meanwhile Patterson retired from the firm and Willits became
the sole owner of its assets. For convenience of operation and to serve his
own purpose, Willits organized a corporation under the laws of California with
its principal office at San Francisco, in and by which he subscribed for, and
became the exclusive owner of all the capital stock except a few shares for
organization purposes only, and the name of the firm was used as the name
of the corporation. A short time after that Willits came to Manila and
organized a corporation here known as Willits & Patterson, Ltd., in and to
which he again subscribed for all of the capital stock except the nominal
shares necessary to qualify the directors. In legal effect, the San Francisco
corporation took over and acquired all of the assets and liabilities of the
G.R. No. L-20214 March 17, 1923 Manila corporation. At the time that Willits was in Manila and while to all
intents and purposes he was the sole owner of the stock of corporations,
G. C. ARNOLD, plaintiff-appellant, there was a conference between him and the plaintiff over the disputed
vs. construction of Exhibit A. As a result of which another instrument, known in
WILLITS & PATTERSON, LTD., defendant-appellee. the record as Exhibit B, was prepared in the form of a letter which the plaintiff
addressed to Willits at Manila on November 10, 1919, the purpose of which
Fisher, DeWitt, Perkins and Brady for appellant. was to more clearly define and specify the compensation which the plaintiff
Ross and Lawrence for appellee. was to receive for his services. Willits received and confirmed this letter by
signing the name of Willits & Patterson, By C.d. Willits. At the time both

385
corporations were legally organized, and there is nothing in the corporate Upon such issues a trial was had, and the lower court rendered judgment in
minutes to show that Exhibit B was ever formally ratified or approved by favor of the defendant as prayed for in its counterclaim, from which the
either corporation. After its organization, the Manila corporation employed a plaintiff appeals, contending that the trial court erred in not holding that the
regular accountant whose duty it was to audit the accounts of the company contract between the parties is that which is embodied in Exhibits A and B,
and render financial statements both for the use of the local banks and the and that the defendant assumed all partnership obligations, and in failing to
local and parent corporations at San Francisco. From time to time and in the render judgment for the plaintiff, as prayed for, and in dismissing his
ordinary course of business such statements of account were prepared by complaint, and denying plaintiff's motion for a new trial.
the accountant and duly forwarded to the home office, and among other
things was a statement of July 31, 1921, showing that there was due and
owing the plaintiff under Exhibit B the sum of P106,277.50. A short time
previous to that date, the San Francisco corporation became involved in
financial trouble, and all of its assets were turned over to a "creditors' JOHNS, J.:
committee." When this statement was received, the "creditors' committee"
immediately protested its allowance. An attempt was made without success
In their respective briefs opposing counsel agree that the important questions
to adjust the matter on a friendly basis and without litigation. January 10,
involved are "what was the contract under which the plaintiff rendered
1922, the plaintiff brought this action to recover from the defendant the sum
services for five years ending July 31, 1921," and "what is due the plaintiff
of P106,277.50 with legal interest and costs, and written instruments known
under that contract." Plaintiff contends that his services were performed
in the record as Exhibits A and B were attached to, and made a part of, the
under Exhibits A and B, and that the defendant assumed all of the obligations
complaint.
of the original partnership under Exhibit A, and is now seeking to deny its
liability under, and repudiate, Exhibit B. The defendant admits that Exhibit A
For answer, the defendant admits the formal parts of the complaint, the was the original contract between Arnold and the firm of Willits & Patterson
execution of Exhibit A and denies each and every other allegation, except as by which he came to the Philippine Islands, and that it was therein agreed
specifically admitted, and alleges that what is known as Exhibit B was signed that he was to be employed for a period of five years as the agent of Willits &
by Willits without the authority of the defendant corporation or the firm of Patterson in the Philippine Islands to operate a certain oil mill, and to do such
Willits & Patterson, and that it is not an agreement which was ever entered other business as might be deemed advisable for which he was to receive,
into with the plaintiff by the defendant or the firm, and, as a separate defense first, the travelling expenses of his wife and self from San Francisco to
and counterclaim, it alleges that on the 30th of June, 1920, there was a Manila, second, the minimum salary of $200 per month, third, a brokerage of
balance due and owing the plaintiff from the defendant under the contract 1 per cent upon all purchases and sales of merchandise, except for the
Exhibit A of the sum of P8,741.05. That his salary from June 30, 1920, to July account of the coconut oil mill, fourth, one-half of the profits on any
31, 1921, under Exhibit A was $400 per month, or a total of P10,400. That transaction in the name of the firm or himself not provided for in the
about July 6, 1921, the plaintiff wrongfully took P30,000 from the assets of agreement. That the agreement also provided that if it be found that the
the firm, and that he is now indebted to the firm in the sum of P10,858.95, business was operated at a loss, Arnold should receive a monthly salary of
with interest and costs, from which it prays judgement. $400 during such period. That the business was operated at a loss from June
30, 1920, to July 31, 1921, and that for such reason, he was entitled to
The plaintiff admits that he withdrew the P30,000, but alleges that it was with nothing more than a salary of $400 per month, or for that period P10,400.
the consent and authority of the defendant, and denies all other new matter Adding this amount to the P8,741.05, which the defendant admits he owed
in the answer. Arnold on June 30, 1920, makes a total of P19,141.05, leaving a balance due

386
the defendant as set out in the counterclaim. In other words, that the On all other business, such as the Cooperative Coconut Products
plaintiff's compensation was measured by, and limited to, the above specified Co. account, or any other business we may undertake as agents or
provisions in the contract Exhibit A, and that the defendant corporation is not managers, half the profits are to be credited to my account and half
bound by the terms or provisions of Exhibit B, which is as follows: to the Profit & Loss account of Willits & Patterson, Ltd., Manila.

WILLITS & PATTERSON, LTD. Where Willits & Patterson, San Francisco, or Willits & Patterson,
Ltd., Manila, have their own funds invested in the capital stock or a
MANILA, P. I., Nov. 10, 1919. corporation, I of course do not participate in the earnings of such
stock, any more than Willits & Patterson would participate in the
CHAS. D. WILLITS, Esq., earnings of stock held by me on my account.

Present. If the foregoing conforms to your understanding of our agreement,


please confirm below.
DEAR MR. WILLITS: My understanding of the intent of my
agreement with Willits & Patterson is as under: Yours faithfully,

Commissions. Willits & Patterson, San Francisco, pay me a (Sgd.) G. C. ARNOLD


commission of one per cent on all purchases made for them in the
Philippines or sales made to them by Manila and one per cent on all Confirmed:
sales made for them in the Philippines, or purchases made from
them by Manila. If such purchases or sales are on an f. o. b. basis WILLITS & PATTERSON
the commission is on the f. o. b. price; if on a c. i. f. basis the
commission is computed on the c. i. f. price By (Sgd.) CHAS. D. WILLITS

These commissions are credited to me in San Francisco. There is no dispute about any of the following facts: That at the inception
C.D. Willits and I. L. Patterson constituted the firm of Willits & Patterson
I do not participate in any profits on business transacted between doing business in the City of San Francisco; that later Patterson retired from
Willits & Patterson, San Francisco, and Willits & Patterson, Ltd., the firm, and Willits acquired all of his interests and thereafter continued the
Manila. business under the name and style of Willits & Patterson; that the original
contract Exhibit A was made between the plaintiff and the old firm at San
Profits. On all business transacted between Willits & Patterson, Ltd. Francisco on July 31, 1916, to cover a period of five years from that date;
and others than Willits & Patterson, San Francisco, half the profits that plaintiff entered upon the discharged of his duties and continued his
are to be credited to my account and half to the Profit & Loss services in the Philippine Islands to someone for the period of five years; that
account of Willits & Patterson, Ltd., Manila. on November 10, 1919, and as a result of conferences between Willits and
the plaintiff, Exhibit B was addressed and signed in the manner and form
above stated in the City of Manila. A short time prior to that date Willits

387
organized a corporation in San Francisco, in the State of California, which all purchases and sales of merchandise, except for the account of the
took over and acquired all of the assets of the firm's business in California coconut mills; (c) that the net profits from all other business should be divided
then being conducted under the name and style of Willits & Patterson; that in equal half shares between the parties hereto."
he subscribed for all of the capital stock of the corporation, and that in truth
and in fact he was the owner of all of its capital stock. After this was done he Under the above provisions, the plaintiff might well contend that he was
caused a new corporation to be organized under the laws of the Philippine entitled to one-half of all the profits and a brokerage of 1 per cent from all
Islands with principal office at Manila, which took over and acquired all the purchases and sales, except those for the account of the coconut oil mills,
business and assets of the firm of Willits & Patterson in the Philippine which under the volume of business then existing would run into a very large
Islands, in and to which, in legal effect, he subscribed for all of its capital sum of money. It was for such reason and after personal conferences
stock, and was the owner of all of its stock. After both corporations were between them, and to settle all disputed questions, that Exhibit B was
organized the above letter was drafted and signed. The plaintiff contends that prepared and signed.
the signing of Exhibit B in the manner and under the conditions in which it
was signed, and through the subsequent acts and conduct of the parties, The record recites that "the defendant admits that from July 31, 1916 to July
was ratified and, in legal effect, became and is now binding upon the 31, 1921, the plaintiff faithfully performed all the duties incumbent upon him
defendant. under his contract of employment, it being understood, however, that this
admission does not include an admission that the plaintiff placed a proper
It will be noted that Exhibit B was executed in Manila, and that at the time it interpretation upon his right to remuneration under said contract of
was signed by Willits, he was to all intents and purposes the legal owner of employment."
all the stock in both corporations. It also appears from the evidence that the
parent corporation at San Francisco took over and acquired all of the assets It being admitted that the plaintiff worked "under his contract of employment"
and liabilities of the local corporation at Manila. That after it was organized for the period of five years, the question naturally arises, for whom was he
the Manila corporation kept separate records and account books of its own, working? His contract was made with the original firm of Willits & Patterson,
and that from time to time financial statements were made and forwarded to and that firm was dissolved and it ceased to exist, and all of its assets were
the home office, from which it conclusively appears that plaintiff was basing merged in, and taken over by, the parent corporation at San Francisco. In the
his claim for services upon Exhibit A, as it was modified by Exhibit B. That at very nature of things, after the corporation was formed, the plaintiff could not
no time after Exhibit B was signed was there ever any dispute between and did not continue to work for the firm, and, yet, he continued his
plaintiff and Willits as to the compensation for plaintiff's services. That is to employment for the full period of five years. For whom did he work after the
say, as between the plaintiff and Willits, Exhibit B was approved, followed partnership was merged in the corporation and ceased to exist?
and at all times in force and effect, after it was signed November 10, 1919. It
appears from an analysis of Exhibit B that it was for the mutual interest of
It is very apparent that, under the conditions then existing, the signing of
both parties. From a small beginning, the business was then in a very
Exhibit B was for the mutual interests of both parties, and that if the contract
flourishing conditions and growing fast, and the profits were very large and
Exhibit A was to be enforced according to its terms, that Arnold might well
were running into big money.
contend for a much larger sum of money for his services. In truth and in fact
Willits and both corporations recognized his employment and accepted the
Among other things, Exhibit A provided: "(a) That the net profits from said benefits of his services. He continued his employment and rendered his
coconut oil business shall be divided in equal shares between the said services after the corporation were organized and Exhibit B was signed just
parties hereto; (b) that Arnold should receive a brokerage of 1 per cent from

388
the same as he did before, and both corporations recognized and accepted associates. This separate existence is to a certain extent a legal
his services. Although the plaintiff was president of the local corporation, the fiction. Whenever necessary for the interests of the public or for the
testimony is conclusive that both of them were what is known as a one man protection or enforcement of the rights of the membership, courts will
corporation, and Willits, as the owner of all of the stock, was the force and disregard this legal fiction and operate upon both the corporation and
dominant power which controlled them. After Exhibit B was signed it was the persons composing it.
recognized by Willits that the plaintiff's services were to be performed and
measured by its term and provisions, and there never was any dispute In the same section, the author quotes from a decision in 49 Ohio State,
between plaintiff and Willits upon that question. 1371; 15 L. R. A., 145, in which the Supreme Court of Ohio says:

The controversy first arose after the corporation was in financial trouble and "So long as a proper use is made of the fiction that a corporation is
the appointment of what is known in the record as a "creditors' committee." an entity apart from its shareholders, it is harmless, and, because
There is no claim or pretense that there was any fraud or collusion between convenient, should not be called in question; but where it is urged to
plaintiff and Willits, and it is very apparent that Exhibit B was to the mutual an end subversive of its policy, or such is the issue, the fiction must
interest of both parties. It is elementary law that if Exhibit B is a binding be ignored, and the question determined whether the act in question,
contract between the plaintiff and Willits and the corporations, it is equally though done by shareholders, that is to say, by the persons
binding upon the creditors' committee. It would not have any higher or better uniting in one body, was done simply as individuals, and with
legal right than the corporation itself, and could not make any defense which respect to their individual interest as shareholders, or was done
it could not make. It is very significant that the claim or defense which is now ostensibly as such, but, as a matter of fact, to control the corporation,
interposed by the creditors' committee was never made or asserted at any and affect the transaction of its business, in the same manner as if
previous time by the defendant, and that it never was made by Willits, and it the act had been clothed with all the formalities of a corporate act.
is very apparent that if he had remained in control of the corporation, it would This must be so, because, the stockholders having a dual capacity,
never have made the defense which is now made by the creditors' and capable of acting in either, and a possible interest to conceal
committee. The record is conclusive that at the time he signed Exhibit B, their character when acting in their corporate capacity, the absence
Willits was, in legal effect, the owner and holder of all the stock in both of the formal evidence of the character of the act cannot preclude
corporations, and that he approved it in their interest, and to protect them judicial inquiry on the subject. If it were otherwise, then in that
from the plaintiff having and making a much larger claim under Exhibit A. As department of the law fraud would enjoy an immunity awarded to it in
a matter of fact, it appears from the statement of Mr. Larkin, the accountant, no other."
in the record that if plaintiff's cause of action was now founded upon Exhibit
A, he would have a claim for more than P160,000. Where the stock of a corporation is owned by one person whereby
the corporation functions only for the benefit of such individual
Thompson on Corporations, 2d ed., vol. I, section 10, says: owner, the corporation and the individual should be deemed to be
the same. (U. S. Gypsum Co. vs. Mackay Wall Plaster Co., 199 Pac.,
The proposition that a corporation has an existence separate and 249.)
distinct from its membership has its limitations. It must be noted that
this separate existence is for particular purposes. It must also be Ruling Case Law, vol. 7, section 663, says:
remembered that there can be no corporate existence without
persons to compose it; there can be no association without

389
While of course a corporation cannot ratify a contract which is In its article on corporations, Corpus Juris, in section 2241 says:
strictly ultra vires, and which it in the first instance could not have
made, it may by ratification render binding on it a contract, entered Ratification by a corporation of a transaction not previously
into on its behalf by its officers or agents without authority. As a authorized is more easily inferred where the corporation receives
general rule such ratification need not be manifested by any voted or and retains property under it, and as a general rule where a
formal resolution of the corporation or be authenticated by the corporation, through its proper officers or board, takes and retains
corporate seal; no higher degree of evidence is requisite in the benefits of the unauthorized act or contract of an officer or agent,
establishing ratification on the part of a corporation, than is requisite with full knowledge of all the material facts, it thereby ratifies and
in showing an antecedent authorization. becomes bound by such act of contract, together with all the
liabilities and burdens resulting therefrom, and in some jurisdiction
xxx xxx xxx this rule is, in effect, declared by statute. Thus the corporation is
liable on the ground of ratification where, with knowledge of the facts,
SEC. 666. The assent or approval of a corporation to acts done on it accepts the benefit of services rendered under an unauthorized
its account may be inferred in the same manner that the absent of a contract of employment . . . .
natural person may be, and it is well settled that where a corporation
with full knowledge of the unauthorized act of its officer or agents Applying the law to the facts.
acquiesces in and consents to such acts, it thereby ratifies them,
especially where the acquiescence results in prejudice to a third Mr. Larkin, an experienced accountant, was employed by the local
person. corporation, and from time to time and in the ordinary course of business
made and prepared financial statements showing its assets and liabilities,
xxx xxx xxx true copies of which were sent to the home office in San Francisco. It
appears upon their face that plaintiff's compensation was made and founded
SEC. 669. So, when, in the usual course of business of a on Exhibit B, and that such statements were made and prepared by the
corporation, an officer has been allowed in his official capacity to accountant on the assumption that Exhibit B was in full force and effect as
manage its affair, his authority to represent the corporation may be between the plaintiff and the defendant. In the course of business in the early
inferred from the manner in which he has been permitted by the part of 1920, plaintiff, as manager of the defendant, sold 500 tons of oil for
directors to transact its business. future delivery at P740 per ton. Due to break in the market, plaintiff was able
to purchase the oil at P380 per ton or a profit of P180,000.
SEC. 656. In accordance with a well-known rule of the law of agency,
notice to corporate officers or agents within the scope or apparent It appears from Exhibit B under the heading of "Profits" that:
scope of their authority is attributed to the corporation.
On all the business transacted between Willits & Patterson, Ltd. and
SEC. 667. As a general rule, if a corporation with knowledge of its others than Willits & Patterson, San Francisco, half the profit are to
agents unauthorized act received and enjoys the benefits thereof, it be credited to may account and half to the Profit & Loss account
impliedly ratifies the unauthorized act if it is one capable of Willits & Patterson, Ltd., Manila.
ratification by parol.

390
The purchasers paid P105,000 on the contracts and gave their notes for oil at the time it sold the oil. It appears that at the time of the sale the
P75,000, and it was agreed that all of the oil purchased should be held as defendant only received P105,000 in cash, and that it took and accepted the
security for the full payment of the purchase price. As a result, the defendant promissory notes of Cruz & Tan Chong Say, the purchasers, for P75,000
itself received the P105,000 in cash, P75,000 in notes, and still holds the 500 more which have been collected and may never be. Hence, it must follow
tons of oil as security for the balance of the purchase price. This transaction that the amount evidence by the notes cannot now be deemed or treated as
was shown in the semi-annual financial statement for the period ending profits on the deal and cannot be until such times as the notes are paid.
December 31, 1920. That is to say, the business was transacted by and
through the plaintiff, and the defendant received and accepted all of the The judgment of the lower court is reversed, and a money judgment will be
profits on the deal, and the statement which was rendered gave him a credit entered here in favor of the plaintiff and against the defendant for the sum of
for P90,737.88, or half the profit as provided in the contract Exhibit B, with P68,527.50, with thereon at the rate of 6 per cent per annum from the 10th
interest. day of January, 1922. In addition thereto, judgment will be rendered against
the defendant in substance and to the effect that the plaintiff is the owner of
Although the previous financial statements show upon their face that the an undivided one-half interest in the promissory notes for P75,000 which
account of plaintiff was credit with several small items on the same basis, it were executed by Cruz & Tan Chong Say, as a part of the purchase price of
was not until the 23d of March, 1921, that any objection was ever made by the oil, and that he is entitled to have and receive one-half of all the proceeds
anyone, and objection was made for the first time by the creditors' committee from the notes or either of them, and that also he have judgment against the
in a cable of that date. defendant for costs. So ordered.

As we analyze the facts Exhibit B was, in legal effect, ratified and approved Araullo, C. J., Street, Malcolm, Avancea, Ostrand, and Romualdez, JJ.,
and is now binding upon the defendant corporation, and the plaintiff is concur.
entitled to recover for his services on that writing as it modified the original
contract Exhibit A.

It appears from the statement prepared by accountant Larkin founded upon


Exhibit B that the plaintiff is entitled to recover P106,277.50. It is very
apparent that his statement was based upon the assumption that there was a
net profit of P180,000 on the 500 tons of oil, of which the plaintiff was entitled
to one-half.

In the absence of any other proof, we have the right to assume that the 500
G.R. No. L-5677 May 25, 1953
tons of oil was worth the amount which the defendant paid for them at the
time of the purchase or P380 per ton, and the record shows that the
defendant took and now has the possession of all of the oil secure the LA CAMPANA FACTORY, INC., and TAN TONG doing business under
payment of the price at which it was sold. Hence, the profit on the deal to the the trial name "LA CAMPANA GAUGAU PACKING", petitioners,
defendant at the time of the sale would amount to the difference between vs.
what the defendant paid for the oil and the amount which it received for the KAISAHAN NG MGA MANGGAGAWA SA LA CAMPANA (KKM) and THE
COURT OF INDUSTRIAL RELATIONS, respondents.

391
Ceferino de los Santos, R., Ceferino de los Santos, Jr. and Manuel V. Roxas On July 19, 1951, the Kaisahan Ng Mga Manggagawa Sa La Campana,
for petitioners. hereinafter to be referred to as the respondent Kaisahan, which, as of that
Carlos E. Santiago for respondent union. date, counted with 66 members workers all of them of both La Campana
Gaugau Packing and La Campana Coffee Factory Co., Inc. presented a
REYES, J.: demand for higher wages and more privileges, the demand being addressed
to La Campana Starch and Coffee Factory, by which name they sought to
Tan Tong, one of the herein petitioners, has since 1932 been engaged in the designate, so it appears, the La Campana Gaugau Packing and the La
business of buying and selling gaugau under the trade name La Campana Campana Coffee Factory Co., Inc. As the demand was not granted and an
Gaugau Packing with an establishment in Binondo, Manila, which was later attempt at settlement through the mediation of the Conciliation Service of the
transferred to Espaa Extension, Quezon City. But on July 6, 1950, Tan Department of Labor had given no result, the said Department certified the
Tong, with himself and members of his family corporation known as La dispute to the Court of Industrial Relations on July 17, 1951, the case being
Campana Factory Co., Inc., with its principal office located in the same place there docketed as Case No. 584-V.
as that of La Campana Gaugau Packing.
With the case already pending in the industrial court, the Secretary of Labor,
About a year before the formation of the corporation, or on July 11, 1949, Tan on September 5, 1951, revoked the Kalipunan Ng Mga Kaisahang
Tong had entered into a collective bargaining agreement with the Philippine Manggagawa's permit as a labor union on the strength of information
Legion of Organized Workers, known as PLOW for short, to which the union received that it was dominated by subversive elements, and, in
of Tan Tong's employees headed by Manuel E. Sadde was then affiliated. consequence, on the 20th of the same month, also suspended the permit of
Seceding, however, from the PLOW, Tan Tong's employees later formed their its affiliate, the respondent Kaisahan.
own organization known as Kaisahan Ng Mga Manggagawa Sa La
Campana, one of the herein respondents, and applied for registration in the We have it from the court's order of January 15, 1952, which forms one of the
Department of Labor as an independent entity. Pending consideration of this annexes to the present petition, that following the revocation of the
application, the Department gave the new organization legal standing by Kaisahan's permit, "La Campana Gaugau and Coffee Factory" (obviously the
issuing it a permit as an affiliate to the Kalipunan Ng Mga Manggagawa. combined name of La Campana Gaugau Packing and La Campana Coffee
Factory Co., Inc,) and the PLOW, which had been allowed to intervene as a
party having an interest in the dispute, filed separate motions for the
dismissal of the case on the following grounds:

1. That the action is directed against two different entities with


distinct personalities, with "La Campana Starch Factory" and the "La
Campana Coffee Factory, Inc.";

2. That the workers of the "La Campana Coffee Factory, Inc." are
less than thirty-one;

392
3. That the petitioning union has no legal capacity to sue, because its E. There has been only one payroll for the entire La Campana
registration as an organized union has been revoked by the personnel and only one person preparing the same Miss
Department of Labor on September 5, 1951; and Natividad Garcia, secretary of Mr. Tan Tong. But after the case at bar
was certified to this Court on July 17, 1951, the company began
4. That there is an existing valid contract between the respondent making separate payrolls for the coffee factory (Exhibits M-2 and M-
"La Campana Gaugau Packing" and the intervenor PLOW, where-in 3, and for the gaugau factory (Exhibits O-2, O-3 and O-4). It is to be
the petitioner's members are contracting parties bound by said noted that before July 21, 1951, the coffee payrolls all began with
contract. number "41-Maria Villanueva" with 24 or more laborers (Exhibits M
and M-1), whereas beginning July 21, 1951, the payrolls for the
Several hearings were held on the above motions, in the course of which coffee factory began with No. 1-Loreta Bernabe with only 14 laborers
ocular inspections were also made, and on the basis of the evidence (Exhibits M-2 and M-3).
received and the facts observed in the ocular inspections, the Court of
Industrial Relations denied the said motions in its order of January 14, 1952, F. During the ocular inspection made in the factory on August 26,
because if found as a fact that: 1951 the Court has found the following:

A. While the coffee corporation is a family corporation with Mr. Tan In the ground floor and second floor of the gaugau factory there were
Tong, his wife, and children as the incorporations and stockhelders hundreds of bags of raw coffee behind the pile of gaugau sacks.
(Exhibit 1), the La Campana Gaugau Packing is merely a business There were also women employees working paper wrappers for
name (Exhibit 4). gaugau, and, in the same place there were about 3,000 cans to be
used as containers for coffee.
B. According to the contract of lease (Exhibit 23), Mr. Tan Tong.,
propriety and manager of the Ka Campana Gaugau Factory, leased The Court found out also that there were 16 trucks used both for the
a space of 200 square meters in the bodega housing the gaugau delivery of coffee and gaugau. To show that those trucks carried both
factory to his son Tan Keng Lim, manager of the La Campana Coffee coffee and gaugau, the union president invited the Court to examine
Factory. But the lease was executed only on September 1, 1951, the contents of delivery truck No. T-582 parked in a garage between
while the dispute between the parties was pending before the Court. the gaugau building and the coffee factory, and upon examination,
there were found inside the said truck boxes of gaugau and cans of
C. There is only one entity La Campana Starch and Coffee Factory, coffee,
as shown by the signboard (Exhibit 1), the advertisement in the
delivery trucks (Exhibit I-1), the packages of gaugau(Exhibit K), and and held that:
delivery forms (Exhibits J, J-1, and J-2).
. . . there is only one management for the business of gaugau and
D. All the laborers working in the gaugau or in the coffee factory coffee with whom the laborers are dealing regarding their work.
receive their pay from the same person, the cashier, Miss Natividad Hence, the filing of action against the Ka Campana Starch and
Garcia, secretary of Mr. Tan Tong; and they are transferred from the Coffee Factory is proper and justified.
gaugau to the coffee and vice-versa as the management so requires.

393
With regards to the alleged lack of personality, it is to be noted that family. But it is settled that this fiction of law, which has been introduced as a
before the certification of the case to this Court on July 17, 1951, the matter of convenience and to subserve the ends of justice cannot be invoked
petitioner Kaisahan Ng Mga Manggagawa Sa La Campana, had a to further an end subversive of that purpose.
separate permit from the Department of Labor. This permit was
suspended on September 30, 1951. (Exhibit M-Intervenor, page 55, Disregarding Corporate Entity. The doctrine that a corporation is a
of the record). It is not true that, on July 17, 1951, when this case legal entity existing separate and apart from the person composing it
forwarded to this Court, the petitioner's permit, as an independent is a legal theory introduced for purposes of convenience and to
union, had not yet been issued, for the very Exhibit MM-Intervenor subserve the ends of justice. The concept cannot, therefore, be
regarding the permit, conclusively shows the preexistence of said extended to a point beyond its reason and policy, and when invoked
permit. (Annex G.) in support of an end subversive of this policy, will be disregarded by
the courts. Thus, in an appropriate case and in furtherance of the
Their motion for reconsideration of the above order having been denied, Tan ends of justice, a corporation and the individual or individuals owning
Tong and La Campana Coffee Factory, Inc. (same as La Campana Coffee all its stocks and assets will be treated as identical, the corporate
Factory Co., Inc.), later joined by the PLOW, filed the present petition for entity being disregarded where used as a cloak or cover for fraud or
certiorari on the grounds that the Court of Industrial Relations had no illegality. (13 Am. Jur., 160-161.)
jurisdiction to take cognizance of the case, for the reason, according to them,
"(1) that the petitioner La Campana Coffee Factory, Inc. has only 14 . . . A subsidiary or auxiliary corporation which is created by a parent
employees, only 5 of whom are members of the respondent union and corporation merely as an agency for the latter may sometimes be
therefore the absence of the jurisdictional number (30) as provided by regarded as identical with the parent corporation, especially if the
sections 1 and 4 of Commonwealth Act No. 103; and, (2) that the suspension stockholders or officers of the two corporations are substantially the
of respondent union's permit by the Secretary of Labor has the effect of same or their system of operation unified. (Ibid. 162; see Annotation
taking away the union's right to collective bargaining under section 2 of 1 A. L. R. 612, s. 34 A. L. R. 599.)
Commonwealth Act No. 213 and consequently, its personality to sue for ad in
behalf of its members." In the present case Tan Tong appears to be the owner of the gaugau factory.
And the coffee factory, though an incorporated business, is in reality owned
As to the first ground, petitioners obviously do not question the fact that the exclusively by Tan Tong and his family. As found by the Court of industrial
number of employees of the La Campana Gaugau Packing involved in the Relations, the two factories have but one office, one management and one
case is more than the jurisdictional number (31) required bylaw, but they do payroll, except after July 17, the day the case was certified to the Court of
contend that the industrial court has no jurisdiction to try the case as against Industrial Relations, when the person who was discharging the office of
La Campana Coffee Factory, Inc. because the latter has allegedly only 14 cashier for both branches of the business began preparing separate payrolls
laborers and only of these are members of the respondent Kaisahan. This for the two. And above all, it should not be overlooked that, as also found by
contention loses force when it is noted that, as found by the industrial court the industrial court, the laborers of the gaugau factory and the coffee factory
and this finding is conclusive upon us La Campana Gaugau Packing were interchangeable, that is, the laborers from the gaugau factory were
and La Campana Coffee Factory Co. Inc., are operating under one single sometimes transferred to the coffee factory and vice-versa. In view of all
management, that is, as one business though with two trade names. True, these, the attempt to make the two factories appears as two separate
the coffee factory is a corporation and, by legal fiction, an entity existing businesses, when in reality they are but one, is but a device to defeat the
separate and apart fro the persons composing it, that is, Tan Tong and his

394
ends of the law (the Act governing capital and labor relations) and should not
be permitted to prevail.

The second point raised by petitioners is likewise with-out merit. In the first
place, there being more than 30 laborers involved and the Secretary of Labor
having certified the dispute to the Court of Industrial Relations, that court duly
acquired jurisdiction over the case (International Oil Factory vs. NLU, Inc. 73
Phil., 401; section 4, C. A. 103). This jurisdiction was not when the
Department of Labor suspended the permit of the respondent Kaisahan as a
labor organization. For once jurisdiction is acquired by the Court of Industrial
Relations it is retained until the case is completely decided. (Manila Hotel
Employees Association vs. Manila Hotel Co. et al., 73 Phil., 374.)

In view of the foregoing, the petition is denied, with costs against the
petitioner.

Paras, C.J., Feria, Pablo, Bengzon, Tuason, Montemayor, Jugo, Bautista


Angelo and Labrador, JJ., concur.

395
YUTIVO SONS HARDWARE COMPANY, petitioner,
vs.
COURT OF TAX APPEALS and COLLECTOR OF INTERNAL
REVENUE, respondents.

Sycip, Quisumbing, Salazar & Associates for petitioner.


Office of the Solicitor General for respondents.

GUTIERREZ DAVID, J.:

This is a petition for review of a decision of the Court of Tax Appeals ordering
petitioner to pay to respondent Collector of Internal Revenue the sum of
P1,266,176.73 as sales tax deficiency for the third quarter of 1947 to the
fourth quarter of 1950; inclusive, plus 75% surcharge thereon, equivalent to
P349,632.54, or a sum total of P2,215,809.27, plus costs of the suit.

From the stipulation of facts and the evidence adduced by both parties, it
appears that petitioner Yutivo Sons Hardware Co. (hereafter referred to as
Yutivo) is a domestic corporation, organized under the laws of the
Philippines, with principal office at 404 Dasmarias St., Manila. Incorporated
in 1916, it was engaged, prior to the last world war, in the importation and
sale of hardware supplies and equipment. After the liberation, it resumed its
business and until June of 1946 bought a number of cars and trucks from
General Motors Overseas Corporation (hereafter referred to as GM for
short), an American corporation licensed to do business in the Philippines. As
importer, GM paid sales tax prescribed by sections 184, 185 and 186 of the
Tax Code on the basis of its selling price to Yutivo. Said tax being collected
only once on original sales, Yutivo paid no further sales tax on its sales to the
public.

On June 13, 1946, the Southern Motors, Inc. (hereafter referred to as SM)
was organized to engage in the business of selling cars, trucks and spare
parts. Its original authorized capital stock was P1,000,000 divided into 10,000
shares with a par value of P100 each.
G.R. No. L-13203 January 28, 1961

396
At the time of its incorporation 2,500 shares worth P250,000 appear to have gathered to sustain the assessment of this Office based on the theory that
been subscribed into equal proportions by Yu Khe Thai, Yu Khe Siong, Hu Southern Motors is a mere instrumentality or subsidiary of Yutivo." The
Kho Jin, Yu Eng Poh, and Washington Sycip. The first three named withdrawal was subject, however, to the general power of review by the now
subscribers are brothers, being sons of Yu Tiong Yee, one of Yutivo's defunct Board of Tax Appeals. The Secretary of Finance to whom the papers
founders. The latter two are respectively sons of Yu Tiong Sin and Albino relative to the case were endorsed, apparently not agreeing with the
Sycip, who are among the founders of Yutivo. withdrawal of the assessment, returned them to the respondent Collector for
reinvestigation.
After the incorporation of SM and until the withdrawal of GM from the
Philippines in the middle of 1947, the cars and tracks purchased by Yutivo After another investigation, the respondent Collector, in a letter to petitioner
from GM were sold by Yutivo to SM which, in turn, sold them to the public in dated December 16, 1954, redetermined that the aforementioned tax
the Visayas and Mindanao. assessment was lawfully due the government and in addition assessed
deficiency sales tax due from petitioner for the four quarters of 1950; the
When GM decided to withdraw from the Philippines in the middle of 1947, the respondents' last demand was in the total sum of P2,215,809.27 detailed as
U.S. manufacturer of GM cars and trucks appointed Yutivo as importer for follows:
the Visayas and Mindanao, and Yutivo continued its previous arrangement of
selling exclusively to SM. In the same way that GM used to pay sales taxes
based on its sales to Yutivo, the latter, as importer, paid sales tax prescribed Deficiency 75% Total Am
on the basis of its selling price to SM, and since such sales tax, as already Sales Tax Surcharge Due
stated, is collected only once on original sales, SM paid no sales tax on its
sales to the public.

On November 7, 1950, after several months of investigation by revenue Assessment (First) of November 7, 1950
officers started in July, 1948, the Collector of Internal Revenue made an for deficiency sales Tax for the period
assessment upon Yutivo and demanded from the latter P1,804,769.85 as from 3rd Qrtr 1947 to 4th Qrtr 1949
deficiency sales tax plus surcharge covering the period from the third quarter inclusive P1,031,296.60 P773,473.45 P1,804,7
of 1947 to the fourth quarter of 1949; or from July 1, 1947 to December 31,
1949, claiming that the taxable sales were the retail sales by SM to the public
and not the sales at wholesale made by, Yutivo to the latter inasmuch as SM
and Yutivo were one and the same corporation, the former being the Additional Assessment for period from
subsidiary of the latter. 1st to 4th Qrtr 1950, inclusive 234,880.13 176,160.09 411,040.2

The assessment was disputed by the petitioner, and a reinvestigation of the


case having been made by the agents of the Bureau of Internal Revenue, the Total amount demanded per letter of
respondent Collector in his letter dated November 15, 1952 countermanded December 16, 1954 P1,266,176.73 P949,632.54 P2,215,8
his demand for sales tax deficiency on the ground that "after several
investigations conducted into the matter no sufficient evidence could be

397
This second assessment was contested by the petitioner Yutivo before the After going over the voluminous record of the present case, we are inclined
Court of Tax Appeals, alleging that there is no valid ground to disregard the to rule that the Court of Tax Appeals was not justified in finding that SM was
corporate personality of SM and to hold that it is an adjunct of petitioner organized for no other purpose than to defraud the Government of its lawful
Yutivo; (2) that assuming the separate personality of SM may be revenues. In the first place, this corporation was organized in June, 1946
disregarded, the sales tax already paid by Yutivo should first be deducted when it could not have caused Yutivo any tax savings. From that date up to
from the selling price of SM in computing the sales tax due on each vehicle; June 30, 1947, or a period of more than one year, GM was the importer of
and (3) that the surcharge has been erroneously imposed by respondent. the cars and trucks sold to Yutivo, which, in turn resold them to SM. During
Finding against Yutivo and sustaining the respondent Collector's theory that that period, it is not disputed that GM as importer, was the one solely liable
there was no legitimate or bona fide purpose in the organization of SM the for sales taxes. Neither Yutivo or SM was subject to the sales taxes on their
apparent objective of its organization being to evade the payment of taxes sales of cars and trucks. The sales tax liability of Yutivo did not arise until
and that it was owned (or the majority of the stocks thereof are owned) and July 1, 1947 when it became the importer and simply continued its practice of
controlled by Yutivo and is a mere subsidiary, branch, adjunct, conduit, selling to SM. The decision, therefore, of the Tax Court that SM was
instrumentality or alter ego of the latter, the Court of Tax Appeals with organized purposely as a tax evasion device runs counter to the fact that
Judge Roman Umali not taking part disregarded its separate corporate there was no tax to evade.
existence and on April 27, 1957, rendered the decision now complained of.
Of the two Judges who signed the decision, one voted for the modification of Making the observation from a newspaper clipping (Exh. "T") that "as early
the computation of the sales tax as determined by the respondent Collector as 1945 it was known that GM was preparing to leave the Philippines and
in his decision so as to give allowance for the reduction of the tax already terminate its business of importing vehicles," the court below speculated that
paid (resulting in the reduction of the assessment to P820,509.91 exclusive Yutivo anticipated the withdrawal of GM from business in the Philippines in
of surcharges), while the other voted for affirmance. The dispositive part of June, 1947. This observation, which was made only in the resolution on the
the decision, however, affirmed the assessment made by the Collector. motion for reconsideration, however, finds no basis in the record. On the
Reconsideration of this decision having been denied, Yutivo brought the case other hand, GM had been an importer of cars in the Philippines even before
to this Court thru the present petition for review. the war and had but recently resumed its operation in the Philippines in 1946
under an ambitious plan to expand its operation by establishing an assembly
It is an elementary and fundamental principle of corporation law that a plant here, so that it could not have been expected to make so drastic a
corporation is an entity separate and distinct from its stockholders and from turnabout of not merely abandoning the assembly plant project but also
other corporation petitions to which it may be connected. However, "when the totally ceasing to do business as an importer. Moreover, the newspaper
notion of legal entity is used to defeat public convenience, justify wrong, clipping, Exh. "T", was published on March 24, 1947, and clipping, merely
protect fraud, or defend crime," the law will regard the corporation as an reported a rumored plan that GM would abandon the assembly plant project
association of persons, or in the case of two corporations merge them into in the Philippines. There was no mention of the cessation of business by GM
one. (Koppel [Phil.], Inc. vs. Yatco, 77 Phil. 496, citing I Fletcher Cyclopedia which must not be confused with the abandonment of the assembly plant
of Corporation, Perm Ed., pp. 135 136; United States vs. Milwaukee project. Even as respect the assembly plant, the newspaper clipping was
Refrigeration Transit Co., 142 Fed., 247, 255 per Sanborn, J.) Another rule is quite explicit in saying that the Acting Manager refused to confirm that rumor
that, when the corporation is the "mere alter ego or business conduit of a as late as March 24, 1947, almost a year after SM was organized.
person, it may be disregarded." (Koppel [Phil.], Inc. vs. Yatco, supra.)
At this juncture, it should be stated that the intention to minimize taxes, when
used in the context of fraud, must be proved to exist by clear and convincing

398
evidence amounting to more than mere preponderance, and cannot be In the third place, sections 184 to 186 of the said Code provides that the
justified by a mere speculation. This is because fraud is never lightly to be sales tax shall be collected "once only on every original sale, barter,
presumed. (Vitelli & Sons vs. U.S 250 U.S. 355; Duffin vs. Lucas, 55 F (2d) exchange . . , to be paid by the manufacturer, producer or importer." The use
786; Budd vs. Commr., 43 F (2d) 509; Maryland Casualty Co. vs. Palmette of the word "original" and the express provision that the tax was collectible
Coal Co., 40 F (2d) 374; Schoonfield Bros., Inc. vs. Commr., 38 BTA 943; "once only" evidently has made the provisions susceptible of different
Charles Heiss vs. Commr 36 BTA 833; Kerbaugh vs. Commr 74 F (2d) 749; interpretations. In this connection, it should be stated that a taxpayer has the
Maddas vs. Commr., 114 F. (2d) 548; Moore vs. Commr., 37 BTA 378; legal right to decrease the amount of what otherwise would be his taxes or
National City Bank of New York vs. Commr., 98 (2d) 93; Richard vs. Commr., altogether avoid them by means which the law permits. (U.S. vs. Isham 17
15 BTA 316; Rea Gane vs. Commr., 19 BTA 518). (See also Balter, Fraud Wall. 496, 506; Gregory vs. Helvering 293 U.S. 465, 469; Commr. vs. Tower,
Under Federal Law, pp. 301-302, citing numerous authorities: Arroyo vs. 327 U.S. 280; Lawton vs. Commr 194 F (2d) 380). Any legal means by the
Granada, et al., 18 Phil. 484.) Fraud is never imputed and the courts never taxpayer to reduce taxes are all right Benry vs. Commr. 25 T. Cl. 78). A man
sustain findings of fraud upon circumstances which, at the most, create only may, therefore, perform an act that he honestly believes to be sufficient to
suspicion. (Haygood Lumber & Mining Co. vs. Commr., 178 F (2d) 769; exempt him from taxes. He does not incur fraud thereby even if the act is
Dalone vs. Commr., 100 F (2d) 507). thereafter found to be insufficient. Thus in the case of Court Holding Co. vs.
Commr. 2 T. Cl. 531, it was held that though an incorrect position in law had
In the second place, SM was organized and it operated, under circumstance been taken by the corporation there was no suppression of the facts, and a
that belied any intention to evade sales taxes. "Tax evasion" is a term that fraud penalty was not justified.
connotes fraud thru the use of pretenses and forbidden devices to lessen or
defeat taxes. The transactions between Yutivo and SM, however, have The evidence for the Collector, in our opinion, falls short of the standard of
always been in the open, embodied in private and public documents, clear and convincing proof of fraud. As a matter of fact, the respondent
constantly subject to inspection by the tax authorities. As a matter of fact, Collector himself showed a great deal of doubt or hesitancy as to the
after Yutivo became the importer of GM cars and trucks for Visayas and existence of fraud. He even doubted the validity of his first assessment dated
Mindanao, it merely continued the method of distribution that it had initiated November 7, 1959. It must be remembered that the fraud which respondent
long before GM withdrew from the Philippines. Collector imputed to Yutivo must be related to its filing of sales tax returns of
less taxes than were legally due. The allegation of fraud, however, cannot be
On the other hand, if tax saving was the only justification for the organization sustained without the showing that Yutivo, in filing said returns, did so fully
of SM, such justification certainly ceased with the passage of Republic Act knowing that the taxes called for therein called for therein were less than
No. 594 on February 16, 1951, governing payment of advance sales tax by what were legally due. Considering that respondent Collector himself with the
the importer based on the landed cost of the imported article, increased by aid of his legal staff, and after some two years of investigation and duty of
mark-ups of 25%, 50%, and 100%, depending on whether the imported investigation and study concluded in 1952 that Yutivo's sales tax returns
article is taxed under sections 186, 185 and 184, respectively, of the Tax were correct only to reverse himself after another two years it would
Code. Under Republic Act No. 594, the amount at which the article is sold is seem harsh and unfair for him to say in 1954 that Yutivo fully knew in
immaterial to the amount of the sales tax. And yet after the passage of that October 1947 that its sales tax returns were inaccurate.
Act, SM continued to exist up to the present and operates as it did many
years past in the promotion and pursuit of the business purposes for which it On this point, one other consideration would show that the intent to save
was organized. taxes could not have existed in the minds of the organizers of SM. The sales
tax imposed, in theory and in practice, is passed on to the vendee, and is

399
usually billed separately as such in the sales invoice. As pointed out by the aforenamed subscribers, but actually the said sum was advanced by
petitioner Yutivo, had not SM handled the retail, the additional tax that would Yutivo. The additional subscriptions to the capital stock of SM and
have been payable by it, could have been easily passed off to the consumer, subsequent transfers thereof were paid by Yutivo itself. The payments were
especially since the period covered by the assessment was a "seller's made, however, without any transfer of funds from Yutivo to SM. Yutivo
market" due to the post-war scarcity up to late 1948, and the imposition of simply charged the accounts of the subscribers for the amount allegedly
controls in the late 1949. advanced by Yutivo in payment of the shares. Whether a charge was to be
made against the accounts of the subscribers or said subscribers were to
It is true that the arrastre charges constitute expenses of Yutivo and its non- subscribe shares appears to constitute a unilateral act on the part of Yutivo,
inclusion in the selling price by Yutivo cost the Government P4.00 per there being no showing that the former initiated the subscription.
vehicle, but said non-inclusion was explained to have been due to an
inadvertent accounting omission, and could hardly be considered as proof of The transactions were made solely by and between SM and Yutivo. In effect,
willful channelling and fraudulent evasion of sales tax. Mere understatement it was Yutivo who undertook the subscription of shares, employing the
of tax in itself does not prove fraud. (James Nicholson, 32 BTA 377, affirmed persons named or "charged" with corresponding account as nominal
90 F. (2) 978, cited in Merten's Sec. 55.11 p. 21) The amount involved, stockholders. Of course, Yu Khe Thai, Yu Khe Jin, Yu Khe Siong and Yu Eng
moreover, is extremely small inducement for Yutivo to go thru all the trouble Poh were manifestly aware of these subscriptions, but considering that they
of organizing SM. Besides, the non-inclusion of these small arrastre charges were the principal officers and constituted the majority of the Board of
in the sales tax returns of Yutivo is clearly shown in the records of Yutivo, Directors of both Yutivo and SM, their subscriptions could readily or easily be
which is uncharacteristic of fraud (See Insular Lumber Co. vs. Collector, G.R. that of Yutivo's Moreover, these persons were related to death other as
No. L-719, April 28, 1956.) brothers or first cousins. There was every reason for them to agree in order
to protect their common interest in Yutivo and SM.
We are, however, inclined to agree with the court below that SM was actually
owned and controlled by petitioner as to make it a mere subsidiary or branch The issued capital stock of SM was increased by additional subscriptions
of the latter created for the purpose of selling the vehicles at retail and made by various person's but except Ng Sam Bak and David Sycip,
maintaining stores for spare parts as well as service repair shops. It is not "payments" thereof were effected by merely debiting 'or charging the
disputed that the petitioner, which is engaged principally in hardware supplies accounts of said stockholders and crediting the corresponding amounts in
and equipment, is completely controlled by the Yutivo, Young or Yu family. favor of SM, without actually transferring cash from Yutivo. Again, in this
The founders of the corporation are closely related to each other either by instance, the "payments" were Yutivo, by effected by the mere unilateral act
blood or affinity, and most of its stockholders are members of the Yu (Yutivo of Yutivo a accounts of the virtue of its control over the individual persons
or Young) family. It is, likewise, admitted that SM was organized by the charged, would necessarily exercise preferential rights and control directly or
leading stockholders of Yutivo headed by Yu Khe Thai. At the time of its indirectly, over the shares, it being the party which really undertook to pay or
incorporation 2,500 shares worth P250,000.00 appear to have been underwrite payment thereof.
subscribed in five equal proportions by Yu Khe Thai, Yu Khe Siong, Yu Khe
Jin, Yu Eng Poh and Washington Sycip. The first three named subscribers The shareholders in SM are mere nominal stockholders holding the shares
are brothers, being the sons of Yu Tien Yee, one of Yutivo's founders. Yu Eng for and in behalf of Yutivo, so even conceding that the original subscribers
Poh and Washington Sycip are respectively sons of Yu Tiong Sing and were stockholders bona fide Yutivo was at all times in control of the majority
Alberto Sycip who are co-founders of Yutivo. According to the Articles of of the stock of SM and that the latter was a mere subsidiary of the former.
Incorporation of the said subscriptions, the amount of P62,500 was paid by

400
True, petitioner and other recorded stockholders transferred their Yutivo's corporate officers, without any copy thereof being furnished to SM.
shareholdings, but the transfers were made to their immediate relatives, All detailed records such as cash disbursements, such as expenses,
either to their respective spouses and children or sometimes brothers or purchases, etc. for the account of SM, are kept by Yutivo and SM merely
sisters. Yutivo's shares in SM were transferred to immediate relatives of keeps a summary record thereof on the basis of information received from
persons who constituted its controlling stockholders, directors and officers. Yutivo.
Despite these purported changes in stock ownership in both corporations,
the Board of Directors and officers of both corporations remained unchanged All the above plainly show that cash or funds of SM, including those of its
and Messrs. Yu Khe Thai, Yu Khe Siong Hu Khe Jin and Yu Eng Poll (all of branches which are directly remitted to Yutivo, are placed in the custody and
the Yu or Young family) continued to constitute the majority in both boards. control of Yutivo, resources and subject to withdrawal only by Yutivo. SM's
All these, as observed by the Court of Tax Appeals, merely serve to being under Yutivo's control, the former's operations and existence became
corroborate the fact that there was a common ownership and interest in the dependent upon the latter.
two corporations.
Consideration of various other circumstances, especially when taken
SM is under the management and control of Yutivo by virtue of a together, indicates that Yutivo treated SM merely as its department or
management contract entered into between the two parties. In fact, the adjunct. For one thing, the accounting system maintained by Yutivo shows
controlling majority of the Board of Directors of Yutivo is also the controlling that it maintained a high degree of control over SM accounts. All transactions
majority of the Board of Directors of SM. At the same time the principal between Yutivo and SM are recorded and effected by mere debit or credit
officers of both corporations are identical. In addition both corporations have entries against the reciprocal account maintained in their respective books of
a common comptroller in the person of Simeon Sy, who is a brother-in-law of accounts and indicate the dependency of SM as branch upon Yutivo.
Yutivo's president, Yu Khe Thai. There is therefore no doubt that by virtue of
such control, the business, financial and management policies of both Apart from the accounting system, other facts corroborate or independently
corporations could be directed towards common ends. show that SM is a branch or department of Yutivo. Even the branches of SM
in Bacolod, Iloilo, Cebu, and Davao treat Yutivo Manila as their "Head
Another aspect relative to Yutivo's control over SM operations relates to its Office" or "Home Office" as shown by their letters of remittances or other
cash transactions. All cash assets of SM were handled by Yutivo and all cash correspondences. These correspondences were actually received by Yutivo
transactions of SM were actually maintained thru Yutivo. Any and all receipts and the reference to Yutivo as the head or home office is obvious from the
of cash by SM including its branches were transmitted or transferred fact that all cash collections of the SM's branches are remitted directly to
immediately and directly to Yutivo in Manila upon receipt thereof. Likewise, Yutivo. Added to this fact, is that SM may freely use forms or stationery of
all expenses, purchases or other obligations incurred by SM are referred to Yutivo
Yutivo which in turn prepares the corresponding disbursement vouchers and
payments in relation there, the payment being made out of the cash deposits The fact that SM is a mere department or adjunct of Yutivo is made more
of SM with Yutivo, if any, or in the absence thereof which occurs generally, a patent by the fact that arrastre conveying, and charges paid for the
corresponding charge is made against the account of SM in Yutivo's books. "operation of receiving, loading or unloading" of imported cars and trucks on
The payments for and charges against SM are made by Yutivo as a matter of piers and wharves, were charged against SM. Overtime charges for the
course and without need of any further request, the latter would advance all unloading of cars and trucks as requested by Yutivo and incurred as part of
such cash requirements for the benefit of SM. Any and all payments and its acquisition cost thereof, were likewise charged against and treated as
cash vouchers are made on Yutivo stationery and made under authority of

401
expenses of SM. If Yutivo were the importer, these arrastre and overtime 1947 to 1949. The corresponding returns filed by petitioner covering the said
charges were Yutivo's expenses in importing goods and not SM's. But since period was made at the earliest on October 1, as regards the third quarter of
those charges were made against SM, it plainly appears that Yutivo had sole 1947, so that it cannot be claimed that the assessment was not made within
authority to allocate its expenses even as against SM in the sense that the the five-year period prescribed in section 331 of the Tax Code invoked by
latter is a mere adjunct, branch or department of the former. petitioner. The assessment, it is admitted, was withdrawn by the Collector on
insufficiency of evidence, but November 15, 1952 due to insufficiency of
Proceeding to another aspect of the relation of the parties, the management evidence, but the withdrawal was made subject to the approval of the
fees due from SM to Yutivo were taken up as expenses of SM and credited Secretary of Finance and the Board of Tax Appeals, pursuant to the
to the account of Yutivo. If it were to be assumed that the two organizations provisions of section 9 of Executive Order No. 401-A, series of 1951. The
are separate juridical entities, the corresponding receipts or receivables decision of the previous assessment of November 7, Collector
should have been treated as income on the part of Yutivo. But such countermanding the as 1950 was forwarded to the Board of Tax Appeals
management fees were recorded as "Reserve for Bonus" and were therefore through the Secretary of Finance but that official, apparently disagreeing with
a liability reserve and not an income account. This reserve for bonus were the decision, sent it back for re-investigation. Consequently, the assessment
subsequently distributed directly to and credited in favor of the employees of November 7, 1950 cannot be considered to have been finally withdrawn.
and directors of Yutivo, thereby clearly showing that the management fees That the assessment was subsequently reiterated in the decision of
were paid directly to Yutivo officers and employees. respondent Collector on December 16, 1954 did not alter the fact that it was
made seasonably. In this connection, it would appear that a warrant of
Briefly stated, Yutivo financed principally, if not wholly, the business of SM distraint and levy had been issued on March 28, 1951 in relation with this
and actually extended all the credit to the latter not only in the form of starting case and by virtue thereof the properties of Yutivo were placed under
capital but also in the form of credits extended for the cars and vehicles constructive distraint. Said warrant and constructive distraint have not been
allegedly sold by Yutivo to SM as well as advances or loans for the expenses lifted up to the present, which shows that the assessment of November 7,
of the latter when the capital had been exhausted. Thus, the increases in the 1950 has always been valid and subsisting.
capital stock were made in advances or "Guarantee" payments by Yutivo and
credited in favor of SM. The funds of SM were all merged in the cash fund of Anent the deficiency sale tax for 1950, considering that the assessment
Yutivo. At all times Yutivo thru officers and directors common to it and SM, thereof was made on December 16, 1954, the same was assessed well
exercised full control over the cash funds, policies, expenditures and within the prescribed five-year period.
obligations of the latter.
Petitioner argues that the original assessment of November 7, 1950 did not
Southern Motors being but a mere instrumentality, or adjunct of Yutivo, the extend the prescriptive period on assessment. The argument is untenable,
Court of Tax Appeals correctly disregarded the technical defense of separate for, as already seen, the assessment was never finally withdrawn, since it
corporate entity in order to arrive at the true tax liability of Yutivo. was not approved by the Secretary of Finance or of the Board of Tax
Appeals. The authority of the Secretary to act upon the assessment cannot
Petitioner contends that the respondent Collector had lost his right or be questioned, for he is expressly granted such authority under section 9 of
authority to issue the disputed assessment by reason of prescription. The Executive Order No. 401-And under section 79 (c) of the Revised
contention, in our opinion, cannot be sustained. It will be noted that the first Administrative Code, he has "direct control, direction and supervision over all
assessment was made on November 7, 1950 for deficiency sales tax from bureaus and offices under his jurisdiction and may, any provision of existing

402
law to the contrary not withstanding, repeal or modify the decision of the chief control upon the powers of the Collector of Internal Revenue in respect to
of said Bureaus or offices when advisable in public interest." assessment and refunds of taxes. If it be conceded that a decision of the
Collector of Internal Revenue on partial remission of taxes is subject to
It should here also be stated that the assessment in question was review by the Secretary of Finance and the Board of Tax Appeals, then with
consistently protested by petitioner, making several requests for more reason should the power of the Collector to withdraw totally an
reinvestigation thereof. Under the circumstances, petitioner may be assessment be subject to such review.
considered to have waived the defense of prescription.
We find merit, however, in petitioner's contention that the Court of Tax
"Estoppel has been employed to prevent the application of the Appeals erred in the imposition of the 5% fraud surcharge. As already shown
statute of limitations against the government in certain instances in in the early part of this decision, no element of fraud is present.
which the taxpayer has taken some affirmative action to prevent the
collection of the tax within the statutory period. It is generally held Pursuant to Section 183 of the National Internal Revenue Code the 50%
that a taxpayer is estopped to repudiate waivers of the statute of surcharge should be added to the deficiency sales tax "in case a false or
limitations upon which the government relied. The cases frequently fraudulent return is willfully made." Although the sales made by SM are in
involve dissolved corporations. If no waiver has been given, the substance by Yutivo this does not necessarily establish fraud nor the willful
cases usually show come conduct directed to a postponement of filing of a false or fraudulent return.
collection, such, for example, as some variety of request to apply an
overassessment. The taxpayer has 'benefited' and 'is not in a The case of Court Holding Co. v. Commissioner of Internal Revenue (August
position to contest' his tax liability. A definite representation of implied 9, 1943, 2 TC 531, 541-549) is in point. The petitioner Court Holding Co. was
authority may be involved, and in many cases the taxpayer has a corporation consisting of only two stockholders, to wit: Minnie Miller and
received the 'benefit' of being saved from the inconvenience, if not her husband Louis Miller. The only assets of third husband and wife
hardship of immediate collection. " corporation consisted of an apartment building which had been acquired for a
very low price at a judicial sale. Louis Miller, the husband, who directed the
Conceivably even in these cases a fully informed Commissioner may company's business, verbally agreed to sell this property to Abe C. Fine and
err to the sorrow of the revenues, but generally speaking, the cases Margaret Fine, husband and wife, for the sum of $54,000.00, payable in
present a strong combination of equities against the taxpayer, and various installments. He received $1,000.00 as down payment. The sale of
few will seriously quarrel with their application of the doctrine of this property for the price mentioned would have netted the corporation a
estoppel." (Mertens Law of Federal Income Taxation, Vol. 10-A, pp. handsome profit on which a large corporate income tax would have to be
159-160.) paid. On the afternoon of February 23, 1940, when the Millers and the Fines
got together for the execution of the document of sale, the Millers announced
It is also claimed that section 9 of Executive Order No. 401-A, series of 1951 that their attorney had called their attention to the large corporate tax which
es involving an original assessment of more than P5,000 refers only to would have to be paid if the sale was made by the corporation itself. So
compromises and refunds of taxes, but not to total withdrawal of the instead of proceeding with the sale as planned, the Millers approved a
assessment. The contention is without merit. A careful examination of the resolution to declare a dividend to themselves "payable in the assets of the
provisions of both sections 8 and 9 of Executive Order No. 401-A, series of corporation, in complete liquidation and surrender of all the outstanding
1951, reveals the procedure prescribed therein is intended as a check or corporate stock." The building, which as above stated was the only property
of the corporation, was then transferred to Mr. and Mrs. Miller who in turn

403
sold it to Mr. and Mrs. Fine for exactly the same price and under the same denying fraud and averring that the loss reported on its return was
terms as had been previously agreed upon between the corporation and the correct to the best of its knowledge and belief. We think the
Fines. respondent has not sustained the burden of proving a fraudulent
intent. We have concluded that the sale of the petitioner's property
The return filed by the Court Holding Co. with the respondent Commissioner was in substance a sale by the petitioner, and that the liquidating
of Internal Revenue reported no taxable gain as having been received from dividend to stockholders had no purpose other than that of tax
the sale of its assets. The Millers, of course, reported a long term capital gain avoidance. But the attempt to avoid tax does not necessarily
on the exchange of their corporate stock with the corporate property. The establish fraud. It is a settled principle that a taxpayer may diminish
Commissioner of Internal Revenue contended that the liquidating dividend to his liability by any means which the law permits. United States v.
stockholders had no purpose other than that of tax avoidance and that, Isham, 17 Wall. 496; Gregory v. Helvering, supra; Chrisholm v.
therefore, the sale by the Millers to the Fines of the corporation's property Commissioner, 79 Fed. (2d) 14. If the petitioner here was of the
was in substance a sale by the corporation itself, for which the corporation is opinion that the method by which it attempted to effect the sale in
subject to the taxable profit thereon. In requiring the corporation to pay the question was legally sufficient to avoid the imposition of tax upon it,
taxable profit on account of the sale, the Commissioner of Internal Revenue, its adoption of that method is not subject to censure. Petitioner took
imposed a surcharge of 25% for delinquency, plus an additional surcharge as a position with respect to a question of law, the substance of which
fraud penalties. was disclosed by the statement endorsed on its return. We can not
say, under the record before us, that that position was taken
The U. S. Court of Tax Appeals held that the sale by the Millers was for no fraudulently. The determination of the fraud penalties is reversed."
other purpose than to avoid the tax and was, in substance, a sale by the
Court Holding Co., and that, therefore, the said corporation should be liable When GM was the importer and Yutivo, the wholesaler, of the cars and
for the assessed taxable profit thereon. The Court of Tax Appeals also trucks, the sales tax was paid only once and on the original sales by the
sustained the Commissioner of Internal Revenue on the delinquency penalty former and neither the latter nor SM paid taxes on their subsequent sales.
of 25%. However, the Court of Tax Appeals disapproved the fraud penalties, Yutivo might have, therefore, honestly believed that the payment by it, as
holding that an attempt to avoid a tax does not necessarily establish fraud; importer, of the sales tax was enough as in the case of GM Consequently, in
that it is a settled principle that a taxpayer may diminish his tax liability by filing its return on the basis of its sales to SM and not on those by the latter to
means which the law permits; that if the petitioner, the Court Holding Co., the public, it cannot be said that Yutivo deliberately made a false return for
was of the opinion that the method by which it attempted to effect the sale in the purpose of defrauding the government of its revenues which will justify
question was legally sufficient to avoid the imposition of a tax upon it, its the imposition of the surcharge penalty.
adoption of that methods not subject to censure; and that in taking a position
with respect to a question of law, the substance of which was disclosed by We likewise find meritorious the contention that the Tax Court erred in
the statement indorsed on it return, it may not be said that that position was computing the alleged deficiency sales tax on the selling price of SM without
taken fraudulently. We quote in full the pertinent portion of the decision of the previously deducting therefrom the sales tax due thereon. The sales tax
Court of Tax Appeals: . provisions (sees. 184.186, Tax Code) impose a tax on original sales
measured by "gross selling price" or "gross value in money". These terms, as
". . . The respondent's answer alleges that the petitioner's failure to interpreted by the respondent Collector, do not include the amount of the
report as income the taxable profit on the real estate sale was sales tax, if invoiced separately. Thus, General Circular No. 431 of the
fraudulent and with intent to evade the tax. The petitioner filed a reply

404
Bureau of Internal Revenue dated July 29, 1939, which implements sections . . . is unfair, because . . .(it is) practically imposing tax on a tax
184.186 of the Tax Code provides: " already paid. Besides, the adoption of the procedure would in certain
cases elevate the bracket under which the tax is based. The late
. . .'Gross selling price' or gross value in money' of the articles sold, payment is already penalized, thru the imposition of surcharges, by
bartered, exchanged, transferred as the term is used in the adopting the theory of the Collector, we will be creating an additional
aforecited sections (sections 184, 185 and 186) of the National penalty not contemplated by law."
Internal Revenue Code, is the total amount of money or its
equivalent which the purchaser pays to the vendor to receive or get If the taxes based on the sales of SM are computed in accordance with Gen.
the goods. However, if a manufacturer, producer, or importer, in fixing Circulars Nos. 431 and 440 the total deficiency sales taxes, exclusive of the
the gross selling price of an article sold by him has included an 25% and 50% surcharges for late payment and for fraud, would amount only
amount intended to cover the sales tax in the gross selling price of to P820,549.91 as shown in the following computation:
the articles, the sales tax shall be based on the gross selling price
less the amount intended to cover the tax, if the same is billed to the
purchaser as a separate item.
Sales Taxes Due
Gross Sales of Total Gross Selling
Rates of and Computed
General Circular No. 440 of the same Bureau reads: Vehicles Exclusive Price Charged to
Sales Tax under Gen. Cir
of Sales Tax the Public
Nos. 431 & 400
Amount intended to cover the tax must be billed as a separate em so
as not to pay a tax on the tax. On sales made after he third
quarter of 1939, the amount intended to cover the sales tax must be
billed to the purchaser as separate items in the, invoices in order that 5% P11,912,219.57 P595,610.98 P12,507,83055
the reduction thereof from the gross ailing price may be allowed in
the computation of the merchants' percentage tax on the sales.
Unless billed to the purchaser as a separate item in the invoice, the
amounts intended to cover the sales tax shall be considered as part 7% 909,559.50 63,669.16 973,228.66
of the gross selling price of the articles sold, and deductions thereof
will not be allowed, (Cited in Dalupan, Nat. Int. Rev. Code,
Annotated, Vol. II, pp. 52-53.) 10% 2,618,695.28 261,869.53 2,880,564.81

Yutivo complied with the above circulars on its sales to SM, and as
separately billed, the sales taxes did not form part of the "gross selling price"
as the measure of the tax. Since Yutivo had previously billed the sales tax 15% 3,602,397.65 540,359.65 4,142,757.30
separately in its sales invoices to SM General Circulars Nos. 431 and 440
should be deemed to have been complied. Respondent Collector's method of
computation, as opined by Judge Nable in the decision complained of
20% 267,150.50 53,430.10 320,580.60

405
two judges shall be necessary to promulgate decision thereof. . . . " It is on
30% 837,146.97 251,114.09 1,088,291.06 record that the present case was heard by two judges of the lower court. And
while Judge Nable expressed his opinion on the issue of whether or not the
amount of the sales tax should be excluded from the gross selling price in
computing the deficiency sales tax due from the petitioner, the opinion,
50% 74,244.30 37,122.16 111,366.46 apparently, is merely an expression of his general or "private sentiment" on
the particular issue, for he concurred the dispositive part of the decision. At
any rate, assuming that there is no valid decision for lack of concurrence of
two judges, the case was submitted for decision of the court below on March
75% 8,000.00 6,000.00 14,000.00 28, 1957 and under section 13 of Republic Act 1125, cases brought before
said court hall be decided within 30 days after submission thereof. "If no
decision is rendered by the Court within thirty days from the date a case is
submitted for decision, the party adversely affected by said ruling, order or
TOTAL P20,220,413.77 P1,809,205.67 P22,038,619.44
decision, may file with said Court a notice of his intention to appeal to the
Supreme Court, and if no decision has as yet been rendered by the Court,
the aggrieved party may file directly with the Supreme Court an appeal from
said ruling, order or decision, notwithstanding the foregoing provisions of this
section." The case having been brought before us on appeal, the question
raised by petitioner as become purely academic.
Less Taxes Paid by
Yutivo 988,655.76 IN VIEW OF THE FOREGOING, the decision of the Court of Tax Appeals
under review is hereby modified in that petitioner shall be ordered to pay to
respondent the sum of P820,549.91, plus 25% surcharge thereon for late
payment.
Deficiency Tax still due P820,549.91
So ordered without costs.

This is the exact amount which, according to Presiding Judge Nable of the Bengzon, Labrador, Concepcion, Reyes, J.B.L., Barrera and Paredes,
Court of Tax Appeals, Yutivo would pay, exclusive of the surcharges. JJ., concur.
Padilla, J., took no part.
Petitioner finally contends that the Court of Tax Appeals erred or acted in
excess of its jurisdiction in promulgating judgment for the affirmance of the G.R. No. L-9687 June 30, 1961
decision of respondent Collector by less than the statutory requirement of at
least two votes of its judges. Anent this contention, section 2 of Republic Act LIDDELL & CO., INC., petitioner-appellant,
No. 1125, creating the Court of Tax Appeals, provides that "Any two judges of vs.
the Court of Tax Appeals shall constitute a quorum, and the concurrence of THE COLLECTOR OF INTERNAL REVENUE, respondent-appellee.

406
Ozaeta, Lichauco and Picazo for petitioner-appellant. On January 31, 1947, with the limited paid-in capital of P20,000,
Office of the Solicitor General for respondent-appellee. Liddell & Co. was able to declare a 90% stock dividend after which
declaration on, Frank Liddells holding in the Company increased to
BENGZON, C.J.: 1,960 shares and the employees, Charles Kurz E.J. Darras, Angel
Manzano and Julian Serrano at 10 share each. The declaration of
Statement. This is an appeal from the decision of the Court of Tax Appeals stock dividend was followed by a resolution increasing the authorized
imposing a tax deficiency liability of P1,317,629.61 on Liddell & Co., Inc. capital of the company to P1,000.000 which the Securities &
Exchange Commission approved on March 3, 1947. Upon such
approval, Frank Liddell subscribed to 3,000 additional shares, for
Said Company lists down several issues which may be boiled to the
which he paid into the corporation P300,000 so that he had in his
following:
own name 4,960 shares.

(a) Whether or not Judge Umali of the Tax Court below could validly
On May 24, 1957, Frank Liddell, on one hand and Messrs. Kurz,
participate in the making of the decision;
Darras, Manzano and Serrano on the other, executed an agreement
(Exhibit A) which was further supplemented by two other agreements
(b) Whether or not Liddell & Co. Inc., and the Liddell Motors, Inc. are (Exhibits B and C) dated May 24, 1947 and June 3, 1948, wherein
(practically) identical corporations, the latter being merely .the alter Frank Liddell transferred (On June 7, 1948) to various employees of
ego of the former; Liddell & Co. shares of stock.

(c) Whether or not, granting the identical nature of the corporations, At the annual meeting of stockholders of Liddell & Co. held on March
the assessment of tax liability, including the surcharge thereon by the 9, 1948, a 100% stock dividend was declared, thereby increasing the
Court of Tax Appeals, is correct. issued capital stock of aid corporation from P1,000.000 to P
3,000,000 which increase was duly approved by the Securities and
Undisputed Facts. The parties submitted a partial stipulation of facts, each Exchange Commission on June 7, 1948. Frank Liddell subscribed to
reserving the right to present additional evidence. and paid 20% of the increase of P400,000. He paid 25% thereof in
the amount of P100,000 and the balance of P3,000,000 was merely
Said undisputed facts are substantially as follows: debited to Frank Liddell-Drawing Account and credited to Subscribed
Capital Stock on December 11, 1948.
The petitioner, Liddell & Co. Inc., (Liddell & Co. for short) is a
domestic corporation establish in the Philippines on February 1, On March 8, 1949, stock dividends were again issued by Liddell &
1946, with an authorized capital of P100,000 divided into 1000 share Co. and in accordance with the agreements, Exhibits A, B, and C,
at P100 each. Of this authorized capital, 196 shares valued at the stocks of said company stood as follows:
P19,600 were subscribed and paid by Frank Liddell while the other
four shares were in the name of Charles Kurz, E.J. Darras, Angel
Manzano and Julian Serrano at one shares each. Its purpose was to
Name No. of Amount Per Cent
engage in the business of importing and retailing Oldsmobile and
Chevrolet passenger cars and GMC and Chevrolet trucks..

407
Shares

Frank Liddell 13,688 P1,368,800 72.00%

Irene Liddell 1 100 .01%

Mercedes Vecin 1 100 .01%

Charles Kurz 1,225 122,500 6.45%

E.J. Darras 1,225 122,500 6.45%

Angel Manzano 1,150 115,000 6.06%

Julian Serrano 710 71,000 3.74%

E. Hasim 500 50,000 2.64%

408
409
G. W. Kernot 500 50,000 2.64%

19,000 P1,900,000 100.00% Angel Manzano 1,810 181,000 6.031%

On November 15, 1948, in accordance with a resolution of a special meeting Julian Serrano 1,700 170,000 5.670%
of the Board of Directors of Liddell & Co., stock dividends were again
declared. As a result of said declaration and in accordance with the
agreements, Exhibits, A, B, and C, the stockholdings in the company
appeared to be: E. Hasim 830 83,000 2.770%

No. of G. W. Kernot 1,490 149,000 4.970%


Name Amount Per Cent
Shares

30,000 P3,000,000 100.000%


Frank Liddell 19,738 P1,973,800 65.791%

On the basis of the agreement Exhibit A, (May, 1947) "40%" of the earnings
Irene Liddell 1 100 .003% available for dividends accrued to Frank Liddell although at the time of the
execution of aid instrument, Frank Liddell owned all of the shares in said
corporation. 45% accrued to the employees, parties thereto; Kurz 12-1/2%;
Darras 12-1/2%; A. Manzano 12-1/2% and Julian Serrano 7-1/2%. The
Mercedes Vecin 1 100 .003% agreement Exhibit A was also made retroactive to 1946. Frank Liddell
reserved the right to reapportion the 45% dividends pertaining to the
employees in the future for the purpose of including such other faithful and
efficient employees as he may subsequently designate. (As a matter of fact,
Charles Kurz 2,215 221,500 7.381%
Frank Liddell did so designate two additional employees namely: E. Hasim
and G. W. Kernot). It was for such inclusion of future faithful employees that
Exhibits B-1 and C were executed. As per Exhibit C, dated May 13, 1948, the
E.J. Darras 2,215 221,500 7.381% 45% given by Frank Liddell to his employees was reapportioned as follows:

410
C. Kurz 12,%; E. J. Darras 12%; A. Manzano l2%; J. Serrano 3- considered as the original sales of Liddell & Co. Accordingly, the Collector of
1/2%; G. W. Kernot 2%. Internal Revenue assessed against Liddell & Co. a sales tax deficiency,
including surcharges, in the amount of P1,317,629.61. In the computation,
Exhibit B contains the employees' definition in detail of the manner by which the gross selling price of Liddell Motors, Inc. to the general public from
they sought to prevent their share-holdings from being transferred to others January 1, 1949 to September 15, 1950, was made the basis without
who may be complete strangers to the business on Liddell & Co. deducting from the selling price, the taxes already paid by Liddell & Co. in its
sales to the Liddell Motors Inc.
From 1946 until November 22, 1948 when the purpose clause of the Articles
of Incorporation of Liddell & Co. Inc., was amended so as to limit its business The Court of Tax Appeals upheld the position taken by the Collector of
activities to importations of automobiles and trucks, Liddell & Co. was Internal Revenue.
engaged in business as an importer and at the same time retailer of
Oldsmobile and Chevrolet passenger cars and GMC and Chevrolet trucks. A. Judge Umali: Appellant urges the disqualification on of Judge Roman M.
Umali to participate in the decision of the instant case because he was Chief
On December 20, 1948, the Liddell Motors, Inc. was organized and of the Law Division, then Acting Deputy Collector and later Chief Counsel of
registered with the Securities and Exchange Commission with an authorized the Bureau of Internal Revenue during the time when the assessment in
capital stock of P100,000 of which P20,000 was subscribed and paid for as question was made.1 In refusing to disqualify himself despite admission that
follows: Irene Liddell wife of Frank Liddell 19,996 shares and Messrs. Marcial had held the aforementioned offices, Judge Umali stated that he had not in
P. Lichauco, E. K. Bromwell, V. E. del Rosario and Esmenia Silva, 1 share any way participated, nor expressed any definite opinion, on any question
each. raised by the parties when this case was presented for resolution before the
said bureau. Furthermore, after careful inspection of the records of the
At about the end of the year 1948, Messrs. Manzano, Kurz and Kernot Bureau, he (Judge Umali as well as the other members of the court below),
resigned from their respective positions in the Retail Dept. of Liddell & Co. had not found any indication that he had expressed any opinion or made any
and they were taken in and employed by Liddell Motors, Inc.: Kurz as decision that would tend to disqualify him from participating in the
Manager-Treasurer, Manzano as General Sales Manager for cars and Kernot consideration of the case in the Tax Court.
as General Sales Manager for trucks.
At this juncture, it is well to consider that petitioner did not question the truth
Beginning January, 1949, Liddell & Co. stopped retailing cars and trucks; it of Judge Umali's statements. In view thereof, this Tribunal is not inclined to
conveyed them instead to Liddell Motors, Inc. which in turn sold the vehicles disqualify said judge. Moreover, in furtherance of the presumption of the
to the public with a steep mark-up. Since then, Liddell & Co. paid sales taxes judge's moral sense of responsibility this Court has adopted, and now here
on the basis of its sales to Liddell Motors Inc. considering said sales as its repeats, the ruling that the mere participation of a judge in prior proceedings
original sales. relating to the subject in the capacity of an administrative official does not
necessarily disqualify him from acting as judge.2
Upon review of the transactions between Liddell & Co. and Liddell Motors,
Inc. the Collector of Internal Revenue determined that the latter was but Appellant also contends that Judge Umali signed the said decision contrary
an alter ego of Liddell & Co. Wherefore, he concluded, that for sales tax to the provision of Section 13, Republic Act No. 1125; 3 that whereas the case
purposes, those sales made by Liddell Motors, Inc. to the public were was submitted for decision of the Court of Tax Appeals on July 12, 1955, and

411
the decision of Associate Judge Luciano and Judge Nable were both signed These stipulations and conditions appear in Exhibit A: (1) that Frank Liddell
on August 11, 1955 (that is, on the last day of the 30-day period provided for had the authority to designate in the future the employee who could receive
in Section 13, Republic Act No. 1125), Judge Umali signed the decision earnings of the corporation; to apportion among the stock holders the share
August 31, 1955 or 20 days after the lapse of the 30-day period allotted by in the profits; (2) that all certificates of stock in the names of the employees
law. should be deposited with Frank Liddell duly indorsed in blank by the
employees concerned; (3) that each employee was required to sign an
By analogy it may be said that inasmuch as in Republic Act No. 1125 (law agreement with the corporation to the effect that, upon his death or upon his
creating the Court of Tax Appeals) like the law governing the procedure in the retirement or separation for any cause whatsoever from the corporation, the
court of Industrial Relations, there is no provision invalidating decisions said corporation should, within a period of sixty days therefor, have the
rendered after the lapse of 30 days, the requirement of Section 13, Republic absolute and exclusive option to purchase and acquire the whole of the stock
Act No. 1125 should be construed as directory.4 interest of the employees so dying, resigning, retiring or separating.

Besides as pointed out by appellee, the third paragraph of Section 13 of These stipulations in our opinion attest to the fact that Frank Liddell also
Republic Act No. 1125 (quoted in the margin)5 confirms this view; because in owned it. He supplied the original his complete control over the corporation.
providing for two thirty-day periods, the law means that decision may still be
rendered within the second period of thirty days (Judge Umali signed his As to Liddell Motors, Inc. we are fully persuaded that Frank Liddell also
decision within that period). owned it. He supplied the original capital funds. 6 It is not proven that his wife
Irene, ostensibly the sole incorporator of Liddell Motors, Inc. had money of
B. Identity of the two corporations: On the question whether or not Liddell her own to pay for her P20,000 initial subscription. 7 Her income in the United
Motors, Inc. is the alter ego of Liddell & Co. Inc., we are fully convinced that States in the years 1943 and 1944 and the savings therefrom could not be
Liddell & Co. is wholly owned by Frank Liddell. As of the time of its enough to cover the amount of subscription, much less to operate an
organization, 98% of the capital stock belonged to Frank Liddell. The 20% expensive trade like the retail of motor vehicles. The alleged sale of her
paid-up subscription with which the company began its business was paid by property in Oregon might have been true, but the money received therefrom
him. The subsequent subscriptions to the capital stock were made by him was never shown to have been saved or deposited so as to be still available
and paid with his own money. at the time of the organization of the Liddell Motors, Inc.

The evidence at hand also shows that Irene Liddell had scant participation in
the affairs of Liddell Motors, Inc. She could hardly be said to possess
business experience. The income tax forms record no independent income of
her own. As a matter of fact, the checks that represented her salary and
bonus from Liddell Motors, Inc. found their way into the personal account of
Frank Liddell. Her frequent absences from the country negate any active
participation in the affairs of the Motors company.

There are quite a series of conspicuous circumstances that militate against


the separate and distinct personality of Liddell Motors, Inc. from Liddell &

412
Co.8 We notice that the bulk of the business of Liddell & Co. was channeled exceed P5000, and 15% of the price if more than P5000 but not more than
through Liddell Motors, Inc. On the other hand, Liddell Motors, Inc. pursued P7000, etc. This progressive rate of the sales tax naturally would tempt the
no activities except to secure cars, trucks, and spare parts from Liddell & Co. taxpayer to employ a way of reducing the price of the first sale. And Liddell
Inc. and then sell them to the general public. These sales of vehicles by Motors, Inc. was the medium created by Liddell & Co. to reduce the price and
Liddell & Co. to Liddell Motors, Inc. for the most part were shown to have the tax liability.
taken place on the same day that Liddell Motors, Inc. sold such vehicles to
the public. We may even say that the cars and trucks merely touched the Let us illustrate: a car with engine motor No. 212381 was sold by Liddell &
hands of Liddell Motors, Inc. as a matter of formality. Co. Inc. to Liddell Motors, Inc. on January 17, 1948 for P4,546,000.00
including tax; the price of the car was P4,133,000.23, the tax paid being
During the first six months of 1949, Liddell & Co. issued ten (10) checks P413.22, at 10%. And when this car was later sold (on the same day) by
payable to Frank Liddell which were deposited by Frank Liddell in his Liddell Motors, Inc. to P.V. Luistro for P5500, no more sales tax was paid. 11 In
personal account with the Philippine National Bank. During this time also, he this price of P5500 was included the P413.32 representing taxes paid by
issued in favor of Liddell Motors, Inc. six (6) checks drawn against his Liddell & Co. Inc. in the sale to Liddell Motors, Inc. Deducting P413.32
personal account with the same bank. The checks issued by Frank Liddell to representing taxes paid by Liddell & Co., Inc. the price of P5500, the balance
the Liddell Motors, Inc. were significantly for the most part issued on the of P5,087.68 would have been the net selling price of Liddell & Co., Inc. to
same day when Liddell & Co. Inc. issued the checks for Frank Liddell 9 and the general public (had Liddell Motors, Inc. not participated and intervened in
for the same amounts. the sale), and 15% sales tax would have been due. In this transaction,
P349.68 in the form of taxes was evaded. All the other transactions
It is of course accepted that the mere fact that one or more corporations are (numerous) examined in this light will inevitably reveal that the Government
owned and controlled by a single stockholder is not of itself sufficient ground coffers had been deprived of a sizeable amount of taxes.
for disregarding separate corporate entities. Authorities 10 support the rule that
it is lawful to obtain a corporation charter, even with a single substantial As opined in the case of Gregory v. Helvering, 12 "the legal right of a taxpayer
stockholder, to engage in a specific activity, and such activity may co-exist to decrease the amount of what otherwise would be his taxes, or altogether
with other private activities of the stockholder. If the corporation is a avoid them by means which the law permits, cannot be doubted." But, as
substantial one, conducted lawfully and without fraud on another, its separate held in another case,13 "where a corporation is a dummy, is unreal or a sham
identity is to be respected. and serves no business purpose and is intended only as a blind, the
corporate form may be ignored for the law cannot countenance a form that is
Accordingly, the mere fact that Liddell & Co. and Liddell Motors, Inc. are bald and a mischievous fiction."
corporations owned and controlled by Frank Liddell directly or indirectly is not
by itself sufficient to justify the disregard of the separate corporate identity of Consistently with this view, the United States Supreme Court 14 held that "a
one from the other. There is, however, in this instant case, a peculiar taxpayer may gain advantage of doing business thru a corporation if he
consequence of the organization and activities of Liddell Motors, Inc. pleases, but the revenue officers in proper cases, may disregard the
separate corporate entity where it serves but as a shield for tax evasion and
Under the law in force at the time of its incorporation the sales tax on original treat the person who actually may take the benefits of the transactions as the
sales of cars (sections 184, 185 and 186 of the National Internal Revenue person accordingly taxable."
Code), was progressive, i.e. 10% of the selling price of the car if it did not

413
Thus, we repeat: to allow a taxpayer to deny tax liability on the ground that Padilla, Labrador, Concepcion, Reyes, J.B.L., Barrera, Paredes, Dizon, De
the sales were made through an other and distinct corporation when it is Leon and Natividad, JJ., concur.
proved that the latter is virtually owned by the former or that they are
practically one and the same is to sanction a circumvention of our tax laws. 15

C. Tax liability computation: In the Yutivo case16 the same question involving
the computation of the alleged deficiency sales tax has been raised. In
accordance with our ruling in said case we hold as correctly stated by Judge
Nable in his concurring and dissenting opinion on this case, that the
deficiency sales tax should be based on the selling price obtained by Liddell
Motors, Inc. to the public AFTER DEDUCTING THE TAX ALREADY PAID BY
LIDDELL & CO., INC. in its sales to Liddell Motors, Inc.

On the imposition of the 50% surcharge by reason of fraud, we see that the
transactions between Liddell Motors Inc. and Liddell & Co., Inc. have always G.R. No. L-22614 August 29, 1969
been embodied in proper documents, constantly subject to inspection by the
tax authorities. Liddell & Co., Inc. have always made a full report of its RAMIREZ TELEPHONE CORPORATION, petitioner,
income and receipts in its income tax returns. vs.
BANK OF AMERICA, E.F. HERBOSA, THE SHERIFF OF MANILA and
Paraphrasing our decision in the Yutivo case, we may now say, in filing its THE COURT OF APPEALS,respondents.
return on the basis of its sales to Liddell Motors, Inc. and not on those by the
latter to the public, it cannot be held that the Liddell & Co., Inc. deliberately Quijano and Arroyo, for petitioner.
made a false return for the purpose of defrauding the government of its Lichauco, Picazo and Agcaoili for respondent Bank of America.
revenue, and should suffer a 50% surcharge. But penalty for late payment Vicente M. Magpoc for respondent E.F. Herbosa.
(25%) should be imposed. Fiscal Eulogio S. Serrano for respondent Sheriff of Manila.

In view of the foregoing, the decision appealed from is hereby modified: CAPISTRANO, J.:
Liddell & Co., Inc. is declared liable only for the amount of P426,811.67 with
25% surcharge for late payment and 6% interest thereon from the time the This is a petition for review on certiorari of a decision of the Court of Appeals
judgment becomes final. of February 27, 1964, wherein the judgment of the lower court was reversed
and another entered dismissing the complaint of plaintiff, now petitioner,
As it appears that, during the pendency of this litigation appellant paid under Ramirez Telephone Corporation, and ordering it to pay to defendant, now
protest to the Government the total amount assessed by the Collector, the respondent, Bank of America, the sum of P500.00 and to the third-party
latter is hereby required to return the excess to the petitioner. No costs. defendant E.F. Herbosa, now likewise respondent, the same amount, both in
the concept of attorney's fees, the costs being adjudged likewise against

414
petitioner. The judgment of the Court of First Instance which was reversed by Civil Case No. 10620
the Court of Appeals reads as follows:1 E.F. Herbosa, Plaintiff

In view of the foregoing considerations, judgment is hereby rendered -- versus -- GARNISHMENT


in favor of the plaintiff and against the defendant Bank of America
ordering the latter to pay the former the sum of P3,000.00 in the form Ruben R. Ramirez, Defendant
of actual damages, and to pay the costs of these proceedings.
To: Bank of America
Likewise, judgment is hereby rendered sentencing the third-party Manila
defendant, E.F. Herbosa, to indemnify or reimburse the third-party
plaintiff, Bank of America, any sum or sums which the latter may pay Greetings:
the plaintiff by virtue of this judgment.
You and each of you are hereby notified that, by virtue of an order of
The third-party complaint against the Sheriff of Manila as well as the attachment issued by the Court of First Instance of Manila, copy of
counterclaim of defendant Bank of America and third-party defendant which is hereto attached, levy is hereby made (or attachment is
E.F. Herbosa are hereby ordered dismissed. hereby levied) upon all the goods, effects, interests, credits, money,
stocks, shares, any interests in stocks and shares and all debts
The facts as found by the Court of Appeals, which we cannot review are set owing by you to the defendant, Ruben R. Ramirez ---------, in the
forth in its decision, thus:2 above entitled case, and any other personal property in your
possession or under your control, belonging to the said defendant
Resultando: Que los hechos al parecer, no son muy embrollados; el --------- on this date, to cover the amount of P2,400.00 and specially
demandado, Herbosa era y es dueno del edificio No. 612, Int. 3 Sta. the ... .
Mesa; se lo habia dado en arrendamiento a Ruben R. Ramirez, y
como este era el presidente de la Ramirez Telephone Corporation, el xxx xxx xxx
taller de la corporacion aunque su oficina central estaba en la
Escolta, Natividad Building, Exh. D. fue trasladado al local: pero Manila, Philippines, October 13, 1950
habiendose amontonado los alquilares sin pagar, Herbosa presento
demanda de desahucio contra Ramirez en el Juzgado Municipal de
MACARIO M. OFILADA
Manila el 10 de Noviembre, 1949, y elevada la causa al Juzgado del
Sheriff of Manila
1.a Instancia, Herbosa pudo conseguir decision favorable alli el 14
(Exh. 2);
de Octubre, 1950, pero en la vispera de la promulgacion de la
sentencia a su favor habia ya conseguido mandamiento de embargo
preventivo contra Ramirez, Exh. A, y el mismo, servido al Bank of y fue contestado por el banco el mismo dia de la siguiente manera:
America el 13 de Octubre, 1950, Exh. 2, lease como sigue:
Dear Sir:

415
In reply to your Garnishment of October 13, 1950, issued under the 1950, envio carta de requerimiento al Bank of America, Exh. 14,
above-subject case, we wish to inform you that we do not hold any manifestando que su cliente habia sufrido "considerable damage and
fund in the name of the defendant, Ruben R. Ramirez. embarrassment," y advirtiendole que si no se le diera completa
satisfaccion el dia siguiente, el presentaria la demanda
Yours very truly, (Exh. 3); correspondiente, "without further notice," Exh. 14; esta carta la
contesto la institucion bancaria el 24 de Octubre, 1950, alegando
pero el Sheriff reitero el embargo el 17 de Octubre, 1950, Exh. B, que,
notificando al Bank of America de que quedaba embargado,
"With reference to your letter dated October 23, 1950, in
"... the interest or participation which the defendant Ruben R. which you are writing in behalf of the Ramirez Telephone
Ramirez may or might have in the deposit of the Ramirez Corporation, it is suggested that you obtain a release from
Telephone, Inc., with that Bank sufficient to cover the said the Court on Civil Case No. 10620, Ruben E. Ramirez,
amount of P2,400.00"; Exh. B; y defendant.

la institucion bancaria en contestacion al Sheriff, de fecha 17 de "This Bank is acting only in accordance with the garnishment
Octubre, 1950 o sea el mismo dia, hizo constar que: and has no interest whatsoever in the funds held," Exh. 15;

"... we are holding the amount of P2,400.00 in the name of pero conforme con su advertencia, el abogado dela Ramirez
the Ramirez Telephone, Inc. subject to your further orders," Telephone, Inc., incoo esta accion el 28 de Octubre, cuatro dias
Exh. G; despues; y el motivo deaccion se de hace consistir en que el banco,

es decir acato la notificacion del embargo de los fondos de la "... knows or should have known that Ruben N. Ramirez the
Ramirez Telephone; ahora bien, recuerdase de que en aquella defendant in said Civil Case and whose property or fund was
fecha, 17 de Octubre, 1950, es Ramirez Telephone tenia en deposito ordered attached has no personal deposit in that bank and
con el Bank of America, la suma de P4,789.53, Exh. 9; de manera that the Ramirez Telephone Corporation is entirely a distinct
que con el embargo, se redujo los fondos libres a la cantidad de and separate entity regardless of the fact that Ruben R.
P2,389.53; pero el dia siguiente, el Ramirez Telephone retiro la Ramirez happens to be its President and General Manager.'
suma de P1,500.00, quedando por tanto como ultimo balance, nada par. 4, demanda; y alegando que con motivo de ello y la
mas que unos P889.00; de esto surgio la presente contienda, pues, siguiente devolucion de su cheque a favor de la Ray
el 19 de Octubre, 1950, la Ramirez Telephone por medio de su Electronics sin pagar, esta habia cancelado su pedido para
presidente, el mismo demandado, Ruben Ramirez, ya mencionado, los equipos necesarios en la construccion de sus lineas
habiendo expedido el 19 de Octubre, 1950, otro cheque en la suma telefonicas en la region bicolana, asi que todas sus
de P2,320.00 a favor de la Ray Electronics, en pago de ciertos operaciones se habian quedado paralizadas, par. 5 id.; la
equipos vendidos por este ultimo, Exhs. 15, 17, L, el cheque Exh. N, demandada Bank of America, emplazada de la demandada,
este cheque al ser presentado a la Bank of America, fue rechazado presento mocion de sobresimiento, que denegada, el 4 de
por lo que el abogado de la Ramirez Telephone el 23 de Octubre, Diciembre, 1950, el banco sometio su contestacion el 23 de

416
Diciembre, 1950 con reconvencion para despues presentar buenasanta podia disponer su Presidente, Ruben R. Ramirez, para
demanda contra el Sheriff, el 25 de Agosto, 1953, y Contra el pago de los alquilares por el debidos a Herbosa, y luego, tambien
Herbosa, el 16 de Agosto, 1955; y este ultimo a su vez en resulta evidente de que la casa por el alquilada Ramirez Telephone,
contestacion, presento contra reclamacion o mejor dicho, y estos hechos agregados el otro hecho tambien probado, de que el
reconvencion contra la misma demandante, Ramirez 75% de las acciones de la compania pertenecia a Ruben Ramirez y
Telephone, y tambien contra el Bank of America, el 10 de su esposa Rizalina P. de Ramirez, Exh. E, todos estos no pueden
Septiembre, 1955, y el Juzgado Inferior, despues de la vista, menos de justificar la conclusion de que el embargo de los fondos
como ya se ha dicho, dictamino en favor de la demandante de la Ramirez Telephone por y en virtud de un mandamiento judicial
contra el Bank of America en la contra-demanda de este de embargo contra Ruben R. Ramirez, especialmente teniendo en
contra aquel; ... ." cuenta que el embargo solo abarcaba,

It was further found by the Court of Appeals:3 "The interest or participation which the defendant Ruben R.
Ramirez may or might have in the deposit of the Ramirez
Considerando: Que el testimonio de Estanislao Herbosa al efecto de Telephone, Inc., in the amount of P2,400.00" Exh. B;
que; si bien Ruben R. Ramirez era su inquilino al principio, pero es
que mas tarde, este lo habia manifestado que "the shop of company cuando entonces estaba depositada la cantidad de P4,857.28, Exh.
was established downstairs," e decir que la Ramirez Telephone 9, era un acto de justicia a favor del acreedor Herbosa y a la verdad,
Corporation a la verdad ocupaba el local alquilado, tanto que Ruben de no haberse permitido el mencionado embargo, este se hubiera
R. Ramirez solia pagar el alquilar en cheques de la Ramirez visto en igual situacion que aquel pobre agraviado que como se dice
Telephone Corporation, y esta declaracion, t.n. 10 y 11, 25 June vulgarmente, tras de cornudo, fue apaleado; ... .
1956, estando corroborada no solamente por el Exh. 12, en donde
Ruben R. Ramirez, en papel con el embrete de la Ramirez The aforestated facts notwithstanding, which must be considered conclusive
Telephone, habia enviado el abogado de Herbosa, el cheque No. C- and binding on us, plaintiff in the lower court, now petitioner, Ramirez
78900, manifestando en la carta de que: Telephone Corporation, as noted, appealed, assigning the following alleged
errors:4
In accordance with your agreement yesterday with my
attorney, Mr. Jose L. de Leon, I am sending you herewith I
check No. C-78900 for the amount of P812.60, rentals for
the premises I am occupying at the rate of P161.00 a month The Court of Appeals erred in not applying the settled legal principle
for the period from February 1, 1949 to June 30, 1949, both that a corporation has a personality separate and distinct from that of
dates, inclusive, plus P7.00 for the court costs.' Exh. 12; its stockholders and, therefore, the funds of a corporation cannot be
reached to satisfy the debt of its stockholders.
y esta carta, leida en relacion con el Exh. 3, en donde se ve que
Ruben R. Ramirez y tenia fondos depositados en el banco II
mencionado, Bank of America, asi que resulta evidente que lo
fondos de la Ramirez Telephone los eran a la verdad, fondos de que

417
The Court of Appeals erred in not taking into account the significant 376; Madrigal Shipping Co., Inc. vs. Ogilvie, L-8431, Oct. 30, 1958; Laguna
fact that when the events that gave rise to this case took place, the Transportation Co., Inc. vs. S.S.S., L-14606, April 28, 1960; McConnel vs.
lawyer of both respondents, i.e., the Bank of America and E.F. C.A., L-10510, March 17, 1961; Liddel & Co., Inc. vs. Collector of Internal
Herbosa, was one and the same. Revenue, L-9687, June 30, 1961; Palacio vs. Fely Transportation Co., L-
15121, August 31, 1962. Hence, to repeat, the first assigned error cannot be
III sustained.

The Court of Appeals erred in not granting petitioner damages as The next two errors assigned likewise fail to call for a reversal of the
awarded by the lower court; likewise, the Court of Appeals erred in judgment now on appeal. The second alleged error would find fault with the
declaring instead that it is petitioner that should pay respondents decision because the Court of Appeals allegedly did not take into account a
attorneys' fees. significant fact, namely, that only one lawyer represented both the
respondent Bank of America and respondent E.F. Herbosa. We are not called
Petitioner's main grievance in the first assigned error is that the Court of upon to consider this particular assignment of error as it is essentially factual,
Appeals disregarded its corporate personality; it relies on the general which is a matter for the Court of Appeals, not for us, to determine. The last
principle "that the corporate entity will not be disregarded no matter how assigned error would in effect seek a restatement of the damages awarded
large the holding a particular stockholder may have in the petitioner on the theory that the Court of Appeals decided the matter
corporation." 5 Petitioner would thus maintain that the personality as an entity erroneously. Since, as we made clear in the foregoing, the decision of the
separate and distinct from its major stockholders, Ruben R. Ramirez and his Court of Appeals is in accordance with law on the facts as found, this alleged
wife, was not to be disregarded even if they did own 75% of the stock of the error likewise is not meritorious.
corporation. 6 The conclusion that would thus emerge, in petitioner's opinion,
is that its funds as a corporation cannot be garnished to satisfy the debts of a PREMISES CONSIDERED, the judgment of the Court of Appeals of
principal stockholder. February 27, 1964 is affirmed, with costs against petitioner Ramirez
Telephone Corporation.
While respect for the corporate personality as such is the general rule, there
are exceptions. In appropriate cases, the veil of corporate fiction may be Concepcion, C.J., Dizon, Makalintal, Sanchez, Castro, Fernando, Teehankee
pierced. From the facts as found which must remain undisturbed, this is such and Barredo, JJ., concur.
a case. This assignment of error has no merit, in view of a number of cases Reyes, J.B.L., and Zaldivar, JJ., are on leave.
decided by this Court, the latest of which is Albert v. Court of First
Instance 7 reaffirming a 1965 resolution in Albert v. University Publishing Co.,
Inc. 8In that resolution, the principle is restated thus: "Even with regard to
corporations duly organized and existing under the law, we have in many a
case pierced the veil of corporate fiction to administer the ends of justice." In
support of the above principle, the following cases were cited: Arnold vs.
Willits & Patterson, Ltd., 44 Phil. 634; Koppel (Phil.), Inc. vs. Yatco, 77 Phil.
496; La Campana Coffee Factory, Inc. vs. Kaisahan ng mga Manggagawa sa
La Campana, 93 Phil. 160; Marvel Building Corporation vs. David, 94 Phil.

418
419
420
From the records it appears that Jolly M. Almoradie was first employed by
Mercury Express International Courier Service, Inc. (MEREX) in October,
G.R. No. 100322 March 9, 1994 1983 as Messenger receiving a monthly salary of P800.00. When it closed its
operations, Almoradie was absorbed by MEREX's sister company Philippine
GUATSON INTERNATIONAL TRAVEL AND TOURS, INC., PHILIPPINE Integrated Labor Assistance Corp. (Philac), likewise as Messenger with an
INTEGRATED LABOR ASSISTANCE CORPORATION, MERCURY increased salary of P1,200.00.
EXPRESS INTERNATIONAL COURIER SERVICES, INC., petitioners,
vs. In September, 1986, Almoradie was transferred to Guatson Travel, allegedly
NATIONAL LABOR RELATIONS COMMISSION AND JOLLY also a sister company of MEREX and Philac, as Liaison Officer with a salary
ALMORADIE, respondents. of P1,864.00. Thereafter, he was promoted to the position of Sales
Representative sometime in April, 1988. On April 30, 1988, Almoradie was
Generosa R. Jacinto for petitioners. issued three separate memoranda as follows:

Donato H. De Castro and Rolando P. Rotairo for private respondent. IOM/88-70

Please explain in writing within 24 hrs. or not


later than Monday morning the reason why
you don't want to sell. 1
NOCON, J.:

IOM/88-71
Petitioners Guatson Travel and Tours, Inc. (hereinafter referred to as
Guatson Travel), Philippine Integrated Labor Assistance Corp. (Philac) and
Mercury Express International Courier Services, Inc. (MEREX) assail the Please explain in writing why did you went
Decision, rendered by the National Labor Relations Commission in Case No. (sic) to BEMIL and who sent you there. 2
NLRC-NCR-00-11-0451-88 entitled "Jolly M. Almoradie v. Guatson's Travel
Company, Philac and MEREX," dated March 21, 1991 and its Resolution, IOM-88
dated May 31, 1991, denying the petitioners' Motion for Reconsideration.
Explain in writing not later than Monday the
In the questioned decision, the NLRC found that Mr. Henry Ocier's (Vice- following:
President and General Manager of petitioner Guatson Travel) actuation of
threatening and forcing private respondent, Jolly M. Almoradie, to resign 1. The reason why you want to be a
amounted to illegal dismissal and thus ordered petitioners to pay private messenger and no more a sales
respondent backwages, computed from the date of his dismissal on representative;
November 1988, until the decision was rendered on February 28, 1991 or the
amount of P50,328.00; and to pay separation pay equivalent to one-half (1/2) 2. That I'm always confronting (sic) you, as
month for every year of service, for seven (7) years or the amount of what you've told me personally;
P6,524.00.

421
3. Why you will not answer in writing the 2. Bemil is a customer of our company. With respect to the
memo issued to you by Lou Cantara on 30 ticketing and booking of Bemil passengers, undertaking (sic)
Apr; by the sales department of our company, I used to go Bemil
(sic) to inquire whether they have passengers for booking
xxx xxx xxx and ticketing. As a matter of fact, I went to Bemil to pick-up
their ticketing and booking for their passengers last Monday,
5. Why when you were asked last Friday to April 29, 1988 (sic) and then returned the following day,
join the Sales Blitz to Sta. Ana you said yes Saturday April 30, 1988, to deliver the ticket.
and you change (sic) your mind when you
were asked again last Saturday; xxx xxx xxx

xxx xxx xxx 3.1. Considering that the job of sales representative entails
so much expense in the performance thereof (sic), as I have
7. Why you have forgotten the situation stated in my number one (1) explanation and I have to use
wherein you refuse (sic) to sell a certain my own personal money to promote and solicit customer
product recommended by Myrna; without any funding of our company (sic), I have taught (sic)
it better that I like my position as messenger, that (sic) as
sales representative, although the later (sic) position is more
8. The meaning of "You pirated me from
dignified, hence I prefer to be entered to my messenger
Philac . . . 3
position.

Within the time frame specified, Almoradie responded to each of the charges,
3.2. That I admit of the often confrontation conducted (sic) by
the essence of which are as follows:
Vice President/General Manager, even in the absence of my
error or fault (sic) . . .
1. It is not true that I do not want to sale (sic) the rates &
package tour of Our Company as imputed and charge (sic),
3.3. It is not true that I did not or fail to answer the memo
because since April, 1988 (sic) when I was transferred from
issued by Lou Cantara, since I was given until May 2, 1988
Accounting to sales department of our Company I was able
to answer the same . . .
to sale (sic) almost 110 dollars to 21 passengers. The truth
however is that, I am hampered in my sales promotion and
solicitation of customer, due to financial constraint xxx xxx xxx
considering that the kind and nature of work entails much
expenses for which I shouldered (sic) with my personal 3.5. As scheduled, I said yes to the sales blitz to Sta. Ana,
money. As a matter of fact I have brought this matter to the because in truth I am very interested in such sales business
Vice President and General Manager if only an appropriation attack since it is in connection with my function as a sales
be set aside for the expenses in going around, meeting representative that will surely enhance and sharpen my
people and soliciting prospective clients. sales acumen, but if I was not able to join it is not the reason

422
my change of mind (sic), but because the Vice- In the morning of October 1, 1988, Almoradie was allegedly summoned by
President/General Manager of Our Company, Henry Ocier Henry Ocier to his office and was there and then forced by the latter to
summoned me to his office and had a very lengthy resign. Ocier taunted Almoradie with threats that it he will not resign, he will
confrontation of me (sic), and when I go out (sic) after the file charges against him which would adversely affect his chances of getting
confrontation to join the sales blitz-krieg to Sta. Ana last employed in the future. Ocier allegedly even provided the pen and paper on
Saturday, April 30, 1988, Mr. Oscar Vanderlipe who heads which Almoradie wrote and signed the resignation letter dictated by Ocier
the sales Group (sic) were (sic) already gone. himself. 5

xxx xxx xxx On that same day, Almoradie sought the help of a friend, Isagani Mallari, who
advised him to report the matter to the Barangay Captain. 6 Subsequently,
3.7. I deny vehemently that I refuse to sale (sic) a certain Almoradie filed a complaint for illegal Dismissal on November 14, 1988. The
product recommended by Myrna de Vera because the same Labor Arbiter, however dismissed his case based on the following
is totally false. Since April 1, 1988 when I was transferred to conclusions:
the sales department of our company where from the very
beginning I was briefed and taught and learned about the In examining the facts and the arguments, it is difficult to
nature of my job and the product to sale (sic) by Myrna (sic) abide by the impression that complainant was forced to
de Vera herself, I have ever since until now ventured and resign. Apart from the averment of respondent Guatson that
performed the selling of rates and package tour which are Mr. Ocier was out of town when the resignation letter was
every products (sic) for sales department of our company. If executed that he just saw the resignation letter when he
sometimes I make no sales, which all sales representative arrived. 7There is reason to believe that complainant
suffer and are beset such (sic), however, cannot be apparently defied the order for his transfer or designation as
considered as refusal to sale (sic). The only product of our account executive earlier before he executed his resignation
Company that Myrna briefed, taught and required as to (sic) letter.
our rates and Package Tours which I've been selling since
April 1, 1988 up to present. (sic) It must be concluded that his designation as account
executive is a management prerogative which under the
xxx xxx xxx 4 circumstance is untainted with any unfair labor practice.
Apparently, complainant resented his resignation without any
On May 4, 1988, Almoradie was reverted to the position of Messenger, yet plausible or cogent reason as he had earlier resented to be a
sometime in September, 1988, he was again given the position of Account sales representative for which he was made to explain the
Executive, the nature of work of which is similar to that of a sales reasons why. The only graceful exit to the complainant was
representative. Almoradie accepted the transfer with the understanding that to execute his letter of resignation. As his letter of resignation
he will solely discharge the duties of an account executive and will no longer shows, it was executed in his own handwriting
be required to do messengerial work. spontaneously out of his own free will. 8

423
Upon Almoradie's appeal, the NLRC reversed the decision of the Labor We do not agree with petitioners' proposition that Mr. Ocier's mere utterances
Arbiter on his finding that complainant was not forced to resign, anchoring its of the words "I will file charges against you," and "I have a very good lawyer,"
conclusion to the fact that Almoradie was a permanent employee who has do not constitute force or coercion as to vitiate the free will of Almoradie in
been working for the Ocier's for five long years; that he was receiving a fairly writing his resignation letter.
good salary considering that he is single; that he had no potential employer
at the time of his resignation; that there was no evidence to show that Mr. Intimidation may vitiate consent when the following requisites are present: (1)
Henry Ocier was indeed not in town on October 1, 1988, when he allegedly that the intimidation caused the consent to be given; 2) that the threatened
forced Almoradie to resign; and his reaction immediately after his forced act be unjust or unlawful; 3) that the threat be real or serious, there being
resignation by seeking the assistance of a friend who was placed in a similar evident disproportion between the evil and the resistance which all man can
situation before and in reporting the incident to the Barangay Chairman to offer, leading to the choice of doing that act which is forced on the person to
seek redress. do as the lesser evil; and 4) that it produces a well-grounded fear from the
fact that the person from whom it comes has the necessary means or ability
The issue therefore, boils down to the question of whether Jolly Almoradie to inflict the threatened injury to his person or property. 9
was indeed illegally dismissed by being forced to resign in the manner
narrated by him. The moment that a person by whom respect and reverence are due should
wrongly exert pressure upon his subordinates, amounting to intimidation in
From a synthesis of the evidence on record, we fully agree with the finding of the manner stated in the Lichauco de Leon case, supra, in order to exact
the NLRC that Jolly Almoradie's resignation was NOT voluntary. The NLRC from said subordinates an act against their will, the same is enough to vitiate
did not err in disregarding the conclusions reached by the Labor Arbiter consent.
because the latter's findings are not supported by substantial evidence.
Henry Ocier did not only say that he will file charges against Almoradie and
It appears that as early as April, 1988, when Almoradie was promoted as that he has a good lawyer but he even threatened to block his future
Sales Representative he had caught the ire of management, so much so that employment should the latter not file his resignation. This threat is not
he was issued no less than three memoranda on one day ordering him to farfetched. Almoradie is not even a college graduate. 10 With his limited skills
answer certain charges. Why he was again promoted to the position of and the scarcity of employment opportunities it would really be difficult for
Account Executive after he was reverted back to the rank of a messenger him to find a job. Considering further the influence of Mr. Henry Ocier and his
from being a Sales Representative is rather intriguing, unless it was a capacity to make good his threat by refusing to give a favorable
scheme of management to really rid him from the company. Apparently, recommendation on Almoradie's performance, the latter is helpless in not
Almoradie is not cut out for a sales job, and hence could be dismissed or complying with the former's demand for his resignation.
forced to resign for failing to make good on his job on sales. On the other
hand, it would be difficult to dismiss him while being a messenger since he is Anent NLRC's grant of separation pay and backwages to private respondent
a permanent employee and there would not be enough basis to make him Jolly M. Almoradie, petitioners argues that the companies, Guatson Travel
resign. Company, Philac Merex have separate and distinct legal personalities such
that the latter companies should not be held liable; assuming, for the sake of
argument that private respondent was illegally dismissed.

424
We uphold the NLRC. The three companies are owned by one family, such should be computed from the time petitioner was employed by Merex and
that majority of the officers of the companies are the same. The companies should include the three-year period as backwages. The petition is hereby
are located in one building and use the same messengerial service. DISMISSED for lack of merit.
Moreover, there was no showing that private respondent was paid separation
pay when he was absorbed by Philac upon closure of Merex; nor was there SO ORDERED.
evidence that he resigned from Philac when he transferred to Guatson
Travel. Under the doctrine of piercing the veil of corporate fiction, when valid Narvasa, C.J., Padilla, Regalado and Puno, JJ., concur.
ground exists, the legal fiction that a corporation is an entity with a juridical
personality separate and distinct from its members or stockholders may be
disregarded. We have applied this doctrine in the case of "Philippine Scout
Veterans Security and Investigation Agency (PSVSIA), et al. v. The Hon.
Secretary of Labor," G.R. No. 92357, July 21, 1993.

Where there is a finding of illegal dismissal, the employee is entitled to both


reinstatement and award of backwages from the time the compensation was
withheld, in this case in 1988, up to a maximum of three years, applying the
Mercury Drug Rule. 11

Reinstatement, however, will not be required not only for the reason that it
was not prayed for by the respondent, but also because the relationship
between Almoradie and Ocier had become strained as to preclude a
harmonious working relationship. In lieu of reinstatement, separation pay is
awarded. 12 As the term suggests, separation pay is the amount that an
employee receives at the time of his severance from the service and is
designed to provide the employee with the wherewithal during the period that
he is looking for another employment. 13

However the award of separation pay should be, as we have consistently


ruled, equivalent to one (1) month for every year of service, 14 instead of one-
half (1/2) month as awarded by the NLRC. In the computation of separation
pay, the three (3) year period wherein backwages are awarded, must be
included. 15

WHEREFORE, the decision of the NLRC is hereby MODIFIED to the extent


that the award of backwages should be computed based on a three-year
period, while the separation pay of one month for every year of service

425
426
Thus, where a sister corporation is used as a shield to evade a corporation's
subsidiary liability for damages, the corporation may not be heard to say that
it has a personality separate and distinct from the other corporation. The
piercing of the corporate veil comes into play.

This special civil action ostensibly raises the question of whether the National
Labor Relations Commission committed grave abuse of discretion when it
issued a "break-open order" to the sheriff to be enforced against personal
property found in the premises of petitioner's sister company.

Petitioner Concept Builders, Inc., a domestic corporation, with principal office


G.R. No. 108734 May 29, 1996
at 355 Maysan Road, Valenzuela, Metro Manila, is engaged in the
construction business. Private respondents were employed by said company
CONCEPT BUILDERS, INC., petitioner, as laborers, carpenters and riggers.
vs.
THE NATIONAL LABOR RELATIONS COMMISSION, (First Division); and
On November, 1981, private respondents were served individual written
Norberto Marabe; Rodolfo Raquel, Cristobal Riego, Manuel Gillego,
notices of termination of employment by petitioner, effective on November
Palcronio Giducos, Pedro Aboigar, Norberto Comendador, Rogelio
30, 1981. It was stated in the individual notices that their contracts of
Salut, Emilio Garcia, Jr., Mariano Rio, Paulina Basea, Alfredo Albera,
employment had expired and the project in which they were hired had been
Paquito Salut, Domingo Guarino, Romeo Galve, Dominador Sabina,
completed.
Felipe Radiana, Gavino Sualibio, Moreno Escares, Ferdinand Torres,
Felipe Basilan, and Ruben Robalos, respondents.
Public respondent found it to be, the fact, however, that at the time of the
termination of private respondent's employment, the project in which they
were hired had not yet been finished and completed. Petitioner had to
engage the services of sub-contractors whose workers performed the
HERMOSISIMA, JR., J.:p functions of private respondents.

The corporate mask may be lifted and the corporate veil may be pierced Aggrieved, private respondents filed a complaint for illegal dismissal, unfair
when a corporation is just but the alter ego of a person or of another labor practice and non-payment of their legal holiday pay, overtime pay and
corporation. Where badges of fraud exist; where public convenience is thirteenth-month pay against petitioner.
defeated; where a wrong is sought to be justified thereby, the corporate
fiction or the notion of legal entity should come to naught. The law in these
On December 19, 1984, the Labor Arbiter rendered judgment 1 ordering
instances will regard the corporation as a mere association of persons and,
petitioner to reinstate private respondents and to pay them back wages
in case of two corporations, merge them into one.
equivalent to one year or three hundred working days.

427
On November 27, 1985, the National Labor Relations Commission (NLRC) 3. Security guards with high-powered guns prevented him from removing the
dismissed the motion for reconsideration filed by petitioner on the ground that properties he had levied upon. 4
the said decision had already become final and executory. 2
The said special sheriff recommended that a "break-open order" be issued to
On October 16, 1986, the NLRC Research and Information Department enable him to enter petitioner's premises so that he could proceed with the
made the finding that private respondents' back wages amounted to public auction sale of the aforesaid personal properties on November 7,
P199,800.00. 3 1989.

On October 29, 1986, the Labor Arbiter issued a writ of execution directing On November 6, 1989, a certain Dennis Cuyegkeng filed a third-party claim
the sheriff to execute the Decision, dated December 19, 1984. The writ was with the Labor Arbiter alleging that the properties sought to be levied upon by
partially satisfied through garnishment of sums from petitioner's debtor, the the sheriff were owned by Hydro (Phils.), Inc. (HPPI) of which he is the Vice-
Metropolitan Waterworks and Sewerage Authority, in the amount of President.
P81,385.34. Said amount was turned over to the cashier of the NLRC.
On November 23, 1989, private respondents filed a "Motion for Issuance of a
On February 1, 1989, an Alias Writ of Execution was issued by the Labor Break-Open Order," alleging that HPPI and petitioner corporation were
Arbiter directing the sheriff to collect from herein petitioner the sum of owned by the same incorporator/stockholders. They also alleged that
P117,414.76, representing the balance of the judgment award, and to petitioner temporarily suspended its business operations in order to evade its
reinstate private respondents to their former positions. legal obligations to them and that private respondents were willing to post an
indemnity bond to answer for any damages which petitioner and HPPI may
On July 13, 1989, the sheriff issued a report stating that he tried to serve suffer because of the issuance of the break-open order.
the alias writ of execution on petitioner through the security guard on duty but
the service was refused on the ground that petitioner no longer occupied the In support of their claim against HPPI, private respondents presented duly
premises. certified copies of the General Informations Sheet, dated May 15, 1987,
submitted by petitioner to the Securities Exchange Commission (SEC) and
On September 26, 1986, upon motion of private respondents, the Labor the General Information Sheet, dated May 25, 1987, submitted by HPPI to
Arbiter issued a second alias writ of execution. the Securities and Exchange Commission.

The said writ had not been enforced by the special sheriff because, as stated The General Information Sheet submitted by the petitioner revealed the
in his progress report, dated November 2, 1989: following:

1. All the employees inside petitioner's premises at 355 Maysan Road, 1. Breakdown of Subscribed Capital
Valenzuela, Metro Manila, claimed that they were employees of Hydro Pipes
Philippines, Inc. (HPPI) and not by respondent; Name of Stockholder Amount Subscribed

2. Levy was made upon personal properties he found in the premises; HPPI P 6,999,500.00

428
Antonio W. Lim 2,900,000.00 355 Maysan Road

Dennis S. Cuyegkeng 300.00 Valenzuela, Metro Manila. 5

Elisa C. Lim 100,000.00 On the other hand, the General Information Sheet of HPPI revealed the
following:
Teodulo R. Dino 100.00
1. Breakdown of Subscribed Capital
Virgilio O. Casino 100.00
Name of Stockholder Amount Subscribed
2. Board of Directors
Antonio W. Lim P 400,000.00
Antonio W. Lim Chairman
Elisa C. Lim 57,700.00
Dennis S. Cuyegkeng Member
AWL Trading 455,000.00
Elisa C. Lim Member
Dennis S. Cuyegkeng 40,100.00
Teodulo R. Dino Member
Teodulo R. Dino 100.00
Virgilio O. Casino Member
Virgilio O. Casino 100.00
3. Corporate Officers
2. Board of Directors
Antonio W. Lim President
Antonio W. Lim Chairman
Dennis S. Cuyegkeng Assistant to the
President Elisa C. Lim Member

Elisa O. Lim Treasurer Dennis S. Cuyegkeng Member

Virgilio O. Casino Corporate Secretary Virgilio O. Casino Member

4. Principal Office Teodulo R. Dino Member

429
3. Corporate Officers Hence, the resort to the present petition.

Antonio W. Lim President Petitioner alleges that the NLRC committed grave abuse of discretion when it
ordered the execution of its decision despite a third-party claim on the levied
Dennis S. Cuyegkeng Assistant to the property. Petitioner further contends, that the doctrine of piercing the
President corporate veil should not have been applied, in this case, in the absence of
any showing that it created HPPI in order to evade its liability to private
Elisa C. Lim Treasurer respondents. It also contends that HPPI is engaged in the manufacture and
sale of steel, concrete and iron pipes, a business which is distinct and
separate from petitioner's construction business. Hence, it is of no
Virgilio O. Casino Corporate Secretary
consequence that petitioner and HPPI shared the same premises, the same
President and the same set of officers and subscribers. 7
4. Principal Office
We find petitioner's contention to be unmeritorious.
355 Maysan Road, Valenzuela, Metro
Manila. 6
It is a fundamental principle of corporation law that a corporation is an entity
separate and distinct from its stockholders and from other corporations to
On February 1, 1990, HPPI filed an Opposition to private respondents' which it may be connected. 8 But, this separate and distinct personality of a
motion for issuance of a break-open order, contending that HPPI is a corporation is merely a fiction created by law for convenience and to promote
corporation which is separate and distinct from petitioner. HPPI also alleged justice. 9 So, when the notion of separate juridical personality is used to
that the two corporations are engaged in two different kinds of defeat public convenience, justify wrong, protect fraud or defend crime, or is
businesses, i.e., HPPI is a manufacturing firm while petitioner was then used as a device to defeat the labor laws, 10 this separate personality of the
engaged in construction. corporation may be disregarded or the veil of corporate fiction pierced. 11 This
is true likewise when the corporation is merely an adjunct, a business conduit
On March 2, 1990, the Labor Arbiter issued an Order which denied private or an alter ego of another corporation. 12
respondents' motion for break-open order.
The conditions under which the juridical entity may be disregarded vary
Private respondents then appealed to the NLRC. On April 23, 1992, the according to the peculiar facts and circumstances of each case. No hard and
NLRC set aside the order of the Labor Arbiter, issued a break-open order and fast rule can be accurately laid down, but certainly, there are some probative
directed private respondents to file a bond. Thereafter, it directed the sheriff factors of identity that will justify the application of the doctrine of piercing the
to proceed with the auction sale of the properties already levied upon. It corporate veil, to wit:
dismissed the third-party claim for lack of merit.
1. Stock ownership by one or common ownership of both
Petitioner moved for reconsideration but the motion was denied by the NLRC corporations.
in a Resolution, dated December 3, 1992.
2. Identity of directors and officers.

430
3. The manner of keeping corporate books and records. 3. The aforesaid control and breach of duty must proximately
cause the injury or unjust loss complained of.
4. Methods of conducting the business. 13
The absence of any one of these elements prevents
The SEC en banc explained the "instrumentality rule" which the courts have "piercing the corporate veil." In applying the "instrumentality"
applied in disregarding the separate juridical personality of corporations as or "alter ego" doctrine, the courts are concerned with reality
follows: and not form, with how the corporation operated and the
individual defendant's relationship to that operation. 14
Where one corporation is so organized and controlled and its
affairs are conducted so that it is, in fact, a mere Thus the question of whether a corporation is a mere alter ego, a mere sheet
instrumentality or adjunct of the other, the fiction of the or paper corporation, a sham or a subterfuge is purely one of fact. 15
corporate entity of the "instrumentality" may be disregarded.
The control necessary to invoke the rule is not majority or In this case, the NLRC noted that, while petitioner claimed that it ceased its
even complete stock control but such domination of business operations on April 29, 1986, it filed an Information Sheet with the
instances, policies and practices that the controlled Securities and Exchange Commission on May 15, 1987, stating that its office
corporation has, so to speak, no separate mind, will or address is at 355 Maysan Road, Valenzuela, Metro Manila. On the other
existence of its own, and is but a conduit for its principal. It hand, HPPI, the third-party claimant, submitted on the same day, a similar
must be kept in mind that the control must be shown to have information sheet stating that its office address is at 355 Maysan Road,
been exercised at the time the acts complained of took Valenzuela, Metro Manila.
place. Moreover, the control and breach of duty must
proximately cause the injury or unjust loss for which the Furthermore, the NLRC stated that:
complaint is made.
Both information sheets were filed by the same Virgilio O.
The test in determining the applicability of the doctrine of piercing the veil of Casio as the corporate secretary of both corporations. It
corporate fiction is as follows: would also not be amiss to note that both corporations had
the same president, the same board of directors,
1. Control, not mere majority or complete stock control, but the same corporate officers, and substantially
complete domination, not only of finances but of policy and the same subscribers.
business practice in respect to the transaction attacked so
that the corporate entity as to this transaction had at the time From the foregoing, it appears that, among other things, the
no separate mind, will or existence of its own; respondent (herein petitioner) and the third-party claimant
shared the same address and/or premises. Under this
2. Such control must have been used by the defendant to circumstances, (sic) it cannot be said that the property levied
commit fraud or wrong, to perpetuate the violation of a upon by the sheriff were not of respondents. 16
statutory or other positive legal duty or dishonest and unjust
act in contravention of plaintiff's legal rights; and

431
Clearly, petitioner ceased its business operations in order to evade the In view of the failure of the sheriff, in the case at bar, to effect a levy upon the
payment to private respondents of back wages and to bar their reinstatement property subject of the execution, private respondents had no other recourse
to their former positions. HPPI is obviously a business conduit of petitioner but to apply for a break-open order after the third-party claim of HPPI was
corporation and its emergence was skillfully orchestrated to avoid the dismissed for lack of merit by the NLRC. This is in consonance with Section
financial liability that already attached to petitioner corporation. 3, Rule VII of the NLRC Manual of Execution of Judgment which provides
that:
The facts in this case are analogous to Claparols v. Court of Industrial
Relations, 17 where we had the occasion to rule: Should the losing party, his agent or representative, refuse or
prohibit the Sheriff or his representative entry to the place
Respondent court's findings that indeed the Claparols Steel where the property subject of execution is located or kept,
and Nail Plant, which ceased operation of June 30, 1957, the judgment creditor may apply to the Commission or Labor
was SUCCEEDED by the Claparols Steel Corporation Arbiter concerned for a break-open order.
effective the next day, July 1, 1957, up to December 7, 1962,
when the latter finally ceased to operate, were not disputed Furthermore, our perusal of the records shows that the twin requirements of
by petitioner. It is very clear that the latter corporation was a due notice and hearing were complied with. Petitioner and the third-party
continuation and successor of the first entity . . . . Both claimant were given the opportunity to submit evidence in support of their
predecessors and successor were owned and controlled by claim.
petitioner Eduardo Claparols and there was no break in the
succession and continuity of the same business. This Hence, the NLRC did not commit any grave abuse of discretion when it
"avoiding-the-liability" scheme is very patent, considering affirmed the break-open order issued by the Labor Arbiter.
that 90% of the subscribed shares of stock of the Claparols
Steel Corporation (the second corporation) was owned by Finally, we do not find any reason to disturb the rule that factual findings of
respondent . . . Claparols himself, and all the assets of the quasi-judicial agencies supported by substantial evidence are binding on this
dissolved Claparols Steel and Nail plant were turned over to Court and are entitled to great respect, in the absence of showing of grave
the emerging Claparols Steel Corporation. abuse of a discretion. 18

It is very obvious that the second corporation seeks the WHEREFORE, the petition is DISMISSED and the assailed resolutions of the
protective shield of a corporate fiction whose veil in the NLRC, dated April 23, 1992 and December 3, 1992, are AFFIRMED.
present case could, and should, be pierced as it was
deliberately and maliciously designed to evade its financial
SO ORDERED.
obligation to its employees.

Padilla, Bellosillo, Vitug and Kapunan, JJ., concur.

432
G.R. No. L-28694 May 13, 1981

TELEPHONE ENGINEERING & SERVICE COMPANY, INC., petitioner,


vs.
WORKMEN'S COMPENSATION COMMISSION, PROVINCIAL SHERIFF
OF RIZAL and LEONILA SANTOS GATUS, for herself and in behalf of
her minor children, Teresita, Antonina and Reynaldo, all surnamed
GATUS, respondents.

MELENCIO-HERRERA, J.:1wph1.t

These certiorari proceedings stem from the award rendered against


petitioner Telephone Engineering and Services, Co., Inc. (TESCO) on
October 6, 1967 by the Acting Referee of Regional Office No. 4, Quezon City
Sub-Regional Office, Workmen's Compensation Section, in favor of
respondent Leonila S. Gatus and her children, dependents of the deceased
employee Pacifico L. Gatus. The principal contention is that the award was
rendered without jurisdiction as there was no employer-employee
relationship between petitioner and the deceased.

Petitioner is a domestic corporation engaged in the business of


manufacturing telephone equipment with offices at Sheridan Street,
Mandaluyong, Rizal. Its Executive Vice-President and General Manager is
Jose Luis Santiago. It has a sister company, the Utilities Management
Corporation (UMACOR), with offices in the same location. UMACOR is also
under the management of Jose Luis Santiago.

433
On September 8, 1964, UMACOR employed the late Pacifica L. Gatus as is not an occupational disease, hence, not compensable under the law. 8 The
Purchasing Agent. On May 16, 1965, Pacifico L. Gatus was detailed with extension requested was denied. The Motion for Reconsideration was
petitioner company. He reported back to UMACOR on August 1, 1965. On likewise denied in an Order issued by the Chief of Section of the Regional
January 13, 1967, he contracted illness and although he retained to work on Office dated December 28, 1967 9 predicated on two grounds: that the
May 10, 1967, he died nevertheless on July 14, 1967 of "liver cirrhosis with alleged mistake or negligence was not excusable, and that the basis of the
malignant degeneration." award was not the theory of direct causation alone but also on that of
aggravation. On January 28, 1968, an Order of execution was issued by the
On August 7, 1967, his widow, respondent Leonila S. Gatus, filed a "Notice same Office.
and Claim for Compensation" with Regional Office No. 4, Quezon City Sub-
Regional Office, Workmen's Compensation Section, alleging therein that her On February 3, 1968, petitioner filed an "Urgent Motion to Compel Referee to
deceased husband was an employee of TESCO, and that he died of liver Elevate the Records to the Workmen's Compensation Commission for
cirrhosis. 1 On August 9, 1967, and Office wrote petitioner transmitting the Review." 10 Meanwhile, the Provincial Sheriff of Rizal levied on and attached
Notice and for Compensation, and requiring it to submit an Employer's the properties of TESCO on February 17, 1968, and scheduled the sale of
Report of Accident or Sickness pursuant to Section 37 of the Workmen's the same at public auction on February 26, 1968. On February 28, 1968, the
Compensation Act (Act No. 3428). 2 An "Employer's Report of Accident or Commission issued an Order requiring petitioner to submit verified or true
Sickness" was thus submitted with UMACOR indicated as the employer of copies of the Motion for Reconsideration and/or Petition to Set Aside Award
the deceased. The Report was signed by Jose Luis Santiago. In answer to and Order of December 28, 1967, and to show proof that said Motion for
questions Nos. 8 and 17, the employer stated that it would not controvert the Reconsideration was filed within the reglementary period, with the warning
claim for compensation, and admitted that the deceased employee that failure to comply would result in the dismissal of the Motion. However,
contracted illness "in regular occupation." 3 On the basis of this Report, the before this Order could be released, TESCO filed with this Court, on
Acting Referee awarded death benefits in the amount of P5,759.52 plus February 22, 1968, The present petition for "Certiorari with Preliminary
burial expenses of P200.00 in favor of the heirs of Gatus in a letter-award Injunction" seeking to annul the award and to enjoin the Sheriff from levying
dated October 6, 1967 4 against TESCO. and selling its properties at public auction.

Replying on October 27, 1967, TESCO, through Jose Luis Santiago, On February 29, 1968, this Court required respondents to answer the
informed the Acting Referee that it would avail of the 15-days-notice given to Petition but denied Injunction. 11 TESCO'S Urgent Motion dated April 2, 1968,
it to state its non-conformity to the award and contended that the cause of for the issuance of a temporary restraining order to enjoin the Sheriff from
the illness contracted by Gatus was in no way aggravated by the nature of proceeding with the auction sale of its properties was denied in our
his work. 5 Resolution dated May 8, 1968.

On November 6, 1967, TESCO requested for an extension of ten days within TESCO asserts: 1wph1.t
which to file a Motion for Reconsideration, 6 and on November 15, 1967,
asked for an additional extension of five days. 7 TESCO filed its "Motion for I. That the respondent Workmen's Compensation
Reconsideration and/or Petition to Set Aside Award" on November 18, 1967, Commission has no jurisdiction nor authority to render the
alleging as grounds therefor, that the admission made in the "Employer's award (Annex 'D', Petition) against your petitioner there
Report of Accident or Sickness" was due to honest mistake and/or excusable
negligence on its part, and that the illness for which compensation is sought

434
being no employer-employee relationship between it and the "Motion for Reconsideration and/or Petition to Set Aside Award," and in its
deceased Gatus; "Urgent Motion to Compel the Referee to Elevate Records to the
Commission for Review," petitioner represented and defended itself as the
II. That petitioner can never be estopped from questioning employer of the deceased. Nowhere in said documents did it allege that it
the jurisdiction of respondent commission especially was not the employer. Petitioner even admitted that TESCO and UMACOR
considering that jurisdiction is never conferred by the acts or are sister companies operating under one single management and housed in
omission of the parties; the same building. Although respect for the corporate personality as such, is
the general rule, there are exceptions. In appropriate cases, the veil of
III. That this Honorable Court has jurisdiction to nullify the corporate fiction may be pierced as when the same is made as a shield to
award of respondent commission. confuse the legitimate issues. 16

TESCO takes the position that the Commission has no jurisdiction to render While, indeed, jurisdiction cannot be conferred by acts or omission of the
a valid award in this suit as there was no employer-employee relationship parties, TESCO'S denial at this stage that it is the employer of the deceased
between them, the deceased having been an employee of UMACOR and not is obviously an afterthought, a devise to defeat the law and evade its
of TESCO. In support of this contention, petitioner submitted photostat obligations. 17 This denial also constitutes a change of theory on appeal
copies of the payroll of UMACOR for the periods May 16-31, 1967 and June which is not allowed in this jurisdiction. 18Moreover, issues not raised before
1-15, 1967 12 showing the name of the deceased as one of the three the Workmen's Compensation Commission cannot be raised for the first time
employees listed under the Purchasing Department of UMACOR. It also on appeal. 19 For that matter, a factual question may not be raised for the first
presented a photostat copy of a check of UMACOR payable to the deceased time on appeal to the Supreme Court. 20
representing his salary for the period June 14 to July 13, 1967. 13
This certiorari proceeding must also be held to have been prematurely
Both public and private respondents contend, on the other hand, that TESCO brought. Before a petition for certiorari can be instituted, all remedies
is estopped from claiming lack of employer employee relationship. available in the trial Court must be exhausted first. 21 certiorari cannot be
resorted to when the remedy of appeal is present. 22 What is sought to be
annulled is the award made by the Referee. However, TESCO did not pursue
To start with, a few basic principles should be re-stated the existence of
the remedies available to it under Rules 23, 24 and 25 of the Rules of the
employer-employee relationship is the jurisdictional foundation for recovery
Workmen's Compensation Commission, namely, an appeal from the award of
of compensation under the Workmen's Compensation Law. 14 The lack of
the Referee, within fifteen days from notice, to the Commission; a petition for
employer-employee relationship, however, is a matter of defense that the
reconsideration of the latter's resolution, if adverse, to the Commission en
employer should properly raise in the proceedings below. The determination
banc; and within ten days from receipt of an unfavorable decision by the
of this relationship involves a finding of fact, which is conclusive and binding
latter, an appeal to this Court. As petitioner had not utilized these remedies
and not subject to review by this Court. 15
available to it, certiorari win not he, it being prematurely filed. As this Court
ruled in the case of Manila Jockey Club, Inc. vs. Del Rosario, 2 SCRA 462
Viewed in the light of these criteria, we note that it is only in this Petition (1961). 1wph1.t
before us that petitioner denied, for the first time, the employer-employee
relationship. In fact, in its letter dated October 27, 1967 to the Acting Referee,
An aggrieved party by the decision of a Commissioner
in its request for extension of time to file Motion for Reconsideration, in its
should seek a reconsideration of the decision by the

435
Commission en banc. If the decision is adverse to him, he pains, fever and headache. He reported to petitioner the fact of his having
may appeal to the Supreme Court. An appeal brought to the vomitted blood. He was sent to petitioner company's physician, Dr. Roman,
Supreme Court without first resorting to the remedy referred who treated him and sent him to Sto. Tomas Hospital where he was confined
to is premature and may be dismissed. for six days. Thereafter, he was admitted at the Quezon Institute. There he
stayed until March 19, 1962 under the medical care of Dr. Mario Lirag. Dr.
Although this rule admits of exceptions, as where public welfare and the Lirag diagnosed his ailment as pulmonary tuberculosis, moderately advanced
advancement of public policy so dictate, the broader interests of justice so in both lungs. Upon his discharge on March 19, 1962, he was clinically
require, or where the Orders complained of were found to be completely null improved. His X-ray examination, however, showed the same finding, i.e.,
and void or that the appeal was not considered the appropriate remedy, 23 the PTB, moderately advanced. He has not resumed work.
case at bar does not fan within any of these exceptions. WHEREFORE, this
Petition is hereby dismissed. Offshoot of the foregoing is respondent's claim filed on May 9, 1962
with the Workmen's Compensation Commission. 1 In affirming the decision of
SO ORDERED. the Hearing Officer, the Commission ordered petitioner:

Teehankee (Chairman), Makasiar, Fernandez and Guerrero, JJ., 1. To pay the claimant, thru this Commission, the sum of THREE
concur.1wph1.t THOUSAND SEVEN HUNDRED THIRTY-TWO and 30/100
(P3,732.30) PESOS as compensation as of August 11, 1964, and
P27.30 thereafter up to a period of 208 weeks, but in no case said
amount of compensation exceed P4,000.00;
G.R. No. L-23586 March 20, 1968
2. To reimburse the claimant, thru this Commission, the sum of
P53.60 which he had actually spent for his treatment;
A. D. SANTOS, INC., petitioner,
vs.
VENTURA VASQUEZ, respondent. 3. To provide claimant continuous medical, surgical and hospital
services and supplies as his illness may warrant;
Emiliano S. Samson and R. Balderrama-Samson for petitioner.
Orlando L. Espinas for respondent. 4. To pay the claimant, also thru this Commission, the sum of
P277.92 as Attorney's fees; and

5. To pay the Commission the sum of P43.00 as costs based on the


amount of compensation already due the claimant as of August 11,
SANCHEZ, J.:
1964, and P1.00 for every hundred pesos which may accrue in his
favor as weekly compensation pursuant to Section 55 of the Act.
Respondent Ventura Vasquez was petitioner's taxi driver. Sometime on
December 22 or 23, 1961, at about 11:00 a.m., while driving petitioner's
The case is now before us on review.
taxicab, he vomitted blood. Aside from his hemoptysis, he suffered back

436
Two questions are raised by petitioner: (1) respondent's claim should his testimony, in the words of this Court in Sugay vs. Reyes, L-20451,
have been dismissed for his failure to file the notice of injury and claim for December 28, 1964, "should not be allowed to confuse the facts relating to
compensation required by Section 24 of the Workmen's Compensation Act; employer-employee relationship" for "when the veil of corporate fiction is
and (2) the claim for compensation is directed against Amador Santos, not made as a shield to perpetrate a fraud and/or confuse legitimate issues
against petitioner. (here, the relation of employer-employee), the same should be pierced."

1. Sickness manifested itself on December 22 or 23, 1961. Claim was For the reasons given, the decision under review is hereby
filed on May 9, 1962. Petitioner argues that by Section 24 of the Workmen's affirmed.1wph1.t
Compensation Act, the claim should be thrown out of court. Because,
according to petitioner, such claim was not filed within two months following Costs against petitioner. So ordered.
illness.
Reyes, J.B.L., Dizon, Makalintal, Bengzon, J.P., Zaldivar, Castro, Angeles
Petitioner's case must fail. Stabilized jurisprudence is that failure of the and Fernando, JJ., concur
employer to file with the Commission notice of controversion set forth in the
second paragraph of Section 45 of the Workmen's Compensation Act is a
waiver of the defense that the claim for compensation was not filed within the
statutory period and a forfeiture of the employer's right to controvert the
G.R. No. L-10510 March 17, 1961
claim. Petitioner here knew of respondent's illness. Yet, it did not controvert
respondent's right to compensation. Constructively, such failure is an
admission that the claim is compensable. 2 M. MC CONNEL, W. P. COCHRANE, RICARDO RODRIGUEZ, ET
AL., petitioners,
vs.
2. Petitioner's averment that respondent driver had no cause of action
THE COURT OF APPEALS and DOMINGA DE LOS REYES, assisted by
against petitioner is equally without merit. Respondent's claim for
her husband, SABINO PADILLA,respondents.
compensation herein is directed against petitioner A.D. Santos, Inc.
Petitioner, in answer to the claim, categorically admitted that claimant was its
taxi driver. Add to this is the fact that the claimant contracted pulmonary Jesus B. Santos and Cornelio Antiquera for petitioners.
tuberculosis by reason of his said employment. And respondent's cause of Teodoro Padilla for respondents.
action against petitioner is complete.
REYES, J.B.L., J.:
But petitioner, cites the fact that respondent driver, in the course of his
testimony, mentioned that he worked for the City Cab operated by Amador The issue before us in the correctness of the decision of the Court of Appeals
Santos. This will not detract from the validity of respondent's right to that, under the circumstances of record, there was justification for
compensation. For, the truth is that really at one time Amador Santos was the disregarding the corporate entity of the Park Rite Co., Inc., and holding its
sole owner and operator of the City Cab. It was subsequently transferred to controlling stockholders personally responsible for a judgment against the
petitioner A.D. Santos, Inc. in which Amador Santos was an officer. The corporation.
mention by respondent of Amador Santos as his employer in the course of

437
The Court of Appeals found that the Park Rite Co., Inc., a Philippine WHEREFORE, premises considered, the decision appealed from is
corporation, was originally organized on or about April 15, 1947, with a reversed. Defendants-appellees Cirilo Paredes and Ursula Tolentino
capital stock of 1,500 shares at P1.00 a share. The corporation leased from are hereby declared liable to the plaintiffs-appellants for the rentals
Rafael Perez Rosales y Samanillo a vacant lot on Juan Luna street (Manila) due on the lot in question from August 22, 1947 to January 31, 1948
which it used for parking motor vehicles for a consideration. at the rate of P1,235.00 a month, with legal interest thereon from the
time of the filing of the complaint. Deducting the P550.00 which was
It turned out that in operating its parking business, the corporation occupied paid at the time when the corporation was already acquired by the
and used not only the Samanillo lot it had leased but also an adjacent lot said defendants-appellees Cirilo Paredes and Ursula Tolentino, they
belonging to the respondents-appellees Padilla, without the owners' are hereby ordered to pay to plaintiffs-appellants Dominga de los
knowledge and consent. When the latter discovered the truth around October Reyes and Sabino Padilla the sum of P6,036.66 with legal interest
of 1947, they demanded payment for the use and occupation of the lot. therein from the time of the filing of the complaint until fully paid.

The corporation (then controlled by petitioners Cirilo Parades and Ursula Defendant-appellee RICARDO RODRIGUEZ is hereby ordered to
Tolentino, who had purchased and held 1,496 of its 1,500 shares) disclaimed pay to the plaintiffs-appellants Dominga de los Reyes and Sabino
liability, blaming the original incorporators, McConnel, Rodriguez and Padilla the sum of P1,742.64 with legal interest thereon from the time
Cochrane. Whereupon, the lot owners filed against it a complaint for forcible of the filing of the complaint and until it is fully paid. In addition
entry in the Municipal Court of Manila on 7 October 1947 (Civil Case No. thereto the defendants-appellees Cirilo Paredes, Ursula Tolentino
4031). and Ricardo Rodriguez shall pay the costs proportionately in both
instances.
Judgment was rendered in due course on 13 November 1947, ordering the
Park Rite Co., Inc. to pay P7,410.00 plus legal interest as damages from IT IS SO ORDERED.
April 15, 1947 until return of the lot. Restitution not having been made until
31 January 1948, the entire judgment amounted to P11,732.50. Upon Cirilo Paredes and Ursula Tolentino then resorted to this court. We
execution, the corporation was found without any assets other than P550.00 granted certiorari.
deposited in Court. After their application to the judgment credit, there
remained a balance of P11,182.50 outstanding and unsatisfied. On the main issue whether the individual stockholders maybe held liable for
obligations contracted by the corporation, this Court has already answered
The judgment creditors then filed suit in the Court of First Instance of Manila the question in the affirmative wherever circumstances have shown that the
against the corporation and its past and present stockholders, to recover corporate entity is being used as an alter ego or business conduit for the sole
from them, jointly and severally, the unsatisfied balance of the judgment, plus benefit of the stockholders, or else to defeat public convenience, justify
legal interest and costs. The Court of First Instance denied recovery; but on wrong, protect fraud, or defend crime (Koppel [Phil.] Inc. vs. Yatco, 77 Phil.
appeal, the Court of Appeals (CA-G.R. No. 8434-R) reversed, finding that the 496; Arnold vs. Willits and Patterson, 44 Phil. 364).
corporation was a mere alter ego or business conduit of the principal
stockholders that controlled it for their own benefit, and adjudged them The Court of Appeals has made express findings to the following effect:
responsible for the amounts demanded by the lot owners, as follows:

438
There is no question that a wrong has been committed by the so- The facts thus found can not be varied by us, and conclusively show that the
called Park Rite Co., Inc., upon the plaintiffs when it occupied the lot corporation is a mere instrumentality of the individual stockholder's, hence
of the latter without its prior knowledge and consent and without the latter must individually answer for the corporate obligations. While the
paying the reasonable rentals for the occupation of said lot. There is mere ownership of all or nearly all of the capital stock of a corporation is a
also no doubt in our mind that the corporation was a mere alter ego mere business conduit of the stockholder, that conclusion is amply justified
or business conduit of the defendants Cirilo Paredes and Ursula where it is shown, as in the case before us, that the operations of the
Tolentino, and before them the defendants M. McConnel, W. P. corporation were so merged with those of the stockholders as to be
Cochrane, and Ricardo Rodriguez. The evidence clearly shows that practically indistinguishable from them. To hold the latter liable for the
these persons completely dominated and controlled the corporation corporation's obligations is not to ignore the corporation's separate entity, but
and that the functions of the corporation were solely for their merely to apply the established principle that such entity can not be invoked
benefits. or used for purposes that could not have been intended by the law that
created that separate personality.
When it was originally organized on or about April 15, 1947, the
original incorporators were M. McConnel, W. P. Cochrane, Ricardo The petitioners-appellants insist that the Court could have no jurisdiction over
Rodriguez, Benedicto M. Dario and Aurea Ordrecio with a capital an action to enforce a judgment within five (5) years from its rendition, since
stock of P1,500.00 divided into 1,500 shares at P1.00 a share. the Rules of Court provide for enforcement by mere motion during those five
McConnel and Cochrane each owned 500 shares, Ricardo years. The error of this stand is apparent, because the second action,
Rodriguez 408 shares, and Dario and Ordrecio 1 share each. It is originally begun in the Court of First Instance, was not an action to enforce
obvious that the shares of the last two named persons were merely the judgment of the Municipal Court, but an action to have non-parties to the
qualifying shares. Then or about August 22, 1947 the defendants judgment held responsible for its payment.
Cirilo Paredes and Ursula Tolentino purchased 1,496 shares of the
said corporation and the remaining four shares were acquired by Finding no error in the judgment appealed from, the same is hereby affirmed,
Bienvenido J. Claudio, Quintin C. Paredes, Segundo Tarictican, and with costs against petitioners-appellants Cirilo Paredes and Ursula Tolentino.
Paulino Marquez at one share each. It is obvious that the last four
shares bought by these four persons were merely qualifying shares Bengzon, Actg. C.J., Bautista, Angelo, Labrador, Barrera and Dizon,
and that to all intents and purposes the spouses Cirilo Paredes and JJ., concur.
Ursula Tolentino composed the so-called Park Rite Co., Inc. That the Concepcion and Paredes, JJ., took no part.
corporation was a mere extension of their personality is shown by
the fact that the office of Cirilo Paredes and that of Park Rite Co.,
Inc. were located in the same building, in the same floor and in the
same room at 507 Wilson Building. This is further shown by the
fact that the funds of the corporation were kept by Cirilo Paredes in
his own name (p. 14, November 8, 1950, T.S.N.) The corporation
itself had no visible assets, as correctly found by the trial court,
except perhaps the toll house, the wire fence around the lot and the
signs thereon. It was for this reason that the judgment against it
could not be fully satisfied. (Emphasis supplied).

439
G.R. No. L-20502 February 26, 1965

EMILIO CANO ENTERPRISES, INC., petitioner,


vs.
COURT OF INDUSTRIAL RELATIONS, ET AL., respondents.

D. T. Reyes and Associates for petitioner.


Mariano B. Tuason for respondent Court of Industrial Relations.
C. E. Santiago for respondent Honorata Cruz.

BAUTISTA ANGELO, J.:

In a complaint for unfair labor practice filed before the Court of Industrial
Relations on June 6, 1956 by a prosecutor of the latter court, Emilio, Ariston
and Rodolfo, all surnamed Cano, were made respondents in their capacity as
president and proprietor, field supervisor and manager, respectively, of Emilio
Cano Enterprises, Inc.

After trial, Presiding Judge Jose S. Bautista rendered decision finding Emilio
Cano and Rodolfo Cano guilty of the unfair labor practice charge, but
absolved Ariston for insufficiency of evidence. As a consequence, the two
were ordered, jointly and severally, to reinstate Honorata Cruz, to her former
position with payment of backwages from the time of her dismissal up to her
reinstatement, together with all other rights and privileges thereunto
appertaining.

440
Meanwhile, Emilio Cano died on November 14, 1958, and the attempt to when invoked in support of an end subversive of this policy it should be
have the case dismissed against him having failed, the case was appealed to disregarded by the courts (12 Am. Jur. 160-161).
the court en banc, which in due course affirmed the decision of Judge
Bautista. An order of execution was issued on August 23, 1961 the A factor that should not be overlooked is that Emilio and Rodolfo Cano are
dispositive part of which reads: (1) to reinstate Honorata Cruz to her former here indicted, not in their private capacity, but as president and manager,
position as ordered in the decision; and (2) to deposit with the court the respectively, of Emilio Cano Enterprises, Inc. Having been sued officially their
amount of P7,222.58 within ten days from receipt of the order, failing which connection with the case must be deemed to be impressed with the
the court will order either a levy on respondents' properties or the filing of an representation of the corporation. In fact, the court's order is for them to
action for contempt of court. reinstate Honorata Cruz to her former position in the corporation and
incidentally pay her the wages she had been deprived of during her
The order of execution having been directed against the properties of Emilio separation. Verily, the order against them is in effect against the corporation.
Cano Enterprises, Inc. instead of those of the respondents named in the No benefit can be attained if this case were to be remanded to the court a
decision, said corporation filed an ex parte motion to quash the writ on the quomerely in response to a technical substitution of parties for such would
ground that the judgment sought to be enforced was not rendered against it only cause an unwarranted delay that would work to Honorata's prejudice.
which is a juridical entity separate and distinct from its officials. This motion This is contrary to the spirit of the law which enjoins a speedy adjudication of
was denied. And having failed to have it reconsidered, the corporation labor cases disregarding as much as possible the technicalities of procedure.
interposed the present petition for certiorari.1wph1.t We, therefore, find unmeritorious the relief herein prayed for.

The issue posed before us is: Can the judgment rendered against Emilio and WHEREFORE, petition is dismissed, with costs.
Rodolfo Cano in their capacity as officials of the corporation Emilio Cano
Enterprises, Inc. be made effective against the property of the latter which Bengzon, C.J., Concepcion, Reyes, J.B.L., Barrera, Paredes, Dizon, Regala,
was not a party to the case? Makalintal, Bengzon, J.P., and Zaldivar, JJ., concur.

The answer must be in the affirmative. While it is an undisputed rule that a


corporation has a personality separate and distinct from its members or
stockholders because of a fiction of the law, here we should not lose sight of
the fact that the Emilio Cano Enterprises, Inc. is a closed family corporation
where the incorporators and directors belong to one single family. Thus, the
following are its incorporators: Emilio Cano, his wife Juliana, his sons
Rodolfo and Carlos, and his daughter-in-law Ana D. Cano. Here is an
instance where the corporation and its members can be considered as one.
And to hold such entity liable for the acts of its members is not to ignore the
legal fiction but merely to give meaning to the principle that such fiction
cannot be invoked if its purpose is to use it as a shield to further an end
subversive of justice. 1 And so it has been held that while a corporation is a
legal entity existing separate and apart from the persons composing it, that
concept cannot be extended to a point beyond its reason and policy, and

441
bars of "Busilak" and 17,285.08 piculs of "Pasumil" domestic raw sugar. As
ASSOCIATED failed to deliver to NAMARCO the 22,516 bags of "Victoria"
G.R. No. L-20886 April 27, 1967 and/or "National" refined sugar agreed upon, the latter, on January 12, 1959,
demanded in writing from the ASSOCIATED either (a) immediate delivery
NATIONAL MARKETING CORPORATION (NAMARCO), plaintiff-appellant, thereof before January 20, or (b) payment of its equivalent cash value
vs. amounting to P372,639.80.
ASSOCIATED FINANCE COMPANY, INC., and FRANCISCO
SYCIP, defendants. On January 19, 1959, ASSOCIATED, through Sycip, offered to pay
FRANCISCO SYCIP, defendant-appellee. NAMARCO the value of 22,516 bags of refined sugar at the rate of P15.30
per bag, but the latter rejected the offer. Instead, on January 21 of the same
Tomas P. Matic, Jr,. for plaintiff and appellant. year it demanded payment of the 7,732.71 bags of "Busilak" raw sugar at
Francisco Sycip in his behalf as defendant and appellee. P15.30 per bag, amounting to P118,310.40. and of the 17,285.08 piculs of
"Pasumil" raw sugar at P16.50 per picul, amounting, to P285.203.82, or a
total price of P403,514.28 for both kinds of sugar, based on the sugar
DIZON, J.:
quotations (Exh. H) as of March 20, 1958 the date when the exchange
agreement was entered into.
Appeal by the National Marketing Corporation hereinafter referred to as
NAMARCO, from the decision of the Court of First Instance of Manila in Civil
As ASSOCIATED refused to deliver the raw sugar or pay for the refined
Case No. 45770 ordering the Associated Finance Company, Inc.
sugar delivered to it, inspite of repeated demands therefore, NAMARCO
hereinafter referred to as the ASSOCIATED to pay the NAMARCO the
instituted the present action in the lower court to recover the sum of
sum of P403,514.28, with legal interest thereon from the date of filing of the
P403,514.28 in payment of the raw sugar received by defendants from it;
action until fully paid, P80,702.26 as liquidated damages, P5,000.00 as
P80,702.86 as liquidated damages; P10,000.00 as attorney's fees, expenses
attorney's fees, plus costs, but dismissing the complaint insofar as defendant
of litigation and exemplary damages, with legal interest thereon from the
Francisco Sycip was concerned, as well as the latter's counterclaim. The
filing of the complaint until fully paid.
appeal is only from that portion of the decision dismissing the case as
against Francisco Sycip.
In their amended answer defendants, by way of affirmative defenses, alleged
that the correct value of the sugar delivered by NAMARCO to them was
On March 25, 1958, ASSOCIATED, a domestic corporation, through its
P259,451.09 or P13.30 per bag of 100 lbs. weight (quedan basis) and not
President, appellee Francisco Sycip, entered into an agreement to exchange
P403,514.38 as claimed by NAMARCO. As counterclaim they prayed for the
sugar with NAMARCO, represented by its then General Manager, Benjamin
award of P500,000.00 as moral damages, P100,000.00 as exemplary
Estrella, whereby the former would deliver to the latter 22,516 bags (each
damages and P10,000.00 as attorney's fees.
weighing 100 pounds) of "Victorias" and/or "National" refined sugar in
exchange for 7,732.71 bags of "Busilak" and 17,285.08 piculs of "Pasumil"
raw sugar belonging to NAMARCO, both agreeing to pay liquidated damages After due trial court rendered the appealed judgment. The appeal was taken
equivalent to 20% of the contractual value of the sugar should either party fail to the Court of Appeals, but on January 15, 1963 the latter certified the case
to comply with the terms and conditions stipulated (Exhibit A). Pursuant to us for final adjudication pursuant to sections 17 and 31 of the Judiciary Act
thereto, on May 19,1958, NAMARCO delivered to ASSOCIATED 7,732.71

442
of 1948, as amended, the amount involved being more than P200,000.00, liable for the corporate obligations. To the contrary, upon the proven facts,
exclusive of interests and cost. We feel perfectly justified in "piercing the veil of corporate fiction" and in
holding Sycip personally liable, jointly and severally with his co-defendant, for
The only issue to be resolved is whether, upon the facts found by the trial the sums of money adjudged in favor of appellant. It is settled law in this and
court, which, in our opinion, are fully supported by the evidence other jurisdictions that when the corporation is the mere alter ego of a
Francisco Sycip may be held liable, jointly and severally with his co- person, the corporate fiction may be disregarded; the same being true when
defendant, for the sums of money adjudged in favor of NAMARCO. the corporation is controlled, and its affairs are so conducted as to make it
merely an instrumentality, agency or conduit of another (Koppel Phils., etc.
The evidence of record shows that, of the capital stock of ASSOCIATED, vs. Yatco, etc., 43 O.G. No. 11. Nov. 1947; Yutivo Sons, etc. vs. Court of Tax
Sycip owned P60,000.00 worth of shares, while his wife the second Appeals, etc., G.R. No. L-13203, promulgated on January 28, 1961).
biggest stockholder owned P20,000.00 worth of shares; that the par value
of the subscribed capital stock of ASSOCIATED was only P105,000.00; that Wherefore, the decision appealed from is modified by sentencing defendant-
negotiations that lead to the execution of the exchange agreement in appellee Francisco Sycip to pay, jointly and severally with the Associated
question were conducted exclusively by Sycip on behalf of ASSOCIATED; Finance Company, Inc., the sum of money which the trial court sentenced the
that, as a matter of fact, in the course of his testimony, Sycip referred to latter to pay to the National Marketing Corporation, as follows: the sum of
himself as the one who contracted or transacted the business in his personal FOUR HUNDRED THREE THOUSAND FIVE HUNDRED FOURTEEN
capacity, and asserted that the exchange agreement was his personal PESOS, and TWENTY-EIGHT CENTAVOS P403,514.28), with interest at the
contract; that it was Sycip who made personal representations and gave legal rate from the date of the filing of the action until fully paid plus an
assurances that ASSOCIATED was in actual possession of the 22,516 bags additional amount of EIGHTY THOUSAND SEVEN HUNDRED TWO PESOS
of "Victorias" and/or "National" refined sugar which the latter had agreed to and EIGHTY-SIX CENTAVOS (P80,702.86) as liquidated damages and
deliver to NAMARCO, and that the same was ready for delivery; that, as a P5,000.00 as attorney's fees and further to pay the costs. With costs.
matter of fact, ASSOCIATED was at that time already insolvent; that when
NAMARCO made demands upon ASSOCIATED to deliver the 22,516 bags Concepcion, C.J., Reyes, J.B.L., Regala, Makalintal, Bengzon, J.P., Zaldivar
of refined sugar it was under obligation to deliver to the former, and Sanchez JJ., concur.
ASSOCIATED and Sycip, instead of making delivery of the sugar, offered to
pay its value at the rate of P15.30 per bag a clear indication that they did
not have the sugar contracted for.1wph1.t

The foregoing facts, fully established by the evidence, can lead to no other
conclusion than that Sycip was guilty of fraud because through false
representations he succeeded in inducing NAMARCO to enter into the
aforesaid exchange agreement, with full knowledge, on his part, on the fact
that ASSOCIATED whom he represented and over whose business and
affairs he had absolute control, was in no position to comply with the
obligation it had assumed. Consequently, he can not now seek refuge behind
the general principle that a corporation has a personality distinct and
separate from that of its stockholders and that the latter are not personally

443
444
thereon at the rate of 16 % per annum from January 1, 1979 up to
the time the said amount is fully paid, plus the sum of P20,000.00 as
attorney's fees. Said defendants are further ordered to pay
in solidum the costs of this suit.

SO ORDERED.2

Petitioner's co-defendant in the courts below, Inland Industries Inc., just as in


the case of petitioner's motion to reconsider the questioned decision, 3 chose
not to join him in this appeal.
G.R. No. 80043 June 6, 1991
In Our resolution of 28 August 1988 We required the respondent to comment
on the petition. Respondent Metropolitan Bank and Trust Co. filed its
ROBERTO A. JACINTO, petitioner,
comment4 on 12 October 1988. We required the petitioner to file a reply
vs.
thereto,5 which he comment plied with on 20 December 1988.6
HONORABLE COURT OF APPEALS and METROPOLITAN BANK AND
TRUST COMPANY, respondents.
We gave due course to the petition on 8 May 1989 7 and required the parties
to submit their respective memoranda.
Romeo G. Carlos for petitioner.
Jorge, Perez & Associates for private respondents.
Private respondent filed its memorandum on 29 June 1989 8 while petitioner
asked leave to adopt his petition and reply as his memorandum, 9 which We
granted on 14 June 1989.10

Petitioner submits the following issues:


DAVIDE, JR., J.:
1. Whether or not the respondent Court of Appeals can validly pierce
This is an appeal by certiorari to partially set aside the Decision of the Court
the fiction of corporate identity of the defendant corporation Inland
of Appeals in C.A-G.R. CV No. 08153 1.promulgated on 19 August 1987,
Industries, Inc. even if there is no allegation in the complaint
which affirmed in toto the decision of the Regional Trial Court of Manila,
regarding the same, nor is there anything in the prayer demanding
Branch 11, in Civil Case No. 133164 entitled "Metropolitan Bank and Trust
the piercing of the corporate veil of the corporation Inland Industries,
Co. vs. Inland Industries Inc. and Roberto Jacinto," the dispositive portion of
Inc.;
which reads:
2. Whether or not the Court of Appeals can validly pierce the fiction
WHEREFORE, judgment is hereby rendered ordering defendants to
of corporate identity of the defendant Inland Industries, Inc. even if
pay, jointly and severally, the plaintiff, the principal obligation of
absolutely no proof was presented in court to serve as legal
P382,015.80 (Annex J-1 to J-3 of Stipulation), with interest/charges
justification for the same.

445
We find this petition to be bereft of merit. The issues are basically factual and applied with great caution and not precipitately, because a dual
a careful scrutiny of the decisions of both courts below reveals that their personality by a corporation and its stockholders would defeat the
findings and conclusions on the matter of piercing the veil of corporate fiction principal purpose for which a corporation is formed. Upon the other
and on the liability of herein petitioner are overwhelmingly supported by the hand, plaintiff-appellee reiterated its allegation in the complaint that
evidence. defendant corporation is just a mere alter ego of defendant Roberto
Jacinto who is its President and General Manager, while the wife of
Insofar as material and relevant to the issues raised, the trial court found and the latter owns a majority of its shares of stock.
held:11
Defendants-appellants' assertion is plainly without legal basis. This is
As to [the] liability of [the] defendant Roberto A. Jacinto, it would shown by the undisputed fact that Roberto Jacinto even admitted
appear that he is in factetum (sic), or, in fact, the corporation itself that he and his wife own 52% of the stocks of defendant corporation
known as Inland Industries, Inc. Aside from the fact that he is (TSN, April 22, 1985, p. 6). We cannot accept as true the assertion of
admittedly the President and General Manager of the corporation defendant Jacinto that he only acted in his official capacity as
and a substantial stockholders (sic) thereof, it was defendant President and General Manager of Inland Industries, Inc. when he
Roberto A. Jacinto who dealt entirely with the plaintiff in those signed the aforesaid trust receipts. To Our mind the same is just a
transactions. In the Trust Receipts that he signed supposedly in clever ruse and a convenient ploy to thwart his personal liability
behalf of Inland Industries, Inc., it is not even mentioned that he did therefor by taking refuge under the protective mantle of the separate
so in this official capacity. corporate personality of defendant corporation.

xxx xxx xxx As could be expected, Roberto Jacinto in his direct testimony
presented a different corporate scenario regarding Inland Industries,
In this case, the Court is satisfied that Roberto A. Jacinto was Inc. and vehemently declared that it is Bienvenida Catabas who is its
practically the corporation itself, the Inland industries, Inc. President, while Aurora Heresa is its Chairman of the Board. His
assertion on this point, however, is not convincing in view of his
admission in the same breath, that his wife, Hedy U. Jacinto, own
In a detailed fashion, the respondent Court of Appeals brushed aside the
(sic) with him 52% of the shares of stock of said corporation. Indeed,
posturing of petitioner as follows:
this circumstance even if standing alone cannot but engender
in the most unprejudiced mind doubt and misgiving why Catabas and
Defendant Roberto Jacinto, tried to escape liability and shift the Heresa would be defendant corporation's President and Chairman of
entire blame under the trust receipts solely and exclusively on the Board, respectively. Pertinent portion of his testimony on this
defendant-appellant corporation. He asserted that he cannot be held point is quoted hereunder:
solidarily liable with the latter (defendant corporation) because he
just signed said instruments in his official capacity as president of
Atty. Carlos Do you know the defendant Inland Industries,
Inland Industries, Inc. and the latter (defendant corporation) has a
Inc.?
juridical personality distinct and separate from its officers and
stockholders. It is likewise asserted, citing an American case, that
the principle of piercing the fiction of corporate entity should be

446
A Yes, sir. Because I am the General Manager of this A About 52 % (Ibid., pp. 3-6)
corporation.
Furthermore, a cursory perusal of the Stipulation of facts clearly
Q Aside from being the General Manager of the defendant shows that defendant Roberto Jacinto acted in his capacity as
corporation are you in any other way connected with the President and General Manager of Inland Industries, Inc. when he
same? signed said trust receipts. Pertinent portion of his testimony are
quoted below:
A I am also a stockholder.
(d) All the goods covered by the three (3) Letters of Credit
Q Does your corporation have a Board of Directors? (Annexes "A", "B" & "C") and paid for under the Bills of
Exchange (Annexes "D", "E" & "F") were delivered to and
A Yes, sir. received by defendant Inland Industries, Inc. through its co-
defendant Roberto A. Jacinto, its President and General
Manager, who signed for and in behalf of defendant Inland
Q By the way, who are the stockholders of this corporation?
and agreed to the terms and conditions of three (3) separate
trust receipts covering the same and herein identified as
A Bienvenida Catabas, Aurora Heresa, Paz Yulo, Hedy Y. follows: . . . (p. 3 of Stipulations of Facts and Formulation of
Jacinto and myself. Issues [p. 95, Records]).

Q Who is the President of the defendant corporation? The conflicting statements by defendant Jacinto place in extreme
doubt his credibility anent his alleged participation in said
A Bienvenida Catabas. transactions and We are thus persuaded to agree with the findings of
the lower court that the latter (Roberto Jacinto) was practically the
Q Who is the Chairman of the Board? corporation itself. Indeed, a painstaking examination of the records
show that there is no clear-cut delimitation between the personality
A Aurora Heresa. of Roberto Jacinto as an individual and the personality of Inland
Industries, Inc. as a corporation.
Q Do you have any relation with Hedy Y. Jacinto?
The circumstances aforestated lead Us to conclude that the
A She is my wife. corporate veil that en-shrouds defendant Inland Industries, Inc. could
be validly pierced, and a host of cases decided by our High Court is
supportive of this view. Thus it held that "when the veil of corporate
Q If you combine the stockholdings of your wife together with
fiction is made as a shield to perpetuate fraud and/or confuse
yours and percentage wise, how much is your equity?
legitimate issues, the same should be pierced." (Republic vs. Razon,
20 SCRA 234; A.D. Santos, Inc. vs. Vasquez, 22 SCRA 1156; Emilio
Atty. Dizon raised some objections. However, the Court Cano Enterprises, Inc. vs. Court of Appeals, 13 SCRA 290). Almost
allowed the same.

447
in the same vein is the dictum enunciated by the same court in the Sec. 5. Amendment to conform to or authorize presentation of
case of Commissioner of Internal Revenue vs. Norton & Harrison evidence. When issues not raised by the pleadings are tried by
Co., (11 SCRA 714), that "Where a corporation is merely an adjunct, express or implied consent of the parties, they shall be treated in all
business conduit or alter ego, the fiction of separate and distinct respects, as if they had been raised in the pleadings. Such
corporate entity should be disregarded." amendment of the pleadings as may be necessary to cause them to
conform to the evidence and to raise these issues may be made
In its resolution of 29 September 1987, the respondent Court of Appeals, on upon motion of any party at any time, even after judgment; but failure
the contention again of petitioner that the finding that defendant corporation so to amend does not affect the trial of these issues. If the evidence
is his mere alter ego is not supported by the evidence and has no legal is objected to at the time of trial on the ground that it is not within the
justification, ruled that: issues made by the pleadings, the court may allow the pleadings to
be amended and shall do so freely when the presentation of the
The contention . . . is nothing but an empty assertion. A cursory merits of the action will be subserved thereby and the objecting party
perusal of the decision would at once readily show on pages 11-13 of fails to satisfy the court that the admission of such evidence would
the same that said factual findings of the court is well grounded as prejudice him in maintaining his action or defense upon the merits.
the same in fact even include a portion of the very testimony of said The court may grant continuance to enable the objecting party to
defendant-appellant admitting that he and his wife own 52% of the meet such evidence.
stocks of defendant corporation. The stipulation of facts also show
(sic) that appellant Roberto Jacinto acted in his capacity as Pursuant thereto, "when evidence is presented by one party, with the express
President/General Manager of defendant corporation and that "all the or implied consent of the adverse party, as to issues not alleged in the
goods covered by the three (3) Letters of Credit (Annexes "A", "B" & pleadings, judgment may be rendered validly as regards those issues, which
"C") and paid for under the Bills of Exchange (Annexes "D", "E" & shall be considered as if they have been raised in the pleadings. There is
"F") were delivered to and received by defendant Inland Industries, implied consent to the evidence thus presented when the adverse party fails
Inc. through its co-defendant Roberto A. Jacinto, its President and to object thereto.12
General Manager, who signed for and in behalf of defendant Inland
and agreed to the terms and conditions of three (3) separate trust WHEREFORE, for lack of merit, the Petition is DISMISSED with costs
receipts covering the same. against petitioner.

Petitioner, however, faults the courts below for piercing the veil of corporate SO ORDERED.
fiction despite the absence of any allegation in the complaint questioning the
separate identity and existence of Inland Industries, Inc. This is not Fernan, C.J., Gutierrez, Jr., Feliciano and Bidin, JJ., concur.
accurate.1wphi1 While on the face of the complaint there is no specific
allegation that the corporation is a mere alter ego of petitioner, subsequent
developments, from the stipulation of facts up to the presentation of evidence
and the examination of witnesses, unequivocally show that respondent
Metropolitan Bank and Trust Company sought to prove that petitioner and the
corporation are one or that he is the corporation. No serious objection was
heard from petitioner. Section 5 of Rule 10 of the Rules of Court provides:

448
G.R. No. 89804 October 23, 1992

CALVIN S. ARCILLA, petitioner,


vs.
THE HONORABLE COURT OF APPEALS and EMILIO
RODULFO, respondents.

DAVIDE, JR., J.:

This petition is a belated attempt to avoid the adverse amended decision of


public respondent, promulgated on 31 May 1989 in C.A.-G.R. No.
11389, 1 on the ground that petitioner is not personally liable for the amount
adjudged since the same constitutes a corporate liability which nevertheless
cannot even bind or be enforced against the corporation because it is not a
party in the collection suit filed before the trial court.

The procedural antecedents are not complicated.

On 4 June 1985, private respondent filed with the Regional Trial Court (RTC)
of Catanduanes a complaint for a sum of money against petitioner. 2 The
case was docketed as Civil Case No. 1992 and was assigned to Branch 42
thereof. It is alleged therein:

449
xxx xxx xxx In his Answer, 3 petitioner does not deny having had business transactions
with the private respondent but alleges that the professional relationship
3. That from late 1981 up to early 1983, the defendant, began only in August of 1982 when he "was looking for a "pro-forma" invoice
taking advantage of his close friendship with the plaintiff, to support his loan with the Kilusang Kabuhayan at Kaunlaran (KKK for short)
succeeded in securing on credit from the plaintiff, various under the Ministry of Human Settlement (sic)." 4 He explicitly admits that
items, cash and checks which the defendant encashed, in "(H)is loan was in the same of his family corporation, CSAR Marine
the total amount of P93,358.51, which the plaintiff willingly Resources,
extended because of the representations of the defendant Inc.;" 5 however, the "vales", more specifically Annexes "A" to "DD" of the
that he was a successful financial consultant of local and complaint, "were liquidated in the bank loan releases." 6 It is thus clear that
international businessmen; his main defense is payment; he did not interpose any other affirmative
defense.
4. That defendant's indebtedness referred to in the next
preceding paragraph, is shown and described in thirty (30) In his Pre-Trial Brief, 7 petitioner reiterated the earlier claim that his first
"vales" signed by him or by persons authorized by him, all of business dealing with the plaintiff (private respondent herein) was in August
which documents are in the possession of the plaintiff for of 1982. This time, however, he alleges that "as President of CSAR Marine
being unredeemed or unpaid, xerox copies attached as Resources, Inc., he requested for a pro-forma Invoice for said corporation to
Annexes "A" to "Z" and "AA" to "DD" which are hereby made support the loan application with the Kilusang Kabuhayan at Kaunlaran (KKK
integral parts hereof; for short), with the Ministry of Human Settlement (sic)." 8

5. That commencing with the summer months of 1983 up to In its Decision of 1 August 1986, 9 the trial court made the following findings
the time immediately before the filing of this complaint, the of fact:
plaintiff had made numerous demands for payment but the
respondent acted in gross and evident bad faith in refusing Defendant admitted the genuineness (sic) and due execution
to satisfy the plaintiff's plainly valid, just and demandable of Exhibits "A" to "DD" but, according to him, he already paid
claim; plaintiff P56,098.00 thru PNB Virac Branch, per Cash
Voucher dated September 28, 1982 (Exh. 3) and then
6. That the plaintiff is left without any recourse other than to P42,363.75 also thru PNB Virac Branch, per PNB check No.
enforce his claim in court and had to secure the services of 628861K dated December 16, 1982 (Exh. 1).
the undersigned counsel who charged the plaintiff with
P1,000.00 for accepting the case, P200.00 appearance fee Analyzing the evidence adduced by both parties, it ruled that since Exhibit "3"
for every appearance before this Court, and attorney's is dated 28 September 1982 and the "vales", Exhibits "A" to "DD", with the
contingent fee of 25% of the award in favor of the plaintiff; exception of Exhibits "K" in the amount of P1,730.00 and "Q" in the amount
plaintiff shall incur litigation expenses which may amount to of P10,765.00, were issued after said date, it could not have been in
no less than P5,000.00, all of which amounts are payment of the "vales" other than that evidenced by Exhibits "K" and "Q"
recoverable from the defendant. Considering, however, that the "vales" remained in the possession of the
private respondent, they are presumed to remain unpaid; in fact, private

450
respondent so testified that they were not paid at all. The court therefore February 1983, signed by Rafael Rodulfo, General Manager of the private
ordered petitioner to pay private respondent: respondent and addressed to Brig. Gen. Clemente Racela, then KKK
General Action Officer, categorically stating that "the account of CSAR
(a) the total amount of P92,358.43 covered by the "vales", Marine Resources, Inc. c/o Atty. Calvin Arcilla" is only P23,639.33; and (c)
plus interest thereon at the rate of twelve (12%) per cent per the evidence presented by both parties disclosure that "the subject account
annum from June 4, 1985 when the complaint was filed; are (sic) all in the name of CSAR MARINE RESOURCES, INC., a
corporation separate and distinct from the appellant;" such fact remains
(b) P9,000.00 for and as attorney's fees; and "uncontroverted" as shown by Exhibits "1", "3", "A" to "DD" adopted as
Exhibits "7" to "25" for the appellant." 14 He then prays that:
(c) the cost of suit. 10
. . . considering that appellees was not able to prove by
preponderance of evidence the alleged unpaid account of
Petitioner appealed this decision to the public respondent which docketed the
appellant, the decision promulgated on January 14, 1988 be
case as C.A.-G.R. CV No. 11389.
RECONSIDERED and a new one be entered REVERSING
the lower court decision and thereby ordering the
The public respondent affirmed the trial court's decision in its Decision of 14 DISMISSAL of plaintiff-appellee's complaint, with damages
January 1988. 11 As could be gleaned therefrom, petitioner's assigned errors and costs against appellee.
are as follows:
In the remote possibility, that the appellee's complaint cannot
. . . defendant raised as error of the court a quo in (sic) be dismissed outrightly, it is further prayed that his
holding that the "vales" (Exhs. A to DD) have not been paid; Honorable Tribunal orders (sic) a new trial for appellant to
that the presumption in favor of the plaintiff-appellee that present additional evidence he wanted to present in his
since he was in possession of the "vales" the same have not motion for new trial. 15
been paid, remained undisputed; that the total transaction
between the parties amount to more than P200,000.00; and
xxx xxx xxx
in rendering a decision in favor of the plaintiff-appellee plus
the award of attorney's fees in his favor. 12
Reacting to this motion, private respondent, in a "Manifestation dated 7
February 1988, informed the public respondent that in the interest of justice
On 5 February 1988, petitioner filed a motion to reconsider the aforesaid
and fair play, he interposes no objection to the alternative prayer for a new
decision 13 alleging therein, inter alia, that (a) the evidence showing payment
trial. 16 Hearing was thereafter conducted to receive the petitioner's so-called
of the "vales" is "uncontroverted", hence the presumption that they were not
newly discovered evidence consisting of the abovementioned letter of Rafael
paid simply because they remain in the possession of the creditor cannot
Rodulfo, dated 7 February 1983, to General Clemente A. Racela (Exh. "1"-
arise; (b) the alleged non-payment of the "vales" could have been further
Motion) wherein the former, as General Manager of private respondent's
explained if the trial court gave the appellant the opportunity to present sur-
Universal Enterprises, informed the latter that:
rebuttal witness and documentary evidence; besides, he has newly
discovered evidence invoked in a prayer for a new trial that was
nevertheless denied by the lower court which consists of a letter, dated 7

451
. . . Csar Marine Resources, Inc. c/o Atty. Calvin Arcilla has 3.3. Csar Marine Resources, Inc. is not a
an outstanding obligation of TWENTY THREE THOUSAND party in this case;
SIX PESOS to Universal Enterprises as a result of various
purchases of construction materials. 17 xxx xxx xxx

Thereafter, on 31 May 1989, the public respondent promulgated an Amended 5. It is rather confusing (sic) that defendant-appellant is
Decision, 18 the dispositive portion of which reads as follows: ordered to pay plaintiff-appellee in his capacity as President
of Csar Marine Resources, Inc. the said amount of
WHEREFORE, the decision of this Court promulgated on P23,639.33, when plaintiff-appellee for ulterior motives
January 14, 1988 is hereby reconsidered and a new one choose (sic) not to implead said corporation. It need not be
rendered, ordering defendant-appellant to pay plaintiff- emphasized that the personality and liability of the
appellee in his capacity as President of Csar Marine defendant-appellant and that of Csar Marine Resources,
Resources, Inc. the outstanding balance of P23,639.33 to Inc., as a corporation, are separate and distinct from its (sic)
Universal Enterprises, owned and operated by plaintiff- other. . . . . 21
appellee, plus interest at 12% per annum from June 4, 1985
when the complaint was filed; attorney's fees of P1,000.00, He then prays that:
P200.00 per court appearance of counsel and 25% of the
amount awarded; plus the costs of the suit. 19 . . . an order be issued clarifying the liability of defendant-
appellant in his personal capacity as regards the amount of
On 4 January 1989, petitioner filed a Motion For Clarificatory P23,639.33, if any, otherwise, the case be dismissed against
Judgment 20 alleging therein that: him. 22

3. It is very clear from the findings of this Honorable Court Public respondent denied this motion in its Resolution of 17 August
contained in the amended decision promulgated on May 31, 1989 23 on these grounds: (a) the veil of corporate fiction should be pierced in
1989 that: this case; (b) since petitioner did not raise the issue of separate corporate
identity in the pleadings in the trial court or in his Brief, he cannot raise it for
3.1. Defendant Calvin S. Arcilla never had the first time in a Motion for Clarificatory Judgment; in his answer to
any personal business transaction (sic) in paragraphs 3 and 4 of the complaint, he admits that it was he and not his
the plaintiff; corporation who transacted business with the private respondent; and (c) the
"vales" refer not only to construction materials for which the loan to Csar
3.2. Csar Marine Resources, Inc. has an Marine Resources, Inc. was supposed to be used, but also to consumables
outstanding balance in the amount of such as salt, rice, food seasoning, cigarettes, coffee, etc.; this indicates that
P23,636.33 with plaintiff-appellee out of the the petitioner himself did not seriously treat the corporate affairs of Csar
KKK loan transaction; Marine Resources, Inc. as separate and distinct from his own.

452
Not satisfied with the Resolution, petitioner filed this petition. He alleges during pendency of the proceedings before the trial court. It is to be noted
therein that respondent Court of Appeals: that he failed to assign as error in his Brief the denial by the said court of his
motion for new trial on the basis thereof.
I
The grant of affirmative relief based on the first assigned error would really
. . . ERRED IN HOLDING CSAR MARINE RESOURCES, redound to the benefit of an entirety which was not made a party in the main
INC., A DOMESTIC CORPORATION DULY ORGANIZED case and which did not seek to intervene therein. Therefore, it has no
ACCORDING TO LAW, WHERE PETITIONER THE personality to seek as review of the public respondent's Amended Decision
PRESIDENT (sic), LIABLE TO THE PRIVATE under Rule 45 of the Rules of Court. Only the original parties to the main
RESPONDENT IN THE AMOUNT AWARDED IN THE case may do so. 26 Moreover, by no stretch of even the most fertile
APPEALED DECISION WITHOUT BEING IMPLEADED AS imagination may one be able to conclude that the challenged Amended
A PARTY IN THE CASE IN VIOLATION OF LAW AND THE Decision directed Csar Marine Resources, Inc. to pay the amounts adjudge.
APPLICABLE DECISIONS OF THE SUPREME COURT; and By its clear and unequivocal language, it is the petitioner who was declared
liable therefor and consequently made to pay. That the latter was ordered to
II do so as president of the corporation would not free him from the
responsibility of paying the due amount simply because according to him, he
had ceased to be corporate president; such conclusion stems from the fact
. . . IN NOT DISMISSING THE CASE AGAINST THE
that the public respondent, in resolving his motion for clarificatory judgment,
PETITIONER. 24
pierced the veil of corporate fictional and cast aside the contention that both
he and the corporation have separate and distinct personalities. In short,
After the filing of the Comment, the Reply thereto and the Rejoinder to the even if We are to assume arguendo that the obligation was incurred in the
latter, this Court gave due course to the petition and required the parties to name of the corporation, the petitioner would still be personally liable therefor
submit their respective Memoranda. 25 because for all legal intents and purposes, he and the corporation are one
and the same. Csar Marine Resources, Inc. is nothing more than his
The records bear nothing to prop up the instant petition. The arguments business conduit and alter ego. The fiction of a separate juridical personality
adduced by the petitioner breathe no life to it. conferred upon such corporation by law should be
disregarded. 27 Significantly, petitioner does not seriously challenge the public
On the contrary, the pleadings lead Us to the inescapable conclusion that the respondent's application of the doctrine which permits the piercing of the
petitioner, who is himself a lawyer, is merely taking advantage of the use of corporate veil and the disregarding of the fiction of a separate juridical
the innocuous phrase "in his capacity as President" found in the dispositive personality; this is because he knows only too well that from the very
portion of the challenged Amended Decision making the same a sanctuary beginning, he merely used the corporation for his personal purposes.
for a defense which he, as hereinafter discussed, had long since abandoned
or waived either deliberately or through his obliviscence. His sole purpose, of In his answer to the complaint, petitioner volunteered the information that
course, is to avoid complying with the liability adjudged against him by the the pro-forma invoice which he obtained from the private respondent and
public respondent; such avoidance is premiered on the so-called newly which became the source of the obligations reflected in the "vales" was to
discovered evidence offered after the public respondent had bent over support his loan. He states in part:
backwards to grant him a new trial despite the availability of such evidence

453
. . . when defendant was looking for a "pro-forma" invoice to set it up as an affirmative defense amounted to a waiver thereof. Section 2,
support his loan with the Kilusang Kabuhayan at Kaunlaran . Rule 9 of the Rules of Court expressly proved that defenses and objections,
. . His loan was in the name of his family corporation, CSAR other than the failure to state a cause of action and lack of jurisdiction, not
Marine Resources, Inc. . . . . 28 pleaded either in a motion to dismiss or in the answer are deemed waved.
Petitioner, as a lawyer, knows or is supposed to know this rule. Since he
That it was indeed his loan is further borne out by his allegations prepared the Answer himself, We cannot think of any possible reason why he
therein part: failed to set up this defense other than his realization of its inherent
weakness or his outright inexcusable negligence of forgetfulness. And even if
(a) The accounting between plaintiff and it were due to inadvertence, he could still have subsequently availed of
defendant, however, was not closed because adjustments Section 2, Rule 10 of the Rules of Court which allows a party to amend his
were needed in the following points: 29 answer as a matter of right within the period therein stated. Failing that, he
could have resorted to Section 3 thereof which allows the making of
amendments upon leave of court. On the other hand, if the lapse was due to
(b) 5. While it is true that plaintiff made demands for payment
forgetfulness, it is just unfortunate that he did not exercise due diligence in
of an alleged balance of P23,000.00 in March 1983, which
the conduct of his won affairs. He can expect no reward for it.
demand was even coursed thru the KKK Regional and
Provincial Offices, after the demand of
P23,000.00 defendant paid additional P5,000.00 cash to Then too, as correctly noted by the public respondent, petitioner, in his Brief,
plaintiff. 30 did not assign as error the holding of the trial court that he is solely liable for
the obligation.
In his motion to reconsider the public respondent's original decision,
petitioner becomes more candid in his admissions that indeed, the Petitioner's volunteered admission that he procured the pro-forma invoice
transaction with the private respondent and the loan obtained from the private respondent in connection with his loan from the KKK, using
previously were for his personal account. Thus he asserts that: his family corporation in the process, and his deliberate waiver of the
aforementioned defense provide an insurmountable obstacle to the viability
of this petition.
(a) the first document made between appellee and
appellant was the pro-forma invoice. 31
WHEREFORE, for utter lack of merit, the instant petition is DENIED with
costs against petitioner.
(b) [c]considering that appellant had already an approved
loan and was ready for release . . . . 32
This decision is immediately executory.
Moreover, petitioner neglected to set up in his Answer the defense that he is
not personally liable to private respondent because the "vales" were SO ORDERED.
corporate obligations of Csar Marine Resources, Inc.. Of course, that
defense would have been inconsistent with his volunteered admission that Gutierrez, Jr., Bidin, Romero and Melo, JJ., concur.
the KKK loan which resulted in the procurement of the pro-forma invoice
from the private respondent was for his benefit. In any case, the failure to

454
3. During 1969, the same Hernandez family organized another corporation,
Rosario Industrial Corporation (ROSARIO, for short) which also engaged, in
the RANSOM Compound, in the business of manufacture of ink and products
associated with ink.

4. The strike became the subject of Cases Nos. 2848 ULP and 2880
ULP of the Court of Industrial Relations which, on December 19, 1972,
ordered RANSOM "its officers and agents to reinstate the 22 strikers with
back wages from July 25, 1969.
G.R. No. L-69494 June 10, 1986

5. On April 2, 1973, RANSOM filed an application for clearance to close or


A.C. RANSOM LABOR UNION-CCLU, petitioner,
cease operations effective May 1, 1973, which was granted by the Ministry of
vs.
Labor and Employment in its Order of June 7, 1973, without prejudice to the
NATIONAL LABOR RELATIONS COMMISSION, First Division, A.C.
right of employees to seek redress of grievance, if any. Although it has
RANSOM (PHILS.) CORPORATION, RUBEN HERNANDEZ, MAXIMO C.
stopped operations, RANSOM has continued its personality as a corporation.
HERNANDEZ, JR., PORFIRIO R. VALENCIA, LAURA H. CORNEJO,
For practical purposes, reinstatement of the 22 strikers has been precluded.
FRANCISCO HERNANDEZ, CELESTINO C. HERNANDEZ & MA.
As a matter of fact, reinstatement is not an issue in this case.
ROSARIO HERNANDEZ, respondents.

6. Back wages of the 22 strikers were subsequently computed at


P164,984.00, probably in early 1974. The exact date is not reflected in the
record.
MELENCIO-HERRERA, J.:
7. Up to September 9, 1976, petitioner UNION had filed about ten (10)
The facts relevant to this case may be related as follows: motions for execution against RANSOM; but all of them could not be
implemented, presumably for failure to find leviable assets of RANSOM;
1. Respondent A. C. Ransom (Philippines) Corporation (RANSOM, for short) although it appears that, in 1975, RANSOM had sold machineries and
was established in 1933 by Maximo C. Hernandez, Sr. It was a "family" equipment for P28million to Revelations Manufacturing Corporation.
corporation, the stockholders of which were/are members of the Hernandez
family. It has a compound in Las Pinas Rizal, where it has been engaged in 8. Directly related to this case is the last Motion for Execution, dated
the manufacture mainly of ink and articles associated with ink. December 18, 1978, filed by petitioner UNION wherein it asked that officers
and agents of RANSOM be held personally liable for payment of the back
2. On June 6, 1961, employees of RANSOM, most of them being members wages. That Motion was granted by Labor Arbiter, Tito F. Genilo, on March
of petitioner Labor UNION, went on strike and established a picket line 11, 1980 (The GENILO ORDER), wherein he expressly authorized a Writ of
which, however, was lifted on June 21st with most of the strikers returning Execution to be issued for P164,984.00 (the back wages) against RANSOM
and being allowed to resume their work by RANSOM Twenty-two (22) and seven officers and directors of the Company who are the named
strikers were refused reinstatement by the Company. individual respondents herein. RANSOM took an appeal to NLRC which

455
affirmed the GENILO ORDER, except as modified in the body of its decision As to the liability of the respondent's officers and agents, we
of July 31, 1984. agree with the contention of the respondent-appellant that
there is nothing in the Order dated May 11, 1986 that would
9. In RANSOM's appeal to the NLRC, two issues were raised: justify the holding of the individual officers and agents of
respondent in their personal capacity. As a general rule,
(a) One of the issues was: officers of the corporation are not liable personally for the
official acts unless they have exceeded the scope of their
authority. In the absence of evidence showing that the
THE DECISION OF THE INDUSTRIAL RELATIONS COURT
officers mentioned in the Order of the Labor Arbiter dated
HAVING BECOME FINAL AND EXECUTORY IN 1973, IS IT
March 11, 1980 have exceeded their authority, the writ of
ENFORCEABLE BY A WRIT OF EXECUTION ISSUED IN
execution can not be enforced against them, especially so
1980 OR MORE THAN FIVE YEARS AFTER THE FINALITY
since they were not given a chance to be heard.
OF THE DECISION SOUGHT TO BE ENFORCED?

RANSOM and the seven individual respondents in this case have not
The corresponding ruling made by NLRC was:
appealed from the ruling of the NLRC that Section 6, Rule 39, is not
invocable by them in regards to the execution of the decision of December
Perforce respondent's theory that execution proceedings 19, 1972. Hence, the issue can no longer be raised herein. Even if the said
must stop after the lapse of five (5) years and that a motion section were applicable, the 5-year period therein mentioned may not have
to revive need be filed, must fail. Suffice it to state also that expired by December 18, 1978 because the period should be counted only
the statute of limitations has been devised to operate from the time the back wages were determined, which could have been in
primarily against those who sleep on their rights, not against early 1974.
those who assert their right but fail for causes beyond their
control. The above recital of facts contradicts respondent's
We now come to the NLRC's decision upholding non-personal liabilities of
contention that the CIR decision of August 19, 1972 had
the individual respondents herein for back wages of the 22 strikers.
remained dormant to require a motion to revive.

(a) Article 265 of the labor Code, in part. expressly provides:


(b) The second issue raised was:

Any worker whose employment has been terminated as a


IS THE JUDGMENT AGAINST A CORPORATION TO
consequence of an unlawful lockout shall be entitled
REINSTATE ITS DISMISSED EMPLOYEES WITH
to reinstatement with fill back wages.
BACKWAGES, ENFORCEABLE AGAINST ITS OFFICERS
AND AGENTS IN THEIR INDIVIDUAL, PRIVATE AND
PERSONAL CAPACITIES WHO WERE NOT PARTIES IN Article 273 of the Code provides that:
THE CASE WHERE THE JUDGMENT WAS RENDERED;
Any person violating any of the provisions of Article 265 of
The NLRC ruling was: this Code shall be punished by a fine of not exceeding five

456
hundred pesos and/or imprisonment for not less than one (1) (c) If the policy of the law were otherwise, the corporation employer can have
day nor more than six (6) months. devious ways for evading payment of back wages. in the instant case, it
would appear that RANSOM, in 1969, foreseeing the possibility or probability
(b) How can the foregoing provisions be implemented when the employer is of payment of back wages to the 22 strikers, organized ROSARIO to replace
a corporation? The answer is found in Article 212 (c) of the Labor Code RANSOM, with the latter to be eventually phased out if the 22 strikers win
which provides: their case. RANSOM actually ceased operation on May 1, 1973, after the
December 19, 1972 Decision of the Court of Industrial Relations was
(c) 'Employer includes any person acting in the interest of an promulgated against RANSOM.
employer directly or indirectly. The term shall not include any
labor organization or any of its officers or agents except (d) The record does not clearly Identify "the officer or officers" of RANSOM
when acting as employer. directly responsible for failure to pay the back wages of the 22 strikers. In the
absence of definite proof in that regard, we believe it should be presumed
The foregoing was culled from Section 2 of RA 602, the Minimum Wage Law. that the responsible officer is the President of the corporation who can be
Since RANSOM is an artificial person, it must have an officer who can be deemed the chief operation officer thereof. Thus, in RA 602, criminal
presumed to be the employer, being the "person acting in the interest of (the) responsibility is with the "Manager" or in his default, the person acting as
employer" RANSOM. The corporation, only in the technical sense, is the such. In RANSOM, the President appears to be the Manager.
employer.
(e) Considering that non-payment of the back wages of the 22 strikers has
The responsible officer of an employer corporation can be held personally, been a continuing situation, it is our opinion What the personal liability of the
not to say even criminally, liable for non-payment of back wages. That is the RANSOM President, at the time the back wages were ordered to be paid
policy of the law. In the Minimum Wage Law, Section 15(b) provided: should also be a continuing joint and several personal liabilities of all who x-
ray have thereafter succeeded to the office of president; otherwise, the 22
strikers may be deprived of their rights by the election of a president without
(b) If any violation of his Act is committed by a corporation,
leviable assets.
trust, partnership or association, the manager or in his
default, the person acting as such when the violation took
place, shall be responsible. In the case of a government WHEREFORE, the questioned Decision of the National Labor Relations
corporation, the managing head shall be made responsible, Commission is SET ASIDE, and the Order of Labor Arbiter Tito F. Genilo of
except when shown that the violation was due to an act or March 11, 1980 is reinstated with the modification that personal liability for
commission of some other person, over whom he has no the back wages due the 22 strikers shall be limited to Ruben Hernandez,
control, in which case the latter shall be held responsible. who was President of RANSOM in 1974, jointly and severally with other
Presidents of the same corporation who had been elected as such after 1972
or up to the time the corporate life was terminated.
In PD 525, where a corporation fails to pay the emergency allowance therein
provided, the prescribed penalty "shall be imposed upon the guilty officer or
officers" of the corporation. SO ORDERED.

Abad Santos (Chairman), Yap, Cruz and Paras, * , JJ., concur.

457
Narvasa, J., took no part.

458
459
1. In G.R. No. 79975, whether or not the private respondent
was an employee of the petitioner and, if so, had been
illegally dismissed; and corollarily, whether or not the NLRC
had jurisdiction over their dispute.

2. In G.R. No. 79907, whether or not the petitioner could be


G.R. No. 79907 March 16, 1989 held solidarity liable with Sweet Lines, Inc. to the private
respondent.
SAMUEL CASAS LIM, petitioner,
vs. The record shows that private respondent Victoria Calsado was hired by
THE NATIONAL LABOR RELATIONS COMMISSION and VICTORIA R. Sweet Lines, Inc. on March 5, 1981, as Senior Branch Officer of its
CALSADO, respondents. International Accounts Department for a fixed salary and a stipulated 5 %
commission on sales production. On December 1, 1983, after tendering her
resignation to accept another offer of employment, she was persuaded to
G.R. No. 79975. March 16, 1989
remain with an offer of her promotion to Manager of the Department with
corresponding increase in compensation, which she accepted. She was also
SWEET LINES, INC., petitioner, allowed to buy a second-hand Colt Lancer pursuant to a liberal car plan
vs. under which one-half of the cost was to be paid by the company and the
NATIONAL LABOR RELATIONS COMMISSION; HON. NESTOR C. LIM (In other half was to be deducted from her salary. Relations began to sour later,
his capacity as Labor Arbiter of the Ministry of Labor and Employment however, when she repeatedly asked for payment of her commissions, which
and VICTORIA R. CALSADO, respondents. had accumulated and were long overdue. She also complained of the
inordinate demands on her time even when she was sick and in the hospital.
Puruganan, Chato, Chato & Tan Law Office for petitioner. Finally, on July 16, 1985, she was served with a letter from Samuel Casas
Lim, the other petitioner, informing her that her "employment with Sweet
Leo C. Romero for petitioner Sweet Lines, Inc. Lines" would terminate on August 5, 1985. Efforts were also taken by Sweet
Lines to forcibly take the car from her, culminating in an action for replevin
Andrea R. dela Cueva for Victoria R. Calsado. against her in the regional trial court of Manila.

On August 14, 1985, Calsado filed a complaint against both petitioners for
illegal dismissal, illegal deduction, and unpaid wages and commissions plus
CRUZ, J.: moral and exemplary damages, among other claims. 1 There followed an
extended hearing where she testified on the details of her employment,
emphasizing her unsatisfactory treatment by the management of Sweet
These two cases have been consolidated because they relate to the same
Lines and especially the termination of her services without the required
factual antecedents and the same private respondent. The issues are:
notice and hearing and without valid cause. She also presented four other
witnesses to corroborate her charges.

460
The respondents' defenses were based mainly on the claim that Calsado (d) Sales commission in the sum of P432,656.68;
was not an employee of Sweet Lines but an independent contractor and that
therefore their dispute with her came under the jurisdiction of the civil courts (e) Moral damages of P100,000.00;
and not of the Labor Arbiter. 2 On this matter the private respondent
pointedly comments: (f) Exemplary damages of P10,000.00; and

At this point, private respondent would like to underscore the (g) Attorney's fees of P10,000.00 plus 25 % of the total
fact that while private respondent in the proceedings before monetary awards in favor of the complainant.
the Labor Arbiter presented five witnesses including herself,
all of whom were cross-examined by petitioners, and
The decision was appealed to the National Labor Relations Commission and
numerous documents which were marked as Exhs. "A" to
affirmed in toto except as to the attorney's fees, which were reduced to 10%
"GG-8d" and 858 receipts and bills, all of which were duly
of the total award. 5 Both Sweet Lines and Lim then came to us in separate
identified and testified to by private respondent and her
petitions to raise the above-stated issues. On October 14, 1987, we issued a
witnesses and examined by petitioners, petitioner failed to
temporary restraining order against the enforcement of the decision of the
present any single evidence, testimonial or documentary, to
public respondent dated September 11, 1987. 6 The petitions were
controvert private respondent's evidence. All that they
consolidated on December 7, 1987, and given due course on May 16, 1987,
presented were their unsubstantiated pleadings not one of
with the parties being required to submit their respective memoranda. On the
which was under oath, not even their position paper which,
first question, we hold that the employee-employer relations between
under the NLRC rules (Sec. 2, Rule 7, Revised Rules of the
Calsado and Sweet Lines have been sufficiently established. The following
NLRC), have to be verified. 3
documents submitted by the former and not controverted by the latter should
belie the claim that Calsado was only an independent contractor over whom
On December 29, 1986, decision was rendered against the two petitioners by Sweet Lines had no control.
the Labor Arbiter 4 who held them liable in solidum to the complainant for the
following amounts:
1. Certification issued by Sweet Lines, lnc. dated May
2l,1984, stating that private respondent 'is employed with this
(a) Separation pay equivalent to one month pay for every company since March 5, 1982 up to the present, presently
year of service based on her latest basic salary of P2,500.00 designated as International Accounts Manager of the Sweet
plus allowance of P500.00, or a total monthly pay of Lines, Inc., Manila Branch." (Exh. "W" )
P3,000.00;
2. Termination letter issued by Samuel Casas Lim to private
(b) Backwages based on her last monthly pay rate of respondent reading. 'Your employment with Sweet lines, Inc.
P3,000.00 to be computed from the time of her dismissal to will cease effective August 15, 1985. In connection with the
the actual payment of her separation pay; foregoing, you are entitled to (1) separation pay equivalent to
one half month of every year of service ... ; (2) The
(c) Proportionate 13th month pay for the year 1985; computed money value of unused vacation leave ... ; (3)
Thirteenth month pay ... ;" (Exh. "W")

461
3. Notice of private respondent's promotion effective Sweet Lines has also failed to substantiate its allegation that Calsado was an
December 1, 1982 from Senior Branch Officer to Manager, independent contractor, as it should have, with evidence showing inter
International Accounts, with an increase in basic salary from alia that she had the financial resources and other means or equipment to
P1,250 to P2,500 a month; (Exh. "D") operate as such. One must prove what one alleges, but Sweet Lines
confined itself to mere denials.
4. Computation of her salary, allowance and 13th month pay
differentials on account of her promotion, prepared and At any rate, the determination of the existence of employee-employer
approved by the proper officials of petitioner Sweet Lines, relations is a factual finding which this Court will not disturb or reverse in the
Inc. whose signatures appear thereon; (Exh. "E") absence of a showing of grave abuse of discretion. We do not see such
justification here. On the contrary, the ascertainment of the employment
5. Certification dated September 6, 19M issued by the status of the private respondent was made on the basis of the criteria
petitioner company, subscribed and sworn to before a notary consistently employed by the Court in the determination of the employee-
public declaring that private respondent was then an Account employer relationship. 7 We find from the record that all these test have been
Executive of Sweet Lines, Inc.; (Exh. "E") satisfied.

6. Certification, notarized on January 10, 1985, by Atty. Such relationship having been established, the third issue is automatically
Gregorio Francisco, counsel for petitioner company, that resolved and requires not much elaboration. Suffice it only to stress that the
private respondent "is a bona fide employee of Sweet Lines, damages claimed by private respondent as a result of her illegal dismissal
Inc. and presently holding the position of Manager, and the violation of the terms and conditions of her employment also come
International Account.' (Exh. "Y") within the jurisdiction of the Labor Arbiter as a contrary rule would result in
the splitting of actions and the consequent multiplication of suits. So we
7. Approved application for sick leave of private respondent recently affirmed in Limquiaco v. Ramolete 8 and more positively in National
for 15 days from March 7, 1985 to April 3, 1985. (Exh. "I") Union of Bank Employees v. Lazaro, 9 where we declared:

There is in the above exhibits a consistent and categorical recognition of As we stated, the damages (allegedly) suffered by the
Calsado as an employee of petitioner Sweet Lines. Indeed, its notarized petitioners only form part of the civil component of the injury
certification that Calsado was its bona fide employee is irrefutable. The arising from the unfair labor practice. Under Article 247 of the
petitioner cannot now argue that the grant to her of the 13th month pay and Code, "the civil aspects of all cases involving unfair labor
even the differential pay was a mere accomodation like the car plan (which, practices which may include claims for damages and other
for that matter, is a benefit usually extended only to employees). If it is true affirmative relief, shall be under the jurisdiction of the labor
that Sweet Lines had no control over her and left her free to determine her arbiters.
work schedule, there would have been no reason at all for its approval of her
application for sick leave from March 7, 1985 to April 3, 1985. The On the fourth issue, we agree with petitioner Lim that he cannot be held
termination letter itself, which was signed by the other petitioner as Vice personally liable with Sweet Lines for merely having signed the letter
President of Sweet Lines, said she was "entitled" to certain payments as a informing Calsado of her separation. There is no evidence that he acted with
result of the cessation of her "employment with Sweet Lines, Inc."

462
malice or bad faith. The letter, in fact, informed her not only of her separation The case of Ransom v. NLRC 12 is not in point because there the debtor
but also of the benefits due her as a result of the termination of her services. corporation actually ceased operations after the decision of the Court of
Industrial Relations was promulgated against it, making it necessary to
It is true that Lim has raised this matter rather tardily and also that he enforce it against its former president. Sweet lines is still existing and able to
belongs to a closed corporation controlled by the members of one family satisfy the judgment in favor of the private respondent.
only. But these circumstances should not be allowed to operate against him if
he is to be accorded substantial justice in the resolution of the private The Solicitor General, invoking equity rather than law, observes that making
respondent's claim. As we said in Ortigas vs. Lufthansa German Lim solidarity liable with Sweet Lines will ensure payment of Calsado's claim.
Airlines, 10 the Court is "clothed with ample authority to review matters, even But this precaution, even assuming it to be valid, is really unnecessary. in
if they are not assigned as errors in the appeal, if it finds that its fact, as a condition for the issuance of our temporary restraining order of
consideration is necessary in arriving at a just decision of the case." As for October 14, 1987, Sweet Lines posted as required a bond in the amount of
the second charge, the mere fact that Lim is part of the family corporation P850,000.00, which should cover the amounts awarded to the private
does not mean that all its acts are imputable to him directly and personally. respondent.13
His acts were official acts, done in his capacity as Vice President of Sweet
Lines and on its behalf. There is no showing that he acted without or in We especially uphold the award of moral and exemplary damages in view of
excess of his authority or was motivated by personal ill-will toward Calsado. the acts of harassment and bad faith testified to by the private respondent
The applicable decision is Sunio v. NLRC, 11 where it was held: and not refuted by Sweet Lines. Her treatment during her employment, the
delays in the payment of her commissions, the pressures exerted upon her
Petitioner Sunio was impleaded in the Complaint in his even when she was sick in the hospital, the suggestion of one of the
capacity as General Manager of petitioner corporation. There company officers that she discuss her complaints with him alone in a private
appears to be no evidence on record that he acted place, her arbitrary separation, the questionable attempts to get the vehicle
maliciously or in bad faith in terminating the services of from her after her dismissal, among other aggravations, clearly demonstrate
private respondents. His act, therefore, was within the scope the validity of the private respondent's complaints.
of his authority and was a corporate act.
Finally, we hold that the contention of Sweet Lines that separation pay and
It is basic that a corporation is invested by law with a back wages are inconsistent with each other is not well-taken. Separation
personality separate and distinct from those of the persons pay is granted where reinstatement is no longer advisable because of
composing it as well as from that of any other entity to which strained relations between the employee and the employer. Back wages
it may be related. Mere ownership by a single stockholder or represent compensation that should have been earned but were not
by another corporation of all or nearly all of the capital stock collected because of the unjust dismissal. The bases for computing the two
of a corporation is not of itself sufficient ground for are different, the first being usually the length of the employee's service and
disregarding the separate corporate personality. Petitioner the second the actual period when he was unlawfully prevented from
Sunio, therefore, should not have been made personally working.
answerable for the payment of private respondents' back
salaries. We have ordered the payment of both in proper case 14 as otherwise the
employee might be deprived of benefits justly due him. Thus, if an employee

463
who has worked only one year is sustained by the labor court after three
years from his unjust dismissal, granting him separation pay only would
entitle him to only one month salary. There is no reason why he should not
also be paid three years back wages corresponding to the period when he
could not return to his work or could not find employment elsewhere.

WHEREFORE, subject to the modification that the award of backwages shall


be limited to only three years, in accordance with existing policy, G.R. No.
79975 is DISMISSED, with costs against the petitioner, G.R. No. 79907 is
GRANTED and petitioner Samuel Casas Lim is hereby absolved of liability in
his personal capacity. The temporary restraining order dated October 14,
1987, is LIFTED. It is so ordered.

Narvasa, Gancayco, Grio-Aquino and Medialdea, JJ., concur.

464
465
Labor Relations Commission in NLRC-NCR Case No. 7-2739-86 and
ordered as follows:

WHEREFORE, the questioned decision is AFFIRMED but with


modification that the petitioner shall not be held jointly and severally
liable with AMAL for the private respondents' money claims against
the latter. However, for his bad faith in arrogating to himself AMAL's
properties to the prejudice of the private respondents, the petitioner
is ordered: 1) to pay the private respondents moral damages in the
sum of P20,000.00 and exemplary damages in the sum of
P20,000.00; and 2) to return the assets of AMAL that he has
appropriated, or the value thereof, with legal interest thereon from
the date of the appropriation until they are actually restored, these
amounts to be proportionately distributed among private respondents
in satisfaction of the judgment rendered in their favor against AMAL. 2

Petitioner was the general manager of the Manila Office of Affiliated


Machineries Agency, Ltd. (AMAL) and among the respondents in a complaint
for illegal dismissal and non-payment of statutory benefits filed by herein
G.R. No. 90856 February 1, 1996 respondents who were former employees of AMAL.

ARTURO DE GUZMAN, petitioner, Respondent employees initiated the complaint following AMAL's refusal to
vs. pay the former's monetary claims after AMAL decided to cease its operations
NATIONAL LABOR RELATIONS COMMISSION, (2ND DIVISION), LABOR in 1986. Petitioner was impleaded for allegedly selling part of AMAL's assets
ARBITER MA. LOURDES A. SALES, AVELINO D. VALLES-TEROL, and applying the proceeds of the same, as well as the remaining assets, to
ALEJANDRO Q. FRIAS, LINDA DELA CRUZ, CORAZON M. DELA satisfy his own claims against the company. He also formed a new company
FUENTE, LILIA F. FLORO, and MARIO F. JAYME, respondents. named Susarco, Inc. and engaged in the same line of business with the
former clients of AMAL.
RESOLUTION
On September 30, 1987, the Labor Arbiter rendered judgment and held
FRANCISCO, R., J.: petitioner jointly and severally liable with AMAL for respondent employees'
claims.3 Upon appeal to the National Labor Relations Commission, the
decision was affirmed in toto.4
This is a motion filed by petitioner Arturo de Guzman seeking the
reconsideration of the decision of this Court, promulgated on July 23,
1992,1 which modified the decisions of the Labor Arbiter and the National Not satisfied, petitioner proceeded to this Court on certiorari assailing the
aforementioned decision and claiming grave abuse of discretion.

466
As initially mentioned, this Court modified the decision of the NLRC and and not to the Labor Arbiter or to the NLRC. In such situations,
absolved petitioner from his solidary liability for respondent employees' resolution of the dispute requires expertise, not in labor management
claims. This was based on a finding that as mere managerial employee, relation nor in wage structures and other terms and conditions of
petitioner had no participation in the decision to cease operations and employment, but rather in the application of the general civil law.
terminate the services of respondent employees which was the exclusive Clearly, such claims fall outside of the area of competence or
responsibility of AMAL alone. Nevertheless, for having acted in bad faith by expertise ordinarily ascribed to Labor Arbiters and the NLRC and the
appropriating the assets of AMAL to satisfy his own claims to the prejudice of rationale for granting jurisdiction over such claims to these agencies
respondent employees' pending claims, petitioner was held directly liable for disappears.5
moral and exemplary damages based on the provisions of Articles 19, 21,
2219(10) and 2229 of the Civil Code. While it is conceded that no employer-employee ties existed between the
petitioner and respondent employees, this does not preclude this Court from
In this motion, petitioner assails the award of damages and the order to adjudging him liable for damages. In labor disputes like the instant suit, it is
return the assets of AMAL which he appropriated for being unwarranted, not required that the claim for relief should directly result from an employer-
arguing that the same were beyond the jurisdiction of this Court to grant in a employee relationship. It suffices that there be a showing of a reasonable
complaint for illegal dismissal in the absence of an employer-employee causal connection between the claim asserted and the employer-employee
relationship between petitioner and respondent employees. relations.6

The argument is premised on the following pronouncements on previous Respondent employees could have been afforded relief in their suit for illegal
decisions that: dismissal and non-payment of statutory benefits were it not for petitioner's
unscrupulous acts of appropriating for himself the assets of AMAL which
. . . The Court, therefore, believes and so holds that the "money rendered the satisfaction of respondent employees' claims impossible. By
claims of workers" referred to in paragraph 3 of Article 217 embraces taking undue advantage of his position as general manager of AMAL,
money claims which arise out of or in connection with the employer- petitioner was able to facilitate the consummation of his acts as he had
employee relationship, or some aspect or incident of such access over the company's assets.
relationship. Put a little differently, that money claims of workers
which now fall within the original and exclusive jurisdiction of Labor On this score, it is evident that petitioner's acts of bad faith were offshoots of
Arbiters are those money claims which have some reasonable the termination of their employment relations with AMAL. The company's
causal connection with the employer-employee relationship. decision to close down its business impelled petitioner to act precipitately in
appropriating the assets of AMAL, fearing perhaps that the same might not
xxx xxx xxx be enough to satisfy all the legitimate claims against it.

. . . The important principle that runs through these three (3) cases is Petitioner's contention that his application of AMAL's assets to satisfy his own
that where the claim to the principal relief sought is to be resolved claims against the company is nothing more than a simple legal
not by reference to the Labor Code or other labor relations statute or compensation or set-off deserves scant consideration as it was done without
a collective bargaining agreement but by the general civil law, the deference to the legitimate claims of respondent employees and other
jurisdiction over the dispute belongs to the regular courts of justice creditors of AMAL, in contravention of the provisions on concurrence and

467
preference of credits under the Civil Code. Although his legitimate claims are
not disputed, the same, however, are properly cognizable at the proceedings
for AMAL's dissolution.

Thus, we affirm our previous conclusion that although the question of


damages arising from petitioner's bad faith has not directly sprung from the
illegal dismissal, it is clearly intertwined therewith. Accordingly, petitioner's
bad faith having been sufficiently established, the award of damages against
him and the order for him to return the assets of AMAL which he
appropriated, or their value, are in order.

Finally, we underscore the fact that this case has already dragged on for the
past nine years, making it extremely urgent that it be resolved with finality
and for this Court not to sanction any further delay or attempts to frustrate the
disposition of the legitimate claims of respondent employees.

WHEREFORE, the motion for reconsideration is hereby DENIED for lack of


merit. The denial is final.

SO ORDERED.

Padilla, Davide, Jr., Bellosillo and Hermosisima, Jr., JJ., concur.

468
CALTEX PHILIPPINES INC., PILIPINAS SHELL PETROLEUM CORP. and
PETRON CORP., respondents.

YNARES-SANTIAGO, J.:

On November 5, 1997, this Court in Tatad v. Secretary of the Department of


Energy and Lagman, et al., v. Hon.Ruben Torres, et al., 1 declared Republic
Act No. 8180, entitled "An Act Deregulating the Downstream Oil Industry and
For Other Purposes", unconstitutional, and its implementing Executive Order
No. 392 void.

R.A. 8180 was struck down as invalid because three key provisions intended
to promote free competition were shown to achieve the opposite result. More
specifically, this Court ruled that its provisions on tariff differential, stocking of
inventories, and predatory pricing inhibit fair competition, encourage
monopolistic power, and interfere with the free interaction of the market
forces.

While R.A. 8180 contained a separability clause, it was declared


unconstitutional in its entirety since the three (3) offending provisions so
permeated the law that they were so intimately the esse of the law. Thus, the
whole statute had to be invalidated.

As a result of the Tatad decision, Congress enacted Republic Act No. 8479, a
new deregulation law without the offending provisions of the earlier law.
Petitioner Enrique T. Garcia, a member of Congress, has now brought this
petition seeking to declare Section 19 thereof, which sets the time of full
G.R. No. 132451 December 17, 1999 deregulation, unconstitutional. After failing in his attempts to have Congress
incorporate in the law the economic theory he espouses, petitioner now asks
us, in the name of upholding the Constitution, to undo a violation which he
CONGRESSMAN ENRIQUE T. GARCIA, petitioner,
claims Congress has committed.
vs.
HON. RENATO C. CORONA, in his capacity as the Executive Secretary,
HON. FRANCISCO VIRAY, in his capacity as the Secretary of Energy, The assailed Section 19 of R.A. 8479 states in full:

469
Sec. 19. Start of Full Deregulation. Full deregulation of Provided, however, That in case full deregulation is started
the Industry shall start five (5) months following the effectivity by the President in the exercise of the authority provided in
of this Act: Provided, however, That when the public interest this Section, the foregoing laws shall continue to be in force
so requires, the President may accelerate the start of full and effect with respect to LPG, regular gasoline and
deregulation upon the recommendation of the DOE and the kerosene for the rest of the five (5)-month period.
Department of Finance (DOF) when the prices of crude oil
and petroleum products in the world market are declining Petitioner contends that Section 19 of R.A. 8479, which prescribes the period
and the value of the peso in relation to the US dollar is for the removal of price control on gasoline and other finished products and
stable, taking into account relevant trends and for the full deregulation of the local downstream oil industry, is patently
prospects; Provided, further, That the foregoing provision contrary to public interest and therefore unconstitutional because within the
notwithstanding, the five (5)-month Transition Phase shall short span of five months, the market is still dominated and controlled by an
continue to apply to LPG, regular gasoline and kerosene as oligopoly of the three (3) private respondents, namely, Shell, Caltex and
socially-sensitive petroleum products and said petroleum Petron.
products shall be covered by the automatic pricing
mechanism during the said period. The objective of the petition is deceptively simple. It states that if the
constitutional mandate against monopolies and combinations in restraint of
Upon the implementation of full deregulation as provided trade 2 is to be obeyed, there should be indefinite and open-ended price
herein, the Transition Phase is deemed terminated and the controls on gasoline and other oil products for as long as necessary. This will
following laws are repealed: allegedly prevent the "Big 3" Shell, Caltex and Petron from price-fixing
and overpricing. Petitioner calls the indefinite retention of price controls as
a) Republic Act No. 6173, as amended; "partial deregulation".

b) Section 5 of Executive Order No. 172, as amended; The grounds relied upon in the petition are:

c) Letter of Instruction No. 1431, dated October 15, 1984; A.

d) Letter of Instruction No. 1441, dated November 20, 1984, Sec. 19 OF R.A. NO. 8479 WHICH PROVIDES FOR FULL
as amended; DEREGULATION FIVE (5) MONTHS OR EARLIER
FOLLOWING THE EFFECTIVITY OF THE LAW, IS
e) Letter of Instruction No. 1460, dated May 9, 1985; GLARINGLY PRO-OLIGOPOLY, ANTI-COMPETITION AND
ANTI-PEOPLE, AND IS THEREFORE PATENTLY
f) Presidential Decree No. 1889; and UNCONSTITUTIONAL FOR BEING IN GROSS AND
CYNICAL CONTRAVENTION OF THE CONSTITUTIONAL
POLICY AND COMMAND EMBODIED IN ARTCLE XII,
g) Presidential Decree No. 1956, as amended by Executive
SECTION 19 OF THE 1987 CONSTITUTION AGAINST
Order No. 137:

470
MONOPOLIES AND COMBINATIONS IN RESTRAINT OF The issues involved in the deregulation of the downstream oil industry are of
TRADE. paramount significance. The ramifications, international and local in scope,
are complex. The impact on the nation's economy is pervasive and far-
B. reaching. The amounts involved in the oil business are immense.
Fluctuations in the supply and price of oil products have a dramatic effect on
SAID SECTION 19 OF R.A. No. 8479 IS GLARINGLY PRO- economic development and public welfare. As pointed out in the Tatad
OLIGOPOLY, ANTI-COMPETITION AND ANTI-PEOPLE, decision, few cases carry a surpassing importance on the daily life of every
FOR THE FURTHER REASON THAT IT PALPABLY AND Filipino. The issues affect everybody from the poorest wage-earners and
CYNICALLY VIOLATES THE VERY OBJECTIVE AND their families to the richest entrepreneurs, from industrial giants to humble
PURPOSE OF R.A. NO. 8479, WHICH IS TO ENSURE A consumers.
TRULY COMPETITIVE MARKET UNDER A REGIME OF
FAIR PRICES. Our decision in this case is complicated by the unstable oil prices in the world
market. Even as this case is pending, the price of OPEC oil is escalating to
C. record levels. We have to emphasize that our decision has nothing to do with
worldwide fluctuations in oil prices and the counter-measures of Government
each time a new development takes place.
SAID SECTION 19 OF R.A. No. 8479, BEING GLARINGLY
PRO-OLIGOPOLY, ANTI-COMPETITION AND ANTI-
PEOPLE, BEING PATENTLY UNCONSTITUTIONAL AND The most important part of deregulation is freedom from price control.
BEING PALPABLY VIOLATIVE OF THE LAW'S POLICY Indeed, the free play of market forces through deregulation and when to
AND PURPOSE OF ENSURING A TRULY COMPETITIVE implement it represent one option to solve the problems of the oil-consuming
MARKET UNDER A REGIME OF FAIR PRICES, IS A VERY public. There are other considerations which may be taken into account such
GRAVE AND GRIEVOUS ABUSE OF DISCRETION ON as the reduction of taxes on oil products, the reinstitution of an Oil Price
THE PART OF THE LEGISLATIVE AND EXECUTIVE Stabilization Fund, the choice between government subsidies taken from the
BRANCHES OF GOVERNMENT. regular taxpaying public on one hand and the increased costs being
shouldered only by users of oil products on the other, and most important,
the immediate repeal of the oil deregulation law as wrong policy. Petitioner
D.
wants the setting of prices to be done by Government instead of being
determined by free market forces. His preference is continued price control
PREMATURE FULL DEREGULATION UNDER SECTION 19 with no fixed end in sight. A simple glance at the factors surrounding the
OF R.A. NO. 8479 MAY AND SHOULD THEREFORE BE present problems besetting the oil industry shows that they are economic in
DECLARED NULL AND VOID EVEN AS THE REST OF ITS nature.
PROVISIONS REMAIN IN FORCE, SUCH AS THE
TRANSITION PHASE OR PARTIAL DEREGULATION WITH
R.A. 8479, the present deregulation law, was enacted to implement Article
PRICE CONTROLS THAT ENSURES THE PROTECTION
XII, Section 19 of the Constitution which provides:
OF THE PUBLIC INTEREST BY PREVENTING THE BIG 3
OLIGOPOLY'S PRICE-FIXING AND OVERPRICING. 3

471
The State shall regulate or prohibit monopolies when the indefinite price control in the manner proposed by petitioner the only feasible
public interest so requires. No combinations in restraint of and legal way to achieve it?
trade or unfair competition shall be allowed.
Petitioner has taken upon himself a most challenging task. Unquestionably,
This is so because the Government believes that deregulation will eventually the direction towards which the nation's efforts at economic and social
prevent monopoly. The simplest form of monopoly exists when there is only upliftment should be addressed is a function of Congress and the President.
one seller or producer of a product or service for which there are no In the exercise of this function, Congress and the President have obviously
substitutes. In its more complex form, monopoly is defined as the joint determined that speedy deregulation is the answer to the acknowledged
acquisition or maintenance by members of a conspiracy, formed for that dominion by oligopolistic forces of the oil industry. Thus, immediately after
purpose, of the power to control and dominate trade and commerce in a R.A. 8180 was declared unconstitutional in the Tatad case, Congress took
commodity to such an extent that they are able, as a group, to exclude actual resolute steps to fashion new legislation towards the objective of the earlier
or potential competitors from the field, accompanied with the intention and law. Invoking the Constitution, petitioner now wants to slow down the
purpose to exercise such power. 4 process.

Where two or three or a few companies act in concert to control market While the Court respects the firm resolve displayed by Congress and the
prices and resultant profits, the monopoly is called an oligopoly or cartel. It is President, all departments of Government are equally bound by the
a combination in restraint of trade. sovereign will expressed in the commands of the Constitution. There is a
need for utmost care if this Court is to faithfully discharge its duties as arbitral
The perennial shortage of oil supply in the Philippines is exacerbated by the guardian of the Constitution. We cannot encroach on the policy functions of
further fact that the importation, refining, and marketing of this precious the two other great departments of Government. But neither can we ignore
commodity are in the hands of a cartel, local but made up of foreign-owned any overstepping of constitutional limitations. Locating the correct balance
corporations. Before the start of deregulation, the three private respondents between legality and policy, constitutional boundaries and freedom of action,
controlled the entire oil industry in the Philippines. and validity and expedition is this Court's dilemma as it resolves the
legitimacy of a Government program aimed at giving every Filipino a more
It bears reiterating at the outset that the deregulation of the oil industry is a secure, fulfilling and abundant life.
policy determination of the highest order. It is unquestionably a priority
program of Government. The Department of Energy Act of 1992 5 expressly Our ruling in Tatad is categorical that the Constitution's Article XII, Section
mandates that the development and updating of the existing Philippine 19, is anti-trust in history and spirit. It espouses competition. We have stated
energy program "shall include a policy direction towards deregulation of the that only competition which is fair can release the creative forces of the
power and energy industry." market. We ruled that the principle which underlies the constitutional
provision is competition. Thus:
Be that as it may, we are not concerned with whether or not there should be
deregulation. This is outside our jurisdiction. The judgment on the issue is a Sec. 19, Article XII of our Constitution is anti-trust in history
settled matter and only Congress can reverse it. Rather, the question that we and in spirit. It espouses competition. The desirability of
should address here is are the method and the manner chosen by competition is the reason for the prohibition against restraint
Government to accomplish its cherished goal offensive to the Constitution? Is of trade, the reason for the interdiction of unfair competition,

472
and the reason for regulation of unmitigated monopolies. measures to address these evils are for Congress to determine, but they
Competition is thus the underlying principle of section 19, have to meet the test of constitutional validity.
Article XII of our Constitution which cannot be violated by
R.A. No. 8180. We subscribe to the observation of Prof. The Court respects the legislative finding that deregulation is the policy
Gellhorn that the objective of anti-trust law is "to assure a answer to the problems. It bears stressing that R.A. 8180 was declared
competitive economy, based upon the belief that through invalid not because deregulation is unconstitutional. The law was struck
competition producers will strive to satisfy consumer wants down because, as crafted, three key provisions plainly encouraged the
at the lowest price with the sacrifice of the fewest resources. continued existence if not the proliferation of the constitutionally proscribed
Competition among producers allows consumers to bid for evils of monopoly and restraint of trade.
goods and services, and thus matches their desires with
society's opportunity costs." He adds with appropriateness In sharp contrast, the present petition lacks a factual foundation specifically
that there is a reliance upon "the operation of the "market" highlighting the need to declare the challenged provision unconstitutional.
system (free enterprise) to decide what shall be produced, There is a dearth of relevant, reliable, and substantial evidence to support
how resources shall be allocated in the production process, petitioner's theory that price control must continue even as Government is
and to whom the various products will be distributed. The trying its best to get out of regulating the oil industry. The facts of the petition
market system relies on the consumer to decide what and are, in the main, a general dissertation on the evils of monopoly.
how much shall be produced, and on competition, among
producers to determine who will manufacture it." 6
Petitioner overlooks the fact that Congress enacted the deregulation law
exactly because of the monopoly evils he mentions in his petition. Congress
In his recital of the antecedent circumstances, petitioner repeats in instituted the lifting of price controls in the belief that free and fair competition
abbreviated form the factual findings and conclusions which led the Court to was the best remedy against monopoly power. In other words, petitioner's
declare R.A. 8180 unconstitutional. The foreign oligopoly or cartel formed by facts are also the reasons why Congress lifted price controls and why the
respondents Shell, Caltex and Petron, their indulging in price-fixing and President accelerated the process. The facts adduced in favor of continued
overpricing, their blockade tactics which effectively obstructed the entry of and indefinite price control are the same facts which supported what
genuine competitors, the dangers posed by the oil cartel to national security Congress believes is an exercise of wisdom and discretion when it chose the
and economic development, and other prevailing sentiments are stated as path of speedy deregulation and rejected Congressman Garcia's economic
axiomatic truths. They are repeated in capsulized context as the current theory.
background facts of the present petition.
The petition states that it is using the very thoughts and words of the Court in
The empirical existence of this deplorable situation was precisely the reason its Tatad decision. Those thoughts and words, however, were directed
why Congress enacted the oil deregulation law. The evils arising from against the tariff differential, the inventory requirement, and predatory pricing,
conspiratorial acts of monopoly are recognized as clear and present. But the not against deregulation as a policy and not against the lifting of price
enumeration of the evils by our Tatad decision was not for the purpose of controls.
justifying continued government control, especially price control. The
objective was, rather, the opposite. The evils were emphasized to show the
A dramatic, at times expansive and grandiloquent, reiteration of the same
need for free competition in a deregulated industry. And to be sure, the
background circumstances narrated in Tatad does not squarely sustain

473
petitioner's novel thesis that there can be deregulation without lifting price develop free from government interference. R.A. 8479 actually allows the
controls. free play of supply and demand to dictate prices. Petitioner wants a
government official or board to continue performing this task. Indefinite and
Petitioner may call the industry subject to price controls as deregulated. In open-ended price control as advocated by petitioner would be to continue a
enacting the challenged provision, Congress, on the other hand, has regime of legislated regulation where free competition cannot possibly
declared that any industry whose prices and profits are fixed by government flourish. Control is the antithesis of competition. To grant the petition would
authority remains a highly regulated one. mean that the Government is not keen on allowing a free market to develop.
Petitioner's "self-evident truth" thus supports the validity of the provision of
Petitioner, therefore, engages in a legal paradox. He fails to show how there law he opposes.
can be deregulation while retaining government price control. Deregulation
means the lifting of control, governance and direction through rule or New players in the oil industry intervened in this case. According to them, it is
regulation. It means that the regulated industry is freed from the controls, the free market policy and atmosphere of deregulation which attracted and
guidance, and restrictions to which it used to be subjected. The use of the brought the new participants, themselves included, into the market. The
word "partial" to qualify deregulation is sugar-coating. Petitioner is really intervenors express their fear that this Court would overrule legislative policy
against deregulation at this time. and replace it with petitioner's own legislative program.

Petitioner states that price control is good. He claims that it was the The factual allegations of the intervenors have not been refuted and we see
regulation of the importation of finished oil products which led to the exit of no reason to doubt them. Their argument that the co-existence of many
competitors and the consolidation and dominion of the market by an viable rivals create free market conditions induces competition in product
oligopoly, not price control. Congress and the President think otherwise. quality and performance and makes available to consumers an expanded
range of choices cannot be seriously disputed.
The argument that price control is not the villain in the intrusion and growth of
monopoly appears to be pure theory not validated by experience. There can On the other hand, the pleadings of public and private respondents both put
be no denying the fact that the evils mentioned in the petition arose while forth the argument that the challenged provision is a policy decision of
there was price control. The dominance of the so-called "Big 3" became Congress and that the wisdom of the provision is outside the authority of this
entrenched during the regime of price control. More importantly, the Court to consider. We agree. As we have ruled in Morfe v. Mutuc 7:
ascertainment of the cause and the method of dismantling the oligopoly thus
created are a matter of legislative and executive choice. The judicial process (I)t is well to remember that this Court, in the language of
is equipped to handle legality but not wisdom of choice and the efficacy of Justice Laurel, "does not pass upon question or wisdom,
solutions. justice or expediency of legislation." As expressed by Justice
Tuason: "It is not the province of the courts to supervise
Petitioner engages in another contradiction when he puts forward what he legislation and keep it within the bounds of propriety and
calls a self-evident truth. He states that a truly competitive market and fair common sense. That is primarily and exclusively a legislative
prices cannot be legislated into existence. However, the truly competitive concern." There can be no possible objection then to the
market is not being created or fashioned by the challenged legislation. The observation of Justice Montemayor: "As long as laws do not
market is simply freed from legislative controls and allowed to grow and violate any Constitutional provision, the Courts merely

474
interpret and apply them regardless of whether or not they monthly price ceiling of each and every petroleum fuel
are wise or salutary." For they, according to Justice Labrador, product, using the automatic pricing formula. While the
"are not supposed to override legitimate policy and . . . never OPSF would return, this coverage would be limited to
inquire into the wisdom of the law." monthly price increases in excess of P0.50 per liter.

It is thus settled, to paraphrase Chief Justice Concepcion We are not impressed by petitioner Garcia's submission.
in Gonzales v. Commission on Elections, that only Petitioner has no basis in condemning as
congressional power or competence, not the wisdom of the unconstitutional per se the date fixed by Congress for the
action taken, may be the basis for declaring a statute invalid. beginning of the full deregulation of the downstream oil
This is as it ought to be: The principle of separation of industry. Our Decision merely faulted the Executive for
powers has in the main wisely allocated the respective factoring the depletion of OPSF in advancing the date of full
authority of each department and confined its jurisdiction to deregulation to February 1997. Nonetheless, the error of the
such a sphere. There would then be intrusion not allowable Executive is now a non-issue for the full deregulation set by
under the Constitution if on a matter left to the discretion of a Congress itself at the end of March 1997 has already come
coordinate branch, the judiciary would substitute its own. If to pass. March 1997 is not an arbitrary date. By that date,
there be adherence to the rule of law, as there ought to be, the transition period has ended and it was expected that the
the last offender should be the courts of justice, to which people would have adjusted to the role of market forces in
rightly litigants submit their controversy precisely to maintain shaping the prices of petroleum and its products. The choice
unimpaired the supremacy of legal norms and prescriptions. of March 1997 as the date of full deregulation is a judgment
The attack on the validity of the challenged provision of Congress and its judgment call cannot be impugned by
likewise insofar as there may be objections, even if valid and this Court. 8
cogent, on its wisdom cannot be sustained.
Reduced to its basic arguments, it can be seen that the challenge in this
In this petition, Congressman Garcia seeks to revive the long settled issue of petition is not against the legality of deregulation. Petitioner does not
the timeliness of full deregulation, which issue he had earlier submitted to expressly challenge deregulation. The issue, quite simply, is the timeliness or
this Court by way of a Partial Motion for Reconsideration in the Tatadcase. In the wisdom of the date when full deregulation should be effective.
our Resolution dated December 3, 1997, which has long become final and
executory, we stated: In this regard, what constitutes reasonable time is not for judicial
determination. Reasonable time involves the appraisal of a great variety of
We shall first resolve petitioner Garcia's linchpin contention relevant conditions, political, social and economic. They are not within the
that the full deregulation decreed by R.A. No. 8180 to start at appropriate range of evidence in a court of justice. It would be an extravagant
the end of March 1997 is unconstitutional. For prescinding extension of judicial authority to assert judicial notice as the basis for the
from this premise, petitioner suggests that "we simply go determination. 9
back to the transition period, price control will be revived
through the automatic pricing mechanism based on We repeat that what petitioner decries as unsuccessful is not a final result. It
Singapore Posted Prices. The Energy Regulatory Board . . . is only a beginning. The Court is not inclined to stifle deregulation as enacted
would play a limited and ministerial role of computing the

475
by Congress from its very start. We leave alone the program of deregulation determined, consistent and faithful execution of the law is the function of the
at this stage. Reasonable time will prove the wisdom or folly of the President. As stated by public respondents, the remedy against
deregulation program for which Congress and not the Court is accountable. unreasonable price increases is not the nullification of Section 19 of R.A.
8479 but the setting into motion of its various other provisions.
Petitioner argues further that the public interest requires price controls while
the oligopoly exists, for that is the only way the public can be protected from For this Court to declare unconstitutional the key provision around which the
monopoly or oligopoly pricing. But is indefinite price control the only feasible law's anti-trust measures are clustered would mean a constitutionally
and legal way to enforce the constitutional mandate against oligopolies? interdicted distrust of the wisdom of Congress and of the determined
exercise of executive power.
Art. 186 of the Revised Penal Code, as amended, punishes as a felony the
creation of monopolies and combinations in restraint of trade. The Solicitor Having decided that deregulation is the policy to follow, Congress and the
General, on the other hand, cites provisions of R.A. 8479 intended to prevent President have the duty to set up the proper and effective machinery to
competition from being corrupted or manipulated. Section 11, entitled "Anti- ensure that it works. This is something which cannot be adjudicated into
Trust Safeguards", defines and prohibits cartelization and predatory pricing. It existence. This Court is only an umpire of last resort whenever the
penalizes the persons and officers involved with imprisonment of three (3) to Constitution or a law appears to have been violated. There is no showing of a
seven (7) years and fines ranging from One million to Two million pesos. For constitutional violation in this case.
this purpose, a Joint Task Force from the Department of Energy and
Department of Justice is created under Section 14 to investigate and order WHEREFORE, the petition is DISMISSED.
the prosecution of violations.
SO ORDERED.
Sec. 8 and 9 of the Act, meanwhile, direct the Departments of Foreign Affairs,
Trade and Industry, and Energy to undertake strategies, incentives and Bellosillo, Melo, Puno, Kapunan, Mendoza, Quisumbing, Purisima, Pardo,
benefits, including international information campaigns, tax holidays and Buena and De Leon, Jr., JJ., concur.
various other agreements and utilizations, to invite and encourage the entry
of new participants. Section 6 provides for uniform tariffs at three percent
Davide, Jr., C.J., in the result. I also join Mr. Justice Panganiban in his
(3%).
separate opinion.

Sec. 13 of the Act provides for "Remedies", under which the filing of actions
Vitug, J., in the result.
by government prosecutors and the investigation of private complaints by the
Task Force is provided. Sections 14 and 15 provide how the Department of
Energy shall monitor and prevent the occurrence of collusive pricing in the Panganiban, J., please see Separate Opinion.
industry.
Gonzaga-Reyes, J., took no part. Spouse with counsel for intervenors.
It can be seen, therefore, that instead of the price controls advocated by the
petitioner, Congress has enacted anti-trust measures which it believes will Separate Opinions
promote free and fair competition. Upon the other hand, the disciplined,

476
PANGANIBAN, J., separate opinion; than promotes free trade, in contravention of the Constitution. True, the
President and Congress, not this Court, have the power and the prerogative
In essence, deregulation shifts the burden of price control from the to determine whether to adopt such market policies and, if so, under what
government to the "market forces" in order (1) to eliminate government conditions and circumstances. However, all such policies and their
intervention that may "do more harm than good" 1 and (2) to achieve a truly ramifications must conform to the Constitution. Otherwise, this Court has the
competitive market of fair prices. 2 It is also aimed at removing government duty to strike them down, not because they are unwise or inconvenient, but
abuse and corruption in price-setting. At bottom, deregulation is supposed to because they are constitutionally impermissible.
provide the best goods and services at the cheapest prices.
Doctrinally, policies and acts of the political departments of government may
The policy, however, is not an infallible cure to abuse, for the evil sought to be voided by this Court on either of two grounds infringement of the
be avoided may well pass on to the market players, particularly when they Constitution or grave abuse of discretion. 4 An infringement may be proven by
combine to restrain trade or engage in unfair competition. In the words of demonstrating that the words of the law directly contradict a provision of the
Prof. Romulo L. Neri of the Asian Institute of Management, "[t]he marker is fundamental law, or by presenting proof that the law authorizes or enables
motivated by price and profits (and sadly, not by moral values [or public the respondents to violate the Constitution.
interest]). The market does not automatically supply those who need (no
matter how badly they need it) but only those who have the money to buy." 3 Petitioner Garcia's Thesis on

The buzz words of the third millennium are "deregulation," "globalization" and Unconstitutionality Concerns Policy
"liberalization." Territorial frontiers are virtually erased by these schemes, as
goods and services are exchanged across borders unhampered by Having set down the doctrinal legal parameters, let me now discuss the
traditional tariffs, taxes, currency controls, quantitative restrictions and other petitioner's thesis. Petitioner Enrique T. Garcia anchors his position on the
protective barriers. Thus, states and governments tend to surrender some of alleged unconstitutionality of Section 19 of RA 8479, 5 which sets the full
their authorities and powers to the "market" and to the renewed energy of deregulation of the oil industry five months from the effectivity of the law, on
laissez faire, such that the threats to civil liberties and human rights, including the argument that said provision directly violates Section 19, Article XII of the
economic rights, may shift from government abuses to the more bedeviling Constitution, which reads as follows:
market forces that transcend boundaries and sovereignties. In developing
countries more than in developed ones, such threats are real and ever Sec. 19. The State shall regulate or prohibit monopolies
present. when the public interest so requires. No combinations in
restraint of trade or unfair competition shall be allowed.
Judicial Review
He maintains that once Section 19 of RA 8479 is struck down, the
to Checks Abuses government will be able to fix and lower petroleum prices indefinitely while
awaiting the advent of "real" competition in the market.
This is where the power of judicial review comes in to examine the legal
effects of these new economic paradigms and, in the present controversy, to Petitioner contends that the three largest oil companies (the "Big Three")
check whether the present Oil Deregulation Law (RA 8479) restrains rather comprise an oligopoly of the downstream oil industry. Oligopolies, he claims,

477
"negate free market competition and fair prices." He submits that "regulation In my Concurring Opinion in Tatad, I labeled RA 8180 as "a pseudo
through price control . . . is patently required by the public interest [and] the deregulation law which in reality restrains free trade and perpetuates a cartel,
failure to regulate the oligopoly through price control is patently inimical to the an oligopoly" because of the aforecited three provisions, and because
national interest and patently negates, circumvents and contravenes Section petitioners therein demonstrated to the Court "that the Big Three oil
19, Article XII of the Constitution." companies were producing and processing almost identical products which
they were selling to the general public at identical prices. When one company
In Tatad v. Secretary of the Department of Energy, 6 this Court defined a adjusted its prices upwards or downwards, the other two followed suit at the
monopoly and a combination in restraint of trade as follows: same time and by the same amount." 7

A monopoly is a privilege or peculiar advantage vested in In his present Petition, petitioner persistently alleges that "[i]t is self-evident
one or more persons or companies, consisting in the truth that public interest requires the prevention of monopolistic/oligopolitic
exclusive right or power to carry on a particular business or pricing . . . ," and that such "monopolistic/oligopolistic pricing may be
trade, manufacture a particular article, or control the sale or prevented only through price control during the regime of monopoly/oligopoly
the whole supply of a particular commodity. It is a form of or through a truly competitive market under a regime of fair prices." In
market structure in which one or only a few firms dominate support of his allegations, he cites "self-evident truths [which] have
the total sales of a product or service. On the other hand, a . . . been officially recognized and implemented during more than 20 years of
combination in restraint of trade is an agreement or price control before the passage of the two oil deregulation laws" and which
understanding between two or more persons, in the form of "have also been recognized and upheld by no less than the Supreme
a contract, trust, pool, holding company, or other form of Court En Banc in the Tatad and Lagman cases . . . ." He contends that "the
association, for the purpose of unduly restricting competition, Big 3 remain as strong and dominant as ever."
monopolizing trade and commerce in a certain commodity,
controlling its production, distribution and price, or otherwise In other words, petitioner believes that there is no valid reason to lift price
interfering with freedom of trade without statutory authority. control at this time when allegedly there still exists an oligopoly in the
Combination in restraint of trade refers to the means, while industry. He proposes instead that government control should stand for an
monopoly refers to the end. indefinite period until the new players are able to capture a substantial part of
the market.
In that case, RA 8180, the predecessor of RA 8479, was struck down by this
Court for being contrary to Section 19, Article XII of the Constitution. We took Unfortunately, however, the foregoing thematic statements and economic
this action because we found that its provisions on (1) tariff differential, (2) theory of Petitioner Garcia are policy in nature and are arguments supporting
minimum inventory and (3) predatory pricing "inhibit fair competition, the wisdom of interim government price control. Indeed, "self-evident truths,"
encourage monopolistic power and interfere with the free interaction of economic theories, deeply-held beliefs, speculative assumptions and
market forces." We concluded, "The aftermath of R.A. No. 8180 is a generalizations may be the bases of legislative and executive actions, but
deregulated market where competition can be corrupted and where market they cannot be substitutes for evidence and legal arguments in a judicial
forces can be manipulated by oligopolies." proceeding. Considered judgment calls of the legislative and the executive
departments are the issues of whether the country should adopt the policy of
complete or partial deregulation, and when such policy should take effect and
over what products or services. These issues come within judicial

478
determination only when there is clear and substantial proof that said policy In this context, I have pored over the records of this case and searched long
and its concomitant variations are violative of the Constitution or are made by and wide for such factual and legal bases but, other than presumptions and
those agencies in grave abuse of their discretion. generalizations that are unsupported by hard evidence, I could not find any.
Petitioner fails to substantiate his allegations that the three oil giants have
The Legal Issue Is Whether Petitioner engaged, directly or indirectly, in an unholy alliance to fix prices and restrain
trade.
Has Submitted Sufficient Proof That the
True, retail prices of petroleum products have been increased, to the
Big Three Have Violated the Constitution consternation of the public, but petitioner has not shown by specific fact or
clear proof how the questioned provision of RA 8479 has been used to
transgress the Constitution. He has not demonstrated that the Big Three
To be more specific, the pivotal issue before this Court is not whether it is
arbitrarily dictate and corrupt the price of oil in a manner violative of the
wiser and more beneficial to empower the government to fix fuel prices;
Constitution.
rather, it is whether petitioner has submitted enough factual bases to justify
the legal conclusion that the Big Three Petron, Shell and Caltex have
combined themselves "in restraint of trade or [to cause] unfair competition," Petitioner merely resurrects and relies heavily on the arguments, the
to such an extent as to legally justify a striking down of Section 19 of RA statistics and the proofs he submitted two years ago in the first oil
8479. The task of proving this issue is not easy; in fact, it is formidable and deregulation case, Tatad v. Secretary of the Department of Energy. Needless
daunting. This is because laws are prima facie presumed constitutional and, to state, those reasons were taken into consideration in said case, and they
unless clearly shown to be infirm, they will always be upheld. 8 So, too, indeed helped show the unconstitutionality of RA 8180. But exactly the same
regularity in the performance of official functions is the postulate, and any old grounds cannot continue to support petitioner's present allegation that the
allegation of grave abuse or irregularity must be proven cogently. major oil companies Petron, Shell and Caltex persist to this date in their
oligopolistic practices, as a consequence of the current Oil Deregulation Law
and in violation of the Constitution. In brief, the legal cause and effect
Deregulation per se Is
relationship has not been amply shown.

Not Constitutionally Infirm


Petitioner Has Not Proven

A close perusal of the assailed Section 19 of RA 8479 and Section 19 of


Arbitrariness or Despotism
Article XII of the Constitution does not readily reveal their irreconcilability.
Indeed, even petitioner admits that the deregulation policy per se is not
contrary to the Constitution. Neither could it be successfully argued that the Petitioner harps at the five-month period of transition from price control to full
implementation of such policy within the five-month phase-in period is per se deregulation provided under Section 19 of RA 8479. He claims that such
anathema to our fundamental law. It is his imperative task therefore to short period is not enough to ensure a "truly competitive market" in the
adduce before the Court factual and legal bases to demonstrate clearly and supposed oligopoly of the oil industry. Again, his statement is not backed up
cogently the unconstitutionality of the acts of Congress and the President in by evidentiary basis. He offers no substantial proof that Congress, in
adopting and implementing full deregulation of petroleum prices at this time. deciding to lift price controls five months from the effectivity of RA 8475,
gravely abused its discretion. To repeat, it is not within the province of the

479
judiciary to determine whether five months is indeed short and, for that Indeed, they believe that the questioned provision has given them the
matter, what length of time is adequate. That is a matter of legislation impetus to compete and thereby eventually show the benefits of
addressed to the discretion of our policy makers. deregulation; namely, the best products at the cheapest prices.

It is basic to our form of government that the Court cannot inquire into the With this factual backdrop and in the dire absence of contrary proof, it would
wisdom or expediency of the acts of the executive or the legislative be specious to conclude that under the aegis of Section 19 of RA 8479, the
department, unless there is a clear showing of constitutional infirmity or grave Big Three have restrained trade or unduly restricted competition.
abuse of discretion amounting to lack or excess of jurisdiction. 9 "By grave
abuse of discretion is such capricious and whimsical exercise of judgment as Moreover, the three provisions in RA 8180 which were adjudged abhorrent to
is equivalent to lack of jurisdiction. Mere abuse of discretion is not enough. It the fundamental principles of free enterprise are no longer found in RA 8479.
must be grave abuse of discretion, as when the power is exercised in an The depletion of the Oil Price Stabilization Fund, the extraneous factor that
arbitrary or despotic manner by reason of passion or personal hostility, and was considered by the President in accelerating the implementation of full
must be so patent and so gross as to amount to an evasion of a positive duty deregulation under RA 8180, was no longer taken into account in the present
or to a virtual refusal to perform the duty enjoined or to act at all in milieu. The Court's reasons for declaring the unconstitutionality of RA 8180
contemplation of law." 10 These jurisprudential elements of arbitrariness, are, therefore, not germane to the validity of RA 8479. The petitioner cannot
despotism, passion and hostility have not been shown to exist under the rely on the same rationale for the purpose of successfully assailing RA 8479.
present circumstances. Indeed, he admits that "the Tatad and Lagman cases . . . did not consider
and adjudicate on the lifting of price control per se, under RA 8180, as an
Market Share of New Players issue."

Has Increased Under RA 8479 Epilogue

Historically, deregulation as a policy in the downstream oil industry was In sum, I make no secret of my sympathy for petitioner's frustration at the
begun in 1996 when new players started to set up and operate their inability of our government to arrest the spiraling cost of fuel and energy. 13 I
businesses in the country. That was practically a full three years of hear the cry of the poor that life has become more miserable day by day. I
operations, the last two of which saw no significant barriers in terms of tariff feel their anguish, pain and seeming hopelessness in securing their material
differential, minimum inventory or predatory pricing. needs.

Obviously, the conditions prevailing when the Court struck down RA 8180 However, the power to lower petroleum prices through the adoption or the
two years ago have not been proven to be prevalent at present. In 1996, the rejection of viable economic policies or theories does not lie in the Court or
new players had a market share of barely one percent. 11 The new players its members. Furthermore, absent sufficient factual evidence and legal
have since expanded or increased in number (46 as of June 30, 1999), and moorings, I cannot vote to declare a law or any provision thereof to be
they now have about nine percent share of the market. 12 Significantly, these unconstitutional simply because, theoretically, such action may appear to be
new players have intervened in this case in defense of the law. These are the wise or beneficial or practical. Neither can I attribute grave abuse of
little Davids who claim that with RA 8479 as their slingshot, they can, given discretion to another branch of government without an adequate showing of
enough time, fight and win against the three erstwhile unbeatable Goliaths.

480
patent arbitrariness, whim or caprice. Should I do so, I myself will be gravely Consequently, Executive Order No. 392 (E.O. 392) implementing the
abusing my discretion, the very evil that petitioner attributes to the legislature. provision of said law was voided. On December 3, 1997, the motions for
reconsideration were denied for utter lack of merit.
WHEREFORE, I vote to DISMISS the Petition.
Now before us is a challenge to the second oil industry deregulation law, R.A.
8479. The relevant factual and procedural antecedents of the present petition
are as follows:
QUISUMBING, J., concurring opinion;
In 1992, the Philippine government welcomed more liberal economic policies
I fully concur in the ponencia of Justice Consuelo Ynares-Santiago. What I and started the ground work for privatization of some government-owned or
would like to stress here and now is that, contrary to certain ill-informed controlled corporations and deregulation of the oil industry. In due time,
comments in media, petitioner's pleadings were thoroughly dissected at the Congress enacted Republic Act No. 7638 on December 9, 1992. It created
hearing where he and his counsel as well as the respondents amply the Department of Energy (DOE). Among others, it was tasked, at the end of
presented their arguments. Questions of law and policy were also illuminated four years from the effectivity of R.A. No. 7638 and upon approval of the
from different perspectives in sessions and in memoranda internally President, to institute the "programs and the timetable for the deregulation of
exchanged by members of the Court. Right away, it must be added, no delay appropriate projects and activities of the energy industry." 2
attended the resolution of this petition. For while the Constitution allows two
years, this case was decided en banc in less than half that period, from the Following the intent of R.A. 7638, the Philippine National Oil Company
time of submission of the parties' memoranda. Below is a full presentation of (PNOC) sold 40% of its equity in Petron Corporation to the Aramco Overseas
my view on the controversy generated by petitioner's insistence that the Company.
Court overturn an act passed by his own branch of government and
approved by the Chief Executive. Sometime in March 1996, Congress made that daring step towards the
realization of liberating the oil industry from government regulation and
At issue in this special civil action for certiorari under Rule 65 is the enacted R.A. 8180. On February 8, 1997, President Fidel V. Ramos issued
constitutionality of Sec. 19 of Republic Act No. 8479, 1 entitled "An Act E.O. 392, which signaled the implementation or start of deregulation in the oil
Deregulating the Downstream Oil Industry and for other Purposes". The law industry.
was enacted pursuant to the policy of the State to liberalize and deregulate
the downstream oil industry. R.A. 8479 is the remedial legislation passed by Senator Francisco Tatad and Congressmen Enrique Garcia, Edcel Lagman,
Congress to cure the infirmities found in Republic Act No. 8180, the first oil Joker Arroyo and Wigberto Taada, among others, filed separate petitions
industry deregulation law, otherwise known as the "Downstream Oil Industry docketed as G.R. Nos. 124360 and 127867, before the Court. The petitioners
Deregulation Act of 1996". contended that some of the provisions of R.A. No. 8180 violated Section 19
of Article XII of the 1987 Constitution, which states:
In a banc decision promulgated on November 5, 1997, the Court declared
R.A. 8180 unconstitutional for having transgressed the constitutional The State shall regulate or prohibit monopolies when the
prohibition against monopolies and combinations in restraint of trade, public interest so requires. No combinations in restraint of
specifically mandated in Section 19, Article XII of the Constitution. trade or unfair competition shall be allowed.

481
The challenged provisions in R.A. 8180 were: In declaring provisions of R.A. 8180 unconstitutional, the Court held:

(1) the provision on tariff differential found in Section 5 (b) which states: . . . Petron, Shell and Caltex stand as the only major league
players in the oil market. . . . The tariff differential of 4%
Sec. 5 (b) Any law to the contrary notwithstanding and therefore works to their immense benefit. . . . New players
starting with the effectivity of this Act, tariff duty shall be that intend to equalize the market power of Petron, Shell and
imposed and collected on imported crude oil at the rate of Caltex by building refineries of their own will have to spend
three percent (3%) and imported refined petroleum products billions of pesos. Those who will not build refineries but
at the rate of seven percent (7%), except fuel oil and LPG, compete with them will suffer the huge disadvantage of
the rate for which shall be the same as that for imported increasing their product cost by 4%. They will be competing
crude oil: Provided, that beginning on January 1, 2004 the on an uneven field.
tariff rate on imported crude oil and refined petroleum
products shall be the same. Provided, further, That this The provision on inventory widens the balance of advantage
provision may be amended only by an Act of Congress. of Petron, Shell and Caltex against prospective new players.
Petron, Shell and Caltex can easily comply with the
(2) the minimum inventory clause, in Section 6 which provides: inventory requirement of R.A. No. 8180 in view of their
existing storage facilities. Prospective competitors again will
Sec. 6 To ensure the security and continuity of petroleum find compliance with this requirement difficult as it will entail
crude and products supply, the DOE shall require the a prohibitive cost. . . .
refiners and importers to maintain a minimum inventory
equivalent to ten percent (10%) of their respective annual Finally, we come to the provision on predatory pricing which
sales volume or forty (40) days of supply, whichever is lower. is defined as ". . . selling or offering to sell any product at a
price unreasonably below the industry average cost so as to
(3) the predatory pricing scheme in Section 9: attract customers to the detriment of competitors." . . . The
ban on predatory pricing cannot be analyzed in isolation. Its
validity is interlocked with the barriers imposed by R.A. No.
Sec. 9 To ensure fair competition and prevent cartels and
8180 on the entry of new players. 3
monopolies in the downstream oil industry, the following acts
shall be prohibited:
That decision came under sharp attack by critics who accused the Court of
improvidently intervening in the economic affairs of the State. Economists
xxx xxx xxx
and businessmen remarked that the decision was a major blow to economic
reforms and an additional burden to the government's already huge budget
(b) Predatory pricing which means selling or offering to sell deficit as it would require reinstating a subsidy on oil products. 4 Pertinent
any product at a price unreasonably below the industry portions of the Decision decreed:
average cost so as to attract customers to the detriment of
competitors.

482
With this Decision, some circles will chide the Court for The Court found no merit in the motion for reconsideration, motion for
interfering with an economic decision of Congress. Such intervention, and partial motion for reconsideration. Despite the separability
criticism is charmless for the Court is annulling R.A. No. clause, the Court ruled that the three questioned provisions cannot be struck
8180 not because it disagrees with deregulation as an down alone, for they were the ones intended to carry out the policy of the law
economic policy but because as cobbled by Congress in its as embodied in Section 2. 6
present form, the law violates the Constitution. The right call
therefor should be for Congress to write a new oil On the question of the validity of E.O. 392, the Court held that the Executive
deregulation law that conforms with the Constitution and not Department failed to follow faithfully the standards set by R.A. 8180 when it
for this Court to shirk its duty of striking down a law that considered the extraneous factor of depletion of the Oil Price Stabilization
offends the Constitution. . . . Indeed when confronted by a Fund (OPSF) fund, instead of limiting the basis for the acceleration of full
law violating the Constitution, the Court has no option but to deregulation of the industry to only two factors, viz: (1) the time when the
strike it down dead. . . . Hence, for as long as the prices of crude oil and petroleum products in the world market are declining,
Constitution reigns supreme so long will this Court be vigilant and (2) the time when the exchange rate of the peso in relation to the US
in upholding the economic rights of our people especially dollar is stable. 7 By considering another factor, the Executive Department
from the onslaught of the powerful. Our defense of the rewrote the standards set forth in R.A. 8180. 8 In light of the uncertainty of the
people's economic rights may appeal heartless because it consideration given by the Executive department to the depletion of the
cannot be half-hearted. OPSF fund for the full deregulation of the oil industry, we ruled that E.O. 392
constituted a misapplication of R.A. 8180. In sum, the implementing order
IN VIEW WHEREFORE, the petitions are granted. R.A. No. was found void, while the basic law was held unconstitutional.
8180 is declared unconstitutional and E.O. No. 372 [392]
void. 5 On reconsideration, our December 3, 1997 Resolution stressed that R.A.
8180 is unconstitutional because (1) it gave more power to an already
Public respondents filed their consolidated motion for reconsideration. Some powerful oil oligopoly; (2) it blocked the entry of effective competitors; and (3)
of the new players, in the industry: Eastern Petroleum Corp., Seaoil it will sire an even more powerful oligopoly whose unchecked power will
Petroleum Corp., Subic Bay Distribution, Inc., TWA, Inc., and Dubphil Gas prejudice the interest of the consumers and compromise the general
moved to intervene and aired their stand against the total nullification of R.A. welfare. 9 The Court reiterated, however, that there was no impediment in re-
8180. They also averred that they were in favor of declaring the three enacting R.A. 8180 minus the provisions which are anti-competition.
offensive provisions unconstitutional. Petitioner Enrique T. Garcia, likewise,
filed a partial motion for reconsideration and pushed for a return only to Consequently, Congress fast-tracked a new oil deregulation law, R.A. 8479,
partial deregulation in which the main features of deregulation would be which was approved and duly signed on February 10, 1998. It took effect an
allowed free reign, but the retail price of oil products would still be regulated February 12, 1998 upon the completion of its publication in a newspaper of
through the Energy Regulatory Board. general circulation.

Dissatisfied with the amendments incorporated into the new law by his own
colleagues in Congress, Honorable Enrique T. Garcia filed the instant
petition.

483
The Court is the ultimate guardian of our Constitution. By virtue of its power warranted, pursuant to Republic Act No. 1937, as amended,
of judicial review, it is duty-bound in an appropriate case to ascertain whether otherwise known as the "Tariff and Customs
a law is free from constitutional flaws. While favoring free competition in the Code": Provided, further, That beginning January 1, 2004 or
oil industry, the Court struck down R.A. 8180 because of provisions therein upon implementation of the Uniform Tariff Program under the
that contravened the basic law, our Constitution. Before dwelling into the World Trade Organization and ASEAN Free Trade Area
issues now raised by the petitioner, we must determine whether R.A. 8479 commitments, the tariff rate shall be automatically adjusted
truly cured the invalid portions of R.A. 8180. When we advocated vigilance in to the appropriate level notwithstanding the provisions under
upholding the economic rights of our people, we truly hoped that Congress this Section.
would address the defects of R.A. 8180 and not re-enact R.A. 8180 through
the guise of R.A. 8479. Second, the minimum inventory level requirement. R.A. 8479 eliminated the
provision in R.A. 8180 requiring the refiners and importers to maintain a
It bears recalling, however, that when the Supreme Court mediates to minimum inventory equivalent to ten percent (10%) of their respective annual
allocate constitutional boundaries or invalidates the acts of a coordinate sales volume or forty (40) days' supply. The minimum inventory requirement
body, what it is upholding is not its own supremacy but the supremacy of the was removed, giving the new entrants opportunities to use their resources to
Constitution. With this in mind, we now focus on the provisions of R.A. 8479, be more competitive.
in particular the 4% tariff differential, minimum inventory level, and predatory
pricing provisions, which aim to prevent the big three oil companies from Third, predatory pricing. In the December 3, 1997 Resolution of the Court in
taking advantage of deregulation as a means of cartelizing their operations, G.R. Nos. 124360 and 127867, we expressed the view that the definition of
and thereby result in monopolistic and oligopolistic practices condemned by predatory pricing was too loose to be a real deterrent. 10 Congressman Dante
the basic law of the land. O. Tinga acknowledged in his explanatory note of House Bill 10057 (H.B.
10057) that the definition of predatory pricing needed specificity, particularly
First, the 4% tariff differential. On December 31, 1997, after the Court with respect to the definitive benchmark price and the express anti-
declared with finality that R.A. 8180 is unconstitutional, President Ramos competitive intent. He suggested the Areeda-Turner test and proposed to
issued Executive Order No. 461. The Order imposed a three percent (3%) redefine predatory pricing. Section 11 par. (b) of R.A. 8479 adopted
import duty on petroleum products enumerated therein. The President's Congressman Tinga's recommendation, to wit:
move avoided the revival of the old tariff rates of 10% on crude oil and 20%
on refined oil while the legislative department was in the process of crafting a b) Predatory pricing which means selling or
new oil deregulation law. Noteworthy, Sec. 6 of R.A. 8479 imposed the same offering to sell any oil product at a price
tariff treatment on petroleum products. Section 6 reads: below the seller's or offeror's average
variable cost for the purpose of destroying
Sec. 6 a) Any law to the contrary notwithstanding and competition, eliminating a competitor or
starting with the effectivity of this Act, a single and uniform discouraging a potential competitor from
tariff duty shall be imposed and collected both on imported entering the market: Provided, however,
crude oil and imported refined petroleum products at the rate That pricing below average variable cost in
of three percent (3%): Provided, however, That the President order to match the lower price of the
of the Philippines may, in the exercise of his powers, reduce competitor and not for the purpose of
such tariff rate when on his judgment such reduction is destroying competition shall not be deemed

484
predatory pricing. For purposes of this After the transition phase comes full deregulation as provided by Sec. 19 of
prohibition, "variable cost" as distinguished R.A. 8479, which reads thus:
from "fixed cost", refers to costs such as
utilities or raw materials, which vary as the Sec. 19. Start of Full Deregulation. Full deregulation of
output increases or decreases and "average the Industry shall start five (5) months following the effectivity
variable cost" refers to the sum of all of this Act: Provided however, That when the public interest
variable costs divided by the number of units so requires, the President may accelerate the start of full
of outputs. deregulation upon the recommendation of the Department of
Energy (DOE) and the Department of Finance (DOF) when
To strengthen the anti-trust safeguards of R.A. 8479, respondents argue that the prices of crude oil and petroleum products in the world
there are enough provisions to encourage entry of new participants. For market are declining and the value of the peso in relation to
instance, R.A. 8479 allows for active participation of the private sector and the US dollar is stable, taking into account relevant trends
cooperatives in the retail of petroleum through joint ventures to establish and prospects: Provided, further, That the foregoing
gasoline stations. Moreover, R.A. 8479 requires initial public offering of provision notwithstanding, the five (5)-month Transition
shares equivalent to 10% of the capital investments by oil companies. Phase shall continue to apply to LPG, regular gasoline and
Respondents also cite that the enforcement of monitoring activities by the kerosene as socially-sensitive petroleum products and said
DOE encourages consumer vigilance over unwarranted increase in the petroleum products shall be covered by the automatic pricing
prices of petroleum products. Another safeguard against collusion among mechanism during the said period. 12
oligopolists is the creation of a task force with members from the DOE and
the Department of Justice (DOJ) to investigate complaints for violations of Note that the abovecited transition phase of five months could be
R.A. 8479. They assert that the mere dominance of Petron, Pilipinas Shell, abbreviated when public interest so requires. The President's power to
and Caltex, is not per se a combination in restraint of trade. Combination in accelerate the start of full deregulation, however, depended upon the
restraint of trade, they claim, is the means to achieve monopoly. recommendation of the Departments of Energy and Finance.

Petitioner Garcia adverts to oil deregulation in phases. The new oil Accordingly as recommended, on March 14, 1998, President Ramos issued
deregulation law has two phases: (1) the transition phase and (2) the full E.O. 471 to accelerate the implementation of full deregulation. Partinently the
deregulation phase. E.O., which implements R.A. 8479, provides:

During the transition period, all non-pricing aspects were lifted. Although the WHEREAS, Republic Act No. 7638, otherwise known as the
Oil Price Stabilization Fund was abolished, a buffer fund 11 was created to "Department of Energy Act of 1992," provides that, "at the
cover increases in the prices of petroleum products, except premium end of four years from its effectivity last December 1992, the
gasoline. The Automatic Oil Pricing Mechanism was maintained to Department [of Energy] shall, upon approval of the
approximate the domestic prices of petroleum products in the international President, institute the programs and timetable of
market. The Energy Regulatory Board (ERB) approved a market-oriented deregulation of appropriate energy projects and activities of
formula to determine the Wholesale Posted Price of petroleum products the energy sector;"
based solely on the changes of either the Singapore Posting of refined
petroleum products, the Singapore Import Parity or the crude landed cost.

485
WHEREAS, Section 19 of Republic Act No. 8479, otherwise WHEREAS, pursuant to Republic Act No. 8479, the
known as the "Downstream Oil Industry Deregulation Act of deregulation of the industry will foster a truly competitive
1998," provides that [T]hat "when the public interest so market which can better achieve the social policy objectives
requires, the President may accelerate the start of full of fair prices and adequate, continuous supply of
deregulation upon the recommendation of the Department of environmentally-clean and high quality petroleum products;
Energy (DOE) and the Department of Finance (DOF) when
the prices of crude oil and petroleum products in the world NOW, THEREFORE, I, FIDEL V. RAMOS, President of the
market are declining and the value of the peso in relation to Philippines, by the powers vested in me by law, do hereby
the US dollar is stable, taking into account relevant trends declare the full deregulation of the downstream oil
and prospects: Provided, further, That the foregoing industry; provided, however, that LPG, regular gasoline and
provision notwithstanding, the five (5)-month Transition kerosene shall be covered by the Automatic Pricing Formula
Phase shall continue to apply to LPG, regular gasoline and pursuant to R.A. No. 8479. 13
kerosene as socially-sensitive petroleum products and said
petroleum products shall be covered by the automatic pricing The implementing guidelines for the acceleration of full deregulation of the
mechanism during said period; industry, set forth in E.O. 471, required the concurrence of two
conditions, viz.: (1) the downtrend of prices of oil and petroleum products,
WHEREAS, pursuant to the joint recommendation of the and (2) stability of exchange rate of peso in relation to US dollar, taking into
Department of Energy and the Department of Finance, and account relevant trends and prospects.
in the interest of the consuming public, recent developments
favor the acceleration of the start of full deregulation of the However, E.O. 471 carried an additional proviso, the transition phase was
downstream oil industry because: (i) the prices of crude oil continued for LPG, regular gas and kerosene. These socially sensitive
and petroleum products in the world market are beginning to products continued to be covered by the automatic pricing mechanism until
be stable and on a downtrend since January 1998; and (ii) July of 1998. Only then was full deregulation of the industry effected, and the
the exchange rate of the peso in relation to the US dollar has automatic pricing mechanism was also lifted for LPG, regular gas and
been stable for the past three months, averaging at around kerosene.
P40.00 to one US dollar;
Turning now to herein petition, Congressman Enrique Garcia raised the
WHEREAS, Executive Order No. 377 dated 31 October following issues to assail the provision implementing full deregulation of the
1996 provides for an institutional framework for the oil industry:
administration of the deregulated industry by defining the
functions and responsibilities of various government
I. Sec. 19 OF R.A. NO. 8479 which provides for full
agencies;
deregulation five (5) months or earlier following the
effectivity of the law, is glaringly pro-oligopoly, anti-
competition and anti-people, and is therefore patently
unconstitutional for being in gross and cynical contravention
of the constitutional policy and command embodied in

486
Article XII, Section 19 of the 1987 Constitution against deregulation decreed under E.O. 471. Petitioner insists that the short
monopolies and combinations in restraint of trade. transition period is pro-oligopoly, anti-competition and anti-people and is
patently unconstitutional because the period is too short to establish true
II. Said Section 19 of R.A. No. 8479 is glaringly pro-oligopoly, competition in the local oil industry. True competition, he claims, exists only
anti-competition and anti-people, for the further reason that it when there can be a sizable number of players, and at present, the new
palpably and cynically violates the very objective and players comprise only 3% of the market share which does not put up real
purpose of R.A. No. 8479, which is to ensure a truly competition against the "Big Three" oil companies (Caltex, Shell and Petron).
competitive market under a regime of fair prices. What he suggests is to prolong the transition phase or partial deregulation
with price controls while the big oil companies are still dominating the market,
III. Said Section 19 of R.A. No. 8479, being glaringly pro- to ensure the protection of the public interest and prevent the big three
oligopoly, anti-competition and anti-people, being patently oligopolies from fixing the price or overpricing. He further contends that the
unconstitutional and being palpably violative of the law's automatic oil pricing mechanism will enable the domestic price of petroleum
policy and purpose of ensuring a truly competitive market products to approximate and promptly reflect the price of oil in the
under a regime of fair prices, is a very grave and grievous international market. He also stressed that new players may come under an
abuse of discretion on the part of the legislative and indefinite or open-ended transition phase.
executive branches of government.
Commenting on the petition, respondents claim that the propriety of full
IV. Premature full deregulation under Section 19 of R.A. No. deregulation involves the wisdom of Congress and is therefore, a non-
8479 may and should therefore be declared null and void justiciable issue. They counter petitioner's arguments by pointing out that the
even as the rest of its provisions remain in force, such as the shortening of the transition period and acceleration of full deregulation were
transition phase or partial deregulation with price controls decreed pursuant to the joint recommendation of the DOE and DOF, based
that ensures the protection of the public interest by on the concurring conditions of a downtrend of crude oil in world market and
preventing the big 3 oligopoly's price-fixing and overpricing. the stability of the exchange rate of P40.00 to US$1.

These issues may be synthesized into one: Whether or not the full The respondents argue that the short transition period is not violative of the
implementation of deregulating the downstream oil industry as provided in Constitution because the new players were given until July 1998 to set up
Section 19 of R.A. 8479 violates the Constitutional mandate of free their businesses as they have in fact, and they have captured at least 3% of
competition in a liberalized oil industry under Section 19, Article XII of the the total oil market.
1987 Philippine Constitution?
Respondent Petron asserts that full deregulation protects the public from the
Petitioner Garcia principally faults Section 19 of the new R.A. 8479 as well as greed and exploitation of business. Petron further contends that competition
E.O. 471 now for violating the constitutional prohibition against monopoly, can be ushered in only with the certainty of price deregulation and the short
and being anti-competition. transition period would guarantee the investors that within a manageable
period, they would be able to set prices, taking into account their investment
and operating costs. It claims an indefinite transition period would discourage
Petitioner claims that there was a premature full deregulation under Section
new investors because the new players had hoped that within a reasonable
19 of R.A. 8479. He protests the acceleration of the full implementation of
time, price regulation would be lifted.

487
The Solicitor General filed a comment on behalf of the public respondents, time it as far as practicable when the prices of crude oil and
interposing economic arguments that price regulation reduces economic petroleum products in the world market are declining and
efficiency and is prejudicial to the public. 14 Public respondents assert that the when the exchange rate of the peso in relation to the US
acceleration of full deregulation is based on existing conditions and sound dollar is stable.
economic theory.
xxx xxx xxx
Respondent Shell filed a rejoinder, stating that to prolong the transition period
will revive the automatic pricing mechanism which means that it will only It ought to follow that the argument that E.O. No. 392 is null
replace the mode of price regulation by still another regulatory scheme. It and void as it was based on indeterminate standards set by
argues that if Sec. 19 of R.A. 8479 were to be struck down, full deregulation R.A. 8180 must likewise fail. If that were all to the attack
will never take place and it would render the entire law different from what against the validity of E.O. No. 392, the issue need not
was passed by Congress. further detain our discourse. 17

Petitioner counters that he is questioning the constitutionality rather than the In G.R. No. 127867, Congressman Garcia filed an Urgent Motion for Partial
wisdom of Sec. 19 of R.A. 8479; it is pro-oligopoly, hence patently Reconsideration from the November 5, 1997, decision of the Court. He
unconstitutional. Petitioner further avers that condemnation against sought to strike down only the premature full deregulation but maintain partial
monopolies and combination in restraint of trade should be given legal deregulation under R.A. No. 8180 with price controls and price mechanism
sanction by the Court. Petitioner maintains that the nullification of Sec. 19 of based on Singapore Posted Prices. The Court resolved the issue this way:
R.A. 8479 will result in partial deregulation, where there will be no regulation
as regards the importation of petroleum products and the establishment of We shall first resolve petitioner Garcia's linchpin contention
gas station, but oil pricing would be regulated based on the Automatic Pricing that the full deregulation decreed by R.A. No. 8180 to start at
Mechanism. the end of March 1997 is unconstitutional. For prescinding
from this premise petitioner suggests that "we simply go
Note that during the review of R.A. 8180 by the Court in G.R. No. 127867, back to the transition period under R.A. No. 8180." Under the
petitioners Edcel C. Lagman, Arroyo, et al., likewise questioned the transition period, price control will be revived through the
constitutionality of Section 15 of R.A. No. 8180 15 as well as E.O. No. automatic pricing mechanism based on Singapore Posted
392 16 which provided for the implementation of full deregulation. The Court Prices. The Energy Regulatory Board . . . would play a
decreed thus: limited and ministerial role of computing the monthly price
ceiling of each and every petroleum fuel product, using the
. . . Full deregulation at the end of March 1997 is mandatory automatic pricing formula. . . .
and the Executive has no discretion to postpone it for any
purported reason. Thus, the law is complete on the question We are not impressed by petitioner Garcia's submission.
of the final date of full deregulation. The discretion given to Petitioner has no basis in condemning as
the President is to advance the date of full deregulation unconstitutional per se the date fixed by Congress for the
before the end of March 1997. Section 15 lays down the beginning of the full deregulation of the downstream oil
standard to guide the judgment of the President he is to industry. . . . The choice of March 1997 as the date of full

488
deregulation is a judgment of Congress and its judgment call be duly suspended only by authority of the legislature. 21 Indeed, a
cannot be impugned by this Court. 18 suspension of a valid statute must rest upon legislative action; 22 it may not
be effected solely by a judicial act. 23 Clearly it is a policy decision of the
Now in the present petition, Garcia insists on his old plea for a return only to legislative and executive departments in whose turf we must not tread, under
partial deregulation of the downstream oil industry, wherein the main features the principle of separation of powers. The term "political question" connotes
of deregulation would be permitted but the retail prices of oil products would what it means in ordinary parlance, namely, a question of policy. 24 It refers to
still be regulated through an Automatic Pricing Mechanism. "those questions which, under the Constitution, are to be decided by the
people in their sovereign capacity, or in regard to which full discretionary
However, I find his contentions to be lacking legal basis, even if his proposal authority has been delegated to the legislative or executive of the
appears to be expedient, or even beneficial, especially to the poor. As the government." 25 If is concerned with issues dependent upon the wisdom, not
Court said Taada vs. Tuvera, 19 "[T]his Court is not called upon to rule on the legality, of a particular measure. 26 The judiciary does not directly settle policy
wisdom of the law or to repeal it or modify it if we find it impractical. That is issues. Under our system of government, policy issues are within the domain
not our function. That function belongs to the legislator. Our task is merely to of the political branches of government and of the people themselves as the
interpret and apply the law as conceived and approved by the political repository of all state powers. 27
departments of the government in accordance with the prescribed
procedure." 20 In PLDT vs. National Telecommunications Commission, 28 the ultimate
considerations cited in matters affecting vital industries, are the public need,
For if we allow an open-ended transition period to maintain government public interest, and the common good. In that case, the Court said:
pricing regulation, we would have suspended the much-needed liberalization
of the downstream oil industry. It would certainly run counter to the Free competition in the industry may also provide the answer
government's policy of allowing free interplay of market forces, with minimal to a much-desired improvement in the quality and delivery of
government supervision. In fact, it could defeat full deregulation to ensure fair this type of public utility, to improved technology, fast and
competition in the downstream oil industry, where new and prospective handy mobile service, and reduced user dissatisfaction. 29
players are on even level playing field with the Big Three.
Similarly, the above-mentioned considerations could undergird the nation's
Furthermore, to base the implementation of full deregulation on the presence energy and other economic policies. The liberalization of the oil industry is a
of a sizable number of new investors, as petitioner would want us to do, reform program initiated by Congress to free the government from the
would be to legislate a floating provision dependent on the happening of a obligation of infusing funds to subsidize increases in the prices of oil
contingent event. To do so, would be to undermine the very purpose of the products. Such funds may now be utilized for other much needed programs
law, which is to liberalize and deregulate the downstream oil industry in order with a public purpose.
to ensure a truly competitive market under a regime of fair prices, adequate
and continuous supply, environmentally clean and high-quality petroleum Well-established is the principle that every law has in its favor the
products. presumption of constitutionality. 30 To declare a law unconstitutional, the
repugnancy of that law to the Constitution must be clear and unequivocal.
Consequently, to heed the petitioner's prayer, this Court would have to But we recognize that even if a law is aimed at the attainment of some public
legislate, a power granted only to Congress. The operation of a statute may good, still its provisions cannot infringe upon constitutional rights. 31That

489
infringement, however, must be proved and established persuasively to concern is that the government might be helpless in case the Big 3 (Shell,
invalidate a provision of a law, if not the entire law itself. Petron and Caltex) overprice their petroleum products. But the people are not
without legal recourse. The public can manifest outright objections to
Petitioner ought to have demonstrated the need for the extension of the overpricing and report to the Department of Energy any unreasonable
transition period. But, in fact, he could not downplay the DOE report that new increase in the prices of these oil products. The monitoring power of the DOE
players accounted for a sizable share of the market, some 18.1 percent of is embodied in Sec. 14 of R.A. 8479, and its implementing rule, Section 18 of
the total product imports, and competing companies are keen in joining the DOE Circular No. 98-03-004, thus:
Philippine oil industry since the full implementation of deregulation. And, as
stressed by the public respondents in the rejoinder dated January 7, 1999: R.A. 8479, Sec. 14 Powers and Functions of the DOE and
DOE Secretary:
Since 1996, new players have taken a significant share in
the market, to wit: (a) seven (7) new players have entered Monitoring
the downstream oil industry before RA No. 8180; (b) during
the effectivity of RA No. 8180, twenty eight (28) new players a) The DOE shall monitor and publish daily international
have engaged in a number of downstream oil industry crude oil prices, as well as follow the movements of
activities; and (c) three (3) new players have engaged in fuel domestic oil prices. It shall likewise monitor the quality of
bulk marketing, while two (2) new players have started to petroleum products and stop the operation of business
establish gasoline service stations immediately before and involved in the sale of petroleum products which do not
during the effectivity of RA No. 8479. At the same time, many comply with the national standards of quality that are aligned
more companies have indicated their intention to enter the with the national standards/protocols of quality. . . .
downstream oil industry business. 32
xxx xxx xxx
The new players, according to industry experts, are gradually making
a dent in the local market and their share is expected to surge in a d) Any report from any person of an unreasonable rise in the
few years when their retail stations are established. 33 prices of petroleum products shall be immediately acted
upon. For this purpose, the creation of DOE-DOJ Task Force
However, the presence or entry of numerous players in the oil industry is not is hereby mandated to determine within thirty (30) days the
a condition precedent before a full deregulated petroleum industry could be merits of the report and initiate the necessary actions
had. But we recognize that it is precisely the implementation of full warranted under the circumstances: Provided that nothing
deregulation that would serve to entice new players to compete against the herein shall prevent the said task force from investigating
so-called Big Three. Hopefully, this move would prevent the powerful oil and/or filing the necessary complaint with the proper court or
companies from manipulating prices, to the prejudice of the consumers and agency motu propio.
the public in general.
Department Circular No. 98-03-004, Sec. 18 Powers and
The petitioner strongly manifested his fears concerning pernicious Functions of the DOE and DOE Secretary
consequences of total lifting of price control in the oil industry. His main

490
Monitoring color. The numeric entries in
these boards shall be at
The DOE shall monitor the following pursuant to Section 14 least six (6) inches in
of the Act. Any misrepresentation, mislabeling, concealment height.
or fraud, shall be subject to penalties under existing
applicable laws. The price display boards
shall be properly installed
a. Prices and labeled not later than
June 30, 1998. Failure to
The DOE shall monitor and publish comply with this
international oil prices as well as follow the requirements shall be
movement of domestic oil prices. penalized pursuant to
Section 24 of the Act.
(1) Price Display Boards
(2) Unreasonable Rise in
Prices
For the convenience of the
public, all retailers of
petroleum products shall Any report from any person
display the prices of each of an unreasonable rise in
type of petroleum product the prices of petroleum
sold in gasoline stations in products shall be
prominently installed price immediately acted upon by
display boards with the DOE-DOJ Task Force in
backgrounds preferably accordance with Section 17
conforming to the color of this IRR. The said Task
coding scheme for the force shall determine within
product, such as: green for thirty (30) days the merits of
Unleaded Premium the report and shall initiate
Gasoline, red for Premium the necessary actions
Low Lead Gasoline, orange warranted under the
for Regular Gasoline, yellow circumstances.
for Diesel Fuel, and white
for Kerosene. In the case of A calculus of fear and pessimism, however, does not justify the remedy
LPG (which has no product petitioner seeks: that we now overturn a law enacted by Congress and
color), the price display approved by the Chief Executive. The Court must act on valid legal reasons
board may be light blue in that will explain why we should interfere with vital legislation. 34 To strike

491
down a provision of law we need a clear showing that what the Constitution
prohibits, the statute has allowed to be done. 35 Since there is no clear
showing that Section 19 of R.A. 8479 has violated the constitutional
prohibition against monopolies and combinations in restraint of trade, I vote
that the present petition be DISMISSED.

492
493
The Anti-Trust Act of July 2, 1890, c. 647, 26 Stat. 209, should be construed
in the light of reason; and, as so construed, it prohibits all contracts and
combination which amount to an unreasonable or undue restraint of trade in
interstate commerce.

The combination of the defendants in this case is an unreasonable and


undue restraint of trade in petroleum and its products moving in interstate
commerce, and falls within the prohibitions of the act as so construed.

Where one of the defendants in a suit, brought by the Government in a


Circuit Court of the United States under the authority of 4 of the Anti-Trust
Act of July 2, 1890, is within the district, the court, under the authority of 5
of that act, can take jurisdiction and order notice to be served upon the
U.S. Supreme Court nonresident defendants.

Standard Oil Co. of New Jersey v. United States, 221 U.S. Allegations as to facts occurring prior to the passage of the Anti-Trust Act
1 (1910) may be considered solely to throw light on acts done after the passage of the
act.
Standard Oil Co. of New Jersey v. United States
Page 221 U. S. 2
Argued March 14, 15, 16, 1910
The debates in Congress on the Anti-Trust Act of 1890 show that one of the
Restored to docket for reargument April 11, 1910 influences leading to the enactment of the statute was doubt as to whether
there is a common law of the United States governing the making of
Reargued January 12, 13, 16, 17, 1911 contracts in restraint of trade and the creation and maintenance of
monopolies in the absence of legislation.
Decided May 15, 1911
While debates of the body enacting it may not be used as means for
221 U.S. 1 interpreting a statute, they may be resorted to as a means of ascertaining the
conditions under which it was enacted.
APPEAL FROM THE CIRCUIT COURT OF THE UNITED STATES
The terms "restraint of trade," and "attempts to monopolize," as used in the
FOR THE EASTERN DISTRICT OF MISSOURI Anti-Trust Act, took their origin in the common law, and were familiar in the
law of this country prior to and at the time of the adoption of the act, and their
meaning should be sought from the conceptions of both English and
Syllabus
American law prior to the passage of the act.

494
The original doctrine that all contracts in restraint of trade were illegal was whether resulting from combinations or otherwise, which do not unduly
long since so modified in the interest of freedom of individuals to contract that restrain interstate or foreign commerce, but to protect that commerce from
the contract was valid if the resulting restraint was only partial in its contracts or combinations by methods, whether old or new, which would
operation, and was otherwise reasonable. constitute an interference with, or an undue restraint upon, it.

The early struggle in England against the power to create monopolies The Anti-Trust Act contemplated and required a standard of interpretation,
resulted in establishing that those institutions were incompatible with the and it was intended that the standard of reason which had been applied at
English Constitution. the common law should be applied in determining whether particular acts
were within its prohibitions.
At common law, monopolies were unlawful because of their restriction upon
individual freedom of contract and their injury to the public and at common The word "person" in 2 of the Anti-Trust Act, as construed by reference to
law, and contracts creating the same evils were brought within the prohibition 8 thereof, implies a corporation as well as an individual.
as impeding the due course of, or being in restraint of, trade.
The commerce referred to by the words "any part" in 2 of the Antitrust Act,
At the time of the passage of the Anti-Trust Act, the English rule was that the as construed in the light of the manifest purpose of that act, includes
individual was free to contract and to abstain from contracting and to geographically any part of the United States and also any of the classes of
exercise every reasonable right in regard thereto, except only as he was things forming a part of interstate or foreign commerce.
restricted from voluntarily and unreasonably or for wrongful purposes
restraining his right to carry on his trade. Mogul Steamship Co. v. The words "to monopolize" and "monopolize" as used in 2 of the Anti-Trust
McGregor, 1892, A.C. 25. Act reach every act bringing about the prohibited result.

A decision of the House of Lords, although announced after an event, may Freedom to contract is the essence of freedom from undue restraint on the
serve reflexly to show the state of the law in England at the time of such right to contract.
event.
In prior cases where general language has been used, to the effect that
This country has followed the line of development of the law of England, and reason could not be resorted to in determining whether a particular case was
the public policy has been to prohibit, or treat as illegal, contracts, or acts within the prohibitions of the Anti-Trust Act, the unreasonableness of the acts
entered into with intent to wrong the public and which unreasonably restrict under consideration was pointed out, and those cases are only authoritative
competitive conditions, limit the right of individuals, restrain the free flow of by the certitude that the rule of reason was applied; United States v. Trans-
commerce, or bring about public evils such as the enhancement of prices. Missouri Freight Association, 166 U. S. 290, and United States v. Joint Traffic
Association, 171 U. S. 505, limited and qualified so far as they conflict with
Page 221 U. S. 3 the construction now given to the Anti-Trust Act of 1890.

The Anti-Trust Act of 1890 was enacted in the light of the then existing The application of the Anti-Trust Act to combinations involving the production
practical conception of the law against restraint of trade, and the intent of of commodities within the States does not so extend the power of Congress
Congress was not to restrain the right to make and enforce contracts,

495
to subjects dehors its authority as to render the statute means whatever, and a dissolution of the offending combination should not
unconstitutional. United States v. E. C. Knight Co., 156 U. S. 1, distinguished. deprive the constituents of the right to live under the law, but should compel
them to obey it.
The Anti-Trust Act generically enumerates the character of the acts prohibited
and the wrongs which it intends to prevent, and is susceptible of being In determining the remedy against an unlawful combination, the court must
enforced without any judicial exertion of legislative power. consider the result, and not inflict serious injury on the public by causing a
cessation of interstate commerce in a necessary commodity.
The unification of power and control over a commodity such as petroleum
173 Fed. Rep. 177, modified and affirmed.
Page 221 U. S. 4
The facts, which involve the construction of the Sherman Anti-Trust Act of
and its products by combining in one corporation the stocks of many other July 2, 1890, and whether defendants had violated its provisions, are stated
corporations aggregating a vast capital gives rise, of itself, to the prima in the opinion.
facie presumption of an intent and purpose to dominate the industry
connected with, and gain perpetual control of the movement of, that Page 221 U. S. 30
commodity and its products in the channels of interstate commerce in
violation of the Anti-Trust Act of 1890, and that presumption is made MR. CHIEF JUSTICE WHITE delivered the opinion of the court.
conclusive by proof of specific acts such as those in the record of this case.
The Standard Oil Company of New Jersey and 33 other corporations, John
The fact that a combination over the products of a commodity such as D. Rockefeller, William Rockefeller, and five other individual defendants
petroleum does not include the crude article itself does not take the prosecute this appeal to reverse a decree of the court below. Such decree
combination outside of the Anti-Trust Act when it appears that the was entered upon a bill filed by the United States under authority of 4 of the
monopolization of the manufactured products necessarily controls the crude act of July 2, 1890, c. 647, p. 209, known as the Anti-Trust Act, and had for
article. its object the enforcement of the provisions of that act. The record is
inordinately voluminous, consisting of twenty-three volumes of printed matter,
Penalties which are not authorized by the law cannot be inflicted by judicial aggregating about twelve thousand pages, containing a vast amount of
authority. confusing and conflicting testimony

The remedy to be administered in case of a combination violating the Anti- Page 221 U. S. 31
Trust Act is two-fold: first, to forbid the continuance of the prohibited act, and
second, to so dissolve the combination as to neutralize the force of the relating to innumerable, complex and varied business transactions, extending
unlawful power. over a period of nearly forty years. In an effort to pave the way to reach the
subjects which we are called upon to consider, we propose at the outset,
The constituents of an unlawful combination under the Anti-Trust Act should following the order of the bill, to give the merest possible outline of its
not be deprived of power to make normal and lawful contracts, but should be contents, to summarize the answer, to indicate the course of the trial, and
restrained from continuing or recreating the unlawful combination by any point out briefly the decision below rendered.

496
The bill and exhibits, covering one hundred and seventy pages of the printed To establish this charge, it was averred that John D. and William Rockefeller
record, was filed on November 15, 1906. Corporations known as Standard and several other named individuals, who, prior to 1870, composed three
Oil Company of New Jersey, Standard Oil Company of California, Standard separate partnerships engaged in the business of refining crude oil and
Oil Company of Indiana, Standard Oil Company of Iowa, Standard Oil shipping its products in interstate commerce, organized in the year 1870 a
Company of Kansas, Standard Oil Company of Kentucky, Standard Oil corporation known as the Standard Oil Company of Ohio and transferred to
Company of Nebraska, Standard Oil Company of New York, Standard Oil that company the business of the said partnerships, the members thereof
Company of Ohio, and sixty-two other corporations and partnerships, as also becoming, in proportion to their prior ownership, stockholders in the
seven individuals were named as defendants. The bill was divided into thirty corporation. It was averred that the other individual defendants soon
numbered sections, and sought relief upon the theory that the various afterwards became participants in the illegal combination and either
defendants were engaged in conspiring transferred property to the corporation or to individuals to be held for the
benefit of all parties in interest in proportion to their respective interests in the
"to restrain the trade and commerce in petroleum, commonly called 'crude combination; that is, in proportion to their stock ownership in the Standard Oil
oil,' in refined oil, and in the other products of petroleum, among the several Company of Ohio. By the means thus stated, it was charged that, by the year
States and Territories of the United States and the District of Columbia and 1872, the combination had acquired substantially all but three or four of the
with foreign nations, and to monopolize the said commerce." thirty-five or forty oil refineries located in Cleveland, Ohio. By reason of the
power thus obtained and in further execution of the intent and purpose to
The conspiracy was alleged to have been formed in or about the year 1870 restrain trade and to monopolize the commerce, interstate as well as
by three of the individual defendants, viz: John D. Rockefeller, William intrastate, in petroleum and its products, the bill alleged that the combination
Rockefeller, and Henry M. Flagler. The detailed averments concerning the and its members
alleged conspiracy were arranged with reference to three periods, the first
from 1870 to 1882, the second from 1882 to 1899, and the third from 1899 to Page 221 U. S. 33
the time of the filing of the bill.
obtained large preferential rates and rebates in many and devious ways over
The general charge concerning the period from 1870 to 1882 was as follows: their competitors from various railroad companies, and that, by means of the
advantage thus obtained, many, if not virtually all, competitors were forced
Page 221 U. S. 32 either to become members of the combination or were driven out of
business, and thus, it was alleged, during the period in question, the
following results were brought about: a. that the combination, in addition to
"That during said first period, the said individual defendants, in connection
the refineries in Cleveland which it had acquired as previously stated, and
with the Standard Oil Company of Ohio, purchased and obtained interests
which it had either dismantled to limit production or continued to operate,
through stock ownership and otherwise in, and entered into agreements with,
also from time to time acquired a large number of refineries of crude
various persons, firms, corporations, and limited partnerships engaged in
petroleum, situated in New York, Pennsylvania, Ohio and elsewhere. The
purchasing, shipping, refining, and selling petroleum and its products among
properties thus acquired, like those previously obtained, although belonging
the various States for the purpose of fixing the price of crude and refined oil
to and being held for the benefit of the combination, were ostensibly
and the products thereof, limiting the production thereof, and controlling the
divergently controlled, some of them being put in the name of the Standard
transportation therein, and thereby restraining trade and commerce among
Oil Company of Ohio, some in the name of corporations or limited
the several States, and monopolizing the said commerce."
partnerships affiliated therewith, or some being left in the name of the original

497
owners, who had become stockholders in the Standard Oil Company of Ohio, The trust agreement thus referred to was set out in the bill. It was made in
and thus members of the alleged illegal combination. b. That the combination January, 1882. By its terms, the stock of forty corporations, including the
had obtained control of the pipelines available for transporting oil from the oil Standard Oil Company of Ohio, and a large quantity of various properties
fields to the refineries in Cleveland, Pittsburgh, Titusville, Philadelphia, New which had been previously acquired by the alleged combination and which
York and New Jersey. c. That the combination during the period named had was held in diverse forms, as we have previously indicated, for the benefit of
obtained a complete mastery over the oil industry, controlling 90 percent of the members of the combination, was vested in the trustees and their
the business of producing, shipping, refining and selling petroleum and its successors, "to be held for all parties in interest jointly." In the body of the
products, and thus was able to fix the price of crude and refined petroleum trust agreement was contained a list of the various individuals and
and to restrain and monopolize all interstate commerce in those products. corporations and limited partnerships whose stockholders and members, or a
portion thereof, became parties to the agreement. This list is in the margin.
The averments bearing upon the second period (1882 to 1899) had relation [Footnote 1]
to the claim:
Page 221 U. S. 35
"That, during the said second period of conspiracy, the defendants entered
into a contract and trust agreement The agreement made provision for the method of controlling and managing
the property by the trustees, for the formation of additional manufacturing,
Page 221 U. S. 34 etc., corporations

by which various independent firms, corporations, limited partnerships and Page 221 U. S. 36
individuals engaged in purchasing, transporting, refining, shipping and selling
oil and the products thereof among the various States turned over the in various States, and the trust, unless terminated by a mode specified, was
management of their said business, corporations and limited partnerships to to continue "during the lives of the survivors and survivor of the trustees
nine trustees, composed chiefly of certain individuals defendant herein, named in the agreement and for twenty-one years thereafter." The
which said trust agreement was in restraint of trade and commerce and in agreement provided for the issue of Standard Oil Trust certificates to
violation of law, as hereinafter more particularly alleged." represent the interest arising under the trust in the properties affected by the
trust, which, of course, in view of the provisions of the agreement and the
subject to which it related, caused the interest in the certificates to be
coincident with and the exact representative of the interest in the
combination, that is, in the Standard Oil Company of Ohio. Soon afterwards,
it was alleged, the trustees organized the Standard Oil Company of New
Jersey and the Standard Oil Company of New York, the former having a
capital stock of $3,000,000 and the latter a capital stock of $5,000,000,
subsequently increased to $10,000,000 and $15,000,000, respectively. The
bill alleged

498
"that, pursuant to said trust agreement, the said trustees caused to be Page 221 U. S. 40
transferred to themselves the stocks of all corporations and limited
partnerships named in said trust agreement, and caused various of the was in restraint of trade and amounted to the creation of an unlawful
individuals and copartnerships, who owned apparently independent refineries monopoly. It was alleged that shortly after this decision, seemingly for the
and other properties employed in the business of refining and transporting purpose of complying therewith, voluntary proceedings were had apparently
and selling oil in and among said various States and Territories to dissolve the trust, but that these proceedings were a subterfuge and a
sham, because they simply amounted to a transfer of the stock held by the
Page 221 U. S. 37 trust in 64 of the companies which it controlled to some of the remaining 20
companies, it having controlled before the decree 84 in all, thereby, while
of the United States as aforesaid, to transfer their property situated in said seemingly in part giving up its dominion, yet in reality preserving the same by
several States to the respective Standard Oil Companies of said States of means of the control of the companies as to which it had retained complete
New York, New Jersey, Pennsylvania and Ohio, and other corporations authority. It was charged that especially was this the case as the stock in the
organized or acquired by said trustees from time to time. . . ." companies selected for transfer was virtually owned by the nine trustees or
the members of their immediate families or associates. The bill further
For the stocks and property so acquired, the trustees issued trust certificates. alleged that, in 1897, the Attorney-General of Ohio instituted contempt
It was alleged that, in 1888, the trustees proceedings in the quo warranto case based upon the claim that the trust
had not been dissolved as required by the decree in that case. About the
same time also, proceedings in quo warranto were commenced to forfeit the
"unlawfully controlled the stock and ownership of various corporations and
charter of a pipeline known as the Buckeye Pipe Line Company, an
limited partnerships engaged in such purchase and transportation, refining,
selling, and shipping of oil,"
Page 221 U. S. 41
as per a list which is excerpted in the margin. [Footnote 2]
Ohio corporation, whose stock, it was alleged, was owned by the members of
the combination, on the ground of its connection with the trust which had
Page 221 U. S. 38
been held to be illegal.

The bill charged that, during the second period, quo warranto proceedings
The result of these proceedings, the bill charged, caused a resort to the
were commenced against the Standard Oil Company of Ohio, which resulted
alleged wrongful acts asserted to have been committed during the third
in the entry by the Supreme Court of Ohio, on March 2, 1892, of a decree
period, as follows:

Page 221 U. S. 39
"That during the third period of said conspiracy and in pursuance thereof, the
said individual defendants operated through the Standard Oil Company of
adjudging the trust agreement to be void, not only because the Standard Oil New Jersey, as a holding corporation, which corporation obtained and
Company of Ohio was a party to the same, but also because the agreement, acquired the majority of the stocks of the various corporations engaged in
in and of itself, purchasing, transporting, refining, shipping, and selling oil into and among
the various States and Territories of the United States and the District of

499
Columbia and with foreign nations, and thereby managed and controlled the $97,250,000. The bill contained allegations referring to the development of
same, in violation of the laws of the United States, as hereinafter more new oil fields, for example, in California, southeastern Kansas, northern
particularly alleged." Indian Territory, and northern Oklahoma, and made reference to the building
or otherwise acquiring by the combination of refineries and pipelines in the
It was alleged that, in or about the month of January, 1899, the individual new fields for the purpose of restraining and monopolizing the interstate
defendants caused the charter of the Standard Oil Company of New Jersey trade in petroleum and its products.
to be amended
Reiterating in substance the averments that both the Standard Oil Trust from
"so that the business and objects of said company were stated as follows, to- 1882 to 1899 and the Standard Oil Company of New Jersey since 1899 had
wit: 'To do all kinds of mining, manufacturing, and trading business; monopolized and restrained interstate commerce in petroleum and its
transporting goods and merchandise by land or water in any manner; to buy, products, the bill at great length additionally set forth various means by
sell, lease, and improve land; build houses, structures, vessels, cars, which, during the second and third periods, in addition to the effect
wharves, docks, and piers; to lay and operate pipelines; to erect lines for occasioned by the combination of alleged previously independent concerns,
conducting electricity; to enter into and carry out contracts of every kind the monopoly and restraint complained of was continued. Without attempting
pertaining to its business; to acquire, use, sell, and grant licenses under to follow the elaborate averments on these subjects spread over fifty-seven
patent rights; to purchase or otherwise acquire, hold, sell, assign, and pages of the printed record, it suffices to say that such averments may
transfer shares of capital stock and bonds or other evidences of properly be grouped under the following heads: rebates, preferences and
indebtedness of corporations, and to exercise all the privileges of ownership, other discriminatory practises in favor of the combination by railroad
including voting upon the stock so held; to carry on its business and have companies; restraint and monopolization by control of pipelines, and unfair
offices and agencies therefor in all parts of the world, and practises against competing

Page 221 U. S. 42 Page 221 U. S. 43

to hold, purchase, mortgage, and convey real estate and personal property pipelines; contracts with competitors in restraint of trade; unfair methods of
outside the State of New Jersey.'" competition, such as local price-cutting at the points where necessary to
suppress competition; espionage of the business of competitors, the
The capital stock of the company -- which, since March 19, 1892, had been operation of bogus independent companies, and payment of rebates on oil,
$10,000,000 -- was increased to $110,000,000, and the individual with the like intent; the division of the United States into districts and the
defendants, as theretofore, continued to be a majority of the board of limiting of the operations of the various subsidiary corporations as to such
directors. districts so that competition in the sale of petroleum products between such
corporations had been entirely eliminated and destroyed, and, finally,
reference was made to what was alleged to be the "enormous and
Without going into detail, it suffices to say that it was alleged in the bill that,
unreasonable profits" earned by the Standard Oil Trust and the Standard Oil
shortly after these proceedings, the trust came to an end, the stock of the
Company as a result of the alleged monopoly, which presumably was
various corporations which had been controlled by it being transferred by its
averred as a means of reflexly inferring the scope and power acquired by the
holders to the Standard Oil Company of New Jersey, which corporation
alleged combination.
issued therefor certificates of its common stock to the amount of

500
Coming to the prayer of the bill, it suffices to say that, in general terms, the "It is sufficient to say that, whilst admitting many of the alleged acquisitions of
substantial relief asked was, first, that the combination in restraint of property, the formation of the so-called trust of 1882, its dissolution in 1892,
interstate trade and commerce and which had monopolized the same, as and the acquisition by the Standard Oil Company of New Jersey of the
alleged in the bill, be found to have existence, and that the parties thereto be stocks of the various corporations in 1899, they deny all the allegations
perpetually enjoined from doing any further act to give effect to it; second, respecting combinations or conspiracies to restrain or monopolize the oil
that the transfer of the stocks of the various corporations to the Standard Oil trade, and particularly that the so-called trust of 1882, or the acquisition of
Company of New Jersey, as alleged in the bill, be held to be in violation of the shares of the defendant companies by the Standard Oil Company of New
the first and second sections of the Antitrust Act, and that the Standard Oil Jersey in 1899, was a combination of independent or competing concerns or
Company of New Jersey be enjoined and restrained from in any manner corporations. The averments of the petition respecting the means adopted to
continuing to exert control over the subsidiary corporations by means of monopolize the oil trade are traversed either by a denial of the acts alleged
ownership of said stock or otherwise; third, that specific relief by injunction be or of their purpose, intent or effect."
awarded against further violation of the statute by any of the acts specifically
complained of in the bill. There was also a prayer for general relief. On June 24, 1907, the cause being at issue, a special examiner was
appointed to take the evidence, and his report was filed March 22, 1909. It
Of the numerous defendants named in the bill, the Waters-Pierce Oil was heard on April 5
Company was the only resident of the
Page 221 U. S. 45
Page 221 U. S. 44
to 10, 1909, under the expediting act of February 11, 1903, before a Circuit
district in which the suit was commenced and the only defendant served with Court consisting of four judges.
process therein. Contemporaneous with the filing of the bill, the court made
an order, under 5 of the Anti-Trust Act, for the service of process upon all The court decided in favor of the United States. In the opinion delivered, all
the other defendants, wherever they could be found. Thereafter, the various the multitude of acts of wrongdoing charged in the bill were put aside, insofar
defendants unsuccessfully moved to vacate the order for service on as they were alleged to have been committed prior to the passage of the
nonresident defendants or filed pleas to the jurisdiction. Joint exceptions Anti-Trust Act, "except as evidence of their (the defendants') purpose, of their
were likewise unsuccessfully filed, upon the ground of impertinence, to many continuing conduct and of its effect." (173 Fed.Rep. 177.)
of the averments of the bill of complaint, particularly those which related to
acts alleged to have been done by the combination prior to the passage of By the decree which was entered, it was adjudged that the combining of the
the Anti-Trust Act and prior to the year 1899. stocks of various companies in the hands of the Standard Oil Company of
New Jersey in 1899 constituted a combination in restraint of trade and also
Certain of the defendants filed separate answers, and a joint answer was an attempt to monopolize and a monopolization under 2 of the Anti-Trust
filed on behalf of the Standard Oil Company of New Jersey and numerous of Act. The decree was against seven individual defendants, the Standard Oil
the other defendants. The scope of the answers will be adequately indicated Company of New Jersey, thirty-six domestic companies, and one foreign
by quoting a summary on the subject made in the brief for the appellants. company which the Standard Oil Company of New Jersey controls by stock
ownership; these 38 corporate defendants being held to be parties to the
combination found to exist. [Footnote 3]

501
The bill was dismissed as to all other corporate defendants, 33 in number, it Second. The overruling of the exceptions taken to so much of the bill as
being adjudged by 3 of the decree that they "have not been proved to be counted upon facts occurring prior to the passage of the Anti-Trust Act --
engaged in the operation or carrying out of the combination." [Footnote 4] whatever may be the view as an original question of the duty to restrict the
controversy to a much narrower area than that propounded by the bill -- we
Page 221 U. S. 46 think by no possibility in the present stage of the case can the action of the
court be treated as prejudicial error justifying reversal. We say this because
The Standard Oil Company of New Jersey was enjoined from voting the the court, as we shall do, gave no weight to the testimony adduced under the
stocks or exerting any control over the said 37 subsidiary companies, and the averments complained of except insofar as it tended to throw light upon the
subsidiary companies were enjoined from paying any dividends as to the acts done after the
Standard Oil Company or permitting it to exercise any control over them by
virtue of the stock ownership or power acquired by means of the Page 221 U. S. 47
combination. The individuals and corporations were also enjoined from
entering into or carrying into effect any like combination which would evade passage of the Anti-Trust Act and the results of which it was charged were
the decree. Further, the individual defendants, the Standard Oil Company, being participated in and enjoyed by the alleged combination at the time of
and the 37 subsidiary corporations were enjoined from engaging or the filing of the bill.
continuing in interstate commerce in petroleum or its products during the
continuance of the illegal combination. We are thus brought face to face with the merits of the controversy.

At the outset a question of jurisdiction requires consideration, and we shall, Both as to the law and as to the facts, the opposing contentions pressed in
also, as a preliminary, dispose of another question, to the end that our the argument are numerous, and, in all their aspects, are so irreconcilable
attention may be completely concentrated upon the merits of the controversy that it is difficult to reduce them to some fundamental generalization which,
when we come to consider them. by being disposed of, would decide them all. For instance, as to the law.
While both sides agree that the determination of the controversy rests upon
First. We are of opinion that, in consequence of the presence within the the correct construction and application of the first and second sections of
district of the Waters-Pierce Oil Company, the court, under the authority of the Anti-Trust Act, yet the views as to the meaning of the act are as wide
5 of the Anti-Trust Act, rightly took jurisdiction over the cause and properly apart as the poles, since there is no real point of agreement on any view of
ordered notice to be served upon the nonresident defendants. the act. And this also is the case as to the scope and effect of authorities
relied upon, even although, in some instances, one and the same authority is
asserted to be controlling.

So also is it as to the facts. Thus, on the one hand, with relentless pertinacity
and minuteness of analysis, it is insisted that the facts establish that the
assailed combination took its birth in a purpose to unlawfully acquire wealth
by oppressing the public and destroying the just rights of others, and that its
entire career exemplifies an inexorable carrying out of such wrongful intents,
since, it is asserted, the pathway of the combination, from the beginning to

502
the time of the filing of the bill, is marked with constant proofs of wrong every aspect is controlled by a correct conception of the meaning of the first
inflicted upon the public, and is strewn with the wrecks resulting from and second sections of the Anti-Trust Act. We shall
crushing out, without regard to law, the individual rights of others. Indeed, so
conclusive, it is urged, is the proof on these subjects that it is asserted that Page 221 U. S. 49
the existence of the principal corporate defendant -- the Standard Oil
Company of New Jersey -- with the vast accumulation of property which it therefore -- departing from what otherwise would be the natural order of
owns or controls, because of its infinite potency analysis -- make this one point of harmony the initial basis of our examination
of the contentions, relying upon the conception that, by doing so, some
Page 221 U. S. 48 harmonious resonance may result adequate to dominate and control the
discord with which the case abounds. That is to say, we shall first come to
for harm and the dangerous example which its continued existence affords, consider the meaning of the first and second sections of the Anti-Trust Act by
is an open and enduring menace to all freedom of trade, and is a byword and the text, and, after discerning what by that process appears to be its true
reproach to modern economic methods. On the other hand, in a powerful meaning, we shall proceed to consider the respective contentions of the
analysis of the facts, it is insisted that they demonstrate that the origin and parties concerning the act, the strength or weakness of those contentions, as
development of the vast business which the defendants control was but the well as the accuracy of the meaning of the act as deduced from the text in
result of lawful competitive methods, guided by economic genius of the the light of the prior decisions of this court concerning it. When we have done
highest order, sustained by courage, by a keen insight into commercial this, we shall then approach the facts. Following this course, we shall make
situations, resulting in the acquisition of great wealth, but at the same time our investigation under four separate headings: First. The text of the first and
serving to stimulate and increase production, to widely extend the distribution second sections of the act originally considered, and its meaning in the light
of the products of petroleum at a cost largely below that which would have of the common law and the law of this country at the time of its adoption.
otherwise prevailed, thus proving to be, at one and the same time, a Second. The contentions of the parties concerning the act, and the scope
benefaction to the general public as well as of enormous advantage to and effect of the decisions of this court upon which they rely. Third. The
individuals. It is not denied that, in the enormous volume of proof contained application of the statute to facts, and, Fourth. The remedy, if any, to be
in the record in the period of almost a lifetime to which that proof is afforded as the result of such application.
addressed, there may be found acts of wrongdoing, but the insistence is that
they were rather the exception than the rule, and, in most cases, were either First. The text of the act and its meaning.
the result of too great individual zeal in the keen rivalries of business or of the
methods and habits of dealing which, even if wrong, were commonly We quote the text of the first and second sections of the act, as follows:
practised at the time. And, to discover and state the truth concerning these
contentions, both arguments call for the analysis and weighing, as we have
"SECTION 1. Every contract, combination in the form of trust or otherwise, or
said at the outset, of a jungle of conflicting testimony covering a period of
conspiracy, in restraint of trade or commerce, among the several States, or
forty years, a duty difficult to rightly perform and, even if satisfactorily
with foreign nations, is hereby declared to be illegal. Every person who shall
accomplished, almost impossible to state with any reasonable regard to
make any such contract, or engage in any such combination or conspiracy,
brevity.
shall be deemed guilty of a misdemeanor, and, on conviction thereof, shall be
punished by fine not exceeding five thousand dollars, or by
Duly appreciating the situation just stated, it is certain that only one point of
concord between the parties is discernable, which is that the controversy in

503
Page 221 U. S. 50 is concerned. It is certain that those terms, at least in their rudimentary
meaning, took their origin in the common law, and were also familiar in the
imprisonment not exceeding one year, or by both said punishments, in the law of this country prior to and at the time of the adoption of the act in
discretion of the court." question.

"SEC. 2. Every person who shall monopolize, or attempt to monopolize, or We shall endeavor then, first to seek their meaning not by indulging in an
combine or conspire with any other person or persons, to monopolize any elaborate and learned analysis of the English law and of the law of this
part of the trade or commerce among the several States, or with foreign country, but by making a very brief reference to the elementary and
nations, shall be deemed guilty of a misdemeanor, and, on conviction indisputable conceptions of both the English and American law on the subject
thereof, shall be punished by fine not exceeding five thousand dollars, or by prior to the passage of the Anti-Trust Act.
imprisonment not exceeding one year, or by both said punishments, in the
discretion of the court." a. It is certain that, at a very remote period, the words "contract in restraint of
trade" in England came to refer to some voluntary restraint put by contract by
The debates show that doubt as to whether there was a common law of the an individual on his right to carry on his trade or calling. Originally all such
United States which governed the subject in the absence of legislation was contracts were considered to be illegal, because it was deemed they were
among the influences leading to the passage of the act. They conclusively injurious to the public, as well as to the individuals who made them. In the
show, however, that the main cause which led to the legislation was the interest of the freedom of individuals to contract, this doctrine was modified
thought that it was required by the economic condition of the times, that is, so that it was only when a restraint by contract was so general as to be
the vast accumulation of wealth in the hands of corporations and individuals, coterminous with the kingdom that it was treated as void. That is to say, if the
the enormous development of corporate organization, the facility for restraint was partial in its operation and was otherwise reasonable, the
combination which such organizations afforded, the fact that the facility was contract was held to be valid.
being used, and that combinations known as trusts were being multiplied,
and the widespread impression that their power had been and would be b. Monopolies were defined by Lord Coke as follows:
exerted to oppress individuals and injure the public generally. Although
debates may not be used as a means for interpreting a statute (United "A monopoly is an institution, or allowance by the king by his grant,
States v. Trans-Missouri Freight Association, 166 U. S. 318, and cases cited) commission, or otherwise to any person or persons, bodies politic or
that rule, in the nature of things, is not violated by resorting to debates as a corporate, of or for the sole buying, selling, making, working, or using of
means of ascertaining the environment at the time of the enactment of a anything, whereby any person or persons, bodies politic or corporate, are
particular law, that is, the history of the period when it was adopted. sought to be restrained of any freedom or liberty that they had before, or
hindered in their lawful trade."
There can be no doubt that the sole subject with which the first section deals
is restraint of trade as therein contemplated, and that the attempt to (3 Inst. 181, c. 85.)
monopolize and monopolization is the subject with which the second section
Hawkins thus defined them:
Page 221 U. S. 51

504
"A monopoly is an allowance by the king to a particular person or persons of Page 221 U. S. 53
the sole buying, selling, making,
This is illustrated by the definition of engrossing found in the statute, 5 and 6
Page 221 U. S. 52 Edw. VI, ch. 14, as follows:

working, or using of anything whereby the subject in general is restrained "Whatsoever person or persons . . . shall engross or get into his or their
from the freedom of manufacturing or trading which he had before." hands by buying, contracting, or promise-taking, other than by demise, grant,
or lease of land, or tithe, any corn growing in the fields, or any other corn or
(Hawk. P. C. bk. 1, c. 29.) grain, butter, cheese, fish, or other dead victual, whatsoever, within the realm
of England, to the intent to sell the same again, shall be accepted, repute,
The frequent granting of monopolies and the struggle which led to a denial of and taken an unlawful engrosser or engrossers."
the power to create them, that is to say, to the establishment that they were
incompatible with the English constitution, is known to all, and need not be As, by the statutes providing against engrossing, the quantity engrossed was
reviewed. The evils which led to the public outcry against monopolies and to not required to be the whole or a proximate part of the whole of an article, it
the final denial of the power to make them may be thus summarily stated: 1. is clear that there was a wide difference between monopoly and engrossing,
The power which the monopoly gave to the one who enjoyed it to fix the price etc. But, as the principal wrong which it was deemed would result from
and thereby injure the public; 2. The power which it engendered of enabling a monopoly, that is, an enhancement of the price, was the same wrong to
limitation on production; and, 3. The danger of deterioration in quality of the which it was thought the prohibited engrossment would give rise, it came to
monopolized article which it was deemed was the inevitable resultant of the pass that monopoly and engrossing were regarded as virtually one and the
monopolistic control over its production and sale. As monopoly as thus same thing. In other words, the prohibited act of engrossing, because of its
conceived embraced only a consequence arising from an exertion of inevitable accomplishment of one of the evils deemed to be engendered by
sovereign power, no express restrictions or prohibitions obtained against the monopoly, came to be referred to as being a monopoly or constituting an
creation by an individual of a monopoly as such. But as it was considered, at attempt to monopolize. Thus, Pollexfen, in his argument in East India
least so far as the necessaries of life were concerned, that individuals, by the Company v. Sandys, Skin. 165, 169, said:
abuse of their right to contract, might be able to usurp the power arbitrarily to
enhance prices, one of the wrongs arising from monopoly, it came to be that "By common law, he said that trade is free, and for that cited 3 Inst. 81; F.B.
laws were passed relating to offenses such as forestalling, regrating and 65; 1 Roll. 4; that the common law is as much against 'monopoly' as
engrossing by which prohibitions were placed upon the power of individuals 'engrossing;' and that they differ only, that a 'monopoly' is by patent from the
to deal under such circumstances and conditions as, according to the king, the other is by the act of the subject between party and party; but that
conception of the times, created a presumption that the dealings were not the mischiefs are the same from both, and there is the same law against
simply the honest exertion of one's right to contract for his own benefit both. Moore, 673; 11 Rep. 84. The sole trade of anything is 'engrossing' ex
unaccompanied by a wrongful motive to injure others, but were the rei natura, for whosoever hath the sole trade of buying and selling hath
consequence of a contract or course of dealing of such a character as to give 'engrossed' that trade, and whosoever
rise to the presumption of an intent to injure others through the means, for
instance, of a monopolistic increase of prices. Page 221 U. S. 54

505
hath the sole trade to any country hath the sole trade of buying and selling was void. And that, at common law, the evils consequent upon engrossing,
the produce of that country, at his own price, which is an 'engrossing.'" etc., caused those things to be treated as coming within monopoly, and
sometimes to be called monopoly, and the same considerations caused
And, by operation of the mental process which led to considering as a monopoly, because of its operation and effect, to be brought within and
monopoly acts which, although they did not constitute a monopoly, were spoken of generally as impeding the due course of, or being in restraint of,
thought to produce some of its baneful effects, so also because of the trade.
impediment or burden to the due course of trade which they produced, such
acts came to be referred to as in restraint of trade. This is shown by my Lord From the development of more accurate economic conceptions and the
Coke's definition of monopoly as being changes in conditions of society, it came to be recognized that the acts
prohibited by the engrossing, forestalling, etc., statutes did not have the
"an institution or allowance . . . whereby any person or persons, bodies politic harmful tendency which they were presumed to have when the legislation
or corporate, are sought to be restrained of any freedom or liberty that they concerning them was enacted, and therefore did not justify the presumption
had before or hindered in their lawful trade." which had previously been deduced from them, but, on the contrary, such
acts tended to fructify and develop trade. See the statutes of 12th George III,
It is illustrated also by the definition which Hawkins gives of monopoly ch. 71, enacted in 1772, and statute of 7 and 8 Victoria, ch. 24, enacted in
wherein it is said that the effect of monopoly is to restrain the citizen "from 1844, repealing the prohibitions against engrossing, forestalling, etc., upon
the freedom of manufacturing or trading which he had before." And see the express ground that the prohibited acts had come to be considered as
especially the opinion of Parker, C.J., in Mitchel v. Reynolds (1711), 1 P. favorable to the development of, and not in restraint of, trade. It is remarkable
Williams, 181, where a classification is made of monopoly which brings it that nowhere at common law can there be found a prohibition against the
generically within the description of restraint of trade. creation of monopoly by an individual. This would seem to manifest, either
consciously or intuitively, a profound conception as to the inevitable operation
of economic forces and the equipoise or balance in favor of the protection of
Generalizing these considerations, the situation is this: 1. That, by the
the rights of individuals which resulted. That is to say, as it was deemed that
common law, monopolies were unlawful because of their restriction upon
monopoly in the concrete could only arise from an act of sovereign power,
individual freedom of contract and their injury to the public. 2. That as to
and, such sovereign power being restrained, prohibitions as to individuals
necessaries of life, the freedom of the individual to deal was restricted where
were directed not against the creation of monopoly, but were only applied to
the nature and character of the dealing was such as to engender the
such acts in relation to particular subjects as to which it was deemed, if not
presumption of intent to bring about at least one of the injuries which it was
restrained, some of the consequences of monopoly might result. After all, this
deemed would result from monopoly, that is, an undue enhancement of price.
was but an instinctive recognition
3. That, to protect the freedom of contract of the individual not only in his own
interest, but principally in the interest of the common weal, a contract of an
individual by which he put an unreasonable restraint upon himself as to Page 221 U. S. 56
carrying on his trade or business
of the truisms that the course of trade could not be made free by obstructing
Page 221 U. S. 55 it, and that an individual's right to trade could not be protected by destroying
such right.

506
From the review just made, it clearly results that, outside of the restrictions trade. The dread of monopoly as an emanation of governmental power, while
resulting from the want of power in an individual to voluntarily and it passed at an early date out of mind in this country as a result of the
unreasonably restrain his right to carry on his trade or business, and outside structure of our Government, did not serve to assuage the fear as to the evil
of the want of right to restrain the free course of trade by contracts or acts consequences which might arise from the acts of individuals producing or
which implied a wrongful purpose, freedom to contract and to abstain from tending to produce the consequences of monopoly. It resulted that treating
contracting, and to exercise every reasonable right incident thereto, became such acts as we have said as amounting to monopoly, sometimes
the rule in the English law. The scope and effect of this freedom to trade and constitutional restrictions, again legislative enactments or judicial decisions,
contract is clearly shown by the decision in Mogul Steamship Co. v. served to enforce and illustrate the purpose to prevent the occurrence of the
McGregor (1892), A.C. 25. While it is true that the decision of the House of evils recognized in the mother country as consequent upon monopoly, by
Lords in the case in question was announced shortly after the passage of the providing against contracts or acts of individuals or combinations of
Anti-Trust Act, it serves reflexly to show the exact state of the law in England individuals or corporations deemed to be conducive to such results. To refer
at the time the Antitrust statute was enacted. to the constitutional or legislative provisions on the subject or many judicial
decisions which illustrate it would unnecessarily prolong this opinion. We
In this country also, the acts from which it was deemed there resulted a part, append in the margin a note to treatises, &c., wherein are contained
if not all, of the injurious consequences ascribed to monopoly came to be references to constitutional and statutory provisions and to numerous
referred to as a monopoly itself. In other words, here, as had been the case decisions, etc., relating to the subject. [Footnote 5]
in England, practical common sense caused attention to be concentrated not
upon the theoretically correct name to be given to the condition or acts which It will be found that, as modern conditions arose, the trend of legislation and
gave rise to a harmful result, but to the result itself, and to the remedying of judicial decision came more and more to adapt the recognized restrictions to
the evils which it produced. The statement just made is illustrated by an early new manifestations of conduct or of dealing which it was thought
statute of the Province of Massachusetts, that is, chap. 31 of the laws of
1778-1779, by which monopoly and forestalling were expressly treated as Page 221 U. S. 58
one and the same thing.
justified the inference of intent to do the wrongs which it had been the
It is also true that, while the principles concerning contracts in restraint of purpose to prevent from the beginning. The evolution is clearly pointed out
trade, that is, voluntary restraint put by a person on his right to pursue his in National Cotton Oil Co. v. Texas, 197 U. S. 115, and Shawnee Compress
calling, hence only operating subjectively, came generally to be recognized Co. v. Anderson, 209 U. S. 423; and, indeed, will be found to be illustrated in
various aspects by the decisions of this court which have been concerned
Page 221 U. S. 57 with the enforcement of the act we are now considering.

in accordance with the English rule, it came moreover to pass that contracts Without going into detail, and but very briefly surveying the whole field, it may
or acts which it was considered had a monopolistic tendency, especially be with accuracy said that the dread of enhancement of prices and of other
those which were thought to unduly diminish competition, and hence to wrongs which it was thought would flow from the undue limitation on
enhance prices -- in other words, to monopolize -- came also in a generic competitive conditions caused by contracts or other acts of individuals or
sense to be spoken of and treated, as they had been in England, as corporations led, as a matter of public policy, to the prohibition or treating as
restricting the due course of trade, and therefore as being in restraint of illegal all contracts or acts which were unreasonably restrictive of competitive

507
conditions, either from the nature or character of the contract or act or where In view of the common law and the law in this country as to restraint of trade,
the surrounding circumstances were such as to justify the conclusion that which we have reviewed, and the illuminating effect which that history must
they had not been entered into or performed with the legitimate purpose of have under the rule to which we have referred, we think it results:
reasonably forwarding personal interest and developing trade, but, on the
contrary, were of such a character as to give rise to the inference or a. That the context manifests that the statute was drawn in the light of the
presumption that they had been entered into or done with the intent to do existing practical conception of the law of restraint of trade, because it groups
wrong to the general public and to limit the right of individuals, thus as within that class not only contracts which were in restraint of trade in the
restraining the free flow of commerce and tending to bring about the evils, subjective sense, but all contracts or acts which theoretically were attempts
such as enhancement of prices, which were considered to be against public to monopolize, yet which, in practice, had come to be considered as in
policy. It is equally true to say that the survey of the legislation in this country restraint of trade in a broad sense.
on this subject from the beginning will show, depending as it did upon the
economic conceptions which obtained at the time when the legislation was b. That, in view of the many new forms of contracts and combinations which
adopted or judicial decision was rendered, that contracts or acts were at one were being evolved from existing economic conditions, it was deemed
time deemed to be of such a character as to justify the inference of wrongful essential by an all-embracing enumeration to make sure that no form of
intent which were, at another period, thought not to be contract or combination by which an undue restraint of

Page 221 U. S. 59 Page 221 U. S. 60

of that character. But this again, as we have seen, simply followed the line of interstate or foreign commerce was brought about could save such restraint
development of the law of England. from condemnation. The statute, under this view, evidenced the intent not to
restrain the right to make and enforce contracts, whether resulting from
Let us consider the language of the first and second sections, guided by the combination or otherwise, which did not unduly restrain interstate or foreign
principle that, where words are employed in a statute which had at the time a commerce, but to protect that commerce from being restrained by methods,
well known meaning at common law or in the law of this country, they are whether old or new, which would constitute an interference that is an undue
presumed to have been used in that sense unless the context compels to the restraint.
contrary. [Footnote 6]
c. And as the contracts or acts embraced in the provision were not expressly
As to the first section, the words to be interpreted are: defined, since the enumeration addressed itself simply to classes of acts,
those classes being broad enough to embrace every conceivable contract or
"Every contract, combination in the form of trust or otherwise, or conspiracy combination which could be made concerning trade or commerce or the
in restraint of trade or commerce . . . is hereby declared to be illegal." subjects of such commerce, and thus caused any act done by any of the
enumerated methods anywhere in the whole field of human activity to be
As there is no room for dispute that the statute was intended to formulate a illegal if in restraint of trade, it inevitably follows that the provision necessarily
rule for the regulation of interstate and foreign commerce, the question is called for the exercise of judgment which required that some standard should
what was the rule which it adopted? be resorted to for the purpose of determining whether the prohibitions
contained in the statute had or had not in any given ease been violated.

508
Thus, not specifying but indubitably contemplating and requiring a standard, monopolizing trade, that is, unduly restraining it by means of every contract,
it follows that it was intended that the standard of reason which had been combination, etc., the second section seeks, if possible, to make the
applied at the common law, and in this country, in dealing with subjects of the prohibitions of the act all the more complete and perfect by embracing all
character embraced by the statute, was intended to be the measure used for attempts to reach the end prohibited by the first section, that is, restraints of
the purpose of determining whether, in a given case, a particular act had or trade, by any attempt to monopolize, or monopolization thereof, even
had not brought about the wrong against which the statute provided. although the acts by which such results are attempted to be brought about or
are brought about be not embraced within the general enumeration of the
And a consideration of the text of the second section serves to establish that first section. And, of course, when the second section is thus harmonized
it was intended to supplement the first, and to make sure that, by no possible with and made as it
guise could the public policy embodied in the first section be frustrated or
evaded. The prohibitions of the second embrace Page 221 U. S. 62

Page 221 U. S. 61 was intended to be the complement of the first, it becomes obvious that the
criteria to be resorted to in any given case for the purpose of ascertaining
"Every person who shall monopolize or attempt to monopolize, or combine or whether violations of the section have been committed is the rule of reason,
conspire with any other person or persons, to monopolize any part of the guided by the established law and by the plain duty to enforce the
trade or commerce among the several states, or with foreign nations, . . ." prohibitions of the act, and thus the public policy which its restrictions were
obviously enacted to subserve. And it is worthy of observation, as we have
"By reference to the terms of 8, it is certain that the word person clearly previously remarked concerning the common law, that, although the statute,
implies a corporation, as well as an individual." by the comprehensiveness of the enumerations embodied in both the first
and second sections, makes it certain that its purpose was to prevent undue
restraints of every kind or nature, nevertheless, by the omission of any direct
The commerce referred to by the words "any part," construed in the light of
prohibition against monopoly in the concrete, it indicates a consciousness
the manifest purpose of the statute, has both a geographical and a
that the freedom of the individual right to contract, when not unduly or
distributive significance, that is, it includes any portion of the United States
improperly exercised, was the most efficient means for the prevention of
and anyone of the classes of things forming a part of interstate or foreign
monopoly, since the operation of the centrifugal and centripetal forces
commerce.
resulting from the right to freely contract was the means by which monopoly
would be inevitably prevented if no extraneous or sovereign power imposed it
Undoubtedly, the words "to monopolize" and "monopolize," as used in the and no right to make unlawful contracts having a monopolistic tendency were
section, reach every act bringing about the prohibited results. The ambiguity, permitted. In other words, that freedom to contract was the essence of
if any, is involved in determining what is intended by monopolize. But this freedom from undue restraint on the right to contract.
ambiguity is readily dispelled in the light of the previous history of the law of
restraint of trade to which we have referred, and the indication which it gives
Clear as it seems to us is the meaning of the provisions of the statute in the
of the practical evolution by which monopoly and the acts which produce the
light of the review which we have made, nevertheless, before definitively
same result as monopoly, that is, an undue restraint of the course of trade, all
applying that meaning, it behooves us to consider the contentions urged on
came to be spoken of as, and to be indeed synonymous with, restraint of
one side or the other concerning the meaning of the statute, which, if
trade. In other words, having, by the first section, forbidden all means of
maintained, would give to it, in some aspects a much wider, and, in every

509
view, at least a somewhat different, significance. And to do this brings us to of the act by precise definition, but, while clearly fixing a standard, that is, by
the second question, which, at the outset, we have stated it was our purpose defining the ulterior boundaries which could not be transgressed with
to consider and dispose of. impunity, to leave it to be determined by the light of reason, guided by the
principles of law and the duty to apply and enforce the public policy
Page 221 U. S. 63 embodied in the statute, in every given case whether any particular act or
contract was within the contemplation of the statute.
Second. The contentions of the parties as to the meaning of the statute and
the decisions of this court relied upon concerning those contentions. But, it is said, persuasive as these views may be, they may not be here
applied, because the previous decisions of this court have given to the
In substance, the propositions urged by the Government are reducible to this: statute a meaning which expressly excludes the construction which must
that the language of the statute embraces every contract, combination, etc., result from the reasoning stated. The cases are United States v. Freight
in restraint of trade, and hence its text leaves no room for the exercise of Association, 166 U. S. 290, and United States v. Joint Traffic
judgment, but simply imposes the plain duty of applying its prohibitions to Association, 171 U. S. 505. Both the cases involved the legality of
every case within its literal language. The error involved lies in assuming the combinations or associations of railroads engaged in interstate commerce for
matter to be decided. This is true because, as the acts which may come the purpose of controlling the conduct of the parties to the association or
under the classes stated in the first section and the restraint of trade to which combination in many particulars. The association or combination was
that section applies are not specifically enumerated or defined, it is obvious assailed in each case as being in violation of the statute. It was held that they
that judgment must, in every case, be called into play in order to determine were. It is undoubted that, in the opinion in each case, general language was
whether a particular act is embraced within the statutory classes, and made use of which, when separated from its context, would justify the
whether, if the act is within such classes, its nature or effect causes it to be a conclusion that it was decided that reason could not be resorted to for the
restraint of trade within the intendment of the act. To hold to the contrary purpose of determining whether the acts complained of were within the
would require the conclusion either that every contract, act or combination, of statute. It is, however, also true that the nature and character of the contract
any kind or nature, whether it operated a restraint on trade or not, was within or agreement in each case was fully referred to, and suggestions as to their
the statute, and thus the statute would be destructive of all right to contract or unreasonableness pointed out, in order to indicate that they were within the
agree or combine in any respect whatever as to subjects embraced in prohibitions of the statute. As the cases cannot by any possible conception
interstate trade or commerce, or if this conclusion were not reached, then the be treated as authoritative without the certitude that reason was resorted to
contention would require it to be held that, as the statute did not define the for the purpose of deciding them, it follows as a matter of course that it must
things to which it related, and excluded resort to the only means by which the have been held by the light of reason, since the conclusion could not have
acts to which it relates could be ascertained -- the light of reason -- the been otherwise reached, that the assailed
enforcement of the statute was impossible because of its uncertainty. The
merely generic enumeration which the statute makes of the acts to which it Page 221 U. S. 65
refers and the absence of any definition of restraint of trade as used in the
statute leaves room for but one conclusion, which is that it was expressly contracts or agreements were within the general enumeration of the statute,
designed not to unduly limit the application and that their operation and effect brought about the restraint of trade which
the statute prohibited. This being inevitable, the deduction can in reason only
Page 221 U. S. 64 be this: that, in the cases relied upon, it having been found that the acts
complained of were within the statute and operated to produce the injuries

510
which the statute forbade, that resort to reason was not permissible in order "To treat as condemned by the act all agreements under which, as a result,
to allow that to be done which the statute prohibited. This being true, the the cost of conducting an interstate commercial business may be increased
rulings in the cases relied upon, when rightly appreciated, were therefore would enlarge the application of the act far beyond the fair meaning of the
this, and nothing more: that, as considering the contracts or agreements, language used. There must be some direct and immediate effect upon
their necessary effect and the character of the parties by whom they were interstate commerce in order to come within the act."
made, they were clearly restraints of trade within the purview of the statute,
they could not be taken out of that category by indulging in general reasoning And, in the Joint Traffic case, this statement was expressly reiterated and
as to the expediency or nonexpediency of having made the contracts or the approved and illustrated by example; like limitation on the general language
wisdom or want of wisdom of the statute which prohibited their being made. used in Freight Association and Joint Traffic cases is also the clear result
That is to say, the cases but decided that the nature and character of the of Bement v. National Harrow Co., 186 U. S. 70, 186 U. S. 92, and especially
contracts, creating as they did a conclusive presumption which brought them of Cincinnati Packet Co. v. Bay, 200 U. S. 179.
within the statute, such result was not to be disregarded by the substitution of
a judicial appreciation of what the law ought to be for the plain judicial duty of If the criterion by which it is to be determined in all cases whether every
enforcing the law as it was made. contract, combination, etc., is a restraint of trade within the intendment of the
law is the direct or indirect effect of the acts involved, then, of course, the rule
But, aside from reasoning, it is true to say that the cases relied upon do not, of reason becomes the guide, and the construction which we have given the
when rightly construed, sustain the doctrine contended, for is established by statute, instead of being refuted by the cases relied upon, is by those cases
all of the numerous decisions of this court which have applied and enforced demonstrated to be correct. This is true because, as the construction which
the Anti-Trust Act, since they all, in the very nature of things, rest upon the we have deduced from the history of the act and the analysis of its text is
premise that reason was the guide by which the provisions of the act were in simply that, in every case where it is claimed that an act or acts are in
every case interpreted. Indeed, intermediate the decision of the two cases, violation of the statute, the rule of reason, in the light of the principles of law
that is, after the decision in the Freight Association Case and before the and the public policy which the act embodies, must be applied. From this it
decision in the Joint Traffic Case, the case of Hopkins v. United States,171 U. follows, since that rule and the result of the test as to direct or indirect, in
S. 578, was decided, their ultimate aspect, come to one and the same thing, that the difference
between the two is therefore only that which obtains between things which do
Page 221 U. S. 66 not differ at all.

the opinion being delivered by Mr. Justice Peckham, who wrote both the Page 221 U. S. 67
opinions in the Freight Association and the Joint Traffic cases. And, referring
in the Hopkins case to the broad claim made as to the rule of interpretation If it be true that there is this identity of result between the rule intended to be
announced in the Freight Association case, it was said (p. 171 U. S. 592): applied in the Freight Association Case, that is, the rule of direct and indirect,
and the rule of reason which, under the statute as we construe it, should be
here applied, it may be asked how was it that, in the opinion in the Freight
Association Case, much consideration was given to the subject of whether
the agreement or combination which was involved in that case could be
taken out of the prohibitions of the statute upon the theory of its

511
reasonableness. The question is pertinent, and must be fully and frankly met, those cases applied the rule of reason for the purpose of determining
for if it be now deemed that the Freight Association Case was mistakenly whether the subject before the court was within the statute. The second is
decided or too broadly stated, the doctrine which it announced should be also true, since, as we have already pointed out, unaided by the light of
either expressly overruled or limited. reason, it is impossible to understand how the statute may in the future be
enforced and the public policy which it establishes be made efficacious.
The confusion which gives rise to the question results from failing to
distinguish between the want of power to take a case which, by its terms, or So far as the objections of the defendants are concerned, they are all
the circumstances which surrounded it, considering among such embraced under two headings:
circumstances the character of the parties, is plainly within the statute, out of
the operation of the statute by resort to reason in effect to establish that the a. That the act, even if the averments of the bill be true, cannot be
contract ought not to be treated as within the statute, and the duty in every constitutionally applied, because to do so would extend the power of
case where it becomes necessary, from the nature and character of the Congress to subjects dehors the reach of its authority to regulate commerce,
parties, to decide whether it was within the statute to pass upon that question by enabling that body to deal with mere questions of production of
by the light of reason. This distinction, we think, serves to point out what, in commodities within the States. But all the structure upon which this argument
its ultimate conception, was the thought underlying the reference to the rule proceeds is based upon the decision in United States v. E. C. Knight
of reason made in the Freight Association Case, especially when such Co., 156 U. S. 1. The view, however, which the argument takes of that case
reference is interpreted by the context of the opinion and in the light of the and the arguments based upon that view have been so repeatedly pressed
subsequent opinion in the Hopkins Case and in Cincinnati Packet Company upon this court in connection with the interpretation and enforcement of the
v. Bay, 200 U. S. 179. Anti-Trust Act, and have been so necessarily and expressly decided to be
unsound as to cause the contentions to be plainly foreclosed and to require
And in order not in the slightest degree to be wanting in frankness, we say no express
that, insofar, however, as by separating the general language used in the
opinions in the Freight Association and Joint Traffic cases from the context Page 221 U. S. 69

Page 221 U. S. 68 notice. United States v. Northern Securities Co., 193 U. S. 197, 193 U. S.
334; Loewe v.Lawlor, 208 U. S. 274; Swift & Co. v. United States, 196 U. S.
and the subject and parties with which the cases were concerned, it may be 375; Montague v. Lowry, 193 U. S. 38; Shawnee Compress Co. v.
conceived that the language referred to conflicts with the construction which Anderson, 209 U. S. 423.
we give the statute, they are necessarily now limited and qualified. We see
no possible escape from this conclusion if we are to adhere to the many b. Many arguments are pressed in various forms of statement which, in
cases decided in this court in which the Anti-Trust Law has been applied and substance, amount to contending that the statute cannot be applied under
enforced and if the duty to apply and enforce that law in the future is to the facts of this case without impairing rights of property and destroying the
continue to exist. The first is true because the construction which we now freedom of contract or trade, which is essentially necessary to the wellbeing
give the statute does not in the slightest degree conflict with a single previous of society and which it is insisted is protected by the constitutional guaranty
case decided concerning the Anti-Trust Law aside from the contention as to of due process of law. But the ultimate foundation of all these arguments is
the Freight Association and Joint Traffic cases, and because every one of the assumption that reason may not be resorted to in interpreting and

512
applying the statute, and therefore that the statute unreasonably restricts the Beyond dispute, the proofs establish substantially as alleged in the bill the
right to contract and unreasonably operates upon the right to acquire and following facts:
hold property. As the premise is demonstrated to be unsound by the
construction we have given the statute, of course, the propositions which rest 1. The creation of the Standard Oil Company of Ohio;
upon that premise need not be further noticed.
2. The organization of the Standard Oil Trust of 1882, and also a previous
So far as the arguments proceed upon the conception that, in view of the one of 1879, not referred to in the bill, and the proceedings in the Supreme
generality of the statute, it is not susceptible of being enforced by the courts Court of Ohio, culminating in a decree based upon the finding that the
because it cannot be carried out without a judicial exertion of legislative company was unlawfully a party to that trust; the transfer by the trustees of
power, they are clearly unsound. The statute certainly generically stocks in certain of the companies; the contempt proceedings; and, finally,
enumerates the character of acts which it prohibits and the wrong which it the increase of the capital of the Standard Oil Company of New Jersey and
was intended to prevent. The propositions therefore but insist that, the acquisition by that company of the shares of the stock of the other
consistently with the fundamental principles of due process of law, it never corporations in exchange for its certificates.
can be left to the judiciary to decide whether, in a given case, particular acts
come within a generic statutory provision. But to reduce the propositions, The vast amount of property and the possibilities of far-reaching control
however, to this their final meaning makes it clear that, in substance, they which resulted from the facts last stated are shown by the statement which
deny the existence of essential legislative authority and challenge the right of we have previously annexed concerning the parties to the trust agreement of
the judiciary to perform duties which that department of the government has 1882, and the corporations whose stock was held by the trustees under the
exerted from trust and which came therefore to be held by the New Jersey corporation.
But these statements do not with accuracy convey an appreciation of the
Page 221 U. S. 70
Page 221 U. S. 71
the beginning. This is so clear as to require no elaboration. Yet let us
demonstrate that which needs no demonstration by a few obvious examples. situation as it existed at the time of the entry of the decree below, since,
Take, for instance, the familiar cases where the judiciary is called upon to during the more than ten years which elapsed between the acquiring by the
determine whether a particular act or acts are within a given prohibition, New Jersey corporation of the stock and other property which was formerly
depending upon wrongful intent. Take questions of fraud. Consider the power held by the trustees under the trust agreement, the situation, of course, had
which must be exercised in every case where the courts are called upon to somewhat changed, a change which, when analyzed in the light of the proof,
determine whether particular acts are invalid which are, abstractly speaking, we think establishes that the result of enlarging the capital stock of the New
in and of themselves valid, but which are asserted to be invalid because of Jersey company and giving it the vast power to which we have referred
their direct effect upon interstate commerce. produced its normal consequence, that is, it gave to the corporation, despite
enormous dividends and despite the dropping out of certain corporations
We come then to the third proposition requiring consideration, viz: enumerated in the decree of the court below, an enlarged and more perfect
sway and control over the trade and commerce in petroleum and its
Third. The facts and the application of the statute to them. products. The ultimate situation referred to will be made manifest by an

513
examination of 2 and 4 of the decree below, which are excerpted in the so many other corporations, aggregating so vast a capital, gives rise, in and
margin. [Footnote 7] of itself, in the absence of countervailing circumstances, to say the least, to
the prima facie presumption of intent and purpose to maintain the dominancy
Page 221 U. S. 72 over the oil industry, not as a result of normal methods of industrial
development, but by new means of combination which were resorted to in
Giving to the facts just stated, the weight which it was deemed they were order that greater power might be added than would otherwise have arisen
entitled to, in the light afforded by the had normal methods been followed, the whole with the purpose of excluding
others from the trade, and thus centralizing in the combination a perpetual
control of the movements of petroleum and its products in the channels of
Page 221 U. S. 73
interstate commerce.

proof of other cognate facts and circumstances, the court below held that the
b. Because the prima facie presumption of intent to restrain trade, to
acts and dealings established by the
monopolize, and to bring about monopolization resulting from the act of
expanding the stock of the New Jersey corporation and vesting it with such
Page 221 U. S. 74 vast control of the oil industry, is made conclusive by considering, 1, the
conduct of the persons or corporations who were mainly instrumental in
proof operated to destroy the "potentiality of competition" which otherwise bringing about the extension of power in the New Jersey corporation before
would have existed to such an extent as to cause the transfers of stock which the consummation of that result and prior to the formation of the trust
were made to the New Jersey corporation and the control which resulted agreements of 1879 and 1882 2, by considering the proof as to what was
over the many and various subsidiary corporations to be a combination or done under those agreements and the acts which immediately preceded the
conspiracy in restraint of trade in violation of the first section of the act, but vesting of power in the New Jersey corporation, as well as by weighing the
also to be an attempt to monopolize and a monopolization bringing about a modes in which the power vested in that corporation has been exerted and
perennial violation of the second section. the results which have arisen from it.

We see no cause to doubt the correctness of these conclusions, considering Recurring to the acts done by the individuals or corporations who were
the subject from every aspect, that is, both in view of the facts established by mainly instrumental in bringing about the
the record and the necessary operation and effect of the law as we have
Page 221 U. S. 76
Page 221 U. S. 75
expansion of the New Jersey corporation during the period prior to the
construed it upon the inferences deducible from the facts, for the following formation of the trust agreements of 1879 and 1882, including those
reasons: agreements, not for the purpose of weighing the substantial merit of the
numerous charges of wrongdoing made during such period, but solely as an
a. Because the unification of power and control over petroleum and its aid for discovering intent and purpose, we think no disinterested mind can
products which was the inevitable result of the combining in the New Jersey survey the period in question without being irresistibly driven to the
corporation by the increase of its stock and the transfer to it of the stocks of conclusion that the very genius for commercial development and

514
organization which it would seem was manifested from the beginning soon power to control the other, and if the inferences which this situation suggests
begot an intent and purpose to exclude others which was frequently were developed, which we deem it unnecessary to do, they might well serve
manifested by acts and dealings wholly inconsistent with the theory that they to add additional cogency to the presumption of intent to monopolize which
were made with the single conception of advancing the development of we have found arises from the unquestioned proof on other subjects.
business power by usual methods, but which, on the contrary, necessarily
involved the intent to drive others from the field, and to exclude them from We are thus brought to the last subject which we are called upon to
their right to trade, and thus accomplish the mastery which was the end in consider, viz:
view. And, considering the period from the date of the trust agreements of
1879 and 1882 up to the time of the expansion of the New Jersey Fourth. The remedy to be administered.
corporation, the gradual extension of the power over the commerce in oil
which ensued, the decision of the Supreme Court of Ohio, the tardiness or
It may be conceded that, ordinarily, where it was found that acts had been
reluctance in conforming to the commands of that decision, the method first
done in violation of the statute, adequate measure of relief would result from
adopted and that which finally culminated in the plan of the New Jersey
restraining the doing of such acts in the future. Swift v. United States, 196 U.
corporation, all additionally serve to make manifest the continued existence
S. 375. But in a case like this, where the condition which has been brought
of the intent which we have previously indicated and which, among other
about in violation of the statute, in and of itself, is not only a continued
things, impelled the expansion of the New Jersey corporation. The exercise
attempt to monopolize, but also a monopolization, the duty to enforce the
of the power which resulted from that organization fortifies the foregoing
statute requires the application of broader and more controlling remedies. As
conclusions, since the development which came, the acquisition here and
penalties which are not authorized by law may not be inflicted by judicial
there which ensued of every efficient means by which competition could have
authority, it follows that, to meet the situation with which we are confronted,
been asserted, the slow but resistless methods which followed by which
means of transportation were absorbed and brought under control,
Page 221 U. S. 78
Page 221 U. S. 77
the application of remedies two-fold in character becomes essential: 1st. To
forbid the doing in the future of acts like those which we have found to have
the system of marketing which was adopted by which the country was
been done in the past which would be violative of the statute. 2d. The
divided into districts and the trade in each district in oil was turned over to a
exertion of such measure of relief as will effectually dissolve the combination
designated corporation within the combination and all others were excluded,
found to exist in violation of the statute, and thus neutralize the extension and
all lead the mind up to a conviction of a purpose and intent which we think is
continually operating force which the possession of the power unlawfully
so certain as practically to cause the subject not to be within the domain of
obtained has brought and will continue to bring about.
reasonable contention.

In applying remedies for this purpose, however, the fact must not be
The inference that no attempt to monopolize could have been intended, and
overlooked that injury to the public by the prevention of an undue restraint
that no monopolization resulted from the acts complained of, since it is
on, or the monopolization of, trade or commerce is the foundation upon
established that a very small percentage of the crude oil produced was
which the prohibitions of the statute rest, and, moreover, that one of the
controlled by the combination, is unwarranted. As substantial power over the
fundamental purposes of the statute is to protect, not to destroy, rights of
crude product was the inevitable result of the absolute control which existed
property.
over the refined product, the monopolization of the one carried with it the

515
Let us then, as a means of accurately determining what relief we are to enjoined from engaging in or carrying on interstate commerce. And, by 9,
afford, first come to consider what relief was afforded by the court below, in among other things, a delay of thirty days was granted for the carrying into
order to fix how far it is necessary to take from or add to that relief, to the end effect of the directions of the decree.
that the prohibitions of the statute may have complete and operative force.
So far as the decree held that the ownership of the stock of the New Jersey
The court below, by virtue of 1, 2, and 4 of its decree, which we have in corporation constituted a combination in violation of the first section and an
part previously excerpted in the margin, adjudged that the New Jersey attempt to create a monopoly or to monopolize under the second section and
corporation, insofar as it held the stock of the various corporations recited in commanded the dissolution of the combination, the decree was clearly
2 and 4 of the decree or controlled the same was a combination in appropriate. And this also is true of 5 of the decree, which restrained both
violation of the first section of the act, and an attempt to monopolize or a the New Jersey corporation and the subsidiary corporations from doing
monopolization contrary to the second section of the act. It commanded the anything which would recognize or give effect to further ownership
dissolution of the combination, and therefore, in effect, directed the transfer
by the New Jersey corporation back to the stockholders of the various Page 221 U. S. 80
subsidiary corporations entitled to the same of the stock which had been
turned over to the New Jersey company in exchange for its stock. To in the New Jersey corporation of the stocks which were ordered to be
retransferred.
Page 221 U. S. 79
But the contention is that, insofar as the relief by way of injunction which was
make this command effective, 5 of the decree forbade the New Jersey awarded by 6 against the stockholders of the subsidiary corporations or the
corporation from in any form or manner exercising any ownership or exerting subsidiary corporations themselves after the transfer of stock by the New
any power directly or indirectly in virtue of its apparent title to the stocks of Jersey corporation was completed in conformity to the decree, the relief
the subsidiary corporations, and prohibited those subsidiary corporations awarded was too broad: a. Because it was not sufficiently specific and
from paying any dividends to the New Jersey corporation or doing any act tended to cause those who were within the embrace of the order to cease to
which would recognize further power in that company, except to the extent be under the protection of the law of the land and required them to thereafter
that it was necessary to enable that company to transfer the stock. So far as conduct their business under the jeopardy of punishments for contempt for
the owners of the stock of the subsidiary corporations and the corporations violating a general injunction. New Haven R.R. v. Interstate Commerce
themselves were concerned, after the stock had been transferred, 6 of the Commission, 200 U. S. 404. Besides, it is said that the restraint imposed by
decree enjoined them from in any way conspiring or combining to violate the 6 -- even putting out of view the consideration just stated -- was moreover
act or to monopolize or attempt to monopolize in virtue of their ownership of calculated to do injury to the public, and, it may be, in and of itself, to produce
the stock transferred to them, and prohibited all agreements between the the very restraint on the due course of trade which it was intended to prevent.
subsidiary corporations or other stockholders in the future, tending to We say this since it does not necessarily follow, because an illegal restraint
produce or bring about further violations of the act. of trade or an attempt to monopolize or a monopolization resulted from the
combination and the transfer of the stocks of the subsidiary corporations to
By 7, pending the accomplishment of the dissolution of the combination by the New Jersey corporation, that a like restraint or attempt to monopolize or
the transfer of stock and until it was consummated, the defendants named in monopolization would necessarily arise from agreements between one or
2, constituting all the corporations to which we have referred, were more of the subsidiary corporations after the transfer of the stock by the New

516
Jersey corporation. For illustration, take the pipelines. By the effect of the Our conclusion is that the decree below was right, and
transfer of the stock, the pipelines would come under the control of various
corporations, instead of being subjected to a uniform control. If various Page 221 U. S. 82
corporations owning the lines determined in the public interests to so
combine as to make a continuous line, such agreement or combination would should be affirmed except as to the minor matters concerning which we have
not be repugnant to the act, and yet it might be restrained by the decree. As indicated the decree should be modified. Our order will therefore be one of
another example, take the affirmance, with directions, however, to modify the decree in accordance with
this opinion. The court below to retain jurisdiction to the extent necessary to
Page 221 U. S. 81 compel compliance in every respect with its decree.

Union Tank Line Company, one of the subsidiary corporations, the owner And it is so ordered.
practically of all the tank cars in use by the combination. If no possibility
existed of agreements for the distribution of these cars among the subsidiary
corporations, the most serious detriment to the public interest might result.
Conceding the merit, abstractly considered, of these contentions, they are
irrelevant. We so think, since we construe the sixth paragraph of the decree
not as depriving the stockholders or the corporations, after the dissolution of
the combination, of the power to make normal and lawful contracts or
agreements, but as restraining them from, by any device whatever,
recreating directly or indirectly the illegal combination which the decree
dissolved. In other words, we construe the sixth paragraph of the decree not
as depriving the stockholders or corporations of the right to live under the law
of the land, but as compelling obedience to that law. As therefore the sixth
paragraph, as thus construed, is not amenable to the criticism directed
against it, and cannot produce the harmful results which the arguments
suggest it was obviously right. We think that, in view of the magnitude of the
interests involved and their complexity, that the delay of thirty days allowed
for executing the decree was too short, and should be extended so as to
embrace a period of at least six months. So also, in view of the possible
serious injury to result to the public from an absolute cessation of interstate
commerce in petroleum and its products by such vast agencies as are
embraced in the combination, a result which might arise from that portion of
the decree which enjoined carrying on of interstate commerce not only by the
New Jersey corporation, but by all the subsidiary companies until the
dissolution of the combination by the transfer of the stocks in accordance
with the decree, the injunction provided for in 7 thereof should not have
been awarded.

517
[G.R. No. 131214. July 27, 2000]

BA SAVINGS BANK, petitioner, vs. ROGER T. SIA, TACIANA U. SIA and


JOHN DOE, respondents.

DECISION

PANGANIBAN, J.:

518
The certificate of non-forum shopping required by Supreme Court On October 24, 1997, the Motion for Reconsideration was denied by the
Circular 28-91 may be signed, for and on behalf of a corporation, by a Court of Appeals on the ground that Supreme Court Revised Circular No. 28-
specifically authorized lawyer who has personal knowledge of the facts 91 requires that it is the petitioner, not the counsel, who must certify under
required to be disclosed in such document. Unlike natural persons, oath to all of the facts and undertakings required therein.
corporations may perform physical actions only through properly delegated
individuals; namely, its officers and/or agents. Hence, this appeal.[5]

The Case Issue

Before us is a Petition for Review on Certiorari under Rule 45 of the In its Memorandum, petitioner submits the following issues for the
Rules of Court, assailing the August 6, 1997 Resolution [1] of the Court of consideration of the Court:
Appeals (CA) in CA-GR SP No. 43209.[2]
I Whether or not petitioner-corporations lawyers are authorized to
Also challenged by petitioner is the October 24, 1997 CA execute and sign the certificate of non-forum shopping. x x x
Resolution[3] denying its Motion for Reconsideration.
II Whether or not the certification of petitioners authorized lawyers will
The Facts
bind the corporation.

On August 6, 1997, the Court of Appeals issued a Resolution denying III Whether or not the certification by petitioner corporations lawyers is in
due course to a Petition for Certiorari filed by BA Savings Bank, on the compliance with the requirements on non-forum shopping. [6]
ground that the Certification on anti-forum shopping incorporated in the
petition was signed not by the duly authorized representative of the Simply stated, the main issue is whether Supreme Court Revised
petitioner, as required under Supreme Court Circular No. 28-91, but by its Circular No. 28-91 allows a corporation to authorize its counsel to execute a
counsel, in contravention of said circular x x x. certificate of non-forum shopping for and on its behalf.

A Motion for Reconsideration was subsequently filed by the petitioner, The Courts Ruling

attached to which was a BA Savings Bank Corporate Secretarys Certificate,


[4]
dated August 14, 1997. The Certificate showed that the petitioners Board
The Petition is meritorious.
of Directors approved a Resolution on May 21, 1996, authorizing the
petitioners lawyers to represent it in any action or proceeding before any Main Issue:
court, tribunal or agency; and to sign, execute and deliver the Certificate of
Non-forum Shopping, among others.
Authority of Counsel

A corporation, such as the petitioner, has no powers except those


expressly conferred on it by the Corporation Code and those that are implied

519
by or are incidental to its existence. In turn, a corporation exercises said In fact, not only was BA Savings Bank authorized to name an agent to
powers through its board of directors and/or its duly authorized officers and sign the certificate; it also exercised its appointing authority reasonably
agents. Physical acts, like the signing of documents, can be performed only well. For who else knows of the circumstances required in the Certificate but
by natural persons duly authorized for the purpose by corporate bylaws or by its own retained counsel. Its regular officers, like its board chairman and
a specific act of the board of directors. All acts within the powers of a president, may not even know the details required therein.
corporation may be performed by agents of its selection; and, except so far
as limitations or restrictions which may be imposed by special charter, by- Consistent with this rationale, the Court en banc in Robern Development
law, or statutory provisions, the same general principles of law which govern Corporation v. Judge Jesus Quitain[8] has allowed even an acting regional
the relation of agency for a natural person govern the officer or agent of a counsel of the National Power Corporation to sign, among others, the
corporation, of whatever status or rank, in respect to his power to act for the certificate of non-forum shopping required by Circular 28-91. The Court held
corporation; and agents once appointed, or members acting in their stead, that the counsel was in the best position to verify the truthfulness and the
are subject to the same rules, liabilities and incapacities as are agents of correctness of the allegations in the Complaint and to know and to certify if
individuals and private persons.[7] an action x x x had already been filed and pending with the courts. [9]

In the present case, the corporations board of directors issued a Circular 28-91 was prescribed by the Supreme Court to prohibit and
Resolution specifically authorizing its lawyers to act as their agents in any penalize the evils of forum shopping. We see no circumvention of this
action or proceeding before the Supreme Court, the Court of Appeals, or any rationale if the certificate was signed by the corporations specifically
other tribunal or agency[;] and to sign, execute and deliver in connection authorized counsel, who had personal knowledge of the matters required in
therewith the necessary pleadings, motions, verification, affidavit of the Circular. In Bernardo v. NLRC,[10] we explained that a literal interpretation
merit, certificate of non-forum shopping and other instruments necessary for of the Circular should be avoided if doing so would subvert its very
such action and proceeding. The Resolution was sufficient to vest such rationale. Said the Court:
persons with the authority to bind the corporation and was specific enough as
to the acts they were empowered to do. x x x. Indeed, while the requirement as to certificate of non-forum shopping is
mandatory, nonetheless the requirements must not be interpreted too literally
In the case of natural persons, Circular 28-91 requires the parties and thus defeat the objective of preventing the undesirable practice of forum-
themselves to sign the certificate of non-forum shopping. However, such shopping.
requirement cannot be imposed on artificial persons, like corporations, for the
simple reason that they cannot personally do the task themselves. As Finally, we stress that technical rules of procedure should be used to
already stated, corporations act only through their officers and duly promote, not frustrate, justice.[11] While the swift unclogging of court dockets
authorized agents. In fact, physical actions, like the signing and the delivery is a laudable objective, the granting of substantial justice is an even more
of documents, may be performed, on behalf of the corporate entity, only by urgent ideal.
specifically authorized individuals.
WHEREFORE, the Petition is GRANTED and the appealed Resolution
It is noteworthy that the Circular does not require corporate officers to is REVERSED and SET ASIDE. The case is REMANDED to the Court of
sign the certificate. More important, there is no prohibition against authorizing Appeals, which is directed to continue the proceedings in CA-GR SP No.
agents to do so. 43209 with all deliberate speed. No costs.

520
SO ORDERED.

Melo, (Chairman), Vitug, Purisima, and Gonzaga-Reyes, JJ., concur.

G.R. No. L-48237

521
MADRIGAL & COMPANY, INC., petitioner, petitioner to its stockholders in exchange for their shares in an equivalent
vs. amount in the corporation.7
HON. RONALDO B. ZAMORA, PRESIDENTIAL ASSISTANT FOR LEGAL
AFFAIRS, THE HON. SECRETARY OF LABOR, and MADRIGAL On August 22, 1975, by yet another alleged stockholders' action, the
CENTRAL OFFICE EMPLOYEES UNION, respondents. petitioner reduced its authorized capitalization from 267,366 shares to
110,085 shares, again, through the same scheme.8
June 30, 1987
After the petitioner's failure to sit down with the respondent union, the latter,
No. L-49023 on August 28, 1974, commenced Case No. LR-5415 with the National Labor
Relations Commission on a complaint for unfair labor practice. 9 In due time,
MADRIGAL & COMPANY, INC., petitioner, the petitioner filed its position paper, 10 alleging operational losses. Pending
vs. the resolution of Case No. LR-5415, the petitioner, in a letter dated
HON. MINISTER OF LABOR and MADRIGAL CENTRAL OFFICE November 17, 1975, 11 informed the Secretary of Labor that Rizal Cement
EMPLOYEES UNION, respondents. Co., Inc., "from which it derives income" 12 "as the General Manager or
Agent" 13 had "ceased operating temporarily." 14 "In addition, "because of the
desire of the stockholders to phase out the operations of the Madrigal & Co.,
Inc. due to lack of business incentives and prospects, and in order to prevent
further losses," 15it had to reduce its capital stock on two occasions "As the
SARMIENTO, J.:
situation, therefore, now stands, the Madrigal & Co., Inc. is without
substantial income to speak of, necessitating a reorganization, by way of
These are two petitions for certiorari and prohibition filed by the retrenchment, of its employees and operations." 16 The petitioner then
petitioner, the Madrigal & Co., Inc. The facts are undisputed. requested that it "be allowed to effect said reorganization gradually
considering all the circumstances, by phasing out in at least three (3) stages,
The petitioner was engaged, among several other corporate objectives, or in a manner the Company deems just, equitable and convenient to all
in the management of Rizal Cement Co., Inc. 1 Admittedly, the petitioner concerned, about which your good office will be apprised accordingly." 17 The
and Rizal Cement Co., Inc. are sister companies. 2 Both are owned by the letter, however, was not verified and neither was it accompanied by the
same or practically the same stockholders. 3 On December 28, 1973, the proper supporting papers. For this reason, the Department of Labor took no
respondent, the Madrigal Central Office Employees Union, sought for the action on the petitioner's request.
renewal of its collective bargaining agreement with the petitioner, which was
due to expire on February 28, 1974. 4 Specifically, it proposed a wage On January 19, 1976, the labor arbiter rendered a decision 18 granting,
increase of P200.00 a month, an allowance of P100.00 a month, and other among other things, a general wage increase of P200.00 a month beginning
economic benefits.5 The petitioner, however, requested for a deferment in the March 1, 1974 plus a monthly living allowance of P100.00 monthly in favor of
negotiations. the petitioner's employees. The arbiter specifically found that the petitioner
"had been making substantial profits in its operation" 19 since 1972 through
On July 29, 1974, by an alleged resolution of its stockholders, the petitioner 1975. The petitioner appealed.
reduced its capital stock from 765,000 shares to 267,366 shares. 6 This was
effected through the distribution of the marketable securities owned by the

522
On January 29, 1976, the petitioner applied for clearance to terminate the Meanwhile, on May 25, 1977, the National Labor Relations Commission
services of a number of employees pursuant supposedly to its retrenchment rendered a decision affirming the labor arbiter's judgment in Case No. LR-
program. On February 3, 1976, the petitioner applied for clearance to 5415. 27 The petitioner appealed to the Secretary of Labor. On June 9, 1978,
terminate 18 employees more. 20 On the same date, the respondent union the Secretary of Labor dismissed the appeal. 28 Following these successive
went to the Regional Office (No. IV) of the Department of Labor (NLRC Case reversals, the petitioner came anew to this court. (G.R. No. 49023.)
No. R04-2-1432-76) to complain of illegal lockout against the
petitioner. 21Acting on this complaint, the Secretary of 22 Labor, in a decision By our resolution dated October 9, 1978, we consolidated G.R. No. 48237
dated December 14, 1976, 22 found the dismissals "to be contrary to with G.R. No. 49023. 29 We likewise issued temporary restraining orders. 30
law" 23 and ordered the petitioner to reinstate some 40 employees, 37 of
them with backwages. 24 The petitioner then moved for reconsideration, In G.R. No. 48237, the petitioner argues, that.
which the Acting Labor Secretary, Amado Inciong, denied. 25
xxx xxx xxx
Thereafter, the petitioner filed an appeal to the Office of the President. The
respondent, the Presidential Assistant on Legal Affairs, affirmed with
I. SAID RESPONDENTS ERRED IN HOLDING THAT THERE WAS NO
modification the Labor Department's decision, thus:
VALID COMPLIANCE WITH THE CLEARANCE REQUIREMENT.

xxx xxx xxx


II. SAID RESPONDENTS ERRED IN NOT HOLDING THAT THERE IS NO
LOCKOUT HERE IN LEGAL CONTEMPLATION, MUCH LESS FOR UNION-
1. Eliseo Dizon, Eugenio Evangelista and Benjamin Victorio are BUSTING PURPOSES.
excluded from the order of reinstatement.
III. RESPONDENT PRESIDENTIAL ASSISTANT ERRED IN ORDERING
2. Rogelio Meneses and Roberto Taladro who appear to have THE REINSTATEMENT OF THE REST OF AFFECTED MEMBERS OF
voluntarily retired and paid their retirement pay, their cases are left to RESPONDENT UNION WITH SIX (6) MONTHS BACKWAGES, EXCEPT
the judgment of the Secretary of Labor who is in a better position to ALELI CONTRERAS, TERESITA EUSEBIO AND NORMA PARLADE WHO
assess appellant's allegation as to their retirement. ARE TO BE REINSTATED WITHOUT BACKWAGES.

3. The rest are hereby reinstated with six (6) months backwages, IV. RESPONDENT PRESIDENTIAL ASSISTANT ERRED IN LEAVING TO
except Aleli Contreras, Teresita Eusebio and Norma Parlade who are THE JUDGMENT OF RESPONDENT SECRETARY THE CASES OF
to be reinstated without backwages. ROGELIO MENESES AND ROBERTO TALADRO WHO HAD
VOLUNTARILY RETIRED AND PAID THEIR RETIREMENT PAY.31
SO ORDERED. 26
xxx xxx xxx
xxx xxx xxx
while in G.R. No. 49023, it submits that:
On May 15, 1978, the petitioner came to this court. (G.R. No. 48237.)

523
xxx xxx xxx Anent Case No. R04-2-1432-76 (G.R. No. 48237), we are satisfied with the
correctness of the respondent Presidential Assistant for Legal Affairs'
1. RESPONDENT MINISTER ERRED IN AFFIRMING THE DECISION EN findings. We quote:
BANC OF THE NATIONAL LABOR RELATIONS COMMISSION DESPITE
CLEAR INDICATIONS IN THE RECORD THAT THE AWARD WAS xxx xxx xxx
PREMATURE IN THE ABSENCE OF A DEADLOCK IN NEGOTIATION AND
THE FAILURE ON THE PART OF THE LABOR ARBITER TO RESOLVE In urging reversal of the appealed decision, appellant contends that
THE MAIN IF NOT ONLY ISSUE OF REFUSAL TO BARGAIN, THEREBY (1) its letter dated November 17, 1975, constitute "substantial
DEPRIVING PETITIONER OF ITS RIGHT TO DUE PROCESS. compliance with the clearance requirement to terminate;" and (2)
individual appellees' dismissal had no relation to any union activities,
2. ASSUMING ARGUENDO THAT THERE WAS A DEADLOCK IN but was the result of an honest-to-goodness retrenchment policy
NEGOTIATION, RESPONDENT MINISTER ERRED NEVERTHELESS IN occasioned by loss of income due to cessation of operation.
NOT FINDING THAT THE ECONOMIC BENEFITS GRANTED IN THE
FORM OF SALARY INCREASES ARE UNFAIR AND VIOLATIVE OF THE We find the first contention to be without merit. Aside from the fact
MANDATORY GUIDELINES PRESCRIBED UNDER PRESIDENTIAL that the controversial letter was unverified, with not even a single
DECREE NO. 525 AND IGNORING THE UNDISPUTED FACT THAT document submitted in support thereof, the same failed to specify the
PETITIONER HAD VIRTUALLY CEASED OPERATIONS AFTER HAVING individual employees to be affected by the intended retrenchment.
TWICE DECREASED ITS CAPITAL STOCKS AND, THEREFORE, NOT Not only this, but the letter is so vague and indefinite regarding the
FINANCIALLY CAPABLE TO ABSORB SUCH AWARD OF BENEFITS. 32 manner of effecting appellant's retrenchment plan as to provide the
Secretary of (sic) a reasonable basis on which to determine whether
xxx xxx xxx the request for retrenchment was valid or otherwise, and whether the
mechanics in giving effect thereto was just or unjust to the
There is no merit in these two (2) petitions. employees concerned. In fact, to be clearly implied from the letter is
that the implementary measures needed to give effect to the
As a general rule, the findings of administrative agencies are accorded not intended retrenchment are yet to be thought of or concretized in the
only respect but even finality. 33 This is especially true with respect to the indefinite future, measures about which the office of the Secretary
Department of Labor, which performs not only a statutory function but carries "will be apprised accordingly." All these, and more, as correctly found
out a Constitutional mandate as well. 34 Our jurisdiction, as a rule, is confined by the Acting Secretary, cannot but show that the letter is insufficient
to cases of grave abuse of discretion. 35 But for certiorari to lie, there must be in form and substance to constitute a valid compliance with the
such arbitrary and whimsical exercise of power, or that discretion was clearance requirement. That being so, it matters little whether or not
exercised despotically. 36 complainant union or any of its members failed to interpose any
opposition thereto.
In no way can the questioned decisions be seen as arbitrary. The decisions
themselves show why. It cannot be over-emphasized that the purpose in requiring a prior
clearance by the Secretary of Labor, in cases of shutdown or
dismissal of employees, is to afford said official ample opportunity to

524
examine and determine the reasonableness of the request. This is "respondent's action [was] a systematic and deliberate attempt to get
made imperative in order to give meaning and substance to the rid of complainants because of their union activities.
constitutional mandate that the State must "afford protection to
labor," and guarantee their "security of tenure." Indeed, the rules We now come to the individual cases of Aleli Contreras, Teresita
require that the application for clearance be filed ten (10) days before Eusebio and Norma Parlade. It is appellant's claim that these three
the intended shutdown or dismissal, serving a copy thereof to the (3) should not be reinstated inasmuch as they have abandoned their
employees affected in order that the latter may register their own work by their continued absences, and moreover in the case of
individual objections against the grant of the clearance. But how Contreras, she failed to oppose the application for clearance filed
could this requirement of notice to the employees have been against her on October 24, 1975. However, appellant's payrolls for
complied with, when, as observed by the Acting Secretary in his December 16-31, 1975, January 1-15, 1976 and January 16-31,
modificatory decision dated June 30, 1977 "the latter of November 1976, show that the three (3) were "on leave without pay." As
17, 1975 does not even state definitely the employees involved" correctly appreciated by the Acting Secretary, these "payrolls prove,
upon whom service could be made. first, that "leave" has been granted to these employees, and, second,
that it is a practice in the company to grant "leaves without pay"
With respect to appellant's second contention, we agree with the without loss of employment status, to those who have exhausted
Acting Secretary's findings that individual appellee's dismissal was their authorized leaves." As regards, Norma Parlade, the records
an offshoot of the union's demand for a renegotiation of the then show that she "truly incurred illness and actually underwent surgery
validly existing collective bargaining Agreement. in Oct., 1975." As to Aleli Contreras, there is no showing that the
Secretary of Labor or appellant ever acted on the clearance. If we
xxx xxx xxx were to follow the logic of appellant, Contreras should not have been
included in the application for clearance filed on Feb. 3, 1976. The
The pattern of appellant's acts after the decision of the Labor Arbiter fact that she was included shows that up to that time, she was still
in Case No. LR-5415 has convinced us that its sole objective was to considered as a regular employee. It was for these reasons, coupled
render moot and academic the desire of the union to exercise its with the length of service that these employees have rendered
right to bargain collectively with management, especially so when it appellant, that the Acting Secretary ordered their reinstatement but
is considered in the light of the fact that under the said decision the without backwages. 37
demand by the union for wage increase and allowances was
granted. What renders appellant's motive suspect was its haste in xxx xxx xxx
terminating the services of individual appellees, without waiting the
outcome of its appeal in Case No. LR-5415. The amount involved by With respect Lo Case No. LR-5415 (G.R. No. 49023), we are likewise
its offer to pay double separation could very well have been used to content with the findings of the National Labor Relations Commission. Thus:
pay the salaries of those employees whose services were sought to
be terminated, until the resolution of its appeal with the NLRC, since xxx xxx xxx
anyway, if its planned retrenchment is found to be justifiable and
done in good faith, its only liability is to answer for the separation pay Appellant now points that the only issue certified to compulsory
provided by law. By and large, therefore, we agree with the Acting arbitration is "refusal to bargain" and it is, therefore, premature to
Secretary that, under the circumstances obtaining in this case,

525
dictate the terms of the CBA on the assumption that there was We now come to the question concerning the equitableness of the
already a deadlock in negotiation. Appellant further contends that, economic benefits granted below. It requires no evidence to show
assuming there was deadlock in negotiation, the economic benefits that the employees concerned deserve some degree of upliftment
granted are unreasonable and violative of the guideline prescribed due to the unabated increase in the cost of living especially in Metro
by P.D. 525. Manila. Of course the company would like us to believe that it is
losing and is therefore not financially capable of improving the
On the other hand, it is the union's stance that its economic demands present CBA to favor its employees. In support of such assertion, the
are justified by, the persistent increase in the cost of living and the company points that the profits reflected in its yearly Statement of
substantial earnings of the company from 1971 to 1975. Income and Expenses are dividends from security holdings. We,
however, reject as puerile its suggestion to dissociate the dividends it
It bears to stress that although the union's petition was precipitated received from security holdings on the pretext that they belong
by the company's refusal to bargain, there are glaring circumstances exclusively to its stockholders. The dividends received by the
pointing out that the parties also submitted "deadlock" to arbitration. company are corporate earnings arising from corporate investment
The petition itself is couched in general terms, praying for arbitration which no doubt are attended to by the employees involved in this
of the union's "dispute" with the respondent concerning proposed proceedings. Otherwise. it would not have been reflected as part of
changes in the collective bargaining agreement." It is supported with profits in the company's yearly financial statements. In determining
a copy of the proposed changes which just goes to show that the the reasonableness of the economic grants below, we have,
union, aside from the issue concerning respondent's refusal to therefore, scrutinized the company's Statement of Income and
bargain, sought determination of the merit of its proposals. On the Expenses from 1972 to 1975 and after equating the welfare of the
part of the appellant company, it pleaded financial incapacity to employees with the substantial earnings of the company, we find the
absorb the proposed economic benefits during the initial stage of the award to be predicated on valid justifications.
proceedings below. Even the evidence and arguments proferred
below by both parties are relevant to deadlock issue. In the face of The salary increase we herein sanction is also in keeping with the
these factual environment, it is our view that the Labor Arbiter below rational that made imperative the enactment of the Termination Pay
did not commit a reversible error in rendering judgment on the Law since in case the respondent company really closes down, the
proposed CBA changes. At any rate, the minimum requirements of employees will receive higher separation pay or retirement benefits
due process was satisfied because as heretofore stated, the to tide them over while seeking another employment. 38
appellant was given Opportunity, and had in fact, presented evidence
and argument in avoidance of the proposed CBA changes. What clearly emerges from the recorded facts is that the petitioner, awash
with profits from its business operations but confronted with the demand of
We do not also subscribe to appellant's argument that by reducing its the union for wage increases, decided to evade its responsibility towards the
capital, it is made evident that it is phasing out its operations. On the employees by a devised capital reduction. While the reduction in capital
contrary, whatever may be the reason behind such reductions, it is stock created an apparent need for retrenchment, it was, by all indications,
indicative of an intention to keep the company a going concern. So just a mask for the purge of union members, who, by then, had agitated for
much so that until now almost four (4) years later, it is still very much wage increases. In the face of the petitioner company's piling profits, the
in existence and operational as before. unionists had the right to demand for such salary adjustments.

526
That the petitioner made quite handsome profits is clear from the records. according to it, "are the absolute property of the stockholders and cannot be
The labor arbiter stated in his decision in the collective agreement case made available for disposition if only to meet the employees' economic
(Case No. LR-5415): demands." 41

xxx xxx xxx There is no merit in this contention. We agree with the National Labor
Relations Commission that "[t]he dividends received by the company are
A clear scrutiny of the financial reports of the respondent [herein corporate earnings arising from corporate investment." 42 Indeed, as found by
petitioner] reveals that it had been making substantial profits in the the Commission, the petitioner had entered such earnings in its financial
operation. statements as profits, which it would not have done if they were not in fact
profits. 43
In 1972, when it still had 765,000 common shares, of which 305,000
were unissued and 459,000 outstanding capitalized at Moreover, it is incorrect to say that such profits in the form of dividends
P16,830,000.00, the respondent made a net profit of P2,403,211.58. are beyond the reach of the petitioner's creditors since the petitioner had
Its total assets were P70,821,317.81. received them as compensation for its management services in favor of the
companies it managed as a shareholder thereof. As such shareholder, the
In 1973, based on the same capitalization, its profit increased to dividends paid to it were its own money, which may then be available for
P2,724,465.33. Its total assets increased to P83,240,473.73. wage increments. It is not a case of a corporation distributing dividends in
favor of its stockholders, in which case, such dividends would be the
absolute property of the stockholders and hence, out of reach by creditors of
In 1974, although its capitalization was reduced from
the corporation. Here, the petitioner was acting as stockholder itself, and in
P16,830,000.00 to P11,230,459.36, its profits were further increased
that case, the right to a share in such dividends, by way of salary increases,
to P2,922,349.70. Its assets were P78,842,175.75.
may not be denied its employees.

The reduction in its assets by P4,398,297.98 was due to the fact that
Accordingly, this court is convinced that the petitioner's capital reduction
its capital stock was reduced by the amount of P5,599,540.54.
efforts were, to begin with, a subterfuge, a deception as it were, to
camouflage the fact that it had been making profits, and consequently, to
In 1975, for the period of only six months, the respondent reported a justify the mass layoff in its employee ranks, especially of union members.
net profit of P547,414.72, which when added to the surplus of They were nothing but a premature and plain distribution of corporate assets
P5,591.214.19, makes a total surplus of P6,138,628.91 as of June to obviate a just sharing to labor of the vast profits obtained by its joint efforts
30, 1975. 39 with capital through the years. Surely, we can neither countenance nor
condone this. It is an unfair labor practice.
xxx xxx xxx
As we observed in People's Bank and Trust Company v. People's Bank and
The petitioner would, however, have us believe that it in fact sustained Trust Co. Employees Union: 44
losses. Whatever profits it earned, so it claims were in the nature of
dividends "declared on its shareholdings in other companies in the earning of xxx xxx xxx
which the employees had no participation whatsoever." 40 "Cash dividends,"

527
As has been held by this Court in Insular Lumber Company vs. CA, the clearance required under Presidential Decree No. 850 has been done
et al., L-23875, August 29, 1969, 29 SCRA 371, retrenchment can away with by Batas Blg. 130, approved on August 21, 1981.
only be availed of if the company is losing or meeting financial
reverses in its operation, which certainly is not the case at bar. During the pendency of these petitions, the petitioner submitted
Undisputed is the fact, that the Bank "at no time incurred losses. " As manifestations to the effect that certain employees have accepted retirement
a matter of fact, "the net earnings of the Bank would be in the benefits pursuant to its retrenchment scheme. 50 This is a matter of defense
average of P2,000,000.00 a year from 1960 to 1969 and, during this that should be raised before the National Labor Relations Commission.
period of nine (9) years, the Bank continuously declared dividends to
its stockholders." Thus the mass lay-off or dismissal of the 65 To do away with the protracted process of determining the earnings acquired
employees under the guise of retrenchment policy of the Bank is a by the employees as a result of ad interim employment, and to erase any
lame excuse and a veritable smoke-screen of its scheme to bust the doubt as to the amount of backwages due them, this court, in line with the
Union and thus unduly disturb the employment tenure of the precedent set in Mercury Drug Co., Inc. v. Court of Industrial
employees concerned, which act is certainly an unfair labor Relations, 51 affirmed in a long line of decisions that came later, 52 hereby
practice. 45 fixes the amount of backwages at three (3) years pay reckoned at the
increased rates decreed by the labor arbiter in Case No. LR-5415 without
Yet, at the same tune, the petitioner would claim that "the phasing out of its deduction or qualification.
operations which brought about the retrenchment of the affected employees
was mainly dictated be the necessity of its stockholders in their capacity as WHEREFORE, the petitions are hereby DISMISSED. Subject to the
heirs of the late Don Vicente Madrigal to partition the estate left by him." 46 It modification as to the amount of backwages hereby awarded, the challenged
must be noted, however, that the labor cases were tried on the theory of decisions are AFFIRMED. The temporary restraining orders are LIFTED.
losses the petitioner was supposed to have incurred to justify retrenchment. With costs against the petitioner.
The petitioner cannot change its theory in the Supreme Court. Moreover,
there is nothing in the records that will substantiate this claim. But what is
This decision is IMMEDIATELY EXECUTORY.
more important is the fact that it is not impossible to partition the Madrigal
estate assuming that the estate is up for partition without the
petitioner's business closing shop and inevitably, without the petitioner laying SO ORDERED.
off its employees.
Yap (Chairman), Narvasa, Melencio-Herrera, Cruz, Feliciano and Gancayco,
As regards the question whether or not the petitioner's letter dated November JJ., concur.
17, 1975 47 was in substantial compliance with legal clearance requirements,
suffice it to state that apart from the Secretary of Labor's valid observation
that the same "did not constitute a sufficient clearance as contemplated by
law, " 48 the factual circumstances show that the letter in question was itself a
part of the "systematic and deliberate attempt to get rid of [the union
members] because of their union activities." 49 Hence, whether or not the said
letter complied with the legal formalities is beside the point since under the
circumstances, retrenchment was, in all events, unjustified. Parenthetically,

528
ISLAMIC DIRECTORATE OF THE PHILIPPINES, MANUEL F. PEREA and
SECURITIES & EXCHANGE COMMISSION, petitioners,
vs.
COURT OF APPEALS and IGLESIA NI CRISTO, respondents.

HERMOSISIMA, JR., J.:

The subject of this petition for review is the Decision of the public respondent
Court of Appeals, 1 dated October 28, 1994, setting aside the portion of the
Decision of the Securities and Exchange Commission (SEC, for short) in
SEC Case No. 4012 which declared null and void the sale of two (2) parcels
of land in Quezon City covered by the Deed of Absolute Sale entered into by
and between private respondent Iglesia Ni Cristo (INC, for short) and the
Islamic Directorate of the Philippines, Inc., Carpizo Group, (IDP, for short).

The following facts appear of record.

Petitioner IDP-Tamano Group alleges that sometime in 1971, Islamic leaders


of all Muslim major tribal groups in the Philippines headed by Dean Cesar
Adib Majul organized and incorporated the ISLAMIC DIRECTORATE OF
THE PHILIPPINES (IDP), the primary purpose of which is to establish an
Islamic Center in Quezon City for the construction of a "Mosque (prayer
place), Madrasah (Arabic School), and other religious infrastructures" so as
to facilitate the effective practice of Islamic faith in the area. 2

Towards this end, that is, in the same year, the Libyan government donated
money to the IDP to purchase land at Culiat, Tandang Sora, Quezon City, to
be used as a Center for the Islamic populace. The land, with an area of
49,652 square meters, was covered by two titles: Transfer Certificate of Title
Nos. RT-26520 (176616) 3 and RT-26521 (170567), 4 both registered in the
name of IDP.

G.R. No. 117897 May 14, 1997


It appears that in 1971, the Board of Trustees of the IDP was composed of
the following per Article 6 of its Articles of Incorporation:

529
Senator Mamintal Tamano 5 and void, a vacuum is created as to who should adopt the
Congressman Ali Dimaporo by-laws and certify its adoption. To remedy this unfortunate
Congressman Salipada Pendatun situation that the association has found itself in, the
Dean Cesar Adib Majul members of the petitioning corporation are hereby
Sultan Harun Al-Rashid Lucman authorized to prepare and adopt their by-laws for submission
Delegate Ahmad Alonto to the Commission. Once approved, an election of the
Commissioner Datu Mama Sinsuat members of the Board of Trustees shall immediately be
Mayor Aminkadra Abubakar 6 called pursuant to the approved by-laws.

According to the petitioner, in 1972, after the purchase of the land by the SO ORDERED. 9
Libyan government in the name of IDP, Martial Law was declared by the late
President Ferdinand Marcos. Most of the members of the 1971 Board of Neither group, however, took the necessary steps prescribed by the SEC in
Trustees like Senators Mamintal Tamano, Salipada Pendatun, Ahmad Alonto, its October 3, 1986 Decision, and, thus, no valid election of the members of
and Congressman Al-Rashid Lucman flew to the Middle East to escape the Board of Trustees of IDP was ever called. Although the Carpizo
political persecution. Group 10 attempted to submit a set of by-laws, the SEC found that, aside
from Engineer Farouk Carpizo and Atty. Musib Buat, those who prepared and
Thereafter, two Muslim groups sprung, the Carpizo Group, headed by adopted the by-laws were not bona fide members of the IDP, thus rendering
Engineer Farouk Carpizo, and the Abbas Group, led by Mrs. Zorayda the adoption of the by-laws likewise null and void.
Tamano and Atty. Firdaussi Abbas. Both groups claimed to be the legitimate
IDP. Significantly, on October 3, 1986, the SEC, in a suit between these two On April 20, 1989, without having been properly elected as new members of
contending groups, came out with a Decision in SEC Case No. 2687 the Board of Trustee of IDP, the Carpizo Group caused to be signed an
declaring the election of both the Carpizo Group and the Abbas Group as alleged Board Resolution 11 of the IDP, authorizing the sale of the subject two
IDP board members to be null and void. The dispositive portion of the SEC parcels of land to the private respondent INC for a consideration of
Decision reads: P22,343,400.00, which sale was evidenced by a Deed of Absolute
Sale 12 dated April 20, 1989.
WHEREFORE, judgment is hereby rendered declaring the
elections of both the petitioners 7 and respondents 8 as null On May 30, 1991, the petitioner 1971 IDP Board of Trustees headed by
and void for being violative of the Articles of Incorporation of former Senator Mamintal Tamano, or the Tamano Group, filed a petition
petitioner corporation. With the nullification of the election of before the SEC, docketed as SEC Case No. 4012, seeking to declare null
the respondents, the approved by-laws which they certified and void the Deed of Absolute Sale signed by the Carpizo Group and the
to this Commission as members of the Board of Trustees INC since the group of Engineer Carpizo was not the legitimate Board of
must necessarily be likewise declared null and void. Trustees of the IDP.
However, before any election of the members of the Board of
Trustees could be conducted, there must be an approved by- Meanwhile, private respondent INC, pursuant to the Deed of Absolute Sale
laws to govern the internal government of the association executed in its favor, filed an action for Specific Performance with Damages
including the conduct of election. And since the election of against the vendor, Carpizo Group, before Branch 81 of the Regional Trial
both petitioners and respondents have been declared null

530
Court of Quezon City, docketed as Civil Case No. Q-90-6937, to compel said Private respondent INC opposed the motion arguing, inter alia, that the issue
group to clear the property of squatters and deliver complete and full physical sought to be litigated by way of intervention is an intra-corporate dispute
possession thereof to INC. Likewise, INC filed a motion in the same case to which falls under the jurisdiction of the SEC. 14
compel one Mrs. Leticia P. Ligon to produce and surrender to the Register of
Deeds of Quezon City the owner's duplicate copy of TCT Nos. RT-26521 and Judge Celia Lipana-Reyes of Branch 81, Regional Trial Court of Quezon City,
RT-26520 covering the aforementioned two parcels of land, so that the sale denied petitioner's motion to intervene on the ground of lack of juridical
in INC's favor may be registered and new titles issued in the name of INC. personality of the IDP-Tamano Group and that the issues being raised by
Mrs. Ligon was alleged to be the mortgagee of the two parcels of land way of intervention are intra-corporate in nature, jurisdiction thereto properly
executed in her favor by certain Abdulrahman R.T. Linzag and Rowaida pertaining to the SEC. 15
Busran-Sampaco claimed to be in behalf of the Carpizo Group.
Apprised of the pendency of SEC Case No. 4012 involving the controverted
The IDP-Tamano Group, on June 11, 1991, sought to intervene in Civil Case status of the IDP-Carpizo Group but without waiting for the outcome of said
No. Q-90-6937 averring, inter alia: case, Judge Reyes, on September 12, 1991, rendered Partial Judgment in
Civil Case No. Q-90-6937 ordering the IDP-Carpizo Group to comply with its
xxx xxx xxx obligation under the Deed of Sale of clearing the subject lots of squatters and
of delivering the actual possession thereof to INC. 16
2. That the Intervenor has filed a case before the Securities
and Exchange Commission (SEC) against Mr. Farouk Thereupon, Judge Reyes in another Order, dated March 2, 1992, pertaining
Carpizo, et. al., who, through false schemes and also to Civil Case No. Q-90-6937, treated INC as the rightful owner of the
machinations, succeeded in executing the Deed of Sale real properties and disposed as follows:
between the IDP and the Iglesia Ni Kristo (plaintiff in the
instant case) and which Deed of Sale is the subject of the WHEREFORE, Leticia P. Ligon is hereby ordered to produce
case at bar; and/or surrender to plaintiff 17 the owner's copy of RT-26521
(170567) and RT-26520 (176616) in open court for the
3. That the said case before the SEC is docketed as Case registration of the Deed of Absolute Sale in the latter's name
No. 04012, the main issue of which is whether or not the and the annotation of the mortgage executed in her favor by
aforesaid Deed of Sale between IDP and the Iglesia ni Kristo herein defendant Islamic Directorate of the Philippines on
is null and void, hence, Intervenor's legal interest in the the new transfer certificate of title to be issued to plaintiff.
instant case. A copy of the said case is hereto attached as
Annex "A"; SO ORDERED. 18

4. That, furthermore, Intervenor herein is the duly constituted On April 6, 1992, the above Order was amended by Judge Reyes directing
body which can lawfully and legally represent the Islamic Ligon "to deliver the owner's duplicate copies of TCT Nos. RT-26521
Directorate of the Philippines; (170567) and RT-26520 (176616) to the Register of Deeds of Quezon
City for the purposes stated in the Order of March 2, 1992." 19
xxx xxx xxx 13

531
Mortgagee Ligon went to the Court of Appeals, thru a petition for certiorari, INC elevated SEC Case No. 4012 to the public respondent Court of Appeals
docketed as CA-G.R No. SP-27973, assailing the foregoing Orders of Judge by way of a special civil action for certiorari, docketed as CA-G.R SP No.
Reyes. The appellate court dismissed her petition on October 28, 1992. 20 33295. On October 28, 1994, the court a quo promulgated a Decision in CA-
G.R. SP No. 33295 granting INC's petition. The portion of the SEC Decision
Undaunted, Ligon filed a petition for review before the Supreme Court which in SEC Case No. 4012 which declared the sale of the two (2) lots in question
was docketed as G.R. No. 107751. to INC as void was ordered set aside by the Court of Appeals.

In the meantime, the SEC, on July 5, 1993, finally came out with a Decision Thus, the IDP-Tamano Group brought the instant petition for review, dated
in SEC Case No. 4012 in this wise: December 21, 1994, submitting that the Court of Appeals gravely erred in:

1. Declaring the by-laws submitted by the respondents 21


as 1) Not upholding the jurisdiction of the SEC to declare the nullity of the sale;
unauthorized, and hence, null and void.
2) Encouraging multiplicity of suits; and
2. Declaring the sale of the two (2) parcels of land in Quezon
26
City covered by the Deed of Absolute Sale entered into by 3) Not applying the principles of estoppel and laches.
Iglesia ni Kristo and the Islamic Directorate of the
Philippines, Inc. 22 null and void; While the above petition was pending, however, the Supreme Court rendered
judgment in G.R. No. 107751 on the petition filed by Mrs. Leticia P. Ligon.
23
3. Declaring the election of the Board of Directors, of the The Decision, dated June 1, 1995, denied the Ligon petition and affirmed the
corporation from 1986 to 1991 as null and void; October 28, 1992 Decision of the Court of Appeals in CA-G.R. No. SP-27973
which sustained the Order of Judge Reyes compelling mortgagee Ligon to
4. Declaring the acceptance of the respondents, except surrender the owner's duplicate copies of TCT Nos. RT-26521 (170567) and
Farouk Carpizo and Musnib Buat, as members of the IDP RT-26520 (176616) to the Register of Deeds of Quezon City so that the
null and void. Deed of Absolute Sale in INC's favor may be properly registered.

No pronouncement as to cost. Before we rule upon the main issue posited in this petition, we would like to
point out that our disposition in G.R. No. 107751 entitled, "Ligon v. Court of
SO ORDERED. 24 Appeals," promulgated on June 1, 1995, in no wise constitutes res
judicatasuch that the petition under consideration would be barred if it were
the ease. Quite the contrary, the requisites or res judicata do not obtain in the
Private respondent INC filed a Motion for Intervention, dated September 7,
case at bench.
1993, in SEC Case No. 4012, but the same was denied on account of the
fact that the decision of the case had become final and executory, no appeal
having been taken therefrom. 25 Section 49, Rule 39 of the Revised Rules of Court lays down the dual
aspects of res judicata in actions in personam, to wit:

532
Effect of judgment. The effect of a judgment or final order Neither of these concepts of res judicata find relevant application in the case
rendered by a court or judge of the Philippines, having at bench. While there may be identity of subject matter (IDP property) in both
jurisdiction to pronounce the judgment or order, may be as cases, there is no identity of parties. The principal parties in G.R. No. 107751
follows: were mortgagee Leticia P. Ligon, as petitioner, and the Iglesia Ni Cristo, as
private respondent. The IDP, as represented by the 1971 Board of Trustees
xxx xxx xxx or the Tamano Group, was only made an ancillary party in G.R. No. 107751
as intervenor. 28 It was never originally a principal party thereto. It must be
(b) In other cases the judgment or order is, with respect to noted that intervention is not an independent action, but is merely collateral,
the matter directly adjudged or as to any other matter that accessory, or ancillary to the principal action. It is just an interlocutory
could have been raised in relation thereto, conclusive proceeding dependent on or subsidiary to the case between the original
between the parties and their successors in interest by title parties. 29 Indeed, the IDP-Tamano Group cannot be considered a principal
subsequent to the commencement of the action or special party in G.R. No. 107751 for purposes of applying the principle of res
proceeding, litigating for the same thing and under the same judicata since the contrary goes against the true import of the action of
title and in the same capacity; intervention as a mere subsidiary proceeding without an independent life
apart from the principal action as well as the intrinsic character of the
intervenor as a mere subordinate party in the main case whose right may be
(c) In any other litigation between the same parties or their
said to be only in aid of the right of the original party. 30 It is only in the
successors in interest, that only is deemed to have been
present case, actually, where the IDP-Tamano Group became a principal
adjudged in a former judgment which appears upon its face
party, as petitioner, with the Iglesia Ni Cristo, as private respondent. Clearly,
to have been so adjudged, or which was actually and
there is no identity of parties in both cases.
necessarily included therein or necessary thereto.

In this connection, although it is true that Civil Case No. Q-90-6937, which
Section 49(b) enunciates the first concept of res judicata known as "bar by
gave rise to G.R. No. 107751, was entitled, "Iglesia Ni Kristo, Plaintiff
prior judgment," whereas, Section 49(c) is referred to as "conclusiveness of
v. Islamic Directorate of the Philippines, Defendant," 31 the IDP can not be
judgment."
considered essentially a formal party thereto for the simple reason that it was
not duly represented by a legitimate Board of Trustees in that case. As a
There is "bar by former judgment" when, between the first case where the necessary consequence, Civil Case No. Q-90-6937, a case for Specific
judgment was rendered, and the second case where such judgment is Performance with Damages, a mere action in personam, did not become
invoked, there is identity of parties, subject matter and cause of action. When final and executory insofar as the true IDP is concerned since petitioner
the three identities are present, the judgment on the merits rendered in the corporation, for want of legitimate representation, was effectively deprived of
first constitutes an absolute bar to the subsequent action. But where between its day in court in said case. Res inter alios judicatae nullum allis
the first case wherein judgment is rendered and the second case wherein praejudicium faciunt. Matters adjudged in a cause do not prejudice those
such judgment is invoked, there is only identity of parties but there is no who were not parties to it. 32 Elsewise put, no person (natural or juridical)
identity of cause of action, the judgment is conclusive in the second case, shall be affected by a proceeding to which he is a stranger. 33
only as to those matters actually and directly controverted and determined,
and not as to matters merely involved therein. This is what is termed
Granting arguendo, that IDP may be considered a principal party in Ligon,
"conclusiveness of judgment." 27
res judicata as a "bar by former judgment" will still not set in on the ground

533
that the cause of action in the two cases are different. The cause of action in Sec. 3. The Commission shall have absolute jurisdiction,
G.R. No. 107751 is the surrender of the owner's duplicate copy of the supervision and control over all corporations, partnership or
transfer certificates of title to the rightful possessor thereof, whereas the associations, who are the grantees of primary franchises
cause of action in the present case is the validity of the Carpizo Group-INC and/or a license or permit issued by the government to
Deed of Absolute Sale. operate in the Philippines . . . .

Res Judicata in the form of "conclusiveness of judgment" cannot likewise xxx xxx xxx
apply for the reason that any mention at all in Ligon as to the validity of the
disputed Carpizo Board-INC sale may only be deemed incidental to the Sec. 5. In addition to the regulatory and adjudicative
resolution of the primary issue posed in said case which is: Who between functions of the Securities and Exchange Commission over
Ligon and INC has the better right of possession over the owner's duplicate corporations, partnerships and other forms of associations
copy of the TCTs covering the IDP property? G.R. No. 107751 cannot be registered with it as expressly granted under existing laws
considered determinative and conclusive on the matter of the validity of the and decrees, it shall have original and exclusive jurisdiction
sale for this particular issue was not the principal thrust of Ligon. To rule to hear and decide cases involving:
otherwise would be to cause grave and irreparable injustice to IDP which
never gave its consent to the sale, thru a legitimate Board of Trustees. xxx xxx xxx

In any case, while it is true that the principle of res judicata is a fundamental c) Controversies in the selection or appointment of directors,
component of our judicial system, it should be disregarded if its rigid trustees, officers, or managers of such corporations,
application would involve the sacrifice of justice to technicality. 34 partnerships or associations. . . . .

The main question though in this petition is: Did the Court of Appeals commit If the SEC can declare who is the legitimate IDP Board, then by
reversible error in setting aside that portion of the SEC's Decision in SEC parity of reasoning, it can also declare who is not the legitimate IDP
Case No. 4012 which declared the sale of two (2) parcels of land in Quezon Board. This is precisely what the SEC did in SEC Case No. 4012
City between the IDP-Carpizo Group and private respondent INC null and when it adjudged the election of the Carpizo Group to the IDP Board
void? of Trustees to be null and
void. 35 By this ruling, the SEC in effect made the unequivocal finding
We rule in the affirmative. that the IDP-Carpizo Group is a bogus Board of Trustees.
Consequently, the Carpizo Group is bereft of any authority
There can be no question as to the authority of the SEC to pass upon the whatsoever to bind IDP in any kind of transaction including the sale
issue as to who among the different contending groups is the legitimate or disposition of ID property.
Board of Trustees of the IDP since this is a matter properly falling within the
original and exclusive jurisdiction of the SEC by virtue of Sections 3 and 5(c) It must be noted that SEC Case No. 4012 is not the first case wherein the
of Presidential Decree No. 902-A: SEC had the opportunity to pass upon the status of the Carpizo Group. As far
back as October 3, 1986, the SEC, in Case No. 2687, 36 in a suit between the
Carpizo Group and the Abbas Group, already declared the election of the

534
Carpizo Group (as well as the Abbas Group) to the IDP Board as null and Sec. 40. Sale or other disposition of assets. Subject to the
void for being violative of the Articles of Incorporation. 37 Nothing thus provisions of existing laws on illegal combinations and
becomes more settled than that the IDP-Carpizo Group with whom private monopolies, a corporation may, by a majority vote of its
respondent INC contracted is a fake Board. board of directors or trustees, sell, lease, exchange,
mortgage, pledge or otherwise dispose of all or substantially
Premises considered, all acts carried out by the Carpizo Board, particularly all of its property and assets, including its goodwill, upon
the sale of the Tandang Sora property, allegedly in the name of the IDP, have terms and conditions and for such consideration, which may
to be struck down for having been done without the consent of the IDP thru a be money, stocks, bonds or other instruments for the
legitimate Board of Trustees. Article 1318 of the New Civil Code lays down payment of money or other property or consideration, as its
the essential requisites of contracts: board of directors or trustees may deem expedient, when
authorized by the vote of the stockholders representing at
There is no contract unless the following requisites concur: least two-thirds (2/3) of the outstanding capital stock; or in
case of non-stock corporation, by the vote of at least two-
thirds (2/3) of the members, in a stockholders' or members'
(1) Consent of the contracting parties;
meeting duly called for the purpose. Written notice of the
proposed action and of the time and place of the meeting
(2) Object certain which is the subject matter of the contract; shall be addressed to each stockholder or member at his
place of residence as shown on the books of the corporation
(3) Cause of the obligation which is established. and deposited to the addressee in the post office with
postage prepaid, or served personally: Provided, That any
All these elements must be present to constitute a valid contract. For, dissenting stockholder may exercise his appraisal right
where even one is absent, the contract is void. As succinctly put by under the conditions provided in this Code.
Tolentino, consent is essential for the existence of a contract, and
where it is wanting, the contract is non-existent. 38 In this case, the A sale or other disposition shall be deemed to cover
IDP, owner of the subject parcels of land, never gave its consent, substantially all the corporate property and assets if thereby
thru a legitimate Board of Trustees, to the disputed Deed of Absolute the corporation would be rendered incapable of continuing
Sale executed in favor of INC. This is, therefore, a case not only of the business or accomplishing the purpose for which it was
vitiated consent, but one where consent on the part of one of the incorporated.
supposed contracting parties is totally wanting. Ineluctably, the
subject sale is void and produces no effect whatsoever. xxx xxx xxx

The Carpizo Group-INC sale is further deemed null and void ab The Tandang Sora property, it appears from the records, constitutes the only
initio because of the Carpizo Group's failure to comply with Section 40 of the property of the IDP. Hence, its sale to a third-party is a sale or disposition of
Corporation Code pertaining to the disposition of all or substantially all assets all the corporate property and assets of IDP falling squarely within the
of the corporation: contemplation of the foregoing section. For the sale to be valid, the majority
vote of the legitimate Board of Trustees, concurred in by the vote of at least
2/3 of the bona fide members of the corporation should have been obtained.

535
These twin requirements were not met as the Carpizo Group which voted to Case No. 4012 came out nullifying the sale, INC came forward, this time,
sell the Tandang Sora property was a fake Board of Trustees, and those quibbling over the issue that it is the regional trial court, and not the SEC,
whose names and signatures were affixed by the Carpizo Group together which has jurisdiction to rule on the validity of the sale. INC is here trifling
with the sham Board Resolution authorizing the negotiation for the sale were, with the courts. We cannot put a premium on this clever legal maneuverings
from all indications, not bona fide members of the IDP as they were made to of private respondent which, if countenanced, would result in a failure of
appear to be. Apparently, there are only fifteen (15) official members of the justice.
petitioner corporation including the eight (8) members of the Board of
Trustees. 39 Furthermore, the Court observes that the INC bought the questioned
property from the Carpizo Group without even seeing the owner's duplicate
All told, the disputed Deed of Absolute Sale executed by the fake Carpizo copy of the titles covering the property. This is very strange considering that
Board and private respondent INC was intrinsically void ab initio. the subject lot is a large piece of real property in Quezon City worth millions,
and that under the Torrens System of Registration, the minimum requirement
Private respondent INC nevertheless questions the authority of the SEC to for one to be a good faith buyer for value is that the vendee at least sees the
nullify the sale for being made outside of its jurisdiction, the same not being owner's duplicate copy of the title and relies upon the same. 41 The private
an intra-corporate dispute. respondent, presumably knowledgeable on the aforesaid workings of the
Torrens System, did not take heed of this and nevertheless went through with
The resolution of the question as to whether or not the SEC had jurisdiction the sale with undue haste. The unexplained eagerness of INC to buy this
to declare the subject sale null and void is rendered moot and academic by valuable piece of land in Quezon City without even being presented with the
the inherent nullity of the highly dubious sale due to lack of consent of the owner's copy of the titles casts very serious doubt on the rightfulness of its
IDP, owner of the subject property. No end of substantial justice will be position as vendee in the transaction.
served if we reverse the SEC's conclusion on the matter, and remand the
case to the regular courts for further litigation over an issue which is already WHEREFORE, the petition is GRANTED. The Decision of the public
determinable based on what we have in the records. respondent Court of Appeals dated October 28, 1994 in CA-G.R. SP No.
33295 is SET ASIDE. The Decision of the Securities and Exchange
It is unfortunate that private respondent INC opposed the motion for Commission dated July 5, 1993 in SEC Case No. 4012 is REINSTATED. The
intervention filed by the 1971 Board of Trustees in Civil Case. No. Q-90- Register of Deeds of Quezon City is hereby ordered to cancel the registration
6937, a case for Specific Performance with Damages between INC and the of the Deed of Absolute Sale in the name of respondent Iglesia Ni Cristo, if
Carpizo Group on the subject Deed of Absolute Sale. The legitimate IDP one has already been made. If new titles have been issued in the name of
Board could have been granted ample opportunity before the regional trial Iglesia Ni Cristo, the Register of Deeds is hereby ordered to cancel the
court to shed light on the true status of the Carpizo Board and settled the same, and issue new ones in the name of petitioner Islamic Directorate of the
matter as to the validity of the sale then and there. But INC, wanting to Philippines. Petitioner corporation is ordered to return to private respondent
acquire the property at all costs and threatened by the participation of the whatever amount has been initially paid by INC as consideration for the
legitimate IDP Board in the civil suit, argued for the denial of the motion property with legal interest, if the same was actually received by IDP.
averring, inter alia, that the issue sought to be litigated by the movant is intra- Otherwise, INC may run after Engineer Farouk Carpizo and his group for the
corporate in nature and outside the jurisdiction of the regional trial court. 40 As amount of money paid.
a result, the motion for intervention was denied. When the Decision in SEC
SO ORDERED.

536
G.R. No. L-56655 July 25, 1983 issue (worth P110,980.00) of previously subscribed shares of the corporation
was made in violation of his pre-emptive right to said additional issue and
DATU TAGORANAO BENITO, petitioner, that the increase in the authorized capital stock of the corporation from
vs. P200,000.00 to P1,000,000.00 was illegal considering that the stockholders
SECURITIES AND EXCHANGE COMMISSION and JAMIATUL of record were not notified of the meeting wherein the proposed increase was
PHILIPPINE-AL ISLAMIA, INC., respondents. in the agenda. Petitioner prayed that the additional issue of shares of
previously authorized capital stock as well as the shares issued from the
The Solicitor General for respondent. increase in capital stock of respondent corporation be cancelled; that the
secretary of respondent corporation be ordered to register the 2,540 shares
acquired by him (petitioner) from Domocao Alonto and Moki-in Alonto; and
Tacod D. Macaraya for private respondent.
that the corporation be ordered to render an accounting of funds to the
stockholders.

In their answer, respondents denied the material allegations of the petition


RELOVA, J.: and, by way of special defense, claimed that petitioner has no cause of
action and that the stock certificates covering the shares alleged to have
On February 6, 1959, the Articles of Incorporation of respondent Jamiatul been sold to petitioner were only given to him as collateral for the loan of
Philippine-Al Islamia, Inc. (originally Kamilol Islam Institute, Inc.) were filed Domocao Alonto and Moki-in Alonto.
with the Securities and Exchange Commission (SEC) and were approved on
December 14, 1962. The corporation had an authorized capital stock of On July 11, 1980, Hearing Officer Ledor E. Macalalag of the Securities and
P200,000.00 divided into 20,000 shares at a par value of P10.00 each. Of Exchange Commission, after due proceedings, rendered a decision which
the authorized capital stock, 8,058 shares worth P80,580.00 were subscribed was affirmed by the Commission En Banc during its executive session held
and fully paid for. Herein petitioner Datu Tagoranao Benito subscribed to 460 on March 9, 1981, as follows:
shares worth P4,600.00.
RESOLVED, That the decision of the hearing Officer in SEC
On October 28, 1975, the respondent corporation filed a certificate of Case No. 1392, dated July 11, 1980, the dispositive portion
increase of its capital stock from P200,000.00 to P1,000,000.00. It was of which reads as follows:
shown in said certificate that P191,560.00 worth of shares were represented
in the stockholders' meeting held on November 25, 1975 at which time the
WHEREFORE, in view of the foregoing
increase was approved. Thus, P110,980.00 worth of shares were
considerations, this Commission hereby
subsequently issued by the corporation from the unissued portion of the
rules: (a) That the issuance by the
authorized capital stock of P200,000.00. Of the increased capital stock of
corporation of its unissued shares was
P1,000,000.00, P160,000.00 worth of shares were subscribed by Mrs.
validly made and was not subject to the pre-
Fatima A. Ramos, Mrs. Tarhata A. Lucman and Mrs. Moki-in Alonto.
emptive rights of stockholders, including the
petitioner, herein; (b) That there is no
On November 18, 1976, petitioner Datu Tagoranao filed with respondent sufficient legal basis to set aside the
Securities and Exchange Commission a petition alleging that the additional

537
certificate issued by this Commission said stockholders' meeting be called within
authorizing the increase in capital stock of thirty (30) days from the time petitioner shall
respondent corporation from P200,000.00 to have subscribed to the increased
Pl,000,000.00. Considering, however, that capitalization.'
petitioner has not waived his pre-emptive
right to subscribe to the increased be, as the same is hereby AFFIRMED, the same being in
capitalization, respondent corporation is accordance with law and the facts of the case. (pp. 28-29,
hereby directed to allow petitioner to Reno)
subscribe thereto, at par value,
proportionate to his present shareholdings, Hence, this petition for review by way of appeal from the aforementioned
adding thereto the 2,540 shares transferred decision of the Securities and Exchange Commission, petitioner contending
to him by Mr. Domocao Alonto and Mrs. that (1) the issuance of the 11,098 shares without the consent of the
Moki-in Alonto; (c) To direct as it hereby stockholders or of the Board of Directors, and in the absence of
directs, the respondent corporation to consideration, is null and void; (2) the increase in the authorized capital stock
immediately cancel Certificates of Stock from P200,000.00 to P1,000,000.00 without the consent or express waiver of
Nos. 216, 223, 302, all in the name of the stockholders, is null and void; (3) he is entitled to attorneys' fees,
Domocao Alonto, and Certificate of Stock damages and expenses of litigation in filing this suit against the directors of
No. 217, in the name of Moki-in Alonto, upon respondent corporation.
their presentation by the petitioner and to
issue new certificates corresponding thereto
We are not persuaded. As aptly stated by the Securities and Exchange
in the name of petitioner herein; (d) To
Commission in its decision:
direct, as it hereby directs, respondent
corporation to religiously comply with the
requirement of filing annual financial xxx xxx xxx
statements under pain of a more drastic
action; (e) To declare, as it hereby declares, ... the questioned issuance of the unsubscribed portion of
as irregular, the election of the nine (9) the capital stock worth P110,980.00 is ' not invalid even if
members of the Board of Trustees of assuming that it was made without notice to the stockholders
respondent corporation on October 30, as claimed by petitioner. The power to issue shares of stocks
1976, for which reason, respondent in a corporation is lodged in the board of directors and no
corporation is hereby ordered to call a stockholders' meeting is necessary to consider it because
stockholders' meeting to elect a new set of additional issuance of shares of stocks does not need
five (5) members of the Board of Trustees, approval of the stockholders. The by-laws of the corporation
unless in the meantime the said number is itself states that 'the Board of Trustees shall, in accordance
accordingly increased and the requirement with law, provide for the issue and transfer of shares of stock
of law to make such increase effective have of the Institute and shall prescribe the form of the certificate
been complied with. It is understood that the of stock of the Institute. (Art. V, Sec. 1).

538
Petitioner bewails the fact that in view of the lack of notice to details of various items taken up therein that would negate any claim that it
him of such subsequent issuance, he was not able to was not authentic. Another thing that petitioner was able to disprove was the
exercise his right of pre-emption over the unissued shares. allegation in the certificate of increase (Exh. 'E-l') that all stockholders who
However, the general rule is that pre-emptive right is did not subscribe to the increase of capital stock have waived their pre-
recognized only with respect to new issue of shares, and not emptive right to do so. As far as the petitioner is concerned, he had not
with respect to additional issues of originally authorized waived his pre-emptive right to subscribe as he could not have done so for
shares. This is on the theory that when a corporation at its the reason that he was not present at the meeting and had not executed a
inception offers its first shares, it is presumed to have offered waiver, thereof. Not having waived such right and for reasons of equity, he
all of those which it is authorized to issue. An original may still be allowed to subscribe to the increased capital stock proportionate
subscriber is deemed to have taken his shares knowing that to his present shareholdings." (pp. 36-37, Rollo)
they form a definite proportionate part of the whole number
of authorized shares. When the shares left unsubscribed are Well-settled is the rule that the findings of facts of administrative bodies will
later re-offered, he cannot therefore claim a dilution of not be interfered with by the courts in the absence of grave abuse of
interest. (Campos and Lopez-Campos Selected Notes and discretion on the part of said agencies, or unless the aforementioned findings
Cases on Corporation Law, p. 855, citing Yasik V. Wachtel are not supported by substantial evidence. (Gokongwei, Jr. vs. SEC, 97
25 Del. Ch. 247,17A. 2d 308 (1941). (pp. 33-34, Rollo) SCRA 78). In a long string of cases, the Supreme Court has consistently
adhered to the rule that decisions of administrative officers are not to be
With respect to the claim that the increase in the authorized capital stock was disturbed by the courts except when the former have acted without or in
without the consent, expressed or implied, of the stockholders, it was the excess of their jurisdiction or with grave abuse of discretion (Sichangco vs.
finding of the Securities and Exchange Commission that a stockholders' Board of Commissioners of Immigration, 94 SCRA 61). Thus, in the case
meeting was held on November 25,1975, presided over by Mr. Ahmad of Deluao vs. Casteel ( L-21906, Dec. 24, 1968, 26 SCRA 475, 496, citing
Domocao Alonto, Chairman of the Board of Trustees and, among the many Pajo vs. Ago, et al., L-15414, June 30, 1960) and Genitano vs. Secretary of
items taken up then were the change of name of the corporation from Kamilol Agriculture and Natural Resources, et al. (L-2ll67, March 31, 1966), the
Islam Institute Inc. to Jamiatul Philippine-Al Islamia, Inc., the increase of its Supreme Court held that:
capital stock from P200,000.00 to P1,000,000.00, and the increase of the
number of its Board of Trustees from five to nine. "Despite the insistence of ... Findings of fact by an administrative board or official,
petitioner, this Commission is inclined to believe that there was a following a hearing, are binding upon the courts and win not
stockholders' meeting on November 25, 1975 which approved the increase. be disturbed except where the board or official has gone
The petitioner had not sufficiently overcome the evidence of respondents that beyond his statutory authority, exercised unconstitutional
such meeting was in fact held. What petitioner successfully proved, however, powers or clearly acted arbitrarily and without regard to his
was the fact that he was not notified of said meeting and that he never duty or with grave abuse of discretion. ...
attended the same as he was out of the country at the time. The
documentary evidence of petitioner conclusively proved that he was ACCORDINGLY, this petition is hereby dismissed for lack of merit.
attending the Mecca pilgrimage when the meeting was held on November
25, 1975. (Exhs. 'Q', 'Q-14', 'R', 'S' and 'S-l'). While petitioner doubts the
SO ORDERED.
authenticity of the alleged minutes of the proceedings (Exh. '4'), the
Commission notes with significance that said minutes contain numerous

539
Plana, Escolin and Gutierrez, Jr., JJ., concur. purpose of recovering a balance of P22,500, alleged to be due upon
defendant's subscription to the capital stock of said insolvent corporation.
Teehankee, J., concurs in the result. The trial judge having given judgment in favor of the plaintiff for the amount
sued for, the defendant appealed.
Melencio-Herrera and Vasquez, JJ., are on leave.
It appears in evidence that in 1918 the Cooperativa Naval Filipina was duly
incorporated under the laws of the Philippine Islands, with a capital of
P100,000, divided into one thousand shares of a par value of P100 each.
Among the incorporators of this company was numbered the defendant
Mariano Rivera, who subscribed for 450 shares representing a value of
P45,000, the remainder of the stock being taken by other persons. The
articles of incorporation were duly registered in the Bureau of Commerce and
Industry on October 30 of the same year.

In the course of time the company became insolvent and went into the hands
of the Philippine Trust Company, as assignee in bankruptcy; and by it this
action was instituted to recover one-half of the stock subscription of the
defendant, which admittedly has never been paid.

The reason given for the failure of the defendant to pay the entire
subscription is, that not long after the Cooperativa Naval Filipina had been
G.R. No. L-19761 January 29, 1923 incorporated, a meeting of its stockholders occurred, at which a resolution
was adopted to the effect that the capital should be reduced by 50 per
PHILIPPINE TRUST COMPANY, as assignee in insolvency of "La centum and the subscribers released from the obligation to pay any unpaid
Cooperativa Naval Filipina," plaintiff-appellee, balance of their subscription in excess of 50 per centum of the same. As a
vs. result of this resolution it seems to have been supposed that the subscription
MARCIANO RIVERA, defendant-appellant. of the various shareholders had been cancelled to the extent stated; and fully
paid certificate were issued to each shareholders for one-half of his
subscription. It does not appear that the formalities prescribed in section 17
Araneta and Zaragoza for appellant.
of the Corporation Law (Act No. 1459), as amended, relative to the reduction
Ross and Lawrence for appellee.
of capital stock in corporations were observed, and in particular it does not
appear that any certificate was at any time filed in the Bureau of Commerce
STREET, J.: and Industry, showing such reduction.

This action was instituted on November 21, 1921, in the Court of First
Instance of Manila, by the Philippine Trust Company, as assignee in
insolvency of La Cooperativa Naval Filipina, against Marciano Rivera, for the

540
His Honor, the trial judge, therefore held that the resolution relied upon the Renato J. Dilag for private respondent.
defendant was without effect and that the defendant was still liable for the
unpaid balance of his subscription. In this we think his Honor was clearly
right.
GRIO-AQUINO, J.:
It is established doctrine that subscription to the capital of a corporation
constitute a find to which creditors have a right to look for satisfaction of their The only issue in this case is whether or not a suit brought by a withdrawing
claims and that the assignee in insolvency can maintain an action upon any stockholder against the corporation to enforce payment of the balance due
unpaid stock subscription in order to realize assets for the payment of its on the consideration (evidenced by a corporate promissory note) for the
debts. (Velasco vs. Poizat, 37 Phil., 802.) A corporation has no power to surrender of his shares of stock and interests in the corporation, involves an
release an original subscriber to its capital stock from the obligation of paying intra-corporate dispute. The resolution of that issue will determine whether
for his shares, without a valuable consideration for such release; and as the Securities and Exchange Commission (SEC) or a regular court has
against creditors a reduction of the capital stock can take place only in the jurisdiction over the action.
manner an under the conditions prescribed by the statute or the charter or
the articles of incorporation. Moreover, strict compliance with the statutory
On May 7, 1984, respondent Nilcar Y. Fajilan offered in writing to resign as
regulations is necessary (14 C. J., 498, 620).
President and Member of the Board of Directors of petitioner, Boman
Environmental Development Corporation (BEDECO), and to sell to the
In the case before us the resolution releasing the shareholders from their company all his shares, rights, and interests therein for P 300,000 plus the
obligation to pay 50 per centum of their respective subscriptions was an transfer to him of the company's Isuzu pick-up truck which he had been
attempted withdrawal of so much capital from the fund upon which the using. The letter-offer (Exh. A-1) reads as follows:
company's creditors were entitled ultimately to rely and, having been effected
without compliance with the statutory requirements, was wholly ineffectual.
07 May 1984

The judgment will be affirmed with cost, and it is so ordered.


THE BOARD OF DIRECTORS,
BOMAN ENVIRONMENTAL DEVELOPMENT
Araullo, C. J., Malcolm, Avancea, Villamor, Ostrand, Johns, and CORPORATION
Romualdez, JJ., concur. 2nd Floor, AGS Building,
466 EDSA, Makati,
G.R. No. 77860 November 22, 1988 Metro Manila

BOMAN ENVIRONMENTAL DEVELOPMENT CORPORATION, petitioners, Gentlemen:


vs.
HON. COURT OF APPEALS and NILCAR Y. FAJILAN, respondents. With deepest regrets, I am tendering my resignation as
member of the Board of Directors and President of the
Lim, Duran & Associates for petitioner. Company effective as soon as my shares and interests
thereto are sold and fully paid.

541
It is really painful to leave the Company which we interest in the company at the price of P300,000.00,
painstakingly labored and nortured for years to attain its inclusive of your unpaid salary from February 1984 to May
success today, however, family interests and other 31, 1984, loan principal, interest on loan, profit sharing and
considerations dictate me otherwise. share on book value of the corporation as at May 31, 1984.
Payment of the P300,000.00 shall be as follows:
Thank you for your interest of buying my shares and other
interests on the Company. It is really my intention to divest
myself of these investments and sell them all for PESOS:
THREE HUNDRED THOUSAND (P 300,000) payable in July 15, 1984 P
cash in addition to the Isuzu pick up I am presently using for 100,000.00
and in behalf of the Company.

Thank you.
September 15, 1984 P
75,000.00
NILCAR Y. FAJILAN
Director/President (p. 239, Rollo.)

At a meeting of the Board of Directors of BEDECO on June 14, 1984,


October 15, 1984 P
Fajilan's resignation as president was accepted and new officers were
62,500.00
elected. Fajilan's offer to sell his shares back to the corporation was
approved, the Board promising to pay for them on a staggered basis from
July 15, 1984 to December 15, 1984 (Annex B).<re||an1w>The
resolution of the Board was communicated to Fajilan in the following letter-
agreement dated June 25, 1984 to which he affixed his conformity (Annex December 15, 1984 P
C): 62,500.00

June 25, 1984

Mr. Nilcar Y. Fajilan P


No. 159 Aramismis Street 300,000.00.
Project 7, Quezon City

Dear Mr. Fajilan: To assure you of payment of the above amount on


respective due dates, the company will execute the
Please be informed that after due deliberation the Board of necessary promissory note.
Directors has accepted your offer to sellyour share and

542
In addition to the above, the Ford Courier Pick-up will belong 1984
to you subject to your assumption of the outstanding PTR No.
obligation thereof with Fil-Invest. It is understood that upon Issued
your full payment of the pick-up, arrangement will be made on January
and negotiated with Fil-Invest regarding the transfer of the at
ownership of the vehicle to your name. Makati,
Manila
If the above meets your requirements, kindly signify your conformity/approval
by signing below. Doc. No. 392
Page No. 80
Very truly yours, Book No. X
(SGD) JAMES C. Series of 1984. (p. 245, Rollo.)
PERALTA
Corporate
A Secretary
promissory note dated July 3, 1984, was signed by BEDECO'S new
president, Alfredo Pangilinan, in the presence of two directors, committing
CONFORME: BEDECO to pay him P300,000 over a six-month period from July 15, 1984 to
December 15, 1984. The promissory note (Exh. D) provided as follows:
(SGD) NILCAR Y. FAJILAN
PROMISSORY NOTE
Noted:
Makati,
(SGD) ALFREDO S. PANGILINAN (SGD) MAXIMO R. July 3, 1984
REBALDO (SGD) BENEDICTO M. EMPAYNADO
FOR VALUE RECEIVED, BOMAN ENVIRONMENTAL
SUBSCRIBED AND SWORN TO before me, this 3rd day of DEVELOPMENT CORPORATION, a domestic corporation
July, 1984, Alfredo S. Pangilinan exhibiting to me his duly registered with the Securities and Exchange
Residence Certificate No. 1696224 issued at Makati, Metro Commission, with office at Rm. 608, Metro Bank Bldg., Ayala
Manila on January 24, 1984, in his capacity as President of Blvd., Makati, Metro Manila, promise to pay NILCAR Y.
Boman Environmental Development Corporation with FAJILAN of 17 Aramismis St., Project 7, Quezon City, the
Corporate Residence Certificate No. 207911 issued at sum of PESOS: THREE HUNDRED THOUSAND
Makati, Metro Manila on March 26, 1984. (P300,000.00), Philippine Currency payable as follows:

(SGD) ERNESTO B.
DURAN P J
NOTARY PUBLIC 1 u
Until December 31,

543
0 l ,
0, y 1
0 1 9
0 5 8
0. , 4
0 1
0 9
8
4 6 D
2, e
5 c
0 .
7 S 0. 1
5, e 0 5
0 p 0 ,
0 t. 1
0. 1 9
0 5 8
0 , 4
1
9
8
4 P300,000.00

6 O BO
2, c MA
5 t N
0 o EN
0. b VIR
0 e ON
0 r ME
1 NTA
5 L

544
DE On April 30, 1985, Fajilan filed a complaint in the Regional Trial Court of
VEL Makati for collection of that balance from BEDECO.
OP
ME In an order dated September 9, 1985, the trial court, through Judge Ansberto
NT Paredes, dismissed the complaint for lack of jurisdiction. It ruled that the
CO controversy arose out of intracorporate relations, hence, the Securities and
RP Exchange Commission has original and exclusive jurisdiction to hear and
OR decide it.
ATI
ON His motion for reconsideration of that order having been denied, Fajilan filed
By: a "Petition for Certiorari, and mandamus with Preliminary Attachment" in the
(SG Intermediate Appellate Court.
D)
ALF
In a decision dated March 2, 1987, the Court of Appeals set aside Judge
RE
Paredes' order of dismissal and directed him to take cognizance of the case.
DO
BEDECO's motion for reconsideration was denied in a resolution dated
S.
March 24, 1987 of the Court of Appeals.
PAN
GILI
NA In its decision, the Appellate Court characterized the case as a suit for
N collection of a sum of money as Fajilan "was merely suing on the balance of
Pre the promissory note" (p. 4, Decision; p. 196, Rollo) which BEDECO failed
side and refused to pay in full. More particularly, the Court of Appeals held:
nt
While it is true that the circumstances which led to the
Signed in the presence of: execution of the promissory note by the Board of Directors of
respondent corporation was an intra- corporate matter, there
arose no controversy as to the sale of petitioner's interests
(SGD) MAXIMO R. REBALDO
and rights as well as his shares as Member of the Board of
Directors and President of respondent corporation. The intra-
(SGD) BENEDICTO M. EMPAYNADO corporate matter of the resignation of petitioner as Member
(Annex D, p. 247, Rollo.) of the Board of Directors and President of respondent
corporation has long been settled without issue.
However, BEDECO paid only P50,000 on July 15, 1984 and another P50,000
on August 31, 1984 and defaulted in paying the balance of P200,000. The Board of Directors of respondent corporation has
likewise long settled the sale by petitioner of all his shares,
rights and interests in favor of the corporation. No

545
controversy arose out of this transaction. The jurisdiction of transfer of his shares to the corporation. In the books of the corporation he is
the Securities and Exchange Commission therefore need not still a stockholder.
be invoked on this matter. (p. 196, Rollo.)
Fajilan's suit against the corporation to enforce the latter's promissory note or
The petition is impressed with merit. compel the corporation to pay for his shareholdings is cognizable by the SEC
alone which shall determine whether such payment will not constitute a
Section 5(b) of P.D. No. 902-A, as amended, grants the SEC original and distribution of corporate assets to a stockholder in preference over creditors
exclusive jurisdiction to hear and decide cases involving of the corporation. The SEC has exclusive supervision, control and
regulatory jurisdiction to investigate whether the corporation has unrestricted
b) Controversies arising out of intra-corporate or partnership retained earnings to cover the payment for the shares, and whether the
relations, between and among stockholders members, or purchase is for a legitimate corporate purpose as provided in Sections 41
associates; between any or all of them and the corporation, and 122 of the Corporation Code, which reads as follows:
partnership or association of which they are stockholders,
members or associates, respectively; ... (Emphasis SEC. 41. Power to acquire own shares.A stock corporation
supplied.) shall have the power to purchase or acquire its own shares
for a legitimate corporate purpose or purposes, including but
This case involves an intra-corporate controversy because the parties are a not limited to the following cases: Provided, That the
stockholder and the corporation. As correctly observed by the trial court, the corporation has unrestricted retained earnings in its books to
perfection of the agreement to sell Fajilan's participation and interests in cover the shares to be purchased or acquired;
BEDECO and the execution of the promissory note for payment of the price
of the sale did not remove the dispute from the coverage of Section 5(b) of 1. To eliminate fractional shares arising out of stock
P.D. No. 902, as amended, for both the said agreement (Annex C) and the dividends;
promissory note (Annex D) arose from intra-corporate relations. Indeed, all
the signatories of both documents were stockholders of the corporation at the 2. To collect or compromise an indebtedness to the
time of signing the same. It was an intra-corporate transaction, hence, this corporation, arising out of unpaid subscription, in a
suit is an intra-corporate controversy. delinquency sale, and to purchase delinquent shares sold
during said sale; and
Fajilan's offer to resign as president and director "effective as soon as my
shares and interests thereto (sic) are sold and fully paid" (Annex A-1, p. 239, 3. To pay dissenting or withdrawing stockholders entitled to
Rollo) implied that he would remain a stockholder until his shares and payment for their shares under the provisions of this Code,
interests were fully paid for, for one cannot be a director or president of a
corporation unless he is also a stockholder thereof. The fact that he was Sec. 12. Corporate liquidation. ...
replaced as president of the corporation did not necessaryily mean that he
ceased to be a stockholder considering how the corporation failed to xxx xxx xxx
complete payment of the consideration for the purchase of his shares of
stock and interests in the goodwill of the business. There has been no actual

546
Except by decrease of capital stock and as otherwise
allowed by this Code, no corporation shall distribute any of
its assets or property except upon lawful dissolution and
after payment of all its debts and liabilities, (77a, 89a, 16a).
G.R. No. L-17504 & L-17506 February 28, 1969
These provisions of the Corporation Code should be deemed written into the
agreement between the corporation and the stockholders even if there is no RAMON DE LA RAMA, FRANCISCO RODRIGUEZ, HORTENCIA SALAS,
express reference to them in the promissory note. The principle is well PAZ SALAS and PATRIA SALAS, heirs of Magdalena Salas, as
settled that an existing law enters into and forms part of a valid contract stockholders on their own behalf and for the benefit of the Ma-ao Sugar
without need for the parties' expressly making reference to it (Lakas ng Central Co., Inc., and other stockholders thereof who may wish to join
Manggagawang Makabayan vs. Abiera, 36 SCRA 437). in this action, plaintiffs-appellants,
vs.
The requirement of unrestricted retained earnings to cover the shares is MA-AO SUGAR CENTRAL CO., INC., J. AMADO ARANETA, MRS.
based on the trust fund doctrine which means that the capital stock, property RAMON S. ARANETA, ROMUALDO M. ARANETA, and RAMON A. YULO,
and other assets of a corporation are regarded as equity in trust for the defendants-appellants.
payment of corporate creditors. The reason is that creditors of a corporation
are preferred over the stockholders in the distribution of corporate assets. San Juan, Africa and Benedicto for plaintiffs-appellants.
There can be no distribution of assets among the stockholders without first Vicente Hilado and Gianzon, Sison, Yulo and Associates for defendants-
paying corporate creditors. Hence, any disposition of corporate funds to the appellants.
prejudice of creditors is null and void. "Creditors of a corporation have the
right to assume that so long as there are outstanding debts and liabilities, the
CAPISTRANO, J.:
board of directors will not use the assets of the corporation to purchase its
own stock ..."(Steinberg vs. Velasco, 52 Phil. 953.)
This was a representative or derivative suit commenced on October 20,
1953, in the Court of First Instance of Manila by four minority stockholders
WHEREFORE, the petition for certiorari is granted. The decision of the Court
against the Ma-ao Sugar Central Co., Inc. and J. Amado Araneta and three
of Appeals is reversed and set aside. The order of the trial court dismissing
other directors of the corporation.
the complaint for lack of jurisdiction is hereby reinstated. No costs.

The complaint comprising the period November, 1946 to October, 1952,


SO ORDERED.
stated five causes of action, to wit: (1) for alleged illegal and ultra-vires acts
consisting of self-dealing irregular loans, and unauthorized investments; (2)
Narvasa, Cruz, Gancayco and Medialdea, JJ., concur. for alleged gross mismanagement; (3) for alleged forfeiture of corporate
rights warranting dissolution; (4) for alleged damages and attorney's fees;
and (5) for receivership.

Plaintiffs prayed, in substance, as follows:

547
Under the FIRST CAUSE OF ACTION, that the defendant J. Amado Araneta BY WAY OF SPECIAL DEFENSES, the defendants alleged, among other
and his individual co-defendants be ordered to render an accounting of all things: (1) that the complaint "is premature, improper and unjustified"; (2) that
transactions made and carried out by them for defendant corporation, and "to plaintiffs did not make an "earnest, not simulated effort" to exhaust first their
collect, produce and/or pay to the defendant corporation the outstanding remedies within the corporation before filing their complaint; (3) that no actual
balance of the amounts so diverted and still unpaid to defendant loss had been suffered by the defendant corporation on account of the
corporation"; transactions questioned by plaintiffs; (4) that the payments by the debtors of
all amounts due to the defendant corporation constituted a full, sufficient and
Under the SECOND CAUSE OF ACTION, that the individual defendants be adequate remedy for the grievances alleged in the complaint and (5) that the
held liable and be ordered to pay to the defendant corporation "whatever dissolution and/or receivership of the defendant corporation would violate
amounts may be recovered by the plaintiffs in Civil Case No. 20122, entitled and impair the obligation of existing contracts of said corporation.
'Francisco Rodriguez vs. Ma-ao Sugar Central Co.'"; to return to the
defendant corporation all amounts withdrawn by way of discretionary funds BY WAY OF COUNTERCLAIM, the defendants in substance further alleged,
or backpay, and to account for the difference between the corporation's crop among others, that the complaint was premature, improper and malicious,
loan accounts payable and its crop loan accounts receivable; and that the language used was "unnecessarily vituperative abusive and
insulting, particularly against defendant J. Amado Araneta who appears to be
Under the THIRD CAUSE OF ACTION, that the corporation be dissolved and the main target of their hatred." Wherefore, the defendant sought to recover
its net assets be distributed to the stockholders; and "compensation for damages, actual, moral, exemplary and corrective,
including reasonable attorney's fees."
Under the FOURTH CAUSE OF ACTION, that the defendants be ordered "to
pay the sum of P300,000.00 by way of compensatory, moral and exemplary After trial, the Lower Court rendered its Decision (later supplemented by an
damages and for expenses of litigation, including attorney's fees and costs of Order resolving defendants' Motion for Reconsideration), the dispositive
the suit." portion of which reads:

THE FIFTH CAUSE OF ACTION was an application for the provisional IN VIEW WHEREOF, the Court dismisses the petition for dissolution
remedy of receivership. but condemns J. Amado Araneta to pay unto Ma-ao Sugar Central
Co., Inc. the amount of P46,270.00 with 8% interest from the date of
In their answer originally filed on December 1, 1953, and amended on the filing of this complaint, plus the costs; the Court reiterates the
February 1, 1955, defendants denied "the allegations regarding the preliminary injunction restraining the Ma-ao Sugar Central Co., Inc.
supposed gross mismanagement, fraudulent use and diversion of corporate management to give any loans or advances to its officers and orders
funds, disregard of corporate requirements, abuse of trust and violation of that this injunction be as it is hereby made, permanent; and orders it
fiduciary relationship, etc., supposed to have been discovered by plaintiffs, all to refrain from making investments in Acoje Mining, Mabuhay
of which are nothing but gratuitous, unwarranted, exaggerated and distorted Printing, and any other company whose purpose is not connected
conclusions not supported by plain and specific facts and transactions with the Sugar Central business; costs of plaintiffs to be borne by the
alleged in the complaint." Corporation and J. Amado Araneta.

From this judgment both parties appealed directly to the Supreme Court.

548
Before taking up the errors respectively, assigned by the parties, we should
state that the following findings of the Lower Court on the commission of
corporate irregularities by the defendants have not been questioned by the
defendants:

1. Failure to hold stockholders' meetings regularly. No stockholders' General Securities 86,743.65


meetings were held in 1947, 1950 and 1951;

2. Irregularities in the keeping of the books. Untrue entries were


made in the books which could not simply be considered as innocent Bacolod Murcia 501,030.61
errors;

3. Illegal investments in the Mabuhay Printing, P2,280,00, and the


Acoje Mining, P7,000.00. The investments were made not in Central Azucarera del Danao 97,884.42
pursuance of the corporate purpose and without the requisite
authority of two-thirds of the stockholders;

4. Unauthorized loans to J. Amado Araneta totalling P132,082.00 Talisay-Silay 4,365.90


(which, according to the defendants, had been fully paid), in violation
of the by-laws of the corporation which prohibits any director from
borrowing money from the corporation; The Court found that sums were taken out of the funds of the Ma-ao Sugar
Central Co., Inc. and delivered to these affiliated companies, and vice versa,
5. Diversion of corporate funds of the Ma-ao Sugar Central Co., Inc. without the approval of the Ma-ao Board of Directors, in violation of Sec. III,
to: Art. 6-A of the by-laws.

The errors assigned in the appeal of the plaintiffs, as appellants, are as


J. Amado Araneta & Co. P243,415.62 follows:

I.

Luzon Industrial Corp. 585,918.17 THE LOWER COURT ERRED IN HOLDING THAT THE
INVESTMENT OF CORPORATE FUNDS OF THE MA-AO SUGAR
CENTRAL CO., INC., IN THE PHILIPPINE FIBER PROCESSING
CO., INC. WAS NOT A VIOLATION OF SEC. 17- OF THE
Associated Sugar 463,860.36 CORPORATION LAW.

549
II. the affirmative vote of stockholders holding shares in the
corporation entitling them to exercise at least two-thirds of
THE LOWER COURT ERRED IN NOT FINDING THAT THE MA-AO the voting power on such proposal at the stockholders'
SUGAR CENTRAL CO., INC. WAS INSOLVENT. meeting called for the purpose.

III. the Court is convinced that that law should be understood to mean
as the authorities state, that it is prohibited to the Corporation to
THE LOWER COURT ERRED IN HOLDING THAT THE invest in shares of another corporation unless such an investment is
DISCRIMINATORY ACTS COMMITTED AGAINST PLANTERS DID authorized by two-thirds of the voting power of the stockholders, if
NOT CONSTITUTE MISMANAGEMENT. the purpose of the corporation in which investment is made is foreign
to the purpose of the investing corporation because surely there is
more logic in the stand that if the investment is made in a corporation
IV.
whose business is important to the investing corporation and would
aid it in its purpose, to require authority of the stockholders would be
THE LOWER COURT ERRED IN HOLDING THAT ITS CULPABLE to unduly curtail the Power of the Board of Directors; the only trouble
ACTS WERE INSUFFICIENT FOR THE DISSOLUTION OF THE here is that the investment was made without any previous authority
CORPORATION. of the Board of Directors but was only ratified afterwards; this of
course would have the effect of legalizing the unauthorized act but it
The portions of the Decision of the Lower Court assailed by the plaintiffs as is an indication of the manner in which corporate business is
appellants are as follows: transacted by the Ma-ao Sugar administration, the fact that off and
on, there would be passed by the Board of Directors, resolutions
(1) ".... Finally, as to the Philippine Fiber, the Court takes it that ratifying all acts previously done by the management, e.g.
defendants admit having invested P655,000.00 in shares of stock of resolutions passed on February 25, 1947, and February 25, 1952, by
this company but that this was ratified by the Board of Directors in the Board of Directors as set forth in the affidavit of Isidro T. Dunca p.
Resolutions 60 and 80, Exhibits "R" and "R-2"; more than that, 127, etc. Vol. 1. (Decision, pp. 239-241 of Record on Appeal.)
defendants contend that since said company was engaged in the
manufacture of sugar bags it was perfectly legitimate for Ma-ao xxx xxx xxx
Sugar either to manufacture sugar bags or invest in another
corporation engaged in said manufacture, and they quote authorities (2) "On the other hand, the Court has noted against plaintiffs that
for the purpose, pp. 28-31, memorandum; the Court is persuaded to their contention that Ma-ao Sugar is on the verge of bankruptcy has
believe that the defendants on this point are correct, because while not been clearly shown; against this are Exh. C to Exh. C-3 perhaps
Sec. 17-1/2 of the Corporation Law provides that: the best proof that insolvency is still far is that this action was filed in
1953 and almost seven years have passed since then without the
No corporation organized under this act shall invest its funds company apparently getting worse than it was before; ..." (Decision,
in any other corporation or business or for any purpose other pp. 243-244, supra.)
than the main purpose for which it was organized unless its
board of directors has been so authorized in a resolution by

550
xxx xxx xxx Going to the discriminatory acts of J. Amado Araneta, namely,
manipulation of cane allotments, withholding of molasses and
(3) "As to the crop loan anomalies in that instead of giving unto the alcohol shares, withholding of trucking allowance, formation of rival
planters the entire amount alloted for that, the Central withheld a planters associations, refusal to deal with legitimate planters group,
certain portion for their own use, as can be seen in Appendix A of Exh. S; the Court notices that as to the failure to provide hauling
Exh. C-1, while the theory of plaintiffs is that since between the transportation, this in a way is corroborated by Exh. 7, that part
amount of P3,791,551.78 the crop loan account payable, and the containing the decision of the Court of First Instance of Manila, civil
amount of P1,708,488.22, the crop loan receivable, there is a 20122, Francisco Rodriguez v. Ma-ao Sugar; for the reason,
difference of P2,083,063.56, this would indicate that this latter sum however, that even if these were true, those grievances were
had been used by the Central itself for its own purposes; on the other grievances of plaintiffs as planters and not as stockholders just as
hand, defendants contend that the first amount did not represent the the grievance as to the crop loans already adverted to, this Court
totality of the crop loans obtained from the Bank for the purpose of will find insufficient merit on this count. (Decision, pp. 230-
relending to the planters, but that it included the Central's own credit 231, supra.)
line on its 40% share in the standing crop; and that this irregularity
amounts to a grievance by plaintiffs as planters and not as xxx xxx xxx
stockholders, the Court must find that as to this count, there is really
reason to find that said anomaly is not a clear basis for the derivative (4) "...; for the Court must admit its limitations and confess that it
suit, first, because plaintiffs' evidence is not very sufficient to prove cannot pretend to know better than the Board in matters where the
clearly the alleged diversion in the face of defendants' defense; there Board has not transgressed any positive statute or by-law especially
should have been a showing that the Central had no authority to where as here, there is the circumstance that presumably, an
make the diversion; and secondly, if the anomaly existed, there is impartial representative in the Board of Directors, the one from the
ground to hold with defendants that it was an anomaly pernicious not Philippine National Bank, against whom apparently plaintiffs have
to the Central but to the planters; it was not even pernicious to the no quarrel, does not appear to have made any protest against the
stockholders. same; the net result will be to hold that the culpable acts proved are
not enough to secure a dissolution; the Court will only order the
correction of abuses, proved as already mentioned; nor will the Court
grant any more damages one way or the other. (Decision, p.
244, supra.)

On the other hand, the errors assigned in the appeal of the defendants as
appellants are as follows:

I.

THE LOWER COURT ERRED IN ADJUDGING J. AMADO


ARANETA TO PAY TO MA-AO SUGAR CENTRAL CO., INC., THE

551
AMOUNT OF P46,270.00, WITH 8% INTEREST FROM THE DATE October, 1953, it was resolved to collect 8%, the Court does not see
OF FILING OF THE COMPLAINT. how such a unilateral action of the Board could bind the borrowers.
Be it stated that defendants have presented in evidence Exh. 5
II. photostatic copy of the page in loan receivable and it is sought to be
proved that J. Amado Araneta's debt was totally paid on 31 October,
THE LOWER COURT ERRED IN NOT ORDERING THE 1953; to the Court, in the absence of definite primary proof of actual
PLAINTIFFS TO PAY THE DEFENDANTS, PARTICULARLY J. payment having found out that there had already been a juggling of
AMADO ARANETA, THE DAMAGES PRAYED FOR IN THE books, it cannot just believe that the amount had been paid as noted
COUNTERCLAIM OF SAID DEFENDANTS. in the books. (Decision, pp. 233-235 of Record on Appeal.)

The portions of the Decision of the Lower Court assailed by the defendants (2) "With respect to the second point in the motion for
as appellants are as follows: reconsideration to the effect that the Court did not make any findings
of fact on the counterclaim of defendants, although the Court did not
say that in so many words, the Court takes it that its findings of fact
(1) "As to the alleged juggling of books in that the personal account
on pages 17 to 21 of its decision were enough to justify a dismissal
of J. Amado Araneta of P46,270.00 was closed on October 31, 1947
of the counterclaim, because the counterclaims were based on the
by charges transferred to loans receivable nor was interest paid on
fact that the complaint was premature, improper, malicious and that
this amount, the Court finds that this is related to charge No. 1,
the language is unnecessarily vituperative abusive and insulting; but
namely, the granting of personal loans to J. Amado Araneta; it is
the Court has not found that the complaint is premature; nor has the
really true that according to the books, and as admitted by
Court found that the complaint was malicious; these findings can be
defendants, J. Amado Araneta secured personal loans; in 1947, the
gleaned from the decision with respect to the allegation that the
cash advance to him was P132,082.00 (Exh. A); the Court has no
complaint was abusive and insulting, the Court does not concur; for it
doubt that this was against the By-Laws which provided that:
has not seen anything in the evidence that would justify a finding that
plaintiffs and been actuated by bad faith, nor is there anything in the
The Directors shall not in any case borrow money from the complaint essentially libelous; especially as the rule is that
Company. (Sec. III, Art. 7); allegations in pleading where relevant, are privileged even though
they may not clearly proved afterwards; so that the Court has not
the Court therefore finds this count to be duly proved; worse, the seen any merit in the counterclaims; and the Court had believed that
Court also finds that as plaintiffs contend, while the books of the the decision already carried with it the implication of the dismissal of
Corporation would show that the last balance of P46,270.00 was the counterclaims, but if that is not enough, the Court makes its
written off as paid, as testified to by Auditor Mr. Sanchez, the position clear on this matter in this order, and clarifies that it has
payment appeared to be nothing more than a transfer of his loan dismissed the counterclaims of defendant; ..." (Order of September
receivable account, stated otherwise, the item was only transferred 3, 1960, pp. 248-249, supra.)
from the personal account to the loan receivable account, so that
again the Court considers established the juggling of the books; and Regarding Assignment of Errors Nos. 2, 3 and 4 contained in the brief of the
then again, it is also true that the loans were secured without any plaintiffs as appellants, it appears to us that the Lower Court was correct in
interest and while it is true that in the Directors' meeting of 21 its appreciation (1) that the evidence presented did not show that the

552
defendant Ma-ao Sugar Company was insolvent (2) that the alleged authorizing the investment; and that it was only on November 26, 1951, that
discriminatory acts committed by the defendant Central against the planters the President of Ma-ao Sugar Central Co., Inc., was so authorized by the
were not a proper subject of derivative suit, but, at most, constituted a cause Board of Directors.
of action of the individual planters; and (3) that the acts of mismanagement
complained of and proved do not justify a dissolution of the corporation. In addition, 355,000 shares of stock of the same Philippine Fiber Processing
Co., Inc., owned by Luzon Industrial, corporation were transferred on May
Whether insolvency exists is usually a question of fact, to be 31, 1952, to the defendant Ma-ao Sugar Central Co., Inc., with a valuation of
determined from an inventory of the assets and their value, as well P355,000.00 on the basis of P1.00 par value per share. Again the
as a consideration of the liabilities.... But the mere impairment of "investment" was made without prior board resolution, the authorizing
capital stock alone does not establish insolvency there being other resolution having been subsequentIy approved only on June 4, 1952.
evidence as to the corporation being a going concern with sufficient
assets. Also, the excess of liabilities over assets does not establish Plaintiffs-appellants also contend that even assuming, arguendo, that the
insolvency, when other assets are available. (Fletcher Cyc. of the said Board Resolutions are valid, the transaction, is still wanting in legality,
Law of Private Corporations, Vol. 15A, 1938 Ed pp. 34-37; Emphasis no resolution having been approved by the affirmative vote of stockholders
supplied). holding shares in the corporation entitling them to exercise at least two-thirds
of the voting power, as required in Sec. 17- of the Corporation Law.
But relief by dissolution will be awarded in such cases only where no
other adequate remedy is available, and is not available where the The legal provision invoked by the plaintiffs, as appellants, Sec. 17- of the
rights of the stockholders can be, or are, protected in some other Corporation Law, provides:
way. (16 Fletcher Cyc. Corporations, 1942 Ed., pp. 812-813, citing
"Thwing v. McDonald", 134 Minn. 148, 156 N.W. 780, 158 N.W. 820, No corporation organized under this act shall invest its funds in any
159 N.W. 564, Ann. Cas. 1918 E 420; Mitchell v. Bank of St. Paul, 7 other corporation or business, or for any purpose other than the main
Minn. 252). purpose for which it was organized, unless its board of directors has
been so authorized in a resolution by the affirmative vote of
The First Assignment of Error in the brief of the plaintiffs as appellants, stockholders holding shares in the corporation entitling them to
contending that the investment of corporate funds by the Ma-ao Sugar Co., exercise at least two-thirds of the voting power on such proposal at a
Inc., in another corporation (the Philippine Fiber Processing Co., Inc.) stockholders' meeting called for the purpose ....
constitutes a violation of Sec. 17- of the Corporation Law, deserves
consideration. On the other hand, the defendants, as appellees, invoked Sec. 13, par. 10 of
the Corporation Law, which provides:
Plaintiffs-appellants contend that in 1950 the Ma-ao Sugar Central Co., Inc.,
through its President, J. Amado Araneta,, subscribed for P300,000.00 worth SEC. 13. Every corporation has the power:
of capital stock of the Philippine Fiber Processing Co. Inc., that payments on
the subscription were made on September 20, 1950, for P150,000.00, on
xxx xxx xxx
April 30, 1951, for P50,000.00, and on March 6, 1952, for P100,000.00; that
at the time the first two payments were made there was no board resolution

553
(9) To enter into any obligation or contract essential to the proper such holdings shall be solely for investment and not for the purpose
administration of its corporate affairs or necessary for the proper of bringing about a monopoly in any line of commerce or combination
transaction of the business or accomplishment of the purpose for in restraint of trade. (The Philippine Corporation Law by Sulpicio S.
which the corporation was organized; Guevara, 1967 Ed., p. 89.) (Emphasis ours.)lawphi1.nt

(10) Except as in this section otherwise provided, and in order to 40. Power to invest corporate funds. A private corporation has the
accomplish its purpose as stated in the articles of incorporation, to power to invest its corporate funds in any other corporation or
acquire, hold, mortgage, pledge or dispose of shares, bonds, business, or for any purpose other than the main purpose for which it
securities and other evidences of indebtedness of any domestic or was organized, provided that 'its board of directors has been so
foreign corporation. authorized in a resolution by the affirmative vote of stockholders
holding shares in the corporation entitling them to exercise at least
A reading of the two afore-quoted provisions shows that there is need for two-thirds of the voting power on such a proposal at a stockholders'
interpretation of the apparent conflict. meeting called for that purpose,' and provided further, that no
agricultural or mining corporation shall in anywise be interested in
In his work entitled "The Philippine Corporation Law," now in its 5th edition, any other agricultural or mining corporation. When the investment is
Professor Sulpicio S. Guevara of the University of the Philippines, College of necessary to accomplish its purpose or purposes as stated in it
Law, a well-known authority in commercial law, reconciled these two articles of incorporation, the approval of the stockholders is not
apparently conflicting legal provisions, as follows: necessary. (Id., p. 108.) (Emphasis ours.)

j. Power to acquire or dispose of shares or securities. A private We agree with Professor Guevara.
corporation, in order to accomplish its purpose as stated in its
articles of incorporation, and subject to the limitations imposed by the We therefore agree with the finding of the Lower Court that the investment in
Corporation Law, has the power to acquire, hold, mortgage, pledge question does not fall under the purview of Sec. 17- of the Corporation
or dispose of shares, bonds, securities, and other evidences of Law.
indebtedness of any domestic or foreign corporation. Such an act, if
done in pursuance of the corporate purpose, does not need the With respect to the defendants' assignment of errors, the second (referring to
approval of the stockholders; but when the purchase of shares of the counterclaim) is clearly without merit. As the Lower Court aptly ruled in its
another corporation is done solely for investment and not to Order of September 3, 1960 (resolving the defendants' Motion for
accomplish the purpose of its incorporation, the vote of approval of Reconsideration) the findings of fact were enough to justify a dismissal of the
the stockholders is necessary. In any case, the purchase of such counterclaim, "because the counterclaims were based on the fact that the
shares or securities must be subject to the limitations established by complaint was premature, improper, malicious and that the language is
the Corporation Law; namely, (a) that no agricultural or mining unnecessarily vituperative abusive and insulting; but the Court has not found
corporation shall in anywise be interested in any other agricultural or that the complaint is premature; nor has the Court found that the complaint
mining corporation; or (b) that a non-agricultural or non-mining was malicious; these findings can be gleaned from the decision; with respect
corporation shall be restricted to own not more than 15% of the to the allegation that the complaint was abusive and insulting, the Court does
voting stock of any agricultural or mining corporation; and (c) that not concur; for it has not seen anything in the evidence that would justify a

554
finding that plaintiffs had been actuated by bad faith, nor is there anything in
the complaint essentially libelous especially as the rule is that allegations in
pleadings where relevant, are privileged even though they may not be clearly
proved afterwards; ..."
G.R. No. 76801 August 11, 1995
As regards defendants' first assignment of error, referring to the status of the
account of J. Amado Araneta in the amount of P46,270.00, this Court LOPEZ REALTY, INC., AND ASUNCION LOPEZ GONZALES, petitioners,
likewise agrees with the finding of the Lower Court that Exhibit 5, photostatic vs.
copy of the page on loans receivable does not constitute definite primary FLORENTINA FONTECHA, ET AL., AND THE NATIONAL LABOR
proof of actual payment, particularly in this case where there is evidence that RELATIONS COMMISSION, respondents.
the account in question was transferred from one account to another. There
is no better substitute for an official receipt and a cancelled check as
evidence of payment.

PUNO, J.:
In the judgment, the lower court ordered the management of the Ma-ao
Sugar Central Co., Inc. "to refrain from making investments in Acoje Mining,
Mabuhay Printing and any other company whose purpose is not connected The controversy at bench arose from a complaint filed by private
with the sugar central business." This portion of the decision should be respondents, 1 namely, Florentina Fontecha, Mila Refuerzo, Marcial Mamaril,
reversed because, Sec. 17- of the Corporation Law allows a corporation to Perfecto Bautista, Edward Mamaril, Marissa Pascual and Allan Pimentel,
"invest its fund in any other corporation or business, or for any purpose other against their employer Lopez Realty Incorporated (petitioner) and its majority
than the main purpose for which it was organized," provided that its board of stockholder, Asuncion Lopez Gonzales, for alleged non-payment of their
directors has been so authorized by the affirmative vote of stockholders gratuity pay and other benefits. 2 The case was docketed as NLRC-NCR
holding shares entitling them to exercise at least two-thirds of the voting Case No. 2-2176-82.
power.
Lopez Realty, Inc., is a corporation engaged in real estate business, while
IN VIEW OF ALL THE FOREGOING, that part of the judgment which orders petitioner Asuncion Lopez Gonzales is one of its majority shareholders. Her
the Ma-ao Sugar Central Co., Inc. "to refrain from making investments in interest in the company vis-a-vis the other shareholders is as follows:
Acoje Mining, Mabuhay Printing, and any other: company whose purpose is
not connected with the sugar central business," is reversed. The other parts
of the judgment are, affirmed. No special pronouncement as to costs. 1 Asuncion Lopez Gonzales 7831 shares

Concepcion, C.J., Reyes, J.B.L., Dizon, Zaldivar, Castro, Fernando and


Barredo, JJ., concur.
2 Teresita Lopez Marquez 7830 shares
Makalintal, Sanchez and Teehankee, JJ., took no part.

555
the amount of P157,750.00 as Gratuity Fund covering the period
3 Arturo F. Lopez 7830 shares from 1950 up to 1980.

Meanwhile, on July 28, 1981, board member and majority


stockholder Teresita Lopez Marquez died.
4 Rosendo de Leon 4 shares
On August 17, 1981, except for Asuncion Lopez Gonzales who was
then abroad, the remaining members of the Board of Directors,
5 Benjamin Bernardino 1 share namely: Rosendo de Leon, Benjamin Bernardino, and Leo Rivera,
convened a special meeting and passed a resolution which reads:

Resolved, as it is hereby resolved that the gratuity (pay) of


6 Leo Rivera 1 share
the employees be given as follows:

(a) Those who will be laid off be given the full amount of
Except for Arturo F. Lopez, the rest of the shareholders also sit as gratuity;
members of the Board of Directors.
(b) Those who will be retained will receive 25% of their
As found by the Labor arbiter. 3 sometime in 1978, Arturo Lopez gratuity (pay) due on September 1, 1981, and another 25%
submitted a proposal relative to the distribution of certain assets of on January 1, 1982, and 50% to be retained by the office in
petitioner corporation among its three (3) main shareholders. The the meantime. (emphasis supplied)
proposal had three (3) aspects, viz: (1) the sale of assets of the
company to pay for its obligations; (2) the transfer of certain assets Private respondents were the retained employees of petitioner
of the company to its three (3) main shareholders, while some other corporation. In a letter, dated August 31, 1981, private respondents
assets shall remain with the company; and (3) the reduction of requested for the full payment of their gratuity pay. Their request was
employees with provision for their gratuity pay. The proposal was granted in a special meeting held on September 1, 1981. The
deliberated upon and approved in a special meeting of the board of relevant, portion of the minutes of the said board meeting reads:
directors held on April 17, 1978.
In view of the request of the employees contained in the
It appears that petitioner corporation approved two (2) resolutions letter dated August 31, 1981, it was also decided that, all
providing for the gratuity pay of its employees, viz: (a) Resolution those remaining employees will receive another 25% (of
No. 6, Series of 1980, passed by the stockholders in a special their gratuity) on or before October 15, 1981 and another
meeting held on September 8, 1980, resolving to set aside, twice a 25% on or before the end of November, 1981 of their
year, a certain sum of money for the gratuity pay of respective gratuity.
its retiring employees and to create a Gratuity Fund for the said
contingency; and (b) Resolution No. 10,Series of 1980, setting aside

556
At that, time, however, petitioner Asuncion Lopez Gonzales was still the private respondents was a mere "mistake" on the part of
abroad. Allegedly, while she was still out of the country, she sent a petitioner corporation since, pursuant to Resolution No. 6, dated
cablegram to the corporation, objecting to certain matters taken up September 8, 1980, and Resolution No. 10, dated October 6, 1980,
by the board in her absence, such as the sale of some of the assets said gratuity pay should be given only upon the employees'
of the corporation. Upon her return, she flied a derivative suit with the retirement.
Securities and Exchange Commission (SEC) against majority
shareholder Arturo F. Lopez. On November 20, 1985, public respondent, through its Second
Division, dismissed the appeal for lack of merit, the pertinent portion
Notwithstanding the "corporate squabble" between petitioner of which states: 6
Asuncion Lopez Gonzales and Arturo Lopez, the first two (2)
installments of the gratuity pay of private respondents Florentina We cannot agree with the contention of respondents
Fontecha, Mila Refuerzo, Marcial Mamaril and Perfecto Bautista (petitioners') that the Labor Arbiter a quo committed abuse of
were paid by petitioner corporation. discretion in his decision.

Also, petitioner corporation had prepared the cash vouchers and Respondents' (petitioners') contention that, the two (2)
checks for the third installments of gratuity pay of said private resolutions dated 17 August 1981 and 1 September 1981 . . .
respondents (Florentina Fontecha, Mila Refuerzo, Marcial Mamaril which were not approved in the annual stockholders meeting
and Perfecto Bautista). For some reason, said vouchers were had no force and effect, deserves scant consideration. The
cancelled by petitioner Asuncion Lopez Gonzales. records show that the stockholders did not revoke nor nullify
these resolutions granting gratuities to complainants.
Likewise, the first, second and third installments of gratuity pay of the
rest of private respondents, particularly, Edward Mamaril, Marissa On record, it appears that the said resolutions arose from the
Pascual and Allan Pimentel, were prepared but cancelled by legitimate creation of the Board of Directors who steered the
petitioner Asuncion Lopez Gonzales. Despite private respondents' corporate affairs of the corporation. . . .
repeated demands for their gratuity pay, corporation refused to pay
the same. 4 Respondents' (petitioners') allegation that the three (3)
complainants, Mila E. Refuerzo, Marissa S. Pascual and
On July 23, 1984, Labor Arbiter Raymundo R. Valenzuela rendered Edward Mamaril, who had resigned after filing the complaint
judgment in favor of private respondents. 5 on February 8, 1982, were precluded to (sic) receive gratuity
because the said resolutions referred to
Petitioners appealed the adverse ruling of the Labor arbiter to public only retiring employee could not be given credence. A
respondent National Labor Relations Commission. The appeal reading of Resolutions dated 17 August 1981 and 1
focused on the alleged non-ratification and non-approval of the September 1981 disclosed that there were periods
assailed August 17, 1981 and September 1, 1981 Board Resolutions mentioned for the payment of complainants' gratuities. This
during the Annual Stockholders' Meeting held on March 1, 1982. disproves respondents' argument allowing gratuities upon
Petitioners further insisted that the payment of the gratuity to some of retirement of employees. Additionally, the proposed

557
distribution of assets (Exh. C-1) filed by Mr. Arturo F. Lopez Petitioners reconsidered. 7 In their motion for reconsideration,
also made mention of gratuity pay, " . . . (wherein) an petitioners assailed the validity of the board resolutions passed on
employee who desires to resign from the LRI will be given August 17, 1981 and September 1, 1981, respectively, and claimed,
the gratuity pay he or she earned." (Emphasis supplied) Let for the first time, that petitioner Asuncion Lopez Gonzales was not
us be reminded, too, that the complainants' resignation was notified of the special board meetings held on said dates. The motion
not voluntary but it was pressurized (sic) due to "power for reconsideration was denied by the Second Division on July 24,
struggle" which was evident between Arturo Lopez and 1986.
Asuncion Gonzales.
On September 4, 1986, petitioners filed another motion for
The respondents' (petitioners') contention of a mistake to reconsideration. Again, the motion was denied by public respondent
have been committed in granting the first two (2) installments in a Minute Resolution dated November 19, 1986. 8
of gratuities to complainants Perfecto Bautista, Florentina
Fontecha, Marcial Mamaril and Mila Refuerzo, (has) no legal Hence, the petition. As prayed for, we issued a Temporary
leg to stand on. The record is bereft of any evidence that the Restraining Order, 9 enjoining public respondent from enforcing or
Board of Directors had passed a resolution nor is there any executing the Resolution, dated November 20, 1986 (sic), in NLRC-
minutes of whatever nature proving mistakes in the award of NCR-2-2176-82. 10
damages (sic).
The sole issue is whether or not public respondent acted with grave
With regard to the award of service incentive leave and abuse of discretion in holding that private respondents are entitled to
others, the Commission finds no cogent reason to disturb the receive their gratuity pay under the assailed board resolutions dated
appealed decision. August 17, 1951 and September 1, 1981.

We affirm. Petitioners contend that the board resolutions passed on August 17,
1981 and September 1, 1981, granting gratuity pay to their retained
WHEREFORE, let the appealed decision be, as it is hereby, employees, are ultra vires on the ground that petitioner Asuncion
AFFIRMED and let the instant appeal (be) dismissed for lack Lopez Gonzales was not duly notified of the said special meetings.
of merit. They aver, further, that said board resolutions were not ratified by the
stockholders of the corporation pursuant to Section 28 1/2 of the
SO ORDERED. Corporation Law (Section 40 of the Corporation Code). They also
insist that the gratuity pay must be given only to the retiring
employees, to the exclusion of the retained employees or those who
voluntarily resigned from their posts.

At the outset, we note that petitioners allegation on lack of notice to


petitioner Asuncion Lopez Gonzales was raised for the first time in
the in their motion for reconsideration filed before public respondent

558
National Labor Relations Commission, or after said public The general rule is that a corporation, through its board of directors,
respondent had affirmed the decision of the labor arbiter. To stress, should act in the manner and within the formalities, if any, prescribed
in their appeal before the NLRC, petitioners never raised the issue of by its charter or by the general law. 14 Thus, directors must act as a
lack of notice to Asuncion Lopez Gonzales. The appeal dealt with (a) body in a meeting called pursuant to the law or the corporation's by-
the failure of the stockholders to ratify the assailed resolutions and laws, otherwise, any action taken therein may be questioned by any
(b) the alleged "mistake" committed by petitioner corporation in objecting director or shareholder. 15
giving the gratuity pay to some of its employees who are yet to retire
from employment. Be that as it may, jurisprudence 16 tells us that an action of the board
of directors during a meeting, which was illegal for lack of notice,
In their comment, 11 private respondents maintain that the new may be ratified either expressly, by the action of the directors in
ground of lack of notice was not raised before the labor arbiter, subsequent legal meeting, or impliedly, by the corporation's
hence, petitioners are barred from raising the same on appeal. subsequent course of conduct. Thus, in one case, 17 it was held:
Private respondents claim, further, that such failure on the part of
petitioners, had deprived them the opportunity to present evidence . . . In 2 Fletcher, Cyclopedia of the Law of Private
that, in a subsequent special board meeting held on September 29, Corporations (Perm. Ed.) sec. 429, at page 290, it is stated:
1981, the subject resolution dated September 1, 1981, was
unanimously approved by the board of directors of petitioner Thus, acts of directors at a meeting which
corporation, including petitioner Asuncion Lopez Gonzales. 12 was illegal because of want of notice may be
ratified by the directors at a subsequent
Indeed, it would be offensive to the basic rules of fair play and justice legal meeting, or by the corporations course
to allow petitioners to raise questions which have not been passed of conduct
upon by the labor arbiter and the public respondent NLRC. It is well ...
settled that questions not raised in the lower courts cannot, be raised
for the first time on appeal. 13 Hence, petitioners may not invoke any Fletcher, supra, further states in sec. 762, at page 1073-
other ground, other than those it specified at the labor arbiter level, to 1074:
impugn the validity of the subject resolutions.
Ratification by directors may be by an
We now come to petitioners' argument that the resolutions passed by express resolution or vote to that effect, or it
the board of directors during the special meetings on August 1, 1981, may be implied from adoption of the act,
and September 1, 1981, were ultra vires for lack of notice. acceptance or acquiescence. Ratification
may be effected by a resolution or vote of
the board of directors expressly ratifying
previous acts either of corporate officers or
agents; but it is not necessary, ordinarily, to
show a meeting and formal action by the

559
board of directors in order to establish a 1981 and September 1, 1981, it is erroneous to state that the
ratification. resolutions passed by the board during the said meetings were ultra
vires. In legal parlance, "ultra vires" act refers to one which is not
In American Casualty Co., v. Dakota Tractor and Equipment within the corporate powers conferred by the Corporation Code or
Co., 234 F. Supp. 606, 611 (D.N.D. 1964), the court stated: articles of incorporation or not necessary or incidental in the exercise
of the powers so conferred. 19
Moreover, the unauthorized acts of an officer
of a corporation may be ratified by the The assailed resolutions before us cover a subject which concerns
corporation by conduct implying approval the benefit and welfare of the company's employees. To stress,
and adoption of the act in question. Such providing gratuity pay for its employees is one of the express powers
ratification may be express or may be of the corporation under the Corporation Code, hence, petitioners
inferred from silence and inaction. cannot invoke the doctrine of ultra vires to avoid any liability arising
from the issuance the subject resolutions. 20
In the case at bench, it was established that petitioner corporation
did not issue any resolution revoking nor nullifying the board We reject petitioners' allegation that private respondents, namely,
resolutions granting gratuity pay to private respondents. Instead, Mila Refuerzo, Marissa Pascual and Edward Mamaril who resigned
they paid the gratuity pay, particularly, the first two (2) installments from petitioner corporation after the filing of the case, are precluded
thereof, of private respondents Florentina Fontecha, Mila Refuerzo, from receiving their gratuity pay. Pursuant to board resolutions dated
Marcial Mamaril and Perfecto Bautista. August 17, 1981 and September 1, 1981, respectively, petitioner
corporation obliged itself to give the gratuity pay of its retained
Despite the alleged lack of notice to petitioner Asuncion Lopez employees in four (4) installments: on September 1, 1981; October
Gonzales at that time the assailed resolutions were passed, we can 15, 1981; November, 1981; and January 1, 1982. Hence, at the time
glean from the records that she was aware of the corporation's the aforenamed private respondents tendered their resignation, the
obligation under the said resolutions. More importantly, she aforementioned private respondents were already entitled to receive
acquiesced thereto. As pointed out by private respondents, petitioner their gratuity pay.
Asuncion Lopez Gonzales affixed her signature on Cash Voucher
Nos. 81-10-510 and 81-10-506, both dated October 15, 1981, Petitioners try to convince us that the subject resolutions had no
evidencing the 2nd installment of the gratuity pay of private force and effect in view of the non-approval thereof during the Annual
respondents Mila Refuerzo and Florentina Fontecha. 18 Stockholders' Meeting held on March 1, 1982. To strengthen their
position, petitioners cite section 28 1/2 of the Corporation Law
We hold, therefore, that the conduct of petitioners after the passage (Section 40 of the Corporation Code). We are not persuaded.
of resolutions dated August, 17, 1951 and September 1, 1981, had
estopped them from assailing the validity of said board resolutions. The cited provision is not applicable to the case at bench as it refers
to the sale, lease, exchange or disposition of all or substantially all of
Assuming, arguendo, that there was no notice given to Asuncion the corporation's assets, including its goodwill. In such a case, the
Lopez Gonzalez during the special meetings held on August 17,

560
action taken by the board of directors requires the authorization of Taada, Teehankee and Carreon for plaintiffs-appellants.
the stockholders on record. Hilado and Hilado for defendant-appellee.

It will be observed that, except far Arturo Lopez, the stockholders of REYES, J.B.L., J.:
petitioner corporation also sit as members of the board of directors.
Under the circumstances in field, it will be illogical and superfluous to Appeal on points of law from a judgment of the Court of First Instance of
require the stockholders' approval of the subject resolutions. Thus, Occidental Negros, in its Civil Case No. 2603, dismissing plaintiff's complaint
even without the stockholders' approval of the subject resolutions, that sought to compel the defendant Milling Company to increase plaintiff's
petitioners are still liable to pay private respondents' gratuity pay. share in the sugar produced from their cane, from 60% to 62.33%, starting
from the 1951-1952 crop year.1wph1.t
IN VIEW WHEREOF, the instant petition is DISMISSED for lack of
merit and the temporary restraining order we issued on February 9, It is undisputed that plaintiffs-appellants, Alfredo Montelibano, Alejandro
1987 is LIFTED. Accordingly, the assailed resolution of the National Montelibano, and the Limited co-partnership Gonzaga and Company, had
Labor Relations Commission in NLRC-NCR-2176-82 is AFFIRMED. been and are sugar planters adhered to the defendant-appellee's sugar
This decision is immediately executory. Costs against petitioners. central mill under identical milling contracts. Originally executed in 1919, said
contracts were stipulated to be in force for 30 years starting with the 1920-21
SO ORDERED. crop, and provided that the resulting product should be divided in the ratio of
45% for the mill and 55% for the planters. Sometime in 1936, it was
Narvasa, C.J., Regalado, Mendoza and Francisco, JJ., concur. proposed to execute amended milling contracts, increasing the planters'
share to 60% of the manufactured sugar and resulting molasses, besides
other concessions, but extending the operation of the milling contract from
the original 30 years to 45 years. To this effect, a printed Amended Milling
Contract form was drawn up. On August 20, 1936, the Board of Directors of
the appellee Bacolod-Murcia Milling Co., Inc., adopted a resolution (Acts No.
11, Acuerdo No. 1) granting further concessions to the planters over and
above those contained in the printed Amended Milling Contract. The bone of
contention is paragraph 9 of this resolution, that reads as follows:

ACTA No. 11
SESSION DE LA JUNTA DIRECTIVA
AGOSTO 20, 1936
G.R. No. L-15092 May 18, 1962
xxx xxx xxx
ALFREDO MONTELIBANO, ET AL., plaintiffs-appellants,
vs. Acuerdo No. 1. Previa mocion debidamente secundada, la Junta
BACOLOD-MURCIA MILLING CO., INC., defendant-appellee. en consideracion a una peticion de los plantadores hecha por un

561
comite nombrado por los mismos, acuerda enmendar el contrato de After trial, the court below rendered judgment upholding the stand of the
molienda enmendado medientelas siguentes: defendant Milling company, and dismissed the complaint. Thereupon,
plaintiffs duly appealed to this Court.
xxx xxx xxx
We agree with appellants that the appealed decisions can not stand. It must
9.a Que si durante la vigencia de este contrato de Molienda be remembered that the controverted resolution was adopted by appellee
Enmendado, lascentrales azucareras, de Negros Occidental, cuya corporation as a supplement to, or further amendment of, the proposed
produccion anual de azucar centrifugado sea mas de una tercera milling contract, and that it was approved on August 20, 1936, twenty-one
parte de la produccion total de todas lascentrales azucareras de days prior to the signing by appellants on September 10, of the Amended
Negros Occidental, concedieren a sus plantadores mejores Milling Contract itself; so that when the Milling Contract was executed, the
condiciones que la estipuladas en el presente contrato, entonces concessions granted by the disputed resolution had been already
esas mejores condiciones se concederan y por el presente se incorporated into its terms. No reason appears of record why, in the face of
entenderan concedidas a los platadores que hayan otorgado este such concessions, the appellants should reject them or consider them as
Contrato de Molienda Enmendado. separate and apart from the main amended milling contract, specially taking
into account that appellant Alfredo Montelibano was, at the time, the
Appellants signed and executed the printed Amended Milling Contract on President of the Planters Association (Exhibit 4, p. 11) that had agitated for
September 10, 1936, but a copy of the resolution of August 10, 1936, signed the concessions embodied in the resolution of August 20, 1936. That the
by the Central's General Manager, was not attached to the printed contract resolution formed an integral part of the amended milling contract, signed on
until April 17, 1937; with the notation September 10, and not a separate bargain, is further shown by the fact that a
copy of the resolution was simply attached to the printed contract without
special negotiations or agreement between the parties.
Las enmiendas arriba transcritas forman parte del contrato de
molienda enmendado, otorgado por y la Bacolod-Murcia Milling
Co., Inc. It follows from the foregoing that the terms embodied in the resolution of
August 20, 1936 were supported by the same causa or consideration
underlying the main amended milling contract; i.e., the promises and
In 1953, the appellants initiated the present action, contending that three
obligations undertaken thereunder by the planters, and, particularly, the
Negros sugar centrals (La Carlota, Binalbagan-Isabela and San Carlos), with
extension of its operative period for an additional 15 years over and beyond
a total annual production exceeding one-third of the production of all the
the 30 years stipulated in the original contract. Hence, the conclusion of the
sugar central mills in the province, had already granted increased
court below that the resolution constituted gratuitous concessions not
participation (of 62.5%) to their planters, and that under paragraph 9 of the
supported by any consideration is legally untenable.
resolution of August 20, 1936, heretofore quoted, the appellee had become
obligated to grant similar concessions to the plaintiffs (appellants herein). The
appellee Bacolod-Murcia Milling Co., inc., resisted the claim, and defended All disquisition concerning donations and the lack of power of the directors of
by urging that the stipulations contained in the resolution were made without the respondent sugar milling company to make a gift to the planters would be
consideration; that the resolution in question was, therefore, null and void ab relevant if the resolution in question had embodied a separate
initio, being in effect a donation that was ultra vires and beyond the powers of agreement after the appellants had already bound themselves to the terms of
the corporate directors to adopt. the printed milling contract. But this was not the case. When the resolution
was adopted and the additional concessions were made by the company, the

562
appellants were not yet obligated by the terms of the printed contract, since Much is made of the circumstance that the report submitted by the Board of
they admittedly did not sign it until twenty-one days later, on September 10, Directors of the appellee company in November 19, 1936 (Exhibit 4) only
1936. Before that date, the printed form was no more than a proposal that made mention of 90%, the planters having agreed to the 60-40 sharing of the
either party could modify at its pleasure, and the appellee actually modified it sugar set forth in the printed "amended milling contracts", and did not make
by adopting the resolution in question. So that by September 10, 1936 any reference at all to the terms of the resolution of August 20, 1936. But a
defendant corporation already understood that the printed terms were not reading of this report shows that it was not intended to inventory all the
controlling, save as modified by its resolution of August 20, 1936; and we are details of the amended contract; numerous provisions of the printed terms
satisfied that such was also the understanding of appellants herein, and that are alao glossed over. The Directors of the appellee Milling Company had no
the minds of the parties met upon that basis. Otherwise there would have reason at the time to call attention to the provisions of the resolution in
been no consent or "meeting of the minds", and no binding contract at all. question, since it contained mostly modifications in detail of the printed
But the conduct of the parties indicates that they assumed, and they do not terms, and the only major change was paragraph 9 heretofore quoted; but
now deny, that the signing of the contract on September 10, 1936, did give when the report was made, that paragraph was not yet in effect, since it was
rise to a binding agreement. That agreement had to exist on the basis of the conditioned on other centrals granting better concessions to their planters,
printed terms as modified by the resolution of August 20, 1936, or not at all. and that did not happen until after 1950. There was no reason in 1936 to
Since there is no rational explanation for the company's assenting to the emphasize a concession that was not yet, and might never be, in effective
further concessions asked by the planters before the contracts were signed, operation.
except as further inducement for the planters to agree to the extension of the
contract period, to allow the company now to retract such concessions would There can be no doubt that the directors of the appellee company had
be to sanction a fraud upon the planters who relied on such additional authority to modify the proposed terms of the Amended Milling Contract for
stipulations. the purpose of making its terms more acceptable to the other contracting
parties. The rule is that
The same considerations apply to the "void innovation" theory of appellees.
There can be no novation unless two distinct and successive binding It is a question, therefore, in each case of the logical relation of the
contracts take place, with the later designed to replace the preceding act to the corporate purpose expressed in the charter. If that act is
convention. Modifications introduced before a bargain becomes obligatory one which is lawful in itself, and not otherwise prohibited, is done for
can in no sense constitute novation in law. the purpose of serving corporate ends, and is reasonably tributary to
the promotion of those ends, in a substantial, and not in a remote
Stress is placed on the fact that the text of the Resolution of August 20, 1936 and fanciful sense, it may fairly be considered within charter powers.
was not attached to the printed contract until April 17, 1937. But, except in The test to be applied is whether the act in question is in direct and
the case of statutory forms or solemn agreements (and it is not claimed that immediate furtherance of the corporation's business, fairly incident to
this is one), it is the assent and concurrence (the "meeting of the minds") of the express powers and reasonably necessary to their exercise. If
the parties, and not the setting down of its terms, that constitutes a binding so, the corporation has the power to do it; otherwise, not. (Fletcher
contract. And the fact that the addendum is only signed by the General Cyc. Corp., Vol. 6, Rev. Ed. 1950, pp. 266-268)
Manager of the milling company emphasizes that the addition was made
solely in order that the memorial of the terms of the agreement should be full As the resolution in question was passed in good faith by the board of
and complete. directors, it is valid and binding, and whether or not it will cause losses or
decrease the profits of the central, the court has no authority to review them.

563
They hold such office charged with the duty to act for the corporation
according to their best judgment, and in so doing they cannot be
63.5% for 1955-56,
controlled in the reasonable exercise and performance of such duty.
Whether the business of a corporation should be operated at a loss
during depression, or close down at a smaller loss, is a purely
business and economic problem to be determined by the directors of the appellee Bacolod-Murcia Milling Company is, under the terms of its
the corporation and not by the court. It is a well-known rule of law Resolution of August 20, 1936, duty bound to grant similar increases to
that questions of policy or of management are left solely to the plaintiffs-appellants herein.
honest decision of officers and directors of a corporation, and the
court is without authority to substitute its judgment of the board of WHEREFORE, the decision under appeal is reversed and set aside; and
directors; the board is the business manager of the corporation, and judgment is decreed sentencing the defendant-appellee to pay plaintiffs-
so long as it acts in good faith its orders are not reviewable by the appellants the differential or increase of participation in the milled sugar in
courts. (Fletcher on Corporations, Vol. 2, p. 390). accordance with paragraph 9 of the appellee Resolution of August 20, 1936,
over and in addition to the 60% expressed in the printed Amended Milling
And it appearing undisputed in this appeal that sugar centrals of La Carlota, Contract, or the value thereof when due, as follows:
Hawaiian Philippines, San Carlos and Binalbagan (which produce over one-
third of the entire annual sugar production in Occidental Negros) have 0,333% to appellants Montelibano for the 1951-1952 crop year, said
granted progressively increasing participations to their adhered planter at an appellants having received an additional 2% corresponding to said
average rate of year in October, 1953;

2.333% to appellant Gonzaga & Co., for the 1951-1952 crop year;
62.333% for the 1951-52 crop year; and to all appellants thereafter
4.2% for the 1952-1953 crop year;
4.3% for the 1953-1954 crop year;
4.5% for the 1954-1955 crop year;
64.2% for 1952-53; 3.5% for the 1955-1956 crop year;

with interest at the legal rate on the value of such differential during the time
they were withheld; and the right is reserved to plaintiffs-appellants to sue for
64.3% for 1953-54; such additional increases as they may be entitled to for the crop years
subsequent to those herein adjudged.

Costs against appellee, Bacolod-Murcia Milling Co.


64.5% for 1954-55; and
Padilla, Bautista Angelo, Labrador, Concepcion, Barrera, Paredes and Dizon,
JJ., concur.

564
G.R. No. L-5377 December 29, 1954

MARIA CLARA PIROVANA ET AL., plaintiffs-appellees,


vs.
THE DE LA RAMA STEAMSHIP CO., defendant-appellant.

Del Rosario and Garcia for appellant.


Vicente J. Francisco for appellees.

BAUTISTA ANGELO, J.:

565
This is an appeal from a decision of the Court of First Instance of Rizal Fourth. That the condition mentioned in the donation is null and
declaring the donation made by the defendant in favor of the minor children void because it depends on the provisions of Article 1115 of the old
of the late Enrico Pirovano of the proceeds of the insurance policies taken on Civil Code.
his life valid and binding, and ordering said defendant to pay to said minor
children the sum of P583,813.59, with interest thereon at the rate of per cent Fifth. That if the condition is valid, its non-fulfillment is due to the
from the date of filing of the complaint, plus an additional amount equivalent desistance of the defendant company from obeying and doing the
to 20 per cent of said sum of P538,813.59 as damages by way of attorney's wishes and mandates of the majority of the stockholders.
fees and the costs of action.
Sixth. That the non-payment of the debt in favor of the National
Plaintiffs herein are the minor children of the late Enrico Pirovano Development Company is not due to the lack of funds, nor to lack of
represented by their mother and judicial guardian Estefania R. Pirovano. authority, but the desire of the President of the corporation to
They seek to enforce certain resolutions adopted by the Board of Directors preserve and continue the Government participation in the company.
and stockholders of the defendant company giving to said minor children of
the proceeds of the insurance policies taken on the life of their deceased Seventh. That due demands were made by the plaintiffs and their
father Enrico Pirovano with the company as beneficiary. Defendant's main attorneys and these demands were rejected for no justifiable or legal
defense is: that said resolutions and the contract executed pursuant thereto grounds.
are ultra vires, and, if valid, the obligation to pay the amount given is not yet
due and demandable.
The important facts which need to be considered for purposes of this appeal
may be briefly stated as follows: Defendant is a corporation duly organized in
The trial court resolved all the issues raised by the parties in favor of the accordance with law with an authorized capital of P500,000, divided into
plaintiffs and, after considering the evidence, both oral and documentary, 5,000 shares, with a par value of P100 each share. The stockholders were:
arrived at the following conclusions: Esteban de la Rama, 1,800 shares, Leonor de la Rama, 100 shares,
Estefania de la Rama, 100 shares, and Eliseo Hervas, Tomas Concepcion,
First. That the contract executed between the plaintiffs and the Antonio G. Juanco, and Gaudencio Volasote with 5 shares each. Leonor and
defendant is a renumerative donation. Estefania are daughters of Don Esteban, while the rest his employees.
Estefania de la Rama was married to the late Enrico Pirovano and to them
Second. That said contract or donation is not ultra vires, but an four children were born who are the plaintiffs in this case.
act executed within the powers of the defendant corporation in
accordance with its articles of incorporation and by laws, sanctioned Enrico Pirovano became the president of the defendant company and under
and approved by its Board of Directors and stockholders; and his management the company grew and progressed until it became a multi-
subsequently ratified by other subsequent acts of the defendant million corporation by the time Pirovano was executed by the Japanese
company. during the occupation. On May 13, 1941, the capital stock of the corporation
was increased to P2,000,000, after which a 100 per cent stock dividend was
Third. That the said donation is in accordance with the trend of declared. Subsequently, or before the outbreak of the war , new stock
modern and more enlightened legislation in its treatment of questions dividends of 200 per cent and 33 1/3 per cent were again declared. On
between labor and capital. December 4, 1941, the capital stock was once more increased to

566
P5,000,000. Under Pirovano's management, the assets of the company grew carried out on September 23, 1949, when the National Development
and increased from an original paid up capital of around P240,000 to Company executed a "Deed of Termination of Trust and Release of
P15,538,024.37 by September 30, 1941 (Exhibit HH). Mortgage" in favor of the De la Rama company (Exhibit 6.) The immediate
effect of this conversion was the released from incumbrance of all the
In the meantime, Don Esteban de la Rama, who practically owned and properties Of Don Esteban and of the Hijos de I. de la Rama and Co., Inc.,
controlled the stock of the defendant corporation, distributed his shareholding which was apparently favorable to the interests of the De la Rama company,
among his five daughters, namely, Leonor, Estefania, Lourdes, Lolita and but, on the other hand, it resulted in the inconvenience that, as holder of the
Conchita and his wife Natividad Aguilar so that, at that time, or on July 10, preferred stock, the National Development Company, was given to the right
1946, the stockholding of the corporation stood as follows: Esteban de la to 40 per cent of the membership of the Board of Directors of the De la Rama
Rama, 869 shares, Leonor de la Rama, 3,375 shares, Estefania de la Rama, company, which meant an increase in the representation of the National
3,368 shares, Lourdes de la Rama, 3,368 shares, Lolita de la Rama, 3,368 Development Company from 2 to 4 of the 9 members of said Board of
shares, Conchita de la Rama, 3,376 shares, and Natividad Aguilar, 2,136 Directors.
shares. The other stockholders , namely, Eliseo Hervas, Tomas Concepcion,
Antonio Juanco, and Jose Aguilar, who were merely employees of Don The first resolution granting to the Pirovano children the proceeds of the
Esteban, were given 40 shares each, while Pio Pedrosa, Marcial P. Lichauco insurance policies taken on his life by the defendant company was adopted
and Rafael Roces, one share each, because they merely represented the by the Board of Directors at a meeting held on July 10, 1946, (Exhibit B).
National Development Company. This Company was given representation in This grant was called in the resolution as "Special Payment to Minor Heirs of
the Board Of Directors of the corporation because at that time the latter had the late Enrico Pirovano". Because of its direct hearing on the issues
an outstanding bonded indebtedness to the National Development Company. involved in this case, said resolution is hereunder reproduced in toto:

This bonded indebtedness was incurred on February 26, 1940 and was in SPECIAL PAYMENT TO MINORS HEIRS OF THE LATE ENRICO
the amount of P7,500.00. The bond held by the National Development PIROVANO
Company was redeemable within a period of 20 years from March 1, 1940,.
bearing interest at the rate of 5 per cent per annum. To secure said bonded The President stated that the principal purpose for which the meeting
indebtedness, all the assets of the De la Rama Steamship Co., Inc., and had been called was to discuss the advisability of making some form
properties of Don Esteban de la Rama, as well as those of the Hijos de I. de of compensation to the minor heirs of the late Enrico Pirovano,
la Rama and Co., Inc., a sister corporation owned by Don Esteban and his former President and General Manager of the Company. As every
family, were mortgaged to the National Development Company (Annexes A, member of the Board knows, said the President, the late Enrico
B, C, D of Exhibit 3, Deed of Trust). Payments made by the corporation Pirovano who was largely responsible for the very successful
under the management of Pirovano reduced this bonded indebtedness to development of the activities of the Company prior to war was killed
P3,260,855.77. by the Japanese in Manila sometime in 1944 leaving as his only
heirs four minor children, Maria Carla, Esteban, Enrico and John
Upon arrangement made with the National Development Company, the Albert. Early in 1941, explained the President, the Company had
outstanding bonded indebtedness was converted into non-voting preferred insured the life of Mr. Pirovano for a million pesos. Following the
shares of stock of the De la Rama company under the express condition that occupation of the Philippines by Japanese forces the Company was
they would bear affixed cumulative dividend of 6 per cent per annum and unable to pay the premiums on those policies issued by Filipino
would be redeemable within 15 years (Exhibits 5 and 7). This conversion was companies and these policies had lapsed. But with regards to the

567
York Office of the De la Rama Steamship Co., Inc. had kept up Whereas, this company paid premium on Mr. Pirovano's life
payment of the premiums from year to year. The payments made on insurance policies for a period of only 4 years so that it will receive
account of these premiums, however, are very small compared to the from the insurance companies sums of money greatly in excess of
amount which the Company will now receive as a result of Mr. the premiums paid by this company.
Pirovano's death. The President proposed therefore that out of the
proceeds of these policies the sum of P400,000 be set aside for the Be it resolved, That out of the proceeds to be collected from the life
minor children of the deceased, said sum of money to be convertible insurance policies on the life of the late Enrico Pirovano, the sum of
into 4,000 shares of the stock of the Company, at par, or 1,000 P400,000 be set aside for equal division among the 4 minor children
shares for each child. This proposal, explained the President as of the deceased, to wit: Esteban, Maria Carla, Enrico and John
being made by him upon suggestion of President Roxas, but, he Albert, all surnamed Pirovano, which sum of money shall be
added, that he himself was very much in favor of it also. On motion convertible into shares of stock of the De la Rama Steamship
of Miss Leonor de la Rama duly seconded by Mrs. Lourdes de la Company, at par and, for that purpose, that the present registered
Rama de Osmea, the following resolution was, thereupon, stockholders of the corporation be requested to waive their
unanimously approved: preemptive right to 4,000 shares of the unissued stock of the
company in order to enable each of the 4 minor heirs of the
Whereas, the late Enrico Pirovano, President and General Manager deceased, to wit: Esteban, Maria Carla, Enrico and John Albert, all
of the De la Rama Steamship Company, died in Manila sometime in surnamed Pirovano, to obtain 1,000 shares at par;
November, 1944:
Resolved, further, that in view of the fact that under the provisions of
Whereas, the said Enrico Pirovano was largely responsible for the the indenture with the National Development Company, it is
rapid and very successful development of the activities of thus necessary that action herein proposed to be confirmed by the Board
company; of Directors of that company, the Secretary is hereby instructed to
send a copy of this resolution to the proper officers of the National
Whereas, early in 1941 this company insured the life of said Enrico Development Company for appropriate action. (Exhibit B)
Pirovano in various Philippine and American Life Insurance
companies for the total sum of P1,000,000; The above resolution, which was adopted on July 10, 1946, was submitted to
the stockholders of the De la Rama company at a meeting properly
Whereas, the said Enrico Pirovano is survived by his widow, convened, and on that same date, July 10, 1946, the same was duly
Estefania Pirovano and four minor children, to wit: Esteban, Maria approved.
Carla, Enrico and John Albert, all surnamed Pirovano;lawphil.net
It appears that, although Don Esteban and the Members of his family were
Whereas, said Enrico Pirovano left practically nothing to his heirs agreeable to giving to the Pirovano children the amount of P400,000 out of
and it is but fit proper that this company which owes so much to the the proceeds of the insurance policies taken on the life of Enrico Pirovano,
deceased should make some provision for his children; they did not realize that when they provided in the above referred two
resolutions that said Amount should be paid in the form of shares of stock,
they would be actually giving to the Pirovano children more than what they

568
intended to give. This came about when Lourdes de la Rama, wife of Sergio approximately P5,000,000" (Exhibit C). This resolution was concurred in by
Osmea, Jr., showed to the latter copies of said resolutions and asked him to the representatives of the National Development Company. The pertinent
explain their import and meaning, and it was value then that Osmea portion of the resolution reads as follows:
explained that because the value then of the shares of stock was actually 3.6
times their par value, the donation their value, the donation, although Be resolved, that out of gratitude to the late Enrico Pirovano this
purporting to be only P400,00, would actually amount to a total of Company renounce as it hereby renounces, all of his right, title, and
P1,440,000. He further explained that if the Pirovano children would given interest as beneficiary in and to the proceeds of the abovementioned
shares of stock in lieu of the amount to be donated, the voting strength of the life insurance policies in favor of Esteban, Maria Carla, Enrico and
five daughters of Don Esteban in the company would be adversely affected John Albert, all surnamed Pirovano, subject to the terms and
in the sense that Mrs. Pirovano would be adversely affected in the sense that conditions herein after provided;
Mrs. Pirovano would have a voting power twice as much as that of her
sisters. This caused Lourdes de la Rama to write to the secretary of the That the proceeds of said insurance policies shall be retained by the
corporation, Atty. Marcial Lichauco, asking him to cancel the waiver she Company in the nature of a loan drawing interest at the rate of 5 per
supposedly gave of her pre-emptive rights. Osmea elaborated on this cent annum from the date of receipt of payment by the Company
matter at the annual meeting of the stockholders held on December 12, 1946 from the various insurance companies above-mentioned until the
but at said meeting it was decided to leave the matter in abeyance pending time the time the same amounts are paid to the minor heirs of Enrico
further action on the part of the members of the De la Rama family. Pirovano previously mentioned;

Osmea, in the meantime, took up the matter with Don Esteban and, as That all amounts received from the above-mentioned policies shall
consequence, the latter, on December 30, 1946, addressed to Marcial be divided equally among the minors heirs of said Enrico Pirovano;
Lichauco a letter stating, among other things, that "in view of the total lack of
understanding by me and my daughters of the two Resolutions
That the company shall proceed to pay the proceeds of said
abovementioned, namely, Directors' and Stockholders' dated July 10, 1946,
insurance policies plus interests that may have accrued to each of
as finally resolved by the majority of the Stockholders and Directors present
the heirs of the said Enrico Pirovano or their duly appointed
yesterday, that you consider the abovementioned resolutions nullified."
representatives after the Company shall have first settled in full the
(Exhibit CC).
balance of its present remaining bonded indebtedness in the sum of
the approximately P5,000,000.
On January 6, 1947, the Board of Directors of the De la Rama company, as a
consequence of the change of attitude of Don Esteban, adopted a resolution
The above resolution was carried out by the company and Mrs. Estefania R.
changing the form of the donation to the Pirovano children from a donation of
Pirovano, the latter acting as guardian of her children, by executing a
4,000 shares of stock as originally planned into a renunciation in favor of the
Memorandum Agreement on January 10, 1947 and June 17, 1947,
children of all the company's "right, title, and interest as beneficiary in and to
respectively, stating therein that the De la Rama Steamship Co., Inc., shall
the proceeds of the abovementioned life insurance policies", subject to the
enter in its books as a loan the proceeds of the life insurance policies taken
express condition that said proceeds should be retained by the company as
on the life of Pirovano totalling S321,500, which loan would earn interest at
a loan drawing interest at the rate of 5 per cent per annum and payable to
the rate of 5 per cent per annum. Mrs. Pirovano, in executing the agreement,
the Pirovano children after the company "shall have first settled in full the
balance of its present remaining bonded indebtedness in the sum of

569
acted with the express authority granted to her by the court in an order dated liquidated its present bonded indebtedness in the amount of
March 26, 1947. P3,260,855.77 with The National Development Company, or fully
redeemed the preferred shares of stock in the amount which shall be
On June 24, 1947, the Board of Directors approved a resolution providing issued to the National Development Company in lieu thereof;
therein that instead of the interest on the loan being payable, together with
the principal, only after the company shall have first settled in full its bonded 2. That any and all taxes, legal fees, and expenses in any way
indebtedness, said interest may be paid to the Pirovano children "whenever connected with the above transaction shall be chargeable and
the company is in a position to met said obligation" (Exhibit D), and on deducted from the proceeds of the life insurance policies mentioned
February 26, 1948, Mrs. Pirovano executed a public document in which she in the resolutions of the Board of Directors. (Exhibit E)
formally accepted the donation (Exhibit H). The Dela Rama company took
"official notice" of this formal acceptance at a meeting held by its Board of Sometime in March 1950, the President of the corporation, Sergio Osmea,
Directors on February 26, 1948. Jr., addressed an inquiry to the Securities and Exchange Commission asking
for opinion regarding the validity of the donation of the proceeds of the
In connection with the above negotiations, the Board of Directors took up at insurance policies to the Pirovano children. On June 20, 1950 that office
its meeting on July 25, 1949, the proposition of Mrs. Pirovano to buy the rendered its opinion that the donation was void because the corporation
house at New Rochelle, New York, owned by the Demwood Realty, a could not dispose of its assets by gift and therefore the corporation acted
subsidiary of the De la Rama company at its original costs of $75,000, which beyond the scope of its corporate powers. This opinion was submitted to the
would be paid from the funds held in trust belonging to her minor children. Board of Directors at its meting on July 12, 1950, on which occasion the
After a brief discussion relative to the matter, the proposition was approved in president recommend that other legal ways be studied whereby the donation
a resolution adopted on the same date. could be carried out. On September 14, 1950, another meeting was held to
discuss the propriety of the donation. At this meeting the president expressed
The formal transfer was made in an agreement signed on September 5, 1949 the view that, since the corporation was not authorized by its charter to make
by Mrs. Pirovano, as guardian of her children, and by the De la Rama the donation to the Pirovano children and the majority of the stockholders
company, represented by its new General Manager, Sergio Osmea, Jr. The was in favor of making provision for said children, the manner he believed
transfer of this property was approved by the court in its order of September this could be done would be to declare a cash dividend in favor of the
20, 1949.lawphil.net stockholders in the exact amount of the insurance proceeds and thereafter
have the stockholders make the donation to the children in their individual
On September 13, 1949, or two years and 3 months after the donation had capacity. Notwithstanding this proposal of the president, the board took no
been approved in the various resolutions herein above mentioned, the action on the matter, and on March 8, 1951, at a stockholders' meeting
stockholders of the De la Rama company formally ratified the donation convened on that date the majority of the stockholders' voted to revoke the
(Exhibit E), with certain clarifying modifications, including the resolution resolution approving the donation to the Pirovano children. The pertinent
approving the transfer of the Demwood property to the Pirovano children. portion of the resolution reads as follows:
The clarifying modifications are quoted hereunder:
Be it resolved, as it is hereby resolved, that in view of the failure of
1. That the payment of the above-mentioned donation shall not be compliance with the above conditions to which the above donation
affected until such time as the Company shall have first duly was made subject, and in view of the opinion of the Securities and
Exchange Commissioner, the stockholders revoke, rescind and

570
annul, as they do thereby revoke, rescind and annul, its ratification prevented the fulfillment of the condition precedent to the payment of said
and approval on September 13, 1949 of the aforementioned donation such that it can be said it has forfeited its right to demand its
resolution of the Board of Directors of January 6, 1947, as amended fulfillment and has made the donation entirely due and demandable?
on June 24, 1947. (Exhibit T)
We will discuss these issues separately.
In view of the resolution declaring that the corporation failed to comply with
the condition set for the effectivity of the donation and revoking at the same 1. To determine the nature of the grant made by the defendant corporation to
time the approval given to it by the corporation, and considering that the the minor children of the late Enrico Pirovano, we do not need to go far nor
corporation can no longer set aside said donation because it had no longer dig into the voluminous record that lies at the bottom of this case. We do not
set aside said donation because it had long been perfected and even need to inquire into the interest which has allegedly been shown by
consummated, the minor children of the late Enrico Pirovano, represented by President Roxas in the welfare of the children of his good friend Enrico
their mother and guardian, Estefania R. de Pirovano, demanded the payment Pirovano. Whether President Roxas has taken the initiative in the move to
of the credit due them as of December 31, 1951, amounting to P564,980.89, give something to said children which later culminated in the donation now in
and this payment having been refused, they instituted the present action in dispute, is of no moment for the fact is that, from the mass of evidence on
the Court of First Instance of Rizal wherein they prayed that the be granted hand, such a donation has been given the full indorsement and encouraging
an alternative relief of the following tenor: (1) sentencing defendant to pay to support by Don Esteban de la Rama who was practically the owner of the
the plaintiff the sum of P564,980.89 as of December 31, 1951, with the corporation. We only need to fall back to accomplish this purpose on the
corresponding interest thereon; (2) as an alternative relief, sentencing several resolutions of the Board of Directors of the corporations containing
defendant to pay to the plaintiffs the interests on said sum of P564,980.89 at said grant for they clearly state the reasons and purposes why the donation
the rate of 5 per cent per annum, and the sum of P564,980.89 after the has been given.
redemption of the preferred shares of the corporation held by the National
Development Company; and (3) in any event, sentencing defendant to pay Before we proceed further, it is convenient to state here in passing that,
the plaintiffs damages in the amount of not less than 20 per cent of the sum before the Board of Directors had approved its resolution of January 6, 1947,
that may be adjudged to the plaintiffs, and the costs of action. as later amended by another resolution adopted on June 24, 1947, the
corporation had already decided to give to the minor children of the late
The only issues which in the opinion of the court need to be determined in Enrico Pirovano the sum of P400,000 out of the proceeds of the insurance
order to reach a decision in this appeal are: (1) Is the grant of the proceeds policies taken on his life in the form of shares, and that when this form was
of the insurance policies taken on the life of the late Enrico Pirovano as considered objectionable because its result and effect would be to give to
embodied in the resolution of the Board of Directors of defendant corporation said children a much greater amount considering the value then of the stock
adopted on January 6, 1947 and June 24, 1947 a remunerative donation as of the corporation, the Board of Directors decided to amend the donation in
found by the lower court?; (2) IN the affirmative case, has that donation been the form and under the terms stated in the aforesaid resolutions. Thus, in the
perfected before its rescission or nullification by the stockholders of the original resolution approved by the Board of Directors on July 10, 1946,
corporation on March 8, 1951?; (3) Can defendant corporation give by way of wherein the reasons for granting the donation to the minor children of the late
donation the proceeds of said insurance policies to the minor children of the Enrico Pirovano were clearly, we find out the following revealing statements:
late Enrico Pirovano under the law or its articles of corporation, or is that
donation an ultra vires act?; and (4) has the defendant corporation, by the
acts it performed subsequent to the granting of the donation, deliberately

571
Whereas, the late Enrico Pirovano President and General Manager Whereas, early in 1941, the life of the said Enrico Pirovano was
of the De la Rama Steamship Company, died in Manila sometime in insured in various life companies, to wit:
November, 1944;
Whereas, the said Enrico Pirovano is survived by 4 minor children, to
Whereas, the said Enrico Pirovano was largely responsible for the wit: Esteban, Maria Carla, Enrico and John Albert, all surnamed
rapid and very successful development of the activities of this Pirovano; and
company;
Whereas, the said Enrico Pirovano left practically nothing to his heirs
Whereas, early in 1941 this company insured the life of said Enrico and it is but fit and proper that this Company which owes so much to
Pirovano in various Philippine and American Life Insurance the deceased should make some provision for his children;
companies for the total sum of P1,000,000;
Be it resolved, that out of gratitude to the late Enrico Pirovano this
Whereas, the said Enrico Pirovano is survived by his widow, Company renounce as it hereby renounces, . . . .
Estefania Pirovano and 4 minor children, to wit: Esteban, Maria
Carla, Enrico and John Albert, all surnamed Pirovano; From the above it clearly appears that the corporation thought of giving the
donation to the children of the late Enrico Pirovano because he "was to a
Whereas, the said Enrico Pirovano left practically nothing to his heirs large extent responsible for the rapid and very successful development and
and it is but fit and proper that this company which owes so much to expansion of the activities of this company"; and also because he "left
the deceased should make some provisions for his children; practically nothing to his heirs and it is but fit and proper that this company
which owes so much to the deceased should make some provision to his
Whereas, this company paid premiums on Mr. Pirovano's life children", and so, the donation was given "out of gratitude to the late Enrico
insurance policies for a period of only 4 years so that it will receive Pirovano." We do not need to stretch our imagination to see that a grant or
from the insurance companies sums of money greatly in excess of donation given under these circumstances is remunerative in nature in
the premiums paid by the company, contemplation of law.

Again, in the resolution approved by the Board of Directors on January 6, That which is made to a person in consideration of his merits or for
1947, we also find the following expressive statements which are but a services rendered to the donor, provided they do not constitute
reiteration of those already expressed in the original resolution: recoverable debts, or that in which a burden less than the value of
the thing given is imposed upon the donee, is also a donation." (Art.
Whereas, the late Enrico Pirovano, President and General Manager 619, old Civil Code.)
of the De la Rama Steamship Co., Inc., died in Manila sometime
during the latter part of the year 1944; In donations made to a person for services rendered to the donor,
the donor's will is moved by acts which directly benefit him. The
Whereas, the said Enrico Pirovano was to a large extent responsible motivating cause is gratitude, acknowledgment of a favor, a desire to
for the rapid and very successful development and expansion of the compensate. A donation made to one who saved the donor's life, or a
activities of this company; lawyer who renounced his fees for services rendered to the donor,

572
would fall under this class of donations. These donations are (c) While the donation can be considered as duly executed by the execution
called remunerative donations . (Sinco and Capistrano, The Civil of the document stated in the preceding paragraph, and by the entry in the
Code, Vol. 1, p. 676; Manresa, 5th ed., pp. 72-73.) books of the corporation of the donation as a loan, a further record of said
execution was made when Mrs. Pirovano executed a public document on
2. The next question to be determined is whether the donation has been February 26, 1948 making similar acceptance of the donation. And this
perfected such that the corporation can no longer rescind it even if it wanted acceptance was officially recorded by the corporation when on the same date
to. The answer to this question cannot but be in the affirmative considering its Board of Directors approved a resolution taking "official notice" of said
that the same has not only been granted in several resolutions duly adopted acceptance.
by the Board of Directors of the defendant corporation, and in all these
corporate acts the concurrence of the representatives of the National (d) On July 25, 1949, the Board of Directors approved the proposal of Mrs.
Development Company, the only creditor whose interest may be affected by Pirovano to buy the house at New Rochelle, New York, owned by a
the donation, has been expressly given. The corporation has even gone subsidiary of the corporation at the costs of S75,000 which would be paid
further. It actually transferred the ownership of the credit subject of donation from the sum held in trust belonging to her minor children. And this
to the Pirovano children with the express understanding that the money agreement was actually carried out in a document signed by the general
would be retained by the corporation subject to the condition that the latter manager of the corporation and by Mrs. Pirovano, who acted on the matter
would pay interest thereon at the rate of 5 per cent per annum payable with the express authority of the court.
whenever said corporation may be in a financial position to do so. Thus, the
following acts of the corporation as reflected from the evidence bear this out: (e) And on September 30, 1949, or two years and 3 months after the
donation had been executed, the stockholders of the defendant corporation
(a) The donation was embodied in a resolution duly approved by the Board of formally ratified and gave approval to the donation as embodied in the
Directors on January 6, 19437. In this resolution, the representatives of the resolutions above referred to, subject to certain modifications which did not
National Development Company, have given their concurrence. This is the materially affect the nature of the donation.
only creditor which can be considered as being adversely affected by the
donation. The resolution of June 24, 1947 did not modify the substance of There can be no doubt from the foregoing relation of facts the donation was a
the former resolution for it merely provided that instead of the interest on the corporate act carried out by the corporation not only with the sanction of its
loan being payable, together with the principal, only after the corporation had Board of Directors but also of its stockholders. It is evident that the donation
first settled in full its bonded indebtedness, said interest would be paid has reached the stage of perfection which is valid and binding upon the
"whenever the company is in a position to meet said obligation." corporation and as such cannot be rescinded unless there is exists legal
grounds for doing so. In this case, we see none. The two reasons given for
(b) The resolution of January 6, 1947 was actually carried out when the the rescission of said donation in the resolution of the corporation adopted on
company and Mrs. Estefania R. Pirovano, executed a memorandum March 8, 1951, to wit: that the corporation failed to comply with the conditions
agreement stating therein hat the proceeds of the insurance policies would to which the above donation was made subject, and that in the opinion of the
be entered in the books of the corporation as a loan which would bear an Securities and Exchange Commission said donation is ultra vires, are not, in
interest at the rate of 5 per cent per annum, and said agreement was signed our opinion, valid and legal as to justify the rescission of a perfected
by Mrs. Pirovano as judicial guardian of her children after she had been donation. These reasons, as we will discuss in the latter part of this decision,
expressly authorized by the court to accept the donation in behalf of her cannot be invoked by the corporation to rescind or set at naught the
children. donation, and the only way by which this can be done is to show that the

573
donee has been in default, or that the donation has not been validly (f) To promote any company or companies for the purposes of
executed, or is illegal or ultra vires, and such is not the case as we will see acquiring all or any of the property or liabilities of this company, or
hereafter. We therefore declare that the resolution approved by the both, or for any other purpose which may seem directly or indirectly
stockholders of the defendant corporation on March 8, 1951 did not and calculated to benefit the company.
cannot have the effect of nullifying the donation in question.
(g) To invest and deal with the moneys of the company and
3. The third question to be determined is: Can defendant corporation give by immediately required, in such manner as from time to time may be
way of donation the proceeds of said insurance policies to the minor children determined.
of the late Enrico Pirovano under the law or its articles of corporation, or is
that donation an ultra vires act? To answer this question it is important for us (h) To borrow, or raise, or secure the payment of money in such
to examine the articles of incorporation of the De la Rama company to see manner as the company shall think fit.
this question it is important for us to examine the articles of incorporation of
the De la Rama company to see if the act or donation is outside of their (i) Generally, to do all such other thing and to transact all business as
scope. Paragraph second of said articles provides: may be directly or indirectly incidental or conducive to the attainment
of the above object, or any of them respectively.
Second. The purposes for which said corporation is formed are:
(j) Without in any particular limiting or restricting any of the objects
(a) To purchase, charter, hire, build, or otherwise acquire steam or and powers of the corporation, it is hereby expressly declared and
other ships or vessels, together with equipments and furniture provided that the corporation shall have power to issue bonds and
therefor, and to employ the same in conveyance and carriage of provided that the corporation shall have power to issue bonds and
goods, wares and merchandise of every description, and of other obligations, to mortgage or pledge any stocks, bonds or other
passengers upon the high seas. obligations or any property which may be required by said
corporations; to secure any bonds, guarantees or other obligations
(b) To sell, let, charter, or otherwise dispose of the said vessels or by it issued or incurred; to lend money or credit to and to aid in any
other property of the company. other manner any person, association, or corporation of which any
obligation or in which any interest is held by this corporation or in the
(c) To carry on the business of carriers by water. affairs or prosperity of which this corporation or in the affairs or
prosperity of which this corporation has a lawful interest, and to do
(d) To carry on the business of shipowners in all of its branches. such acts and things as may be necessary to protect, preserve,
improve, or enhance the value of any such obligation or interest;
and, in general, to do such other acts in connection with the
(e) To purchase or take on lease, lands, wharves, stores, lighters,
purposes for which this corporation has been formed which is
barges and other things which the company may deem necessary or
calculated to promote the interest of the corporation or to enhance
advisable to be purchased or leased for the necessary and proper
the value of its property and to exercise all the rights, powers and
purposes of the business of the company, and from time to time to
privileges which are now or may hereafter be conferred by the laws
sell the dispose of the same.
of the Philippines upon corporations formed under the Philippine

574
Corporation Act; to execute from time to time general or special one of the ships of the company who died in Japan, a gratuity of P7,000,
powers of attorney to persons, firms, associations or corporations equivalent to one month salary for each year of service. It also gave to
either in the Philippines, in the United States, or in any other country Ramon Pons, a captain of one of its ships , a retirement gratuity equivalent to
and to revoke the same as and when the Directors may determine one month salary for every year of service, the same to be based upon his
and to do any and or all of the things hereinafter set forth and to the highest salary. And it contributed P2,000 to the fund raised by the Associated
same extent as natural persons might or could do. Steamship Lines for the widow of the late Francis Gispert, secretary of said
Association, of which the De la Rama Steamship Co., Inc., was a member
After a careful perusal of the provisions above quoted we find that the along with about 30 other steamship companies. In this instance, Gispert
corporation was given broad and almost unlimited powers to carry out the was not even an employee of the corporation. And invoking this vast power,
purposes for which it was organized among them, (1) "To invest and deal the corporation even went to the extent of contributing P100,000 to the
with the moneys of the company not immediately required, in such manner Liberal Party campaign funds, apparently in the hope that by conserving its
as from time to time may be determined" and, (2) "to aid in any other manner cordial relations with that party it might continue to retain the patronage of the
any person, association, or corporation of which any obligation or in which administration. All these acts executed before and after the donation in
any interest is held by this corporation or in the affairs or prosperity of which question have never been questioned and were willingly and actually carried
this corporation has a lawful interest." The world deal is broad enough to out.
include any manner of disposition, and refers to moneys not immediately
required by the corporation, and such disposition may be made in such We don't see much distinction between these acts of generosity or
manner as from time to time may be determined by the corporations. The benevolence extended to some employees of the corporation, and even to
donation in question undoubtedly comes within the scope of this broad power some in whom the corporation was merely interested because of certain
for it is a fact appearing in the evidence that the insurance proceeds were not moral or political considerations, and the donation which the corporation has
immediately required when they were given away. In fact, the evidence seen fit to give to the children of the late Enrico Pirovano from the point of
shows that the corporation declared a 100 per cent cash dividend, or view of the power of the corporation as expressed in its articles of
P2,000,000, and later on another 30 per cent cash dividend. This is clear incorporation. And if the former had been sanctioned and had been
proof of the solvency of the corporation. It may be that, as insinuated, Don considered valid and intra vires, we see no plausible reasons why the latter
Esteban wanted to make use of the insurance money to rehabilitate the should now be deemed ultra vires. It may perhaps be argued that the
central owned by a sister corporation, known as Hijos de I. de la Rama and donation given to the children of the late Enrico Pirovano is so large and
Co., Inc., situated in Bago, Negros Occidental, but this, far from reflecting disproportionate that it can hardly be considered a pension of gratuity that
against the solvency of the De la Rama company, only shows that the funds can be placed on a par with the instances above mentioned, but this
were not needed by the corporation. argument overlooks one consideration: the gratuity here given was not
merely motivated by pure liberality or act of generosity, but by a deep sense
Under the second broad power we have the above stated, that is, to aid in of recognition of the valuable services rendered by the late Enrico Pirovano
any other manner any person in the affairs and prosperity of whom the which had immensely contributed to the growth of the corporation to the
corporation has a lawful interest, the record of this case is replete with extent that from its humble capitalization it blossomed into a multi-million
instances which clearly show that the corporation knew well its scope and corporation that it is today. In other words of the very resolutions granting the
meaning so much so that, with the exception of the instant case, no one has donation or gratuity, said donation was given not only because the company
lifted a finger to dispute their validity. Thus, under this broad grant of power, was so indebted to him that it saw fit and proper to make provisions for his
this corporation paid to the heirs of one Florentino Nonato, an engineer of children, but it did so out of a sense of gratitude. Another factor that we

575
should bear in mind is that Enrico Pirovano was not only a high official of the with less fair rivals. Thus, an incorporated fire insurance company
company but was at the same time a member of the De la Rama family, and which policies except losses from explosions may nevertheless pay
the recipient of the donation are the grandchildren of Don Esteban de la a loss from that cause when other companies are accustomed to do
Rama. This we, may say, is the motivating root cause behind the grant of this so, such liberal dealing being deemed conducive to the prosperity of
bounty. the corporation." (Modern Law of Corporations, Machen, Vol. 1, p.
81).
It may be contended that a donation is different from a gratuity. While
technically this may be so in substance they are the same. They are even So, a bank may grant a five years pension to the family at one of its
similar to a pension. Thus, it was granted for services previously rendered, officers. In all cases in this sorts, the amount of the gratuity rests
and which at the time they were rendered gave rise to no legal obligation. " entirely within the discretion of the company, unless indeed it be all
(Words and Phrases, Permanent Edition, p. 675; O'Dea vs. Cook,, 169 Pac., together out of the reason and fitness. But where the company has
306, 176 Cal., 659.) Or stated in another way, a "Gratuity is mere bounty ceased to be going concerned, this power to make gifts or present it
given by the Government in consideration or recognition or meritorious at the end. (Modern Law of Corporations, Machen, Vol. 1, p. 82.).
services and springs from the appreciation an d graciousness of the
Government", (Ilagan vs. Ilaya, G.R. No. 33507, Dec. 20 1930) or "A gratuity Payment of Gratitude out of Capital. There seems on principle no
is something given freely, or without recompense, a gift, something reason to doubt that gifts or gratuities wherever they are lawful may
voluntarily given in return for a favor or services; a bounty; a tip." Wood be paid out of capital as well as out of profits. (Modern Law of
Mercantile Co. vs. Cole, 209 S.W. 2d. 290; Mendoza vs. Dizon, 77 Phil., 533, corporations, Machen, Vol. 1 p. 83.).
43 Off. Gaz. p. 4633. We do not see much difference between this definition
of gratuity and a remunerative donation contemplated in the Civil Code. In Whether desirable to supplement implied powers of this kind by
essence they are the same. Such being the case, it may be said that this express provisions. Enough has been said to show that the implied
donation is gratuity in a large sense for it was given for valuable services powers of a corporation to give gratuities to its servants and officers,
rendered an ultra vires act in the light of the following authorities: as well as to strangers, are ample, so that there is therefore no need
to supplement them by express provisions." (modern Law of
Indeed, some cases seem to hold that the giving of a pure gratuity to Corporations, Machen, Vol. 1, p. 83.) 1
directors is ultra vires of corporation, so that it could not be legalized
even if the approval of the shareholders; but this position has no Granting arguendo that the donation given by Pirovano children is outside
sound reason to support it, and is opposed to the weight of the scope of the powers of the defendant corporation, or the scope of the
authority (Suffaker vs. Kierger's Assignee, 53 S.W. Rep. 288; !07 Ky. powers that it may exercise under the law, or it is an ultra vires act, still it may
200; 46 L.R.A. 384). said that the same can not be invalidated, or declared legally ineffective for
the reason alone, it appearing that the donation represents not only the act of
But although business corporations cannot contribute to charity or the Board of Directors but of the stockholders themselves as shown by the
benevolence, yet they are not required always to insist on the full fact that the same has been expressly ratified in a resolution duly approved
extent of their legal rights. They are not forbidden for the recognizing by the latter. By this ratification, the infirmity of the corporate act, it may has
moral obligation of which strict law takes no cognizance. They are been obliterated thereby making the cat perfectly valid and enforceable. This
not prohibited from establishing a reputation for board, liberal, is specially so if the donation is not merely executory but executed and
equitable dealing which may stand them in good stead in competition

576
consummated and no creditors are prejudice, or if there are creditors Corporate transactions which are illegal because prohibited by
affected, the latter has expressly given their confirmity. statute or against public policy are ordinarily void and unenforceable
regardless of the part performance, ratification, or estoppel; but
In making this pronouncement, advertence should made of the nature of general prohibitions against exceeding corporate powers and
the ultra vires act that is in question. A little digression needs be made on this prohibitions intended to protect a particular class or specifying the
matter to show the different legal effect that may result consequent upon the consequences of violation may not preclude enforcement of the
performance of a particular ultra vires act on the part of the corporation. may transaction and an action may be had for the part unaffected by the
authorities may be cited interpreting or defining, extent, and scope of an ultra illegality or for equitable restitution. (19 C.J.S. 421.)
vires act, but all of them are uniform and unanimous that the same may be
either an act performed merely outside the scope of the powers granted to it Generally, a transaction within corporate powers but executed in an
by it articles of incorporation, or one which is contrary to law or violative of irregular or unauthorized manner is voidable only, and may become
any principle which will void any contract whether done individually or enforceable by reason of ratification or express or implied assent by
collectively. In other words, a distinction should be made between corporate the stockholders or by reason of estoppel of the corporation or the
acts or contracts which are illegal and those which are merely ultra vires. The other party to the transaction to raise the objection, particularly
former contemplates the doing of an act which is contrary to law, morals, or where the benefits are retained
public policy or public duty, and are, like similar transactions between the
individuals void. They cannot serve as basis of a court action, nor require As appears in paragraphs 960-964 supra, the general rule is that a
validity ultra vires acts on the other hand, or those which are not illegal and corporation must act in the manner and with the formalities, if any,
void ab initio, but are merely within are not illegal and void ab initio, but are prescribed by its character or by the general law. However, a
not merely within the scope of the articles of incorporation, are merely corporation transaction or contract which is within the corporation
voidable and may become binding and enforceable when ratified by the powers, which is neither wrong in itself nor against public policy, but
stockholders. which is defective from a failure to observe in its execution a
requirement of law enacted for the benefit or protection of a certain
Strictly speaking, an ultra vires act is one outside the scope of the class, is voidable and is valid until avoided, not void until validated;
power conferred by the legislature, and although the term has been the parties for whose benefit the requirement was enacted may ratify
used indiscriminately, it is properly distinguishable from acts which it or be estoppel to assert its invalidity, and third persons acting in
are illegal, in excess or abuse of power, or executed in an good faith are not usually affected by an irregularity on the part of the
unauthorized manner, or acts within corporate powers but outside corporation in the exercise of its granted powers. (19 C.J.S., 423-24.)
the authority of particular officers or agents (19 C. J. S. 419).
It is true that there are authorities which told that ultra vires acts, or those
performed beyond the powers conferred upon the corporation either by law
or by its articles of incorporation, are not only voidable, but wholly void and of
no legal effect, and that such acts cannot be validated by ratification or be
the basis of any action in court; but such ruling does not constitute the weight
of authority, the reason being that they fail to make the important distinction
we have above adverted to. Because rule has been rejected by most of the
state courts and even by the modern treaties or corporations (7 Flethcer,

577
Cyc. Corps., 563-564). And now it can be said that the majority of the cases property in New York, U.S.A., the price of which was paid by her but of the
hold that acts which are merely ultra vires, or acts which are not illegal, may proceeds of the insurance policies given as donation. To allow the
be ratified by the stockholders of a corporation (Brooklyn Heights R. Co. vs. corporation to undo what it has done would only be most unfair but would
Brooklyn City R. Co., 135 N.Y. Supp. 1001). contravene the well-settled doctrine that the defense of ultra vires cannot be
set up or availed of in completed transactions (7 Fletcher, Cyc. Corps.
Strictly speaking, an act of a corporation outside of its character Section 3497, p. 652; 19 C.J.S., 431).
powers is just as such ultra vires where all the stockholders consent
thereto as in a case where none of the stockholders expressly or 4. We now come to the fourth and last question that the defendant
cannot be ratified so as to make it valid, even though all the corporation, by the acts it has performed subsequent to the granting of the
stockholders consent thereto; but inasmuch as the stockholders in donation, deliberately prevented the fulfillment of the condition precedent to
reality constitute the corporation, it should , it would seem, be the payment of said donation such that it can be said it has forfeited entirely
estopped to allege ultra vires, and it is generally so held where there due and demandable.
are no creditors, or the creditors are not injured thereby, and where
the rights of the state or the public are not involved, unless the act is It should be recalled that the original resolution of the Board of Directors
not only ultra vires but in addition illegal and void. of course, such adopted on July 10, 1946 which provided for the donation of P400,000 out of
consent of all the stockholders cannot adversely affect creditors of the proceeds which the De la Rama company would collect on the insurance
the corporation nor preclude a proper attack by the state because of policies taken on the life of the late Enrico Pirovano was, as already stated
such ultra vires act. (7 Fletcher Corp., Sec. 3432, p. 585) above, amended on January 6, 1947 to include, among the conditions
therein provided, that the corporation shall proceed to pay said amount, as
Since it is not contended that the donation under consideration is illegal, or well as the interest due thereon, after it shall have settled in full balance of its
contrary to any of the express provision of the articles of incorporation, nor bonded indebtedness in the sum of P5,000,000. It should be recalled that on
prejudicial to the creditors of the defendant corporation, we cannot but September 13, 1949, or more than 2 years after the last amendment referred
logically conclude, on the strength of the authorities we have quoted above, too above, the stockholders adopted another resolution whereby they
that said donation, even if ultravires in the supposition we have adverted to, formally ratified said donation but subject to the following clarifications: (1)
is not void, and if voidable its infirmity has been cured by ratification and that the amount of the donation shall not be effected until such time as the
subsequent acts of the defendant corporation. The defendant corporation, company shall have first duly liquidated its present bonded indebtedness in
therefore, is now prevented or estopped from contesting the validity of the the amount of P3,260,855.77 to the National Development Company, or shall
donation. This is specially so in this case when the very directors who have first fully redeemed the preferred shares of stock in the amount to be
conceived the idea of granting said donation are practically the stockholders issued to said company in lieu thereof, and (2) that any and all taxes, legal
themselves, with few nominal exception. This applies to the new stockholder fees, and expenses connected with the transaction shall be chargeable from
Jose Cojuangco who acquired his interest after the donation has been made the proceeds of said insurance policies.
because of the rule that a "purchaser of shares of stock cannot avoid ultra
vires acts of the corporation authorized by its vendor, except those done after The trial court, in considering these conditions in the light of the acts
the purchase" (7 Fletcher, Cyc. Corps. section 3456, p. 603; Pascual vs. Del subsequently performed by the corporation in connection with the proceeds
Saz Orozco, 19 Phil., 82.) Indeed, how can the stockholders now pretend to of the insurance policies, considered said conditions null and void, or at most
revoke the donation which has been partly consummated? How can the not written because in its pinion their non-fulfillment was due to a deliberate
corporation now set at naught the transfer made to Mrs. Pirovano of the desistance of the corporation and not to lack of funds to redeem the

578
preferred shares of the National Development Company. The conclusions It is plain from the text of the above resolution that the defendant corporation
arrived at by the trial court on this point are as follows: had 15 years from February 18, 1949, or until 1964, within which to effect the
redemption of the preferred shares issued to the National Development
Fourth. that the condition mentioned in the donation is null and Company. This condition cannot but be binding and obligatory upon the
void because it depends on the exclusive will of the donor, in donees, if they desire to maintain the validity of the donation, for it is not only
accordance with the provisions of Article 1115 of the Old Civil Code. the basis upon which the stockholders of the defendant corporation
expressed their willingness to ratify the donation, but it is also by way which
Fifth. That if the condition is valid, its non-fulfillment is due to the its creditor, the National Development Company, would want it to be. If the
desistance of the defendant company from obeying and doing the defendant corporation is given 15 years within which to redeem the preferred
wishes and mandate of the majority of the stockholders. shares, and that period would expire in 1964, one cannot blame the
corporation for availing itself of this period if in its opinion it would redound to
its best interest. It cannot therefore be said that the fulfillment of the condition
Sixth. That the non-payment of the debt in favor of the National
for the payment of the donation is one that wholly depends on the exclusive
Development Company is due to the lack of funds, nor to lack of
will of the donor, as the lower court has concluded, simply because it failed to
authority, but to the desire of the President of the corporation to
meet the redemption of said shares in her manner desired by the donees.
preserve and continue the Government participation in the company.
While it may be admitted that because of the disposition of the assets of the
corporation upon the suggestion of its general manager more than enough
To this views of the trial court, we fail to agree. There are many factors we funds had been raised to effect the immediate redemption of the above
can consider why the failure to immediately redeem the preferred shares shares, it is not correct to say that the management has completely failed in
issued to the National Development Company as desired by the minor its duty to pay its obligations for, according to the evidence, a substantial
children of the late Enrico Pirovano cannot or should not be attributed to a portion of the indebtedness has been paid and only a balance of about
mere desire on the part of the corporation to delay the redemption, or to P1,805,169.98 was outstanding when the stockholders of the corporation
prejudice the interest of the minors, but rather to protect the interest of the decided to revoke or cancel the donation. (Exhibit P.)
corporation itself. One of them is the text of the very resolution approved by
the National Development Company on February 18, 1949 which prescribed
But there are other good reasons why all the available funds have not been
the terms and conditions under which it expressed its conformity to the
actually applied to the redemption of the preferred shares, one of them being
conversion of the bonded indebtedness into preferred shares of stock. The
the "desire of the president of the corporation to preserve and continue the
text of the resolution above mentioned reads:
government participation in the company" which even the lower court found it
to be meritorious, which is one way by which it could continue receiving the
Resolved: That the outstanding bonded indebtedness of the Dela patronage and protection of the government. Another reason is that the
Rama Steamship Co., Inc., in the approximate amount of redemption of the shares does not depend on the will of the corporation
P3,260,855.77 be converted into non-voting preferred shares of alone but to a great extent on the will of a third party, the National
stock of said company, said shares to bear a fixed dividend of 6 Development Company. In fact, as the evidence shows, this Company had
percent per annum which shall be cumulative and redeemable within pledged these shares to the Philippine National Bank and the Rehabilitation
15 years. Said shares shall be preferred as to assets in the event of Finance Corporation as a security to obtain certain loans to finance the
liquidation or dissolution of said company but shall be non- purchase of certain ships to be built for the use of the company under
participating. management contract entered into between the corporation and the National

579
Development Company, and this was what prevented the corporation from Having reached the foregoing conclusion, we deem it unnecessary to discuss
carrying out its offer to pay the sum P1,956,513.07 on April 5, 1951. Had this the other issues raised by the parties in their briefs.
offer been accepted, or favorably acted upon by the National Development
Company, the indebtedness would have been practically liquidated, leaving The lower court adjudicated to plaintiff an additional amount equivalent to 20
outstanding only one certificate worth P217,390.45. Of course, the per cent of the amount claimed as damages by way of attorney's fees, and in
corporation could have insisted in redeeming the shares if it wanted to even our opinion, this award can be justified under Article 2208, paragraph 2, of
to the extent of taking a court action if necessary to force its creditor to the new Civil Code, which provides: "When the defendant's act or omission
relinquish the shares that may be necessary to accomplish the redemption, has compelled the plaintiff to litigate with third persons or to incur expenses
but such would be a drastic step which would have not been advisable to protect his interest", attorney's fees nay be awarded as damages.
considering the policy right along maintained by the corporation to preserve However, the majority believes that this award should be reduced to 10 per
its cordial and smooth relation with the government. At any rate, whether cent.
such attitude be considered as a mere excuse to justify the delay in effecting
the redemption of the shares, or a mere desire on the part of the corporation Wherefore, the decision appealed from should be modified as follows: (a)
to retain in its possession more funds available to attend to other pressing that the donation made in favor of the children of the late Enrico Pirovano of
need as demanded by the interest of the corporation, we fail to see in such the proceeds of the insurance policies taken on his life is valid and binding on
an attitude an improper motive to circumvent the early realization of the the defendant corporation, (b) that said donation, which amounts to a total of
desire of the minors to obtain the immediate payment of the donation which P583,813.59, including interest, as it appears in the books of the corporation
was made dependent upon the redemption of said shares there being no as of August 31, 1951, plus interest thereon at the rate of 5 per cent per
clear evidence that may justify such design. Anyway, a great portion of the annum from the filing of the complaint, should be paid to the plaintiffs after
funds went to the stockholders themselves by way of dividends to offset, so it the defendant corporation shall have fully redeemed the preferred shares
appears, the huge advances that the corporation had made to them which issued to the National Development Company under the terms and
were entered in the books of the corporation as loans and, therefore, they conditions stated in the resolutions of the Board of Directors of January 6,
were invested for their own benefit. As General Manager Osmea said, "we 1947 and June 24, 1947, as amended by the resolution of the stockholders
were first confronted with the problem of the withdrawals of the family which adopted on September 13,1949; and (c) defendant shall pay to plaintiffs an
had to be repaid back to the National Development Company and one of the additional amount equivalent to 10 per cent of said amount of P583,813.59
most practical solutions to that was to declare dividends and reduce the as damages by way of attorney's fees, and to pay the costs of action.
amounts of their withdrawals", which then totalled about P3,000,000.
Paras, C. J., Pablo Bengzon, Padilla, Montemayor, Jugo, Concepcion, and
All things considered, we are of the opinion that the finding of the lower court Reyes, J. B. L., concur.
that the failure of the defendant corporation to comply with the condition of Reyes, A., concurs in the result.
the donation is merely due to its desistance from obeying the mandate of the
majority of the stockholders and not to lack of funds, or to lack of authority,
has no foundation in law or in fact, and, therefore, its conclusion that
because of such desistance that condition should be deemed as fulfilled and
the payment of the donation due and demandable, is not justified. In this
respect, the decision of the lower court should be reversed.

580
The certificate was later sold to Francisco Benitez, Jr., who resold it to Rodi
Taxicab Company. Both sales were made with assumption of the mortgage in
favor of the RFC, and were also approved provisionally by the Commission,
subject to petitioner's lien.
G.R. No. L-17716 July 31, 1962
On October 10, 1953 petitioner filed an action to foreclose the chattel
LUNETA MOTOR COMPANY, petitioner, mortgage executed in its favor by Concepcion (Civil Case No. 20853 of the
vs. Court of First Instance of Manila) in view of the failure of the latter and his
A.D. SANTOS, INC., ET AL., respondents. guarantor, Placido Esteban, to pay their overdue account.

Jose Agbulos for petitioner. While the above case was pending, the RFC also instituted foreclosure
Graciano C. Regala and Angel A. Sison for respondents. proceedings on its second chattel mortgage, and as a result of the decision
in its favor therein rendered, the certificate of public convenience was sold at
public auction in favor of Amador D. Santos for P24,010.00 on August 31,
DIZON, J.:
1956. Santos immediately applied with the Commission for the approval of
the sale, and the same was approved on January 26, 1957, subject to the
Appeal from the decision of the Public Service Commission in case No. mortgage lien in favor of petitioner.
123401 dismissing petitioner's application for the approval of the sale in its
favor, made by the Sheriff of the City of Manila, of the certificate of public
On June 9, 1958 the Court of First Instance of Manila rendered judgment in
convenience granted before the war to Nicolas Concepcion (Commission
Civil Case No. 20853, amended on August 1, 1958, adjudging Concepcion
Cases Nos. 60604 and 60605, reconstituted after the war in Commission
indebted to petitioner in the sum of P15,197.84, with 12% interest thereon
Case No. 1470) to operate a taxicab service of 27 units in the City of Manila
from December 2, 1941 until full payment, plus other assessments, and
and therefrom to any point in Luzon.
ordered that the certificate of public convenience subject matter of the chattel
mortgage be sold at public auction in accordance with law. Accordingly, on
It appears that on December 31, 1941, to secure payment of a loan March 3, 1959 said certificate was sold at public auction to petitioner, and six
evidenced by a promissory note executed by Nicolas Concepcion and days thereafter the Sheriff of the City of Manila issued in its favor the
guaranteed by one Placido Esteban in favor of petitioner, Concepcion corresponding certificate of sale. Thereupon petitioner filed the application
executed a chattel mortgage covering the above mentioned certificate in mentioned heretofore for the approval of the sale. In the meantime and
favor of petitioner. before his death, Amador D. Santos sold and transferred (Commission Case
No. 1272231) all his rights and interests in the certificate of public
To secure payment of a subsequent loan obtained by Concepcion from the convenience in question in favor of the now respondent A.D. Santos, Inc.,
Rehabilitation Finance Corporation (now Development Bank of the who opposed petitioner's application.
Philippines) he constituted a second mortgage on the same certificate. This
second mortgage was approved by the respondent Commission, subject to The record discloses that in the course of the hearing on said application and
the mortgage lien in favor of petitioner. after petitioner had rested its case, the respondent A. D. Santos, Inc., with
leave of court, filed a motion to dismiss based on the following grounds:

581
a) under the petitioner's Articles of Incorporation, it was not It is not denied that under Section 13 (5) of the Corporation Law, a
authorized to engage in the taxicab business or operate as a corporation created thereunder may purchase, hold, etc., and otherwise deal
common carrier; in such real and personal property is the purpose for which the corporation
was formed may permit, and the transaction of its lawful business may
b) the decision in Civil Case No. 20853 of the Court of First Instance reasonably and necessarily require. The issue here is precisely whether the
of Manila did not affect the oppositor nor its predecessor Amador D. purpose for which petitioner was organized and the transaction of its lawful
Santos inasmuch as neither of them had been impleaded into the business reasonably and necessarily require the purchase and holding by it
case; of a certificate of public convenience like the one in question and thus give it
additional authority to operate thereunder as a common carrier by land.
c) that what was sold to the petitioner were only the "rights, interests
and participation" of Nicolas Concepcion in the certificate that had Petitioner claims in this regard that its corporate purposes are to carry on a
been granted to him which were no longer existing at the time of the general mercantile and commercial business, etc., and that it is authorized in
sale. its articles of incorporation to operate and otherwise deal in and concerning
automobiles and automobile accessories' business in all its multifarious
On October 18, 1960, the respondent Commission, after considering the ramification (petitioner's brief p. 7) and to operate, etc., and otherwise
memoranda submitted by the parties, rendered the appealed decision dispose of vessels and boats, etc., and to own and operate steamship and
sustaining the first ground relied upon in support thereof, namely, that under sailing ships and other floating craft and deal in the same and engage in the
petitioner's articles of incorporation it had no authority to engage in the Philippine Islands and elsewhere in the transportation of persons,
taxicab business or operate as a common carrier, and that, is a result, it merchandise and chattels by water; all this incidental to the transportation of
could not acquire by purchase the certificate of public convenience referred automobiles (id. pp. 7-8 and Exhibit B).
to above. Hence, the present appeal interposed by petitioner who claims
that, in accordance with the Corporation Law and its articles of incorporation, We find nothing in the legal provision and the provisions of petitioner's
it can acquire by purchase the certificate of public convenience in question, articles of incorporation relied upon that could justify petitioner's contention in
maintaining inferentially that, after acquiring said certificate, it could make this case. To the contrary, they are precisely the best evidence that it has no
use of it by operating a taxicab business or operate is a common carrier by authority at all to engage in the business of land transportation and operate a
land. taxicab service. That it may operate and otherwise deal in automobiles and
automobile accessories; that it may engage in the transportation of
There is no question that a certificate of public convenience granted to the persons by water does not mean that it may engage in the business of land
public operator is liable to execution (Raymundo vs. Luneta Motor Co., 58 transportation an entirely different line of business. If it could not thus
Phil. 889) and may be acquired by purchase. The question involved in the engage in the line of business, it follows that it may not acquire an certificate
present appeal, however, is not only whether, under the Corporation Law and of public convenience to operate a taxicab service, such as the one in
petitioner's articles of incorporation, it may acquire by purchase a certificate question, because such acquisition would be without purpose and would
of public convenience, such as the one in question, but also whether, after its have no necessary connection with petitioner's legitimate business.
acquisition, petitioner may hold the certificate and thereunder operate as a
common carrier by land. In view of the conclusion we have arrived at on the decisive issue involved in
this appeal, we deem it unnecessary to resolve the other incidental questions
raised by petitioner.

582
WHEREFORE, the appealed decision is affirmed, with costs. On May 17, 1948, the Acoje Mining Company, Inc. wrote the Director of
Posts requesting the opening of a post, telegraph and money order offices at
Bengzon, C.J., Padilla, Concepcion, Barrera, Paredes and Makalintal, JJ., its mining camp at Sta. Cruz, Zambales, to service its employees and their
concur. families that were living in said camp. Acting on the request, the Director of
Regala, J., took no part. Posts wrote in reply stating that if aside from free quarters the company
would provide for all essential equipment and assign a responsible employee
to perform the duties of a postmaster without compensation from his office
until such time as funds therefor may be available he would agree to put up
the offices requested. The company in turn replied signifying its willingness to
comply with all the requirements outlined in the letter of the Director of Posts
requesting at the same time that it be furnished with the necessary forms for
the early establishment of a post office branch.

On April 11, 1949, the Director of Posts again wrote a letter to the company
stating among other things that "In cases where a post office will be opened
G.R. No. L-18062 February 28, 1963 under circumstances similar to the present, it is the policy of this office to
have the company assume direct responsibility for whatever pecuniary loss
REPUBLIC OF THE PHILIPPINES, plaintiff-appellee, may be suffered by the Bureau of Posts by reason of any act of dishonesty,
vs. carelessness or negligence on the part of the employee of the company who
ACOJE MINING COMPANY, INC., defendant-appellant. is assigned to take charge of the post office," thereby suggesting that a
resolution be adopted by the board of directors of the company expressing
Office of the Solicitor General for plaintiff-appellee. conformity to the above condition relative to the responsibility to be assumed
Jalandoni & Jamir for defendant-appellant. buy it in the event a post office branch is opened as requested. On
September 2, 1949, the company informed the Director of Posts of the
BAUTISTA ANGELO, J.: passage by its board of directors of a resolution of the following tenor: "That
the requirement of the Bureau of Posts that the Company should accept full
responsibility for all cash received by the Postmaster be complied with, and
that a copy of this resolution be forwarded to the Bureau of Posts." The letter
further states that the company feels that that resolution fulfills the last
condition imposed by the Director of Posts and that, therefore, it would
request that an inspector be sent to the camp for the purpose of acquainting
the postmaster with the details of the operation of the branch office.

The post office branch was opened at the camp on October 13, 1949 with
one Hilario M. Sanchez as postmaster. He is an employee of the company.
On May 11, 1954, the postmaster went on a three-day leave but never
returned. The company immediately informed the officials of the Manila Post

583
Office and the provincial auditor of Zambales of Sanchez' disappearance with Posts was prevailed upon to agree to the request only after studying the
the result that the accounts of the postmaster were checked and a shortage necessity for its establishment and after imposing upon the company certain
was found in the amount of P13,867.24. requirements intended to safeguard and protect the interest of the
government. Thus, after the company had signified its willingness to comply
The several demands made upon the company for the payment of the with the requirement of the government that it furnish free quarters and all
shortage in line with the liability it has assumed having failed, the government the essential equipment that may be necessary for the operation of the office
commenced the present action on September 10, 1954 before the Court of including the assignment of an employee who will perform the duties of a
First Instance of Manila seeking to recover the amount of Pl3,867.24. The postmaster, the Director of Posts agreed to the opening of the post office
company in its answer denied liability for said amount contending that the stating that "In cases where a post office will be opened under circumstances
resolution of the board of directors wherein it assumed responsibility for the similar to the present, it is the policy of this office to have the company
act of the postmaster is ultra vires, and in any event its liability under said assume direct responsibility for whatever pecuniary loss may be suffered by
resolution is only that of a guarantor who answers only after the exhaustion the Bureau of Posts by reason of any act of dishonesty, carelessness or
of the properties of the principal, aside from the fact that the loss claimed by negligence on the part of the employee of the company who is assigned to
the plaintiff is not supported by the office record. take charge of the post office," and accepting this condition, the company,
thru its board of directors, adopted forthwith a resolution of the following
Wherefore, the parties respectfully pray that the foregoing stipulation of facts tenor: "That the requirement of the Bureau of Posts that the company should
be admitted and approved by this Honorable Court, without prejudice to the accept full responsibility for all cash received by the Postmaster, be complied
parties adducing other evidence to prove their case not covered by this with, and that a copy of this resolution be forwarded to the Bureau of Posts."
stipulation of facts. 1wph1.t On the basis of the foregoing facts, it is evident that the company cannot now
be heard to complain that it is not liable for the irregularity committed by its
employee upon the technical plea that the resolution approved by its board of
After trial, the court a quo found that, of the amount claimed by plaintiff
directors is ultra vires. The least that can be said is that it cannot now go
totalling P13,867.24, only the sum of P9,515.25 was supported by the
back on its plighted word on the ground of estoppel.
evidence, and so it rendered judgment for the plaintiff only for the amount
last mentioned. The court rejected the contention that the resolution adopted
by the company is ultra vires and that the obligation it has assumed is merely The claim that the resolution adopted by the board of directors of appellant
that of a guarantor. company is an ultra vires act cannot also be entertained it appearing that the
same covers a subject which concerns the benefit, convenience and welfare
of its employees and their families. While as a rule an ultra vires act is one
Defendant took the present appeal.
committed outside the object for which a corporation is created as defined by
the law of its organization and therefore beyond the powers conferred upon it
The contention that the resolution adopted by the company dated August 31, by law (19 C.J.S., Section 965, p. 419), there are however certain corporate
1949 is ultra vires in the sense that it has no authority to act on a matter acts that may be performed outside of the scope of the powers expressly
which may render the company liable as a guarantor has no factual or legal conferred if they are necessary to promote the interest or welfare of the
basis. In the first place, it should be noted that the opening of a post office corporation. Thus, it has been held that "although not expressly authorized to
branch at the mining camp of appellant corporation was undertaken because do so a corporation may become a surety where the particular transaction is
of a request submitted by it to promote the convenience and benefit of its reasonably necessary or proper to the conduct of its business," 1 and here it is
employees. The idea did not come from the government, and the Director of undisputed that the establishment of the local post office is a reasonable and

584
proper adjunct to the conduct of the business of appellant company. Indeed, The acceptance of benefits arising from the performance by the
such post office is a vital improvement in the living condition of its employees other party may give rise to an estoppel precluding repudiation of the
and laborers who came to settle in its mining camp which is far removed from transaction. (19 C.J.S., Section 976, p. 433.)
the postal facilities or means of communication accorded to people living in a
city or municipality.. The current of modern authorities favors the rule that where the ultra
vires transaction has been executed by the other party and the
Even assuming arguendo that the resolution in question constitutes an ultra corporation has received the benefit of it, the law interposes an
vires act, the same however is not void for it was approved not in estoppel, and will not permit the validity of the transaction or contract
contravention of law, customs, public order or public policy. The term ultra to be questioned, and this is especially true where there is nothing in
viresshould be distinguished from an illegal act for the former is merely the circumstances to put the other party to the transaction on notice
voidable which may be enforced by performance, ratification, or estoppel, that the corporation has exceeded its powers in entering into it and
while the latter is void and cannot be validated. 2 It being merely voidable, an has in so doing overstepped the line of corporate privileges. (19
ultra vires act can be enforced or validated if there are equitable grounds for C.J.S., Section 977, pp. 435-437, citing Williams v. Peoples Building
taking such action. Here it is fair that the resolution be upheld at least on the & Loan Ass'n, 97 S.W. 2d 930, 193 Ark. 118; Hays v. Galion Gas
ground of estoppel. On this point, the authorities are overwhelming: Light Co., 29 Ohio St. 330)

The weight of authority in the state courts is to the effect that a Neither can we entertain the claim of appellant that its liability is only that of a
transaction which is merely ultra vires and not malum in se or malum guarantor. On this point, we agree with the following comment of the court a
prohibitum, is, if performed by one party, not void as between the quo: "A mere reading of the resolution of the Board of Directors dated August
parties to all intents and purposes, and that an action may be 31, 1949, upon which the plaintiff based its claim would show that the
brought directly on the transaction and relief had according to its responsibility of the defendant company is not just that of a guarantor. Notice
terms. (19 C.J.S., Section 976, p. 432, citing Nettles v. Rhett, that the phraseology and the terms employed are so clear and sweeping and
C.C.A.S.C., 94 F. 2d, reversing, D.C., 20 F. Supp. 48) that the defendant assumed 'full responsibility for all cash received by the
Postmaster.' Here the responsibility of the defendant is not just that of a
This rule is based on the consideration that as between private guarantor. It is clearly that of a principal."
corporations, one party cannot receive the benefits which are
embraced in total performance of a contract made with it by another WHEREFORE, the decision appealed from is affirmed. No costs.
party and then set up the invalidity of the transaction as a defense."
(London & Lancashire Indemnity Co. of America v. Fairbanks Steam Bengzon, C.J., Padilla, Labrador, Concepcion, Reyes, J.B.L., Barrera,
Shovel Co., 147 N.E. 329, 332, 112 Ohio St. 136.) Paredes, Dizon, Regala and Makalintal, JJ. concur.

The defense of ultra vires rests on violation of trust or duty toward


stockholders, and should not be entertained where its allowance will
do greater wrong to innocent parties dealing with corporation..

585
G.R. No. 80599 September 15, 1989

ERNESTINA CRISOLOGO-JOSE, petitioner,


vs.
COURT OF APPEALS and RICARDO S. SANTOS, JR. in his own behalf
and as Vice-President for Sales of Mover Enterprises, Inc., respondents.

Melquiades P. de Leon for petitioner.

Rogelio A. Ajes for private respondent.

REGALADO, J.:

Petitioner seeks the annulment of the decision 1 of respondent Court of


Appeals, promulgated on September 8, 1987, which reversed the decision of
the trial Court 2 dismissing the complaint for consignation filed by therein
plaintiff Ricardo S. Santos, Jr.

The parties are substantially agreed on the following facts as found by both
lower courts:

In 1980, plaintiff Ricardo S. Santos, Jr. was the vice-


president of Mover Enterprises, Inc. in-charge of marketing
and sales; and the president of the said corporation was Atty.
Oscar Z. Benares. On April 30, 1980, Atty. Benares, in
accommodation of his clients, the spouses Jaime and Clarita
Ong, issued Check No. 093553 drawn against Traders Royal
Bank, dated June 14, 1980, in the amount of P45,000.00
(Exh- 'I') payable to defendant Ernestina Crisologo-Jose.
Since the check was under the account of Mover
Enterprises, Inc., the same was to be signed by its president,
Atty. Oscar Z. Benares, and the treasurer of the said
corporation. However, since at that time, the treasurer of
Mover Enterprises was not available, Atty. Benares prevailed

586
upon the plaintiff, Ricardo S. Santos, Jr., to sign the before Assistant City Fiscal Alfonso T. Llamas, plaintiff
aforesaid chEck as an alternate story. Plaintiff Ricardo S. Ricardo S. Santos, Jr. tendered cashier's check No. CC
Santos, Jr. did sign the check. 160152 for P45,000.00 dated April 10, 1981 to the defendant
Ernestina Crisologo-Jose, the complainant in that criminal
It appears that the check (Exh. '1') was issued to defendant case. The defendant refused to receive the cashier's check
Ernestina Crisologo-Jose in consideration of the waiver or in payment of the dishonored check in the amount of
quitclaim by said defendant over a certain property which the P45,000.00. Hence, plaintiff encashed the aforesaid
Government Service Insurance System (GSIS) agreed to cashier's check and subsequently deposited said amount of
sell to the clients of Atty. Oscar Benares, the spouses Jaime P45,000.00 with the Clerk of Court on August 14, 1981
and Clarita Ong, with the understanding that upon approval (Exhs. 'D' and 'E'). Incidentally, the cashier's check adverted
by the GSIS of the compromise agreement with the spouses to above was purchased by Atty. Oscar Z. Benares and
Ong, the check will be encashed accordingly. However, since given to the plaintiff herein to be applied in payment of the
the compromise agreement was not approved within the dishonored check. 3
expected period of time, the aforesaid check for P45,000.00
(Exh. '1') was replaced by Atty. Benares with another Traders After trial, the court a quo, holding that it was "not persuaded to believe that
Royal Bank cheek bearing No. 379299 dated August 10, consignation referred to in Article 1256 of the Civil Code is applicable to this
1980, in the same amount of P45,000.00 (Exhs. 'A' and '2'), case," rendered judgment dismissing plaintiff s complaint and defendant's
also payable to the defendant Jose. This replacement check counterclaim. 4
was also signed by Atty. Oscar Z. Benares and by the
plaintiff Ricardo S. Santos, Jr. When defendant deposited As earlier stated, respondent court reversed and set aside said judgment of
this replacement check (Exhs. 'A' and '2') with her account at dismissal and revived the complaint for consignation, directing the trial court
Family Savings Bank, Mayon Branch, it was dishonored for to give due course thereto.
insufficiency of funds. A subsequent redepositing of the said
check was likewise dishonored by the bank for the same Hence, the instant petition, the assignment of errors wherein are prefatorily
reason. Hence, defendant through counsel was constrained stated and discussed seriatim.
to file a criminal complaint for violation of Batas Pambansa
Blg. 22 with the Quezon City Fiscal's Office against Atty.
1. Petitioner contends that respondent Court of Appeals
Oscar Z. Benares and plaintiff Ricardo S. Santos, Jr. The
erred in holding that private respondent, one of the
investigating Assistant City Fiscal, Alfonso Llamas,
signatories of the check issued under the account of Mover
accordingly filed an amended information with the court
Enterprises, Inc., is an accommodation party under the
charging both Oscar Benares and Ricardo S. Santos, Jr., for
Negotiable Instruments Law and a debtor of petitioner to the
violation of Batas Pambansa Blg. 22 docketed as Criminal
extent of the amount of said check.
Case No. Q-14867 of then Court of First Instance of Rizal,
Quezon City.
Petitioner avers that the accommodation party in this case is Mover
Enterprises, Inc. and not private respondent who merely signed the check in
Meanwhile, during the preliminary investigation of the
question in a representative capacity, that is, as vice-president of said
criminal charge against Benares and the plaintiff herein,

587
corporation, hence he is not liable thereon under the Negotiable Instruments The aforequoted provision of the Negotiable Instruments Law which holds an
Law. accommodation party liable on the instrument to a holder for value, although
such holder at the time of taking the instrument knew him to be only an
The pertinent provision of said law referred to provides: accommodation party, does not include nor apply to corporations which are
accommodation parties. 7 This is because the issue or indorsement of
Sec. 29. Liability of accommodation party an accommodation negotiable paper by a corporation without consideration and for the
party is one who has signed the instrument as maker, accommodation of another is ultra vires. 8 Hence, one who has taken the
drawer, acceptor, or indorser, without receiving value instrument with knowledge of the accommodation nature thereof cannot
therefor, and for the purpose of lending his name to some recover against a corporation where it is only an accommodation party. If the
other person. Such a person is liable on the instrument to a form of the instrument, or the nature of the transaction, is such as to charge
holder for value, notwithstanding such holder, at the time of the indorsee with knowledge that the issue or indorsement of the instrument
taking the instrument, knew him to be only an by the corporation is for the accommodation of another, he cannot recover
accommodation party. against the corporation thereon. 9

Consequently, to be considered an accommodation party, a person must (1) By way of exception, an officer or agent of a corporation shall have the power
be a party to the instrument, signing as maker, drawer, acceptor, or indorser, to execute or indorse a negotiable paper in the name of the corporation for
(2) not receive value therefor, and (3) sign for the purpose of lending his the accommodation of a third person only if specifically authorized to do
name for the credit of some other person. so. 10 Corollarily, corporate officers, such as the president and vice-president,
have no power to execute for mere accommodation a negotiable instrument
of the corporation for their individual debts or transactions arising from or in
Based on the foregoing requisites, it is not a valid defense that the
relation to matters in which the corporation has no legitimate concern. Since
accommodation party did not receive any valuable consideration when he
such accommodation paper cannot thus be enforced against the corporation,
executed the instrument. From the standpoint of contract law, he differs from
especially since it is not involved in any aspect of the corporate business or
the ordinary concept of a debtor therein in the sense that he has not received
operations, the inescapable conclusion in law and in logic is that the
any valuable consideration for the instrument he signs. Nevertheless, he is
signatories thereof shall be personally liable therefor, as well as the
liable to a holder for value as if the contract was not for accommodation 5in
consequences arising from their acts in connection therewith.
whatever capacity such accommodation party signed the instrument, whether
primarily or secondarily. Thus, it has been held that in lending his name to the
accommodated party, the accommodation party is in effect a surety for the The instant case falls squarely within the purview of the aforesaid decisional
latter. 6 rules. If we indulge petitioner in her aforesaid postulation, then she is
effectively barred from recovering from Mover Enterprises, Inc. the value of
the check. Be that as it may, petitioner is not without recourse.
Assuming arguendo that Mover Enterprises, Inc. is the accommodation party
in this case, as petitioner suggests, the inevitable question is whether or not
it may be held liable on the accommodation instrument, that is, the check The fact that for lack of capacity the corporation is not bound by an
issued in favor of herein petitioner. accommodation paper does not thereby absolve, but should render
personally liable, the signatories of said instrument where the facts show that
the accommodation involved was for their personal account, undertaking or
We hold in the negative.
purpose and the creditor was aware thereof.

588
Petitioner, as hereinbefore explained, was evidently charged with the between Atty. Benares and respondent Santos, on the one hand, and
knowledge that the cheek was issued at the instance and for the personal petitioner, on the other. This circumstance enables respondent Santos to
account of Atty. Benares who merely prevailed upon respondent Santos to resort to an action of consignation where his tender of payment had been
act as co-signatory in accordance with the arrangement of the corporation refused by petitioner.
with its depository bank. That it was a personal undertaking of said corporate
officers was apparent to petitioner by reason of her personal involvement in We interpose the caveat, however, that by holding that the remedy of
the financial arrangement and the fact that, while it was the corporation's consignation is proper under the given circumstances, we do not thereby rule
check which was issued to her for the amount involved, she actually had no that all the operative facts for consignation which would produce the effect of
transaction directly with said corporation. payment are present in this case. Those are factual issues that are not clear
in the records before us and which are for the Regional Trial Court of Quezon
There should be no legal obstacle, therefore, to petitioner's claims being City to ascertain in Civil Case No. Q-33160, for which reason it has advisedly
directed personally against Atty. Oscar Z. Benares and respondent Ricardo been directed by respondent court to give due course to the complaint for
S. Santos, Jr., president and vice-president, respectively, of Mover consignation, and which would be subject to such issues or claims as may be
Enterprises, Inc. raised by defendant and the counterclaim filed therein which is hereby
ordered similarly revived.
2. On her second assignment of error, petitioner argues that
the Court of Appeals erred in holding that the consignation of 3. That respondent court virtually prejudged Criminal Case
the sum of P45,000.00, made by private respondent after his No. Q-14687 of the Regional Trial Court of Quezon City filed
tender of payment was refused by petitioner, was proper against private respondent for violation of Batas Pambansa
under Article 1256 of the Civil Code. Blg. 22, by holding that no criminal liability had yet attached
to private respondent when he deposited with the court the
Petitioner's submission is that no creditor-debtor relationship exists between amount of P45,000.00 is the final plaint of petitioner.
the parties, hence consignation is not proper. Concomitantly, this argument
was premised on the assumption that private respondent Santos is not an We sustain petitioner on this score.
accommodation party.
Indeed, respondent court went beyond the ratiocination called for in the
As previously discussed, however, respondent Santos is an accommodation appeal to it in CA-G.R. CV. No. 05464. In its own decision therein, it declared
party and is, therefore, liable for the value of the check. The fact that he was that "(t)he lone issue dwells in the question of whether an accommodation
only a co-signatory does not detract from his personal liability. A co-maker or party can validly consign the amount of the debt due with the court after his
co-drawer under the circumstances in this case is as much an tender of payment was refused by the creditor." Yet, from the commercial and
accommodation party as the other co-signatory or, for that matter, as a lone civil law aspects determinative of said issue, it digressed into the merits of
signatory in an accommodation instrument. Under the doctrine in Philippine the aforesaid Criminal Case No. Q-14867, thus:
Bank of Commerce vs. Aruego, supra, he is in effect a co-surety for the
accommodated party with whom he and his co-signatory, as the other co- Section 2 of B.P. 22 establishes the prima facie evidence of
surety, assume solidary liability ex lege for the debt involved. With the knowledge of such insufficiency of funds or credit. Thus, the
dishonor of the check, there was created a debtor-creditor relationship, as making, drawing and issuance of a check, payment of which

589
is refused by the drawee because of insufficient funds in or consignation of payment, where no such period and its legal effects are
credit with such bank is prima facie evidence of knowledge involved.
of insufficiency of funds or credit, when the check is
presented within 90 days from the date of the check. These are aside from the considerations that the disputed period involved in
the criminal case is only a presumptive rule, juris tantum at that, to determine
It will be noted that the last part of Section 2 of B.P. 22 whether or not there was knowledge of insufficiency of funds in or credit with
provides that the element of knowledge of insufficiency of the drawee bank; that payment of civil liability is not a mode for
funds or credit is not present and, therefore, the crime does extinguishment of criminal liability; and that the requisite quantum of
not exist, when the drawer pays the holder the amount due evidence in the two types of cases are not the same.
or makes arrangements for payment in full by the drawee of
such check within five (5) banking days after receiving notice To repeat, the foregoing matters are properly addressed to the trial court in
that such check has not been paid by the drawee. Criminal Case No. Q-14867, the resolution of which should not be interfered
with by respondent Court of Appeals at the present posture of said case,
Based on the foregoing consideration, this Court finds that much less preempted by the inappropriate and unnecessary holdings in the
the plaintiff-appellant acted within Ms legal rights when he aforequoted portion of the decision of said respondent court. Consequently,
consigned the amount of P45,000.00 on August 14, 1981, we modify the decision of respondent court in CA-G.R. CV No. 05464 by
between August 7, 1981, the date when plaintiff-appellant setting aside and declaring without force and effect its pronouncements and
receive (sic) the notice of non-payment, and August 14, findings insofar as the merits of Criminal Case No. Q-14867 and the liability
1981, the date when the debt due was deposited with the of the accused therein are concerned.
Clerk of Court (a Saturday and a Sunday which are not
banking days) intervened. The fifth banking day fell on WHEREFORE, subject to the aforesaid modifications, the judgment of
August 14, 1981. Hence, no criminal liability has yet respondent Court of Appeals is AFFIRMED.
attached to plaintiff-appellant when he deposited the amount
of P45,000.00 with the Court a quo on August 14, 1981. 11 SO ORDERED.

That said observations made in the civil case at bar and the intrusion into the Paras, Padilla and Sarmiento, JJ., concur
merits of the criminal case pending in another court are improper do not have
to be belabored. In the latter case, the criminal trial court has to grapple with
G.R. No. L-37331 March 18, 1933
such factual issues as, for instance, whether or not the period of five banking
days had expired, in the process determining whether notice of dishonor
should be reckoned from any prior notice if any has been given or from FRED M. HARDEN, J.D. HIGHSMITH, and JOHN C. HART, in their own
receipt by private respondents of the subpoena therein with supporting behalf and in that all other stockholders of the Balatoc Mining
affidavits, if any, or from the first day of actual preliminary investigation; and Company, etc., plaintiffs-appellants,
whether there was a justification for not making the requisite arrangements vs.
for payment in full of such check by the drawee bank within the said period. BENGUET CONSOLIDATED MINING COMPANY, BALATOC MINING
These are matters alien to the present controversy on tender and COMPANY, H. E. RENZ, JOHN W. JAUSSERMANN, and A. W.
BEAM, defendants-appellees.

590
Gibbs and McDonough and Roman Ozaeta for appellants. The facts which have given rise this lawsuit are simple, as the financial
DeWitt, Perkins and Brady for appellees. interests involve are immense. Briefly told these facts are as follows: The
Ross, Lawrence and Selph for appellee Balatoc Mining Company. Benguet Consolidated Mining Co. was organized in June, 1903, as
a sociedad anonima in conformity with the provisions of Spanish law; while
STREET, J.: the Balatoc Mining Co. was organized in December 1925, as a corporation,
in conformity with the provisions of the Corporation Law (Act No. 1459). Both
This action was originally instituted in the Court of First Instance of the City of entities were organized for the purpose of engaging in the mining of gold in
Manila by F. M. Harden, acting in his own behalf and that of all other the Philippine Islands, and their respective properties are located only a few
stockholders of the Balatoc Mining Co. who might join in the action and miles apart in the subprovince of Benguet. The capital stock of the Balatoc
contribute to the expense of the suit. With the plaintiff Harden two others, J. Mining Co. consists of one million shares of the par value of one peso (P1)
D. Highsmith and John C. Hart, subsequently associated themselves. The each.
defendants are the Benguet Consolidated Mining Co., the Balatoc Mining
Co., H. E. Renz, John W. Haussermann, and A. W. Beam. The principal When the Balatoc Mining Co. was first organized the properties acquired by it
purpose of the original action was to annul a certificate covering 600,000 were largely undeveloped; and the original stockholders were unable to
shares of the stock of the Balatoc Mining Co., which have been issued to the supply the means needed for profitable operation. For this reason, the board
Benguet Consolidated Mining Co., and to secure to the Balatoc Mining Co., of directors of the corporation ordered a suspension of all work, effective July
the restoration of a large sum of money alleged to have been unlawfully 31, 1926. In November of the same year a general meeting of the company's
collected by the Benguet Consolidated Mining Co., with legal interest, after stockholders appointed a committee for the purpose of interesting outside
deduction therefrom of the amount expended by the latter company under a capital in the mine. Under the authority of this resolution the committee
contract between the two companies, bearing date of March 9, 1927. The approached A. W. Beam, then president and general manager of the
complaint was afterwards amended so as to include a prayer for the Benguet Company, to secure the capital necessary to the development of the
annulment of this contract. Shortly prior to the institution of this lawsuit, the Balatoc property. As a result of the negotiations thus begun, a contract,
Benguet Consolidated Mining Co., transferred to H. E. Renz, as trustee, the formally authorized by the management of both companies, was executed on
certificate for 600,000 shares of the Balatoc Mining Co. which constitute the March 9, 1927, the principal features of which were that the Benguet
principal subject matter of the action. This was done apparently to facilitate Company was to proceed with the development and construct a milling plant
the splitting up to the shares in the course of the sale or distribution. To for the Balatoc mine, of a capacity of 100 tons of ore per day, and with an
prevent this the plaintiffs, upon filing their original complaint, procured a extraction of at least 85 per cent of the gold content. The Benguet Company
preliminary injunction restraining the defendants, their agents and servants, also agreed to erect an appropriate power plant, with the aerial tramlines and
from selling, assigning or transferring the 600,000 shares of the Balatoc such other surface buildings as might be needed to operate the mine. In
Mining Co., or any part thereof, and from removing said shares from the return for this it was agreed that the Benguet Company should receive from
Philippine Islands. This explains the connection of Renz with the case. The the treasurer of the Balatoc Company shares of a par value of P600,000, in
other individual defendants are made merely as officials of the Benguet payment for the first P600,000 be thus advanced to it by the Benguet
Consolidated Mining Co. Upon hearing the cause the trial court dismissed Company.
the complaint and dissolved the preliminary injunction, with costs against the
plaintiffs. From this judgment the plaintiffs appealed. The performance of this contract was speedily begun, and by May 31, 1929,
the Benguet Company had spent upon the development the sum of
P1,417,952.15. In compensation for this work a certificate for six hundred

591
thousand shares of the stock of the Balatoc Company has been delivered to Under the guidance of this and certain other provisions thus enacted by
the Benguet Company, and the excess value of the work in the amount of Congress, the Philippine Commission entered upon the enactment of a
P817,952.15 has been returned to the Benguet Company in cash. Meanwhile general law authorizing the creation of corporations in the Philippine Islands.
dividends of the Balatoc Company have been enriching its stockholders, and This rather elaborate piece of legislation is embodied in what is called our
at the time of the filing of the complaint the value of its shares had increased Corporation Law (Act No. 1459 of the Philippine Commission). The evident
in the market from a nominal valuation to more than eleven pesos per share. purpose of the commission was to introduce the American corporation into
While the Benguet Company was pouring its million and a half into the the Philippine Islands as the standard commercial entity and to hasten the
Balatoc property, the arrangements made between the two companies day when the sociedad anonima of the Spanish law would be obsolete. That
appear to have been viewed by the plaintiff Harden with complacency, he statute is a sort of codification of American corporate law.
being the owner of many thousands of the shares of the Balatoc Company.
But as soon as the success of the development had become apparent, he For the purposes general description only, it may be stated that the sociedad
began this litigation in which he has been joined by two others of the eighty anonima is something very much like the English joint stock company, with
shareholders of the Balatoc Company. features resembling those of both the partnership is shown in the fact that
sociedad, the generic component of its name in Spanish, is the same word
Briefly, the legal point upon which the action is planted is that it is unlawful for that is used in that language to designate other forms of partnership, and in
the Benguet Company to hold any interest in a mining corporation and that its organization it is constructed along the same general lines as the ordinary
the contract by which the interest here in question was acquired must be partnership. It is therefore not surprising that for purposes of loose translation
annulled, with the consequent obliteration of the certificate issued to the the expression sociedad anonima has not infrequently the other hand, the
Benguet Company and the corresponding enrichment of the shareholders of affinity of this entity to the American corporation has not escaped notice, and
the Balatoc Company. the expression sociedad anonima is now generally translated by the word
corporation. But when the word corporation is used in the sense of sociedad
When the Philippine Islands passed to the sovereignty of the United States, anonima and close discrimination is necessary, it should be associated with
in the attention of the Philippine Commission was early drawn to the fact that the Spanish expression sociedad anonima either in a parenthesis or
there is no entity in Spanish law exactly corresponding to the notion of the connected by the word "or". This latter device was adopted in sections 75
corporation in English and American law; and in the Philippine Bill, approved and 191 of the Corporation Law.
July 1, 1902, the Congress of the United States inserted certain provisions,
under the head of Franchises, which were intended to control the lawmaking In drafting the Corporation Law the Philippine Commission inserted bodily, in
power in the Philippine Islands in the matter of granting of franchises, subsection (5) of section 13 of that Act (No. 1459) the words which we have
privileges and concessions. These provisions are found in section 74 and 75 already quoted from section 75 of the Act of Congress of July 1, 1902
of the Act. The provisions of section 74 have been superseded by section 28 (Philippine Bill); and it is of course obvious that whatever meaning originally
of the Act of Congress of August 29, 1916, but in section 75 there is a attached to this provision in the Act of Congress, the same significance
provision referring to mining corporations, which still remains the law, as should be attached to it in section 13 of our Corporation Law.
amended. This provisions, in its original form, reads as follows: "... it shall be
unlawful for any member of a corporation engaged in agriculture or mining As it was the intention of our lawmakers to stimulate the introduction of the
and for any corporation organized for any purpose except irrigation to be in American Corporation into Philippine law in the place of the sociedad
any wise interested in any other corporation engaged in agriculture or in anonima, it was necessary to make certain adjustments resulting from the
mining." continued co-existence, for a time, of the two forms of commercial entities.

592
Accordingly, in section 75 of the Corporation Law, a provision is found As originally drawn, our Corporation Law (Act No. 1459) did not contain any
making the sociedad anonima subject to the provisions of the Corporation appropriate clause directly penalizing the act of a corporation, a member of a
Law "so far as such provisions may be applicable", and giving to corporation , in acquiring an interest contrary to paragraph (5) of section 13
the sociedades anonimas previously created in the Islands the option to of the Act. The Philippine Legislature undertook to remedy this situation in
continue business as such or to reform and organize under the provisions of section 3 of Act No. 2792 of the Philippine Legislature, approved on February
the Corporation Law. Again, in section 191 of the Corporation Law, the Code 18, 1919, but this provision was declared invalid by this court in Government
of Commerce is repealed in so far as it relates to sociedades anonimas. The of the Philippine Islands vs. El Hogar Filipino (50 Phil., 399), for lack of an
purpose of the commission in repealing this part of the Code of Commerce adequate title to the Act. Subsequently the Legislature reenacted
was to compel commercial entities thereafter organized to incorporate under substantially the same penal provision in section 21 of Act No. 3518, under a
the Corporation Law, unless they should prefer to adopt some form or other title sufficiently broad to comprehend the subject matter. This part of Act No.
of the partnership. To this provision was added another to the effect that 3518 became effective upon approval by the Governor-General, on
existing sociedades anonimas, which elected to continue their business as December 3, 1928, and it was therefore in full force when the contract now in
such, instead of reforming and reorganizing under the Corporation Law, question was made.
should continue to be governed by the laws that were in force prior to the
passage of this Act "in relation to their organization and method of This provision was inserted as a new section in the Corporation Law, forming
transacting business and to the rights of members thereof as between section 1990 (A) of said Act as it now stands. Omitting the proviso, which
themselves, but their relations to the public and public officials shall be seems not to be pertinent to the present controversy, said provision reads as
governed by the provisions of this Act." follows:

As already observed, the provision above quoted from section 75 of the Act SEC. 190 (A). Penalties. The violation of any of the provisions of
Congress of July 1, 1902 (Philippine Bill), generally prohibiting corporations this Act and its amendments not otherwise penalized therein, shall
engaged in mining and members of such from being interested in any other be punished by a fine of not more than five thousand pesos and by
corporation engaged in mining, was amended by section 7 of Act No. 3518 of imprisonment for not more than five years, in the discretion of the
the Philippine Legislature, approved by Congress March 1, 1929. The court. If the violation is committed by a corporation, the same shall,
change in the law effected by this amendment was in the direction of upon such violation being proved, be dissolved by quo
liberalization. Thus, the inhibition contained in the original provision against warranto proceedings instituted by the Attorney-General or by any
members of a corporation engaged in agriculture or mining from being provincial fiscal by order of said Attorney-General: . . . .
interested in other corporations engaged in agriculture or in mining was so
modified as merely to prohibit any such member from holding more than Upon a survey of the facts sketched above it is obvious that there are two
fifteen per centum of the outstanding capital stock of another such fundamental questions involved in this controversy. The first is whether the
corporation. Moreover, the explicit prohibition against the holding plaintiffs can maintain an action based upon the violation of law supposedly
by any corporation (except for irrigation) of an interest in any other committed by the Benguet Company in this case. The second is whether,
corporation engaged in agriculture or in mining was so modified as to limit assuming the first question to be answered in the affirmative, the Benguet
the restriction to corporations organized for the purpose of engaging in Company, which was organized as a sociedad anonima, is a corporation
agriculture or in mining. within the meaning of the language used by the Congress of the United
States, and later by the Philippine Legislature, prohibiting a mining
corporation from becoming interested in another mining corporation. It is

593
obvious that, if the first question be answered in the negative, it will be safely vaulted, the general remedy supplied in article 1305 of the Civil Code
unnecessary to consider the second question in this lawsuit. cannot be invoked where an adequate special remedy is supplied in a
special law. It has been so held by this court in Go Chioco vs. Martinez (45
Upon the first point it is at once obvious that the provision referred to was Phil., 256, 280), where we refused to apply that article to a case of nullity
adopted by the lawmakers with a sole view to the public policy that should arising upon a usurious loan. The reason given for the decision on this point
control in the granting of mining rights. Furthermore, the penalties imposed in was that the Usury Act, as amended, contains all the provisions necessary
what is now section 190 (A) of the Corporation Law for the violation of the for the effectuation of its purposes, with the result that the remedy given in
prohibition in question are of such nature that they can be enforced only by a article 1305 of the Civil Code is unnecessary. Much more is that idea
criminal prosecution or by an action of quo warranto. But these proceedings applicable to the situation now before us, where the special provisions give
can be maintained only by the Attorney-General in representation of the ample remedies for the enforcement of the law by action in the name of the
Government. Government, and where no civil wrong has been done to the party here
seeking redress.
What room then is left for the private action which the plaintiffs seek to assert
in this case? The defendant Benguet Company has committed no civil wrong The view of the case presented above rest upon considerations arising upon
against the plaintiffs, and if a public wrong has been committed, the directors our own statutes; and it would seem to be unnecessary to ransack the
of the Balatoc Company, and the plaintiff Harden himself, were the active American decisions for analogies pertinent to the case. We may observe,
inducers of the commission of that wrong. The contract, supposing it to have however, that the situation involved is not unlike that which has frequently
been unlawful in fact, has been performed on both sides, by the building of arisen in the United States under provisions of the National Bank Act
the Balatoc plant by the Benguet Company and the delivery to the latter of prohibiting banks organized under that law from holding real property. It has
the certificate of 600,000 shares of the Balatoc Company. There is no been uniformly held that a trust deed or mortgaged conveying property of this
possibility of really undoing what has been done. Nobody would suggest the kind to a bank, by way of security, is valid until the transaction is assailed in a
demolition of the mill. The Balatoc Company is secure in the possession of direct proceeding instituted by the Government against the bank, and the
that improvement, and talk about putting the parties in status quo ante by illegality of such tenure supplies no basis for an action by the former private
restoring the consideration with interest, while the Balatoc Company remains owner, or his creditor, to annul the conveyance. (National Bank vs. Matthews,
in possession of what it obtained by the use of that money, does not quite 98 U. S., 621; Kerfoot vs. Farmers & M. Bank, 218 U. S., 281.) Other
meet the case. Also, to mulct the Benguet Company in many millions of analogies point in the same direction. (South & Ala. R. Ginniss vs. B. & M.
dollars in favor of individuals who have not the slightest equitable right to that Consol. etc. Mining Co., 29 Mont., 428; Holmes & Griggs Mfg. Co. vs.
money in a proposition to which no court can give a ready assent. Holmes & Wessell Metal Co., 127 N. Y., 252; Oelbermann vs. N. Y. & N. R.
Co., 77 Hun., 332.)
The most plausible presentation of the case of the plaintiffs proceeds on the
assumption that only one of the contracting parties has been guilty of a
misdemeanor, namely, the Benguet Company, and that the other party, the
Balatoc Company, is wholly innocent to participation in that wrong. The
plaintiffs would then have us apply the second paragraph of article 1305 of
the Civil Code which declares that an innocent party to an illegal contract
may recover anything he may have given, while he is not bound to fulfill any
promise he may have made. But, supposing that the first hurdle can be

594
Most suggestive perhaps of all the cases in Compaia Azucarera de
Carolina vs. Registrar (19 Porto Rico, 143), for the reason that this case
arose under a provision of the Foraker Act, a law analogous to our Philippine
Bill. It appears that the registrar had refused to register two deeds in favor of
the Compaia Azucarera on the ground that the land thereby conveyed was
in excess of the area permitted by law to the company. The Porto Rican court
reversed the ruling of the registrar and ordered the registration of the deeds,
saying:

Thus it may be seen that a corporation limited by the law or by its


charter has until the State acts every power and capacity that any
other individual capable of acquiring lands, possesses. The
corporation may exercise every act of ownership over such lands; it
may sue in ejectment or unlawful detainer and it may demand
specific performance. It has an absolute title against all the world
except the State after a proper proceeding is begun in a court of
law. ... The Attorney General is the exclusive officer in whom is
confided the right to initiate proceedings for escheat or attack the
right of a corporation to hold land.

Having shown that the plaintiffs in this case have no right of action against
the Benguet Company for the infraction of law supposed to have been
committed, we forego cny discussion of the further question whether
a sociedad anonima created under Spanish law, such as the Benguet
Company, is a corporation within the meaning of the prohibitory provision
already so many times mentioned. That important question should, in our
opinion, be left until it is raised in an action brought by the Government.

The judgment which is the subject of his appeal will therefore be affirmed,
and it is so ordered, with costs against the appellants.

Avancea, C.J., Villamor, Ostrand, Villa-Real, Abad Santos, Hull, Vickers,


Imperial and Butte, JJ., concur.

595
thereon, issued by the Mindoro Sugar Company and placed in trust with the
Philippine Trust Company which, in turn, guaranteed them for value received.
Said plaintiff appealed from the judgment rendered by the Court of First
Instance of Manila absolving the defendants from the complaint, excepting
the Mindoro Sugar Company, which was sentenced to pay the value of the
four bonds with interest at 8 per cent per annum, plus costs.

The Mindoro Sugar Company is a corporation constituted in accordance with


the laws of the country and registered on July 30, 1917. According to its
articles of incorporation, Exhibit 5, one of its principal purposes was to
acquire and exercise the franchise granted by Act No. 2720 to George H.
Fairchild, to substitute the organized corporation, the Mindoro Company, and
to acquire all the rights and obligations of the latter and of Horace
Havemeyer and Charles J. Welch in the so-called San Jose Estate in the
Province of Mindoro.

The Philippine Trust Company is another domestic corporation, registered on


October 21, 1917. In its articles of incorporation, Exhibit A, some of its
purposes are expressed thus: "To acquire by purchase, subscription, or
otherwise, and to invest in, hold, sell, or otherwise dispose of stocks, bonds,
mortgages, and other securities, or any interest in either, or any obligations
G.R. No. L-36207 October 26, 1932 or evidences of indebtedness, of any other corporation or corporations,
domestic or foreign. . . . Without in any particular limiting any of the powers of
the corporation, it is hereby expressly declared that the corporation shall
IRINEO G. CARLOS, plaintiff-appellant,
have power to make any guaranty respecting the dividends, interest, stock,
vs.
bonds, mortgages, notes, contracts or other obligations of any corporation,
MINDORO SUGAR CO., ET AL., defendants-appellees.
so far as the same may be permitted by the laws of the Philippine Islands
now or hereafter in force." Its principal purpose, then, as its name indicates,
Jose Ayala for appellant. is to engage in the trust business.
Ross, Lawrence & Selph for appellees.
On November 17, 1917, the board of directors of the Philippine Trust
Company, composed of Phil, C. Whitaker, chairman, and James Ross, Otto
Vorster, Charles D. Ayton, and William J. O'Donovan, members, adopted a
IMPERIAL, J.: resolution authorizing its president, among other things, to purchase at par
and in the name and for the use of the trust corporation all or such part as he
The plaintiff brought this action to recover from the defendants the value of may deem expedient, of the bonds in the value of P3,000,000 that the
four bonds, Nos. 1219, 1220, 1221, and 1222, with due and unpaid interest Mindoro Sugar Company was about to issue, and to resell them, with or

596
without the guarantee of said trust corporation, at a price not less than par, Charles J. Welch and Horace Havemeyer up to the aforesaid amount of
and to guarantee to the Philippine National Bank the payment of the P2,000,000. The aforementioned deed was approved by his Excellency, the
indebtedness to said bank by the Mindoro Sugar Company or Charles J. Governor-General, upon recommendation of the Secretary of Agriculture and
Welch and Horace Havemeyer, up to P2,000,000. The relevant part of the Natural Resources, and in accordance with the provisions of Act No. 2720 of
resolution, Exhibit 3, reads as follows: the Philippine Legislature. Following are the clauses of said Exhibit 6 material
to this decision:
Resolved that Mr. Phil. C. Whitaker, president of this company, be
and he hereby is authorized to purchase at par in the name and for Whereas, for the purposes aforesaid, and in further pursuance of
the use of this company all, or such part as he may deem expedient, said resolutions of its board of directors and of its stockholders, the
of the said P3,000,000 of 20-year 8 per cent coupon bonds of the company, in order to secure the payment of said First Mortgage,
said Mindoro Sugar Company, and to resell or otherwise dispose of Twenty Year, Eight Per Cent, Gold Bonds, has determined to execute
the said bonds, with or without this company's guaranty, at a price and deliver to said Philippine Trust Company, as trustee, a deed of
not less than par; and it was further trust of its properties hereinafter described, and the board of
directors of the Company has approved the form of this indenture
Resolved that Mr. Phil. C. Whitaker, president of the company be and and directed that the same be executed and delivered to said
he hereby is authorized in the name of this company alone or in trustee; and
connection with others, by joint and several obligations, to guarantee
to the Philippine National Bank the due and punctual payment of any Whereas, all things necessary to make said bonds, when certified by
and all indebtedness owing to the said Bank by either the Mindoro said trustee as in this indenture provided, valid, binding, legal and
Sugar Company, the Mindoro Company, or Charles J. Welch and negotiable obligations of the company and this indenture a valid
Horace Havemeyer, up to P2,000,000; and it was further deed of trust to secure the payment of said bonds, have been done
and performed, and the creation and issue of said bonds, and the
Resolved that the said president, Mr. Phil. C. Whitaker, be and he execution, acknowledgment and delivery of this deed of trust have
hereby is authorized to execute in the name of this company any and been duly authorized;
all notes, mortgages, bonds, guaranties, or instruments in writing
whatever necessary for the carrying into effect of the authority Now, therefore, in order to secure the payment of the principal and
hereby granted. interest of all such bonds at any time issued and outstanding under
this indenture, according to their tenor, purport and effect, and to
In pursuance of this resolution, on December 21, 1917, the Mindoro Sugar secure the performance and observance of all the covenants and
Company executed in favor of the Philippine Trust Company the deed of conditions herein contained and to declare the terms and conditions
trust, Exhibit 6, transferring all of its property to it in consideration of the upon which said bonds are issued, received and held, and for and in
bonds it had issued to the value of P3,000,000, the value of each bond being consideration of the premises, and of the purchase or acceptance of
$1,000, which par value, with interest at 8 per cent per annum, the Philippine such bonds by the holders thereof, and of the sum of one dollar,
Trust Company had guaranteed to the holders, and in consideration, United States currency, to it duly paid at or before the ensealing and
furthermore, of said trust corporation having guaranteed to the Philippine delivery of these presents, the receipt whereof is hereby
National Bank all the obligations contracted by the Mindoro Sugar Company, acknowledged, the Mindoro Sugar Company, party of the first part,
has sold and conveyed, and by these presents does sell and convey

597
to the Philippine Trust Company, party of the second part, its The appellant now contends that the judgment appealed from is untenable,
successors and assigns forever; assigning the following errors:

(Description of the property.) FIRST ERROR

In consequence of this transaction, the bonds, with their coupons were The lower court erred in sustaining the demurrer against the
placed on the market and sold by the Philippine Trust Company, all endorsed amended complaint, filed by defendant J. S. Reis (Reese) and
as follows: consequently in dismissing the same with regard to this defendant.

This is to certify that the within bond is one of the series described in SECOND ERROR
the trust deed therein mentioned.
The lower court, without a proof to support it or an averment in
PHILIPPINE TRUST COMPANY defense by the defendant Philippine Trust Company, erred in finding
by: (Sgd.) PHIL. C. WHITAKER hypothetically that if the guarantee made by this company be held
President valid, the trust funds and deposits in its hands would probably be
endangered.
For values received, the Philippine Trust Company hereby
guarantees the payment of principal and interest of the within bond. THIRD ERROR

Manila, Jan.2, 1918 The lower court erred in holding that the Philippine Trust Company
has no power to guarantee the obligation of another juridical
PHILIPPINE TRUST COMPANY personality, for value received.
by: (Sgd.) PHIL. C. WHITAKER
President FOURTH ERROR

The Philippine Trust Company sold thirteen bonds, Nos. 1219 to 1231, to The lower court erred in not recognizing the validity and effect of the
Ramon Diaz for P27,300, at a net profit of P100 per bond. The four bonds guarantee subscribed by the Philippine Trust Company for the
Nos. 1219, 1220, 1221, and 1222, here in litigation, are included in the payment of the four bonds claimed in the complaint, endorsed upon
thirteen sold to Diaz. them, and in absolving said institution from the complaint.

The Philippine Trust Company paid the appellant, upon presentation of the FIFTH ERROR
coupons, the stipulated interest from the date of their maturity until the 1st of
July, 1928, when it stopped payments; and thenceforth it alleged that it did The lower court erred in absolving the ex-directors of the Philippine
not deem itself bound to pay such interest or to redeem the obligation Trust Company, Phil. C. Whitaker, O. Vorster, and Charles D. Ayton,
because the guarantee given for the bonds was illegal and void. from the complaint.

598
We shall not follow the order of the appellant's argument, deeming it "Whenever a corporation has the power to take and dispose of the securities
unnecessary, but shall decide only the third and fourth assignments of error of another corporation, of whatsoever kind, it may, for the purpose of giving
upon which the merits of the case depend. For the clear understanding of them a marketable quality, guarantee their payment, even though the amount
this decision and to avoid erroneous interpretations, however, we wish to involved in the guaranty may subject the corporation to liabilities in excess of
state that in this decision we shall decide only the rights of the parties with the limit of indebtedness which it is authorized to incur. A corporation which
regard to the four bonds in question and whatever we say in no wise affects has power by its charter to issue its own bonds has power to guarantee the
or applies to the rest of the bonds. bonds of another corporation, which has been taken in payment of a debt
due to it, and which it sells or transfers in payment of its own debt, the
We shall begin by saying that the majority of the justices of this court who guaranty being given to enable it to dispose of the bond to better advantage.
took part in the case are of opinion that the only point of law to be decided is And so guaranties of payment of bonds taken by a loan and trust company in
whether the Philippine Trust Company acquired the four bonds in question, the ordinary course of its business, made in connection with their sale, are
and whether as such it bound itself legally and acted within its corporate not ultra vires, and are binding." (14-A C. J., pp. 742-743 and cases cited);
powers in guaranteeing them. This question was answered in the thirdly, that although it does not clearly appear in the deed of trust (Exhibit 6)
affirmative.1awphil.net that the Mindoro Sugar Company transferred the bonds therein referred to, to
the Philippine Trust Company, nevertheless, in the resolution of the board of
In adopting this conclusion we have relied principally upon the following facts directors (Exhibit 3), the president of the Philippine Trust Company was
and circumstances: Firstly, that the Philippine Trust Company, although expressly authorized to purchase all or some of the bonds and to guarantee
secondarily engaged in banking, was primarily organized as a trust them; whence it may be inferred that subsequent purchasers of the bonds in
corporation with full power to acquire personal property such as the bonds in the market relied upon the belief that they were acquiring securities of the
question according to both section 13 (par. 5) of the Corporation Law and its Philippine Trust Company, guaranteed by this corporation; fourthly, that as
duly registered by-laws and articles of incorporation; secondly, that being soon as P3,000,000 worth of bonds was issued, and by the deed of trust the
thus authorized to acquire the bonds, it was given implied power to Mindoro, Sugar Company transferred all its real property to the Philippine
guarantee them in order to place them upon the market under better, more Trust Company, the cause or consideration of the transfer being, (1) the
advantageous conditions, and thereby secure the profit derived from their guarantee given by the purchaser to the bonds, and (2) its having likewise
sale: guaranteed its obligations and those of Welch and Havemeyer in favor of the
Philippine National Bank up to the amount of P2,000,000; fifthly, that in
transferring its real property as aforesaid the Mindoro Sugar Company was
It is not, however, ultra vires for a corporation to enter into contracts
reduced to a real state of bankruptcy, as the parties specifically agreed
of guaranty or suretyship where it does so in the legitimate
during the hearing of the case, to the point of having become a nominal
furtherance of its purposes and business. And it is well settled that
corporation without any assets whatsoever; sixthly, that such operation or
where a corporation acquires commercial paper or bonds in the
transaction cannot mean anything other than that the real intention of the
legitimate transaction of its business it may sell them, and in
parties was that the Philippine Trust Company acquired the bonds issued
furtherance of such a sale it may, in order to make them the more
and at the same time guaranteed the payment of their par value with interest,
readily marketable, indorse or guarantee their payment. (7 R. C. L.,
because otherwise the transaction would be fraudulent, inasmuch as nobody
p. 604 and cases cited.)
would be answerable to the bond-holders for their value and interest;
seventhly, that the Philippine Trust Company had been paying the appellant
the interest accrued upon the four bonds from the date of their issuance until

599
July 1, 1928, such payment of interest being another proof that said or authorize an action to be maintained directly upon it. However, the
corporation had really become the owner of the aforesaid bonds; and, eightly, great weight of authority in the state courts is to the effect that a
that the Philippine Trust Company has not adduced any evidence to show transaction which is merely ultra vires and not malum in se or malum
any other conclusions. prohibitum although it may be made by the state a basis for the
forfeiture of the corporate charter or the dissolution of the
There are other considerations leading to the same result even in the corporation, is, if performed by one party, not void as between the
supposition that the Philippine Trust Company did not acquire the bonds in parties to all intents and purposes, and that an action may be
question, but only guaranteed them. In such a case the guarantee of these brought directly upon the transaction and relief had according to its
bonds would at any rate, be valid and the said corporation would be bound to terms. ( 14-A C. J., pp. 319-320.)
pay the appellant their value with the accrued interest in view of the fact that
they become due on account of the lapse of sixty (60) days, without the When a contract is not on its face necessarily beyond the scope of
accrued interest due having been paid; and the reason is that it is estopped the power of the corporation by which it was made, it will, in the
from denying the validity of its guarantee. absence of proof to the contrary, be presumed to be valid.
Corporations are presumed to contract within their powers. The
. . . On the other hand, according to the view taken by other courts, doctrine of ultra vires, when invoked for or against a corporation,
which it must be acknowledged are in the majority, a recovery should not be allowed to prevail where it would defeat the ends of
directly upon the contract is permitted, on the ground that the justice or work a legal wrong. (Coleman vs. Hotel de France Co., 29
corporation, having received money or property by virtue of a Phil., 323.)
contract not immoral or illegal of itself, is estopped to deny liability;
and that the only remedy is one on behalf of the state to punish the Guaranties of payment of bonds taken by a loan and trust company
corporation for violating the law. (7 R. C. L., pp. 680-681 and cases in the ordinary course of its business, made in connection with their
cited.) sale, are not ultra vires, and are binding. (Broadway Nat. Bank vs.
Baker, 57 N. E., p. 603.)
. . . The doctrine of ultra vires has been declared to be entirely the
creation of the courts and is of comparatively modern origin. The It has been intimated according to section 121 of the Corporation Law, the
defense is by some courts regarded as an ungracious and odious Philippine Trust Company, as a banking institution, could not guarantee the
one, to be sustained only where the most persuasive considerations bonds to the value of P3,000,000 because this amount far exceeds its capital
of public policy are involved, and there are numerous decisions and of P1,000,000 of which only one-half has been subscribed and paid. Section
dicta to the effect that the plea should not as a general rule prevail 121 reads as follows:
whether interposed for or against the corporation, where it will not
advance justice but on the contrary will accomplish a legal wrong. SEC. 212. No such bank shall at any time be indebted or in any way
(14-A C. J., pp. 314-315.) liable to an amount exceeding the amount of its capital stock at such
time actually paid in and remaining undiminished by losses or
The doctrine of the Supreme Court of the United States together with otherwise, except on account of demands of the following nature:
the English courts and some of the state courts is that no
performance upon either side can validate an ultra vires transaction (1) Moneys deposited with or collected by the bank;

600
(2) Bills of exchange or drafts drawn against money actually
on deposit to the credit of the bank or due thereto;

(3) Liabilities to the stockholders of the bank for dividends


and reserve profits.

This difficulty is easily obviated by bearing in mind that, as we stated above,


the banking operations are not the primary aim of said corporation, which is
engaged essentially in the trust business, and that the prohibition of the law
is not applicable to the Philippine Trust Company, for the evidence shows
that Mindoro Sugar Company transferred all its real property, with the
improvements, to it, and the value of both, which surely could not be less
than the value of the obligation guaranteed, became a part of its capital and
assets; in other words, with the value of the real property transferred to it, the G.R. No. L-27155 May 18, 1978
Philippine Trust Company had enough capital and assets to meet the amount
of the bonds guaranteed with interest thereon. PHILIPPINE NATIONAL BANK, petitioner,
vs.
Wherefore, the decision appealed from is reversed and the Philippine Trust THE COURT OF APPEALS, RITA GUECO TAPNIO, CECILIO GUECO and
Company is sentenced to pay to the appellant the sum of four thousand THE PHILIPPINE AMERICAN GENERAL INSURANCE COMPANY,
dollars ($4,000) with interest at eight per cent (8%) per annum from July 1, INC., respondents.
1928 until fully paid, and the costs of both instances. So ordered.
Medina, Locsin, Corua, & Sumbillo for petitioner.
Avancea, C.J., Ostrand, Villa-Real, Abad Santos and Butte, JJ., concur.
Malcolm and Hull, JJ., concur in the result. Manuel Lim & Associates for private respondents.

ANTONIO, J.:

Certiorari to review the decision of the Court of Appeals which affirmed the
judgment of the Court of First Instance of Manila in Civil Case No. 34185,
ordering petitioner, as third-party defendant, to pay respondent Rita Gueco
Tapnio, as third-party plaintiff, the sum of P2,379.71, plus 12% interest per
annum from September 19, 1957 until the same is fully paid, P200.00
attorney's fees and costs, the same amounts which Rita Gueco Tapnio was
ordered to pay the Philippine American General Insurance Co., Inc., to be

601
paid directly to the Philippine American General Insurance Co., Inc. in full It is not disputed that defendant Rita Gueco Tapnio was
satisfaction of the judgment rendered against Rita Gueco Tapnio in favor of indebted to the bank in the sum of P2,000.00, plus
the former; plus P500.00 attorney's fees for Rita Gueco Tapnio and costs. accumulated interests unpaid, which she failed to pay
The basic action is the complaint filed by Philamgen (Philippine American despite demands. The Bank wrote a letter of demand to
General Insurance Co., Inc.) as surety against Rita Gueco Tapnio and Cecilio plaintiff, as per Exh. C; whereupon, plaintiff paid the bank on
Gueco, for the recovery of the sum of P2,379.71 paid by Philamgen to the September 18, 1957, the full amount due and owing in the
Philippine National Bank on behalf of respondents Tapnio and Gueco, sum of P2,379.91, for and on account of defendant Rita
pursuant to an indemnity agreement. Petitioner Bank was made third-party Gueco's obligation (Exhs. D and D-1).
defendant by Tapnio and Gueco on the theory that their failure to pay the
debt was due to the fault or negligence of petitioner. Plaintiff, in turn, made several demands, both verbal and
written, upon defendants (Exhs. E and F), but to no avail.
The facts as found by the respondent Court of Appeals, in affirming the
decision of the Court of First Instance of Manila, are quoted hereunder: Defendant Rita Gueco Tapnio admitted all the foregoing
facts. She claims, however, when demand was made upon
Plaintiff executed its Bond, Exh. A, with defendant Rita her by plaintiff for her to pay her debt to the Bank, that she
Gueco Tapnio as principal, in favor of the Philippine National told the Plaintiff that she did not consider herself to be
Bank Branch at San Fernando, Pampanga, to guarantee the indebted to the Bank at all because she had an agreement
payment of defendant Rita Gueco Tapnio's account with said with one Jacobo-Nazon whereby she had leased to the latter
Bank. In turn, to guarantee the payment of whatever amount her unused export sugar quota for the 1956-1957 agricultural
the bonding company would pay to the Philippine National year, consisting of 1,000 piculs at the rate of P2.80 per picul,
Bank, both defendants executed the indemnity agreement, or for a total of P2,800.00, which was already in excess of
Exh. B. Under the terms and conditions of this indemnity her obligation guaranteed by plaintiff's bond, Exh. A. This
agreement, whatever amount the plaintiff would pay would lease agreement, according to her, was with the knowledge
earn interest at the rate of 12% per annum, plus attorney's of the bank. But the Bank has placed obstacles to the
fees in the amount of 15 % of the whole amount due in case consummation of the lease, and the delay caused by said
of court litigation. obstacles forced 'Nazon to rescind the lease contract. Thus,
Rita Gueco Tapnio filed her third-party complaint against the
The original amount of the bond was for P4,000.00; but the Bank to recover from the latter any and all sums of money
amount was later reduced to P2,000.00. which may be adjudged against her and in favor of the plaitiff
plus moral damages, attorney's fees and costs.

Insofar as the contentions of the parties herein are


concerned, we quote with approval the following findings of
the lower court based on the evidence presented at the trial
of the case:

602
It has been established during the trial that "2-Gueco") informing them that "the
Mrs. Tapnio had an export sugar quota of minimum lease rental acceptable to the
1,000 piculs for the agricultural year 1956- Bank, is P2.80 per picul." In a letter
1957 which she did not need. She agreed to addressed to the branch manager on August
allow Mr. Jacobo C. Tuazon to use said 10, 1956, Mr. Tuazon informed the manager
quota for the consideration of P2,500.00 that he was agreeable to raising the
(Exh. "4"-Gueco). This agreement was consideration to P2.80 per picul. He further
called a contract of lease of sugar allotment. informed the manager that he was ready to
pay said amount as the funds were in his
At the time of the agreement, Mrs. Tapnio folder which was kept in the bank.
was indebted to the Philippine National Bank
at San Fernando, Pampanga. Her Explaining the meaning of Tuazon's
indebtedness was known as a crop loan and statement as to the funds, it was stated by
was secured by a mortgage on her standing him that he had an approved loan from the
crop including her sugar quota allocation for bank but he had not yet utilized it as he was
the agricultural year corresponding to said intending to use it to pay for the quota.
standing crop. This arrangement was Hence, when he said the amount needed to
necessary in order that when Mrs. Tapnio pay Mrs. Tapnio was in his folder which was
harvests, the P.N.B., having a lien on the in the bank, he meant and the manager
crop, may effectively enforce collection understood and knew he had an approved
against her. Her sugar cannot be exported loan available to be used in payment of the
without sugar quota allotment Sometimes, quota. In said Exh. "6-Gueco", Tuazon also
however, a planter harvest less sugar than informed the manager that he would want
her quota, so her excess quota is utilized by for a notice from the manager as to the time
another who pays her for its use. This is the when the bank needed the money so that
arrangement entered into between Mrs. Tuazon could sign the corresponding
Tapnio and Mr. Tuazon regarding the promissory note.
former's excess quota for 1956-1957 (Exh.
"4"-Gueco). Further Consideration of the evidence discloses that when
the branch manager of the Philippine National Bank at San
Since the quota was mortgaged to the Fernando recommended the approval of the contract of
P.N.B., the contract of lease had to be lease at the price of P2.80 per picul (Exh. 1 1-Bank), whose
approved by said Bank, The same was recommendation was concurred in by the Vice-president of
submitted to the branch manager at San said Bank, J. V. Buenaventura, the board of directors
Fernando, Pampanga. The latter required required that the amount be raised to 13.00 per picul. This
the parties to raise the consideration of act of the board of directors was communicated to Tuazon,
P2.80 per picul or a total of P2,800.00 (Exh. who in turn asked for a reconsideration thereof. On

603
November 19, 1956, the branch manager submitted were secured by chattel mortgage on standing crops,
Tuazon's request for reconsideration to the board of directors assignment of leasehold rights and interests on her
with another recommendation for the approval of the lease at properties, and surety bonds, aside from the fact that from
P2.80 per picul, but the board returned the recommendation Exh. 8-Bank, it appears that she was offering to execute a
unacted upon, considering that the current price prevailing at real estate mortgage in favor of the Bank to replace the
the time was P3.00 per picul (Exh. 9-Bank). surety bond This statement is further bolstered by the fact
that Rita Gueco Tapnio apparently had the means to pay her
The parties were notified of the refusal on the part of the obligation fact that she has been granted several value of
board of directors of the Bank to grant the motion for almost P80,000.00 for the agricultural years from 1952 to
reconsideration. The matter stood as it was until February 56. 1
22, 1957, when Tuazon wrote a letter (Exh. 10-Bank
informing the Bank that he was no longer interested to Its motion for the reconsideration of the decision of the Court of Appeals
continue the deal, referring to the lease of sugar quota having been denied, petitioner filed the present petition.
allotment in favor of defendant Rita Gueco Tapnio. The result
is that the latter lost the sum of P2,800.00 which she should The petitioner contends that the Court of Appeals erred:
have received from Tuazon and which she could have paid
the Bank to cancel off her indebtedness, (1) In finding that the rescission of the lease contract of the 1,000 piculs of
sugar quota allocation of respondent Rita Gueco Tapnio by Jacobo C.
The court below held, and in this holding we concur that Tuazon was due to the unjustified refusal of petitioner to approve said lease
failure of the negotiation for the lease of the sugar quota contract, and its unreasonable insistence on the rental price of P3.00 instead
allocation of Rita Gueco Tapnio to Tuazon was due to the of P2.80 per picul; and
fault of the directors of the Philippine National Bank, The
refusal on the part of the bank to approve the lease at the (2) In not holding that based on the statistics of sugar price and prices of
rate of P2.80 per picul which, as stated above, would have sugar quota in the possession of the petitioner, the latter's Board of Directors
enabled Rita Gueco Tapnio to realize the amount of correctly fixed the rental of price per picul of 1,000 piculs of sugar quota
P2,800.00 which was more than sufficient to pay off her leased by respondent Rita Gueco Tapnio to Jacobo C. Tuazon at P3.00 per
indebtedness to the Bank, and its insistence on the rental picul.
price of P3.00 per picul thus unnecessarily increasing the
value by only a difference of P200.00. inevitably brought
Petitioner argued that as an assignee of the sugar quota of Tapnio, it has the
about the rescission of the lease contract to the damage and
right, both under its own Charter and under the Corporation Law, to
prejudice of Rita Gueco Tapnio in the aforesaid sum of
safeguard and protect its rights and interests under the deed of assignment,
P2,800.00. The unreasonableness of the position adopted by
which include the right to approve or disapprove the said lease of sugar
the board of directors of the Philippine National Bank in
quota and in the exercise of that authority, its
refusing to approve the lease at the rate of P2.80 per picul
and insisting on the rate of P3.00 per picul, if only to
increase the retail value by only P200.00 is shown by the Board of Directors necessarily had authority to determine and fix the rental
fact that all the accounts of Rita Gueco Tapnio with the Bank price per picul of the sugar quota subject of the lease between private

604
respondents and Jacobo C. Tuazon. It argued further that both under its folder which was kept in the said Bank. This referred to the approved loan of
Charter and the Corporation Law, petitioner, acting thru its Board of Tuazon from the Bank which he intended to use in paying for the use of the
Directors, has the perfect right to adopt a policy with respect to fixing of rental sugar quota. The Branch Manager submitted the contract of lease of sugar
prices of export sugar quota allocations, and in fixing the rentals at P3.00 per quota allocation to the Head Office on September 7, 1956, with a
picul, it did not act arbitrarily since the said Board was guided by statistics of recommendation for approval, which recommendation was concurred in by
sugar price and prices of sugar quotas prevailing at the time. Since the fixing the Vice-President of the Bank, Mr. J. V. Buenaventura. This notwithstanding,
of the rental of the sugar quota is a function lodged with petitioner's Board of the Board of Directors of petitioner required that the consideration be raised
Directors and is a matter of policy, the respondent Court of Appeals could not to P3.00 per picul.
substitute its own judgment for that of said Board of Directors, which acted in
good faith, making as its basis therefore the prevailing market price as shown Tuazon, after being informed of the action of the Board of Directors, asked
by statistics which were then in their possession. for a reconsideration thereof. On November 19, 1956, the Branch Manager
submitted the request for reconsideration and again recommended the
Finally, petitioner emphasized that under the appealed judgment, it shall approval of the lease at P2.80 per picul, but the Board returned the
suffer a great injustice because as a creditor, it shall be deprived of a just recommendation unacted, stating that the current price prevailing at that time
claim against its debtor (respondent Rita Gueco Tapnio) as it would be was P3.00 per picul.
required to return to respondent Philamgen the sum of P2,379.71, plus
interest, which amount had been previously paid to petitioner by said On February 22, 1957, Tuazon wrote a letter, informing the Bank that he was
insurance company in behalf of the principal debtor, herein respondent Rita no longer interested in continuing the lease of sugar quota allotment. The
Gueco Tapnio, and without recourse against respondent Rita Gueco Tapnio. crop year 1956-1957 ended and Mrs. Tapnio failed to utilize her sugar quota,
resulting in her loss in the sum of P2,800.00 which she should have received
We must advert to the rule that this Court's appellate jurisdiction in had the lease in favor of Tuazon been implemented.
proceedings of this nature is limited to reviewing only errors of law, accepting
as conclusive the factual fin dings of the Court of Appeals upon its own It has been clearly shown that when the Branch Manager of petitioner
assessment of the evidence. 2 required the parties to raise the consideration of the lease from P2.50 to
P2.80 per picul, or a total of P2,800-00, they readily agreed. Hence, in his
The contract of lease of sugar quota allotment at P2.50 per picul between letter to the Branch Manager of the Bank on August 10, 1956, Tuazon
Rita Gueco Tapnio and Jacobo C. Tuazon was executed on April 17, 1956. informed him that the minimum lease rental of P2.80 per picul was
This contract was submitted to the Branch Manager of the Philippine National acceptable to him and that he even offered to use the loan secured by him
Bank at San Fernando, Pampanga. This arrangement was necessary from petitioner to pay in full the sum of P2,800.00 which was the total
because Tapnio's indebtedness to petitioner was secured by a mortgage on consideration of the lease. This arrangement was not only satisfactory to the
her standing crop including her sugar quota allocation for the agricultural Branch Manager but it was also approves by Vice-President J. V.
year corresponding to said standing crop. The latter required the parties to Buenaventura of the PNB. Under that arrangement, Rita Gueco Tapnio could
raise the consideration to P2.80 per picul, the minimum lease rental have realized the amount of P2,800.00, which was more than enough to pay
acceptable to the Bank, or a total of P2,800.00. Tuazon informed the Branch the balance of her indebtedness to the Bank which was secured by the bond
Manager, thru a letter dated August 10, 1956, that he was agreeable to of Philamgen.
raising the consideration to P2.80 per picul. He further informed the manager
that he was ready to pay the said sum of P2,800.00 as the funds were in his

605
There is no question that Tapnio's failure to utilize her sugar quota for the respondents would be unable to utilize the sugar quota in question. In failing
crop year 1956-1957 was due to the disapproval of the lease by the Board of to observe the reasonable degree of care and vigilance which the
Directors of petitioner. The issue, therefore, is whether or not petitioner is surrounding circumstances reasonably impose, petitioner is consequently
liable for the damage caused. liable for the damages caused on private respondents. Under Article 21 of
the New Civil Code, "any person who wilfully causes loss or injury to another
As observed by the trial court, time is of the essence in the approval of the in a manner that is contrary to morals, good customs or public policy shall
lease of sugar quota allotments, since the same must be utilized during the compensate the latter for the damage." The afore-cited provisions on human
milling season, because any allotment which is not filled during such milling relations were intended to expand the concept of torts in this jurisdiction by
season may be reallocated by the Sugar Quota Administration to other granting adequate legal remedy for the untold number of moral wrongs which
holders of allotments. 3 There was no proof that there was any other person is impossible for human foresight to specifically provide in the statutes. 5
at that time willing to lease the sugar quota allotment of private respondents
for a price higher than P2.80 per picul. "The fact that there were isolated A corporation is civilly liable in the same manner as natural persons for torts,
transactions wherein the consideration for the lease was P3.00 a picul", because "generally speaking, the rules governing the liability of a principal or
according to the trial court, "does not necessarily mean that there are always master for a tort committed by an agent or servant are the same whether the
ready takers of said price. " The unreasonableness of the position adopted principal or master be a natural person or a corporation, and whether the
by the petitioner's Board of Directors is shown by the fact that the difference servant or agent be a natural or artificial person. All of the authorities agree
between the amount of P2.80 per picul offered by Tuazon and the P3.00 per that a principal or master is liable for every tort which he expressly directs or
picul demanded by the Board amounted only to a total sum of P200.00. authorizes, and this is just as true of a corporation as of a natural person, A
Considering that all the accounts of Rita Gueco Tapnio with the Bank were corporation is liable, therefore, whenever a tortious act is committed by an
secured by chattel mortgage on standing crops, assignment of leasehold officer or agent under express direction or authority from the stockholders or
rights and interests on her properties, and surety bonds and that she had members acting as a body, or, generally, from the directors as the governing
apparently "the means to pay her obligation to the Bank, as shown by the body." 6
fact that she has been granted several sugar crop loans of the total value of
almost P80,000.00 for the agricultural years from 1952 to 1956", there was WHEREFORE, in view of the foregoing, the decision of the Court of Appeals
no reasonable basis for the Board of Directors of petitioner to have rejected is hereby AFFIRMED.
the lease agreement because of a measly sum of P200.00.
Fernando, Aquino, Concepcion, Jr., and Santos, JJ., concur.
While petitioner had the ultimate authority of approving or disapproving the
proposed lease since the quota was mortgaged to the Bank, the latter
certainly cannot escape its responsibility of observing, for the protection of
the interest of private respondents, that degree of care, precaution and
vigilance which the circumstances justly demand in approving or
disapproving the lease of said sugar quota. The law makes it imperative that
every person "must in the exercise of his rights and in the performance of his
duties, act with justice, give everyone his due, and observe honesty and
good faith, 4 This petitioner failed to do. Certainly, it knew that the agricultural
year was about to expire, that by its disapproval of the lease private

606
That on and during the four quarters of the year 1924, in the
municipality of Iloilo, Province of Iloilo, Philippine Islands, the said
accused, as corporation organized under the laws of the Philippine
Islands and engaged in the purchase and the sale of sugar, "bayon,"
coprax, and other native products and as such object to the payment
of internal-revenue taxes upon its sales, did then and there
voluntarily, illegally, and criminally declare in 1924 for the purpose of
taxation only the sum of P2,352,761.94, when in truth and in fact,
and the accused well knew that the total gross sales of said
corporation during that year amounted to P2543,303.44, thereby
failing to declare for the purpose of taxation the amount of
P190,541.50, and voluntarily and illegally not paying the Government
as internal-revenue percentage taxes the sum of P2,960.12,
corresponding to 1 per cent of said undeclared sales.

The question to be decided is whether the information sets forth facts


rendering the defendant, as manager of the corporation liable criminally
under section 2723 of Act No. 2711 for violation of section 1458 of the same
act for the benefit of said corporation. Section 1458 and 2723 read as
G.R. No. L-35262 March 15, 1930 follows:

THE PEOPLE OF THE PHILIPPINE ISLANDS, plaintiff-appellant, SEC. 1458. Payment of percentage taxes Quarterly reports of
vs. earnings. The percentage taxes on business shall be payable at
TAN BOON KONG, defendant-appellee. the end of each calendar quarter in the amount lawfully due on the
business transacted during each quarter; and it shall be on the duty
of every person conducting a business subject to such tax, within the
Attorney-General Jaranilla for appellant.
same period as is allowed for the payment of the quarterly
Alejandro de Aboitiz Pinaga for appellee.
installments of the fixed taxes without penalty, to make a true and
complete return of the amount of the receipts or earnings of his
OSTRAND, J.: business during the preceeding quarter and pay the tax due thereon.
. . . (Act No. 2711.)
This is an appeal from an order of the Judge of the Twenty-third Judicial
District sustaining to demurrer to an information charging the defendant Tan SEC. 2723. Failure to make true return of receipts and sales. Any
Boon Kong with the violation of section 1458 of Act No. 2711 as amended. person who, being required by law to make a return of the amount of
The information reads as follows: his receipts, sales, or business, shall fail or neglect to make such
return within the time required, shall be punished by a fine not

607
exceeding two thousand pesos or by imprisonment for a term not
exceeding one year, or both.

And any such person who shall make a false or fraudulent return
shall be punished by a fine not exceeding ten thousand pesos or by
imprisonment for a term not exceeding two years, or both. (Act No.
2711.)

Apparently, the court below based the appealed ruling on the ground that the
offense charged must be regarded as committed by the corporation and not
by its officials or agents. This view is in direct conflict with the great weight of
authority. a corporation can act only through its officers and agent s, and
where the business itself involves a violation of the law, the correct rule is
that all who participate in it are liable (Grall and Ostrand's Case, 103 Va.,
855, and authorities there cited.)

In case of State vs. Burnam (17 Wash., 199), the court went so far as to hold
that the manager of a diary corporation was criminally liable for the violation
of a statute by the corporation through he was not present when the offense
was committed.

In the present case the information or complaint alleges that he defendant


was the manager of a corporation which was engaged in business as a
merchant, and as such manager, he made a false return, for purposes of
taxation, of the total amount of sale made by said false return constitutes a
violation of law, the defendant, as the author of the illegal act, must
necessarily answer for its consequences, provided that the allegation are
proven.

The ruling of the court below sustaining the demurrer to the complaint is
therefore reversed, and the case will be returned to said court for further
proceedings not inconsistent with our view as hereinafter stated. Without
costs. So ordered.

Johnson, Malcolm, Villamor, Johns, Romualdez and Villa-Real, JJ., concur.

608
DE CASTRO, J.:

Petition for review of the decision of the Court of Appeals affirming the
decision of the Court of First Instance of Manila convicting the appellant of
estafa, under an information which reads:

That in, about or during the period comprised' between July


24, 1963 and December 31, 1963, both dates inclusive, in
the City of Manila, Philippines, the said accused did then and
there willfully, unlawfully and feloniously defraud the
Continental Bank, a banking institution duly organized and
doing business in the City of Manila, in the following manner,
to wit: the said accused, in his capacity as president and
general manager of the Metal Manufacturing of the
Philippines, Inc. (MEMAP) and on behalf of said company,
obtained delivery of 150 M/T Cold Rolled Steel Sheets
valued at P 71,023.60 under a trust receipt agreement under
L/C No. 63/109, which cold rolled steel sheets were
consigned to the Continental Bank, under the express
obligation on the part of said accused of holding the said
steel sheets in trust and selling them and turning over the
proceeds of the sale to the Continental Bank; but the said
accused, once in possession of the said goods, far from
complying with his aforesaid obligation and despite demands
made upon him to do so, with intent to defraud, failed and
refused to return the said cold rolled sheets or account for
the proceeds thereof, if sold, which the said accused willfully,
unlawfully and feloniously misappropriated, misapplied and
G.R. No. L-30896 April 28, 1983 converted to his own personal use and benefit, to the
damage and prejudice of the said Continental Bank in the
JOSE O. SIA, petitioner, total amount of P146,818.68, that is the balance including
vs. the interest after deducting the sum of P28,736.47 deposited
THE PEOPLE OF THE PHILIPPINES, respondent. by the said accused with the bank as marginal deposit and
forfeited by the said from the value of the said goods, in the
said sum of P71,023.60. (Original Records, p. 1).

609
In reviewing the evidence, the Court of Appeals came up with the following The first issue raised, which in effect combines the first three errors
findings of facts which the Solicitor General alleges should be conclusive assigned, is whether petitioner Jose O. Sia, having only acted for and in
upon this Court: behalf of the Metal Manufacturing Company of the Philippines (Metal
Company, for short) as President thereof in dealing with the complainant, the
There is no debate on certain antecedents: Accused Jose 0. Continental Bank, (Bank for short) he may be liable for the crime charged.
Sia sometime prior to 24 May, 1963, was General Manager
of the Metal Manufacturing Company of the Philippines, Inc. In discussing this question, petitioner proceeds, in the meantime, on the
engaged in the manufacture of steel office equipment; on 31 assumption that the acts imputed to him would constitute the crime of estafa,
May, 1963, because his company was in need of raw which he also disputes, but seeks to avoid liability on his theory that the Bank
materials to be imported from abroad, he applied for a letter knew all along that petitioner was dealing with him only as an officer of the
of credit to import steel sheets from Mitsui Bussan Kaisha, Metal Company which was the true and actual applicant for the letter of credit
Ltd. of Tokyo, Japan, the application being directed to the (Exhibit B) and which, accordingly, assumed sole obligation under the trust
Continental Bank, herein complainant, Exhibit B and his receipt (Exhibit A). In disputing the theory of petitioner, the Solicitor General
application having been approved, the letter of credit was relies on the general principle that when a corporation commits an act which
opened on 5 June, 1963 in the amount of $18,300, Exhibit D; would constitute a punishable offense under the law, it is the responsible
and the goods arrived sometime in July, 1963 according to officers thereof, acting for the corporation, who would be punished for the
accused himself, tsn. II:7; now from here on there is some crime, The Court of Appeals has subscribed to this view when it quoted
debate on the evidence; according to Complainant Bank, approvingly from the decision of the trial court the following:
there was permitted delivery of the steel sheets only upon
execution of a trust receipt, Exhibit A; while according to the A corporation is an artificial person, an abstract being. If the
accused, the goods were delivered to him sometime before defense theory is followed unscrupulously legions would
he executed that trust receipt in fact they had already been form corporations to commit swindle right and left where
converted into steel office equipment by the time he signed nobody could be convicted, for it would be futile and
said trust receipt, tsn. II:8; but there is no question - and this ridiculous to convict an abstract being that can not be
is not debated - that the bill of exchange issued for the pinched and confined in jail like a natural, living person,
purpose of collecting the unpaid account thereon having hence the result of the defense theory would be hopeless
fallen due (see Exh. B) neither accused nor his company chose in business and finance. It is completely untenable.
having made payment thereon notwithstanding demands, (Rollo [CA], p. 108.)
Exh. C and C-1, dated 17 and 27 December, 1963, and the
accounts having reached the sum in pesos of P46,818.68 The above-quoted observation of the trial court would seem to be merely
after deducting his deposit valued at P28,736.47; that was restating a general principle that for crimes committed by a corporation, the
the reason why upon complaint by Continental Bank, the responsible officers thereof would personally bear the criminal liability.
Fiscal filed the information after preliminary investigation as (People vs. Tan Boon Kong, 54 Phil. 607. See also Tolentino, Commercial
has been said on 22 October, 1964. (Rollo [CA], pp. 103- Laws of the Philippines, p. 625, citing cases.)
104).

610
The case cited by the Court of Appeals in support of its stand-Tan Boon Kong The next question is whether the violation of a trust receipt constitutes estafa
case, supra-may however not be squarely applicable to the instant case in under Art. 315 (1-[2]) of the Revised Penal Code, as also raised by the
that the corporation was directly required by law to do an act in a given petitioner. We now entertain grave doubts, in the light of the promulgation of
manner, and the same law makes the person who fails to perform the act in P.D. 115 providing for the regulation of trust receipts transaction, which is a
the prescribed manner expressly liable criminally. The performance of the act very comprehensive piece of legislation, and includes an express provision
is an obligation directly imposed by the law on the corporation. Since it is a that if the violation or offense is committed by a corporation, partnership,
responsible officer or officers of the corporation who actually perform the act association or other juridical entities the penalty provided for in this Decree
for the corporation, they must of necessity be the ones to assume the shall be imposed upon the directors, officers, employees or other officials or
criminal liability; otherwise this liability as created by the law would be persons therein responsible for the offense, without prejudice to civil liabilities
illusory, and the deterrent effect of the law, negated. arising from the criminal offense. The question that suggests itself is,
therefore, whether the provisions of the Revised Penal Code, Article 315, par.
In the present case, a distinction is to be found with the Tan Boon Kong case 1 (b) are not adequate to justify the punishment of the act made punishable
in that the act alleged to be a crime is not in the performance of an act by P.D. 115, that the necessity was felt for the promulgation of the decree. To
directly ordained by law to be performed by the corporation. The act is answer this question, it is imperative to make an indepth analysis of the
imposed by agreement of parties, as a practice observed in the usual pursuit conditions usually embodied in a trust receipt to best their legal sufficiency to
of a business or a commercial transaction. The offense may arise, if at all, constitute the basis for holding the violation of said conditions as estafa
from the peculiar terms and condition agreed upon by the parties to the under Article 315 of the Revised Penal Code which P.D. 115 now seeks to
transaction, not by direct provision of the law. The intention of the parties, punish expressly.
therefore, is a factor determinant of whether a crime was committed or
whether a civil obligation alone intended by the parties. With this explanation, As executed, the trust receipt in question reads:
the distinction adverted to between the Tan Boon Kong case and the case at
bar should come out clear and meaningful. In the absence of an express I/WE HEREBY AGREE TO HOLD SAID GOODS IN TRUST
provision of law making the petitioner liable for the criminal offense FOR THE SAID BANK as its property with liberty to sell the
committed by the corporation of which he is a president as in fact there is no same for its account but without authority to make any other
such provisions in the Revised Penal Code under which petitioner is being disposition whatsoever of the said goods or any part thereof
prosecuted, the existence of a criminal liability on his part may not be said to (or the proceeds thereof) either way of conditional sale,
be beyond any doubt. In all criminal prosecutions, the existence of criminal pledge or otherwise;
liability for which the accused is made answerable must be clear and certain.
The maxim that all doubts must be resolved in favor of the accused is always In case of sale I/we further agree to hand the proceeds as
of compelling force in the prosecution of offenses. This Court has thus far not soon as received to the BANK to apply against the relative
ruled on the criminal liability of an officer of a corporation signing in behalf of acceptance (as described above) and for the payment of any
said corporation a trust receipt of the same nature as that involved herein. In other indebtedness of mine/ours to CONTINENTAL BANK.
the case of Samo vs. People, L-17603-04, May 31, 1962, the accused was (Original Records, p. 108)
not clearly shown to be acting other than in his own behalf, not in behalf of a
corporation.
One view is to consider the transaction as merely that of a security of a loan,
and that the trust element is but and inherent feature of the security aspect of

611
the arrangement where the goods are placed in the possession of the the transaction would be to disregard the loan feature thereof, a feature
"entrustee," to use the term used in P.D. 115, violation of the element of trust totally absent in the case of the transaction between the jewel-owner and his
not being intended to be in the same concept as how it is understood in the agent.
criminal sense. The other view is that the bank as the owner and "entrustor"
delivers the goods to the "entrustee, " with the authority to sell the goods, but Consequently, if only from the fact that the trust receipt transaction is
with the obligation to give the proceeds to the "entrustor" or return the goods susceptible to two reasonable interpretation, one as giving rise only to civil
themselves if not sold, a trust being thus created in the full sense as liability for the violation of the condition thereof, and the other, as generating
contemplated by Art. 315, par. 1 (b). also criminal liability, the former should be adopted as more favorable to the
supposed offender. (Duran vs. CA, L-39758, May 7, 1976, 71 SCRA 68;
We consider the view that the trust receipt arrangement gives rise only to People vs. Parayno, L-24804, July 5, 1968, 24 SCRA 3; People vs. Abendan,
civil liability as the more feasible, before the promulgation of P.D. 115. The L-1481, January 28,1949,82 Phil. 711; People vs. Bautista, L-1502, May 24,
transaction being contractual, the intent of the parties should govern. Since 1948, 81 Phil. 78; People vs. Abana, L-39, February 1, 1946, 76 Phil. 1.)
the trust receipt has, by its nature, to be executed upon the arrival of the
goods imported, and acquires legal standing as such receipt only upon There is, moreover, one circumstance appearing on record, the significance
acceptance by the "entrustee," the trust receipt transaction itself, the of which should be properly evaluated. As stated in petitioner's brief (page 2),
antecedent acts consisting of the application of the L/C, the approval of the not denied by the People, "before the Continental Bank approved the
L/C and the making of the marginal deposit and the effective importation of application for a letter of credit (Exhibit 'D'), subsequently covered by the
the goods, all through the efforts of the importer who has to find his supplier, trust receipt, the Continental Bank examined the financial capabilities of the
arrange for the payment and shipment of the imported goods-all these applicant, Metal Manufacturing Company of the Philippines because that was
circumstances would negate any intent of subjecting the importer to criminal the bank's standard procedure (Testimony of Mr. Ernesto Garlit, Asst.
prosecution, which could possibly give rise to a case of imprisonment for Manager of the Foreign Department, Continental Bank, t.s.n., August 30,
non-payment of a debt. The parties, therefore, are deemed to have 1965). The Continental Bank did not examine the financial capabilities of
consciously entered into a purely commercial transaction that could give rise herein petitioner, Jose O. Sia, in connection with the same letter of credit.
only to civil liability, never to subject the "entrustee" to criminal prosecution. (Ibid). " From this fact, it would appear as positively established that the
Unlike, for instance, when several pieces of jewelry are received by a person intention of the parties in entering into the "trust receipt" agreement is merely
from the owner for sale on commission, and the former misappropriates for to afford a stronger security for the loan evidenced by the letter of credit, may
his personal use and benefit, either the jewelries or the proceeds of the sale, be not as an ordinary pledge as observed in P.N.B. vs. Viuda e Hijos de
instead of returning them to the owner as is his obligation, the bank is not in Angel Jose, et al., 63 Phil. 814, citing In re Dunlap C (206 Fed. 726) but
the same concept as the jewelry owner with full power of disposition of the neither as a transaction falling under Article 315-1 (b) of the Revised Penal
goods, which the bank does not have, for the bank has previously extended Code giving rise to criminal liability, as previously explained and
a loan which the L/C represents to the importer, and by that loan, the demonstrated.
importer should be the real owner of the goods. If under the trust receipt the
bank is made to appear as the owner, it was but an artificial expedient, more It is worthy of note that the civil liability imposed by the trust receipt is
of a legal fiction than fact, for if it were really so, it could dispose of the goods exclusively on the Metal Company. Speaking of such liability alone, as one
in any manner it wants, which it cannot do, just to give consistency with the arising from the contract, as distinguished from the civil liability arising out of
purpose of the trust receipt of giving a stronger security for the loan obtained a crime, the petitioner was never intended to be equally liable as the
by the importer. To consider the bank as the true owner from the inception of corporation. Without being made so liable personally as the corporation is,

612
there would then be no basis for holding him criminally liable, for any
violation of the trust receipt. This is made clearly so upon consideration of the
fact that in the violation of the trust agreement and in the absence of positive
evidence to the contrary, only the corporation benefited, not the petitioner
personally, yet, the allegation of the information is to effect that the
misappropriation or conversion was for the personal use and benefit of the
petitioner, with respect to which there is variance between the allegation and
the evidence.

It is also worthy of note that while the trust receipt speaks of authority to sell,
the fact is undisputed that the imported goods were to be manufactured into
finished products first before they could be sold, as the Bank had full
knowledge of. This fact is, however, not embodied in the trust agreement,
thus impressing on the trust receipt vagueness and ambiguity which should
not be the basis for criminal prosecution, in the event of a violation of the
terms of the trust receipt. Again, P.D. 115 has express provision relative to [G.R. No. 128690. January 21, 1999]
the "manufacture or process of the good with the purpose of ultimate sale,"
as a distinct condition from that of "to sell the goods or procure their sale" ABS-CBN BROADCASTING CORPORATION, petitioners,
(Section 4, (1). Note that what is embodied in the receipt in question is vs. HONORABLE COURT OF APPEALS, REPUBLIC
the sale of imported goods, the manufacture thereof not having been BROADCASTING CORP., VIVA PRODUCTIONS, INC., and
mentioned. The requirement in criminal prosecution, that there must be strict VICENTE DEL ROSARIO, respondents.
harmony, not variance, between the allegation and the evidence, may
therefore, not be said to have been satisfied in the instance case. DECISION

FOR ALL THE FOREGOING, We reverse the decision of the Court of DAVIDE, JR., C.J.:
Appeals and hereby acquit the petitioner, with costs de oficio.
In this petition for review on certiorari, petitioners ABS-CBN
SO ORDERED. Broadcasting Corp. (hereinafter ABS-CBN) seeks to reverse and set aside
the decision[1] of 31 October 1996 and the resolution [2] of 10 March 1997 of
Concepcion, Jr., Guerrero, Vasquez, Relova and Gutierrez, JJ., concur. the Court of Appeals in CA-G.R. CV No. 44125. The former affirmed with
modification the decision[3] of 28 April 1993 of the Regional Trial Court (RTC)
Fernando, CJ., Escolin, Plana, Abad Santos, JJ., concur in the result. of Quezon City, Branch 80, in Civil Case No. Q-12309. The latter denied the
motion to reconsider the decision of 31 October 1996.

The antecedents, as found by the RTC and adopted by the Court of


Appeals, are as follows:

613
In 1990, ABS-CBN and VIVA executed a Film Exhibition Agreement (Exh. A) big action stars. As a film producer, I am sure you understand what I am
whereby Viva gave ABS-CBN an exclusive right to exhibit some Viva trying to say as Viva produces only big action pictures.
films. Sometime in December 1991, in accordance with paragraph 2.4 [sic] of
said agreement stating that- In fact, I would like to request two (2) additional runs for these movies as I
can only schedule them in out non-primetime slots. We have to cover the
1.4 ABS-CBN shall have the right of first refusal to the next twenty-four (24) amount that was paid for these movies because as you very well know that
Viva films for TV telecast under such terms as may be agreed upon by the non-primetime advertising rates are very low. These are the unaired titles in
parties hereto, provided, however, that such right shall be exercised by ABS- the first contract.
CBN from the actual offer in writing.
1. Kontra Persa [sic]
Viva, through defendant Del Rosario, offered ABS-CBN, through its vice-
president Charo Santos-Concio, a list of three (3) film packages (36 title) 2. Raider Platoon
from which ABS-CBN may exercise its right of first refusal under the afore-
said agreement (Exhs. 1 par. 2, 2, 2-A and 2-B Viva). ABS-CBN, however 3. Underground guerillas
through Mrs. Concio, can tick off only ten (10) titles (from the list) we can
purchase (Exh. 3 Viva) and therefore did not accept said list (TSN, June 8,
4. Tiger Command
1992, pp. 9-10). The titles ticked off by Mrs. Concio are not the subject of the
case at bar except the film Maging Sino Ka Man.
5. Boy de Sabog
For further enlightenment, this rejection letter dated January 06, 1992 (Exh 3
Viva) is hereby quoted: 6. lady Commando

6 January 1992 7. Batang Matadero

Dear Vic, 8. Rebelyon

This is not a very formal business letter I am writing to you as I would like to I hope you will consider this request of mine.
express my difficulty in recommending the purchase of the three film
packages you are offering ABS-CBN. The other dramatic films have been offered to us before and have been
rejected because of the ruling of MTRCB to have them aired at 9:00 p.m. due
From among the three packages I can only tick off 10 titles we can to their very adult themes.
purchase. Please see attached. I hope you will understand my position. Most
of the action pictures in the list do not have big action stars in the cast. They As for the 10 titles I have choosen [sic] from the 3 packages please consider
are not for primetime. In line with this I wish to mention that I have not including all the other Viva movies produced last year, I have quite an
scheduled for telecast several action pictures in our very first contract attractive offer to make.
because of the cheap production value of these movies as well as the lack of

614
Thanking you and with my warmest regards. On April 07, 1992, defendant Del Rosario received through his secretary , a
handwritten note from Ms. Concio, (Exh. 5 Viva), which reads: Heres the
(Signed) draft of the contract. I hope you find everything in order, to which was
attached a draft exhibition agreement (Exh. C ABS-CBN; Exh. 9 Viva p. 3) a
Charo Santos-Concio counter-proposal covering 53 films, 52 of which came from the list sent by
defendant Del Rosario and one film was added by Ms. Concio, for a
consideration of P35 million. Exhibit C provides that ABS-CBN is granted film
On February 27, 1992, defendant Del Rosario approached ABS-CBNs Ms.
rights to 53 films and contains a right of first refusal to 1992 Viva Films. The
Concio, with a list consisting of 52 original movie titles (i.e., not yet aired on
said counter proposal was however rejected by Vivas Board of Directors [in
television) including the 14 titles subject of the present case, as well as 104
the] evening of the same day, April 7, 1992, as Viva would not sell anything
re-runs (previously aired on television) from which ABS-CBN may choose
less than the package of 104 films for P60 million pesos (Exh. 9 Viva), and
another 52 titles, as a total of 156 titles, proposing to sell to ABS-CBN airing
such rejection was relayed to Ms. Concio.
rights over this package of 52 originals and 52 re-runs for P60,000,000.00 of
which P30,000,000.00 will be in cash and P30,000,000.00 worth of television
spots (Exh. 4 to 4-C Viva; 9 Viva). On April 29, 1992, after the rejection of ABS-CBN and following several
negotiations and meetings defendant Del Rosario and Vivas President
Teresita Cruz, in consideration of P60 million, signed a letter of agreement
On April 2, 1992, defendant Del Rosario and ABS-CBNs general manager,
dated April 24, 1992, granting RBS the exclusive right to air 104 Viva-
Eugenio Lopez III, met at the Tamarind Grill Restaurant in Quezon City to
produced and/or acquired films (Exh. 7-A - RBS; Exh. 4 RBS) including the
discuss the package proposal of VIVA. What transpired in that lunch meeting
fourteen (14) films subject of the present case.[4]
is the subject of conflicting versions. Mr. Lopez testified that he and Mr. Del
Rosario allegedly agreed that ABS-CBN was granted exclusive film rights to
fourteen (14) films for a total consideration of P36 million; that he allegedly On 27 May 1992, ABS-CBN filed before the RTC a complaint for specific
put this agreement as to the price and number of films in a napkin and signed performance with a prayer for a writ of preliminary injunction and/or
it and gave it to Mr. Del Rosario (Exh. D; TSN, pp. 24-26, 77-78, June 8, temporary restraining order against private respondents Republic
1992).On the other hand. Del Rosario denied having made any agreement Broadcasting Corporation[5] (hereafter RBS), Viva Production (hereafter
with Lopez regarding the 14 Viva films; denied the existence of a napkin in VIVA), and Vicente del Rosario. The complaint was docketed as Civil Case
which Lopez wrote something; and insisted that what he and Lopez No. Q-92-12309.
discussed at the lunch meeting was Vivas film package offer of 104 films (52
originals and 52 re-runs) for a total price of P60 million. Mr. Lopez promising On 28 May 1992, the RTC issued a temporary restraining
[sic]to make a counter proposal which came in the form of a proposal order[6] enjoining private respondents from proceeding with the airing,
contract Annex C of the complaint (Exh. 1 Viva; Exh C ABS-CBN). broadcasting, and televising of the fourteen VIVA films subject of the
controversy, starting with the film Maging Sino Ka Man, which was scheduled
On April 06, 1992, Del Rosario and Mr. Graciano Gozon of RBS Senior vice- to be shown on private respondent RBS channel 7 at seven oclock in the
president for Finance discussed the terms and conditions of Vivas offer to evening of said date.
sell the 104 films, after the rejection of the same package by ABS-CBN.
On 17 June 1992, after appropriate proceedings, the RTC issued an
order[7] directing the issuance of a writ of preliminary injunction upon ABS-

615
CBNs posting of a P35 million bond. ABS-CBN moved for the reduction of On 3 November 1992, the Court of Appeals issued a temporary
the bond,[8] while private respondents moved for reconsideration of the order restraining order[18] to enjoin the airing, broadcasting, and televising of any or
and offered to put up a counterbond.[9] all of the films involved in the controversy.

In the meantime, private respondents filed separate answer with On 18 December 1992, the Court of Appeals promulgated a
counterclaim.[10] RBS also set up a cross-claim against VIVA. decision[19] dismissing the petition in CA-G.R. SP No. 29300 for being
premature. ABS-CBN challenged the dismissal in a petition for review filed
On 3 August 1992, the RTC issued an order [11] dissolving the writ of with this Court on 19 January 1993, which was docketed s G.R. No. 108363.
preliminary injunction upon the posting by RBS of a P30 million counterbond
to answer for whatever damages ABS-CBN might suffer by virtue of such In the meantime the RTC received the evidence for the parties in Civil
dissolution. However, it reduced petitioners injunction bond to P15 million as Case No. Q-92-12309. Thereafter, on 28 April 1993, it rendered a
a condition precedent for the reinstatement of the writ of preliminary decision[20] in favor of RBS and VIVA and against ABS-CBN disposing as
injunction should private respondents be unable to post a counterbond. follows:

At the pre-trial[12] on 6 August 1992, the parties upon suggestion of the WHEREFORE, under cool reflection and prescinding from the foregoing,
court, agreed to explore the possibility of an amicable settlement. In the judgment is rendered in favor of defendants and against the plaintiff.
meantime, RBS prayed for and was granted reasonable time within which to
put up a P30 million counterbond in the event that no settlement would be (1) The complaint is hereby dismissed;
reached.
(2) Plaintiff ABS-CBN is ordered to pay defendant RBS the
As the parties failed to enter into an amicable settlement, RBS posted following:
on 1 October 1992 a counterbond, which the RTC approved in its Order of 15
October 1992.[13] a) P107,727.00 the amount of premium paid by RBS to the
surety which issued defendants RBSs bond to lift the
On 19 October 1992, ABS-CBN filed a motion for reconsideration [14] of injunction;
the 3 August and 15 October 1992 Orders, which RBS opposed. [15]
b) P191,843.00 for the amount of print advertisement for
On 29 October, the RTC conducted a pre-trial.[16] Maging Sino Ka Man in various newspapers;

Pending resolution of its motion for reconsideration, ABS-CBN filed with c) Attorneys fees in the amount of P1 million;
the Court of Appeals a petition [17] challenging the RTCs Order of 3 August
and 15 October 1992 and praying for the issuance of a writ of preliminary d) P5 million as and by way of moral damages;
injunction to enjoin the RTC from enforcing said orders. The case was
docketed as CA-G.R. SP No. 29300.
e) P5 million as and by way of exemplary damages;

616
(3) For the defendant VIVA, plaintiff ABS-CBN is ordered to rejected ABS-CBNs insistence on its right of first refusal and ratiocinated as
pay P212,000.00 by way of reasonable attorneys fees. follows:

(4) The cross-claim of defendant RBS against defendant VIVA is As regards the matter of right of first refusal, it may be true that a Film
dismissed. Exhibition Agreement was entered into between Appellant ABS-CBN and
appellant VIVA under Exhibit A in 1990 and that parag. 1.4 thereof provides:
(5) Plaintiff to pay the costs.
1.4 ABS-CBN shall have the right of first refusal to the next twenty-four (24)
According to the RTC, there was no meeting of minds on the price and VIVA films for TV telecast under such terms as may be agreed upon by the
terms of the offer. The alleged agreement between Lopez III and Del Rosario parties hereto, provided, however, that such right shall be exercised by ABS-
was subject to the approval of the VIVA Board of Directors, and said CBN within a period of fifteen (15) days from the actual offer in writing
agreement was disapproved during the meeting of the Board on 7 April (Records, p. 14).
1992. Hence, there was no basis for ABS-CBNs demand that VIVA signed
the 1992 Film Exhibition Agreement.Furthermore, the right of first refusal [H]owever, it is very clear that said right of first refusal in favor of ABS-CBN
under the 1990 Film Exhibition Agreement had previously been exercised per shall still be subjected to such terms as may be agreed upon by the parties
Ms. Concios letter to Del Rosario ticking off ten titles acceptable to them, thereto, and that the said right shall be exercised by ABS-CBN within fifteen
which would have made the 1992 agreement an entirely new contract. (15) days from the actual offer in writing.

On 21 June 1993, this Court denied[21] ABS-CBNs petition for review in Said parag. 1.4 of the agreement Exhibit A on the right of first refusal did not
G.R. No. 108363, as no reversible error was committed by the Court of fix the price of the film right to the twenty-four (24) films, nor did it specify the
Appeals in its challenged decision and the case had become moot and terms thereof. The same are still left to be agreed upon by the parties.
academic in view of the dismissal of the main action by the court a quo in its
decision of 28 April 1993. In the instant case, ABS-CBNs letter of rejection Exhibit 3 (Records, p. 89)
stated that it can only tick off ten (10) films, and the draft contract Exhibit C
Aggrieved by the RTCs decision, ABS-CBN appealed to the Court of accepted only fourteen (14) films, while parag. 1.4 of Exhibit A speaks of the
Appeals claiming that there was a perfected contract between ABS-CBN and next twenty-four (24) films.
VIVA granting ABS-CBN the exclusive right to exhibit the subject
films. Private respondents VIVA and Del Rosario also appealed seeking The offer of VIVA was sometime in December 1991, (Exhibits 2, 2-A, 2-B;
moral and exemplary damages and additional attorneys fees. Records, pp. 86-88; Decision, p. 11, Records, p. 1150), when the first list of
VIVA films was sent by Mr. Del Rosario to ABS-CBN. The Vice President of
In its decision of 31 October 1996, the Court of Appeals agreed with the ABS-CBN, Mrs. Charo Santos-Concio, sent a letter dated January 6, 1992
RTC that the contract between ABS-CBN and VIVA had not been perfected, (Exhibit 3, Records, p. 89) where ABS-CBN exercised its right of refusal by
absent the approval by the VIVA Board of Directors of whatever Del Rosario, rejecting the offer of VIVA. As aptly observed by the trial court, with the said
its agent, might have agreed with Lopez III. The appellate court did not even letter of Mrs. Concio of January 6, 1992, ABS-CBN had lost its right of first
believe ABS-CBNs evidence that Lopez III actually wrote down such an refusal. And even if We reckon the fifteen (15) day period from February 27,
agreement on a napkin, as the same was never produced in court. It likewise 1992 (Exhibit 4 to 4-C) when another list was sent to ABS-CBN after the

617
letter of Mrs. Concio, still the fifteen (15) day period within which ABS-CBN IN AWARDING ACTUAL AND COMPENSATORY DAMAGES IN
shall exercise its right of first refusal has already expired. [22] FAVOR OF PRIVATE RESPONDENT RBS.

Accordingly, respondent court sustained the award factual damages III


consisting in the cost of print advertisements and the premium payments for
the counterbond, there being adequate proof of the pecuniary loss which IN AWARDING MORAL AND EXEMPLARY DAMAGES IN
RBS has suffered as a result of the filing of the complaint by ABS-CBN. As to FAVOR OF PRIVATE RESPONDENT RBS.
the award of moral damages, the Court of Appeals found reasonable basis
therefor, holding that RBSs reputation was debased by the filing of the IV
complaint in Civil Case No. Q-92-12309 and by the non-showing of the
film Maging Sino Ka Man. Respondent court also held that exemplary
IN AWARDING ATORNEYS FEES OF RBS.
damages were correctly imposed by way of example or correction for the
public good in view of the filing of the complaint despite petitioners
knowledge that the contract with VIVA had not been perfected. It also upheld ABS-CBN claims that it had yet to fully exercise its right of first refusal
the award of attorneys fees, reasoning that with ABS-CBNs act of instituting over twenty-four titles under the 1990 Film Exhibition Agreement, as it had
Civil Case No. Q-92-12309, RBS was unnecessarily forced to litigate. The chosen only ten titles from the first list. It insists that we give credence to
appellate court, however, reduced the awards of moral damages to P 2 Lopezs testimony that he and Del Rosario met at the Tamarind Grill
million, exemplary damages to P2 million, and attorneys fees Restaurant, discussed the terms and conditions of the second list (the 1992
to P500,000.00. Film Exhibition Agreement) and upon agreement thereon, wrote the same on
a paper napkin. It also asserts that the contract has already been effective,
as the elements thereof, namely, consent, object, and consideration were
On the other hand, respondent Court of Appeals denied VIVA and Del
established. It then concludes that the Court of Appeals pronouncements
Rosarios appeal because it was RBS and not VIVA which was actually
were not supported by law and jurisprudence, as per our decision of 1
prejudiced when the complaint was filed by ABS-CBN.
December 1995 in Limketkai Sons Milling, Inc. v. Court of Appeals, [23] which
cited Toyota Shaw, Inc. v. Court of Appeals;[24] Ang Yu Asuncion v. Court of
Its motion for reconsideration having been denied, ABS-CBN filed the Appeals,[25] and Villonco Realty Company v. Bormaheco, Inc.[26]
petition in this case, contending that the Court of Appeals gravely erred in
Anent the actual damages awarded to RBS, ABS-CBN disavows liability
I therefor. RBS spent for the premium on the counterbond of its own volition in
order to negate the injunction issued by the trial court after the parties had
RULING THAT THERE WAS NO PERFECTED CONTRACT ventilated their respective positions during the hearings for the purpose. The
BETWEEN PETITIONER AND PRIVATE RESPONDENT VIVA filing of the counterbond was an option available to RBS, but it can hardly be
NOTWITHSTANDING PREPONFERANCE OF EVIDENCE argued that ABS-CBN compelled RBS to incur such expense. Besides, RBS
ADDUCED BY PETITIONER TO THE CONTRARY. had another available option, i.e., move for the dissolution of the injunction;
or if it was determined to put up a counterbond, it could have presented a
II cash bond. Furthermore under Article 2203 of the Civil Code, the party
suffering loss injury is also required to exercise the diligence of a good father

618
of a family to minimize the damages resulting from the act or omission. As On the other hand, RBS asserts that there was no perfected contract
regards the cost of print advertisements, RBS had not convincingly between ABS-CBN and VIVA absent meeting of minds between them
established that this was a loss attributable to the non-showing of Maging regarding the object and consideration of the alleged contract.It affirms that
Sino Ka Man; on the contrary, it was brought out during trial that with or ABS-CBNs claim of a right of first refusal was correctly rejected by the trial
without the case or injunction, RBS would have spent such an amount to court. RBS insists the premium it had paid for the counterbond constituted a
generate interest in the film. pecuniary loss upon which it may recover. It was obliged to put up the
counterbond due to the injunction procured by ABS-CBN. Since the trial court
ABS-CBN further contends that there was no other clear basis for the found that ABS-CBN had no cause of action or valid claim against RBS and,
awards of moral and exemplary damages. The controversy involving ABS- therefore not entitled to the writ of injunction, RBS could recover from ABS-
CBN and RBS did not in any way originate from business transaction CBN the premium paid on the counterbond. Contrary to the claim of ABS-
between them. The claims for such damages did not arise from any CBN, the cash bond would prove to be more expensive, as the loss would be
contractual dealings or from specific acts committed by ABS-CBN against equivalent to the cost of money RBS would forego in case the P30 million
RBS that may be characterized as wanton, fraudulent, or reckless; they came from its funds or was borrowed from banks.
arose by virtue only of the filing of the complaint. An award of moral and
exemplary damages is not warranted where the record is bereft of any proof RBS likewise asserts that it was entitled to the cost of advertisements
that a party acted maliciously or in bad faith in filing an action. [27] In any case, for the cancelled showing of the film Maging Sino Ka Man because the print
free resort to courts for redress of wrongs is a matter of public policy. The law advertisements were out to announce the showing on a particular day and
recognizes the right of every one to sue for that which he honestly believes to hour on Channel 7, i.e., in its entirety at one time, not as series to be shown
be his right without fear of standing trial for damages where by lack of on a periodic basis. Hence, the print advertisements were good and relevant
sufficient evidence, legal technicalities, or a different interpretation of the laws for the particular date of showing, and since the film could not be shown on
on the matter, the case would lose ground. [28] One who, makes use of his that particular date and hour because of the injunction, the expenses for the
own legal right does no injury.[29] If damage results from filing of the advertisements had gone to waste.
complaint, it is damnum absque injuria.[30] Besides, moral damages are
generally not awarded in favor of a juridical person, unless it enjoys a good As regards moral and exemplary damages, RBS asserts that ABS-CBN
reputation that was debased by the offending party resulting in social filed the case and secured injunctions purely for the purpose of harassing
humiliation.[31] and prejudicing RBS. Pursuant then to Articles 19 and 21 of the Civil Code,
ABS-CBN must be held liable for such damages. Citing Tolentino,
[34]
As regards the award of attorneys fees, ABS-CBN maintains that the damages may be awarded in cases of abuse of rights even if the done is
same had no factual, legal, or equitable justification. In sustaining the trial not illicit, and there is abuse of rights where a plaintiff institutes an action
courts award, the Court of Appeals acted in clear disregard of the doctrine purely for the purpose of harassing or prejudicing the defendant.
laid down in Buan v. Camaganacan[32] that the text of the decision should
state the reason why attorneys fees are being awarded; otherwise, the award In support of its stand that a juridical entity can recover moral and
should be disallowed. Besides, no bad faith has been imputed on, much less exemplary damages, private respondent RBS cited People v. Manero,
proved as having been committed by, ABS-CBN. It has been held that where [35]
where it was stated that such entity may recover moral and exemplary
no sufficient showing of bad faith would be reflected in a partys persistence damages if it has a good reputation that is debased resulting in social
in a case other than an erroneous conviction of the righteousness of his humiliation. It then ratiocinates; thus:
cause, attorneys fees shall not be recovered as cost. [33]

619
There can be no doubt that RBS reputation has been debased by ABS-CBNs The key issues for our consideration are (1) whether there was a
acts in this case. When RBS was not able to fulfill its commitment to the perfected contract between VIVA and ABS-CBN, and (2) whether RBS is
viewing public to show the film Maging Sino Ka Man on the scheduled dates entitled to damages and attorneys fees. It may be noted that that award of
and times (and on two occasions that RBS advertised), it suffered serious attorneys fees of P212,000 in favor of VIVA is not assigned as another error.
embarrassment and social humiliation. When the showing was cancelled,
irate viewers called up RBS offices and subjected RBS to verbal abuse I
(Announce kayo ng announce, hindi ninyo naman ilalabas, nanloloko yata
kayo) (Exh. 3-RBS, par.3). This alone was not something RBS brought upon The first issue should be resolved against ABS-CBN. A contract is a
itself. It was exactly what ABS-CBN had planted to happen. meeting of minds between two persons whereby one binds himself to give
something or render some service to another[37] for a consideration. There is
The amount of moral and exemplary damages cannot be said to be no contract unless the following requisites concur: (1) consent of the
excessive. Two reasons justify the amount of the award. contracting parties; (2) object certain which is the subject of the contract; and
(3) cause of the obligation, which is established. [38] A contract undergoes
The first is that the humiliation suffered by RBS, is national in extent. RBS three stages:
operations as a broadcasting company is [sic] nationwide. Its clientele, like
that of ABS-CBN, consists of those who own and watch television. It is not an (a) preparation, conception, or generation, which is the period of
exaggeration to state, and it is a matter of judicial notice that almost every negotiation and bargaining, ending at the moment of
other person in the country watches television. The humiliation suffered by agreement of the parties;
RBS is multiplied by the number of televiewers who had anticipated the
showing of the film, Maging Sino Ka Man on May 28 and November 3, 1992 (b) perfection or birth of the contract, which is the moment when
but did not see it owing to the cancellation. Added to this are the advertisers the parties come to agree on the terms of the contract; and
who had placed commercial spots for the telecast and to whom RBS had a
commitment in consideration of the placement to show the film in the dates
(c) consummation or death, which is the fulfillment or performance
and times specified.
of the terms agreed upon in the contract.[39]

The second is that it is a competitor that caused RBS suffer the


Contracts that are consensual in nature are perfected upon mere
humiliation. The humiliation and injury are far greater in degree when caused
meeting of the minds. Once there is concurrence between the offer and the
by an entity whose ultimate business objective is to lure customers (viewers
acceptance upon the subject matter, consideration, and terms of payment a
in this case) away from the competition.[36]
contract is produced. The offer must be certain. To convert the offer into a
contract, the acceptance must be absolute and must not qualify the terms of
For their part, VIVA and Vicente del Rosario contend that the findings of the offer; it must be plain, unequivocal, unconditional, and without variance of
fact of the trial court and the Court of Appeals do not support ABS-CBNs any sort from the proposal. A qualified acceptance, or one that involves a
claim that there was a perfected contract. Such factual findings can no longer new proposal, constitutes a counter-offer and is a rejection of the original
be disturbed in this petition for review under Rule 45, as only questions of offer. Consequently, when something is desired which is not exactly what is
law can be raised, not questions of fact. On the issue of damages and proposed in the offer, such acceptance is not sufficient to generate consent
attorneys fees, they adopted the arguments of RBS.

620
because any modification or variation from the terms of the offer annuls the its counter-proposals or counter-offer in a draft contract. VIVA through its
offer.[40] Board of Directors, rejected such counter-offer. Even if it be
conceded arguendo that Del Rosario had accepted the counter-offer, the
When Mr. Del Rosario of Viva met Mr. Lopez of ABS-CBN at the acceptance did not bind VIVA, as there was no proof whatsoever that Del
Tamarind Grill on 2 April 1992 to discuss the package of films, said package Rosario had the specific authority to do so.
of 104 VIVA films was VIVAs offer to ABS-CBN to enter into a new Film
Exhibition Agreement. But ABS-CBN, sent through Ms. Concio, counter- Under the Corporation Code,[46] unless otherwise provided by said Code,
proposal in the form a draft contract proposing exhibition of 53 films for a corporate powers, such as the power to enter into contracts, are exercised by
consideration of P35 million. This counter-proposal could be nothing less the Board of Directors. However, the Board may delegate such powers to
than the counter-offer of Mr. Lopez during his conference with Del Rosario at either an executive committee or officials or contracted managers. The
Tamarind Grill Restaurant. Clearly, there was no acceptance of VIVAs offer, delegation, except for the executive committee, must be for specific
for it was met by a counter-offer which substantially varied the terms of the purposes.[47] Delegation to officers makes the latter agents of the corporation;
offer. accordingly, the general rules of agency as to the binding effects of their acts
would apply.[48] For such officers to be deemed fully clothed by the
ABS-CBNs reliance in Limketkai Sons Milling, Inc. v. Court of corporation to exercise a power of the Board, the latter must specially
Appeals[41] and Villonco Realty Company v. Bormaheco, Inc.,[42] is authorize them to do so. that Del Rosario did not have the authority to accept
misplaced. In these cases, it was held that an acceptance may contain a ABS-CBNs counter-offer was best evidenced by his submission of the draft
request for certain changes in the terms of the offer and yet be a binding contract to VIVAs Board of Directors for the latters approval. In any event,
acceptance as long as it is clear that the meaning of the acceptance is there was between Del Rosario and Lopez III no meeting of minds. The
positively and unequivocally to accept the offer, whether such request is following findings of the trial court are instructive:
granted or not. This ruling was, however, reversed in the resolution of 29
March 1996,[43] which ruled that the acceptance of an offer must be A number of considerations militate against ABS-CBNs claim that a contract
unqualified and absolute, i.e., it must be identical in all respects with that of was perfected at that lunch meeting on April 02, 1992 at the Tamarind Grill.
the offer so as to produce consent or meetings of the minds.
FIRST, Mr. Lopez claimed that what was agreed upon at the Tamarind Grill
On the other hand, in Villonco, cited in Limketkai, the alleged changes in referred to the price and the number of films, which he wrote on a
the revised counter-offer were not material but merely clarificatory of what napkin. However, Exhibit C contains numerous provisions which were not
had previously been agreed upon. It cited the statement in Stuart v. Franklin discussed at the Tamarind Grill, if Lopez testimony was to be believed nor
Life Insurance Co.[44] that a vendors change in a phrase of the offer to could they have been physically written on a napkin. There was even doubt
purchase, which change does not essentially change the terms of the offer, as to whether it was a paper napkin or cloth napkin. In short what were
does not amount to a rejection of the offer and the tender of a counter-offer. written in Exhibit C were not discussed, and therefore could not have been
[45]
However, when any of the elements of the contract is modified upon agreed upon, by the parties. How then could this court compel the parties to
acceptance, such alteration amounts to a counter-offer. sign Exhibit C when the provisions thereof were not previously agreed upon?

In the case at bar, ABS-CBN made no unqualified acceptance of VIVAs SECOND, Mr. Lopez claimed that what was agreed upon as the subject
offer hence, they underwent period of bargaining. ABS-CBN then formalized matter of the contract was 14 films. The complaint in fact prays for delivery of

621
14 films. But Exhibit C mentions 53 films as its subject matter. Which is FOURTH. Mrs. Concio, testifying for ABS-CBN stated that she transmitted
which? If Exhibit C reflected the true intent of the parties, then ABS-CBNs Exhibit C to Mr. Del Rosario with a handwritten note, describing said Exhibit
claim for 14 films in its complaint is false or if what it alleged in the complaint C as a draft. (Exh. 5 Viva; tsn pp. 23-24, June 08, 1992). The said draft has a
is true, then Exhibit C did not reflect what was agreed upon by the well defined meaning.
parties. This underscores the fact that there was no meeting of the minds as
to the subject matter of the contract, so as to preclude perfection thereof. For Since Exhibit C is only a draft, or a tentative, provisional or preparatory
settled is the rule that there can be no contract where there is no object writing prepared for discussion, the terms and conditions thereof could not
certain which is its subject matter (Art. 1318, NCC). have been previously agreed upon by ABS-CBN and Viva.Exhibit C could not
therefore legally bind Viva, not having agreed thereto. In fact, Ms. Concio
THIRD, Mr. Lopez [sic] answer to question 29 of his affidavit testimony (Exh. admitted that the terms and conditions embodied in Exhibit C were prepared
D) States: by ABS-CBNs lawyers and there was no discussion on said terms and
conditions.
We were able to reach an agreement. VIVA gave us the exclusive license to
show these fourteen (14) films, and we agreed to pay Viva the amount As the parties had not yet discussed the proposed terms and conditions in
of P16,050,000.00 as well as grant Viva commercial slots Exhibit C, and there was no evidence whatsoever that Viva agreed to the
worth P19,950,000.00. We had already earmarked this P16,050,000.00. terms and conditions thereof, said document cannot be a binding
contract. The fact that Viva refused to sign Exhibit C reveals only two [sic]
which gives a total consideration of P36 million (P19,951,000.00 well that it did not agree on its terms and conditions, and this court has no
plus P16,050,000.00 equals P36,000,000.00). authority to compel Viva to agree thereto.

On cross-examination Mr. Lopez testified: FIFTH. Mr. Lopez understand [sic] that what he and Mr. Del Rosario agreed
upon at the Tamarind Grill was only provisional, in the sense that it was
Q What was written in this napkin? subject to approval by the Board of Directors of Viva. He testified:

A The total price, the breakdown the known Viva movies, the 7 Q Now, Mr. Witness, and after that Tamarinf meeting the second meeting
blockbuster movies and the other 7 Viva movies because the price wherein you claimed that you have the meeting of the minds
was broken down accordingly. The none [sic] Viva and the seven between you and Mr. Vic del Rosario, what happened?
other Viva movies and the sharing between the cash portion and the
concerned spot portion in the total amount of P35 million pesos. A Vic Del Rosario was supposed to call us up and tell us specifically the
result of the discussion with the Board of Directors.
Now, which is which? P36 million or P35 million? This weakens ABS-CBNs
claim. Q And you are referring to the so-called agreement which you wrote in
[sic] a piece of paper?

A Yes, sir.

622
Q So, he was going to forward that to the board of Directors for approval? previous contract is untenable. As observed by the trial court, ABS-CBNs
right of first refusal had already been exercised when Ms. Concio wrote to
A Yes, sir (Tsn, pp. 42-43, June 8, 1992) Viva ticking off ten films.Thus:

Q Did Mr. Del Rosario tell you that he will submit it to his Board for [T]he subsequent negotiation with ABS-CBN two (2) months after this
approval? letter was sent, was for an entirely different package. Ms. Concio herself
admitted on cross-examination to having used or exercised the right of
A Yes, sir. (Tsn, p. 69, June 8, 1992). first refusal. She stated that the list was not acceptable and was indeed
not accepted by ABS-CBN, (Tsn, June 8, 1992, pp. 8-10). Even Mr.
Lopez himself admitted that the right of first refusal may have been
The above testimony of Mr. Lopez shows beyond doubt that he knew Mr. Del
already exercised by Ms. Concio (as she had). (TSN, June 8, 1992, pp.
Rosario had no authority to bind Viva to a contract with ABS-CBN until and
71-75). Del Rosario himself knew and understand [sic] that ABS-CBN
unless its Board of Directors approved it. The complaint, in fact, alleges that
has lost its right of first refusal when his list of 36 titles were rejected
Mr. Del Rosario is the Executive Producer of defendant Viva which is a
(Tsn, June 9, 1992, pp. 10-11).[50]
corporation. (par. 2, complaint). As a mere agent of Viva, Del Rosario could
not bind Viva unless what he did is ratified by its Directors. (Vicente
vs.Geraldez, 52 SCRA 210; Arnold vs. Willets and Paterson, 44 Phil. II
634). As a mere agent, recognized as such by plaintiff, Del Rosario could not
be held liable jointly and severally with Viva and his inclusion as party However, we find for ABS-CBN on the issue of damages. We shall first
defendant has no legal basis. (Salonga vs. Warner Barnes [sic],COLTA, 88 take up actual damages. Chapter 2, Title XVIII, Book IV of the Civil Code is
Phil. 125; Salmon vs. Tan, 36 Phil. 556). the specific law on actual or compensatory damages. Except as provided by
law or by stipulation, one is entitled to compensation for actual damages only
The testimony of Mr. Lopez and the allegations in the complaint are clear for such pecuniary loss suffered by him as he has duly proved. [51] The
admissions that what was supposed to have been agreed upon at the indemnification shall comprehend not only the value of the loss suffered, but
Tamarind Grill between Mr. Lopez and Del Rosario was not a binding also that of the profits that the obligee failed to obtain. [52] In contracts and
agreement. It is as it should be because corporate power to enter into a quasi-contracts the damages which may be awarded are dependent on
contract is lodged in the Board of Directors. (Sec. 23, Corporation whether the obligor acted with good faith or otherwise. In case of good faith,
Code). Without such board approval by the Viva board, whatever agreement the damages recoverable are those which are the natural and probable
Lopez and Del Rosario arrived at could not ripen into a valid binding upon consequences of the breach of the obligation and which the parties have
Viva (Yao Ka Sin Trading vs. Court of Appeals, 209 SCRA 763). The foreseen or could have reasonably foreseen at the time of the constitution of
evidence adduced shows that the Board of Directors of Viva rejected Exhibit the obligation. If the obligor acted with fraud, bad faith, malice, or wanton
C and insisted that the film package for 104 films be maintained (Exh. 7-1 attitude, he shall be responsible for all damages which may be reasonably
Cica).[49] attributed to the non-performance of the obligation. [53] In crimes and quasi-
delicts, the defendants shall be liable for all damages which are the natural
and probable consequences of the act or omission complained of, whether or
The contention that ABS-CBN had yet to fully exercise its right of first
not such damages have been foreseen or could have reasonably been
refusal over twenty-four films under the 1990 Film Exhibition Agreement and
foreseen by the defendant.[54]
that the meeting between Lopez and Del Rosario was a continuation of said

623
Actual damages may likewise be recovered for loss or impairment of It may further be observed that in cases where a writ of preliminary
earning capacity in cases of temporary or permanent personal injury, or for injunction is issued, the damages which the defendant may suffer by reason
injury to the plaintiffs business standing or commercial credit. [55] of the writ are recoverable from the injunctive bond. [57] In this case, ABS-CBN
had not yet filed the required bond; as a matter of fact, it asked for reduction
The claim of RBS for actual damages did not arise from contract, quasi- of the bond and even went to the Court of Appeals to challenge the order on
contract, delict, or quasi-delict. It arose from the fact of filing of the complaint the matter. Clearly then, it was not necessary for RBS to file a
despite ABS-CBNs alleged knowledge of lack of cause of action. Thus counterbond. Hence, ABS-CBN cannot be held responsible for the premium
paragraph 12 of RBSs Answer with Counterclaim and Cross-claim under the RBS paid for the counterbond.
heading COUNTERCLAIM specifically alleges:
Neither could ABS-CBN be liable for the print advertisements for Maging
12. ABS-CBN filed the complaint knowing fully well that it has no Sino Ka Man for lack of sufficient legal basis. The RTC issued a temporary
cause of action against RBS. As a result thereof, RBS suffered restraining order and later, a writ of preliminary injunction on the basis of its
actual damages in the amount of P6,621,195.32.[56] determination that there existed sufficient ground for the issuance
thereof. Notably, the RTC did not dissolve the injunction on the ground of lack
Needless to state the award of actual damages cannot be comprehended of legal and factual basis, but because of the plea of RBS that it be allowed
under the above law on actual damages. RBS could only probably take to put up a counterbond.
refuge under Articles 19, 20, and 21 of the Civil Code, which read as follows:
As regards attorneys fees, the law is clear that in the absence of
ART. 19. Every person must, in the exercise of hid rights and in the stipulation, attorneys fees may be recovered as actual or compensatory
performance of his duties, act with justice, give everyone his due, and damages under any of the circumstances provided for in Article 2208 of the
observe honesty and good faith. Civil Code.[58]

ART. 20. Every person who, contrary to law, wilfully or negligently causes The general rule is that attorneys fees cannot be recovered as part of
damage to another shall indemnify the latter for the same. damages because of the policy that no premium should be placed on the
right to litigate.[59] They are not to be awarded every time a party wins a
suit. The power of the court t award attorneys fees under Article 2208
ART. 21. Any person who wilfully causes loss or injury to another in a manner
demands factual, legal, and equitable justification. [60] Even when a claimant is
that is contrary to morals, good customs or public policy shall compensate
compelled to litigate with third persons or to incur expenses to protect his
the latter for the damage.
rights, still attorneys fees may not be awarded where no sufficient showing of
bad faith could be reflected in a partys persistence in a case other than an
erroneous conviction of the righteousness of his cause. [61]

As to moral damages the law is Section 1, Chapter 3, Title XVIII, Book


IV of the Civil Code. Article 2217 thereof defines what are included in moral
damages, while Article 2219 enumerates the cases where they may be
recovered. Article 2220 provides that moral damages may be recovered in

624
breaches of contract where the defendant acted fraudulently or in bad negligence;[70] and in contracts and quasi-contracts, if the defendant acted in
faith. RBSs claim for moral damages could possibly fall only under item (10) a wanton, fraudulent, reckless, oppressive, or malevolent manner.[71]
of Article 2219, thereof which reads:
It may be reiterated that the claim of RBS against ABS-CBN is not
(10) Acts and actions referred to in Articles 21, 26, 27, 28, 29, 30, 32, 34 and based on contract, quasi-contract, delict, or quasi-delict. Hence, the claims
35. for moral and exemplary damages can only be based on Articles 19, 20, and
21 of the Civil Code.
Moral damages are in the category of an award designed to
compensate the claimant for actual injury suffered and not to impose a The elements of abuse of right under Article 19 are the following: (1) the
penalty on the wrongdoer.[62] The award is not meant to enrich the existence of a legal right or duty, (2) which is exercised in bad faith, and (3)
complainant at the expense of the defendant, but to enable the injured party for the sole intent of prejudicing or injuring another.Article 20 speaks of the
to obtain means, diversion, or amusements that will serve to obviate the general sanction for all provisions of law which do not especially provide for
moral suffering he has undergone. It is aimed at the restoration, within the their own sanction; while Article 21 deals with acts contra bonus mores, and
limits of the possible, of the spiritual status quo ante, and should be has the following elements: (1) there is an act which is legal, (2) but which is
proportionate to the suffering inflicted. [63] Trial courts must then guard against contrary to morals, good custom, public order, or public policy, and (3) and it
the award of exorbitant damages; they should exercise balanced restrained is done with intent to injure.[72]
and measured objectivity to avoid suspicion that it was due to passion,
prejudice, or corruption or the part of the trial court. [64] Verily then, malice or bad faith is at the core of Articles 19, 20, and
21. Malice or bad faith implies a conscious and intentional design to do a
The award of moral damages cannot be granted in favor of a wrongful act for a dishonest purpose or moral obliquity.[73]Such must be
corporation because, being an artificial person and having existence only in substantiated by evidence.[74]
legal contemplation, it has no feelings, no emotions, no senses. It cannot,
therefore, experience physical suffering and mental anguish, which can be There is no adequate proof that ABS-CBN was inspired by malice or
experienced only by one having a nervous system. [65] The statement bad faith. It was honestly convinced of the merits of its cause after it had
in People v. Manero[66] and Mambulao Lumber Co. v. PNB[67] that a undergone serious negotiations culminating in its formal submission of a draft
corporation may recover moral damages if it has a good reputation that is contract. Settled is the rule that the adverse result of an action does not per
debased, resulting in social humiliation is an obiter dictum. On this score se make the action wrongful and subject the actor to damages, for the law
alone the award for damages must be set aside, since RBS is a corporation. could not have meant impose a penalty on the right to litigate. If damages
result from a persons exercise of a right, it is damnum absque injuria.[75]
The basic law on exemplary damages is Section 5 Chapter 3, Title XVIII,
Book IV of the Civil Code. These are imposed by way of example or WHEREFORE, the instant petition is GRANTED. The challenged
correction for the public good, in addition to moral, temperate, liquidated, or decision of the Court of Appeals in CA-G.R. CV No. 44125 is hereby
compensatory damages.[68] They are recoverable in criminal cases as part of REVERSED except as to unappealed award of attorneys fees in favor of
the civil liability when the crime was committed with one or more aggravating VIVA Productions, Inc.
circumstances;[69] in quasi-delicts, if the defendant acted with gross
No pronouncement as to costs.

625
SO ORDERED. FILIPINAS BROADCASTING NETWORK, INC., petitioner, vs. AGO
MEDICAL AND EDUCATIONAL CENTER-BICOL CHRISTIAN
Melo, Kapunan, Martinez, and Pardo, JJ., concur. COLLEGE OF MEDICINE, (AMEC-BCCM) and ANGELITA F.
AGO, respondents.

DECISION

CARPIO, J.:

The Case

This petition for review[1] assails the 4 January 1999 Decision[2] and 26
January 2000 Resolution of the Court of Appeals in CA-G.R. CV No. 40151.
The Court of Appeals affirmed with modification the 14 December 1992
Decision[3] of the Regional Trial Court of Legazpi City, Branch 10, in Civil
Case No. 8236. The Court of Appeals held Filipinas Broadcasting Network,
Inc. and its broadcasters Hermogenes Alegre and Carmelo Rima liable for
libel and ordered them to solidarily pay Ago Medical and Educational Center-
Bicol Christian College of Medicine moral damages, attorneys fees and costs
of suit.

The Antecedents

Expos is a radio documentary[4] program hosted by Carmelo Mel Rima


(Rima) and Hermogenes Jun Alegre (Alegre). [5] Expos is aired every morning
over DZRC-AM which is owned by Filipinas Broadcasting Network, Inc.
(FBNI). Expos is heard over Legazpi City, the Albay municipalities and other
Bicol areas.[6]

In the morning of 14 and 15 December 1989, Rima and Alegre exposed


various alleged complaints from students, teachers and parents against Ago
Medical and Educational Center-Bicol Christian College of Medicine (AMEC)
and its administrators. Claiming that the broadcasts were defamatory, AMEC
and Angelita Ago (Ago), as Dean of AMECs College of Medicine, filed a
[G.R. No. 141994. January 17, 2005]
complaint for damages[7] against FBNI, Rima and Alegre on 27 February
1990. Quoted are portions of the allegedly libelous broadcasts:

626
JUN ALEGRE: cross purpose with its reason for being it is possible for these foreign
foundations to lift or suspend their donations temporarily.[8]
Let us begin with the less burdensome: if you have children taking
medical course at AMEC-BCCM, advise them to pass all subjects xxx
because if they fail in any subject they will repeat their year level, taking
up all subjects including those they have passed already. Several On the other hand, the administrators of AMEC-BCCM, AMEC Science
students had approached me stating that they had consulted with the DECS High School and the AMEC-Institute of Mass Communication in their
which told them that there is no such regulation. If [there] is no such effort to minimize expenses in terms of salary are absorbing or
regulation why is AMEC doing the same? continues to accept rejects. For example how many teachers in AMEC are
former teachers of Aquinas University but were removed because of
xxx immorality? Does it mean that the present administration of AMEC have the
total definite moral foundation from catholic administrator of Aquinas
Second: Earlier AMEC students in Physical Therapy had complained University. I will prove to you my friends, that AMEC is a dumping ground,
that the course is not recognized by DECS. xxx garbage, not merely of moral and physical misfits. Probably they only
qualify in terms of intellect. The Dean of Student Affairs of AMEC is Justita
Third: Students are required to take and pay for the subject even if the Lola, as the family name implies. She is too old to work, being an old woman.
subject does not have an instructor - such greed for money on the part Is the AMEC administration exploiting the very [e]nterprising or compromising
of AMECs administration. Take the subject Anatomy: students would pay and undemanding Lola? Could it be that AMEC is just patiently making use of
for the subject upon enrolment because it is offered by the school. However Dean Justita Lola were if she is very old. As in atmospheric situation zero
there would be no instructor for such subject. Students would be informed visibility the plane cannot land, meaning she is very old, low pay follows. By
that course would be moved to a later date because the school is still the way, Dean Justita Lola is also the chairman of the committee on
searching for the appropriate instructor. scholarship in AMEC. She had retired from Bicol University a long time ago
but AMEC has patiently made use of her.
xxx
xxx
It is a public knowledge that the Ago Medical and Educational Center has
survived and has been surviving for the past few years since its inception MEL RIMA:
because of funds support from foreign foundations. If you will take a look at
the AMEC premises youll find out that the names of the buildings there are xxx My friends based on the expose, AMEC is a dumping ground for moral
foreign soundings. There is a McDonald Hall. Why not Jose Rizal or and physically misfit people. What does this mean? Immoral and physically
Bonifacio Hall? That is a very concrete and undeniable evidence that the misfits as teachers.
support of foreign foundations for AMEC is substantial, isnt it? With the report
which is the basis of the expose in DZRC today, it would be very easy for May I say Im sorry to Dean Justita Lola. But this is the truth. The truth is this,
detractors and enemies of the Ago family to stop the flow of support of that your are no longer fit to teach. You are too old. As an aviation, your case
foreign foundations who assist the medical school on the basis of the latters is zero visibility. Dont insist.
purpose. But if the purpose of the institution (AMEC) is to deceive students at

627
xxx Why did AMEC still absorb her as a teacher, a dean, and chairman of the an application; (2) be interviewed; and (3) undergo an apprenticeship and
scholarship committee at that. The reason is practical cost saving in salaries, training program after passing the interview. FBNI likewise claimed that it
because an old person is not fastidious, so long as she has money to buy the always reminds its broadcasters to observe truth, fairness and objectivity in
ingredient of beetle juice. The elderly can get by thats why she (Lola) was their broadcasts and to refrain from using libelous and indecent language.
taken in as Dean. Moreover, FBNI requires all broadcasters to pass the Kapisanan ng mga
Brodkaster sa Pilipinas (KBP) accreditation test and to secure a KBP permit.
xxx
On 14 December 1992, the trial court rendered a Decision [12] finding
xxx On our end our task is to attend to the interests of students. It is likely FBNI and Alegre liable for libel except Rima. The trial court held that the
that the students would be influenced by evil. When they become members broadcasts are libelous per se. The trial court rejected the broadcasters claim
of society outside of campus will be liabilities rather than assets. What that their utterances were the result of straight reporting because it had no
do you expect from a doctor who while studying at AMEC is so much factual basis. The broadcasters did not even verify their reports before airing
burdened with unreasonable imposition? What do you expect from a student them to show good faith. In holding FBNI liable for libel, the trial court found
who aside from peculiar problems because not all students are rich in their that FBNI failed to exercise diligence in the selection and supervision of its
struggle to improve their social status are even more burdened with false employees.
regulations. xxx[9] (Emphasis supplied)
In absolving Rima from the charge, the trial court ruled that Rimas only
The complaint further alleged that AMEC is a reputable learning participation was when he agreed with Alegres expos. The trial court found
institution. With the supposed exposs, FBNI, Rima and Alegre transmitted Rimas statement within the bounds of freedom of speech, expression, and of
malicious imputations, and as such, destroyed plaintiffs (AMEC and Ago) the press. The dispositive portion of the decision reads:
reputation. AMEC and Ago included FBNI as defendant for allegedly failing to
exercise due diligence in the selection and supervision of its employees, WHEREFORE, premises considered, this court finds for the
particularly Rima and Alegre. plaintiff. Considering the degree of damages caused by the
controversial utterances, which are not found by this court to be really
On 18 June 1990, FBNI, Rima and Alegre, through Atty. Rozil Lozares, very serious and damaging, and there being no showing that indeed the
filed an Answer[10] alleging that the broadcasts against AMEC were fair and enrollment of plaintiff school dropped, defendants Hermogenes Jun
true. FBNI, Rima and Alegre claimed that they were plainly impelled by a Alegre, Jr. and Filipinas Broadcasting Network (owner of the radio station
sense of public duty to report the goings-on in AMEC, [which is] an institution DZRC), are hereby jointly and severally ordered to pay plaintiff Ago Medical
imbued with public interest. and Educational Center-Bicol Christian College of Medicine (AMEC-BCCM)
the amount of P300,000.00 moral damages, plus P30,000.00 reimbursement
Thereafter, trial ensued. During the presentation of the evidence for the of attorneys fees, and to pay the costs of suit.
defense, Atty. Edmundo Cea, collaborating counsel of Atty. Lozares, filed a
Motion to Dismiss[11] on FBNIs behalf. The trial court denied the motion to SO ORDERED. [13] (Emphasis supplied)
dismiss. Consequently, FBNI filed a separate Answer claiming that it
exercised due diligence in the selection and supervision of Rima and Alegre. Both parties, namely, FBNI, Rima and Alegre, on one hand, and AMEC
FBNI claimed that before hiring a broadcaster, the broadcaster should (1) file and Ago, on the other, appealed the decision to the Court of Appeals. The

628
Court of Appeals affirmed the trial courts judgment with modification. The The Court of Appeals found Rima also liable for libel since he remarked
appellate court made Rima solidarily liable with FBNI and Alegre. The that (1) AMEC-BCCM is a dumping ground for morally and physically misfit
appellate court denied Agos claim for damages and attorneys fees because teachers; (2) AMEC obtained the services of Dean Justita Lola to minimize
the broadcasts were directed against AMEC, and not against her. The expenses on its employees salaries; and (3) AMEC burdened the students
dispositive portion of the Court of Appeals decision reads: with unreasonable imposition and false regulations. [16]

WHEREFORE, the decision appealed from is hereby AFFIRMED, subject to The Court of Appeals held that FBNI failed to exercise due diligence in
the modification that broadcaster Mel Rima is SOLIDARILY the selection and supervision of its employees for allowing Rima and Alegre
ADJUDGED liable with FBN[I] and Hermo[g]enes Alegre. to make the radio broadcasts without the proper KBP accreditation. The
Court of Appeals denied Agos claim for damages and attorneys fees because
SO ORDERED.[14] the libelous remarks were directed against AMEC, and not against her. The
Court of Appeals adjudged FBNI, Rima and Alegre solidarily liable to pay
FBNI, Rima and Alegre filed a motion for reconsideration which the AMEC moral damages, attorneys fees and costs of suit.
Court of Appeals denied in its 26 January 2000 Resolution.
Issues
[15]
Hence, FBNI filed this petition.
FBNI raises the following issues for resolution:
The Ruling of the Court of Appeals
I. WHETHER THE BROADCASTS ARE LIBELOUS;
The Court of Appeals upheld the trial courts ruling that the questioned
broadcasts are libelous per se and that FBNI, Rima and Alegre failed to II. WHETHER AMEC IS ENTITLED TO MORAL DAMAGES;
overcome the legal presumption of malice. The Court of Appeals found Rima
and Alegres claim that they were actuated by their moral and social duty to III. WHETHER THE AWARD OF ATTORNEYS FEES IS PROPER; and
inform the public of the students gripes as insufficient to justify the utterance
of the defamatory remarks. IV. WHETHER FBNI IS SOLIDARILY LIABLE WITH RIMA AND
ALEGRE FOR PAYMENT OF MORAL DAMAGES, ATTORNEYS
Finding no factual basis for the imputations against AMECs FEES AND COSTS OF SUIT.
administrators, the Court of Appeals ruled that the broadcasts were made
with reckless disregard as to whether they were true or false. The appellate The Courts Ruling
court pointed out that FBNI, Rima and Alegre failed to present in court any of
the students who allegedly complained against AMEC. Rima and Alegre We deny the petition.
merely gave a single name when asked to identify the students. According to
the Court of Appeals, these circumstances cast doubt on the veracity of the
This is a civil action for damages as a result of the allegedly defamatory
broadcasters claim that they were impelled by their moral and social duty to
remarks of Rima and Alegre against AMEC. [17] While AMEC did not point out
inform the public about the students gripes.
clearly the legal basis for its complaint, a reading of the complaint reveals

629
that AMECs cause of action is based on Articles 30 and 33 of the Civil Code. Every defamatory imputation is presumed malicious. [25] Rima and Alegre
Article 30[18] authorizes a separate civil action to recover civil liability arising failed to show adequately their good intention and justifiable motive in airing
from a criminal offense. On the other hand, Article 33 [19] particularly provides the supposed gripes of the students. As hosts of a documentary or public
that the injured party may bring a separate civil action for damages in cases affairs program, Rima and Alegre should have presented the public issues
of defamation, fraud, and physical injuries. AMEC also invokes Article 19 [20] of free from inaccurate and misleading information.[26] Hearing the students
the Civil Code to justify its claim for damages. AMEC cites Articles alleged complaints a month before the expos, [27] they had sufficient time to
2176[21] and 2180[22] of the Civil Code to hold FBNI solidarily liable with Rima verify their sources and information. However, Rima and Alegre hardly made
and Alegre. a thorough investigation of the students alleged gripes. Neither did they
inquire about nor confirm the purported irregularities in AMEC from the
I. Department of Education, Culture and Sports. Alegre testified that he merely
went to AMEC to verify his report from an alleged AMEC official who refused
Whether the broadcasts are libelous to disclose any information. Alegre simply relied on the words of the students
because they were many and not because there is proof that what they are
saying is true.[28] This plainly shows Rima and Alegres reckless disregard of
A libel[23] is a public and malicious imputation of a crime, or of a vice or
whether their report was true or not.
defect, real or imaginary, or any act or omission, condition, status, or
circumstance tending to cause the dishonor, discredit, or contempt of a
natural or juridical person, or to blacken the memory of one who is dead. [24] Contrary to FBNIs claim, the broadcasts were not the result of straight
reporting. Significantly, some courts in the United States apply the privilege
of neutral reportage in libel cases involving matters of public interest or public
There is no question that the broadcasts were made public and imputed
figures. Under this privilege, a republisher who accurately and disinterestedly
to AMEC defects or circumstances tending to cause it dishonor, discredit and
reports certain defamatory statements made against public figures is
contempt. Rima and Alegres remarks such as greed for money on the part of
shielded from liability, regardless of the republishers subjective awareness of
AMECs administrators; AMEC is a dumping ground, garbage of xxx moral
the truth or falsity of the accusation. [29] Rima and Alegre cannot invoke the
and physical misfits; and AMEC students who graduate will be liabilities
privilege of neutral reportage because unfounded comments abound in the
rather than assets of the society are libelous per se. Taken as a whole, the
broadcasts. Moreover, there is no existing controversy involving AMEC when
broadcasts suggest that AMEC is a money-making institution where
the broadcasts were made. The privilege of neutral reportage applies where
physically and morally unfit teachers abound.
the defamed person is a public figure who is involved in an existing
controversy, and a party to that controversy makes the defamatory
However, FBNI contends that the broadcasts are not malicious. FBNI statement.[30]
claims that Rima and Alegre were plainly impelled by their civic duty to air the
students gripes. FBNI alleges that there is no evidence that ill will or spite
However, FBNI argues vigorously that malice in law does not apply to
motivated Rima and Alegre in making the broadcasts. FBNI further points out
this case. Citing Borjal v. Court of Appeals,[31] FBNI contends that the
that Rima and Alegre exerted efforts to obtain AMECs side and gave Ago the
broadcasts fall within the coverage of qualifiedly privileged communications
opportunity to defend AMEC and its administrators. FBNI concludes that
for being commentaries on matters of public interest. Such being the case,
since there is no malice, there is no libel.
AMEC should prove malice in fact or actual malice. Since AMEC allegedly
failed to prove actual malice, there is no libel.
FBNIs contentions are untenable.

630
FBNIs reliance on Borjal is misplaced. In Borjal, the Court elucidated Secondly, there is reason to believe that defendant radio broadcasters,
on the doctrine of fair comment, thus: contrary to the mandates of their duties, did not verify and analyze the truth
of the reports before they aired it, in order to prove that they are in good faith.
[F]air commentaries on matters of public interest are privileged and constitute
a valid defense in an action for libel or slander. The doctrine of fair comment Alegre contended that plaintiff school had no permit and is not accredited to
means that while in general every discreditable imputation publicly made is offer Physical Therapy courses. Yet, plaintiff produced a certificate coming
deemed false, because every man is presumed innocent until his guilt is from DECS that as of Sept. 22, 1987 or more than 2 years before the
judicially proved, and every false imputation is deemed malicious, controversial broadcast, accreditation to offer Physical Therapy course had
nevertheless, when the discreditable imputation is directed against a public already been given the plaintiff, which certificate is signed by no less than the
person in his public capacity, it is not necessarily actionable. In order that Secretary of Education and Culture herself, Lourdes R. Quisumbing (Exh. C-
such discreditable imputation to a public official may be actionable, it rebuttal). Defendants could have easily known this were they careful enough
must either be a false allegation of fact or a comment based on a false to verify. And yet, defendants were very categorical and sounded too positive
supposition. If the comment is an expression of opinion, based on when they made the erroneous report that plaintiff had no permit to offer
established facts, then it is immaterial that the opinion happens to be Physical Therapy courses which they were offering.
mistaken, as long as it might reasonably be inferred from the facts.
[32]
(Emphasis supplied) The allegation that plaintiff was getting tremendous aids from foreign
foundations like Mcdonald Foundation prove not to be true also. The truth is
True, AMEC is a private learning institution whose business of educating there is no Mcdonald Foundation existing. Although a big building of plaintiff
students is genuinely imbued with public interest. The welfare of the youth in school was given the name Mcdonald building, that was only in order to
general and AMECs students in particular is a matter which the public has honor the first missionary in Bicol of plaintiffs religion, as explained by Dr. Lita
the right to know. Thus, similar to the newspaper articles in Borjal, the Ago. Contrary to the claim of defendants over the air, not a single centavo
subject broadcasts dealt with matters of public interest. However, unlike appears to be received by plaintiff school from the aforementioned McDonald
in Borjal, the questioned broadcasts are not based on established facts. Foundation which does not exist.
The record supports the following findings of the trial court:
Defendants did not even also bother to prove their claim, though denied by
xxx Although defendants claim that they were motivated by consistent reports Dra. Ago, that when medical students fail in one subject, they are made to
of students and parents against plaintiff, yet, defendants have not presented repeat all the other subject[s], even those they have already passed, nor their
in court, nor even gave name of a single student who made the complaint to claim that the school charges laboratory fees even if there are no
them, much less present written complaint or petition to that effect. To accept laboratories in the school. No evidence was presented to prove the bases for
this defense of defendants is too dangerous because it could easily give these claims, at least in order to give semblance of good faith.
license to the media to malign people and establishments based on flimsy
excuses that there were reports to them although they could not satisfactorily As for the allegation that plaintiff is the dumping ground for misfits, and
establish it. Such laxity would encourage careless and irresponsible immoral teachers, defendant[s] singled out Dean Justita Lola who is said to
broadcasting which is inimical to public interests. be so old, with zero visibility already. Dean Lola testified in court last Jan. 21,
1991, and was found to be 75 years old. xxx Even older people prove to be
effective teachers like Supreme Court Justices who are still very much in

631
demand as law professors in their late years. Counsel for defendants is past 8. It shall be the responsibility of the newscaster, commentator, host
75 but is found by this court to be still very sharp and effective. So is plaintiffs and announcer to protect public interest, general welfare and
counsel. good order in the presentation of public affairs and public
issues.[36](Emphasis supplied)
Dr. Lola was observed by this court not to be physically decrepit yet, nor
mentally infirmed, but is still alert and docile. The broadcasts fail to meet the standards prescribed in the Radio Code,
which lays down the code of ethical conduct governing practitioners in the
The contention that plaintiffs graduates become liabilities rather than assets radio broadcast industry. The Radio Code is a voluntary code of conduct
of our society is a mere conclusion. Being from the place himself, this court is imposed by the radio broadcast industry on its own members. The Radio
aware that majority of the medical graduates of plaintiffs pass the board Code is a public warranty by the radio broadcast industry that radio
examination easily and become prosperous and responsible professionals. [33] broadcast practitioners are subject to a code by which their conduct are
measured for lapses, liability and sanctions.
Had the comments been an expression of opinion based on established
facts, it is immaterial that the opinion happens to be mistaken, as long as it The public has a right to expect and demand that radio broadcast
might reasonably be inferred from the facts. [34] However, the comments of practitioners live up to the code of conduct of their profession, just like other
Rima and Alegre were not backed up by facts. Therefore, the broadcasts are professionals. A professional code of conduct provides the standards for
not privileged and remain libelous per se. determining whether a person has acted justly, honestly and with good faith
in the exercise of his rights and performance of his duties as required by
The broadcasts also violate the Radio Code[35] of the Kapisanan ng mga Article 19[37] of the Civil Code. A professional code of conduct also provides
Brodkaster sa Pilipinas, Ink. (Radio Code). Item I(B) of the Radio Code the standards for determining whether a person who willfully causes loss or
provides: injury to another has acted in a manner contrary to morals or good customs
under Article 21[38] of the Civil Code.
B. PUBLIC AFFAIRS, PUBLIC ISSUES AND COMMENTARIES
II.
1. x x x
Whether AMEC is entitled to moral damages
4. Public affairs program shall present public issues free
from personal bias, prejudice and inaccurate and misleading FBNI contends that AMEC is not entitled to moral damages because it is
information. x x x Furthermore, the station shall strive to a corporation.[39]
present balanced discussion of issues. x x x.
A juridical person is generally not entitled to moral damages because,
xxx unlike a natural person, it cannot experience physical suffering or such
sentiments as wounded feelings, serious anxiety, mental anguish or moral
shock.[40] The Court of Appeals cites Mambulao Lumber Co. v. PNB, et al.
7. The station shall be responsible at all times in the supervision of [41]
to justify the award of moral damages. However, the Courts statement
public affairs, public issues and commentary programs so that
in Mambulao that a corporation may have a good reputation which, if
they conform to the provisions and standards of this code.

632
besmirched, may also be a ground for the award of moral damages is courts failed to explicitly state in their respective decisions the rationale for
an obiter dictum.[42] the award of attorneys fees. [49] In Inter-Asia Investment Industries, Inc. v.
Court of Appeals,[50] we held that:
Nevertheless, AMECs claim for moral damages falls under item 7 of
Article 2219[43] of the Civil Code. This provision expressly authorizes the [I]t is an accepted doctrine that the award thereof as an item of damages is
recovery of moral damages in cases of libel, slander or any other form of the exception rather than the rule, and counsels fees are not to be awarded
defamation. Article 2219(7) does not qualify whether the plaintiff is a natural every time a party wins a suit. The power of the court to award attorneys
or juridical person. Therefore, a juridical person such as a corporation can fees under Article 2208 of the Civil Code demands factual, legal and
validly complain for libel or any other form of defamation and claim for moral equitable justification, without which the award is a conclusion without
damages.[44] a premise, its basis being improperly left to speculation and conjecture.
In all events, the court must explicitly state in the text of the decision, and not
Moreover, where the broadcast is libelous per se, the law implies only in the decretal portion thereof, the legal reason for the award of
damages.[45] In such a case, evidence of an honest mistake or the want of attorneys fees.[51](Emphasis supplied)
character or reputation of the party libeled goes only in mitigation of
damages.[46] Neither in such a case is the plaintiff required to introduce While it mentioned about the award of attorneys fees by stating that it
evidence of actual damages as a condition precedent to the recovery of lies within the discretion of the court and depends upon the circumstances of
some damages.[47] In this case, the broadcasts are libelous per se. Thus, each case, the Court of Appeals failed to point out any circumstance to justify
AMEC is entitled to moral damages. the award.

However, we find the award of P300,000 moral damages unreasonable. IV.


The record shows that even though the broadcasts were libelous per se,
AMEC has not suffered any substantial or material damage to its reputation. Whether FBNI is solidarily liable with Rima and Alegre
Therefore, we reduce the award of moral damages from P300,000
to P150,000. for moral damages, attorneys fees

III. and costs of suit

Whether the award of attorneys fees is proper FBNI contends that it is not solidarily liable with Rima and Alegre for the
payment of damages and attorneys fees because it exercised due diligence
FBNI contends that since AMEC is not entitled to moral damages, there in the selection and supervision of its employees, particularly Rima and
is no basis for the award of attorneys fees. FBNI adds that the instant case Alegre. FBNI maintains that its broadcasters, including Rima and Alegre,
does not fall under the enumeration in Article 2208 [48] of the Civil Code. undergo a very regimented process before they are allowed to go on air.
Those who apply for broadcaster are subjected to interviews, examinations
The award of attorneys fees is not proper because AMEC failed to justify and an apprenticeship program.
satisfactorily its claim for attorneys fees. AMEC did not adduce evidence to
warrant the award of attorneys fees. Moreover, both the trial and appellate

633
FBNI further argues that Alegres age and lack of training are irrelevant Rima and Alegre. FBNI merely showed that it exercised diligence in
to his competence as a broadcaster. FBNI points out that the minor the selection of its broadcasters without introducing any evidence to prove
deficiencies in the KBP accreditation of Rima and Alegre do not in any way that it observed the same diligence in the supervision of Rima and Alegre.
prove that FBNI did not exercise the diligence of a good father of a family in FBNI did not show how it exercised diligence in supervising its broadcasters.
selecting and supervising them. Rimas accreditation lapsed due to his non- FBNIs alleged constant reminder to its broadcasters to observe truth,
payment of the KBP annual fees while Alegres accreditation card was fairness and objectivity and to refrain from using libelous and indecent
delayed allegedly for reasons attributable to the KBP Manila Office. FBNI language is not enough to prove due diligence in the supervision of its
claims that membership in the KBP is merely voluntary and not required by broadcasters. Adequate training of the broadcasters on the industrys code of
any law or government regulation. conduct, sufficient information on libel laws, and continuous evaluation of the
broadcasters performance are but a few of the many ways of showing
FBNIs arguments do not persuade us. diligence in the supervision of broadcasters.

The basis of the present action is a tort. Joint tort feasors are jointly and FBNI claims that it has taken all the precaution in the selection of Rima
severally liable for the tort which they commit. [52] Joint tort feasors are all the and Alegre as broadcasters, bearing in mind their qualifications. However, no
persons who command, instigate, promote, encourage, advise, countenance, clear and convincing evidence shows that Rima and Alegre underwent FBNIs
cooperate in, aid or abet the commission of a tort, or who approve of it after it regimented process of application. Furthermore, FBNI admits that Rima and
is done, if done for their benefit. [53] Thus, AMEC correctly anchored its cause Alegre had deficiencies in their KBP accreditation, [56] which is one of FBNIs
of action against FBNI on Articles 2176 and 2180 of the Civil Code. requirements before it hires a broadcaster. Significantly, membership in the
KBP, while voluntary, indicates the broadcasters strong commitment to
As operator of DZRC-AM and employer of Rima and Alegre, FBNI is observe the broadcast industrys rules and regulations. Clearly, these
solidarily liable to pay for damages arising from the libelous broadcasts. As circumstances show FBNIs lack of diligence in selecting and supervising
stated by the Court of Appeals, recovery for defamatory statements Rima and Alegre. Hence, FBNI is solidarily liable to pay damages together
published by radio or television may be had from the owner of the station, a with Rima and Alegre.
licensee, the operator of the station, or a person who procures, or
participates in, the making of the defamatory statements. [54] An employer and WHEREFORE, we DENY the instant petition. We AFFIRM the Decision
employee are solidarily liable for a defamatory statement by the employee of 4 January 1999 and Resolution of 26 January 2000 of the Court of
within the course and scope of his or her employment, at least when the Appeals in CA-G.R. CV No. 40151 with the MODIFICATION that the award
employer authorizes or ratifies the defamation. [55] In this case, Rima and of moral damages is reduced from P300,000 to P150,000 and the award of
Alegre were clearly performing their official duties as hosts of FBNIs radio attorneys fees is deleted. Costs against petitioner.
program Expos when they aired the broadcasts. FBNI neither alleged nor
proved that Rima and Alegre went beyond the scope of their work at that SO ORDERED.
time. There was likewise no showing that FBNI did not authorize and ratify
the defamatory broadcasts. Davide, Jr., C.J., (Chairman), Quisumbing, Ynares-
Santiago, and Azcuna, JJ., concur.
Moreover, there is insufficient evidence on record that FBNI exercised
due diligence in the selection and supervision of its employees, particularly

634
entered into a Purchase Agreement under which the plaintiff
agreed to purchase from the said defendant preferred
shares of stock worth ONE MILLION PESOS
[P1,000,000.00] subject to the conditions set forth in such
agreement;...

2. That pursuant to the Purchase Agreement of September


4, 1961, the plaintiff, on January 31, 1962, paid the
defendant Lirag Textile Mills, Inc. the sum of FIVE
HUNDRED THOUSAND PESOS [P500,000.00] for which
the said defendant issued to plaintiff 5,000 preferred shares
with a par value of one hundred pesos [P10000] per share
as evidenced by stock Certificate No. 128, ...

3. That further in pursuance of the Purchase Agreement of


G.R. No. L-33205 August 31, 1987 September 4, 1961, the plaintiff paid to the Lirag Textile Mills,
Inc. the sum of FIVE UNDRED THOUSAND PESOS
LIRAG TEXTILE MILLS, INC., and BASILIO L. LIRAG, petitioners, [P500,000.00] for which the said defendant issued to plaintiff
vs. 5,000 preferred shares with a par value of one hundred
SOCIAL SECURITY SYSTEM, and HON. PACIFICO DE pesos [P100.00] per share as evidenced by Stock Certificate
CASTRO, respondents. No. 139, ...

4. That in accordance with paragraph 3 of the Purchase


Agreement of September 4, 1961 which provides for the
FERNAN, J.: repurchase by the Lirag Textile Mills, Inc. of the shares of
stock at regular intervals of one year beginning with the 4th
This is an appeal by certiorari involving purely questions of law from the year following the date of issue, Stock Certificates Nos. 128
decision rendered by respondent judge in Civil Case No. Q-12275 entitled and 139 were to be repurchased by the Lirag Textile Mills,
"Social Security System versus Lirag Textile Mills, Inc. and Basilio L. Lirag." Inc. thus:

The antecedent facts, as stipulated by the parties during the trial, are as CERT. No. AMOUNT DATE OF REDEMPTION
follows:
128 P100,000.00 February 14, 1965
1. That on September 4, 1961, the plaintiff [herein
respondent Social Security System] and the defendants 100,000.00 February 14, 1966
[herein petitioners] Lirag Textile Mills, Inc. and Basilio Lirag

635
100,000.00 February 14, 1967 6. That defendant corporation failed to redeem certificates of
Stock Nos. 128 and 139 by payment of the amounts
100,000.00 February 14, 1968 mentioned in paragraph 4 above;

100,000.00 February 14, 1969 7. That the Lirag Textile Mills, lnc. has not paid dividends in
the amounts and within the period set forth in paragraph 10
139 P100,000.00 July 3, 1966 of the complaint;*

100,000.00 July 3,1967 8. That letters of demands have been sent by the plaintiff to
the defendant to redeem the foregoing stock certificates and
pay the dividends set forth in paragraph 10 of the complaint,
100,000.00 July 3,1968
but the Lirag Textile Mills, Inc. has not made such
redemption nor made such dividend payments;
100,000.00 July 3, 1969
9. That defendant Basilio L. Lirag likewise received letters of
100,000.00 July 3,1970 demand from the plaintiff requiring him to make good his
obligation as surety;
5. That to guarantee the redemption of the stocks purchased
by the plaintiff, the payment of dividends, as well as the other 10. That notwithstanding such letters of demand to the
obligations of the Lirag Textile Mills, Inc., defendants Basilio defendant Basilio L. Lirag, Stock Certificates Nos. 128 and
L. Lirag signed the Purchase Agreement of September 4, 139 issued to plaintiff are still unredeemed and no dividends
1961 not only as president of the defendant corporation, but have been paid on said stock certificates;
also as surety so that should the Lirag Textile Mills, Inc. fail
to perform any of its obligations in the said Purchase
11. That paragraph 5 of the Purchase Agreement provides
Agreement, the surety shall immediately pay to the vendee
that should the Lirag Textile Mills, Inc. fail to effect any of the
the amounts then outstanding pursuant to Condition No. 4,
redemptions stipulated therein, the entire obligation shall
to wit:
immediately become due and demandable and the Lirag
Textile Mills, Inc., shall, furthermore, be liable to the plaintiff
To guarantee the redemption of the stocks in an amount equivalent to twelve per cent [12%] of the
herein purchased, the payment of the amount then outstanding as liquidated damages;
dividends, as well as other obligations of the
VENDOR herein, the SURETY hereby binds
12. That the failure of the Lirag Textile Mills, Inc. to redeem
himself jointly and severally liable with the
the foregoing certificates of stock and pay dividends thereon
VENDOR so that should the VENDOR fail to
were due to financial reverses, to wit:
perform any of its obligations hereunder, the
SURETY shall immediately pay to the
VENDEE the amounts then outstanding. '

636
[a] Unrestrained smuggling into the country investments. Thus, pursuant to paragraph 6 of the Purchase
of textiles from the United States and other Agreement of September 4, 1961 which provides as follows:
countries;
The VENDEE shall be allowed to have a
[b] Unrestricted entry of supposed remmants representative in the Board of Directors of
which competed with textiles of domestic the VENDOR with the right to participate in
produce to the disadvantage and economic the discussions and to vote therein;
prejudice of the latter;
14. That Messrs. Rene Espina, Bernardino Abes and Heber
[c] Scarcity of money and the unavailability Catalan were each issued one common share of stock as a
of financing facilities; qualifying share to their election to the Board of Directors of
the Lirag Textiles Mills, Inc.;
[d] Payment of interest on matured loans
extended to defendant corporation; 15. That Messrs. Rene Espina, Bernardino Abes and Heber
Catalan, during their respective tenure as member of the
[e] Construction of the Montalban plant of Board of Directors of the Lirag Textile Mills, Inc. attended the
the defendant corporation financed largely meetings of the said Board, received per diems for their
through reparation benefits; attendance therein in the same manner and in the same
amount as any other member of the Board of Directors,
[f] Labor problems occasioned by the fact participated in the deliberations therein and freely exercised
that the defendant company is financial (sic) their right to vote in such meetings. However, the per diems
unable to improve, in a substantial way, the received by the SSS representative do not go to the coffers
economic plight of its workers as a result of of the System but personally to the representative in the said
which two costly strikes had occurred, one in board of directors. 1
1965 and another in 1968; and
For failure of Lirag Textile Mills, Inc. and Basilio L. Lirag to comply with the
[g] The occurrence of a fire which destroyed terms of the Purchase Agreement, the SSS filed an action for specific
more than 1 million worth of raw cotton, performance and damages before the then Court of First Instance of Rizal,
paralyzed operations partially, increased Quezon City, praying that therein defendants Lirag Textile Mills, Inc. and
overhead costs and wiped out any expected Basilio L. Lirag be adjudged liable for [1] the entire obligation of P1M which
profits that year; became due and demandable upon defendants' failure to repurchase the
stocks as scheduled; [21 dividends in the amount of P220,000.00; [31
liquidated damages in an amount equivalent to twelve percent (12%) of the
13. That it has been the policy of the plaintiff to be
amount then outstanding; [4] exemplary damages in the amount of
represented in the board of directors of the corporation or
P100,000.00 and [5] attorney's fees of P20,000.00.
entity which has obtained financial assistance from the
System be it in terms of loans, mortgages or equity

637
Lirag Textile Mills, Inc. and Basilio L. Lirag moved for the dismissal of the agreement provides that said dividends shall be paid from
complaint, but were denied the relief sought. Thus, they filed their answer the net profits and earned surplus of petitioner corporation
with counterclaim, denying the existence of any obligation on their part to and respondent SSS has admitted that due to losses
redeem the preferred stocks, on the ground that the SSS became and still is sustained since -1964, no dividends had been and can be
a preferred stockholder of the corporation so that redemption of the shares declared by petitioner corporation;
purchased depended upon the financial ability of said corporation. Insofar as
defendant Basilio Lirag is concerned, it was alleged that his liability arises 3. Respondent judge erred in sentencing petitioners to pay
only if the corporation is liable and does not perform its obligations under the P146,400.00 in liquidated damages;
Purchase Agreement. They further contended that no liability on their part
has arisen because of the financial condition of the corporation upon which 4. Respondent judge erred in sentencing petitioners to pay
such liability was made to depend, particularly the non-realization of any P10,000.00 by way of attorney's fees;
profit or earned surplus. Thus, the other claims for dividends, liquidated
damages and exemplary damages are allegedly without basis.
5. Respondent judge erred in sentencing petitioners to pay
interest from the time of firing the complaint u to the time of
After entering into the Stipulation of Facts above-quoted, the parties filed full payment both on the P1,000,000.00 invested by
their respective memoranda and submitted the case for decision. respondent SSS in petitioner's corporation and on the
P220,000.00 which the SSS claims as dividends due on its
The lower court, ruling that the purchase agreement was a debt instrument, investments;
decided in favor of SSS and sentenced Lirag Textile Mills, Inc. and Basilio L.
Lirag to pay SSS jointly and severally P1,000,000.00 plus legal interest until 6. Respondent judge erred in holding that petitioner Lirag is
the said amount is fully paid; P220,000.00 representing the 8% per annum liable to redeem the P1,000,000.00 worth of preferred
dividends on the preferred shares plus legal interest up to the time of actual shares purchased by respondent SSS from petitioner
payment; P146,400.00 as liquidated damages; and P10,000.00 as attorney's corporation and the 8% cumulative dividend, it appearing
fees. The counterclaim of Lirag Textile Mills, Inc. and Basilio L. Lirag was that Lirag was merely a surety and not an insurer of the
dismissed. obligation;

Hence, this petition. 7. Respondent judge erred in dismissing the counterclaim of


petitioners.
Petitioners assign the following errors:
The fundamental issue in this case is whether or not the Purchase
1. The trial court erred in deciding that the Purchase Agreement entered into by petitioners and respondent SSS is a debt
Agreement is a debt instrument; instrument.

2. Respondent judge erred in holding petitioner corporation Petitioners claim that respondent SSS merely became and still is a preferred
liable for the payment of the 8% preferred and cumulative stockholder of the petitioner corporation, the redemption of the shares
dividends on the preferred shares since the purchase purchased by said respondent being dependent upon the financial ability of

638
petitioner corporation. Petitioner corporation, thus, has no obligation to agreement between the parties, though the precise terms and conditions
redeem the preferred stocks. thereof must be read together with, and regarded as qualified by the terms
and conditions of the Purchase Agreement.
On the other hand, respondent SSS claims that the Purchase Agreement is a
debt instrument, imposing upon the petitioners the obligation to pay the The rights given by the Purchase Agreement to respondent SSS are rights
amount owed, and creating as between them the relation of creditor and not enjoyed by ordinary stockholders. This fact could only lead to the
debtor, not that of a stockholder and a corporation. conclusion made by the trial court that:

We uphold the lower court's finding that the Purchase Agreement is, indeed, The aforementioned rights specially stipulated for the benefit
a debt instrument. Its terms and conditions unmistakably show that the of the plaintiff [respondent SSS] suggest eloquently an
parties intended the repurchase of the preferred shares on the respective intention on the part of the plaintiff [respondent SSS] to
scheduled dates to be an absolute obligation which does not depend upon facilitate a loan to the defendant corporation upon the latter's
the financial ability of petitioner corporation. This absolute obligation on the request. In order to afford protection to the plaintiff which
part of petitioner corporation is made manifest by the fact that a surety was otherwise is provided by means of collaterals, as the plaintiff
required to see to it that the obligation is fulfilled in the event of the principal exacts in its grants of loans in its ordinary transactions of this
debtor's inability to do so. The unconditional undertaking of petitioner kind, as it is looked upon more as a lending institution rather
corporation to redeem the preferred shares at the specified dates constitutes than as an investing agency, the purchase agreement
a debt which is defined "as an obligation to pay money at some fixed future supplied these protective rights which would otherwise be
time, or at a time which becomes definite and fixed by acts of either party furnished by collaterals to the loan. Thus, the membership in
and which they expressly or impliedly, agree to perform in the contract. 2 the board is to have a watchdog in the operation of the
business of the corporation, so as to insure against
A stockholder sinks or swims with the corporation and there is no obligation mismanagement which may result in losses not entirely
to return the value of his shares by means of repurchase if the corporation unavoidable since payment for purposes of redemption as
incurs losses and financial reverses, much less guarantee such repurchase well as the dividends is expressly stipulated to come from
through a surety. profits and/or surplus. Such a right is never exacted by an
ordinary stockholder merely investing in the corporation. 3
As private respondent rightly contends, if the parties intended it [SSS] to be
merely a stockholder of petitioner corporation, it would have been sufficient Moreover, the Purchase Agreement provided that failure on the part of
that Preferred Certificates Nos. 128 and 139 were issued in its name as the petitioner to repurchase the preferred shares on the scheduled due dates
preferred certificates contained all the rights of a stockholder as well as renders the entire obligation due and demandable, with petitioner in such
certain obligations on the part of petitioner corporation. However, the parties eventuality liable to pay 12% of the then outstanding obligation as liquidated
did in fact execute the Purchase Agreement, at the same time that the damages. These features of the Purchase Agreement, taken collectively,
petitioner corporation issued its preferred stock to the respondent SSS. The clearly show the intent of the parties to be bound therein as debtor and
Purchase Agreement serves to define the rights and obligations of the parties creditor, and not as corporation and stockholder.
and to establish firmly the liability of petitioners in case of breach of contract.
The Certificates of Preferred Stock serve as additional evidence of the

639
Petitioners' contention that it is beyond the power and competence of The award of the sum of P146,400.00 in liquidated damages representing
petitioner corporation to redeem the preferred shares or pay the accrued 12% of the amount then outstanding is correct, considering that petitioners in
dividends due to financial reverses can not serve as legal justification for the stipulation of facts admitted having failed to fulfill their obligations under
their failure to perform under the Purchase Agreement. The Purchase the Purchase Agreement. The grant of liquidated damages in the amount
Agreement constitutes the law between the parties and obligations arising ex stated is expressly provided for in the Purchase Agreement in case of
contractu must be fulfilled in accordance with the stipulations. 4 Besides, it contractual breach.
was precisely this eventuality that was sought to be avoided when
respondent SSS required a surety for the obligation. The pronouncement of the lower court for the payment of interests on both
the unredeemed shares and unpaid dividends is also in order. Per stipulation
Thus, it follows that petitioner Basilio L. Lirag cannot deny liability for of facts, petitioners did not deny the fact of non-payment of dividends nor
petitioner corporation's default. As surety, Basilio L. Lirag is bound their failure to purchase the preferred shares. Since these involve sums of
immediately to pay respondent SSS the amount then outstanding. money which are overdue, they are bound to earn legal interest from the time
of demand, in this case, judicial, i.e., the time of filing the action.
The obligation of a surety differs from that of a guarantor in
that the surety insures the debt, whereas the guarantor Petitioner Basilio L. Lirag is precluded from denying his liability under the-
merely insures solvency of the debtor; and the surety Purchase Agreement. After his firm representation to "pay immediately to the
undertakes to pay if the principal does not pay, whereas a VENDEE the amounts then outstanding" evidencing his commitment as
guarantor merely binds itself to pay if the principal is unable SURETY, he is estopped from denying the same. His signature in the
to pay. 5 agreement carries with it the official imprimatur as petitioner corporation's
president, in his personal capacity as majority stockholder, as surety and as
On the liability of petitioners to pay 8% cumulative dividend, We agree with solidary obligor. The essence of his obligation as surety is to pay immediately
the observation of the lower court that the dividends stipulated by the parties without qualification whatsoever if petitioner corporation does not pay. To
served evidently as interests. 6 The amount thereof was fixed at 8% per have another interpretation of petitioner Lirag's liability as surety would
annum and was not made to depend upon or to fluctuate with the amount of violate the integrity of the Purchase Agreement as well as the clear and
profits or surplus realized, a clear indication that the parties intended to give unmistakable intent of the parties to the same.
a sure and fixed earnings on the principal loan. The fact that the dividends
were supposed to be paid out of net profits and earned surplus, of which WHEREFORE, the decision in Civil Case No. Q-12275 entitled "Social
there were none, does not excuse petitioners from the payment thereof, Security System vs. Lirag Textile Mills, Inc. and Basilio L. Lirag" is hereby
again for the reason that the undertaking of petitioner Basilio L. Lirag as affirmed in toto. Costs against petitioners.
surety, included the payment of dividends and other obligations then
outstanding. SO ORDERED.

Gutierrez, Jr., Feliciano, Bidin and Cortes, JJ., concur.

640
G.R. No. L-39427 February 24, 1934

TIRSO GARCIA, in his capacity as receiver of the Mercantile Bank of


China, plaintiff-appellee,
vs.
LIM CHU SING, defendant-appellant.

Marcelino Lontok for appellant.


Nicolas Santiago for appellee.

VILLA-REAL, J.:

This is an appeal taken by the defendant Lim Chu Sing from the judgment
rendered by the Court of First Instance of Manila, the dispositive part of
which reads as follows:

Wherefore, judgment is rendered sentencing the defendant to pay


the sum of P9,105.17 with interest thereon at the rate of six per cent

641
per annum from September 1, 1932, until fully paid, plus the sum of 2. The plaintiff admits the allegations in the answer, particularly with
P910.51, as attorney's fees, with the costs of this suit. reference to the fact that the defendant is the owner of two hundred
shares at a par value of fifty pesos (P50) each, that is (Pl0,000).
In conformity with the stipulation, this judgment shall be subject to
execution after ninety (90) days. So ordered. 3. The court may render judgment in accordance with this stipulation,
but the same shall be subject to execution after ninety (90) days.
In support of his appeal, the appellant assigns the following alleged errors as
committed by the court a quo in its decision, to wit: Wherefore, they respectfully submit this stipulation and pray that
judgment be rendered in accordance therewith.
1. In denying the motion dated December 27, 1932, praying for the
inclusion of Lim Cuan Sy, being the principal debtor, as party to this The facts alleged in the complaint and admitted by both parties under the
suit. above quoted stipulation of facts are as follows:

2. In holding as improper the compensation of the defendant's debt On June 20, 1930, the defendant-appellant Lim Chu Sing executed and
of P9,106.17, claimed in the complaint, with his credit amounting to delivered to the Mercantile Bank of China promissory note for the sum of
P10,000 with the Mercantile Bank of China. P19,605.17 with interest thereon at 6 per cent per annum, payable monthly
as follows: P1,000 on July 1, 1930; P500 on August 1, 1930; and P500 on
3. In not ordering that after the compensation the plaintiff-appellee, the first of every month thereafter until the amount of the promissory note
as receiver of the Mercantile Bank of China, should liquidate the together with the interest thereon is fully paid (Exhibit A). One of the
dividends of the defendant-appellant's shares. conditions stipulated in said promissory note is that in case of defendant's
default in the payment of any of the monthly installments, as they become
4. In sentencing the defendant-appellant to pay to the plaintiff- due, the entire amount or the unpaid balance thereof together with interest
appellee the sum of P910.51 as attorney's fees, plus interest at 6 per thereon at 6 per cent per annum, shall become due and payable on demand.
cent per annum on the sum of P9,105.17, with costs. The defendant had been, making several partial payments thereon, leaving
an unpaid balance of P9,105.17. However, he defaulted in the payment of
several installments by reason of which the unpaid balance of P9,105.17 on
5. In denying the motion for a new trial.
the promissory note has ipso factobecome due and demandable.

When the case was called for hearing, the parties submitted the following
The facts alleged in the answer and admitted by both parties under the same
stipulation of facts for the consideration of the trial court, to wit:
stipulation of facts are as follows:

Come now both parties and to this Honorable Court respectfully


The debt which is the subject matter of the complaint was not really an
submit the following stipulation:
indebtedness of the defendant but of Lim Cuan Sy, who had an account with
the plaintiff bank in the form of "trust receipts" guaranteed by the defendant
1. The defendant admits the facts alleged in the complaint. as surety and with chattel mortgage securities. The plaintiff bank, without the
knowledge and consent of the defendant, foreclosed the chattel mortgage

642
and privately sold the property covered thereby. Inasmuch as Lim Cuan Sy specific to direct the attention of the court to the alleged defects.' (8 Enc. Pl
failed to comply with his obligations, the plaintiff required the defendant, as and Pr., 289.)" (Garcia de Lara vs. Gonzales de Lara, 2 Phil., 297.) Inasmuch
surety, to sign a promissory note for the sum of P19,105.17 payable in the as an exception is an objection taken to the decision of the trial court upon a
manner hereinbefore stated (Exhibit A). The defendant had been paying the matter of law and is a notice that the party taking it will submit for the
corresponding installments until the debt was reduced to the sum of consideration of the appellate court the ruling deemed erroneous, failure to
P9,105.17 claimed in the complaint. The defendant is the owner of shares of interpose it deprived the appellant of the right to raise the question whether
stock of the plaintiff Mercantile Bank of China amounting to P10,000. The or not the court a quo committed the alleged error attributed to it in its ruling
plaintiff bank is now under liquidation. which had not been excepted to by the said appellant. The inclusion in, or
exclusion from an action of a certain party is a question of law. The herein
On December 27, 1932, the defendant-appellant Lim Chu Sing filed a motion defendant-appellant, not having excepted to the order of the Court of First
praying for the inclusion of the principal debtor Lim Cuan Sy as party Instance of Manila denying his motion for the inclusion of Lim Cuan Sy as
defendant so that he could avail himself of the benefit of the exhaustion of party defendant, is estopped from raising such question upon appeal (Roman
the property of said Lim Cuan Sy. Said motion was denied in open court by Catholic Bishop of Lipa vs. Municipality of San Jose, 27 Phil., 571;
the presiding judge without the defendant-appellant having excepted to such Vergara vs. Laciapag, 28 Phil., 439; Andrews vs. Morente Rosario, 9 Phil.,
order of denial. 634).

The proceeds of the sale of the mortgaged chattels together with other The second question to be decided is whether or not it is proper to
payments made were applied to the amount of the promissory note in compensate the defendant-appellant's indebtedness of P9,105.17, which is
question, leaving the balance which the plaintiff now seeks to collect. claimed in the complaint, with the sum of P10,000 representing the value of
his shares of stock with the plaintiff entity, the Mercantile Bank of China.
The first question to be decided in this appeal is whether or not the court a
quo erred in denying the motion for inclusion of a party a defendant, filed by According to the weight of authority, a share of stock or the certificate thereof
the defendant-appellant. is not an indebtedness to the owner nor evidence of indebtedness and,
therefore, it is not a credit (14 Corpus Juris, p. 388, see. 511). Stockholders,
According to the provisions of section 141 of the Code of Civil Procedure, as such, are not creditors of the corporation (14 Corpus Juris, p. 848, Sec.
". . . Rulings of the court upon minor matters, such as adjournments, 1289). It is the prevailing doctrine of the American courts, repeatedly
postponements of trials, the extension of time for filing pleadings or motions, asserted in the broadest terms, that the capital stock of a corporation is a
and other matters addressed to the discretion of the court in the performance trust fund to be used more particularly for the security of creditors of the
of its duty, shall not be subject to exception. But exception may be taken to corporation, who presumably deal with it on the credit of its capital stock (14
any other ruling, order, or judgment of the court made during the pendency of Corpus Juris, p. 383, sec. 505). Therefore, the defendant-appellant Lim Chu
the action in the Court of First Instance." "An `exception' has been defined as Sing not being a creditor of the Mercantile Bank of China, although the latter
an objection taken to the decision of the trial court upon a matter of law, and is a creditor of the former, there is no sufficient ground to justify a
is a notice that the party taking it preserves for the consideration of the compensation (art. 1195, Civil Code; Acua Co Chongco vs. Dievas, 12 Phil.,
appellate court a ruling deemed erroneous. (8 Am. Enc. P. and P., 157.)" " 250).
`Errors in a judgment or decree will not be noticed on appeal in the absence
of objections and exceptions taken below, and they should be sufficiently The third question to be decided in this appeal is whether or not the court a
quo erred in sentencing the said defendant-appellant to pay the sum of

643
P910.51 as attorney's fees in addition to interest at 6 per cent per annum on Wherefore, with the sole modification that the costs be eliminated from the
the amount sought in the complaint. appealed judgment, the same is hereby affirmed, without special
pronouncement as to costs of this instance. So ordered.
The pertinent clause of the promissory note Exhibit A reads as follows: "In
case of default of any of the above installments, the total amount of the Malcolm, Hull, Imperial, and Goddard, JJ., concur.
balance still unpaid of this note will become due and payable on demand
plus interest thereon at the rate of 6 per cent per annum from date of this
note until payment is made. And I further agree to pay an additional sum
equivalent to 10 per cent of the said note to cover cost and attorney's fees for
collection."

The stipulation relative to the payment of interest at the rate of 6 per cent per
annum on the unpaid balance of the promissory note Exhibit A refers to the
capital and the 10 per cent stipulated for costs and attorney's fees cannot be
considered as interest but an indemnity for damages occasioned by the
collection of the indebtedness through judicial process. Therefore the two
rates in question cannot be combined and considered usurious interest.

With reference to the costs, the 10 per cent stipulated in the promissory note
is for costs and attorney's fees which may be incurred in the collection of the
indebtedness through judicial process. Therefore, the defendant-appellant
should not again be made to pay for them (Bank of the Philippine Islands vs.
Yulo, 31 Phil., 476).

In view of the foregoing, this court is of the opinion and so holds: (1) That
failure to file an exception to a ruling rendered in open court denying a
motion for the inclusion of a party as defendant deprives the petitioner, upon
appeal of the right to raise the question whether such denial proper or
improper; (2) that the shares of a banking corporation do not constitute an
indebtedness of the corporation to the stockholder and, therefore, the latter is
G.R. No. L-27872 February 25, 1928
not a creditor of the former for such shares; (3) that the indebtedness of a
shareholder to a banking corporation cannot be compensated with the
amount of his shares therein, there being no relation of creditor and debtor THE NATIONAL EXCHANGE CO., INC., plaintiff-appellee,
with respect to such shares; and (4) that the percentage stipulated in a vs.
contract, for costs and attorney's fees for the collection of an indebtedness, I. B. DEXTER, defendant-appellant.
includes judicial costs.

644
Ross, Lawrence & Selph and Antonio T. Carrascoso, Jr., for appellant. As the case reaches this court the sole question here presented for
Lucio Javillonar for appellee. consideration is one of law, namely, whether the stipulation contained in the
subscription to the effect that the subscription is payable from the first
STREET, J.: dividends declared on the shares has the effect of relieving the subscriber
from personal liability in an action to recover the value of the shares. The trial
This action was instituted in the Court of First Instance of Manila by the court held, in effect, that the stipulation mentioned is invalid.
National Exchange Co., Inc., as assignee (through the Philippine National
Bank) of C. S. Salmon & Co., for the purpose of recovering from I. B. Dexter In discussing this problem we accept as sound law the proposition
a balance of P15,000, the par value of one hundred fifty shares of the capital propounded by the appellant's attorneys and taken from Fletcher's
stock of C. S. Salmon & co., with interest and costs. Upon hearing the cause Cyclopedia as follows:
the trial judge gave judgment for the plaintiff to recover the amount claimed,
with lawful interest from January 1, 1920, and with costs. From this judgment In the absence of restrictions in its character, a corporation, under its
the defendant appealed. general power to contract, has the power to accept subscriptions
upon any special terms not prohibited by positive law or contrary to
It appears that on August 10, 1919, the defendant, I. B. Dexter, signed a public policy, provided they are not such as to require the
written subscription to the corporate stock of C. S. Salmon & Co. in the performance of acts which are beyond the powers conferred upon
following form: the corporation by its character, and provided they do not constitute
a fraud upon other subscribers or stockholders, or upon persons who
I hereby subscribe for three hundred (300) shares of the capital stock are or may become creditors of the corporation. (Fletcher, Cyc.
of C. S. Salmon and Company, payable from the first dividends Corp., sec. 602, p. 1314.)
declared on any and all shares of said company owned by me at the
time dividends are declared, until the full amount of this subscription Under the American regime corporate franchises in the Philippine Islands are
has been paid. granted subject to the provisions of section 74 of the Organic Act of July 1,
1902, which, in the part here material, is substantially reproduced in section
Upon this subscription the sum of P15,000 was paid in January, 1920, from a 28 of the Autonomy Act of August 29, 1916. In the Organic Act it is among
dividend declared at about that time by the company, supplemented by other things, declared: "That all franchises, privileges, or concessions
money supplied personally by the subscriber. Beyond this nothing has been granted under this Act shall forbid the issue of stock or bonds except in
paid on the shares and no further dividend has been declared by the exchange for actual cash or for property at a fair valuation equal to the par
corporation. There is therefore a balance of P15,000 still paid upon the value of the stock or bonds so issued; . . . ." (Act of Congress of July 1, 1902,
subscription. sec. 74.)

Pursuant to this provision we find that the Philippine Commission inserted in


the Corporation Law, enacted March 1, 1906, the following provision: ". . . no
corporation shall issue stock or bonds except in exchange for actual cash
paid to the corporation or for property actually received by it at a fair

645
valuation equal to the par value of the stock or bonds so issued." (Act No. The rule thus stated is supported by a long line of decisions from numerous
1459, sec. 16 as amended by Act No. 2792, sec. 2.) courts, with little or no diversity of opinion. As stated in the headnote to the
opinion of the Supreme Court of United States in the case of Putnan vs.New
The prohibition against the issuance of shares by corporations except for Albany, etc. Railroad Co. as reported in 21 Law. ed., 361, the rule is that
actual cash to the par value of the stock to its full equivalent in property is "Conditions attached to subcriptions, which, if valid, lessen the capital of the
thus enshrined in both the organic and statutory law of the Philippine; company, are a fraud upon the grantor of the franchise, and upon those who
Islands; and it would seem that our lawmakers could scarely have chosen may become creditors of the corporation, and upon unconditional
language more directly suited to secure absolute equality stockholders with stockholders."
respect to their liability upon stock subscriptions. Now, if it is unlawful to issue
stock otherwise than as stated it is self-evident that a stipulation such as that In the appellant's brief attention is called to the third headnote to
now under consideration, in a stock subcription, is illegal, for this stipulation Bank vs. Cook (125 Iowa, 111), where it is stated that a collateral agreement
obligates the subcriber to pay nothing for the shares except as dividends with a subcriber to stock that his subcription shall not be collectible except
may accrue upon the stock. In the contingency that dividends are not paid, from dividends on the stock, is valid as between the parties and a complete
there is no liability at all. This is a discrimination in favor of the particular defense to a suit on notes given for the amount of the subscription. A careful
subcriber, and hence the stipulation is unlawful. persual of the decision will show that the rule thus broadly stated in the
headnote is not justified by anything in the reported decision; for what the
The general doctrine of corporation law is in conformity with this conclusion, court really held was that the making of such promise by the agent of the
as may be seen from the following proposition taken from the standard corporation who sold the stock is admissible in evidence in support of the
encyclopedia treatise, Corpus Juris: defense of fraud and failure of consideration. Moreover, even if the decision
had been to the effect supposed, the relu announced in the headnote could
Nor has a corporation the power to receive a subscription upon such have no weight in a jurisdiction like this were there is a statutory provision
terms as will operate as a fraud upon the other subscribers or prohibiting such agreements.
stockholders by subjecting the particular subcriber to lighter burdens,
or by giving him greater rights and privileges, or as a fraud upon We may add that the law in force in this jurisdiction makes no distinction, in
creditors of the corporation by withdrawing or decreasing the capital. respect to the liability of the subcriber, between shares subscribed before
It is well settled therefore, as a general rule, that an agreement incorporation is effected and shares subscribed thereafter. All like are bound
between a corporation and a particular subscriber, by which the to pay full value in cash or its equivalent, and any attempt to discriminate in
subscription is not to be payable, or is to be payable in part only, favor of one subscriber by relieving him of this liability wholly or in part is
whether it is for the purpose of pretending that the stock is really forbidden. In what is here said we have reference of course primarily to
greater than it is, or for the purpose of preventing the predominance subcriptions to shares that have not been previously issued. It is conceivable
of certain stockholders, or for any other purpose, is illegal and void that the power of the corporation to make terms with the purchaser would be
as in fraud of other stockholders or creditors, or both, and cannot be greater where the shares which are the subject of the transaction have been
either enforced by the subcriber or interposed as a defense in an acquired by the corporation in course of commerce, after they have already
action on the subcription. (14 C. J., p. 570.) been once issued. But the shares with which are here concerned are not of
this sort.

646
The judgment appealed from must be affirmed, and it is so ordered, with directly to this court under G.R. No. L-4824. Defendant Irineo Baltazar
costs against the appellant. appealed to the Court of Appeals. By a resolution of that appellate tribunal,
the appeal was certified to this court pursuant to section 17, (5) and (6) of the
Malcolm, Ostrand, Johns, Romualdez and Villa-Real, JJ., concur. Judiciary Act of 1948, and is now listed here under G.R. No. L-6344.

The main facts of the case are not disputed, and we are reproducing and
making our own the relation of facts contained in the decision appealed from.

The plaintiff, Lingayen Gulf Electric Power Company is a domestic


corporation with an authorized capital stock of P300,000 divided into 3,000
shares with a par value of P100 per share. The defendant, Irineo Baltazar
appears to have subscribed for 600 shares on account of which he had paid
upon the organization of the corporation the sum of P15,000. (See Exhibit A,
page 2). After incorporation, the defendant made further payments on
G.R. No. L-4824 June 30, 1953 account of his subscription, leaving a balance of P18,500 unpaid for, which
amount, the plaintiff now claims in this action.
LINGAYEN GULF ELECTRIC POWER COMPANY, INC., plaintiff-appellant,
vs. On July 23, 1946, a majority of the stockholders of the corporation, among
IRINEO BALTAZAR, defendant-appellee. them the herein defendant, held a meeting and adopted stockholders'
resolution No. 17. By said resolution, it was agreed upon by the stockholders
x---------------------------------------------------------x present to call the balance of all unpaid subscribed capital stock as of July
23, 1946, the first 50 per cent payable within 60 days beginnning August 1,
G.R. No. L-6244 June 30, 1953 1946, and the remaining 50 per cent payable within 60 days beginning
October 1, 1946. The resolution also provided, that all unpaid subscription
LINGAYEN GULF ELECTRIC POWER COMPANY, INC., plaintiff-appellee, after the due dates of both calls would be subject to 12 per cent interest per
vs. annum. Lastly, the resolution provided, that after the expiration of 60 days'
IRINEO BALTAZAR, defendant and appellant. grace which would be on December 1, 1946, for the first call, and on
February 1, 1947, for the second call, all subscribed stocks remaining unpaid
Manuel L. Fernandez for appellant. would revert to the corporation. (See Exhibit F and Exhibit I).
Sofronio C. Quimson and daniel C. Macaraeg for appellee.
On September 22, 1946, the plaintiff corporation wrote a letter to the
MONTEMAYOR, J.: defendant reminding him that the first 50 per cent of his unpaid subscription
would be due on October 1, 1946. The plaintiff requested the defendant to
"kindly advise the company thru the undersigned your decision regarding this
These two cases here on appeal stem from the same case, that of civil case
matter." (See Exhibit 4). The defendant answered on September 25, 1946,
No. 10944 of the Court of First Instance of Pangasinan. From the trial court's
asking the corporation that he be allowed to pay his unpaid subscription by
decision, plaintiff Lingayen Gulf Electric Power Company, Inc. appealed

647
February 1, 1947. In his answer, the defendant also agreed that if he could On September 28, 1949, the legal counsel of the plaintiff corporation wrote a
not pay the balance of his subscription by February 1, 1947, his unpaid letter to the defendant, demanding the payment of the unpaid balance of his
subscription would be reverted to the corporation. (See Exhibit 5). subscription amounting to P18,500. Copy of this letter was sent by registered
mail to the defendant on September 29,1 949. (See Exhibit G). The
On December 19, 1947, the defendant wrote another letter to the members defendant ignored the said demand. Hence this action.
of the Board of Directors of the plaintiff corporation, offering to withdraw
completely from the corporation by selling out to the corporation all his The defendant, in his answer, disclaims liability tot he plaintiff corporation on
shares of stock in the total amount of P23,000. (See Exhibit 8). Apparently the following grounds:
this offer of the defendant was left unacted upon by the plaintiff.
1. That the plaintiffs' action is premature because there was no valid call; and
On April 17, 1948, the Board of Directors of the plaintiff corporation held a
meeting, and in the course of the said meeting they adopted Resolution No. 2. That granting that there was a valid call, he was released from the
17. This resolution in effect set aside the stockholders resolution approved on obligation of the balance of his subscription by stockholders' resolution No.
June 23, 1946 (Exhibit D), on the ground that said stockholders' resolution 17 and No. 4.
was null and void, and because the plaintiff corporation was not in a financial
position to absorb the unpaid balance of the subscribed capital stock. At the By way of counterclaim, the defendant also claims from the plaintiff a
said meeting the directors also decided to call 50 per cent of the unpaid reasonable compensation at the rate of P700 per month as president of the
subscription within 30 days from April 17, 1948, the call payable within 60 company, for the period from March 1, 1946 to December 31, 1948.
days from receipt of notice from the Secretary-Treasurer. This resolution also
authorized legal counsel of the company to take all the necessary legal steps
In the light of the foregoing undisputed facts, the only questions are as
for the collection of the payment of the call. (See Exhibit E-2).
follows:

On June 10, 1949, the stockholders of the corporation held another meeting
1. Was the call Exhibit E-2 valid?
in which the stockholders were all present, either in person or by proxy. At
such meeting, the stockholders adopted resolution No. 4, whereby it was
agreed to revalue the stocks and assets of the company so as to attract 2. Was the defendant released from the obligation of the unpaid balance of
outside investors to put in money for the rehabilitation of the company. The his subscription by virtue of stockholders' resolution Nos. 17 and 4?
president was authorized to make all arrangement for such appraisal and the
Secretary to call a meeting upon completion of the reassessment. (See 3. Is the defendant entitled to compensation as president of the plaintiff
Exhibit 2). corporation?

It was admitted by the defendant that he received notice from the Secretary- In an exhaustive and well prepared decision, Judge M. Mejia of the lower
Treasurer of the company, demanding payment of the unpaid balance of his court found that the call for payment embodied in resolution No. 17 of July
subscription. It was agreed by the parties that the call of the Board of 23, 1946 was null and void for lack of publication; consequently, he
Directors was not published in a newspaper of general circulation as required dismissed the complaint as premature. He further held said resolution null
by section 40 of the Corporation Law. and void in so far as it tried to relieve the defend- ant from liability on his
unpaid subscription, on the ground that the resolution was not approved by

648
all the stockholders of the corporation. He also dismissed the defendant's SEC. 3744. Provisions requiring notice of calls. The governing statute,
counterclaim for compensation as president of the corporation. charter or by-laws usually require that notice of calls be given the subscriber
or stockholder. If any particular notice or demand is required by either of
Inasmuch as in the two appeals, the assignment of errors are related to each these, or by the contract of subscription, then such notice or demand must be
other, and because they refer to the same case, we propose to determine given, and must be alleged and proved in order to maintain an action for the
both appeals in one single decision. call.

We agree with the lower court that the law requires that notice of any call for xxx xxx xxx
the payment of unpaid subscription should be made not only personally but
also by publication. This is clear from the provisions of section 40 of the SEC. 3745. Notice. Compliance with requirements-From what has
Corporation Law, Act No. 1459, as amended, which reads as follows: preceded it is clear that where any particular form or kind of notice is
required, such form or kind must be given-the requirement must be complied
SEC. 40. Notice of call for unpaid subscriptions must be either personally with. Thus, where the charter expressly required notice to be given in certain
served upon each stockholder or deposited in the post office, postage newspapers for a certain number of days, the corporation must show
prepaid, addressed to him at his place of residence, if known, and if not compliance with the conditions before recovery on the call. An action is
known, addressed to the place where the principal office of the corporation is ordinarily made effective by notice thereof to the subscribers, in accordance
situated. The notice must also be published once a week for four successive with the by-laws or general regulations of the corporation in that regard. So,
weeks in some newspaper of general circulation devoted to the publication of where there are statutory or other regulations as to the form and sufficiency
general news published at the place where the principal office of the of the notice, these must be followed. Thus, where such a notice was
corporation is established or located, and posted in some prominent place at required to be signed by the directors, a notice with the names of the
the works of the corporation if any such there be. If there be no newspaper directors signed by a clerk, was held insufficient. These cases and others
published at the place where the principal office of the corporation is proceed on the theory that where the manner of giving notice is prescribed
established or located, then such notice may be published in any newspaper by law every condition precedent must be strictly and literally complied with.
of general news in the Philippines. (Thompson on Corporations, Vol. 5, 3rd ed.)

It will be noted that section 40 is mandatory as regards publication, using the This view is shared by Justice Fisher. In his book "The Philippine Law on
word "must". As correctly stated by the trial court, the reason for the Stock Corporations" he says: "Not only must personal notice be given in one
mandatory provision is not only to assure notice to all subscribers, but also to of these manners, but the notice must also be published once a week, for
assure equality and uniformity in the assessment on stockholders. (14 C.J. four consecutive weeks, in some newspaper." (p. 110.).
639).
We find the citation of authorities made by the plaintiff and appellant
This rule finds support in authorities on corporation law, such as, Thompson inapplicable. In the case of Velasco vs. Poizat(37 Phil. 805), the corporation
on Corporations, Vol. 5, 3rd edition, pages 588-590, from which we make the involved was insolvent, in which case all unpaid stock subscriptions become
following quotation: payable on demand and are immediately recoverable in an action instituted
by the assignee. Said the court in that case:

649
. . . . it is now quite well settled that when the corporation becomes insolvent, In particular circumstances, as where it is given pursuant to a bona fide
with proceedings instituted by creditors to wind up and distribute its assets, compromise, or to set off a debt due from the corporation, a release,
no call or assessment is necessary before the institution of suits to collect supported by consideration, will be effectual as against dissenting
unpaid balance on subscription. stockholders and subsequent and existing creditors. A release which might
originally have been held invalid may be sustained after a considerable lapse
But when the corporation is a solvent concern, the rule is: of time. (18 C.J.S. 874).

It is again insisted that plaintiffs cannot recover because the suit was not In the present case, the release claimed by defendant and appellant does not
proceeded by a call or assessment against the defendant as a subscriber, fall under the exception above referred to, because it was not given pursuant
and that until this is done no right of action accrues. In a suit by a solvent to a bona fide compromise, or to set off a debt due from the corporation, and
going corporation to collect a subscription, and in certain suits provided by there was no consideration for it.
statute this would be true;. . . . . (Id.)
Another authority:
Going to the claim of defendant and appellant that Resolution No. 17 of 1946
released him from the obligation to pay for his unpaid subscription, the SEC. 850. Unanimous consent of stockholders necessary to release
authorities are generally agreed that in order to effect the release, there must subscriber. It may be asserted as the first rule under this proposition that,
be unanimous consent of the stockholders of the corporation. We quote after a valid subscription to the capital stock of a corporation has been made
some authorities: and accepted, there can be no cancellation or release from the obligation
without the consent of the corporation and all the stockholders; . . . . (2
Subject to certain exceptions, considered in subdivision (3) of this section, Thompson on Corporation, p. 186).
the general rule is that a valid and binding subscription for stock of a
corporation cannot be cancelled so as to release the subscriber from liability He states the reason for the rule as follows:
thereon without the consent of all the stockholders or subscribers.
Furthermore, a subscription cannot be cancelled by the company, even SEC. 855. Right to withdraw as against subscribers. A contract of
under a secret or collateral agreement for cancellation made with the subscription is, at least in the sense which creates as estoppel, a contract
subscriber at the time of the subscription, as against persons who among the several subscribers. For this reason no one of the subscribers
subsequently subscribed or purchased without notice of such agreement. (18 can withdraw from the contract without the consent of all the others, and
C.J.S. 874). thereby diminish, without the universal consent, the common fund in which all
have acquired an interest. . . . (2 Thompson on Corporations, p. 194.).
(3) Exceptions.
As already found by the trial court, the release attempted in Resolution No.
17 of 1946 was not valid for lack of a unanimous vote. If found that at least
seven stockholders were absent from the meeting when said resolution was
approved.

650
Defendant and appellant, however, contends that after dismissing the In conclusion we hold that under the Corporation Law, notice of call for
complaint for being premature, there was no necessity or reason for the trial payment for unpaid subscribed stock must be published, except when the
court to go further and say that defendant was not validly released from the corporation is insolvent, in which case, payment is immediately demandable.
payment for his unpaid subscription. It must be borne in mind, however, that We also rule that release from such payment must be made by all the
this was one of the principal issues involved in the case and the trial court stockholders.
was called upon to pass upon it, because unless so passed upon and deter-
mined, it might decisively affect the case on appeal. Supposing that on In view of the foregoing and finding no reversible error in the decision
appeal the appellate court decides that the call was valid, then it would be appealed, the same is hereby affirmed.
important to know whether or not in spite of the validity of the call, defendant
was nevertheless not liable because he had been validly released by a No pronouncement as to costs.
resolution of the corporation. If that question was not decided by the trial
court, and naturally was not touched upon in the appeal, then the appellate
Paras, C.J., Pablo, Bengzon, Padilla, Tuason, Reyes, Jugo, Bautista Angelo
court would have no occasion to pass upon it, and it might be necessary to
and Labrador, JJ., concur.
bring another action to determine the point, which means multiplicity of suits.
Moreover, the authority given to the courts to render judgments for
declaratory relief in order to determine the rights or duties of parties over a
certain transaction or under a certain written instrument, or to remove the
uncertainty or controversy over the same (Rule 66 of the Rules of Court),
justified the trial court in passing upon this question of release.

As regards the compensation of President claimed by defendant and


appellant, it is clear that he is not entitled to the same. The by-laws of the
company are silent as to the salary of the President. And, while resolutions of
the incorporators and stockholders (Exhibits G-1 and I-1) provide salaries for
the general manager, secretary-treasurer and other employees, there was no
provision for the salary of the President. On the other hand, other resolutions
(Exhibits H-1 and J-3) provide for per diems to be paid to the President and
the directors of each meeting attended, P10 for the President and P8 for
each director, which were later increased to P25 and P15, respectively. This
leads to the conclusions that the President and the board of directors were
expected to serve without salary, and that the per diems paid to them were
sufficient compensation for their services. Furthermore, for defendant's
several years of service as President and up to the filing of the action against
him, he never filed a claim for salary. He thought of claiming it only when this
suit was brought against him.

651
G.R. No. L-68097 January 16, 1986

EDWARD A. KELLER & CO., LTD., petitioner-appellant,


vs.
COB GROUP MARKETING, INC., JOSE E. BAX, FRANCISCO C. DE
CASTRO, JOHNNY DE LA FUENTE, SERGIO C. ORDOEZ, TRINIDAD C.
ORDOEZ, MAGNO C. ORDOEZ, ADORACION C. ORDOEZ, TOMAS
C. LORENZO, JR., LUIZ M. AGUILA-ADAO, MOISES P. ADAO,
ASUNCION MANAHAN and INTERMEDIATE APPELLATE
COURT, respondents-appellees.

Sycip, Salazar, Feliciano & Hernandez Law Office for petitioner.

Vicente G. Gregorio for private respondents.

Roberto P. Vega for respondent Asuncion Manahan.

AQUINO, C.J.:

This case is about the liability of a marketing distributor under its sales
agreements with the owner of the products. The petitioner presented its
evidence before Judges Castro Bartolome and Benipayo. Respondents
presented their evidence before Judge Tamayo who decided the case.

A review of the record shows that Judge Tamayo acted under a


misapprehension of facts and his findings are contradicted by the evidence.
The Appellate Court adopted the findings of Judge Tamayo. This is a case
where this Court is not bound by the factual findings of the Appellate Court.

652
(See Director of Lands vs. Zartiga, L-46068-69, September 30, 1982, 117 1. Increase of mortgaged collaterals to the full market value
SCRA 346, 355). (estimated by Edak at P90,000.00).

Edward A. Keller & Co., Ltd. appointed COB Group Marketing, Inc. as 2. Turn-over of receivables (estimated outstandings
exclusive distributor of its household products, Brite and Nuvan in Panay and P70,000.00 to P80,000.00).
Negros, as shown in the sales agreement dated March 14, 1970 (32-33 RA).
Under that agreement Keller sold on credit its products to COB Group 3. Turn-over of 4 (four) trucks for outright sale to Edak, to be
Marketing. credited against C.0.B.'s account.

As security for COB Group Marketing's credit purchases up to the amount of 4. Remaining 8 (eight) trucks to be assigned to Edak, C.O.B
P35,000, one Asuncion Manahan mortgaged her land to Keller. Manahan will continue operation with these 8 trucks. They win be
assumed solidarily with COB Group Marketing the faithful performance of all returned to COB after settlement of full account.
the terms and conditions of the sales agreement (Exh. D).
5. C.O.B has to put up securities totalling P200,000.00.
In July, 1970 the parties executed a second sales agreement whereby COB P100,000.00 has to be liquidated within one year. The
Group Marketing's territory was extended to Northern and Southern Luzon. remaining P100,000.00 has to be settled within the second
As security for the credit purchases up to P25,000 of COB Group Marketing year.
for that area, Tomas C. Lorenzo, Jr. and his father Tomas, Sr. (now
deceased) executed a mortgage on their land in Nueva Ecija. Like Manahan, 6. Edak wig agree to allow C.O.B. to buy goods to the value
the Lorenzos were solidarily liable with COB Group Marketing for its of the difference between P200,000.00 and their
obligations under the sales agreement (Exh. E). outstandings, provided C.O.B. is in a position to put up
securities amounting to P200,000.00.
The credit purchases of COB Group Marketing, which started on October 15,
1969, limited up to January 22, 1971. On May 8, the board of directors of Discussion held on May 8, 1971.
COB Group Marketing were apprised by Jose E. Bax the firm's president and
general manager, that the firm owed Keller about P179,000. Bax was
Twelve days later, or on May 20, COB Group Marketing, through Bax
authorized to negotiate with Keller for the settlement of his firm's liability
executed two second chattel mortgages over its 12 trucks (already
(Exh. 1, minutes of the meeting).
mortgaged to Northern Motors, Inc.) as security for its obligation to Keller
amounting to P179,185.16 as of April 30, 1971 (Exh. PP and QQ). However,
On the same day, May 8, Bax and R. Oefeli of Keller signed the conditions the second mortgages did not become effective because the first mortgagee,
for the settlement of COB Group Marketing's liability, Exhibit J, reproduced as Northern Motors, did not give its consent. But the second mortgages served
follows: the purpose of being admissions of the liability COB Group Marketing to
Keller.
This formalizes our conditions for the settlement of C.O.B.'s
account with Edward Keller Ltd. The stockholders of COB Group Marketing, Moises P. Adao and Tomas C.
Lorenzo, Jr., in a letter dated July 24, 1971 to Keller's counsel, proposed to

653
pay Keller P5,000 on November 30, 1971 and thereafter every thirtieth day of Keller sued on September 16, 1971 COB Group Marketing, its stockholders
the month for three years until COB Group Marketing's mortgage obligation and the mortgagors, Manahan and Lorenzo.
had been fully satisfied. They also proposed to substitute the Manahan
mortgage with a mortgage on Adao's lot at 72 7th Avenue, Cubao, Quezon COB Group Marketing, Trinidad C. Ordonez and Johnny de la Fuente were
City (Exh. L). declared in default (290 Record on Appeal).

These pieces of documentary evidence are sufficient to prove the liability of After trial, the lower court (1) dismissed the complaint; (2) ordered Keller to
COB Group Marketing and to justify the foreclosure of the two mortgages pay COB Group Marketing the sum of P100,596.72 with 6% interest a year
executed by Manahan and Lorenzo (Exh. D and E). from August 1, 1971 until the amount is fully paid: (3) ordered Keller to pay
P100,000 as moral damages to be allocated among the stockholders of COB
Section 22, Rule 130 of the Rules of Court provides that the act, declaration Group Marketing in proportion to their unpaid capital subscriptions; (4)
or omission of a party as to a relevant fact may be given in evidence against ordered the petitioner to pay Manahan P20,000 as moral damages; (5)
him "as admissions of a party". ordered the petitioner to pay P20,000 as attomey's fees to be divided among
the lawyers of all the answering defendants and to pay the costs of the suit;
The admissions of Bax are supported by the documentary evidence. It is (6) declared void the mortgages executed by Manahan and Lorenzo and the
noteworthy that all the invoices, with delivery receipts, were presented in cancellation of the annotation of said mortgages on the Torrens titles thereof,
evidence by Keller, Exhibits KK-1 to KK-277-a and N to N-149-a, together and (7) dismissed Manahan's cross-claim for lack of merit.
with a tabulation thereof, Exhibit KK, covering the period from October 15,
1969 to January 22, 1971. Victor A. Mayo, Keller's finance manager, The petitioner appealed. The Appellate Court affirmed said judgment except
submitted a statement of account showing that COB Group Marketing owed the award of P20,000 as moral damages which it eliminated. The petitioner
Keller P184,509.60 as of July 31, 1971 (Exh. JJ). That amount is reflected in appealed to this Court.
the customer's ledger, Exhibit M.
Bax and the other respondents quoted the six assignments of error made by
On the other hand, Bax although not an accountant, presented his own the petitioner in the Appellate Court, not the four assignments of error in its
reconciliation statements wherein he showed that COB Group Marketing brief herein. Manahan did not file any appellee's brief.
overpaid Keller P100,596.72 (Exh. 7 and 8). He claimed overpayment
although in his answer he did not allege at all that there was an overpayment We find that the lower courts erred in nullifying the admissions of liability
to Keller. made in 1971 by Bax as president and general manager of COB Group
Marketing and in giving credence to the alleged overpayment computed by
The statement of the Appellate Court that COB Group Marketing alleged in Bax .
its answer that it overpaid Keller P100,596.72 is manifestly erroneous first,
because COB Group Marketing did not file any answer, having been The lower courts not only allowed Bax to nullify his admissions as to the
declared in default, and second, because Bax and the other stockholders, liability of COB Group Marketing but they also erroneously rendered
who filed an answer, did not allege any overpayment. As already stated, even judgment in its favor in the amount of its supposed overpayment in the sum
before they filed their answer, Bax admitted that COB Group Marketing owed of P100,596.72 (Exh. 8-A), in spite of the fact that COB Group Marketing was
Keller around P179,000 (Exh. 1).

654
declared in default and did not file any counterclaim for the supposed COB Group marketing, Inc. is ordered to pay Edward A. Keller & Co., Ltd. the
overpayment. sum of P182,994.60 with 12% interest per annum from August 1, 1971 up to
the date of payment plus P20,000 as attorney's fees.
The lower courts harped on Keller's alleged failure to thresh out with
representatives of COB Group Marketing their "diverse statements of credits Asuncion Manahan and Tomas C. Lorenzo, Jr. are ordered to pay solidarity
and payments". This contention has no factual basis. In Exhibit J, quoted with COB Group Marketing the sums of P35,000 and P25,000, respectively.
above, it is stated by Bax and Keller's Oefeli that "discussion (was) held on
May 8, 1971." The following respondents are solidarity liable with COB Group Marketing up
to the amounts of their unpaid subscription to be applied to the company's
That means that there was a conference on the COB Group Marketing's liability herein: Jose E. Bax P36,000; Francisco C. de Castro, P36,000;
liability. Bax in that discussion did not present his reconciliation statements to Johnny de la Fuente, P12,000; Sergio C. Ordonez, P12,000; Trinidad C.
show overpayment. His Exhibits 7 and 8 were an afterthought. He presented Ordonez, P3,000; Magno C. Ordonez, P3,000; Adoracion C. Ordonez
them long after the case was filed. The petitioner regards them as P3,000; Tomas C. Lorenzo, Jr., P3,000 and Luz M. Aguilar-Adao, P6,000.
"fabricated" (p. 28, Appellant's Brief).
If after ninety (90) days from notice of the finality of the judgment in this case
Bax admitted that Keller sent his company monthly statements of accounts the judgment against COB Group Marketing has not been satisfied fully, then
(20-21 tsn, September 2, 1976) but he could not produce any formal protest the mortgages executed by Manahan and Lorenzo should be foreclosed and
against the supposed inaccuracy of the said statements (22). He lamely the proceeds of the sales applied to the obligation of COB Group Marketing.
explained that he would have to dig up his company's records for the formal Said mortgage obligations should bear six percent legal interest per
protest (23-24). He did not make any written demand for reconciliation of annum after the expiration of the said 90-day period. Costs against the
accounts (27-28). private respondents.

As to the liability of the stockholders, it is settled that a stockholder is SO ORDERED.


personally liable for the financial obligations of a corporation to the extent of
his unpaid subscription (Vda. de Salvatierra vs. Garlitos 103 Phil. 757, 763; Concepcion, Jr. (Chairman), Escolin, Cuevas and Alampay, JJ., concur.
18 CJs 1311-2).
Abad Santos, J., took no part.
While the evidence shows that the amount due from COB Group Marketing is
P184,509.60 as of July 31, 1971 or P186,354.70 as of August 31, 1971 (Exh.
JJ), the amount prayed for in Keller's complaint is P182,994.60 as of July 31,
1971 (18-19 Record on Appeal). This latter amount should be the one
awarded to Keller because a judgment entered against a party in default
cannot exceed the amount prayed for (Sec. 5, Rule 18, Rules of Court).

WHEREFORE, the decisions of the trial court and the Appellate Court are
reversed and set aside.

655
G.R. No. L-35961 December 2, 1932

ROMANA MIRANDA, in her capacity as judicial administratrix of the


intestate estate of her deceased father, Alberto Miranda, plaintiff-
appellant,
vs.
THE TARLAC RICE MILL CO., INC., defendant-appellee.

Fausto and Ramos for appellant.


Enrique Maglanoc for appellee.

VICKERS, J.:

656
This is an appeal by the plaintiff from a decision of Judge A. M. Recto of the contrajo una deuda de P10,000 a los Sres. Mariano Tablante y
Court of First Instance of Tarlac, dismissing the case without a special finding Carmen Gueco, de Angeles, Pampanga, como se acredita por la
as to costs. escritura de prestamo hipotecario otorgada al efecto, que tambien se
adjunta a la presente, como Exhibits C y C-1;
The case was tried on the following agreed statement of facts:
5. Que la demandada no ha pagado en ningun tiempo ni el capital, ni
Comparecen las partes la demandante, asistida de su infrascrito los intereses, del prestamo arriba mencionado, motivo por el cual el
abogado, y la demandada, por medio de su presidente y abogado referido Don Alberto Miranda hubo de entrar en arreglo amistoso con
que subscriben y para abreviar la vista de esta causa y sin los acreedores, al expirar el plazo convenido para el pago,
perjuicio de practicarse pruebas adicionales sobre hechos en los satisfaciendo dicho prestamo y sus intereses devengados, segun
que las partes no estan de acuerdo, respetuosamente someten, consta en la carta de pago extendida al efecto, que se hace parte
para la decision de esta causa, las siguientes estipulaciones: integrante del presente convenio como Anexo o Exhibt D;

1. Que la demandante Romana Miranda es la administradora 6. Que, a partir desde el ano 1928 hasta esta fecha, la demandada
judicial, debidamente nombrada, del Intestado del finado Don Alberto ha dejado de hacer negocios y operaciones de ninguna clase;
Miranda, Civil No. 3090, de este mismo Juzgado; y la entidad
demandada es una corporacion debidamente organizada de 7. Que, con excepcion del citado Don Alberto Miranda, ninguno de
acuerdo con las leyes en vigor en estas Islas, teniendo su domicilio las otras accionistas y directores de la corporacion demandada ha
legal, lo mismo que la demandante, en esta cabecera de Tarlac, pagado o se le ha hecho pagar, conforme los terminos de los
Provincia de Tarlac; contratos de subscripcion otorgados al efecto, el importe de sus
respectivas acciones, y a pesar de esta morosidad de los fereridos
2. Que, con fecha 8 de junio de 1926, el hoy difunto Don Alberto accionistas y directores, la corporacion demandada no ha dado,
Miranda de cuyo intestado es administradora judicial la aqui hasta la fecha, ningun paso tendente a compeler la efectividad de
demandante subscribo acciones de la corporacion demandada, las referidas acciones morosas.
otorgando al efecto un contrato de subscripcion, copia auntentica del
cual se une al presente y se hace parte integrante del mismo, como The only additional evidence presented was the testimony of Marciano
exhibit A; David, which is of no consequence in our view of the case.

3. Que, en relacion con el contrato de subcripcion Exhibit A, a que se The appellant makes the following assignment of errors:
contrae el parrafo que precede, Don Alberto Miranda otorgo luego
una escritura de poder a favor de la demandada, cuyo original se The trial court erred:
une asimismo al presente, haciendose parte integrante del mismo,
como Exhibit B; 1. In declaring that the defendant corporation did not violate the
terms of the power of attorney Exhibit B, for the plaintiff, when she
4. Que, por virtud de los documentos a que se contraen los dos obtained the loan Exhibit C;
parrafos inmediatamente anteriores la corporacion demandada

657
2. In declaring that "all responsibility originating in the execution by
the officers of the defendant corporation of the mortgage contract On or before January 21, 1929 2,500.00
Exhibit C has already ceased";

3. In pretending to base the decision in this case upon theories


On or before January 21, 1930 2,500.00
neither presented by the pleadings of the parties nor deduced from
the evidence produced by the parties;

4. In denying the motion for new trial of the plaintiff-appellant; and On July 10, 1926 Alberto Miranda by means of a public document "assigned"
mortgaged, or transferred in lieu of cash for the benefit and to the credit of
the Tarlac Rice Mill Company, Inc., a corporation to be organized and to exist
5. In not sentencing the defendant to pay the plaintiff the sum of
under and by virtue of the laws of the Philippine Islands", the parcel of land
P10,000, with interest thereon at P1,200 a year, from the year 1927
described in certificate No. 751 in the land records of the Province of Tarlac;
until paid, plus the sum of P1,500, which the principal had to pay in
and "to carry out the true intent, meaning, and purposes thereof I have
the form of a penal clause for the violation of the terms of the
hereby further voluntarily made, constituted, and appointed and by these
mortgage contract Exhibit C, aside from the legal interests of all
presents do make, constitute and appoint, either jointly, Evaristo Magbag,
these amounts from the presentation of the present complaint, and
duly elected President and Treasurer of said Company, Eusebio R. Cabrera
the costs of the suit.
and Marcos P. Puno, duly elected Vice-Presidents of the same company, or
anyone of the three named elected officers of the Tarlac Rice Mill Company,
It appears from the evidence that on June 8, 1926 Alberto Miranda executed Inc., jointly with C. M. Dizon to be my true and lawful attorney-in-fact, for me
a written contract whereby he subscribed for 100 shares of the capital stock and in my name, in my behalf to transfer, mortgage, convey or confirm or in
of a corporation to be organized under the laws of the Philippine Islands for any way convenient to them to any local or foreign bank, firm or individual in
the purpose of operating a rice mill in Tarlac, said corporation to be known as order to obtain, secure or solicit credit against my above described property
Tarlac Rice Mill Company, Inc., that the par value of each share was P100; in an amount not to exceed ten thousand pesos (P10,000), Philippine
and that Alberto Miranda obligated himself to pay to the treasurer of the currency, in accordance with the subscription contract voluntary executed by
corporation or its assign the sum of P10,000 as follows: me, for or to increase the capital of the said Tarlac Rice Mill Company, Inc., in
order to carry out the purposes for which such firm is to be organized.

On or before September 21, 1926 P1,000.00 That for the foregoing purposes, I hereby transfer my right and
interest in the said described properties, and by these presents do
hereby give and grant unto my said attorneys-in fact full power and
On or before January 21, 1927 2,000.00 authority to do and perform all and every act and thing whatsoever
requisite and necessary to be done in all about the premises as fully
to all intents and purposes as I might or could do if personally
present with full power of substitution or revocation, hereby ratify and
On or before January 21, 1928 2,000.00 confirm all that my said attorneys-in-fact, anyone or all of the three
Evaristo Magbag, Eusebio R. Cabrera, and Marcos P. Puno, jointly

658
with C. M. Dizon or their substitutes shall lawfully do or cause to be ceased to do business, and it had taken no steps to compel the other
done by virtue of these presents. stockholders to pay for the shares for which they had subscribed.

On February 19, 1927 the president and vice-president of the Tarlac Rice Mill No question as to the validity of subscription agreement is raised, and no
Company, Inc., and C. M. Dizon, acting on behalf of said corporation and fraud on the part of the officers of the corporation is alleged or proved. We
Alberto Miranda, borrowed P10,000 from Mariano Tablante, and agreed to shall therefore confine ourselves to the issues raised by the pleading.
repay said sum on or before February 19, 1928, with interest at 12 per cent
per annum, and to pay a further sum of 25 per cent of the principal for It is true that when the property was mortgaged on February 19, 1927 the
attorney's fees and expenses of collection in case the promissory note amount due from Alberto Miranda in accordance with the subscription
should not be paid at maturity. Marcos Puno, Evaristo Magbag, and Dizon & agreement was only P3,000, and it is likewise true that it does not appear
Co., Inc., jointly and severally guaranteed the payment of this sum; and the from the evidence that any call was issued by the directors for the payment
president and vice-president of the Tarlac Rice Mill Company, Inc., and C. M. of any subscriptions.
Dizon as attorneys-in-fact of Alberto Miranda mortgaged to Mariano Tablante
the aforementioned parcel of land to secure the payment of said promissory The fact that Alberto Miranda agreed on June 8, 1926 to pay the amount of
note. his subscription installments on certain fixed dates did not, of course prevent
him from authorizing the officers of the corporation as his attorneys-in-fact to
The sum of P10,000 obtained from Mariano Tablante was retained by the pay his subscription prior to the dates fixed in the subscription agreement.
corporation. When the promissory note became due, Alberto Miranda Great stress is laid by the appellant upon the fact that in one paragraph of
arranged for an extension of time in which to pay it, and on July 19, 1929 he the power of attorney it is stated that the attorneys-in-fact of Alberto Miranda
sold the aforementioned parcel of land under pacto de retro to Vicente are authorized to mortgage or convey the property in any way convenient to
Panlilio for P10,000, and paid Mariano Tablante. them in the amount not to exceed P10,000 in accordance with the
subscription contract, but the phrase "in accordance with the subscription
According to an allegation in the complaint, Alberto Miranda died on May 24, contract" is followed by the following words "for or to increase the capital of
1930. the said Tarlac Rice Mill Company, Inc., in order to carry out the purposes for
which said firm is to be organized." Under the circumstances, it seems to us
It is agreed that the defendant corporation ceased to do business from the that it would be a strained construction of the power of attorney, taking into
year 1928, and that the other stockholders have not paid for their shares in consideration the whole document, to hold that the officers of the corporation
accordance with their subscription agreement, and that no action has been acting as attorneys-in-fact- of Alberto Miranda were authorized to mortgage
taken by the corporation to require them to do so. or convey the land for only the amount then due from Alberto Miranda in
accordance with the subscription agreement. It can hardly be contended that
The principal contention of the appellant is that the officers of the corporation the power of attorney contemplated that the property should be mortgaged
violated the terms of the power of attorney in mortgaging the land on three times, that is, each time that an instalment became due. We are
February 19, 1927 for P10,000, because the only sum then due and payable inclined to the view that it was the intention of the parties that the property
by Alberto Miranda to the corporation was P3,000, and that when the should be mortgaged immediately for a sum not to exceed P10,000, not only
remaining instalments of the stock subscription became due, Alberto Miranda for the purpose of paying the subscription agreement of Alberto Miranda, but
was under no obligation to pay them, because the corporation had already also for the purpose, as stated in the power of attorney, of increasing the
capital of the corporation, not the capital stock, in order to carry out the

659
purposes for which it was to be organized. This view of the matter is No call is necessary when a subscription is payable, not upon call or
confirmed by the subsequent conduct of the parties. Although the corporation demand by the directors or stockholders, but immediately, or on
retained the full amount of the loan obtained from Mariano Tablante, and specified day, or on or before a specified day, or when it is payable in
Alberto Miranda had to pay that obligation, he never sought, so far as the installments at specified times. In such cases it is the duty of the
record shows, to recover from the corporation any part of the sum of subscriber to pay the subscription or instalment thereof as soon as it
P10,000. As we have already stated, the mortgage was executed on is due, without any call or demand, and, if he fails to do so, an action
February 19, 1927; it was satisfied by Alberto Miranda on July 19, 1929, and may be brought at any time. (Fletcher: Cyclopedia of the Law of
he lived until May 24, 1930. It does not appear that he ever sought to evade Private Corporations, vol. 2, page 1509.)
the satisfaction of the mortgage by alleging that his attorneys-in-fact
exceeded their authority in mortgaging the property on February 19, 1927 for When this action was filed on September 2, 1930, the last of the instalments
P10,000. On the contrary he repaid to Mariano Tablante the amount which had already become payable in accordance with the subscription agreement.
the officers of the corporation had borrowed. The fact that he at no time it must be borne in mind that this is not an action by the corporation to
sought to recover from the corporation any part of the sum borrowed by the recover on a subscription agreement, but an action by the administratrix of a
officers of the corporation in his name certainly tends to show that he stockholder to recover what was paid in to the corporation by the stockholder.
acquiesced in the action taken by them. The phrase "in accordance with the It does not appear from the evidence whether or not the corporation has any
subscription contract" found in the power of attorney probably was intended debts. Neither the fact that the corporation has ceased to do business nor the
to mean "in pursuance of the subscription agreement", that is, it referred to fact that the other stockholders have not been required to pay for their shares
the obligation, and had no particular reference to the dates when the different in accordance with their subscription agreement justifies us in ordering the
installments were to be paid. corporation to return to the plaintiff the amount paid in by Alberto Miranda. If
the directors have failed to perform their duty with respect to the other
Section 38 of the Corporation Law provides that the board of directors of stockholders, the law provides a remedy therefor.
every corporation may at any time declare due and payable to the
corporation unpaid subscriptions to the capital stock and may collect the In the case of Velasco vs. Poizat (37 Phil., 802), this court held that a stock
same with interest accrued thereon or such percentage of said unpaid subscription is a contract between the corporation and the subscriber, and
subscriptions as it may deem necessary. In his work, "The Philippine Law of courts will enforce it for or against either; that a corporation has no legal
Stock Corporations", page 97, Justice Fisher expresses the opinion that this capacity to release a subscriber to its capital stock from the obligation to pay
power of the directors is absolute and cannot be limited by the subscription for his shares, and that any agreement to this effect is invalid.
contract, but this does not mean that the directors may not rely on the
subscription contract if they see fit to do so. In the case at bar it is not contended that Alberto Miranda cancelled his
subscription agreement, or that the corporation attempted to release him
therefrom.

For the foregoing reasons, the decision appealed from is affirmed, with the
costs against the appellant.

Street, Malcolm, Ostrand and Imperial, JJ., concur.

660
was notified by the secretary of the corporation of a resolution adopted by the
board of directors of the corporation on the preceding day, declaring the
unpaid subscriptions to the capital stock of the corporation to have become
due and payable on the following May 31st at the office thereof, the payment
to be made to the treasurer, and stating that all such shares as may have not
been paid then, with the accrued interest up to that date, will be declared
delinquent, advertised for sale at public auction, and sold on the following
June 16th, for the purpose of paying up the amount of the subscription and
accrued interest, with the expenses of the advertisement and sale, unless
said payment was made before. The proper advertisement having been
published, as announced in the aforesaid notice, the plaintiff filed a complaint
in the Court of First Instance of Cebu on May 5th of the same year against
the said corporation, wherein, after relating the above-mentioned facts, he
prayed for a judgment in his favor, decreeing that, in prescribing another
method of paying the subscription to the capital stock different from that
provided in article 46 of its by-laws, in declaring the aforesaid 450 shares
delinquent, and in directing the sale thereof, as advertised, the corporation
had exceeded its executive authority, and as a consequence thereof he
asked that a writ of injunction be issued against the said defendant, enjoining
it from taking any further action of whatever nature in connection with the
acts complained of and that it pay the costs of this suit.

G.R. No. L-19893 March 31, 1923


The plaintiff alleged as the grounds of his petition: (1) That, according to
aforesaid article 46 of the by-law of the corporation, which was inserted in the
ARNALDO F. DE SILVA, plaintiff-appellant, complaint, all the shares subscribed to by the incorporation that were not
vs. paid for at the time of the incorporation, shall be paid out of the 70 per cent of
ABOITIZ & COMPANY, INC., defendant-appellee. the profit obtained, the same to be distributed among the subscribers, who
shall not receive any dividend until said shares were paid in full; (2) that in
Del Rosario and Del Rosario and Andres Jayme for appellant. declaring the plaintiff's unpaid subscription to the capital stock to have
Rodriguez and Zacarias for appellee. become due and payable on May 31st, and in publishing the aforesaid notice
declaring his unpaid shares delinquent, the defendant corporation has
ARAULLO, C. J.: violated the aforesaid article, which prescribes an operative method of paying
for the shares continuously until their full amortization, thus violating and
The plaintiff subscribed for 650 shares of stock of the defendant corporation disregarding a right of the plaintiff vested under the said by-laws; (3) that the
of the value of P500 each, of which he has paid only the total value of 200 aforesaid acts of the defendant corporation were in excess of its powers and
shares, there remaining 450 shares unpaid, for which he was indebted to the executive authority and the plaintiff had no other plain, speedy and adequate
corporation in the sum of P225,000, the value thereof. On April 22, 1922, he remedy in the ordinary course of law than that prayed for in the said

661
complaint, to prevent the defendant from taking any further action in laws; ten per cent (10%) for the general manager; ten per cent (10%)
connection with the sale and alienation of the said shares. for the reserve fund, and seventy per cent (70%) for the
shareholders in equal parts; Provided, however, That from this
A preliminary injunction having been issued against the defendant, as prayed seventy per cent dividend the Board of Directors may deduct such
for by the plaintiff, upon the giving of the proper bond, and the defendant amount as it may deem fit for the payment of the unpaid subscription
having been summoned, the latter filed a demurrer to the complaint on the to the capital stock and not pay any dividend to the holders of the
ground that the facts alleged therein did not constitute a cause of action, and said unpaid shares until they are fully paid; Provided, further, That
that even supposing the plaintiff to have any lawful claim against the when all the shares have been paid in full as provided in the
defendant corporation, the special remedy applied for by the plaintiff was not preceding paragraph, the Board of Directors may also deduct such
the most adequate and speedy. amount as it may deem fit for the creation of an emergency special
fund, or extraordinary reserve fund when in its judgment the same
Hearing having been had the court below by an order dated September 21, may convenient for the development of the business of the
1922, sustained the aforesaid demurrer on the first ground, giving the plaintiff corporation or for meeting any such contingencies as may arise from
five days within which to amend his complaint, but the said period having its operation, whenever the distributable dividend is found, after the
elapsed without the plaintiff having amended his complaint, upon motion of foregoing deduction, to be not less than ten per cent (10%) of the
the defendant, that court, by an order dated the 2d of the following month of paid up capital stock.
October, dismissed the complaint and ordered the dissolution of the
preliminary injunction previously issued, with costs, to which orders the No dividend shall be declared or paid, except when there remains a
plaintiff excepted, asking at the same time for the annulment thereof and a net profit after the payment of all the expenses incurred, or
new hearing, which motion was denied by the lower court. To that ruling the allowances made, by the corporation to carry out the operation of its
plaintiff also excepted, and brought the case to this court by the proper bill of business; so that no such dividend may be declared as may affect
exceptions. the capital of the corporation.

Assuming the truth of the facts alleged in the complaint filed against the As will be seen from the context of the said article, its first part specifies the
herein defendant, as the filling of a demurrer to a complaint is made on that manner in which the net profit from the annual liquidation should be
assumption, the question to be decided reduces itself to determining whether distributed, fixing a certain per cent for the board of directors; another for the
or not, under the provision of article 46 of the by-laws of the defendant general manager; another for the reserve fund, and the remaining 70 per
corporation, the latter may declare the unpaid shares delinquent, or collect cent to be distributed in equal parts among the shareholders. But it
their value by another method different from that prescribed in the aforecited authorizes or empowers the board of directors to collect the value of the
article. shares subscribed to and not fully paid by deducting from the 70 per cent,
distributable in equal parts among the shareholders, such amount as may be
Said article reads thus: deemed convenient, to be applied on the payment of the said shares, and
not to pay the subscriber until the same are fully paid up. In no other way can
the words "Provided, however, that from this seventy per cent dividend the
ART. 46. The net profit resulting from the annual liquidation shall be
board of directors may deduct such amount as it may deem fit for the
distributed as follows: Ten per cent (10%) for the Board of Directors
payment, etc." And this is so clear that in that same article the board of
and in the manner prescribed in article twenty-six (26) of these by-
directors is also authorized to create a special emergency fund or

662
extraordinary reserve fund, when, in its judgment, and in case all the shares subscription, together with accrued interest and costs and expenses
subscribed to have been fully paid, the same is convenient for the incurred."
development of the business of the corporation or for meeting any such
contingencies as my arise from its operation, applying said 70 per cent of the In the instant case the board of directors of the defendant corporation elected
profit on the payment of the shares that may have not been fully paid, to avail itself of the first of said two remedies, and, complying strictly with the
provided that the distributable dividend remaining after the deduction to be provisions of sections 37 to 49, inclusive, of the aforesaid Corporation Law,
made for the creation of the said special emergency fund or extraordinary which is binding upon it and its stockholders. it being an artificial entity
reserve fund is not less than 10 per cent of the capital actually paid. So that it created by virtue of that same law (sec. 2), the board of directors made use
is discretionary on the part of the board of directors to do whatever is of the discretionary power granted to it by that law and declared that payment
provided in the said article relative to the application of a part of the 70 per of plaintiff's subscription to 450 shares which had not been paid by him was
cent of the profit distributable in equal parts on the payment of the shares due, and that said shares were delinquent, and performed all the other acts
subscribed to and not fully paid, and to the creation of a special emergency subsequent to said declaration that are mentioned in the complaint, as it did
fund or extraordinary reserve fund; and the fact itself that said special fund not deem it advantageous to the corporation to apply on the payment of said
may not be created when the dividend appearing to be distributable, after shares, as was authorized by the by-law, a part of the profit that was, or
deducting from the said 70 per cent the amount to be applied on the payment might have been realized, and was distributable among the stockholders in
of the unpaid subscription, is less than 10 per cent of the capital actually equal parts, as to the existence of which profit no allegation is made in the
paid, shows that it is the board of directors and not the delinquent subscriber complaint, or to enforce payment of such shares by bringing in court the
that may and must judge and decide whether or not such value must be paid proper action against the debtor or delinquent stockholders. It is, however,
out of a part of the 70 per cent of the profit distributable in equal parts among alleged by the appellant that the by-law of the corporation being of the nature
the shareholders, as provided in the first part of the said article. It lies of a contract between it and its stockholders or members, and article 46 of
therefore, within the discretion of the board of directors to make use of such the by-laws of the said corporation providing an operative method for the
authority. payment of stock subscriptions continuously until the full amortization
thereof, application cannot be made in the present case of the provisions
If the board of directors does not wish to make, or does not make, use of said above cited of the Corporation Law for the purpose contemplated by the
authority it has two other remedies for accomplishing the same purpose. As defendant, as the provision of said article must prevail against that law.
was said by this court in the case of Velasco vs. Poizat (37 Phil., 802):
Admitting that the provision of article 46 of the said by-laws maybe regarded
The first and most special remedy given by the statute consists in as a contract between the defendant corporation and its stockholders , yet as
permitting the corporation to put the unpaid stock for sale and it is only to the board of directors of the corporation that said articles gives
dispose of it for the account of the delinquent subscriber. In this case the authority or right to apply on the payment of unpaid subscriptions such
the provisions of sections 38 to 48, inclusive, of the Corporation Law amount of the 70 per cent of the profit distributable among the shareholders
are applicable and must be followed. The other remedy is by action in equal parts as may be deemed fit, it cannot be maintained that the said
in court concerning which we find in section 49 the following article has prescribe an operative method for the payment of said
provision: subscription continuously until their full amortization, or, what would be the
same thing, that said article has prescribe that sole and exclusive method for
"Nothing in this Act prevent the directors from collecting, by action in that purpose, for, in the first place, the adoption of that method for the
any court of proper jurisdiction, the amount due on any unpaid purpose of collecting the value of subscriptions due and unpaid lies,

663
according to said article, within the discretion of the board of directions, that
is, it is subject to this condition, and this can in no way be reconciled with the
idea of method, which implies something fixed as a rule or permanent
standard, and not variable at the will of somebody and according to the
circumstances; and, in the second place, in connection with the provision of G.R. No. 51765 March 3, 1997
the said article relative to the aforesaid discretionary power of the board of
directors to adopt that method, there is also the discretionary power granted
REPUBLIC PLANTERS BANK, petitioner,
the same board of directors to avail itself, for the same purpose, to either of
vs.
the two remedies prescribed in sections 38 to 49, inclusive, of the aforecited
HON. ENRIQUE A. AGANA, SR., as Presiding Judge, Court of First
Corporation Law.
Instance of Rizal, Branch XXVIII, Pasay City, ROBES-FRANCISCO
REALTY & DEVELOPMENT CORPORATION and ADALIA F.
In the instant case, the defendant corporation, through its board of directors, ROBES, respondents.
made use of its discretionary power, taking advantage of the first of the two
remedies provided by the aforesaid law. On the other hand, the plaintiff has
no right whatsoever under the provision of the above cited article 46 of the
said by-laws to prevent the board of directors from following, for that
purpose, any other method than that mentioned in the said article, for the HERMOSISIMA, JR., J.:
very reason that the same does not give the stockholders any right in
connection with the determination of the question whether or not there should This is a petition for certiorari seeking the annulment of the Decision 1 of the
be deducted from the 70 per cent of the profit distributable among the then Court of First Instance of Rizal 2 for having been rendered in grave
stockholders such amount as may be deemed fit for the payment of abuse of discretion. Private respondents Robes-Francisco Realty and
subscriptions due and unpaid. Therefore, it is evident that the defendant Development Corporation (hereafter, "the Corporation") and Adalia F. Robes
corporation has not violated, nor disregarded any right of the plaintiff filed in the court a quo, an action for specific performance to compel
recognized by the said by-laws, nor exceeded its authority in the discharge of petitioner to redeem 800 preferred shares of stock with a face value of
its executive functions, nor abused its discretion when it performed the acts P8,000.00 and to pay 1% quarterly interest thereon as quarterly dividend
mentioned in the complaint as grounds thereof, and, consequently, the facts owing them under the terms and conditions of the certificates of stock.
therein alleged do not constitute a cause of action.
The court a quo rendered judgment in favor of private respondents; hence,
For the foregoing, the orders appealed from are affirmed, with the costs of this instant petition.
both instances against the appellant. So ordered.
Herein parties debate only legal issues, no issues of fact having been raised
Street, Malcolm, Avancea, Ostrand, Johns, and Romualdez, JJ., concur. by them in the court a quo. For ready reference, however, the following
narration of pertinent transactions and events is in order:

On September 18, 1961, private respondent Corporation secured a loan from


petitioner in the amount of P120,000.00. As part of the proceeds of the loan,
preferred shares of stocks were issued to private respondent Corporation,

664
through its officers then, private respondent Adalia F. Robes and one Carlos Petitioner's Motion to Dismiss was denied by the trial court in an Order dated
F. Robes. In other words, instead of giving the legal tender totaling to the full March 16, 1979. 4 Petitioner then filed its Answer on May 2,
amount of the loan, which is P120,000.00, petitioner lent such amount 1979. 5 Thereafter, the trial court gave the parties ten (10) days from July 30,
partially in the form of money and partially in the form of stock certificates 1979 to submit their respective memoranda after the submission of which the
numbered 3204 and 3205, each for 400 shares with a par value of P10.00 case would be deemed submitted for resolution. 6
per share, or for P4,000.00 each, for a total of P8,000.00. Said stock
certificates were in the name of private respondent Adalia F. Robes and On September 7, 1979, the trial court rendered the herein assailed decision
Carlos F. Robes, who subsequently, however, endorsed his shares in favor of in favor of private respondents. In ordering petitioner to pay private
Adalia F. Robes. respondents the face value of the stock certificates as redemption price, plus
1% quarterly interest thereon until full payment, the trial court ruled:
Said certificates of stock bear the following terms and conditions:
There being no issue of fact raised by either of the parties
The Preferred Stock shall have the following rights, who filed their respective memoranda delineating their
preferences, qualifications and limitations, to wit: respective contentions, a judgment on the pleadings,
conformably with an earlier order of the Court, appears to be
1. Of the right to receive a quarterly dividend of One Per in order.
Centum (1%), cumulative and participating.
From a further perusal of the pleadings, it appears that the
xxx xxx xxx provision of the stock certificates in question to the effect that
the plaintiffs shall have the right to receive a quarterly
2. That such preferred shares may be redeemed, by the dividend of One Per Centum (1%), cumulative and
system of drawing lots, at any time after two (2) years from participating, clearly and unequivocably [sic] indicates that
the date of issue at the option of the Corporation. . . . the same are "interest bearing stocks" which are stocks
issued by a corporation under an agreement to pay a certain
rate of interest thereon (5 Thompson, Sec. 3439). As such,
On January 31, 1979, private respondents proceeded against petitioner and
plaintiffs become entitled to the payment thereof as a matter
filed a Complaint anchored on private respondents' alleged rights to collect
of right without necessity of a prior declaration of dividend.
dividends under the preferred shares in question and to have petitioner
redeem the same under the terms and conditions of the stock certificates.
Private respondents attached to their complaint, a letter-demand dated On the question of the redemption by the defendant of said
January 5, 1979 which, significantly, was not formally offered in evidence. preferred shares of stock, the very wordings of the terms and
conditions in said stock certificates clearly allows the same.
Petitioner filed a Motion to Dismiss 3 private respondents' Complaint on the
following grounds: (1) that the trial court had no jurisdiction over the subject- To allow the herein defendant not to redeem said preferred
matter of the action; (2) that the action was unenforceable under substantive shares of stock and/or pay the interest due thereon despite
law; and (3) that the action was barred by the statute of limitations and/or the clear import of said provisions by the mere invocation of
laches. alleged Central Bank Circulars prohibiting the same is

665
tantamount to an impairment of the obligation of contracts REDEEMING ITS PREFERRED SHARES AND FROM
enshrined in no less than the fundamental law itself. PAYING DIVIDENDS THEREON . . . .

Moreover, the herein defendant is considered in estoppel D. THE TRIAL COURT ERRED IN NOT HOLDING THAT
from taking shelter behind a General Banking Act provision THE COMPLAINT DOES NOT STATE A CAUSE OF
to the effect that it cannot buy its own shares of stocks ACTION.
considering that the very terms and conditions in said stock
certificates allowing their redemption are its own handiwork. E. THE TRIAL COURT ERRED IN NOT HOLDING THAT
THE CLAIM OF RESPONDENT ADALIA F. ROBES IS
As to the claim by the defendant that plaintiffs' cause of BARRED BY PRESCRIPTION OR LACHES. 8
action is barred by prescription, suffice it to state that the
running of the prescriptive period was considered interrupted The petition is meritorious.
by the written extrajudicial demands made by the plaintiffs
from the defendant. 7 Before passing upon the merits of this petition, it may be pertinent to provide
an overview on the nature of preferred shares and the redemption thereof,
Aggrieved by the decision of the trial court, petitioner elevated the case considering that these issues lie at the heart of the dispute.
before us essentially on pure questions of law. Petitioner's statement of the
issues that it submits for us to adjudicate upon, is as follows: A preferred share of stock, on one hand, is one which entitles the holder
thereof to certain preferences over the holders of common stock. The
A. RESPONDENT JUDGE COMMITTED A GRAVE ABUSE preferences are designed to induce persons to subscribe for shares of a
OF DISCRETION AMOUNTING TO LACK OR EXCESS OF corporation. 9 Preferred shares take a multiplicity of forms. The most common
JURISDICTION IN ORDERING PETITIONER TO PAY forms may be classified into two: (1) preferred shares as to assets; and (2)
RESPONDENT ADALIA F. ROBES THE AMOUNT OF preferred shares as to dividends. The former is a share which gives the
P8213.69 AS INTERESTS FROM 1961 TO 1979 ON HER holder thereof preference in the distribution of the assets of the corporation in
PREFERRED SHARES. case of liquidation; 10 the latter is a share the holder of which is entitled to
receive dividends on said share to the extent agreed upon before any
B. RESPONDENT JUDGE COMMITTED A GRAVE ABUSE dividends at all are paid to the holders of common stock. 11 There is no
OF DISCRETION AMOUNTING TO LACK OR EXCESS OF guaranty, however, that the share will receive any dividends. Under the old
JURISDICTION IN ORDERING PETITIONER TO REDEEM Corporation Law in force at the time the contract between the petitioner and
RESPONDENT ADALIA F. ROBES' PREFERRED SHARES the private respondents was entered into, it was provided that "no
FOR P8,000.00. corporation shall make or declare any dividend except from the surplus
profits arising from its business, or distribute its capital stock or property
C. RESPONDENT JUDGE COMMITTED A GRAVE ABUSE other than actual profits among its members or stockholders until after the
OF DISCRETION AMOUNTING TO LACK OR EXCESS OF payment of its debts and the termination of its existence by limitation or
JURISDICTION IN DISREGARDING THE ORDER OF THE lawful dissolution." 12 Similarly, the present Corporation Code 13 provides that
CENTRAL BANK TO PETITIONER TO DESIST FROM the board of directors of a stock corporation may declare dividends only out

666
of unrestricted retained earnings. 14 The Code, in Section 43, adopting the On the question of the redemption by the defendant of said
change made in accounting terminology, substituted the phrase "unrestricted preferred shares of stock, the very wordings of the terms and
retained earnings," which may be a more precise term, in place of "surplus conditions in said stock certificates clearly allows the
profits arising from its business" in the former law. Thus, the declaration of same. 21
dividends is dependent upon the availability of surplus profit or unrestricted
retained earnings, as the case may be. Preferences granted to preferred What respondent judge failed to recognize was that while the stock
stockholders, moreover, do not give them a lien upon the property of the certificate does allow redemption, the option to do so was clearly
corporation nor make them creditors of the corporation, the right of the vested in the petitioner bank. The redemption therefore is clearly the
former being always subordinate to the latter. Dividends are thus payable type known as "optional". Thus, except as otherwise provided in the
only when there are profits earned by the corporation and as a general rule, stock certificate, the redemption rests entirely with the corporation
even if there are existing profits, the board of directors has the discretion to and the stockholder is without right to either compel or refuse the
determine whether or not dividends are to be declared. 15 Shareholders, both redemption of its stock. 22 Furthermore, the terms and conditions set
common and preferred, are considered risk takers who invest capital in the forth therein use the word "may". It is a settled doctrine in statutory
business and who can look only to what is left after corporate debts and construction that the word "may" denotes discretion, and cannot be
liabilities are fully paid. 16 construed as having a mandatory effect. We fail to see how
respondent judge can ignore what, in his words, are the "very
Redeemable shares, on the other hand, are shares usually preferred, which wordings of the terms and conditions in said stock certificates" and
by their terms are redeemable at a fixed date, or at the option of either construe what is clearly a mere option to be his legal basis for
issuing corporation, or the stockholder, or both at a certain redemption compelling the petitioner to redeem the shares in question.
price.17 A redemption by the corporation of its stock is, in a sense, a
repurchase of it for cancellation. 18 The present Code allows redemption of The redemption of said shares cannot be allowed. As pointed out by the
shares even if there are no unrestricted retained earnings on the books of the petitioner, the Central Bank made a finding that said petitioner has been
corporation. This is a new provision which in effect qualifies the general rule suffering from chronic reserve deficiency, 23 and that such finding resulted in
that the corporation cannot purchase its own shares except out of current a directive, issued on January 31, 1973 by then Gov. G.S. Licaros of the
retained earnings. 19 However, while redeemable shares may be redeemed Central Bank, to the President and Acting Chairman of the Board of the
regardless of the existence of unrestricted retained earnings, this is subject petitioner bank prohibiting the latter from redeeming any preferred share, on
to the condition that the corporation has, after such redemption, assets in its the ground that said redemption would reduce the assets of the Bank to the
books to cover debts and liabilities inclusive of capital stock. Redemption, prejudice of its depositors and creditors. 24 Redemption of preferred shares
therefore, may not be made where the corporation is insolvent or if such was prohibited for a just and valid reason. The directive issued by the Central
redemption will cause insolvency or inability of the corporation to meet its Bank Governor was obviously meant to preserve the status quo, and to
debts as they mature. 20 prevent the financial ruin of a banking institution that would have resulted in
adverse repercussions, not only to its depositors and creditors, but also to
We come now to the merits of the case. The petitioner argues that it cannot the banking industry as a whole. The directive, in limiting the exercise of a
be compelled to redeem the preferred shares issued to the private right granted by law to a corporate entity, may thus be considered as an
respondent. We agree. Respondent judge, in ruling that petitioner must exercise of police power. The respondent judge insists that the directive
redeem the shares in question, stated that: constitutes an impairment of the obligation of contracts. It has, however,
been settled that the Constitutional guaranty of non-impairment of obligations

667
of contract is limited by the exercise of the police power of the state, the right of the private respondents to any relief under the law has already
reason being that public welfare is superior to private rights. 25 prescribed. Moreover, the claim of the private respondents is also barred by
laches. Laches has been defined as the failure or neglect, for an
The respondent judge also stated that since the stock certificate granted the unreasonable length of time, to do that which by exercising due diligence
private respondents the right to receive a quarterly dividend of One Per could or should have been done earlier; it is negligence or omission to assert
Centum (1%) cumulative and participating, it "clearly and unequivocably (sic) a right within a reasonable time, warranting a presumption that the party
indicates that the same are "interest bearing stocks" or stocks issued by a entitled to assert it either has abandoned it or declined to assert it. 28
corporation under an agreement to pay a certain rate of interest thereon. As
such, plaintiffs (private respondents herein) become entitled to the payment Considering that the terms and conditions set forth in the stock certificate
thereof as a matter of right without necessity of a prior declaration of clearly indicate that redemption of the preferred shares may be made at any
dividend." 26 There is no legal basis for this observation. Both Sec. 16 of the time after the lapse of two years from the date of issue, private respondents
Corporation Law and Sec. 43 of the present Corporation Code prohibit the should have taken it upon themselves, after the lapse of the said period, to
issuance of any stock dividend without the approval of stockholders, inquire from the petitioner the reason why the said shares have not been
representing not less than two-thirds (2/3) of the outstanding capital stock at redeemed. As it is, not only two years had lapsed, as agreed upon, but an
a regular or special meeting duly called for the purpose. These provisions additional sixteen years passed before the private respondents saw it fit to
underscore the fact that payment of dividends to a stockholder is not a matter demand their right. The petitioner, at the time it issued said preferred shares
of right but a matter of consensus. Furthermore, "interest bearing stocks", on to the private respondents in 1961, could not have known that it would be
which the corporation agrees absolutely to pay interest before dividends are suffering from chronic reserve deficiency twelve years later. Had the private
paid to common stockholders, is legal only when construed as requiring respondents been vigilant in asserting their rights, the redemption could have
payment of interest as dividends from net earnings or surplus only. 27 Clearly, been effected at a time when the petitioner bank was not suffering from any
the respondent judge, in compelling the petitioner to redeem the shares in financial crisis.
question and to pay the corresponding dividends, committed grave abuse of
discretion amounting to lack or excess of jurisdiction in ignoring both the WHEREFORE, the instant petition, being impressed with merit, is hereby
terms and conditions specified in the stock certificate, as well as the clear GRANTED. The challenged decision of respondent judge is set aside and
mandate of the law. the complaint against the petitioner is dismissed. Costs against the private
respondents. SO ORDERED.
Anent the issue of prescription, this Court so holds that the claim of private
respondent is already barred by prescription as well as laches. Art. 1144 of [G.R. No. L-28398. August 6, 1975.]
the New Civil Code provides that a right of action that is founded upon a
written contract prescribes in ten (10) years. The letter-demand made by the COMMISSIONER OF INTERNAL REVENUE, Petitioner, v. JOHN L.
private respondents to the petitioner was made only on January 5, 1979, or MANNING, W.D. McDONALD, E.E. SIMMONS and THE COURT OF TAX
almost eighteen years after receipt of the written contract in the form of the APPEALS, Respondents.
stock certificate. As noted earlier, this letter-demand, significantly, was not
formally offered in evidence, nor were any other evidence of demand Solicitor General Antonio P. Barredo, Solicitor Lolita O. Gal-lang and
presented. Therefore, we conclude that the only time the private respondents Special Attorney Virgilio J. Saldajena for Petitioner.
saw it fit to assert their rights, if any, to the preferred shares of stock, was
after the lapse of almost eighteen years. The same clearly indicates that the Manuel O. Chan for Private Respondents.

668
SYLLABUS
SYNOPSIS

Under a trust agreement, Julius Reese who owned 24,700 shares of the
25,000 common shares of MANTRASCO, and the three private respondents 1. PRIVATE CORPORATIONS; SHARES OF STOCKS; TREASURY;
who owned the rest, at 100 shares each, deposited all their shares with the SHARES. Treasury shares are stocks issued and fully paid for and re-
Trustees. The trust agreement provided that upon Reeses death acquired by the corporation either by purchase, donation, forfeiture or other
MANTRASCO shall purchase Reeses shares. The trust agreement was means. They are therefore issued shares, but being in the treasury they do
executed in view of Reeses desire that upon his death the Company would not have the status of outstanding shares. Consequently, although a treasury
continue under the management of respondents. Upon Reeses death and share, not having been retired by the corporation re-acquiring it, may be re-
partial payment by the company of Reesess share, a new certificate was issued or sold again, such share, as long as it is held by the corporation as a
issued in the name of MANTRASCO, and the certificate indorsed to the treasury share, participates neither in dividends, because dividends cannot
Trustees. Subsequently, the stockholders reverted the 24,700 shares in the be declared by the corporation to itself, nor in the meetings of the
Treasury to the capital account of the company as stock dividends to be corporations as voting stock, for otherwise equal distribution of voting powers
distributed to the stockholders. When the entire purchase price of Reeses among stockholders will be effectively lost and the directors will be able to
interest in the company was paid in full by the latter, the trust agreement was perpetuate their control of the corporation though it still represent a paid
terminated, and the shares held in trust were delivered to the company. for interest in the property of the corporation.

The Bureau of Internal Revenue concluded that the distribution of the 24,700 2. ID.; ID.; ID.; DECLARATION OF QUESTIONED SHARES AS TREASURY
shares of Reese as stock dividends was in effect a distribution of the "assets STOCK DIVIDENDS, A NULLITY. Where the manifest intention of the
or property of the corporation." It therefore assessed respondents for parties to the trust agreement was, in sum and substance, to treat the shares
deficiency income taxes as well as for fraud penalty and interest charges. of a deceased stockholder as absolutely outstanding shares of said
The Court of Tax Appeals absolved respondent from any liability for receiving stockholders estate until they were fully paid. the declaration of said shares
the questioned stock dividends on the ground that their respective one-third as treasury stock dividend was a complete nullity and plainly violative of
interest in the Company remained the same before and after the declaration public policy.
of the stock dividends and only the number of shares held by each of them
had changed. 3. ID.; ID.; STOCK DIVIDEND PAYABLE ONLY FROM RETAINED
EARNINGS. A stock dividend, being one payable in capital stock, cannot
On a petition for review, the Supreme Court held that the newly acquired be declared out of outstanding corporate stock, but only from retained
shares were not treasury shares; their declaration as treasury stock earnings.
dividends was a complete nullity and that the assessment by the
Commissioner of fraud penalty and the imposition of interest charges 4. ID.; ID.; PURCHASE OF HOLDING RESULTING IN DISTRIBUTION OF
pursuant to the provision of the Tax Code were made in accordance with law. EARNINGS TAXABLE. Where by the use of a trust instrument as a
convenient technical device, respondents bestowed unto themselves the full
Judgment of the Court of Tax Appeals se aside. worth and value of a deceased stockholders corporate holding acquired with
the very earnings of the companies, such package device which obviously is
not designed to carry out the usual stock dividend purpose of corporate

669
expansion reinvestment, e.g., the acquisition of additional facilities and other 6. TAXATION; INCOME TAX; ASSESSMENT OF FRAUD PENALTY AND
capital budget items, but exclusively for expanding the capital base of the IMPOSITION OF INTEREST CHARGES IN ACCORDANCE WITH LAW
surviving stockholders in the company, cannot be allowed to deflect the DESPITE NULLITY OF RESOLUTION AUTHORIZING DISTRIBUTION OF
latters responsibilities toward our income tax laws. The conclusion is EARNINGS. The fact that the resolution authorizing the distribution of
ineluctable that whenever the company parted with a portion of its earnings earnings is null and void is of no moment. Under the National Internal
"to buy" the corporate holdings of the deceased stockholders, it was in Revenue Code, income tax is assessed on income received from any
ultimate effect and result making a distribution of such earnings to the property, activity or service that produces income. The Tax Code stands as
surviving stockholders. All these amounts are consequently subject to an indifferent, neutral party on the matter of where the income comes from.
income tax as being, in truth and in fact, a flow of cash benefits to the The action taken by the Commissioner of assessing fraud penalty and
surviving stockholders. imposing interest charges pursuant to the provisions of the Tax Code is in
accordance with law.
5. ID.; ID.; ID.; COMMISSIONER ASSESSMENT BASED ON THE TOTAL
ACQUISITION COST OF THE ALLEGED TREASURY STOCK DIVIDENDS,
ERROR. Where the surviving stockholders, by resolution, partitioned
among themselves, as treasury stock dividends, the deceased stockholders
interest, and earnings of the corporation over a period of years were used to DECISION
gradually wipe out the holdings therein of said deceased stockholder, the
earnings (which in effect have been distributed to the surviving stockholders
when they appropriated among themselves the deceased stockholders
interest), should be taxed for each of the corresponding years when
payments were made to the deceaseds estate on account of his shares. In
other words, the Tax Commissioner may not asses the surviving CASTRO, J.:
stockholders, for income tax purposes, the total acquisition cost of the
alleged treasury stock dividends in one lump sum. However, with regard to
payment made with the corporations earnings before the passage of the
resolution declaring as stock dividends the deceased stockholders interest
(while indeed those earnings were utilized in those years to gradually pay off This is a petition for review of the decision of the Court of Tax Appeals, in
the value of the deceased stockholders holdings), the surviving stockholders CTA case 1626, which set aside the income tax assessments issued by the
should be liable (in the absence of evidence that prior to the passage of the Commissioner of Internal Revenue against John L. Manning, W.D. McDonald
stockholders resolution the contributed of each of the surviving stockholder and E.E. Simmons (hereinafter referred to as the respondents), for alleged
rose corresponding), for income tax purposes, to the extent of the aggregate undeclared stock dividends received in 1958 from the Manila Trading and
amount paid by the corporation (prior to such resolution) to buy off the Supply Co. (hereinafter referred to as the MANTRASCO) valued at
deceased stockholders shares. The reason is that it was only by virtue of the P7,973,660.
authority contained in said resolution that the surviving stockholders actually,
albeit illegally, appropriated and petitioned among themselves the In 1952 the MANTRASCO had an authorized capital stock of P2,500,000
stockholders equity representing the deceased stockholders interest. divided into 25,000 common shares; 24,700 of these were owned by Julius
S. Reese, and the rest, at 100 shares each, by the three respondents.

670
"4. (a) Upon the death of the OWNER and the receipt by the TRUSTEES of
On February 29, 1952, in view of Reeses desire that upon his death the initial payment from the company purchasing the OWNERS SHARES,
MANTRASCO and its two subsidiaries, MANTRASCO (Guam), Inc. and the the TRUSTEES shall cause the OWNERS SHARES to be transferred into
Port Motors, Inc., would continue under the management of the respondents, the name of such company and such company shall thereupon transfer such
a trust agreement on his and the respondents interests in MANTRASCO was shares into the name of the TRUSTEES and the TRUSTEES shall hold such
executed by and among Reese (therein referred to as OWNER), shares until payment for all such shares shall have been made by the
MANTRASCO (therein referred to as COMPANY), the law firm of Ross, company as provided in this agreement.
Selph, Carrascoso and Janda (therein referred to as TRUSTEES), and the
respondents (therein referred to as MANAGERS). x x x

The trust agreement pertinently provides as follows:jgc:chanrobles.com.ph

"1. Upon the execution of this agreement the OWNER shall deposit with the "(c) The TRUSTEES shall vote all stock standing in their name or the name
TRUSTEES, duly endorsed and ready for transfer Twenty-Four Thousand of their nominees at all meetings and shall be in all respects entitled to all the
Seven Hundred (24,700) shares of the capital stock of the COMPANY, these rights as owners of said shares, subject, however, to the provisions of this
shares being all shares of the capital stock of the COMPANIES belonging to agreement of trust.
him . . .
"(d) Any and all dividends paid on said shares after the death of the OWNER
"2. Upon the execution of this Agreement the MANAGERS shall deposit with shall be subject to the provisions of this agreement.
the TRUSTEES, duly endorsed and ready for transfer, all shares of the
capital stock of the COMPANIES belonging to any of them.
x x x
"3. (a) The OWNER and the MANAGERS, and each of them, agree that if
any of them shall at any time during the life of this trust acquire any additional
shares of stock of any of the COMPANIES, or of any successor company, or
any shares in substitution, exchange or replacement of the shares subject to "5. (b) It is expressly agreed and understood, however, that the declaration of
this agreement, they shall forthwith endorse and deposit such shares with the dividends and amount of earnings transferred to surplus shall be subject to
TRUSTEES hereunder and such additional or other shares shall become the approval of the TRUSTEES and the TRUSTEES shall participate to such
subject to this agreement; shares deposited by the OWNER and shares extent in the affairs of the COMPANIES as they deem necessary to insure
received by the TRUSTEES as stock dividends on, or in substitution, the carrying out of this agreement and the discharge of the obligations of the
exchange or replacement of, such shares so deposited under this agreement COMPANIES and each of them and of the MANAGERS hereunder.
being MANAGERS SHARES.
"(c) The TRUSTEES shall designate one or more directors of each of the
"(b) All shares deposited under paragraphs 1, 2 and 3(a) hereof shall, during COMPANIES as they shall consider advisable and corresponding shares
the life of the OWNER, remain in the name of and shall be voted by the shall be transferred to such directors to qualify them to act.
respective parties making the deposit ...
x x x

671
terminate this agreement and dispose of all the shares of stock deposited
hereunder, whether or not payment shall have been made for part of such
"8. (a) Upon the death of the OWNER, the COMPANIES or any one or more stock, applying the proceeds of such sale or disposition to the unpaid
of them shall purchase the OWNERS SHARES; it being the intent that any balance of the purchase price:jgc:chanrobles.com.ph
of the COMPANIES shall purchase all or a proportionate part of the
OWNERS SHARES . . . "(a) If, upon any such sale or disposition of the stock, the TRUSTEES shall
receive an amount in excess of the unpaid balance of the purchase price
"(b) The purchase price of such shares shall be the book value of such share agreed to be paid by the COMPANIES for the OWNERS SHARES such
computed in United States dollars . . . excess, after deducting all expenses, charges and taxes, shall be paid to the
then MANAGERS.
x x x
x x x

"(d) All dividends paid on stock that had been OWNERS SHARES, from the
time of the transfer of such shares by one or more of the COMPANIES to the "17. Until the delivery to him of the shares purchased by him, no MANAGER,
TRUSTEES as provided in Article 4 until payment in full for such OWNERS shall sell, assign, mortgage, pledge, transfer or in anywise encumber or
SHARES shall have been made by each of the COMPANIES which shall hypothecate such shares or his interest in this agreement.
have purchased the same, shall be credited as payments on account of the
purchase price of such shares and shall be a prepayment on account of the x x x
next due installment or installments of such purchase price.

x x x
"19. After the death of the OWNER and during the period of this trust the
COMPANIES shall pay no dividends except as may be authorized by the
TRUSTEES. Dividends on MANAGERS SHARES shall, so long as they
"12. The TRUSTEES may from time to time increase or decrease the unpaid shall not be in default under this agreement, be paid over by the TRUSTEES
balance of the purchase price of the shares being purchased by any to the MANAGERS. Dividends on OWNERS SHARES shall be applied in
COMPANY or COMPANIES should they in their exclusive discretion liquidation of the COMPANIES liabilities hereunder as provided in Article
determine that such increase or decrease would be necessary to carry out 8(d).
the intention of the parties that the Estate and heirs of the OWNER shall
receive the fair value of the shares deposited in Trust as such value existed x x x
at the date of the death of the OWNER. . .

"13. Should the said COMPANIES or any of them be unable or unwilling to


comply with their obligations hereunder when due, the TRUSTEES may "26. The TRUSTEES may, after the death of the OWNER and during the life

672
of this trust, vote any and all shares held in trust, at any general and special December 19, 1958 which action is hereby approved and confirmed."cralaw
meeting of stockholders for all purposes, including but not limited to wholly or virtua1aw library
partially liquidating or reducing the capital of any COMPANY or
COMPANIES, authorizing the sale of any or all assets, and election of On November 25, 1963 the entire purchase price of Reeses interest in
directors . . . MANTRASCO was finally paid in full by the latter, On May 4, 1964 the trust
agreement was terminated and the trustees delivered to MANTRASCO all
x x x the shares which they were holding in trust.

Meanwhile, on September 14, 1962, an examination of MANTRASCOs


books was ordered by the Bureau of Internal Revenue. The examination
"28. The COMPANIES and each of them undertake and agree by proper disclosed that (a) as of December 31, 1958 the 24,700 shares declared as
corporate act to reduce their capitalization, sell or encumber their assets, dividends had been proportionately distributed to the respondents,
amend their articles of incorporation, reorganize, liquidate, dissolve and do representing a total book value or acquisition cost of P7,973,660; (b) the
all other things the TRUSTEES in their discretion determine to be necessary respondents failed to declare the said stock dividends as part of their taxable
to enable them to comply with their obligations hereunder and the income for the year 1958; and (c) from 1956 to 1961 the following amounts
TRUSTEES are hereby irrevocably authorized to vote all shares of the were paid by MANTRASCO to Reeses estate by virtue of the trust
COMPANIES and each of them at any general or special meeting for the agreement, to wit:chanrob1es virtual 1aw library
accomplishment of such purposes. . . ."cralaw virtua1aw library
Amounts
On October 19, 1954 Reese died. The projected transfer of his shares in the
name of MANTRASCO could not, however, be immediately effected for lack Year Liabilities Paid
of sufficient funds to cover initial payment on the shares.
1956 P5,830,587.86 P 2,143,073.00
On February 2, 1955, after MANTRASCO made a partial payment of Reeses
shares, the certificate for the 24,700 shares in Reeses name was cancelled 1957 5,317,137.86 513,450.00
and a new certificate was issued in the name of MANTRASCO. On the same
date, and in the meantime that Reeses interest had not been fully paid, the 1958 4,824,059.28 493,078.58
new certificate was endorsed to the law firm of Ross, Selph, Carrascoso and
Janda, as trustees for and in behalf of MANTRASCO. 1959 4,319,420.14 504,639.14

On December 22, 1958, at a special meeting of MANTRASCO stockholders, 1960 3,849,720.14 469,700.00
the following resolution was passed:jgc:chanrobles.com.ph
1961 3,811,387.69 38,332.45
"RESOLVED, that the 24,700 shares in the Treasury be reverted back to the
capital account of the company as a stock dividend to be distributed to On the basis of their examination, the BIR examiners concluded that the
shareholders of record at the close of business on December 22, 1958, in distribution of Reeses shares as stock dividends was in effect a distribution
accordance with the action of the Board of Directors at its meeting on of the "asset or property of the corporation as may be gleaned from the

673
payment of cash for the redemption of said stock and distributing the same A. National Internal Revenue Code
as stock dividend." On April 14, 1965 the Commissioner of Internal Revenue
issued notices of assessment for deficiency income taxes to the respondents "SEC. 83. Distribution of dividends or assets by corporations (a) Definition
for the year 1958, as follows:chanrob1es virtual 1aw library of Dividends The term dividends when used in this Title means any
distribution made by a corporation to its shareholders out of its earnings or
J.L. Manning W.D. McDonald E.E. Simmons profits accrued since March first, nineteen hundred and thirteen, and payable
to its shareholders, whether in money or in other property.
Deficiency Income Tax P1,416,469.00 P1,442,719.00 P1,450,434.00
"Where a corporation distributes all of its assets in complete liquidation or
Add 50% surcharge* 723,234.50 721,359.507 25,217.00 dissolution the gain realized or loss sustained by the stockholder, whether
individual or corporate, is a taxable income or deductible loss, as the case
1/2% monthly interest from may be.

6-20-59 to 6-20-62 260,364.42 259,689.42 261,078.12 "(b) Stock dividend. A stock dividend representing the transfer of surplus
to capital account shall not be subject to tax. However, if a corporation
cancels or redeems stock issued as a dividend at such time and in such
manner as to make the distribution and cancellation or redemption, in whole
TOTAL AMOUNT DUE or in part, essentially equivalent to the distribution of a taxable dividend, the
amount so distributed in redemption or cancellation of the stock shall be
& COLLECTIBLE P2,430,067.92 P2,423,767.92 2,436,729.12 considered as taxable income to the extent that it represents a distribution of
earnings or profits accumulated after March first, nineteen hundred and
The respondents unsuccessfully challenged the foregoing assessments and, thirteen."cralaw virtua1aw library
failing to secure a favorable reconsideration, appealed to the Court of Tax
Appeals. B. B.I.R. Regulations

On October 30, 1967 the CTA rendered judgment absolving the respondents "SEC. 251. Dividends paid in property. Dividends paid in securities or
from any liability for receiving the questioned stock dividends on the ground other property (other than its own stock), in which the earnings of the
that their respective one-third interest in MANTRASCO remained the same corporation have been invested, are income to the recipients to the amount
before and after the declaration of stock dividends and only the number of of the full market value of such property when receivable by individual
shares held by each of them had changed. stockholders . . .

Hence, the present recourse. "SEC. 252. Stock dividend. A stock dividend which represents the transfer
of surplus to capital account is not subject to income tax. However, a
All the parties rely upon the same provisions of the Tax Code and internal dividend in stock may constitute taxable income to the recipients thereof
revenue regulations to bolster their respective positions. These notwithstanding the fact that the officers or directors of the corporation (as
are:chanrob1es virtual 1aw library defined in section 84) choose to call such distribution as a stock dividend.
The distinction between a stock dividend which does not, and one which

674
does, constitute income taxable to the shareholders is the distinction Treasury shares are therefore issued shares, but being in the treasury they
between a stock dividend which works no change in the corporate entity, the do not have the status of outstanding shares. 3 Consequently, although a
same interest in the same corporation being represented after the distribution treasury share, not having been retired by the corporation re-acquiring it,
by more shares of precisely the same character, and a stock dividend where may be re-issued or sold again, such share, as long as it is held by the
there either has been change of corporate identity or a change in the nature corporation as a treasury share, participates neither in dividends, because
of the shares issued as dividends whereby the proportional interest of the dividends cannot be declared by the corporation to itself, 4 nor in the
shareholder after the distribution is essentially different from the former meetings of the corporation as voting stock, for otherwise equal distribution
interest. A stock dividend constitutes income if it gives the shareholder an of voting powers among stockholders will be effectively lost and the directors
interest different from that which his former stockholdings represented. A will be able to perpetuate their control of the corporation, 5 though it still
stock dividend does not constitute income if the new shares confer no represents a paid-for interest in the property of the corporation. 6 The
different rights or interests than did the old the new certificate plus the old foregoing essential features of a treasury stock are lacking in the questioned
representing the same proportionate interest in the net assets of the shares. Thus,
corporation as did the old."cralaw virtua1aw library
(a) under paragraph 4(c) of the trust agreement, the trustees were authorized
The parties differ, however, on the taxability of the "treasury" stock dividends to vote all stock standing in their names at all meetings and to exercise all
received by the respondents. rights "as owners of said shares" this authority is reiterated in paragraphs
26 and 28 of the trust agreement;
The respondents anchor their argument on the same basis as the Court of
Tax Appeals; whereas the Commissioner maintains that the full value (b) under paragraph 4(d), "Any and all dividends paid on said shares after the
(P7,973,660) of the shares redeemed from Reese by MANTRASCO which death of the OWNER shall be subject to the provisions of this agreement;"
were subsequently distributed to the respondents as stock dividends in 1958
should be taxed as income of the respondents for that year, the said (c) under paragraph 5(b), the amount of retained earnings to be declared as
distribution being in effect a distribution of cash. The respondents interests in dividends was made subject to the approval of the trustees of the 24,700
MANTRASCO, he further argues, were only .4% prior to the declaration of shares;
the stock dividends in 1958, but rose to 33 1/3% each after the said
declaration. (d) under paragraph 5(c), the choice of corporate directors was delegated
exclusively to the trustees who were also given the authority to transfer
In submitting their respective contentions, it is the assumption of both parties qualifying shares to such directors; and
that the 24,700 shares declared as stock dividends were treasury shares. We
are however convinced, after a careful study of the trust agreement, that the (e) under paragraph 19, MANTRASCO and its two subsidiaries were
said shares were not, on December 22, 1958 or at anytime before or after expressly prohibited from paying "dividends except as may be authorized by
that date, treasury shares. The reasons are quite plain. the TRUSTEES;" in the same paragraph mention was also made of
"dividends on OWNERS SHARES" which shall be applied to the liquidation
Although authorities may differ on the exact legal and accounting status of of the liabilities of the three companies for the price of Reeses shares.
so-called "treasury shares," 1 they are more or less in agreement that
treasury shares are stocks issued and fully paid for and re-acquired by the The manifest intention of the parties to the trust agreement was, in sum and
corporation either by purchase, donation, forfeiture or other means. 2 substance, to treat the 24,700 shares of Reese as absolutely outstanding

675
shares of Reeses estate until they were fully paid. Such being the true technical device, bestowed unto themselves the full worth and value of
nature of the 24,700 shares, their declaration as treasury stock dividend in Reeses corporate holdings with the use of the very earnings of the
1958 was a complete nullity and plainly violative of public policy. A stock companies. Such package device, obviously not designed to carry out the
dividend, being one payable in capital stock, cannot be declared out of usual stock dividend purpose of corporate expansion reinvestment, e.g. the
outstanding corporate stock, but only from retained earnings: 7 acquisition of additional facilities and other capital budget items, but
exclusively for expanding the capital base of the respondents in
Of pointed relevance is this useful discussion of the nature of a stock MANTRASCO, cannot be allowed to deflect the respondents responsibilities
dividend: 8 toward our income tax laws. The conclusion is thus ineluctable that whenever
the companies involved herein parted with a portion of their earnings "to buy"
"A stock dividend always involves a transfer of surplus (or profit) to capital the corporate holdings of Reese, they were in ultimate effect and result
stock. Graham and Katz, Accounting in Law Practice, 2d ed. 1938, No. 70. making a distribution of such earnings to the respondents. All these amounts
As the court said in United States v. Siegel, 8 Cir., 1931, 52 F 2d 63, 65, 78 are consequently subject to income tax as being, in truth and in fact, a flow of
ALR 672: A stock dividend is a conversion of surplus or undivided profits into cash benefits to the respondents.
capital stock, which is distributed to stockholders in lieu of a cash dividend.
Congress itself has defined the term dividend in No. 115(a) of the Act as We are of the opinion, however, that the Commissioner erred in assessing
meaning any distribution made by a corporation to its shareholders, whether the respondents the total acquisition cost (P7,973,660) of the alleged
in money or in other property, out of its earnings or profits. In Eisner v. treasury stock dividends in one lump sum. The record shows that the
Macomber, 1920, 252 US 189, 40 S Ct 189, 64 L Ed 521, 9 ALR 1570, both earnings of MANTRASCO over a period of years were used to gradually
the prevailing and the dissenting opinions recognized that within the meaning wipe out the holdings therein of Reese. Consequently, those earnings, which
of the revenue acts the essence of a stock dividend was the segregation out we hold, under the facts disclosed in the case at bar, as in effect having been
of surplus account of a definite portion of the corporate earnings as part of distributed to the respondents, should be taxed for each of the corresponding
the permanent capital resources of the corporation by the device of years when payments were made to Reeses estate on account of his 24,700
capitalizing the same, and the issuance to the stockholders of additional shares. With regard to payments made with MANTRASCO earnings in 1958
shares of stock representing the profits so capitalized."cralaw virtua1aw and the years before, while indeed those earnings were utilized in those
library years to gradually pay off the value of Reeses holdings in MANTRASCO,
there is no evidence from which it can be inferred that prior to the passage of
The declaration by the respondents and Reeses trustees of MANTRASCOs the stockholders resolution of December 22, 1958 the contributed equity of
alleged treasury stock dividends in favor of the former, brings, however, into each of the respondents rose correspondingly. It was only by virtue of the
clear focus the ultimate purpose which the parties to the trust instrument authority contained in the said resolution that the respondents actually, albeit
aimed to realize: to make the respondents the sole owners of Reeses illegally, appropriated and partitioned among themselves the stockholders
interest in MANTRASCO by utilizing the periodic earnings of that company equity representing Reeses interests in MANTRASCO. As those payments
and its subsidiaries to directly subsidize their purchase of the said interests, accrued in favor of the respondents in 1958 they are and should be liable, for
and by making it appear outwardly, through the formal declaration of non- income tax purposes, to the extent of the aggregate amount paid, from 1955
existent stock dividends in the treasury, that they have not received any to 1958, by MANTRASCO to buy off Reeses shares.
income from those firms when, in fact, by that declaration they secured to
themselves the means to turn around as full owners of Reeses shares. In The fact that the resolution authorizing the distribution of the said earnings is
other words, the respondents, using the trust instrument as a convenient null and void is of no moment. Under the National Internal Revenue Code,

676
income tax is assessed on income received from any property, activity or
service that produces income. 9 The Tax Code stands as an indifferent,
neutral party on the matter of where the income comes from. 10

Subject to the foregoing qualifications, we find the action taken by the G.R. No. L-21601 December 28, 1968
Commissioner in all other respects that is, the assessment of a fraud
penalty and imposition of interest charges pursuant to the provisions of the
NIELSON & COMPANY, INC., plaintiff-appellant,
Tax Code to be in accordance with law.
vs.
LEPANTO CONSOLIDATED MINING COMPANY, defendant-appellee.
ACCORDINGLY, the judgment of the Court of Tax Appeals absolving the
respondents from any deficiency income tax liability is set aside, and this
case is hereby remanded to the Court of Tax Appeals for further proceedings. RESOLUTION
More specifically, the Court of Tax Appeals shall recompute the income tax
liabilities of the respondents in accordance with this decision and with the Tax ZALDIVAR, J.:
Code, and thereafter pronounce and enter judgment accordingly. No costs.
Lepanto seeks the reconsideration of the decision rendered on December
Makasiar, Esguerra, Muoz Palma and Martin, JJ., concur. 17, 1966. The motion for reconsideration is based on two sets of grounds
the first set consisting of four principal grounds, and the second set
Teehankee, J., is on leave. consisting of five alternative grounds, as follows:

Principal Grounds:

1. The court erred in overlooking and failing to apply the proper law
applicable to the agency or management contract in question,
namely, Article 1733 of the Old Civil Code (Article 1920 of the new),
by virtue of which said agency was effectively revoked and
terminated in 1945 when, as stated in paragraph 20 of the complaint,
"defendant voluntarily ... prevented plaintiff from resuming
management and operation of said mining properties."

2. The court erred in holding that paragraph II of the management


contract (Exhibit C) suspended the period of said contract.

3. The court erred in reversing the ruling of the trial judge, based on
well-settled jurisprudence of this Supreme Court, that the

677
management agreement was only suspended but not extended on said contract. Lepanto now asserts for the first time and this is done in a
account of the war. motion for reconsideration - that the management contract in question is a
contract of agency such that it has the right to revoke and terminate the said
4. The court erred in reversing the finding of the trial judge that contract, as it did terminate the same, under the law of agency, and
Nielson's action had prescribed, but considering only the first claim particularly pursuant to Article 1733 of the Old Civil Code (Article 1920 of the
and ignoring the prescriptibility of the other claims. New Civil Code).

Alternative Grounds: We have taken note that Lepanto is advancing a new theory. We have
carefully examined the pleadings filed by Lepanto in the lower court, its
5. The court erred in holding that the period of suspension of the memorandum and its brief on appeal, and never did it assert the theory that it
contract on account of the war lasted from February 1942 to June has the right to terminate the management contract because that contract is
26, 1948. one of agency which it could terminate at will. While it is true that in its ninth
and tenth special affirmative defenses, in its answer in the court below,
Lepanto pleaded that it had the right to terminate the management contract
6. Assuming arguendo that Nielson is entitled to any relief, the court
in question, that plea of its right to terminate was not based upon the ground
erred in awarding as damages (a) 10% of the cash dividends
that the relation between Lepanto and Nielson was that of principal and
declared and paid in December, 1941; (b) the management fee of
agent but upon the ground that Nielson had allegedly not complied with
P2,500.00 for the month of January, 1942; and (c) the full contract
certain terms of the management contract. If Lepanto had thought of
price for the extended period of sixty months, since these damages
considering the management contract as one of agency it could have
were neither demanded nor proved and, in any case, not allowable
amended its answer by stating exactly its position. It could have asserted its
under the general law of damages.
theory of agency in its memorandum for the lower court and in its brief on
appeal. This, Lepanto did not do. It is the rule, and the settled doctrine of this
7. Assuming arguendo that appellant is entitled to any relief, the Court, that a party cannot change his theory on appeal that is, that a party
court erred in ordering appellee to issue and deliver to appellant cannot raise in the appellate court any question of law or of fact that was not
shares of stock together with fruits thereof. raised in the court below or which was not within the issue made by the
parties in their pleadings (Section 19, Rule 49 of the old Rules of Court, and
8. The court erred in awarding to appellant an undetermined amount also Section 18 of the new Rules of Court; Hautea vs. Magallon, L-20345,
of shares of stock and/or cash, which award cannot be ascertained November 28, 1964; Northern Motors, Inc. vs. Prince Line, L-13884,
and executed without further litigation. February 29, 1960; American Express Co. vs. Natividad, 46 Phil. 207;
Agoncillo vs. Javier, 38 Phil. 424 and Molina vs. Somes, 24 Phil 49).
9. The court erred in rendering judgment for attorney's fees.
At any rate, even if we allow Lepanto to assert its new theory at this very late
We are going to dwell on these grounds in the order they are presented. stage of the proceedings, this Court cannot sustain the same.

1. In its first principal ground Lepanto claims that its own counsel and this Lepanto contends that the management contract in question (Exhibit C) is
Court had overlooked the real nature of the management contract entered one of agency because: (1) Nielson was to manage and operate the mining
into by and between Lepanto and Nielson, and the law that is applicable on properties and mill on behalf, and for the account, of Lepanto; and (2)

678
Nielson was authorized to represent Lepanto in entering, on Lepanto's services does not represent his employer, while the agent represents his
behalf, into contracts for the hiring of laborers, purchase of supplies, and the principal. Manresa, in his "Commentarios al Codigo Civil Espaol" (1931,
sale and marketing of the ores mined. All these, Lepanto claims, show that Tomo IX, pp. 372-373), points out that the element of representation
Nielson was, by the terms of the contract, destined to execute juridical acts distinguishes agency from lease of services, as follows:
not on its own behalf but on behalf of Lepanto under the control of the Board
of Directors of Lepanto "at all times". Hence Lepanto claims that the contract Nuestro art. 1.709 como el art. 1.984 del Codigo de Napoleon y
is one of agency. Lepanto then maintains that an agency is revocable at the cuantos textos legales citamos en las concordancias, expresan
will of the principal (Article 1733 of the Old Civil Code), regardless of any claramente esta idea de la representacion, "hacer alguna cosa por
term or period stipulated in the contract, and it was in pursuance of that right cuenta o encargo de otra" dice nuestro Codigo; "poder de hacer
that Lepanto terminated the contract in 1945 when it took over and assumed alguna cosa para el mandante o en su nombre" dice el Codigo de
exclusive management of the work previously entrusted to Nielson under the Napoleon, y en tales palabras aparece vivo y luminoso el concepto y
contract. Lepanto finally maintains that Nielson as an agent is not entitled to la teoria de la representacion, tan fecunda en ensenanzas, que a su
damages since the law gives to the principal the right to terminate the agency sola luz es como se explican las diferencias que separan el mandato
at will. del arrendamiento de servicios, de los contratos inominados, del
consejo y de la gestion de negocios.
Because of Lepanto's new theory We consider it necessary to determine
the nature of the management contract whether it is a contract of agency En efecto, en el arrendamiento de servicios al obligarse para su
or a contract of lease of services. Incidentally, we have noted that the lower ejecucion, se trabaja, en verdad, para el dueno que remunera la
court, in the decision appealed from, considered the management contract labor, pero ni se le representa ni se obra en su nombre....
as a contract of lease of services.
On the basis of the interpretation of Article 1709 of the old Civil Code, Article
Article 1709 of the Old Civil Code, defining contract of agency, provides: 1868 of the new Civil Code has defined the contract of agency in more
explicit terms, as follows:
By the contract of agency, one person binds himself to render some
service or do something for the account or at the request of another. By the contract of agency a person binds himself to render some
service or to do something in representation or on behalf of another,
Article 1544, defining contract of lease of service, provides: with the consent or authority of the latter.

In a lease of work or services, one of the parties binds himself to There is another obvious distinction between agency and lease of services.
make or construct something or to render a service to the other for a Agency is a preparatory contract, as agency "does not stop with the agency
price certain. because the purpose is to enter into other contracts." The most characteristic
feature of an agency relationship is the agent's power to bring about
In both agency and lease of services one of the parties binds himself to business relations between his principal and third persons. "The agent is
render some service to the other party. Agency, however, is distinguished destined to execute juridical acts (creation, modification or extinction of
from lease of work or services in that the basis of agency is representation, relations with third parties). Lease of services contemplate only material
while in the lease of work or services the basis is employment. The lessor of

679
(non-juridical) acts." (Reyes and Puno, "An Outline of Philippine Civil Law," It thus appears that the principal and paramount undertaking of Nielson
Vol. V, p. 277). under the management contract was the operation and development of the
mine and the operation of the mill. All the other undertakings mentioned in
In the light of the interpretations we have mentioned in the foregoing the contract are necessary or incidental to the principal undertaking these
paragraphs let us now determine the nature of the management contract in other undertakings being dependent upon the work on the development of
question. Under the contract, Nielson had agreed, for a period of five years, the mine and the operation of the mill. In the performance of this principal
with the right to renew for a like period, to explore, develop and operate the undertaking Nielson was not in any way executing juridical acts for Lepanto,
mining claims of Lepanto, and to mine, or mine and mill, such pay ore as destined to create, modify or extinguish business relations between Lepanto
may be found therein and to market the metallic products recovered and third persons. In other words, in performing its principal undertaking
therefrom which may prove to be marketable, as well as to render for Nielson was not acting as an agent of Lepanto, in the sense that the term
Lepanto other services specified in the contract. We gather from the contract agent is interpreted under the law of agency, but as one who was performing
that the work undertaken by Nielson was to take complete charge subject at material acts for an employer, for a compensation.
all times to the general control of the Board of Directors of Lepanto, of the
exploration and development of the mining claims, of the hiring of a sufficient It is true that the management contract provides that Nielson would also act
and competent staff and of sufficient and capable laborers, of the prospecting as purchasing agent of supplies and enter into contracts regarding the sale of
and development of the mine, of the erection and operation of the mill, and of mineral, but the contract also provides that Nielson could not make any
the benefication and marketing of the minerals found on the mining purchase, or sell the minerals, without the prior approval of Lepanto. It is
properties; and in carrying out said obligation Nielson should proceed clear, therefore, that even in these cases Nielson could not execute juridical
diligently and in accordance with the best mining practice. In connection with acts which would bind Lepanto without first securing the approval of Lepanto.
its work Nielson was to submit reports, maps, plans and recommendations Nielson, then, was to act only as an intermediary, not as an agent.
with respect to the operation and development of the mining properties,
make recommendations and plans on the erection or enlargement of any Lepanto contends that the management contract in question being one of
existing mill, dispatch mining engineers and technicians to the mining agency it had the right to terminate the contract at will pursuant to the
properties as from time to time may reasonably be required to investigate provision of Article 1733 of the old Civil Code. We find, however, a proviso in
and make recommendations without cost or expense to Lepanto. Nielson the management contract which militates against this stand of Lepanto.
was also to "act as purchasing agent of supplies, equipment and other Paragraph XI of the contract provides:
necessary purchases by Lepanto, provided, however, that no purchase shall
be made without the prior approval of Lepanto; and provided further, that no Both parties to this agreement fully recognize that the terms of this
commission shall be claimed or retained by Nielson on such purchase"; and Agreement are made possible only because of the faith or
"to submit all requisition for supplies, all constricts and arrangement with confidence that the Officials of each company have in the other;
engineers, and staff and all matters requiring the expenditures of money, therefore, in order to assure that such confidence and faith shall
present or future, for prior approval by Lepanto; and also to make contracts abide and continue, NIELSON agrees that LEPANTO may cancel
subject to the prior approve of Lepanto for the sale and marketing of the this Agreement at any time upon ninety (90) days written notice, in
minerals mined from said properties, when said products are in a suitable the event that NIELSON for any reason whatsoever, except acts of
condition for marketing."1 God, strike and other causes beyond its control, shall cease to
prosecute the operation and development of the properties herein

680
described, in good faith and in accordance with approved mining Re: Mankayan Copper Mines
practice.
GENTLEMEN:
It is thus seen, from the above-quoted provision of paragraph XI of the
management contract, that Lepanto could not terminate the agreement at After an examination of your property by our engineers, we
will. Lepanto could terminate or cancel the agreement by giving notice of have decided to offer as we hereby offer to underwrite the
termination ninety days in advance only in the event that Nielson should entire issue of stock of a corporation to be formed for the
prosecute in bad faith and not in accordance with approved mining practice purpose of taking over said properties, said corporation to
the operation and development of the mining properties of Lepanto. Lepanto have an authorized capital of P1,750,000.00, of which
could not terminate the agreement if Nielson should cease to prosecute the P700,000.00 will be issued in escrow to the claim-owners in
operation and development of the mining properties by reason of acts of exchange for their claims, and the balance of P1,050,000.00
God, strike and other causes beyond the control of Nielson. we will sell to the public at par or take ourselves.

The phrase "Both parties to this agreement fully recognize that the terms of The arrangement will be under the following conditions:
this agreement are made possible only because of the faith and confidence
of the officials of each company have in the other" in paragraph XI of the 1. The subscriptions for cash shall be payable 50% at time of
management contract does not qualify the relation between Lepanto and subscription and the balance subject to the call of the Board
Nielson as that of principal and agent based on trust and confidence, such of Directors of the proposed corporation.
that the contractual relation may be terminated by the principal at any time
that the principal loses trust and confidence in the agent. Rather, that phrase
2. We shall have an underwriting and brokerage commission
simply implies the circumstance that brought about the execution of the
of 10% of the P1,050,000.00 to be sold for cash to the
management contract. Thus, in the annual report for 1936 2, submitted by Mr.
public, said commission to be payable from the first payment
C. A. Dewit, President of Lepanto, to its stockholders, under date of March
of 50% on each subscription.
15, 1937, we read the following:

3. We will bear the cost of preparing and mailing any


To the stockholders
prospectus that may be required, but no such prospectus will
be sent out until the text thereof has been first approved by
xxx xxx xxx the Board of Directors of the proposed corporation.

The incorporation of our Company was effected as a result of


negotiations with Messrs. Nielson & Co., Inc., and an offer by these
gentlemen to Messrs. C. I. Cookes and V. L. Lednicky, dated August
11, 1936, reading as follows:

Messrs. Cookes and Lednicky,


Present

681
4. That after the organization of the corporation, all operating was made with the consent of all the then stockholders, in virtue of
contract be entered into between ourselves and said which the compensation of Messrs. Nielson & Co., was increased to
corporation, under the terms which the property will be P2,500.00 per month when mill construction began. The formal
developed and mined and a mill erected, under our Management Contract was not entered into until January 30, 1937.
supervision, our compensation to be P2,000.00 per month
until the property is put on a profitable basis and P2,500.00 xxx xxx xxx
per month plus 10% of the net profits for a period of five
years thereafter. Manila, March 15, 1937

5. That we shall have the option to renew said operating (Sgd.) C. A. DeWitt
contract for an additional period of five years, on the same President
basis as the original contract, upon the expiration thereof.
We can gather from the foregoing statements in the annual report for 1936,
It is understood that the development and mining operations and from the provision of paragraph XI of the Management contract, that the
on said property, and the erection of the mill thereon, and the employment by Lepanto of Nielson to operate and manage its mines was
expenditures therefor shall be subject to the general control principally in consideration of the know-how and technical services that
of the Board of Directors of the proposed corporation, and, in Nielson offered Lepanto. The contract thus entered into pursuant to the offer
case you accept this proposition, that a detailed operating made by Nielson and accepted by Lepanto was a "detailed operating
contract will be entered into, covering the relationships contract". It was not a contract of agency. Nowhere in the record is it shown
between the parties. that Lepanto considered Nielson as its agent and that Lepanto terminated the
management contract because it had lost its trust and confidence in Nielson.
Yours very truly,
(Sgd.) L. R. Nielson The contention of Lepanto that it had terminated the management contract in
1945, following the liberation of the mines from Japanese control, because
Pursuant to the provisions of paragraph 2 of this offer, Messrs. the relation between it and Nielson was one of agency and as such it could
Nielson & Co., took subscriptions for One Million Fifty Thousand terminate the agency at will, is, therefore, untenable. On the other hand, it
Pesos (P1,050,000.00) in shares of our Company and their can be said that, in asserting that it had terminated or cancelled the
underwriting and brokerage commission has been paid. More than management contract in 1945, Lepanto had thereby violated the express
fifty per cent of these subscriptions have been paid to the Company terms of the management contract. The management contract was renewed
in cash. The claim owners have transferred their claims to the to last until January 31, 1947, so that the contract had yet almost two years
Corporation, but the P700,000.00 in stock which they are to receive to go upon the liberation of the mines in 1945. There is no showing that
therefor, is as yet held in escrow. Nielson had ceased to prosecute the operation and development of the
mines in good faith and in accordance with approved mining practice which
Immediately upon the formation of the Corporation Messrs. Nielson would warrant the termination of the contract upon ninety days written notice.
& Co., assumed the Management of the property under the control of In fact there was no such written notice of termination. It is an admitted fact
the Board of Directors. A modification in the Management Contract that Nielson ceased to operate and develop the mines because of the war

682
a cause beyond the control of Nielson. Indeed, if the management contract in unable to perform the work of mining and milling because of the adverse
question was intended to create a relationship of principal and agent effects of the war on the work of mining and milling.
between Lepanto and Nielson, paragraph XI of the contract should not have
been inserted because, as provided in Article 1733 of the old Civil Code, It is the contention of Lepanto that the happening of those events, and the
agency is essentially revocable at the will of the principal that means, with effects of those events, simply suspended the performance of the obligations
or without cause. But precisely said paragraph XI was inserted in the by either party in the contract, but did not suspend the period of the contract,
management contract to provide for the cause for its revocation. The much less extended the period of the contract.
provision of paragraph XI must be given effect.
We have conscientiously considered the arguments of Lepanto in support of
In the construction of an instrument where there are several provisions or these three grounds, but We are not persuaded to reconsider the rulings that
particulars, such a construction is, if possible, to be adopted as will give We made in Our decision.
effect to all,3 and if some stipulation of any contract should admit of several
meanings, it shall be understood as bearing that import which is most We want to say a little more on these points, however. Paragraph II of the
adequate to render it effectual.4 management contract provides as follows:

It is Our considered view that by express stipulation of the parties, the In the event of inundation, flooding of the mine, typhoon, earthquake
management contract in question is not revocable at the will of Lepanto. We or any other force majeure, war, insurrection, civil commotion,
rule that this management contract is not a contract of agency as defined in organized strike, riot, fire, injury to the machinery or other event or
Article 1709 of the old Civil Code, but a contract of lease of services as cause reasonably beyond the control of NIELSON and which
defined in Article 1544 of the same Code. This contract can not be adversely affects the work of mining and milling; NIELSON shall
unilaterally revoked by Lepanto. report such fact to LEPANTO and without liability or breach of the
terms of this Agreement,the same shall remain in suspense, wholly
The first ground of the motion for reconsideration should, therefore, be or partially during the terms of such inability. (Emphasis supplied)
brushed aside.
A reading of the above-quoted paragraph II cannot but convey the idea that
2. In the second, third and fifth grounds of its motion for reconsideration, upon the happening of any of the events enumerated therein, which
Lepanto maintains that this Court erred, in holding that paragraph 11 of the adversely affects the work of mining and milling, the agreement is deemed
management contract suspended the period of said contract, in holding that suspended for as long as Nielson is unable to perform its work of mining and
the agreement was not only suspended but was extended on account of the milling because of the adverse effects of the happening of the event on the
war, and in holding that the period of suspension on account of the war work of mining and milling. During the period when the adverse effects on the
lasted from February, 1942 to June 26, 1948. We are going to discuss these work of mining and milling exist, neither party in the contract would be held
three grounds together because they are interrelated. liable for non-compliance of its obligation under the contract. In other words,
the operation of the contract is suspended for as long as the adverse effects
In our decision we have dwelt lengthily on the points that the management of the happening of any of those events had impeded or obstructed the work
contract was suspended because of the war, and that the period of the of mining and milling. An analysis of the phraseology of the above-quoted
contract was extended for a period equivalent to the time when Nielson was paragraph II of the management contract readily supports the conclusion that

683
it is the agreement, or the contract, that is suspended. The phrase "the In Our decision We pointed out that the agreement in the management
same" can refer to no other than the term "Agreement" which immediately contract would be suspended when two conditions concur, namely: (1) the
precedes it. The "Agreement" may be wholly or partially suspended, and this happening of the event constituting a force majeure that was reasonably
situation will depend on whether the event wholly or partially affected beyond the control of Nielson, and (2) that the event constituting the force
adversely the work of mining and milling. In the instant case, the war had majeure adversely affected the work of mining and milling. The suspension,
adversely affected and wholly at that the work of mining and milling. We therefore, would last not only while the event constituting the force majeure
have clearly stated in Our decision the circumstances brought about by the continued to occur but also for as long as the adverse effects of the force
war which caused the whole or total suspension of the agreement or of the majeure on the work of mining and milling had not been eliminated. Under
management contract. the management contract the happening alone of the event constituting the
force majeure which did not affect adversely the work of mining and milling
LEPANTO itself admits that the management contract was suspended. We would not suspend the period of the contract. It is only when the two
quote from the brief of LEPANTO: conditions concur that the period of the agreement is suspended.

Probably, what Nielson meant was, it was prevented by Lepanto to It is not denied that because of the war, in February 1942, the mine, the
assume again the management of the mine in 1945, at the precise original mill, the original power plant, the supplies and equipment, and all
time when defendant was at the feverish phase of rehabilitation and installations at the Mankayan mines of Lepanto, were destroyed upon order
although the contract had already been suspended. (Lepanto's Brief, of the United States Army, to prevent their utilization by the enemy. It is not
p. 9). denied that for the duration of the war Nielson could not undertake the work
of mining and milling. When the mines were liberated from the enemy in
... it was impossible, as a result of the destruction of the mine, for the August, 1945, the condition of the mines, the mill, the power plant and other
plaintiff to manage and operate the same and because, as provided installations, was not the same as in February 1942 when they were ordered
in the agreement, the contract was suspended by reason of the war destroyed by the US army. Certainly, upon the liberation of the mines from
(Lepanto's Brief, pp. 9-10). the enemy, the work of mining and milling could not be undertaken by
Nielson under the same favorable circumstances that obtained before
February 1942. The work of mining and milling, as undertaken by Nielson in
Clause II, by its terms, is clear that the contract is suspended in case
January, 1942, could not be resumed by Nielson soon after liberation
fortuitous event or force majeure, such as war, adversely affects the
because of the adverse effects of the war, and this situation continued until
work of mining and milling. (Lepanto's Brief, p. 49).
June of 1948. Hence, the suspension of the management contract did not
end upon the liberation of the mines in August, 1945. The mines and the mill
Lepanto is correct when it said that the obligations under the contract were and the installations, laid waste by the ravages of war, had to be
suspended upon the happening of any of the events enumerated in reconstructed and rehabilitated, and it can be said that it was only on June
paragraph II of the management contract. Indeed, those obligations were 26, 1948 that the adverse effects of the war on the work of mining and milling
suspended because the contract itself was suspended. When we talk of a had ended, because it was on that date that the operation of the mines and
contract that has been suspended we certainly mean that the contract the mill was resumed. The period of suspension should, therefore, be
temporarily ceased to be operative, and the contract becomes operative reckoned from February 1942 until June 26, 1948, because it was during this
again upon the happening of a condition or when a situation obtains period that the war and the adverse effects of the war on the work of mining
which warrants the termination of the suspension of the contract. and milling had lasted. The mines and the installations had to be rehabilitated

684
because of the adverse effects of the war. The work of rehabilitation started It is the considered view of this court that it would not be fair to Nielson to
soon after the liberation of the mines in August, 1945 and lasted until June consider the suspension of the contract as terminated upon the liberation of
26, 1948 when, as stated in Lepanto's annual report to its stockholders for the mines because then Nielson would be placed in a situation whereby it
the year 1948, "June 28, 1948 marked the official return to operation of this would have to suffer the adverse effects of the war on the work of mining and
company at its properties at Mankayan, Mountain Province, Philippines" milling. The evidence shows that as of January 1942 the operation of the
(Exh. F-1). mines under the management of Nielson was already under beneficial
conditions, so much so that dividends were already declared by Lepanto for
Lepanto would argue that if the management contract was suspended at all the years 1939, 1940 and 1941. To make the management contract
the suspension should cease in August of 1945, contending that the effects immediately operative after the liberation of the mines from the Japanese, at
of the war should cease upon the liberation of the mines from the enemy. the time when the mines and all its installations were laid waste as a result of
This contention cannot be sustained, because the period of rehabilitation was the war, would be to place Nielson in a situation whereby it would lose all the
still a period when the physical effects of the war the destruction of the benefits of what it had accomplished in placing the Lepanto mines in
mines and of all the mining installations adversely affected, and made profitable operation before the outbreak of the war in December, 1941. The
impossible, the work of mining and milling. Hence, the period of the record shows that Nielson started its management operation way back in
reconstruction and rehabilitation of the mines and the installations must be 1936, even before the management contract was entered into. As early as
counted as part of the period of suspension of the contract. August 1936 Nielson negotiated with Messrs. C. I. Cookes and V. L. Lednicky
for the operation of the Mankayan mines and it was the result of those
Lepanto claims that it would not be unfair to end the period of suspension negotiations that Lepanto was incorporated; that it was Nielson that helped to
upon the liberation of the mines because soon after the liberation of the capitalize Lepanto, and that after the formation of the corporation (Lepanto)
mines Nielson insisted to resume the management work, and that Nielson Nielson immediately assumed the management of the mining properties of
was under obligation to reconstruct the mill in the same way that it was under Lepanto. It was not until January 30, 1937 when the management contract in
obligation to construct the mill in 1937. This contention is untenable. It is true question was entered into between Lepanto and Nielson (Exhibit A).
that Nielson insisted to resume its management work after liberation, but this
was only for the purpose of restoring the mines, the mill, and other A contract for the management and operation of mines calls for a speculative
installations to their operating and producing condition as of February 1942 and risky venture on the part of the manager-operator. The manager-
when they were ordered destroyed. It is not shown by any evidence in the operator invests its technical know-how, undertakes back-breaking efforts
record, that Nielson had agreed, or would have agreed, that the period of and tremendous spade-work, so to say, in the first years of its management
suspension of the contract would end upon the liberation of the mines. This is and operation of the mines, in the expectation that the investment and the
so because, as found by this Court, the intention of the parties in the efforts employed might be rewarded later with success. This expected
management contract, and as understood by them, the management success may never come. This had happened in the very case of the
contract was suspended for as long as the adverse effects of the force Mankayan mines where, as recounted by Mr. Lednicky of Lepanto, various
majeure on the work of mining and milling had not been removed, and the persons and entities of different nationalities, including Lednicky himself,
contract would be extended for as long as it was suspended. Under the invested all their money and failed. The manager-operator may not strike
management contract Nielson had the obligation to erect and operate the sufficient ore in the first, second, third, or fourth year of the management
mill, but not to erect or reconstruct the mill in case of its destruction by force contract, or he may not strike ore even until the end of the fifth year. Unless
majeure. the manager-operator strikes sufficient quantity of ore he cannot expect
profits or reward for his investment and efforts. In the case of Nielson, its

685
corps of competent engineers, geologists, and technicians begun working on In Our decision We stated that the claims of Nielson are based on a written
the Mankayan mines of Lepanto since the latter part of 1936, and continued document, and, as such, the cause of action prescribes in ten
their work without success and profit through 1937, 1938, and the earlier part years.5 Inasmuch as there are different claims which accrued on different
of 1939. It was only in December of 1939 when the efforts of Nielson started dates the prescriptive periods for all the claims are not the same. The claims
to be rewarded when Lepanto realized profits and the first dividends were of Nielson that have been awarded by this Court are itemized in the
declared. From that time on Nielson could expect profit to come to it as in dispositive part of the decision.
fact Lepanto declared dividends for 1940 and 1941 if the development and
operation of the mines and the mill would continue unhampered. The The first item of the awards in Our decision refers to Nielson's compensation
operation, and the expected profits, however, would still be subject to in the sum of P17,500.00, which is equivalent to 10% of the cash dividends
hazards due to the occurrence of fortuitous events, fires, earthquakes, declared by Lepanto in December, 1941. As we have stated in Our decision,
strikes, war, etc., constituting force majeure, which would result in the this claim accrued on December 31, 1941, and the right to commence an
destruction of the mines and the mill. One of these diverse causes, or one action thereon started on January 1, 1942. We declared that the action on
after the other, may consume the whole period of the contract, and if it should this claim did not prescribe although the complaint was filed on February 6,
happen that way the manager-operator would reap no profit to compensate 1958 or after a lapse of 16 years, 1 month and 5 days because of the
for the first years of spade-work and investment of efforts and know-how. operation of the moratorium law.
Hence, in fairness to the manager-operator, so that he may not be deprived
of the benefits of the work he had accomplished, the force majeure clause is We declared that under the applicable decisions of this Court 6 the
incorporated as a standard clause in contracts for the management and moratorium period of 8 years, 2 months and 8 days should be deducted from
operation of mines. the period that had elapsed since the accrual of the cause of action to the
date of the filing of the complaint, so that there is a period of less than 8
The nature of the contract for the management and operation of mines years to be reckoned for the purpose of prescription.
justifies the interpretation of the force majeure clause, that a period equal to
the period of suspension due to force majeure should be added to the This claim of Nielson is covered by Executive Order No. 32, issued on March
original term of the contract by way of an extension. We, therefore, reiterate 10, 1945, which provides as follows:
the ruling in Our decision that the management contract in the instant case
was suspended from February, 1942 to June 26, 1948, and that from the
Enforcement of payments of all debts and other monetary
latter date the contract had yet five years to go.
obligations payable in the Philippines, except debts and other
monetary obligations entered into in any area after declaration by
3. In the fourth ground of its motion for reconsideration, Lepanto maintains Presidential Proclamation that such area has been freed from enemy
that this Court erred in reversing the finding of the trial court that Nielson's occupation and control, is temporarily suspended pending action by
action has prescribed, by considering only the first claim and ignoring the the Commonwealth Government. (41 O.G. 56-57; Emphasis
prescriptibility of the other claims. supplied)

This ground of the motion for reconsideration has no merit. Executive Order No. 32 covered all debts and monetary obligation contracted
before the war (or before December 8, 1941) and those contracted
subsequent to December 8, 1941 and during the Japanese occupation.

686
Republic Act No. 342, approved on July 26, 1948, lifted the moratorium We therefore reiterate the ruling in Our decision that the claim involved in the
provided for in Executive Order No. 32 on pre-war (or pre-December 8, 1941) first item awarded to Nielson had not prescribed.
debts of debtors who had not filed war damage claims with the United States
War Damage Commission. In other words, after the effectivity of Republic Act What we have stated herein regarding the non-prescription of the cause of
No. 342, the debt moratorium was limited: (1) to debts and other monetary action of the claim involved in the first item in the award also holds true with
obligations which were contracted after December 8, 1941 and during the respect to the second item in the award, which refers to Nielson's claim for
Japanese occupation, and (2) to those pre-war (or pre-December 8, 1941) management fee of P2,500.00 for January, 1942. Lepanto admits that this
debts and other monetary obligations where the debtors filed war damage second item, like the first, is a monetary obligation. The right of action of
claims. That was the situation up to May 18, 1953 when this Court declared Nielson regarding this claim accrued on January 31, 1942.
Republic Act No. 342 unconstitutional. 7 It has been held by this Court,
however, that from March 10, 1945 when Executive Order No. 32 was As regards items 3, 4, 5, 6 and 7 in the awards in the decision, the
issued, to May 18, 1953 when Republic Act No. 342 was declared moratorium law is not applicable. That is the reason why in Our decision We
unconstitutional or a period of 8 years, 2 months and 8 days the debt did not discuss the question of prescription regarding these items. The claims
moratorium was in force, and had the effect of suspending the period of of Nielson involved in these items are based on the management contract,
prescription.8 and Nielson's cause of action regarding these claims prescribes in ten years.
Corollary to Our ruling that the management contract was suspended from
Lepanto is wrong when in its motion for reconsideration it claims that the February, 1942 until June 26, 1948, and that the contract was extended for
moratorium provided for in Executive Order No. 32 was continued by five years from June 26, 1948, the right of action of Nielson to claim for what
Republic Act No. 342 "only with respect to debtors of pre-war obligations or is due to it during that period of extension accrued during the period from
those incurred prior to December 8, 1941," and that "the moratorium June 26, 1948 till the end of the five-year extension period or until June 26,
was lifted and terminated with respect to obligations incurred after December 1953. And so, even if We reckon June 26, 1948 as the starting date of the
8, 1941."9 ten-year period in connection with the prescriptibility of the claims involved in
items 3, 4, 5, 6 and 7 of the awards in the decision, it is obvious that when
This Court has held that Republic Act No. 342 does not apply to debts the complaint was filed on February 6, 1958 the ten-year prescriptive period
contracted during the war and did not lift the moratorium in relations had not yet lapsed.
thereto.10 In the case of Abraham, et al. vs. Intestate Estate of Juan C.
Ysmael, et al., L-16741, Jan. 31, 1962, this Court said: In Our decision We have also ruled that the right of action of Nielson against
Lepanto had not prescribed because of the arbitration clause in the
Respondents, however, contend that Republic Act No. 342, which Management contract. We are satisfied that there is evidence that Nielson
took effect on July 26, 1948, lifted the moratorium on debts had asked for arbitration, and an arbitration committee had been constituted.
contracted during the Japanese occupation. The court has already The arbitration committee, however, failed to bring about any settlement of
held that Republic Act No. 342 did not lift the moratorium on debts the differences between Nielson and Lepanto. On June 25, 1957 counsel for
contracted during the war (Uy vs. Kalaw Katigbak, G.R. No. L-1830, Lepanto definitely advised Nielson that they were not entertaining any claim
Dec. 31, 1949) but modified Executive Order No. 32 as to pre-war of Nielson. The complaint in this case was filed on February 6, 1958.
debts, making the protection available only to debtors who had war
damage claims (Sison v. Mirasol, G.R. No. L-4711, Oct. 3, 1952).

687
4. In the sixth ground of its motion for reconsideration, Lepanto maintains because this evidence was not presented during the trial in the court below;
that this Court "erred in awarding as damages (a) 10% of the cash dividends (2) there is no showing that this piece of evidence is newly discovered and
declared and paid in December, 1941; (b) the management fee of P2,500.00 that Lepanto was not in possession of said evidence when this case was
for the month of January 1942; and (c) the full contract price for the extended being tried in the court below; and (3) according to Exhibit L cash dividends
period of 60 months, since the damages were never demanded nor proved of P175,000.00 were declared in December, 1941, and so the sum of
and, in any case, not allowable under the general law on damages." P17,500.00 which appears to have been paid to Nielson in October 1941
could not be payment of the equivalent of 10% of the cash dividends that
We have stated in Our decision that the original agreement in the were later declared in December, 1941.
management contract regarding the compensation of Nielson was modified,
such that instead of receiving a monthly compensation of P2,500.00 plus As regards the management fee of Nielson corresponding to January, 1942,
10% of the net profits from the operation of the properties for the preceding in the sum of P2,500.00, We have also found that Nielson is entitled to be
month,11 Nielson would receive a compensation of P2,500.00 a month, plus paid this amount, and that this amount was not paid by Lepanto to Nielson.
(1) 10% of the dividends declared and paid, when and as paid, during the Whereas, Lepanto was able to prove that it had paid the management fees of
period of the contract, and at the end of each year, (2) 10% of any depletion Nielson for November and December, 1941,13 it was not able to present any
reserve that may be set up, and (3) 10% of any amount expended during the evidence to show that the management fee of P2,500.00 for January, 1942
year out of surplus earnings for capital account. had been paid.

It is shown that in December, 1941, cash dividends amounting to It having been declared in Our decision, as well as in this resolution, that the
P175,000.00 was declared by Lepanto.12Nielson, therefore, should receive management contract had been extended for 5 years, or sixty months, from
the equivalent of 10% of this amount, or the sum of P17,500.00. We have June 27, 1948 to June 26, 1953, and that the cause of action of Nielson to
found that this amount was not paid to Nielson. claim for its compensation during that period of extension had not prescribed,
it follows that Nielson should be awarded the management fees during the
In its motion for reconsideration, Lepanto inserted a photographic copy of whole period of extension, plus the 10% of the value of the dividends
page 127 of its cash disbursement book, allegedly for 1941, in an effort to declared during the said period of extension, the 10% of the depletion
show that this amount of P17,500.00 had been paid to Nielson. It appears, reserve that was set up, and the 10% of any amount expended out of surplus
however, in this photographic copy of page 127 of the cash disbursement earnings for capital account.
book that the sum of P17,500.00 was entered on October 29 as "surplus a/c
Nielson & Co. Inc." The entry does not make any reference to dividends or 5. In the seventh ground of its motion for reconsideration, Lepanto maintains
participation of Nielson in the profits. On the other hand, in the photographic that this Court erred in ordering Lepanto to issue and deliver to Nielson
copy of page 89 of the 1941 cash disbursement book, also attached to the shares of stock together with fruits thereof.
motion for reconsideration, there is an entry for P17,500.00 on April 23, 1941
which states "Accts. Pay. Particip. Nielson & Co. Inc." This entry for April 23, In Our decision, We declared that pursuant to the modified agreement
1941 may really be the participation of Nielson in the profits based on regarding the compensation of Nielson which provides, among others, that
dividends declared in April 1941 as shown in Exhibit L. But in the same Nielson would receive 10% of any dividends declared and paid, when and as
Exhibit L it is not stated that any dividend was declared in October 1941. On paid, Nielson should be paid 10% of the stock dividends declared by Lepanto
the contrary it is stated in Exhibit L that dividends were declared in December during the period of extension of the contract.
1941. We cannot entertain this piece of evidence for several reasons: (1)

688
It is not denied that on November 28, 1949, Lepanto declared stock outstanding and entitled to vote at a general meeting of the
dividends worth P1,000,000.00; and on August 22, 1950, it declared stock corporation or at a special meeting duly called for the purpose.
dividends worth P2,000,000.00). In other words, during the period of
extension Lepanto had declared stock dividends worth P3,000,000.00. We xxx xxx xxx
held in Our decision that Nielson is entitled to receive l0% of the stock
dividends declared, or shares of stock worth P300,000.00 at the par value of No corporation shall make or declare any dividend except from the
P0.10 per share. We ordered Lepanto to issue and deliver to Nielson those surplus profits arising from its business, or divide or distribute its
shares of stocks as well as all the fruits or dividends that accrued to said capital stock or property other than actual profits among its members
shares. or stockholders until after the payment of its debts and the
termination of its existence by limitation or lawful
In its motion for reconsideration, Lepanto contends that the payment to dissolution: Provided, That banking, savings and loan, and trust
Nielson of stock dividends as compensation for its services under the corporations may receive deposits and issue certificates of deposit,
management contract is a violation of the Corporation Law, and that it was checks, drafts, and bills of exchange, and the like in the transaction
not, and it could not be, the intention of Lepanto and Nielson as of the ordinary business of banking, savings and loan, and trust
contracting parties that the services of Nielson should be paid in shares of corporations. (As amended by Act No. 2792, and Act No. 3518;
stock taken out of stock dividends declared by Lepanto. We have Emphasis supplied.)
assiduously considered the arguments adduced by Lepanto in support of its
contention, as well as the answer of Nielson in this connection, and We have From the above-quoted provision of Section 16 of the Corporation Law, the
arrived at the conclusion that there is merit in the contention of Lepanto. consideration for which shares of stock may be issued are: (1) cash; (2)
property; and (3) undistributed profits. Shares of stock are given the special
Section 16 of the Corporation Law, in part, provides as follows: name "stock dividends" only if they are issued in lieu of undistributed profits.
If shares of stocks are issued in exchange of cash or property then those
No corporation organized under this Act shall create or issue bills, shares do not fall under the category of "stock dividends". A corporation may
notes or other evidence of debt, for circulation as money, and no legally issue shares of stock in consideration of services rendered to it by a
corporation shall issue stock or bonds except in exchange for actual person not a stockholder, or in payment of its indebtedness. A share of stock
cash paid to the corporation or for: (1) property actually received by it issued to pay for services rendered is equivalent to a stock issued in
at a fair valuation equal to the par or issued value of the stock or exchange of property, because services is equivalent to property.14 Likewise
bonds so issued; and in case of disagreement as to their value, the a share of stock issued in payment of indebtedness is equivalent to issuing a
same shall be presumed to be the assessed value or the value stock in exchange for cash. But a share of stock thus issued should be part
appearing in invoices or other commercial documents, as the case of the original capital stock of the corporation upon its organization, or part of
may be; and the burden or proof that the real present value of the the stocks issued when the increase of the capitalization of a corporation is
property is greater than the assessed value or value appearing in properly authorized. In other words, it is the shares of stock that are originally
invoices or other commercial documents, as the case may be, shall issued by the corporation and forming part of the capital that can be
be upon the corporation, or for (2) profits earned by it but not exchanged for cash or services rendered, or property; that is, if the
distributed among its stockholders or members; Provided, however, corporation has original shares of stock unsold or unsubscribed, either
That no stock or bond dividend shall be issued without the approval coming from the original capitalization or from the increased capitalization.
of stockholders representing not less than two-thirds of all stock then Those shares of stock may be issued to a person who is not a stockholder, or

689
to a person already a stockholder in exchange for services rendered or for It is Our considered view, therefore, that under Section 16 of the Corporation
cash or property. But a share of stock coming from stock dividends declared Law stock dividends can not be issued to a person who is not a stockholder
cannot be issued to one who is not a stockholder of a corporation. in payment of services rendered. And so, in the case at bar Nielson can not
be paid in shares of stock which form part of the stock dividends of Lepanto
A "stock dividend" is any dividend payable in shares of stock of the for services it rendered under the management contract. We sustain the
corporation declaring or authorizing such dividend. It is, what the term itself contention of Lepanto that the understanding between Lepanto and Nielson
implies, a distribution of the shares of stock of the corporation among the was simply to make the cash value of the stock dividends declared as the
stockholders as dividends. A stock dividend of a corporation is a dividend basis for determining the amount of compensation that should be paid to
paid in shares of stock instead of cash, and is properly payable only out of Nielson, in the proportion of 10% of the cash value of the stock dividends
surplus profits.15 So, a stock dividend is actually two things: (1) a dividend, declared. And this conclusion of Ours finds support in the record.
and (2) the enforced use of the dividend money to purchase additional
shares of stock at par.16 When a corporation issues stock dividends, it shows We had adverted to in Our decision that in 1940 there was some dispute
that the corporation's accumulated profits have been capitalized instead of between Lepanto and Nielson regarding the application and interpretation of
distributed to the stockholders or retained as surplus available for certain provisions of the original contract particularly with regard to the 10%
distribution, in money or kind, should opportunity offer. Far from being a participation of Nielson in the net profits, so that some adjustments had to be
realization of profits for the stockholder, it tends rather to postpone said made. In the minutes of the meeting of the Board of Directors of Lepanto on
realization, in that the fund represented by the new stock has been August 21, 1940, We read the following:
transferred from surplus to assets and no longer available for actual
distribution.17 Thus, it is apparent that stock dividends are issued only to The Chairman stated that he believed that it would be better to tie
stockholders. This is so because only stockholders are entitled to dividends. the computation of the 10% participation of Nielson & Company, Inc.
They are the only ones who have a right to a proportional share in that part of to the dividend, because Nielson will then be able to definitely
the surplus which is declared as dividends. A stock dividend really adds compute its net participation by the amount of the dividends
nothing to the interest of the stockholder; the proportional interest of each declared. In addition to the dividend, we have been setting up a
stockholder remains the same.18If a stockholder is deprived of his stock depletion reserve and it does not seem fair to burden the 10%
dividends - and this happens if the shares of stock forming part of the stock participation of Nielson with the depletion reserve, as the depletion
dividends are issued to a non-stockholder then the proportion of the reserve should not be considered as an operating expense. After a
stockholder's interest changes radically. Stock dividends are civil fruits of the prolonged discussion, upon motion duly made and seconded, it was
original investment, and to the owners of the shares belong the civil fruits. 19

The term "dividend" both in the technical sense and its ordinary acceptation, RESOLVED, That the President, be, and he hereby is, authorized to
is that part or portion of the profits of the enterprise which the corporation, by enter into an agreement with Nielson & Company, Inc., modifying
its governing agents, sets apart for ratable division among the holders of the Paragraph V of management contract of January 30, 1937, effective
capital stock. It means the fund actually set aside, and declared by the January 1, 1940, in such a way that Nielson & Company, Inc. shall
directors of the corporation as dividends and duly ordered by the director, or receive 10% of any dividends declared and paid, when and as paid
by the stockholders at a corporate meeting, to be divided or distributed during the period of the contract and at the end of each year, 10% of
among the stockholders according to their respective interests. 20 any depletion reserve that may be set up and 10% of any amount

690
expended during the year out of surplus earnings for capital account. We, therefore, reconsider that part of Our decision which declares that
(Emphasis supplied.) Nielson is entitled to shares of stock worth P300,000.00 based on the stock
dividends declared on November 28, 1949 and on August 20, 1950, together
From the sentence, "The Chairman stated that he believed that it would be with all the fruits accruing thereto. Instead, We declare that Nielson is entitled
better to tie the computation of the 10% participation of Nielson & Company, to payment by Lepanto of P300,000.00 in cash, which is equivalent to 10% of
Inc., to the dividend, because Nielson will then be able to definitely compute the money value of the stock dividends worth P3,000,000.00 which were
its net participation by the amount of the dividends declared" the idea is declared on November 28, 1949 and on August 20, 1950, with interest
conveyed that the intention of Lepanto, as expressed by its Chairman C. A. thereon at the rate of 6% from February 6, 1958.
DeWitt, was to make the value of the dividends declared whether the
dividends were in cash or in stock as the basis for determining the amount 6. In the eighth ground of its motion for reconsideration Lepanto maintains
of compensation that should be paid to Nielson, in the proportion of 10% of that this Court erred in awarding to Nielson an undetermined amount of
the cash value of the dividends so declared. It does not mean, however, that shares of stock and/or cash, which award can not be ascertained and
the compensation of Nielson would be taken from the amount actually executed without further litigation.
declared as cash dividend to be distributed to the stockholder, nor from the
shares of stocks to be issued to the stockholders as stock dividends, but In view of Our ruling in this resolution that Nielson is not entitled to receive
from the other assets or funds of the corporation which are not burdened by shares of stock as stock dividends in payment of its compensation under the
the dividends thus declared. In other words, if, for example, cash dividends of management contract, We do not consider it necessary to discuss this
P300,000.00 are declared, Nielson would be entitled to a compensation of ground of the motion for reconsideration. The awards in the present case are
P30,000.00, but this P30,000.00 should not be taken from the P300,000.00 all reduced to specific sums of money.
to be distributed as cash dividends to the stockholders but from some other
funds or assets of the corporation which are not included in the amount to 7. In the ninth ground of its motion for reconsideration Lepanto maintains that
answer for the cash dividends thus declared. This is so because if the this Court erred in rendering judgment or attorney's fees.
P30,000.00 would be taken out from the P300,000.00 declared as cash
dividends, then the stockholders would not be getting P300,000.00 as
The matter of the award of attorney's fees is within the sound discretion of
dividends but only P270,000.00. There would be a dilution of the dividend
this Court. In Our decision We have stated the reason why the award of
that corresponds to each share of stock held by the stockholders. Similarly, if
P50,000.00 for attorney's fees is considered by this Court as reasonable.
there were stock dividends worth one million pesos that were declared, which
means an issuance of ten million shares at the par value of ten centavos per
share, it does not mean that Nielson would be given 100,000 shares. It only Accordingly, We resolve to modify the decision that We rendered on
means that Nielson should be given the equivalent of 10% of the aggregate December 17, 1966, in the sense that instead of awarding Nielson shares of
cash value of those shares issued as stock dividends. That this was the stock worth P300,000.00 at the par value of ten centavos (P0.10) per share
understanding of Nielson itself is borne out by the fact that in its appeal brief based on the stock dividends declared by Lepanto on November 28, 1949
Nielson urged that it should be paid "P300,000.00 being 10% of the and August 20, 1950, together with their fruits, Nielson should be awarded
P3,000,000.00 stock dividends declared on November 28, 1949 and August the sum of P300,000.00 which is an amount equivalent to 10% of the cash
20, 1950...."21 value of the stock dividends thus declared, as part of the compensation due
Nielson under the management contract. The dispositive portion of the
decision should, therefore, be amended, to read as follows:

691
IN VIEW OF THE FOREGOING CONSIDERATIONS, We hereby reverse the (7) Six hundred ninety four thousand three hundred sixty four pesos and
decision of the court a quo and enter in lieu thereof another, ordering the seventy six centavos (P694,364.76), equivalent to 10% of the expenses for
appellee Lepanto to pay the appellant Nielson the different amounts as capital account during the period of extension, with legal interest thereon
specified hereinbelow: from the date of the filing of the complaint;

(1) Seventeen thousand five hundred pesos (P17,500.00), equivalent to 10% (8) Fifty thousand pesos (P50,000.00) as attorney's fees; and
of the cash dividends of December, 1941, with legal interest thereon from the
date of the filing of the complaint; (9) The costs.

(2) Two thousand five hundred pesos (P2,500.00) as management fee for It is so ordered.
January 1942, with legal interest thereon from the date of the filing of the
complaint; Concepcion, C.J., Reyes, J.B.L., Dizon, Makalintal, Sanchez and Castro,
JJ., concur.
(3) One hundred fifty thousand pesos (P150,000.00), representing
management fees for the sixty-month period of extension of the management Fernando, Capistrano, Teehankee and Barredo, JJ., took no part.
contract, with legal interest thereon from the date of the filing of the
complaint;

(4) One million four hundred thousand pesos (P1,400,000.00), equivalent to


10% of the cash dividends declared during the period of extension of the
management contract, with legal interest thereon from the date of the filing of
the complaint; G.R. No. 108576 January 20, 1999

(5) Three hundred thousand pesos (P300,000.00), equivalent to 10% of the COMMISSIONER OF INTERNAL REVENUE, petitioner,
cash value of the stock dividends declared on November 28, 1949 and vs.
August 20, 1950, with legal interest thereon from the date of the filing of the THE COURT OF APPEALS, COURT OF TAX APPEALS and A. SORIANO
complaint; CORP., respondents.

(6) Fifty three thousand nine hundred twenty eight pesos and eighty eight
centavos (P53,928.88), equivalent to 10% of the depletion reserve set up
during the period of extension, with legal interest thereon from the date of the MARTINEZ, J.:
filing of the complaint;
Petitioner Commissioner of Internal Revenue (CIR) seeks the reversal of the
decision of the Court of Appeals (CA) 1 which affirmed the ruling of the Court
of Tax Appeals (CTA) 2 that private respondent A. Soriano Corporation's
(hereinafter ANSCOR) redemption and exchange of the stocks of its foreign

692
stockholders cannot be considered as "essentially equivalent to a distribution ANSCOR. Hence, increasing their accumulated shareholdings to 138,867
of taxable dividends" under, Section 83(b) of the 1939 Internal Revenue Act. 3 and 138,864 19 common shares each. 20

The undisputed facts are as follows: On December 28, 1967, Doa Carmen requested a ruling from the United
States Internal Revenue Service (IRS), inquiring if an exchange of common
Sometime in the 1930s, Don Andres Soriano, a citizen and resident of the with preferred shares may be considered as a tax avoidance scheme 21under
United States, formed the corporation "A. Soriano Y Cia", predecessor of Section 367 of the 1954 U.S. Revenue Act. 22 By January 2, 1968, ANSCOR
ANSCOR, with a P1,000,000.00 capitalization divided into 10,000 common reclassified its existing 300,000 common shares into 150,000 common and
shares at a par value of P100/share. ANSCOR is wholly owned and 150,000 preferred shares. 23
controlled by the family of Don Andres, who are all non-resident aliens. 4 In
1937, Don Andres subscribed to 4,963 shares of the 5,000 shares originally In a letter-reply dated February 1968, the IRS opined that the exchange is
issued. 5 only a recapitalization scheme and not tax avoidance. 24 Consequently, 25 on
March 31, 1968 Doa Carmen exchanged her whole 138,864 common
On September 12, 1945, ANSCOR's authorized capital stock was increased shares for 138,860 of the newly reclassified preferred shares. The estate of
to P2,500,000.00 divided into 25,000 common shares with the same par Don Andres in turn, exchanged 11,140 of its common shares, for the
value of the additional 15,000 shares, only 10,000 was issued which were all remaining 11,140 preferred shares, thus reducing its (the estate) common
subscribed by Don Andres, after the other stockholders waived in favor of the shares to 127,727. 26
former their pre-emptive rights to subscribe to the new issues. 6 This
increased his subscription to 14,963 common shares. 7 A month later, 8 Don On June 30, 1968, pursuant to a Board Resolution, ANSCOR redeemed
Andres transferred 1,250 shares each to his two sons, Jose and Andres, Jr., 28,000 common shares from the Don Andres' estate. By November 1968, the
as their initial investments in ANSCOR. 9 Both sons are foreigners. 10 Board further increased ANSCOR's capital stock to P75M divided into
150,000 preferred shares and 600,000 common shares. 27 About a year later,
By 1947, ANSCOR declared stock dividends. Other stock dividend ANSCOR again redeemed 80,000 common shares from the Don Andres'
declarations were made between 1949 and December 20, 1963. 11 On estate, 28 further reducing the latter's common shareholdings to 19,727. As
December 30, 1964 Don Andres died. As of that date, the records revealed stated in the Board Resolutions, ANSCOR's business purpose for both
that he has a total shareholdings of 185,154 shares 12 50,495 of which are redemptions of stocks is to partially retire said stocks as treasury shares in
original issues and the balance of 134.659 shares as stock dividend order to reduce the company's foreign exchange remittances in case cash
declarations. 13Correspondingly, one-half of that shareholdings or dividends are declared. 29
92,577 14 shares were transferred to his wife, Doa Carmen Soriano, as her
conjugal share. The other half formed part of his estate. 15 In 1973, after examining ANSCOR's books of account and records, Revenue
examiners issued a report proposing that ANSCOR be assessed for
A day after Don Andres died, ANSCOR increased its capital stock to deficiency withholding tax-at-source, pursuant to Sections 53 and 54 of the
P20M 16 and in 1966 further increased it to P30M. 17 In the same year 1939 Revenue Code, 30 for the year 1968 and the second quarter of 1969
(December 1966), stock dividends worth 46,290 and 46,287 shares were based on the transactions of exchange 31 and redemption of stocks. 31The
respectively received by the Don Andres estate 18 and Doa Carmen from Bureau of Internal Revenue (BIR) made the corresponding assessments
despite the claim of ANSCOR that it availed of the tax amnesty under

693
Presidential Decree Specifically, the issue is whether ANSCOR's redemption of stocks
(P.D.) 23 32 which were amended by P.D.'s 67 and 157. 33 However, petitioner from its stockholder as well as the exchange of common with
ruled that the invoked decrees do not cover Sections 53 and 54 in relation to preferred shares can be considered as "essentially equivalent to the
Article 83(b) of the 1939 Revenue Act under which ANSCOR was distribution of taxable dividend" making the proceeds thereof taxable
assessed. 34ANSCOR's subsequent protest on the assessments was denied under the provisions of the above-quoted law.
in 1983 by petitioner. 35
Petitioner contends that the exchange transaction a tantamount to
Subsequently, ANSCOR filed a petition for review with the CTA assailing the "cancellation" under Section 83(b) making the proceeds thereof taxable. It
tax assessments on the redemptions and exchange of stocks. In its decision, also argues that the Section applies to stock dividends which is the bulk of
the Tax Court reversed petitioner's ruling, after finding sufficient evidence to stocks that ANSCOR redeemed. Further, petitioner claims that under the "net
overcome the prima facie correctness of the questioned assessments. 36 In a effect test," the estate of Don Andres gained from the redemption.
petition for review the CA as mentioned, affirmed the ruling of the Accordingly, it was the duty of ANSCOR to withhold the tax-at-source arising
CTA. 37 Hence, this petition. from the two transactions, pursuant to Section 53 and 54 of the 1939
Revenue Act. 39
The bone of contention is the interpretation and application of Section 83(b)
of the 1939 Revenue Act 38 which provides: ANSCOR, however, avers that it has no duty to withhold any tax either from
the Don Andres estate or from Doa Carmen based on the two transactions,
Sec. 83. Distribution of dividends or assets by corporations. because the same were done for legitimate business purposes which are (a)
to reduce its foreign exchange remittances in the event the company would
declare cash dividends, 40 and to (b) subsequently "filipinized" ownership of
(b) Stock dividends A stock dividend representing the ANSCOR, as allegedly, envisioned by Don Andres. 41 It likewise invoked the
transfer of surplus to capital account shall not be subject to amnesty provisions of P.D. 67.
tax. However, if a corporation cancels or redeems stock
issued as a dividend at such time and in such manner as to We must emphasize that the application of Sec. 83(b) depends on the
make the distribution and cancellation or redemption, in special factual circumstances of each case. 42 The findings of facts of a
whole or in part, essentially equivalent to the distribution of special court (CTA) exercising particular expertise on the subject of tax,
a taxable dividend, the amount so distributed in redemption generally binds this Court, 43 considering that it is substantially similar to the
or cancellation of the stock shall be considered as taxable findings of the CA which is the final arbiter of questions of facts. 44 The issue
income to the extent it represents a distribution of earnings in this case does not only deal with facts but whether the law applies to a
or profits accumulated after March first, nineteen hundred particular set of facts. Moreover, this Court is not necessarily bound by the
and thirteen. (Emphasis supplied) lower courts' conclusions of law drawn from such facts. 45

AMNESTY:

We will deal first with the issue of tax amnesty. Section 1 of P.D.
67 46 provides:

694
1. In all cases of voluntary disclosures of previously untaxed the taxing authority. 51 In other words, the withholding agent is merely a tax
income and/or wealth such as earnings, receipts, gifts, collector, not a taxpayer. Under the withholding system, however, the agent-
bequests or any other acquisitions from any source payor becomes a payee by fiction of law. His (agent) liability is direct and
whatsoever which are taxable under the National Internal independent from the taxpayer, 52 because the income tax is still impose on
Revenue Code, as amended, realized here or abroad by any and due from the latter. The agent is not liable for the tax as no wealth flowed
taxpayer, natural or judicial; the collection of all internal into him he earned no income. The Tax Code only makes the agent
revenue taxes including the increments or penalties or personally liable for the tax 53arising from the breach of its legal duty to
account of non-payment as well as all civil, criminal or withhold as distinguish from its duty to pay tax since:
administrative liabilities arising from or incident to such
disclosures under the National Internal Revenue Code, the the government's cause of action against the withholding is
Revised Penal Code, the Anti-Graft and Corrupt Practices not for the collection of income tax, but for the enforcement
Act, the Revised Administrative Code, the Civil Service laws of the withholding provision of Section 53 of the Tax Code,
and regulations, laws and regulations on Immigration and compliance with which is imposed on the withholding agent
Deportation, or any other applicable law or proclamation, are and not upon the taxpayer. 54
hereby condoned and, in lieu thereof, a tax of ten (10%) per
centum on such previously untaxed income or wealth, is Not being a taxpayer, a withholding agent, like ANSCOR in this
hereby imposed, subject to the following conditions: transaction is not protected by the amnesty under the decree.
(conditions omitted) [Emphasis supplied].
Codal provisions on withholding tax are mandatory and must be complied
The decree condones "the collection of all internal revenue taxes with by the withholding agent. 55 The taxpayer should not answer for the non-
including the increments or penalties or account of non-payment as performance by the withholding agent of its legal duty to withhold unless
well as all civil, criminal or administrative liable arising from or there is collusion or bad faith. The former could not be deemed to have
incident to" (voluntary) disclosures under the NIRC of previously evaded the tax had the withholding agent performed its duty. This could be
untaxed income and/or wealth "realized here or abroad by any the situation for which the amnesty decree was intended. Thus, to curtail tax
taxpayer, natural or juridical." evasion and give tax evaders a chance to reform, 56 it was deemed
administratively feasible to grant tax amnesty in certain instances. In
May the withholding agent, in such capacity, be deemed a taxpayer for it to addition, a "tax amnesty, much like a tax exemption, is never favored nor
avail of the amnesty? An income taxpayer covers all persons who derive presumed in law and if granted by a statute, the term of the amnesty like that
taxable income. 47 ANSCOR was assessed by petitioner for deficiency of a tax exemption must be construed strictly against the taxpayer and
withholding tax under Section 53 and 54 of the 1939 Code. As such, it is liberally in favor of the taxing authority. 57 The rule on strictissimi juris equally
being held liable in its capacity as a withholding agent and not its personality applies. 58 So that, any doubt in the application of an amnesty law/decree
as a taxpayer. should be resolved in favor of the taxing authority.

In the operation of the withholding tax system, the withholding agent is the Furthermore, ANSCOR's claim of amnesty cannot prosper.
payor, a separate entity acting no more than an agent of the government for The implementing rules of P.D. 370 which expanded
the collection of the tax 48 in order to ensure its payments; 49 the payer is the
taxpayer he is the person subject to tax impose by law; 50 and the payee is

695
amnesty on previously untaxed income under P.D. 23 is very capital
explicit, to wit: investment." 65 As capital, the stock dividends postpone the realization of
profits because the "fund represented by the new stock has been transferred
Sec. 4. Cases not covered by amnesty. The following from surplus to capital and no longer available for actual
cases are not covered by the amnesty subject of these distribution." 66 Income in tax law is "an amount of money coming to a person
regulations: within a specified time, whether as payment for services, interest, or profit
from investment." 67 It means cash or its equivalent. 68 It is gain derived and
xxx xxx xxx severed from capital, 69 from labor or from both combined 70 so that to tax
a stock dividend would be to tax a capital increase rather than the
income. 71 In a loose sense, stock dividends issued by the corporation, are
(2) Tax liabilities with or without assessments, on withholding
considered unrealized gain, and cannot be subjected to income tax until that
tax at source provided under Section 53 and 54 of the
gain has been realized. Before the realization, stock dividends are nothing
National Internal Revenue Code, as amended; 59
but a representation of an interest in the corporate properties. 72 As capital, it
is not yet subject to income tax. It should be noted that capital and income
ANSCOR was assessed under Sections 53 and 54 of the 1939 Tax are different. Capital is wealth or fund; whereas income is profit or gain or the
Code. Thus, by specific provision of law, it is not covered by the flow of wealth.73 The determining factor for the imposition of income tax is
amnesty. whether any gain or profit was derived from a transaction. 74

TAX ON STOCK DIVIDENDS The Exception

General Rule However, if a corporation cancels or redeems stock issued


as a dividend at such time and in such manner as to make
Sec. 83(b) of the 1939 NIRC was taken from the Section 115(g)(1) of the the distribution and cancellation or redemption, in whole or in
U.S. Revenue Code of 1928. 60 It laid down the general rule known as the part, essentially equivalent to the distribution of a taxable
proportionate test 61 wherein stock dividends once issued form part of the dividend, the amount so distributed in redemption or
capital and, thus, subject to income tax. 62 Specifically, the general rule states cancellation of the stock shall be considered as taxable
that: income to the extent it represents a distribution of earnings
or profits accumulated after March first, nineteen hundred
A stock dividend representing the transfer of surplus to and thirteen. (Emphasis supplied).
capital account shall not be subject to tax.
In a response to the ruling of the American Supreme Court in the case
Having been derived from a foreign law, resort to the jurisprudence of its of Eisner v. Macomber 75 (that pro ratastock dividends are not taxable
origin may shed light. Under the US Revenue Code, this provision originally income), the exempting clause above quoted was added because provision
referred to "stock dividends" only, without any exception. Stock dividends, corporation found a loophole in the original provision. They resorted to
strictly speaking, represent capital and do not constitute income to its devious means to circumvent the law and evade the tax. Corporate earnings
recipient. 63 So that the mere issuance thereof is not yet subject to income would be distributed under the guise of its initial capitalization by declaring
tax 64 as they are nothing but an "enrichment through increase in value of the stock dividends previously issued and later redeem said dividends by

696
paying cash to the stockholder. This process of issuance-redemption of a "taxable dividend" is a question of fact, 82 which is determinable on "the
amounts to a distribution of taxable cash dividends which was lust delayed basis of the particular facts of the transaction in question. 83 No decisive test
so as to escape the tax. It becomes a convenient technical strategy to avoid can be used to determine the application of the exemption under Section
the effects of taxation. 83(b). The use of the words "such manner" and "essentially equivalent"
negative any idea that a weighted formula can resolve a crucial issue
Thus, to plug the loophole the exempting clause was added. It provides Should the distribution be treated as taxable dividend. 84 On this aspect,
that the redemption or cancellation of stock dividends, depending on the American courts developed certain recognized criteria, which includes the
"time" and "manner" it was made, is essentially equivalent to a distribution of following: 85
taxable dividends," making the proceeds thereof "taxable income" "to the
extent it represents profits". The exception was designed to prevent the 1) the presence or absence of real business
issuance and cancellation or redemption of stock dividends, which is purpose,
fundamentally not taxable, from being made use of as a device for the actual
distribution of cash dividends, which is taxable. 76 Thus, 2) the amount of earnings and profits
available for the declaration of a regular
the provision had the obvious purpose of preventing a dividends and the corporation's past record
corporation from avoiding dividend tax treatment by with respect to the declaration of dividends,
distributing earnings to its shareholders in two transactions
a pro rata stock dividend followed by a pro 3) the effect of the distribution, as compared
rata redemption that would have the same economic with the declaration of regular dividend,
consequences as a simple dividend. 77
4) the lapse of time between issuance and
Although redemption and cancellation are generally considered redemption, 86
capital transactions, as such. they are not subject to tax. However, it
does not necessarily mean that a shareholder may not realize a 5) the presence of a substantial
taxable gain from such transactions. 78 Simply put, depending on the surplus 87 and a generous supply of cash
circumstances, the proceeds of redemption of stock dividends are which invites suspicion as does a meager
essentially distribution of cash dividends, which when paid becomes policy in relation both to current earnings
the absolute property of the stockholder. Thereafter, the latter and accumulated surplus, 88
becomes the exclusive owner thereof and can exercise the freedom
of choice. 79Having realized gain from that redemption, the income
REDEMPTION AND CANCELLATION
earner cannot escape income tax. 80

For the exempting clause of Section, 83(b) to apply, it is


As qualified by the phrase "such time and in such manner," the exception
indispensable that: (a) there is redemption or cancellation; (b) the
was not intended to characterize as taxable dividend every distribution of
transaction involves stock dividends and (c) the "time and manner" of
earnings arising from the redemption of stock dividend. 81 So that, whether
the transaction makes it "essentially equivalent to a distribution of
the amount distributed in the redemption should be treated as the equivalent
taxable dividends." Of these, the most important is the third.

697
Redemption is repurchase, a reacquisition of stock by a corporation which issuance and the redemption. The "time" element is a factor to show a device
issued the stock 89 in exchange for property, whether or not the acquired to evade tax and the scheme of cancelling or redeeming the same shares is
stock is cancelled, retired or held in the treasury. 90 Essentially, the a method usually adopted to accomplish the end sought. 96 Was this
corporation gets back some of its stock, distributes cash or property to the transaction used as a "continuing plan," "device" or "artifice" to evade
shareholder in payment for the stock, and continues in business as before. payment of tax? It is necessary to determine the "net effect" of the
The redemption of stock dividends previously issued is used as a veil for the transaction between the shareholder-income taxpayer and the acquiring
constructive distribution of cash dividends. In the instant case, there is no (redeeming) corporation. 97 The "net effect" test is not evidence or testimony
dispute that ANSCOR redeemed shares of stocks from a stockholder (Don to be considered; it is rather an inference to be drawn or a conclusion to be
Andres) twice (28,000 and 80,000 common shares). But where did the reached. 98 It is also important to know whether the issuance of stock
shares redeemed come from? If its source is the original capital subscriptions dividends was dictated by legitimate business reasons, the presence of
upon establishment of the corporation or from initial capital investment in an which might negate a tax evasion plan. 99
existing enterprise, its redemption to the concurrent value of acquisition may
not invite the application of Sec. 83(b) under the 1939 Tax Code, as it is not The issuance of stock dividends and its subsequent redemption must be
income but a mere return of capital. On the contrary, if the redeemed shares separate, distinct, and not related, for the redemption to be considered a
are from stock dividend declarations other than as initial capital investment, legitimate tax scheme. 100 Redemption cannot be used as a cloak to
the proceeds of the redemption is additional wealth, for it is not merely a distribute corporate earnings. 101 Otherwise, the apparent intention to avoid
return of capital but a gain thereon. tax becomes doubtful as the intention to evade becomes manifest. It has
been ruled that:
It is not the stock dividends but the proceeds of its redemption that may be
deemed as taxable dividends. Here, it is undisputed that at the time of the [A]n operation with no business or corporate purpose is a
last redemption, the original common shares owned by the estate were only mere devise which put on the form of a corporate
25,247.5 91 This means that from the total of 108,000 shares redeemed from reorganization as a disguise for concealing its real character,
the estate, the balance of 82,752.5 (108,000 less 25,247.5) must have come and the sole object and accomplishment of which was the
from stock dividends. Besides, in the absence of evidence to the contrary, consummation of a preconceived plan, not to reorganize a
the Tax Code presumes that every distribution of corporate property, in whole business or any part of a business, but to transfer a parcel of
or in part, is made out of corporate profits 92such as stock dividends. The corporate shares to a stockholder. 102
capital cannot be distributed in the form of redemption of stock dividends
without violating the trust fund doctrine wherein the capital stock, property Depending on each case, the exempting provision of Sec. 83(b) of the 1939
and other assets of the corporation are regarded as equity in trust for the Code may not be applicable if the redeemed shares were issued with bona
payment of the corporate creditors. 93 Once capital, it is always fide business purpose, 103 which is judged after each and every step of the
capital. 94 That doctrine was intended for the protection of corporate transaction have been considered and the whole transaction does not
creditors. 95 amount to a tax evasion scheme.

With respect to the third requisite, ANSCOR redeemed stock dividends ANSCOR invoked two reasons to justify the redemptions (1) the alleged
issued just 2 to 3 years earlier. The time alone that lapsed from the issuance "filipinization" program and (2) the reduction of foreign exchange remittances
to the redemption is not a sufficient indicator to determine taxability. It is a in case cash dividends are declared. The Court is not concerned with the
must to consider the factual circumstances as to the manner of both the

698
wisdom of these purposes but on their relevance to the whole transaction legitimate business reasons would defeat the very purpose of imposing tax
which can be inferred from the outcome thereof. Again, it is the "net effect on income. Such argument would open the door for income earners not to
rather than the motives and plans of the taxpayer or his corporation" 104 that pay tax so long as the person from whom the income was derived has
is the fundamental guide in administering Sec. 83(b). This tax provision is legitimate business reasons. In other words, the payment of tax under the
aimed at the result. 105 It also applies even if at the time of the issuance of the exempting clause of Section 83(b) would be made to depend not on the
stock dividend, there was no intention to redeem it as a means of distributing income of the taxpayer, but on the business purposes of a third party (the
profit or avoiding tax on dividends. 106 The existence of legitimate business corporation herein) from whom the income was earned. This is absurd,
purposes in support of the redemption of stock dividends is immaterial in illogical and impractical considering that the Bureau of Internal Revenue
income taxation. It has no relevance in determining "dividend (BIR) would be pestered with instances in determining the legitimacy of
equivalence". 107 Such purposes may be material only upon the issuance of business reasons that every income earner may interposed. It is not
the stock dividends. The test of taxability under the exempting clause, when administratively feasible and cannot therefore be allowed.
it provides "such time and manner" as would make the redemption
"essentially equivalent to the distribution of a taxable dividend", is whether The ruling in the American cases cited and relied upon by ANSCOR that "the
the redemption resulted into a flow of wealth. If no wealth is realized from the redeemed shares are the equivalent of dividend only if the shares were
redemption, there may not be a dividend equivalence treatment. In the not issued for genuine business purposes", 111 or the "redeemed shares have
metaphor of Eisner v. Macomber, income is not deemed "realize" until the been issued by a corporation bona fide" 112 bears no relevance in determining
fruit has fallen or been plucked from the tree. the non-taxability of the proceeds of redemption ANSCOR, relying heavily
and applying said cases, argued that so long as the redemption is supported
The three elements in the imposition of income tax are: (1) there must be by valid corporate purposes the proceeds are not subject to tax. 113 The
gain or and profit, (2) that the gain or profit is realized or received, actually or adoption by the courts below 114 of such argument is misleading if not
constructively, 108 and (3) it is not exempted by law or treaty from income tax. misplaced. A review of the cited American cases shows that the presence or
Any business purpose as to why or how the income was earned by the absence of "genuine business purposes" may be material with respect to the
taxpayer is not a requirement. Income tax is assessed on income received issuance or declaration of stock dividends but not on its subsequent
from any property, activity or service that produces the income because the redemption. The issuance and the redemption of stocks are two different
Tax Code stands as an indifferent neutral party on the matter of where transactions. Although the existence of legitimate corporate purposes may
income comes justify a corporation's acquisition of its own shares under Section 41 of the
from. 109 Corporation Code, 115such purposes cannot excuse the stockholder from the
effects of taxation arising from the redemption. If the issuance of stock
As stated above, the test of taxability under the exempting clause of Section dividends is part of a tax evasion plan and thus, without legitimate business
83(b) is, whether income was realized through the redemption of stock reasons, the redemption becomes suspicious which exempting clause. The
dividends. The redemption converts into money the stock dividends which substance of the whole transaction, not its form, usually controls the tax
become a realized profit or gain and consequently, the stockholder's consequences. 116
separate property. 110 Profits derived from the capital invested cannot escape
income tax. As realized income, the proceeds of the redeemed stock The two purposes invoked by ANSCOR, under the facts of this case are no
dividends can be reached by income taxation regardless of the existence of excuse for its tax liability. First, the alleged "filipinization" plan cannot be
any business purpose for the redemption. Otherwise, to rule that the said considered legitimate as it was not implemented until the BIR started making
proceeds are exempt from income tax when the redemption is supported by assessments on the proceeds of the redemption. Such corporate plan was

699
not stated in nor supported by any Board Resolution but a mere afterthought company. But the unfairness may not be true to an original subscriber like
interposed by the counsel of ANSCOR. Being a separate entity, the Don Andres, who holds stock dividends as gains from his investments. The
corporation can act only through its Board of Directors. 117 The Board subsequent buyer who buys stock dividends is investing capital. It just so
Resolutions authorizing the redemptions state only one purpose reduction happen that what he bought is stock dividends. The effect of its (stock
of foreign exchange remittances in case cash dividends are declared. Not dividends) redemption from that subsequent buyer is merely to return his
even this purpose can be given credence. Records show that despite the capital subscription, which is income if redeemed from the original
existence of enormous corporate profits no cash dividend was ever declared subscriber.
by ANSCOR from 1945 until the BIR started making assessments in the early
1970's. Although a corporation under certain exceptions, has the prerogative After considering the manner and the circumstances by which the issuance
when to issue dividends, yet when no cash dividends was issued for about and redemption of stock dividends were made, there is no other conclusion
three decades, this circumstance negates the legitimacy of ANSCOR's but that the proceeds thereof are essentially considered equivalent to a
alleged purposes. Moreover, to issue stock dividends is to increase the distribution of taxable dividends. As "taxable dividend" under Section 83(b), it
shareholdings of ANSCOR's foreign stockholders contrary to its "filipinization" is part of the "entire income" subject to tax under Section 22 in relation to
plan. This would also increase rather than reduce their need for foreign Section 21 120 of the 1939 Code. Moreover, under Section 29(a) of said Code,
exchange remittances in case of cash dividend declaration, considering that dividends are included in "gross income". As income, it is subject to income
ANSCOR is a family corporation where the majority shares at the time of tax which is required to be withheld at source. The 1997 Tax Code may have
redemptions were held by Don Andres' foreign heirs. altered the situation but it does not change this disposition.

Secondly, assuming arguendo, that those business purposes are legitimate, EXCHANGE OF COMMON WITH PREFERRED SHARES 121

the same cannot be a valid excuse for the imposition of tax. Otherwise, the
taxpayer's liability to pay income tax would be made to depend upon a third Exchange is an act of taking or giving one thing for another
person who did not earn the income being taxed. Furthermore, even if the involving 122 reciprocal transfer 123 and is generally considered as a taxable
said purposes support the redemption and justify the issuance of stock transaction. The exchange of common stocks with preferred stocks, or
dividends, the same has no bearing whatsoever on the imposition of the tax preferred for common or a combination of either for both, may not produce a
herein assessed because the proceeds of the redemption are deemed recognized gain or loss, so long as the provisions of Section 83(b) is not
taxable dividends since it was shown that income was generated therefrom. applicable. This is true in a trade between two (2) persons as well as a trade
between a stockholder and a corporation. In general, this trade must be parts
Thirdly, ANSCOR argued that to treat as "taxable dividend" the proceeds of of merger, transfer to controlled corporation, corporate acquisitions or
the redeemed stock dividends would be to impose on such stock an corporate reorganizations. No taxable gain or loss may be recognized on
undisclosed lien and would be extremely unfair to intervening purchase, i.e. exchange of property, stock or securities related to reorganizations. 124
those who buys the stock dividends after their issuance. 118 Such argument,
however, bears no relevance in this case as no intervening buyer is involved. Both the Tax Court and the Court of Appeals found that ANSCOR reclassified
And even if there is an intervening buyer, it is necessary to look into the its shares into common and preferred, and that parts of the common shares
factual milieu of the case if income was realized from the transaction. Again, of the Don Andres estate and all of Doa Carmen's shares were exchanged
we reiterate that the dividend equivalence test depends on such "time and for the whole 150.000 preferred shares. Thereafter, both the Don Andres
manner" of the transaction and its net effect. The undisclosed lien 119 may be estate and Doa Carmen remained as corporate subscribers except that
unfair to a subsequent stock buyer who has no capital interest in the

700
their subscriptions now include preferred shares. There was no change in herein considered as essentially equivalent to a distribution of taxable
their proportional interest after the exchange. There was no cash flow. Both dividends for which it is LIABLE for the withholding tax-at-source. The
stocks had the same par value. Under the facts herein, any difference in their decision is AFFIRMED in all other respects.
market value would be immaterial at the time of exchange because no
income is yet realized it was a mere corporate paper transaction. It would SO ORDERED.
have been different, if the exchange transaction resulted into a flow of
wealth, in which case income tax may be imposed. 125 Davide, Jr., C.J., Melo, Kapunan and Pardo, JJ., concur.

Reclassification of shares does not always bring any substantial alteration in


the subscriber's proportional interest. But the exchange is different there
would be a shifting of the balance of stock features, like priority in dividend
declarations or absence of voting rights. Yet neither the reclassification nor
exchange per se, yields realize income for tax purposes. A common stock
represents the residual ownership interest in the corporation. It is a basic G.R. No. L-30460 March 12, 1929
class of stock ordinarily and usually issued without extraordinary rights or
privileges and entitles the shareholder to a pro rata division of C. H. STEINBERG, as Receiver of the Sibuguey Trading Company,
profits. 126 Preferred stocks are those which entitle the shareholder to some Incorporated, plaintiff-appellant,
priority on dividends and asset distribution. 127 vs.
GREGORIO VELASCO, ET AL., defendants-appellees.
Both shares are part of the corporation's capital stock. Both stockholders are
no different from ordinary investors who take on the same investment risks. Frank H. Young for appellant.
Preferred and common shareholders participate in the same venture, willing Pablo Lorenzo and Delfin Joven for appellees.
to share in the profits and losses of the enterprise. 128 Moreover, under the
doctrine of equality of shares all stocks issued by the corporation are STATEMENT
presumed equal with the same privileges and liabilities, provided that the
Articles of Incorporation is silent on such differences. 129 Plaintiff is the receiver of the Sibuguey Trading Company, a domestic
corporation. The defendants are residents of the Philippine Islands.
In this case, the exchange of shares, without more, produces no realized
income to the subscriber. There is only a modification of the subscriber's It is alleged that the defendants, Gregorio Velasco, as president, Felix del
rights and privileges which is not a flow of wealth for tax purposes. The Castillo, as vice-president, Andres L. Navallo, as secretary-treasurer, and
issue of taxable dividend may arise only once a subscriber disposes of his Rufino Manuel, as director of Trading Company, at a meeting of the board of
entire interest and not when there is still maintenance of proprietary directors held on July 24, 1922, approved and authorized various lawful
interest. 130 purchases already made of a large portion of the capital stock of the
company from its various stockholders, thereby diverting its funds to the
WHEREFORE, premises considered, the decision of the Court of Appeals is injury, damage and in fraud of the creditors of the corporation. That pursuant
MODIFIED in that ANSCOR's redemption of 82,752.5 stock dividends is to such resolution and on March 31, 1922, the corporation purchased from

701
the defendant S. R. Ganzon 100 shares of its capital stock of the par value of directors of the corporation "when the business of the company was going on
P10, and on June 29, 1922, it purchased from the defendant Felix D. very well." That the defendant is one of the principal shareholders, and that
Mendaros 100 shares of the par value of P10, and on July 16, 1922, it about the same time, he purchase other shares for his own account, because
purchased from the defendant Felix D. Mendaros 100 shares of the par value he thought they would bring profits. As to the second cause of action, he
of P10, each, and on April 5, 1922, it purchased from the defendant Dionisio admits that the dividends described in paragraph 4 of the complaint were
Saavedra 10 shares of the same par value, and on June 29, 1922, it distributed, but alleges that such distribution was authorized by the board of
purchased from the defendant Valentin Matias 20 shares of like value. That directors, "and that the amount represented by said dividends really
the total amount of the capital stock unlawfully purchased was P3,300. That constitutes a surplus profit of the corporation," and as counterclaim, he asks
at the time of such purchase, the corporation had accounts payable for judgment against the receiver for P12,512.47 for and on account of his
amounting to P13,807.50, most of which were unpaid at the time petition for negligence in failing to collect the accounts.
the dissolution of the corporation was financial condition, in contemplation of
an insolvency and dissolution. Although duly served, the defendant Mendaros did not appear or answer.
The defendant Navallo was not served, and the case against him was
As a second cause of action, plaintiff alleges that on July 24, 1922, the dismissed.
officers and directors of the corporation approved a resolution for the
payment of P3,000 as dividends to its stockholders, which was wrongfully April 30, 1928, the case was tried and submitted on a stipulation of facts,
done and in bad faith, and to the injury and fraud of its creditors. That at the based upon which the lower court dismissed plaintiff's complaint, and
time the petition for the dissolution of the corporation was presented it had rendered judgment for the defendants, with costs against the plaintiff, and
accounts payable in the sum of P9,241.19, "and practically worthless absolved him from the cross-complaint of the defendant Velasco, and on
accounts receivable." appeal, the plaintiff assigns the following errors:

Plaintiff prays judgment for the sum of P3,300 from the defendants Gregorio 1. In holding that the Sibuguey Trading Company, Incorporated,
Velasco, Felix del Castillo, Andres L. Navallo and Rufino Manuel, personally could legally purchase its own stock.
as members of the Board of Directors, or for the recovery from the
defendants S. R. Ganzon, of the sum of P1,000, from the defendant Felix D. 2. In holding that the Board of Directors of the said Corporation could
Mendaros, P2,000, and from the defendant Dionisio Saavedra, P100, and legally declared a dividend of P3,000, July 24, 1922.
under his second cause of action, he prays judgment for the sum of P3,000,
with legal interest against the board of directors, and costs.
JOHNS, J.:

For answer the defendants Felix del Castillo, Rufino Manuel, S. R. Ganzon,
It is stipulated that on July 24, 1922, the directors of the corporation
Dionisio Saavedra and Valentin Matias made a general and specific denial.
approved the purchase of stocks as follows:

In his amended answer, the defendant Gregorio Velasco admits paragraphs,


One hundred shares from S. R. Ganzon for P1,000;
1, 2 and 3 of each cause of action of the complaint, and that the shares
mentioned in paragraph 4 of the first cause of action were purchased, but
alleges that they were purchased by virtue of a resolution of the board of One hundred shares from Felix D. Mendaros at the same price; which
purchase was made on June 29, 1922; another

702
One hundred shares from Felix D. Mendaros at the same price on July 16, That the same defendants, mentioned in paragraph 2 of this
1922; stipulation of facts and in the same capacity, on the same date of
July 24, 1922, and at the said meeting of the said Board of Directors,
Ten shares from Dionisio Saavedra at the same price on June 29, 1922. approved and authorized by resolution the payment of dividends to
its stockholders, in the sum of three thousand pesos (P3,000),
That during such times, the defendant Gregorio Velasco purchased 13 Philippine currency, which payments were made at different dates,
shares for the corporation for P130; Felix del Castillo 42 shares for P420; between September 30, 1922, and May 12, 1923, both dates
Andres Navallo 15 shares for P150; and the defendant Mendaros 10 inclusive, at a time when the corporation had accounts less in
shares for P100. That during the time these various purchases were made, amount than the accounts receivable, which resolution was based
the total amount of subscribed and paid up capital stock of the corporation upon the balance sheet made as June 30, 1922, said balance sheet
was P10,030, out of the authorized capital stock 2,000 shares of the par showing that the corporation had a surplus of P1,069.41, and a profit
value of P10 each. on the same date of P2,656.08, or a total surplus amount of
P3,725.49, and a reserve fund of P2,889.23 for bad and doubtful
accounts and depreciation of equipment, thereby leaving a balance
Paragraph 4 of the stipulation also recites:
of P3,314.72 of net surplus profit after paying this dividend.

Be it also admitted as a fact that the time of the said purchases there
It is also stipulated at a meeting of the board of directors held on July 24,
was a surplus profit of the corporation above-named of P3,314.72.
1922, as follows:

Paragraph 5 is as follows:
6. The president and manager submitted to the Board of Directors
his statement and balance sheet for the first semester ending June
That at the time of the repeatedly mentioned various purchases of 30, 1922 and recommended that P3,000 out of the surplus
the said capital stock were made, the said corporation had Accounts account be set aside for dividends payable, and that payments be
Payable in the total amount of P13,807.50 as shown by the made in installments so as not to effect the financial condition of the
statement of the corporation, dated June 30, 1922, and the Accounts corporation. That stockholders having outstanding account with the
Receivable in the sum of P19,126.02 according to the books, and corporation should settle first their accounts before payments of their
that the intention of the Board of Directors was to resell the stocks dividends could be made. Mr. Castillo moved that the statement and
purchased by the corporations at a sum above par for each stock, balance sheet be approved as submitted, and also the
this expectation being justified by the then satisfactory and sound recommendations of the president. Seconded by Mr. Manuel.
financial condition of the business of the corporation. Approved.

It is also stipulated that on September 11, 1923, when the petition for the Paragraph 8 of the stipulation is as follows:
dissolution of the corporation was presented to the court, according to a
statement made June 30, 1923, it has accounts payable aggregating
That according to the balance sheet of the corporation, dated June
P9,41.19, and accounts receivable for P12,512.47.
30, 1923, it had accounts receivable in the sum of P12,512.47, due
from various contractor and laborers of the National Coal Company,
Paragraph 7 of the stipulation recites:

703
and also employees of the herein corporation, which the herein It is very apparent that on June 24, 1922, the board of directors acted on
receiver, after his appointment on February 28, 1924, although he assumption that, because it appeared from the books of the corporation that
made due efforts by personally visiting the location of the it had accounts receivable of the face value of P19,126.02, therefore it had a
corporation, and of National Coal Company, at its offices, at surplus over and above its debts and liabilities. But as stated there is no
Malangas, Mindanao, and by writing numerous letters of demand to stipulation as to the actual cash value of those accounts, and it does appear
the debtors of the corporation, in order to collect these accounts from the stipulation that on February 28, 1924, P12,512.47 of those accounts
receivable, he was unable to do so as most of them were without had but little, if any, value, and it must be conceded that, in the purchase of
goods or property, and he could not file any suit against them that its own stock to the amount of P3,300 and in declaring the dividends to the
might have any property, for the reason that he had no funds on amount of P3,000, the real assets of the corporation were diminished
hand with which to pay the filing and sheriff fees to Malangas, and P6,300. It also appears from paragraph 4 of the stipulation that the
other places of their residences. corporation had a "surplus profit" of P3,314.72 only. It is further stipulated
that the dividends should "be made in installments so as not to effect
From all of which, it appears that on June 30, 1922, the board of directors of financial condition of the corporation." In other words, that the corporation did
the corporation authorized the purchase of, purchased and paid for, 330 not then have an actual bona fidesurplus from which the dividends could be
shares of the capital stock of the corporation at the agreed price of P3,300, paid, and that the payment of them in full at the time would "affect the
and that at the time the purchase was made, the corporation was indebted in financial condition of the corporation."
the sum of P13,807.50, and that according to its books, it had accounts
receivable in the sum of P19,126.02. That on September 11, 1923, when the It is, indeed, peculiar that the action of the board in purchasing the stock from
petition was filed for its dissolution upon the ground that it was insolvent, its the corporation and in declaring the dividends on the stock was all done at
accounts payable amounted to P9,241.19, and its accounts receivable the same meeting of the board of directors, and it appears in those minutes
P12,512.47, or an apparent asset of P3,271.28 over and above its liabilities. that the both Ganzon and Mendaros were formerly directors and resigned
But it will be noted that there is no stipulation or finding of facts as to what before the board approved the purchase and declared the dividends, and
was the actual cash value of its accounts receivable. Neither is there any that out of the whole 330 shares purchased, Ganzon, sold 100 and
stipulation that those accounts or any part of them ever have been or will be Mendaros 200, or a total of 300 shares out of the 330, which were purchased
collected, and it does appear that after his appointment on February 28, by the corporation, and for which it paid P3,300. In other words, that the
1924, the receiver made a diligent effort to collect them, and that he was directors were permitted to resign so that they could sell their stock to the
unable to do so, and it also appears from the minutes of the board of corporation. As stated, the authorized capital stock was P20,000 divided into
directors that the president and manager "recommended that P3,000 out 2,000 shares of the par value of P10 each, which only P10,030 was
of the surplus account to be set aside for dividends payable, and that subscribed and paid. Deducting the P3,300 paid for the purchase of the
payments be made in installments so as not to effect the financial condition stock, there would be left P7,000 of paid up stock, from which deduct P3,000
of the corporation." paid in dividends, there would be left P4,000 only. In this situation and upon
this state of facts, it is very apparent that the directors did not act in good
If in truth and in fact the corporation had an actual bona fide surplus of faith or that they were grossly ignorant of their duties.
P3,000 over and above all of its debt and liabilities, the payment of the
P3,000 in dividends would not in the least impair the financial condition of the Upon each of those points, the rule is well stated in Ruling Case Law, vol. 7,
corporation or prejudice the interests of its creditors. p. 473, section 454 where it is said:

704
General Duty to Exercise Reasonable Care. The directors of a The amount involved in this case is not large, but the legal principles are
corporation are bound to care for its property and manage its affairs important, and we have given them the consideration which they deserve.
in good faith, and for a violation of these duties resulting in waste of
its assets or injury to the property they are liable to account the same The judgment of the lower court is reversed, and (a), as to the first cause of
as other trustees. Are there can be no doubt that if they do acts action, one will be entered for the plaintiff and against the defendant S. R.
clearly beyond their power, whereby loss ensues to the corporation, Ganzon for the sum of P1,000, with legal interest from the 10th of February,
or dispose of its property or pay away its money without authority, 1926, and against the defendant Felix D. Medaros for P2,000, with like
they will be required to make good the loss out of their private interests, and against the defendant Dionisio Saavedra for P100, with like
estates. This is the rule where the disposition made of money or interest, and against each of them for costs, each on their primary liability as
property of the corporation is one either not within the lawful power of purchasers of stock, and (b) against the defendants Gregorio Velasco, Felix
the corporation, or, if within the authority of the particular officer or del Castillo and Rufino Manuel, personally, as members of the board of
officers. directors of the Sibuguey Trading Company, Incorporated, as secondarily
liable for the whole amount of such stock sold and purchased as above
And section 458 which says: stated, and on the second cause of action, judgment will be entered (c) for
the plaintiff and jointly and severally against the defendants Gregorio
Want of Knowledge, Skill, or Competency. It has been said that Velasco, Felix del Castillo and Rufino Manuel, personally, as members of the
directors are not liable for losses resulting to the corporation from board of directors of the Sibuguey Trading Company, Incorporated, for
want of knowledge on their part; or for mistake of judgment, provided P3,000, with interest thereon from February 10, 1926, at the rate of 6 per
they were honest, and provided they are fairly within the scope of the cent per annum, and costs. So ordered.
powers and discretion confided to the managing body. But the
acceptance of the office of a director of a corporation implies a Johnson, Street, Malcolm, Ostrand, Romualdez and Villa-Real, JJ., concur.
competent knowledge of the duties assumed, and directors cannot
excuse imprudence on the ground of their ignorance or inexperience;
and if they commit an error of judgment through mere recklessness
or want of ordinary prudence or skill, they may be held liable for the
consequences. Like a mandatory, to whom he has been likened, a
director is bound not only to exercise proper care and diligence, but
ordinary skill and judgment. As he is bound to exercise ordinary skill
and judgment, he cannot set up that he did not possess them.

Creditors of a corporation have the right to assume that so long as there are
outstanding debts and liabilities, the board of directors will not use the assets
of the corporation to purchase its own stock, and that it will not declare
dividends to stockholders when the corporation is insolvent. G.R. No. L-42091 November 2, 1935

GONZALO CHUA GUAN, plaintiff-appellant,


vs.

705
SAMAHANG MAGSASAKA, INC., and SIMPLICIO OCAMPO, ADRIANO The debtor, Gonzalo H. Co Toco, having defaulted in the payment of said
G. SOTTO, and EMILIO VERGARA, as president, secretary and debt at maturity, the plaintiff foreclosed said mortgage and delivered the
treasurer respectively of the same, defendants-appellees. certificates of stock and copies of the mortgage and assignment to the sheriff
of the City of Manila in order to sell the said shares at public auction. The
Buenaventura C. Lopez for appellant. sheriff auctioned said 5,894 shares of stock on December 22, 1932, and the
Domingo L. Vergara for appellees. plaintiff having been the highest bidder for the sum of P14,390, the sheriff
executed in his favor a certificate of sale of said shares.

The plaintiff tendered the certificates of stock standing in the name of


BUTTE, J.: Gonzalo H. Co Toco to the proper officers of the corporation for cancellation
and demanded that they issue new certificates in the name of the plaintiff.
The said officers (the individual defendants) refused and still refuse to issue
This is an appeal from a judgment of the Court of First Instance of Nueva
said new shares in the name of the plaintiff.
Ecija in an action for a writ of mandamus. The case is remarkable for the
following reason: that the parties entered into a stipulation in which the
defendants admitted all of the allegations of the complaint and the plaintiff The prayer is that a writ of mandamus be issued requiring the defendants to
admitted all of the special defenses in the answer of the defendants, and on transfer the said 5,894 shares of stock to the plaintiff by cancelling the old
this stipulation they submitted the case for decision. certificates and issuing new ones in their stead.

The complaint alleges that the defendant Samahang Magsasaka, Inc., is a The special defenses set up in the answer are as follows: that the defendants
corporation duly organized under the laws of the Philippine Islands with refuse to cancel the said certificates standing in the name of Gonzalo H. Co
principal office in Cabanatuan, Nueva Ecija, and that the individual Toco on the books of the corporation and to issue new ones in the name of
defendants are the president, secretary and treasurer respectively of the the plaintiff because prior to the date when the plaintiff made his demand, to
same; that on June 18, 1931, Gonzalo H. Co Toco was the owner of 5,894 wit, February 4, 1933, nine attachments had been issued and served and
shares of the capital stock of the said corporation represented by nine noted on the books of the corporation against the shares of Gonzalo H. Co
certificates having a par value of P5 per share; that on said date Gonzalo H. Toco and the plaintiff objected to having these attachments noted on the new
Co Toco, a resident of Manila, mortgaged said 5,894 shares to Chua Chiu to certificates which he demanded. These attachments noted on the books of
guarantee the payment of a debt of P20,000 due on or before June 19, 1932. the corporation against the shares of Gonzalo H. Co Toco are as follows:
The said certificates of stock were delivered with the mortgage to the
mortgagee, Chua Chiu. The said mortgage was duly registered in the office MISSING PAGES: 475-477.
of the register of deeds of Manila on June 23, 1931, and in the office of the
said corporation on September 30, 1931. It will be noted that the first eight of the said writs of attachment were served
on the corporation and noted on its records before the corporation received
On November 28, 1931, Chua Chiu assigned all his right and interest in the notice from the mortgagee Chua Chiu of the mortgage of said shares dated
said mortgage to the plaintiff and the assignment was registered in the office June 18, 1931. No question is raised as to the validity of said mortgage or of
of the register of deeds in the City of Manila on December 28, 1931, and in said writs of attachment and the sole question presented for decision is
the office of the said corporation on January 4, 1932. whether the said mortgage takes priority over the said writs of attachment.

706
It is not alleged that the said attaching creditors had actual notice of the said The practical application of the Chattel Mortgage Law to shares of stock of a
mortgage and the question therefore narrows itself down to this: Did the corporation presents considerable difficulty and we have obtained little aid
registration of said chattel mortgage in the registry of chattel mortgages in from the decisions of other jurisdictions because that form of mortgage is ill
the office of the register of deeds of Manila, under date of July 23, 1931, give suited to the hypothecation of shares of stock and has been rarely used
constructive notice to the said attaching creditors? elsewhere. In fact, it has been doubted whether shares of stock in a
corporation are chattels in the sense in which that word is used chattel
In passing, let it be noted that the registration of the said chattel mortgage in mortgage statutes. This doubt is reflected in our own decision in the case
the office of the corporation was not necessary and had no legal effect. of Fua Cun vs. Summers and China Banking Corporation (44 Phil., 705), in
(Monserrat vs. Ceron, 58 Phil., 469.) The long mooted question as to which we said:
whether or not shares of a corporation could be hypothecated by placing a
chattel mortgage on the certificate representing such shares we now regard ". . . an equity in shares of stock is of such an intangible character that it is
as settled by the case of Monserrat vs. Ceron, supra. But that case did not somewhat difficult to see how it can be treated as a chattel and mortgaged in
deal with any question relating to the registration of such a mortgage or the such a manner that the recording of the mortgage will furnish constructive
effect of such registration. Nothing appears in the record of that case even notice to third parties. . . ."And we held that the chattel mortgage there
tending to show that the chattel mortgage there involved was ever registered involved: "at least operated as a conditional equitable assignment." In that
anywhere except in the office of the corporation, and there was no question case we quoted the following from Spalding vs. Paine's Adm'r. (81 Ky., 416),
involved there as to the right of priority among conflicting claims of creditors with regard to a chattel mortgage of shares of stock:
of the owner of the shares.
"These certificates of stock are in the pockets of the owner, and go
The Chattel Mortgage Law, Act No. 1508, as amended by Act No. 2496, with him where he may happen to locate, as choses in action, or
contains the following provision: evidence of his right, without any means on the part of those with
whom he proposes to deal on the faith of such a security of
SEC. 4. A chattel mortgage shall not be valid against any person ascertaining whether or not this stock is in pledge or mortgaged to
except the mortgagor, his executors or administrators, unless the others. He finds the name of the owner on the books of the company
possession of the property is delivered to and retained by the as a subscriber of paid-up stock, amounting to 180 shares, with the
mortgagee or unless the mortgage is recorded in the office of the certificates in his possession, pays for these certificates their full
register of deeds of the province in which the mortgagor resides at value, and has the transfer to him made on the books of the
the time of making the same, or, if he resides the Philippine Islands, company, thereby obtaining a perfect title. What other inquiry is he to
in the province in which the property is situated: Provided, make, so as to make his investment certain and secure? Where is he
however, That if the property is situated in a different province from to look, in order to ascertain whether or not this stock has been
that in which the mortgagor resides, the mortgage shall be recorded mortgaged? The chief office of the company may be at one place
in the office of the register of deeds of both the province in which the today and at another tomorrow. The owner may have no fixed or
mortgagor resides and that in which the property is situated, and for permanent abode, and with his notes in one pocket and his
the purposes of this Act the City of Manila Shall be deemed to be a certificates of stock in the other the one evidencing the extent of
province. his interest in the stock of the corporation, the other his right to
money owing him by his debtor, we are asked to say that the
mortgage is effectual as to the one and inoperative as to the other."

707
But the case of Fua Cun vs. Summers and China Banking Corporation, owner and for others at the domicile of the corporation; and even elsewhere.
supra, did not decide the question here presented and gave no light as to the (Cf. Vidal vs. South American Securities Co., 276 Fed., 855; Black Eagle
registration of a chattel mortgage of shares of stock of a corporation under Min. Co. vs. Conroy, 94 Okla., 199; 221 Pac,, 425 Norrie vs. Kansas City
the provisions of section 4 of the Chattel Mortgage Law, supra. Southern Ry. Co., 7 Fed. [2d]. 158.) It is a general rule that for purposes of
execution, attachment and garnishment, it is not the domicile of the owner of
Section 4 of Act No. 1508 provides two ways for executing a valid chattel a certificate but the domicile of the corporation which is decisive. (Fletcher,
mortgage which shall be effective against third persons. First, the possession Cyclopedia of the Law of Private Corporations, vol. 11, paragraph 5106. Cf.
of the property mortgage must be delivered to and retained by the sections 430 and 450, Code of Civil Procedure.)
mortgagee; and, second, without such delivery the mortgage must be
recorded in the proper office or offices of the register or registers of deeds. If By analogy with the foregoing and considering the ownership of shares in a
a chattel mortgage of shares of stock of a corporation may validly be made corporation as property distinct from the certificates which are merely the
without the delivery of possession of the property to the mortgagee and the evidence of such ownership, it seems to us a reasonable construction of
mere registration of the mortgage is sufficient to constructive notice to third section 4 of Act No. 1508 to hold that the property in the shares may be
parties, we are confronted with the question as to the proper place of deemed to be situated in the province in which the corporation has its
registration of such a mortgage. Section 4 provides that in such a case the principal office or place of business. If this province is also the province of the
mortgage resides at the time of making the same or, if he is a non-resident, owner's domicile, a single registration sufficient. If not, the chattel mortgage
in the province in which the property is situated; and it also provides that if should be registered both at the owner's domicile and in the province where
the property is situated in a different province from that in which the the corporation has its principal office or place of business. In this sense the
mortgagor resides the mortgage shall be recorded both in the province of the property mortgaged is not the certificate but the participation and share of the
mortgagor's residence and in the province where the property is situated. owner in the assets of the corporation.

If with respect to a chattel mortgage of shares of stock of a corporation, Apart from the cumbersome and unusual method of hypothecating shares of
registration in the province of the owner's domicile should be sufficient, those stock by chattel mortgage, it appears that in the present state of our law, the
who lend on such security would be confronted with the practical difficulty of only safe way to accomplish the hypothecation of share of stock of a
being compelled not only to search the records of every province in which the Philippine corporation is for the creditor to insist on the assignment and
mortgagor might have been domiciled but also every province in which a delivery of the certificate and to obtain the transfer of the legal title to him on
chattel mortgage by any former owner of such shares might be registered. the books of the corporation by the cancellation of the certificate and the
We cannot think that it was the intention of the legislature to put this almost issuance of a new one to him. From the standpoint of the debtor this may be
prohibitive impediment upon the hypothecation of shares of stock in view of unsatisfactory because it leaves the creditor as the ostensible owner of the
the great volume of business that is done on the faith of the pledge of shares shares and the debtor is forced to rely upon the honesty and solvency of the
of stock as collateral. creditor. Of course, the mere possession and retention of the debtor's
certificate by the creditor gives some security to the creditor against an
It is a common but not accurate generalization that the situs of shares of attempted voluntary transfer by the debtor, provided the by-laws of the
stock is at the domicile of the owner. The term situs is not one of fixed of corporation expressly enact that transfers may be made only upon the
invariable meaning or usage. Nor should we lose sight of the difference surrender of the certificate. It is to be noted, however, that section 35 of the
between the situs of the shares and the situs of the certificates of shares. Corporation Law (Act No. 1459) enacts that shares of stock "may be
The situs of shares of stock for some purposes may be at the domicile of the transferred by delivery of the certificate endorsed by the owner or his

708
attorney in fact or other person legally authorized to make the transfer." The
use of the verb "may" does not exclude the possibility that a transfer may be
made in a different manner, thus leaving the creditor in an insecure position
even though he has the certificate in his possession. Moreover, the shares
still standing in the name of the debtor on the books of the corporation will be
liable to seizure by attachment or levy on execution at the instance of other
creditors. (Cf. Uy Piaoco vs.McMicking, 10 Phil., 286, and
Uson vs. Diosomito, 61 Phil., 535.) This unsatisfactory state of our law is well
known to the bench and bar. (Cf. Fisher, The Philippine Law of Stock
Corporations, pages 163-168.) Loans upon stock securities should be
facilitated in order to foster economic development. The transfer by
endorsement and delivery of a certificate with intention to pledge the shares
covered thereby should be sufficient to give legal effect to that intention and
to consummate the juristic act without necessity for registration.lawphil.net

We are fully conscious of the fact that our decisions in the case of Monserrat
vs. Ceron, supra, and in the present case have done little perhaps to
ameliorate the present uncertain and unsatisfactory state of our law
applicable to pledges and chattel mortgages of shares of stock of Philippine
corporations. The remedy lies with the legislature.

In view of the premises, the attaching creditors are entitled to priority over the
defectively registered mortgage of the appellant and the judgment appealed
from must be affirmed without special pronouncement as to costs in this
instance. 1

Malcolm, Villa-Real, Imperial, and Goddard, JJ., concur.

709
In support of this appeal, the appellants assign nine alleged errors in the
decision of the trial court, which we shall discuss in the course of this
decision.

Some of the following facts are undisputed and others proven by a


preponderance of the evidence:

The plaintiff herein, Enrique Monserrat, was the president and manager of
the Manila Yellow Taxicab Co., Inc., and the owner of P1,200 common
shares of stock thereof.
G.R. No. 37078 September 27, 1933
On March 25, 1930, in consideration of the interest shown and the financial
aid extended him in the organization of the corporation by Carlos G. Ceron,
ENRIQUE MONSERRAT, plaintiff-appellee,
one of the defendants herein, Enrique Monserrat assigned to the former the
vs.
usufruct of half of the aforesaid common shares of stock, the corresponding
CARLOS G. CERON, ET AL., defendants.
certificate of stock No. 7, having been issued in the name of said Carlos G,
ERMA, INC., and, THE SHERIFF OF MANILA, respondents.
Ceron to that effect on March 24, 1930. (Exhibit 1.) Said assignment or
transfer only gave the transferee the right to enjoy, during his lifetime, the
Juan T. Santos and Arsenio Solidum for appellants. profits which might be derived from the shares assigned him, prohibiting him
Cardenas and Casal for appellee. from selling, mortgaging, encumbering, alienating or otherwise exercising
any act implying absolute ownership of all or any of the shares in question,
VILLA-REAL, J.: the transferor having reserved for himself and his heirs the right to vote
derived from said shares of stock and to recover the ownership thereof at the
This is an appeal taken by the defendant-entity, Erma, Inc., and by the sheriff termination of the usufruct (Exhibit A). Stock certificate No. 7 was recorded in
of the City of Manila, from the judgment rendered by the Court of First the name of Carlos G. Ceron and the aforesaid deed of transfer Exhibit A,
Instance of Manila, the dispositive part of which reads as follows: was noted by himself as secretary, on page 22 of the Stock and Transfer
Book of the Manila Yellow Taxicab Co., Inc.
In view of the foregoing considerations, judgment is rendered in favor
of the plaintiff declaring the preliminary injunction issued herein final By way of defense, the defendants herein alleged that on February 20, 1931,
and permanent; declaring the plaintiff herein the owner of the 600 Eduardo R. Matute, president of the defendant corporation, Erma, Inc., and
shares of stock, Exhibit 1; declaring the mortgage constituted on the the defendant Carlos G. Ceros. appeared at the plaintiff's office on Mabini
ownership of the shares of stock in question null and void and Street, Manila, and there Ceron, at a distance of about three meters from the
without force and effect, although the mortgage on the usufruct plaintiff, showed Matute the stock book of the Manila Yellow Taxicab Co.,
enjoyed by the mortgage debtor Carlos G. Ceron in the said 600 Inc., Matute did not see the annotation on page 22 thereof regarding exhibit A
shares of stock is hereby declared valid, with costs against the which, according to Ceron, was executed two months after March 25, 1930,
defendants. It is so ordered. the date on which it appears to have been executed. Ceron alleges that,

710
upon instructions of the plaintiff, he did not make any notation of said No share of stock against which the corporation hold, any unpaid
document in the stock book until May 5, 1931, the date on which the shares claim shall be transferable on the books of the corporation.
of stock in question were to be sold at public auction to satisfy his debt to
Matute. The legal provision just quoted does not require any entry except of transfers
of shares of stock in order that such transfers may be valid as against third
On February 26, 1931, Carlos G. Ceron mortgaged to Eduardo R. Matute persons. Now, what did the Legislature mean in using the word "transfer"?
some shares of stock of the Manila Yellow Taxicab Co., Inc., among which
were the 600 common shares of stock in question, for the sum of P30,000. It is a rule of statutory construction that the words of a statute are to be taken
Ceron endorsed to Matute the certificate of stock Exhibit 1, of which Matute in their natural, plain and ordinary signification in accordance with the
has been in possession ever since. When Ceron mortgaged the shares in common and approved usage of the language, giving to words of common
question to Matute, he did not inform Matute of the existence of the use their popularly accepted meaning and to technical terms or words of art,
document, Exhibit A, and the latter never had any knowledge thereof. When their accepted special signification, unless there is reason to believe from the
he was asked by the plaintiff whether he succeeded in carrying out his context of the statute that such words have been used in another sense.
transaction with Matute, Carlos G. Ceron informed him of the aforesaid (Black, Construction and Interpretation of Laws, section 57.) Inasmuch as it
mortgage at the beginning of March 1931. Ceron continued as secretary of does not appear from the text of the Corporation Law that an attempt was
the Manila Yellow Taxicab Co., Inc., until May 5, 1931. made to give a special signification to the word "transfer", we shall construe it
according to its accepted meaning in ordinary parlance.
The first question to decide in the present appeal is whether or not it is
necessary to enter upon the books of the corporation a mortgage constituted The word "transferencia" (transfer) is defined by the "Diccionario de la
on common shares of stock in order that such mortgage may be valid and Academia de la Lengua Castellana" as "accion y efecto de transferir" (the act
may have force and effect as against third persons. and effect of transferring); and the verb "transferir", as "ceder o renunciar en
otro el derecho o dominio que se tiene sobre una cosa, haciendole dueno de
Section 35 of the Corporation Law provides the following: ella" (to assign or waive the right in, or absolute ownership of, a thing in favor
of another, making him the owner thereof).
SEC. 35. The capital stock of stock corporations shall be divided into
shares for which certificates signed by the president or the vice- In the Law Dictionary of "Words and Phrases", third series, volume 7, p. 589,
president, counter signed by the secretary or clerk and sealed with the word "transfer" is defined as follows:
the seal of the corporation, shall be issued in accordance with the
by-laws. Shares of stock so issued are personal property and may "Transfer" means any act by which property of one person is vested
be transferred by delivery of the certificate indorsed by the owner or in another, and "transfer of shares", as used in Uniform Stock
his attorney in fact or other person legally authorized to make the Transfer Act (Comp. St. Supp., 690), implies any means whereby
transfer. No transfer, however, shall be valid, except as between the one may be divested of and another acquire ownership of stock.
parties, until the transfer is entered and noted upon the books of the (Wallach vs. Stein [N.J.], 136 A., 209, 210.)"
corporation so as to show the names of the parties to the
transaction, the date of the transfer the number of the certificate, and In view of the definitions cited above, the question arises as to whether or not
the number of shares transferred. a mortgage constituted on certain shares of stock in accordance with Act No.

711
1508, as amended by Act No. 2496, is a transfer of such shares in the If, in accordance with said section 35 of the Corporation Law, only the
abovementioned sense. transfer or absolute conveyance of the ownership of the title to a share need
be entered and noted upon the books of the corporation in order that such
Section 3 of the aforesaid Act No. 1508, as amended by Act No. 2496, transfer may ba valid, therefore, inasmuch as a chattel mortgage of the
defines the phrase "hipoteca mobiliaria" (chattel mortgage) as follows: aforesaid title is not a complete and absolute alienation of the dominion and
ownership thereof, its entry and notation upon the books of the corporation is
SEC. 3. A chattel mortgage is a conditional sale of personal property not necessary requisite to its validity.
as security for the payment of a debt, or the performance of some
other obligation specified therein, the condition being that the sale The second question to decide is whether or not the defendant entity, Erma,
shall be avoided upon the seller paying to the purchaser a sum of Inc., had knowledge of the execution of the deed Exhibit A, dated March 25,
money or doing some other act named. If the condition is performed 1930, wherein the defendant Carlos G. Ceron states that the transfer to him
according to its terms the mortgage and sale immediately become by Enrique Monserrat of 600 shares of common stock of the Manila Yellow
void, and the mortgage is hereby divested of his title. Taxicab Co., Inc., covered by certificate No. 7, was only for the usufruct
thereof, reserving to the assignor the right to vote said share and binding
According to the legal provision just quoted, although a chattel mortgage, himself not to alienate nor encumber them.
accompanied by delivery of the mortgaged thing, transfers the title and
ownership thereof to the mortgage creditor, such transfer is not absolute but The evidence shows that when Matute as president of Erma, Inc., went to the
constitutes a mere security for the payment of the mortgage debt, the office of the Manila Yellow Taxicab Co., Inc., at Mabini Street, manila, on
transfer in question becoming null and void from the time the mortgage February 20, 1931, to examine the Stock and Transfer Book of the said
debtor complies with his obligation to pay his debt. corporation, for the purpose of ascertaining the actual status of Carlos G.
Ceron's shares of stock, Ceron as secretary of said corporation and in
In the case of Noble vs. Ft. Smith Wholesale Grocery Co. (127 Pac., 14, 17; charge of said stock book, showed it to him, and Matute found nothing but
34 Okl., 662; 46 L. R. A. [N.S.], 455), cited in Words and Phrases, second that the shares in question were recorded therein in the name of said Carlos
series, vol. 4, p. 978, the following appears: G. Ceron, free from all liens and encumbrances and no reference made to
the Exhibit A. the defendant, Carlos G. Ceron himself, testified that when he
mortgaged his shares, he said nothing to Erma, Inc., about the existence of
A "transfer" is the act by which owner of a thing delivers it to another
the deed, Exhibit A, for fear he might not succeed in obtaining the loan he
with the intent of passing the rights which he has in it to the latter,
applied for, with the said shares as security, and that the notation of Exhibit A
and a chattel mortgage is not within the meaning of such term.
in question appearing in the books of the corporation was place there only on
May 5, 1931, the same date on which the 600 common shares were to have
Therefore, the chattel mortgage is not the transfer referred to in section 35 of been sold at public auction, together with the preferred shares, which were
Act No. 1459 commonly known as the Corporation law, which transfer should delivered to the sheriff for that purpose by Erma, Inc., in view of Carlos G.
be entered and noted upon the books of a corporation in order to be valid, Ceron's default in the payment of the loan secured by them. From the time
and which, as has already been said, means the absolute and unconditional said shares of stocks in question were mortgaged by Carlos G. Ceron on
conveyance of the title and ownership of a share of stock. February 26, 1931, the corresponding certificate has been in possession of
the defendant entity, Erma, Inc., without any notation thereon relative to the
deed Exhibit A. It is obvious, therefore, that the defendant entity Erma, Inc.,

712
as a conditional purchaser of the shares of stock in question given as OSTRAND, J.:
security for the payment of his credit, acquired in good faith Carlos G.
Ceron's right and title to the 600 common shares of stock evidenced by It appears from the evidence that on August 26, 1920, one Chua Soco
certificate No. 7 of the Manila Yellow Taxicab Co., Inc., and as such subscribed for five hundred shares of stock of the defendant Banking
conditional purchaser in good faith, it is entitled to the protection of the law. Corporation at a par value of P100 per share, paying the sum of P25,000,
one-half of the subscription price, in cash, for which a receipt was issued in
In view of the foregoing considerations, we are of the opinion and so hold the following terms:
that, inasmuch as section 35 of the Corporation Law does not require the
notation upon the books of a corporation of transactions relating to its shares, This is to certify, That Chua Soco, a subscriber for five hundred
except the transfer of possession and ownership thereof, as a necessary shares of the capital stock of the China Banking Corporation at its
requisite to the validity of such transfer, the notation upon the aforesaid par value of P100 per share, has paid into the Treasury of the
books of the corporation, of a chattel mortgage constituted on the shares of Corporation, on account of said subscription and in accordance with
stock in question is not necessary to its validity. its terms, the sum of twenty-five thousand pesos (P25,000),
Philippine currency.
Wherefore, the judgment appealed from is hereby reversed and the
defendants are absolved from the complaint herein which is dismissed with Upon receipt of the balance of said subscription in accordance with
costs against the appellee. So ordered. the terms of the calls of the Board of Directors, and surrender of this
certificate, duly executed certificates for said five hundred shares of
Malcolm, Abad Santos, Hull, and Imperial, JJ., concur. stock will be issued to the order of the subscriber.

It is expressly understood that the total number of shares specified in


this receipt is subject to sale by the China Banking Corporation for
the payment of any unpaid subscriptions, should the subscriber fail
to pay the whole or any part of the balance of his subscription upon
30 days' notice issued therefor by the Board of Directors.

G.R. No. L-19441 March 27, 1923 Witness our official signatures at Manila, P. I., this 25th day of
August, 1920.
FUA CUN (alias Tua Cun), plaintiff-appellee,
vs. (Sgd.) MERVIN WEBSTER
RICARDO SUMMERS, in his capacity as Sheriff ex-oficio of the City of Cashier
Manila, and the CHINA BANKING CORPORATION, defendants-appellants.
(Sgd.) DEE C. CHUAN
Araneta and Zaragoza for appellants. President
Canillas and Cardenas for appellee.

713
On May 18, 1921, Chua Soco executed a promissory note in favor of the Though the court below erred in holding that Chua Soco, by paying one-half
plaintiff Fua Cun for the sum of P25,000 payable in ninety days and drawing of the subscription price of five hundred shares, in effect became the owner
interest at the rate of 1 per cent per month, securing the note with a chattel of two hundred and fifty shares, the judgment appealed from is in the main
mortgage on the shares of stock subscribed for by Chua Soco, who also correct.
endorsed the receipt above mentioned and delivered it to the mortgagee.
The plaintiff thereupon took the receipt to the manager of the defendant Bank The claim of the defendant Banking Corporation upon which it brought the
and informed him of the transaction with Chua Soco, but was told to await action in which the writ of attachment was issued, was for the non-payment
action upon the matter by the Board of Directors. of drafts accepted by Chua Soco and had no direct connection with the
shares of stock in question. At common law a corporation has no lien upon
In the meantime Chua Soco appears to have become indebted to the China the shares of stockholders for any indebtedness to the corporation (Jones on
Banking Corporation in the sum of P37,731.68 for dishonored acceptances of Liens, 3d ed., sec. 375) and our attention has not been called to any statute
commercial paper and in an action brought against him to recover this creating such lien here. On the contrary, section 120 of the Corporation Act
amount, Chua Soco's interest in the five hundred shares subscribed for was provides that "no bank organized under this Act shall make any loan or
attached and the receipt seized by the sheriff. The attachment was levied discount on the security of the shares of its own capital stock, nor be the
after the defendant bank had received notice of the facts that the receipt had purchaser or holder of any such shares, unless such security or purchase
been endorsed over to the plaintiff. shall be necessary to prevent loss upon a debt previously contracted in good
faith, and stock so purchased or acquired shall, within six months from the
Fua Cun thereupon brought the present action maintaining that by virtue of time of its purchase, be sold or disposed of at public or private sale, or, in
the payment of the one-half of the subscription price of five hundred shares default thereof, a receiver may be appointed to close up the business of the
Chua Soco in effect became the owner of two hundred and fifty shares and bank in accordance with law."
praying that his, the plaintiff's, lien on said shares, by virtue of the chattel
mortgage, be declared to hold priority over the claim of the defendant Section 35 of the United States National Banking Act of 1864 contains a
Banking Corporation; that the defendants be ordered to deliver the receipt in similar provision and it has been held in various decisions of the United
question to him; and that he be awarded the sum of P5,000 in damages for States Supreme Court that a bank organized under that Act can have no lien
wrongful attachment. on its own stock for the indebtedness of the stockholders even when the by-
laws provide that the shares shall be transferable only on the books of the
The trial court rendered judgment in favor of the plaintiff declaring that Chua corporation and that no such transfer shall be made if the holder of the
Soco, through the payment of the P25,000, acquired the right to two hundred shares is indebted to the corporation. (Jones on Liens, 3d ed., sec. 384; First
and fifty shares fully paid up, upon which shares the plaintiff holds a lien National Bank of South Bend vs.Lanier and Handy, 11 Wall., 369;
superior to that of the defendant Banking Corporation and ordering that the Bullard vs. National Eagle Bank, 18 Wall., 589; First National Bank of
receipt be returned to said plaintiff. From this judgment the defendants Xenia vs.Stewart and McMillan, 107 U.S., 676.) The reasons for this doctrine
appeal. are obvious; if banking corporations were given a lien on their own stock for
the indebtedness of the stockholders, the prohibition against granting loans
or discounts upon the security of the stock would become largely ineffective.

Turning now to the rights of the plaintiff in the stock in question, it is argued
that the interest held by Chua Soco was merely an equity which could not be

714
made the subject of a chattel mortgage. Though the courts have uniformly For value received, I assign all my rights in these shares in favor of
held that chattel mortgages on shares of stock and other choses in action are Mr. Tua Cun.
valid as between the parties, there is still much to be said in favor of the
defendants' contention that the chattel mortgage here in question would not Manila, P. I., May 18, 1921.
prevail over liens of third parties without notice; an equity in shares of stock is
of such an intangible character that it is somewhat difficult to see how it can (Sgd.) CHUA SOCO
be treated as a chattel and mortgaged in such a manner that the recording of
the mortgage will furnish constructive notice to third parties. As said by the
This endorsement was accompanied by the delivery of the receipt to the
court in the case of Spalding vs. Paine's Adm'r. (81 Ky., 416), in regard to a
plaintiff and further strengthened by the execution of the chattel mortgage,
chattel mortgage of shares of stock:
which mortgage, at least, operated as a conditional equitable assignment.

These certificates of stock are in the pockets of the owner, and go


As against the rights of the plaintiff the defendant bank had, as we have
with him where he may happen to locate, as choses in action, or
seen, no lien unless by virtue of the attachment. But the attachment was
evidence of his right, without any means on the part of those with
levied after the bank had received notice of the assignment of Chua Soco's
whom he proposes to deal on the faith of such a security of
interests to the plaintiff and was therefore subject to the rights of the latter. It
ascertaining whether or not this stock is in pledge or mortgaged to
follows that as against these rights the defendant bank holds no lien
others. He finds the name of the owner on the books of the company
whatever.
as a subscriber of paid-up stock, amounting to 180 shares, with the
certificates in his possession, pays for these certificates their full
value, and has the transfer to him made on the books of the As we have already stated, the court erred in holding the plaintiff as the
company, thereby obtaining a perfect title. What other inquiry is he to owner of two hundred and fifty shares of stock; "the plaintiff's rights consist in
make, so as to make his investment certain and secure? Where is he an equity in five hundred shares and upon payment of the unpaid portion of
to look, in order to ascertain whether or not this stock has been the subscription price he becomes entitled to the issuance of certificate for
mortgaged? The chief office of the company may be at one place to- said five hundred shares in his favor."
day and at another tomorrow. The owner may have no fixed or
permanent abode, and with his notes in one pocket and his The judgment appealed from is modified accordingly, and in all other
certificates of stock in the other the one evidencing the extent of respects it is affirmed, with the costs against the appellants Banking
his interest in the stock of the corporation, the other his right to Corporation. So ordered.
money owing him by his debtor, we are asked to say that the
mortgage is effectual as to the one and inoperative as to the other. Araullo, C.J., Street, Malcolm, Avancea, Villamor, Johns, and Romualdez,
JJ., concur.
But a determination of this question is not essential in the present case.
There can be no doubt that an equity in shares of stock may be assigned and
that the assignment is valid as between the parties and as to persons to
whom notice is brought home. Such an assignment exists here, though it
was made for the purpose of securing a debt. The endorsement to the
plaintiff of the receipt above mentioned reads:

715
G.R. No. L-28120 November 25, 1976

RICARDO A. NAVA, petitioner-appellant.


vs.
PEERS MARKETING CORPORATION, RENATO R. CUSI and AMPARO
CUSI, respondents-appellees.

Rolando M. Medalla, for appellant.

Jose Y. Montalvo, for appellees.

AQUINO, J:

This is a mandamus case, Teofilo Po as an incorporator subscribed to eighty


shares of Peers Marketing Corporation at one hundred pesos a share or a
total par value of eight thousand pesos. Po paid two thousand pesos or
twenty-five percent of the amount of his subscription. No certificate of stock
was issued to him or, for that matter, to any incorporator, subscriber or
stockholder.

On April 2, 1966 Po sold to Ricardo A. Nava for two thousand pesos twenty
of his eighty shares. In the deed of sale Po represented that he was "the
absolute and registered owner of twenty shares" of Peers Marketing
Corporation.

716
Nava requested the officers of the corporation to register the sale in the The issue is whether the officers of Peers Marketing Corporation can be
books of the corporation. The request was denied because Po has not paid compelled by mandamus to enter in its stock and transfer book the sale
fully the amount of his subscription. Nava was informed that Po was made by Po to Nava of the twenty shares forming part of Po's subscription of
delinquent in the payment of the balance due on his subscription and that the eighty shares, with a total par value of P8,000 and for which Po had paid only
corporation had a claim on his entire subscription of eighty shares which P2,000, it being admitted that the corporation has an unpaid claim of P6,000
included the twenty shares that had been sold to Nava. as the balance due on Po's subscription and that the twenty shares are not
covered by any stock certificate.
On December 21, 1966 Nava filed this mandamus action in the Court of First
Instance of Negros Occidental, Bacolod City Branch to compel the Apparently, no provision of the by-laws of the corporation covers that
corporation and Renato R. Cusi and Amparo Cusi, its executive vice- situation. The parties did not bother to submit in evidence the by-laws nor
president and secretary, respectively, to register the said twenty shares in invoke any of its provisions. The corporation can include in its by-laws rules,
Nava's name in the corporation's transfer book. not inconsistent with law, governing the transfer of its shares of stock (Sec.
137 , Act No. 1459; Fleischer vs. Botica Nolasco Co., 47 Phil. 583, 589).
The respondents in their answer pleaded the defense that no shares of stock
against which the corporation holds an unpaid claim are transferable in the We hold that the transfer made by Po to Nava is not the "alienation, sale, or
books of the corporation. transfer of stock" that is supposed to be recorded in the stock and transfer
book, as contemplated in section 52 of the Corporation Law.
After hearing, the trial court dismissed the petition. Nava appealed on the
ground that the decision "is contrary to law ". His sole assignment of error is As a rule, the shares which may be alienated are those which are covered by
that the trial court erred in applying the ruling in Fua Cun vs. Summers and certificates of stock, as shown in the following provisions of the Corporation
China Banking Corporation, 44 Phil. 705 to justify respondents' refusal in Law and as intimated in Hager vs. Bryan, 19 Phil. 138 (overruling the
registering the twenty shares in Nava's name in the books of the corporation. decision in Hager vs. Bryan, 21 Phil. 523. See 19 Phil. 616, notes, and
Hodges vs. Lezama, 14 SCRA 1030).
The rule enunciated in the Fua Cun case is that payment of one-half of the
subscription does not entitle the subscriber to a certificate of stock for one- SEC. 35. The capital stock of stock corporations shall be
half of the number of shares subscribed. divided into shares for which certificatessigned by the
president or the vice-president, countersigned by the
Appellant Nava contends that the Fua Cun case was decided under section secretary or clerk and sealed with the seal of the
36 of the Corporation Law which provides that "no certificate of stock shall be corporation, shall be issued in accordance with the by-laws.
issued to a subscriber as fully paid up until the full par value thereof has been Shares of stock so issued are personal property and may be
paid by him to the corporation". Section 36 was amended by Act No. 3518. It transferred by delivery of the certificate indorsed by the
is now section 37. Section 37 provides that "no certificate of stock shall be owner or his attorney in fact or other person legally
issued to a subscriber as fully paid up until the full par valuethereof, or authorized to make the transfer. No transfer, however, shall
the full subscription in case of no par stock, has been paid by him to the be valid, except as between the, parties, until the transfer is
corporation". entered and noted upon the books of the corporation so as
to show the names of the parties to the transaction, the date

717
of the transfer, the number of the certificate, and the number corporation so that the transfer may be entered in the corporation's books.
of shares transferred. The certificate is then surrendered and a new one issued to the transferee.
(Hager vs. Bryan, 19 Phil. 138, 143-4).
No share of stock against which the corporation holds any
unpaid claim shall be transferable on the books of the That procedure cannot be followed in the instant case because, as already
corporation. noted, the twenty shares in question are not covered by any certificate of
stock in Po's name. Moreover, the corporation has a claim on the said shares
SEC. 36. (re voting trust agreement) ... for the unpaid balance of Po's subscription. A stock subscription is a
subsisting liability from the time the subscription is made. The subscriber is
The certificates of stock so transferred shall be surrendered as much bound to pay his subscription as he would be to pay any other debt.
and cancelled, and new certificates therefor issued to such The right of the corporation to demand payment is no less incontestable.
person or persons, or corporation, as such trustee or (Velasco vs. Poizat, 37 Phil. 802; Lumanlan vs. Cura, 59 Phil. 746).
trustees, in which new certificates it shall appear that they
are issued pursuant to said agreement. A corporation cannot release an original subscriber from paying for his
shares without a valuable consideration (Philippine National Bank vs. Bitulok
xxx xxx xxx Sawmill, Inc.,
L-24177-85, June 29, 1968, 23 SCRA 1366) or without the unanimous
consent of the stockholders (Lingayen Gulf Electric Power Co., Inc. vs.
(Emphasis supplied).
Baltazar, 93 Phil 404).

(In the case of nonstock corporations a membership certificate is usually


Under the facts of this case, there is no clear legal duty on the part of the
issued. Lee E. Won vs. Wack Wack Golf & Country Club, Inc., 104 Phil. 466;
officers of the corporation to register the twenty shares in Nava's name,
Wack Wack Golf & Country Club, Inc. vs. Won, L-23851, March 26, 1976, 70
Hence, there is no cause of action for mandamus.
SCRA 165).

Nava argues that under section 37 a certificate of stock may be issued for
As prescribed in section 35, shares of stock may be transferred by delivery to
shares the par value of which have already been paid for although the entire
the transferee of the certificate properly indorsed. "Title may be vested in the
subscription has not been fully paid. He contends that Peers Marketing
transferee by delivery of the certificate with a written assignment or
Corporation should issue a certificate of stock for the twenty shares,
indorsement thereof" (18 C.J.S. 928). There should be compliance with the
notwithstanding that Po had not paid fully his subscription for the eighty
mode of transfer prescribed by law (18 C.J.S. 930).
shares, because section 37 requires full payment for the subscription, as a
condition precedent for the issuance of the certificate of stock, only in the
The usual practice is for the stockholder to sign the form on the back of the case of no par stock.
stock certificate. The certificate may thereafter be transferred from one
person to another. If the holder of the certificate desires to assume the legal
Nava relies on Baltazar v Lingayen Gulf Electric Power Co., Inc., L-16236-38,
rights of a shareholder to enable him to vote at corporate elections and to
June 30, 1965, 14 SCRA 522, where it was held that section 37 "requires as
receive dividends, he fills up the blanks in the form by inserting his own name
a condition before a shareholder can vote his shares that his full subscription
as transferee. Then he delivers the certificate to the secretary of the

718
be paid in the case of no par value stock; and in case of stock corporation
with par value, the stockholder can vote the shares fully paid by him only,
irrespective of the unpaid delinquent shares".

There is no parallelism between this case and the Baltazar case. It is


noteworthy that in the Baltazar case the stockholder, an incorporator, was the
holder of a certificate of stock for the shares the par value of which had been
paid by him. The issue was whether the said shares had voting rights
although the incorporator had not paid fully the total amount of his
subscription. That is not the issue in this case.

In the Baltazar case, it was held that where a stockholder subscribed to a


certain number of shares with par value and he made a partial payment and
was issued a certificate for the shares covered by his partial payment, he is
entitled to vote the said shares, although he has not paid the balance of his
subscription and a call or demand had been made for the payment of the par
value of the delinquent shares.

As already stressed, in this case no stock certificate was issued to Po.


Without stock certificate, which is the evidence of ownership of corporate
stock, the assignment of corporate shares is effective only between the
parties to the transaction (Davis vs. Wachter, 140 So. 361).

The delivery of the stock certificate, which represents the shares to be G.R. No. L-42135 June 17, 1935
alienated , is essential for the protection of both the corporation and its
stockholders (Smallwood vs. Moretti, 128 So. 2d 628). TORIBIA USON, plaintiff-appellee,
vs.
In view of the foregoing considerations, the trial court's judgment dismissing VICENTE DIOSOMITO, ET AL., defendants.
the petition for mandamus is affirmed. Costs against the petitioner-appellant. VICENTE DIOSOMITO, EMETERIO BARCELON, H.P.L. JOLLYE and
NORTH ELECTRIC COMPANY, INC.,appellants.
SO ORDERED.
Lino S. Gabriel for appellant Diosomito.
Fernando (Chairman), Barredo, Antonio and Concepcion, Jr., JJ., concur. Emeterio Barcelon and M.B. Villanueva for appellant Barcelon.
Ross, Lawrence and Selph for appellants Jollye and North Electric Co., Inc.
Encarnacion and Arca for appellee.

719
BUTTE, J.: It will be seen, therefore, that the transfer of said shares by Vicente
Diosomito, the judgment debtor in suit No. 2525, to Barcelon was not
This is an appeal from a decision of the Court of First Instance of Cavite registered and noted on the books of the corporation until September 16,
involving the ownership of seventy-five shares of stock in the North Electric 1932, which was some nine months after the attachment had been levied on
Company, Inc. The plaintiff-appellee claims to be the owner of these shares said shares in civil case No. 2525 as above stated.
by virtue of purchase at a sheriff's sale for the sum of P2,617.18.
Thus arises in this case one of the most vexing questions in the law of
It appears that Toribia Uson had filed a civil action for debt in the Court of corporations, namely, whether a bona fidetransfer of the shares of a
First Instance of Cavite, No. 2525, against Vicente Diosomito and that upon corporation, not registered or noted on the books of the corporation, is valid
institution of said action an attachment was duly issued and levied upon the as against a subsequent lawful attachment of said shares, regardless of
property of the defendant Diosomito, including seventy-five shares of the whether the attaching creditor had actual notice of said transfer or not. This is
North Electric Co., Inc., which stood in his name on the books of the the first case in which this question has been squarely presented to us for
company when the attachment was levied on January 18, 1932. decision. The case of Uy Piaco vs. McMicking (10 Phil., 286), decided in
Subsequently, on June 23, 1932, in said civil case No. 2525, Toribia Uson 1908, arose before the Philippine Corporation Law, Act No. 1459, took effect
obtained judgment against the defendant Diosomito for the sum of P2,300 (April 1, 1906). The cases of Fua Cun vs. Summer and China Banking
with interest and costs. To satisfy said judgment, the sheriff sold said shares Corporation, 44 Phil., 705 [1923] and Fleischer vs. Botica Nolasco Co., 47
at public auction in accordance with law on March 20, 1933. The plaintiff Phil., 583 [1925] are not in point.
Toribia Uson was the highest bidder and said shares were adjudicated to her.
(See Exhibit K.) In the present action, H.P.L. Jollye claims to be the owner of Section 35 of the Corporation Law is as follows:
said 75 shares of the North Electric Co., Inc., and presents a certificate of
stock issued to him by the company on February 13, 1933. SEC. 35. The capital stock of stock corporations shall be divided into
shares for which certificates signed by the president or the vice-
There is no dispute that the defendant Vicente Diosomito was the original president, countersigned by the secretary or clerk and sealed with
owner of said shares of stock, having a par value of P7,500, and that on the by-laws. Shares of stock so issued are personal property and
February 3, 1931, he sold said shares to Emeterio Barcelon and delivered to may be transferred by delivery of the certificate indorsed by the
the latter the corresponding certificates Nos. 2 and 19. But Barcelon did not owner or his attorney in fact or other person legally authorized to
present these certificates to the corporation for registration until the 16th of make the transfer. No transfer, however, shall be valid, except as
September, 1932, when they were cancelled and a new certificate, No. 29, between the parties, until the transfer is entered and noted upon the
was issued in favor of Barcelon, who transferred the same of the defendant books of the corporation so as to show the names of the parties to
H.P.L. Jollye to whom a new certificate No. 25 was issued on February 13, the transaction, the date of the transfer, the number of the certificate,
1933. and the number of shares transferred.

No shares of stock against which the corporation holds any unpaid


claim shall be transferable on the books of the corporation.

720
The sentence of the foregoing section immediately applicable in the present amended it by statute. Second, that group which, like our own Act No. 1459,
case is as follows: holds to the rule that no transfer shall be valid except as between the parties
until the transfer is duly registered. This group, according to the best
No transfer, however, shall be valid, except as between the parties, information available here, includes or has included the State of Arizona,
until the transfer is entered and noted upon the books of the California, the Territory of Hawaii, Idaho, Iowa, Nevada, New Mexico, North
corporation so as to show the names of the parties to the Dakota, Oklahoma, South Dakota, Washington, Wisconsin. The thirds group
transaction, the date of the transfer, the number of the certificate, which includes the remaining jurisdictions follows the rule and the doctrine
and the number of shares transferred. invoked by the appellant in this case, which, by amendment of the statutes, is
becoming the prevailing rule in the United States.
The appellants cites decision from a number of states of the American Union
which hold that an unregistered transfer is valid as against the lien of a The decision of the Supreme Court of California in the case of National Bank
subsequent attachment sued out by a creditor of the assignor, whether such of the Pacific vs. Western Pacific Railway Company (157 Cal., 573 [1910];
creditor has notice of the transfer or not. These decisions are founded upon 108 Pac., 676), sitting in division of three, construed section 324 of the Civil
the theory that the attachment reaches only such title or interest as the Code of California which is identical with section 35, supra, of the Philippine
defendant may have in the property at the time of the levy; and if all title and Corporation Law. The court stressed the provision that the shares of stock in
interest had previously passed by assignment from the debtor to a third a corporation are personal property and may be transferred by endorsement
person, the attaching creditor obtains nothing by the levy; that the owner of and delivery of the certificate. The opinion also endeavors to distinguish the
shares of stock has the common law right to dispose of the same as personal prior decisions of Weston vs. Bear River and Auburn Water and Mining Co.
property. But with the exception of California, to which reference will be made (5 Cal., 186); Strout vs. Natoma Water and Mining Company (9 Cal., 78), and
later, none of the decisions cited by the appellants construed statues Naglee vs. Pacific Wharf Company (20 Cal., 529), which are frequently cited
identical with ours. Much of the confusion which is to be found in the decision in other jurisdictions as sustaining the theory of the superiority of the
has arisen because the courts have failed to note the difference in the attachment lien over the unregistered stock transfer. (See Lyndonville
various statutes of the American Union on the question considered here. For National Bank vs. Folsom, 7 N.M., 611 [1894]; 38 Pac., 253.) The California
an illuminating discussion of this confusion the following authorities may be decision leaves us unconvinced that the statutes which fall in the second
consulted: group above mentioned should be given the same effect as the statute in the
third group without any necessity for legislative amendment.
Fletcher, Cyclopedia of the Law of Private Corporations (1932), vol.
12, pages 358-389. We prefer to adopt the line followed by the Supreme Courts of
American and English Annotated Cases, vol. 21, pages 1391-1407. Massachusetts and of Wisconsin. (See Clews vs. Friedman, 182 Mass., 555;
American Law Review, vol. 35, pages 238-251. 55 Cent. L. J., 243- 66 N.E. 201, and In re Murphy, 51 Wis., 519; 8 N.W., 419.)
251.
In the latter case the court had under consideration a statute identical with
The statutes on this point may be put roughly in three groups: First, those our own section 35, supra, and the court said:
that provide, in substance, that no transfer of shares is valid for any purpose
unless registered on the books of the corporation. This rule apparently once We think the true meaning of the language is, and the obvious
prevailed in Colorado and the District of Columbia both of which have since intention of the legislature in using it was, that all transfers of shares

721
should be entered, as here required, on the books of the corporation.
And it is equally clear to us that all transfers of shares not so entered
are invalid as to attaching or execution creditors of the assignors, as
well as to the corporation and to subsequent purchasers in good
faith, and indeed, as to all persons interested, except the parties to
such transfers. All transfers not so entered on the books of the
corporation are absolutely void; not because they are without notice
or fraudulent in law or fact, but because they are made so void by
statute.

Some of the states, including Wisconsin, which has held to the rather, strict
but judicial interpretation of the statutory language here in question have
amended the statute so as to fall in line with the more liberal and rational
doctrine of the third group referred to above. This court still adheres to the
principle that its function is jus dicere non jus dare. To us the language of the
legislature is plain to the effect that the right of the owner of the shares of
stock of a Philippine corporation to transfer the same by delivery of the
certificate, whether it be regarded as statutory on common law right, is
limited and restricted by the express provision that "no transfer, however,
shall be valid, except as between the parties, until the transfer is entered and
noted upon the books of the corporation." Therefore, the transfer of the 75
shares in the North Electric Company, Inc., made by the defendant Diosomito
to the defendant Barcelon was not valid as to the plaintiff-appellee, Toribia
Uson, on January 18, 1932, the date on which she obtained her attachment
lien on said shares of stock which still stood in the name of Diosomito on the
books of the corporation.

We have considered the remaining assignments of error of the appellants


and finding no merit in them in results that the judgment must be affirmed
with costs against the appellants.

Malcolm and Diaz, JJ., concur.


Goddard, J., I agree with Justice Hull.

722
The present case, which was instituted by Antonio Escao against the
Filipinas Mining Corporational and the Standard Investment of the
Philippines, relates to the escrow shares involved in the garnishment
proceeding above mentioned. It appears that after the complaint in the
original case of Escao vs. Salvosa was filed but before judgment we as
rendered therein, that lis to say, on November 21, 1936, Silverio Salvosa sold
to Jose P. Bengzon all his right, title, and interest in and to 18,580 shares of
G.R. No. L-49003 July 28, 1944
stock of the Filipinas Mining Corporation held in escrow which the said
Salvosa was entitled to receive, and which Bengzon in turn subsequently
ANTONIO ESCAO, plaintiff-appellee, sold and transferred to the present defendant-appellant, Standard Investment
vs. of the Philippines. Neither Salvosa's sale to Bengzon nor Bengzon's sale to
FILIPINAS MINING CORPORATION, ET Al., defendants. the Standard Investment of the Philippines was notified to and recorded in
STANDARD INVESTMENT OF THE PHILIPPINES, appellant. the books of the Filipinas Mining Corporation until December 7, 1940, that is
to say, more than three years after the escrow shares in question were
Jose P. Bengzon for appellant. attached by garnishment served on the Filipinas Mining Corporation as
Matias E. Vergara and Jose Ma. Reyes for appellee. hereinbefore set forth. On January 24, 1941, the defendant Filipinas Mining
Corporation issued in favor of the defendant Standard Investment of the
OZAETA, J.: Philippines certificate of stock for the 18,580 shares formerly held in escrow
by Silverio Salvosa and which had been adversely by the present plaintiff-
This case was submitted to and decided by the Court of First Instance of appellee on the one hand and the Standard Investment of the Philippines on
Manila upon an agreed statement of facts which may be restated as follows: the other, the first by virtue of garnishment proceedings and the second by
virtue of the sale made to it by Jose P. Bengzon as aforesaid.
On March 8, 1937, the plaintiff-appellee obtained judgment in the Court of
First Instance of Manila against Silverio Salvosa whereby the latter was The question to determine is whether the issuance by the Filipinas Mining
ordered to transfer and deliver to the former 116 active shares and an Corporation of the said 18,580 shares of its stock to the Standard Investment
undetermined number of shares in escrow of the Filipinas Mining Corporation of the Philippines was valid as against the attaching judgment creditor of the
and to pay the sum of P500 as damages, with the proviso that the escrow original owner, Silverio Salvosa, namely, the present plaintiff-appellee
shares shall be transferred and delivered to the plaintiff only after they shall Antonio Escao.
have been released by the company. On June 25, 1937, a writ of
garnishment was served by the sheriff of Manila upon the Filipinas Mining In addition to the above stipulated facts, the trial court found from the
Corporation to satisfy the said judgment; and on July 29, 1937, the Filipinas supplementary oral evidence adduced by the plaintiff "that several promises
Mining Corporation advised the sheriff of Manila that according to its books were made by the secretary of the defendant Filipinas Mining Corporation
the judgment debtor Silverio Salvosa was the registered owner of 1,000 that as soon as the escrow shares pertaining to Silverio Salvosa were
active shares and about 21,339 unissued shares held in escrow by the said released he (the secretary) would notify the plaintiff so that the latter might
corporation. The sheriff sold the 1,000 active shares at public auction, take the proper action for the execution of the judgment rendered in the said
realizing therefrom only the sum of P10, which was applied in partial case entitled "Antonio Escao vs. Silverio Salvosa," civil case No. 50575 of
satisfaction of the judgment for damages in the sum of P500. the Court of First Instance of Manila. But the secretary, instead of complying

723
with his promises, issued the escrow shares to the defendant Standard 1. Sections 431 and 432 of the Code of Civil Procedure (now sections 7 and
Investment of the Philippines . . ." 8 of Rule 59), which were in force at the time the garnishment in question
was served on the defendant Filipinas Mining Corporation, provide as
The trial court held that the transfer of the escrow shares in question from follows:
Salvosa to Bengzon and from Bengzon to the Standard Investment of the
Philippines, not having been recorded in the books of the corporation as Sec. 431. Executing Order of Attachment as to debts and Credits.
required by section 35 of the Corporation Law, could not prevail over the Debts and credits, and other personal property not capable of
garnishment previously made by the plaintiff of the said shares, and rendered manual delivery, shall be attached by leaving with the person owing
judgment "ordering the defendants Filipinas Mining Corporation and the such debts or having in his possession or under his control, such
Standard Investment of the Philippines to issue to the plaintiff out of the credits and other personal property, a copy of the order of
escrow shares which formerly belonged to Silverio Salvosa, 4,152 shares of attachment and a notice that the debts owing by him to the
the Filipinas Mining Corporation and to pay to him the dividends which have defendant, or the credits and other personal property in his
been and may be declared on said shares until the delivery thereof to the possession or under his control, belonging to the defendant, are
plaintiff; and ordering the sheriff to levy execution on the remaining shares attached in pursuance of such order.
which formerly belonged to Silverio Salvosa in order to satisfy the balance of
the judgment rendered in the civil case entitled "Antonio Escao vs. Silverio Sec. 432. Effect of Attachment of Debts and Credits. All persons
Salvosa," civil case No. 50575 of the Court of First Instance of Manila, with having in their possession or under their control any credits or other
costs against the defendants." From that judgment the Standard Investment personal property belonging to the defendant, or owing any debts to
of the Philippines has appealed to this Court and makes the following the defendant at the time of service upon them of a copy of the order
assignment of errors: of attachment and notice as provided in the last section, shall be,
unless such property be delivered up or transferred, or such debts be
1. The trial court erred in holding that section 35 of Act 14599 and paid to the clerk of the court in which the action is pending, liable to
the doctrine laid down in the case of Uson vs. Diosomito, 61 Phil., the plaintiff for the amount of such credits, property, or debts, until
535, are applicable to the case at bar. the attachment be discharged, or any judgment recovered by him be
satisfied."
2. The trial court erred in "ordering the sheriff to levy execution on
the remaining shares of the 18,580 shares to satisfy the balance of Under the section last above quoted, the Filipinas Mining Corporation
the judgment rendered in civil case No. 50575 of the Court of First became liable to the plaintiff for the shares of stock mentioned in its return to
Instance of Manila"; and in not holding that because of the delay or the sheriff of July 29, 1937, wherein it informed the latter in response to the
neglect for an unreasonable length of time by the plaintiff to enforce notice of garnishment "that according to its books said Silverio Salvosa was
his execution, the 18,580 shares affected in this litigation has been the registered owner of 1,000 active shares evidence by certificate of stock
discharged thru his waiver or abandonment. No. 235 and about 21,338 unissued shares held in escrow by the defendant
Filipinas Mining Corporation."

Counsel for the appellant Standard Investment of the Philippines contends


that a distinction should be drawn between issued shares evidenced by

724
certificates of stock and unissued shares held in escrow, in that while the possibility of fictitious or fraudulent transfers exists. The only reason
transfer of the former is subject to the restriction contained in section 35 of advanced by the appellant for exempting the transfer of unissued shares
the Corporation Law, that of the latter is not. The said section, insofar as from recording is that in case of unissued shares there is no certificate
pertinent here, reads as follows: number to be recorded. But that is a mere detail which does not affect the
reasons behind the rule. The lack of such detail does not make it impossible
. . . Shares of stock so issued are personal property and may be to record the transfer upon the books of the corporation so as to show the
transferred by delivery of the certificate indorsed by the owner or his names of the parties to the transaction, the date of the transfer, and the
attorney in fact or other person legally authorized to make the number of shares transferred, which are the most essential data. As a matter
transfer. No transfer, however, shall be valid, except as between the of fact, the defendant Filipinas Mining Corporation was able to take not of the
parties, until the transfer is entered and noted upon the books of the transfer of the escrow shares in question to the Standard Investment of the
corporation so as to show the names of the parties to the Philippines on December 7, 1940, without knowing the certificate number
transaction, the date of the transfer, the number of the certificate, that would correspond to said shares.
and the number of shares transferred.
Moreover, it seems illogical and unreasonable to hold that inactive or
It is admitted that under this legal provision and the decision of this Court unissued shares still held by the corporation in escrow pending receipt of
in Uson vs. Diosomito, 61 Phil. 535, the transfer of duly issued shares of authorization from the Government to issue them, may be negotiated or
stock is not valid as against third parties and the corporation until it is noted transferred unrestrictedly and more freely than active or issued shares
upon the books of the corporation; but it is contended that the transfer of evidenced by certificates of stock.
unissued shares of stock held in escrow is valid against the whole world
although not notified to the corporation and not noted upon its books. Since We are, therefore, of the opinion and so hold that section 35 of the
the sale, transfer, or assignment of unissued shares of stock held in escrow Corporation Law, which requires the registration of transfers of shares stock
is not specifically provided for by law, the question has to be resolved by upon the books of the corporation as a condition precedent to their validity
resorting to analogy. What is the reason of the law for requiring the recording against the corporation and third parties, is also applicable to unissued
upon the books of the corporation of transfers of shares of stock as a shares held by the corporation in escrow.
condition precedent to their validity against the corporation, and third parties?
We imagine that it is (1) to enable the corporation to know at all times who its 2. Under its second assignment of error appellant contends that appellee has
actual stockholders are, because mutual rights and obligations exist between been guilty of laches in neglecting for an unreasonable length of time to
the corporation and its stockholders; (2) to afford to the corporation an enforce its levy on the 18,580 shares of stock in question by having them
opportunity to object or refuse its consent to the transfer in case it has any sold at public auction, and that, consequently, said levy should be considered
claim against the stock sought to be transferred, or for any other valid discharged through waiver or abandonment. We find no factual basis for the
reason; and (3) to avoid fictitious or fraudulent transfers. Do these reasons alleged laches and abandonment. The trial court found that the secretary of
hold as to the transfer of unissued shares held in escrow? To sustain the defendant Filipinas Mining Corporation had repeatedly promised the
appellant's contention is to declare that they do not. But we see no valid plaintiff that he would notify the latter as soon as the escrow shares
reason for treating unissued shares held in escrow differently from issued pertaining to Silverio Salvosa were released so that he ((plaintiff) might take
shares insofar as their sale and transfer is concerned. In both cases the the proper action for the execution of his judgment. The Filipinas Mining
corporation is entitled to know who the actual owners of the shares are, and Corporation having advised the sheriff that it was holding the escrow shares
to object to the transfer upon any valid ground. Likewise, in both cases the of the judgment debtor Silverio Salvosa, the plaintiff as execution creditor

725
had the right to wait for the release or issuance of said shares before having Securities and Exchange Commission (SEC) En Banc in SEC-AC No. 545
the same sold at public auction, so long as the period of five years within and reinstated the order[3] of the Hearing Officer dismissing herein petitioners
which to execution his judgment had not yet lapsed. Moreover, the judgment complaint. Also assailed is the CAs resolution [4] of August 10, 1999, denying
itself provided "that the escrow shares shall be transferred and delivered to petitioners motion for reconsideration.
the plaintiff only after they have been released by the company." It is stated
in the stipulation of facts that it was only after shares in favor of the Standard On January 25, 1996, plaintiff (now petitioner) Vicente C. Ponce, filed a
Investment of the Philippines that the plaintiff Antonio Escao came to know complaint[5] with the SEC for mandamus and damages against defendants
that Jose P. Bengzon and the Standard Investment of the Philippines had (now respondents) Alsons Cement Corporation and its corporate secretary
acquired Silverio Salvosa's rights to the shares in question. Upon these facts, Francisco M. Giron, Jr. In his complaint, petitioner alleged, among others,
together with the consideration that the delay had not in any way misled the that:
appellant to its prejudice, we find appellant's second assignment of error
untenable. xxx

The judgment appealed from is affirmed, with costs. 5. The late Fausto G. Gaid was an incorporator of Victory Cement
Corporation (VCC), having subscribed to and fully paid 239,500 shares of
Yulo, C.J., Moran, Horrilleno, and Bacobo, JJ., concur. said corporation.
Paras, J., I concurs in the result.
6. On February 8, 1968, plaintiff and Fausto Gaid executed a Deed of
Undertaking and Indorsement whereby the latter acknowledges that the
former is the owner of said shares and he was therefore assigning/endorsing
the same to the plaintiff. A copy of the said deed/indorsement is attached as
Annex A.

7. On April 10, 1968, VCC was renamed Floro Cement Corporation (FCC for
[G.R. NO. 139802. December 10, 2002] brevity).

VICENTE C. PONCE, petitioner, vs. ALSONS CEMENT CORPORATION, 8. On October 22, 1990, FCC was renamed Alsons Cement Corporation
and FRANCISCO M. GIRON, JR., respondents. (ACC for brevity) as shown by the Amended Articles of Incorporation of ACC,
a copy of which is attached as Annex B.
DECISION
9. From the time of incorporation of VCC up to the present, no certificates of
QUISUMBING, J.: stock corresponding to the 239,500 subscribed and fully paid shares of Gaid
were issued in the name of Fausto G. Gaid and/or the plaintiff.
This petition for review seeks to annul the decision [1] of the Court of
Appeals, in CA-G.R. SP No. 46692, which set aside the decision [2] of the 10. Despite repeated demands, the defendants refused and continue to
refuse without any justifiable reason to issue to plaintiff the certificates of

726
stocks corresponding to the 239,500 shares of Gaid, in violation of plaintiffs the 239,500 shares of stocks and its legal increments and (b) to pay him
right to secure the corresponding certificate of stock in his name. [6] damages.[8]

Attached to the complaint was the Deed of Undertaking and Instead of filing an answer, respondents moved to dismiss the complaint
Indorsement[7] upon which petitioner based his petition for mandamus. Said on the grounds that: (a) the complaint states no cause of action; mandamus
deed and indorsement read as follows: is improper and not available to petitioner; (b) the petitioner is not the real
party in interest; (c) the cause of action is barred by the statute of limitations;
DEED OF UNDERTAKING and (d) in any case, the petitioners cause of action is barred by laches.
[9]
They argued, inter alia, that there being no allegation that the alleged
KNOW ALL MEN BY THESE PRESENTS: INDORSEMENT was recorded in the books of the corporation, said
indorsement by Gaid to the plaintiff of the shares of stock in
questionassuming that the indorsement was in fact a transfer of stockswas
I, VICENTE C. PONCE, is the owner of the total subscription of Fausto Gaid
not valid against third persons such as ALSONS under Section 63 of the
with Victory Cement Corporation in the total amount of TWO HUNDRED
Corporation Code.[10] There was, therefore, no specific legal duty on the part
THIRTY NINE THOUSAND FIVE HUNDRED (P239,500.00) PESOS and that
of the respondents to issue the corresponding certificates of stock, and
Fausto Gaid does not have any liability whatsoever on the subscription
mandamus will not lie.[11]
agreement in favor of Victory Cement Corporation.

Petitioner filed his opposition to the motion to dismiss on February 19,


(SGD.) VICENTE C. PONCE
1996 contending that: (1) mandamus is the proper remedy when a
corporation and its corporate secretary wrongfully refuse to record a transfer
February 8, 1968 of shares and issue the corresponding certificates of stocks; (2) he is the
proper party in interest since he stands to be benefited or injured by a
CONFORME: judgment in the case; (3) the statute of limitations did not begin to run until
defendant refused to issue the certificates of stock in favor of the plaintiff on
(SGD.) FAUSTO GAID April 13, 1992.

INDORSEMENT After respondents filed their reply, SEC Hearing Officer Enrique L.
Flores, Jr. granted the motion to dismiss in an Order dated February 29,
I, FAUSTO GAID is indorsing the total amount of TWO HUNDRED THIRTY 1996, which held that:
NINE THOUSAND FIVE HUNDRED (239,500.00) stocks of Victory Cement
Corporation to VICENTE C. PONCE. xxx

(SGD.) FAUSTO GAID Insofar as the issuance of certificates of stock is concerned, the real party in
interest is Fausto G. Gaid, or his estate or his heirs. Gaid was an
With these allegations, petitioner prayed that judgment be rendered incorporator and an original stockholder of the defendant corporation who
ordering respondents (a) to issue in his name certificates of stocks covering subscribed and fully paid for 239,500 shares of stock (Annex "B"). In

727
accordance with Section 37 of the old Corporation Law (Act No. 1459) rights as such stockholder. This is because the SEC by express mandate has
obtaining in 1968 when the defendant corporation was incorporated, as well absolute jurisdiction, supervision and control over all corporations and is
as Section 64 of the present Corporation Code (Batas Pambansa Blg. 68), a called upon to enforce the provisions of the Corporation Code, among which
stockholder who has fully paid for his subscription together with interest and is the stock purchasers right to secure the corresponding certificate in his
expenses in case of delinquent shares, is entitled to the issuance of a name under the provisions of Section 63 of the Code. Needless to say, any
certificate of stock for his shares. According to paragraph 9 of the Complaint, problem encountered in securing the certificates of stock representing the
no stock certificate was issued to Gaid. investment made by the buyer must be expeditiously dealt with through
administrative mandamus proceedings with the SEC, rather than through the
Comes now the plaintiff who seeks to step into the shoes of Gaid and usual tedious regular court procedure. xxx
thereby become a stockholder of the defendant corporation by demanding
issuance of the certificates of stock in his name. This he cannot do, for two Applying this principle in the case on hand, a transfer or assignment of
reasons: there is no record of any assignment or transfer in the books of the stocks need not be registered first before the Commission can take
defendant corporation, and there is no instruction or authority from the cognizance of the case to enforce his rights as a stockholder. Also, the
transferor (Gaid) for such assignment or transfer. Indeed, nothing is alleged problem encountered in securing the certificates of stock made by the buyer
in the complaint on these two points. must be expeditiously taken up through the so-called administrative
mandamus proceedings with the SEC than in the regular courts. [13]
xxx
The Commission En Banc also found that the Hearing Officer erred in
In the present case, there is not even any indorsement of any stock holding that petitioner is not the real party in interest.
certificate to speak of. What the plaintiff possesses is a document by which
Gaid supposedly transferred the shares to him. Assuming the document has xxx
this effect, nevertheless there is neither any allegation nor any showing that it
is recorded in the books of the defendant corporation, such recording being a As appearing in the allegations of the complaint, plaintiff-appellant is the
prerequisite to the issuance of a stock certificate in favor of the transferee. [12] transferee of the shares of stock of Gaid and is therefore entitled to avail of
the suit to obtain the proper remedy to make him the rightful owner and
Petitioner appealed the Order of dismissal. On January 6, 1997, the holder of a stock certificate to be issued in his name. Moreover, defendant-
Commission En Banc reversed the appealed Order and directed the Hearing appellees failed to show that the transferor nor his heirs have refuted the
Officer to proceed with the case. In ruling that a transfer or assignment of ownership of the transferee. Assuming these allegations to be true, the
stocks need not be registered first before it can take cognizance of the case corporation has a mere ministerial duty to register in its stock and transfer
to enforce the petitioners rights as a stockholder, the Commission En Banc book the shares of stock in the name of the plaintiff-appellant subject to the
cited our ruling in Abejo vs. De la Cruz, 149 SCRA 654 (1987) to the effect determination of the validity of the deed of assignment in the proper
that: tribunal. [14]

xxx As the SEC maintains, There is no requirement that a stockholder of a Their motion for reconsideration having been denied, herein
corporation must be a registered one in order that the Securities and respondents appealed the decision [15] of the SEC En Banc and the
Exchange Commission may take cognizance of a suit seeking to enforce his resolution[16] denying their motion for reconsideration to the Court of Appeals.

728
In its decision, the Court of Appeals held that in the absence of any and cancel the old one. A transferee who requests for the issuance of a stock
allegation that the transfer of the shares between Fausto Gaid and Vicente certificate need not spell out each and every act that needs to be done by the
C. Ponce was registered in the stock and transfer book of ALSONS, Ponce corporate secretary, as a request for issuance of stock certificates
failed to state a cause of action. Thus, said the CA, the complaint for necessarily includes a request for the recording of the transfer. Ergo, the
mandamus should be dismissed for failure to state a cause of action. failure to record the transfer does not mean that the transferee cannot ask for
[17]
petitioners motion for reconsideration was likewise denied in a the issuance of stock certificates.
resolution[18] dated August 10, 1999.
Secondly, according to petitioner, there is no law, rule or regulation
Hence, the instant petition for review on certiorari alleging that: requiring a transferor of shares of stock to first issue express instructions or
execute a power of attorney for the transfer of said shares before a certificate
I. THE HONORABLE COURT OF APPEALS ERRED IN HOLDING THAT of stock is issued in the name of the transferee and the transfer registered in
THE COMPLAINT FOR ISSUANCE OF A CERTIFICATE OF STOCK FILED the books of the corporation. He contends that Hager vs. Bryan, 19 Phil. 138
BY PETITIONER FAILED TO STATE A CAUSE OF ACTION BECAUSE IT (1911), and Rivera vs. Florendo, 144 SCRA 643 (1986), cited by
DID NOT ALLEGE THAT THE TRANSFER OF THE SHARES (SUBJECT respondents, do not apply to this case. These cases contemplate a situation
MATTER OF THE COMPLAINT) WAS REGISTERED IN THE STOCK AND where a certificate of stock has been issued by the company whereas in this
TRANSFER BOOK OF THE CORPORATION, CITING SECTION 63 OF THE case at bar, no stock certificates have been issued even in the name of the
CORPORATION CODE. original stockholder, Fausto Gaid.

II. THE HONORABLE COURT OF APPEALS ERRED IN NOT APPLYING Finally, petitioner maintains that since he is under no compulsion to
THE CASES OF ABEJO VS. DE LA CRUZ, 149 SCRA 654 AND RURAL register the transfer or to secure stock certificates in his name, his cause of
BANK OF SALINAS, INC., ET AL VS. COURT OF APPEALS, ET AL., G.R. action is deemed not to have accrued until respondent ALSONS denied his
NO. 96674, JUNE 26, 1992. request.

III. THE HONORABLE COURT OF APPEALS ERRED IN APPLYING A 1911 Respondents, in their comment, maintain that the transfer of shares of
CASE, HAGER VS. BRYAN, 19 PHIL. 138, TO DISMISS THE COMPLAINT stock not recorded in the stock and transfer book of the corporation is non-
FOR ISSUANCE OF A CERTIFICATE OF STOCK.[19] existent insofar as the corporation is concerned and no certificate of stock
can be issued in the name of the transferee. Until the recording is made, the
At issue is whether the Court of Appeals erred in holding that herein transfer cannot be the basis of issuance of a certificate of stock. They add
petitioner has no cause of action for a writ of mandamus. that petitioner is not the real party in interest, the real party in interest being
Fausto Gaid since it is his name that appears in the records of the
corporation. They conclude that petitioners cause of action is barred by
Petitioner first contends that the act of recording the transfer of shares in
prescription and laches since 24 years elapsed before he made any demand
the stock and transfer book and that of issuing a certificate of stock for the
upon ALSONS.
transferred shares involves only one continuous process. Thus, when a
corporate secretary is presented with a document of transfer of fully paid
shares, it is his duty to record the transfer in the stock and transfer book of
the corporation, issue a new stock certificate in the name of the transferee,

729
We find the instant petition without merit. The Court of Appeals did not transfer book that a corporation may rightfully regard the transferee as one of
err in ruling that petitioner had no cause of action, and that his petition for its stockholders. From this time, the consequent obligation on the part of the
mandamus was properly dismissed. corporation to recognize such rights as it is mandated by law to recognize
arises.
There is no question that Fausto Gaid was an original subscriber of
respondent corporations 239,500 shares. This is clear from the numerous Hence, without such recording, the transferee may not be regarded by
pleadings filed by either party. It is also clear from the Amended Articles of the corporation as one among its stockholders and the corporation may
Incorporation[20] approved on August 9, 1995[21] that each share had a par legally refuse the issuance of stock certificates in the name of the transferee
value of P1.00 per share. And, it is undisputed that petitioner had not made a even when there has been compliance with the requirements of Section
previous request upon the corporate secretary of ALSONS, respondent 64[24] of the Corporation Code. This is the import of Section 63 which states
Francisco M. Giron Jr., to record the alleged transfer of stocks. that No transfer, however, shall be valid, except between the parties, until the
transfer is recorded in the books of the corporation showing the names of the
The Corporation Code states that: parties to the transaction, the date of the transfer, the number of the
certificate or certificates and the number of shares transferred. The situation
SEC. 63. Certificate of stock and transfer of shares.The capital stock of stock would be different if the petitioner was himself the registered owner of the
corporations shall be divided into shares for which certificates signed by the stock which he sought to transfer to a third party, for then he would be
president or vice-president, countersigned by the secretary or assistant entitled to the remedy of mandamus.[25]
secretary, and sealed with the seal of the corporation shall be issued in
accordance with the by-laws. Shares of stock so issued are personal From the corporations point of view, the transfer is not effective until it is
property and may be transferred by delivery of the certificate or certificates recorded. Unless and until such recording is made the demand for the
indorsed by the owner or his attorney-in-fact or other person legally issuance of stock certificates to the alleged transferee has no legal basis. As
authorized to make the transfer. No transfer, however, shall be valid, except between the corporation on the one hand, and its shareholders and third
as between the parties, until the transfer is recorded in the books of the persons on the other, the corporation looks only to its books for the purpose
corporation so as to show the names of the parties to the transaction, the of determining who its shareholders are. [26] In other words, the stock and
date of the transfer, the number of the certificate or certificates and the transfer book is the basis for ascertaining the persons entitled to the rights
number of shares transferred. and subject to the liabilities of a stockholder. Where a transferee is not yet
recognized as a stockholder, the corporation is under no specific legal duty to
No shares of stock against which the corporation holds any unpaid claim issue stock certificates in the transferees name.
shall be transferable in the books of the corporation.
It follows that, as held by the Court of Appeals:
Pursuant to the foregoing provision, a transfer of shares of stock not
recorded in the stock and transfer book of the corporation is non-existent as x x x until registration is accomplished, the transfer, though valid between the
far as the corporation is concerned.[22]As between the corporation on the one parties, cannot be effective as against the corporation. Thus, in the absence
hand, and its shareholders and third persons on the other, the corporation of any allegation that the transfer of the shares between Gaid and the private
looks only to its books for the purpose of determining who its shareholders respondent [herein petitioner] was registered in the stock and transfer book
are.[23] It is only when the transfer has been recorded in the stock and

730
of the petitioner corporation, the private respondent has failed to state a That cannot be said of this case. The deed of undertaking with
cause of action.[27] indorsement presented by petitioner does not establish, on its face, his right
to demand for the registration of the transfer and the issuance of certificates
Petitioner insists that it is precisely the duty of the corporate secretary, of stocks. In Hager vs. Bryan, 19 Phil. 138 (1911), this Court held that a
when presented with the document of fully paid shares, to effect the transfer petition for mandamus fails to state a cause of action where it appears that
by recording the transfer in the stock and transfer book of the corporation the petitioner is not the registered stockholder and there is no allegation that
and to issue stock certificates in the name of the transferee. On this point, he holds any power of attorney from the registered stockholder, from whom
the SEC En Banc cited Rural Bank of Salinas, Inc. vs. Court of he obtained the stocks, to make the transfer, thus:
Appeals, [28] where we held that:
It appears, however, from the original as well as the amended petition, that
For the petitioner Rural Bank of Salinas to refuse registration of the this petitioner is not the registered owner of the stock which he seeks to have
transferred shares in its stock and transfer book, which duty is ministerial on transferred, and except in so far as he alleges that he is the owner of the
its part, is to render nugatory and ineffectual the spirit and intent of Section stock and that it was "indorsed" to him on February 5 by the Bryan-Landon
63 of the Corporation Code. Thus, respondent Court of Appeals did not err in Company, in whose name it is registered on the books of the Visayan Electric
upholding the decision of respondent SEC affirming the Decision of its Company, there is no allegation that the petitioner holds any power of
Hearing Officer directing the registration of the 473 shares in the stock and attorney from the Bryan-Landon Company authorizing him to make demand
transfer book in the names of private respondents. At all events, the on the secretary of the Visayan Electric Company to make the transfer which
registration is without prejudice to the proceedings in court to determine the petitioner seeks to have made through the medium of the mandamus of this
validity of the Deeds of Assignment of the shares of stock in question. court.

In Rural Bank of Salinas, Inc., however, private respondent Melania Without discussing or deciding the respective rights of the parties which
Guerrero had a Special Power of Attorney executed in her favor by Clemente might be properly asserted in an ordinary action or an action in the nature of
Guerrero, the registered stockholder.It gave Guerrero full authority to sell or an equitable suit, we are all agreed that in a case such as that at bar, a
otherwise dispose of the 473 shares of stock registered in Clementes name mandamus should not issue to compel the secretary of a corporation
and to execute the proper documents therefor. Pursuant to the authority so to make a transfer of the stock on the books of the company, unless it
given, Melania assigned the 473 shares of stock owned by Guerrero and affirmatively appears that he has failed or refused so to do, upon the
presented to the Rural Bank of Salinas the deeds of assignment covering the demand either of the person in whose name the stock is registered, or
assigned shares. Melania Guerrero prayed for the transfer of the stocks in of some person holding a power of attorney for that purpose from the
the stock and transfer book and the issuance of stock certificates in the name registered owner of the stock.There is no allegation in the petition that the
of the new owners thereof. Based on those circumstances, there was a clear petitioner or anyone else holds a power of attorney from the Bryan-Landon
duty on the part of the corporate secretary to register the 473 shares in favor Company authorizing a demand for the transfer of the stock, or that the
of the new owners, since the person who sought the transfer of shares had Bryan-Landon Company has ever itself made such demand upon the
express instructions from and specific authority given by the registered Visayan Electric Company, and in the absence of such allegation we are not
stockholder to cause the disposition of stocks registered in his name. able to say that there was such a clear indisputable duty, such a clear legal
obligation upon the respondent, as to justify the issuance of the writ to
compel him to perform it.

731
Under the provisions of our statute touching the transfer of stock (secs. 35 corporation, Pocket Bell, due to the refusal of the corporate secretary to
and 36 of Act No. 1459),[29] the mere indorsement of stock certificates does record the transfers in favor of Telectronics of the corporations controlling
not in itself give to the indorsee such a right to have a transfer of the shares 56% shares of stock which were covered by duly endorsed stock
of stock on the books of the company as will entitle him to the writ of certificates. As aforesaid, the request for the recording of a transfer is
mandamus to compel the company and its officers to make such transfer at different from the request for the issuance of stock certificates in the
his demand, because, under such circumstances the duty, the legal transferees name. Finally, in Abejo we did not say that transfer of shares
obligation, is not so clear and indisputable as to justify the issuance of the need not be recorded in the books of the corporation before the transferee
writ. As a general rule and especially under the above-cited statute, as may ask for the issuance of stock certificates. The Courts statement, that
between the corporation on the one hand, and its shareholders and third there is no requirement that a stockholder of a corporation must be a
persons on the other, the corporation looks only to its books for the purpose registered one in order that the Securities and Exchange Commission may
of determining who its shareholders are, so that a mere indorsee of a stock take cognizance of a suit seeking to enforce his rights as such stockholder
certificate, claiming to be the owner, will not necessarily be recognized as among which is the stock purchasers right to secure the corresponding
such by the corporation and its officers, in the absence of express certificate in his name,[32] was addressed to the issue of jurisdiction, which is
instructions of the registered owner to make such transfer to the indorsee, or not pertinent to the issue at hand.
a power of attorney authorizing such transfer.[30]
Absent an allegation that the transfer of shares is recorded in the stock
In Rivera vs. Florendo, 144 SCRA 643, 657 (1986), we reiterated that a and transfer book of respondent ALSONS, there appears no basis for a clear
mere indorsement by the supposed owners of the stock, in the absence of and indisputable duty or clear legal obligation that can be imposed upon the
express instructions from them, cannot be the basis of an action for respondent corporate secretary, so as to justify the issuance of the writ of
mandamus and that the rights of the parties have to be threshed out in an mandamus to compel him to perform the transfer of the shares to
ordinary action. That Hager and Rivera involved petitions for mandamus to petitioner. The test of sufficiency of the facts alleged in a petition is whether
compel the registration of the transfer, while this case is one for issuance of or not, admitting the facts alleged, the court could render a valid judgment
stock, is of no moment. It has been made clear, thus far, that before a thereon in accordance with the prayer of the petition. [33] This test would not
transferee may ask for the issuance of stock certificates, he must first cause be satisfied if, as in this case, not all the elements of a cause of action are
the registration of the transfer and thereby enjoy the status of a stockholder alleged in the complaint.[34] Where the corporate secretary is under no clear
insofar as the corporation is concerned. A corporate secretary may not be legal duty to issue stock certificates because of the petitioners failure to
compelled to register transfers of shares on the basis merely of an record earlier the transfer of shares, one of the elements of the cause of
indorsement of stock certificates. With more reason, in our view, a corporate action for mandamus is clearly missing.
secretary may not be compelled to issue stock certificates without such
registration.[31] That petitioner was under no obligation to request for the registration of
the transfer is not in issue. It has no pertinence in this controversy. One may
Petitioners reliance on our ruling in Abejo vs. De la Cruz, 149 SCRA 654 own shares of corporate stock without possessing a stock certificate. In Tan
(1987), that notice given to the corporation of the sale of the shares and vs. SEC, 206 SCRA 740 (1992), we had occasion to declare that a certificate
presentation of the certificates for transfer is equivalent to registration is of stock is not necessary to render one a stockholder in a corporation. But a
misplaced. In this case there is no allegation in the complaint that petitioner certificate of stock is the tangible evidence of the stock itself and of the
ever gave notice to respondents of the alleged transfer in his favor.Moreover, various interests therein. The certificate is the evidence of the holders
that case arose between and among the principal stockholders of the interest and status in the corporation, his ownership of the share represented

732
thereby. The certificate is in law, so to speak, an equivalent of such
ownership. It expresses the contract between the corporation and the
stockholder, but it is not essential to the existence of a share in stock or the
creation of the relation of shareholder to the corporation. [35] In fact, it rests on
the will of the stockholder whether he wants to be issued stock certificates,
and a stockholder may opt not to be issued a certificate. In Won vs. Wack
Wack Golf and Country Club, Inc., 104 Phil. 466 (1958), we held that
considering that the law does not prescribe a period within which the
registration should be effected, the action to enforce the right does not
accrue until there has been a demand and a refusal concerning the
transfer. In the present case, petitioners complaint for mandamus must fail,
not because of laches or estoppel, but because he had alleged no cause of
action sufficient for the issuance of the writ.

WHEREFORE, the petition is DENIED for lack of merit. The decision of


the Court of Appeals, in CA-G.R. SP No. 46692, which set aside that of the
Securities and Exchange Commission En Banc in SEC-AC No. 545 and
reinstated the order of the Hearing Officer, is hereby AFFIRMED.

No pronouncement as to costs.

SO ORDERED.

Bellosillo, (Chairman), Mendoza, Austria-Martinez, and Callejo, Sr.,


JJ., concur.

733
1. Confirming the validity of the resolution of the board of
directors of the Visayan Educational Supply Corporation so
far as it cancelled Stock Certificate No. 2 and split the same
into Stock Certificates No. 6 (for Angel S. Tan) and No. 8 (for
Alfonso S. Tan);

2. Invalidating the sale of shares represented under Stock


Certificate No. 8 between Alfonso S. Tan and the respondent
corporation which converted the said stocks into treasury
shares, as well as those transactions involved in the
withdrawal of the stockholders from the respondent
corporation for being contrary to law, but ordering the neither
party may recover pursuant to Article 1412 (1) Civil Code of
G.R. No. 95696 March 3, 1992 the Philippines; and

ALFONSO S. TAN, Petitioner, 3. Revoking the Order of Hearing Officer Felix Chan to
vs. reinstate complainant's original 400 shares of stock in the
SECURITIES AND EXCHANGE COMMISSION, VISAYAN EDUCATIONAL books of the corporation in view of the validity of the sale of
SUPPLY CORP., TAN SU CHING, ALFREDO B. UY, ANGEL S. TAN and 50 shares represented under stock certificate No. 6; and the
PATRICIA AGUILAR, Respondents. nullity of the sale 350 shares represented under stock
certificate No. 8, pursuant to the "in pari delicto" doctrine
aforecited. (Rollo, p. 4)

PARAS, J.: The antecedent facts of the case are as follows:

Petitioner filed a petition for certiorari against the public respondent Respondent corporation was registered on October 1, 1979. As incorporator,
Securities and Exchange Commission and its co-respondents, after the petitioner had four hundred (400) shares of the capital stock standing in his
former in an en banc Order, overturned with modification, the decision of its name at the par value of P100.00 per share, evidenced by Certificate of
Cebu SEC Extension hearing officer, Felix Chan, in SEC Case No. C-0096, Stock No. 2. He was elected as President and subsequently reelected,
dated May 23, 1989, on October 10, 1990, under SEC-AC No. 263. holding the position as such until 1982 but remained in the Board of Directors
(Rollo, pp. 3 and 4) until April 19, 1983 as director. (Rollo, p. 5)

Sought to be reversed by petitioner, is the ruling of the Commission, On January 31, 1981, while petitioner was still the president of the
specifically declaring that: respondent corporation, two other incorporators, namely, Antonia Y. Young
and Teresita Y. Ong, assigned to the corporation their shares, represented by

734
certificate of stock No. 4 and 5 after which, they were paid the corresponding of Stock No. 2 and returned only to me Certificate of Stock
40% corporate stock-in-trade. (Rollo, p. 43) No. 8, which I delivered to Tan Su Ching.

Petitioner's certificate of stock No. 2 was cancelled by the corporate 10. That the cancellation of his stock (Stock No. 2) was
secretary and respondent Patricia Aguilar by virtue of Resolution No. 1981 known by him in 1981; that it was Stock No. 8, that was
(b), which was passed and approved while petitioner was still a member of delivered in March 1983 for his endorsement and
the Board of Directors of the respondent corporation. (Rollo, p. 6) cancellation. (Ibid, p. 18)

Due to the withdrawal of the aforesaid incorporators and in order to complete From the same Affidavit, it was alleged that Atty. Ramirez prepared a
the membership of the five (5) directors of the board, petitioner sold fifty (50) Memorandum of Agreement with respect to the transaction of the fifty (50)
shares out of his 400 shares of capital stock to his brother Angel S. Tan. shares of stock part of the Stock Certificate No. 2 of petitioner, which was
Another incorporator, Alfredo B. Uy, also sold fifty (50) of his 400 shares of submitted to its former owner, Alfonso Tan, but which the purposely did not
capital stock to Teodora S. Tan and both new stockholders attended the return. (Ibid., p. 18)
special meeting, Angel Tan was elected director and on March 27, 1981, the
minutes of said meeting was filed with the SEC. These facts stand On January 29, 1983, during the annual meeting of the corporation,
unchallenged. (Rollo, p. 43) respondent Tan Su Ching was elected as President while petitioner was
elected as Vice-president. He, however, did not sign the minutes of said
Accordingly, as a result of the sale by petitioner of his fifty (50) shares of meeting which was submitted to the SEC on March 30, 1983. (Rollo, p. 43)
stock to Angel S. Tan on April 16, 1981, Certificate of Stock No. 2 was
cancelled and the corresponding Certificates Nos. 6 and 8 were issued, When petitioner was dislodged from his position as president, he withdrew
signed by the newly elected fifth member of the Board, Angel S. Tan as Vice- from the corporation on February 27, 1983, on condition that he be paid with
president, upon instruction of Alfonso S. Tan who was then the president of stocks-in-trade equivalent to 33.3% in lieu of the stock value of his shares in
the Corporation.(Memorandum of the Private Respondent, p. 15) the amount of P35,000.00. After the withdrawal of the stocks, the board of
the respondent corporation held a meeting on April 19, 1983, effecting the
With the cancellation of Certificate of stock No. 2 and the subsequent cancellation of Stock Certificate Nos. 2 and 8 (Exh. 278-C) in the corporate
issuance of Stock Certificate No. 6 in the name of Angel S. Tan and for the stock and transfer book 1 (Exh. 1-1-A) and submitted the minutes thereof to
remaining 350 shares, Stock Certificate No. 8 was issued in the name of the SEC on May 18, 1983. (Rollo, p. 44)
petitioner Alfonso S. Tan, Mr. Buzon, submitted an Affidavit (Exh. 29), alleging
that: Five (5) years and nine (9) months after the transfer of 50 shares to Angel S.
Tan, brother of petitioner Alfonso S. Tan, and three (3) years and seven (7)
9. That in view of his having taken 33 1/3 interest, I was months after effecting the transfer of Stock Certificate Nos. 2 and 8 from the
personally requested by Mr. Tan Su Ching to request Mr. original owner (Alfonso S. Tan) in the stock and transfer book of the
Alfonso Tan to make proper endorsement in the cancelled corporation, the latter filed the case before the Cebu SEC Extension Office
Certificate of Stock No. 2 and Certificate No. 8, but he did under SEC Case No. C-0096, more specifically on December 3, 1983,
not endorse, instead he kept the cancelled (1981) Certificate questioning for the first time, the cancellation of his aforesaid Stock
Certificates Nos. 2 and 8. (Rollo, p. 44)

735
The bone of centention raised by the petitioner is that the deprivation of his 1. The meaning of shares of stock are personal property and may be
shares despite the non-endorsement or surrender of his Stock Certificate transferred by delivery of the certificate or certificates indorsed by the owner
Nos. 2 and 8, was without the process contrary to the provision of Section 63 or his attorney-in-fact or other person legally authorized to make the transfer.
of the Corporation Code (Batas Pambansa Blg. 68), which requires that: (Rollo, p. 10)

. . . No transfer, however, shall be valid, except as between The case of Nava vs. peers Marketing corporation (74 SCRA 65) was cited
the parties, until the transfer is recorded to the books of the by petitioner making the reference to commentaries taken from 18 C.J.S.
corporation so as to show the names of the parties to the 928-930, that the transfer by delivery to the transferee of the certificate
transaction, the date of the transfer, the number of the should be properly indorsed, and that "There should be compliance with the
certificate or certificates and the number of shares mode of transfer prescribed by law." Using Section 35, now Section 63 of the
transferred. Corporation Code, the provision of the law, reads:

After hearing, the Cebu SEC Extension Office Hearing Officer, Felix Chan SEC. 63. Certificate of stock and transfer of shares. The
ruled, that: capital stock and stock and corporations shall be divided into
shares for which certificates signed by the president and vice
a) The cancellation of the complainant's shares of stock with president, countersigned by the secretary or assistant
the Visayan Educational Supply Corporation is null and void; secretary, and sealed with the seal of the corporation shall
be issued in accordance with the by-laws. Shares of stocks
b) The earlier cancellation of stock certificate No. 2 and the so issued are personal property and may be transferred by
subsequent issuance of stock certificate No. 8 is also hereby delivery of the certificate or certificates indorsed by the
declared null and void; owner or his attorney-in-fact or other person legally
authorized to make the transfer. No transfer, however, shall
be valid, except as between the parties, until the transfer is
c) The Secretary of the Corporation is hereby ordered to
recorded in the books of the corporation so as to show the
make the necessary corrections in the books of the
names of the parties to the transaction, the date of the
corporation reinstating thereto complainant's original 400
transfer, the number of the certificate or certificates and the
shares of stock. (Rollo, pp. 39-40)
number of shares transferred.

Private respondent in the original complaint went to the Securities and


No shares of stocks against which the corporation holds any
Exchange and Commission on appeal, and on October 10, 1990, the
unpaid claim shall be transferable in the books of the
commission en banc unanimously overturned the Decision of the Hearing
corporations.
Officer under SEC-AC No. 263. (Order, Rollo, pp. 42-49)

There is no doubt that there was delivery of Stock Certificate No. 2 made by
The petition for certiorari centered on three major issues, with other issues
the petitioner to the Corporation before its replacement with the Stock
considered as subordinate to them, to wit:
Certificate No. 6 for fifty (50) shares to Angel S. Tan and Stock Certificate No.
8 for 350 shares to the petitioner, on March 16, 1981. The problem arose

736
when petitioner was given back Stock Certificate No. 2 for him to endorse It was very obvious that petitioner devised the scheme of not returning the
and he deliberately witheld it for reasons of his own. That the Stock cancelled Stock Certificate No. 2 which was returned to him for his
Certificate in question was returned to him for his purpose was attested to by endorsement, to skim off the largesse of the corporation as shown by the
Mr. Buzon in his Affidavit, the pertinent portion of which has been earlier trading of his Stock Certificate No. 8 for goods of the corporation valued at
quoted. P2 million when the par value of the same was only worth P35,000.00. (Ibid.,
p. 470) He also used this scheme to renege on his indebtedness to
The proof that Stock Certificate No. 2 was split into two (2) consisting of respondent Tan Su Ching in the amount of P1 million. (Decision, p. 6)
Stock Certificate No. 6 for fifty (50) shares and Stock Certificate No. 8 for 350
shares, is the fact that petitioner surrendered the latter stock (No. 8) in lieu of It is not remote that if petitioner could have cashed in on Stock Certificate No.
P2 million pesos 1 worth of stocks, which the board passed in a resolution in 2 with the remainder of the goods that he padlocked, he would have done so,
its meeting on April 19, 1983. Thus, on February 27, 1983, petitioner until the respondent corporation was bled entirely.
indicated he was withdrawing from the corporation on condition that he be
paid with stock-in-trade corresponding to 33.3% (Exh. 294), which had only a Along this line, petitioner put up the argument that he was responsible for the
par value of P35,000.00. In this same meeting, the transfer of Stock growth of the corporation by the alleging that during his incumbency, the
Certificate Nos. 2 and 8 from the original owner, Alfonso S. Tan was ordered corporation grew, prospered and flourished in the court of business as
to be recorded in the corporate stock and transfer book (Exh. "I-1-A") evidenced by its audited financial statements, and grossed the following
thereafter submitting the minutes of said meeting to the SEC on May 18, incomes from: 1980 P8,658,414.10, 1981 P8,039,816.67, 1982
1983 (Exhs. 12 and I). (Order, Rollo, p. 44) P7,306,168.67, 1983 P5,874,453.55, 1984 P3,911,667.76. (Ibid., Rollo,
p. 24)
It is also doubtless that Stock Certificate No. 8 was exchanged by petitioner
for stocks-in-trade since he was operating his own enterprise engaged in the Moreover, petitioner asserted that he was ousted from the corporation by
same business, otherwise, why would a businessman be interested in reason of his efforts to establish fiscal controls and to demand an accounting
acquiring P2,000,000.00 worth of goods which could possibly at that time, fill of corporate funds which were accordingly being transferred and diverted to
up warehouse? In fact, he even padlocked the warehouse of the respondent certain of private respondents' personal accounts which were allegedly
corporation, after withdrawing the thirty-three and one-third (33 1/3%) misapplied, misappropriated and converted to their own personal use and
percent stocks. Accordingly, the Memorandum of Agreement prepared by the benefit. (Ibid., p. 125)
respondents' counsel, Atty. Ramirez evidencing the transaction, was also
presented to petitioner for his signature, however, this document was never 2. Petitioner further claims that "(T)he cancellation and transfer of petitioner's
returned by him to the corporate officer for the signature of the other officers shares and Certificate of Stock No. 2 (Exh. A) as well as the issuance and
concerned. (Rollo, p. 28) cancellation of Certificate of Stock No. 8 (Exh. M) was patently and palpably
unlawful, null and void, invalid and fraudulent." (Rollo, p. 9) And, that Section
At the time the warehouse was padlocked by the petitioner, the remaining 63 of the Corporation Code of the Philippines is "mandatory in nature",
stock inventory was valued at P7,454,189.05 of which 66 2/3 percent thereof meaning that without the actual delivery and endorsement of the certificate in
belonged to the private respondents. (Ibid., p. 28) question, there can be no transfer, or that such transfer is null and void.
(Rollo, p. 10)

737
These arguments are all motivated by self-interest, using foreign authorities For all intents and purposes, however, since this was already cancelled
that are slanted in his favor and even misquoting local authorities to prop up which cancellation was also reported to the respondent Commission, there
his erroneous posture and all these attempts are intended to stifle justice, was no necessity for the same certificate to be endorsed by the petitioner. All
truth and equity. the acts required for the transferee to exercise its rights over the acquired
stocks were attendant and even the corporation was protected from other
Contrary to the understanding of the petitioner with respect to the use of the parties, considering that said transfer was earlier recorded or registered in
word "may", in the case of Shauf v. Court of Appeals, (191 SCRA 713, 27 the corporate stock and transfer book.
November 1990), this Court held, that "Remedial law statues are to be
construed liberally." The term 'may' as used in adjective rules, is only Following the doctrine enunciated in the case of Tuazon v. La Provisora
permissive and not mandatory. In several earlier cases, the usage of the Filipina, where this Court held, that:
word "may" was described as follows:
But delivery is not essential where it appears that the
The word "may"is an auxilliary verb showing among others, persons sought to be held as stockholders are officers of the
opportunity or possibility. Under ordinary circumstances, the corporation, and have the custody of the stock book . . . (67
phrase "may be" implies the possible existence of Phi. 36).
something. In this case, the "something" is a law governing
sectoral representation. The phrase in question should, Furthermore, there is a necessity to delineate the function of the stock itself
therefore, be understood to mean as prescribed by such law from the actual delivery or endorsement of the certificate of stock itself as is
that governs the matter at the time . . . The phrase does not the question in the instant case. A certificate of stock is not necessary to
and cannot, by its very wording, restrict itself to the render one a stockholder in corporation.
uncertainly of future legislation. (Legaspi v. Estrella, 189
SCRA 58, 24 Aug. 1990, En Banc) Nevertheless, a certificate of stock is the paper representative or tangible
evidence of the stock itself and of the various interests therein. The certificate
Years before the above rulings concerning the interpretation of the word is not stock in the corporation but is merely evidence of the holder's interest
"may", this Court held in Chua v. Samahang Magsasaka, that "the word and status in the corporation, his ownership of the share represented
"may" indicates that the transfer may be effected in a manner different from thereby, but is not in law the equivalent of such ownership. It expresses the
that provided for in the law." (62 Phil. 472) contract between the corporation and the stockholder, but is not essential to
the existence of a share in stock or the nation of the relation of shareholder
Moreover, it is safe to infer from the facts deduced in the instant case that, to the corporation. (13 Am. Jur. 2d, 769)
there was already delivery of the unendorsed Stock Certificate No. 2, which
is essential to the issuance of Stock Certificate Nos. 6 and 8 to angel S. Tan Under the instant case, the fact of the matter is, the new holder, Angel S. Tan
and petitioner Alfonso S. Tan, respectively. What led to the problem was the has already exercised his rights and prerogatives as stockholder and was
return of the cancelled certificate (No. 2) to Alfonso S. Tan for his even elected as member of the board of directors in the respondent
endorsement and his deliberate non-endorsement. corporation with the full knowledge and acquiescence of petitioner. Due to
the transfer of fifty (50) shares, Angel S. Tan was clothed with rights and

738
responsibilities in the board of the respondent corporation when he was 350 shares of stock with a par value of P35,000.00 only, and thereafter
elected as officer thereof. withdrawing from the respondent corporation.

Besides, in Philippine jurisprudence, a certificate of stock is not a negotiable Not content with the fantastic return of his investment in the corporation and
instrument. "Although it is sometime regarded as quasi-negotiable, in the bent on sucking out the corporate resources by filing the instant case for
sense that it may be transferred by endorsement, coupled with delivery, it is damages and seeking the nullity of the cancellation of his Certificate of Stock
well-settled that it is non-negotiable, because the holder thereof takes it Nos. 2 and 8, petitioner even attempted to mislead the Court by erroneously
without prejudice to such rights or defenses as the registered owner/s or quoting the ruling of the Court in C. N. Hodges v. Lezama, which has some
transferror's creditor may have under the law, except insofar as such rights or parallelism with the instant case was the parties involved therein were also
defenses are subject to the limitations imposed by the principles governing close relatives as in this case.
estoppel." (De los Santos vs. McGrath, 96 Phil. 577)
The quoted portion appearing on p. 11 of the petition, was cut short in such a
To follow the argument put up by petitioner which was upheld by the Cebu way that relevant portions thereof were purposely left out in order to impress
SEC Extension Office Hearing Officer, Felix Chan, that the cancellation of upon the Court that the unendorsed and uncancelled stock certificate No. 17,
Stock Certificate Nos. 2 and 8 was null and void for lack of delivery of the was unconditionally declared null and void, flagrantly omitting the justifying
cancelled "mother" Certificate No. 2 whose endorsement was deliberately circumstances regarding its acquisition and the reason given by the Court
withheld by petitioner, is to prescribe certain restrictions on the transfer of why it was declared so. The history of certificate No. 17 is quoted below,
stock in violation of the corporation law itself as the only law governing showing the reason why the certificate in question was considered null and
transfer of stocks. While Section 47(s) grants a stock corporations the void, as follows:
authority to determine in the by-laws "the manner of issuing certificates" of
shares of stock, however, the power to regulate is not the power to prohibit, (P)etitioner Hodges did not cause to be entered in the books
or to impose unreasonable restrictions of the right of stockholders to transfer of the corporation as he had his stock certificate No. 17
their shares. (Emphasis supplied) which, therefore had not been endorsed by him to anybody
or cancelled and which he considered still subsisting. On
In Fleisher v. Botica Nolasco Co., Inc., it was held that a by-law which September 18, 1958, petitioner Hodges again sold his
prohibits a transfer of stock without the consent or approval of all the aforesaid 2,230 shares of stock covered by his stock
stockholders or of the president or board of directors is illegal as constituting certificate No. 17 on installment basis to his co-petitioner
undue limitation on the right of ownership and in restraint of trade. (47 Phil. Ricardo Gurrea, but continued keeping the stock certificate
583) in his possession without endorsing it to Gurrea or causing
the sale to be entered in the books of the corporation,
3. Attempt to mislead Petitioner should be held guilty of manipulating the believing that said shares of stock were his until fully paid
provision of Section 63 of the Corporation Law for contumaciously for. Up to the present, petitioner Hodges has in his
withholding the endorsement of Stock Certificate No. 2 which was returned to possession and under his control his aforesaid stock
him for the purpose, wasting time and resources of the Court, even after he certificate No. 17, unendorsed and uncancelled (Exhs. A &
had received the stocks-in-trade equivalent to P2,000,000.00 in lieu of his A-1), a fact known to the respondents. (14 SCRA p. 1032)

739
The pertinent misquoted portion follows: such interest at the public auction held in the proceedings for
the foreclosure of his chattel was rendered making said
Before the stockholders' meeting of the La Paz ice Plant & preliminary injunction permanent and declaring Hodges as
Cold Storage Co., Inc., hereinafter referred to as the the one entitled to vote on the shares of stock in question.
Corporation - which was scheduled to be held on August 6,
1959, petitioners C.N. Hodges and Ricardo Gurrea filed with Petitioner ought to have even included the following which was the reason for
the CFI of Iloilo, a petition docketed as Civil Case No. declaring the following which was the reason for declaring the unedorsed,
5261 of said court for a writ of prohibition with preliminary unsurrendered and uncancelled stock certificate, null and void:
injunction, to restrain respondents Jose Manuel Lezama, as
president and secretary, respectively, of said Corporation . . . It is, moreover, obvious that Hodges retained it (stock
from allowing their brother-in-law and brother, respectively, certificate no. 17) with Borja's consent. It was evidently part
respondent Benjamin L. Borja, to vote in said meeting on the of their agreement, or implied therein, that Hodges would
aforementioned 2,230 shares of stock. Upon the filing of said keep the stock certificate and thus remain in the records of
petition and of a bond in the sum of P1,000, the writ of the Corporation as owner of the shares, despite the
preliminary injunction prayed for was issued. After due trial, aforementioned sale thereof and the chattel mortgage
or on March 28, 1960, (start of petitioner's quotation) "The thereon. In other words, the parties thereto intended Hodges
court of origin rendered a decision holding that, in view of the to continue, for all intents and purposes, as owner of said
provision in stock certificate no. 17, in the name of Hodges, share, until Borja shall have fully paid its stipulated price.
to the effect that he (Ibid, pp. 1033-1034)

. . . is the owner of Two Thousand Two Other issues raised by the petitioner, subordinate to the principal issues
Hundred Thirty shares of the capital stock of above, (except the ruling by the respondent Commission with respect to the
La Paz Ice Plant & Cold Storage Co., Inc., "pari delicto" doctrine which is not acceptable to this Court) are of no
transferrable only on the books of the moment.
corporation by the holder hereof in person or
by attorney upon surrender of this certificate Considering the circumstances of the case, it appearing that petitioner is
properly endorsed. guilty of manipulation, and high-handedness, circumventing the clear
provisions of law in shielding himself from his wrongdoing contrary to the
stock certificate no. 18, issued in favor of Borja and the entry protective mantle that the law intended for innocent parties, the Court finds
thereof at his instance in the books of the corporation without the excuses of the petitioner as unworthy of belief.
stock certificate no. 17 being first properly endorsed,
surrendered and cancelled, is null and void. . . . " (end of WHEREFORE, in view of the foregoing, the Order of the Commission under
quotation by petitioner, but the ruling, continues without the SEC-AC No. 263 dated October 10, 1990 is hereby AFFIRMED but modified
period after the word void.) "and that it would be with respect to the "nullity of the sale of 350 shares represented under stock
unconscionable and for Borja to vote on said shares of certification No. 8, pursuant to the "in pari delicto" doctrine. The court holds
stock, knowing that he had ceased to have actual interest that the conversion of the 350 shares with a par value of only P35,000.00 at
therein since September 17, 1958, when Hodges bought

740
P100.00 per share into treasury stocks after petitioner exchanged them with
P2,000,000.00 worth of stocks-in-trade of the corporation, is valid and lawful.
With regard to the damages being claimed by the petitioner, the respondent
Commission is not empowered to award such, other than the imposition of
fine and imprisonment under Section 56 of the Corporation Code of the
Philippines, as amended.

SO ORDERED.

Melencio-Herrera, Padilla, Regalado and Nocon, JJ., concur.

G.R. No. L-6230 January 18, 1911

A. R. HAGER, petitioner,
vs.
ABERT J. BRYAN, respondent.

741
Wolfson & Wolfson for petitioner. Visayan Electric Company, being the one hundred shares
Haussermann, Cohn & Fisher for respondent. hereinbefore mentioned and including the 25 shares hereinbefore
particularly mentioned. That this agreement was finally signed on
JOHNSON, J.: February 25, 1910.

This was an original action commenced in the Supreme Court to secure a That the aforementioned agreement between your petitioner and the
writ of mandamus against the defendant to compel him, as secretary of the said Levering grew out of the fact that certain parties among whom
Visayan Electric Company, to transfer upon the books of said company were Bryan-London & Co., of which the respondent was and is
certain shares mentioned in the petition. The petition alleges: member, were at that time trying by various means and
manipulations to get control of the said Visayan Electric Company,
That your petitioner is a resident of the city of Manila, Philippine and the said Levering and your petitioner were desirous of
Islands, but temporarily residing in the city of Shanghai, China, and preventing their thus securing control.
that the respondent is a resident of and domiciled in the city of Cebu,
Philippine Islands. That in the aforesaid agreement between your petitioner and the said
Levering it was understood and agreed that delivery of all the stock
That on January first, 1910, the respondent qualified as the secretary should be made free from all incumbrances and the necessary
of the Visayan Electric Company, after having been duly elected, and transfers made on the books of the company.
that defendant was on January 1, 1910, ever since has been and
now is the secretary of the Visayan Electric Company, a corporation That the respondent does not question the title of your petitioner to
duly organized and existing under and by virtue of the laws of the the aforementioned shares of the said Visayan Electric Company,
Philippines Islands. and that he has no reason to question such title.

That prior to February 5, 1910, your petitioner was the sole owner of That the shares of the said Visayan Electric Company are
one hundred shares of capital stock of the said Visayan Electric transferable only on the books of the company and therefore your
Company and among which shares were: petitioner has repeatedly demanded and requested the respondent,
as secretary of the said Visayan Electric Company to transfer on the
Certificate No. 55 representing 5 shares; books of the company the aforesaid shares, viz:
Certificate No. 62 representing 10 shares;
Certificate No. 63 representing 10 shares. Certificate No. 55 representing 5 shares;
Certificate No. 62 representing 10 shares;
These certificates were issued in the name of Bryan-London & Co. and by Certificate No. 63 representing 10 shares;
them indorsed to your petitioner.
to the name of A. R. Hager, who was the true owner of same of A. R. Hager,
That on February 5, 1910, your petitioner entered into an agreement your petitioner, may fully and faithfully carry out his agreement with the said
with one Martin M. Levering for the purchase and sale from your Levering hereinbefore mentioned.
petitioner to the said Levering of your petitioner's interest in the said

742
That the said Albert J. Bryan, the respondent, individually, as a Company, for the costs of this action, together with such further and
member of the firm of Bryan-London & Company, and as secretary of other relief as this honorable court may deem meet and proper and
the said Visayan Electric Company has refused and neglects to that these proceedings be expedited under section 518, Code of Civil
transfer or permit the transfer of the said aforementioned shares on Procedure.
the books of the company in the name of your petitioner. That this
refusal on the part of the respondent is arbitrary, malicious, and To this petition the defendants presented the following demurrer:
contrary to law.
Now comes the defendant in the case named above and demurs to
That the respondent as secretary of the said Visayan Electric the complaint on the ground that acts constituting cause of action are
Company is the only person whose duty it is to transfer shares on not therein alleged.
the books of the said company.
The question presented is, Will the courts of the Philippine Islands, under the
That your petitioner has no plain, speedy, adequate, and complete provisions of the Code of Procedure in Civil Actions (secs. 222-225, 515)
remedy at law, as a suit in damages for the value of the shares issue the writ of mandamus, under the facts alleged in the complaint, for the
would not be speedy, adequate, nor complete, because then other purpose of compelling the secretary of a private corporation to transfer stock
parties than your petitioner and Martin M. Levering might become the upon the books of the corporation?
owners of the said shares, this being the apparent desire of the
respondent and being the express condition that your petitioner The Honorable Arthur L. Sanborn, judge of the United States district court for
desires to prevent, and being the foundation of the agreement the western district of Wisconsin, in his article entitled "Mandamus" (26 Cy. of
entered into by your petitioner and the said Martin M. Levering. Law and Procedure (Cyc), at p. 347), said:

Wherefore your petitioner respectfully prays that this honorable court By the weight of authority mandamus will not lie in ordinary cases to
issue a writ of mandamus addressed to Albert J. Bryan, as secretary compel a corporation of its officers to transfer stock on its books and
of the Visayan Electric Company ordering the said Albert J. Bryan to issue new certificates to the transferee, since the writ (in such a
forthwith transfer on the books of the company to the name to your case) is a purely private one, and there is generally an adequate
petitioner the following shares in the said Visayan Electric remedy by an action against the corporation for damages.
Company viz:
In support of this conclusion Judge Sanborn cites cases from the States as
Certificate No. 55 representing 5 shares; follows:
Certificate No. 62 representing 10 shares;
Certificate No. 63 representing 10 shares.
California. (Kimball vs. The Union Water Company, 44 Cal., 173; 13
American Reports, 157);
An order addressed to Albert J. Bryan, individually, as a member of
the firm of Bryan-London & Company, restraining the said Albert J.
Connecticut. (Tobey vs. Hakes, 54 Conn., 274; 1 American State Reports,
Bryan from in any way interfering with the transfer of the
114);
aforementioned shares on the books of the said Visayan Electric

743
Georgio. (Terrel vs. Georgia Ry. Co., 115 Ga., 104); To permit the writ of mandamus to issue for the purpose of compelling the
officers of a corporation, in cases like the present one, to transfer stock upon
Massachusetts. (Stockpole vs. Seymour, 127 Mass., 104); the books of the corporation, might, under certain circumstances, require
such officers to transfer stock against which the corporation holds unpaid
Michigan. (Clark vs. Hill, 132 Mich., 434); claims. These claims might easily arise between the time of the issuance of
the writ and the service of the same upon such officers. If the court should
issue the writ, it might require an officer to transfer stock under conditions
Minnesota. (Berker vs. Marshall, 15 Minn., [136] 177);
where the law expressly prohibited such transfer. The writ of mandamus will
never issue to compel a person to violate an express provision of the law.
Missouri. (State vs. Rombauer, 46 Mo., 155); The act required to be performed must be one which the law specially enjoin
as a duty resulting from an office, trust, or station or unlawfully excludes the
Montana. (Durfee vs. Harper, 22 Mon., 354); plaintiff from the used and enjoyment of a right or office to which he is entitled
and from which he is unlawfully precluded. (Sec. 222 of Act No. 190.)
New Jersy. (Galbraith vs. People's Building Associations etc., 43 N. J.
Law, 389); No law has been called to our attention which specially requires the
performance of the act of transferring the stock, while there is a law
New York. (People vs. Miller, 114 N. Y., 636); expressly prohibiting its transfer, except under certain conditions. (Sec. 35,
Act No. 1459.)
Ohio. (Freon vs. Carriage Company, 42 Ohio St., 30; 51 Am. Reps., 794);
In the case of Tobey vs. Hakes (54 Conn., 274) Justice Carpenter, in
Oregon. (Slemmons vs. Thompson, 23 Oregon, 215); discussing the question whether mandamus would issue or not, in an
application for such writ to compel the secretary of a private corporation to
Pennsylvania. (Burmingham Fire Ins. Co. vs. Commonwealth, 92 Pa. St., transfer stock on the books of the corporation, to a purchaser, and to issue a
72); certificate thereof, said:

Rhode Island. (Wilkinson vs. Providence Bank, 3 R. I., 22.) "This suit is against a private corporation, and its object is to enforce a mere
private right. It is in no sense a proceeding to enforce the performance of a
public duty. We have no precedent in this State for allowing this writ to
The above doctrine is the general rule in England, as well as in Canada.
compel the transfer of stock in a private corporation, and the authorities
(Rex vs. London Assurance Company, 5 B. and Ald., 899).
elsewhere are against it," citing in support of his conclusions cases:
Cushman vs. Thayer Manufacturing Co., (76 N. Y., 365), Town vs. Nichols,
Section 35, Act No. 1459, provides among other things, that: (73 Me., 515), States vs. Peoples' Building Association (43 N. J. Law, 389),
Bank vs. Harrison (66 Ga., 696).
No share of stock against which the corporation holds any unpaid
claim, shall be transferable on the books of the corporation. Justice Carpenter continues by saying:

744
There is another ground on which the writ was properly refused. It is For the reasons above stated the demurrer is hereby sustained and the
familiar law that the writ will not ordinarily issue if the plaintiff has petition is hereby dismissed with costs, without prejudice to the right of the
other remedies. If the corporation improperly refuses to transfer the plaintiff to commence an ordinary action in the Court of First Instance to
stock it is clearly liable for the damages in an action at law. secure an order directing the defendant to transfer the stock in question upon
the books of the company, after it is alleged and proved that the corporation
In the case of Kimball vs. Union Water Company (44 Cal., 173) the supreme holds no unpaid claims against it.
court of California said:
Arellano, C.J., and Torres, J., concur.
It has been so frequently decided that a party entitled to stock in a Mapa, J., concurs in the result.
private corporation has an action for damages against the
corporation for the refusal of its officers to transfer the stock to him
upon the company's books, that it must be considered as a settled
principle of law. (King vs. Bank of England, 2 Douglass, 526; Shipley
vs. Mechanics' Bank, 10 Johnson, 484; Wilkinson vs. Providence
Bank, 3 R. I., 22; Ex parte Fireman's Ins. Co., 6th Hill (N. Y.), 243;
American Asylum, etc., vs. Phoenix Bank, 4 Conn., 172; Sargeant
vs. Franklin Ins. Co., 8 Pickering (Mass.), 90.)

In the case of Shipley vs. Mechanics' Bank (10 Johnson, N. Y., 484) an
application was made for mandamus to compel a corporation to transfer to
the petitioners certain stock owned by them, and in deciding the question, the
court said:

The applicants have an adequate remedy, by special action on the


case, to recover the value of the stock, if the bank have unduly
refused to transfer it. There is no need of the extraordinary remedy
by mandamus in so ordinary a case. It might as well be required in
every case in which an ordinary action would lie.

This court has frequently decided that mandamus, being an extraordinary


remedy, will not issue when another adequate remedy exists under the
ordinary procedure. (Hoey vs. Baldwin, 1 Phi. Rep., 551 Manotoc vs.
McMicking, 10 Phil. Rep., 119; Peterson vs. Peterson, Case No. 2360; 1 Cruz
Herrera vs. McMicking, 14 Phil. Rep., 641; Tabigue et al. vs. Duval, 16 Phil.
Rep., 324.)

745
WHEREFORE, the Court renders judgment enjoining the
defendants to effect the transfer of the shares covered by
Stock Certificate No. 16807 to and in the name of plaintiff
INCORPORATED Mining Corporation, and the writ of
preliminary mandatory injunction issued on March 16, 1970
is hereby declared permanent.

SO ORDERED.

Upon the other hand, the decretal portion of the CA decision states:
G.R. No. L-45048 January 7, 1987
WHEREFORE, the judgment appealed from is hereby
modified by adding the following to the dispositive portion
BATONG BUHAY GOLD MINES, INC., petitioner,
thereof:
vs.
THE COURT OF APPEALS and INC. MINING
CORPORATION, respondents. Ordering defendant Batong Buhay Gold Mines, Inc. to pay to
the plaintiff the sum of P5,625.55, with interest at the legal
rate from March 5, 1970 until full payment; and dismissing
Taada, Sanchez, Taada & Taada Law Office for petitioner.
the complaint with respect to defendant Del Rosario and
Company. Defendant Batong Buhay shall pay the costs.
Quisumbing, Caparas, Ilagan Alcantara & Mosqueda Law Office for private
respondent.
IT IS SO ORDERED.

(pp. 67-68, Rollo)

PARAS, J.:
The antecedent facts, as found by the Court of Appeals, are as follows:

This is a petition to review the decision dated August 27, 1976 of the Court of
The defendant Batong Buhay Gold Mines, Inc. issued Stock
Appeals (CA) in CA-G.R. No. 51313-R which modified the decision of the
Certificate No. 16807 covering 62,495 shares with a par
then Court of First Instance (CFI) of Manila, Branch 11 in Civil Case No.
value of P0.01 per share to Francisco Aguac who was then
79183 Also sought for review are the resolutions of the aforenamed court
legally married to Paula G. Aguac, but the said spouses had
dated October 21, 1976 and November 12, 1976 which denied petitioner's
lived separately for more than fourteen (14) years prior to the
motion for reconsideration of the subject decision and petition and/or motion
said date. On December 16, 1969, Francisco Aguac sold his
for new trial, respectively.
62,495 shares covered by Stock Certificate No. 16807 for
the sum of P9,374.70 in favor of the plaintiff, the said
The dispositive portion of the CFI judgment reads: transaction being evidenced by a deed of sale (Exhibit D).

746
The said sale was made by Francisco Aguac without the On March 2, 1970 Francisco Aguac was charged in a
knowledge or consent of his wife Paula G. Aguac. criminal complaint Pasil Kalinga-Apayao, docketed as
Criminal Case No. 10, entitled "People vs. Francisco Aguac,
On the same date of the sale, December 16, 1969, Paula G. et al."
Aguac wrote a letter to the president of defendant Batong
Buhay Gold Mines, Inc. asking that the transfer of the shares The defendants justify their refusal to transfer the shares of
sold by her husband be withheld, inasmuch as the same stock of Francisco Aguac in the name of the plaintiff in view
constituted conjugal property and her share of proceeds of of their apprehension that they might he held liable for
the sale was not given to her (Exhibit 1). damages under Article 173 of the Civil Code and the ruling of
the Supreme Court in Bucoy vs. Paulino, 23 SCRA 248.
On January 5, 1970, under a covering letter dated December
26, 1969, plaintiff's counsel presented Stock Certificate No. On March 5, 1970, in view of the defendant's inaction on the
16807 duly endorsed by Francisco Aguac for registration and request for the transfer of the stock certificate in its name,
transfer of the said stock certificate in the name of the the plaintiff commenced this action before the Court of First
plaintiff (Exhibit F). The said letter was addressed to Instance of Manila, praying that the defendants be ordered
defendant Del Rosario and Company which was the transfer to issue and release the transfer stock certificate covering
agent of Batong Buhay at that time. In a letter dated 62,495 shares of defendant Batong Buhay, formerly
February 24, 1970 also addressed to Del Rosario and registered in the name of Francisco Aguac, in favor of the
Company, plaintiff's counsel requested information as to the plaintiff, and for the recovery of compensatory, exemplary
action taken on the transfer of Stock Certificate No. 16807 in and corrective damages and attorney's fees. A writ of
favor of the plaintiff, nothing about which having heard preliminary mandatory injunction was prayed for to order the
despite the lapse of over a month (Exhibit H). In a reply letter defendants to issue immediately the transfer certificate
dated February 28, 1970, Del Rosario and Company covering the aforesaid shares of stock of defendant Batong
informed plaintiff's counsel that Batong Buhay has referred Buhay in the name of the plaintiff.
the matter to their attorneys, inasmuch as there was a
"technical problem that has developed in the transfer of The trial court granted the prayer for the issuance of the writ
stock," and further advised that the plaintiff communicate of preliminary mandatory injunction in its order of March 16,
directly with Batong Buhay for further details (Exhibit 1970. In compliance therewith, Stock Certificate No. 16807
1).lwphl@it was cancelled and new Stock Certificate No. 27650 dated
January 5, 1970 was issued to and received by the plaintiff
It developed that when Batong Buhay was about to effect the on July 20, 1970."
cancellation of Stock Certificate No. 16807 and transfer the
62,495 shares covered thereby to the plaintiff and had, in On October 28, 1971, the trial court handed down its judgment ordering the
fact, prepared new Stock Certificate No. 27650 dated defendant (herein petitioner) to effect the transfer of the shares covered by
January 5, 1970, it received the letter of Paula G. Aguac Stock Certificate No. 16807 in the name of herein respondent Incoporated
advising it to withhold the transfer of the subject shares of
stock on the ground that the same are conjugal property.

747
Mining Corporation and declaring permanent the writ of preliminary The petitioner alleges that the appellate court gravely and categorically erred
mandatory injunction issued on March 16, 1970. in awarding damages by way of unrealized profit (or lucro cesante) to private
respondent. Petitioner company also alleges that the claim for unrealized
Private respondent seasonably appealed the aforesaid decision to the Court profit must be duly and sufficiently established, that is, that the claimant must
of Appeals anchored on the lower court's alleged failure to award damages submit proof that it was in fact damaged because of petitioner's act or
for the wrongful refusal of petitioner to transfer the subject shares of stock omission.
and alleged failure to award attorney's fees, cost of injunction bond and
expenses of litigation. The stipulation of facts of the parties does not at all show that private
respondent intended to sell, or would sell or would have sold the stocks in
On August 27, 1986, respondent appellate court rendered the subject question on specified dates. While it is true that shares of stock may go up or
decision the dispositive portion of which has already been quoted down in value (as in fact the concerned shares here really rose from fifteen
hereinabove. (15) centavos to twenty three or twenty four (23/24) centavos per share and
then fell to about two (2) centavos per share, still whatever profits could have
Hence, this petition. been made are purely SPECULATIVE, for it was difficult to predict with any
decree of certainty the rise and fall in the value of the shares. Thus this Court
has ruled that speculative damages cannot be recovered.
In assailing the decision of the Court of Appeals, petitioner poses the
following issues:
It is easy to say now that had private respondent gained legal title to the
shares, it could have sold the same and reaped a profit of P5,624.95 but it
1. May the Court of Appeals award damages by way of unrealized profits
could not do so because of petitioner's refusal to transfer the stocks in the
despite the absence of supporting evidence, or merely on the basis of pure
former's name at the time demand was made, but then it is also true that
assumption, speculation or conjecture; or can the respondent recover
human nature, being what it is, private respondent's officials could also have
damages by way of unrealized profits when it has not shown that it was
refused to sell and instead wait for expected further increases in value.
damaged in any manner by the act of petitioner?

In view of what has been said, We find no necessity to discuss the second
2. May the appellate court deny the petitioner the chance to present evidence
issue.
discovered after judgment which were not only very material to its case, but
would also show the untenability and illegality of private respondent's
position? WHEREFORE, the assailed decision and resolutions of the Court of Appeals
are hereby SET ASIDE, and a new one is hereby rendered REINSTATING
the decision of the trial court. No costs.
We answer the first issue in the negative.

SO ORDERED.

Feria (Chairman), Fernan, Alampay and Gutierrez, Jr., JJ., concur.

748
Leonardo Abola for appellant.

Alfonso V. Agcaoli & Ramon A. Barcelona for appellee Lee E. Won.

Bienvenido A. Tan in his own behalf.

CASTRO, C.J.:

This is an appeal from the order of the Court of First Instance of Rizal, in civil
case 7656, dismissing the plaintiff-appellant's complaint of interpleader upon
the grounds of failure to state a cause of action and res judicata.

In its amended and supplemental complaint of October 23, 1963, the Wack
Wack Golf & Country Club, Inc., a non-stock, civic and athletic corporation
duly organized under the laws of the Philippines, with principal office in
Mandaluyong, Rizal (hereinafter referred to as the Corporation), alleged, for
its first cause of action, that the defendant Lee E. Won claims ownership of
its membership fee certificate 201, by virtue of the decision rendered in civil
case 26044 of the CFI of Manila, entitled "Lee E. Won alias Ramon Lee vs.
Wack Wack Golf & Country Club, Inc." and also by virtue of membership fee
certificate 201-serial no. 1478 issued on October 17, 1963 by Ponciano B.
Jacinto, deputy clerk of court of the said CFI of Manila, for and in behalf of
the president and the secretary of the Corporation and of the People's Bank
& Trust Company as transfer agent of the said Corporation, pursuant to the
order of September 23, 1963 in the said case; that the defendant Bienvenido
A. Tan, on the other hand, claims to be lawful owner of its aforesaid
membership fee certificate 201 by virtue of membership fee certificate 201-
G.R. No. L-23851 March 26, 1976 serial no. 1199 issued to him on July 24, 1950 pursuant to an assignment
made in his favor by "Swan, Culbertson and Fritz," the original owner and
holder of membership fee certificate 201; that under its articles of
WACK WACK GOLF & COUNTRY CLUB, INC., plaintiff-appellant,
incorporation and by-laws the Corporation is authorized to issue a maximum
vs.
of 400 membership fee certificates to persons duly elected or admitted to
LEE E. WON alias RAMON LEE and BIENVENIDO A. TAN, defendants-
proprietary membership, all of which have been issued as early as December
appellees.
1939; that it claims no interest whatsoever in the said membership fee

749
certificate 201; that it has no means of determining who of the two In this appeal, the Corporation contends that the court a quo erred (1) in
defendants is the lawful owner thereof; that it is without power to issue two finding that the allegations in its amended and supplemental complaint do not
separate certificates for the same membership fee certificate 201, or to issue constitute a valid ground for an action of interpleader, and in holding that "the
another membership fee certificate to the defendant Lee, without violating its principal motive for the present action is to reopen the Manila Case and
articles of incorporation and by-laws; and that the membership fee certificate collaterally attack the decision of the said Court"; (2) in finding that the
201-serial no. 1199 held by the defendant Tan and the membership fee decision in civil case 26044 of the CFI of Manila constitutes res judicata and
certificate 201-serial No. 1478 issued to the defendant Lee proceed from the bars its present action; and (3) in dismissing its action instead of compelling
same membership fee certificate 201, originally issued in the name of "Swan, the appellees to interplead and litigate between themselves their respective
Culbertson and Fritz". claims.

For its second cause of action. it alleged that the membership fee certificate The Corporations position may be stated elsewise as follows: The trial court
201-serial no. 1478 issued by the deputy clerk of court of court of the CFI of erred in dismissing the complaint, instead of compelling the appellees to
Manila in behalf of the Corporation is null and void because issued in interplead because there actually are conflicting claims between the latter
violation of its by-laws, which require the surrender and cancellation of the with respect to the ownership of membership fee certificate 201, and, as
outstanding membership fee certificate 201 before issuance may be made to there is not Identity of parties, of subject-matter, and of cause of action,
the transferee of a new certificate duly signed by its president and secretary, between civil case 26044 of the CFI of Manila and the present action, the
aside from the fact that the decision of the CFI of Manila in civil case 26044 complaint should not have been dismissed upon the ground of res judicata.
is not binding upon the defendant Tan, holder of membership fee certificate
201-serial no. 1199; that Tan is made a party because of his refusal to join it On the other hand, the appellees argue that the trial court properly dismissed
in this action or bring a separate action to protect his rights despite the fact the complaint, because, having the effect of reopening civil case 26044, the
that he has a legal and beneficial interest in the subject matter of this present action is barred by res judicata.
litigation; and that he is made a part so that complete relief may be accorded
herein. Although res judicata or bar by a prior judgment was the principal ground
availed of by the appellees in moving for the dismissal of the complaint and
The Corporation prayed that (a) an order be issued requiring Lee and Tan to upon which the trial court actually dismissed the complaint, the determinative
interplead and litigate their conflicting claims; and (b) judgment. be rendered, issue, as can be gleaned from the pleadings of the parties, relates to the
after hearing, declaring who of the two is the lawful owner of membership fee propriety and timeliness of the remedy of interpleader.
certificate 201, and ordering the surrender and cancellation of membership
fee certificate 201-serial no. 1478 issued in the name of Lee. The action of interpleader, under section 120 of the Code of Civil
Procedure, 2 is a remedy whereby a person who has personal property in his
In separate motions the defendants moved to dismiss the complaint upon the possession, or an obligation to render wholly or partially, without claiming any
grounds of res judicata, failure of the complaint to state a cause of action, right to either, comes to court and asks that the persons who claim the said
and bar by prescription. 1 These motions were duly opposed by the personal property or who consider themselves entitled to demand
Corporation. Finding the grounds of bar by prior judgment and failure to state compliance with the obligation, be required to litigate among themselves in
a cause of action well taken, the trial court dismissed the complaint, with order to determine finally who is entitled to tone or the one thing. The remedy
costs against the Corporation. is afforded to protect a person not against double liability but against double

750
vexation in respect of one liability. 3 The procedure under the Rules of claims prior to the rendition of the judgment and neglected the opportunity to
Court 4 is the same as that under the Code of Civil Procedure, 5except that implead the adverse claimants in the suit where judgment was entered. This
under the former the remedy of interpleader is available regardless of the must be so, because once judgment is obtained against him by one claimant
nature of the subject-matter of the controversy, whereas under the latter an he becomes liable to the latter. 14 In once case, 15 it was declared:
interpleader suit is proper only if the subject-matter of the controversy is
personal property or relates to the performance of an obligation. The record here discloses that long before the rendition of
the judgment in favor of relators against the Hanover Fire
There is no question that the subject matter of the present controversy, i.e., Insurance Company the latter had notice of the adverse
the membership fee certificate 201, is proper for an interpleader suit. What is claim of South to the proceeds of the policy. No reason is
here disputed is the propriety and timeliness of the remedy in the light of the shown why the Insurance Company did not implead South in
facts and circumstances obtaining. the former suit and have the conflicting claims there
determined. The Insurance Company elected not to do so
A stakeholder 6 should use reasonable diligence to hale the contending and that suit proceeded to a final judgment in favor of
claimants to court. 7 He need not await actual institution of independent suits relators. The Company thereby became independently liable
against him before filing a bill of interpleader. 8 He should file an action of to relators. It was then too late for such company to invoke
interpleader within a reasonable time after a dispute has arisen without the remedy of interpleader
waiting to be sued by either of the contending claimants. 9 Otherwise, he may
be barred by laches 10 or undue delay. 11 But where he acts with reasonable The Corporation has not shown any justifiable reason why it did not file an
diligence in view of the environmental circumstances, the remedy is not application for interpleader in civil case 26044 to compel the appellees herein
barred. 12 to litigate between themselves their conflicting claims of ownership. It was
only after adverse final judgment was rendered against it that the remedy of
Has the Corporation in this case acted with diligence, in view of all the interpleader was invoked by it. By then it was too late, because to he entitled
circumstances, such that it may properly invoke the remedy of interpleader? to this remedy the applicant must be able to show that lie has not been made
We do not think so. It was aware of the conflicting claims of the appellees independently liable to any of the claimants. And since the Corporation is
with respect to the membership fee certificate 201 long before it filed the already liable to Lee under a final judgment, the present interpleader suit is
present interpleader suit. It had been recognizing Tan as the lawful owner clearly improper and unavailing.
thereof. It was sued by Lee who also claimed the same membership fee
certificate. Yet it did not interplead Tan. It preferred to proceed with the It is the general rule that before a person will be deemed to
litigation (civil case 26044) and to defend itself therein. As a matter of fact, be in a position to ask for an order of intrepleader, he must
final judgment was rendered against it and said judgment has already been be prepared to show, among other prerequisites, that he has
executed. It is not therefore too late for it to invoke the remedy of not become independently liable to any of the claimants. 25
interpleader. Tex. Jur. p. 52, Sec. 3; 30 Am. Jur. p. 218, Section 8.

It has been held that a stakeholder's action of interpleader is too late when It is also the general rule that a bill of interpleader comes too
filed after judgment has been rendered against him in favor of one of the late when it is filed after judgment has been rendered in
contending claimants, 13 especially where he had notice of the conflicting favor of one of the claimants of the fund, this being

751
especially true when the holder of the funds had notice of the of such judgment and an opportunity to implead the adverse
conflicting claims prior to the rendition of the judgment and claimants in the suit in which such judgment was rendered.
had an opportunity to implead the adverse claimants in the (See notes and cases cited 36 Am. Dec. 703, Am. St. Rep.
suit in which the judgment was rendered. United Procedures 598, also 5 Pomeroy's Eq. Juris. Sec. 41.)
Pipe Line Co. v. Britton, Tex. Civ. App. 264 S.W. 176; Nash v.
McCullum, Tex. Civ. 74 S.W. 2d 1046; 30 Am. Jur. p. 223, The evidence in the opinion of the majority shows beyond
Sec. 11; 25 Tex. Jur. p. 56, Sec. 5; 108 A.L.R., note 5, p. dispute that the appellant permitted the Parker county suit to
275. 16 proceed to judgment in favor of Britton with full notice of the
adverse claims of the defendants in the present suit other
Indeed, if a stakeholder defends a suit filed by one of the adverse claimants than the assignees of the judgment (the bank and Mrs.
and allows said suit to proceed to final judgment against him, he cannot later Pabb) and no excuse is shown why he did not implead them
on have that part of the litigation repeated in an interpleader suit. In the case in the suit. 18
at hand, the Corporation allowed civil case 26044 to proceed to final
judgment. And it offered no satisfactory explanation for its failure to implead To now permit the Corporation to bring Lee to court after the latter's
Tan in the same litigation. In this factual situation, it is clear that this successful establishment of his rights in civil case 26044 to the membership
interpleader suit cannot prosper because it was filed much too late. fee certificate 201, is to increase instead of to diminish the number of suits,
which is one of the purposes of an action of interpleader, with the possibility
If a stakeholder defends a suit by one claimant and allows it that the latter would lose the benefits of the favorable judgment. This cannot
to proceed so far as a judgment against him without filing a be done because having elected to take its chances of success in said civil
bill of interpleader, it then becomes too late for him to do case 26044, with full knowledge of all the fact, the Corporation must submit
so. Union Bank v. Kerr, 2 Md. Ch. 460; Home Life Ins. Co. v. to the consequences of defeat.
Gaulk, 86 Md. 385, 390, 38 A. 901; Gonia v. O'Brien, 223
Mass. 177, 111 N.E. 787. It is one o the main offices of a bill The act providing for the proceeding has nothing to say
of interpleader to restrain a separate proceeding at law by touching the right of one, after contesting a claim of one of
claimant so as to avoid the resulting partial judgment; and if the claimants to final judgment unsuccessfully, to involve the
the stakeholder acquiesces in one claimant's trying out his successful litigant in litigation anew by bringing an
claim and establishing it at law, he cannot then have that interpleader action. The question seems to be one of first
part of the litigation repeated in an interpleader suit. 4 impression here, but, in other jurisdictions, from which the
Pomeroy's Eq. Juris. No. 162; Mitfor's Eq. Pleading (Tyler's substance of the act was apparently taken, the rule prevails
Ed.) 147 and 236; Langdell's Summary of Eq. Pleading, No. that the action cannot be resorted to after an unsuccessful
162' De Zouche v. Garrizon, 140 Pa. 430, 21 A/450. 17 trial against one of the claimants.

It is the general rule that a bill of interpleader comes too late It is well settled, both by reasons and authority, that one who
when application therefore is delayed until after judgment asks the interposition of a court of equity to compel others,
has been rendered in favor of one of the claimants of the claiming property in his hands, to interplead, must do so
fund, and that this is especially true where the holder of the before putting them to the test of trials at law. Yarborough v.
fund had notice of the conflicting claims prior to the rendition

752
Thompson, 3 Smedes & M. 291 (41 Am. Dec. 626); Gornish The jurisprudence of this state and the common law states is
v. Tanner, 1 You. & Jer. 333; Haseltine v. Brickery, 16 Grat. well-settled that a claimant who has been put to test of a trial
(Va.) 116. The remedy by interpleader is afforded to protect by a surety, and has establish his claim, may not be
the party from the annoyance and hazard of two or more impleaded later by the surety in an interpleader suit, and
actions touching the same property or demand; but one who, compelled to prove his claim again with other adverse
with knowledge of all the facts, neglects to avail himself of claimants. American Surety Company of New York v. Brim,
the relief, or elects to take the chances for success in the 175 La. 959, 144 So. 727; American Surety Company of
actions at law, ought to submit to the consequences of New York v. Brim (In Re Lyong Lumber Company), 176 La.
defeat. To permit an unsuccessful defendant to compel the 867, 147 So. 18; Dugas v. N.Y. Casualty Co., 181 La. 322,
successful plaintiffs to interplead, is to increase instead of to 159 So. 572; 15 Ruling Case Law, 228; 33 Corpus Juris,
diminish the number of suits; to put upon the shoulders of 477; 4 Pomeroy's Jurisprudence, 1023; Royal Neighbors of
others the burden which he asks may be taken from his own. America v. Lowary (D.C.) 46 F2d 565; Brackett v. Graves, 30
....' App. Div. 162, 51 N.Y.S. 895; De Zouche v. Garrison, 140
Pa. 430, 21 A. 450, 451; Manufacturer's Finance Co. v. W.I.
It is urged, however, that the American Surety Company of Jones Co. 141 Ga., 519, 81 S.E. 1033; Hancock Mutual Life
New York was not in position to file an interpleader until it Ins. Co. v. Lawder, 22 R.I. 416, 84 A. 383.
had tested the claim of relatrix to final judgment, and that,
failing to meet with success, it promptly filed the interpleader. There can be no doubt that relator's claim has been finally
The reason why, it urges, it was not in such position until and definitely established, because that matter was passed
then is that had it succeeded before this court in sustaining upon by three courts in definitive judgments. The only
its construction of the bond and the law governing the bond, remaining item is the value of the use of the land during the
it would not have been called upon to file an interpleader, time that relator occupied it. The case was remanded solely
since there would have been sufficient funds in its hands to and only for the purpose of determining the amount of that
have satisfied all lawful claimants. It may be observed, credit. In all other aspects the judgment is final. 20
however, that the surety company was acquainted with all of
the facts, and hence that it simply took its chances of It is generally held by the cases it is the office of interpleader
meeting with success by its own construction of the bond to protect a party, not against double liability, but against
and the law. Having failed to sustain it, it cannot now force double vexation on account of one liability. Gonia v. O'Brien,
relatrix into litigation anew with others, involving most likely a 223 Mass. 177, 111 N.E. 787. And so it is said that it is too
repetition of what has been decided, or force her to accept a late for the remedy of interpleader if the party seeking this
pro rata part of a fund, which is far from benefits of the relef has contested the claim of one of the parties and
judgment. 19 suffered judgment to be taken.

Besides, a successful litigant cannot later be impleaded by his defeated In United P.P.I. Co. v. Britton (Tex. Civ. App.) 264 S.W. 576.
adversary in an interpleader suit and compelled to prove his claim anew 578, it was said: 'It is the general rule that a bill of
against other adverse claimants, as that would in effect be a collateral attack interpleader comes too late when application therefor is
upon the judgment. delayed until after judgment has been rendered in favor of

753
one of the claimants of the fund, and this is especially true ACCORDINGLY, the order of May 28, 1964, dismissing the complaint, is
where the holder of the fund had notice of the conflicting affirmed, at appellant's cost.
claims prior to the rendition of such judgment and an
opportunity to implead the adverse claimants in the suit in Teehankee, Makasiar, Antonio, Esguerra, Muoz Palma, Aquino and
which such judgment was rendered. See notes and cases Concepcion, Jr., JJ., concur.
cited 35 Am. Dec. 703; 91 An. St. Rep. 598; also 5
Pomeroy's Equity Jurisprudence No. 41.' Barredo and Martin, JJ., took no part.

The principle thus stated has been recognized in many Fernando, J., is on leave.
cases in other jurisdictions, among which may be
cited American Surety Co. v. O'Brien, 223 Mass. 177, 111
N.E. 787; Phillips v. Taylor, 148 Md. 157, 129 A. 18; Moore v.
Hill, 59 Ga. 760, 761; Yearborough v. Thompson, 3 Smedes
& M. (11 Miss.) 291, 41 Am. Dec. 626. See, also, 33 C.J. p.
447, No. 30; Nash v. McCullum, (Tex. Civ. App.) 74 S.W. 2d
1042, 1047.

It would seem that this rule should logically follow since, after
the recovery of judgment, the interpleading of the judgment
creditor is in effect a collateral attack upon the judgment. 21

In fine, the instant interpleader suit cannot prosper because the Corporation
had already been made independently liable in civil case 26044 and,
therefore, its present application for interpleader would in effect be a G.R. No. L-7991 January 29, 1914
collateral attack upon the final judgment in the said civil case; the appellee
Lee had already established his rights to membership fee certificate 201 in LEON J. LAMBERT, plaintiff-appellant,
the aforesaid civil case and, therefore, this interpleader suit would compel vs.
him to establish his rights anew, and thereby increase instead of diminish T. J. FOX, defendant-appellee.
litigations, which is one of the purposes of an interpleader suit, with the
possiblity that the benefits of the final judgment in the said civil case might
O'Brien and DeWitt and C. W. Ney, for appellant.
eventually be taken away from him; and because the Corporation allowed
J. C. Hixon, for appellee.
itself to be sued to final judgment in the said case, its action of interpleader
was filed inexcusably late, for which reason it is barred by laches or
unreasonable delay. MORELAND, J.:

This is an action brought to recover a penalty prescribed on a contract as


punishment for the breach thereof.

754
Early in 1911 the firm known as John R. Edgar & Co., engaged in the retail This sale was made by the defendant against the protest of the plaintiff and
book and stationery business, found itself in such condition financially that its with the warning that he would be held liable under the contract hereinabove
creditors, including the plaintiff and the defendant, together with many others, set forth and in accordance with its terms. In fact, the defendant Foz offered
agreed to take over the business, incorporate it and accept stock therein in to sell his shares of stock to the plaintiff for the same sum that McCullough
payment of their respective credits. This was done, the plaintiff and the was paying them less P1,000, the penalty specified in the contract.
defendant becoming the two largest stockholders in the new corporation
called John R. Edgar & Co., Incorporated. A few days after the incorporation The learned trial court decided the case in favor of the defendant upon the
was completed plaintiff and defendant entered into the following agreement: ground that the intention of the parties as it appeared from the contract in
question was to the effect that the agreement should be good and continue
Whereas the undersigned are, respectively, owners of large amounts only until the corporation reached a sound financial basis, and that that event
of stock in John R. Edgar and Co, Inc; and, having occurred some time before the expiration of the year mentioned in the
contract, the purpose for which the contract was made and had been fulfilled
Whereas it is recognized that the success of said corporation and the defendant accordingly discharged of his obligation thereunder. The
depends, now and for at least one year next following, in the larger complaint was dismissed upon the merits.
stockholders retaining their respective interests in the business of
said corporation: It is argued here that the court erred in its construction of the contract. We
are of the opinion that the contention is sound. The intention of parties to a
Therefore, the undersigned mutually and reciprocally agree not to contract must be determined, in the first instance, from the words of the
sell, transfer, or otherwise dispose of any part of their present contract itself. It is to be presumed that persons mean what they say when
holdings of stock in said John R. Edgar & Co. Inc., till after one year they speak plain English. Interpretation and construction should by the
from the date hereof. instruments last resorted to by a court in determining what the parties agreed
to. Where the language used by the parties is plain, then construction and
Either party violating this agreement shall pay to the other the sum of interpretation are unnecessary and, if used, result in making a contract for
one thousand (P1,000) pesos as liquidated damages, unless the parties. (Lizarraga Hermanos vs. Yap Tico, 24 Phil. Rep., 504.)
previous consent in writing to such sale, transfer, or other disposition
be obtained. In the case cited the court said with reference to the construction and
interpretation of statutes: "As for us, we do not construe or interpret this law.
Notwithstanding this contract the defendant Fox on October 19, 1911, sold It does not need it. We apply it. By applying the law, we conserve both
his stock in the said corporation to E. C. McCullough of the firm of E. C. provisions for the benefit of litigants. The first and fundamental duty of courts,
McCullough & Co. of Manila, a strong competitor of the said John R. Edgar & in our judgment, is to apply the law. Construction and interpretation come
Co., Inc. only after it has been demonstrated that application is impossible or
inadequate without them. They are the very last functions which a court
should exercise. The majority of the law need no interpretation or
construction. They require only application, and if there were more
application and less construction, there would be more stability in the law,
and more people would know what the law is."

755
What we said in that case is equally applicable to contracts between the sum stipulated without the necessity of proving damages. Indeed one of
persons. In the case at bar the parties expressly stipulated that the contract the primary purposes in fixing a penalty or in liquidating damages, is to avoid
should last one year. No reason is shown for saying that it shall last only nine such necessity.
months. Whatever the object was in specifying the year, it was their
agreement that the contract should last a year and it was their judgment and It is also urged by the appelle in this case that the stipulation in the contract
conviction that their purposes would not be subversed in any less time. What suspending the power to sell the stock referred to therein is an illegal
reason can give for refusing to follow the plain words of the men who made stipulation, is in restraint of trade and, therefore, offends public policy. We do
the contract? We see none. not so regard it. The suspension of the power to sell has a beneficial
purpose, results in the protection of the corporation as well as of the
The appellee urges that the plaintiff cannot recover for the reason that he did individual parties to the contract, and is reasonable as to the length of time of
not prove damages, and cites numerous American authorities to the effect the suspension. We do not here undertake to discuss the limitations to the
that because stipulations for liquidated damages are generally in excess of power to suspend the right of alienation of stock, limiting ourselves to the
actual damages and so work a hardship upon the party in default, courts are statement that the suspension in this particular case is legal and valid.
strongly inclined to treat all such agreements as imposing a penalty and to
allow a recovery for actual damages only. He also cites authorities holding The judgment is reversed, the case remanded with instructions to enter a
that a penalty, as such, will not be enforced and that the party suing, in spite judgment in favor of the plaintiff and against the defendant for P1,000, with
of the penalty assigned, will be put to his proof to demonstrate the damages interest; without costs in this instance.
actually suffered by reason of defendants wrongful act or omission.
Arellano, C.J., Trent and Araullo, JJ., concur.
In this jurisdiction penalties provided in contracts of this character are
enforced . It is the rule that parties who are competent to contract may make
such agreements within the limitations of the law and public policy as they
desire, and that the courts will enforce them according to their terms. (Civil
Code, articles 1152, 1153, 1154, and 1155; Fornow vs. Hoffmeister, 6 Phil.
Rep., 33; Palacios vs. Municipality of Cavite, 12 Phil. Rep., 140; Gsell vs.
Koch, 16 Phil. Rep., 1.) The only case recognized by the Civil Code in which
the court is authorized to intervene for the purpose of reducing a penalty
stipulated in the contract is when the principal obligation has been partly or
irregularly fulfilled and the court can see that the person demanding the
penalty has received the benefit of such or irregular performance. In such
case the court is authorized to reduce the penalty to the extent of the benefits
received by the party enforcing the penalty. G.R. No. L-23241 March 14, 1925

In this jurisdiction, there is no difference between a penalty and liquidated HENRY FLEISCHER, plaintiff-appellee,
damages, so far as legal results are concerned. Whatever differences exists vs.
between them as a matter of language, they are treated the same legally. In BOTICA NOLASCO CO., INC., defendant-appellant.
either case the party to whom payment is to be made is entitled to recover

756
Antonio Gonzalez for appellant. special defense, alleged that the defendant, pursuant to article 12 of its by-
Emilio M. Javier for appellee. laws, had preferential right to buy from the plaintiff said shares at the par
value of P100 a share, plus P90 as dividends corresponding to the year
JOHNSON, J.: 1922, and that said offer was refused by the plaintiff. The defendant prayed
for a judgment absolving it from all liability under the complaint and directing
This action was commenced in the Court of First Instance of the Province of the plaintiff to deliver to the defendant the five shares of stock in question,
Oriental Negros on the 14th day of August, 1923, against the board of and to pay damages in the sum of P500, and the costs.
directors of the Botica Nolasco, Inc., a corporation duly organized and
existing under the laws of the Philippine Islands. The plaintiff prayed that said Upon the issue presented by the pleadings above stated, the cause was
board of directors be ordered to register in the books of the corporation five brought on for trial, at the conclusion of which, and on August 21, 1924, the
shares of its stock in the name of Henry Fleischer, the plaintiff, and to pay Honorable N. Capistrano, judge, held that, in his opinion, article 12 of the by-
him the sum of P500 for damages sustained by him resulting from the refusal laws of the corporation which gives it preferential right to buy its shares from
of said body to register the shares of stock in question. The defendant filed a retiring stockholders, is in conflict with Act No. 1459 (Corporation Law),
demurrer on the ground that the facts alleged in the complaint did not especially with section 35 thereof; and rendered a judgment ordering the
constitute sufficient cause of action, and that the action was not brought defendant corporation, through its board of directors, to register in the books
against the proper party, which was the Botica Nolasco, Inc. The demurrer of said corporation the said five shares of stock in the name of the plaintiff,
was sustained, and the plaintiff was granted five days to amend his Henry Fleischer, as the shareholder or owner thereof, instead of the original
complaint. owner, Manuel Gonzalez, with costs against the defendant.

On November 15, 1923, the plaintiff filed an amended complaint against the The defendant appealed from said judgment, and now makes several
Botica Nolasco, Inc., alleging that he became the owner of five shares of assignment of error, all of which, in substance, raise the question whether or
stock of said corporation, by purchase from their original owner, one Manuel not article 12 of the by-laws of the corporation is in conflict with the provisions
Gonzalez; that the said shares were fully paid; and that the defendant of the Corporation Law (Act No. 1459).
refused to register said shares in his name in the books of the corporation in
spite of repeated demands to that effect made by him upon said corporation, There is no controversy as to the facts of the present case. They are simple
which refusal caused him damages amounting to P500. Plaintiff prayed for a and may be stated as follows:
judgment ordering the Botica Nolasco, Inc. to register in his name in the
books of the corporation the five shares of stock recorded in said books in That Manuel Gonzalez was the original owner of the five shares of stock in
the name of Manuel Gonzalez, and to indemnify him in the sum of P500 as question, Nos. 16, 17, 18, 19 and 20 of the Botica Nolasco, Inc.; that on
damages, and to pay the costs. The defendant again filed a demurrer on the March 11, 1923, he assigned and delivered said five shares to the plaintiff,
ground that the amended complaint did not state facts sufficient to constitute Henry Fleischer, by accomplishing the form of endorsement provided on the
a cause of action, and that said amended complaint was ambiguous, back thereof, together with other credits, in consideration of a large sum of
unintelligible, uncertain, which demurrer was overruled by the court. money owed by Gonzalez to Fleischer (Exhibits A, B, B-1, B-2, B-3, B-4); that
on March 13, 1923, Dr. Eduardo Miciano, who was the secretary-treasurer of
The defendant answered the amended complaint denying generally and said corporation, offered to buy from Henry Fleischer, on behalf of the
specifically each and every one of the material allegations thereof, and, as a corporation, said shares of stock, at their par value of P100 a share, for

757
P500; that by virtue of article 12 of the by-laws of Botica Nolasco, Inc., said el derecho de adquirir para si la accion o acciones que se traten de
corporation had the preferential right to buy from Manuel Gonzalez said transferir. (Exhibit 2.)
shares (Exhibit 2); that the plaintiff refused to sell them to the defendant; that
the plaintiff requested Doctor Miciano to register said shares in his name; The above-quoted article constitutes a by-law or regulation adopted by the
that Doctor Miciano refused to do so, saying that it would be in contravention Botica Nolasco, Inc., governing the transfer of shares of stock of said
of the by-laws of the corporation. corporation. The latter part of said article creates in favor of the Botica
Nolasco, Inc., a preferential right to buy, under the same conditions, the
It also appears from the record that on the 13th day of March, 1923, two days share or shares of stock of a retiring shareholder. Has said corporation any
after the assignment of the shares to the plaintiff, Manuel Gonzales made a power, under the Corporation Law (Act. No. 1459), to adopt such by-law?
written statement to the Botica Nolasco, Inc., requesting that the five shares
of stock sold by him to Henry Fleischer be noted transferred to Fleischer's The particular provisions of the Corporation Law referring to transfer of
name. He also acknowledged in said written statement the preferential right shares of stock are as follows:
of the corporation to buy said five shares (Exhibit 3). On June 14, 1923,
Gonzalez wrote a letter to the Botica Nolasco, withdrawing and cancelling his SEC. 13. Every corporation has the power:
written statement of March 13, 1923 (Exhibit C), to which letter the Botica
Nolasco on June 15, 1923, replied, declaring that his written statement was
xxx xxx xxx
in conformity with the by-laws of the corporation; that his letter of June 14th
was of no effect, and that the shares in question had been registered in the
name of the Botica Nolasco, Inc., (Exhibit X). (7) To make by-laws, not inconsistent with any existing law, for the
fixing or changing of the number of its officers and directors within
the limits prescribed by law, and for the transferring of its stock, the
As indicated above, the important question raised in this appeal is whether or
administration of its corporate affairs, etc.
not article 12 of the by-laws of the Botica Nolasco, Inc., is in conflict with the
provisions of the Corporation Law (Act No. 1459). Appellant invoked said
article as its ground for denying the request of the plaintiff that the shares in xxx xxx xxx
question be registered in his (plaintiff's) name, and for claiming that it (Botica
Nolasco, Inc.) had the preferential right to buy said shares from Gonzalez. SEC. 35. The capital stock of stock corporations shall de divided into
Appellant now contends that article 12 of the said by-laws is in conformity shares for which certificates signed by the president or the vice-
with the provisions of Act No. 1459. Said article is as follows: president, countersigned by the secretary or clerk and sealed with
the seal of the corporation, shall be issued in accordance with the
ART. 12. Las acciones de la Corporacion pueden ser transferidas a by-laws. Shares of stock so issued are personal property and may
otra persona, pero para que estas transferencias tengan validez be transferred by delivery of the certificate indorsed by the owner or
legal, deben constar en los registros de la Corporacion con el debido his attorney in fact or other person legally authorized to make the
endoso del accionista a cuyo nombre se ha expedido la accion o transfer. No transfer, however, shall be valid, except as between the
acciones que se transfieran, o un documento de parties, until the transfer is entered and noted upon the books of the
transferencia. Entendiendose que, ningun accionista transferira corporation so as to show the names of the parties to the
accion alguna a otra persona sin participar antes por escrito al transaction, that date of the transfer, the number of the certificate,
Secretario-Tesorero. En igualdad de condiciones, la sociedad tendra and the number of shares transferred.

758
No share of stock against which the corporation holds any unpaid On the other hand, it is equally well settled that by-laws of a corporation must
claim shall be transferable on the books of the corporation. be reasonable and for a corporate purpose, and always within the charter
limits. They must always be strictly subordinate to the constitution and the
Section 13, paragraph 7, above-quoted, empowers a corporation to general laws of the land. They must not infringe the policy of the state, nor be
make by-laws, not inconsistent with any existing law, for the transferring of its hostile to public welfare. (46 Am. Rep., 332.) They must not disturb vested
stock. It follows from said provision, that a by-law adopted by a corporation rights or impair the obligation of a contract, take away or abridge the
relating to transfer of stock should be in harmony with the law on the subject substantial rights of stockholder or member, affect rights of property or create
of transfer of stock. The law on this subject is found in section 35 of Act No. obligations unknown to the law. (People's Home Savings Bank vs. Superior
1459 above quoted. Said section specifically provides that the shares of Court, 104 Cal., 649; 43 Am. St. Rep., 147; Ireland vs. Globe Milling Co., 79
stock "are personal property and may be transferred by delivery of the Am. St. Rep., 769.)
certificate indorsed by the owner, etc."Said section 35 defines the nature,
character and transferability of shares of stock. Under said section they are The validity of the by-law of a corporation is purely a question of law. (South
personal property and may be transferred as therein provided. Said section Florida Railroad Co. vs. Rhodes, 25 Fla., 40.)
contemplates no restriction as to whom they may be transferred or sold. It
does not suggest that any discrimination may be created by the corporation The power to enact by-laws restraining the sale and transfer of stock
in favor or against a certain purchaser. The holder of shares, as owner of must be found in the governing statute or the charter. Restrictions
personal property, is at liberty, under said section, to dispose of them in favor upon the traffic in stock must have their source in legislative
of whomsoever he pleases, without any other limitation in this respect, than enactment, as the corporation itself cannot create such impediments.
the general provisions of law. Therefore, a stock corporation in adopting a by- By-law are intended merely for the protection of the corporation, and
law governing transfer of shares of stock should take into consideration the prescribe regulation and not restriction; they are always subject to
specific provisions of section 35 of Act No. 1459, and said by-law should be the charter of the corporation. The corporation, in the absence of
made to harmonize with said provisions. It should not be inconsistent such a power, cannot ordinarily inquire into or pass upon the legality
therewith. of the transaction by which its stock passes from one person to
another, nor can it question the consideration upon which a sale is
The by-law now in question was adopted under the power conferred upon based. A by-law cannot take away or abridge the substantial rights of
the corporation by section 13, paragraph 7, above quoted; but in adopting stockholder. Under a statute authorizing by- laws for the transfer of
said by-law the corporation has transcended the limits fixed by law in the stock, a corporation can do no more than prescribe a general mode
same section, and has not taken into consideration the provisions of section of transfer on the corporate books and cannot justify an
35 of Act No. 1459. unreasonable restriction upon the right of sale. (4 Thompson on
Corporations, sec. 4137, p. 674.
As a general rule, the by-laws of a corporation are valid if they are
reasonable and calculated to carry into effect the objects of the corporation, The right of unrestrained transfer of shares inheres in the very nature
and are not contradictory to the general policy of the laws of the land. of a corporation, and courts will carefully scrutinize any attempt to
(Supreme Commandery of the Knights of the Golden Rule vs. Ainsworth, 71 impose restrictions or limitations upon the right of stockholders to sell
Ala., 436; 46 Am. Rep., 332.) and assign their stock. The right to impose any restraint in this
respect must be conferred upon the corporation either by the
governing statute or by the articles of the corporation. It cannot be

759
done by a by-law without statutory or charter authority. (4 Thompson nature of that imposed in the by-law now in question, is ultra vires, violative
on Corporations, sec. 4334, pp. 818, 819.) of the property rights of shareholders, and in restraint of trade.

The jus disponendi, being an incident of the ownership of property, And moreover, the by-laws now in question cannot have any effect on the
the general rule (subject to exceptions hereafter pointed out and appellee. He had no knowledge of such by-law when the shares were
discussed) is that every owner of corporate shares has the same assigned to him. He obtained them in good faith and for a valuable
uncontrollable right to alien them which attaches to the ownership of consideration. He was not a privy to the contract created by said by-law
any other species of property. A shareholder is under no obligation to between the shareholder Manuel Gonzalez and the Botica Nolasco, Inc. Said
refrain from selling his shares at the sacrifice of his personal interest, by-law cannot operate to defeat his rights as a purchaser.
in order to secure the welfare of the corporation, or to enable another
shareholder to make gains and profits. (10 Cyc., p. 577.) An unauthorized by-law forbidding a shareholder to sell his shares
without first offering them to the corporation for a period of thirty days
It follows from the foregoing that a corporation has no power to is not binding upon an assignee of the stock as a personal contract,
prevent or to restrain transfers of its shares, unless such power is although his assignor knew of the by-law and took part in its
expressly conferred in its charter or governing statute. This adoption. (10 Cyc., 579; Ireland vs. Globe Milling Co., 21 R.I., 9.)
conclusion follows from the further consideration that by-laws or
other regulations restraining such transfers, unless derived from When no restriction is placed by public law on the transfer of
authority expressly granted by the legislature, would be regarded as corporate stock, a purchaser is not affected by any contractual
impositions in restraint of trade. (10 Cyc., p. 578.) restriction of which he had no notice. (Brinkerhoff-Farris Trust and
Savings Co. vs. Home Lumber Co., 118 Mo., 447.)
The foregoing authorities go farther than the stand we are taking on this
question. They hold that the power of a corporation to enact by-laws The assignment of shares of stock in a corporation by one who has
restraining the sale and transfer of shares, should not only be in harmony assented to an unauthorized by-law has only the effect of a contract
with the law or charter of the corporation, but such power should be by, and enforceable against, the assignor; the assignee is not bound
expressly granted in said law or charter. by such by-law by virtue of the assignment alone. (Ireland vs. Globe
Milling Co., 21 R.I., 9.)
The only restraint imposed by the Corporation Law upon transfer of shares is
found in section 35 of Act No. 1459, quoted above, as follows: "No transfer, A by-law of a corporation which provides that transfers of stock shall
however, shall be valid, except as between the parties, until the transfer is not be valid unless approved by the board of directors, while it may
entered and noted upon the books of the corporation so as to show the be enforced as a reasonable regulation for the protection of the
names of the parties to the transaction, the date of the transfer, the number corporation against worthless stockholders, cannot be made
of the certificate, and the number of shares transferred." This restriction is available to defeat the rights of third persons. (Farmers' and
necessary in order that the officers of the corporation may know who are the Merchants' Bank of Lineville vs. Wasson, 48 Iowa, 336.)
stockholders, which is essential in conducting elections of officers, in calling
meeting of stockholders, and for other purposes. but any restriction of the Counsel for defendant incidentally argues in his brief, that the plaintiff does
not have any right of action against the defendant corporation, but against

760
the president and secretary thereof, inasmuch as the signing and registration
of shares is incumbent upon said officers pursuant to section 35 of the
Corporation Law. This contention cannot be sustained now. The question
should have been raised in the lower court. It is too late to raise it now in this
appeal. Besides, as stated above, the corporation was made defendant in
this action upon the demurrer of the attorney of the original defendant in the
lower court, who contended that the Botica Nolasco, Inc., should be made
the party defendant in this action. Accordingly, upon order of the court, the
complaint was amended and the said corporation was made the party
defendant.

Whenever a corporation refuses to transfer and register stock in cases like


the present, mandamus will lie to compel the officers of the corporation to
transfer said stock upon the books of the corporation. (26 Cyc. 347;
Hager vs. Bryan, 19 Phil., 138.)

In view of all the foregoing, we are of the opinion, and so hold, that the
decision of the lower court is in accordance with law and should be and is
hereby affirmed, with costs. So ordered.

Malcolm, Villamor, Ostrand, Johns, and Romualdez, JJ., concur.

761
thereon. In this respect it is hereby held that there has been no such
contract, either express or implied, between the plaintiff and the defendants.

3. ID.; NON-EXISTENT OR IMAGINARY OBLIGATION. In the absence of


a similar contractual obligation and a legal provision applicable thereto, it is
logical to conclude that it would be unjust and unreasonable to compel the
said defendants to comply with a non- existent or imaginary obligation.
Whereupon, the judgment originally rendered to that effect is untenable and
should be set aside.

DECISION

[G.R. No. 38684. December 21, 1933.]

CYRUS PADGETT, Plaintiff-Appellee, v. BABCOCK & TEMPLETON, INC.,


and W. R. BABCOCK, Defendants-Appellants.
IMPERIAL, J.:
J. F. Boomer for appellant Babcock & Templeton, Inc.

W. R. Babcock in his own behalf.

Vicente Pelaez for Appellee. By resolution approved on November 25, 1933, this court set aside its
decision in this case, which was promulgated on October 13th of the same
SYLLABUS year, and thereby granted a rehearing before the second division. The
defendant W. R. Babcock and his counsel J. F. Boomer, both of whom were
1. SHARES OF CAPITAL STOCK; ILLEGAL RESTRICTION IMPOSED present during the said rehearing again argued the merits of the case.
THEREON. The restriction consisting in the word "nontransferable" Nobody appeared for the plaintiff.
appearing on the twelve (12) certificates, Exhibits F to F-11, is illegal on the
ground that it constitutes an undue limitation of the right of ownership and is The facts of the case have not suffered any change. They remain the same
in restraint of trade. It should, therefore, be eliminated. as those which we stated in the original decision as follows: "The appellee
was an employee of the appellant corporation and rendered services as such
2. ID.; OBLIGATION TO PURCHASE. There is no existing law nor from January 1, 1923, to April 15, 1929. During that period he bought 35
authority in support of the plaintiffs claim to the effect that the defendants are shares thereof at P100 a share at the suggestion of the president of said
obliged to purchase his shares at par value plus the interest demanded corporation. He was also the recipient of 9 shares by way of bonus during

762
Christmas seasons. In this way the said appellee became the owner of 44 "Any restriction on a stockholders right to dispose of his shares must be
shares for which the 12 certificates, Exhibits F to F-11, were issued in his construed strictly; and any attempt to restrain a transfer of shares is regarded
favor. The word nontransferable appears on each and every one of these as being in restraint of trade, in the absence of a valid lien upon its shares,
certificates. Before severing his connections with the said corporation, the and except to the extent that valid restrictive regulations and agreements
appellee proposed to the president that the said corporation buy his 44 exist and are applicable. Subject only to such restrictions, a stockholder
shares at par value plus the interest thereon, or that he be authorized to sell cannot be controlled in or restrained from exercising his right to transfer by
them to other persons. The corporation bought similar shares belonging to the corporation or its officers or by other stockholders, even though the sale
other employees, at par value. Sometime later, the said president offered to is to a competitor of the company, or to an insolvent person, or even though
buy the appellees shares first at P85 each and then at P80. The appellee did a controlling interest is sold to one purchaser." (Ibid., sec. 1035, pp. 665,
not agree thereto."cralaw virtua1aw library 666.)

The defendants admit that the 44 shares in question have become the In the case of Fleischer v. Botica Nolasco Co. (47 Phil., 583), we have
property of the plaintiff. They likewise grant that under the law the said discussed the validity of a clause in the by-laws of the defendant corporation,
appellee has the right to have the restriction" nontransferable" appearing on which provided that, under the same conditions, the owner of a share of
the 12 certificates eliminated therefrom. However, they vigorously contend stock could not sell it to another person except to the defendant corporation.
that there is no existing law nor authority in support of the proposition that In deciding the legality and validity of said restriction, we
they are bound to redeem or buy said shares at par value. Their admission is held:jgc:chanrobles.com.ph
only limited to the proposition that after the restriction appearing thereon is
eliminated, the plaintiff may sell the said shares to anybody, at their market "The only restraint imposed by the Corporation Law upon transfer of shares
value or at any price he sees fit. is found in section 35 of Act No. 1459, quoted above, as follows: No transfer,
however, shall be valid, except as between the parties, until the transfer is
We have not had the opportunity of hearing the opinion of the counsel for the entered and noted upon the books of the corporation so as to show the
plaintiff. We have again studied the laws applicable thereto and have names of the parties to the transaction, the date of the transfer, the number
searched for more authorities on the subject under discussion, but we have of the certificate, and the number of shares transferred. This restriction is
not found anything that bears directly on the question whether or not the necessary in order that the officers of the corporation may know who are the
defendants may be compelled, in this case, to buy the shares in question at stockholders, which is essential in conducting elections of officers, in calling
par value. However, the opinion seems to be unanimous that a restriction meetings of stockholders, and for other purposes. But any restriction of the
imposed upon a certificate of shares, similar to the ones under consideration, nature of that imposed in the by-law now in question, is ultra vires, violative
is null and void on the ground that it constitutes an unreasonable limitation of of the property rights of shareholders, and in restraint of trade." (Id., p. 592.)
the right of ownership and is in restraint of trade.
It is obvious, therefore, that the restriction consisting in the word
"Shares of corporate stock being regarded as property, the owner of such "nontransferable," appearing on the 12 certificates, Exhibits F to F-11, is
shares may, as a general rule, dispose of them as he sees fit, unless the illegal and should be eliminated.
corporation has been dissolved, or unless the right to do so is properly
restricted, or the owners privilege of disposing of his shares has been As we have hereinbefore stated, there is no existing law nor authority in
hampered by his own action." (14 C. J., sec. 1033, pp. 663, 664.) support of the plaintiffs claim to the effect that the defendants are obliged to
buy his shares of stock at par value, plus the interest demanded thereon. In

763
this respect, we hold that there has been no such contract, either express or Nicodemus L. Dasig and Sotto and Sotto for plaintiff and appellant.
implied, between the plaintiff and the defendants. In the absence of a similar Quijano, Rosete and Tizon for defendants and appellants.
contractual obligation and of a legal provision applicable thereto, it is logical
to conclude that it would be unjust and unreasonable to compel the said BAUTISTA ANGELO, J.:
defendants to comply with a non-existent or imaginary obligation.
Whereupon, we are likewise compelled to conclude that the judgment This is an appeal from a decision of the Court of First Instance of Manila
originally rendered to that effect is untenable and should be set aside. ordering the Hongkong and Shanghai Banking Corporation to pay the plaintiff
the sum of P8,041.20 plus the costs of suit. The case was certified to this
Wherefore, the judgment appealed from is hereby reversed, and the Court of Appeals.
restriction consisting in the word "non-transferable" appearing on the 12
certificates of shares of stock, is declared null and void. The defendants
The facts of this case found by the Court of Appeals are as follows:
herein are hereby ordered to cancel the certificates in question and to issue
in lieu thereof new ones without any restriction whatsoever, with the costs of
both instances against the said defendants-appellants. So ordered. Sometime in February, 1937, Mrs. Josefa T. Santamaria bought
10,000 shares of the Batangas Minerals, Inc., through the offices of
Malcolm, Villa-Real, Abad Santos, and Hull, JJ., concur. Woo, Uy-Tioco & Naftaly, a stock brokerage firm and pay therefore
the sum of P8,041.20 as shown by receipt Exh. B. The buyer
received Stock Certificate No. 517, Exh. "F", issued in the name of
Woo, Uy-Tioco & Naftaly and indorsed in bank by this firm.

On March 9, 1937, Mrs. Santamaria placed an order for the


purchase of 10,000 shares of the Crown Mines, Inc. with R.J.
Campos & Co., a brokerage firm, and delivered Certificate No. 517 to
the latter as security therefor with the understanding that said
certificate would be returned to her upon payment of the 10,000
Crown Mines, Inc. shares. Exh. D. is the receipt of the certificate in
question signed by one Mr. Cosculluela, Manager of the R.J.
Campos & Co., Inc. According to certificate Exh. E, R. J. Campos &
G.R. No. L-2808 August 31, 1951 Co., Inc. bought for Mrs. Josefa Santamaria 10,000 shares of the
Crown Mines, Inc. at .225 a share, or the total amount of P2,250.
JOSEFA SANTAMARIA, assisted by her husband, FRANCISCO
SANTAMARIA, Jr., plaintiff-appellee, At the time of the delivery of a stock Certificate No. 517 to R.J.
vs. Campos & Co., Inc. this certificate was in the same condition as that
THE HONGKONG AND SHANGHAI BANKING CORPORATION and R. W. when Mrs. Santamaria received from Woo, Uy-Tioco & Naftaly, with
TAPLIN, defendants-appellant. the sole difference that her name was later written in lead pencil on
the upper right hand corner thereof.

764
Two days later, on March 11, Mrs. Santamaria went to R.J. Campos the Crown Mines, Inc., or else on March 13, 1937. In her interview
& Co., Inc. to pay for her order of 10,000 Crown Mines shares and to with Taplin, the bank's representative, she informed him that the
get back Certificate No. 517. Cosculluela then informed her that R.J. certificate belonged to her, and she demanded that it be returned to
Campos & Co., Inc. was no longer allowed to transact business due her. Taplin then replied that the bank did not know anything about the
to a prohibition order from Securities and Exchange Commission. transaction had between her and R.J. Campos & Co., Inc., and that
She was also inform that her Stock certificate was in the possession he could not do anything until the case of the bank with Campos
of the Hongkong and Shanghai Banking Corporation. shall have been terminated. This declaration was not contradicted by
the adverse party.
Certificate No. 517 came into possession of the Hongkong and
Shanghai Banking Corporation because R.J. Campos & Co., Inc. "In Civil Case No. 51224, R.J. Campos & Co., Inc. was declared
had opened an overdraft account with this bank and to this effect it insolvent, and on July 12, 1937, the Hongkong & Shanghai Banking
had executed on April 16, 1936 a document of hypothecation, Exhibit Corporation asked permission in the insolvency court to sell the R.J.
1, by the term of which R.J. Campos & Co., Inc. pledged to the said Campos & Co., Inc., securities listed in its motion by virtue of the
bank "all stocks, shares and securities which I/we may hereafter document of hypothecation Exhibit 1. In an order dated July 15,
come into their possession of my/our account and whether originally 1937, the insolvency court granted this motion.
deposited for safe custody only or for any other purpose whatever or
which may hereinafter be deposited by me/us in lieu of or in addition "On June 3, 1938, to 10,000 shares of Batangas Minerals, Inc.
to the Stocks Shares and Securities now deposited or for any other represented by Certificate No. 715, were sold to the same bank by
purposes whatsoever." the Sheriff for P300 at the foreclosure sale authorized by said order.
(Exhibits F, 2 and 3.)
On March 11, 1937, as shown by Exhibit G. Certificate No. 517,
already indorsed by R.J. Campos Co. Inc. to the Hongkong & R.J. Campos, the president of R.J. Campos & Co., Inc., was
Shanghai Banking Corporation, was sent by the latter to the office of prosecuted for estafa and found guilty of this crime and was
the Batangas Minerals, Inc. with the request that the same be sentenced by the Manila Court of First Instance in Criminal Case No.
cancelled and a new certificate be issued in the name of R.W. Taplin 54428, to an imprisonment and to indemnify the offended party, Mrs.
as trustee and nominee of the banking corporation. Robert W. Taplin Josefa Santamaria, in the amount of P8,041.20 representing the
was an officer of this institution in charge of the securities belonging value of the 10,000 shares of Batangas Minerals, Inc. (Exhibits I and
to or claimed by the bank. As per this request the Batangas Minerals, J.) The decision was later confirmed by the Court of Appeals.
Inc. on March 12, 1937, issued Certificate No. 715 in lieu of (Exhibits J.) The offended party and R. W. Taplin were among the
Certificate No. 517, in the name of Robert W. Taplin as trustee and witnesses for the prosecution in this criminal case No. 54428.
nominee of the Hongkong & Shanghai Banking Corporation. (Exhibits 4.).
(Exhibits G, H, I, J, 1, 4 and 5.)
When Mrs. Santamaria failed in her efforts to force the civil judgment
According to Mrs. Santamaria, she made the claim to the bank for rendered in her favor in the criminal case because the accused
her certificate, though she did not remember the exact date, but it became insolvent, she filed her complaint in this case on October 11,
was most likely on the following day of that when she went to 1940. At the trial both parties agreed that the 10,000 Batangas
Cosculluela for the purpose of paying her order for 10,000 shares of

765
Minerals shares formerly represented by Certificate No. 517 and issued said certificate to cancel it and issue another in lieu thereof in her
thereafter by Certificate No. 715, have no actual market value. name to apprise the holder that she was the owner of said certificate. This
she failed to do, and instead she delivered said certificate, as it was, to R.J.
The errors assigned by the defendants-appellants as committed by the lower Campos & Co., Inc., thereby clothing the latter with apparent title to the
court are: shares represented by said certificate including apparent authority to
negotiate it by delivering it to said company while it was indorsed in blank by
I the person or firm appearing on its face as the owner thereof. The defendant
Bank had no knowledge of the circumstances under which the certificate of
stock was delivered to R.J. Campos & Co., Inc., and had a perfect right to
The trial court erred in finding that the plaintiff-appellee was not
assume that R.J. Campos & Co., Inc. was lawfully in possession of the
chargeable with negligence in the transaction which gave rise to this
certificate in view of the fact that it was a street certificate, and was in such
case.
form as would entitle any possessor thereof to a transfer of the stock on the
books of the corporation concerned. There is no question that, in this case,
II plaintiff made the negotiation of the certificate of stock to other parties
possible and the confidence she placed in R.J. Campos & Co., Inc. made the
The trial court erred in holding that it was the obligation of the bank wrong done possible. This was the proximate cause of the damage suffered
to have inquired into the ownership of the certificate when it received by her. She is, therefore, estopped from claiming further title to or interest
it from R.J. Campos & Company and in concluding that the bank was therein as against a bona fide pledge or transferee thereof, for it is a well-
negligent for not having done so. known rule that a bona fide pledgee or transferee of a stock from the
apparent owner is not chargeable with knowledge of the limitations placed on
III it by the real owner, or of any secret agreement relating to the use which
might be made of the stock by the holder (Fletcher, Cyclopedia of
The trial court erred on ordering defendants-appellants to pay to Corporations, section 5562, Vol. 12, p. 521).
plaintiff the sum of P8,041.20.
On the other hand, it appears that this certificate of stock, indorsed as it was
1. Defendants-appellants contend in the first place that the trial court erred in in blank by Woo, Uy-Tioco & Naftaly, stock brokers, was delivered to The
finding that the plaintiff-appellee was not chargeable with negligence in the Hongkong and Shanghai Banking Corporation by R.J. Campos & Co., Inc.,
transaction which gave rise to this case. duly indorsed by the latter, pursuant to a letter of hypothecation executed by
R.J. Campos & Co., Inc., in favor of said Bank (Exhibit "1"). The said
A careful analysis of the facts seems to justify this contention. Certificate of certificate was delivered to the Bank in the ordinary course of business,
stock No. 517 was made out in the name of Wo, Uy-Tioco & Naftaly, brokers, together with many other securities, and at the time it was delivered, the
and was duly indorsed in bank by said brokers. This certificate of stock was Bank had no Knowledge that the shares represented by the certificate
delivered by plaintiff to R.J. Campos & Co., Inc. to comply with a requirement belonged to the plaintiff for, as already said, it was in the form of street
that she deposit something on account if she wanted to buy 10,000 shares of certificate which was transferable by mere delivery. The rule is "where one of
Crown Mines Inc. In making said deposit, plaintiff did not take any precaution two innocent parties must suffer by reason of a wrongful or unauthorized act,
to protect herself against the possible misuse of the shares represented by the loss must fall on the one who first trusted the wrong doer and put in his
the certificate of stock. Plaintiff could have asked the corporation that had

766
hands the means of inflicting such loss" (Fletcher Cyclopedia of the question that asserts itself is: was the defendants Bank obligated to
Corporations, supra). inquire who was the real owner of the shares represented by the certificate of
stock, and could it be charged with negligence for having failed to do so?
It is therefore clear that plaintiff, in failing to take the necessary precautions
upon delivering the certificate of stock to her broker, was chargeable with It should be noted that the certificate of stock in question was issued in the
negligence in the transaction which resulted to her own prejudice, and as name of the brokerage firm-Woo, Uy-Tioco & Naftaly and that it was duly
such, she is estopped from asserting title to it as against the defendant Bank. indorsed in blank by said firm, and that said indorsement was guaranteed by
R.J. Campos & Co., Inc., which in turn indorsed it in blank. This certificate is
2. The next contention of the defendant is that the trial court erred in holding what it is known as street certificate. Upon its face, the holder was entitled to
that it was the obligation of the defendant Bank to have inquired into the demand its transfer into his name from the issuing corporation. The Bank
ownership of the certificate when it received it from R.J. Campos & Co., Inc. was not obligated to look beyond the certificate to ascertain the ownership of
and in concluding that the Bank was negligent for not having done so, the stock at the time it received the same from R.J. Campos & Co., Inc., for it
contrary to the claim of the plaintiff that defendant Bank acted negligently, if was given to the Bank pursuant to their letter of hypothecation. Even if said
not in bad faith, in accepting delivery of said certificate from RJ. Campos & certificate had been in the name of the plaintiff but indorsed in blank, the
Co., Inc. Bank would still have been justified in believing that R.J. Campos & Co., Inc.
had title thereto for the reason that it is a well-known practice that a
Let us now see the material facts on this point. Certificate No. 517 came into certificate of stock, indorsed in blank, is deemed quasi negotiable, and as
the possession of the defendant Bank because R.J. Campos & Co., Inc. had such the transferee thereof is justified in believing that it belongs to the
opened an overdraft account with said Bank and to this effect it had executed holder and transferor (Heyman vs. Hamilton National Bank, 266 S.W. 1043;
on April 16, 1946, a letter of hypothecation by the terms of which R.J. Fletcher, Cyclopedia of Corporations, Vol. 12, pp. 521-524, 525-527;
Campos & Co., Inc. pledged to the said Bank "all Stocks, Shares and McNeil vs. Tenth National Bank, 7 Am. Rep. 341).
Securities which I/we may hereafter come into their possession on my/our
account and whether originally deposited for safe custody only or for any The only evidence in the record to show that the certificate of stock in
other purpose whatever or which may hereafter be deposited by me/us in lieu question may not have belonged to R.J. Campos & Co., Inc. is the testimony
of or in addition to the Stocks, Shares, and Securities now deposited or for of the plaintiff to the effect that she had approached Robert W. Taplin on
any other purpose whatsoever." On March 13, 1937, plaintiff went to the March 13, 1937, and informed him that she was the true owner of said
office of the Bank to claim for her certificate. In her interview with one Robert certificate and demanded the return thereof, or its value, but even assuming
W. Taplin, the officer in charge of the securities of that institution, she for the sake of argument that what plaintiff has stated is true, such an
informed him that the certificate belonged to her and she demanded that it be incident would merely show that plaintiff has an adverse claim to the
returned to her. Taplin then replied that the Bank did not know anything about ownership of said certificate of stock, but that would not necessarily place the
the transaction had between her and that he could not do anything until the Bank in the position to inquire as to the real basis of her claim, nor would it
case of the Bank with R.J. Campos & Co., Inc. had been terminated. It place the Bank in the obligation to recognize her claim and return to her the
further appears that when the certificate of stock was delivered by plaintiff to certificate outright. A mere claim and of ownership does not establish the fact
R.J. Campos & Co., Inc., the manager thereof, Sebastian Cosculluela, wrote of ownership. The right of the plaintiff in such a case would be against the
in pencil on the right margin the name of Josefa T. Santamaria, pursuant to transferor. In fact, this is the attitude plaintiff has adopted when she filed a
the practice followed by said firm to write on that part of the certificate the charge for estafa against Rafael J. Campos, which culminated in his
name of the owner for purposes of identification. Upon the facts thus stated, prosecution and conviction, and it is only when she found him to be insolvent

767
that she decided to go against the Bank. The fact that on the right margin of
the said certificate the name of the plaintiff appeared written, granting it to be
true, can not be considered sufficient reason to indicate that its owner was
the plaintiff considering that said certificate was indorsed in blank by her
brokers Woo, Uy-Tioco & Naftaly, was guaranteed by indorsement in blank
by R.J. Campos & Co., Inc., and was transferred in due course by the latter
to the Bank under their letter of hypothecation. Said indicium could at best
give the impression that the plaintiff was the original holder of the certificate.

The Court has noticed that the defendant Bank was willing from the very
beginning to compromise this case by delivering to the plaintiff certificate of
stock No. 715 that was issued to said Bank by the issuer corporation in lieu
of the original as alleged and prayed for in its amended answer to the
complaint dated April 2, 1941. Considering that in the light of the law and G.R. No. L-4818 February 28, 1955
precedents applicable in this case, the most that plaintiff could claim is the
return to her of the said certificate of stock (Howson vs. Mechanics Sav. APOLINARIO G. DE LOS SANTOS and ISABELO
Bank, 183 Atl., p. 697), the Court, regardless of the conclusions arrived at as ASTRAQUILLO, plaintiffs-appellees,
above stated, is inclined to grant the formal tender made by the defendant to vs.
the plaintiff of said certificate. J. HOWARD MCGRATH ATTORNEY GENERAL OF THE UNITED STATES,
SUCCESSOR TO THE PHILIPPINE ALIEN PROPERTY ADMINISTRATION
Wherefore, the decision of the lower court is hereby modified in the sense of OF THE UNITED STATES, defendant-appellant.
ordering the defendant to deliver to the plaintiff certificate of stock No. 715, REPUBLIC OF THE PHILIPPINES, intervenor-appellant.
without pronouncement as to costs.
Jose P. Laurel, Adolfo A. Scheerer, Antonio Quirino, and J. C. Orendain, for
Paras, C.J., Feria, Bengzon and Jugo, JJ., concur. appellees.
Padilla, J., concurs in the result. Harold I. Baynton, Stanley Gilbert, Juan T. Santos, and Lino M. Patajo, and
Perkins, Ponce Enrile & Associates, for appellant.
Office of the Solicitor General Pompeyo Diaz and Solicitor Pacifico P. de
Castro for intervenor-appellant.

CONCEPCION, J.:

This action involves the title to 1,600,000 shares of stock of the Lepanto
Consolidated Mining Co., Inc., a corporation duly organized and existing
under the laws of the Philippines, hereinafter referred to, for the sake of
brevity, as the Lepanto. Originally, one-half of said shares of stock were

768
claimed by plaintiff, Apolinario de los Santos, and the other half, by his co- to as the "Mitsuis"), a corporation organized in accordance with the laws of
plaintiff Isabelo Astraquillo. During the pendency of this case, the latter has Japan, the true owner thereof, with branch office in the Philippines; that on or
allegedly conveyed and assigned his interest in and to said half claimed by before March, 1942, Madrigal delivered the corresponding stock certificates,
him to the former. The shares of stock in question are covered by several with his blank indorsement thereon, to the Mitsuis, which kept said
stock certificates issued in favor of Vicente Madrigal, who is registered in the certificates, in the files of its office in Manila, until the liberation of the latter by
books of the Lepanto as owner of said stocks and whose indorsement in the American forces early in 1945; that the Mitsuis had never sold, or
blank appears on the back of said certificates, all of which, except certificates otherwise disposed of, said shares of stock; and that the stock certificates
No. 2279 marked Exhibit 2 covering 55,000 shares, are in plaintiffs' aforementioned must have been stolen or looted, therefore, during the
possession. So was said Exhibit 2, up to sometime in 1945 or 1946 when emergency resulting from said liberation.
said possession was lost under the conditions set forth in subsequent pages.
Inasmuch as, pursuant to the Philippine Property Act, all property vested in
Briefly stated, plaintiffs contend that De los Santos bought 55,000 shares the United States, or any of its officials, under the Trading with the Enemy
from Juan Campos, in Manila, early in December, 1942; that he bought Act, as amended, located in the Philippines at the time of such vesting, or the
300,000 shares from Carl Hess, in the same city, several days later; and that, proceeds thereof, shall be transferred to the Republic of the Philippines, the
before Christmas of 1942, be bought 800,000 shares from Carl Hess, this latter sought permission, and was allowed, to intervene in this case and filed
time for the account and benefit of Astraquillo. By virtue of vesting P-12, an answer adopting in substance the theory of the defendant.
dated February 18, 1945, title to the 1,600,000 shares of stock in dispute
was, however, vested in the Alien Property Custodian of the U. S. After due hearing, the Court of First Instance of Manila, presided over by
(hereinafter referred to as the Property Custodian) as Japanese property. Honorable Higinio B. Macadaeg, Judge, rendered a decision the dispositive
Hence, plaintiffs filed their respective claims with the Property Custodian. In part of which reads, as follows:
due course, the Vested Property Claims Committee of the Philippine Alien
Property Administration made a "determination," dated March 9, 1948, In view of the foregoing consideration, judgment is hereby rendered
allowing said claims, which were considered and heard jointly as Claim No. in favor of the plaintiffs and against the defendant, declaring the
535, but, upon personal review, the Philippine Alien Property Administration former the absolute owners of the shares of stock of the Lepanto
made by said Committee and decreed that "title to the shares in question consolidated Mining Company, covered by the certificates of stock,
shall remain in the name of the Philippine Alien Property Administrator." respectively, in their (plaintiffs') possession. The transfer of said
Consequently, plaintiffs instituted the present action to establish title to the shares of stock in favor of the Alien Property Custodian of the U. S.
aforementioned shares of stock. In their complaint, they pray that judgment of America, now Philippine Alien Property Administration, is hereby
be rendered declaring them lawful owners of said shares of stock, with such declared null and void and of no effect. Consequently, the Lepanto
dividends, profits and rights as may have accrued thereto; requiring the consolidated mining Company is ordered to cancel the certificates of
defendant to render accounts and to transfer said shares of stock to plaintiffs' stock issued in the name of the Philippine Alien Property Custodian
names; and sentencing the former to pay the costs. or Philippine Alien Property Administrator, as the case may be.
Defendant shall pay the cost of the proceeding. (p. 67, R.A.)
The defendant herein is the Attorney General of the U. S., successor to the
"Administrator". He contends, substantially, that, prior to the outbreak of the The defendant and the intervenor have appealed from this decision. The
war in the Pacific, said shares of stock were bought by Vicente Madrigal, in main question for determination in this appeal is whether or not plaintiffs had
trust for, and for the benefit of, the Mitsui Bussan Kaisha (hereinafter referred

769
purchased the shares of stock in question. In support of the negative answer, possession, as branch manager of the Mitsuis; that said shares of stock were
appellants have introduced the testimony of Vicente Madrigal, Matsune never sold or otherwise disposed of by the Mitsuis; that, late in September
Kitajima, Kingy Miwa, Miguel Simon, E. A. Perkins and Victor E. Lednicky, as 1944, he bade his assistant, one Miyazawa, to transfer all important
well as several pieces of documentary evidence. documents to their residence and headquarters, at Taft Avenue, Manila,
although he did not know personally whether or not the transfer was actually
Mr. Madrigal, whose testimony before the claims Committee of the Philippine carried out; and that in January 1945, when the Japanese were about to
Alien Property Administration was admitted with plaintiffs' consent, stated that evacuate Manila, he told his Assistant Manager, one Shinoda, to burn all
he purchased the shares of stock in question, among others, for the Mitsuis important papers before leaving the city.
and at their request; that he paid with his own funds the corresponding price,
which was later reimbursed to him by the Mitsuis; that he held the Miguel Simon, brother of Carl Hess, from whom plaintiffs claim to have
corresponding stock certificates, which were issued in his name, with the purchased 1,100,000 shares of stock, affirmed that Hess lived in front of his
understanding that he would effect the necessary transfer, to the Mitsuis, (Simon's) house; that they were close to each other and had long been
upon demand; and that, shortly before the outbreak of war, he delivered said associated in business; that he was the office manager of "Hess and Zeitling"
stock certificates, with his blank endorsement thereon, to the Mitsuis, to before the war; that Hess used to tell him his daily transactions during the
whom said stock belonged. occupation; that at that time, Hess did not have in possession any certificates
of stock of the Lepanto in the name of Vicente Madrigal; that neither did
Matsune Kitajima declared that in June 1941 he relieved one Kobayashi, as Hess, during that period, operate as broker, for being American, he was
manager of the branch office of the Mitsuis in Manila; that he then receive under Japanese surveillance, and that Hess had made, during the
from Kobayashi the stock certificates for about 1,900,000 shares of the occupation, no transaction involving mining shares, except when he sold
Lepanto, belonging to the Mitsuis, but issued in favor of the Vicente Madrigal, 12,000 shares of the Benguet Consolidated, inherited from his mother,
except the certificates for 200,000 shares, which were in the name of the sometime in 1943.
Mitsuis; that all these certificates were in kept in a steel safe in said office of
the Mitsuis; that, in July 1941, he returned the stock certificates to Madrigal, E. A. Perkins, a member of the law firm DeWitt, Perkins & Ponce Enrile
with the request that he buy for the Mitsuis, from time to time, some more testified substantially as follows: On October 27, 1945, Leonardo Recio
shares of stock, in small lots; that Madrigal bought 200,000 additional shares brought stock certificate no. 2279 (Exhibit 2) and offered the same for sale to
of the Lepanto for the Mitsuis; that, late in November or early in December, Clyde DeWitt, who in turn, asked Perkins, whose room adjoined that of
1941, the stock certificates of the aforementioned 2,100,000 shares were DeWitt, to join them. Recio showed Exhibit 2 to DeWitt stating that he (Recio)
returned to the Mitsuis, which had decided to stop buying, in view of the wanted P0.13 per share. DeWitt handed Exhibit 2 over to Perkins, who, after
strained international situation then prevailing; that, as branch manager of examining the instrument, returned it to DeWitt. The latter, thereafter,
the Mitsuis, he was the only official authorized to dispose of the shares in checked it with a communication of the Property Custodian and then advised
question, none of which was alienated by him; and that he had the Recio that said Exhibit 2 was one of the stock certificates looted from the
aforementioned stock certificates in his possession continuously until early in Mitsuis and that he (DeWitt) would have to report the matter to said official.
April 1943, when he delivered the same to his successor in office, Kingy As DeWitt, thereupon, telephoned one Mr. Erickson, of the Property
Miwa. Custodian's office, Recio stepped out of the room without Exhibit 2, which
neither he or plaintiffs had ever tried to recover.
Apart from corroborating Kitajima's testimony relative to said delivery of stock
certificates in April 1943, Kingy Miwa testified that he kept the latter in his

770
Victor E. Lednicky, one of the organizers and prewar directors of the Lepanto, Such is the situation obtaining in the case at bar. Indeed, upon careful
and present vice-president and member of its board of director, asserted that, examination of the record before us, we find it impossible to share the
having learned from a soldier of the existence of mining papers and conclusions, made in the decision appealed from, relative to the alleged
securities of the Lepanto in the offices of the Mitsuis at the Ayala Building, flaws in the version of the defense.
formerly known as the National city Bank Building in Manila, he went thereto
in February 1945 and saw many documents scattered on the desks and floor Let us, first, examine the evidence for the plaintiffs, consisting, mainly, of their
of said premises. Among said papers, he noticed two stock certificates of the own testimony and that of Primitivo Javier and Leonardo Recio.
Lepanto, one, in the name of either a Japanese or Chinese, and the other, in
the name of Vicente Madrigal, endorsed in blank. Soon, however, he heard According to De los Santos, on or about December 8, 1942, he purchased
voices from the stairs, whereupon he departed hurriedly, for fear of being from Juan Campos, in Manila, 500,000 shares of stock of the Lepanto, for
mistaken for a looter. the aggregate sum of P30,000.00, or about P0.06 each share, paid in cash,
in exchange for the corresponding stock certificates, which were delivered to
After analyzing the foregoing evidence for the defense, the lower court found him. Several days later, he bought from Carl Hess, in Manila, 300,000 shares
the same "inherently improbable" and seemingly concluded that, as a of the Lepanto, at the same rate. Soon after, he visited his daughter in
consequence, it should accept plaintiffs' version, for which reason judgment Baguio, where he, likewise, saw his co-plaintiff, and former secretary, Isabelo
was rendered as above stated. It is well settled, in this jurisdiction, that the Astraquillo. Before leaving Astraquillo's house, De los Santos happened to
findings of fact particularly those relating to the credibility of the opposing mention his aforesaid purchases of Lepanto shares, at P0.06 each,
witnesses made by the Judge a quo, should not be disturbed on appeal, in whereupon, Astraquillo expressed the wish to buy 800,000 shares at the
the absence of strong and cogent reasons therefor. This policy is predicated same price, the amount of which he delivered to De los Santos the next day.
upon the circumstance that the trial court has had an opportunity, denied to Upon his return to Manila, De los Santos purchased from Hess said 800,000
the appellate court to observe the behaviour of the witnesses during the shares, the certificates of which were turned over by the former to Astraquillo,
hearing, a potent factor in gauging their bias and veracity. In the case at bar, in Baguio, at about Christmas time. Over 3 years later, or in January 1946,
however, we notice that, rejecting the theory of the defense, the court of De los Santos repaired to the offices of the Lepanto in Manila to ascertain
origin was guided, not by the conduct of the witnesses in the name course of whether it accepted certificates of stock for registration. He then received a
their testimony, but by what His Honor, the trial Judge, regarded as the negative answer. Upon further inquiry, he learned, in February 1946, that the
inherent weakness thereof, in the evaluation of which court does not enjoy shares in the name of Madrigal were blocked. So engaged the services of
the advantage already adverted to. Atty. A. Scheerer, who secured an order of release from the Freezing Control
Office of the United States Treasury Department. As he brought a copy of
Moreover, the decision appealed from appears to have assumed that this order to the offices of the Lepanto, on or about May 1, 1946, he was
plaintiffs' pretense must necessarily be relied upon, owing to the infirmities advised that no transfer could be affected without the authority of Clyde
said to have been found in the theory of the defense. This view suffers from a DeWitt, the company president. Thereupon, De los Santos caused to be
fatal defect. It overlooks that fact that the burden of proof is upon the filed, with the offices of the Property Custodian, the corresponding claim for
plaintiffs, and that, accordingly, a decision in their favor is not in order unless the shares of stock in question, with the result already adverted to.
a preponderance of the evidence supports their claim. To put it differently, the
alleged improbabilities in the testimony of the witnesses for the defense will Astraquillo tried to corroborate the testimony of De los Santos, concerning
not justify a judgment against the latter, if the evidence for the plaintiffs is the purchase of 800,000 shares of stock on behalf of the former. Moreover,
more improbable than, or, at least, as improbable as, that of the defense. Astraquillo declared that, being in need of money, he came to Manila in

771
November or December 1945, and delivered to stock broker Leonardo Recio introduced him to Mr. DeWitt, who stated that the shares of stock covered by
stock certificate No. 2279 (Exhibit 2) for 55,000 shares, with a view to Exhibit 2 were included in the list of questioned shares. DeWitt, also, asked
disposing of the same at a price ranging from P0.13 to P0.15 each. He him whether he would leave the certificate, to which Recio replied
advised Recio that, in the absence of any buyer, hew could see Mr. DeWitt, affirmatively. While he was away, several months later, or shortly before
who, probably, would be interested in purchasing the shares. Sometime later, Christmas, his house at Blumentrit Street, San Juan del Monte, Rizal, and
Astraquillo learned that, according to Recio, upon seeing Exhibit 2, DeWitt everything contained therein, including the aforementioned receipt, which
retained it upon the ground that the shares represented therein had been was in his wallet, were destroyed by fire.
blocked by the United States and that he (Recio) got therefor a receipt,
which was subsequently lost in a fire that destroyed his (Recio's) dwelling. As It thus appears that the only evidence on the alleged sale of the shares of
Astraquillo hurried to Manila, he was told that representatives of the CIC stock in question to the plaintiffs the main issue in the case at bar is the
would go to Baguio to investigate. So, he returned to Baguio, but he did not testimony of Apolinario de los Santos, who now claims to be the sole owner
wait for the investigation in that city. Late in February or early in March, 1946, thereof. Juan Campos and Carl Hess, the alleged vendors, could not take the
he came back to Manila and asked the assistance of De los Santos, witness stand, for Hess was executed by the Japanese, and Campos died
whereupon both contacted Atty. Scheerer for the purpose already stated. during the liberation of Manila. Thus, death has sealed the lips of the only
persons who could have positively corroborated or contradicted the
Primitivo Javier narrated that, late in 1945, he received Exhibit 2 from his aforementioned testimony of De los Santos. Was this a mere accident of fate,
uncle, Astraquillo, who wanted to sell the 55,000 shares represented by said as plaintiffs would have us believe? Or were Campos and Hess named by
stock certificate (No. 2279) at a price ranging from P0.12 to P0.15 each the plaintiffs as their immediate predecessors in interest precisely because,
share. He, in turn, delivered the certificate to Recio, a licensed broker. as contended by appellants, said deceased persons could no longer said
Subsequently, Recio reported to him that he (Recio) had brought Exhibit 2 to testimony?
the office of Mr. DeWitt, whom he did not see on his first visit; that he then left
Exhibit 2 in the hands of a person who worked in said office, one Atty. Orlina, For obvious reasons, the Court can not answer these questions with absolute
who issued a receipt therefor; that, when Recio came back, later on, DeWitt certainty. It can only explore the possibilities and probabilities of the case, in
told him that Exhibit 2 was defective; and that, accordingly, Exhibit 2 was left the light of human experience. And, viewed from this angle, it can not be
in the possession of Mr. DeWitt. Javier relayed this information to Astraquillo, denied that the demise of Campos and Hess before the filing of plaintiffs
who, thereupon, came to Manila. Both went to the temporary residence of claim seriously impairs the weight thereof. That the Grim Reaper had chosen
Recio in Sampaloc, his house in San Juan del Monte, Rizal, having been to strike at one of the alleged predecessors of the plaintiffs is a matter that
destroyed by fire late in December 1945. Recio then advised them that said may be attributed to sheer fortuitousness. When, as in the case at bar, not
receipt had been burned with his house. one, but both have thus been eliminated,, it is clear, however, that this
circumstances is most unusual, and most place the Court on guard.
Leonardo Recio said that sometime in 1945, Javier gave him Exhibit 2,
stating that it belonged to his uncle, who wanted to alienate the The need for caution becomes more imperative when we bear in mind that
corresponding shares of stock at P0.15, more or less, each, and suggesting an important piece of documentary evidence, which allegedly existed after
that he offer the same to Mr. DeWitt: In the latter's office, Atty. Orlina told liberation, and could have effectively corroborated one phase of the plaintiff's
Recio that DeWitt was busy and bade him (Recio) to return later. Recio contention, had, according to their evidence, disappeared through still
delivered Exhibit 2 to Orlina, who gave him a receipt, which, subsequently, another unfortunate turn of the wheel of fate. It will be recalled that late in
he showed to Javier. When, soon after, he went back to Orlina, the latter 1945, Leonardo Recio, allegedly acting on behalf of Astraquillo, offered to

772
sell to Atty. DeWitt the 55,000 shares represented by stock certificate No. as admitted by Santos even, no market value at all (p. 132, t.s.n). Indeed,
2279 (Exhibit 2). Recio testified that, having been unable to see DeWitt, the stockholders could neither collect dividends nor exercise their voting
when he (Recio) went to the latter's office, for the first time, said Exhibit 2 power, or otherwise participate in the operation of the enterprise. Moreover,
was left by him (Recio) in the hands of Atty. Orlina who worked therein and there was a possibility of its assets being fully confiscated, for all practical
gave him a receipt therefor. This receipt, if produced, would have surely purposes, should Japan emerge victorious in the was in the Pacific, which it
afforded us tangible proof of the veracity of, at least this part of plaintiffs' appeared to be winning easily up to that time (December, 1942).
story. Yet, we are now told that, one day in December, 1945, Recio's
house accidentally caught fire, and that the latter consumed, also, said (2) Inasmuch as citizens of the United States held a majority of the shares of
receipt, kept in a wallet, which, by accident, he had failed to bring with him. stock of the Lepanto, the same had from the view point of the Japanese, an
Aren't there too many accidents in plaintiffs' version? At any rate, we have enemy character, and the purchase of said stocks was, therefore, a hostile
thus been deprived of all means to check with reasonable certainty the truth act. As a matter of fact, in the proceedings before the Vested Property Claims
of any of the controverted portions of their pretense. In other words, the Committee, the parties including plaintiffs herein had stipulated "that
same is based, and must stand or fall, therefore, upon the uncorroborated such transfers and dealings in said stock were prohibited by the Japanese
testimony of plaintiff Apolinario de los Santos, and the credence and weight during the occupation and hence were dangerous." (Record on Appeal, p.
that may be given thereto. Upon a review of the record, we find, however, 110). Said transactions could jeopardize the life of the parties thereto and De
that said testimony is highly improbable and inherently weak, for, among los Santos was aware of the "highly dangerous" or "very risky" nature of the
other things: "mere possession" of the stock certificates in question. (pp. 141, 143, t. s. n.)

(1) De los Santos declared that, in December, 1942, he purchased 300,000 (3) Astraquillo is merely a former employee of De los Santos, who had,
shares from Juan Campos and 1,300,000 shares from Carl Hess, at P0.06 therefore, no reason to risk his neck, not only by allegedly buying 800,000
each share. As an enterprise controlled by Americans, the Lepanto had been shares of stock for Astraquillo, but, also, by avowedly bringing with him (De
seized by the Japanese who, accordingly, were operating it. At that time, los Santos) the corresponding stock certificates from Manila to Baguio, to
there were no clear, or, even, substantial, indications that changes would make delivery thereof to Astraquillo, as the defense would have us believe,
take place, either in the local or in the international situation in the near of notwithstanding the many Japanese check points in the 250 kilometers
foreseeable future. In deed, the morale of the population in democratic highway connecting both cities and the absence of any monetary or other
countries, particularly in the Philippines, was then at its lowest ebb. Both in gain he could have derived from the acts he professes to have performed.
Europe and in the Pacific, the Axis powers had reached in enemy territories
the highest degree of penetration attained during the last war. Before the (4) According to the Ballantyne schedule the accuracy of which has not
world had recovered from the shock produced by the German blitzkrieg been impugned by plaintiffs herein the Japanese war notes in the
operations in the low countries and in France, the Nazis were already Philippines had the same exchange of purchase value as the currency of our
knocking at the gates of Stalingrad entrenched in New Guinea and the legitimate government, in December, 1942 and this was conceded by De
Soloman Islands. The people had a hazy notion about the facts pertinent to los Santos (p. 136, t. s. n.) when they claim to have purchased the
the Battle of Midway (June 3-6, 1942) and the implications thereof were by Lepanto stocks. The P48,000 supposedly paid by the De los Santos, and the
and large unknown. In other words, the conditions were such as to warrant identical sum allegedly disbursed by Astraquillo, for their respective stock,
the general belief that the Lepanto would remain under the authority and represented, therefore, the same amount in legal tender of the
management of the Japanese Imperial forces for an indefinite period of time. Commonwealth of the Philippines. In fact, according to the evidence for the
As a consequence, the Lepanto stock had not merely a doubtful value, but plaintiffs, part of the price allegedly paid by Astraquillo, or P6,000, were in the

773
genuine Philippine money, representing his savings for 25 years. Said sum of (7) De los Santos could not have purchased 1,300,000 shares of stock, from
P6,000 being insufficient to cover the cost of 800,000 shares of stock, Hess, and received from him the corresponding stock certificates, endorsed
Astraquillo, it is urged, alienated other properties to raise the amount in blank by Vicente Madrigal, for Hess had never had such stock certificates
necessary thereof. It is very difficult to believe that the plaintiffs would have in his possession during the occupation. There is no plausible reason to
parted with P48,000 each precisely when, owing to the abnormal doubt the veracity of the testimony of Miguel Simon to this effect, for the
conditions brought about by the occupation, said funds might be needed, at latter had no possible motive to commit perjury, and was in a position to
any time, to meet unforeseen emergencies of the gravest and most vital know what he was talking about. Apart from being a brother-in-law of Hess,
nature for shares of stock of dubious value then and in the foreseeable Simon was manager of the firm Hess & Zeitling, of which Hess was the
future. senior partner, who used to inform him (Simon) of his (Hess) business
transactions.
(5) We are not satisfied that either De los Santos or Astraquillo possessed
enough resources to have P48,000, in cash, each, in December 1942. Their (8) Campos and Hess could not have delivered the stock certificates for the
evidence on this point is too general apart from being based exclusively 1,600,000 shares of stock in question, and, consequently, said shares of
upon their respective oral testimonies, which are absolutely uncorroborated stock could not have been sold by them, to De los Santos in December 1942,
to support their contention. At any rate, De los Santos admitted that he is inasmuch as from December 1941 to April 1943, said stock certificates were
"not yet" rich (p. 134, t. s. n.), and his testimony suggests that he did not continuously in the custody of Matsume Kitajima, manager of the Mitsuis in
even own the house in which he lived. Manila, whose testimony was corroborated by his successor in office, Kingy
Miwa, to whom Kitajima turned over the stock certificates in April 1943. The
(6) Campos offered to sell his stocks, according to De los Santos, at P0.06 sincerity of Matsume Kitajima and Kingy Miwa can not doubted, for neither
each (although its par value was P0.10), stating that "he (Campos) needed appears to have any possible reason to trifle with the facts. Indeed, their
money" (p. 43, t.s.n.), and advised him that Hess was, also, willing to dispose testimony, if accepted as true, would ultimately result in the confiscation, by
of his own stocks at the same price. Being, accordingly, aware that Campos the Republic of the Philippines, of the shares of stock in question and, thus,
and Hess were in need of money and considering the risks attending the place the same beyond the reach of the Mitsuis.
transaction, it is but logical to expect De los Santos, an experienced trader in
stocks, to bargain for a lower price. Yet, the evidence for the plaintiffs shows It has been intimated that Kitajima and Kingy may have testified as they did,
that neither he nor Astraquillo tried to do so, contrary to the normal course of either to protect themselves, because they might have disposed of the
events. shares of stock in question for their personal benefit, or because there had
been undue influence or pressure from the authorities presumably officers
of the government of the United States. But these are mere speculations,
without sufficient basis. Besides, judicial notice may be taken of the
circumstance that, during the occupation, even minor Japanese officials
could easily make money, in the Japanese properties. Again, in December,
1942, the Japanese in the Philippines appeared to have no doubts that, in
effect, Japan had already won the war. In short, Kitajima and Kingy must
have thought that, sooner or later, Japan would own the Lepanto and that,
therefore, they would have to account for the shares of stock under

774
consideration. Consequently, it is most unlikely that neither would have and dwellings were looted during the liberation of Manila. The possibility that
misappropriated said shares of stock as suggested by the plaintiffs. possession of the stock certificates in question may have been secured by
looting should not be "ruled out," therefore, irrespective of the credence and
The benefits which the Mitsuis and Japan may derive from a decision against weight given to the testimony of Lednicky. Actually, said certificates are
the plaintiffs inasmuch as the value of the shares of stock in question included in the list of stocks certificates of the Lepanto which, soon after
would then be credited in payment of the reparation which may be demanded liberation, were reported and considered looted from the Mitsuis, and,
by the Philippines and/or the United States has been pointed out, in the accordingly, "blocked" or "frozen" by the authorities. Irrespective of the
dissenting opinion, as a possible motive for the commission of perjury by foregoing, De los Santos could not have obtained those certificates from
Kitajima and Kingy. Besides being purely conjectural in nature, this line of Campos and Hess in December 1942, inasmuch, as, from December 1941 to
thought which not even the plaintiffs have taken would have no leg to April 1943, Kitajima had been continuously in possession of said documents,
stand on, unless we assume that the Mitsuis had sold or otherwise disposed none of which had been held by Hess during the occupation.
of said stocks during the year 1942, but before the alleged transactions
between Campos and Hess, on the one hand, and the plaintiffs on the other, The lower court considered against the defense the circumstance that
in December of that year. It is inconceivable, however, that the Mitsuis would Lednicky, Simon and Perkins had not testified before the Vested Property
part with the stocks in question, precisely when Japanese was at the crest of Claims Committee. There is no evidence, however, that any of them knew of
its military and political victories. Indeed, even if its officers had already the proceedings before said committee. Furthermore, none of them has any
foreseen, at the time, the eventual defeat of the axis powers and personal interest in the outcome of this action. Consequently, they have no
everything then appeared to indicate the contrary the Mitsuis could not possible motive to distort the truth, unlike De los Santos, who, as the present
have disposed of said stocks without thereby revealing their own lack of faith claimant of all shares of stock in dispute, will de directly affected by the
in the ability of Japan to achieve final victory. Thus, the Mitsuis would have outcome of the case at bar. His testimony, therefore, cannot be more weighty
caused a grave injury upon the Japanese propaganda and thereby earned than that of the aforementioned witnesses for the defense.
severe punishment from the Imperial Government. Nothing, absolutely
nothing, in the record, or in contemporary history, warrants the belief that the The decision appealed from criticizes the testimony of Perkins upon the
Mitsuis, who were closely associated with the Japanese Government, could following grounds:
be guilty of such folly.
(1) Having taken no part in the alleged looting of Exhibit 2, Recio had nothing
Let us now turn our attention to the evidence for the defense, beginning with to fear in connection therewith and, so, he could not have left the office of Mr.
the testimony of Victor E. Lednicky. It will be recalled that this witness DeWitt, while the latter was talking over the telephone with a representative
claimed to have gone to the premises of the Mitsuis, sometime in February of the Alien Property Custodian; .
1945, including two (2) Lepanto certificates of stock, one of which was in the
name of Vicente Madrigal, whose blank indorsement appeared thereon. (2) Inasmuch as DeWitt had stated that Exhibit 2 was included in the list of
Thus, the defense sought to prove that the certificates of the shares of stock looted stock certificates, Perkins should have known that, as holder of the
involved in this case have probably been looted. The lower court found certificate, Recio is presumed to be the one who stole the same. Why then
Lednicky's story inherently improbable and then concluded that the theory of plaintiffs inquire did Perkins fail to prevent Recio from leaving said office?
the looting must, consequently, be "ruled out". To our mind, however, the
testimony of Lednicky is not inherently improbable. Besides, it is a matter of
common knowledge, of which judicial notice may be taken, that many offices

775
As regards the first observation, suffice it to say that, as bearer of the Exhibit conscience of a man who came to the Court with a mental
2, Recio who, according to the lower court, is an intelligent man must reservation, but who may have been compelled under the
have realized the danger, probably unforeseen by him, of being considered a circumstances to play the role of a willing tool. (p. 54, R.A.)
privy to the looting of said stock certificates, of which he might have been
unaware before the conference with Mr. DeWitt. Hence, Recio's fright and The following portion of Miwa's testimony illustrates the point referred to in
virtual flight. Verily, the testimony of Perkins on this point is borne out by the the decision appealed from:
undisputed fact that Exhibit 2 was left by Recio in the hands of DeWitt, and
that neither Astraquillo, nor his alleged successor in interest, De los Santos, ATTY. QUIRINO:
has ever demand from DeWitt the return of said certificate, or even
recriminated Recio for having voluntarily parted with its possession, as he
Q. Will you please go over this paper which for purposes of
would have us believe, without authority therefor, as a broker or agent who
identification we request that it be marked as Exhibit M for the
was supposed merely to find a buyer.
plaintiffs and which was marked Exhibit 6-b before the Vested
Property Claims Committee, and tell us if you know that document?
As to the second observation, Perkins knew that Recio was acting solely as a A. No. I do not remember this paper.
broker or agent. As such, he was not the real holder of Exhibit 2, and,
consequently, the presumption adverted to did not apply to him. Even if it did,
Q. Mr. Miwa, at the bottom of this certificate or Exhibit M, which
however, what could Perkins have done? Use force or violence upon the
was Exhibit 6b in the committee and submitted by the Alien Property
person of Recio, or ask a policeman to detain him? Neither step, however,
Administration, there is a typewritten name, Kingy Miwa, and above it
could have been taken without some risks. To begin with, Perkins could not
is a signature. Will you kindly tell the Court if that is your signature or
have properly taken the law in his own hands. Had he done so, Recio could
not? Please look over it again. A. No. It is not mine.
have legally used force against force. Moreover, said presumption is
rebuttable and would have easily been offset by the undeniable fact that
Recio had acted merely in a representative capacity. Again, why should Q. Please examine it carefully and tell the Court afterwards if you
Perkins take the initiative in the matter? Was it not being handled by his recognize that signature. Examine it carefully. A. It looks
associate in the law firm, Mr. DeWitt, one of the most able members of the very similar to my signature.
Philippine Bar? It may not be amiss to add that the record before us
discloses absolutely nothing that may cast even a shadow of doubt upon the Q. But would you want or are you willing to go on record and say
honesty of Mr. Perkins. that it is not your signature? A. I can not say. I don't exactly
remember that I signed this, but it looks very similar to my signature.
The language of the lower court in commenting on the testimony of Miwa
was: Q. You will not testify under oath that this is your signature?
A. Yes. sir.
. . . In general, the testimony of Miwa is unreliable. His behaviour in
Court in denying first and then in accepting later his own signature Q. What do you mean to say by "yes, sir"? Do you swear that this
throws him to a position where the Court must look upon him with is your signature or not your signature? A. I think this is my
suspicion and distrust. His prevarication before the Court as to the signature.
genuineness of his own signature was probably due to the

776
Q. So, you are willing to go on record now that that signature countersigned by the secretary or clerk and sealed with the seal of
appearing in Exhibit "M" is your signature? A. Yes, I think the corporation, shall be issued in accordance with the by-laws.
so. (pp. 125-126, t. s. n.) Shares of stock so issued are personal property and may be
transferred by delivery of the certificate endorsed by the owner or his
We do not agree with its appraisal by the lower court. It is clear that, as he attorney in fact or other person legally authorized to make the
did not remember the execution of Exhibit M several years before the hearing transfer. No transfer, however, shall be valid, except as between the
of this case, Miwa had doubts about the genuineness of the signature parties, until the transfer is entered and noted upon the books of the
thereon, but the appearance thereof, similar or identical to that of his own corporation so as to show the names of the parties to the
signature, prevented him from denying its authenticity. This does not indicate transaction, the date of the transfer, the number of the certificate,
lack of veracity on his part. At any rate, plaintiffs claim to have bought the and the number of shares transferred.
shares of stock in question in December, 1942, or during the management of
Kitajima, who held the corresponding stock certificates continuously from No shares of stock against which the corporation holds any unpaid
December, 1941, to April, 1943, when Miwa substituted him, so that neither claim shall be transferable on the books of the corporation.
Campos nor Hess could have delivered those certificates to De los Santos in (Emphasis supplied.)
December 1942. Apart from this, if there are flaws in the proof for the
defense, those of the evidence for the plaintiffs are much bigger and more Pursuant to this provision, a share of stock may be transferred by
substantial and vital. Consequently, we hold that plaintiffs have not endorsement of the corresponding stock certificate, coupled with its delivery.
established their pretense by a preponderance of the evidence. However, the transfer shall "not be valid, except as between the parties," until
it is "entered and noted upon the books of the corporation." no such entry in
Even, however, if Juan Campos and Carl Hess had sold the shares of stock the name of the plaintiffs herein having been made, it follows that the transfer
in question, as testified to by De los Santos, the result, insofar as plaintiffs allegedly effected by Juan Campos and Carl Hess in their favor is "not valid,
are concerned, would be the same. It is not disputed that said shares of except as between" themselves. It does not bind either Madrigal or the
stock were registered, in the records of the Lepanto, in the name of Vicente Mitsuis, who are not parties to said alleged transaction. What is more, the
Madrigal. Neither it is denied that the latter was, as regards said shares of same is "not valid," or, in the words of the Supreme Court of Wisconsin (Re
stock, a mere trustee for the benefit of the Mitsuis. The record shows and Murphy, 51 Wisc. 519, 8 N. W. 419) which were quoted approval in Uson
there is no evidence to the contrary that Madrigal had never disposed of vs. Diosomito (61 Phil., 535) "absolutely void" and, hence, as good as
said shares of stock in any manner whatsoever, except by turning over the non-existent, insofar as Madrigal and the Mitsuis are concerned. For this
corresponding stock certificates, late in 1941, to the Mitsuis, the beneficial reason, although a stock certificate is sometimes regarded as quasi-
and true owners thereof. It has, moreover, been established,, by the negotiable, in the sense that it may be transferred by endorsement, coupled
uncontradicted testimony of Kitajima and Miwa, the managers of the Mitsuis with delivery, it is well settled that the instrument is non-negotiable, because
in the Philippines, from 1941 to 1945, that the Mitsuis had neither sold, the holder thereof takes it without prejudice to such rights or defenses as the
conveyed, or alienated said shares of stock, nor delivered the registered owner or creditor may have under the law, except insofar as such
aforementioned stock certificates, to anybody during said period. Section 35 rights or defenses are subject to the limitations imposed by the principles
of the Corporation Law reads: governing estoppel.

The capital stock corporations shall be divided into shares for which Certificates of stock are not negotiable instruments (post, Par. 102),
certificates signed by the president or the vice-president, consequently, a transferee under a forged assignment acquires no

777
title which can be asserted against the true owner, unless his own transaction itself "; the negligence must be in or immediately
negligence has been such as to create an estoppel against him connected with the transfer itself . Furthermore, "to establish this
(Clarke on Corporations, Sec. Ed. p. 415). If the owner of the estoppel it must appear that the true owner had conferred upon the
certificate has endorsed it in blank, and it is stolen from him, no title person who has diverted the security the indicia of ownership, or an
is acquired by an innocent purchaser for value (East Birmingham apparent title or authority to transfer the title." So the owner is not
Land Co. vs. Dennis, 85 Ala. 565, 2 L.R.A. 836; Sherwood vs. mining guilty of negligence in merely entrusting another with the possession
co., 50 Calif. 412). As was said by the Supreme Court of the United of his certificate of stock, if he does not, by assignment or otherwise,
States in a leading case (Western Union Telegraph clothe him with the apparent title. Nor is he deprived of his title or his
Co. vs. Davenfort, 97 U. S. 369; 24 L. Ed. 1047) remedy against the corporation because he intrusts a third person
with the key of a box in which the certificate are kept, where the latter
"Neither the absence of blame on the part of the officers of the takes them from the box and by forging the owner's name to a power
company in allowing an unauthorized transfer of stock, nor the good of attorney procures their transfer on the corporate books. Nor is the
faith of the purchaser of stolen property, will avail as an answer to mere indorsement of an assignment and power of attorney in blank
the demand of the true owner. The great principle that no one can on a certificate of stock, which is afterwards lost or stolen, such
deprived of his property without his assent, except by processes of negligence as will estop the owner from asserting his title as against
the law, requires, in the case mentioned, that the property wrongfully a bona fide purchaser from the finder or thief, or from holding the
transferred or stolen should be restored to its rightful owner." (The corporation liable for allowing a transfer on its books, where the loss
Philippine Law of Stock Corporations by Fisher, p. 132.) (Emphasis or theft of the certificate was not due to any negligence on the part of
supplied.) the owner, although there is some dangerous and wholly
unjustifiable dictum to the contrary. So it has been held that the fact
In the language of Fletcher's Cyclopedia Corporations (Vol. 12, pp. 521-534): that stock pledged to a bank is endorsed in blank by the owner does
not estop him from asserting title thereto as against a bona fide
purchaser for value who derives his title from one who stole the
The doctrine that a bona fide purchaser of shares under a forged or
certificate from the pledgee. And this has also been held to be true
unauthorized transfer acquires no title as against the true owner
though the thief was an officer of the pledgee, since his act in
does not apply where the circumstances are such as to estop the
wrongfully appropriating the certificate cannotbe regarded as a
latter from asserting his title. . . .
misappropriation by the bank to whose custody the certificate was
intrusted by the owner, even though the bank may be liable to the
xxx xxx xxx pledgor. . . . . A person is not guilty of negligence in leaving a
certificate of stock endorsed in blank in a safe deposit box used by
A reason often given for the rule is that it is a case for the application himself and another jointly, so as to be estopped from asserting his
of the maxim that where one of two innocent parties must suffer by title after the certificate has been stolen by the other, and sold or
reason of a wrongful or unauthorized act, the loss must fall on the pledged to a bona fide purchaser or pledgee. Nor is he negligent in
one who first trusted the wrongdoer and put in his hands the means putting a certificate so endorsed in a place to which an employee
of inflicting such loss. But "negligence which will work an estoppel of had access, where he has no reason to doubt the latter's
this kind must be a proximate cause of the purchase or advancement honesty, . . . . (Emphasis supplied.)
of money by the holder of the property, and must enter into the

778
In the leading case of Knox vs. Eden Muscee American Co. (42 N. E. 988, or robbery. The system of registry of negotiable bonds, which
992-993), the rule has been forcefully stated as follows: prevails to a considerable extent, authorized by statutes of some of
the states and of the United States, seems to indicate a tendency
The courts have been frequently importuned to extend the qualities to restrict, rather than to extend, the range of negotiable instruments.
of negotiability of stock certificates beyond the limits mentioned, and (Emphasis supplied.)
clothe them with the same character of complete negotiability as
attaches to commercial paper, so as to make a transfer to a The status of quasi-negotiability generally accorded to, and at present
purchaser in good faith for value equivalent to actual title, although enjoyed by, certificates of stock, under the Philippine law, is in itself a
there was no agency in the transferor, and the certificate had been recognition of the fact that the certificates are non-negotiable. Instead of
lost without the fault of the true owner, or had been obtained by theft sustaining appellees' claim, section 5 of the uniform Stock Transfer Act,
or robbery. But the courts have refused to accede to this view, and which "gives full negotiability to certificates of stock," refutes said claim and
we have found no case entitled to be regarded as authority which confirms the non-negotiable character of stock certificates in the absence of
denies to the owner of a stock certificate which has been lost without said Unifrom Act, for, obviously, the same could not have given, negotiability
his negligence, or stolen, the right to reclaim it from the hands of any to an instrument already possessing this attribute prior thereto. Again, apart
person in whose possession it subsequently comes, although the from being distinct from the general Corporation Law, the aforementioned
holder may have taken it in good faith and for value. The precise Uniform Act is not in force in the Philippines. In this connection, it should be
question has not often been presented to the courts, for the reason, noted that this special piece of legislation was adopted in some states of the
probably, that they have with great uniformity held that stock union as early as the year 1910. The failure of the Philippine government to
certificates were not negotiable instruments in the broad meaning of incorporate its provisions in our statute books, for a period of almost 45
that phrase; but whenever the question has a risen it has been held years, is, to our mind, clear proof of the unwillingness of our department to
that the title of the true owner of a lost or stolen certificate may be change the policy set forth in section 35 of Act No. 1459. Needless to say,
asserted against any one subsequently its possession although the this fact negates our authority which is limited to the interpretation of the
holder may be bona fide purchaser. Anderson vs. Nicholas, 28 N. Y. law, and its application, with all its imperfections to abandon what the
600; Power vs. Robinson, 52 Fed. 520; Biddle vs. Bayard, 13 Pa. St. dissenting opinion characterizes as the "civil law standpoint," and substitute,
150; Barstow vs. mining Co., 64 Cal. 388, 1 Pac. 349. See in lieu thereof, the commercial viewpoint, by applying said section 5 of the
Shaw vs.Railroad Co., 101 U. S. 557. . . . It is plain, we think, that the Uniform Stock Transfer Act, although not a part of the law of the land. Indeed,
argument in support of the judgment in this case, base on the even in matters generally considered as falling within "commercial territory",
complete negotiability of stock certificates, is not supported by, but is the Roman Law concept has not given way in the Philippines to the Common
contrary to, the decisions. If public policy requires that a further Law approach, except when there is explicit statutory provision to the
advance should be made in more completely assimilating them to contrary.
commercial paper in the qualities of negotiability, the legislature, and
not the courts, should so declare. Under the law as it has hitherto In the case at bar, neither madrigal nor the Mitsuis had alienated shares of
prevailed there does not seem to have been any serious hindrance stock in question. It is not even claimed that either had, through negligence,
in dealing with property of this character. It may, perhaps, be given occasion for an improper or irregular disposition of the
doubted, taking into consideration the interests of investors as well corresponding stock certificates. Plaintiffs merely argue without any evidence
as dealers, whether it would be wise to remove the protection which whatsoever thereon that Kitajima might have, or must have, assigned the
the true owner of a stock certificate now has against accident, theft, certificates on or before December 1942, although, as above stated, this is,

779
not only, improbable, under the conditions, then obtaining, but, also., by the defense and, to our mind, has not been established by a
impossible, considering that, in April 1943, Kitajima delivered the instruments preponderance of the evidence. Hence, as the undisputed principal or
to Miwa, who kept them in its possession until 1945. At any rate, such beneficiary of the registered owner (Madrigal), the Mitsuis may claim his
assignment by Miwa granting for the sake of argument the accuracy of the rights, which cannot be exercised by the plaintiffs, not only because their
surmise of plaintiffs herein was unauthorized by the mitsuis, who, in the alleged title is not derived either from madrigal or from the Mitsuis, but, also,
light of the precedents cited above, are not chargeable with negligence. In because it is in derogation, of said rights. madrigal and the Mitsuis
other words, assuming that Kitajima had been guilty of embezzlement, by are not privies to the alleged sales by Campos and Hess to the plaintiffs,
negotiating the stock certificates in question for his personal benefit, as contrary to the latter's pretense.
claimed by the plaintiffs, the title of his assignees and successors in interest
would still be subject to the rights of the registered owner, namely, Madrigal, In conclusion, when the Property Custodian issued the Vesting Order
and consequently, of the party for whose benefit and account the latter held complained of, the shares of stock in question belonged to the Mitsuis,
the corresponding shares of stock, that is to say, the Mitsuis. admittedly an enemy corporation, so that Vesting Order is in conformity with
law and should be upheld. Wherefore, the decision appealed from is hereby
At any rate, at the time of the alleged sales in their favor, plaintiffs were reversed, and the complaint, accordingly, dismissed, with costs against the
aware of sufficient facts to put them on notice of the need of inquiring into the plaintiffs-appellees. It is so ordered.
regularity of the transactions and the title of the supposed vendors. Indeed,
the certificates of stock in question were in the name of madrigal. Obviously, Paras, C. J., Pablo, Padilla, Montemayor, Reyes, A., Jugo and Labrador,
therefore, the alleged sellers (Campos and Hess) were not registered owners JJ., concur.
of the corresponding shares of stock. Being presumed to know the law
particularly the provisions of section 35 of Act No. 1459 and, as
experienced traders in shares of stock, plaintiffs must have, accordingly,
been conscious of the consequent infirmities in the title of the supposed
vendors, or of the handicaps thereof. Moreover, the aforementioned sales
were admittedly hostile to the Japanese, who had prohibited it and plaintiffs
had actual knowledge of these facts and of the risks attendant to the alleged
transaction. In other words, plaintiffs advisedly assumed those risks and,
hence, they can not validly claim, against the registered stockholder, the
status of purchasers in good faith.

The lower court held, and plaintiffs maintain that, not being the registered
owners of the shares of stock in question, the Mitsuis can not assert a better
right than said plaintiffs. This pretense is untenable. Inasmuch as Madrigal,
the registered owner of said shares of stock, has always acknowledged that
he held the same merely as an agent of, or trustee for, the mitsuis and this
is not denied it follows that the latter are entitled to invoke such rights as
Madrigal had as registered stockholder. Upon the other hand, even the
alleged sale by Juan Campos and Carl Hess to plaintiffs herein is contested

780
781
Leonito Lopue, and Luisa U. Dacles to nullify the issuance of 823 shares of
stock of the Inocentes de la Rama, Inc. in favor of the said defendants. The
gist of the complaint, filed on April 4, 1972, is that the plaintiffs, with the
exception of Anastacio Dacles who was joined as a formal party, are the
G.R. No. L-40620 May 5, 1979 owners of 1,328 shares of stock of the Inocentes de la Rama, Inc., a
domestic corporation, with an authorized capital stock of 3,000 shares, with a
par value of P100.00 per share, 2,177 of which were subscribed and issued,
RICARDO L. GAMBOA, LYDIA R. GAMBOA, HONORIO DE 1A RAMA,
thus leaving 823 shares unissued; that upon the plaintiffs' acquisition of the
EDUARDO DE LA RAMA, and the HEIRS OF MERCEDES DE LA RAMA-
shares of stock held by Rafael Ledesma and Jose Sicangco, Jr., then
BORROMEO, petitioners,
President and Vice-President of the corporation, respectively, the defendants
vs.
Mercedes R. Borromeo, Honorio de la Rama, and Ricardo Gamboa,
HON. OSCAR R. VICTORIANO as Presiding Judge of the Court of First
remaining members of the board of directors of the corporation, in order to
Instance of Negros Occidental, Branch II, BENJAMIN LOPUE, SR.,
forestall the takeover by the plaintiffs of the afore-named corporation,
BENJAMIN LOPUE, JR., LEONITO LOPUE, and LUISA U.
surreptitiously met and elected Ricardo L. Gamboa and Honorio de la Rama
DACLESrespondents.
as president and vice-president of the corporation, respectively, and
thereafter passed a resolution authorizing the sale of the 823 unissued
Exequiel T. A Alejandro for petitioners. shares of the corporation to the defendants, Ricardo L. Gamboa, Lydia R.
Gamboa, Honorio de la Rama, Ramon de la Rama, Paz R. Battistuzzi
Acua, Lirazan & Associates for private respondents. Eduardo de la Rama, and Mercedes R. Borromeo, at par value, after which
the defendants Honorio de la Rama, Lydia de la Rama-Gamboa, and Enzo
Battistuzzi were elected to the board of directors of the corporation; that the
sale of the unissued 823 shares of stock of the corporation was in violation of
CONCEPCION JR., J,: the plaintiffs' and pre-emptive rights and made without the approval of the
board of directors representing 2/3 of the outstanding capital stock, and is in
Petition for certiorari to review the order of the respondent judge, dated disregard of the strictest relation of trust existing between the defendants, as
January 2, 1975, denying the petitioners' motion to dismiss the complaint stockholders thereof; and that the defendants Lydia de la Rama-Gamboa,
filed in Civil Case No. 10257 of the Court of First Instance of Negros Honorio de la Rama, and Enzo Battistuzzi were not legally elected to the
Occidental, entitled, "Benjamin Lopue Sr., et al., plaintiffs, versus Ricardo board of directors of the said corporation and has unlawfully usurped or
Gamboa, et al., defendants," as well as the order dated April 4, 1975, intruded into said office to the prejudice of the plaintiffs. Wherefore, they
denying the motion for the reconsideration of Said order. prayed that a writ of preliminary injunction be issued restraining the
defendants from committing, or continuing the performance of an act tending
to prejudice, diminish or otherwise injure the plaintiffs' rights in the corporate
In the aforementioned Civil Case No. 10257 of the Court of First Instance of
properties and funds of the corporation, and from disposing, transferring,
Negros Occidental, the herein petitioners, Ricardo L. Gamboa, Lydia R.
selling, or otherwise impairing the value of the 823 shares of stock illegally
Gamboa, Honorio de la Rama, Eduardo de la Rama, and the late Mercedes
issued by the defendants; that a receiver be appointed to preserve and
de la Rama-Borromeo, now represented by her heirs, as well as Ramon de
administer the property and funds of the corporation; that defendants Lydia
la Rama, Paz de la Rama-Battistuzzi, and Enzo Battistuzzi, were sued by the
de la Rama-Gamboa, Honorio de la Rama, and Enzo Battistuzzi be declared
herein private respondents, Benjamin Lopue, Sr., Benjamin Lopue, Jr.,

782
as usurpers or intruders into the office of director in the corporation and, participations or title that the defendants Ramon L. de la
consequently, ousting them therefrom and declare Luisa U. Dacles as a Rama, Paz de la Rama Battistuzzi Enzo Battistuzzi now
legally elected director of the corporation; that the sale of 823 shares of stock have or may have in the eight hundred twenty-three (823)
of the corporation be declared null and void; and that the defendants be shares in the capital stock of the corporation INOCENTES
ordered to pay damages and attorney's fees, as well as the costs of suit . 1 DELA RAMA, INC.' which were issued in the names of the
defendants in the above-entitled case on or about February
Acting upon the complaint, the respondent judge, after proper hearing, 11, 1972, or at any date thereafter and which shares are the
directed the clerk of court "to issue the corresponding writ of preliminary subject-matter of the present suit.
injunction restraining the defendants and/or their representatives, agents, or
persons acting in their behalf from the commission or continuance of any act The compromise agreement was approved by the trial court on December 4,
tending in any way to prejudice, diminish or otherwise injure plaintiffs' rights 1972, 5 As a result, the defendants filed a motion to dismiss the complaint, on
in the corporate properties and funds of the corporation Inocentes de la November 19, 1974, upon the grounds: (1) that the plaintiffs' cause of action
Rama, Inc.' and from disposing, transferring, selling or otherwise impairing had been waived or abandoned; and (2) that they were estopped from further
the value of the certificates of stock allegedly issued illegally in their names prosecuting the case since they have, in effect, acknowledged the validity of
on February 11, 1972, or at any date thereafter, and ordering them to deposit the issuance of the disputed 823 shares of stock. The motion was denied on
with the Clerk of Court the corresponding certificates of stock for the 823 January 2, 1975. 6
shares issued to said defendants on February 11, 1972, upon plaintiffs'
posting a bond in the sum of P50,000.00, to answer for any damages and The defendants also filed a motion to declare the defendants Ramon L. de la
costs that may be sustained by the defendants by reason of the issuance of Rama, Paz de la Rama Battistuzzi and Enzo Battistuzzi in contempt of court,
the writ, copy of the bond to be furnished to the defendants. " 2 Pursuant for having violated the writ of preliminary injunction when they entered into
thereto, the defendants deposited with the clerk of court the corporation's the aforesaid compromise agreement with the plaintiffs, but the respondent
certificates of stock Nos. 80 to 86, inclusive, representing the disputed 823 judge denied the said motion for lack of merit. 7
shares of stock of the corporation. 3
On February 10, 1975, the defendants filed a motion for the reconsideration
On October 31, 1972, the plaintiffs therein, now private respondents, entered of the order denying their motion to dismiss the complaint' and subsequently,
into a compromise agreement with the defendants Ramon de la Rama, Paz an Addendum thereto, claiming that the respondent court has no jurisdiction
de la Rama Battistuzzi and Enzo Battistuzzi , 4 whereby the contracting to interfere with the management of the corporation by the board of directors,
parties withdrew their respective claims against each other and the and the enactment of a resolution by the defendants, as members of the
aforenamed defendants waived and transferred their rights and interests over board of directors of the corporation, allowing the sale of the 823 shares of
the questioned 823 shares of stock in favor of the plaintiffs, as follows: stock to the defendants was purely a management concern which the courts
could not interfere with. When the trial court denied said motion and its
3. That the defendants Ramon L. de la Rama, Paz de la addendum, the defendants filed the instant petition for certiorari for the
Rama Battistuzzi and Enzo Battistuzzi will waive, cede, review of said orders.
transfer or other wise convey, as they hereby waive, cede,
transfer and convey, free from all liens and encumbrances The petition is without merit. The questioned order denying the petitioners'
unto the plaintiffs, in such proportion as the plaintiffs may motion to dismiss the complaint is merely interlocutory and cannot be the
among themselves determine, all of the rights, interests,

783
subject of a petition for certiorari. The proper procedure to be followed in The petitioners further contend that the proper remedy of the plaintiffs would
such a case is to continue with the trial of the case on the merits and, if the be to institute a derivative suit against the petitioners in the name of the
decision is adverse, to reiterate the issue on appeal. It would be a breach of corporation in order to secure a binding relief after exhausting all the possible
orderly procedure to allow a party to come before this Court every time an remedies available within the corporation.
order is issued with which he does not agree.
An individual stockholder is permitted to institute a derivative suit on behalf of
Besides, the order denying the petitioners' motion to dismiss the complaint the corporation wherein he holds stock in order to protect or vindicate
was not capriciously, arbitrarily, or whimsically issued, or that the respondent corporate rights, whenever the officials of the corporation refuse to sue, or
court lacked jurisdiction over the cause as to warrant the issuance of the writ are the ones to be sued or hold the control of the corporation. In such
prayed for. As found by the respondent judge, the petitioners have not actions, the suing stockholder is regarded as a nominal party, with the
waived their cause of action against the petitioners by entering into a corporation as the real party in interest. 12 In the case at bar, however, the
compromise agreement with the other defendants in view of the express plaintiffs are alleging and vindicating their own individual interests or
provision of the compromise agreement that the same "shall not in any way prejudice, and not that of the corporation. At any rate, it is yet too early in the
constitute or be considered a waiver or abandonment of any claim or cause proceedings since the issues have not been joined. Besides, misjoinder of
of action against the other defendants." There is also no estoppel because parties is not a ground to dismiss an action. 13
there is nothing in the agreement which could be construed as an affirmative
admission by the plaintiff of the validity of the resolution of the defendants WHEREFORE, the petition should be, as it is hereby DISMISSED for lack of
which is now sought to be judicially declared null and void. The foregoing merit. With costs against the petitioners.
circumstances and the fact that no consideration was mentioned in the
agreement for the transfer of rights to the said shares of stock to the plaintiffs SO ORDERED.
are sufficient to show that the agreement was merely an admission by the
defendants Ramon de la Rama, Paz de la Rama Battistuzzi and Enzo
Antonio, Aquino, Santos and Abad Santos JJ., concur.
Battistuzzi of the validity of the claim of the plaintiffs.

The claim of the petitioners, in their Addendum to the motion for


reconsideration of the order denying the motion to dismiss the complaint,
questioning the trial court's jurisdiction on matters affecting the management
of the corporation, is without merit. The well-known rule is that courts cannot
undertake to control the discretion of the board of directors about
administrative matters as to which they have legitimate power of, 10 action
and contracts intra vires entered into by the board of directors are binding
upon the corporation and courts will not interfere unless such contracts are
so unconscionable and oppressive as to amount to a wanton destruction of
the rights of the minority. 11 In the instant case, the plaintiffs aver that the
defendants have concluded a transaction among themselves as will result to
serious injury to the interests of the plaintiffs, so that the trial court has
jurisdiction over the case.

784
785
ANTONIO, J.:

The instant petition for certiorari, mandamus and injunction, with prayer for
issuance of writ of preliminary injunction, arose out of two cases filed by
petitioner with the Securities and Exchange Commission, as follows:

SEC CASE NO 1375

On October 22, 1976, petitioner, as stockholder of respondent San Miguel


Corporation, filed with the Securities and Exchange Commission (SEC) a
petition for "declaration of nullity of amended by-laws, cancellation of
certificate of filing of amended by- laws, injunction and damages with prayer
for a preliminary injunction" against the majority of the members of the Board
of Directors and San Miguel Corporation as an unwilling petitioner. The
petition, entitled "John Gokongwei Jr. vs. Andres Soriano, Jr., Jose M.
G.R. No. L-45911 April 11, 1979 Soriano, Enrique Zobel, Antonio Roxas, Emeterio Bunao, Walthrode B.
Conde, Miguel Ortigas, Antonio Prieto and San Miguel Corporation", was
JOHN GOKONGWEI, JR., petitioner, docketed as SEC Case No. 1375.
vs.
SECURITIES AND EXCHANGE COMMISSION, ANDRES M. SORIANO, As a first cause of action, petitioner alleged that on September 18, 1976,
JOSE M. SORIANO, ENRIQUE ZOBEL, ANTONIO ROXAS, EMETERIO individual respondents amended by bylaws of the corporation, basing their
BUNAO, WALTHRODE B. CONDE, MIGUEL ORTIGAS, ANTONIO authority to do so on a resolution of the stockholders adopted on March 13,
PRIETO, SAN MIGUEL CORPORATION, EMIGDIO TANJUATCO, SR., and 1961, when the outstanding capital stock of respondent corporation was only
EDUARDO R. VISAYA, respondents. P70,139.740.00, divided into 5,513,974 common shares at P10.00 per share
and 150,000 preferred shares at P100.00 per share. At the time of the
De Santos, Balgos & Perez for petitioner. amendment, the outstanding and paid up shares totalled 30,127,047 with a
total par value of P301,270,430.00. It was contended that according to
Angara, Abello, Concepcion, Regala, Cruz Law Offices for respondents section 22 of the Corporation Law and Article VIII of the by-laws of the
Sorianos corporation, the power to amend, modify, repeal or adopt new by-laws may
be delegated to the Board of Directors only by the affirmative vote of
Siguion Reyna, Montecillo & Ongsiako for respondent San Miguel stockholders representing not less than 2/3 of the subscribed and paid up
Corporation. capital stock of the corporation, which 2/3 should have been computed on
the basis of the capitalization at the time of the amendment. Since the
R. T Capulong for respondent Eduardo R. Visaya. amendment was based on the 1961 authorization, petitioner contended that
the Board acted without authority and in usurpation of the power of the
stockholders.

786
As a second cause of action, it was alleged that the authority granted in 1961 On October 28, 1976, in connection with the same case, petitioner filed with
had already been exercised in 1962 and 1963, after which the authority of the Securities and Exchange Commission an "Urgent Motion for Production
the Board ceased to exist. and Inspection of Documents", alleging that the Secretary of respondent
corporation refused to allow him to inspect its records despite request made
As a third cause of action, petitioner averred that the membership of the by petitioner for production of certain documents enumerated in the request,
Board of Directors had changed since the authority was given in 1961, there and that respondent corporation had been attempting to suppress
being six (6) new directors. information from its stockholders despite a negative reply by the SEC to its
query regarding their authority to do so. Among the documents requested to
As a fourth cause of action, it was claimed that prior to the questioned be copied were (a) minutes of the stockholder's meeting field on March 13,
amendment, petitioner had all the qualifications to be a director of 1961, (b) copy of the management contract between San Miguel Corporation
respondent corporation, being a Substantial stockholder thereof; that as a and A. Soriano Corporation (ANSCOR); (c) latest balance sheet of San
stockholder, petitioner had acquired rights inherent in stock ownership, such Miguel International, Inc.; (d) authority of the stockholders to invest the funds
as the rights to vote and to be voted upon in the election of directors; and of respondent corporation in San Miguel International, Inc.; and (e) lists of
that in amending the by-laws, respondents purposely provided for petitioner's salaries, allowances, bonuses, and other compensation, if any, received by
disqualification and deprived him of his vested right as afore-mentioned Andres M. Soriano, Jr. and/or its successor-in-interest.
hence the amended by-laws are null and void. 1
The "Urgent Motion for Production and Inspection of Documents" was
As additional causes of action, it was alleged that corporations have no opposed by respondents, alleging, among others that the motion has no legal
inherent power to disqualify a stockholder from being elected as a director basis; that the demand is not based on good faith; that the motion is
and, therefore, the questioned act is ultra vires and void; that Andres M. premature since the materiality or relevance of the evidence sought cannot
Soriano, Jr. and/or Jose M. Soriano, while representing other corporations, be determined until the issues are joined, that it fails to show good cause and
entered into contracts (specifically a management contract) with respondent constitutes continued harrasment, and that some of the information sought
corporation, which was allowed because the questioned amendment gave are not part of the records of the corporation and, therefore, privileged.
the Board itself the prerogative of determining whether they or other persons
are engaged in competitive or antagonistic business; that the portion of the During the pendency of the motion for production, respondents San Miguel
amended bylaws which states that in determining whether or not a person is Corporation, Enrique Conde, Miguel Ortigas and Antonio Prieto filed their
engaged in competitive business, the Board may consider such factors as answer to the petition, denying the substantial allegations therein and stating,
business and family relationship, is unreasonable and oppressive and, by way of affirmative defenses that "the action taken by the Board of
therefore, void; and that the portion of the amended by-laws which requires Directors on September 18, 1976 resulting in the ... amendments is valid and
that "all nominations for election of directors ... shall be submitted in writing to legal because the power to "amend, modify, repeal or adopt new By-laws"
the Board of Directors at least five (5) working days before the date of the delegated to said Board on March 13, 1961 and long prior thereto has never
Annual Meeting" is likewise unreasonable and oppressive. been revoked of SMC"; that contrary to petitioner's claim, "the vote
requirement for a valid delegation of the power to amend, repeal or adopt
It was, therefore, prayed that the amended by-laws be declared null and void new by-laws is determined in relation to the total subscribed capital stock at
and the certificate of filing thereof be cancelled, and that individual the time the delegation of said power is made, not when the Board opts to
respondents be made to pay damages, in specified amounts, to petitioner. exercise said delegated power"; that petitioner has not availed of his intra-
corporate remedy for the nullification of the amendment, which is to secure

787
its repeal by vote of the stockholders representing a majority of the stockholders in his bid to secure a seat in the Board of Directors on the basic
subscribed capital stock at any regular or special meeting, as provided in issue that petitioner was engaged in a competitive business and his securing
Article VIII, section I of the by-laws and section 22 of the Corporation law, a seat would have subjected respondent corporation to grave disadvantages;
hence the, petition is premature; that petitioner is estopped from questioning that "petitioner nevertheless vowed to secure a seat in the Board of Directors
the amendments on the ground of lack of authority of the Board. since he at the next annual meeting; that thereafter the Board of Directors amended
failed, to object to other amendments made on the basis of the same 1961 the by-laws as afore-stated.
authorization: that the power of the corporation to amend its by-laws is broad,
subject only to the condition that the by-laws adopted should not be As counterclaims, actual damages, moral damages, exemplary damages,
respondent corporation inconsistent with any existing law; that respondent expenses of litigation and attorney's fees were presented against petitioner.
corporation should not be precluded from adopting protective measures to
minimize or eliminate situations where its directors might be tempted to put Subsequently, a Joint Omnibus Motion for the striking out of the motion for
their personal interests over t I hat of the corporation; that the questioned production and inspection of documents was filed by all the respondents.
amended by-laws is a matter of internal policy and the judgment of the board This was duly opposed by petitioner. At this juncture, respondents Emigdio
should not be interfered with: That the by-laws, as amended, are valid and Tanjuatco, Sr. and Eduardo R. Visaya were allowed to intervene as
binding and are intended to prevent the possibility of violation of criminal and oppositors and they accordingly filed their oppositions-intervention to the
civil laws prohibiting combinations in restraint of trade; and that the petition petition.
states no cause of action. It was, therefore, prayed that the petition be
dismissed and that petitioner be ordered to pay damages and attorney's fees
On December 29, 1976, the Securities and Exchange Commission resolved
to respondents. The application for writ of preliminary injunction was likewise
the motion for production and inspection of documents by issuing Order No.
on various grounds.
26, Series of 1977, stating, in part as follows:

Respondents Andres M. Soriano, Jr. and Jose M. Soriano filed their


Considering the evidence submitted before the Commission
opposition to the petition, denying the material averments thereof and stating,
by the petitioner and respondents in the above-entitled case,
as part of their affirmative defenses, that in August 1972, the Universal
it is hereby ordered:
Robina Corporation (Robina), a corporation engaged in business competitive
to that of respondent corporation, began acquiring shares therein. until
September 1976 when its total holding amounted to 622,987 shares: that in 1. That respondents produce and permit the inspection,
October 1972, the Consolidated Foods Corporation (CFC) likewise began copying and photographing, by or on behalf of the petitioner-
acquiring shares in respondent (corporation. until its total holdings amounted movant, John Gokongwei, Jr., of the minutes of the
to P543,959.00 in September 1976; that on January 12, 1976, petitioner, who stockholders' meeting of the respondent San Miguel
is president and controlling shareholder of Robina and CFC (both closed Corporation held on March 13, 1961, which are in the
corporations) purchased 5,000 shares of stock of respondent corporation, possession, custody and control of the said corporation, it
and thereafter, in behalf of himself, CFC and Robina, "conducted malevolent appearing that the same is material and relevant to the
and malicious publicity campaign against SMC" to generate support from the issues involved in the main case. Accordingly, the
stockholder "in his effort to secure for himself and in representation of Robina respondents should allow petitioner-movant entry in the
and CFC interests, a seat in the Board of Directors of SMC", that in the principal office of the respondent Corporation, San Miguel
stockholders' meeting of March 18, 1976, petitioner was rejected by the Corporation on January 14, 1977, at 9:30 o'clock in the

788
morning for purposes of enforcing the rights herein granted; Meanwhile, on December 10, 1976, while the petition was yet to be heard,
it being understood that the inspection, copying and respondent corporation issued a notice of special stockholders' meeting for
photographing of the said documents shall be undertaken the purpose of "ratification and confirmation of the amendment to the By-
under the direct and strict supervision of this Commission. laws", setting such meeting for February 10, 1977. This prompted petitioner
Provided, however, that other documents and/or papers not to ask respondent Commission for a summary judgment insofar as the first
heretofore included are not covered by this Order and any cause of action is concerned, for the alleged reason that by calling a special
inspection thereof shall require the prior permission of this stockholders' meeting for the aforesaid purpose, private respondents
Commission; admitted the invalidity of the amendments of September 18, 1976. The
motion for summary judgment was opposed by private respondents. Pending
2. As to the Balance Sheet of San Miguel International, Inc. action on the motion, petitioner filed an "Urgent Motion for the Issuance of a
as well as the list of salaries, allowances, bonuses, Temporary Restraining Order", praying that pending the determination of
compensation and/or remuneration received by respondent petitioner's application for the issuance of a preliminary injunction and/or
Jose M. Soriano, Jr. and Andres Soriano from San Miguel petitioner's motion for summary judgment, a temporary restraining order be
International, Inc. and/or its successors-in- interest, the issued, restraining respondents from holding the special stockholder's
Petition to produce and inspect the same is hereby DENIED, meeting as scheduled. This motion was duly opposed by respondents.
as petitioner-movant is not a stockholder of San Miguel
International, Inc. and has, therefore, no inherent right to On February 10, 1977, respondent Commission issued an order denying the
inspect said documents; motion for issuance of temporary restraining order. After receipt of the order
of denial, respondents conducted the special stockholders' meeting wherein
3. In view of the Manifestation of petitioner-movant dated the amendments to the by-laws were ratified. On February 14, 1977,
November 29, 1976, withdrawing his request to copy and petitioner filed a consolidated motion for contempt and for nullification of the
inspect the management contract between San Miguel special stockholders' meeting.
Corporation and A. Soriano Corporation and the renewal and
amendments thereof for the reason that he had already A motion for reconsideration of the order denying petitioner's motion for
obtained the same, the Commission takes note thereof; and summary judgment was filed by petitioner before respondent Commission on
March 10, 1977. Petitioner alleges that up to the time of the filing of the
4. Finally, the Commission holds in abeyance the resolution instant petition, the said motion had not yet been scheduled for hearing.
on the matter of production and inspection of the authority of Likewise, the motion for reconsideration of the order granting in part and
the stockholders of San Miguel Corporation to invest the denying in part petitioner's motion for production of record had not yet been
funds of respondent corporation in San Miguel International, resolved.
Inc., until after the hearing on the merits of the principal
issues in the above-entitled case. In view of the fact that the annul stockholders' meeting of respondent
corporation had been scheduled for May 10, 1977, petitioner filed with
This Order is immediately executory upon its approval. 2 respondent Commission a Manifestation stating that he intended to run for
the position of director of respondent corporation. Thereafter, respondents
Dissatisfied with the foregoing Order, petitioner moved for its reconsideration. filed a Manifestation with respondent Commission, submitting a Resolution of
the Board of Directors of respondent corporation disqualifying and precluding

789
petitioner from being a candidate for director unless he could submit 6. Re-affirmation of the authorization to the Board of
evidence on May 3, 1977 that he does not come within the disqualifications Directors by the stockholders at the meeting on March 20,
specified in the amendment to the by-laws, subject matter of SEC Case No. 1972 to invest corporate funds in other companies or
1375. By reason thereof, petitioner filed a manifestation and motion to businesses or for purposes other than the main purpose for
resolve pending incidents in the case and to issue a writ of injunction, which the Corporation has been organized, and ratification of
alleging that private respondents were seeking to nullify and render the investments thereafter made pursuant thereto.
ineffectual the exercise of jurisdiction by the respondent Commission, to
petitioner's irreparable damage and prejudice, Allegedly despite a By reason of the foregoing, on April 28, 1977, petitioner filed with the SEC an
subsequent Manifestation to prod respondent Commission to act, petitioner urgent motion for the issuance of a writ of preliminary injunction to restrain
was not heard prior to the date of the stockholders' meeting. private respondents from taking up Item 6 of the Agenda at the annual
stockholders' meeting, requesting that the same be set for hearing on May 3,
Petitioner alleges that there appears a deliberate and concerted inability on 1977, the date set for the second hearing of the case on the merits.
the part of the SEC to act hence petitioner came to this Court. Respondent Commission, however, cancelled the dates of hearing originally
scheduled and reset the same to May 16 and 17, 1977, or after the
SEC. CASE NO. 1423 scheduled annual stockholders' meeting. For the purpose of urging the
Commission to act, petitioner filed an urgent manifestation on May 3, 1977,
Petitioner likewise alleges that, having discovered that respondent but this notwithstanding, no action has been taken up to the date of the filing
corporation has been investing corporate funds in other corporations and of the instant petition.
businesses outside of the primary purpose clause of the corporation, in
violation of section 17 1/2 of the Corporation Law, he filed with respondent With respect to the afore-mentioned SEC cases, it is petitioner's contention
Commission, on January 20, 1977, a petition seeking to have private before this Court that respondent Commission gravely abused its discretion
respondents Andres M. Soriano, Jr. and Jose M. Soriano, as well as the when it failed to act with deliberate dispatch on the motions of petitioner
respondent corporation declared guilty of such violation, and ordered to seeking to prevent illegal and/or arbitrary impositions or limitations upon his
account for such investments and to answer for damages. rights as stockholder of respondent corporation, and that respondent are
acting oppressively against petitioner, in gross derogation of petitioner's
On February 4, 1977, motions to dismiss were filed by private respondents, rights to property and due process. He prayed that this Court direct
to which a consolidated motion to strike and to declare individual respondent SEC to act on collateral incidents pending before it.
respondents in default and an opposition ad abundantiorem cautelam were
filed by petitioner. Despite the fact that said motions were filed as early as On May 6, 1977, this Court issued a temporary restraining order restraining
February 4, 1977, the commission acted thereon only on April 25, 1977, private respondents from disqualifying or preventing petitioner from running
when it denied respondents' motion to dismiss and gave them two (2) days or from being voted as director of respondent corporation and from
within which to file their answer, and set the case for hearing on April 29 and submitting for ratification or confirmation or from causing the ratification or
May 3, 1977. confirmation of Item 6 of the Agenda of the annual stockholders' meeting on
May 10, 1977, or from Making effective the amended by-laws of respondent
Respondents issued notices of the annual stockholders' meeting, including in corporation, until further orders from this Court or until the Securities and Ex-
the Agenda thereof, the following: change Commission acts on the matters complained of in the instant petition.

790
On May 14, 1977, petitioner filed a Supplemental Petition, alleging that after International. Inc. and thereafter to decide SEC Cases No. 1375 and 1423 on
a restraining order had been issued by this Court, or on May 9, 1977, the the merits.
respondent Commission served upon petitioner copies of the following
orders: On May 17, 1977, respondent SEC, Andres M. Soriano, Jr. and Jose M.
Soriano filed their comment, alleging that the petition is without merit for the
(1) Order No. 449, Series of 1977 (SEC Case No. 1375); denying petitioner's following reasons:
motion for reconsideration, with its supplement, of the order of the
Commission denying in part petitioner's motion for production of documents, (1) that the petitioner the interest he represents are engaged in business
petitioner's motion for reconsideration of the order denying the issuance of a competitive and antagonistic to that of respondent San Miguel Corporation, it
temporary restraining order denying the issuance of a temporary restraining appearing that the owns and controls a greater portion of his SMC stock thru
order, and petitioner's consolidated motion to declare respondents in the Universal Robina Corporation and the Consolidated Foods Corporation,
contempt and to nullify the stockholders' meeting; which corporations are engaged in business directly and substantially
competing with the allied businesses of respondent SMC and of corporations
(2) Order No. 450, Series of 1977 (SEC Case No. 1375), allowing petitioner in which SMC has substantial investments. Further, when CFC and Robina
to run as a director of respondent corporation but stating that he should not had accumulated investments. Further, when CFC and Robina had
sit as such if elected, until such time that the Commission has decided the accumulated shares in SMC, the Board of Directors of SMC realized the
validity of the bylaws in dispute, and denying deferment of Item 6 of the clear and present danger that competitors or antagonistic parties may be
Agenda for the annual stockholders' meeting; and elected directors and thereby have easy and direct access to SMC's
business and trade secrets and plans;
(3) Order No. 451, Series of 1977 (SEC Case No. 1375), denying petitioner's
motion for reconsideration of the order of respondent Commission denying (2) that the amended by law were adopted to preserve and protect
petitioner's motion for summary judgment; respondent SMC from the clear and present danger that business
competitors, if allowed to become directors, will illegally and unfairly utilize
It is petitioner's assertions, anent the foregoing orders, (1) that respondent their direct access to its business secrets and plans for their own private gain
Commission acted with indecent haste and without circumspection in issuing to the irreparable prejudice of respondent SMC, and, ultimately, its
the aforesaid orders to petitioner's irreparable damage and injury; (2) that it stockholders. Further, it is asserted that membership of a competitor in the
acted without jurisdiction and in violation of petitioner's right to due process Board of Directors is a blatant disregard of no less that the Constitution and
when it decided en banc an issue not raised before it and still pending before pertinent laws against combinations in restraint of trade;
one of its Commissioners, and without hearing petitioner thereon despite
petitioner's request to have the same calendared for hearing , and (3) that (3) that by laws are valid and binding since a corporation has the inherent
the respondents acted oppressively against the petitioner in violation of his right and duty to preserve and protect itself by excluding competitors and
rights as a stockholder, warranting immediate judicial intervention. antogonistic parties, under the law of self-preservation, and it should be
allowed a wide latitude in the selection of means to preserve itself;
It is prayed in the supplemental petition that the SEC orders complained of
be declared null and void and that respondent Commission be ordered to (4) that the delay in the resolution and disposition of SEC Cases Nos. 1375
allow petitioner to undertake discovery proceedings relative to San Miguel and 1423 was due to petitioner's own acts or omissions, since he failed to

791
have the petition to suspend, pendente lite the amended by-laws calendared Respondent Commission, thru the Solicitor General, filed a separate
for hearing. It was emphasized that it was only on April 29, 1977 that comment, alleging that after receiving a copy of the restraining order issued
petitioner calendared the aforesaid petition for suspension (preliminary by this Court and noting that the restraining order did not foreclose action by
injunction) for hearing on May 3, 1977. The instant petition being dated May it, the Commission en banc issued Orders Nos. 449, 450 and 451 in SEC
4, 1977, it is apparent that respondent Commission was not given a chance Case No. 1375.
to act "with deliberate dispatch", and
In answer to the allegation in the supplemental petition, it states that Order
(5) that, even assuming that the petition was meritorious was, it has become No. 450 which denied deferment of Item 6 of the Agenda of the annual
moot and academic because respondent Commission has acted on the stockholders' meeting of respondent corporation, took into consideration an
pending incidents, complained of. It was, therefore, prayed that the petition urgent manifestation filed with the Commission by petitioner on May 3, 1977
be dismissed. which prayed, among others, that the discussion of Item 6 of the Agenda be
deferred. The reason given for denial of deferment was that "such action is
On May 21, 1977, respondent Emigdio G, Tanjuatco, Sr. filed his comment, within the authority of the corporation as well as falling within the sphere of
alleging that the petition has become moot and academic for the reason, stockholders' right to know, deliberate upon and/or to express their wishes
among others that the acts of private respondent sought to be enjoined have regarding disposition of corporate funds considering that their investments
reference to the annual meeting of the stockholders of respondent San are the ones directly affected." It was alleged that the main petition has,
Miguel Corporation, which was held on may 10, 1977; that in said meeting, in therefore, become moot and academic.
compliance with the order of respondent Commission, petitioner was allowed
to run and be voted for as director; and that in the same meeting, Item 6 of On September 29,1977, petitioner filed a second supplemental petition with
the Agenda was discussed, voted upon, ratified and confirmed. Further it was prayer for preliminary injunction, alleging that the actuations of respondent
averred that the questions and issues raised by petitioner are pending in the SEC tended to deprive him of his right to due process, and "that all possible
Securities and Exchange Commission which has acquired jurisdiction over questions on the facts now pending before the respondent Commission are
the case, and no hearing on the merits has been had; hence the elevation of now before this Honorable Court which has the authority and the
these issues before the Supreme Court is premature. competence to act on them as it may see fit." (Reno, pp. 927-928.)

Petitioner filed a reply to the aforesaid comments, stating that the petition Petitioner, in his memorandum, submits the following issues for resolution;
presents justiciable questions for the determination of this Court because (1)
the respondent Commission acted without circumspection, unfairly and (1) whether or not the provisions of the amended by-laws of respondent
oppresively against petitioner, warranting the intervention of this Court; (2) a corporation, disqualifying a competitor from nomination or election to the
derivative suit, such as the instant case, is not rendered academic by the act Board of Directors are valid and reasonable;
of a majority of stockholders, such that the discussion, ratification and
confirmation of Item 6 of the Agenda of the annual stockholders' meeting of (2) whether or not respondent SEC gravely abused its discretion in denying
May 10, 1977 did not render the case moot; that the amendment to the petitioner's request for an examination of the records of San Miguel
bylaws which specifically bars petitioner from being a director is void since it International, Inc., a fully owned subsidiary of San Miguel Corporation; and
deprives him of his vested rights.

792
(3) whether or not respondent SEC committed grave abuse of discretion in Respondents Andres M. Soriano, Jr. and Jose M. Soriano similarly pray that
allowing discussion of Item 6 of the Agenda of the Annual Stockholders' this Court resolve the legal issues raised by the parties in keeping with the
Meeting on May 10, 1977, and the ratification of the investment in a foreign "cherished rules of procedure" that "a court should always strive to settle the
corporation of the corporate funds, allegedly in violation of section 17-1/2 of entire controversy in a single proceeding leaving no root or branch to bear
the Corporation Law. the seeds of future ligiation", citing Gayong v. Gayos. 3 To the same effect is
the prayer of San Miguel Corporation that this Court resolve on the merits the
I validity of its amended by laws and the rights and obligations of the parties
thereunder, otherwise "the time spent and effort exerted by the parties
Whether or not amended by-laws are valid is purely a legal question which concerned and, more importantly, by this Honorable Court, would have been
public interest requires to be resolved for naught because the main question will come back to this Honorable Court
for final resolution." Respondent Eduardo R. Visaya submits a similar appeal.
It is the position of the petitioner that "it is not necessary to remand the case
to respondent SEC for an appropriate ruling on the intrinsic validity of the It is only the Solicitor General who contends that the case should be
amended by-laws in compliance with the principle of exhaustion of remanded to the SEC for hearing and decision of the issues involved,
administrative remedies", considering that: first: "whether or not the invoking the latter's primary jurisdiction to hear and decide case involving
provisions of the amended by-laws are intrinsically valid ... is purely a legal intra-corporate controversies.
question. There is no factual dispute as to what the provisions are and
evidence is not necessary to determine whether such amended by-laws are It is an accepted rule of procedure that the Supreme Court should always
valid as framed and approved ... "; second: "it is for the interest and guidance strive to settle the entire controversy in a single proceeding, leaving nor root
of the public that an immediate and final ruling on the question be made ... "; or branch to bear the seeds of future litigation. 4 Thus, in Francisco v. City of
third: "petitioner was denied due process by SEC" when "Commissioner de Davao, 5 this Court resolved to decide the case on the merits instead of
Guzman had openly shown prejudice against petitioner ... ", and remanding it to the trial court for further proceedings since the ends of justice
"Commissioner Sulit ... approved the amended by-laws ex-parte and would not be subserved by the remand of the case. In Republic v. Security
obviously found the same intrinsically valid; and finally: "to remand the case Credit and Acceptance Corporation, et al., 6 this Court, finding that the main
to SEC would only entail delay rather than serve the ends of justice." issue is one of law, resolved to decide the case on the merits "because public
interest demands an early disposition of the case", and in Republic v. Central
Surety and Insurance Company, 7 this Court denied remand of the third-party
complaint to the trial court for further proceedings, citing precedent where
this Court, in similar situations resolved to decide the cases on the merits,
instead of remanding them to the trial court where (a) the ends of justice
would not be subserved by the remand of the case; or (b) where public
interest demand an early disposition of the case; or (c) where the trial court
had already received all the evidence presented by both parties and the
Supreme Court is now in a position, based upon said evidence, to decide the
case on its merits. 8 It is settled that the doctrine of primary jurisdiction has no
application where only a question of law is involved. 8a Because uniformity
may be secured through review by a single Supreme Court, questions of law

793
may appropriately be determined in the first instance by courts. 8b In the Upon the other hand, respondents Andres M. Soriano, Jr., Jose M. Soriano
case at bar, there are facts which cannot be denied, viz.: that the amended and San Miguel Corporation content that ex. conclusion of a competitor from
by-laws were adopted by the Board of Directors of the San Miguel the Board is legitimate corporate purpose, considering that being a
Corporation in the exercise of the power delegated by the stockholders competitor, petitioner cannot devote an unselfish and undivided Loyalty to the
ostensibly pursuant to section 22 of the Corporation Law; that in a special corporation; that it is essentially a preventive measure to assure stockholders
meeting on February 10, 1977 held specially for that purpose, the amended of San Miguel Corporation of reasonable protective from the unrestrained
by-laws were ratified by more than 80% of the stockholders of record; that self-interest of those charged with the promotion of the corporate enterprise;
the foreign investment in the Hongkong Brewery and Distellery, a beer that access to confidential information by a competitor may result either in
manufacturing company in Hongkong, was made by the San Miguel the promotion of the interest of the competitor at the expense of the San
Corporation in 1948; and that in the stockholders' annual meeting held in Miguel Corporation, or the promotion of both the interests of petitioner and
1972 and 1977, all foreign investments and operations of San Miguel respondent San Miguel Corporation, which may, therefore, result in a
Corporation were ratified by the stockholders. combination or agreement in violation of Article 186 of the Revised Penal
Code by destroying free competition to the detriment of the consuming
II public. It is further argued that there is not vested right of any stockholder
under Philippine Law to be voted as director of a corporation. It is alleged
Whether or not the amended by-laws of SMC of disqualifying a competitor that petitioner, as of May 6, 1978, has exercised, personally or thru two
from nomination or election to the Board of Directors of SMC are valid and corporations owned or controlled by him, control over the following
reasonable shareholdings in San Miguel Corporation, vis.: (a) John Gokongwei, Jr.
6,325 shares; (b) Universal Robina Corporation 738,647 shares; (c) CFC
Corporation 658,313 shares, or a total of 1,403,285 shares. Since the
The validity or reasonableness of a by-law of a corporation in purely a
outstanding capital stock of San Miguel Corporation, as of the present date,
question of law. 9 Whether the by-law is in conflict with the law of the land, or
is represented by 33,139,749 shares with a par value of P10.00, the total
with the charter of the corporation, or is in a legal sense unreasonable and
shares owned or controlled by petitioner represents 4.2344% of the total
therefore unlawful is a question of law. 10 This rule is subject, however, to the
outstanding capital stock of San Miguel Corporation. It is also contended that
limitation that where the reasonableness of a by-law is a mere matter of
petitioner is the president and substantial stockholder of Universal Robina
judgment, and one upon which reasonable minds must necessarily differ, a
Corporation and CFC Corporation, both of which are allegedly controlled by
court would not be warranted in substituting its judgment instead of the
petitioner and members of his family. It is also claimed that both the
judgment of those who are authorized to make by-laws and who have
Universal Robina Corporation and the CFC Corporation are engaged in
exercised their authority. 11
businesses directly and substantially competing with the alleged businesses
of San Miguel Corporation, and of corporations in which SMC has substantial
Petitioner claims that the amended by-laws are invalid and unreasonable investments.
because they were tailored to suppress the minority and prevent them from
having representation in the Board", at the same time depriving petitioner of
ALLEGED AREAS OF COMPETITION BETWEEN PETITIONER'S
his "vested right" to be voted for and to vote for a person of his choice as
CORPORATIONS AND SAN MIGUEL CORPORATION
director.

794
According to respondent San Miguel Corporation, the areas of, competition 23,436,754 shares in SMC, or more than 90% of the total outstanding shares
are enumerated in its Board the areas of competition are enumerated in its of SMC, rejected petitioner's candidacy for the Board of Directors because
Board Resolution dated April 28, 1978, thus: they "realized the grave dangers to the corporation in the event a competitor
gets a board seat in SMC." On September 18, 1978, the Board of Directors
Product Line Estimated Market Share Total of SMC, by "virtue of powers delegated to it by the stockholders," approved
1977 SMC Robina-CFC the amendment to ' he by-laws in question. At the meeting of February 10,
1977, these amendments were confirmed and ratified by 5,716 shareholders
Table Eggs 0.6% 10.0% 10.6% owning 24,283,945 shares, or more than 80% of the total outstanding shares.
Layer Pullets 33.0% 24.0% 57.0% Only 12 shareholders, representing 7,005 shares, opposed the confirmation
Dressed Chicken 35.0% 14.0% 49.0% and ratification. At the Annual Stockholders' Meeting of May 10, 1977, 11,349
Poultry & Hog Feeds 40.0% 12.0% 52.0% shareholders, owning 27,257.014 shares, or more than 90% of the
Ice Cream 70.0% 13.0% 83.0% outstanding shares, rejected petitioner's candidacy, while 946 stockholders,
Instant Coffee 45.0% 40.0% 85.0% representing 1,648,801 shares voted for him. On the May 9, 1978 Annual
Woven Fabrics 17.5% 9.1% 26.6% Stockholders' Meeting, 12,480 shareholders, owning more than 30 million
shares, or more than 90% of the total outstanding shares. voted against
petitioner.
Thus, according to respondent SMC, in 1976, the areas of competition
affecting SMC involved product sales of over P400 million or more than 20%
of the P2 billion total product sales of SMC. Significantly, the combined AUTHORITY OF CORPORATION TO PRESCRIBE QUALIFICATIONS OF
market shares of SMC and CFC-Robina in layer pullets dressed chicken, DIRECTORS EXPRESSLY CONFERRED BY LAW
poultry and hog feeds ice cream, instant coffee and woven fabrics would
result in a position of such dominance as to affect the prevailing market Private respondents contend that the disputed amended by laws were
factors. adopted by the Board of Directors of San Miguel Corporation a-, a measure
of self-defense to protect the corporation from the clear and present danger
It is further asserted that in 1977, the CFC-Robina group was in direct that the election of a business competitor to the Board may cause upon the
competition on product lines which, for SMC, represented sales amounting to corporation and the other stockholders inseparable prejudice. Submitted for
more than ?478 million. In addition, CFC-Robina was directly competing in resolution, therefore, is the issue whether or not respondent San Miguel
the sale of coffee with Filipro, a subsidiary of SMC, which product line Corporation could, as a measure of self- protection, disqualify a competitor
represented sales for SMC amounting to more than P275 million. The CFC- from nomination and election to its Board of Directors.
Robina group (Robitex, excluding Litton Mills recently acquired by petitioner)
is purportedly also in direct competition with Ramie Textile, Inc., subsidiary of It is recognized by an authorities that 'every corporation has the inherent
SMC, in product sales amounting to more than P95 million. The areas of power to adopt by-laws 'for its internal government, and to regulate the
competition between SMC and CFC-Robina in 1977 represented, therefore, conduct and prescribe the rights and duties of its members towards itself and
for SMC, product sales of more than P849 million. among themselves in reference to the management of its affairs. 12 At
common law, the rule was "that the power to make and adopt by-laws
According to private respondents, at the Annual Stockholders' Meeting of was inherent in every corporation as one of its necessary and inseparable
March 18, 1976, 9,894 stockholders, in person or by proxy, owning legal incidents. And it is settled throughout the United States that in the
absence of positive legislative provisions limiting it, every private corporation

795
has this inherent power as one of its necessary and inseparable legal corporation If the amendment changes, diminishes or restricts the rights of
incidents, independent of any specific enabling provision in its charter or in the existing shareholders then the disenting minority has only one right,
general law, such power of self-government being essential to enable the viz.: "to object thereto in writing and demand payment for his share." Under
corporation to accomplish the purposes of its creation. 13 section 22 of the same law, the owners of the majority of the subscribed
capital stock may amend or repeal any by-law or adopt new by-laws. It
In this jurisdiction, under section 21 of the Corporation Law, a corporation cannot be said, therefore, that petitioner has a vested right to be elected
may prescribe in its by-laws "the qualifications, duties and compensation of director, in the face of the fact that the law at the time such right as
directors, officers and employees ... " This must necessarily refer to a stockholder was acquired contained the prescription that the corporate
qualification in addition to that specified by section 30 of the Corporation Law, charter and the by-law shall be subject to amendment, alteration and
which provides that "every director must own in his right at least one share of modification. 17
the capital stock of the stock corporation of which he is a director ... "
In Government v. El Hogar, 14 the Court sustained the validity of a provision in It being settled that the corporation has the power to provide for the
the corporate by-law requiring that persons elected to the Board of Directors qualifications of its directors, the next question that must be considered is
must be holders of shares of the paid up value of P5,000.00, which shall be whether the disqualification of a competitor from being elected to the Board
held as security for their action, on the ground that section 21 of the of Directors is a reasonable exercise of corporate authority.
Corporation Law expressly gives the power to the corporation to provide in its
by-laws for the qualifications of directors and is "highly prudent and in A DIRECTOR STANDS IN A FIDUCIARY RELATION TO THE
conformity with good practice. " CORPORATION AND ITS SHAREHOLDERS

NO VESTED RIGHT OF STOCKHOLDER TO BE ELECTED DIRECTOR Although in the strict and technical sense, directors of a private corporation
are not regarded as trustees, there cannot be any doubt that their character
Any person "who buys stock in a corporation does so with the knowledge that is that of a fiduciary insofar as the corporation and the stockholders as a
its affairs are dominated by a majority of the stockholders and that he body are concerned. As agents entrusted with the management of the
impliedly contracts that the will of the majority shall govern in all matters corporation for the collective benefit of the stockholders, "they occupy a
within the limits of the act of incorporation and lawfully enacted by-laws and fiduciary relation, and in this sense the relation is one of trust." 18 "The
not forbidden by law." 15 To this extent, therefore, the stockholder may be ordinary trust relationship of directors of a corporation and stockholders",
considered to have "parted with his personal right or privilege to regulate the according to Ashaman v. Miller, 19 "is not a matter of statutory or technical
disposition of his property which he has invested in the capital stock of the law. It springs from the fact that directors have the control and guidance of
corporation, and surrendered it to the will of the majority of his fellow corporate affairs and property and hence of the property interests of the
incorporators. ... It cannot therefore be justly said that the contract, express stockholders. Equity recognizes that stockholders are the proprietors of the
or implied, between the corporation and the stockholders is infringed ... by corporate interests and are ultimately the only beneficiaries thereof * * *.
any act of the former which is authorized by a majority ... ." 16
Justice Douglas, in Pepper v. Litton, 20 emphatically restated the standard of
Pursuant to section 18 of the Corporation Law, any corporation may amend fiduciary obligation of the directors of corporations, thus:
its articles of incorporation by a vote or written assent of the stockholders
representing at least two-thirds of the subscribed capital stock of the

796
A director is a fiduciary. ... Their powers are powers in affairs, and his suppose influence over her. It is perhaps true that such
trust. ... He who is in such fiduciary position cannot serve stockholders ought not to be condemned as selfish and dangerous to the
himself first and his cestuis second. ... He cannot manipulate best interest of the corporation until tried and tested. So it is also true that we
the affairs of his corporation to their detriment and in cannot condemn as selfish and dangerous and unreasonable the action of
disregard of the standards of common decency. He cannot the board in passing the by-law. The strife over the matter of control in this
by the intervention of a corporate entity violate the ancient corporation as in many others is perhaps carried on not altogether in the
precept against serving two masters ... He cannot utilize his spirit of brotherly love and affection. The only test that we can apply is as to
inside information and strategic position for his own whether or not the action of the Board is authorized and sanctioned by law. ...
preferment. He cannot violate rules of fair play by doing . 22
indirectly through the corporation what he could not do so
directly. He cannot violate rules of fair play by doing indirectly These principles have been applied by this Court in previous cases. 23
though the corporation what he could not do so directly. He
cannot use his power for his personal advantage and to the AN AMENDMENT TO THE CORPORATION BY-LAW WHICH RENDERS A
detriment of the stockholders and creditors no matter how STOCKHOLDER INELIGIBLE TO BE DIRECTOR, IF HE BE ALSO
absolute in terms that power may be and no matter how DIRECTOR IN A CORPORATION WHOSE BUSINESS IS IN
meticulous he is to satisfy technical requirements. For that COMPETITION WITH THAT OF THE OTHER CORPORATION, HAS BEEN
power is at all times subject to the equitable limitation that it SUSTAINED AS VALID
may not be exercised for the aggrandizement, preference or
advantage of the fiduciary to the exclusion or detriment of
It is a settled state law in the United States, according to Fletcher, that
the cestuis.
corporations have the power to make by-laws declaring a person employed
21
in the service of a rival company to be ineligible for the corporation's Board of
And in Cross v. West Virginia Cent, & P. R. R. Co., it was said: Directors. ... (A)n amendment which renders ineligible, or if elected, subjects
to removal, a director if he be also a director in a corporation whose business
... A person cannot serve two hostile and adverse master, is in competition with or is antagonistic to the other corporation is
without detriment to one of them. A judge cannot be impartial valid." 24 This is based upon the principle that where the director is so
if personally interested in the cause. No more can a director. employed in the service of a rival company, he cannot serve both, but must
Human nature is too weak -for this. Take whatever statute betray one or the other. Such an amendment "advances the benefit of the
provision you please giving power to stockholders to choose corporation and is good." An exception exists in New Jersey, where the
directors, and in none will you find any express prohibition Supreme Court held that the Corporation Law in New Jersey prescribed the
against a discretion to select directors having the company's only qualification, and therefore the corporation was not empowered to add
interest at heart, and it would simply be going far to deny by additional qualifications. 25 This is the exact opposite of the situation in the
mere implication the existence of such a salutary power Philippines because as stated heretofore, section 21 of the Corporation Law
expressly provides that a corporation may make by-laws for the qualifications
... If the by-law is to be held reasonable in disqualifying a stockholder in a of directors. Thus, it has been held that an officer of a corporation cannot
competing company from being a director, the same reasoning would apply engage in a business in direct competition with that of the corporation where
to disqualify the wife and immediate member of the family of such he is a director by utilizing information he has received as such officer, under
stockholder, on account of the supposed interest of the wife in her husband's "the established law that a director or officer of a corporation may not enter

797
into a competing enterprise which cripples or injures the business of the It is obviously to prevent the creation of an opportunity for an officer or
corporation of which he is an officer or director. 26 director of San Miguel Corporation, who is also the officer or owner of a
competing corporation, from taking advantage of the information which he
It is also well established that corporate officers "are not permitted to use acquires as director to promote his individual or corporate interests to the
their position of trust and confidence to further their private interests." 27 In a prejudice of San Miguel Corporation and its stockholders, that the questioned
case where directors of a corporation cancelled a contract of the corporation amendment of the by-laws was made. Certainly, where two corporations are
for exclusive sale of a foreign firm's products, and after establishing a rival competitive in a substantial sense, it would seem improbable, if not
business, the directors entered into a new contract themselves with the impossible, for the director, if he were to discharge effectively his duty, to
foreign firm for exclusive sale of its products, the court held that equity would satisfy his loyalty to both corporations and place the performance of his
regard the new contract as an offshoot of the old contract and, therefore, for corporation duties above his personal concerns.
the benefit of the corporation, as a "faultless fiduciary may not reap the fruits
of his misconduct to the exclusion of his principal. 28 Thus, in McKee & Co. v. First National Bank of San Diego, supra the court
sustained as valid and reasonable an amendment to the by-laws of a bank,
The doctrine of "corporate opportunity" 29 is precisely a recognition by the requiring that its directors should not be directors, officers, employees,
courts that the fiduciary standards could not be upheld where the fiduciary agents, nominees or attorneys of any other banking corporation, affiliate or
was acting for two entities with competing interests. This doctrine rests subsidiary thereof. Chief Judge Parker, in McKee, explained the reasons of
fundamentally on the unfairness, in particular circumstances, of an officer or the court, thus:
director taking advantage of an opportunity for his own personal profit when
the interest of the corporation justly calls for protection. 30 ... A bank director has access to a great deal of information
concerning the business and plans of a bank which would
It is not denied that a member of the Board of Directors of the San Miguel likely be injurious to the bank if known to another bank, and
Corporation has access to sensitive and highly confidential information, such it was reasonable and prudent to enlarge this minimum
as: (a) marketing strategies and pricing structure; (b) budget for expansion disqualification to include any director, officer, employee,
and diversification; (c) research and development; and (d) sources of agent, nominee, or attorney of any other bank in California.
funding, availability of personnel, proposals of mergers or tie-ups with other The Ashkins case, supra, specifically recognizes protection
firms. against rivals and others who might acquire information
which might be used against the interests of the corporation
as a legitimate object of by-law protection. With respect to
attorneys or persons associated with a firm which is attorney
for another bank, in addition to the direct conflict or potential
conflict of interest, there is also the danger of inadvertent
leakage of confidential information through casual office
discussions or accessibility of files. Defendant's directors
determined that its welfare was best protected if this
opportunity for conflicting loyalties and potential misuse and
leakage of confidential information was foreclosed.

798
In McKee the Court further listed qualificational by-laws upheld by the courts, the passive attitude of directors ... without active and conscientious
as follows: participation in the managerial functions of the company. As directors, it is
their duty to control and supervise the day to day business activities of the
(1) A director shall not be directly or indirectly interested as a company or to promulgate definite policies and rules of guidance with a
stockholder in any other firm, company, or association which vigilant eye toward seeing to it that these policies are carried out. It is only
competes with the subject corporation. then that directors may be said to have fulfilled their duty of fealty to the
corporation."
(2) A director shall not be the immediate member of the
family of any stockholder in any other firm, company, or Sound principles of corporate management counsel against sharing sensitive
association which competes with the subject corporation, information with a director whose fiduciary duty of loyalty may well require
that he disclose this information to a competitive arrival. These dangers are
(3) A director shall not be an officer, agent, employee, enhanced considerably where the common director such as the petitioner is
attorney, or trustee in any other firm, company, or a controlling stockholder of two of the competing corporations. It would seem
association which compete with the subject corporation. manifest that in such situations, the director has an economic incentive to
appropriate for the benefit of his own corporation the corporate plans and
policies of the corporation where he sits as director.
(4) A director shall be of good moral character as an
essential qualification to holding office.
Indeed, access by a competitor to confidential information regarding
marketing strategies and pricing policies of San Miguel Corporation would
(5) No person who is an attorney against the corporation in a
subject the latter to a competitive disadvantage and unjustly enrich the
law suit is eligible for service on the board. (At p. 7.)
competitor, for advance knowledge by the competitor of the strategies for the
development of existing or new markets of existing or new products could
These are not based on theorical abstractions but on human experience enable said competitor to utilize such knowledge to his advantage. 32
that a person cannot serve two hostile masters without detriment to one of
them.
There is another important consideration in determining whether or not the
amended by-laws are reasonable. The Constitution and the law prohibit
The offer and assurance of petitioner that to avoid any possibility of his taking combinations in restraint of trade or unfair competition. Thus, section 2 of
unfair advantage of his position as director of San Miguel Corporation, he Article XIV of the Constitution provides: "The State shall regulate or prohibit
would absent himself from meetings at which confidential matters would be private monopolies when the public interest so requires. No combinations in
discussed, would not detract from the validity and reasonableness of the by- restraint of trade or unfair competition shall be snowed."
laws here involved. Apart from the impractical results that would ensue from
such arrangement, it would be inconsistent with petitioner's primary motive in
Article 186 of the Revised Penal Code also provides:
running for board membership which is to protect his investments in San
Miguel Corporation. More important, such a proposed norm of conduct would
be against all accepted principles underlying a director's duty of fidelity to the Art. 186. Monopolies and combinations in restraint of trade.
corporation, for the policy of the law is to encourage and enforce responsible The penalty of prision correccional in its minimum period
corporate management. As explained by Oleck: 31 "The law win not tolerate

799
or a fine ranging from two hundred to six thousand pesos, or Basically, these anti-trust laws or laws against monopolies or combinations in
both, shall be imposed upon: restraint of trade are aimed at raising levels of competition by improving the
consumers' effectiveness as the final arbiter in free markets. These laws are
1. Any person who shall enter into any contract or agreement designed to preserve free and unfettered competition as the rule of trade. "It
or shall take part in any conspiracy or combination in the rests on the premise that the unrestrained interaction of competitive forces
form of a trust or otherwise, in restraint of trade or commerce will yield the best allocation of our economic resources, the lowest prices and
or to prevent by artificial means free competition in the the highest quality ... ." 34 they operate to forestall concentration of economic
market. power. 35 The law against monopolies and combinations in restraint of trade
is aimed at contracts and combinations that, by reason of the inherent nature
2. Any person who shag monopolize any merchandise or of the contemplated acts, prejudice the public interest by unduly restraining
object of trade or commerce, or shall combine with any other competition or unduly obstructing the course of trade. 36
person or persons to monopolize said merchandise or object
in order to alter the price thereof by spreading false rumors The terms "monopoly", "combination in restraint of trade" and "unfair
or making use of any other artifice to restrain free competition" appear to have a well defined meaning in other jurisdictions. A
competition in the market. "monopoly" embraces any combination the tendency of which is to prevent
competition in the broad and general sense, or to control prices to the
3. Any person who, being a manufacturer, producer, or detriment of the public. 37 In short, it is the concentration of business in the
processor of any merchandise or object of commerce or an hands of a few. The material consideration in determining its existence is not
importer of any merchandise or object of commerce from that prices are raised and competition actually excluded, but
any foreign country, either as principal or agent, wholesale or that power exists to raise prices or exclude competition when
retailer, shall combine, conspire or agree in any manner with desired. 38Further, it must be considered that the Idea of monopoly is now
any person likewise engaged in the manufacture, production, understood to include a condition produced by the mere act of individuals. Its
processing, assembling or importation of such merchandise dominant thought is the notion of exclusiveness or unity, or the suppression
or object of commerce or with any other persons not so of competition by the qualification of interest or management, or it may be
similarly engaged for the purpose of making transactions thru agreement and concert of action. It is, in brief, unified tactics with regard
prejudicial to lawful commerce, or of increasing the market to prices. 39
price in any part of the Philippines, or any such merchandise
or object of commerce manufactured, produced, processed, From the foregoing definitions, it is apparent that the contentions of petitioner
assembled in or imported into the Philippines, or of any are not in accord with reality. The election of petitioner to the Board of
article in the manufacture of which such manufactured, respondent Corporation can bring about an illegal situation. This is because
produced, processed, or imported merchandise or object of an express agreement is not necessary for the existence of a combination or
commerce is used. conspiracy in restraint of trade. 40 It is enough that a concert of action is
contemplated and that the defendants conformed to the arrangements, 41 and
There are other legislation in this jurisdiction, which prohibit monopolies and what is to be considered is what the parties actually did and not the words
combinations in restraint of trade. 33 they used. For instance, the Clayton Act prohibits a person from serving at
the same time as a director in any two or more corporations, if such
corporations are, by virtue of their business and location of

800
operation, competitors so that the elimination of competition between them Obviously, if a competitor has access to the pricing policy and cost conditions
would constitute violation of any provision of the anti-trust laws. 42 There is of the products of San Miguel Corporation, the essence of competition in a
here a statutory recognition of the anti-competitive dangers which may arise free market for the purpose of serving the lowest priced goods to the
when an individual simultaneously acts as a director of two or more consuming public would be frustrated, The competitor could so manipulate
competing corporations. A common director of two or more competing the prices of his products or vary its marketing strategies by region or by
corporations would have access to confidential sales, pricing and marketing brand in order to get the most out of the consumers. Where the two
information and would be in a position to coordinate policies or to aid one competing firms control a substantial segment of the market this could lead
corporation at the expense of another, thereby stifling competition. This to collusion and combination in restraint of trade. Reason and experience
situation has been aptly explained by Travers, thus: point to the inevitable conclusion that the inherent tendency of interlocking
directorates between companies that are related to each other as
The argument for prohibiting competing corporations from competitors is to blunt the edge of rivalry between the corporations, to seek
sharing even one director is that theinterlock permits the out ways of compromising opposing interests, and thus eliminate
coordination of policies between nominally independent competition. As respondent SMC aptly observes, knowledge by CFC-Robina
firms to an extent that competition between them may be of SMC's costs in various industries and regions in the country win enable
completely eliminated. Indeed, if a director, for example, is to the former to practice price discrimination. CFC-Robina can segment the
be faithful to both corporations, some accommodation must entire consuming population by geographical areas or income groups and
result. Suppose X is a director of both Corporation A and change varying prices in order to maximize profits from every market
Corporation B. X could hardly vote for a policy by A that segment. CFC-Robina could determine the most profitable volume at which it
would injure B without violating his duty of loyalty to B at the could produce for every product line in which it competes with SMC. Access
same time he could hardly abstain from voting without to SMC pricing policy by CFC-Robina would in effect destroy free competition
depriving A of his best judgment. If the firms really do and deprive the consuming public of opportunity to buy goods of the highest
compete in the sense of vying for economic advantage at possible quality at the lowest prices.
the expense of the other there can hardly be any
reason for an interlock between competitors other than the Finally, considering that both Robina and SMC are, to a certain extent,
suppression of competition. 43 (Emphasis supplied.) engaged in agriculture, then the election of petitioner to the Board of SMC
may constitute a violation of the prohibition contained in section 13(5) of the
According to the Report of the House Judiciary Committee of the U. S. Corporation Law. Said section provides in part that "any stockholder of more
Congress on section 9 of the Clayton Act, it was established that: "By means than one corporation organized for the purpose of engaging in agriculture
of the interlocking directorates one man or group of men have been able to may hold his stock in such corporations solely for investment and not for the
dominate and control a great number of corporations ... to the detriment of purpose of bringing about or attempting to bring about a combination to
the small ones dependent upon them and to the injury of the public. 44 exercise control of incorporations ... ."

Shared information on cost accounting may lead to price fixing. Certainly, Neither are We persuaded by the claim that the by-law was Intended to
shared information on production, orders, shipments, capacity and prevent the candidacy of petitioner for election to the Board. If the by-law
inventories may lead to control of production for the purpose of controlling were to be applied in the case of one stockholder but waived in the case of
prices. another, then it could be reasonably claimed that the by-law was being
applied in a discriminatory manner. However, the by law, by its terms, applies

801
to all stockholders. The equal protection clause of the Constitution requires consequence that petitioner is ipso facto disqualified. Consonant with the
only that the by-law operate equally upon all persons of a class. Besides, requirement of due process, there must be due hearing at which the
before petitioner can be declared ineligible to run for director, there must be petitioner must be given the fullest opportunity to show that he is not covered
hearing and evidence must be submitted to bring his case within the ambit of by the disqualification. As trustees of the corporation and of the stockholders,
the disqualification. Sound principles of public policy and management, it is the responsibility of directors to act with fairness to the
therefore, support the view that a by-law which disqualifies a competition stockholders. 48 Pursuant to this obligation and to remove any suspicion that
from election to the Board of Directors of another corporation is valid and this power may be utilized by the incumbent members of the Board to
reasonable. perpetuate themselves in power, any decision of the Board to disqualify a
candidate for the Board of Directors should be reviewed by the Securities
In the absence of any legal prohibition or overriding public policy, wide behind Exchange Commission en banc and its decision shall be final unless
latitude may be accorded to the corporation in adopting measures to protect reversed by this Court on certiorari. 49 Indeed, it is a settled principle that
legitimate corporation interests. Thus, "where the reasonableness of a by-law where the action of a Board of Directors is an abuse of discretion, or
is a mere matter of judgment, and upon which reasonable minds must forbidden by statute, or is against public policy, or is ultra vires, or is a fraud
necessarily differ, a court would not be warranted in substituting its judgment upon minority stockholders or creditors, or will result in waste, dissipation or
instead of the judgment of those who are authorized to make by-laws and misapplication of the corporation assets, a court of equity has the power to
who have expressed their authority. 45 grant appropriate relief. 50

Although it is asserted that the amended by-laws confer on the present III
Board powers to perpetua themselves in power such fears appear to be
misplaced. This power, but is very nature, is subject to certain well Whether or not respondent SEC gravely abused its discretion in denying
established limitations. One of these is inherent in the very convert and petitioner's request for an examination of the records of San Miguel
definition of the terms "competition" and "competitor". "Competition" implies a International Inc., a fully owned subsidiary of San Miguel Corporation
struggle for advantage between two or more forces, each possessing, in
substantially similar if not Identical degree, certain characteristics essential to Respondent San Miguel Corporation stated in its memorandum that
the business sought. It means an independent endeavor of two or more petitioner's claim that he was denied inspection rights as stockholder of SMC
persons to obtain the business patronage of a third by offering more "was made in the teeth of undisputed facts that, over a specific period,
advantageous terms as an inducement to secure trade. 46 The test must be petitioner had been furnished numerous documents and information," to wit:
whether the business does in fact compete, not whether it is capable of an (1) a complete list of stockholders and their stockholdings; (2) a complete list
indirect and highly unsubstantial duplication of an isolated or non- of proxies given by the stockholders for use at the annual stockholders'
characteristics activity. 47 It is, therefore, obvious that not every person or meeting of May 18, 1975; (3) a copy of the minutes of the stockholders'
entity engaged in business of the same kind is a competitor. Such factors as meeting of March 18,1976; (4) a breakdown of SMC's P186.6 million
quantum and place of business, Identity of products and area of competition investment in associated companies and other companies as of December
should be taken into consideration. It is, therefore, necessary to show that 31, 1975; (5) a listing of the salaries, allowances, bonuses and other
petitioner's business covers a substantial portion of the same markets for compensation or remunerations received by the directors and corporate
similar products to the extent of not less than 10% of respondent officers of SMC; (6) a copy of the US $100 million Euro-Dollar Loan
corporation's market for competing products. While We here sustain the Agreement of SMC; and (7) copies of the minutes of all meetings of the
validity of the amended by-laws, it does not follow as a necessary

802
Board of Directors from January 1975 to May 1976, with deletions of that "the right to examine the books of the corporation must be exercised in
sensitive data, which deletions were not objected to by petitioner. good faith, for specific and honest purpose, and not to gratify curiosity, or for
specific and honest purpose, and not to gratify curiosity, or for speculative or
Further, it was averred that upon request, petitioner was informed in writing vexatious purposes. The weight of judicial opinion appears to be, that on
on September 18, 1976; (1) that SMC's foreign investments are handled by application for mandamus to enforce the right, it is proper for the court to
San Miguel International, Inc., incorporated in Bermuda and wholly owned by inquire into and consider the stockholder's good faith and his purpose and
SMC; this was SMC's first venture abroad, having started in 1948 with an motives in seeking inspection. 56 Thus, it was held that "the right given by
initial outlay of ?500,000.00, augmented by a loan of Hongkong $6 million statute is not absolute and may be refused when the information is not
from a foreign bank under the personal guaranty of SMC's former President, sought in good faith or is used to the detriment of the corporation." 57 But the
the late Col. Andres Soriano; (2) that as of December 31, 1975, the "impropriety of purpose such as will defeat enforcement must be set up the
estimated value of SMI would amount to almost P400 million (3) that the total corporation defensively if the Court is to take cognizance of it as a
cash dividends received by SMC from SMI since 1953 has amount to US $ qualification. In other words, the specific provisions take from the stockholder
9.4 million; and (4) that from 1972-1975, SMI did not declare cash or stock the burden of showing propriety of purpose and place upon the corporation
dividends, all earnings having been used in line with a program for the setting the burden of showing impropriety of purpose or motive. 58 It appears to be
up of breweries by SMI the general rule that stockholders are entitled to full information as to the
management of the corporation and the manner of expenditure of its funds,
These averments are supported by the affidavit of the Corporate Secretary, and to inspection to obtain such information, especially where it appears that
enclosing photocopies of the afore-mentioned documents. 51 the company is being mismanaged or that it is being managed for the
personal benefit of officers or directors or certain of the stockholders to the
exclusion of others." 59
Pursuant to the second paragraph of section 51 of the Corporation Law,
"(t)he record of all business transactions of the corporation and minutes of
any meeting shall be open to the inspection of any director, member or While the right of a stockholder to examine the books and records of a
stockholder of the corporation at reasonable hours." corporation for a lawful purpose is a matter of law, the right of such
stockholder to examine the books and records of a wholly-owned subsidiary
of the corporation in which he is a stockholder is a different thing.
The stockholder's right of inspection of the corporation's books and records is
based upon their ownership of the assets and property of the corporation. It
is, therefore, an incident of ownership of the corporate property, whether this
ownership or interest be termed an equitable ownership, a beneficial
ownership, or a ownership. 52 This right is predicated upon the necessity of
self-protection. It is generally held by majority of the courts that where the
right is granted by statute to the stockholder, it is given to him as such and
must be exercised by him with respect to his interest as a stockholder and for
some purpose germane thereto or in the interest of the corporation. 53 In
other words, the inspection has to be germane to the petitioner's interest as a
stockholder, and has to be proper and lawful in character and not inimical to
the interest of the corporation. 54 In Grey v. Insular Lumber, 55 this Court held

803
Some state courts recognize the right under certain conditions, while others some documents which for some reason or another, respondent corporation
do not. Thus, it has been held that where a corporation owns approximately is very reluctant in revealing to the petitioner notwithstanding the fact that no
no property except the shares of stock of subsidiary corporations which are harm would be caused thereby to the corporation." 67 There is no question
merely agents or instrumentalities of the holding company, the legal fiction of that stockholders are entitled to inspect the books and records of a
distinct corporate entities may be disregarded and the books, papers and corporation in order to investigate the conduct of the management, determine
documents of all the corporations may be required to be produced for the financial condition of the corporation, and generally take an account of
examination, 60 and that a writ of mandamus, may be granted, as the records the stewardship of the officers and directors. 68
of the subsidiary were, to all incontents and purposes, the records of the
parent even though subsidiary was not named as a party. 61 mandamus was In the case at bar, considering that the foreign subsidiary is wholly owned by
likewise held proper to inspect both the subsidiary's and the parent respondent San Miguel Corporation and, therefore, under its control, it would
corporation's books upon proof of sufficient control or dominion by the parent be more in accord with equity, good faith and fair dealing to construe the
showing the relation of principal or agent or something similar thereto. 62 statutory right of petitioner as stockholder to inspect the books and records of
the corporation as extending to books and records of such wholly subsidiary
On the other hand, mandamus at the suit of a stockholder was refused where which are in respondent corporation's possession and control.
the subsidiary corporation is a separate and distinct corporation domiciled
and with its books and records in another jurisdiction, and is not legally IV
subject to the control of the parent company, although it owned a vast
majority of the stock of the subsidiary. 63 Likewise, inspection of the books of Whether or not respondent SEC gravely abused its discretion in allowing the
an allied corporation by stockholder of the parent company which owns all stockholders of respondent corporation to ratify the investment of corporate
the stock of the subsidiary has been refused on the ground that the funds in a foreign corporation
stockholder was not within the class of "persons having an interest." 64
Petitioner reiterates his contention in SEC Case No. 1423 that respondent
In the Nash case, 65 The Supreme Court of New York held that the corporation invested corporate funds in SMI without prior authority of the
contractual right of former stockholders to inspect books and records of the stockholders, thus violating section 17-1/2 of the Corporation Law, and
corporation included the right to inspect corporation's subsidiaries' books and alleges that respondent SEC should have investigated the charge, being a
records which were in corporation's possession and control in its office in statutory offense, instead of allowing ratification of the investment by the
New York." stockholders.

In the Bailey case, 66 stockholders of a corporation were held entitled to Respondent SEC's position is that submission of the investment to the
inspect the records of a controlled subsidiary corporation which used the stockholders for ratification is a sound corporate practice and should not be
same offices and had Identical officers and directors. thwarted but encouraged.

In his "Urgent Motion for Production and Inspection of Documents" before Section 17-1/2 of the Corporation Law allows a corporation to "invest its
respondent SEC, petitioner contended that respondent corporation "had funds in any other corporation or business or for any purpose other than the
been attempting to suppress information for the stockholders" and that main purpose for which it was organized" provided that its Board of Directors
petitioner, "as stockholder of respondent corporation, is entitled to copies of has been so authorized by the affirmative vote of stockholders holding

804
shares entitling them to exercise at least two-thirds of the voting power. If the pursuance of the corporate purpose, does not need the
investment is made in pursuance of the corporate purpose, it does not need approval of stockholders; but when the purchase of shares
the approval of the stockholders. It is only when the purchase of shares is of another corporation is done solely for investment and not
done solely for investment and not to accomplish the purpose of its to accomplish the purpose of its incorporation, the vote of
incorporation that the vote of approval of the stockholders holding shares approval of the stockholders is necessary. In any case, the
entitling them to exercise at least two-thirds of the voting power is purchase of such shares or securities must be subject to the
necessary. 69 limitations established by the Corporations law; namely, (a)
that no agricultural or mining corporation shall be restricted
As stated by respondent corporation, the purchase of beer manufacturing to own not more than 15% of the voting stock of nay
facilities by SMC was an investment in the same business stated as its main agricultural or mining corporation; and (c) that such holdings
purpose in its Articles of Incorporation, which is to manufacture and market shall be solely for investment and not for the purpose of
beer. It appears that the original investment was made in 1947-1948, when bringing about a monopoly in any line of commerce of
SMC, then San Miguel Brewery, Inc., purchased a beer brewery in Hongkong combination in restraint of trade." The Philippine Corporation
(Hongkong Brewery & Distillery, Ltd.) for the manufacture and marketing of Law by Sulpicio S. Guevara, 1967 Ed., p. 89) (Emphasis
San Miguel beer thereat. Restructuring of the investment was made in 1970- supplied.)
1971 thru the organization of SMI in Bermuda as a tax free reorganization.
40. Power to invest corporate funds. A private corporation
Under these circumstances, the ruling in De la Rama v. Manao Sugar has the power to invest its corporate funds "in any other
Central Co., Inc., supra, appears relevant. In said case, one of the issues corporation or business, or for any purpose other than the
was the legality of an investment made by Manao Sugar Central Co., Inc., main purpose for which it was organized, provide that 'its
without prior resolution approved by the affirmative vote of 2/3 of the board of directors has been so authorized in a resolution by
stockholders' voting power, in the Philippine Fiber Processing Co., Inc., a the affirmative vote of stockholders holding shares in the
company engaged in the manufacture of sugar bags. The lower court said corporation entitling them to exercise at least two-thirds of
that "there is more logic in the stand that if the investment is made in a the voting power on such a propose at a stockholders'
corporation whose business is important to the investing corporation and meeting called for that purpose,' and provided further, that no
would aid it in its purpose, to require authority of the stockholders would be to agricultural or mining corporation shall in anywise be
unduly curtail the power of the Board of Directors." This Court affirmed the interested in any other agricultural or mining
ruling of the court a quo on the matter and, quoting Prof. Sulpicio S. corporation. When the investment is necessary to
Guevara, said: accomplish its purpose or purposes as stated in its articles
of incorporation the approval of the stockholders is not
"j. Power to acquire or dispose of shares or securities. A necessary."" (Id., p. 108) (Emphasis ours.) (pp. 258-259).
private corporation, in order to accomplish is purpose as
stated in its articles of incorporation, and subject to the Assuming arguendo that the Board of Directors of SMC had no authority to
limitations imposed by the Corporation Law, has the power make the assailed investment, there is no question that a corporation, like an
to acquire, hold, mortgage, pledge or dispose of shares, individual, may ratify and thereby render binding upon it the originally
bonds, securities, and other evidence of indebtedness of any unauthorized acts of its officers or other agents. 70 This is true because the
domestic or foreign corporation. Such an act, if done in questioned investment is neither contrary to law, morals, public order or

805
public policy. It is a corporate transaction or contract which is within the the respondent Securities and Exchange Commission deliberating and
corporate powers, but which is defective from a supported failure to observe acting en banc and ultimately to this Court. Unless disqualified in the manner
in its execution the. requirement of the law that the investment must be herein provided, the prohibition in the afore-mentioned amended by-laws
authorized by the affirmative vote of the stockholders holding two-thirds of shall not apply to petitioner.
the voting power. This requirement is for the benefit of the stockholders. The
stockholders for whose benefit the requirement was enacted may, therefore, The afore-mentioned six (6) Justices, together with Justice Fernando, voted
ratify the investment and its ratification by said stockholders obliterates any to declare the issue on the validity of the foreign investment of respondent
defect which it may have had at the outset. "Mere ultra vires acts", said this corporation as moot.
Court in Pirovano, 71 "or those which are not illegal and void ab initio, but are
not merely within the scope of the articles of incorporation, are merely Chief Justice Fred Ruiz Castro reserved his vote on the validity of the
voidable and may become binding and enforceable when ratified by the amended by-laws, pending hearing by this Court on the applicability of
stockholders. section 13(5) of the Corporation Law to petitioner.

Besides, the investment was for the purchase of beer manufacturing and Justice Fernando reserved his vote on the validity of subject amendment to
marketing facilities which is apparently relevant to the corporate purpose. the by-laws but otherwise concurs in the result.
The mere fact that respondent corporation submitted the assailed investment
to the stockholders for ratification at the annual meeting of May 10, 1977
Four (4) Justices, namely, Justices Teehankee, Concepcion, Jr., Fernandez
cannot be construed as an admission that respondent corporation had
and Guerrero filed a separate opinion, wherein they voted against the validity
committed an ultra vires act, considering the common practice of
of the questioned amended bylaws and that this question should properly be
corporations of periodically submitting for the gratification of their
resolved first by the SEC as the agency of primary jurisdiction. They concur
stockholders the acts of their directors, officers and managers.
in the result that petitioner may be allowed to run for and sit as director of
respondent SMC in the scheduled May 6, 1979 election and subsequent
WHEREFORE, judgment is hereby rendered as follows: elections until disqualified after proper hearing by the respondent's Board of
Directors and petitioner's disqualification shall have been sustained by
The Court voted unanimously to grant the petition insofar as it prays that respondent SEC en banc and ultimately by final judgment of this Court.
petitioner be allowed to examine the books and records of San Miguel
International, Inc., as specified by him. In resume, subject to the qualifications aforestated judgment is hereby
rendered GRANTING the petition by allowing petitioner to examine the books
On the matter of the validity of the amended by-laws of respondent San and records of San Miguel International, Inc. as specified in the petition. The
Miguel Corporation, six (6) Justices, namely, Justices Barredo, Makasiar, petition, insofar as it assails the validity of the amended by- laws and the
Antonio, Santos, Abad Santos and De Castro, voted to sustain the validity ratification of the foreign investment of respondent corporation, for lack of
per se of the amended by-laws in question and to dismiss the petition without necessary votes, is hereby DISMISSED. No costs.
prejudice to the question of the actual disqualification of petitioner John
Gokongwei, Jr. to run and if elected to sit as director of respondent San Makasiar, Santos Abad Santos and De Castro, JJ., concur.
Miguel Corporation being decided, after a new and proper hearing by the
Board of Directors of said corporation, whose decision shall be appealable to
Aquino, and Melencio Herrera JJ., took no part.

806
in this matter, being moved by the suggestions and representations of
Vicente Ocampo, manage of the Oriental Theater, to the effect that the
G.R. No. 11897 September 24, 1918 securing of the said films was necessary to the success of the corporation.

J. F. RAMIREZ, plaintiff-appellee, Near the end of July of the year aforesaid, Jose Ramirez, as representative
vs. of his father, placed in the hands of Ramon J. Fernandez an offer, dated July
THE ORIENTALIST CO., and RAMON J. FERNANDEZ, defendants- 4, 1913, stating detail the terms upon which the plaintiff would undertake to
appellants. supply from Paris the aforesaid films. This officer was declared to be good
until the end of July; and as only about for the Orientalist Company to act on
Jose Moreno Lacalle for appellant Fernandez. the matter speedily, if it desired to take advantage of said offer. Accordingly,
Sanz, Opisso & Luzuriaga for appellant "The Orientalist Co." Ramon J. Fernandez, on July 30, had an informal conference with all the
No appearance for appellee. members of the company's board of directors except one, and with approval
of those with whom he had communicated, addressed a letter to Jose
Ramirez, in Manila, accepting the offer contained in the memorandum of July
STREET, J.:
4th for the exclusive agency of the Eclair films. A few days later, on August 5,
he addressed another letter couched in the same terms, likewise accepting
The Orientalist Company is a corporation, duly organized under the laws of the office of the exclusive agency for the Milano Films.
the Philippine Islands, and in 1913 and 1914, the time of the occurrences
which gave rise to this lawsuit, was engaged in the business of maintaining
The memorandum offer contained a statement of the price at which the films
and conducting a theatre in the city of Manila for the exhibition of
would be sold, the quantity which the representative of each was required to
cinematographic films. Under the articles of incorporation the company is
take and information concerning the manner and intervals of time for the
authorized to manufacture, buy, or otherwise obtain all accessories
respective shipments. The expenses of packing, transportation and other
necessary for conducting such a business. The plaintiff J. F. Ramirez was, at
incidentals were to be at the cost of the purchaser. There was added a
the same time, a resident of the city of Paris, France, and was engaged in
clause in which J. F. Ramirez described his function in such transactions as
the business of marketing films for a manufacturer or manufacturers, there
that of a commission agent and stated that he would see to the prompt
engaged in the production or distribution of cinematographic material. In this
shipment of the films, would pay the manufacturer, and take care that the
enterprise the plaintiff was represented in the city of Manila by his son, Jose
films were insured his commission for such services being fixed at 5 per
Ramirez.
cent.

In the month of July, 1913, certain of the directors of the Orientalist Company,
What we consider to be the most portion of the two letters of acceptance
in Manila, became apprised of the fact that the plaintiff in Paris had control of
written by R. J. Fernandez to Jose Ramirez is in the following terms:
the agencies for two different marks of films, namely, the "Eclair Films" and
the "Milano Films;" and negotiations were begun with said officials of the
Orientalist Company by Jose Ramirez, as agent of the plaintiff, for the We willingly accepted the officer under the terms communicated by
purpose of placing the exclusive agency of these films in the hands of the your father in his letter dated at Paris on July 4th of the present year.
Orientalist Company. The defendant Ramon J. Fernandez, one of the
directors of the Orientalist Company and also its treasure, was chiefly active

807
These communications were signed in the following form, in which it will be B. Hernandez, the films which had been procured by he payment of said
noted the separate signature of R. J. Fernandez, as an individual, is placed drafts were treated by him as his own property; and they in fact never came
somewhat below and to the left of the signature of the Orientalist Company into the actual possession of the Orientalist Company as owner at all, though
as singed by R. J. Fernandez, in the capacity of treasurer: it is true Hernandez rented the films to the Orientalist Company and they
were exhibited by it in the Oriental Theater under an arrangement which was
made between him and the theater's manager.

THE ORIENTALIST COMPANY, During the period between February 27, 1914, and April 30, 1914, there
By R. J. FERNANDEZ, arrived in the city of Manila several remittances of films from Paris, and it is
Treasurer, these shipments which have given occasion for the present action. All of the
drafts accompanying these films were drawn, as on former occasions, upon
the Orientalist Company; and all were accepted in the name of B.
R. J. FERNANDEZ. Hernandez, except the last, which was accepted by B. Hernandez
individually. None of the drafts thus accepted were taken up by the drawee or
by B. Hernandez when they fell due; and it was finally necessary for the
Both of these letters also contained a request that Jose Ramirez should at
plaintiff himself to take them up as dishonored by non-payment.
once telegraph to his father in Paris that his offer had been accepted by the
Orientalist Company and instruct him to make a contract with the film
companies, according to the tenor of the offer, and in the capacity of Thereupon this action was instituted by the plaintiff on May 19, 1914, against
attorney-in-fact for the Orientalist Company. The idea behind the latter the Orientalist Company, and Ramon J. Fernandez. As the films which
suggestion apparently was that the contract for the films would have to be accompanied the dishonored were liable to deteriorate, the court, upon
made directly between the film-producing companies and the Orientalist application of the plaintiff, and apparently without opposition on the part of
Company; and it seemed convenient, in order to save time, that the the defendants, appointed a receiver who took charge of the films and sold
Orientalist Company should clothed J. F. Ramirez with full authority as its them. The amount realized from this sale was applied to the satisfaction of
attorney-in-fact. This idea was never given effect; and so far as the record the plaintiff's claim and was accordingly delivered to him in part payment
shows, J. F. Ramirez himself procured the films upon his own responsibility, thereof. At trial judgment was given for the balance due to the plaintiff,
as he indicated in the officer of July 4 that he would do, with the result that namely P6,018.93, with interest from May 19, 1914, the date of the institution
the only contracting parties in this case are J. F. Ramirez of the one part, and of the action. In the judgment of the trial court the Orientalist Company was
the Orientalist Company, with Ramon J. Fernandez of the other. declared to be a principal debtor and Ramon J. Fernandez was declared to
be liable subsidiarily as guarantor. From this judgment both of the parties
defendant appealed.
In due time the films began to arrive in Manila, a draft for the cost and
expenses incident to each shipment being attached to the proper bill of
lading. It appears that the Orientalist Company was without funds to meet In this Court neither of the parties appellant make any question with respect
these obligations and the first few drafts were dealt with in the following to the right of the plaintiff to recover from somebody the amount awarded by
manner: The drafts, upon presented through the bank, were accepted in the the lower court; but each of the defendants insists the other is liable for the
name of the Orientalist Company by its president B. Hernandez, and were whole. It results that the real contention upon this appeal is between the two
taken up by the latter with his own funds. As the drafts had thus been paid by defendants.

808
It is stated in the brief of the appellant Ramon J. Fernandez and the When an action is brought upon a written instrument and the
statement is not challenged by the Orientalist Company that the judgment complaint contains or has annexed or has annexed a copy of such
has already been executed as against the company is exclusively and instrument, the genuineness and due execution of the instrument
primarily liable the entire indebtedness, the question as to the liability of shall be deemed admitted, unless specifically denied under oath in
Ramon J. Fernandez would be academic. But if the latter is liable as principal the answer.
obligor for the whole or any part of the debt, it will be necessary to modify the
judgment in order to adjust the rights of the defendants in accordance with No sworn answer denying the genuineness and due execution of the
such finding. contracts in question or questioning the authority of Ramon J. Fernandez to
bind the Orientalist Company was filed in this case; but evidence was
It will be noted that the action is primarily founded upon the liability created admitted without objection from the plaintiff, tending to show that Ramon J.
by the letters dated July 30th and August 5, 1913, in connection with the Fernandez had no such authority. This evidence consisted of extracts from
plaintiff's offer of July 4, 1913; and both of the letters mentioned are copied the minutes of the proceedings of the company's board of directors and also
into the complaint as the foundation of the action. The action is not based of extracts from the minutes of the proceedings of the company's
upon the dishonored drafts which were accepted by B. Hernandez in the stockholders, showing that the making of this contract had been under
name of the Orientalist Company; and although these drafts, as well as the consideration in both bodies and that the authority to make the same had
last draft, which was accepted by B. Hernandez individually, have been been withheld by the stockholders. It therefore becomes necessary for us to
introduced in evidence, this was evidently done for the purpose of proving consider whether the administration resulting from the failure of the
the amount of damages which the plaintiff was entitled to recover. defendant company to deny the execution of the contracts under oath is
binding upon it for all purposes of this lawsuit, or whether such failure should
In the discussion which is to follow we shall consider, first, the question of the be considered a mere irregularity of procedure which was waived when the
liability of the corporation upon the contracts contained in the letters of July evidence referred to was admitted without objection from the plaintiff. The
30 and August 5, 1913, and, secondly the question of the liability of Ramon J. proper solution of this problem makes it necessary to consider carefully the
Fernandez, based upon his personal signature to the same documents. principle underlying the provision above quoted.

As to the liability of the corporation a preliminary point of importance arises That the situation was one in which an answer under oath denying the
upon the pleadings. The action, as already stated, is based upon documents authority of the agent should have been interposed, supposing that the
purporting to be signed by the Orientalist Company, and copies of the company desired to contest this point, is not open to question. In the case of
documents are set out in the complaint. It was therefore incumbent upon the Merchant vs. International Banking Corporation, (6 Phil. Rep., 314), it
corporation, if it desired to question the authority of Fernandez to bind it, to appeared that one Brown has signed the name of the defendant bank as
deny the due execution of said contracts under oath, as prescribed in section guarantor of a promissory note. The bank was sued upon this guaranty and
103 of the Code of Civil procedure. Said section, in the part pertinent to the at the hearing attempted to prove that Brown had no authority to bind the
situation now under consideration, reads as follows: bank by such contract. It was held that buy failing to deny the contract under
oath, the bank had admitted the genuineness and due execution thereof, and
that this admission extended not only to the authenticity of the signature of
Brown but also to his authority. Said Justice Willard: "The failure of the
defendant to deny the genuineness and due execution of this guaranty under
oath was an admission not only of the signature of Brown, but also his

809
authority to make the contract in behalf of the defendant and of the power the If the person who signed this note for the company, and upon which they are
contract in behalf of the defendant and of the power of the defendant to enter sued, was not authorized to make it, let them say so upon oath, and
into such a contract. the onus is then on the plaintiff to overcome the plea."

The rule thus stated is in entire accord with the doctrine prevailing in the It should be noted that the provision contained in section 103 of our Code of
United States, as will be seen by reference to the following, among other Civil Procedure is embodied in some form or other in the statutes of probably
authorities: all of the American States, and it is not by any means peculiar to the laws of
California, though it appears to have been taken immediately from the
The case of Barrett Mining Co. vs. Tappan (2 Colo., 124) was an action statutes of that State. (Secs. 447, 448, California Code of Civil Procedure.)
against a mining corporation upon an appeal bond. The name of the
company had been affixed to the obligation by an agent, and no sufficient There is really a broader question here involved than that which relates
affidavit was filed by the corporation questioning its signature or the authority merely to the formality of verifying the answer with an affidavit. This question
of the agent to bind the company. It was held that the plaintiff did not have to arises from the circumstance that the answer of the corporation does not in
prove the due execution of the bond and that the corporation as to be taken any was challenge the authority of Ramon J. Fernandez to bind it by the
as admitting the authority of the agent to make the signature. Among other contracts in question and does not set forth, as a special defense, any such
things the court said: "But it is said that the authority of Barrett to execute the lack of authority in him. Upon well-established principles of pleading lack of
bond is distinguishable from the signing and, although the signature must be authority in an officer of a corporation to bind it by a contract executed by him
denied under oath, the authority of the agent need not be. Upon this we in its name is a defense which should be specially pleaded and this quite
observe that the statute manifestly refers to the legal effect of the signature, apart from the requirement, contained in section 103, that the answer setting
rather than the manual act of singing. If the name of the obligor, in a bond, is up such defense should be verified by oath. But is should not here escape
subscribed by one in his presence, and by his direction, the effect is the observation that section 103 also requires in denial contemplated in that
same as if his name should be signed with his own hand, and under such section shall be specific. An attack on the instrument in general terms is
circumstances we do not doubt that the obligor must deny his signature insufficient, even though the answer is under oath. (Songco vs. Sellner, 37
under oath, in order to put the obligee to proof of the fact. Quit facit per aliam Phil. Rep., 254.)
facit per se, and when the name is signed by one thereunto authorized, it is
as much as the signature of the principal as if written with his own hand. In the first edition of a well-known treatise on the laws of corporations we find
Therefore, if the principal would deny the authority of the agent, as the the following proposition:
validity of the signature is thereby directly attacked, the denial must be under
oath. If an action is brought against a corporation upon a contract alleged
to be its contract, if it desires to set up the defense that the contract
In Union Dry Company vs. Reid (26 Ga., 107), an action was brought upon a was executed by one not authorized as its agent, it must plead non
promissory note purporting to have been given by on A. B., as the treasurer est factum. (Thompson on Corporations, 1st ed., vol. 6, sec. 7631.)
of the defendant company. Said the court: "Under the Judiciary Act of 1799,
requiring the defendant to deny on oath an instrument of writing, upon which Again, says the same author:
he is sued, the plea in this case should have been verified.

810
A corporation can not avail itself of the defense that it had no power are from time to time convoked. Whether a particular officer actually
to enter into the obligation to enforce which the suit is brought, possesses the authority which he assumes to exercise is frequently known to
unless it pleads that defense. This principle applies equally where very few, and the proof of it usually is not readily accessible to the stranger
the defendant intends to challenge the power of its officer or agent to who deals with the corporation on the faith of the ostensible authority
execute in its behalf the contract upon which the action brought and exercised by some of the corporate officers. It is therefore reasonable, in a
where it intends to defend on the ground of total want of power in the case where an officer of a corporation has made a contract in its name, that
corporation to make such a contract. (Opus citat. sec. 7619.) the corporation should be required, if it denies his authority, to state such
defense in its answer. By this means the plaintiff is apprised of the fact that
In Simon vs. Calfee (80 Ark., 65), it was said: the agent's authority is contested; and he is given an opportunity to adduce
evidence showing either that the authority existed or that the contract was
Though the power of the officers of a business corporation to issue ratified and approved.
negotiable paper in its name is not presumed, such corporation can
not avail itself of a want of power in its officers to bind it unless the We are of the opinion that the failure of the defendant corporation to make
defense was made on such ground. any issue in its answer with regard to the authority of Ramon J. Fernandez to
bind it, and particularly its failure to deny specifically under oath the
The rule has been applied where the question was whether corporate officer, genuineness and due execution of the contracts sued upon, have the effect
having admitted power to make a contract, had in the particular instance of elimination the question of his authority from the case, considered as a
exceeded that authority, (Merill vs. Consumers' Coal Co., 114 N.Y., 216); and matter of mere pleading. The statute (sec. 103) plainly says that if a written
it has been held that where the answer in a suit against a corporation on its instrument, the foundation of the suit, is not denied upon oath, it shall be
note relies simply on the want of power of the corporation to issue notes, the deemed to be admitted. It is familiar doctrine that an admission made in a
defendant can not afterwards object that the plaintiff has not shown that the pleading can not be controverted by the party making such admission; and
officer executing the note were empowered to do so. (Smith vs. Eureka Flour all proof submitted by him contrary thereto or inconsistent therewith should
Mills Co., 6 Cal., 1.) simply be ignored by the court, whether objection is interposed by the
opposite party or not. We can see no reason why a constructive admission,
created by the express words of the statute, should be considered to have
The reason for the rule enunciated in the foregoing authorities will, we think,
less effect than any other admission.
be readily appreciated. In dealing with corporations the public at large is
bound to rely to a large extent upon outward appearances. If a man is found
acting for a corporation with the external indicia of authority, any person, not The parties to an action are required to submit their respective contentions to
having notice of want of authority, may usually rely upon those appearances; the court in their complaint and answer. These documents supply the
and if it be found that the directors had permitted the agent to exercise that materials which the court must use in order to discover the points of
authority and thereby held him out as a person competent to bind the contention between the parties; and where the statute says that the due
corporation, or had acquiesced in a contract and retained the benefit execution of a document which supplies the foundation of an action is to be
supposed to have been conferred by it, the corporation will be bound, taken as admitted unless denied under oath, the failure of the defendant to
notwithstanding the actual authority may never have been granted. The make such denial must be taken to operate as a conclusive admission, so
public is not supposed nor required to know the transactions which happen long as the pleadings remain that form.
around the table where the corporate board of directors or the stockholders

811
It is true that it is declared in section 109 of the Code of Civil Procedure that would have been more soundly planted if it had been said that the incapacity
immaterial variances between the allegations of a pleading and the proof of the plaintiff, due to his drunken condition, was a matter which did not
shall be disregarded and the facts shall be found according to the evidence. involve either the genuineness or due execution of the release. Like the
The same section, however, recognizes the necessity for an amendment of defenses of fraud, coercion, imbecility, and mistake, it was a matter which
the pleadings. And judgment must be in conformity with the case made in could be proved under the general issue and did not have to be set up in a
conformity with the case made in the pleadings and established by the proof, sworn reply. (Cf. Moore vs. Copp, 119 Cal., 429, 432, 433.) A somewhat
and relief can not be granted that is substantially inconsistent with either. A similar explanation can, we think, be given of the case of Clark vs. Child in
party can no more succeed upon a case proved but not alleged than upon a which the rule declared in the earlier case was followed. With respect to both
case alleged but nor proved. This rule of course operates with like effect decisions which we merely observe that upon point of procedure which they
upon both parties, and applies equality to the defendants special defense as are supposed to maintain, the reasoning of the court is in our opinion
to the plaintiffs cause of action. unconvincing.

Of course this Court, under section 109 of the Code of Civil Procedure, has We shall now consider the liability of the defendant company on the merits
authority even now to permit the answer of the defendant to be amended; just as if that liability had been properly put in issue by a specific answer
and if we believed that the interests of justice so required, we would either under oath denying the authority of Fernandez go to bind it. Upon this
exercise that authority or remand the cause for a new trial in court below. As question it must at the outset be premised that Ramon J. Fernandez, as
will appear further on in this opinion, however, we think that the interests of treasurer, had no independent authority to bind the company by signing its
justice will best be promoted by deciding the case, without more ado, upon name to the letters in question. It is declared by signing its name to the
the issues presented in the record as it now stands. letters in question. It is declared in section 28 of the Corporation Law that
corporate power shall be exercised, and all corporate business conducted by
That we may not appear to have overlooked the matter, we will observe that the board of directors; and this principle is recognized in the by-laws of the
two cases are cited from California in which the Supreme Court of the State corporation in question which contain a provision declaring that the power to
has held that where a release is pleaded by way of defense and evidence make contracts shall be vested in the board of directors. It is true that it is
tending to destroy its effect is introduced without objection, the circumstance also declared in the same by-laws that the president shall have the power,
that it was not denied under oath is immaterial. In the earlier of these cases, and it shall be his duty, to sign contract; but this has reference rather to the
Crowley, vs. Railroad Co. (60 Cal., 628), an action was brought against a formality of reducing to proper form the contract which are authorized by the
railroad company to recover damages for the death of the plaintiff's minor board and is not intended to confer an independent power to make contract
son, alleged to have been killed by the negligence of the defendant. The binding on the corporation.
defendant company pleaded by way of defense a release purporting to be
signed by the plaintiff, and in its answer inserted a copy of the release. The The fact that the power to make corporate contract is thus vested in the
execution of the release was not denied under oath; but at the trial evidence board of directors does not signify that a formal vote of the board must
was submitted on behalf of the plaintiff tending to show that at the time he always be taken before contractual liability can be fixed upon a
signed the release, he was incompetent by reason of drunkenness to bind corporation; for the board can create liability, like an individual, by
himself thereby. It was held that inasmuch as this evidence had been other means than by a formal expression of its will. In this connection the
submitted by the plaintiff without objection, it was proper for the court to case of Robert Gair Co. vs. Columbia Rice Packing Co. (124 La., 194) is
consider it. We do not question the propriety of that decision, especially as instructive. If there appeared that the secretary of the defendant corporation
the issue had been passed upon by a jury; but we believe that the decision had signed an obligation on its behalf binding it as guarantor of the

812
performance of an important contract upon which the name of another The following extracts from the minutes of this meeting are here pertinent:
corporation appeared as principal. The defendant company set up by way of
defense that is secretary had no authority to bind it by such an engagement. Mr. Fernandez informed the stockholders that, in view of the urgency
The court found that the guaranty was given with the knowledge and consent of the matter and for the purpose of avoiding that other importers
of the president and directors, and that this consent of the president and should get ahead of the corporation in this regard, he and Messrs. B.
directors, and that this consent was given with as much observance of Hernandez, Leon Monroy, and Dr. Papa met for the purpose of
formality as was customary in the transaction of the business of the considering the acceptance of the offer together with the
company. It was held that, so far as the authority of the secretary was responsibilities attached thereto, made to the corporation by the film
concerned, the contract was binding. In discussing this point, the court manufacturers of Eclair and Milano of Paris and Italy respectively,
quoted with approval the following language form one of its prior decisions: inasmuch as the first shipment of films was then expected to arrive.

The authority of the subordinate agent of a corporation often At the same time he informed the said stockholders that he had
depends upon the course of dealings which the company or its already made arrangements with respect to renting said films after
director have sanctioned. It may be established sometimes without they have been once exhibited in the Cine Oriental, and that the
reference to official record of the proceedings of the board, by proof corporation could very well meet the expenditure involved and net a
of the usage which the company had permitted to grow up in certain profit, but that, if we could enter into a contract with about
business, and of the acquiescence of the board charged with the nine cinematographs, big gains would be obtained through such a
duty of supervising and controlling the company's business. step.

It appears in evidence, in the case now before us, that on July 30, the date The possibility that the corporation might not see fit to authorize the contract,
upon which the letter accepting the offer of the Eclair films was dispatched or might for lack of funds be unable to make the necessary outlay, was
the board of directors of the Orientalist Company convened in special foreseen; and in such contingency the stockholders were informed, that the
session in the office of Ramon J. Fernandez at the request of the latter. four gentlemen above mentioned (Hernandez, Fernandez, Monroy, and
There were present the four members, including the president, who had Papa) "would continue importing said films at their own account and risk, and
already signified their consent to the making of the contract. At this meeting, shall be entitled only to a compensation of 10 per cent of their outlay in
as appears from the minutes, Fernandez informed the board of the offer importing the films, said payment to be made in shares of said corporation,
which had been received from the plaintiff with reference to the importation of inasmuch as the corporation is lacking available funds for the purpose, and
films. The minutes add that terms of this offer were approved; but at the also because there are 88 shares of stock remaining still unsold."
suggestion of Fernandez it was decided to call a special meeting of the
stockholders to consider the matter and definite action was postponed. In view of this statement, the stockholders adopted a resolution to the effect
that the agencies of the Eclair and Milano films should be accepted, if the
The stockholders meeting was convoked upon September 18, 1913, upon corporation could obtain the money with which to meet the expenditure
which occasion Fernandez informed those present of the offer in question involved, and to this end appointed a committee to apply to the bank for a
and of the terms upon which the films could be procured. He estimated that credit. The evidence shows that an attempt was made, on behalf of the
the company would have to make an outlay of about P5,500 per month, if the corporation, to obtain a credit of P10,000 from the Bank of the Philippine
offer for the two films should be accepted by it. Islands for the purpose indicated, but the bank declined to grant his credit.

813
Thereafter another special meeting of the shareholders of the defendant Ignoring now, for a moment, the transactions of the stockholders, and
corporation was called at which the failure of their committee to obtain a reverting to the proceedings of the board of directors of the Orientalist
credit from the bank was made known. A resolution was thereupon passed to Company, we find that upon October 27, 1913, after Fernandez had departed
the effect that the company should pay to Hernandez, Fernandez, Monroy, from the Philippine Islands, to be absent for many months, said board
and Papa an amount equal to 10 per cent of their outlay in importing the adopted a resolution conferring the following among other powers on Vicente
films, said payment to be made in shares of the company in accordance with Ocampo, the manager of the Oriental theater, namely:
the suggestion made at the previous meeting. At the time this meeting was
held three shipment of the films had already been received in Manila. (1) To rent a box for the films in the "Kneeler Building."

We believe it is a fair inference from the recitals of the minutes of the (4) To be in charge of the films and of the renting of the same.
stockholders meeting of September 18, and especially from the first
paragraph above quoted, that this body was then cognizant that the officer (5) To advertise in the different newspapers that we are importing
had already been accepted in the name of the Orientalist Company and that films to be exhibited in the Cine Oriental.
the films which were then expected to arrive were being imported by virtue of
such acceptance. Certainly four members of the board of directors there
(6) Not to deliver any film for rent without first receiving the rental
present were aware of this fact, as the letter accepting the offer had been
therefor or the guaranty for the payment thereof.
sent with their knowledge and consent. In view of this circumstance, a certain
doubt arises whether they meant to utilize the financial assistance of the four
so-called importers in order that the corporation might bet the benefit of the (7) To buy a book and cards for indexing the names of the films.
contract for the films, just as it would have utilized the credit of the bank if
such credit had been extended. If such was the intention of the stockholders (10) Upon the motion of Mr. Ocampo, it was decided to give ample
their action amounted to a virtual, though indirect, approval of the contract. It powers to the Hon. R. Acua to enter into agreements with
is not however, necessary to found the judgment on this interpretation of the cinematograph proprietors in the provinces for the purpose of renting
stockholders proceedings, inasmuch as we think for reasons presently to be films from us.
stated, that the corporation is bound, and we will here assume that in the end
the contract were not approved by the stockholders. It thus appears that the board of directors, before the financial inability of the
corporation to proceed with the project was revealed, had already recognized
It will be observed that Ramon J. Fernandez was the particular officer and the contract as being in existence and had proceeded to take the steps
member of the board of directors who was most active in the effort to secure necessary to utilize the films. Particularly suggestive is the direction given at
the films for the corporation. The negotiations were conducted by him with this meeting for the publication of announcements in the newspapers to the
the knowledge and consent of other members of the board; and the contract effect that the company was engaged in importing films. In the light of all the
was made with their prior approval. As appears from the papers in this circumstances of the case, we are of the opinion that the contracts in
record, Fernandez was the person to who keeping was confided the printed question were thus inferentially approved by the company's board of
stationery bearing the official style of the corporation, as well as rubber directors and that the company is bound unless the subsequent failure of the
stencil with which the name of the corporation could be signed to documents stockholders to approve said contracts had the effect of abrogating the
bearing its name. liability thus created.

814
Both upon principle and authority it is clear that the action of the In passing upon the liability of a corporation in cases of this kind it is always
stockholders, whatever its character, must be ignored. The functions of well to keep in mind the situation as it presents itself to the third party with
the stockholders of a corporation are, it must be remembered, of a whom the contract is made. Naturally he can have little or no information as
limited nature. The theory of a corporation is that the stockholders may to what occurs in corporate meetings; and he must necessarily rely upon the
have all the profits but shall turn over the complete management of the external manifestations of corporate consent. The integrity of commercial
enterprise to their representatives and agents, called directors. transactions can only be maintained by holding the corporation strictly to the
Accordingly, there is little for the stockholders to do beyond electing liability fixed upon it by its agents in accordance with law, and we would be
directors, making by-laws, and exercising certain other special powers sorry to announce a doctrine which would permit the property of a man in the
defined by-law. In conformity with this idea it is settled that contract city of Paris to be whisked out of his hands and carried into a remote quarter
between a corporation and third person must be made by the director of the earth without recourse against the corporations whose name and
and not by the stockholders. The corporation, in such matters, is authority had been used in the manner disclosed in this case. As already
represented by the former and not by the latter. (Cook on Corporations, observed, it is familiar doctrine that if a corporation knowingly permits
sixth ed., secs. 708, 709.) This conclusion is entirely accordant with the one of its officer, or any other agent, to do acts within the scope of an
provisions of section 28 of our Corporation Law already referred to. It apparent authority, and thus hold him out to the public as possessing
results that where a meeting of the stockholders is called for the power to do those acts, the corporation will as against any one who has
purpose of passing on the propriety of making a corporate contract, its in good faith dealt with the corporation through such agent, be
resolutions are at most advisory and not in any wise binding on the estopped from denying his authority; and where it is said "if the
board. corporation permits" this means the same as "if the thing is permitted
by the directing power of the corporation."

It being determined that the corporation is bound by the contract in question,


it remains to consider the character of the liability assumed by R. J.
Fernandez, in affixing his personal signature to said contract. The question
here is whether Fernandez is liable jointly with the Orientalists Company as a
principal obligor, or whether his liability is that of a guarantor merely.

As appears upon the face of the contracts, the signature of Fernandez, in his
individual capacity, is not in line with the signature of the Orientalist
Company, but is set off to the left of the company's signature and somewhat
who sign contracts in some capacity other than that of principal obligor to
place their signature alone would justify a court in holding that Fernandez
here took upon himself the responsibility of a guarantor rather than that of a
principal obligor. We do, however, think, that the form in which the contract is
signed raises a doubt as to what the real intention was; and we feel justified,
in looking to the evidence to discover that intention. In this connection it is
entirely clear, from the testimony of both Ramirez and Ramon J. Fernandez,
that the responsibility of the latter was intended to be that of guarantor. There

815
is, to be sure, a certain difference between these witnesses as to the nature
of this guaranty, inasmuch as Fernandez would have us believe that his
name was signed as a guaranty that the contract would be approved by the
corporation, while Ramirez says that the name was put on the contract for
the purpose of guaranteeing, not the approval of the contract, but its
performance. We are convinced that the latter was the real intention of the
contracting parties.

We are not unmindful of the force of that rule of law which declares that oral
evidence is admissible to show the character in which the signature was
affixed. This conclusion is perhaps supported by the language of the second
paragraph of article 1281 of the Civil Code, which declares that if the words
of a contract should appear contrary to the evident intention of the parties,
the intention shall prevail. But the conclusion reached is, we think, deducible
from the general principle that in case of ambiguity parol evidence is
admissible to show the intention of the contracting parties.

It should be stated in conclusion that as the issues in this case have been
framed, the only question presented to this court is: To what extent are the
signatory parties to the contract liable to the plaintiff J. F. Ramirez? No
contentious issue is raised directly between the defendants, the Orientalist
Company and Ramon H. Fernandez; nor does the present the present action
involve any question as to the undertaking of Fernandez and his three
associates to effect the importation of the films upon their own account and
risk. Whether they may be bound to hold the company harmless is a matter
upon which we express no opinion.

The judgment appealed from is affirmed, with costs equally against the two
appellant. So ordered.

Torres, Johnson, Malcolm, Avancea and Fisher, JJ., concur.

816
SANCHEZ, J.:

The National Coconut Corporation (NACOCO, for short) was chartered as a


non-profit governmental organization on May 7, 1940 by Commonwealth Act
518 avowedly for the protection, preservation and development of the
coconut industry in the Philippines. On August 1, 1946, NACOCO's charter
was amended [Republic Act 5] to grant that corporation the express power
"to buy, sell, barter, export, and in any other manner deal in, coconut, copra,
and dessicated coconut, as well as their by-products, and to act as agent,
broker or commission merchant of the producers, dealers or merchants"
thereof. The charter amendment was enacted to stabilize copra prices, to
serve coconut producers by securing advantageous prices for them, to cut
down to a minimum, if not altogether eliminate, the margin of middlemen,
mostly aliens.4

General manager and board chairman was Maximo M. Kalaw; defendants


Juan Bocar and Casimiro Garcia were members of the Board; defendant
Leonor Moll became director only on December 22, 1947.

NACOCO, after the passage of Republic Act 5, embarked on copra trading


activities. Amongst the scores of contracts executed by general manager
Kalaw are the disputed contracts, for the delivery of copra, viz:

G.R. No. L-18805 August 14, 1967 (a) July 30, 1947: Alexander Adamson & Co., for 2,000 long tons,
$167.00: per ton, f. o. b., delivery: August and September, 1947. This
THE BOARD OF LIQUIDATORS1 representing THE GOVERNMENT OF contract was later assigned to Louis Dreyfus & Co. (Overseas) Ltd.
THE REPUBLIC OF THE PHILIPPINES, plaintiff-appellant,
vs. (b) August 14, 1947: Alexander Adamson & Co., for 2,000 long tons
HEIRS OF MAXIMO M. KALAW,2 JUAN BOCAR, ESTATE OF THE $145.00 per long ton, f.o.b., Philippine ports, to be shipped:
DECEASED CASIMIRO GARCIA, 3 and LEONOR MOLL, defendants- September-October, 1947. This contract was also assigned to Louis
appellees. Dreyfus & Co. (Overseas) Ltd.

Simeon M. Gopengco and Solicitor General for plaintiff-appellant. (c) August 22, 1947: Pacific Vegetable Co., for 3,000 tons, $137.50
L. H. Hernandez, Emma Quisumbing, Fernando and Quisumbing, Jr.; Ponce per ton, delivery: September, 1947.
Enrile, Siguion Reyna, Montecillo and Belo for defendants-appellees.

817
(d) September 5, 1947: Spencer Kellog & Sons, for 1,000 long tons, date. A meeting was then held. Kalaw made a full disclosure of the situation,
$160.00 per ton, c.i.f., Los Angeles, California, delivery: November, apprised the board of the impending heavy losses. No action was taken on
1947. the contracts. Neither did the board vote thereon at the meeting of January 7,
1948 following. Then, on January 11, 1948, President Roxas made a
(e) September 9, 1947: Franklin Baker Division of General Foods statement that the NACOCO head did his best to avert the losses,
Corporation, for 1,500 long tons, $164,00 per ton, c.i.f., New York, to emphasized that government concerns faced the same risks that confronted
be shipped in November, 1947. private companies, that NACOCO was recouping its losses, and that Kalaw
was to remain in his post. Not long thereafter, that is, on January 30, 1948,
(f) September 12, 1947: Louis Dreyfus & Co. (Overseas) Ltd., for the board met again with Kalaw, Bocar, Garcia and Moll in attendance. They
3,000 long tons, $154.00 per ton, f.o.b., 3 Philippine ports, delivery: unanimously approved the contracts hereinbefore enumerated.
November, 1947.
As was to be expected, NACOCO but partially performed the contracts, as
(g) September 13, 1947: Juan Cojuangco, for 2,000 tons, $175.00 follows:
per ton, delivery: November and December, 1947. This contract was
assigned to Pacific Vegetable Co.
Tons
(h) October 27, 1947: Fairwood & Co., for 1,000 tons, $210.00 per Buyers Undelivered
Delivered
short ton, c.i.f., Pacific ports, delivery: December, 1947 and January,
1948. This contract was assigned to Pacific Vegetable Co.

(i) October 28, 1947: Fairwood & Co., for 1,000 tons, $210.00 per Pacific Vegetable Oil 2,386.45 4,613.55
short ton, c.i.f., Pacific ports, delivery: January, 1948. This contract
was assigned to Pacific Vegetable Co.

An unhappy chain of events conspired to deter NACOCO from fulfilling these Spencer Kellog None 1,000
contracts. Nature supervened. Four devastating typhoons visited the
Philippines: the first in October, the second and third in November, and the
fourth in December, 1947. Coconut trees throughout the country suffered
extensive damage. Copra production decreased. Prices spiralled. Franklin Baker 1,000 500
Warehouses were destroyed. Cash requirements doubled. Deprivation of
export facilities increased the time necessary to accumulate shiploads of
copra. Quick turnovers became impossible, financing a problem.
Louis Dreyfus 800 2,200
When it became clear that the contracts would be unprofitable, Kalaw
submitted them to the board for approval. It was not until December 22, 1947
when the membership was completed. Defendant Moll took her oath on that

818
x x x However, in similar cases brought by the same claimant [Louis
Dreyfus & Co. (Overseas) Ltd.] against Santiago Syjuco for non-
Louis Dreyfus (Adamson contract of July 30,
1,150 850 delivery of copra also involving a claim of P345,654.68 wherein
1947)
defendant set up same defenses as above, plaintiff accepted
a promise of P5,000.00 only (Exhs. 31 & 32 Heirs.) Following the
same proportion, the claim of Dreyfus against NACOCO should have
been compromised for only P10,000.00, if at all. Now, why should
Louis Dreyfus (Adamson Contract of August
1,755 245 defendants be held liable for the large sum paid as compromise by
14, 1947)
the Board of Liquidators? This is just a sample to show how unjust it
would be to hold defendants liable for the readiness with which the
Board of Liquidators disposed of the NACOCO funds, although there
was much possibility of successfully resisting the claims, or at least
settlement for nominal sums like what happened in the Syjuco case.5

T O TALS 7,091.45 9,408.55 All the settlements sum up to P1,343,274.52.

In this suit started in February, 1949, NACOCO seeks to recover the above
The buyers threatened damage suits. Some of the claims were settled, viz: sum of P1,343,274.52 from general manager and board chairman Maximo
Pacific Vegetable Oil Co., in copra delivered by NACOCO, P539,000.00; M. Kalaw, and directors Juan Bocar, Casimiro Garcia and Leonor Moll. It
Franklin Baker Corporation, P78,210.00; Spencer Kellog & Sons, charges Kalaw with negligence under Article 1902 of the old Civil Code (now
P159,040.00. Article 2176, new Civil Code); and defendant board members, including
Kalaw, with bad faith and/or breach of trust for having approved the
But one buyer, Louis Dreyfus & Go. (Overseas) Ltd., did in fact sue before contracts. The fifth amended complaint, on which this case was tried, was
the Court of First Instance of Manila, upon claims as follows: For the filed on July 2, 1959. Defendants resisted the action upon defenses
undelivered copra under the July 30 contract (Civil Case 4459); P287,028.00; hereinafter in this opinion to be discussed.
for the balance on the August 14 contract (Civil Case 4398), P75,098.63; for
that per the September 12 contract reduced to judgment (Civil Case 4322, The lower court came out with a judgment dismissing the complaint without
appealed to this Court in L-2829), P447,908.40. These cases culminated in costs as well as defendants' counterclaims, except that plaintiff was ordered
an out-of-court amicable settlement when the Kalaw management was to pay the heirs of Maximo Kalaw the sum of P2,601.94 for unpaid salaries
already out. The corporation thereunder paid Dreyfus P567,024.52 and cash deposit due the deceased Kalaw from NACOCO.
representing 70% of the total claims. With particular reference to the Dreyfus
claims, NACOCO put up the defenses that: (1) the contracts were void Plaintiff appealed direct to this Court.
because Louis Dreyfus & Co. (Overseas) Ltd. did not have license to do
business here; and (2) failure to deliver was due to force majeure, the Plaintiff's brief did not, question the judgment on Kalaw's counterclaim for the
typhoons. To project the utter unreasonableness of this compromise, we sum of P2,601.94.
reproduce in haec verba this finding below:

819
Right at the outset, two preliminary questions raised before, but adversely affairs; and that, since the three year period has elapsed, the Board of
decided by, the court below, arrest our attention. On appeal, defendants Liquidators may not now continue with, and prosecute, the present case to its
renew their bid. And this, upon established jurisprudence that an appellate conclusion, because Executive Order 372 provides in Section 1 thereof that
court may base its decision of affirmance of the judgment below on a point or
points ignored by the trial court or in which said court was in error.6
Sec.1. The National Abaca and Other Fibers Corporation, the
1. First of the threshold questions is that advanced by defendants that National Coconut Corporation, the National Tobacco Corporation, the
plaintiff Board of Liquidators has lost its legal personality to continue with this National Food Producer Corporation and the former enemy-owned or
suit. controlled corporations or associations, . . . are hereby abolished.
The said corporations shall be liquidated in accordance with law, the
Accepted in this jurisdiction are three methods by which a corporation may provisions of this Order, and/or in such manner as the President of
wind up its affairs: (1) under Section 3, Rule 104, of the Rules of Court [which the Philippines may direct; Provided, however, That each of the said
superseded Section 66 of the Corporation Law] 7 whereby, upon voluntary corporations shall nevertheless be continued as a body corporate for
dissolution of a corporation, the court may direct "such disposition of its a period of three (3) years from the effective date of this Executive
assets as justice requires, and may appoint a receiver to collect such Order for the purpose of prosecuting and defending suits by or
assets and pay the debts of the corporation;" against it and of enabling the Board of Liquidators gradually to settle
and close its affairs, to dispose of and, convey its property in the
(2) under Section 77 of the Corporation Law, whereby a corporation whose manner hereinafter provided.
corporate existence is terminated, "shall nevertheless be continued as a
body corporate for three years after the time when it would have been Citing Mr. Justice Fisher, defendants proceed to argue that even where it
so dissolved, for the purpose of prosecuting and defending suits by or may be found impossible within the 3 year period to reduce disputed claims
against it and of enabling it gradually to settle and close its affairs, to to judgment, nonetheless, "suits by or against a corporation abate when it
dispose of and convey its property and to divide its capital stock, but ceases to be an entity capable of suing or being sued" (Fisher, The Philippine
not for the purpose of continuing the business for which it was Law of Stock Corporations, pp. 390-391). Corpus Juris Secundum likewise is
established;" and authority for the statement that "[t]he dissolution of a corporation ends its
existence so that there must be statutory authority for prolongation of its
(3) under Section 78 of the Corporation Law, by virtue of which the life even for purposes of pending litigation"9 and that suit "cannot be
corporation, within the three year period just mentioned, "is authorized and continued or revived; nor can a valid judgment be rendered therein, and a
empowered to convey all of its property to trustees for the benefit of judgment, if rendered, is not only erroneous, but void and subject to collateral
members, stockholders, creditors, and others interested."8 attack." 10 So it is, that abatement of pending actions follows as a matter of
course upon the expiration of the legal period for liquidation, 11 unless the
statute merely requires a commencement of suit within the added
It is defendants' pose that their case comes within the coverage of the
time. 12 For, the court cannot extend the time alloted by statute. 13
second method. They reason out that suit was commenced in February,
1949; that by Executive Order 372, dated November 24, 1950, NACOCO,
together with other government-owned corporations, was abolished, and the We, however, express the view that the executive order abolishing NACOCO
Board of Liquidators was entrusted with the function of settling and closing its and creating the Board of Liquidators should be examined in context. The
proviso in Section 1 of Executive Order 372, whereby the corporate

820
existence of NACOCO was continued for a period of three years from the first judicial test before this Court is National Abaca and Other Fibers
effectivity of the order for "the purpose of prosecuting and defending suits by Corporation vs. Pore, L-16779, August 16, 1961. In that case, the
or against it and of enabling the Board of Liquidators gradually to settle and corporation, already dissolved, commenced suit within the three-year
close its affairs, to dispose of and convey its property in the manner extended period for liquidation. That suit was for recovery of money
hereinafter provided", is to be read not as an isolated provision but in advanced to defendant for the purchase of hemp in behalf of the corporation.
conjunction with the whole. So reading, it will be readily observed that no She failed to account for that money. Defendant moved to dismiss,
time limit has been tacked to the existence of the Board of Liquidators and its questioned the corporation's capacity to sue. The lower court ordered plaintiff
function of closing the affairs of the various government owned corporations, to include as co-party plaintiff, The Board of Liquidators, to which the
including NACOCO. corporation's liquidation was entrusted by Executive Order 372. Plaintiff failed
to effect inclusion. The lower court dismissed the suit. Plaintiff moved to
By Section 2 of the executive order, while the boards of directors of the reconsider. Ground: excusable negligence, in that its counsel prepared the
various corporations were abolished, their powers and functions and duties amended complaint, as directed, and instructed the board's incoming and
under existing laws were to be assumed and exercised by the Board of outgoing correspondence clerk, Mrs. Receda Vda. de Ocampo, to mail the
Liquidators. The President thought it best to do away with the boards of original thereof to the court and a copy of the same to defendant's counsel.
directors of the defunct corporations; at the same time, however, the She mailed the copy to the latter but failed to send the original to the court.
President had chosen to see to it that the Board of Liquidators step into the This motion was rejected below. Plaintiff came to this Court on appeal. We
vacuum. And nowhere in the executive order was there any mention of the there said that "the rule appears to be well settled that, in the absence of
lifespan of the Board of Liquidators. A glance at the other provisions of the statutory provision to the contrary, pending actions by or against a
executive order buttresses our conclusion. Thus, liquidation by the Board of corporation are abated upon expiration of the period allowed by law for the
Liquidators may, under section 1, proceed in accordance with law, the liquidation of its affairs." We there said that "[o]ur Corporation Law contains
provisions of the executive order, "and/or in such manner as the President of no provision authorizing a corporation, after three (3) years from the
the Philippines may direct." By Section 4, when any property, fund, or project expiration of its lifetime, to continue in its corporate name actions instituted
is transferred to any governmental instrumentality "for administration or by it within said period of three (3) years." 14 However, these precepts
continuance of any project," the necessary funds therefor shall be taken from notwithstanding, we, in effect, held in that case that the Board of Liquidators
the corresponding special fund created in Section 5. Section 5, in turn, talks escapes from the operation thereof for the reason that "[o]bviously, the
of special funds established from the "net proceeds of the liquidation" of the complete loss of plaintiff's corporate existence after the expiration of the
various corporations abolished. And by Section, 7, fifty per centum of the period of three (3) years for the settlement of its affairs is what impelled the
fees collected from the copra standardization and inspection service shall President to create a Board of Liquidators, to continue the management of
accrue "to the special fund created in section 5 hereof for the rehabilitation such matters as may then be pending." 15 We accordingly directed the record
and development of the coconut industry." Implicit in all these, is that the term of said case to be returned to the lower court, with instructions to admit
of life of the Board of Liquidators is without time limit. Contemporary history plaintiff's amended complaint to include, as party plaintiff, the Board of
gives us the fact that the Board of Liquidators still exists as an office with Liquidators.
officials and numerous employees continuing the job of liquidation and
prosecution of several court actions. Defendants' position is vulnerable to attack from another direction.

Not that our views on the power of the Board of Liquidators to proceed to the By Executive Order 372, the government, the sole stockholder, abolished
final determination of the present case is without jurisprudential support. The NACOCO, and placed its assets in the hands of the Board of Liquidators.

821
The Board of Liquidators thus became the trustee on behalf of the The leading expositor of the law on this point is Aguas vs. Llemos, L-18107,
government. It was an express trust. The legal interest became vested in the August 30, 1962. There, plaintiffs sought to recover damages from defendant
trustee the Board of Liquidators. The beneficial interest remained with the Llemos. The complaint averred that Llemos had served plaintiff by registered
sole stockholder the government. At no time had the government mail with a copy of a petition for a writ of possession in Civil Case 4824 of the
withdrawn the property, or the authority to continue the present suit, from the Court of First Instance at Catbalogan, Samar, with notice that the same
Board of Liquidators. If for this reason alone, we cannot stay the hand of the would be submitted to the Samar court on February 23, 1960 at 8:00 a.m.;
Board of Liquidators from prosecuting this case to its final conclusion. 16 The that in view of the copy and notice served, plaintiffs proceeded to the said
provisions of Section 78 of the Corporation Law the third method of court of Samar from their residence in Manila accompanied by their lawyers,
winding up corporate affairs find application. only to discover that no such petition had been filed; and that defendant
Llemos maliciously failed to appear in court, so that plaintiffs' expenditure
We, accordingly, rule that the Board of Liquidators has personality to proceed and trouble turned out to be in vain, causing them mental anguish and undue
as: party-plaintiff in this case. embarrassment. Defendant died before he could answer the complaint. Upon
leave of court, plaintiffs amended their complaint to include the heirs of the
2. Defendants' second poser is that the action is unenforceable against the deceased. The heirs moved to dismiss. The court dismissed the complaint on
heirs of Kalaw. the ground that the legal representative, and not the heirs, should have been
made the party defendant; and that, anyway, the action being for recovery of
money, testate or intestate proceedings should be initiated and the claim filed
Appellee heirs of Kalaw raised in their motion to dismiss, 17 which was
therein. This Court, thru Mr. Justice Jose B. L. Reyes, there declared:
overruled, and in their nineteenth special defense, that plaintiff's action is
personal to the deceased Maximo M. Kalaw, and may not be deemed to
have survived after his death. 18 They say that the controlling statute is Plaintiffs argue with considerable cogency that contrasting the
Section 5, Rule 87, of the 1940 Rules of Court. 19 which provides that "[a]ll correlated provisions of the Rules of Court, those concerning claims
claims for money against the decedent, arising from contract, express or that are barred if not filed in the estate settlement proceedings (Rule
implied", must be filed in the estate proceedings of the deceased. We 87, sec. 5) and those defining actions that survive and may be
disagree. prosecuted against the executor or administrator (Rule 88, sec. 1), it
is apparent that actions for damages caused by tortious conduct of a
defendant (as in the case at bar) survive the death of the latter.
The suit here revolves around the alleged negligent acts of Kalaw for having
Under Rule 87, section 5, the actions that are abated by death are:
entered into the questioned contracts without prior approval of the board of
(1) claims for funeral expenses and those for the last sickness of the
directors, to the damage and prejudice of plaintiff; and is against Kalaw and
decedent; (2) judgments for money; and (3) "all claims for money
the other directors for having subsequently approved the said contracts in
against the decedent, arising from contract express or implied." None
bad faith and/or breach of trust." Clearly then, the present case is not a mere
of these includes that of the plaintiffs-appellants; for it is not enough
action for the recovery of money nor a claim for money arising from contract.
that the claim against the deceased party be for money, but it must
The suit involves alleged tortious acts. And the action is embraced in suits
arise from "contract express or implied", and these words (also used
filed "to recover damages for an injury to person or property, real or
by the Rules in connection with attachments and derived from the
personal", which survive. 20
common law) were construed in Leung Ben vs. O'Brien, 38 Phil. 182,
189-194,

822
"to include all purely personal obligations other than those "intrusted with the general management and control of its business,
which have their source in delict or tort." has implied authority to make any contract or do any other act which is
necessary or appropriate to the conduct of the ordinary business of the
Upon the other hand, Rule 88, section 1, enumerates actions that corporation. 21As such officer, "he may, without any special authority
survive against a decedent's executors or administrators, and they from the Board of Directors perform all acts of an ordinary nature,
are: (1) actions to recover real and personal property from the estate; which by usage or necessity are incident to his office, and may bind the
(2) actions to enforce a lien thereon; and (3) actions to recover corporation by contracts in matters arising in the usual course of
damages for an injury to person or property. The present suit is one business. 22
for damages under the last class, it having been held that "injury to
property" is not limited to injuries to specific property, but extends to The problem, therefore, is whether the case at bar is to be taken out of the
other wrongs by which personal estate is injured or diminished general concept of the powers of a general manager, given the cited
(Baker vs. Crandall, 47 Am. Rep. 126; also 171 A.L.R., 1395). To provision of the NACOCO by-laws requiring prior directorate approval of
maliciously cause a party to incur unnecessary expenses, as NACOCO contracts.
charged in this case, is certainly injury to that party's property (Javier
vs. Araneta, L-4369, Aug. 31, 1953). The peculiar nature of copra trading, at this point, deserves express
articulation. Ordinary in this enterprise are copra sales for future delivery. The
The ruling in the preceding case was hammered out of facts comparable to movement of the market requires that sales agreements be entered into,
those of the present. No cogent reason exists why we should break away even though the goods are not yet in the hands of the seller. Known in
from the views just expressed. And, the conclusion remains: Action against business parlance as forward sales, it is concededly the practice of the trade.
the Kalaw heirs and, for the matter, against the Estate of Casimiro Garcia A certain amount of speculation is inherent in the undertaking. NACOCO was
survives. much more conservative than the exporters with big capital. This short-selling
was inevitable at the time in the light of other factors such as availability of
The preliminaries out of the way, we now go to the core of the controversy. vessels, the quantity required before being accepted for loading, the labor
needed to prepare and sack the copra for market. To NACOCO, forward
3. Plaintiff levelled a major attack on the lower court's holding that Kalaw sales were a necessity. Copra could not stay long in its hands; it would lose
justifiedly entered into the controverted contracts without the prior approval of weight, its value decrease. Above all, NACOCO's limited funds necessitated
the corporation's directorate. Plaintiff leans heavily on NACOCO's corporate a quick turnover. Copra contracts then had to be executed on short notice
by-laws. Article IV (b), Chapter III thereof, recites, as amongst the duties at times within twenty-four hours. To be appreciated then is the difficulty of
of the general manager, the obligation: "(b) To perform or execute on calling a formal meeting of the board.
behalf of the Corporation upon prior approval of the Board, all
contracts necessary and essential to the proper accomplishment for Such were the environmental circumstances when Kalaw went into copra
which the Corporation was organized." trading.

Not of de minimis importance in a proper approach to the problem at hand, is Long before the disputed contracts came into being, Kalaw contracted by
the nature of a general manager's position in the corporate structure. A rule himself alone as general manager for forward sales of copra. For
that has gained acceptance through the years is that a corporate officer the fiscal year ending June 30, 1947, Kalaw signed some 60 such contracts

823
for the sale of copra to divers parties. During that period, from those copra And more. On December 19, 1946, the board resolved to ratify the brokerage
sales, NACOCO reaped a gross profit of P3,631,181.48. So pleased was commission of 2% of Smith, Bell and Co., Ltd., in the sale of 4,300 long tons
NACOCO's board of directors that, on December 5, 1946, in Kalaw's of copra to the French Government. Such ratification was necessary
absence, it voted to grant him a special bonus "in recognition of the signal because, as stated by Kalaw in that same meeting, "under an existing
achievement rendered by him in putting the Corporation's business on a self- resolution he is authorized to give a brokerage fee of only 1% on sales of
sufficient basis within a few months after assuming office, despite numerous copra made through brokers." On January 15, 1947, the brokerage fee
handicaps and difficulties." agreements of 1-1/2% on three export contracts, and 2% on three others, for
the sale of copra were approved by the board with a proviso authorizing the
These previous contract it should be stressed, were signed by Kalaw without general manager to pay a commission up to the amount of 1-1/2% "without
prior authority from the board. Said contracts were known all along to further action by the Board." On February 5, 1947, the brokerage fee of 2%
the board members. Nothing was said by them. The aforesaid contracts of J. Cojuangco & Co. on the sale of 2,000 tons of copra was favorably acted
stand to prove one thing: Obviously, NACOCO board met the upon by the board. On March 19, 1947, a 2% brokerage commission was
difficulties attendant to forward sales by leaving the adoption of means similarly approved by the board for Pacific Trading Corporation on the sale of
to end, to the sound discretion of NACOCO's general manager Maximo 2,000 tons of copra.
M. Kalaw.
It is to be noted in the foregoing cases that only the brokerage fee
Liberally spread on the record are instances of contracts executed by agreements were passed upon by the board, not the sales contracts
NACOCO's general manager and submitted to the board after their themselves. And even those fee agreements were submitted only when the
consummation, not before. These agreements were not Kalaw's alone. One commission exceeded the ceiling fixed by the board.
at least was executed by a predecessor way back in 1940, soon after
NACOCO was chartered. It was a contract of lease executed on November Knowledge by the board is also discernible from other recorded
16, 1940 by the then general manager and board chairman, Maximo instances.1wph1.t
Rodriguez, and A. Soriano y Cia., for the lease of a space in Soriano Building
On November 14, 1946, NACOCO, thru its general manager Kalaw, sold When the board met on May 10, 1947, the directors discussed the copra
3,000 tons of copra to the Food Ministry, London, thru Sebastian Palanca. situation: There was a slow downward trend but belief was entertained that
On December 22, 1947, when the controversy over the present contract the nadir might have already been reached and an improvement in prices
cropped up, the board voted to approve a lease contract previously executed was expected. In view thereof, Kalaw informed the board that "he intends to
between Kalaw and Fidel Isberto and Ulpiana Isberto covering a warehouse wait until he has signed contracts to sell before starting to buy copra."23
of the latter. On the same date, the board gave its nod to a contract for
renewal of the services of Dr. Manuel L. Roxas. In fact, also on that date, the In the board meeting of July 29, 1947, Kalaw reported on the copra price
board requested Kalaw to report for action all copra contracts signed by him conditions then current: The copra market appeared to have become fairly
"at the meeting immediately following the signing of the contracts." This steady; it was not expected that copra prices would again rise very high as in
practice was observed in a later instance when, on January 7, 1948, the the unprecedented boom during January-April, 1947; the prices seemed to
board approved two previous contracts for the sale of 1,000 tons of copra oscillate between $140 to $150 per ton; a radical rise or decrease was not
each to a certain "SCAP" and a certain "GNAPO". indicated by the trends. Kalaw continued to say that "the Corporation has

824
been closing contracts for the sale of copra generally with a margin of P5.00 x x x authority to act for and bind a corporation may be
to P7.00 per hundred kilos." 24 presumed from acts of recognition in other instances where the
power was in fact exercised. 28
We now lift the following excerpts from the minutes of that same board
meeting of July 29, 1947: x x x Thus, when, in the usual course of business of a
corporation, an officer has been allowed in his official capacity
521. In connection with the buying and selling of copra the Board to manage its affairs, his authority to represent the corporation
inquired whether it is the practice of the management to close may be implied from the manner in which he has been permitted
contracts of sale first before buying. The General Manager replied by the directors to manage its business.29
that this practice is generally followed but that it is not always
possible to do so for two reasons: In the case at bar, the practice of the corporation has been to allow its
general manager to negotiate and execute contracts in its copra trading
(1) The role of the Nacoco to stabilize the prices of copra requires activities for and in NACOCO's behalf without prior board approval. If the by-
that it should not cease buying even when it does not have actual laws were to be literally followed, the board should give its stamp of prior
contracts of sale since the suspension of buying by the Nacoco will approval on all corporate contracts. But that board itself, by its acts and
result in middlemen taking advantage of the temporary inactivity of through acquiescence, practically laid aside the by-law requirement of prior
the Corporation to lower the prices to the detriment of the producers. approval.

(2) The movement of the market is such that it may not be practical Under the given circumstances, the Kalaw contracts are valid corporate acts.
always to wait for the consummation of contracts of sale before
beginning to buy copra. 4. But if more were required, we need but turn to the board's ratification of
the contracts in dispute on January 30, 1948, though it is our (and the lower
The General Manager explained that in this connection a certain court's) belief that ratification here is nothing more than a mere formality.
amount of speculation is unavoidable. However, he said that the
Nacoco is much more conservative than the other big exporters in Authorities, great in number, are one in the idea that "ratification by a
this respect.25 corporation of an unauthorized act or contract by its officers or others relates
back to the time of the act or contract ratified, and is equivalent to original
Settled jurisprudence has it that where similar acts have been approved authority;" and that " [t]he corporation and the other party to the transaction
by the directors as a matter of general practice, custom, and policy, the are in precisely the same position as if the act or contract had been
general manager may bind the company without formal authorization of authorized at the time." 30 The language of one case is expressive: "The
the board of directors. 26 In varying language, existence of such adoption or ratification of a contract by a corporation is nothing more or less
authority is established, by proof of the course of business, the usage than the making of an original contract. The theory of corporate ratification
and practices of the company and by the knowledge which the board of is predicated on the right of a corporation to contract, and any ratification or
directors has, or must be presumed to have, of acts and doings of its adoption is equivalent to a grant of prior authority." 31
subordinates in and about the affairs of the corporation. 27So also,

825
Indeed, our law pronounces that "[r]atification cleanses the contract from all approval from Judge Sharswood (in Spering's App., 71 Pa. 11), the following:
its defects from the moment it was constituted." 32 By corporate confirmation, "Upon a close examination of all the reported cases, although there are many
the contracts executed by Kalaw are thus purged of whatever vice or defect dicta not easily reconcilable, yet I have found no judgment or decree which
they may have. 33 has held directors to account, except when they have themselves been
personally guilty of some fraud on the corporation, or have known and
In sum, a case is here presented whereunder, even in the face of an express connived at some fraud in others, or where such fraud might have been
by-law requirement of prior approval, the law on corporations is not to be prevented had they given ordinary attention to their duties. . . ." Plaintiff did
held so rigid and inflexible as to fail to recognize equitable considerations. not even dare charge its defendant-directors with any of these malevolent
And, the conclusion inevitably is that the embattled contracts remain valid. acts.

5. It would be difficult, even with hostile eyes, to read the record in terms of Obviously, the board thought that to jettison Kalaw's contracts would
"bad faith and/or breach of trust" in the board's ratification of the contracts contravene basic dictates of fairness. They did not think of raising their voice
without prior approval of the board. For, in reality, all that we have on the in protest against past contracts which brought in enormous profits to the
government's side of the scale is that the board knew that the contracts so corporation. By the same token, fair dealing disagrees with the idea that
confirmed would cause heavy losses. similar contracts, when unprofitable, should not merit the same treatment.
Profit or loss resulting from business ventures is no justification for turning
As we have earlier expressed, Kalaw had authority to execute the contracts one's back on contracts entered into. The truth, then, of the matter is that
without need of prior approval. Everybody, including Kalaw himself, thought in the words of the trial court the ratification of the contracts was "an act of
so, and for a long time. Doubts were first thrown on the way only when the simple justice and fairness to the general manager and the best interest of
contracts turned out to be unprofitable for NACOCO. the corporation whose prestige would have been seriously impaired by a
rejection by the board of those contracts which proved disadvantageous." 37
Rightfully had it been said that bad faith does not simply connote bad 38
judgment or negligence; it imports a dishonest purpose or some moral The directors are not liable."
obliquity and conscious doing of wrong; it means breach of a known duty thru
some motive or interest or ill will; it partakes of the nature of fraud. 34 Applying 6. To what then may we trace the damage suffered by NACOCO.
this precept to the given facts herein, we find that there was no "dishonest
purpose," or "some moral obliquity," or "conscious doing of wrong," or The facts yield the answer. Four typhoons wreaked havoc then on our copra-
"breach of a known duty," or "Some motive or interest or ill will" that "partakes producing regions. Result: Copra production was impaired, prices spiralled,
of the nature of fraud." warehouses destroyed. Quick turnovers could not be expected. NACOCO
was not alone in this misfortune. The record discloses that private traders,
Nor was it even intimated here that the NACOCO directors acted for personal old, experienced, with bigger facilities, were not spared; also suffered
reasons, or to serve their own private interests, or to pocket money at the tremendous losses. Roughly estimated, eleven principal trading concerns did
expense of the corporation. 35 We have had occasion to affirm that bad faith run losses to about P10,300,000.00. Plaintiff's witness Sisenando Barretto,
contemplates a "state of mind affirmatively operating with furtive design or head of the copra marketing department of NACOCO, observed that from
with some motive of self-interest or ill will or for ulterior purposes." 36 Briggs late 1947 to early 1948 "there were many who lost money in the
vs. Spaulding, 141 U.S. 132, 148-149, 35 L. ed. 662, 669, quotes with trade." 39 NACOCO was not immune from such usual business risk.

826
The typhoons were known to plaintiff. In fact, NACOCO resisted the suits the corporation; that he entered into the contracts in pursuance of an overall
filed by Louis Dreyfus & Co. by pleading in its answers force majeure as an policy to stabilize prices, to free the producers from the clutches of the
affirmative defense and there vehemently asserted that "as a result of the middlemen. The prices for which NACOCO contracted in the disputed
said typhoons, extensive damage was caused to the coconut trees in the agreements, were at a level calculated to produce profits and higher than
copra producing regions of the Philippines and according to estimates of those prevailing in the local market. Plaintiff's witness, Barretto, categorically
competent authorities, it will take about one year until the coconut producing stated that "it would be foolish to think that one would sign (a) contract when
regions will be able to produce their normal coconut yield and it will take you are going to lose money" and that no contract was executed "at a price
some time until the price of copra will reach normal levels;" and that "it had unsafe for the Nacoco." 45 Really, on the basis of prices then prevailing,
never been the intention of the contracting parties in entering into the NACOCO envisioned a profit of around P752,440.00. 46
contract in question that, in the event of a sharp rise in the price of copra in
the Philippine market produce byforce majeure or by caused beyond Kalaw's acts were not the result of haphazard decisions either. Kalaw
defendant's control, the defendant should buy the copra contracted for at invariably consulted with NACOCO's Chief Buyer, Sisenando Barretto, or the
exorbitant prices far beyond the buying price of the plaintiff under the Assistant General Manager. The dailies and quotations from abroad were
contract." 40 guideposts to him.

A high regard for formal judicial admissions made in court pleadings would Of course, Kalaw could not have been an insurer of profits. He could not be
suffice to deter us from permitting plaintiff to stray away therefrom, to charge expected to predict the coming of unpredictable typhoons. And even as
now that the damage suffered was because of Kalaw's negligence, or for that typhoons supervened Kalaw was not remissed in his duty. He exerted efforts
matter, by reason of the board's ratification of the contracts. 41 to stave off losses. He asked the Philippine National Bank to implement its
commitment to extend a P400,000.00 loan. The bank did not release the
Indeed, were it not for the typhoons, 42 NACOCO could have, with ease, met loan, not even the sum of P200,000.00, which, in October, 1947, was
its contractual obligations. Stock accessibility was no problem. NACOCO had approved by the bank's board of directors. In frustration, on December 12,
90 buying agencies spread throughout the islands. It could purchase 2,000 1947, Kalaw turned to the President, complained about the bank's short-
tons of copra a day. The various contracts involved delivery of but 16,500 sighted policy. In the end, nothing came out of the negotiations with the bank.
tons over a five-month period. Despite the typhoons, NACOCO was still able NACOCO eventually faltered in its contractual obligations.
to deliver a little short of 50% of the tonnage required under the contracts.
That Kalaw cannot be tagged with crassa negligentia or as much as simple
As the trial court correctly observed, this is a case of damnum absque injuria. negligence, would seem to be supported by the fact that even as the
Conjunction of damage and wrong is here absent. There cannot be an contracts were being questioned in Congress and in the NACOCO board
actionable wrong if either one or the other is wanting. 43 itself, President Roxas defended the actuations of Kalaw. On December 27,
1947, President Roxas expressed his desire "that the Board of Directors
7. On top of all these, is that no assertion is made and no proof is presented should reelect Hon. Maximo M. Kalaw as General Manager of the National
which would link Kalaw's acts ratified by the board to a matrix for Coconut Corporation." 47 And, on January 7, 1948, at a time when the
defraudation of the government. Kalaw is clear of the stigma of bad faith. contracts had already been openly disputed, the board, at its regular
Plaintiff's corporate counsel 44 concedes that Kalaw all along thought that he meeting, appointed Maximo M. Kalaw as acting general manager of the
had authority to enter into the contracts, that he did so in the best interests of corporation.

827
Well may we profit from the following passage from Montelibano vs.
Bacolod-Murcia Milling Co., Inc., L-15092, May 18, 1962:

"They (the directors) hold such office charged with the duty to act for
the corporation according to their best judgment, and in so doing they
cannot be controlled in the reasonable exercise and performance of
such duty. Whether the business of a corporation should be operated
at a loss during a business depression, or closed down at a smaller
loss, is a purely business and economic problem to be determined by
the directors of the corporation, and not by the court. It is a well known
rule of law that questions of policy of management are left solely to the
honest decision of officers and directors of a corporation, and the court
is without authority to substitute its judgment for the judgment of the
board of directors; the board is the business manager of the 0G.R. No. L-20333 June 30, 1967
corporation, and solong as it acts in good faith its orders are not
reviewable by the courts." (Fletcher on Corporations, Vol. 2, p. 390.) 48 EMILIANO ACUA, plaintiff-appellant,
vs.
Kalaw's good faith, and that of the other directors, clinch the case for BATAC PRODUCERS COOPERATIVE MARKETING ASSOCIATION, INC.,
defendants. 49 JUSTINO GALANO, TEODORO NARCISO, PABLO BACTIN, (DR.)
EMMANUEL BUMANGLAG, VENANCIO DIRIC, MARCOS ESQUIVEL,
Viewed in the light of the entire record, the judgment under review must be, EVARISTO CAOILI, FIDEL BATTULAYAN, DAMIAN ROSSINI,
as it is hereby, affirmed. RAYMUNDO BATALLONES, PLACIDO QUIAOIT, and LEON Q.
VERANO defendants-appellees.
Without costs. So ordered.
Marquez and Marquez for plaintiff-appellant.
Estanislao A. Fernandez for defendants-appellees.
Reyes, J.B.L., Makalintal, Bengzon, J.P., Zaldivar, Castro and Angeles, JJ.,
concur.
Fernando, J., took no part. MAKALINTAL, J.:
Concepcion, C.J. and Dizon, J., are on leave.
Appeal taken from the order dated September 10, 1962 of the Court of First
Instance of Rizal, Branch V (Quezon City) dismissing plaintiff's complaint on
the ground that it states no cause of action, and discharging the writ of
preliminary attachment issued therein.

On August 9, 1962, plaintiff Emiliano Acua filed a complaint, which was later
amended on August 13, against the defendant Batac Producers Cooperative

828
Marketing Association, Inc., hereinafter called the Batac Procoma, Inc., or that a formal approval of said "Agreement" by the Board was no longer
alternatively, against all the other defendants named in the caption. The necessary, as it was a mere "formality" appended to its authorizing resolution
complaint alleged, inter alia, that on or about May 5, 1962 it was tentatively and as all the members of the Board had already agreed to the same; that on
agreed upon between plaintiff and defendant Leon Q. Verano, as Manager of the same date, May 10, 1962, plaintiff gave and turned over to the defendant
the defendant Batac Procoma, Inc., that the former would seek and obtain corporation, thru its treasurer, Dominador T. Cocson the sum of P20,000.00,
the sum of not less, than P20,000.00 to be advanced to the defendant Batac in the presence of defendants Leon Q. Verano, Justino Galano, Dr.
Procoma, Inc., to be utilized by it as additional funds for its Virginia tobacco Emmanuel Bumanglag and Atty. Fernando Alcantara, for which said treasurer
buying operations during the current redrying season; that plaintiff would be issued to plaintiff its corresponding Official Receipt No. 130852; that from
constituted as the corporation's representative in Manila to assist in handling then on, plaintiff diligently and religiously kept his part of the "Agreement;"
and facilitating its continuous shipments of tobacco and their delivery to the that plaintiff even furnished the defendant corporation, upon request of its
redrying plants and in speeding up the prompt payment and collection of all Manager Leon Q. Verano three thousand (3,000) sacks which it utilized in the
amounts due to the corporation for such shipments; that for his services shipment of its tobacco costing P6,000.00 and that plaintiff had personally
plaintiff would be paid a remuneration at the rate of P0.50 per kilo of tobacco; advanced out of his own personal funds the total sum of P5,000.00 with the
that said tentative agreement was favorably received by the Board of full knowledge, acquiescence and consent of all the individual defendants;
Directors of the defendant Batac Procoma Inc., and on May 6, 1962 all the that after the defendant corporation was enabled to replenish its funds with
defendants named above, who constituted the entire Board of Directors of continuous collections from the PVTA for tobacco delivered due to the help,
said corporation (except Leon Q. Verano, who was its Manager), together assistance and intervention of plaintiff, for which the said corporation
with defendants Justino Galano and Teodoro Narciso, as President and Vice- collected from the PVTA the total sum of P381,495.00, the "Agreement" was
President, respectively, unanimously authorized defendant Leon Q. Verano, disapproved by its Board of Directors on June 6, 1962. Upon the foregoing
by a formal resolution, "to execute any agreement with any person or entity, allegations plaintiff prays: (a) that an order of attachment be issued against
on behalf of the corporation, for the purpose of securing additional funds for the properties of defendant corporation; (b) that after due trial, judgment be
the corporation, as well as to secure the services of such person or entity, in rendered condemning defendant corporation, or alternatively, all the other
the collection of all payments due to the corporation from the PVTA for any individual defendants, jointly and severally, to comply with their contractual
tobacco sold and delivered to said administration; giving and conferring upon obligations and to pay plaintiff the sum of P300,000.00 for his services, plus
the Manager, full and complete authority to bind the corporation with such P31,000.00 for cash advances made by him and P25,000.00 for attorney's
person or entity in any agreement, and under such considerations, which the fees.
said Manager may deem expedient and necessary for that purpose; that
plaintiff was made to understand by all of said defendants that the original On August 14, 1962, the lower court ordered the issuance of a writ of
understanding between him and defendant Leon Q. Verano was acceptable preliminary attachment against the properties of the defendants and on the
to the corporation, except that the remuneration for the plaintiff's services following day, after the plaintiff had posted the required bond, the writ was
would be P0.30 per kilo of tobacco; that on May 10, 1962, the formal accordingly issued by the Clerk of Court.1wph1.t
"Agreement" was executed between plaintiff and defendant Leon Q. Verano,
as Manager of the defendant corporation, duly authorized by its Board of On August 22, 1962, the defendants filed a motion to dismiss the complaint
Directors for such purpose, and signed by defendants Justino Galano and Dr. on the ground that it stated no cause of action and to discharge the
Emmanuel Bumanglag as instrumental witnesses and acknowledged by Atty. preliminary attachment on the ground that it was improperly or irregularly
Fernando Alcantara, the Secretary and Legal Counsel of the defendant issued. In support of the motion defendants alleged that the contract for
corporation; that upon plaintiff's inquiry, he was assured by these defendants services was never perfected because it was not approved or ratified but was

829
instead disapproved by the Board of Directors of defendant Batac Procoma, It is a settled principle that when a motion to dismiss is based on the ground
Inc., and that on the basis of plaintiff's pleadings the contract is void and that the complaint does not state a cause of action (Rule 8, Section 1, par. 7
unenforceable. Defendants further denied the fact that plaintiff had performed of the old Rules; Rule 16, Section 1., par. [g] of the Revised Rules) the
his part of the contract, alleging that he had not in any manner intervened in averments in the complaint are deemed hypothetically admitted and the
the delivery and payment of tobacco pertaining to the defendant corporation. inquiry is limited to whether or not they make out a case on which relief can
be granted. If said motion assails directly or indirectly the veracity of the
On August 25, 1962, plaintiff filed a written opposition to the motion to allegations, it is improper to grant the motion upon the assumption that the
dismiss and to discharge the preliminary attachment. averments therein are true and those of the complaint are not (Carreon vs.
Prov. Board of Pampanga, 52 O.G. 6557.) The sufficiency of the motion
On September 10, 1962, the trial court sustained defendants' motion and should be tested on the strength of the allegations of facts contained in the
issued the following order: complaint, and no other. If these allegations show a cause of action, or
furnish sufficient basis by which the complaint can be maintained, the
complaint should not be dismissed regardless of the defenses that may be
In resume the Court believes that the complaint states no cause of
averred by the defendants. (Josefa de Jesus, et al. vs. Santos Belarmino, 50
action and that contract in question is void ab initio.
O.G. 3004-3068; Verzosa vs. Rigonan, G.R. No. L-6459, April 23, 1954;
Dimayuga vs. Dimayuga, 51 O.G. 2397-2400.)
IN VIEW OF THE FOREGOING, the amended complaint filed in this
case is hereby ordered DISMISSED, without special pronouncement
The first ground upon which the order of dismissal issued by the lower court
as to costs. Consequently, the writ of preliminary attachment issued
is predicated is that the Board of Directors of defendant corporation did not
herein is ordered discharged. However, it is of record that the
approve, the agreement in question in fact disapproved it by a resolution
defendants has (sic) deposited the Court the amount of P20,400.00
passed on June 6, 1962 and that as a consequence the "suspensive
representing the amount of money invested by the plaintiff plus the
condition" attached to the agreement was never fulfilled. The specific
corresponding interest thereon. Plaintiff, by virtue of this order, may
stipulation referred to by the Court as a suspensive condition states:
withdraw the same in due time, if he so desires, upon proper receipt
"provided, however that the contract entered into by said manager to carry
therefor.
out the purposes above-mentioned shall be subject to the approve by the
Board."
From the foregoing order plaintiff interposed the present appeal.
A perusal of the complaint reveals that it contains sufficient allegations
Appellant has assigned four errors, which we shall consider seriatim: indicating such approval or at least subsequent ratification. On the first point
we note the following averments: that on May 9th the plaintiff met with each
The first assignment reads: "As the defendants' motion to dismiss the and all of the individual defendants (who constituted the entire Board of
complaint and to discharge the preliminary attachment was based on the Directors) and discussed with them extensively the tentative agreement and
specific ground that the complaint states no cause of action (Sec. 1 [f], Rule he was made to understand that it was acceptable to them, except as to
8, Rules of Court), the lower court should not have gone beyond, and it plaintiff's remuneration; that it was finally agreed between plaintiff and all said
should have limited itself, to the facts alleged in the complaint in considering Directors that his remuneration would be P0.30 per kilo (of tobacco); and that
and resolving said motion to dismiss. after the agreement was formally executed he was assured by said Directors
that there would be no need of formal approval by the Board. It should be

830
noted in this connection that although the contract required such approval it veracity, merely questions their sufficiency to make out a case on which the
did not specify just in what manner the same should be given. court can grant relief. Affidavits, such as those presented by defendants in
support of the motion, can only be considered for the purpose of ascertaining
On the question of ratification the complaint alleges that plaintiff delivered to whether an issue of fact is presented, but not as a basis for deciding the
the defendant corporation the sum of P20,000.00 as called for in the factual issue itself. This should await the trial on the merits.
contract; that he rendered the services he was required to do; that he
furnished said defendant 3,000 sacks at a cost of P6,000.00 and advanced The third assignment of error assails the lower court's ruling that even
to it the further sum of P5,000.00; and that he did all of these things with the assuming that a contract had been perfected no action can be maintained
full knowledge, acquiescence and consent of each and all of the individual thereon because its object was illegal and therefore void. Specific reference
defendants who constitute the Board of Directors of the defendant was made by said court to an affidavit executed by appellant on May 10,
corporation. There is abundant authority in support of the proposition 1962 which reads:
that ratification may be express or implied, and that implied ratification
may take diverse forms, such as by silence or acquiescence; by acts That I, EMILIANO ACUA, the party of the Second Part in the contract
showing approval or adoption of the contract; or by acceptance and entered into with the Batac Procoma, Inc., the party of the First Part
retention of benefits flowing therefrom. in same contract declares that the amount of P0.30 per kilo is
referred to upgraded tobacco only as delivered. This supplements
Significantly the very resolution of the Board of Directors relied upon by paragraph three of the contract referred to. Deliveries downgraded or
defendants appears to militate against their contention. It refers to plaintiff's maintained at the redrying plant are deemed not included.
failure to comply with certain promises he had made, as well as to his
interpretation of the contract with respect to his remuneration which, The lower court, in its order of dismissal, held that "the upgrading of tobaccos
according to the Board, was contrary to the intention of the parties. The is clearly prohibited under our laws," and hence the contract cannot be validly
resolution then proceeds to "disapprove and/or rescind" the said contract. ratified. Evidently the court had in mind a fraudulent upgrading of tobacco by
The idea of conflicting interpretation, or rescission on the ground that one of appellant as part of the services called for under the contract. This
the parties has failed to fulfill his obligation under the contract, is certainly conclusion, however, is squarely traversed by appellant in another affidavit
incompatible with defendants' theory here that no contract had yet been attached to his reply and opposition to the motion to dismiss, in which he
perfected for lack of approval by the Board of Directors. explained the circumstances which led to the execution of the one relied
upon by the court, and the real meaning of the word "upgraded" therein. It is
Appellants' second assignment of error reads: "Assuming that in resolving therein stated:
the defendants' motion to dismiss the lower court could consider the new
facts alleged therein and the documents annexed thereto it committed an That after the execution of the agreement (Annex "B" to the amended
error in extending such consideration beyond ascertaining only if an issue of complaint in said Civil Case No. Q-6547), Messrs. Verano, Galano and Dr.
fact has been presented and in actually deciding instead such fact in issue." Bumanglag of the defendant Corporation indicated to me that if the price of
P0.30 per kilo stipulated there to be paid to me were to be indiscriminately
The assignment is well taken, and is the logical corollary of the rule that a applied to all deliveries of tobaccos, the Corporation would be placed in a
motion to dismiss on the ground that the complaint fails to state a cause of disadvantageous and losing position, and they proceeded to explain to me
action addresses itself to the averments in the complaint and, admitting their the following,

831
(a) that when the farmers sell their tobaccos to the Facoma, they do forthwith prepared, drafted and typed the "addendum" in question in their
so in bunches of assorted qualities which may belong either to Class own typewriter of the Corporation; and as I am not a lawyer and was not well
A, B, C, D and E, and upon such purchase they are initially given an versed with the usage, customs and phraseology usually used in tobacco
arbitrary classification of any of such classes as the case may be, trading, I relied in absolute good faith that, as explained by the defendants,
the tendency generally being to give them a lower classification to there was nothing wrong nor illegal in the use of the words "upgrading" and
equalize or average the assorted qualities as much as possible, and "downgrading" used in said addendum, which Atty. Alcantara unfortunately
this is what is termed "downgrading;" used in the same;

(b) that after the tobaccos have been purchased by the Facoma from Apart from the above, defendants knew the physical impossibility of
the farmers, they are then reassorted and re-classified in accordance "upgrading" the tobaccos at the redrying plant, because at the time of the
with their actual quality or grade as found by the officials of the transaction, only the PTFC & RC was allowed to accept tobacco for redrying
Facoma, thus in a bunch which are purchased as Class C, D or E, and under the existing regulations and practices the delivery area for
upon reclassification those found to belong to Class A are separated tobaccos at the redrying plant is enclosed by a high wire fence inaccessible
from Class B, those belonging to Class B are separated from Class to the general public and the only ones who actually make the grading of
C, and so on, and these bunches so reclassified necessarily have a tobaccos delivered, are the (1) American representative of the redrying plant
higher grade than the farmers, and this is what is termed "upgrading" (PTFC & RC), (2) the PVTA, (3) the BIR, and (4) the General Auditing Office
upon delivery original arbitrary classification given when purchased in the presence of the representative of the FACOMA, and since the redrying
from the which was used in the addendum; plant is compelled to purchase 41% of all tobaccos delivered and redried
under their negotiated management contract, it is highly improbable that the
(c) the Facoma, in turn, delivers these properly re-classified tobaccos representative of the redrying plant (PTFC & RC) whose conformity to the
to the redrying plant, and there, a group of officials composed of a actual grading done must appear in the corresponding "guia" or tally sheet,
representative of the redrying plant, the Bureau of Internal Revenue, would allow the "upgrading" of tobaccos, aside from the fact that stringent
the General Auditing Office, the PVTA and the Facoma measures had been devised under the present administration to prevent the
representative, then examines and grades the tobaccos, and if the "upgrading" of tobaccos by any party. Certainly, an impossible condition
classification given by the Facoma is found correct and not changed, could not have been contemplated by me and the defendants; (Record on
then and only then would or should be entitled to collect the P0.30 Appeal, pp. 171-175).
per kilo, and this they said is what is termed "grade maintained"
on the other hand, if these officials found the classification incorrect The foregoing explanation, on its face, is satisfactory and deprives the term
and lowers the classification given by the Facoma, thus class A to B, "upgraded" of the sinister and illegal connotation attributed to it by the lower
or from B to C, then the tobaccos are considered or said to be court. To be sure, whether the allegations in this subsequent affidavit are true
"downgraded" and in that event I should not receive any centavo for or not is a question of fact; but it is precisely for this reason that they can
such deliveries, and it is in this sense that I was made to understand neither be summarily admitted nor rejected for purposes of a motion to
the term; dismiss. Due process demands that they be the subject of proof and
considered only after trial on the merits.
Believing implicitly in the foregoing explanations of the defendants and in the
reasonableness of their proposal, I agreed readily and Atty. Fernando The other errors assigned by appellant are merely incidental to those already
Alcantara, Legal Counsel and Secretary of the defendant Corporation discussed, and require no separate treatment.

832
Wherefore, the order appealed from is set aside and the case is remanded to
the court a quo for further proceedings, without prejudice to, the right of
plaintiff-appellant to ask for another writ of attachment in said court, as the
circumstances may warrant. Costs against defendants-appellees.
G.R. No. L-37281 November 10, 1933
Concepcion, C.J., Reyes, J.B.L., Bengzon, J.P., Zaldivar, Sanchez and
Castro, JJ., concur. W. S. PRICE and THE SULU DEVELOPMENT COMPANY, plaintiffs-
Dizon, J., took no part. appellants,
vs.
H. MARTIN, defendant-appellant.
THE AGUSAN COCONUT COMPANY, defendant-appellee.

J.W. Ferrier for plaintiff-appellants.


G.E. Campbell and W.A. Caldwell for defendant-appellant.
DeWitt, Perkins and Brady for appellee.

HULL, J.:

Plaintiffs brought suit in the Court of First Instance of Manila praying that a
mortgage executed by the Sulu Development Company on its properties in
favor of the Agusan Coconut Company be dissolved and declared null and
void, the principal contentions being that at the stockholders' meeting in
which the officers of the Sulu Development Company were elected and at
which the proposed mortgage was approved of, 97 shares of stock of the
Sulu Development Company were voted by the proxy of Mrs. Worcester , in
whose name the stock at that time stood upon the books of the company,
whereas defendant Martin claimed that he was the true owner and that he
should have voted the stock.

From the records of the Sulu Development Company it appears that at the
meeting of November 12, 1925, Martin presented evidence to the effect that
he, and not Mrs. Worcester, was the owner of the 97 shares of stock. Copies
of the documents relied upon by Martin were made a part of the record, but
apparently no action was taken by the stockholders or by the directors, and

833
at the meetings of November 12, 17, and 19, Mrs. Worcester's proxy The trial court, on timely objection, refused to receive the parol evidence as
apparently voted the stock without protest on the part of Martin or any other to the cultivation agreement, and after trial and a lengthy opinion, held that
stockholder. the mortgage in question was valid and refused to order its cancellation.

As far as the record shows, every formal action taken at those three From that decision plaintiff appeal and make the following assignments of
meetings was unanimous, and Martin at the last two meetings was error:
accompanied by two members of the Bar of the Philippine Islands as his
counsel. The trial court erred:

The Sulu Development Company from its inception up to the time of 1. In refusing appellants the right to introduce evidence as to the
executing the contract was virtually owned and controlled by Martin. Prince "cultivation agreement" extensively referred to by the parties herein.
purchased one share of stock about a month before the called meeting but
was not present at the meetings in question. 2. In refusing to reopen the case on motion filed in due form and
manner by the plaintiffs and appellants herein, on the ground of
Another ground relied upon by plaintiffs is a claim that the mortgage was newly discovered evidence, such motion having been filed the
without consideration. The evidence shows that for years the Agusan rendition of the judgment herein.
Coconut Company, through its general manager, had been advancing sums
through Martin in order that the Sulu Development Company might secure 3. In finding that the plaintiff, W.S. Price, did not appear here as a
good and sufficient title to a large tract of land situated near Siasi and plaintiff to depend his own right but for the purpose of giving aid to
thereon develop a coconut plantation. The amount of money so advanced the defendant, Harry Martin.
was in dispute, but between the meeting on November 12 and the final action
on November 19, the attorney of the Sulu Development Company, one of
4. In ruling that although the 97 shares voted by Mrs. Nanon L.
whom was also an accountant, and the attorneys of the Agusan Coconut
Worcester at the meetings in question thru her proxy belonged to
Company went over the mutual accounts with care and arrived at the sum
Harry Martin and were only held in trust by her late husband, Dean
set forth in the mortgaged. Had there been no agreement, suit would have
C. Worcester, yet such trusteeship was for the benefit of the Agusan
been instituted by the Agusan Company against the Sulu Development
Coconut Company, and that such company is the actual cestui que
Company.
trust thereunder, in violation of the express terms of the trust
agreement.
There is also a claim that there was a parol agreement between Martin and
Worcester, representing the two companies, that after the death of Mr.
5. In holding that Mrs. Nanon L. Worcester could legally vote the said
Worcester on May 2, 1924, the Agusan Coconut Company failed to comply
97 shares she actually voted at the meeting in question,
with the terms and conditions of the so-called cultivation agreement, and
notwithstanding the facts as found by said court, that said shares
Martin prayed in his special cross-complaint and counter-claim that the
belonged to H. Martin and were merely held in trust by her deceased
Defendant Agusan Coconut Company be required to make such further cash
husband.
advances to "carry out the full scale development of the tract of land in the
cultivation agreement and as contemplated therein."

834
6. In finding that the 97 shares of stock in question had been 14. In dismissing plaintiffs' complaint herein.
adjudicated to Mrs. Nanon L. Worcester by the commissioners on
claims against the estate of her deceased husband; that such 15. In denying plaintiffs' motion for a new trial.
adjudication had been approved by the Court of First Instance of the
City of Manila, and that the said Nanon L. Worcester had inherited While defendant Martin appeals and assigns the following errors:
said shares by virtue of the will of her deceased husband.
1. The trial court erred in refusing to find that the one hundred shares
7. In holding the effect that there was a quorum in the pretended of the capital stock of the appellant, the Sulu Development Company,
meetings of the stockholders of the Sulu Development Company delivered on November 23, 1922, by the appellant, H. Martin, to the
alleged to have taken place on November 12, 17 and 19, 1925, late Dean C. Worcester, were so delivered in trust to be held and
particularly that one asserted to have been held on November 19, used for the benefit of the said H. Martin.
1925, when in law and in fact there was no such quorum.
2. The trial court erred in finding that the voting by Mrs. Nanon L.
8. In finding in effect that the meetings pretended to be held by Sulu Worcester, in the meeting held by the stockholders of the appellant,
Development on the dates aforementioned were validly and legally the Sulu Development Company, on November 12, 17, and 19, 1925,
held and that the action taken and proceedings had thereat were was legal.
valid and effective.
3. The trial court erred in refusing to find that the mortgage involved
9. In finding that if the defendant H. Martin had had the 97 shares in in this litigation, purported to have been executed by the appellant,
question in his own name at the alleged meetings of the Sulu the Sulu Development Company, in favor of the appellee, the Agusan
Development Company, he would have voted them in the same way Coconut Company, is null and void.
and to the same effect as the said Nanon L. Worcester voted them.
4. The trial court erred in excluding, as being within the statute of
10. In not finding that there was attendant fraud, misrepresentation frauds, testimony regarding a certain verbal agreement entered into
and deceit in the execution and issuance of the mortgage contract, by and between the appellee, the Agusan Coconut Company, and
Exhibit U. the appellant, H. Martin, which agreement had been fully performed
by the latter.
11. In not holding that said mortgage is null and void for want of legal
consideration. 5. The trial court erred in excluding as "Hearsay Evidence",
testimony regarding statements made by certain officials of the
12. In finding that the plaintiffs and appellants herein are legally appellee, the Agusan Company.
bound by the said mortgage contract Exhibit U.
6. The trial court erred in excluding the testimony of the appellant, H.
13. In holding that the plaintiffs and appellants herein are legally Martin, regarding matters of fact which occurred between him and
estopped to contest the efficacy and validity of the mortgage certain officials of the appellee, the Agusan Coconut Company, who
contract, Exhibit, U. had died prior to the trial of this action.

835
An examination of the assignments of error will show that although this case It must be remembered that there is no dispute between the Worcester
in its main aspects is a simple one and confined to the questions, first, as to interests and the Agusan Coconut Company as to who advanced the money,
whether the mortgage was duly executed by the Sulu Development namely, the Agusan Coconut Company, nor is there any difficulty in
Company and, second, whether it was given for a valuable consideration, determining to whom the money was advanced. Although Martin was virtually
many side issues of no moment were urged upon the trial court, which the owner of all the capital stock of the Sulu Development Company,
probably accounts for the voluminous record with which we are confronted business was carried on in the name of the company, and the land and
and numerous assignments of error which we do not deem it necessary to properties were secured in the name of the company, and up to the time of
discuss in detail. the execution of the mortgage and some time thereafter there was no claim
from anybody the money had been advanced to Martin instead of the
Plaintiffs contend that the transference on the books of the company of 97 company. Even a repeated use of the questionable phrase "and/or" as to the
shares of stock in the name of Mrs. Worcester was fraudulent and illegal. The grantor "and/or" as to the grantee, will not fabricate a life-raft on which a
evidence of record, however, under all the circumstances of the case, fails to recalcitrant debtor can reach a safe harbor of repudiation.lawphil.net
demonstrate the allegation of fraud, and this court believes that she acted in
good faith and in the honest belief that she had not only a legal right but a We are therefore convinced that the contention that the mortgage was made
duty to participate in the stockholders meeting. without consideration was a afterthought without foundation in fact and in a
vain attempt to avoid a legal and binding obligation.
As to whether the stock was rightfully the property of Martin, that is a
question for the courts and for a stockholder's meeting. Until challenged in a We find no merit in the contention that the trial court should have concerned
proper proceeding, a stockholder according to the books of the company has itself with an alleged parol contract between Martin and Dean C. Worcester,
a right to participate in that meeting, and in the absence of fraud the action of deceased. The alleged contract not being in writing or to be executed within
the stockholders' meeting cannot be collaterally attacked on account of such a year, it is within the statute of frauds. The value of the rule is shown in this
participation. "A person who has purchased stock, and who desires to be case as it was some time after Mr. Worcester's death before anything was
recognized as a stockholder, for the purpose of voting, must secure such a heard of such an alleged agreement. Even if such an agreement had been
standing by having the transfer recorder upon the books. If the transfer is not made and it had been proper to receive proof thereof, it would not benefit
duly made upon request, he has, as his remedy, to compel it to be made." plaintiffs as the mortgage was executed pursuant to a compromise
(Morrill vs. Little Falls Mfg. Co., 53 Minn., 371; 21 L.R.A., 175-178, citing agreement to settle the affairs between the two companies, and all the
Cook, Stock & Stockholders, par. 611; People vs. Robinson, 64 Cal., 373; transactions between the two companies were merged and settle by that
Downing vs. Potts, 23 N.J.L., 66; State vs. Ferris, 42 Conn., 560; New York & compromise.
N.H.R. Co. vs. Schuyler, 34 N.Y., 80; Bank of Commerce's App., 73 Pa., 59;
Hoppin vs. Buffum, 9 R.I., 513; 11 Am. Rep., 219; Re St. Lawrence S.R. Co., The contention that a new trial should have been granted in order that
44 N. J. L., 529.) plaintiffs could present in evidence a letter from Mr. Worcester to the late
Governor-General Wood, is likewise without merit. The letter, even if
As to the question of lack of consideration for the mortgage, throughout the admitted, would not have changed the result of these proceedings, as a fair
brief for appellants it appears by the constant reiteration of the phrase that all reading of the letter is not repugnant to a single contention of defendant-
the advances were made "by the Agusan Coconut Company and/or its then appellee.
General Manager, the late Dean C. Worcester, to H. Martin and/or the Sulu
Development Company."

836
The judgment appealed from is therefore affirmed. Costs against appellants.
So ordered.

Malcolm, Villa-Real, Abad Santos, and Imperial, JJ., concur.

G.R. No. 131394 March 28, 2005

JESUS V. LANUZA, MAGADYA REYES, BAYANI REYES and ARIEL


REYES, Petitioner,
vs.
COURT OF APPEALS, SECURITIES AND EXCHANGE COMMISSION,
DOLORES ONRUBIA, ELENITA NOLASCO, JUAN O. NOLASCO III,
ESTATE OF FAUSTINA M. ONRUBIA, PHILIPPINE MERCHANT MARINE
SCHOOL, INC., Respondents.

DECISION

TINGA, J.:

837
Presented in the case at bar is the apparently straight-forward but questioning the validity of the 06 May 1992 stockholders meeting, alleging
complicated question: What should be the basis of quorum for a that the quorum for the said meeting should not be based on the 165 issued
stockholders meetingthe outstanding capital stock as indicated in the and outstanding shares as per the stock and transfer book, but on the initial
articles of incorporation or that contained in the companys stock and transfer subscribed capital stock of seven hundred seventy-six (776) shares, as
book? reflected in the 1952 Articles of Incorporation. The petition was
dismissed.4 Appeal was made to the SEC En Banc, which granted said
Petitioners seek to nullify the Court of Appeals Decision in CAG.R. SP No. appeal, holding that the shares of the deceased incorporators should be duly
414731 promulgated on 18 August 1997, affirming the SEC Order dated 20 represented by their respective administrators or heirs concerned. The SEC
June 1996, and the Resolution2 of the Court of Appeals dated 31 October directed the parties to call for a stockholders meeting on the basis of the
1997 which denied petitioners motion for reconsideration. stockholdings reflected in the articles of incorporation for the purpose of
electing a new set of officers for the corporation. 5
The antecedents are not disputed.
Petitioners, who are PMMSI stockholders, filed a petition for review with the
In 1952, the Philippine Merchant Marine School, Inc. (PMMSI) was Court of Appeals.6 Rebecca Acayan, Jayne O. Abuid, Willie O. Abuid and
incorporated, with seven hundred (700) founders shares and seventy-six Renato Cervantes, stockholders and directors of PMMSI, earlier filed another
(76) common shares as its initial capital stock subscription reflected in the petition for review of the same SEC En Bancs orders. The petitions were
articles of incorporation. However, private respondents and their thereafter consolidated.7 The consolidated petitions essentially raised the
predecessors who were in control of PMMSI registered the companys stock following issues, viz: (a) whether the basis the outstanding capital stock and
and transfer book for the first time in 1978, recording thirty-three (33) accordingly also for determining the quorum at stockholders meetings it
common shares as the only issued and outstanding shares of PMMSI. should be the 1978 stock and transfer book or if it should be the 1952 articles
Sometime in 1979, a special stockholders meeting was called and held on of incorporation; and (b) whether the Court of Appeals "gravely erred in
the basis of what was considered as a quorum of twenty-seven (27) common applying the Espejo Decision to the benefit of respondents." 8 The "Espejo
shares, representing more than two-thirds (2/3) of the common shares issued Decision" is the decision of the SEC en banc in SEC Case No. 2289 which
and outstanding. ordered the recording of the shares of Jose Acayan in the stock and transfer
book.
In 1982, the heirs of one of the original incorporators, Juan Acayan, filed a
petition with the Securities and Exchange Commission (SEC) for the The Court of Appeals held that for purposes of transacting business,
registration of their property rights over one hundred (120) founders shares the quorum should be based on the outstanding capital stock as found
and twelve (12) common shares owned by their father. The SEC hearing in the articles of incorporation.9 As to the second issue, the Court of
officer held that the heirs of Acayan were entitled to the claimed shares and Appeals held that the ruling in the Acayan case would ipso facto benefit the
called for a special stockholders meeting to elect a new set of officers. 3The private respondents, since to require a separate judicial declaration to
SEC En Banc affirmed the decision. As a result, the shares of Acayan were recognize the shares of the original incorporators would entail unnecessary
recorded in the stock and transfer book. delay and expense. Besides, the Court of Appeals added, the incorporators
have already proved their stockholdings through the provisions of the articles
of incorporation.10
On 06 May 1992, a special stockholders meeting was held to elect a new set
of directors. Private respondents thereafter filed a petition with the SEC

838
In the instant petition, petitioners claim that the 1992 stockholders meeting Res judicata does not apply in
was valid and legal. They submit that reliance on the 1952 articles of the case at bar.
incorporation for determining the quorum negates the existence and validity
of the stock and transfer book which private respondents themselves Res judicata means a matter adjudged, a thing judicially acted upon or
prepared. In addition, they posit that private respondents cannot avail of the decided; a thing or matter settled by judgment. 17 The doctrine
benefits secured by the heirs of Acayan, as private respondents must show of res judicata provides that a final judgment, on the merits rendered by a
and prove entitlement to the founders and common shares in a separate and court of competent jurisdiction is conclusive as to the rights of the parties and
independent action/proceeding. their privies and constitutes an absolute bar to subsequent actions involving
the same claim, demand, or cause of action. 18 The elements of res
In private respondents Memorandum11 dated 08 March 2000, they point out judicata are (a) identity of parties or at least such as representing the same
that the instant petition raises the same facts and issues as those raised in interest in both actions; (b) identity of rights asserted and relief prayed for,
G.R. No. 13131512, which was denied by the First Division of this Court on 18 the relief being founded on the same facts; and (c) the identity in the two (2)
January 1999 for failure to show that the Court of Appeals committed any particulars is such that any judgment which may be rendered in the other
reversible error. They add that as a logical consequence, the instant petition action will, regardless of which party is successful, amount to res judicata in
should be dismissed on the ground of res judicata. Furthermore, private the action under consideration.19
respondents claim that in view of the applicability of the rule on res judicata,
petitioners counsel should be cited for contempt for violating the rule against There is no dispute as to the identity of subject matter since the crucial point
forum-shopping.13 in both cases is the propriety of including the still unproven shares of
respondents for purposes of determining the quorum. Petitioners, however,
For their part, petitioners claim that the principle of res judicata does not deny that there is identity of parties and causes of actions between the two
apply to the instant case. They argue that the instant petition is separate and petitions.
distinct from G.R. No. 131315, there being no identity of parties, and more
importantly, the parties in the two petitions have their own distinct rights and The test often used in determining whether causes of action are identical is
interests in relation to the subject matter in litigation. For the same reasons, to ascertain whether the same facts or evidence would support and establish
they claim that counsel for petitioners cannot be found guilty of forum- the former and present causes of action. 20 More significantly, there is identity
shopping.14 of causes of action when the judgment sought will be inconsistent with the
prior judgment.21 In both petitions, petitioners assert that the Court of
In their Manifestation and Motion15 dated 22 September 2004, private Appeals Decision effectively negates the existence and validity of the stock
respondents moved for the dismissal of the instant petition in view of the and transfer book, as well as automatically grants private respondents
dismissal of G.R. No. 131315. Attached to the said manifestation is a copy of shares of stocks which they do not own, or the ownership of which remains
the Entry of Judgment16 issued by the First Division dated 01 December to be unproved. Petitioners in the two petitions rely on the entries in the stock
1999. and transfer book as the proper basis for computing the quorum, and
consequently determine the degree of control one has over the company.
The petition must be denied, not on res judicata, but on the ground that like Essentially, the affirmance of the SEC Order had the effect of diminishing
the petition in G.R. No. 131315 it fails to impute reversible error to the their control and interests in the company, as it allowed the participation of
challenged Court of Appeals Decision. the individual private respondents in the election of officers of the
corporation.

839
Absolute identity of parties is not a condition sine qua non for res judicata to The crucial issue in this case is whether it is the companys stock and
applya shared identity of interest is sufficient to invoke the coverage of the transfer book, or its 1952 Articles of Incorporation, which determines
principle.22 However, there is no identity of parties between the two cases. stockholders shareholdings, and provides the basis for computing the
The parties in the two petitions have their own rights and interests in relation quorum.
to the subject matter in litigation. As stated by petitioners in their Reply to
Respondents Memorandum,23 there are no two separate actions filed, but We agree with the Court of Appeals.
rather, two separate petitions for review on certiorari filed by two distinct
parties with the Court and represented by their own counsels, arising from an The articles of incorporation has been described as one that defines the
adverse consolidated decision promulgated by the Court of Appeals in one charter of the corporation and the contractual relationships between the State
action or proceeding.24 As such, res judicata is not present in the instant and the corporation, the stockholders and the State, and between the
case. corporation and its stockholders. 27 When PMMSI was incorporated, the
prevailing law was Act No. 1459, otherwise known as "The Corporation Law."
Likewise, there is no basis for declaring petitioners or their counsel guilty of Section 6 thereof states:
violating the rules against forum-shopping. In
the Verification/Certification25 portion of the petition, petitioners clearly stated Sec. 6. Five or more persons, not exceeding fifteen, a majority of
that there was then a pending motion for reconsideration of the 18 August whom are residents of the Philippines, may form a private
1997 Decision of the Court of Appeals in the consolidated cases (CA-G.R. corporation for any lawful purpose or purposes by filing with the
SP No. 41473 and CA-G.R. SP No. 41403) filed by the Abuids, as well as a Securities and Exchange Commission articles of incorporation duly
motion for clarification. Moreover, the records indicate that petitioners filed executed and acknowledged before a notary public, setting forth:
their Manifestation26 dated 20 January 1998, informing the Court of their
receipt of the petition in G.R. No. 131315 in compliance with their duty to
....
inform the Court of the pendency of another similar petition. The Court finds
that petitioners substantially complied with the rules against forum-shopping.
(7) If it be a stock corporation, the amount of its capital stock, in
lawful money of the Philippines, and the number of shares into which
The Decision of the Court of
it is divided, and if such stock be in whole or in part without par value
Appeals must be upheld.
then such fact shall be stated; Provided, however, That as to stock
without par value the articles of incorporation need only state the
The petition in this case involves the same facts and substantially the same number of shares into which said capital stock is divided.
issues and arguments as those in G.R. No. 131315 which the First Division
has long denied with finality. The First Division found the petition before it
(8) If it be a stock corporation, the amount of capital stock or number
inadequate in failing to raise any reversible error on the part of the Court of
of shares of no-par stock actually subscribed, the amount or number
Appeals. We reach a similar conclusion as regards the present petition.
of shares of no-par stock subscribed by each and the sum paid by
each on his subscription. . . .28

A review of PMMSIs articles of incorporation 29 shows that the corporation


complied with the requirements laid down by Act No. 1459. It provides in part:

840
7. That the capital stock of the said corporation is NINETY
THOUSAND PESOS (P90,000.00) divided into two classes, namely:
Martin P. Sagarbarria 100 " 2, 000.00
FOUNDERS STOCK - 1,000 shares at P20 par value- P 20,000.00

COMMON STOCK- 700 shares at P 100 par value P 70,000.00


Mauricio G. Gallaga 50 " 1, 000.00
TOTAL ---------------------1,700 shares----------------------------
P 90,000.00

.... Luis Renteria 50 " 1, 000.00

8. That the amount of the entire capital stock which has been
actually subscribed is TWENTY ONE THOUSAND SIX HUNDRED
PESOS (P21,600.00) and the following persons have subscribed for Faustina M. de Onrubia 140 " 2, 800.00
the number of shares and amount of capital stock set out after their
respective names:

Mrs. Ramon Araneta 40 " 800.00

SUBSCRIBER SUBSCRIBED AMOUNT


SUBSCRIBED

Carlos M. Onrubia 80 " 1,600.00

No. of Shares Par Value

700 P 14,000.00

Crispulo J. Onrubia 120 Founders P 2,400.00

Juan H. Acayan 120 " 2, 400.00

841
SUBSCRIBER SUBSCRIBED AMOUNT
SUBSCRIBED
No. of Shares
Par Value
Carlos M. Onrubia 8" 800.00

Crispulo J. Onrubia 12 Common P 1,200.00


76 P7,600.0030

Juan H. Acayan 12 " 1,200.00


There is no gainsaying that the contents of the articles of incorporation
are binding, not only on the corporation, but also on its shareholders.
In the instant case, the articles of incorporation indicate that at the time
of incorporation, the incorporators were bona fide stockholders of
Martin P. Sagarbarria 8" 800.00
seven hundred (700) founders shares and seventy-six (76) common
shares. Hence, at that time, the corporation had 776 issued and
outstanding shares.

Mauricio G. Gallaga 8" 800.00 On the other hand, a stock and transfer book is the book which records the
names and addresses of all stockholders arranged alphabetically, the
installments paid and unpaid on all stock for which subscription has been
made, and the date of payment thereof; a statement of every alienation, sale
Luis Renteria 8" 800.00 or transfer of stock made, the date thereof and by and to whom made; and
such other entries as may be prescribed by law. 31 A stock and transfer book
is necessary as a measure of precaution, expediency and convenience since
it provides the only certain and accurate method of establishing the various
Faustina M. de Onrubia 12 " 1,200.00 corporate acts and transactions and of showing the ownership of stock and
like matters.32 However, a stock and transfer book, like other corporate
books and records, is not in any sense a public record, and thus is not
exclusive evidence of the matters and things which ordinarily are or
should be written therein.33 In fact, it is generally held that the records
Mrs. Ramon Araneta 8" 800.00 and minutes of a corporation are not conclusive even against the
corporation but are prima facie evidence only,34 and may be impeached

842
or even contradicted by other competent evidence. 35 Thus, parol This case is one instance where resort to documents other than the stock
evidence may be admitted to supply omissions in the records or explain and transfer books is necessary. The stock and transfer book of PMMSI
ambiguities, or to contradict such records.36 cannot be used as the sole basis for determining the quorum as it does not
reflect the totality of shares which have been subscribed, more so when the
In 1980, Batas Pambansa Blg. 68, otherwise known as "The Corporation articles of incorporation show a significantly larger amount of shares issued
Code of the Philippines" supplanted Act No. 1459. BP Blg. 68 provides: and outstanding as compared to that listed in the stock and transfer book. As
aptly stated by the SEC in its Order dated 15 July 1996:38
Sec. 24. Election of directors or trustees.At all elections of
directors or trustees, there must be present, either in person or by It is to be explained, that if at the onset of incorporation a corporation
representative authorized to act by written proxy, the owners of a has 771 shares subscribed, the Stock and Transfer Book should
majority of the outstanding capital stock, or if there be no capital likewise reflect 771 shares. Any sale, disposition or even
stock, a majority of the members entitled to vote. . . . reacquisition of the company of its own shares, in which it becomes
treasury shares, would not affect the total number of shares in the
Sec. 52. Quorum in meetings.- Unless otherwise provided for in this Stock and Transfer Book. All that will change are the entries as to the
Code or in the by-laws, a quorum shall consist of the stockholders owners of the shares but not as to the amount of shares already
representing a majority of the outstanding capital stock or majority of subscribed.
the members in the case of non-stock corporation.
This is precisely the reason why the Stock and Transfer Book was
Outstanding capital stock, on the other hand, is defined by the Code as: not given probative value. Did the shares, which were not recorded
in the Stock and Transfer Book, but were recorded in the Articles of
Iincorporation just vanish into thin air? . . . . 39
Sec. 137. Outstanding capital stock defined. The term
"outstanding capital stock" as used in this code, means the total
shares of stock issued to subscribers or stockholders whether or not As shown above, at the time the corporation was set-up, there were already
fully or partially paid (as long as there is binding subscription seven hundred seventy-six (776) issued and outstanding shares as reflected
agreement) except treasury shares. in the articles of incorporation. No proof was adduced as to any transaction
effected on these shares from the time PMMSI was incorporated up to the
time the instant petition was filed, except for the thirty-three (33) shares
Thus, quorum is based on the totality of the shares which have been
which were recorded in the stock and transfer book in 1978, and the
subscribed and issued, whether it be founders shares or common
additional one hundred thirty-two (132) in 1982. But obviously, the shares so
shares.37 In the instant case, two figures are being pitted against each
ordered recorded in the stock and transfer book are among the shares
other those contained in the articles of incorporation, and those
reflected in the articles of incorporation as the shares subscribed to by the
listed in the stock and transfer book.
incorporators named therein.

To base the computation of quorum solely on the obviously deficient, if not


One who is actually a stockholder cannot be denied his right to vote by
inaccurate stock and transfer book, and completely disregarding the issued
the corporation merely because the corporate officers failed to keep its
and outstanding shares as indicated in the articles of incorporation would
records accurately.40 A corporations records are not the only evidence
work injustice to the owners and/or successors in interest of the said shares.

843
of the ownership of stock in a corporation. 41 In an American WHEREFORE, the petition is DENIED and the assailed Decision is
case,42 persons claiming shareholders status in a professional corporation AFFIRMED. Costs against petitioners.
were listed as stockholders in the amendment to the articles of incorporation.
On that basis, they were in all respects treated as shareholders. In fact, the SO ORDERED.
acts and conduct of the parties may even constitute sufficient evidence of
ones status as a shareholder or member.43 In the instant case, no less than Puno, (Chairman), Austria-Martinez, Callejo, Sr., and Chico-Nazario,
the articles of incorporation declare the incorporators to have in their name JJ., concur.
the founders and several common shares. Thus, to disregard the contents of
the articles of incorporation would be to pretend that the basic document
which legally triggered the creation of the corporation does not exist and
accordingly to allow great injustice to be caused to the incorporators and
their heirs.

Petitioners argue that the Court of Appeals "gravely erred in applying the
Espejo decision to the benefit of respondents." The Court believes that the
more precise statement of the issue is whether in its assailed Decision, the
Court of Appeals can declare private respondents as the heirs of the
incorporators, and consequently register the founders shares in their name.
However, this issue as recast is not actually determinative of the present
controversy as explained below.

Petitioners claim that the Decision of the Court of Appeals unilaterally


divested them of their shares in PMMSI as recorded in the stock and transfer
book and instantly created inexistent shares in favor of private respondents.
We do not agree.

The assailed Decision merely declared that a separate judicial declaration to


recognize the shares of the original incorporators would entail unnecessary
delay and expense on the part of the litigants, considering that the
incorporators had already proved ownership of such shares as shown in the
articles of incorporation.44 There was no declaration of who the individual
owners of these shares were on the date of the promulgation of the Decision.
As properly stated by the SEC in its Order dated 20 June 1996, to which the
appellate courts Decisionshould be related, "if at all, the ownership of these
shares should only be subjected to the proper judicial (probate) or
extrajudicial proceedings in order to determine the respective shares of the
legal heirs of the deceased incorporators."45

844
845
846
On September 17, 1987, the petitioners filed a motion to dismiss the third
party complaint which the Regional Trial Court of Makati, Branch 58 denied in
G.R. No. 93695 February 4, 1992 an Order dated June 27, 1988.

RAMON C. LEE and ANTONIO DM. LACDAO, petitioners, On July 18, 1988, the petitioners filed their answer to the third party
vs. complaint.
THE HON. COURT OF APPEALS, SACOBA MANUFACTURING CORP.,
PABLO GONZALES, JR. and THOMAS GONZALES, respondents. Meanwhile, on July 12, 1988, the trial court issued an order requiring the
issuance of an alias summons upon ALFA through the DBP as a
Cayanga, Zuniga & Angel Law Offices for petitioners. consequence of the petitioner's letter informing the court that the summons
for ALFA was erroneously served upon them considering that the
Timbol & Associates for private respondents. management of ALFA had been transferred to the DBP.

In a manifestation dated July 22, 1988, the DBP claimed that it was not
authorized to receive summons on behalf of ALFA since the DBP had not
taken over the company which has a separate and distinct corporate
GUTIERREZ, JR., J.:
personality and existence.

What is the nature of the voting trust agreement executed between two
On August 4, 1988, the trial court issued an order advising the private
parties in this case? Who owns the stocks of the corporation under the terms
respondents to take the appropriate steps to serve the summons to ALFA.
of the voting trust agreement? How long can a voting trust agreement remain
valid and effective? Did a director of the corporation cease to be such upon
the creation of the voting trust agreement? These are the questions the On August 16, 1988, the private respondents filed a Manifestation and
answers to which are necessary in resolving the principal issue in this Motion for the Declaration of Proper Service of Summons which the trial
petition for certiorari whether or not there was proper service of summons court granted on August 17, 1988.
on Alfa Integrated Textile Mills (ALFA, for short) through the petitioners as
president and vice-president, allegedly, of the subject corporation after the On September 12, 1988, the petitioners filed a motion for reconsideration
execution of a voting trust agreement between ALFA and the Development submitting that Rule 14, section 13 of the Revised Rules of Court is not
Bank of the Philippines (DBP, for short). applicable since they were no longer officers of ALFA and that the private
respondents should have availed of another mode of service under Rule 14,
From the records of the instant case, the following antecedent facts appear: Section 16 of the said Rules, i.e.,through publication to effect proper service
upon ALFA.
On November 15, 1985, a complaint for a sum of money was filed by the
International Corporate Bank, Inc. against the private respondents who, in In their Comment to the Motion for Reconsideration dated September 27,
turn, filed a third party complaint against ALFA and the petitioners on March 1988, the private respondents argued that the voting trust agreement dated
17, 1986. March 11, 1981 did not divest the petitioners of their positions as president

847
and executive vice-president of ALFA so that service of summons upon ALFA same for failure to prosecute. Subsequently, on October 25, 1989 the private
through the petitioners as corporate officers was proper. respondents filed a motion for reconsideration on which the trial court took no
further action.
On January 2, 1989, the trial court upheld the validity of the service of
summons on ALFA through the petitioners, thus, denying the latter's motion On March 19, 1990, after the petitioners filed their answer to the private
for reconsideration and requiring ALFA to filed its answer through the respondents' petition for certiorari, the public respondent rendered its
petitioners as its corporate officers. decision, the dispositive portion of which reads:

On January 19, 1989, a second motion for reconsideration was filed by the WHEREFORE, in view of the foregoing, the orders of
petitioners reiterating their stand that by virtue of the voting trust agreement respondent judge dated April 25, 1989 and August 14, 1989
they ceased to be officers and directors of ALFA, hence, they could no longer are hereby SET ASIDE and respondent corporation is
receive summons or any court processes for or on behalf of ALFA. In support ordered to file its answer within the reglementary period. (CA
of their second motion for reconsideration, the petitioners attached thereto a Decision, p. 8; Rollo, p. 24)
copy of the voting trust agreement between all the stockholders of ALFA (the
petitioners included), on the one hand, and the DBP, on the other hand, On April 11, 1990, the petitioners moved for a reconsideration of the decision
whereby the management and control of ALFA became vested upon the of the public respondent which resolved to deny the same on May 10, 1990.
DBP. Hence, the petitioners filed this certiorari petition imputing grave abuse of
discretion amounting to lack of jurisdiction on the part of the public
On April 25, 1989, the trial court reversed itself by setting aside its previous respondent in reversing the questioned Orders dated April 25, 1989 and
Order dated January 2, 1989 and declared that service upon the petitioners August 14, 1989 of the court a quo, thus, holding that there was proper
who were no longer corporate officers of ALFA cannot be considered as service of summons on ALFA through the petitioners.
proper service of summons on ALFA.
In the meantime, the public respondent inadvertently made an entry of
On May 15, 1989, the private respondents moved for a reconsideration of the judgment on July 16, 1990 erroneously applying the rule that the period
above Order which was affirmed by the court in its Order dated August 14, during which a motion for reconsideration has been pending must be
1989 denying the private respondent's motion for reconsideration. deducted from the 15-day period to appeal. However, in its Resolution dated
January 3, 1991, the public respondent set aside the aforestated entry of
On September 18, 1989, a petition for certiorari was belatedly submitted by judgment after further considering that the rule it relied on applies to appeals
the private respondent before the public respondent which, nonetheless, from decisions of the Regional Trial Courts to the Court of Appeals, not to
resolved to give due course thereto on September 21, 1989. appeals from its decision to us pursuant to our ruling in the case
of Refractories Corporation of the Philippines v. Intermediate Appellate
On October 17, 1989, the trial court, not having been notified of the pending Court, 176 SCRA 539 [1989]. (CA Rollo, pp. 249-250)
petition for certiorari with public respondent issued an Order declaring as
final the Order dated April 25, 1989. The private respondents in the said In their memorandum, the petitioners present the following arguments, to wit:
Order were required to take positive steps in prosecuting the third party
complaint in order that the court would not be constrained to dismiss the

848
(1) that the execution of the voting trust agreement by a by them, of the power to direct how such control shall be
stockholders whereby all his shares to the corporation have used. (98 ALR 2d. 379 sec. 1 [d]; 19 Am J 2d Corp. sec.
been transferred to the trustee deprives the stockholders of 685).
his position as director of the corporation; to rule otherwise,
as the respondent Court of Appeals did, would be violative of Under Section 59 of the new Corporation Code which expressly recognizes
section 23 of the Corporation Code ( Rollo, pp. 270-3273); voting trust agreements, a more definitive meaning may be gathered. The
and said provision partly reads:

(2) that the petitioners were no longer acting or holding any Sec. 59. Voting Trusts One or more stockholders of a
of the positions provided under Rule 14, Section 13 of the stock corporation may create a voting trust for the purpose of
Rules of Court authorized to receive service of summons for conferring upon a trustee or trustees the right to vote and
and in behalf of the private domestic corporation so that the other rights pertaining to the share for a period rights
service of summons on ALFA effected through the petitioners pertaining to the shares for a period not exceeding five (5)
is not valid and ineffective; to maintain the respondent Court years at any one time: Provided, that in the case of a voting
of Appeals' position that ALFA was properly served its trust specifically required as a condition in a loan agreement,
summons through the petitioners would be contrary to the said voting trust may be for a period exceeding (5) years but
general principle that a corporation can only be bound by shall automatically expire upon full payment of the loan. A
such acts which are within the scope of its officers' or agents' voting trust agreement must be in writing and notarized, and
authority (Rollo, pp. 273-275) shall specify the terms and conditions thereof. A certified
copy of such agreement shall be filed with the corporation
In resolving the issue of the propriety of the service of summons in the and with the Securities and Exchange Commission;
instant case, we dwell first on the nature of a voting trust agreement and the otherwise, said agreement is ineffective and unenforceable.
consequent effects upon its creation in the light of the provisions of the The certificate or certificates of stock covered by the voting
Corporation Code. trust agreement shall be cancelled and new ones shall be
issued in the name of the trustee or trustees stating that they
A voting trust is defined in Ballentine's Law Dictionary as follows: are issued pursuant to said agreement. In the books of the
corporation, it shall be noted that the transfer in the name of
(a) trust created by an agreement between a group of the the trustee or trustees is made pursuant to said voting trust
stockholders of a corporation and the trustee or by a group agreement.
of identical agreements between individual stockholders and
a common trustee, whereby it is provided that for a term of By its very nature, a voting trust agreement results in the separation of the
years, or for a period contingent upon a certain event, or voting rights of a stockholder from his other rights such as the right to receive
until the agreement is terminated, control over the stock dividends, the right to inspect the books of the corporation, the right to sell
owned by such stockholders, either for certain purposes or certain interests in the assets of the corporation and other rights to which a
for all purposes, is to be lodged in the trustee, either with or stockholder may be entitled until the liquidation of the corporation. However,
without a reservation to the owners, or persons designated in order to distinguish a voting trust agreement from proxies and other voting
pools and agreements, it must pass three criteria or tests, namely: (1) that

849
the voting rights of the stock are separated from the other attributes of former assigned and transferred all their shares in ALFA to DBP, as trustee.
ownership; (2) that the voting rights granted are intended to be irrevocable They argue that by virtue to of the voting trust agreement the petitioners can
for a definite period of time; and (3) that the principal purpose of the grant of no longer be considered directors of ALFA. In support of their contention, the
voting rights is to acquire voting control of the corporation. (5 petitioners invoke section 23 of the Corporation Code which provides, in part,
Fletcher, Cyclopedia of the Law on Private Corporations, section 2075 [1976] that:
p. 331 citing Tankersly v. Albright, 374 F. Supp. 538)
Every director must own at least one (1) share of the capital
Under section 59 of the Corporation Code, supra, a voting trust agreement stock of the corporation of which he is a director which share
may confer upon a trustee not only the stockholder's voting rights but also shall stand in his name on the books of the corporation. Any
other rights pertaining to his shares as long as the voting trust agreement is director who ceases to be the owner of at least one (1) share
not entered "for the purpose of circumventing the law against monopolies of the capital stock of the corporation of which he is a
and illegal combinations in restraint of trade or used for purposes of fraud." director shall thereby cease to be director . . . (Rollo, p. 270)
(section 59, 5th paragraph of the Corporation Code) Thus, the traditional
concept of a voting trust agreement primarily intended to single out a The private respondents, on the contrary, insist that the voting trust
stockholder's right to vote from his other rights as such and made irrevocable agreement between ALFA and the DBP had all the more safeguarded the
for a limited duration may in practice become a legal device whereby a petitioners' continuance as officers and directors of ALFA inasmuch as the
transfer of the stockholder's shares is effected subject to the specific general object of voting trust is to insure permanency of the tenure of the
provision of the voting trust agreement. directors of a corporation. They cited the commentaries by Prof. Aguedo
Agbayani on the right and status of the transferring stockholders, to wit:
The execution of a voting trust agreement, therefore, may create a dichotomy
between the equitable or beneficial ownership of the corporate shares of a The "transferring stockholder", also called the "depositing
stockholders, on the one hand, and the legal title thereto on the other hand. stockholder", is equitable owner for the stocks represented
by the voting trust certificates and the stock reversible on
The law simply provides that a voting trust agreement is an agreement in termination of the trust by surrender. It is said that the voting
writing whereby one or more stockholders of a corporation consent to trust agreement does not destroy the status of the
transfer his or their shares to a trustee in order to vest in the latter voting or transferring stockholders as such, and thus render them
other rights pertaining to said shares for a period not exceeding five years ineligible as directors. But a more accurate statement seems
upon the fulfillment of statutory conditions and such other terms and to be that for some purposes the depositing stockholder
conditions specified in the agreement. The five year-period may be extended holding voting trust certificates in lieu of his stock and being
in cases where the voting trust is executed pursuant to a loan agreement the beneficial owner thereof, remains and is treated as a
whereby the period is made contingent upon full payment of the loan. stockholder. It seems to be deducible from the case that he
may sue as a stockholder if the suit is in equity or is of an
In the instant case, the point of controversy arises from the effects of the equitable nature, such as, a technical stockholders' suit in
creation of the voting trust agreement. The petitioners maintain that with the right of the corporation. [Commercial Laws of the Philippines
execution of the voting trust agreement between them and the other by Agbayani, Vol. 3 pp. 492-493, citing 5 Fletcher 326, 327]
stockholders of ALFA, as one party, and the DBP, as the other party, the (Rollo, p. 291)

850
We find the petitioners' position meritorious. in their names on the books of the corporation becomes formally legalized
(see Campos and Lopez-Campos, supra, p. 296) Hence, this is a clear
Both under the old and the new Corporation Codes there is no dispute as to indication that in order to be eligible as a director, what is material is the legal
the most immediate effect of a voting trust agreement on the status of a title to, not beneficial ownership of, the stock as appearing on the books of
stockholder who is a party to its execution from legal titleholder or owner the corporation (2 Fletcher, Cyclopedia of the Law of Private Corporations,
of the shares subject of the voting trust agreement, he becomes the section 300, p. 92 [1969] citing People v. Lihme, 269 Ill. 351, 109 N.E. 1051).
equitable or beneficial owner. (Salonga, Philippine Law on Private
Corporations, 1958 ed., p. 268; Pineda and Carlos, The Law on Private The facts of this case show that the petitioners, by virtue of the voting trust
Corporations and Corporate Practice, 1969 ed., p. 175; Campos and Lopez- agreement executed in 1981 disposed of all their shares through assignment
Campos, The Corporation Code; Comments, Notes & Selected Cases, 1981, and delivery in favor of the DBP, as trustee. Consequently, the petitioners
ed., p. 386; Agbayani, Commentaries and Jurisprudence on the Commercial ceased to own at least one share standing in their names on the books of
Laws of the Philippines, Vol. 3, 1988 ed., p. 536). The penultimate question, ALFA as required under Section 23 of the new Corporation Code. They also
therefore, is whether the change in his status deprives the stockholder of the ceased to have anything to do with the management of the enterprise. The
right to qualify as a director under section 23 of the present Corporation petitioners ceased to be directors. Hence, the transfer of the petitioners'
Code which deletes the phrase "in his own right." Section 30 of the old Code shares to the DBP created vacancies in their respective positions as
states that: directors of ALFA. The transfer of shares from the stockholder of ALFA to the
DBP is the essence of the subject voting trust agreement as evident from the
Every director must own in his own right at least one share following stipulations:
of the capital stock of the stock corporation of which he is a
director, which stock shall stand in his name on the books of 1. The TRUSTORS hereby assign and deliver to the
the corporation. A director who ceases to be the owner of at TRUSTEE the certificate of the shares of the stocks owned
least one share of the capital stock of a stock corporation of by them respectively and shall do all things necessary for the
which is a director shall thereby cease to be a director . . . transfer of their respective shares to the TRUSTEE on the
(Emphasis supplied) books of ALFA.

Under the old Corporation Code, the eligibility of a director, strictly speaking, 2. The TRUSTEE shall issue to each of the TRUSTORS a
cannot be adversely affected by the simple act of such director being a party trust certificate for the number of shares transferred, which
to a voting trust agreement inasmuch as he remains owner (although shall be transferrable in the same manner and with the same
beneficial or equitable only) of the shares subject of the voting trust effect as certificates of stock subject to the provisions of this
agreement pursuant to which a transfer of the stockholder's shares in favor of agreement;
the trustee is required (section 36 of the old Corporation Code). No
disqualification arises by virtue of the phrase "in his own right" provided 3. The TRUSTEE shall vote upon the shares of stock at all
under the old Corporation Code. meetings of ALFA, annual or special, upon any resolution,
matter or business that may be submitted to any such
With the omission of the phrase "in his own right" the election of trustees and meeting, and shall possess in that respect the same powers
other persons who in fact are not beneficial owners of the shares registered

851
as owners of the equitable as well as the legal title to the their position as directors of ALFA, the public respondent committed a
stock; reversible error when it ruled that:

4. The TRUSTEE may cause to be transferred to any person . . . while the individual respondents (petitioners Lee and
one share of stock for the purpose of qualifying such person Lacdao) may have ceased to be president and vice-
as director of ALFA, and cause a certificate of stock president, respectively, of the corporation at the time of
evidencing the share so transferred to be issued in the name service of summons on them on August 21, 1987, they were
of such person; at least up to that time, still directors . . .

xxx xxx xxx The aforequoted statement is quite inaccurate in the light of the express
terms of Stipulation No. 4 of the subject voting trust agreement. Both parties,
9. Any stockholder not entering into this agreement may ALFA and the DBP, were aware at the time of the execution of the agreement
transfer his shares to the same trustees without the need of that by virtue of the transfer of shares of ALFA to the DBP, all the directors of
revising this agreement, and this agreement shall have the ALFA were stripped of their positions as such.
same force and effect upon that said stockholder. (CA Rollo,
pp. 137-138; Emphasis supplied) There can be no reliance on the inference that the five-year period of the
voting trust agreement in question had lapsed in 1986 so that the legal title to
Considering that the voting trust agreement between ALFA and the DBP the stocks covered by the said voting trust agreement ipso facto reverted to
transferred legal ownership of the stock covered by the agreement to the the petitioners as beneficial owners pursuant to the 6th paragraph of section
DBP as trustee, the latter became the stockholder of record with respect to 59 of the new Corporation Code which reads:
the said shares of stocks. In the absence of a showing that the DBP had
caused to be transferred in their names one share of stock for the purpose of Unless expressly renewed, all rights granted in a voting trust
qualifying as directors of ALFA, the petitioners can no longer be deemed to agreement shall automatically expire at the end of the
have retained their status as officers of ALFA which was the case before the agreed period, and the voting trust certificate as well as the
execution of the subject voting trust agreement. There appears to be no certificates of stock in the name of the trustee or trustees
dispute from the records that DBP has taken over full control and shall thereby be deemed cancelled and new certificates of
management of the firm. stock shall be reissued in the name of the transferors.

Moreover, in the Certification dated January 24, 1989 issued by the DBP On the contrary, it is manifestly clear from the terms of the voting trust
through one Elsa A. Guevarra, Vice-President of its Special Accounts agreement between ALFA and the DBP that the duration of the agreement is
Department II, Remedial Management Group, the petitioners were no longer contingent upon the fulfillment of certain obligations of ALFA with the DBP.
included in the list of officers of ALFA "as of April 1982." (CA Rollo, pp. 140- This is shown by the following portions of the agreement.
142)
WHEREAS, the TRUSTEE is one of the creditors of ALFA,
Inasmuch as the private respondents in this case failed to substantiate their and its credit is secured by a first mortgage on the
claim that the subject voting trust agreement did not deprive the petitioners of manufacturing plant of said company;

852
WHEREAS, ALFA is also indebted to other creditors for Had the five-year period of the voting trust agreement expired in 1986, the
various financial accomodations and because of the burden DBP would not have transferred all its rights, titles and interests in ALFA
of these obligations is encountering very serious difficulties "effective June 30, 1986" to the national government through the Asset
in continuing with its operations. Privatization Trust (APT) as attested to in a Certification dated January 24,
1989 of the Vice President of the DBP's Special Accounts Department II. In
WHEREAS, in consideration of additional accommodations the same certification, it is stated that the DBP, from 1987 until 1989, had
from the TRUSTEE, ALFA had offered and the TRUSTEE handled APT's account which included ALFA's assets pursuant to a
has accepted participation in the management and control of management agreement by and between the DBP and APT (CA Rollo, p.
the company and to assure the aforesaid participation by the 142) Hence, there is evidence on record that at the time of the service of
TRUSTEE, the TRUSTORS have agreed to execute a voting summons on ALFA through the petitioners on August 21, 1987, the voting
trust covering their shareholding in ALFA in favor of the trust agreement in question was not yet terminated so that the legal title to
TRUSTEE; the stocks of ALFA, then, still belonged to the DBP.

AND WHEREAS, DBP is willing to accept the trust for the In view of the foregoing, the ultimate issue of whether or not there was
purpose aforementioned. proper service of summons on ALFA through the petitioners is readily
answered in the negative.
NOW, THEREFORE, it is hereby agreed as follows:
Under section 13, Rule 14 of the Revised Rules of Court, it is provided that:
xxx xxx xxx
Sec. 13. Service upon private domestic corporation or
6. This Agreement shall last for a period of Five (5) years, partnership. If the defendant is a corporation organized
and is renewable for as long as the obligations of ALFA with under the laws of the Philippines or a partnership duly
DBP, or any portion thereof, remains outstanding; (CA Rollo, registered, service may be made on the president, manager,
pp. 137-138) secretary, cashier, agent or any of its directors.

It is a basic principle in Corporation Law that a corporation has a personality


separate and distinct from the officers or members who compose it.
(See Sulo ng Bayan Inc. v. Araneta, Inc., 72 SCRA 347 [1976]; Osias
Academy v. Department of Labor and Employment, et al., G.R. Nos. 83257-
58, December 21, 1990). Thus, the above rule on service of processes of a
corporation enumerates the representatives of a corporation who can validly
receive court processes on its behalf. Not every stockholder or officer can
bind the corporation considering the existence of a corporate entity separate
from those who compose it.

853
The rationale of the aforecited rule is that service must be made on a
representative so integrated with the corporation sued as to make it a
priori supposable that he will realize his responsibilities and know what he
should do with any legal papers served on him. (Far Corporation v.
Francisco, 146 SCRA 197 [1986] citing Villa Rey Transit, Inc. v. Far East
Motor Corp. 81 SCRA 303 [1978]).

The petitioners in this case do not fall under any of the enumerated officers.
The service of summons upon ALFA, through the petitioners, therefore, is not
valid. To rule otherwise, as correctly argued by the petitioners, will
contravene the general principle that a corporation can only be bound by
such acts which are within the scope of the officer's or agent's authority.
(see Vicente v. Geraldez, 52 SCRA 210 [1973]).

WHEREFORE, premises considered, the petition is hereby GRANTED. The


appealed decision dated March 19, 1990 and the Court of Appeals' resolution
of May 10, 1990 are SET ASIDE and the Orders dated April 25, 1989 and
October 17, 1989 issued by the Regional Trial Court of Makati, Branch 58 are
REINSTATED.

SO ORDERED.

Feliciano, Bidin, Davide, Jr. and Romero, JJ., concur.

854
Cruz, Palafox, Alfonso and Associates for petitioner NIDC in G.R. No. 34192.

The Chief Legal Counsel for petitioner PNB in G.R. No. 34213.

Reyes and Sundiam Law Office for respondent Batjak, Inc.

Duran, Chuanico Oebanda, Benemerito & Associates for private respondents


in G.R. Nos. 34192 & 34213.

Tolentino, Garcia, Cruz & Reyes for movant in G.R. No. L-34192.

PADILLA, J.:

These two (2) separate petitions for certiorari and prohibition, with
preliminary injunction, seek to annul and set aside the orders of respondent
judge, dated 16 August 1971 and 30 September 1971, in Civil Case No.
G.R. No. L-34192 June 30, 1988 14452 of the Court of First Instance of Rizal, entitled Batjak Inc. vs. NIDC et
al." The order of 16 August 1971 1 granted the alternative petition of private
respondent Batjak, Inc. Batjak for short) for the appointment of receiver and
NATIONAL INVESTMENT AND DEVELOPMENT CORPORATION,
denied petitioners' motion to dismiss the complaint of said private
EUSEBIO VILLATUYA MARIO Y. CONSING and ROBERTO S.
respondent. The order dated 30 September 1971 2 denied petitioners' motion
BENEDICTO, petitioners,
for reconsideration of the order dated 16 August 1971.
vs.
HON. BENJAMIN AQUINO, in his official capacity as Presiding Judge of
Branch VIII of the Court of First Instance of Rizal, BATJAK INC., The herein petitions likewise seek to prohibit the respondent judge from
GRACIANO A. GARCIA and MARCELINO CALINAWAN JR., respondents. hearing and/or conducting any further proceedings in Civil Case No. 14452 of
said court.
G.R. No. L-34213 June 30, 1988
Batjak, (Basic Agricultural Traders Jointly Administered Kasamahan) is a
Filipino-American corporation organized under the laws of the Philippines,
PHILIPPINE NATIONAL BANK, petitioner,
primarily engaged in the manufacture of coconut oil and copra cake for
vs.
export. In 1965, Batjak's financial condition deteriorated to the point of
HON. BENJAMIN H. AQUINO, in his capacity as Presiding Judge of the
bankruptcy. As of that year, Batjak's indebtedness to some private banks and
Court of First Instance of Rizal, Branch VIII and BATJAK
to the Philippine National Bank (PNB) amounted to P11,915,000.00, shown
INCORPORATED, respondents.
as follows:

855
Republic Bank P 2,324,000.00 PHILIPPINE NATIONAL BANK

Philippine Commercial and Manila, Philippines

Industrial Bank 1,346,000.00 International Department

Manila Banking Corporation 2,000,000.00 October 5, 1

Manufacturers Bank 440,000.00 BATJAK, INCORPORATED

Hongkong and Shanghai 3rd Floor, G. Puyat Bldg.

Banking Corporation 250,000.00 Escolta, Manila

Foreign Export Advances Attn.: Mr. CIRIACO B. MENDOZA

(against immediate shipment) 555,000.00 Vice-President & General Manager

PNB export advance line Gentlemen:

(against immediate shipment) 5,000,000.00 We are pleased to advise that our Board of
Directors approved for you the following:
TOTAL 11,915,000.00
1) That NIDC shall invest P6,722,500.00 in the form of
As security for the payment of its obligations and advances against preferred shares of stocks at 9% cumulative, participating
shipments, Batjak mortgaged its three (3) coco-processing oil mills in Sasa, and convertible within 5 years at par into common stocks to
Davao City, Jimenez, Misamis Occidental and Tanauan, Leyte to Manila liquidate your accounts with the Republic Bank,
Banking Corporation (Manila Bank), Republic Bank (RB), and Philippine Manufacturers Bank & Trust Company and the PCIB which,
Commercial and Industrial Bank (PCIB), respectively. In need for additional however, shall be applied to the latter three (3) banks
operating capital to place the three (3) coco-processing mills at their optimum accounts with the Loans & Discounts Dept. NIDC shall
capacity and maximum efficiency and to settle, pay or otherwise liquidate match your P 10 million subscription by an additional
pending financial obligations with the different private banks, Batjak applied investment of P3,277,500 within a period of one to two years
to PNB for additional financial assistance. On 5 October 1965, a Financial at NIDC's option;
Agreement was submitted by PNB to Batjak for acceptance. The Financial
Agreement reads:

856
2) That NIDC will guaranty for five (5) years your account 10) That any excess of NIDC investment as required in
with the Manila Banking Corporation; Condition 1 after payment of the obligations to three (3)
Banks (RB, MBTC, & PCIB) shall be applied to reduce the
3) That the above banks (Republic Bank, PCIB, MBTC and above Demand Loan of P 5 million;
Manila Banking Corp.) shall release in favor of PNB the first
and any mortgage they hold on your properties; 11) That we shall grant you an export advance of P3 million
to be used for copra purchases, subject to the following
4) That you shall exercise (execute) a first mortgage on all conditions:
your properties located at Sasa, Davao City; Jimenez,
Misamis Occidental; and Tanauan, Leyte and assign a) That the line shall expire on September
leasehold rights on the property on which your plant at Sasa, 30, 1966 but revocable at the Bank(s)
Davao City is erected in favor of PNB; option;

5) That a voting trust agreement for five (5) years over 60% b) That drawings against the line shall be
of the oustanding paid up and subscribed shares shall be allowed only when an irrevocable export L/C
executed by your stockholders in favor of NIDC; for coconut products has been established
or assigned in your favor and you shall
6) That this accomodation shall be secured by the joint and assign to us all proceeds of negotiations to
several signatures of officers and directors; be received from your letters of credit;

7) That the number of the Board of Directors shall be c) That drawings against the line be limited
increased to seven (7), three (3) from your firm and the other to 60% of the peso value of the export
four (4) from the PNB-NIDC; letters of credit computed at P3.50 per $1.00
but total drawings shall not in any event
8) That a comptroller, at your expense, shall be appointed by exceed P3,000,000.00;
PNB-NIDC to supervise the financial management of your
firm; d) That release or releases against the line
shall be covered by promissory note or
9) That the past due accounts of P 5 million with the notes for 90 days but not beyond the expiry
International Department of the PNB shall be transferred to dates of the coveting L/C and proceeds of
the Loans & Discount Department and to be treated as a said L/C shall first be applied to the
Demand Loan; correspondent drawings on the line;

e) That drawings against the line shall be


charged interest at the rate of 9% per
annum and subject to 1/2% penalty charge

857
on all drawings not paid or extended on balance of the investment was to be applied to Batjak's past due account of
maturity date; and P 5 million with the PNB.

f) That within 90 days from date of release Upon receiving payment, RB, PCIB, and MBTC released in favor of PNB the
against the line, you shall negotiate with us first and any mortgages they held on the properties of Batjak.
on equivalent amount in export bills,
otherwise, the line shag be temporarily As agreed, PNB also granted Batjak an export-advance line of P 3 million,
suspended until the outstanding export later increased to P 5million, and a standby letter of credit facility in the
advance is fully liquidated. amount of P5,850,000.00. As of 29 September 1966, the financial
accomodation that had been extended by PNB to Batjak amounted to a total
We are writing the National Investment & Development of P 14,207,859.51.
Corporation, the Republic Bank, the Philippine Commercial
& Industrial Bank and the Manufacturers Bank & Trust As likewise agreed, Batjak executed a first mortgage in favor of PNB on all its
Company and the Manila Banking Corporation regarding the properties located at Jimenez, Misamis Occidental and Tanauan, Leyte.
above. Batjak's plant in Sasa, Davao City was mortgaged to the Manila Bank which,
in 1967, instituted foreclosure proceedings against the same but which were
In connection with the above, kindly submit to us two (2) aborted by the payment by Batjak of the sum of P2,400,000.00 to Manila
copies of your board resolution certified to under oath by Bank, and which amount was advanced to Batjak by NIDC, a wholly-owned
your corporate secretary accepting the conditions subsidiary of PNB. To secure the advance, Batjak mortgaged the oil mill in
enumerated above authorizing the above transactions and Sasa, Davao City to NIDC. 4
the officer or officers to sign on behalf of the corporation.
Next, a Voting Trust Agreement was executed on 26 October 1965 in favor of
Thank you. NIDC by the stockholders representing 60% of the outstanding paid-up and
subscribed shares of Batjak. This agreement was for a period of five (5)
Very trulyyears
yours,and, upon its expiration, was to be subject to negotiation between the
parties. The voting Trust Agreement reads:
(SGD.) JOSE B.
SAMSON VOTING TRUST AGREEMENT

The terms and conditions of the Financial Agreement were duly accepted by KNOW ALL MEN BY THESE PRESENTS:
Batjak. Under said Agreement, NIDC would, as it actually did, invest
P6,722,500.00 in Batjak in the form of preferred shares of stock convertible This AGREEMENT made and executed by the undersigned
within five (5) years at par into common stock, to liquidate Batjak's stockholders of BATJAK, INC., a corporation duly organized
obligations to Republic Bank (RB), Manufacturers Bank and Trust Company and existing under the laws of the Philippines, whose names
(MBTC) and Philippine Commercial & Industrial Bank (PCIB), and the are hereinbelow subscribed hereinafter caged the
SUBSCRIBERS, and the NATIONAL INVESTMENT AND

858
DEVELOPMENT CORPORATION, hereinafter referred to as 2. ASSIGNMENT OF STOCK CERTIFICATES UPON
the trustee. ISSUANCE The undersigned stockholders hereby
transfer and assign their common shares to the capital stock
WITNESSETH: of the CORPORATION to the extent shown hereunder:

WHEREAS, the SUBSCRIBERS are owners respectively of JAMES A. KEISTER 21,500 shares
the capital stock of the BATJAK, INC. (hereinafter called the
CORPORATION) in the amounts represented by the number JOHNNY LIEUSON 20,300 shares
of shares set fort opposite their respective names hereunder;
CBM FINANCE & INVESTMENT
AND WHEREAS, with a view or establishing a safe and
competent management to operate the corporation for the CORP. (C.B. Mendoza, Pres.) 5,000 shares
best interest of all the stockholders thereof, and as mutually
agreed between the SUBSCRIBERS and the TRUSTEE, this ALEJANDRO G. BELTRAN 4,000 shares
Voting Trust Agreement has been executed under the
following terms and conditions.
ESPERANZA A. ZAMORA 3,000 shares

NOW THEREFORE, the undersigned stockholders, in


CIRIACO B. MENDOZA 2,000 shares
consideration of the premises and of the mutual covenants
and agreements herein contained and to carry out the
foregoing purposes in order to vest in the TRUSTEE the FIDELA DE GUZMAN 2,000 shares
voting rights of the shares of stock held by the undersigned
in the CORPORATION as hereinafter stated it is mutually LLOYD D. COMBS 2,000 shares
agreed as follows:
RENATO B. BEJAR 200 shares
1. PERIOD OF DESIGNATION For a period of five (5)
years from and after date hereof, without power of TOTAL 60,000 shares
revocation on the part of the SUBSCRIBERS, the TRUSTEE
designated in the manner herein provided is hereby made, to the TRUSTEE by virtue of the provisions hereof and do
constituted and appointed as a VOTING TRUSTEE to act for hereby authorize the Secretary of the CORPORATION to
and in the name of the SUBSCRIBERS, it being understood, issue the corresponding certificate directly in the name of the
however, that this Voting Trust Agreement shall, upon its TRUSTEE and on which certificates it shall appear that they
expiration be subject to a re-negotiation between the parties, have been issued pursuant to this Voting Trust Agreement
as may be warranted by the balance and attending and the said TRUSTEE shall hold in escrow all such
circumstance of the loan investment of the TRUSTEE or certificates during the term of the Agreement. In turn, the
otherwise in the CORPORATION. TRUSTEE shall deliver to the undersigned stockholders the

859
corresponding Voting Trust certificates provided for in Sec. stockholdings after deducting therefrom or applying the
36 of Act No. 1459. same to whatever liabilities the stockholders may have in
favor of the TRUSTEE by virtue of any Agreement or
3. VOTING POWER OF TRUSTEE The TRUSTEE and Contract that may have been or will be executed by and
its successors in trust, if anym shall have the power and it between the TRUSTEE and the CORPORATION or between
shall be its duty to vote the shares of the undersigned the former and the undersigned stockholders.
subject hereof and covered by this Agreement at all annual,
adjourned and special meetings of the CORPORATION on 6 COMPENSATION; IMMUNITY The TRUSTEE or its
all questions, motions, resolutions and matters including the successor in trust shall not receive any compensation for its
election of directors and such matters on which the serviceexcept perhaps that which the CORPORATION may
stockholders, by virtue of the by-laws of the CORPORATION grant to the TRUSTEE's authorized representative, if any.
and of the existing legislations are entitled to vote, which Expenses costs, champs, and other liabilities incurred in the
may be voted upon at any and all said meetings and shall carrying out of the but herein established or by reason
also have the power to execute and acknowledge any thereof, shall be paid for with the funds of the
agreements or documents that may be necessary in its CORPORATION. The TRUSTEE or any of its duly
opinion to express the consent or assent of all or any of the authorized representative shall incur no liability by reason of
stockholders of the CORPORATION with respect to any any error of law or of any matter or thing done or omitted
matter or thing to which any consent or assent of the under this Agreement, except for his own individual
stockholders may be necessary, proper or convenient. malfeasance.

4. FILING of AGREEMENT An executed copy of this 7. REPRESENTATION The TRUSTEE, being a


Agreement shall be filed with the CORPORATION at its corporation and a juridical person shall accomplish the
office in the City of Manila wherever it may be transfered foregoing objectives and perform its functions under this
therefrom and shall constitute irrevocable authority and Agreement as well as enjoy and exercise the powers,
absolute direction of the officers of the CORPORATION privileges, rights and interests herein established through its
whose duty is to sign and deliver stock certificates to make duly authorized and accredited re resentatives . p with full
delivery only to said voting trustee of the shares and authority under the specific appointment or designation or
certificates of stock subject to the provisions of this Proxy.
Agreement as aforesaid. Such copy of this Agreement shall
at all times be open to inspection by any stockholder, as 8. IRREVOCABILITY This Agreement shall during its 5-
provided by law. year term or any extension thereof be binding upon and
inure to the benefit of the undersigned stockholders and their
5. DIVIDEND the full and absolute beneficial interest in respective legal representatives, pledges, transferees,
the shares subject of this Agreement shall remain with the and/or assigns and shall be irrevocable during the said terms
stockholders executing the same and any all dividends and/or its extension pursuant to the provisions of paragraph
which may be declared by the CORPORATION shall belong 1 hereof. It is hereby understood and the undersigned
and be paid to them exclusively in accordance with their stockholders have bound as they hereby bind themselves to

860
make a condition of every pledge, transfer of assignment of President
their interests in the CORPORATION that the interests and
participation so pledged, transferred or assigned is ESPERANZA A. ZAMORA (SGD) ALEJANDRO G.
evidenced by annotations in the certificates of stocks or in BELTRAN
the books of the corporation, shall be subject to this
Agreement and the same shall be binding upon the By: (SGD) MARIANO ZAMORA Stockholder
pledgees, transferees and assigns while the trust herein
created still subsists.
ESPERANZA A. ZAMORA

9. TERMINATION Upon termination of this Agreement as


(SGD) FIDELA DE GUZMAN (SGD) CIRIACO B. MENDOZA
heretofore provided, the certificates delivered to the
TRUSTEE by virtue hereof shall be returned and delivered to
the undersigned stockholders as the absolute owners Stockholder Stockholder
thereof, upon surrender of their respective voting trust
certificates, and the duties of the TRUSTEE shall cease and (SGD) RENATO B. BEJAR (SGD) LLOYD D. COMBS
terminate.
Stockholder Stockholder
10. ACCEPTANCE OF TRUST The TRUSTEE hereby
accepts the trust created by this Agreement under the NAT
signature of its duly authorized representative affixed ION
hereinbelow and agrees to perform the same in accordance AL
with the term/s hereof. INV
EST
IN WITNTESS HEREOF, the undersigned stockholders and ME
the TRUSTEE by its representatives, have hereunto affixed NT
their signatures this 26 day of October, 1965 in the City of AN
Manila, Philippines. D

(SGD) JAMES A. KEISER (SGD) JOHNNY LIEUSON

Stockholder Stockholder

CBM FINANCE & INVESTMENT CORPORATION

By: (SGD) C.B. MENDOZA

861
DE As regards the oil mill located at Sasa, Davao City, the same was similarly
VEL foreclosed extrajudicial by NIDC. It was sold to NIDC as the highest bidder.
OP After Batjak failed to redeem the property, NIDC consolidated its ownership
ME of the oil mill. 8
NT
CO Three (3) years thereafter, or on 31 August 1970, Batjak represented by
RP majority stockholders, through Atty. Amado Duran, legal counsel of private
OR respondent Batjak, wrote a letter to NIDC inquiring if the latter was still
ATI interested in negotiating the renewal of the Voting Trust Agreement. 9 On 22
ON September 1970, legal counsel of Batjak wrote another letter to NIDC
informing the latter that Batjak would now safely assume that NIDC was no
By: longer interested in the renewal of said Voting Trust Agreement and, in view
thereof, requested for the turn-over and transfer of all Batjak assets,
(SG properties, management and operations. 10
D)
IGN On 23 September 1970, legal counsel of Batjak sent stin another letter to
ACI NIDC, this time asking for a complete accounting of the assets, properties,
O management and operation of Batjak, preparatory to their turn-over and
DE transfer to the stockholders of Batjak. 11
BU
QU NIDC replied, confirming the fact that it had no intention whatsoever to
E comply with the demands of Batjak. 12
JR.
On 24 February 1971, Batjak filed before the Court of First Instance of Rizal
Vice-President
a special civil action for mandamus with preliminary injunction against herein
petitioners docketed as Civil Case No. 14452. 13
In July 1967, forced by the insolvency of Batjak, PNB instituted extrajudicial
foreclosure proceedings against the oil mills of Batjak located in Tanauan, On 14 April 1971, in said Civil Case No. 14452, Batjak filed an urgent ex
Leyte and Jimenez, Misamis Occidental. The properties were sold to PNB as parte motion for the issuance of a writ of preliminary prohibitory and
the highest bidder. One year thereafter, or in September 1968, final mandatory injunction. 14 On the same day, respondent judge issued a
Certificates of Sale were issued by the provincial sheriffs of Leyte 6 and restraining order "prohibiting defendants (herein petitioners) from removing
Misamis Occidental 7 for the two (2) oil mills in Tanauan and Jimenez in favor any record, books, commercial papers or cash, and leasing, renting out,
of PNB, after Batjak failed to exercise its right to redeem the foreclosed disposing of or otherwise transferring any or all of the properties,
properties within the allowable one year period of redemption. Subsequently, machineries, raw materials and finished products and/or by-products thereof
PNB transferred the ownership of the two (2) oil mills to NIDC which, as now in the factory sites of the three (3) modem coco milling plants situated in
aforestated, was a wholly-owned PNB subsidiary. Jimenez, Misamis Occidental, Sasa, Davao City, and Tanauan, Leyte." 15

862
The order of 14 April 1971 was subsequently amended by respondent judge of the aggrieved party in a denied motion to dismiss is to file an answer and
upon an ex parte motion of private respondent Batjak so as to include the interpose, as defense or defenses, the objection or objections raised by him
premises of NIDC in Makati and those of PNB in Manila, as among the in said motion to dismiss, then proceed to trial and, in case of adverse
premises which private respondent Batjak was authorized to enter in order to decision, to elevate the entire case by appeal in due course. However, under
conduct an inventory. certain situations, recourse to the extraordinary legal remedies of certiorari,
prohibition and mandamus to question the denial of a motion to dismiss or
On 24 April 1971, NIDC and PNB filed an opposition to the ex quash is considered proper, in the interest of more enlightened and
parte application for the issuance of a writ of preliminary prohibitory and substantial justice. As the court said in Pineda and Ampil Manufacturing Co.
mandatory injunction and a motion to set aside restraining order. vs. Bartolome, 95 Phil. 930,938

Before the court could act on the said motion, private respondent Batjak filed For analogous reasons it may be said that the petition for
on 3 May 1971 a petition for receivership as alternative to writ of preliminary certiorari interposed by the accused against the order of the
prohibitory and mandatory injunction. 16 This was opposed by PNB and NIDC court a quo denying the motion to quash may be entertained,
. 17 not only because it was rendered in a criminal case, but
because it was rendered, as claimed, with grave abuse of
On 8 May 1971., NIDC and PNB filed a motion to dismiss Batjak's discretion, as found by the Court of Appeals. ..
complaints. 18
and reiterated in Mead v. Argel 22 citing Yap v. Lutero (105 Phil. 1307):
On 16 August 1971, respondent judge issued the now assailed order denying
petitioners' motion to dismiss and appointing a set of three (3) However, were we to require adherence to this pretense, the
receivers. 19 NIDC moved for reconsideration of the aforesaid order. 20 On 30 case at bar would have to be dismissed and petitioner
September 1971, respondent judge denied the motion for reconsideration. 21 required to go through the inconvenience, not to say the
mental agony and torture, of submitting himself to trial on the
Hence, these two (2) petitions, which have been consolidated, as they merits in Case No. 166443, apart from the expenses
involve a resolution of the same issues. In their manifestation with motion for incidental thereto, despite the fact that his trial and
early decision, dated 25 August 1986, private respondent, Batjak contends conviction therein would violate one of this [sic] constitutional
that the NIDC has already been abolished or scrapped by its parent rights, and that, an appeal to this Court, we would, therefore,
company, the PNB. have to set aside the judgment of conviction of the lower
court. This would, obviously, be most unfair and unjust.
Under the circumstances obtaining the present case, the
After a careful study and examination of the records of the case, the Court
flaw in the procedure followed by petitioner herein may be
finds and holds for the petitioners.
overlooked, in the interest of a more enlightened and
substantial justice.
1. On the denial of petitioners' motion to dismiss.

As a general rule, an order denying a motion to quash or to dismiss is


interlocutory and cannot be the subject of a petition for certiorari. The remedy

863
Thus, where there is patent grave abuse of discretion, in denying the motion In support of the third ground of their motion to dismiss, PNB and NIDC
to dismiss, as in the present case, this Court may entertain the petition for contend that Batjak's complaint for mandamus is based on its claim or right
certiorari interposed by the party against whom the said order is issued. to recovery of possession of the three (3) oil mills, on the ground of an
alleged breach of fiduciary relationship. Noteworthy is the fact that, in the
In their motion to dismiss Batjaks complaint, in Civil Case No. 14452, NIDC Voting Trust Agreement, the parties thereto were NIDC and certain
and PNB raised common grounds for its allowance, to wit: stockholders of Batjak. Batjak itself was not a signatory thereto. Under Sec.
2, Rule 3 of the Rules of Court, every action must be prosecuted and
1. This Honorable Court (the trial court) has no jurisdiction defended in the name of the real party in interest. Applying the rule in the
over the subject of the action or suit; present case, the action should have been filed by the stockholders of
Batjak, who executed the Voting Trust Agreement with NIDC, and not by
Batjak itself which is not a party to said agreement, and therefore, not the
2. The venue is improperly laid; and
real party in interest in the suit to enforce the same.

3. Plaintiff has no legal capacity to sue.


In addition, PNB claims that Batjak has no cause of action and prays that the
petition for mandamus be dismissed. A careful reading of the Voting Trust
In addition, PNB contended that the complaint states no cause of action Agreement shows that PNB was really not a party thereto. Hence,
(Rule 16, Sec. 1, Par. a, c, d & g, Rules of Court). mandamus will not lie against PNB.

Anent the first ground, it is a well-settled rule that the jurisdiction of a Court of Moreover, the action instituted by Batjak before the respondent court was a
First Instance to issue a writ of preliminary or permanent injunction is special civil action for mandamus with prayer for preliminary mandatory
confined within the boundaries of the province where the land in controversy injunction. Generally, mandamus is not a writ of right and its allowance or
is situated. 23 The petition for mandamus of Batjak prayed that NIDC and refusal is a matter of discretion to be exercised on equitable principles and in
PNB be ordered to surrender, relinquish and turnover to Batjak the assets, accordance with well-settled rules of law, and that it should never be used to
management and operation of Batjak particularly the three (3) oil mills effectuate an injustice, but only to prevent a failure of justice. 24 The writ does
located in Sasa, Davao City, Jimenez, Misamis Occidental and Tanauan, not issue as a matter of course. It will issue only where there is a clear legal
Leyte. right sought to be enforced. It will not issue to enforce a doubtful right. A clear
legal right within the meaning of Sec. 3, Rule 65 of the Rules of Court means
Clearly, what Batjak asked of respondent court was the exercise of power or a right clearly founded in or granted by law, a right which is enforceable as a
authority outside its jurisdiction. matter of law.

On the matter of proper venue, Batjak's complaint should have been filed in Applying the above-cited principles of law in the present case, the Court finds
the provinces where said oil mills are located. Under Rule 4, Sec. 2, no clear right in Batjak to be entitled to the writ prayed for. It should be noted
paragraph A of the Rules of Court, "actions affecting title to, or for recovery of that the petition for mandamus filed by it prayed that NIDC and PNB be
possession, or for partition or condemnation of, or foreclosure of mortgage ordered to surrender, relinquish and turn-over to Batjak the assets,
on, real property, shall be commenced and tried in the province where the management, and operation of Batjak particularly the three (3) oil mills and to
property or any part thereof lies." make the order permanent, after trial, and ordering NIDC and PNB to submit
a complete accounting of the assets, management and operation of Batjak

864
from 1965. In effect, what Batjak seeks to recover is title to, or possession of, Batjak premises its right to the possession of the three (3) off mills on the
real property (the three (3) oil mills which really made up the assets of Voting Trust Agreement, claiming that under said agreement, NIDC was
Batjak) but which the records show already belong to NIDC. It is not disputed constituted as trustee of the assets, management and operations of Batjak,
that the mortgages on the three (3) oil mills were foreclosed by PNB and that due to the expiration of the Voting Trust Agreement, on 26 October 1970,
NIDC and acquired by them as the highest bidder in the appropriate NIDC should tum over the assets of the three (3) oil mills to Batjak. The
foreclosure sales. Ownership thereto was subsequently consolidated by PNB relevant provisions of the Voting Trust Agreement, particularly paragraph 4 &
and NIDC, after Batjak failed to exercise its right of redemption. The three (3) No. 1 thereof, are hereby reproduced:
oil mills are now titled in the name of NIDC. From the foregoing, it is evident
that Batjak had no clear right to be entitled to the writ prayed for. In Lamb vs. NOW THEREFORE, the undersigned stockholders, in
Philippines(22 Phil. 456) citing the case of Gonzales V. Salazar vs. The consideration of the premises and of the mutual covenants
Board of Pharmacy, 20 Phil. 367, the Court said that the writ of mandamus and agreements herein contained and to carry out the
will not issue to give to the applicant anything to which he is not entitled by foregoing purposes in order to vest in the TRUSTEE the
law. voting right.8 of the shares of stock held by the undersigned
in the CORPORATION as hereinafter stated it is mutually
2. On the appointment of receiver. agreed as follows:

A receiver of real or personal property, which is the subject of the action, may 1. PERIOD OF DESIGNATION For a period of five (5)
be appointed by the court when it appears from the pleadings that the party years from and after date hereof, without power of revocation
applying for the appointment of receiver has an interest in said on the part of the SUBSCRIBERS, the TRUSTEE
property. 25 The right, interest, or claim in property, to entitle one to a receiver designated in the manner herein provided is hereby made,
over it, must be present and existing. constituted and appointed as a VOTING TRUSTEE to act for
and in the name of the SUBSCRIBERS, it being understood,
As borne out by the records of the case, PNB acquired ownership of two (2) however, that this Voting Trust Agreement shall, upon its
of the three (3) oil mills by virtue of mortgage foreclosure sales. NIDC expiration be subject to a re-negotiation between the parties,
acquired ownership of the third oil mill also under a mortgage foreclosure as may be warranted by the balance and attending
sale. Certificates of title were issued to PNB and NIDC after the lapse of the circumstance of the loan investment of the TRUSTEE or
one (1) year redemption period. Subsequently, PNB transferred the otherwise in the CORPORATION.
ownership of the two (2) oil mills to NIDC. There can be no doubt, therefore,
that NIDC not only has possession of, but also title to the three (3) oil mills and No. 3 thereof reads:
formerly owned by Batjak. The interest of Batjak over the three (3) oil mills
ceased upon the issuance of the certificates of title to PNB and NIDC 3. VOTING POWER OF TRUSTEE The TRUSTEE and its
confirming their ownership over the said properties. More so, where Batjak successors in trust, if any, shall have the power and it shall
does not impugn the validity of the foreclosure proceedings. Neither Batjak be its duty to vote the shares of the undersigned subject
nor its stockholders have instituted any legal proceedings to annul the hereof and covered by this Agreement at all annual,
mortgage foreclosure aforementioned. adjourned and special meetings of the CORPORATION on
all questions, motions, resolutions and matters including the

865
election of directors and all such matters on which the In any event, a voting trust transfers only voting or other rights pertaining to
stockholders, by virtue of the by-laws of the CORPORATION the shares subject of the agreement or control over the stock. The law on the
and of the existing legislations are entitled to vote, which matter is Section 59, Paragraph 1 of the Corporation Code (BP 68) which
may be voted upon at any and all said meetings and shall provides:
also have the power to execute and acknowledge any
agreements or documents that may be necessary in its Sec. 59. Voting Trusts One or more stockholders of a
opinion to express the consent or assent of all or any of the stock corporation may create a voting trust for the purpose of
stockholders of the CORPORATION with respect to any confering upon a trustee or trusties the right to vote and
matter or thing to which any consent or assent of the other rights pertaining to the shares for a period not
stockholders may be necessary, proper or convenient. exceeding five (5) years at any one time: ... 26

From the foregoing provisions, it is clear that what was assigned to NIDC The acquisition by PNB-NIDC of the properties in question was not made or
was the power to vote the shares of stock of the stockholders of Batjak, effected under the capacity of a trustee but as a foreclosing creditor for the
representing 60% of Batjak's outstanding shares, and who are the purpose of recovering on a just and valid obligation of Batjak.
signatories to the agreement. The power entrusted to NIDC also included the
authority to execute any agreement or document that may be necessary to Moreover, the prevention of imminent danger to property is the guiding
express the consent or assent to any matter, by the stockholders. Nowhere in principle that governs courts in the matter of appointing receivers. Under
the said provisions or in any other part of the Voting Trust Agreement is Sec. 1 (b), Rule 59 of the Rules of Court, it is necessary in granting the relief
mention made of any transfer or assignment to NIDC of Batjak's assets, of receivership that the property or fired be in danger of loss, removal or
operations, and management. NIDC was constituted as trustee only of the material injury.
voting rights of 60% of the paid-up and outstanding shares of stock in Batjak.
This is confirmed by paragraph No. 9 of the Voting Trust Agreement, thus:
In the case at bar, Batjak in its petition for receivership, or in its amended
petition therefor, failed to present any evidence, to establish the requisite
9. TERMINATION Upon termination of this Agreement as condition that the property is in danger of being lost, removed or materially
heretofore provided, the certificates delivered to the injured unless a receiver is appointed to guard and preserve it.
TRUSTEE by virtue hereof shall be returned and delivered to
the undersigned stockholders as the absolute owners
WHEREFORE, the petitions are GRANTED. The orders of the respondent
thereof, upon surrender of their respective voting trust
judge, dated 16 August 1971 and 30 September 1971, are hereby
certificates, and the duties of the TRUSTEE shall cease and
ANNULLED and SET ASIDE. The respondent judge and/or his successors
terminate.-
are ordered to desist from hearing and/or conducting any further proceedings
in Civil Case No. 14452, except to dismiss the same. With costs against
Under the aforecited provision, what was to be returned by NIDC as trustee private respondents.
to Batjak's stockholders, upon the termination of the agreement, are the
certificates of shares of stock belonging to Batjak's stockholders, not the
SO ORDERED.
properties or assets of Batjak itself which were never delivered, in the first
place to NIDC, under the terms of said Voting Trust Agreement.
Yap, C.J., Melencio-Herrera, Paras and Sarmiento, JJ., concur.

866
867
G.R. No. L-15568 November 8, 1919

W. G. PHILPOTTS, petitioner,
vs.
PHILIPPINE MANUFACTURING COMPANY and F. N.
BERRY, respondents.

868
Lawrence and Ross for petitioner. with the custody of all documents, correspondence, and records of a
Crossfield and O'Brien for defendants. corporation, and he is presumably the person against whom the personal
orders of the court would be made effective in case the relief sought should
be granted. Certainly there is nothing in the complaint to indicate that the
secretary is an improper person to be joined. The petitioner might have
STREET, J.: named the president of the corporation as a respondent also; and this official
might be brought in later, even after judgment rendered, if necessary to the
effectuation of the order of the court.
The petitioner, W. G. Philpotts, a stockholder in the Philippine Manufacturing
Company, one of the respondents herein, seeks by this proceeding to obtain
a writ of mandamus to compel the respondents to permit the plaintiff, in Section 222 of our Code of Civil Procedure is taken from the California Code,
person or by some authorized agent or attorney, to inspect and examine the and a decision of the California Supreme Court Barber vs. Mulford (117
records of the business transacted by said company since January 1, 1918. Cal., 356) is quite clear upon the point that both the corporation and its
The petition is filed originally in this court under the authority of section 515 of officers may be joined as defendants.
the Code of Civil Procedure, which gives to this tribunal concurrent
jurisdiction with the Court of First Instance in cases, among others, where The real controversy which has brought these litigants into court is upon the
any corporation or person unlawfully excludes the plaintiff from the use and question argued in connection with the second ground of demurrer, namely,
enjoyment of some right to which he is entitled. The respondents interposed whether the right which the law concedes to a stockholder to inspect
a demurrer, and the controversy is now before us for the determination of the the records can be exercised by a proper agent or attorney of the
questions thus presented. stockholder as well as by the stockholder in person. There is no
pretense that the respondent corporation or any of its officials has refused to
The first point made has reference to a supposed defect of parties, and it is allow the petitioner himself to examine anything relating to the affairs of the
said that the action can not be maintained jointly against the corporation and company, and the petition prays for a peremptory order commanding the
its secretary without the addition of the allegation that the latter is the respondents to place the records of all business transactions of the company,
custodian of the business records of the respondent company. during a specified period, at the disposal of the plaintiff or his duly authorized
agent or attorney, it being evident that the petitioner desires to exercise said
right through an agent or attorney. In the argument in support of the demurrer
By the plain language of sections 515 and 222 of our Code of Civil
it is conceded by counsel for the respondents that there is a right of
Procedure, the right of action in such a proceeding as this is given against
examination in the stockholder granted under section 51 of the Corporation
the corporation; and the respondent corporation in this case was the only
Law, but it is insisted that this right must be exercised in person.
absolutely necessary party. In the Ohio case of Cincinnati Volksblatt
Co. vs. Hoffmister (61 Ohio St., 432; 48 L. R. A., 735), only the corporation
was named as defendant, while the complaint, in language almost identical The pertinent provision of our law is found in the second paragraph of
with that in the case at bar, alleged a demand upon and refusal by the section 51 of Act No. 1459, which reads as follows: "The record of all
corporation. business transactions of the corporation and the minutes of any meeting
shall be open to the inspection of any director, member or stockholder of the
corporation at reasonable hours."
Nevertheless the propriety of naming the secretary of the corporation as a
codefendant cannot be questioned, since such official is customarily charged

869
This provision is to be read of course in connecting with the related inspection given by law to the stockholder; as for instance, where a
provisions of sections 51 and 52, defining the duty of the corporation in corporation, engaged in the business of manufacture, has acquired a
respect to the keeping of its records. formula or process, not generally known, which has proved of utility to
it in the manufacture of its products. It is not our intention to declare
Now it is our opinion, and we accordingly hold, that the right of that the authorities of the corporation, and more particularly the Board
inspection given to a stockholder in the provision above quoted can be of Directors, might not adopt measures for the protection of such
exercised either by himself or by any proper representative or attorney process form publicity. There is, however, nothing in the petition which
in fact, and either with or without the attendance of the stockholder. would indicate that the petitioner in this case is seeking to discover
This is in conformity with the general rule that what a man may do in anything which the corporation is entitled to keep secret; and if
person he may do through another; and we find nothing in the statute anything of the sort is involved in the case it may be brought out at a
that would justify us in qualifying the right in the manner suggested by more advanced stage of the proceedings.lawphil.net
the respondents.
The demurrer is overruled; and it is ordered that the writ of mandamus shall
This conclusion is supported by the undoubted weight of authority in the issue as prayed, unless within 5 days from notification hereof the
United States, where it is generally held that the provisions of law conceding respondents answer to the merits. So ordered.
the right of inspection to stockholders of corporations are to be liberally
construed and that said right may be exercised through any other properly Arellano, C.J., Torres, Johnson, Araullo, Malcolm and Avancea, JJ., concur.
authorized person. As was said in Foster vs. White (86 Ala., 467), "The right
may be regarded as personal, in the sense that only a stockholder may enjoy
it; but the inspection and examination may be made by another. Otherwise it
would be unavailing in many instances." An observation to the same effect is
contained in Martin vs. Bienville Oil Works Co. (28 La., 204), where it is said:
"The possession of the right in question would be futile if the possessor of it,
through lack of knowledge necessary to exercise it, were debarred the right
of procuring in his behalf the services of one who could exercise it." In
Deadreck vs. Wilson (8 Baxt. [Tenn.], 108), the court said: "That stockholders
have the right to inspect the books of the corporation, taking minutes from
the same, at all reasonable times, and may be aided in this by experts and
counsel, so as to make the inspection valuable to them, is a principle too well
settled to need discussion." Authorities on this point could be accumulated in
great abundance, but as they may be found cited in any legal encyclopedia
or treaties devoted to the subject of corporations, it is unnecessary here to
refer to other cases announcing the same rule.

In order that the rule above stated may not be taken in too sweeping a
sense, we deem it advisable to say that there are some things which a
corporation may undoubtedly keep secret, notwithstanding the right of

870
G.R. No. L-22442 August 1, 1924

ANTONIO PARDO, petitioner,


vs.
THE HERCULES LUMBER CO., INC., and IGNACIO
FERRER, respondents.

W.J. O'Donovan and M.H. de Joya for petitioner.


Sumulong and Lavides and Ross, Lawrence and Selph for respondents.

871
STREET, J.: The board also resolved to call the usual general (meeting of shareholders)
for March 30 of the present year, with notice to the shareholders that the
The petitioner, Antonio Pardo, a stockholder in the Hercules Lumber books of the company are at their disposition from the 15th to 25th of the
Company, Inc., one of the respondents herein, seeks by this original same month for examination, in appropriate hours.
proceeding in the Supreme Court to obtain a writ of mandamus to compel the
respondents to permit the plaintiff and his duly authorized agent and The contention for the respondent is that this resolution of the board
representative to examine the records and business transactions of said constitutes a lawful restriction on the right conferred by statute; and it
company. To this petition the respondents interposed an answer, in which, is insisted that as the petitioner has not availed himself of the
after admitting certain allegations of the petition, the respondents set forth permission to inspect the books and transactions of the company
the facts upon which they mainly rely as a defense to the petition. To this within the ten days thus defined, his right to inspection and
answer the petitioner in turn interposed a demurrer, and the cause is now examination is lost, at least for this year.
before us for determination of the issue thus presented.
We are entirely unable to concur in this contention. The general right given
It is inferentially, if not directly admitted that the petitioner is in fact a by the statute may not be lawfully abridged to the extent attempted in
stockholder in the Hercules Lumber Company, Inc., and that the respondent, this resolution. It may be admitted that the officials in charge of a
Ignacio Ferrer, as acting secretary of the said company, has refused to corporation may deny inspection when sought at unusual hours or
permit the petitioner or his agent to inspect the records and business under other improper conditions; but neither the executive officers nor
transactions of the said Hercules Lumber Company, Inc., at times desired by the board of directors have the power to deprive a stockholder of the
the petitioner. No serious question is of course made as to the right of the right altogether. A by-law unduly restricting the right of inspection is
petitioner, by himself or proper representative, to exercise the right of undoubtedly invalid. Authorities to this effect are too numerous and direct
inspection conferred by section 51 of Act No. 1459. Said provision was under to require extended comment. (14 C.J., 859; 7 R.C.L., 325; 4 Thompson on
the consideration of this court in the case of Philpotts vs. Philippine Corporations, 2nd ed., sec. 4517; Harkness vs. Guthrie, 27 Utah, 248; 107
Manufacturing Co., and Berry (40 Phil., 471), where we held that the right of Am., St. Rep., 664. 681.) Under a statute similar to our own it has been held
examination there conceded to the stockholder may be exercised either by a that the statutory right of inspection is not affected by the adoption by the
stockholder in person or by any duly authorized agent or representative. board of directors of a resolution providing for the closing of transfer books
thirty days before an election. (State vs. St. Louis Railroad Co., 29 Mo., Ap.,
The main ground upon which the defense appears to be rested has 301.)
reference to the time, or times, within which the right of inspection may be
exercised. In this connection the answer asserts that in article 10 of the By- It will be noted that our statute declares that the right of inspection can
laws of the respondent corporation it is declared that "Every shareholder be exercised "at reasonable hours." This means at reasonable hours on
may examine the books of the company and other documents business days throughout the year, and not merely during some
pertaining to the same upon the days which the board of directors shall arbitrary period of a few days chosen by the directors.
annually fix." It is further averred that at the directors' meeting of the
respondent corporation held on February 16, 1924, the board passed a In addition to relying upon the by-law, to which reference is above made, the
resolution to the following effect: answer of the respondents calls in question the motive which is supposed to
prompt the petitioner to make inspection; and in this connection it is alleged

872
that the information which the petitioner seeks is desired for ulterior purposes
in connection with a competitive firm with which the petitioner is alleged to be
connected. It is also insisted that one of the purposes of the petitioner is to
obtain evidence preparatory to the institution of an action which he means to
bring against the corporation by reason of a contract of employment which
once existed between the corporation and himself. These suggestions are
entirely apart from the issue, as, generally speaking, the motive of the
shareholder exercising the right is immaterial. (7 R.C.L., 327.)

We are of the opinion that, upon the allegations of the petition and the
admissions of the answer, the petitioner is entitled to relief. The demurrer is,
therefore, sustained; and the writ of mandamus will issue as prayed, with the
costs against the respondent. So ordered.

Johnson, Malcolm, Villamor, Ostrand, and Romualdez, JJ., concur.

873
874
directors, and to place at his disposal at reasonable hours, the minutes, and
documents, and books of the aforesaid corporation, for his inspection as
director and stockholder, and to issue, upon payment of the fees, certified
copies of any documentation in connection with said minutes, documents,
and books of the corporation; and (b) that, in view of the memoranda and
hearing of the parties, a final and absolute writ of mandamus be issued to
each and all of the respondents to notify immediately the petitioner within the
reglamentary period, of all regular and special meetings of the board of
directors of the Isabela Sugar Central Company, Inc., and to place at his
disposal at reasonable hours the minutes, documents, and books of said
corporation for his inspection as director and stockholder, and to issue
immediately, upon payment of the fees, certified copies of any documentation
in connection with said minutes, documents, and the books of the aforesaid
G.R. No. L-37064 October 4, 1932 corporation. To the petition an answer has been interposed by the
respondent, too long to be here summarized, which raised questions of fact
EUGENIO VERAGUTH, Director and Stockholder of the Isabela Sugar and law. Following the taking of considerable before the clerk as
Company, Inc., petitioner, commissioner, the case has been submitted on memoranda.
vs.
ISABELA SUGAR COMPANY, INC., GIL MONTILLA, Acting President, It should first be observed that when the case was filed here, it was, in
and AGUSTIN B. MONTILLA, Secretary of the same accordance with settled practice, dismissed without prejudice to the right of
corporation, respondents. the petitioner to present the action before the Court of First Instance of
Occidental Negros. Thereafter, on a motion of reconsideration being
Jose B. Gamboa for petitioner. presented, this order was set aside and the case was permitted to continue
Agustin P. Seva for respondents. in this court. On further reflection, we now feel that this was error, and that it
would have been the correct practice to have required the petitioner to
present the action in a court of First Instance which is better equipped for the
taking of testimony and the resolution of questions of fact than is the
appellate court. Only with considerable difficulty, therefore, can we decide the
MALCOLM, J.:
issues of fact, since none of the members of the court saw or heard the
witnesses testify.
The parties to this action are Eugenio Veraguth, a director and stockholder of
the Isabela Sugar Company, Inc., who is the petitioner, and the Isabela
Speaking to the first point with which the petition is concerned, relating to the
Sugar Company, Inc., Gil Montilla, acting president of the company, and
alleged failure of the secretary of the company to notify the petitioner in due
Agustin B. Montilla, secretary of the company, who are the respondents. The
time of a special meeting of the company, we find by-laws, together with a
petitioner prays: (a) That the respondents be required within five days from
resolution of the board of directors, providing for the holding of ordinary and
receipt of notice of this petition to show cause why they refuse to notify the
special meetings. Whether there was a malicious attempt to keep Director
petitioner, as director, of the regular and special meetings of the board of
Veraguth from attending a special meeting of the board of the board of

875
directors at which the compensation of the attorneys of the company was The record of all business transactions of the corporation and the
fixed, or whether Director Veraguth, in a spirit of antogonism, has made this minutes of any meeting shall be open to the inspection of any
merely a pretext to cause trouble, we are unable definitely to say. This much, director, member, or stockholder of the corporation at reasonable
however, can appropriately be stated and is decisive, and this is that the hours.
meeting in question is in the past and, therefore, now merely presents an
academic question; that no damage was caused to Veraguth by the action The above puts in statutory form the general principles of Corporation
taken at the special meeting which he did not attend, since his interests were Law. Directors of a corporation have the unqualified right to inspect the
fully protected by the Philippine National Bank; and that as to meetings in the books and records of the corporation at all reasonable times. Pretexts
future it is to be presumed that the secretary of the company will fulfill the may not be put forward by officers of corporations to keep a director or
requirements of the resolutions of the company pertaining to regular and shareholder from inspecting the books and minutes of the corporation,
special meetings. It will, of course, be incumbent upon Veraguth to give and the right of inspection is not to be denied on the ground that the
formal notice to the secretary of his post-office address if he desires notice director or shareholder is on unfriendly terms with the officers of the
sent to a particular residence. 1awphil.net corporation whose records are sought to be inspected. A director or
stockholder can not of course make copies, abstracts, and memoranda
On the second question pertaining to the right of inspection of the books of of documents, books, and papers as an incident to the right of
the company, we find Director Veraguth telegraphing the secretary of the inspection, but cannot, without an order of a court, be permitted to take
company, asking the latter to forward in the shortest possible time a certified books from the office of the corporation. We do not conceive, however,
copy of the resolution of the board of directors concerning the payment of that a director or stockholder has any absolute right to secure certified
attorney's fees in the case against the Isabela Sugar Company and others. copies of the minutes of the corporation until these minutes have been
To this the secretary made answer by letter stating that, since the minutes of written up and approved by the directors. (See Fisher's Philippine Law of
the meeting in question had not been signed by the directors present, a Stock Corporations, sec. 153, and Fletcher Cyclopedia Corporations, vol. 4,
certified copy could not be furnished and that as to other proceedings of the Chap. 45.)
stockholders a request should be made to the president of the Isabela Sugar
Company, Inc. It further appears that the board of directors adopted a Combining the facts and the law, we do not think that anything improper
resolution providing for inspection of the books and the taking of copies "by occurred when the secretary declined to furnish certified copies of minutes
authority of the President of the corporation previously obtained in each which had not been approved by the board of directors, and that while so
case." much of the last resolution of the board of directors as provides for
prior approval of the president of the corporation before the books of
The Corporation Law, section 51, provides that: the corporation can be inspected puts an illegal obstacle in the way of a
stockholder or director, that resolution, so far as we are aware, has not
All business corporations shall keep and carefully preserve a record been enforced to the detriment of anyone. In addition, it should be said
of all business transactions, and a minute of all meetings of that this is a family dispute, the petitioner and the individual respondents
directors, members, or stockholders, in which shall be set forth in belonging to the same family; that a test case between the petitioner and the
detail the time and place of holding the meeting was regular or respondents has not been begun in the Court of First Instance of Occidental
special, if special its object, those present and absent, and every act Negros involving hundreds of thousands of pesos, and that the appellate
done or ordered done at the meeting. . . . court should not intrude its views to give an advantage to either party. We
rule that the petitioner has not made out a case for relief by mandamus.

876
Petition denied with costs.

Avancea, C.J., Villamor, Villa-Real, Hull and Imperial, JJ., concur.

877
G.R. No. L-33320 May 30, 1983 Previous to the present action, the petitioner instituted
several cases in this Court questioning different transactions
RAMON A. GONZALES, petitioner, entered into by the Bark with other parties. First among them
vs. is Civil Case No. 69345 filed on April 27, 1967, by petitioner
THE PHILIPPINE NATIONAL BANK, respondent. as a taxpayer versus Sec. Antonio Raquiza of Public Works
and Communications, the Commissioner of Public
Ramon A. Gonzales in his own behalf. Highways, the Bank, Continental Ore Phil., Inc., Continental
Ore, Huber Corporation, Allis Chalmers and General Motors
Corporation In the course of the hearing of said case on
Juan Diaz for respondent.
August 3, 1967, the personality of herein petitioner to sue
the bank and question the letters of credit it has extended for
the importation by the Republic of the Philippines of public
works equipment intended for the massive development
VASQUEZ, J.: program of the President was raised. In view thereof, he
expressed and made known his intention to acquire one
Petitioner Ramon A. Gonzales instituted in the erstwhile Court of First share of stock from Congressman Justiniano Montano
Instance of Manila a special civil action for mandamus against the herein which, on the following day, August 30, 1967, was
respondent praying that the latter be ordered to allow him to look into the transferred in his name in the books of the Bank.
books and records of the respondent bank in order to satisfy himself as to the
truth of the published reports that the respondent has guaranteed the Subsequent to his aforementioned acquisition of one share
obligation of Southern Negros Development Corporation in the purchase of a of stock of the Bank, petitioner, in his dual capacity as a
US$ 23 million sugar-mill to be financed by Japanese suppliers and taxpayer and stockholder, filed the following cases involving
financiers; that the respondent is financing the construction of the P 21 the bank or the members of its Board of Directors to wit:
million Cebu-Mactan Bridge to be constructed by V.C. Ponce, Inc., and the
construction of Passi Sugar Mill at Iloilo by the Honiron Philippines, Inc., as l. On October l8,1967, Civil Case No. 71044 versus the
well as to inquire into the validity of Id transactions. The petitioner has Board of Directors of the Bank; the National Investment and
alleged hat his written request for such examination was denied by the Development Corp., Marubeni Iida Co., Ltd., and Agro-Inc.
respondent. The trial court having dismissed the petition for mandamus, the Dev. Co. or Saravia;
instant appeal to review the said dismissal was filed.
2. On May 11, 1968, Civil Case No. 72936 versus Roberto
The facts that gave rise to the subject controversy have been set forth by the Benedicto and other Directors of the Bank, Passi (Iloilo)
trial court in the decision herein sought to be reviewed, as follows: Sugar Central, Inc., Calinog-Lambunao Sugar Mill Integrated
Farming, Inc., Talog sugar Milling Co., Inc., Safary Central,
Briefly stated, the following facts gathered from the Inc., and Batangas Sugar Central Inc.;
stipulation of the parties served as the backdrop of this
proceeding.

878
3. On May 8, 1969, Civil Case No. 76427 versus Alfredo Assailing the conclusions of the lower court, the petitioner has assigned the
Montelibano and the Directors of both the PNB and DBP; single error to the lower court of having ruled that his alleged improper
motive in asking for an examination of the books and records of the
On January 11, 1969, however, petitioner addressed a letter respondent bank disqualifies him to exercise the right of a stockholder to
to the President of the Bank (Annex A, Pet.), requesting such inspection under Section 51 of Act No. 1459, as amended. Said
submission to look into the records of its transactions provision reads in part as follows:
covering the purchase of a sugar central by the Southern
Negros Development Corp. to be financed by Japanese Sec. 51. ... The record of all business transactions of the
suppliers and financiers; its financing of the Cebu-Mactan corporation and the minutes of any meeting shall be open to
Bridge to be constructed by V.C. Ponce, Inc. and the the inspection of any director, member or stockholder of the
construction of the Passi Sugar Mills in Iloilo. On January 23, corporation at reasonable hours.
1969, the Asst. Vice-President and Legal Counsel of the
Bank answered petitioner's letter denying his request for Petitioner maintains that the above-quoted provision does not justify the
being not germane to his interest as a one-share stockholder qualification made by the lower court that the inspection of corporate records
and for the cloud of doubt as to his real intention and may be denied on the ground that it is intended for an improper motive or
purpose in acquiring said share. (Annex B, Pet.) In view of purpose, the law having granted such right to a stockholder in clear and
the Bank's refusal the petitioner instituted this action.' (Rollo, unconditional terms. He further argues that, assuming that a proper motive or
pp. 16-18.) purpose for the desired examination is necessary for its exercise, there is
nothing improper in his purpose for asking for the examination and inspection
The petitioner has adopted the above finding of facts made by the trial court herein involved.
in its brief which he characterized as having been "correctly stated."
(Petitioner-Appellant"s Brief, pp. 57.) Petitioner may no longer insist on his interpretation of Section 51 of Act No.
1459, as amended, regarding the right of a stockholder to inspect and
The court a quo denied the prayer of the petitioner that he be allowed to examine the books and records of a corporation. The former Corporation
examine and inspect the books and records of the respondent bank Law (Act No. 1459, as amended) has been replaced by Batas Pambansa
regarding the transactions mentioned on the grounds that the right of a Blg. 68, otherwise known as the "Corporation Code of the Philippines."
stockholder to inspect the record of the business transactions of a
corporation granted under Section 51 of the former Corporation Law (Act No. The right of inspection granted to a stockholder under Section 51 of Act No.
1459, as amended) is not absolute, but is limited to purposes reasonably 1459 has been retained, but with some modifications. The second and third
related to the interest of the stockholder, must be asked for in good faith for a paragraphs of Section 74 of Batas Pambansa Blg. 68 provide the following:
specific and honest purpose and not gratify curiosity or for speculative or
vicious purposes; that such examination would violate the confidentiality of The records of all business transactions of the corporation
the records of the respondent bank as provided in Section 16 of its charter, and the minutes of any meeting shag be open to inspection
Republic Act No. 1300, as amended; and that the petitioner has not by any director, trustee, stockholder or member of the
exhausted his administrative remedies. corporation at reasonable hours on business days and he

879
may demand, in writing, for a copy of excerpts from said and the refusal to allow such inspection shall subject the erring officer or
records or minutes, at his expense. agent of the corporation to civil and criminal liabilities.

Any officer or agent of the corporation who shall refuse to


allow any director, trustee, stockholder or member of the
corporation to examine and copy excerpts from its records or However, while seemingly enlarging the right of inspection, the new
minutes, in accordance with the provisions of this Code, Code has prescribed limitations to the same. It is now expressly
shall be liable to such director, trustee, stockholder or required as a condition for such examination that
member for damages, and in addition, shall be guilty of an
offense which shall be punishable under Section 144 of this the one requesting it must not have been guilty of using improperly any
Code: Provided, That if such refusal is made pursuant to a information through a prior examination,
resolution or order of the board of directors or trustees, the
liability under this section for such action shall be imposed
and that the person asking for such examination must be "acting in good faith
upon the directors or trustees who voted for such refusal;
and for a legitimate purpose in making his demand."
and Provided, further, That it shall be a defense to any
action under this section that the person demanding to
examine and copy excerpts from the corporation's records
and minutes has improperly used any information secured
through any prior examination of the records or minutes of
such corporation or of any other corporation, or was not
acting in good faith or for a legitimate purpose in making his The unqualified provision on the right of inspection previously
demand. contained in Section 51, Act No. 1459, as amended, no longer holds
true under the provisions of the present law. The argument of the
As may be noted from the above-quoted provisions, among the petitioner that the right granted to him under Section 51 of the former
changes introduced in the new Code with respect to the right of Corporation Law should not be dependent on the propriety of his
inspection granted to a stockholder are the following: motive or purpose in asking for the inspection of the books of the
respondent bank loses whatever validity it might have had before the
amendment of the law. If there is any doubt in the correctness of the
ruling of the trial court that the right of inspection granted under
Section 51 of the old Corporation Law must be dependent on a showing
the records must be kept at the principal office of the corporation;
of proper motive on the part of the stockholder demanding the same, it
is now dissipated by the clear language of the pertinent provision
the inspection must be made on business days; contained in Section 74 of Batas Pambansa Blg. 68.

the stockholder may demand a copy of the excerpts of the records or Although the petitioner has claimed that he has justifiable motives in seeking
minutes; the inspection of the books of the respondent bank, he has not set forth the
reasons and the purposes for which he desires such inspection, except to

880
satisfy himself as to the truth of published reports regarding certain Sec. 30. Penalties for violation of the provisions of this Act.
transactions entered into by the respondent bank and to inquire into their Any director, officer, employee, or agent of the Bank, who
validity. The circumstances under which he acquired one share of stock in violates or permits the violation of any of the provisions of
the respondent bank purposely to exercise the right of inspection do not this Act, or any person aiding or abetting the violations of
argue in favor of his good faith and proper motivation. Admittedly he sought any of the provisions of this Act, shall be punished by a fine
to be a stockholder in order to pry into transactions entered into by the not to exceed ten thousand pesos or by imprisonment of not
respondent bank even before he became a stockholder. His obvious purpose more than five years, or both such fine and imprisonment.
was to arm himself with materials which he can use against the respondent
bank for acts done by the latter when the petitioner was a total stranger to The Philippine National Bank is not an ordinary corporation. Having a charter
the same. He could have been impelled by a laudable sense of civic of its own, it is not governed, as a rule, by the Corporation Code of the
consciousness, but it could not be said that his purpose is germane to his Philippines. Section 4 of the said Code provides:
interest as a stockholder.
SEC. 4. Corporations created by special laws or charters.
We also find merit in the contention of the respondent bank that the Corporations created by special laws or charters shall be
inspection sought to be exercised by the petitioner would be violative governed primarily by the provisions of the special law or
of the provisions of its charter. (Republic Act No. 1300, as amended.) charter creating them or applicable to them. supplemented
Sections 15, 16 and 30 of the said charter provide respectively as by the provisions of this Code, insofar as they are applicable.
follows:
The provision of Section 74 of Batas Pambansa Blg. 68 of the new
Sec. 15. Inspection by Department of Supervision and Corporation Code with respect to the right of a stockholder to demand an
Examination of the Central Bank. The National Bank shall inspection or examination of the books of the corporation may not be
be subject to inspection by the Department of Supervision reconciled with the abovequoted provisions of the charter of the respondent
and Examination of the Central Bank' bank. It is not correct to claim, therefore, that the right of inspection under
Section 74 of the new Corporation Code may apply in a supplementary
Sec. 16. Confidential information. The Superintendent of capacity to the charter of the respondent bank.
Banks and the Auditor General, or other officers designated
by law to inspect or investigate the condition of the National WHEREFORE, the petition is hereby DISMISSED, without costs.
Bank, shall not reveal to any person other than the President
of the Philippines, the Secretary of Finance, and the Board Richardson v. Arizona Fuels Corp.
of Directors the details of the inspection or investigation, nor
shall they give any information relative to the funds in its
614 P.2d 636 (1980)
custody, its current accounts or deposits belonging to private
individuals, corporations, or any other entity, except by order
of a Court of competent jurisdiction,' Donald J. RICHARDSON, Grove L. Cook and Wayne Weaver, Individually
and for and on behalf of all similarly situated shareholders of Major Oil
Corporation, Plaintiffs and Respondents, v. ARIZONA FUELS
CORPORATION, a Utah Corporation, Eugene Dalton, an Individual, Deanna

881
J. Dalton, an Individual, and Major Oil Corporation, a Utah Corporation, The complaint was subsequently amended, inter alia, to name Major as an
Defendants and Appellants. involuntary defendant, pursuant to Rule 19(a). The amended complaint
describes this action as one brought as a class action pursuant to Rule 23
No. 15691. and as a stockholders' derivative action pursuant to Rule 23.1. Plaintiffs
moved for an order certifying this suit as a class action and for appointment
of a receiver for Major pursuant to Rule 66. Both motions were granted by
the district court.
Supreme Court of Utah.
Defendants attack the order on the grounds (1) that the appointment of a
receiver was not justified, (2) that certification of all the claims in the suit as a
class action was improper, and (3) alternatively, that the district court erred in
not designating under which subsection of Rule 23(b) this action is to
May 1, 1980. proceed. Plaintiffs' motions were granted solely on the basis of the verified
amended complaint.
*637 LeRoy S. Axland and David R. Olsen, of Suitter, Axland & Armstrong,
Salt Lake City, for defendants and appellants. *638 We first consider the propriety of the appointment of a receiver. The
ground for appointment, as stated in the amended complaint, is that the
Paul N. Cotro-Manes, Parker M. Nielson, Salt Lake City, for plaintiffs and defendants' conduct has caused Major to become insolvent or placed Major
respondents. in imminent danger of becoming insolvent. Rule 66(a)(5) permits
appointment of a receiver "[i]n cases where a corporation . . . is insolvent or
STEWART, Justice: in imminent danger of insolvency . . .."

We here review on interlocutory appeal an order of the district court of Salt The authorities are generally in agreement that the appointment of a receiver
Lake County appointing a receiver for defendant Major Oil Corporation for a corporation is permissible at the request of stockholders of the
(hereafter "Major") and certifying this case as a class action. All references to corporation suing either individually or on behalf of the corporation. 3 Clark
rules refer to the Utah Rules of Civil Procedure. on Receivers 738(d) (3rd ed. 1959); 16 Fletcher, Cyclopedia of the Law of
Private Corporations 7688 (rev. perm. ed. 1979) (hereinafter cited as
Plaintiffs Donald J. Richardson, Grove L. Cook, and Wayne Weaver are "Fletcher"); 65 Am.Jur.2d Receivers 11 (1972); 19 C.J.S. Corporations
stockholders of Major who brought this action individually and on behalf of all 833c. (1940). Defendants attack the appointment on the ground that it is not
other stockholders of Major. The original complaint named as defendants justified by allegations on information and belief, even though those
Arizona Fuels, Inc.; Eugene Dalton; and Deanna J. Dalton. Arizona Fuels is allegations were stated in a verified complaint. The amended complaint
alleged to be the legal or beneficial owner of 47% of the issued and contains numerous allegations based on information and belief of fraudulent
outstanding shares of stock of Major. Eugene Dalton is alleged to be the and otherwise wrongful conduct on the part of defendants. The allegations
controlling stockholder, officer and director of Arizona Fuels and the specify suspect transactions and state details of alleged fraud. The
controlling officer and director of Major. Deanna Dalton is alleged to be an allegations requesting appointment of a receiver, however, are made without
officer and director of both Major and Arizona Fuels. any qualification as to information and belief, and these allegations are not
controverted by defendants, either by pleading or affidavit.

882
In determining whether a receiver should be appointed, the district court enforcing their claims. A recovery in a class action is a recovery which
should consider the pleadings as a whole. Receivers have historically been belongs directly to the shareholders. However, in a derivative action, the
appointed in cases where misappropriation of corporate assets by corporate plaintiff shareholder recovers nothing and the judgment runs in favor of
insiders is asserted. Stevens v. South Ogden Land, Bldg. & Improvement the corporation.
Co., 14 Utah 232, 47 P. 81 (1896); Bookout v. Atlas Financial Corp., 395 F.
Supp. 1338 (D.Ga. 1974). If defendants seriously contend that insolvency is The difference in the two procedures and their relationship to underlying
not imminent or that a receivership is not appropriate under the substantive law has been stated as follows:
circumstances, they had abundant opportunity to provide factual support for
their contention. The record discloses that they did not do so. *639 Suits which are said to be derivative, and therefore
come within the rule, are those which seek to enforce any right
The appointment of a receiver is among those discretionary powers subject which belongs to the corporation and is not being enforced, such as
to review for abuse, but we cannot find abuse where the ground for the liability of corporate officers or majority shareholders for
appointment is stated in the language of the rules and remains mismanagement, to recover corporate assets and related claims, to
uncontroverted through a full hearing with extensive preliminary written enforce rights of the corporation by virtue of its contract with a third
memoranda. person, and to enjoin those in charge of the corporation from causing
it to commit an ultra vires act. [Footnotes omitted.] [3B Moore's
The next issue is whether the district court erred in certifying this matter as a Federal Practice 23.1.16[1] (2nd ed. 1980).]
class action. It is alleged in the amended complaint that "some" of the causes
of action found therein belong to Major, and that as to those causes plaintiffs On the other hand,
bring the suit derivatively on behalf of the corporation pursuant to Rule 23.1.
[i]f the injury is one to the plaintiff as a stockholder and to
A class action and a derivative action rest upon fundamentally different him individually, and not to the corporation, as where the action is
principles of substantive law; to ignore those differences is not a minor based on a contract to which he is a party, or on a right belonging
procedural solecism. A derivative action must necessarily be based on a severally to him, or on a fraud affecting him directly, it is an
claim for relief which is owned by the stockholders' corporation. Indeed, a individual action. [Footnotes omitted.] [13 Fletcher 5911 (1970).]
prerequisite for filing a derivative action is the failure of the corporation to
initiate the action in its own name. The stockholder, as a nominal party, has It is the duty of the district court to apply carefully the criteria set forth in Rule
no right, title or interest whatsoever in the claim itself whether the action is 23(a) and (b) to the facts of the case to determine whether an action may be
brought by the corporation or by the stockholder on behalf of the corporation. maintained as a class action. 3B Moore's Federal Practice 23.5. If the
criteria of Rule 23 are complied with, it is within the sound discretion of the
A class action, on the other hand, is predicated on ownership of the claim district court to determine whether a suit, or some of the issues in a lawsuit,
for relief sued upon in the representative of the class and all other class should proceed as a class action. Id.
members in their capacity as individuals. Shareholders of the corporation
may, of course, have claims for relief directly against their corporation In this case, neither the memorandum decision nor the order of the district
because the corporation itself has violated rights possessed by the court does any more than recite that the suit may be maintained as a class
shareholders, and a class action would be an appropriate means for action. Furthermore, the amended complaint in alleging that the action

883
should be maintained as a class action, does no more than mimic the stockholders collectively does not appear to be one of substance in this case.
language of Rule 23. As was pointed out in Jones v. Diamond, 519 F.2d There is no important *640 issue as to whether the cause of action states a
1090, 1098 (5th Cir.1975), "Without more, mere mimicry is insufficient to corporate claim. Although plaintiff frames this claim, in the alternative, as one
undergird a decision either way on the propriety of class certification." See belonging to the shareholders, the claim for relief belongs to the corporation.
also Rossin v. Southern Union Gas Co., 472 F.2d 707, 712 (10th Cir.1973);
Weathers v. Peters Realty Corp., 499 F.2d 1197, 1200 (6th Cir.1974). We The ninth cause of action then goes on to allege that the defendants
therefore direct our attention to the contents of the verified amended "mismanaged the corporate and prudential affairs of Major Oil . . .." The rule
complaint, which was the basis upon which the district court made its in Utah is that mismanagement of the corporation gives rise to a cause of
determination. action in the corporation, even if the mismanagement results in damage to
stockholders by depreciating the value of the corporation's stock. Morris v.
The amended complaint states twelve causes of action, the first eight of Ogden State Bank, 84 Utah 127, 140-41, 28 P.2d 138, 143 (1934).
which allege some fraudulent appropriation of or scheme to appropriate Therefore, any compensatory damages which may be recovered on account
Major's assets by defendants. These causes of action seek to require the of any breach by defendants of their fiduciary duty as directors and officers or
defendants to disgorge and return to Major the assets wrongfully obtained. arising as a result of mismanagement of the corporation by defendants
Of the remaining four causes, three seek compensatory or punitive damages belong to the corporation and not to the stockholders individually.
for injury attributable to alleged breaches of fiduciary duty implicit in the
fraudulent acts enumerated in the first eight causes. The final cause of action The ninth cause of action also prays for punitive damages based on the
seeks appointment of a receiver. nature of defendants' conduct. Because there is no claim for relief for punitive
damages as such, a claim for punitive damages must be related to an
There is no doubt that the first eight causes of action allege injury to the underlying cause of action on which the punitive damages may be based.
corporation only. The injury alleged can be asserted by plaintiffs only Graham v. Street, 2 Utah 2d 144, 270 P.2d 456 (1954); 22 Am.Jur.2d
derivatively as stockholders on behalf of the corporation. This leaves the Damages 241 (1965). Since we have concluded that the other aspects of
ninth, tenth and eleventh causes of action to be analyzed to determine if they the ninth cause of action state a corporate claim, the punitive damages claim
state claims which may be pursued by the stockholders as a class to redress alleged must likewise belong to the corporation. In short, the ninth cause of
injuries to the stockholders as individuals. action states a claim belonging to the corporation and precludes that claim
from being alleged as a class action.
The ninth cause of action alleges initially that the defendants "breached their
fiduciary duties to Major Oil and to its stockholders . . .." As a general rule, The tenth cause of action alleges that the defendants "defrauded the
directors and other officers of a corporation stand in a fiduciary relation to the stockholders of Major Oil Company . . .." The fraud is premised on
corporation. Branch v. Western Factors, Inc., 28 Utah 2d 361, 502 P.2d defendants' fiduciary duty owed to the stockholders of the corporation. That
570 (1972); 3 Fletcher 838. While the statement is made that directors and duty is alleged to have been breached in six particulars. Each of the six
officers stand in a like relation to the stockholders of the corporation, 3 alleged defalcations states a claim belonging to the corporation and not to
Fletcher 838, in Utah it is clear that that relation is to the stockholders the stockholders or any of them individually. The tenth cause of action is
collectively. Jones Min. Co. v. Cardiff Min. & Mill. Co., 56 Utah 449, 456, 191 rounded out by allegations of the defendants' knowledge of their wrongful
P. 426, 428 (1920); accord, Hansen v. Granite Holding Co., 117 Utah conduct, the reasonable reliance of the plaintiffs on defendants' performance
530, 218 P.2d 274 (1950); 3 Fletcher 838 at 144. The distinction between a of defendants' fiduciary duties, and the resulting damage to the stockholders.
fiduciary duty owed to the corporation as a whole as opposed to the However, in no regard can the tenth cause of action be interpreted as stating

884
a claim belonging to the stockholders individually, and therefore that claim for
relief will not support a class action.

The eleventh cause of action alleges the possibility of other conversions of


Major's assets and alleges that the defendants should be required to account
to the stockholders for all of the assets of Major and disgorge themselves of
any assets so converted. This claim also clearly belongs to the corporation.

Although class actions have historical antecedents in rules of equity


that go back several centuries in English jurisprudence,[1] it is not to
be gainsaid that the modern class action rule is one of the most
farreaching and important changes in legal procedure in many a
decade. Its impact on the enforcement of consumer rights, antitrust
claims, securities claims and civil rights actions, to name but a few
areas, has been monumental. However, the class action device, if used
inappropriately and in lieu of a derivative action, is likely to result in
grave injustices, not the least of which is the diversion of assets
recovered in a lawsuit from creditors of a corporation to stockholders,
thereby reversing long established substantive rules of law as to the
relative priorities of the claims of creditors and stockholders to the G.R. No. 123553 July 13, 1998
assets of an insolvent corporation.
(CA-G.R. No. 33291) July 13, 1998
We therefore reverse the district court's certification of this suit as a class
action and remand for further proceedings not inconsistent with this opinion. NORA A. BITONG, petitioner,

Costs to Defendants. vs.

CROCKETT, C.J., and WILKINS, MAUGHAN and HALL, JJ., concur. COURT OF APPEALS (FIFTH DIVISION), EUGENIA D. APOSTOL, JOSE
A. APOSTOL, MR. & MS. PUBLISHING CO., LETTY J. MAGSANOC, AND
ADORACION G. NUYDA, respondents.

(CA-G.R. No. 33873) July 13, 1998

NORA A. BITONG, petitioner,

885
vs. Petitioner further alleged that respondents Eugenia and Jose Apostol were
stockholders, directors and officers in both Mr. & Ms. and PDI. In fact on 2
COURT OF APPEALS (FIFTH DIVISION) and EDGARDO B. May 1986 respondents Eugenia D. Apostol, Leticia J. Magsanoc and
ESPIRITU, respondents. Adoracion G. Nuyda subscribed to PDI shares of stock at P50,000.00 each
or a total of P150,000.00. The stock subscriptions were paid for by Mr. & Ms.
and initially treated, as receivables from officers and employees. But, no
payments were ever received from respondents, Magsanoc and Nuyda.
BELLOSILLO, J.:
The petition principally sought to (a) enjoin respondents Eugenia D. Apostol
and Jose A. Apostol from further acting as president-director and director,
These twin cases originated from a derivative suit 1 filed by petitioner Nora A.
respectively, of Mr. & Ms. and disbursing any money or funds except for the
Bitong before the Securities and Exchange Commission (SEC hereafter)
payment of salaries and similar expenses in the ordinary course of business,
allegedly for the benefit of private respondent Mr. & Ms. Publishing Co.,
and from disposing of their Mr. & Ms. shares; (b) enjoin respondents Apostol
Inc. (Mr. & Ms. hereafter), among others, to hold respondent spouses
spouses, Magsanoc and Nuyda from disposing of the PDI shares of stock
Eugenia D. Apostol and Jose A. Apostol 2 liable for fraud, misrepresentation,
registered in their names; (c) compel respondents Eugenia and Jose Apostol
disloyalty, evident bad faith, conflict of interest and mismanagement in
to account for and reconvey all profits and benefits accruing to them as a
directing the affairs of Mr. & Ms. to the damage and prejudice of Mr. & Ms.
result of their improper and fraudulent acts; (d) compel respondents
and its stockholders, including petitioner.
Magsanoc and Nuyda to account for and reconvey to Mr. & Ms. all shares of
stock paid from cash advances from it and all accessions or fruits thereof; (e)
Alleging before the SEC that she had been the Treasurer and a Member of hold respondents Eugenia and Jose Apostol liable for damages suffered by
the Board of Directors of Mr. & Ms. from the time it was incorporated on 29 Mr. & Ms. and the other stockholders, including petitioner, by reason of their
October 1976 to 11 April 1989, and was the registered owner of 1,000 shares improper and fraudulent acts; (f) appoint a management committee for Mr. &
of stock out of the 4,088 total outstanding shares, petitioner complained of Ms. during the pendency of the suit to prevent further dissipation and loss of
irregularities committed from 1983 to 1987 by Eugenia D. Apostol, President its assets and funds as well as paralyzation of business operations; and, (g)
and Chairperson of the Board of Directors. Petitioner claimed that except for direct the management committee for Mr. & Ms. to file the necessary action
the sale of the name Philippine Inquirer to Philippine Daily Inquirer (PDI to enforce its rights against PDI and other third parties.
hereafter) all other transactions and agreements entered into by Mr. & Ms.
with PDI were not supported by any bond and/or stockholders' resolution.
Private respondents Apostol spouses, Magsanoc, Nuyda, and Mr. & Ms., on
And, upon instructions of Eugenia D. Apostol, Mr. & Ms. made several cash
the other hand, refuted the allegations of petitioner by starting with a
advances to PDI on various occasions amounting to P3.276 million. On some
narration of the beginnings of Mr. & Ms. They recounted that on 9 March
of these borrowings PDI paid no interest whatsoever. Despite the fact that
1976 Ex Libris Publishing Co., Inc. (Ex Libris hereafter) was incorporated for
the advances made by Mr. & Ms. to PDI were booked as advances to an
the purpose of publishing a weekly magazine. Its original principal
affiliate, there existed no board or stockholders' resolution, contract nor any
stockholders were spouses Senator Juan Ponce Enrile (then Minister of
other document which could legally authorize the creation of and support to
National Defense) and Cristina Ponce Enrile through Jaka Investments
an affiliate.
Corporation (JAKA hereafter), and respondents Eugenia and Jose Apostol.
When Ex Libris suffered financial difficulties, JAKA and the Apostols, together

886
with new investors Luis Villafuerte and Ramon Siy, restructured Ex Libris by Private respondents averred that all the PDI shares owned by respondents
organizing a new corporation known as Mr. & Ms. Eugenia and Jose Apostol were acquired through their own private funds and
that the loan of P750,000.00 by PDI from Mr. & Ms. had been fully paid with
The original stockholders of Mr. & Ms., i.e., JAKA, Luis Villafuerte, Ramon 20% interest per annum. And, it was PDI, not Mr. & Ms., which loaned off
Siy, the Apostols and Ex Libriscontinued to be virtually the same up to 1989. P250,000.00 each to respondents Magsanoc and Nuyda. Private
Thereafter it was agreed among them that, they being close friends, Mr. & respondents further argued that petitioner was not the true party to this case,
Ms. would be operated as a partnership or a close corporation; respondent the real party being JAKA which continued to be the true stockholder of Mr. &
Eugenia D. Apostol would manage the affairs of Mr. & Ms.; and, no shares of Ms.; hence, petitioner did not have the personality to initiate and prosecute
stock would be sold to third parties without first offering the shares to the the derivative suit which, consequently, must be dismissed.
other stockholders so that transfers would be limited to and only among the
original stockholders. On 6 December 1990, the SEC Hearing Panel 3 issued a writ of preliminary
injunction enjoining private respondents from disbursing any money except
Private respondents also asserted that respondent Eugenia D. Apostol had for the payment of salaries and other similar expenses in the regular course
been informing her business partners of her actions as manager, and of business. The Hearing Panel also enjoined respondent Apostol spouses,
obtaining their advice and consent. Consequently the other stockholders Nuyda and Magsanoc from disposing of their PDI shares, and further ruled
consented, either expressly or impliedly, to her management. They offered no
objections. As a result, the business prospered. Thus, as shown in a
statement prepared by the accounting firm Punongbayan and Araullo, there . . . respondents' contention that petitioner is not entitled to
were increases from 1976 to 1988 in the total assets of Mr. & Ms. from the provisional reliefs prayed for because she is not the real
P457,569.00 to P10,143,046.00; in the total stockholders' equity from party in interest . . . is bereft of any merit. No less than
P203,378.00 to P2,324,954.00; and, in the net sales, from P301,489.00 to respondents' Amended Answer, specifically paragraph V, No.
P16,325,610.00. Likewise, cash dividends were distributed and received by 8 on Affirmative Allegations/Defenses states that "The
the stockholders. petitioner being herself a minor stockholder and holder-in-
trust of JAKA shares represented and continues to represent
Private respondents further contended that petitioner, being merely a holder- JAKA in the Board." This statement refers to petitioner sitting
in-trust of JAKA shares, only represented and continued to represent JAKA in in the board of directors of Mr. & Ms. in two capacities, one
the board. In the beginning, petitioner cooperated with and assisted the as a minor stockholder and the other as the holder in trust of
management until mid-1986 when relations between her and her principals the shares of JAKA in Mr. & Ms. Such reference alluded to
on one hand, and respondent Eugenia D. Apostol on the other, became by the respondents indicates an admission on respondents'
strained due to political differences. Hence from mid-1986 to mid-1988 part of the petitioner's legal personality to file a derivative suit
petitioner refused to speak with respondent Eugenia D. Apostol, and in 1988 for the benefit of the respondent Mr. & Ms. Publishing Co.,
the former became openly critical of the management of the latter. Inc.
Nevertheless, respondent Eugenia D. Apostol always made available to
petitioner and her representatives all the books of the corporation. The Hearing Panel however denied petitioner's prayer for the
constitution of a management committee.

887
On 25 March 1991 private respondents filed a Motion to Amend Pleadings to any of Mr. & Ms. assets. The Hearing Panel ruled that there was no serious
Conform to Evidence alleging that the issue of whether petitioner is the real mismanagement of Mr. & Ms. which would warrant drastic corrective
party-in-interest had been tried by express or implied consent of the parties measures. It gave credence to the assertion of respondent Eugenia D.
through the admission of documentary exhibits presented by private Apostol that Mr. & Ms. was operated like a close corporation where important
respondents proving that the real party-in-interest was JAKA, not petitioner matters were discussed and approved through informal consultations at
Bitong. As such, No. 8, par. V (Affirmative Allegations/Defenses), Answer to breakfast conferences. The Hearing Panel also concluded that while the
the Amended Petition, was stipulated due to inadvertence and excusable evidence presented tended to show that the real party-in-interest indeed was
mistake and should be amended. On 10 October 1991 the Hearing Panel JAKA and/or Senator Enrile, it viewed the real issue to be the alleged
denied the motion for amendment. mismanagement, fraud and conflict of interest on the part of respondent
Eugenia D. Apostol, and allowed petitioner to prosecute the derivative suit if
Petitioner testified at the trial that she became the registered and beneficial only to resolve the real issues. Hence, for this purpose, the Hearing Panel
owner of 997 shares of stock of Mr. & Ms. out of the 4,088 total outstanding considered petitioner to be the real party-in-interest.
shares after she acquired them from JAKA through a deed of sale executed
on 25 July 1983 and recorded in the Stock and Transfer Book of Mr. & Ms. On 19 August 1993 respondent Apostol spouses sold the PDI shares
under Certificate of Shares of Stock No. 008. She pointed out that Senator registered in the name of their holding company, JAED Management
Enrile decided that JAKA should completely divest itself of its holdings in Mr. Corporation, to Edgardo B. Espiritu. On 25 August 1993 petitioner Bitong
& Ms. and this resulted in the sale to her of JAKA's interest and holdings in appealed to the SEC En Banc.
that publishing firm.
On 24 January 1994 the SEC En Banc 4 reversed the decision of the Hearing
Private respondents refuted the statement of petitioner that she was a Panel and, among others, ordered private respondents to account for, return
stockholder of Mr. & Ms. since 25 July 1983 as respondent Eugenia D. and deliver to Mr. & Ms. any and all funds and assets that they disbursed
Apostol signed Certificate of Stock No. 008 only on 17 March 1989, and not from the coffers of the corporation including shares of stock, profits,
on 25 July 1983. Respondent Eugenia D. Apostol explained that she stopped dividends and/or fruits that they might have received as a result of their
using her long signature (Eugenia D. Apostol) in 1987 and changed it to E.D. investment in PDI, including those arising from the P150,000.00 advanced to
Apostol, the signature which appeared on the face of Certificate of Stock No. respondents Eugenia D. Apostol, Leticia J. Magsanoc and Adoracion G.
008 bearing the date 25 July 1983. And, since the Stock and Transfer Book Nuyda; account for and return any profits and fruits of all amounts irregularly
which petitioner presented in evidence was not registered with the SEC, the or unlawfully advanced to PDI and other third persons; and, cease and desist
entries therein including Certificate of Stock No. 008 were fraudulent. from managing the affairs of Mr. & Ms. for reasons of fraud, mismanagement,
Respondent Eugenia D. Apostol claimed that she had not seen the Stock and disloyalty and conflict of interest.
Transfer Book at anytime until 21 March 1989 when it was delivered by
petitioner herself to the office of Mr. & Ms., and that petitioner repeatedly The SEC En Banc also declared the 19 August 1993 sale of the PDI shares
referred to Senator Enrile as "my principal" during the Mr. & Ms. board of JAED Management Corporation to Edgardo B. Espiritu to be tainted with
meeting of 22 September 1988, seven (7) times no less. fraud, hence, null and void, and considered Mr. & Ms. as the true and lawful
owner of all the PDI shares acquired by respondents Eugenia D. Apostol,
On 3 August 1993, after trial on the merits, the SEC Hearing Panel dismissed Magsanoc and Nuyda. It also declared all subsequent transferees of such
the derivative suit filed by petitioner and dissolved the writ of preliminary shares as trustees for the benefit of Mr. & Ms. and ordered them to forthwith
injunction barring private respondents from disposing of their PDI shares and deliver said shares to Mr. & Ms.

888
Consequently, respondent Apostol spouses, Magsanoc, Nuyda, and Mr. & of fact. In addition, a party against whom a single clause or paragraph of a
Ms. filed a petition for review before respondent Court of Appeals, docketed pleading is offered may have the right to introduce other paragraphs which
as CA-GR No. SP 33291, while respondent Edgardo B. Espiritu filed a tend to destroy the admission in the paragraph offered by the adversary. 6
petition for certiorari and prohibition also before respondent Court of Appeals,
docketed as CA-GR No. SP 33873. On 8 December 1994 the two (2) The Amended Petition before the SEC alleges
petitions were consolidated.
I. THE PARTIES
On 31 August 1995 respondent appellate court rendered a decision reversing
the SEC En Banc and held that from the evidence on record petitioner was 1. Petitioner is a stockholder and director of Mr. & Ms. . . . .
not the owner of any share of stock in Mr. & Ms. and therefore not the real
party-in-interest to prosecute the complaint she had instituted against private
II. THE FACTS
respondents. Accordingly, petitioner alone and by herself as an agent could
not file a derivative suit in behalf of her principal. For not being the real party-
in-interest, petitioner's complaint did not state a cause of action, a defense 1. Petitioner is the registered owner of 1,000 shares of stock
which was never waived; hence, her petition should have been dismissed. of Mr. & Ms. out of the latter's 4,088 total outstanding shares.
Respondent appellate court ruled that the assailed orders of the SEC were Petitioner, at all times material to this petition, is a member of
issued in excess of jurisdiction, or want of it, and thus were null and the Board of Directors of Mr. & Ms. and from the inception of
void. 5 On 18 January 1996, petitioner's motion for reconsideration was Mr. & Ms. until 11 April 1989 was its treasurer . . .
denied for lack of merit.
On the other hand, the Amended Answer to the Amended Petition states
Before this Court, petitioner submits that in paragraph 1 under the caption "I.
The Parties" of her Amended Petition before the SEC, she stated that she I. PARTIES
was a stockholder and director of Mr. & Ms. In par. 1 under the caption "II.
The Facts" she declared that she "is the registered owner of 1,000 shares of 1. Respondents admit the allegations contained in Caption I,
stock of Mr. & Ms. out of the latter's 4,088 total outstanding shares" and that pars. 1 to 4 of the Petition referring to the personality,
she was a member of the Board of Directors of Mr. & Ms. and treasurer from addresses and capacity of the parties to the petition
its inception until 11 April 1989. Petitioner contends that private respondents except . . . but qualify said admission insofar as they are
did not deny the above allegations in their answer and therefore they are limited, qualified and/or expanded by allegations in the
conclusively bound by this judicial admission. Consequently, private Affirmative Allegations/Defenses . . .
respondents' admission that petitioner has 1,000 shares of stock registered
in her name in the books of Mr. & Ms. forecloses any question on her status II. THE FACTS
and right to bring a derivative suit on behalf of Mr. & Ms.
1. Respondents admit paragraph 1 of the Petition, but qualify
Not necessarily. A party whose pleading is admitted as an admission against said admission as to the beneficial ownership of the shares
interest is entitled to overcome by evidence the apparent inconsistency, and of stock registered in the name of the petitioner, the truth
it is competent for the party against whom the pleading is offered to show
that the statements were inadvertently made or were made under a mistake

889
being as stated in the Affirmative Allegations/Defenses of this 4,800 96%
Answer . . .
4. The above-named original stockholders of respondent Mr.
V. AFFIRMATIVE ALLEGATIONS/DEFENSES & Ms. continue to be virtually the same stockholders up to
this date . . . .
Respondents respectfully allege by way of Affirmative
Allegations/Defenses, that . . . . 8. The petitioner being herself a minor stockholder and
holder-in-trust of JAKA shares, represented and continues to
3. Fortunately, respondent Apostol was able to convince Mr. represent JAKA in the Board . . . .
Luis Villafuerte to take interest in the business and he,
together with the original investors, restructured the Ex Libris 21. Petitioner Nora A. Bitong is not the true party to this
Publishing Company by organizing a new corporation known case, the true party being JAKA Investments Corporation
as Mr. & Ms. Publishing Co., Inc. . . . Mr. Luis Villafuerte which continues to be the true stockholder of respondent Mr.
contributed his own P100,000.00. JAKA and respondent & Ms. Publishing Co., Inc., consequently, she does not have
Jose Z. Apostol, original investors of Ex Libris contributed the personality to initiate and prosecute this derivative suit,
P100,000.00 each; Ex Libris Publishing Company was paid and should therefore be dismissed . . . .
800 shares for the name of Mr. & Ms. magazine and
goodwill. Thus, the original stockholders of respondent Mr. & The answer of private respondents shows that there was no judicial
Ms. were: admission that petitioner was a stockholder of Mr. & Ms. to entitle her to file a
derivative suit on behalf of the corporation. Where the statements of the
Cert./No./Date Name of Stockholder No. of Shares private respondents were qualified with phrases such as, "insofar as they are
% limited, qualified and/or expanded by," "the truth being as stated in
the Affirmative Allegations/Defenses of this Answer" they cannot be
001-9-15-76 JAKA Investments Corp. 1,000 21% considered definite and certain enough, cannot be construed as judicial
admissions. 7
002-9-15-76 Luis Villafuerte 1,000 21%
More so, the affirmative defenses of private respondents directly refute the
003-9-15-76 Ramon L. Siy 1,000 21% representation of petitioner that she is a true and genuine stockholder of Mr.
& Ms. by stating unequivocally that petitioner is not the true party to the case
but JAKA which continues to be the true stockholder of Mr. & Ms. In fact, one
004-9-15-76 Jose Z. Apostol 1,000 21%
of the reliefs which private respondents prayed for was the dismissal of the
petition on the ground that petitioner did not have the legal interest to initiate
005-9-15-76 Ex Libris Publishing Co. 800 16% and prosecute the same.

When taken in its totality, the Amended Answer to the Amended Petition, or
even the Answer to the Amended Petition alone, clearly raises an issue as to

890
the legal personality of petitioner to file the complaint. Every alleged For, an interlocutory order refers to something between the commencement
admission is taken as an entirety of the fact which makes for the one side and end of the suit which decides some point or matter but it is not the final
with the qualifications which limit, modify or destroy its effect on the other decision of the whole controversy. 10 Thus, even though the 6 December
side. The reason for this is, where part of a statement of a party is used 1990 Order was adverse to private respondents, they had the legal right and
against him as an admission, the court should weigh any other portion option not to elevate the same to the SEC En Banc but rather to await the
connected with the statement, which tends to neutralize or explain the portion decision which resolves all the issues raised by the parties and to appeal
which is against interest. therefrom by assigning all errors that might have been committed by the
Hearing Panel.
In other words, while the admission is admissible in evidence, its probative
value is to be determined from the whole statement and others intimately On the other hand, the 3 August 1993 Decision of the Hearing Panel
related or connected therewith as an integrated unit. Although acts or facts dismissing the derivative suit for failure to prove the charges of
admitted do not require proof and cannot be contradicted, however, mismanagement, fraud, disloyalty and conflict of interest and dissolving the
evidence aliunde can be presented to show that the admission was made writ of preliminary injunction, was favorable to private respondents. Hence,
through palpable mistake. 8 The rule is always in favor of liberality in they were not expected to appeal therefrom.
construction of pleadings so that the real matter in dispute may be submitted
to the judgment of the court. 9 In fact, in the 3 August 1993 Decision, the Hearing Panel categorically stated
that the evidence presented showed that the real party-in-interest was not
Petitioner also argues that since private respondents failed to appeal the 6 petitioner Bitong but JAKA and/or Senator Enrile. Petitioner was merely
December 1990 Order and the 3 August 1993 Decision of the SEC Hearing allowed to prosecute her complaint so as not to sidetrack "the real issue to
Panel declaring that she was the real party-in-interest and had legal be resolved (which) was the allegation of mismanagement, fraud and conflict
personality to sue, they are now estopped from questioning her personality. of interest allegedly committed by respondent Eugenia D. Apostol." It was
only for this reason that petitioner was considered to be capacitated and
Not quite. The 6 December 1990 Order is clearly an interlocutory order which competent to file the petition.
cannot be considered as having finally resolved on the merits the issue of
legal capacity of petitioner. The SEC Hearing Panel discussed the issue of Accordingly, with the dismissal of the complaint of petitioner against private
legal capacity solely for the purpose of ruling on the application for writ of respondents, there was no compelling reason for the latter to appeal to the
preliminary injunction as an incident to the main issues raised in the SEC En Banc. It was in fact petitioner's turn as the aggrieved party to
complaint. Being a mere interlocutory order, it is not appealable. exercise her right to appeal from the decision. It is worthy to note that even
during the appeal of petitioner before the SEC En Banc private respondents
maintained their vigorous objection to the appeal and reiterated petitioner's
lack of legal capacity to sue before the SEC.

Petitioner then contends that she was a holder of the proper certificates of
shares of stock and that the transfer was recorded in the Stock and Transfer
Book of Mr. & Ms. She invokes Sec. 63 of The Corporation Code which
provides that no transfer shall be valid except as between the parties until the

891
transfer is recorded in the books of the corporation, and upon its recording company. 12 Third, the par value, as to par value shares, or the full
the corporation is bound by it and is estopped to deny the fact of transfer of subscription as to no par value shares, must first be fully paid. Fourth, the
said shares. Petitioner alleges that even in the absence of a stock certificate, original certificate must be surrendered where the person requesting the
a stockholder solely on the strength of the recording in the stock and transfer issuance of a certificate is a transferee from a stockholder.
book can exercise all the rights as stockholder, including the right to file a
derivative suit in the name of the corporation. And, she need not present a The certificate of stock itself once issued is a continuing affirmation or
separate deed of sale or transfer in her favor to prove ownership of stock. representation that the stock described therein is valid and genuine and is at
least prima facie evidence that it was legally issued in the absence of
Sec. 63 of The Corporation Code expressly provides evidence to the contrary. However, this presumption may be
rebutted. 13 Similarly, books and records of a corporation which include even
Sec. 63. Certificate of stock and transfer of shares. The the stock and transfer book are generally admissible in evidence in favor of
capital stock of stock corporations shall be divided into or against the corporation and its members to prove the corporate acts, its
shares for which certificates signed by the president or vice financial status and other matters including one's status as a stockholder.
president, countersigned by the secretary or assistant They are ordinarily the best evidence of corporate acts and proceedings.
secretary, and sealed with the seal of the corporation shall
be issued in accordance with the by-laws. Shares of stock so However, the books and records of a corporation are not conclusive even
issued are personal property and may be transferred by against the corporation but are prima facie evidence only. Parol evidence
delivery of the certificate or certificates indorsed by the may be admitted to supply omissions in the records, explain ambiguities, or
owner or his attorney-in-fact or other person legally show what transpired where no records were kept, or in some cases where
authorized to make the transfer. No transfer however shall such records were contradicted. 14The effect of entries in the books of the
be valid except as between the parties until the transfer is corporation which purport to be regular records of the proceedings of its
recorded in the books of the corporation showing the names board of directors or stockholders can be destroyed by testimony of a more
of the parties to the transaction, the date of the transfer, the conclusive character than mere suspicion that there was an irregularity in the
number of the certificate or certificates and the number of manner in which the books were kept. 15
shares transferred . . . .
The foregoing considerations are founded on the basic principle that stock
This provision above quoted envisions a formal certificate of stock which can issued without authority and in violation of law is void and confers no rights
be issued only upon compliance with certain requisites. First, the certificates on the person to whom it is issued and subjects him to no liabilities. 16 Where
must be signed by the president or vice-president, countersigned by the there is an inherent lack of power in the corporation to issue the stock,
secretary or assistant secretary, and sealed with the seal of the corporation. neither the corporation nor the person to whom the stock is issued is
A mere typewritten statement advising a stockholder of the extent of his estopped to question its validity since an estopped cannot operate to create
ownership in a corporation without qualification and/or authentication cannot stock which under the law cannot have existence. 17
be considered as a formal certificate of stock. 11 Second, delivery of the
certificate is an essential element of its issuance. Hence, there is no As found by the Hearing Panel and affirmed by respondent Court of Appeals,
issuance of a stock certificate where it is never detached from the stock there is overwhelming evidence that despite what appears on the certificate
books although blanks therein are properly filled up if the person whose of stock and stock and transfer book, petitioner was not a bona
name is inserted therein has no control over the books of the

892
fidestockholder of Mr. & Ms. before March 1989 or at the time the complained representative of the stock itself and of the owner's interest therein. Hence,
acts were committed to qualify her to institute a stockholder's derivative suit when Certificate of Stock No. 008 was admittedly signed and issued only on
against private respondents. Aside from petitioner's own admissions, several 17 March 1989 and not on 25 July 1983, even as it indicates that petitioner
corporate documents disclose that the true party-in-interest is not petitioner owns 997 shares of stock of Mr. & Ms., the certificate has no evidentiary
but JAKA. value for the purpose of proving that petitioner was a stockholder since 1983
up to 1989.
Thus, while petitioner asserts in her petition that Certificate of Stock No. 008
dated 25 July 1983 was issued in her name, private respondents argue that And even the factual antecedents of the alleged ownership by petitioner in
this certificate was signed by respondent Eugenia D. Apostol as President 1983 of shares of stock of Mr. & Ms. are indistinctive if not enshrouded in
only in 1989 and was fraudulently antedated by petitioner who had inconsistencies. In her testimony before the Hearing Panel, petitioner said
possession of the Certificate Book and the Stock and Transfer Book. Private that early in 1983, to relieve Mr. & Ms. from political pressure, Senator Enrile
respondents stress that petitioner's counsel entered into a stipulation on decided to divest the family holdings in Mr. & Ms. as he was then part of the
record before the Hearing Panel that the certificate was indeed signed by government and Mr. & Ms. was evolving to be an opposition newspaper. The
respondent Apostol only in 1989 and not in 1983. JAKA shares numbering 1,000 covered by Certificate of Stock No. 001 were
thus transferred to respondent Eugenia D. Apostol in trust or in blank. 18
In her reply, petitioner admits that while respondent Eugenia D. Apostol
signed the Certificate of Stock No. 008 in petitioner's name only in 1989, it Petitioner now claims that a few days after JAKA's shares were transferred to
was issued by the corporate secretary in 1983 and that the other certificates respondent Eugenia D. Apostol, Senator Enrile sold to petitioner 997 shares
covering shares in Mr. & Ms. had not yet been signed by respondent Eugenia of JAKA. For this purpose, a deed of sale was executed and antedated to 10
D. Apostol at the time of the filing of the complaint with the SEC although May 1983. 19 This submission of petitioner is however contradicted by the
they were issued years before. records which show that a deed of sale was executed by JAKA transferring
1,000 shares of Mr. & Ms. to respondent Apostol on 10 May 1983 and not to
Based on the foregoing admission of petitioner, there is no truth to the petitioner. 20
statement written in Certificate of Stock No. 008 that the same was issued
and signed on 25 July 1983 by its duly authorized officers specifically the Then Senator Enrile testified that in May or June 1983 he was asked at a
President and Corporate Secretary because the actual date of signing media interview if his family owned shares of stock in Mr. & Ms. Although he
thereof was 17 March 1989. Verily, a formal certificate of stock could not be and his family were stockholders at that time he denied it so as not to
considered issued in contemplation of law unless signed by the president or embarrass the magazine. He called up petitioner and instructed her to work
vice-president and countersigned by the secretary or assistant secretary. out the documentation of the transfer of shares from JAKA to respondent
Apostol to be covered by a declaration of trust. His instruction was to transfer
In this case, contrary to petitioner's submission, the Certificate of Stock No. the shares of JAKA in Mr. & Ms. and Ex Libris to respondent Apostol as a
008 was only legally issued on 17 March 1989 when it was actually signed by nominal holder. He then finally decided to transfer the shareholdings to
the President of the corporation, and not before that date. While a certificate petitioner. 21
of stock is not necessary to make one a stockholder, e.g., where he is an
incorporator and listed as stockholder in the articles of incorporation although When asked if there was any document or any written evidence of that
no certificate of stock has yet been issued, it is supposed to serve as paper divestment in favor of petitioner, Senator Enrile answered that there was an

893
endorsement of the shares of stock. He said that there was no other And, there is nothing in the records which shows that JAKA had revoked the
document evidencing the assignment to petitioner because the stocks were trust it reposed on respondent Eugenia D. Apostol. Neither was there any
personal property that could be transferred even orally. 22 Contrary to Senator evidence that the principal had requested her to assign and transfer the
Enrile's testimony, however, petitioner maintains that Senator Enrile executed shares of stock to petitioner. If it was true that the shares of stock covered by
a deed of sale in her favor. Certificate of Stock No. 007 had been transferred to petitioner, the person
who could legally endorse the certificate was private respondent Eugenia D.
A careful perusal of the records shows that neither the alleged endorsement Apostol, she being the registered owner and trustee of the shares of stock
of Certificate of Stock No. 001 in the name of JAKA nor the alleged deed of covered by Certificate of Stock No. 007. It is a settled rule that the trustee
sale executed by Senator Enrile directly in favor of petitioner could have should endorse the stock certificate to validate the cancellation of her share
legally transferred or assigned on 25 July 1983 the shares of stock in favor of and to have the transfer recorded in the books of the corporation. 25
petitioner because as of 10 May 1983 Certificate of Stock No. 001 in the
name of JAKA was already cancelled and a new one, Certificate of Stock No. In fine, the records are unclear on how petitioner allegedly acquired the
007, issued in favor of respondent Apostol by virtue of a Declaration of Trust shares of stock of JAKA. Petitioner being the chief executive officer of JAKA
and Deed of Sale. 23 and the sole person in charge of all business and financial transactions and
affairs of JAKA 26 was supposed to be in the best position to show convincing
It should be emphasized that on 10 May 1983 JAKA executed, a deed of sale evidence on the alleged transfer of shares to her, if indeed there was a
over 1,000 Mr. & Ms. shares in favor of respondent Eugenio D. Apostol. On transfer. Considering that petitioner's status is being questioned and several
the same day, respondent Apostol signed a declaration of trust stating that factual circumstances have been presented by private respondents
she was the registered owner of 1,000 Mr. & Ms. shares covered by disproving petitioner's claim, it was incumbent upon her to submit rebuttal
Certificate of Stock No. 007. evidence on the manner by which she allegedly became a stockholder. Her
failure to do so taken in the light of several substantial inconsistencies in her
The declaration of trust further showed that although respondent Apostol was evidence is fatal to her case.
the registered owner, she held the shares of stock and dividends which might
be paid in connection therewith solely in trust for the benefit of JAKA, her The rule is that the endorsement of the certificate of stock by the owner or his
principal. It was also stated therein that being a trustee, respondent Apostol attorney-in-fact or any other person legally authorized to make the transfer
agreed, on written request of the principal, to assign and transfer the shares shall be sufficient to effect the transfer of shares only if the same is coupled
of stock and any and all such distributions or dividends unto the principal or with delivery. The delivery of the stock certificate duly endorsed by the owner
such other person as the principal would nominate or appoint. is the operative act of transfer of shares from the lawful owner to the new
transferee.
Petitioner was well aware of this trust, being the person in charge of this
documentation and being one of the witnesses to the execution of this Thus, for a valid transfer of stocks, the requirements are as follows: (a) There
document. 24 Hence, the mere alleged endorsement of Certificate of Stock must be delivery of the stock certificate; (b) The certificate must be endorsed
No. 001 by Senator Enrile or by a duly authorized officer of JAKA to effect the by the owner or his attorney-in-fact or other persons legally authorized to
transfer of shares of JAKA to petitioner could not have been legally feasible make the transfer; and, (c) to be valid against third parties, the transfer must
because Certificate of Stock No. 001 was already canceled by virtue of the be recorded in the books of the corporation. 27 At most, in the instant case,
deed of sale to respondent Apostol. petitioner has satisfied only the third requirement. Compliance with the first
two requisites has not been clearly and sufficiently shown.

894
Considering that the requirements provided under Sec. 63 of The That JAKA retained its ownership of its Mr. & Ms. shares was clearly shown
Corporation Code should be mandatorily complied with, the rule on by its receipt of the dividends issued in December 1986. 32 This only means,
presumption of regularity cannot apply. The regularity and validity of the very obviously, that Mr. & Ms. shares in question still belonged to JAKA and
transfer must be proved. As it is, even the credibility of the stock and transfer not to petitioner. For, dividends are distributed to stockholders pursuant to
book and the entries thereon relied upon by petitioner to show compliance their right to share in corporate profits. When a dividend is declared, it
with the third requisite to prove that she was a stockholder since 1983 is belongs to the person who is the substantial and beneficial owner of the
highly doubtful. stock at the time regardless of when the distribution profit was earned. 33

The records show that the original stock and transfer book and the stock Finally, this Court takes notice of the glaring and open admissions of
certificate book of Mr. & Ms. were in the possession of petitioner before their petitioner made, not just seven (7) but nine (9) times, during the 22
custody was transferred to the Corporate Secretary, Atty. Augusto San September 1988 meeting of the board of directors that the Enriles were her
Pedro. 28 On 25 May 1988, Assistant Corporate Secretary Renato Jose principals or shareholders, as shown by the minutes thereof which she duly
Unson wrote Mr. & Ms. about the lost stock and transfer book which was also signed 34
noted by the corporation's external auditors, Punongbayan and Araullo, in
their audit. Atty. Unson even informed respondent Eugenia D. Apostol as 5. Mrs. E. Apostol explained to the Directors that through her
President of Mr. & Ms. that steps would be undertaken to prepare and efforts, the asset base of the Company has improved and
register a new Stock and Transfer Book with the SEC. Incidentally, perhaps profits were realized. It is for this reason that the Company
strangely, upon verification with the SEC, it was discovered that the general has declared a 100% cash dividend in 1986. She said that it
file of the corporation with the SEC was missing. Hence, it was even possible is up for the Board to decide based on this performance
that the original Stock and Transfer Book might not have been registered at whether she should continue to act as Board Chairman or
all. not. In this regard, Ms. N.A. Bitong expressed her
recollection of how Ex-Libris/Mr. & Ms. were organized and
On 20 October 1988 respondent Eugenia D. Apostol wrote Atty. Augusto San her participation for and on behalf of her principals, as
Pedro noting the changes he had made in the Stock and Transfer Book follows: She recalled that her principals were invited by Mrs.
without prior notice to the corporate officers. 29 In the 27 October 1988 E. Apostol to invest in Ex-Libris and eventually Mr. & Ms. The
directors' meeting, respondent Eugenia D. Apostol asked about the relationship between her principals and Mrs. E. Apostol
documentation to support the changes in the Stock and Transfer Book with made it possible for the latter to have access to several
regard to the JAKA shares. Petitioner answered that Atty. San Pedro made information concerning certain political events and issues. In
the changes upon her instructions conformably with established practice. 30 many instances, her principals supplied first hand and
newsworthy information that made Mr. & Ms. a popular
This simply shows that as of 1988 there still existed certain issues affecting paper . . . .
the ownership of the JAKA shares, thus raising doubts whether the alleged
transactions recorded in the Stock and Transfer Book were proper, regular 6. According to Ms. Bitong, her principals were instrumental
and authorized. Then, as if to magnify and compound the uncertainties in the in helping Mr. & Ms. survive during those years that it was
ownership of the shares of stock in question, when the corporate secretary cash strapped . . . . Ms. N.A. Bitong pointed out that the
resigned, the Stock and Transfer Book was delivered not to the corporate practice of using the former Minister's influence and stature
office where the book should be kept but to petitioner. 31

895
in the government is one thing which her Corporation Code but is impliedly recognized when the law makes corporate
principals themselves are strongly against . . . . directors or officers liable for damages suffered by the corporation and its
stockholders for violation of their fiduciary duties.
7. . . . . At this point, Ms. N. Bitong again expressed her
recollection of the subject matter as follows: (a) Mrs. E. Hence, a stockholder may sue for mismanagement, waste or dissipation of
Apostol, she remembers, brought up the concept of a corporate assets because of a special injury to him for which he is otherwise
cooperative-ran newspaper company in one of her breakfast without redress. 39 In effect, the suit is an action for specific performance of
session with her principals sometime during the end of an obligation owed by the corporation to the stockholders to assist its rights
1985. Her principals when asked for an opinion, said that of action when the corporation has been put in default by the wrongful refusal
they recognized the concept as something very noble and of the directors or management to make suitable measures for its
visible . . . . Then Ms. Bitong asked a very specific question protection. 40
"When you conceptualized Ex-Libris and Mr. & Ms., did
you not think of my shareholders the Ponce Enriles as The basis of a stockholder's suit is always one in equity. However, it cannot
liabilities? How come you associated yourself with them then prosper without first complying with the legal requisites for its institution. The
and not now? What is the difference?" Mrs. Apostol did not most important of these is the bona fide ownership by a stockholder of a
answer the question. stock in his own right at the time of the transaction complained of which
invests him with standing to institute a derivative action for the benefit of the
The admissions of a party against his interest inscribed upon the record corporation. 41
books of a corporation are competent and persuasive evidence against
him. 35 These admissions render nugatory any argument that petitioner is WHEREFORE, the petition is DENIED. The 31 August 1995 Decision of the
a bona fidestockholder of Mr. & Ms. at any time before 1988 or at the time Court of Appeals dismissing the complaint of petitioner Nora A. Bitong in CA-
the acts complained of were committed. There is no doubt that petitioner was G.R. No. SP 33291, and granting the petition for certiorari and prohibition
an employee of JAKA as its managing officer, as testified to by Senator Enrile filed by respondent Edgardo U. Espiritu as well as annulling the 5 November
himself. 36 However, in the absence of a special authority from the board of 1993, 24 January 1993 and 18 February 1994 Orders of the SEC En Banc in
directors of JAKA to institute a derivative suit for and in its behalf, petitioner is CA-G.R. No. SP 33873, is AFFIRMED. Costs against petitioner.
disqualified by law to sue in her own name. The power to sue and be sued in
any court by a corporation even as a stockholder is lodged in the board of SO ORDERED.
directors that exercises its corporate powers and not in the president or
officer thereof. 37
Davide, Vitug and Quisumbing, JJ., concur.

It is well settled in this jurisdiction that where corporate directors are guilty of
Panganiban, J., took no part.
a breach of trust, not of mere error of judgment or abuse of discretion, and
intracorporate remedy is futile or useless, a stockholder may institute a suit in
behalf of himself and other stockholders and for the benefit of the
corporation, to bring about a redress of the wrong inflicted directly upon the
corporation and indirectly upon the stockholders. 38 The stockholder's right to
institute a derivative suit is not based on any express provision of The

896
897
G.R. No. 85339 August 11, 1989

898
SAN MIGUEL CORPORATION, represented by EDUARDO DE LOS expressed the desire of the 14 corporations to sell the shares of stock "to pay
ANGELES, petitioners, certain outstanding and unpaid debts," and Soriano's own wish to purchase
vs. the same "in order to institutionalize and stabilize the management of the
ERNEST KAHN, ANDRES SORIANO III, BENIGNO TODA, JR., ANTONIO COMPANY in .. (himself) and the professional officer corps, mandated by the
ROXAS, ANTONIO PRIETO, FRANCISCO EIZMENDI, JR., EDUARDO COMPANY's By- laws, and to direct the COMPANY towards giving the
SORIANO, RALPH KAHN and RAMON DEL ROSARIO, JR.,respondents. highest priority to its principal products and extensive support to agriculture
programme of' the Government ... 6 Actually, according to Soriano and the
Romulo, Mabanta, Buenaventura, Sayoc & De Los Angeles petitioner. other private respondents, the buyer of the shares was a foreign company,
Neptunia Corporation Limited (of Hongkong, a wholly owned subsidiary of
Roco & Bunag Law Offices for respondent Ernest Kahn. San Miguel International which is, in turn, a wholly owned subsidiary of San
Miguel Corporation; 7 and it was Neptunia which on or about April 1, 1986
had made the down payment of P500,000,000.00, "from the proceeds of
Siguion Reyna, Montecillo and Ongsiako for other respondents.
certain loans". 8

At this point the 33,133,266 SMC shares were sequestered by the


Presidential Commission on Good Government (PCGG), on the ground that
NARVASA, J.: the stock belonged to Eduardo Cojuangco, Jr., allegedly a close associate
and dummy of former President Marcos, and the sale thereof was "in direct
On December 15, 1983, 33,133,266 shares of the outstanding capital stock contravention of .. Executive Orders Numbered 1 and 2 (.. dated February
of the San Miguel Corporation were acquired 1 by fourteen (14) other 28, 1986 and March 12, 1986, respectively) which prohibit .. the transfer,
corporations, 2 and were placed under a Voting Trust Agreement in favor of conveyance, encumbrance, concealment or liquidation of assets and
the late Andres Soriano, Jr. When the latter died, Eduardo M. Cojuangco, Jr. properties acquired by former President Ferdinand Marcos and/or his wife,
was elected Substitute Trustee on April 9, 1984 with power to delegate the Mrs. Imelda Romualdez Marcos, their close relatives, subordinates, business
trusteeship in writing to Andres Soriano III. 3 Shortly after the Revolution of associates. 9 The sequestration was subsequently lifted, and the sale allowed
February, 1986, Cojuangco left the country amid "persistent reports" that to proceed, on representations by San Miguel Corporation x x that the shares
"huge and unusual cash disbursements from the funds of SMC" had been were 'owned by 1.3 million coconut farmers;' the seller corporations were
irregularly made, and the resources of the firm extensively used in support of 'fully owned' by said farmers and Cojuangco owned only 2 shares in one of
the candidacy of Ferdinand Marcos during the snap elections in February, the companies, etc. However, the sequestration was soon re-imposed by
1986 . 4 Order of the PCGG dated May 19, 1986 .. The same order forbade the SMC
corporate Secretary to register any transfer or encumbrance of any of the
On March 26, 1986, an "Agreement" was executed between Andres Soriano stock without the PCGG's prior written authority. 10
III, as "Buyer," and the 14 corporations, as "Sellers," for the purchase by
Soriano, "for himself and as agent of several persons," of the 33,133,266 San Miguel promptly suspended payment of the other installments of the
shares of stock at the price of P100.00 per share, or "an aggregate sum of price to the fourteen (14) seller corporations. The latter as promptly sued for
Three Billion Three Hundred Thirteen Million Three Hundred Twenty Six rescission and damages. 11
Thousand Six Hundred (P3,313,326,600.00) Pesos payable in specified
installments. 5 The Agreement revoked the voting trust above mentioned, and

899
On June 4,1986, the PCGG directed San Miguel Corporation"to issue resolution authorizing the company to borrow up to US
qualifying shares" in the corporation to seven (7) individuals, including $26,500,000.00 from the Hongkong & Shanghai Banking
Eduardo de los Angeles, "from the sequestered shares registered as street Corporation, Hongkong "to enable the Soriano family to
certificates under the control of Anscor- Hagedorn Securities, Inc.," to "be initiate steps and sign an agreement for the purchase of
held in trust by .. (said seven [7] persons) for the benefit of Anscor-Hagedom some 33,133,266 shares of San ,Miguel Corporation. 16
Securities, Inc. and/or whoever shall finally be determined to be the
owner/owners of said shares. 12 b) The loan of $26,500,000.00 was obtained on the same
day, the corresponding loan agreement having been signed
In December, 1986, the SMC Board, by Resolution No. 86-122, "decided to for Neptunia by Ralph Kahn and Carl Ottiger. At the latter's
assume the loans incurred by Neptunia for the down payment ((P500M)) on request, the proceeds of the loan were deposited in different
the 33,133,266 shares." The Board opined that there was "nothing illegal in banks 17 for the account of "Eduardo J. Soriano".
this assumption (of liability for the loans)," since Neptunia was "an indirectly
wholly owned subsidiary of SMC," there "was no additional expense or c) Three (3) days later, on April 4, 1986, Soriano III sent
exposure for the SMC Group, and there were tax and other benefits which identical letters to the stockholders of San Miguel
would redound to the SMC group of companies. 13 Corporation, 18 inter alia soliciting their proxies and
announcing that "the Soriano family, friends and affiliates
However, at the meeting of the SMC Board on January 30, 1987, Eduardo de acquired a considerable block of San Miguel Corporation
los Angeles, one of the PCGG representatives in the SMC board, impugned shares only a few days ago .., the transaction .. (having
said Resolution No. 86-12-2, denying that it was ever adopted, and stating been) made through the facilities of the Manila Stock
that what in truth was agreed upon at the meeting of December 4, 1986 was Exchange, and 33,133,266 shares .. (having thereby been)
merely a "further study" by Director Ramon del Rosario of a plan presented purchased for the aggregate price of' P3,313,326,600.00."
by him for the assumption of the loan. De los Angeles also pointed out The letters also stated that the purchase was "an exercise of
certain "deleterious effects" thereof. He was however overruled by private the Sorianos' right to buy back the same number of shares
respondents. 14 When his efforts to obtain relief within the corporation and purchased in 1983 by the .. (14 seller corporations)."
later the PCGG proved futile, he repaired to the Securities and Exchange
Commission (SEC).lwph1.t d) In implementing the assumption of the Neptunia loan and
the purchase agreement for which said loan was obtained,
He filed with the SEC in April, 1987, what he describes as a derivative suit in which assumption constituted an improper use of corporate
behalf of San Miguel Corporation, against ten (10) of the fifteen-member funds to pay personal obligations of Andres Soriano III,
Board of Directors who had "either voted to approve and/or refused to enabling him; to purchase stock of the corporation using
reconsider and revoke Board Resolution No. 86-12-2." 15 His Amended funds of' the corporation itself, the respondents, through
Petition in the SEC recited substantially the foregoing antecedents and the various subsequent machinations and manipulations, for
following additional facts, to wit: interior motives and in breach of fiduciary duty, compound
the damages caused San Miguel Corporation by, among
a) On April 1, 1986 Soriano, Kahn and Roxas, as directors other things: (1) agreeing to pay a higher price for the shares
of' Neptunia Corporation, Ltd., had met and passed a than was originally covenanted in order to prevent a
rescission of the purchase agreement by the sellers; (2)

900
urging UCPB to accept San Miguel Corporation and c) he has not come to court
Neptunia as buyers of the shares, thereby committing the with clean hands; and
former to the purchase of its own shares for at least 25%
higher than the price at which they were fairly traded in the 2. The Securities & Exchange Commission has no
stock exchanges, and shifting to said corporations the jurisdiction over the controversy because the matters
personal obligations of Soriano III under the purchase involved are exclusively within the business judgment of the
agreement; and (3) causing to be applied to the part Board of Directors. 19
payment of P1,800,000.00 on said purchase, various assets
and receivables of San Miguel Corporation. Kahn's motion to dismiss was subsequently adopted by his
correspondents . 20
The complaint closed with a prayer for injunction against the execution or
consummation of any agreement causing San Miguel Corporation to The motion to dismiss was denied by SEC Healing Officer Josefina L. Pasay
purchase the shares in question or entailing the use of its corporate funds or Paz, by order dated September 4, 1987. 21 In her view
assets for said purchase, and against Andres Soriano III from further using or
disposing of the funds or assets of the corporation for his obligations; for the
1) the fact that de los Angeles was a PCGG nominee was
nullification of the SMC Board's resolution of April 2, 1987 making San
irrelevant because in law, ownership of even one share only,
Miguel Corporation a party to the purchase agreement; and for damages.
sufficed to qualify a person to bring a derivative suit;

Ernest Kahn moved to dismiss de los Angeles' derivative suit on two


2) it is indisputable that the action had been brought by de
grounds, to wit:
los Angeles for the benefit of the corporation and all the
other stockholders;
1 De los Angeles has no legal capacity to sue because
3) he was a stockholder at the time of the commission of the
a) having been merely acts complained of, the number of shares owned by him
imposed by the PCGG as a being to repeat, immaterial;
director on San Miguel, he
has no standing to bring a
4) there is no merit in the assertion that he had come to
minority derivative suit;
Court with unclean hands, it not having been shown that he
participated in the act complained of or ratified the same;
b) he personally holds only and
20 shares and hence
cannotfairly and adequately
5) where business judgment transgresses the law, the
represent the minority
securities and Exchange Commission always has
stockholders of the
competence to inquire thereinto.
corporation;

901
Kahn filed a petition for certiorari and prohibition with the Court of Appeals, 1) the number of shares owned by him was immaterial, he
seeking the annulment of this adverse resolution of the SEC Hearing Officer being a stockholder in his own right;
and her perpetual inhibition from proceeding with SEC Case No. 3152.
2) he had not voted in favor of the resolution authorizing the
A Special Division of that Court sustained him, upon a vote of three-to-two. purchase of the shares; and
The majority 22 ruled that de los Angeles had no egal capacity to institute the
derivative suit, a conclusion founded on the following propositions: 3) even if PCGG was not the owner of the sequestered
shares, it had the right to seek the protection of the interest
1) a party "who files a derivative suit should adequately of the corporation, it having been held that even an
represent the interests of the minority stockholders;" since unregistered shareholder or an equitable owner of shares
"De los Angeles holds 20 shares of stock out of 121,645,860 and pledgees of shares may be deemed a shareholder for
or 0.00001644% (appearing to be undisputed), (he) cannot purposes of instituting a derivative suit.
even be remotely said to adequately represent the interests
of the minority stockholders, (e)specially so when .. de los De los Angeles has appealed to this Court. He prays for reversal of the
Angeles was put by the PCGG to vote the majority stock," a judgment of the Court of Appeals, imputing to the latter the following errors:
situation generating "a genuine conflict of interest;"
1) having granted the writ of certiorari despite the fact that
2) de los Angeles has not met this conflict-of-interest Kahn had not first resorted to the plain remedy available to
argument, i.e., that his position as PCGG-nominated director him, i.e., appeal to the SEC en banc and despite the fact that
is inconsistent with his assumed role of representative of no question of jurisdiction was involved;
minority stockholders; not having been elected by the
minority, his voting would expectedly consider the interest of 2) having ruled on Kahn's petition on the basis merely of his
the entity which placed him in the board of directors; factual allegations, although he (de los Angeles) had
disputed them and there had been no trial in the SEC; and
3) Baseco v. PCGG, May 27,1987, 23 has laid down the
principle that the (a) PCGG cannot exercise acts of dominion 3) having held that he (de los Angeles) could not file a
over sequestered property, (b) it has only powers of derivative suit as stockholder and/or director of the San
administration, and (c) its voting of sequestered stock must Miguel Corporation.
be done only pursuant to its power of administration; and
For their part, and in this Court, the respondents make the following
4) de los Angeles' suit is not a derivative suit, a derivative assertions:
suit being one brought for the benefit of the corporation.
1) SEC has no jurisdiction over the dispute at bar which
The dissenting Justices, 24 on the other hand, were of the opinion that the suit involves the ownership of the 33,133,266 shares of SMC
had been properly brought by de los Angeles because stock, in light of this Court's Resolution in G.R. Nos. 74910,

902
5075, 75094, 76397, 79459 and 79520, promulgated on De los Angeles' Reply seeks to make the following points:
August 10, 1988; 25
1) since the law lays down three (3) requisites for a
2) de los Angeles was beholden to the controlling derivative suit, viz:
stockholder in the corporation (PCGG), which had "imposed"
him on the corporation; since the PCGG had a clear conflict a) the party bringing suit should be a
of interest with the minority, de los Angeles, as director of the shareholder as of the time of the act or
former, had no legal capacity to sue on behalf of the latter; transaction complained of,

3) even assuming absence of conflict of interest, de los b) he has exhausted intra-corporate


Angeles does not fairly and adequately represent the interest remedies, i.e., has made a demand on the
of the minority stockholders; board of directors for the appropriate relief
but the latter has failed or refused to heed
4) the respondents had properly applied for certiorari with his plea; and
the Court of Appeals because
c) c)the cause of action actually devolves on
a) that Court had, by law, exclusive appellate jurisdiction the corporation, the ,wrongdoing or harm
over officers and agencies exercising quasi-judicial having been caused to the corporation and
functions, and hence file competence to issue the writ not to .the particular stockholder bringing the
of certiorari; suit;

b) the principle of exhaustion of administrative remedies and since (1) he is admittedly the owner of 20 shares of SMC stock in
does not apply since the issue involved is one of law; his own right, having acquired those shares as early as 1977, (2) he had
sought without success to have the board of' directors remedy with wrong,
c) said respondents had no plain, speedy and adequate and (3) that wrong was in truth a 'wrong against the stockholders of the
remedy within SEC; corporation, generally, ,and not against him individually and it was the
corporation, and not he, particularly, that would be entitled to the appropriate'
d) the Order of the SEC Investigating Officer denying the relief the propriety of his suit cannot be gainsaid;
motion to dismiss was issued without or in excess of
jurisdiction, hence was correctly nullified by the Court of 2) Kahn had not limited himself to questions of law in the
Appeals; and proceedings in the Court of Appeals and hence could not
claim exclusion from the scope of the doctrine of exhaustion
e) de los Angeles had not raised the issue of absence of a of remedies; moreover, Rule 65, invoked by him, bars a
motion for reconsideration by respondents in the SEC case; resort to certiorari. where a plain, speedy and accurate
in any event, such a motion was unnecessary in the remedy was available to him in this case, to wit, a motion for
premises. reconsideration before the Sec en banc and, contrary to the

903
respondent's claim, De Los Angeles had in fact asserted to sequestered stock thru its nominee directors as was done
this propositions before the Appellate Tribunal; and by United Coconut Planters Bank and the 14 seller
corporations (in SEC Case No. 3005, later consolidated with
3) the respondent had not raised the issue of jurisdiction SEC Case No. 3000 above mentioned, these two (2) cases
before the Court of Appeals; indeed, they admit to their later having become subject of G.R.No. 76397) as well as by
Comment that that one Clifton Ganay, a UCPB stockholder (in G.R. No. 75094
of this Court).lwph1.t 28
issue has not yet been resolved by the SEC," be this as it
may, the derivative suit does not fall within the BASECO The subject matter of his complaint in the SEC does not therefore fall within
doctrine since it does not involve any question of ownership the ambit of this Court's Resolution of August 10, 1988 on the cases just
of the 33,133,266 sequestered SMC shares but rather, the mentioned, to the effect that, citing PCGG v. Pena, et al 29 an cases of the
validity of the resolution of the board of directors for the Commission regarding 'the funds, moneys, assets, and properties illegally
assumption by the corporation, for the benefit of certain of its acquired or misappropriated by former President Ferdinand Marcos, Mrs.
officers and stockholders, of liability for loans contracted by Imelda Romualdez Marcos, their close relatives, Subordinates, Business
another corporation, which is an intra-corporate dispute Associates, Dummies, Agents, or Nominees, whether civil or criminal, are
within the exclusive jurisdiction of the SEC. lodged within the exclusive and original jurisdiction of the Sandiganbayan,'
and all incidents arising from, incidental to, or related to, such cases
1. De los Angeles is not opposed to the asserted position of necessarily fall likewise under the Sandiganbayan's exclusive and original
the PCGG that the sequestered SMC shares of stock belong jurisdiction, subject to review on certiorari exclusively by the Supreme Court."
to Ferdinand Marcos and/or his dummies and/or cronies. His His complaint does not involve any property illegally acquired or
consent to sit in the board as nominee of PCGG misappropriated by Marcos, et al., or "any incidents arising from, incidental
unquestionably indicates his advocacy of the PCGG to, or related to" any case involving such property, but assets indisputably
position. He does not here seek, and his complaint in the belonging to San Miguel Corporation which were, in his (de los Angeles')
SEC does not pray for, the annulment of the purchase by view, being illicitly committed by a majority of its board of directors to answer
SMC of the stock in question, or even the subsequent for loans assumed by a sister corporation, Neptunia Co., Ltd.
purchase of the same stock by others 26which proposition
was challenged by (1) one Evio, in SEC Case No. 3000; (2) De los Angeles' complaint, in fine, is confined to the issue of the validity of
by the 14 corporations which sold the stock to SMC, in Civil the assumption by the corporation of the indebtedness of Neptunia Co., Ltd.,
Case No. 13865 of the Manila RTC, said cases having later allegedly for the benefit of certain of its officers and stockholders, an issue
become subject of G.R. No. 74910 of this Court; (3) by evidently distinct from, and not even remotely requiring inquiry into the matter
Neptunia, SMC, and others, in G.R. No. 79520 of this Court; of whether or not the 33,133,266 SMC shares sequestered by the PCGG
and (4) by Eduardo Cojuangco and others in Civil Case No. belong to Marcos and his cronies or dummies (on which- issue, as already
16371 of the RTC, Makati, [on the theory that the pointed out, de los Angeles, in common with the PCGG, had in fact espoused
sequestered stock in fact belonged to coconut planters and the affirmative). De los Angeles' dispute, as stockholder and director of SMC,
oil millers], said case later having become subject of G.R. with other SMC directors, an intra-corporate one, to be sure, is of no concern
No. 79459 of this court . 27 Neither does de los Angeles to the Sandiganbayan, having no relevance whatever to the ownership- of
impugn, obviously, the right of the PCGG to vote the the sequestered stock. The contention, therefore, that in view of this Court's

904
ruling as regards the sequestered SMC stock above adverted to, the SEC The bona fide ownership by a stockholder of stock in his own right suffices to
has no jurisdiction over the de los Angeles complaint, cannot be sustained invest him with standing to bring a derivative action for the benefit of the
and must be rejected. The dispute concerns acts of the board of directors corporation. The number of his shares is immaterial since he is not suing in
claimed to amount to fraud and misrepresentation which may be detrimental his own behalf, or for the protection or vindication of his own particular right,
to the interest of the stockholders, or is one arising out of intra-corporate or the redress of a wrong committed against him, individually, but in behalf
relations between and among stockholders, or between any or all of them and for the benefit of the corporation.
and the corporation of which they are stockholders . 30
3. Neither can the "conflict-of-interest" theory be upheld.
2. The theory that de los Angeles has no personality to bring From the conceded premise that de los Angeles now sits in
suit in behalf of the corporation because his stockholding the SMC Board of Directors by the grace of the PCGG, it
is minuscule, and there is a "conflict of interest" between him does not follow that he is legally obliged to vote as the
and the PCGG cannot be sustained, either. PCGG would have him do, that he cannot legitimately take a
position inconsistent with that of the PCGG, or that, not
It is claimed that since de los Angeles 20 shares (owned by him since 1977) having been elected by the minority stockholders, his vote
represent only. 00001644% of the total number of outstanding shares (1 would necessarily never consider the latter's interests. The
21,645,860), he cannot be deemed to fairly and adequately represent the proposition is not only logically indefensible, non sequitur,
interests of the minority stockholders. The implicit argument that a but also constitutes an erroneous conception of a director's
stockholder, to be considered as qualified to bring a derivative suit, must hold role and function, it being plainly a director's duty to vote
a substantial or significant block of stock finds no support whatever in the according to his own independent judgment and his own
law. The requisites for a derivative suit 31 are as follows: conscience as to what is in the best interests of the
company. Moreover, it is undisputed that apart from the
a) the party bringing suit should be a shareholder as of the qualifying shares given to him by the PCGG, he owns 20
time of the act or transaction complained of, the number of shares in his own right, as regards which he cannot from any
his shares not being material; 32 aspect be deemed to be "beholden" to the PCGG, his
ownership of these shares being precisely what he invokes
as the source of his authority to bring the derivative suit.
b) he has tried to exhaust intra-corporate remedies, i.e., has
made a demand on the board of directors for the appropriate
relief but the latter has failed or refused to heed his 4. It is also theorized, on the authority of the BASECO
plea; 33 and decision, that the PCGG has no power to vote sequestered
shares of stock as an act of dominion but only in pursuance
to its power of administration. The inference is that the
c) the cause of action actually devolves on the corporation,
PCGG's act of voting the stock to elect de los Angeles to the
the wrongdoing or harm having been, or being caused to the
SMC Board of Directors was unauthorized and void; hence,
corporation and not to the particular stockholder bringing the
the latter could not bring suit in the corporation's behalf. The
suit. 34
argument is strained and obviously of no merit. As already
more than plainly indicated, it was not necessary for de los
Angeles to be a director in order to bring a derivative action;

905
all he had to be was a stockholder, and that he was owning
in his own right 20 shares of stock, a fact not disputed by the
respondents. G.R. No. L-5174 March 17, 1911

Nor is there anything in the Baseco decision which can be interpreted as CANDIDO PASCUAL, plaintiff-appellant,
ruling that sequestered stock may not under any circumstances be voted by vs.
the PCGG to elect a director in the company in which such stock is held. On EUGENIO DEL SAZ OROZCO, ET AL, defendants-appellees.
the contrary, that it held such act permissible is evident from the context of its
reference to the Presidential Memorandum of June 26, 1986 authorizing the C.W. Ney and O'Brien and De Witt for appellant.
PCGG, "pending the outcome of proceedings to determine the ownership Ortigas and Fisher for appellees.
of .. sequestered shares of stock,"'to vote such shares .. at all stockholders'
meetings called for the election of directors ..," the only caveat being that the
TRENT, J.:
stock is not to be voted simply because the power to do so exists, whether it
be to oust and replace directors or to effect substantial changes in corporate
policy, programs or practice, but only "for demonstrably weighty and This is an appeal by the plaintiff from a judgment sustaining a demurrer to
defensible grounds" or "when essential to prevent disappearance or wastage the first and second causes of action set forth in the amended complaint. The
of corporate property." demurrer as to both causes of action was based upon the following grounds:

The issues raised here do not peremptorily call for a determination of (a) Lack of legal capacity to use on part of plaintiff;
whether or not in voting petitioner de los Angeles to the San Miguel Board, (b) Failure to state facts constituting a cause of action;
the PCGG kept within the parameters announced in Baseco; and absent any (c) Defect of parties plaintiff; and,
showing to the contrary, consistently with the presumption that official duty is (d) Uncertainty.
regularly performed, it must be assumed to have done so.
The lower court sustained the demurrer as to both causes of action upon the
WHEREFORE, the petition is GRANTED. The appealed decision of the second ground above-mentioned.
Court of Appeals in CA- G.R. SP No. 12857 setting aside the order of
September 4, 1987 issued in SEC Case No. 3153 and dismissing said case The following errors have been assigned:
is REVERSED AND SET ASIDE. The further disposition in the appealed
decision for the issuance of a writ of preliminary injunction upon the filing and The court a quo erred in sustaining the demurrer to the first cause of action
approval of a bond of P500,000.00 by respondent Ernest Kahn (petitioner in and dismissing the same because (a) the facts alleged constitute a cause of
the Appellate Court) is also SET ASIDE, and any writ of injunction issued action, and (b) the remedy sought by the plaintiff is the only one available.
pursuant thereto is lifted. Costs against private respondents.
The same errors are assigned as to the second cause of action.
SO ORDERED.
The gist of the first and second causes of action is as follows:
Gancayco, Gri;o-Aquino and Medialdea, JJ., concur.

906
That during the years 1903, 1904, 1905, and 1907 the defendants and This action was brought by the plaintiff Pascual, in his own right as a
appellees, without the knowledge, consent, or acquiescence of the stockholder of the bank, for the benefit of the bank, and all the other
stockholders, deducted their respective compensation from the gross income stockholders thereof. The plaintiff sues on behalf of the corporation, which,
instead of from the net profits of the bank, thereby defrauding the bank and even though nominally a defendant, is to all intents and purposes the real
its stockholders of approximately P20,000 per annum; that though due plaintiff in this case. That such is the case is shown by the prayer of the
demands has been made upon them therefor, defendants refuse to refund to complaint which is that judgment be entered in favor of the bank.
the bank the sums so misappropriated, or any part thereof; that defendants
constitute a majority of the present board of directors of the bank, who alone According to the pleadings, the Banco Espaol-Filipino is a banking
can authorize an action against them in the name of the corporation, and that corporation, constituted as such by royal decree of the Crown of Spain in the
prior to the filing of the present suit plaintiff exhausted every remedy in the year 1854, the original grant having been subsequently extended and
premises within this banking corporation. modified by royal decree of July 14, 1897, and by Act No. 1790 of the
Philippine Commission. From the first it has been a bank of issue, and under
The second cause of action sets forth that defendants' and appellees' the Spanish regime was regarded as a quasi-public institution, its full title
immediate predecessors in office in this bank during the years 1899, 1900, having been originally "Banco Espaol-Filipino de Isabel II." The Captain-
1901, and 1902, committed the same illegality as to their compensation as is General of the Philippine Islands was its protector and supreme head. To him
charged against the defendants themselves; that in the four years belonged the power to appoint its directors and other managing officers,
immediately following the year 1902, the defendants and appellees were the remove them from office for cause, fix the rate of interest demandable by the
only officials or representatives of the bank who could and should investigate bank, resolve all doubts and controversies relating to its management, "and
and take action in regard to the sums of money thus fraudulently finally, exercise, as representative of Her Majesty's Government, the powers
appropriated by their predecessors; that they were the only persons that the laws give him respecting public establishments protected and
interested in the bank who knew of the fraudulent appropriation by their privileged."
predecessors; that they wholly neglected to take any action in the premises
or inform the stockholders thereof; that due demand has been made upon It is alleged in the amended complaint that the only compensation
defendants to reimburse the bank for this loss; that the bank itself can not contemplated or provided for the managing officers of the bank was a certain
bring an action in its own name against the defendants and appellees, for the per cent of the net profits resulting from the bank's operations, as set forth in
reason already stated, and that there remains no remedy within the article 30 of its reformed charter or statutes, which article is as follows:
corporation itself.
Of the profits or gains which may result from the bank's operations,
The questions raised in third cause of action are not before us at this as the after deducting all the expenses of its administration and the part, if
demurrer to that cause of action was overruled. any, which corresponds to the legal reserve fund, there shall be set
apart ten per cent for the directors and five per cent for the board of
The court below sustained the demurrer as to the first and second causes of government, the distribution of which shall be made as provided in
action on the ground that in actions of this character the plaintiff must aver in the regulations. The eighty-five per cent remaining shall belong
his complaint that he was the owner of stock in the corporation at the time of integrally to the shareholders pro rata the number of shares owned
the occurrences complained of, or else that the stock has since devolved by each.
upon him by operation of law.

907
Before proceeding to the determination of the real questions involved in this In suits of this character the corporation itself and not the plaintiff stockholder
case it might be well to note briefly the origin and history of the right of a is the real party in interest. The rights of the individual stockholder are
stockholder in a corporation to maintain a suit of this kind. merged into that of the corporation. It is a universally recognized doctrine that
a stockholder in a corporation has no title legal or equitable to the corporate
A corporation is an artificial being, invinsible, intangible, and existing property; that both of these are in the corporation itself for the benefit of all
only in contemplation of law. (Chief Justice Marshall in Trustees of the stockholders. Text writers illustrate this rule by the familiar example of
Dartmouth College vs. Woodward, 4 Wheat., 636.) one person or entity owning all the stock and still having no greater or
essentially different title than if he owned but one single share. Since,
The word "corporation" is but a collective name for the corporators or therefore, the stockholder has no title, it is evident that what he does have,
members who compose an incorporated association; and where it is with respect to the corporation and his fellow stockholder, are certain
said that a corporation is itself a person, or being, or creature, this rights sui generis. These rights are generally enumerated as being, first, to
must be understood in a figurative sense only. (Morawetz on Private have a certificate or other evidence of his status as stockholder issued to
Corporations, 2nd ed., sec. 1.) him; second, to vote at meetings of the corporation; third, to receive his
proportionate share of the profits of the corporation; and lastly, to participate
proportionately in the distribution of the corporate assets upon the dissolution
A corporation is "an artificial person created by the sovereign from
or winding up. (Purdy's Beach on Private Corporations, sec. 554.)
natural persons and in which artificial person the natural persons of
which it is composed become merged and nonexistent." (Quoted
with approval in case of The People, ex rel. Winchester, etc., The right of individual stockholders to maintain suits for and on behalf of the
respondent, vs. Coleman, et al., commissioners of taxes etc., corporation was denied until within a comparatively short time, but his right is
appellants, 133 N.Y. Appls., 279.) now no longer doubted. On this point Cook on Corporations, 5th ed. (1903),
secs. 644, 645, and 646, says:

Notwithstanding this fact, however, that it was the duty and right of the
corporation to bring suit remedy these wrongs, it gradually became apparent
that frequently the corporation was helpless and unable to institute the suit. It
was found, where the guilty parties themselves controlled the directors and
also a majority of the stock, that the corporation was in their power, was
unable to institute suit, and that the minority of the stockholders were being
defrauded of their rights and were without remedy. The time came when the
minority of the stockholders of the defrauded corporation the corporation
itself being controlled by the guilty parties were given a standing in court
for the purpose of taking up the cause of the corporation, and, in its name
and stead, of bringing the guilty parties to an account. Accordingly, in 1843,
in the leading case of Foss vs. Harbottle, a stockholder brought suit in the
name of himself and other defrauded stockholders, and for the benefit of the
corporation, against the directors, for a breach of their duty to the
corporation. This case was decided against the complaining stockholder, on

908
the ground that the complainant had not proved that the corporation itself So it is clear that the plaintiff, by reason of the fact that he is a stockholder in
was under the control of the guilty parties, and had not proved that it was the bank (corporation) has a right to maintain a suit for and on behalf of the
unable to institute suit. The court, however, broadly intimated that a bank, but the extent of such a right must depend upon when, how, and for
case might arise when a suit instituted by defrauded stockholders would be what purpose he acquired the shares which he now owns. In the
entertained by the court and redress given. Acting upon this suggestion, and determination of these questions we can not see how, if it be true that the
impelled by the utter inadequacy of suits instituted by the corporation, bank is a quasi-public institution, it can affect in any way the final result.
defrauded stockholders continued to institute these suits and to urge the
courts of equity to grant relief. These efforts were unsuccessful in clearly It is alleged that the plaintiff became a stockholder on the 13th of November,
establishing the right of stockholders herein until the cases of Atwol against 1903; that the defendants, as members of the board of directors and board of
Merriwether, in England, 1867, and of Dodge vs. Woolsey, in this country, in government, respectively, during each and all the years 1903, 1904, 1905,
1855. These two great and leading cases have firmly established the law for 1906, and 1907, did fraudulently, and to the great prejudice of the bank and
England and America, that where corporate directors have committed a its stockholders, appropriate to their own use from the profits of the bank
breach of trust either by their frauds, ultra vires acts, or negligence, and the sums of money amounting approximately to P20,000 per annum.
corporation is unable or unwilling to institute suit to remedy the wrong, a
single stockholder may institute that suit, suing on behalf of himself and other Article 31 of the bank's charter provides that dividends shall be declared
stockholders and for the benefit of the corporation, to bring about a redress each semestre. The stockholders meet once a year, in February, to receive
of the wrong done directly to the corporation and indirectly to the and consider the report of the bank's operations contained in the annual
stockholders. balance and memorial. Beyond this they have no direct voice in the affairs of
the bank, but all who are then stockholders and have a right to vote must
It is now no longer doubted, said Mr. Justice Wayne, in the case of clearly have a right to vote upon all the business proceedings of the year,
Dodge vs. Woolsey, 18 How. (U.S.), 331 either in England or the irrespective of the date upon which they may have become stockholders.
United States, that courts of equity, in both, have a jurisdiction over They are entitled to all the dividends that have been earned by their stock
corporations, at the instance of one or more of their members; to during the year which has not been earned by their stock during the year
apply preventive remedies by injunction, to restrain those who which has not been already declared and paid, regardless of the precise
administer them from doing acts which would amount to a violation of period of the year in which it may have accrued. So, in the general meeting
charters, or to prevent any misapplication of their capitals or profits, of the stockholders on February 3, 1904, the plaintiff had a right to
which might result in lessening the dividends of stockholders, or the participate.
value of their shares, as either may be protected by the franchises of
a corporation, if the acts intended to be done create what is in the Neither the charter, the by-laws, nor the regulations prescribe when, within
law denominated a breach of trust. And the jurisdiction extends to the semestre, the dividends shall be declared; but it may be presumed that
inquire into, and to enjoin, as the case may require that to be done, such dividends are declared at the end of the semestre and that the
any proceedings by individuals, in whatever character they may first semestre begins with the first day of January of each year. On this basis
profess to act, if the subject of complaint is an imputed violation of a the owner of stock from whom the plaintiff purchased his ten shares might
corporate franchise, or the denial of a right growing out of it, for have received the dividends corresponding to these ten shares for the
which there is not an adequate remedy at law. first semestre (six months) of the year 1903. The dividends were declared
twice a year, every six months. The times for declaring the dividends are
specifically and distinctly pointed out one period is separated from the

909
other. Every six months forms a period. So if the plaintiff was not entitled to The complaint contains three causes of action, each set forth in a separate
the dividends for the first period (from January to July, 1903), he having paragraph. The matter in the first cause is, as we have said, divisible in its
become a stockholder in September of that year, he would have been nature. The rule above quoted is, therefore, perfectly applicable.
entitled to the dividends on his stock for the second period, or semestre. The
plaintiff was, therefore, a stockholder during all the time for which he seeks The most important question to be decided is, did the lower court err in
recovery in his first cause of action, except the first six months of the year sustaining the demurrer to the second cause of action? If this question be
1903. Then again, as a matter of fact (which we do not now decide), if the decided in the negative, then it will not be necessary to determine whether or
defendants had taken their salaries for the year 1903 at the close of that year not the allegations in this part of the complaint are sufficient to hold the
or at any time after September 13, the plaintiff would then have had an defendants liable for the acts of their predecessors.
interest and, on the theory that he was a stockholder, could have questioned
the legality of the defendants' right to take such salary, inasmuch as his It affirmatively appears from the complaint that the plaintiff was not a
dividends would be directly affected, in that, if the defendants took 10 per stockholder during any of the time in question in this second cause of action.
cent of the gross instead of the net earnings of the bank, his dividend on his Upon the question whether or not a stockholder can maintain a suit of this
ten shares for the second period (from July to December, 1903) would be character upon a cause of action pertaining to the corporation when it
less. appears that he was not a stockholder at the time of the occurrence of the
acts complained of and upon which the action is based, the authorities do not
Conceding that this cause of action is demurrable on the grounds that the agree.
plaintiff was not a stockholder during the first six moths of the year 1903,
should the demurrer have been sustained as to the whole cause of action In the case of Hawes vs. Oakland (14 Otto [104 U.S.], 450, 456), the plaintiff,
when the time for which recovery is sought is clearly divisible? a shareholder in the Contra Costa Waterworks Company, brought a bill in
equity against the company and the city of Oakland in the Circuit Court of the
Section 90 of the Code of Civil Procedure in force in the Philippine Islands United States for California, on the ground that he was a citizen of New York
provides, in part, as follows: and the defendant citizens of California, alleging that the company was
furnishing the city of Oakland with water free charge beyond what the law
2. ... If the complaint contains more than one cause of action, each required it to do, and that, although he had required them to desist, the
distinct cause of action must be set forth in a separate paragraph directors had failed to heed his protest and that unless enjoined they would
containing all the facts constituting the particular cause of action. continue to furnish water to the city in excess of their legal obligations in this
particular, to the damage of plaintiff and the shareholders.
Where the matter in a single count is divisible in its nature, the
demurrer should be confined to those parts which are defective, as To this complaint the city of Oakland demurred upon the ground that the
the same general rule which applies to different counts applies also appellant had shown no capacity in himself to maintain the suit, the injury, if
to divisible matter in the same count constituting different causes of any, being to the corporation and the right to sue pertaining to it solely. The
action; and where one count, containing distinct averments, demurrer was sustained and the bill dismissed, whereupon the plaintiff
discloses a good cause of action in one of such averments, as when carried the case to the Supreme Court of the United States.
several breaches are assigned, some well and others ill, a general
demurrer will be overruled. (6 Ency. Plead. & Prac., 303, 304.)

910
The decision of the court, which was written by Mr. Justice Miller and Every bill brought by one or more stockholders in a corporation
concurred in by all the other justices, contains a review of the earlier against the corporation and other parties, founded on rights which
decisions of the English and American courts with respect to the right of may properly be asserted by the corporation, must be verified by
stockholders of corporations to maintain suits of this character. In concluding, oath, and must contain an allegation that the plaintiff was a
the court, after enumerating a number of circumstances in which a shareholder at the time of the transaction of which he complains, or
stockholder might be permitted to sue upon a cause of action pertaining to that his shares had devolved on him since by operation of law, and
the corporation, said: that the suit is not a collusive one to confer on a court of the United
States jurisdiction of a case of which it would not otherwise have
But in addition to the existence of grievances which call for this kind of relief, cognizance. It must also set forth with particularity the efforts of the
it is equally important that before the shareholder is permitted, in his own plaintiff to secure such action as he desires on the part of the
name to institute and conduct a litigation which usually belongs to the managing directors or trustees, and, if necessary, of the
corporation, he should show to the satisfaction of the court that he has shareholders, and the causes of his failure to obtain such action.
exhausted all the means within his reach to attain within the corporation
itself, the redress of his grievances, or action in conformity to his wishes. He January 21, 1884, the Supreme Court decided the case of Dimpfel vs. Ohio,
must make an earnest, not a simulated effort, with the managing body of the etc., R.R. Co. (110 U.S., 212; 28 Law Ed., 121, 122), which was similar to the
corporation, to induce remedial action on their part, and this must be made Hawes case, above cited. Mr. Justice Field, by whom the opinion of the court
apparent to the court. If the time permits, or has permitted, he must show, if was written, says (p. 122):
he fails with the directors, that he has made an honest effort to obtain action
by the stockholders as a body, in the matter of which he complains. And he The suit was brought to set aside a contract by which the Ohio and
must show a case, if this is not done, where it could not be done, or it was Mississippi Railway Company became the owner of a portion of its
not reasonable to require it. road known as the Springfield Division, and to obtain a decree from
the court declaring that the bonds, issued by the company and
The effort to induce such action as plaintiff desires on the part of the secured by a mortgage upon that division, are null and void. It was
directors, or of the stockholders when that is necessary, and the commenced by Dimpfel, and individual stockholder in the company,
cause of failure in these efforts, and all allegation that plaintiff was a who stated in his bill, that it was filed on behalf of himself and such
shareholder at the time of the transactions of which he complains, or other stockholders as might join him in the suit. Callaghan, another
that the shares have devolved on him since by operation of law and stockholder, is the only one who joined him. The two claim to be
that the suit is not a collusive one to confer on a court otherwise owners of fifteen hundred shares of the stock of the company. The
have no cognizance, should be in the bill, which should be verified by whole number of shares is 240,000. The owners of the balance of
affidavit. this large number make no complaint of the transactions which the
complainants seek to annul. And it does not appear that the
This case was decided January 16, 1882. More than a year afterward the complainant owned their shares when these transactions took place.
Supreme Court embodied the procedural part of this decision in the 94th For aught we can see to the contrary, they may have purchased the
Equity Rule, adopted January 23, 1883. The rule reads as follows: shares long afterwards, expressly to annoy and vex the company, in
the hope that they might thereby extort, from its fears, a larger
benefit than the other stockholders have received or may reasonably
expect from the purchase, or compel the company to buy their

911
shares at prices above the market value. Unfortunately, litigation collusion between the parties, which should property be tried in the State
against large companies is often instituted by individual stockholders court. It is equally true that the court was desirous of preventing a
from no higher motive. continuance of these fraudulent practices, by establishing a test which should
prevent them. The basis of the right to sue in the Federal courts being
The bill in this case was also open to the objection that the plaintiff had not diversity of citizenship, the usual method employed to enable parties to suits
exhausted the means of redress available within the corporation. The next of this kind to invoke the jurisdiction of these courts was to have a few shares
proceeds to consider this point, but prefaces its remarks with the following of stock transferred to some person who was a citizen of a State other than
significant phrase: that of which the proposed defendants were citizens. In a case of this kind
the transfer of the stock would be, of necessity, merely nominal, and the
But assuming that the complainants were the owners of the shares plaintiff, under such circumstances, would not be a bona fide stockholder,
held by them when the transactions of which they complain took and would not be entitled to maintain the suit. Of necessity, in cases of this
place, it does not appear that they made any attempt, etc. kind, of genuine collusion to create a fictitious diversity of citizenship the
nominal transfer of the stock is made at a date subsequent to that of the
occurrence of the acts or omissions complained of. Although the court was
Counsel for the plaintiff in a very able and exhaustive brief sought to show
lawfully entitled to protect itself against such frauds as those of which it
that the doctrine laid down in these two cases is not applicable to the case at
complains in this case, and to refuse to take cognizance of cases in which,
bar, first, because the Supreme Court in these cases merely established a
owing to the purely fictitious nature of the simulated diversity of citizenship,
rule of practice, designed to prevent collusive suits in the Federal court; and,
the proper tribunals were the State courts, on the other hand, in cases of
second, that if such rule is to be regarded as a declaration of substantive law,
genuine diversity of citizenship, it could not lawfully refuse to exercise the
it is wrong on principle and should be disregarded. Many of the authorities
jurisdiction vested in it. No citation of authority is needed to support the
cited by the plaintiff to the effect that the rule is merely one of practice,
proposition that it is the duty of courts to exercise the jurisdiction properly
peculiar to the Federal court, base that conclusion upon the fact that the
conferred upon them. It is elementary that where there is a higher tribunal
requirement of the inclusion of the averments in question in the bill to be
authorized to issue the writ, mandamus will lie to put the judicial machinery in
found in the 94th Equity Rule. Some of the authorities cited, which hold this
motion. (Spelling, Extraordinary Relief, sec. 1394.) This being the case, the
view, are: Pomeroy, Eq. Jur., sec. 1096; Thompson, Corporations, sec. 4570;
conclusion is obvious that the mere fact that in some cases persons suing as
Cook, Corporations vol. 3, secs. 736, 737; Morawetz, Corporations, sec. 209;
stockholders for the redress of grievances anterior to the transfer of the stock
Forrester vs. Mining Co., 55 Pac. Rep., 229.
held by the plaintiff are not acting in good faith would not justify or authorize a
refusal to take jurisdiction in any case in which the plaintiff's stock was
In the first place the doctrine was announced in Hawes vs. Oakland, supra, acquired after the occurrence of the facts supposed to constitute the cause of
more than a year before the 94th Equity Rule was promulgated, so that it can action, unless the court were of the opinion, as a matter of substantive law,
admit of no dispute that in the opinion of the Supreme Court, as least, the that in no event would a stockholder so situated be entitled to maintain such
ownership of stock at the time of the transaction complained of was essential an action.
to the right to maintain such an action as a matter of substantive law, prior to
and independent of the Equity Rule.
It is only upon this assumption that the correctness of the decision in
Hawes vs. Oakland and the legality of the 94th Equity Rule can be
It is true that the court in writing the decision in the Hawes case, had in mind maintained. The court had no authority to change the substantive law either
the prevalence of the practice of bringing suits in the Federal courts, by by its decision or the rule, and it is not to be presumed that it intended to do

912
so. A careful examination of the Hawes case and of the rule will show that no any circumstances, sue upon a cause of action accruing to the corporation
such change was in fact made. The decision is merely declaratory of the prior to such transfer, the rule instead of requiring the plaintiff to allege
preexisting law, as the court understood it to be, and the rule merely provides unconditionally that he was a stockholder at the time of the transaction
a rule of pleading. complained of and that the suit is not collusive, would have provided that the
plaintiff should be required to aver in his sworn bill the date upon which he
The decision in the Hawes case it that among other necessary averments, acquired his stock, and if it appeared that it was acquired after the
the bill should contain "an allegation that the plaintiff was a shareholder at the occurrence of the acts complained of, then that he should also required to
time of the transaction of which he complains ... and that the suit is not a aver under oath that the suit was not collusive.
collusive one to confer jurisdiction on a court of the United States in a case in
which it would otherwise have no cognizance ... ." The language of the 94th Sound reason and good authority sustain the rule that a purchaser of
Equity Rule is practically identical with this. It provides, in terms that a stock can not complain of the prior acts and management of the
stockholder's bill in cases of this character "must contain an allegation that corporation. (Home Fire Ins. Co. vs. Baker, 60 L.R.A., 927, 933,
the plaintiff was a stockholder at the time of the transaction of which he citing Hawes vs. Oakland, supra; Dimpfel vs. Ohio & M.R.
complains ... and that the suit is not a collusive one to confer Co., supra; Taylor vs. Fayette Fuel Gas Co., 146 Pa., 13;
jurisdiction . . . ." Alexander vs. Searcy, 81 Ga., 536; Clark vs. American Coal Co., 86
Iowa, 436; United Electric Securities Co., vs. Louisiana Electric Light
This is, obviously, a mere rule of pleading it requires averments of facts Co., 68 Fed., 673; Venner vs. Atchison T. & S.F.R. Co., 28 Fed., 581;
upon which the plaintiff's cause of action and the jurisdiction of the court rest. Heath vs. Erie R. Co., 8 Blachf., 347; Dannmeyer vs. Coleman, 8
It assumes, as the court had already decided, that the ownership of the stock Sawy., 51; Works vs. Sowers, 2 Walk (Pa.), 416; 4 Thompson Corp.,
at the time of the transaction is a fact essential to the maintenance of the suit 4569.)
in any event. Unless that fact exists no cause of action exists, whether the
suit is collusive or not. Even if the stock was owned prior to the transaction In Alexander vs. Searcy, supra, the court said (p. 550):
complained of, if the suit is collusive as it would be, for instance if one of
the defendants had acquired a merely colorable domicile in another State to The weight of authority seems to be that a person who did not own
support the allegation of diversity of citizenship the plaintiff has no right to stock at the time of the transactions complained of can not complain
maintain the action in a Federal court. Consequently, the rule requires that or bring a suit to have them declared illegal.
these two facts be distinctly averred. The requirement that they be pleaded is
procedural. The necessity of the existence of the facts in order to give rise to In the United Electric Securities Co. vs. Louisiana Electric Light Co., supra, it
the right of action is substantive. is said:

If the Supreme Court had been of the opinion, as are some of the State As a general proposition, the purchaser of stock in a corporation is
courts and text writers cited in plaintiff's brief, that the transferee of shares of not allowed to attack the acts and management of the company prior
stock in a corporation acquires the right to sue upon the causes of action to the acquisition of his stock; otherwise we might have a case where
which accrued before he acquired such shares, it surely would not have stock duly represented in a corporation consented to and
attempted to deprive him of the right to exercise in the Federal court an participated in bad management and waste, and after reaping the
action which, were it not for diversity of citizenship, he might exercise in a benefits from such transaction, could be easily passed into the hands
State court. If the court had believed that the transferee of stock could, under

913
of a subsequent purchaser, who could make his harvest by If a stockholder participates in a wrongful or fraudulent contract, or
appearing and contesting the very acts and conducts which his silently acquiesces until the contract becomes executed, he can not
vendor had consented to. then come into a court of equity to cancel the contract, and more
especially if the company, or himself, as a stockholder, has reaped a
Where stock is required for the purpose of bringing suit it has been held that benefit from the contract; and this rule holds good, although the
the complainant is a mere interloper and entitled to no consideration. And consideration of the contract may be one expressly prohibited by
stockholder suits not brought in good faith in the interest of the corporation statute. The same disability would attach to the transferee of his
have been dismissed on the ground. (Home Fire Ins. Co. vs. Baker, supra, stock who bought with notice.
and cases cited therein.) Some of the State courts hold that a purchaser of
shares in a corporation acquires all the rights of the vendor. The Alabama This rule, in the main, is correctly stated, but we think that the latter part of
Supreme Court has gone so far as to hold that a purchaser in good faith is the same should be modified so as to read: "The same disability would attach
not necessarily disqualified as a suitor in all cases because the prior holder to the transferee of his stock who bought with or without notice." We base our
was personally disqualified. (Parsons vs. Joseph, 92 Ala., 403.) From the modification of this rule upon the ground that a transferee could not sue as
pleadings in this case (it having been decided by the Supreme Court upon a being a bona fide purchaser in ignorance of the disability attaching to his
demurrer) it appears that Joseph sought to have cancelled certain certificates vendor, because shares of stock, strictly speaking, are not negotiable, and
of stock issued by the Street Railway Company to Parsons, on the ground the sale can not pass greater rights than those possessed by the vendor.
that said stock was fictitious and was issued in violation of the constitution (Clark vs. American Coal Co., 86 Iowa, 436; 4 Thomp. Corp., 3410.)
and statute law of the State. It was alleged, as a special defense, that if the
transactions, which form the basis of the issuance of the stock to Parsons, It is self-evident that the plaintiff in the case at bar was not, before he
were illegal, and fraudulent, and not done in good faith, the complainant, acquired in September, 1903, the shares which he now owns, injured or
Joseph, was estopped from setting up fraud in such transactions or, seeking affected in any manner by the transactions set forth in the second cause of
to cancel the stock, because one E. Lesser, who was complainant's action. His vendor could have complained of these transactions, but he did
transferrer, participated in all of said transactions. In this case the court, not choose to do so. The discretion whether to sue to set them aside, or to
speaking through Mr. Justice Coleman, said: acquiesce in and agree to them, is, in our opinion, incapable of transfer. If the
plaintiff himself had been injured by the acts of defendants' predecessors that
If the transferee purchased the shares in good faith, and without is another matter. He ought to take things as he found them when he
notice of the fact that the prior holder had precluded himself from voluntarily acquired his ten shares. If he was defrauded in the purchase of
suing, he would have as just a title to relief as if he had purchased these shares he should sue his vendor.
from a shareholder who was under no disability; but if the purchaser
was aware that the prior holder had barred his right to relief, neither If the party himself, who is the victim of fraud or usury, chooses to
justice nor public policy would require that the transferee, under waive his remedy and release the party, it does not belong to a
these circumstances, should be accorded any greater rights than his subsequent purchaser under him to recall and assume the remedy
transferrer. for him. (Quoted with approval in the case of the Graham vs. La
Crosse and Milwaukee R.R. Co., 102., U.S., 148.)
xxx xxx xxx

914
But it is contended that this is a case in which the debtor corporation Arellano, C.J., Torres, Mapa, and Johnson, JJ., concur.
was defrauded of its property, and that as the company had a right of
proceeding for its recovery, any of its judgment and execution
creditors have an equal right; that it is a property right, and one that
inures to the benefit of creditors.

Conceding that creditors who were such when the fraudulent


procurement of the debtor's property occurred and cases to that
effect have been cited the question still remains, whether, the
debtor being unwilling to disturb the transaction, subsequent
creditors have such an interest that they can reach the property for
the satisfaction of their debts. We doubt whether any case, going as
far as this, can be found. No such case has been cited in the
argument. Dicta of judges to that effect may undoubtedly be
produced, but they are not supported by the facts of the cases under
consideration.

It seems clear that subsequent creditors have no better right than


subsequent purchasers, to question a previous transaction in which
the debtor's property was obtained from him by fraud, which he has
acquiesced in, and which he has manifested no desire to disturb.
Yet, in such a case, subsequent purchasers have no such right. (Id.)

So it seems to be settled by the Supreme Court of the United States, as a


matter of substantive law, that a stockholder in a corporation who was not
such at the time of the transactions complained of, or whose shares had not
devolved upon him since by operation of law, can not maintain suits of this
character, unless such transactions continue and are injurious to the
stockholder, or affect him especially and specifically in some other way.

We are, therefore of the opinion, and so hold, that the judgment appealed
from, sustaining the demurrer to the first cause of action should be, and the
same is hereby reversed; and the judgment sustaining the demurrer to the
second cause of action should be, and is hereby affirmed, without any
special ruling as to costs. The record will be returned to the court whence it
came for further proceedings in accordance with this decision. So ordered.

915
916
3. In holding that it was proper for defendants to compute their
compensation upon the gross profits before charging against such
gross profits the aggregate amount of accounts written off as
uncollectible (dudosa y fallida) as shown in Exhibits C-1 to C-9,
inclusive.

4. In holding that any of the debit items appearing in Exhibits C-1 to


C-9 and especially the industrial and internal-revenue taxes, are
items that should be charged against capital and not against current
profits.1awphil.net

5. In holding that it was within the power of the stockholders of the


bank to ratify the so-called interpretation by defendants of said article
G.R. No. L-7945 December 1, 1914 30.

CANDIDO PASCUAL, plaintiff-appellant, This action was commenced by the plaintiff as a shareholder of the Banco
vs. Espaol-Filipino for the benefit of the bank and all of the stockholders
EUGENIO DEL SAZ OROZCO, ET AL., defendants-appellees. thereof. Its purpose is to require the defendants as former directors and
councilors of the bank to refund a portion of the compensation paid to them
C. W. Ney and O'Brien & De Witt for appellant. for their services, on the ground that the amounts thereof have been
Hausermann, Cohn & Fisher for appellees. wrongfully computed.

The complaint contains three separate causes of action, of which the first
only is here involved. The defendants' demurrer to this cause of action was
TRENT, J.: sustained upon the ground that the facts alleged therein were not sufficient to
entitle the plaintiff to the relief sought. Upon appeal this judgment was
The plaintiff appeals from a judgment upon the merits in favor of the reversed and the record returned for further proceedings. (19 Phil. Rep., 82.)
defendants, and insists that the court erred: The complaint was not thereafter amended.

1. In holding that the interpretation placed upon article 30 of the The question raised by the plaintiff in his first assignment of error requires an
bank's charter by the decision of the Supreme Court herein is not the examination of the pertinent allegations in the first cause of action. These
law of the case. allegations are as follows.

2. In holding that the defendants had a right to deduct their X. That, notwithstanding the fact that article 30 of the said by-laws
compensation from the gross profits of the bank. (Exhibit B) clearly and unequivocally prescribes that the net profits of
the said bank shall be apportioned as follows: Then per cent for the

917
board of directors, five per cent for the board of managers ten per cent remaining shall belong integrally to the shareholders pro data
(composed of counselors and trustees) in compensation for their the number of shares owned by each."
services as such, and the remainder, eighty-five per
cent, integrally for the shareholders of the said bank, the defendants, Since the date on which the plaintiff acquired his shares, the earnings of
as such members of the said boards of directors and managers, each half-year of the bank have been liquidated in the manner set forth in the
respectively, did, during each and all of the years specified, Exhibits C-1 to C-9, inclusive, attached to the agreed statement of facts, and
fraudulently and to the great detriment of the said Bank and its the respective defendants have individually collected for their services the
shareholders, and without the knowledge, consent or acquiescence sums specified in Exhibit D.
of the latter, appropriate to themselves for their own use from the
profits of the said Bank sums of money reaching an approximate Under date of November 15, 2907, the plaintiff addressed to the defendants
amount of twenty thousand pesos, or a total sum of one hundred a letter alleging that the earnings of the bank had not been apportioned in
thousand pesos during the five years aforementioned, by deducting accordance with the provisions of article 30, supra, and making demand
their said ten and five per cent, respectively, from the gross profits upon them for the refund to the bank of a portion of the amounts received by
instead of deducting them from the net profits of the said bank. them in compensation for their services. The defendants refused to comply
with this demand and on December 7, 1907, the plaintiff commenced an
XI. That the said defendants, during the time mentioned, carefully action seeking the same relief herein prayed for. This action was dismissed,
concealed in all the balances and reports of the said Bank published and on December 21, 1907, the shareholders of the bank were convened in
by them every indication that might gave the stockholders of the said a special meeting "for the express purpose of discussing and taking action
Bank the slightest suspicion that the said defendants were relative to the alleged interpretation of article 30 of the by-laws." At this
fraudulently appropriating to themselves the funds of the same; and shareholders' meeting there were present, either in person or by proxy, 183
that the plaintiff learned of such appropriation, by a mere chance, in persons and entities, holding 6,499 shares of the total issue of 7,500. Among
the month of November, 1907. those present at this meeting was plaintiff's attorney. The plaintiff's letter,
referred to above, was read, as was the complaint which the plaintiff had
The Banco Espaol-Filipino was a banking corporation which, until January previously filed, and, after a discussion in which the appellant's attorney took
1, 1908, was controlled by the by-laws and regulations annexed to the part, a resolution was adopted ratifying and approving the distribution of the
complaint as Exhibits A and B. On November 13, 1903, the plaintiff acquired bank's earnings as made, and authorizing the defendants to proceed in the
10 shares of the capital stock and has been the registered holder of these same manner with the earnings of the latter half of the year 1907. In favor of
shares since that date. The defendants filled, during the time mentioned in this resolution there was a total of 555 votes, representing 5,550 shares.
the complaint, the offices of director, consiliario, and sindico, and collectively Soon thereafter the present action was commenced.
constituted the board of government. The only compensation to which the
defendants were entitled for their services is that prescribed by article 30 of As will be seen from the plaintiff's first assignment of error and the argument
the by-laws then in force. of counsel relating thereto, it is strongly urged that inquiry respecting the
interpretation and application of article 30, supra, has been closed by the
This article reads: "Of the profits or gains which may result from the bank's decision of this court rendered upon the demurrer of the defendants to the
operations, after deducting all the expenses of its administration and the part, complaint. Under the doctrine of stare decisis the plaintiff insists that "the law
if any, which corresponds to the legal reserve fund, there shall be set apart of the case" has been established and that it has been necessarily decided

918
that the remuneration received by the defendants for their services was not in the defendants may not fraudulently compute their percentages upon the
accordance with article 30. gross earnings, and that a complaint which alleges that they have done so
states a cause of action. This was question submitted and decided. The
The decision is relied upon by the plaintiff is that of Pascual vs. Del Saz question submitted upon the present appeal is whether the computation
Orozco (19 Phil. Rep., 82). "The law of the case," established by that really made is in accordance with article 30. This holding is not in conflict with
decision, is the law of the case which was before the court and which the the rule announced in the cases cited and relied upon by counsel for the
court thereby decided.1awphil.net plaintiff.

The plaintiff, as will be seen from paragraphs 10 and 11, above quoted, For example, in the case of Heidt vs. Minor (113 Cal., 385), the court said:
whose sufficiency was then and there under consideration, alleged that the "Moreover, the rule of the law of the case only applies when, on subsequent
defendants, in violation of article 30, had fraudulently misappropriated to trial, the issues and the facts found remain substantially the same."itc@alf
themselves certain funds of the bank by computing their percentages upon
the gross earnings of the bank and, by a series of fraudulent concealment's, In the case of Foregerson et al. vs. Smith (104 Ind., 246), the court laid down
had withheld the knowledge thereof from the shareholders. The demurrer this rule: "But where the questions are necessarily involved, . . . the judgment
admitted the facts as alleged and raised the question of the right of the on appeal rules the case throughout all its subsequent stages. The decision
plaintiff to recover upon those facts. The ruling of the lower court was to the is an adjudication concluding the courts and the parties. It is not, of course,
effect that, even assuming the facts to be as alleged in the complaint, the conclusive in judgment in the case in which it was rendered, upon the parties
plaintiff had no right of action. On appeal the Supreme Court considered this and those in privity with them. . . . We regard the former decision as
very question and necessarily none other, which relates to the point now adjudicating all of the controlling questions in the case, for it was not possible
under consideration, and, in reversing the ruling of the lower court, decided to reach the conclusion there announced without deciding that the property in
that, assuming the facts to be as alleged in the complaint, the plaintiff did the promissory notes in controversy was in the administrator of the estate of
have a cause of action. If these were the facts of the case now under Mahala Shaw deceased."
consideration, there would be neither occasion nor opportunity to further
discuss the law applicable thereto. But the case which the present appeal In Standard Sewing Machine Co. vs. Leslie (118 Fed., 557), the court used
presents is not the case at all. Since that decision was rendered the case has this language: "It is a familiar and entirely righteous rule that a court of review
been tried and the facts now before the court for consideration are not the is precluded from agitating the questions that were made, considered, and
allegations that the defendants fraudulently mis-appropriated to themselves decided on previous reviews. The former decision furnishes 'the law of the
certain funds of the bank, and by a series of concealment's withheld the case' not only to the tribunal to which the cause is remanded, but to the
knowledge thereof from the shareholders, but the real facts as they have appellate tribunal itself on a subsequent writ or appeal."
been stipulated in the agreed statement. These facts are that the defendants
did not, as alleged, fraudulently misappropriate certain funds of the bank by Now, has the remuneration of the defendants for their services been
computing their percentages upon the gross earnings, but the first deduct the computed in accordance with article 30 of the by-laws?
expenses of administration, and that none of the acts of the defendants were
tainted in any way with fraud.
The item of "profit and loss" for each half year, during the entire period
covered by the complaint, was made up by crediting to it all the items of net
In this particular the case now under consideration is clearly differentiated profits produced by the various accounts of the bank, including the accounts
and distinguished from the former case. In the case the court decided that

919
of current debtors, the profits from exchange, the profits in the sale of money, To this method of computing the defendants' remuneration the objection of
the profits from the discount of bills and notes, the net proceeds from the real the plaintiff is twofold:
properties of the bank after the payment of all the expense thereof, including
taxes, insurance, and repairs, and all other net profits obtained by the bank. (a) That before computing the defendants' remuneration there was
To the debit of this "profit and loss" account were entered all sums paid out not first deducted from the earnings or gains the amount payable as
by the bank as interest upon fixed deposits or credit balances of current industrial tax (later internal revenue), and
accounts. The items of "general expenses" included salaries, light , water,
stationery, stamps, attorneys' fees, and all other items of general expenses (b) That before computing the defendant's remuneration there was
incurred either by the main office in Manila or by the branch office in Iloilo. In not first deducted from the earnings or gains the amounts retained to
short, it appears that every expenditure of whatever nature made from the cover bad accounts.
funds of the bank, with the exception of the industrial tax (later internal-
revenue tax) and amounts set off against bad accounts, was included in the
From an examination of article 30 it will be seen that only two items from the
item of "general expenses," or, what amounted to the same thing, deducted
gross profits of the bank are to be deducted before computing the
from the "profit and loss" account before computing the remuneration
compensation of the directors and board of government (the defendants
received by the respective defendants for their services. Upon this point it
constituted both the directors and the board of government), to wit: Expenses
might be well to set out in full Exhibit C-1. (Exhibits C-2 to
of administration and the amount, if any, corresponding to the legal reserve
C-9, inclusive, were made up in the same manner.) This exhibit is as follows:
fund. On December 31, 1903, the legal reserve fund of P225,000 was not
only completed, but a voluntary reserve fund of P665,000, authorized by the
1903. charter, had accumulated. From the various Exhibits, C-1 to C-9, inclusive, it
is apparent that nothing whatever was applied to this reserve fund, and, as
December 31. Balance of the account of profit and the correctness of these exhibits is not disputed, it is also apparent that
loss ............................. $196,580.22 Deduction of surplus of June 30, nothing was due this fund at any time during the period covered by the
last ..................................... 7,816.38 complaint. Unless, therefore, the items of industrial tax (later internal-revenue
188,763.84 Do. General tax) and the amounts set aside to cover bad debts that there is no merit in
expenses ............................................................... 51,753.77 the plaintiff's contention.
137,010.07 Compensation for the board of government,
15 At the outset it may be said that the proper disposition of this case is
per cent ....................................................................................... rendered difficult by the inaccurate language used in article 30. This article
20,551.51 116,458.56 Dividend of 4 per cent on provides for a percentage of the profits (utilidades y ganacias), and it may be
$1,500,000 .................................... 60,000.00 56,458.56 at once said that these are not necessarily net profits, as claimed by counsel
Industrial tax (later internal revenue) 5 per cent of for the plaintiff. Profits may be either gross profits or net profits, and there are
$60,000 ....................................................................................... innumerable methods of computing each of these. Likewise, "expenses of
3,000.00 53,458.56 Balance on June 30 carried administration" may or may not include all amounts expended in the conduct
forward ..................................... 7,816.38 61,274.94 of business. Indeed, it is somewhat unusual that a provision of the bank's
Amount for bad accounts .................................................... 60,000.00 charter, so difficult of exact definition, should be so lacking in precision.
Balance for next semester ................................. 1,274.94 Hardly less unusual, from an American point of view, is the incorporation into

920
the bank's charter of the measure of remuneration of the board of the sum of P58,000, how much is to be deducted therefrom as internal-
government. This, is America, has generally been considered a detail in the revenue tax before computing the percentage of the defendants? The law
internal management of a corporation to be controlled by the shareholders said that the amount of this tax should be 5 per cent of the dividends
themselves, who, in many instances, even delegate to the directors the distributed. The amount to be distributed depends upon how much may be
power of fixing their own salaries. left after the remuneraton of the defendants is paid. It is no reply to this
argument to point out that the total profits may be or are usually sufficiently
The remuneration received by the defendants is not even alleged to be great to permit a declaration of the maximum dividend of P60,000 and that in
excessive. The two active managers of the bank received, during the period such cases t is simple matter to compute 5 per cent of P60,000, for the rule
in question, sums amounting to approximately P15,000 per year, while the of computation, established by article 30, is a general one, applicable alike in
other defendants, not participating in the active management of the all cases, whether the earnings of the bank be great in small. This article
corporation, received sums amounting in no instance to a salary of P2,500 does not establish two rules of computations, one which is only feasible or
per year. All of the defendants received, during the four and a half years, practicable when the earnings are sufficiently large to warrant a dividend in
P201,825.81, or an approximate yearly average of P45,000 per year Bearing the maximum amount and another and different rule when the dividend falls
in mind the magnitude of the business and the fact that the bank prospered below that amount.
under the management of the defendants, there is no wonder that no claim is
made of excessive compensation. During all these years the plaintiff, as well Again, in our opinion the nature of the old industrial tax negatives the idea
as the other shareholders of the bank, remained silent, apparently content that it is one of the items of "expenses of administration" referred to in article
with the increased prosperity of the business, although at the increased 30. This tax was levied by law, not upon the earnings or profits of the bank,
prosperity of the business, although at the end of each fiscal year they had but only upon such earnings or profits as were actually distributed among the
the opportunity to examine the books of the bank and inform themselves of shareholders as dividends. It was purely a dividend tax, collected for
the method by which the defendants computed their compensation. And, convenience in a lump sum from the company, but levied solely and
furthermore, an extraordinary meeting of the shareholders was duly exclusively upon the distributed dividends. To deduct this tax from the
convened on December 21, 1907, for the express purpose, as we have amount upon which the remuneration of the defendants was computed would
indicated, of discussing the interpretation placed upon article 30 by the have made the defendants contributors to the tax levied upon the company-
defendants. shareholders. Article 30 does not require the defendants as employees of the
bank to contribute to the payment of the bank's taxes. The discrimination
The industrial tax, which the appellant insists should be first deducted from made by article 30 between "expenses of administration" and other
the earnings before computing the percentages, was fixed by law at 5 per disbursements is reasonable and in accordance with the principles of the
cent of the dividends distributed among the shareholders of the bank. In contract which existed between the bank and the defendants. That was a
order to make the deduction of this tax, its amount must first be a known contract of employment in which one of the contracting parties agreed to
quantity. Since its amount is a percentage of the dividends, the amount of the supply the capital and the other his services, and to divide in a stipulated
latter must likewise by a known quantity before the operation can be made. proportion the proceeds of the application of the services of the one to the
The amount available as dividends is dependent upon the amount due and capital of the other. Since it was incumbent upon the bank to furnish the
payable out of profits to the defendants for their services. Therefore, this capital, so it was incumbent upon it to maintain the same. Any tax which
amount due the defendants from the profits must be known before the tended directly to impair the amount of the capital should consequently have
amount remaining for dividends can be fixed. For example, if the remaining been paid by the hirer of the services and not by the servant. There could be
earnings, after deducting from the gross earnings the general expenses, is no real difference in principle between the failure to furnish the capital in the

921
first place and the failure to replace any part of it which disappears by reason the excess net profits, the application made was in direct accord with the by-
of a tax levied thereon. We, therefore, conclude that the method employed by laws, since the application of the funds to the purposes of the reserve fund is
the defendants for the liquidation of the bank's business, in so far as the exactly the same as if the reserve fund had been employed for the purpose
industrial tax (internal-revenue tax) is concerned, was strictly in accordance and then replenishment by these funds.
with article 30 of the by-laws.
According to article 30, the net profits belonged to the shareholders.
DEDUCTION OF AMOUNTS TO COVER BAD ACCOUNTS According to article 31, these not profits, belonging to the shareholders,
should be partially divided among them and partially kept intact in the bank,
When the defendant Orozco took over the management of the bank, he according to the amount thereof, to the status of the legal reserve, and to the
reported to the board of directors its financial situation, embracing among wishes of the board of government respecting a voluntary reserve. From the
other thins a loss from bad accounts for the past of over P500,000. It fact that part of either the legal or voluntary reserve, it cannot be said that
probably would have been possible to cover this entire amount of losses from such portion of the net profits had ceased to belong to the share-holders.
funds in the reserve, existing for just such purposes, but to have done so these excess net profits are, in a sense, still in the bank and still belong to
would have left the bank without a present reserve. It was decided to the shareholders within the meaning of article 30. to hold that the bad
preserve the reserve fund intact, and carry the bad accounts as accounts in accounts of the bank should have been extinguished by the gross earnings
suspense until the same could be gradually and conveniently wiped out. instead of the net profits, would, in effect, compel the defendants, as
Consequently, in each half yearly liquidation the dividends distributed to the employees, to contribute to the replenishing of the depleted reserves of the
shareholders were strictly limited to 4 per cent per semester, and the bank.
earnings after payment of the expenses of administration, the remuneration
of defendants, the taxes, and the said dividends were applied pro tanto to the It would be wholly unjust to include under "expenses of administration" during
extinction of the ad accounts held in suspense. It does not clearly appear the time the defendants were in charge, the losses previously sustained by
whether these funds, which were used for that purpose, first went into the the defendants' predecessors in office. These defendants were in no wise
voluntary reserve fund and were then applied to the extinction of the bad connected with the bank no were they in any way responsible for those
accounts in suspense or were applied directly by the semiannual liquidation's losses. To interpret article 30 so would result in the incoming manager
from the profit and loss accounts. The process followed is immaterial since becoming an heir to an insolvent inheritance. Under such conditions no one
the result must be the same. The important fact is that in each semester would be found to accept the office and the bank would have to cease its
there was an excess of net profits over and above the 4 per cent provided in operations.
article 31 of the by-laws. This excess of net profits was divisible under that
article, one-half to the shareholders and one-half to the legal reserve, As to the responsibility of the defendants for the losses which occurred
voluntary reserve, or additional dividends as the case might be. Instead, the during the period covered by the complaint, it might be said in the first place
entire excess of net profits went to extinguish bad accounts whose extinction that the greater part of these losses constitutes the third cause of action of
would have exhausted the reserve fund and required its replenishment. To appellant's complaint and was made the subject of a separate appeal to this
the extent that one-half of the excess of net profits were not distributed as court. In the second place, it has not been shown that any part of such losses
dividends, but were put to the purposes of reserve, the shareholders made a were written off as bad debts during the period of time in question. And it is
sacrifice for their own welfare. Whether this was validly done or not is of no upon this fact that we rest our holding on this point. Therefore, we are not
importance of this time for the reason that the remuneration of the now called upon to decide whether the defendants could have treated these
defendants was not affected in any way thereby. As to the remaining half of

922
losses in the same manner as they did those occurring prior to December 31, to render an account of his administration of the corporate affairs and assets:
1905. (2) to pay plaintiffs the value of t heir respective participation in said assets
on the basis of the value of the stocks held by each of them; and (3) to pay
The judgment appealed from is affirmed. 1 In this opinion it has been our the costs of suit. Plaintiffs also ask for such other remedy as may be and
intention to set forth at some length our reasons for affirming this judgment at equitable.
the close of the last session.
The complaint does not give plaintiffs' residence, but, but purposes of venue,
Arellano, C.J., Carson and Araullo, JJ., concur. alleges that defendant resides at 2112 Dewey Boulevard, corner Libertad
Street, Pasay, province of Rizal. Having been served with summons at that
place, defendant filed a motion for the dismissal of the complaint on the
ground of improper venue and also on the ground that the complaint did not
state a cause of action in favor of plaintiffs.
G.R. No. L-1721 May 19, 1950

In support of the objection to the venue, the motion, which is under oath,
JUAN D. EVANGELISTA ET AL., plaintiffs-appellants,
states that defendant is a resident of Iloilo City and not of Pasay, and at the
vs.
hearing of the motion defendant also presented further affidavit to the effect
RAFAEL SANTOS, defendant-appellee.
that while he has a house in Pasay, where members of his family who are
studying in Manila live and where he himself is sojourning for the purpose of
Antonio Gonzales for appellants. attending to his interests in Manila, yet he has permanent residence in the
Benjamin H. Tirol for appellee. City of Iloilo where he is registered as a voter for election purposes and has
been paying his residence certificate. Plaintiffs opposed the motion for
REYES, J.: dismissal but presented no counter proof and merely called attention to the
Sheriff's return showing service of summons on defendant personally at his
This is an action by the minority stockholders of a corporation against its alleged residence at No. 2112 Dewey Boulevard, Pasay.
principal officer for damages resulting from his mismanagement of its affairs
and misuse of its assets. After hearing, the lower court rendered its order, granting the motion for
dismissal upon the two grounds alleged by defendant, and reconsideration of
The complaint alleges that plaintiffs are minority stockholders of the Vitali this order having been denied, plaintiffs have appealed to this Court.
Lumber Company, Inc., a Philippine corporation organized for the exploitation
of a lumber concession in Zamboanga, Philippines; that defendant holds The appeal presents two questions. The first refers to venue and the second,
more than 50 per cent of the stocks of said corporation and also is and to the right of the plaintiffs to bring this action for their benefit.
always has been the president, manager, and treasurer thereof; and that
defendant, in such triple capacity, through fault, neglect, and abandonment As to the first question, it is important to remember that the laying of the
allowed its lumber concession to lapse and its properties and assets, among venue of an action is not left to plaintiff's caprice. The matter is regulated by
them machineries, buildings, warehouses, trucks, etc., to disappear, thus the Rules of Court. And in actions like the present, which is one in personam,
causing the complete ruin of the corporation and total depreciation of its
stocks. The complaint therefore prays for judgment requiring defendant: (1)

923
the regulation applicable is that contained in section 1 of Rule 5, which Section 377 provides that actions of this character "may be brought
provides: in any province where the defendants or any necessary party
defendant may reside or be found, or in any province where the
Civil actions in Courts of First Instance may be commenced and tried plaintiff or one of the plaintiffs resides, at the election of the plaintiff."
where the defendant or any of the defendant resides or may be The plaintiff in this action has no residence in the Philippine Islands.
found, or where the plaintiff or any of the plaintiffs resides, at the Only one of the parties to the action resides here. There can be,
election of the plaintiff. therefore, no election by plaintiff as to the trial. It must be in the
province where the defendant resides. The defendant resides, in the
Objection to improper venue may be interposed at any time prior to the trial. eye of the law, in Baguio. Was it "found" in the city of Manila under
(Moran's Comments on the Rules of Court, Vol. I, 2nd ed., p. 108.) section 377, its president being in that city where the service of
summons was made? We think not. The word "found" as used
section 377 has a different meaning that belongs to it as used in
Believing that defendant resided in the province of Rizal, herein plaintiffs
section 394, which refers exclusively to the place where the
brought their action in the Court of First Instance of that province. But that
summons may be served. As we have said a summons may be
belief proved erroneous, for the lower court found after hearing that
legally served on a defendant wherever he may be "found," i. e.,
defendant had his residence in Iloilo. The finding is based on defendant's
wherever he may be, provided he be in the Philippine Islands; but
sworn statement not rebutted by any proof to the contrary.
the venue cannot be laid wherever the defendant may be "found."
There is an element entering in section 377 which is not present in
There is nothing to the contention that defendant's motion to dismiss section 394, that is a residence. Residence of the plaintiff or
necessarily presupposes a hypothetical admission of the allegations of the defendant does not affect the place where a summons may be
complaint, among them the averment that defendant is a resident of Rizal served; but residence is the vital thing when we deal with venue. The
province, for the motion precisely denies that averment and alleges that his venue must be laid in the province where one of the parties resides.
real residence is in Iloilo City. This, defendant had the right to do in objecting If the plaintiff is a nonresident the venue must laid in the province of
to the court's jurisdiction on the ground of improper venue. the defendant's residence. The venue can be laid in the province
where defendant is "found" only when defendant has no residence in
Section 1 of Rule 5 may seem, at first blush, to authorize the laying of the the Philippine Islands. A defendant can not have a residence in one
venue in the province where the defendant "may be found." But this phrase province and be "found" in another. As long as he has a residence in
has already been held to have a limited application. It is the same phrase the Philippine Islands he can be "found," for the purposes of section
used in section 377 of Act 190 from which section 1 of Rule 5 was taken, and 377, only in the province of his residence. In such case the words
as construed by this Court it applies only to cases where defendant has no "residence" and "found" are synonymous. If he is a nonresident then
residence in the Philippine Islands. This was the construction adopted in the the venue may laid in the province where he is "found" at the time
case of Cohen vs. Benguet Commercial Co., Ltd., 34 Phil. 526, which was an venue the action is commenced or in the province of plaintiff's
action brought in Manila by a nonresident against a corporation which had its residence. This applies also to a domestic corporation.
residence for legal purposes in Baguio but whose President was found in
Manila and there served with summons. This Court there said: While the service of the summons was good in either Baguio or
Manila we are of the opinion that the objection of the defendant to

924
the place of trial was proper in both cases and that the trial court The construction which the moving party asks us to place on that
should have held that the venue was improperly laid. provision of section 377 above quoted would result in the destruction
of the privilege conferred by the section upon a resident defendant
And elaborating on the point when the case came up for reconsideration, the which requires the venue to be laid in the province where he resides.
Court further said: This is clear; for, if the venue may be laid in any province where the
defendant, although a resident of some other province, any be found
The moving party contends that the venue was properly laid under at the time process is served on him, then the provision that it shall
section 377 in that was laid in the province where the defendant was be laid in the province where he resides is no value to him. If a
found at the time summons was served on its president, he having defendant residing in the province of Rizal is helpless when the
been found and served with process in the city of Manila. for the venue is laid in the province of Mindoro in an action in which the
purpose of the discussion we assumed in the main case, as the plaintiff is a nonresident or resides in Manila, what is the value of a
plaintiff claimed, that the defendant was in fact and in law found in residence in Rizal? If a defendant residing in Jolo is without remedy
the city of Manila; and proceeded to decide the cause upon the when a nonresident plaintiff or a plaintiff residing in Jolo lays the
theory that, even if the defendant were found in the city of Manila, venue in Bontoc because the defendant happens to be found there,
that did not justify, under the facts of the case, the laying of the of what significance is a residence in Jolo? The phrases "where the
venue in the city of Manila. defendant ... may reside" and "or be found" must be construed
together and in such manner that both may be given effect. The
construction asked for by the moving party would deprive the phrase
We do not believe that the moving party's objection that our
"where the defendant ... may reside" of all significance, as the
construction deprives the word "found" of all significance and results,
plaintiff could always elect to lay the venue in the province where the
in effect, in eliminating it from the statue, is sound. We do not deprive
defendant was "found" and not where he resided; whereas the
it of all significance and effect and do not eliminate it from the statue.
construction which we place upon these phrases permits both to
We give it the only effect which can be given it and still accord with
have effect. We declare that, when the defendant is a resident of the
the other provisions of the section which give defendant the right to
Philippine Islands, the venue must be laid either in the province
have the venue laid in the province of his residence, the effect which
where the plaintiff resides or in the province where the defendant
it was intended by the legislature they should have. We held that the
resides, and in no other province. Where, however, the defendant is
word "found" was applicable in certain cases, and in such cases
a nonresident the venue may be laid wherever defendant may be
gave it full significance and effect. We declared that it was applicable
found in the Philippine Islands. This construction gives both phrases
and effective in cases where the defendant is a nonresident. In such
their proper and legitimate effect without doing violence to the spirit
cases where the defendant is a nonresident. In such cases the
which informs all laws relating to venue and which insists always that
venue may be laid wherever he may be found in the Philippine
the action shall tried in the place where the greater convenience of
Islands at the time of the service of the process, but we also held
the parties will be served. Ordinarily a defendant's witness are found
that where he is a resident of the Philippine Islands the word "found"
where the defendant resides; and plaintiff's witnesses are generally
has no application and the venue must be laid in the province where
found where he resides or where the defendant resides. It is,
he resides.
therefore, generally desirable to have the action tried where on of the
resides. Where the plaintiff is a nonresident and the contract upon
which suit is brought was made in the Philippine Islands it may safely

925
be asserted that the convenience of the defendant would be best But while it is to the corporation that the action should pertain in cases of this
served by a trial in the province where he resides. nature, however, if the officers of the corporation, who are the ones called
upon to protect their rights, refuse to sue, or where a demand upon them to
The fact that defendant was sojourning in Pasay t the time he was served file the necessary suit would be futile because they are the very ones to be
with summons does not make him a resident of that place for purposes of sued or because they hold the controlling interest in the corporation, then in
venue. Residence is "the permanent home, the place to which, whenever that case any one of the stockholders is allowed to bring suit (3 Fletcher's
absent for business or pleasure, one intends to return, ..." (67 C.J., pp. 123- Cyclopedia of Corporations, pp. 977-980). But in that case it is the
124.) A man can have but one domicile at a time (Alcantara vs. Secretary of corporation itself and not the plaintiff stockholder that is the real property in
Interior, 61 Phil., 459), and residence is anonymous with domicile under interest, so that such damages as may be recovered shall pertain to the
section 1 of Rule 5 (Moran's Comments, supra, p. 104). corporation (Pascual vs. Del Saz Orosco, 19 Phil. 82, 85). In other words, it
is a derivative suit brought by a stockholder as the nominal party plaintiff for
In view of the foregoing, we hold that the objection to the venue was correctly the benefit of the corporation, which is the real property in interest (13
sustained by the lower court. Fletcher, Cyclopedia of Corporations, p. 295).

As to the second question, the complaint shows that the action is for In the present case, the plaintiff stockholders have brought the action not for
damages resulting from mismanagement of the affairs and assets of the the benefit of the corporation but for their own benefit, since they ask that the
corporation by its principal officer, it being alleged that defendant's defendant make good the losses occasioned by his mismanagement and pay
maladministration has brought about the ruin of the corporation and the to them the value of their respective participation in the corporate assets on
consequent loss of value of its stocks. The injury complained of is thus the basis of their respective holdings. Clearly, this cannot be done until all
primarily to the corporation, so that the suit for the damages claimed should corporate debts, if there be any, are paid and the existence of the corporation
be by the corporation rather than by the stockholders (3 Fletcher, Cyclopedia terminated by the limitation of its charter or by lawful dissolution in view of the
of Corporation pp. 977-980). The stockholders may not directly claim those provisions of section 16 of the Corporation Law.
damages for themselves for that would result in the appropriation by, and the
distribution among them of part of the corporate assets before the dissolution It results that plaintiff's complaint shows no cause of action in their favor so
of the corporation and the liquidation of its debts and liabilities, something that the lower court did not err in dismissing the complaint on that ground.
which cannot be legally done in view of section 16 of the Corporation Law,
which provides: While plaintiffs ask for remedy to which they are not entitled unless the
requirement of section 16 of the Corporation Law be first complied with, we
No shall corporation shall make or declare any stock or bond note that the action stated in their complaint is susceptible of being converted
dividend or any dividend whatsoever from the profits arising from its into a derivative suit for the benefit of the corporation by a mere change in
business, or divide or distribute its capital stock or property other the prayer. Such amendment, however, is not possible now, since the
than actual profits among its members or stockholders until after the complaint has been filed in the wrong court, so that the same last to be
payment of its debts and the termination of its existence by limitation dismissed.
or lawful dissolution.

926
The order appealed from is therefore affirmed, but without prejudice to the
filing of the proper action in which the venue shall be laid in the proper
province. Appellant's shall pay costs. So ordered.

Moran, C.J., Ozaeta, Pablo, Bengzon, Tuason, and Montemayor, JJ., concur.

927
G.R. No. L-22399 March 30, 1967

REPUBLIC BANK, represented in this action by DAMASO P. PEREZ,


etc., plaintiff-appellant,
vs.
MIGUEL CUADERNO, BIENVENIDO DIZON, PABLO ROMAN,
THE BOARD OF DIRECTORS OF THE REPUBLIC BANK AND THE
MONETARY BOARD OF THE CENTRAL BANK OF THE
PHILIPPINES, defendants-appellees.
Crispin D. Baizas and Associates and Halili, Bolinao and Associates for
plaintiff-appellant.

928
N. M. Balboa, F.E. Evangelista and S. Malvar for defendant-appellee Board ordered an investigation, which was carried out by Bank Examiners;
Monetary Board. that they and the Superintendent of Banks of the Central Bank reported that
Norberto J. Quisumbing and H.V. Quisumbing for other defendants- certain mortgage loans amounting to P2,303,400.00 were granted in violation
appellees. of sections 77, 78 and 88 of the General Banking Act; that acting on said
reports, the Monetary Board, of which defendant Cuaderno was a member,
REYES, J.B.L., J.: ordered a new Board of Directors of the Republic Bank to be elected, which
was done, and subsequently approved by the Monetary Board; that on
Direct appeal from an order of the Court of First Instance of Manila, in its civil January 5, 1960, the latter accepted the offer of Pablo Roman to put up
case No. 53936, dismissing the petitioner's complaint on the ground of failure adequate security for the questioned loans made by the Republic Bank, and
to state cause of action. such security was made a condition for the resumption of the Bank's normal
operations; that subsequently, the Central Bank through its Governor, Miguel
Cuaderno, referred to special prosecutors of the Department of Justice on
In the Court below, Damaso Perez, a stockholder of the Republic Bank, a
July 22, 1960, the banking frauds and violations of the Banking Act, reported
Philippine banking corporation domiciled in Manila, instituted a derivative suit
by the Superintendent of Banks, for investigation and prosecution, but no
for and in behalf of said Bank, against Miguel Cuaderno, Bienvenido Dizon,
information was filed up to the time of the retirement of Cuaderno in 1961;
the Board of Directors of the Republic Bank, and the Monetary Board of the
that other similar frauds were subsequently discovered; that to neutralize the
Central Bank of the Philippines. Paragraph 6 of the Complaint (Rec. on
impending action against him, Pablo Roman engaged Miguel Cuaderno as
Appeal, p. 7) expressly pleaded the following: .
technical consultant at a compensation of P12,500.00 per month, and
selected Bienvenido Dizon as chairman of the Board of Directors of the
6. That the relator herein filed the present derivative suit without any Republic Bank; that the Board of Directors composed of individuals
further demand on the Board of Directors of the Republic Bank for personally selected and chosen by Roman, connived and confederated in
the reason that such formal demand to institute the present approving the appointment and selection of Cuaderno and Dizon; that such
complaint would be a futile formality since the members of the board action was motivated by bad faith and without intention to protect the interest
are personally chosen by defendant Pablo Roman himself. of the Republic Bank but were prompted to protect Pablo Roman from
criminal prosecution; that the appointment of Cuaderno and his acceptance
For a cause of action plaintiff alleged, inter alia, that Damaso Perez had of the position of technical consultant are immoral, anomalous and illegal,
complained to the Monetary Board of the Central Bank against certain frauds and his compensation highly unconscionable, because court actions
allegedly committed by defendant Pablo Roman, in that being chairman of involving the actuations of Cuaderno as Governor and Member or Chairman
the Board of Directors of the Republic Bank, and of its Executive Loan of the Monetary Board are still pending in court; that as member of the
Committee, in 1957 to 1959, "in grave abuse of his fiduciary duty and taking Monetary Board from 1961 to 1962, Bienvenido Dizon exercised supervision
advantage of his said positions and in connivance with other officials of the over the Republic Bank; that the selection of Dizon as chairman of the Board
Republic Bank", Roman had fraudulently granted or caused to be granted of the Republic Bank after he was forced to resign from the presidency of the
loans to fictitious and non-existing persons and to their close friends, Philippine National Bank and from membership of the Monetary Board and
relatives and/or employees, who were in reality their dummies, on the basis within one year thereafter is in violation of option 3, sub-paragraph (d) of the
of fictitious and inflated appraised values of real estate properties; that said Anti-Graft and Corrupt Practices Act; that both Cuaderno and Dizon
loans amounted to almost 4 million pesos; that acting upon the complaint, were alter egos of Pablo Roman; that the Monetary Board was about to
Miguel Cuaderno (then Governor of the Central Bank) and the Monetary

929
approve the appointment of Cuaderno and Dizon and would do so unless Roman to refund the amounts paid to said defendant Miguel
enjoined. Cuaderno and defendant Bienvenido Dizon, and to pay such
reasonable damages to the plaintiff Republic Bank;
The complaint, therefore, prayed for a writ of preliminary injunction against
the Monetary Board to prevent its confirmation of the appointments of Dizon f) ordering all the defendants to pay the sum of P25,000.00 as
and Cuaderno; against the Board of Directors of the Republic Bank from attorney's fees, including all expenses of litigation and costs of this
recognizing Cuaderno as technical consultant and Dizon as Chairman of the suit.
Board; and against Pablo Roman from appointing or selecting officers or
directors of the Republic Bank, and against the recognition of any such The Monetary Board filed an answer with denials, admissions and affirmative
appointees until final determination of the action. And concluded by praying defenses; but the other defendants filed separate motions to dismiss on
that after due hearing, judgment be rendered, practically the same grounds: no valid cause of action against the individual
movants; lack of legal capacity of plaintiff-relator to sue; and non-exhaustion
a) making the writ of injunction permanent; of intra-corporate remedies. These motions were duly opposed by plaintiff
Damaso Perez.1wph1.t
b) declaring the appointment of defendant Miguel Cuaderno as
technical consultant with monthly compensation of P12,500.00 On October 24, 1963, the court, "taking into consideration the grounds
unconscionable, immoral, illegal and null and void; alleged in the motions to dismiss and the opposition for the issuance of a writ
of preliminary injunction and the affirmative defenses filed by the defendants
c) declaring the selection of defendant Bienvenido Dizon as and the arguments in support thereof", and "that there are already eight
chairman of the Board of Directors of the Republic Bank violative of cases pending in the different branches of this court between practically the
Section 3, sub-paragraph (d) of Republic Act No. 5019, otherwise same parties", denied the petition for a writ of preliminary injunction and
known as the Anti-Graft and Corrupt Practices Act, and therefore, dismissed the case. The court in effect suggested that the matter at issue in
illegal and null and void; the case may be presented in any of the pending eight cases by means of
amended and supplemental pleadings.
d) declaring that defendant Pablo Roman, in view of his criminal
liability for the fraudulent real estate mortgage loans in the Republic Plaintiff Damaso Perez thereupon appealed to this Court.
Bank amounting to P4 million, has no right to select or to be allowed
to select person or persons who are his alter egos to manage the The issue in this appeal, then, is whether or not the Court below erred in
Republic Bank, and enjoining the defendant Board of Directors of the dismissing the complaint. In this connection, it should be remembered that
Republic Bank from recognizing any officers or directors appointed the defenses of the Monetary Board of the Central Bank, being interposed in
or selected by defendant Pablo Roman; an answer and not in a motion to dismiss, are not here at issue. Our sole
concern is with the motions to dismiss of the other defendants, Roman,
e) ordering defendants Miguel Cuaderno and Bienvenido Dizon to Cuaderno, Dizon, and the Board of Directors of the Republic Bank.
return to the Republic Bank all amounts they may have received
either in the form of compensation, remuneration or emolument, with They mainly controvert the right of plaintiff to question the appointment and
an interest thereon at the rate of 6%; or to order defendant Pablo selection of defendants Cuaderno and Dizon, which they contend to be the

930
result of corporate acts with which plaintiff, as stockholder, cannot interfere. cause of action. This is the cardinal principle in the matter. And, it has been
Normally, this is correct, but Philippine jurisprudence is settled that an ruled that the test of sufficiency of the facts alleged is whether or not the
individual stockholder is permitted to institute a derivative or representative Court could render a valid judgment as prayed for, accepting as true the
suit on behalf of the corporation wherein he holds stock in order to protect or exclusive facts set forth in the complaint.1So rigid is the norm prescribed that
vindicate corporate rights, whenever the officials of the corporation refuse to if the Court should doubt the truth of the facts averred it must not dismiss the
sue, or are the ones to be sued or hold the control of the corporation. In such complaint but require an answer and proceed to trial on the merits. 2
actions, the suing stockholder is regarded as a nominal party, with the
corporation as the real party in interest (Pascual vs. Del Saz Orozco, 19 Phil. Defendants urge that the action is improper because the plaintiff was not
82, 85; Everett vs. Asia Banking Corp., 45 Phil. 518; Angeles vs. Santos, 64 authorized by the corporation to bring suit in its behalf. Any such authority
Phil. 697; Evangelista vs. Santos, 86 Phil. 388). Plaintiff-appellant's action could not be expected as the suit is aimed to nullify the action taken by the
here is precisely in conformity, with these principles. He is neither alleging manager and the board of directors of the Republic Bank; and any demand
nor vindicating his own individual interest or prejudice, but the interest of the for intra-corporate remedy would be futile, as expressly pleaded in the
Republic Bank and the damage caused to it. The action he has brought is a complaint. These circumstances permit a stockholder to bring a derivative
derivative one, expressly manifested to be for and in behalf of the Republic suit (Evangelista vs. Santos, 86 Phil. 394). That no other stockholder has
Bank, because it was futile to demand action by the corporation, since its chosen to make common cause with plaintiff Perez is irrelevant, since the
Directors were nominees and creatures of defendant Pablo Roman smallness of plaintiff's holdings is no ground for denying him relief
(Complaint, p. 6). The frauds charged by plaintiff are frauds against the Bank (Ashwander vs. TVA, 80 L. Ed. 688). At any rate, it is yet too early in the
that redounded to its prejudice. proceedings for the absence of other stockholders to be of any significance,
no issues having even been joined.
The complaint expressly pleads that the appointment of Cuaderno as
technical consultant, and of Bienvenido Dizon to head the Board of Directors There remains the procedural question whether the corporation itself must be
of the Republic Bank, were made only to shield Pablo Roman from criminal made party defendant. The English practice is to make the corporation a
prosecution and not to further the interests of the Bank, and avers that both party plaintiff, while in the United States, the usage leans in favor of its being
men are Roman's alter egos. There is no denying that the facts thus pleaded joined as party defendant (see Editorial Note, 51 LRA [NS] 123). Objections
in the complaint constitute a cause of action for the bank: if the questioned can be raised against either method. Absence of corporate authority would
appointments were made solely to protect Roman from criminal prosecution, seem to militate against making the corporation a party plaintiff, while joining
by a Board composed by Roman's creatures and nominees, then the it as defendant places the entity in the awkward position of resisting an action
moneys disbursed in favor of Cuaderno and Dizon would be an unlawful instituted for its benefit. What is important is that the corporation' should be
wastage or diversion of corporate funds, since the Republic Bank would have made a party, in order to make the Court's judgment binding upon it, and thus
no interest in shielding Roman, and the directors in approving the bar future relitigation of the issues. On what side the corporation appears
appointments would be committing a breach of trust; the Bank, therefore, loses importance when it is considered that it lay within the power of the trial
could sue to nullify the appointments, enjoin disbursement of its funds to pay court to direct the making of such amendments of the pleadings, by adding or
them, and recover those paid out for the purpose, as prayed for in the dropping parties, as may be required in the interest of justice (Revised Rule
complaint in this case (Angeles vs. Santos, supra.). 3, sec. 11). Misjoinder of parties is not a ground to dismiss an action. (Ibid.)

Facts pleaded in the complaint are to be deemed accepted by the We see no reason to support the contention of defendant Bienvenido Dizon
defendants who file a motion to dismiss the complaint for failure to state a that the action of plaintiff amounts to a quo warranto proceeding. Plaintiff

931
Perez is not claiming title to Dizon's position as head of the Republic Bank's
board of directors. The suit is aimed at preventing the waste or diversion of
corporate funds in paying officers appointed solely to protect Pablo Roman
from criminal prosecution, and not to carry on the corporation's bank
business. Whether the complaint's allegations to such effect are true or not
must be determined after due hearing.

Independently of the grounds advanced by the defendants in their motions to


dismiss, the Court a quo gave as a further pretext for the dismissal of the
action the pendency of eight other lawsuits between practically the same
parties; reasoning that the question at issue in the present case could be
incorporated in any one of the other actions by amended or supplemental
pleading. We fail to see that this justifies the dismissal of the case under
appeal. In the first place, there is no pretense that the cause of action here
was already included in any of the other pending cases. As a matter of fact,
dismissal of the present action was not sought on the ground of pendency of
another action between the same parties. Secondly, the amendment of a
complaint after a responsive pleading is filed, would rest upon the discretion
of the party and the Court. Hence, this case cannot be dismissed simply
because of the possibility that the cause of action here can be incorporated
or introduced in any of those of the pending cases.

In view of the foregoing, the order dismissing the complaint is reversed and
set aside. The case is remanded to the court of origin with instructions to
overrule the motions to dismiss and require the defendants to answer the
complaint. Thereafter, the case shall be tried and decided on its merits. Costs
against defendants-appellees. So ordered.

Concepcion, C.J., Dizon, Regala, Bengzon, J.P., Zaldivar, Sanchez and


Castro, JJ., concur.
Makalintal, J., took no part.

932
933
G.R. No. L-16982 September 30, 1961

CATALINA R. REYES, petitioner,


vs.
HON. BIENVENIDO A. TAN, as Judge of the Court of First Instance of
Manila, Branch XIII and FRANCISCA R. JUSTINIANI, respondents.

Jose W. Diokno for petitioner.


Norberto J. Quisumbing for respondents.

LABRADOR, J.:

This is a petition for certiorari to review and set aside an order of the Court of
First Instance of Manila, Hon. Bienvenido A. Tan, presiding, in Civil Case No.
42375, entitled "Francisca R. Justiniani vs. Wadhumal Dalamal, et al.",
appointing a receiver of the corporation Roxas-Kalaw Textile Mills, Inc. In
said action, plaintiff Justiniani asks the court to order the directors of the
corporation, jointly and severally, to repair the damage caused to the
corporation, of which all the plaintiff and defendants are members. The action
was filed about January of 1960 and the order for the appointment of the
receiver issued on February 15, 1960, while the designation of the receiver
was made in an order of the court dated April 30, 1960.

In the complaint in said Civil Case No. 42375, it is alleged that the
corporation, Roxas-Kalaw Textile Mills, Inc., was organized on June 5, 1954
by defendants Cesar K. Roxas, Adelia K. Roxas, Benjamin M. Roxas, Jose
Ma. Barcelona and Morris Wilson, for and on behalf of the following primary
principals with the following shareholdings: Adelia K. Roxas, 1200 Class A
shares; I. Sherman, 900 Class A shares; Robert W. Born, 450 Class A shares
and Morris Wilson, 450 Class A shares; that the plaintiff holds both Class A
and Class B shares and number and value thereof are is follows: Class A
50 shares, Class B 1,250 shares; that on May 8, 1957, the Board of

934
Directors approved a resolution designating one Dayaram as co-manager The second ground of the defendant's motion to dismiss and or deny
with the specific understanding that he was to act as defendant Wadhumal the petition is the allegedly want of a cause of action of the plaintiff's
Dalamal's designee, Morris Wilson was likewise designated as co-manager complaint. Philippine jurisprudence is complete with authorities
with responsibilities for the management of the factory only, that an office in upholding the principle that this ground for dismissal must appear in
New York was opened for the purpose of supervising purchases, which the face of the complaint itself; and that to determine the sufficiency
purchases must have the unanimous agreement of Cesar K. Roxas, New of the cause of action, only the facts alleged in the complaint and no
York resident member of the board of directors, Robert Born and Wadhumal other, should be considered; in fine, the test of sufficiency of cause of
Dalamal or their respective representatives; that several purchases action is whether or not, admitting the facts alleged in the complaint,
aggregating $289,678.86 were made in New York for raw materials such as the Court could render a valid judgment upon the same in
greige cloth, rayon and grey goods for the textile mill and shipped to the accordance with the prayer of the petition (e.g., Paminsan v.
Philippines, which shipment were found out to consist not of raw materials Costales, 29 Phil. 587, 489). The complaint in the instant case
but already finished products, such as, West Point Khaki rayon suiting abounds with arguments establishing and supporting plaintiff's cause
materials dyed in the piece, finished rayon tafetta in cubes, cotton eyelets, of action for and in behalf of the Roxas-Kalaw Textile Mills, Inc.
etc., for which reasons the Central Bank of the Philippines stopped all dollar against all the defendants (See e.g. paragraphs 4, 5, 6 and 7 of the
allocations for raw materials for the corporation which necessarily led to the Complaint). Taking these paragraphs of the complaint in context, it is
paralyzation of the operation of the textile mill and its business; that the clear that the plaintiff has sufficient averred facts constituting a cause
supplier of the aforesaid finished goods was the United Commercial or basis for a derivative suit for "injuries to the corporation, as by
Company of New York in which defendant Dalamal had interests and the negligence, mismanagement or fraud of its directors, are normally
letter of credit for said goods were guaranteed by the Indian Commercial dealt with as wrong to the whole group of share holders in their
Company and the Indian Traders in which firms defendant Dalamal likewise corporate capacity, to be redressed in a suit by or on behalf of the
held interests; that the resale of the finished goods was the business of the corporation.1awphl.nt
Indian Commercial Company of Manila, which company could not obtain
dollar allocations for importations of finished goods under the Central Bank Evident from the defendants' motion to dismiss and/or to deny the
regulations; that plaintiff and some members of the board of directors urged petition for receivership is their complete failure to come up with a
defendants to proceed against Dalamal, exposing his offense to the Central valid and substantial defense against or denial of the complaint's
Bank, and to initiate suit against Dalamal for his fraud against the allegations of mismanagement, if not the actual commission of ultra
corporation; that defendants refused to proceed against Dalamal and instead vires and illegal acts. Invariably the props of defendants' motion
continued to deal with the Indian Commercial Company to the damage and consist of the unconvincing countercharges of the plaintiff's non-
prejudice of the corporation. The prayer asks for the appointment of a observance of the technicalities of our procedural law and disregard
receiver and a judgment marking defendants jointly and severally liable for of technical and evidently futile intracorporate remedies to redress
the damages. the violations charged against the defendants. It is clear that the
controlling majority did nothing for two years to protect the interests
After a denial of a motion to dismiss and the filing of an answer alleging that of corporation. (See pars. 5-7, complaint.)
the complaint states no cause of action, the motion for the appointment of a
receiver was set for hearing and subsequently the court entered the order for The defendants themselves having admitted in open court during the
the appointment of a receiver. The court found and held: oral discussion of their motion to dismiss and the plaintiff's motion for
receivership that the majority stockholders will under any condition

935
entertain any suggestion of the minority shareholders, the It is also not denied by petitioner that the allocation of dollars to the
appointment of an independent third party in the management of the corporation for the importation of raw materials was suspended. In the eyes
corporation becomes imperative for the survival of the company. of the court below, as well as in our own, the importation of textiles instead of
(Order dated Feb. 15, 1960). raw materials, as well as the failure of the Board of Directors to take action
against those directly responsible for the misuse of dollar allocations
On April 30, 1960, the court issued mother order which reads as follows: constitute fraud, or consent thereto on the part of the directors. Therefore, a
breach of trust was committed which justified the derivative suit by a minority
After this incident wherein it was clearly shown that the minority stockholder on behalf of the corporation.
stockholders, represented by the plaintiff, have no recourse
whatsoever before the majority stockholders of the company, and It is well settled in this jurisdiction that where corporate directors are
after it has been shown that the majority has violated the law by guilty of a breach of trust not of mere error of judgment or abuse
importing into the Philippines finished goods instead of raw materials of discretion and intracorporate remedy is futile or useless, a
as stipulated in their license, and since these acts are prejudicial to stockholder may institute a suit in behalf of himself and other
the company because it might result in the cancellation of their stockholders and for the benefit of the corporation, to bring about a
license, the Court is of the opinion and so holds that the appointment redress of the wrong inflicted directly upon the corporation and
of a receiver is absolutely necessary for the protection not only of the indirectly upon the stockholders. An illustration of a suit of this kind is
rights of the minority but also those of the majority stockholders of found in the case of Pascual vs. Del Saz Orozco (19 Phil. 82),
the company. decided by this Court as early as 1911. In that case, the Banco
Espaol-Filipino suffered heavy losses due to fraudulent connivance
In the first assignment of error, petitioner claims that respondent Justiniani between a depositor and an employee of the bank, which losses, it
neither alleged nor proved the existence of an emergency requiring the was contended, could have been avoided if the president and
immediate appoinment of a receiver of the Roxas-Kalaw Textile Mill, Inc.; that directors had been more vigilant in the administration of the affairs of
the alleged fraudulent transaction took place more than two years before the the bank. The stockholders constituting the minority brought a suit in
application for receivership, and so was the refusal of the directors to sue or behalf of the bank against the directors to recover damages, and this
prosecute Dalamal. This contention is not well founded. At the hearing of the over the objection of the majority of the stockholders and the
petition for the appointment of a receiver held on January 30, 1960, various directors. This court held that the suit could properly be maintained.
records of shipments of finished textile goods on dollar allocations for raw (64 Phil., Angeles vs. Santos [G.R. No. L-43413, prom. August 31,
materials were exhibited. Publicity had also been given to the importations of 1937] p. 697).
textiles by the corporation, in place of cotton raw materials. The record
shows the list of the various documents proving the purchase of letters of The claim that respondent Justiniani did not take steps to remedy the illegal
credit for textiles. These textiles were denied importation and had to be re- importation for a period of two years is also without merit. During that period
exported. The fact of the importation of finished textiles on dollar allocations of time respondent had the right to assume and expect that the directors
for raw materials in violation of Central Bank regulations was, therefore, would remedy the anomalous situation of the corporation brought about by
conclusively shown. their own wrong doing. Only after such period of time had elapsed could
respondent conclude that the directors were remiss in their duty to protect
the corporation property and business.

936
Counsel for petitioner claims that respondent Justiniani was treasurer of the in the court below. (Tan Machan v. Trinidad, 3 Phil. 684; Ramiro v. Grao, 54
corporation for sometime and had control of funds and this notwithstanding, Phil. 744; Vda. de Villaruel, et al. v. Manila Motor Co., Inc., et al., G.R. No. L-
she had not taken the steps to remedy the situation. In answer we state that 10394, Dec. 13, 1958; Collector of Internal Revenue v. Estate of F. P. Buan,
the fraud consisted in importing finished textile instead of raw cotton for the et al., G.R. Nos. L-11438-39, and L-11542-46, July 31, 1958; S.V.S. Pictures,
textile mill; the fraud, therefore, was committed by the manager of the Inc., et al. v. The Court of Appeals, et al., G.R. No. L-7075, January 29, 1960;
business and was consented to by the directors, evidently beyond reach of Elena Peralta Vda. de Caina vs. Hon. Andres Reyes, et al., G.R. No. L-
respondent. 15792, May 30, 1960).

The directors permitted the fraudulent transaction to go unpunished and The supposed new management, alleged as a ground for the reversal of the
nothing appears to have been done to remove the erring purchasing order of the court below appointing a receiver, is not in itself a ground of
managers. In a way the appointment of a receiver may have been thought of objection to the appointment of a receiver. The parties found to be guilty of
by the court below so that the dollar allocation for raw material may be the fraud, as a cause of which receivership proceedings were instituted, were
revived and the textile mill placed on an operating basis. It is possible that if a the Board of Directors, which took no action to stop the anomalies being
receiver in which the Central Bank may have confidence is appointed, the perpetrated by the management. But it appears that the management must
dollar allocation for raw material may be restored. Claim is made that if a have acted directly under orders of the Board of Directors. The appointment
receiver is appointed, the Philippine National Bank to which the corporation of a new management, therefore, would not remedy the anomalous situation
owes considerable sums of money might be led to foreclose the mortgage. in which the corporation is found, because such situation was not due to the
Precisely the appointment of a receiver in whom the bank may have had management alone but principally because of direction of the Board of
confidence might rehabilitate the business and bring a restoration of the Directors.
dollar allocation much needed for raw material and an improvement in the
business and assets the corporation, thus insuring the collection of the The second ground for the petition is, therefore, also without merit.
bank's loan.
WHEREFORE, the court finds that the court below did not commit an abuse
Considering the above circumstances we are led to agree with the judge of discretion in appointing a receiver for the corporation and the petition to
below that the appointment of a receiver was not only expedient but also set aside the order for the appointment of a receiver should be, as it is
necessary to restore the faith and confidence of the Central Bank authorities hereby, dismissed. With costs against the petitioner.
in the administration of the affairs of the corporation, thus ultimately leading
to a restoration of the dollar allocation so essential to the operation of the Bengzon, C.J., Padilla, Reyes, J.B.L., Paredes and De Leon, JJ., concur.
textile mills. The first assignment of error is, therefore, overruled.

In the second assignment of error, petitioner claims that the management


has been changed and the new management has not been afforded a
chance to show what it can do. This ground of the petition was not mentioned
or raised as a ground of defense or objection to the appointment of a receiver
in the court below. It is only raised for the first time before Us in the petition
for certiorari. The principle has long ago been enunciated by Us that an
appellate court may not consider any ground of objection that was not raised

937
938
When a bank, by its acts and failure to act, has clearly clothed its manager
with apparent authority to sell an acquired asset in the normal course of
business, it is legally obliged to confirm the transaction by issuing a board
resolution to enable the buyers to register the property in their names. It has
a duty to perform necessary and lawful acts to enable the other parties to
enjoy all benefits of the contract which it had authorized.

The Case

Before this Court is a Petition for Review on Certiorari challenging the


December 18, 1998 Decision of the Court of Appeals 1 (CA) in CA-GR SP
No. 46246, which affirmed the May 20, 1997 Decision 2 of the Regional Trial
Court (RTC) of Naga City (Branch 28). The CA disposed as follows:

Wherefore, premises considered, the Judgment appealed from is


hereby AFFIRMED. Costs against the respondent-appellant. 3

The dispositive portion of the judgment affirmed by the CA ruled in this wise:

WHEREFORE, in view of all the foregoing findings, decision is


hereby rendered whereby the [petitioner] Rural Bank of Milaor
(Camarines Sur), Inc. through its Board of Directors is hereby
ordered to immediately issue a Board Resolution confirming the
Deed of Sale it executed in favor of Renato Ocfemia marked Exhibits
C, C-1 and C-2); to pay [respondents] the sum of FIVE HUNDRED
G.R. No. 137686 February 8, 2000 (P500.00) PESOS as actual damages; TEN THOUSAND
(P10,000.00) PESOS as attorney's fees; THIRTY THOUSAND
RURAL BANK OF MILAOR (CAMARINES SUR), petitioner, (P30,000.00) PESOS as moral damages; THIRTY THOUSAND
vs. (P30,000.00) PESOS as exemplary damages; and to pay the costs. 4
FRANCISCA OCFEMIA, ROWENA BARROGO, MARIFE O. NIO,
FELICISIMO OCFEMIA, RENATO OCFEMIA JR, and WINSTON Also assailed is the February 26, 1999 CA Resolution 5 which denied
OCFEMIA, respondents. petitioner's Motion for Reconsideration.

PANGANIBAN, J.: The Facts

939
The trial court's summary of the undisputed facts was reproduced in the CA Felicisimo Ocfemia. During the lifetime of her grandparents,
Decision as follows: [respondents] mortgaged the said five (5) parcels of land and two (2)
others to the [petitioner] Rural Bank of Milaor as shown by the Deed
This is an action for mandamus with damages. On April 10, 1996, of Real Estate Mortgage (Exhs. A and A-1) and the Promissory Note
[herein petitioner] was declared in default on motion of the (Exh. B).
[respondents] for failure to file an answer within the reglementary-
period after it was duly served with summons. On April 26, 1996, The spouses Felicisimo Ocfemia and Juanita Arellano Ocfemia were
[herein petitioner] filed a motion to set aside the order of default with not able to redeem the mortgaged properties consisting of seven (7)
objection thereto filed by [herein respondents]. parcels of land and so the mortgage was foreclosed and thereafter
ownership thereof was transferred to the [petitioner] bank. Out of the
On June 17, 1996, an order was issued denying [petitioner's] motion seven (7) parcels that were foreclosed, five (5) of them are in the
to set aside the order of default. On July 10, 1996, the defendant possession of the [respondents] because these five (5) parcels of
filed a motion for reconsideration of the order of June 17, 1996 with land described in paragraph 6 of the petition were sold by the
objection thereto by [respondents]. On July 12, 1996, an order was [petitioner] bank to the parents of Marife O. Nio as evidenced by a
issued denying [petitioner's] motion for reconsideration. On July 31, Deed of Sale executed in January 1988 (Exhs. C, C-1 and C-2).
1996, [respondents] filed a motion to set case for hearing. A copy
thereof was duly furnished the [petitioner] but the latter did not file The aforementioned five (5) parcels of land subject of the deed of
any opposition and so [respondents] were allowed to present their sale (Exh. C), have not been, however transferred in the name of the
evidence ex-parte. A certiorari case was filed by the [petitioner] with parents of Merife O. Nio after they were sold to her parents by the
the Court of Appeals docketed as CA GR No. 41497-SP but the [petitioner] bank because according to the Assessor's Office the five
petition was denied in a decision rendered on March 31, 1997 and (5) parcels of land, subject of the sale, cannot be transferred in the
the same is now final. name of the buyers as there is a need to have the document of sale
registered with the Register of Deeds of Camarines Sur.
The evidence presented by the [respondents] through the testimony
of Marife O. Nio, one of the [respondents] in this case, show[s] that In view of the foregoing, Marife O. Nio went to the Register of
she is the daughter of Francisca Ocfemia, a co-[respondent] in this Deeds of Camarines Sur with the Deed of Sale (Exh. C) in order to
case, and the late Renato Ocfemia who died on July 23, 1994. The have the same registered. The Register of Deeds, however, informed
parents of her father, Renato Ocfemia, were Juanita Arellano her that the document of sale cannot be registered without a board
Ocfemia and Felicisimo Ocfemia. Her other co-[respondents] resolution of the [petitioner] Bank. Marife Nio then went to the bank,
Rowena O. Barrogo, Felicisimo Ocfemia, Renato Ocfemia, Jr. and showed to if the Deed of Sale (Exh. C), the tax declaration and
Winston Ocfemia are her brothers and sisters.1wphi1.nt receipt of tax payments and requested the [petitioner] for a board
resolution so that the property can be transferred to the name of
Marife O. Nio knows the five (5) parcels of land described in Renato Ocfemia the husband of petitioner Francisca Ocfemia and
paragraph 6 of the petition which are located in Bombon, Camarines the father of the other [respondents] having died already.
Sur and that they are the ones possessing them which [were]
originally owned by her grandparents, Juanita Arellano Ocfemia and

940
The [petitioner] bank refused her request for a board resolution and deed of sale itself already show[ed] that the [respondents were]
made many alibi[s]. She was told that the [petitioner] bank ha[d] a clearly entitled to the land subject of the sale (Exh. F). The manager
new manager and it had no record of the sale. She was asked and of the [petitioner] bank received the letter which was served
she complied with the request of the [petitioner] for a copy of the personally to him and the latter told Marife O. Nio that since he was
deed of sale and receipt of payment. The president of the [petitioner] the one himself who received the letter he would not sign anymore a
bank told her to get an authority from her parents and other copy showing him as having already received said letter (Exh. F).
[respondents] and receipts evidencing payment of the consideration
appearing in the deed of sale. She complied with said requirements After several days from receipt of the letter (Exh. F) when Marife O.
and after she gave all these documents, Marife O. Nio was again Nio went to the [petitioner] again and reiterated her request, the
told to wait for two (2) weeks because the [petitioner] bank would still manager of the [petitioner] bank told her that they could not issue the
study the matter. required board resolution as the [petitioner] bank ha[d] no records of
the sale. Because of this Merife O. Nio already went to their lawyer
After two (2) weeks, Marife O. Nio returned to the [petitioner] bank and ha[d] this petition filed.
and she was told that the resolution of the board would not be
released because the [petitioner] bank ha[d] no records from the old The [respondents] are interested in having the property described in
manager. Because of this, Marife O. Nio brought the matter to her paragraph 6 of the petition transferred to their names because their
lawyer and the latter wrote a letter on December 22, 1995 to the mother and co-petitioner, Francisca Ocfemia, is very sickly and they
[petitioner] bank inquiring why no action was taken by the board of want to mortgage the property for the medical expenses of Francisca
the request for the issuance of the resolution considering that the Ocfemia. The illness of Francisca Ocfemia beg[a]n after her husband
bank was already fully paid [for] the consideration of the sale since died and her suffering from arthritis and pulmonary disease already
January 1988 as shown by the deed of sale itself (Exh. D and D-1 ). became serious before December 1995.

On January 15, 1996 the [petitioner] bank answered [respondents'] Marife O. Nio declared that her mother is now in serious condition
lawyer's letter (Exh. D and D-1) informing the latter that the request and they could not have her hospitalized for treatment as they do not
for board resolution ha[d] already been referred to the board of have any money and this is causing the family sleepless nights and
directors of the [petitioner] bank with another request that the latter mental anguish, thinking that their mother may die because they
should be furnished with a certified machine copy of the receipt of could not submit her for medication as they do not have money. 6
payment covering the sale between the [respondents] and the
[petitioner] (Exh. E). This request of the [petitioner] bank was already The trial court granted the Petition. As noted earlier, the CA affirmed the RTC
complied [with] by Marife O. Nio even before she brought the matter Decision.
to her lawyer.
Hence, this recourse. 7 In a Resolution dated June 23, 1999, this Court
On January 23, 1996 [respondents'] lawyer wrote back the branch issued a Temporary Restraining Order directing the trial court "to refrain and
manager of the [petitioner] bank informing the latter that they were desist from executing [pending appeal] the decision dated May 20, 1997 in
already furnished the receipts the bank was asking [for] and that the Civil Case No. RTC-96-3513, effective immediately until further orders from
[respondents] want[ed] already to know the stand of the bank this Court." 8
whether the board [would] issue the required board resolution as the

941
Ruling of the Court of Appeals We disagree. The well-settled rule is that jurisdiction is determined by the
allegations of the complaint. 11 In the present case, the Petition for
The CA held that herein respondents were "able to prove their present cause Mandamus filed by respondents before the trial court prayed that petitioner-
of action" against petitioner. It ruled that the RTC had jurisdiction over the bank be compelled to issue a board resolution confirming the Deed of Sale
case, because (1) the Petition involved a matter incapable of pecuniary covering five parcels of unregistered land, which the bank manager had
estimation; (2) mandamus fell within the jurisdiction of RTC; and (3) executed in their favor. The RTC has jurisdiction over such action pursuant to
assuming that the action was for specific performance as argued by the Section 21 of BP 129, which provides:
petitioner, it was still cognizable by the said court.
Sec. 21. Original jurisdiction in other cases. Regional Trial Courts
Issues shall exercise original jurisdiction;

In its Memorandum, 9 the bank posed the following questions: (1) in the issuance of writ of certiorari, prohibition, mandamus, quo
warranto, habeas corpus and injunction which may be enforced in
1. Question of Jurisdiction of the Regional Trial Court. Has a any part of their respective regions; and
Regional Trial Court original jurisdiction over an action involving title
to real property with a total assessed value of less than P20,000.00? (2) In actions affecting ambassadors and other public ministers and
consuls.
2. Question of Law. May the board of directors of a rural banking
corporation be compelled to confirm a deed of absolute sale of real A perusal of the Petition shows that the respondents did not raise any
property owned by the corporation which deed of sale was executed question involving the title to the property, but merely asked that petitioner's
by the bank manager without prior authority of the board of directors board of directors be directed to issue the subject resolution. Moreover, the
of the rural banking corporation? 10 bank did not controvert the allegations in the said Petition. To repeat, the
issue therein was not the title to the property; it was respondents' right to
This Court's Ruling compel the bank to issue a board resolution confirming the Deed of Sale.

The present Petition has no merit. Second Issue:


Authority of the Bank Manager
First Issue:
Jurisdiction of the Regional Trial Court Respondents initiated the present proceedings, so that they could transfer to
their names the subject five parcels of land; and subsequently, to mortgage
said lots and to use the loan proceeds for the medical expenses of their ailing
Petitioner submits that the RTC had no jurisdiction over the case. Disputing
mother. For the property to be transferred in their names, however, the
the ruling of the appellate court that the present action was incapable of
register of deeds required the submission of a board resolution from the bank
pecuniary estimation, petitioner argues that the matter in fact involved title to
confirming both the Deed of Sale and the authority of the bank manager, Fe
real property worth less than P20,000. Thus, under RA 7691, the case should
S. Tena, to enter into such transaction. Petitioner refused. After being given
have been filed before a metropolitan trial court, a municipal trial court or a
the runaround by the bank, respondents sued in exasperation.
municipal circuit trial court.

942
Allegations in the Petition for Mandamus Deemed Admitted In any event, the bank acknowledged, by its own acts or failure to act, the
authority of Fe S. Tena to enter into binding contracts. After the execution of
Respondents based their action before the trial court on the Deed of Sale, the Deed of Sale, respondents occupied the properties in dispute and paid
the substance of which was alleged in and a copy thereof was attached to the real estate taxes due thereon. If the bank management believed that it
the Petition for Mandamus. The Deed named Fe S. Tena as the had title to the property, it should have taken some measures to prevent the
representative of the bank. Petitioner, however, failed to specifically deny infringement or invasion of its title thereto and possession thereof.
under oath the allegations in that contract. In fact, it filed no answer at all, for
which reason it was declared in default. Pertinent provisions of the Rules of Likewise, Tena had previously transacted business on behalf of the bank,
Court read: and the latter had acknowledged her authority. A bank is liable to innocent
third persons where representation is made in the course of its normal
Sec. 7. Action or defense based on document. Whenever an business by an agent like Manager Tena, even though such agent is abusing
action or defense is based upon a written instrument or document, her authority. 14 Clearly, persons dealing with her could not be blamed for
the substance of such instrument or document shall be set forth in believing that she was authorized to transact business for and on behalf of
the pleading, and the original or a copy thereof shall be attached to the bank. Thus, this Court has ruled in Board of Liquidators v. Kalaw: 15
the pleading as an exhibit, which shall be deemed to be a part of the
pleading, or said copy may with like effect be set forth in the Settled jurisprudence has it that where similar acts have been
pleading. approved by the directors as a matter of general practice, custom,
and policy, the general manager may bind the company without
Sec. 8. How to contest genuineness of such documents. When an formal authorization of the board of directors. In varying language,
action or defense is founded upon a written instrument, copied in or existence of such authority is established, by proof of the course of
attached to the corresponding pleading as provided in the preceding business, the usages and practices of the company and by the
section, the genuineness and due execution of the instrument shall knowledge which the board of directors has, or must be presumed to
be deemed admitted unless the adverse party, under oath, have, of acts and doings of its subordinates in and about the affairs
specifically denies them, and sets forth what he claims to be the of the corporation. So also,
facts; but this provision does not apply when the adverse party does
not appear to be a party to the instrument or when compliance with . . . authority to act for and bind a corporation may be presumed from
an order for an inspection of the original instrument is refused. 12 acts of recognition in other instances where the power was in fact
exercised.
In failing to file its answer specifically denying under oath the Deed of Sale,
the bank admitted the due execution of the said contract. Such admission . . . Thus, when, in the usual course of business of a corporation, an
means that it acknowledged that Tena was authorized to sign the Deed of officer has been allowed in his official capacity to manage its affairs,
Sale on its behalf. 13 Thus, defenses that are inconsistent with the due his authority to represent the corporation may be implied from the
execution and the genuineness of the written instrument are cut off by an manner in which he has been permitted by the directors to manage
admission implied from a failure to make a verified specific denial. its business.

Other Acts of the Bank

943
Notwithstanding the putative authority of the manager to bind the bank in the In passing upon the liability of a corporation in cases of this
Deed of Sale, petitioner has failed to file an answer to the Petition below kind it is always well to keep in mind the situation as it
within the reglementary period, let alone present evidence controverting such presents itself to the third party with whom the contract is
authority. Indeed, when one of herein respondents, Marife S. Nino, went to made. Naturally he can have little or no information as to
the bank to ask for the board resolution, she was merely told to bring the what occurs in corporate meetings; and he must necessarily
receipts. The bank failed to categorically declare that Tena had no authority. rely upon the external manifestation of corporate consent.
This Court stresses the following: The integrity of commercial transactions can only be
maintained by holding the corporation strictly to the liability
. . . Corporate transactions would speedily come to a standstill were fixed upon it by its agents in accordance with law; and we
every person dealing with a corporation held duty-bound to would be sorry to announce a doctrine which would permit
disbelieve every act of its responsible officers, no matter how regular the property of man in the city of Paris to be whisked out of
they should appear on their face. This Court has observed his hands and carried into a remote quarter of the earth
in Ramirez vs. Orientalist Co., 38 Phil. 634, 654-655, that without recourse against the corporation whose name and
authority had been used in the manner disclosed in this
case. As already observed, it is familiar doctrine that if a
corporation knowingly permits one of its officers, or any other
agent, to do acts within the scope of an apparent authority,
and thus holds him out to the public as possessing power to
do those acts, the corporation will, as against any one who
has in good faith dealt with the corporation through such
agent, be estopped from denying his authority; and where it
is said "if the corporation permits this means the same as "if
the thing is permitted by the directing power of the
corporation." 16

In this light, the bank is estopped from questioning the authority of the bank
manager to enter into the contract of sale. If a corporation knowingly permits
one of its officers or any other agent to act within the scope of an apparent
authority, it holds the agent out to the public as possessing the power to do
those acts; thus, the corporation will, as against anyone who has in good
faith dealt with it through such agent, be estopped from denying the agent's
authority. 17

Unquestionably, petitioner has authorized Tena to enter into the Deed of


Sale. Accordingly, it has a clear legal duty to issue the board resolution
sought by respondent's. Having authorized her to sell the property, it

944
behooves the bank to confirm the Deed of Sale so that the buyers may enjoy
its full use.

The board resolution is, in fact, mere paper work. Nonetheless, it is paper
work necessary in the orderly operations of the register of deeds and the full
enjoyment of respondents' rights. Petitioner-bank persistently and
unjustifiably refused to perform its legal duty. Worse, it was less than candid
in dealing with respondents regarding this matter. In this light, the Court finds
it proper to assess the bank treble costs, in addition to the award of
damages.

WHEREFORE, the Petition is hereby DENIED and the assailed Decision and
Resolution AFFIRMED. The Temporary Restraining Order issued by this
Court is hereby LIFTED. Treble costs against petitioner.

SO ORDERED.

Melo, Purisima and Gonzaga-Reyes, JJ., concur.


Vitug, J., please see concurring opinion.

945
director; and that it was necessary to appoint a receiver to take charge of the
assets and receive the income of the corporation. Plaintiff prayed that a
G.R. No. L-23428 November 29, 1968 preliminary injunction ex-parte be issued restraining defendant from
exercising the functions of managing director and from disbursing and
DETECTIVE & PROTECTIVE BUREAU, INC., petitioner, disposing of its funds; that Jose M. Barredo be appointed receiver; that, after
vs. judgment, the injunction be made permanent and defendant be ordered to
THE HONORABLE GAUDENCIO CLORIBEL, in his capacity as Presiding render an accounting.
Judge of Branch VI, Court of First Instance of Manila, and FAUSTINO S.
ALBERTO, respondents. Herein respondent Judge, the Honorable Gaudencio Cloribel, set for hearing
plaintiff's prayer for ancillary relief and required the parties to submit their
Crispin D. Biazas and Associates and Jose S. Sarte for petitioner. respective memoranda. On June 18, 1964, respondent Judge granted the
Gaudencio T. Bocobo for respondents. writ of preliminary injunction prayed for, conditioned upon plaintiff's filing a
bond of P5,000.00. Plaintiff filed the bond, but while the same was pending
approval defendant Fausto S. Alberto filed, on July 1, 1964, a motion to admit
ZALDIVAR, J.:
a counter-bond for the purpose of lifting the order granting the writ of
preliminary injunction. Inspite of the opposition filed by plaintiff, respondent
The complaint, in Civil Case No. 56949 of the Court of First Instance of Judge issued, on August 5, 1964, an order admitting the counterbond and
Manila, dated May 4, 1964, filed by Detective and Protective Bureau, Inc., setting aside the writ of preliminary injunction.
therein plaintiff (petitioner herein) against Fausto S. Alberto, therein
defendant (respondent herein), for accounting with preliminary injunction and
On the belief that the order approving the counter-bond and lifting the writ of
receivership, alleged that plaintiff was a corporation duly organized and
preliminary injunction was contrary to law and the act of respondent Judge
existing under the laws of the Philippines; that defendant was managing
constituted a grave abuse of discretion, and that there was no plain, speedy
director of plaintiff corporation from 1952 until January 14, 1964; that in June,
and adequate remedy available to it, plaintiff filed with this Court the instant
1963, defendant illegally seized and took control of all the assets as well as
petition for certiorari, praying that a writ of preliminary injunction enjoining
the books, records, vouchers and receipts of the corporation from the
defendant Fausto S. Albert from exercising the functions of managing
accountant-cashier, concealed them illegally and refused to allow any
director be issued, and that the order dated August 5, 1964 of respondent
member of the corporation to see and examine the same; that on January
Judge approving the counter-bond and lifting the writ of preliminary injunction
14, 1964, the stockholders, in a meeting, removed defendant as managing
he had previously issued be set aside and declared null and void. The Court
director and elected Jose de la Rosa in his stead; that defendant not only
gave due course to the petition but did not issue a preliminary injunction.
had refused to vacate his office and to deliver the assets and books to Jose
de la Rosa, but also continued to perform unauthorized acts for and in behalf
of plaintiff corporation; that defendant had been required to submit a financial In his answer, now respondent Fausto S. Alberto traversed the material
statement and to render an accounting of his administration from 1952 but allegations of the petition, justified the order complained of, and prayed for
defendant has failed to do so; that defendant, contrary to a resolution the dismissal of the petition.
adopted by the Board of Directors on November 24, 1963, had been illegally
disposing of corporate funds; that defendant, unless immediately From the pleadings, it appears that the only issue to be resolved is whether
restrained ex-parte, would continue discharging the functions of managing the order of respondent Judge dated August 5, 1964, admitting and

946
approving the counter-bond of P5,000 and setting aside the writ of injunction without any formal petition of the parties and without having
preliminary injunction granted in his order dated June 18, 164, was issued followed the procedure prescribed by the statute. There was, however, a
contrary to law and with grave abuse of discretion. verbal application for the dissolution of the writ, based upon the ground of the
in suficiency of the complaint which was the basis of the application for the
Now petitioner contends that the setting aside of the order granting the writ issuance of said writ of preliminary injunction. This Court said:
was contrary to law and was done with a grave abuse of discretion, because:
(1) the motion to admit defendant's counter-bond was not supported by Section 169 of Act 1909 does not prescribe the manner of filing the
affidavits showing why the counter-bond should be admitted, as required by application to annul or modify a writ of preliminary injunction. It
Section 6 of Rule 58; (2) the preliminary injunction was not issued ex- simply states that if a temporary injunction be granted without notice,
parte but after hearing, and the admission of the counter-bond rendered said the defendant, at any time before trial, may apply, upon reasonable
writ ineffective; (3) the writ was granted in accordance with Rule 58 of the notice to the adverse party, to the judge who granted the injunction,
Rules of Court and established precedents' (4) public interest required that or to the judge of the court of which the action was brought, to
the writ be not set aside because respondent had arrogated unto himself all dissolve or modify the same.
the powers of petitioning corporation, to the irreparable damage of the
corporation; and that (5) the counter-bond could not compensate petitioner's On the strength of the decision in the above-cited case, this Court in Caluya,
damage. et al. vs. Ramos, et al.,2 said;

1. The first reason given by petitioner in support of its contention that the Petitioners' criticism that the motion to dissolve filed by the
dissolution of the writ of preliminary injunction was contrary to law is that the defendants in Civil Case No. 4634 was not verified, is also
motion to admit respondent's counter-bond for the dissolution of the writ was groundless inasmuch as even an indirect verbal application for the
not supported by affidavits as required by section 6 of Rule 58 of the Rules of dissolution of an ex parteorder of preliminary injunction has been
Court. The controverted motion, however, does not appear in the record. held to be a sufficient compliance with the provisions of Section 6 of
However, the record shows that respondent Alberto had filed a verified Rule 60 (Moran, Comments on the Rules of Court, Second Edition,
answer to the complaint and a verified opposition to the issuance of the writ Vol. II, p. 65, citing the case of Sy Yam Bio v. Barrios, etc., 63 Phil.
of preliminary injunction. 206), the obvious reason being that said rule does not prescribe the
form by which an application for the dissolution or modification of an
Regarding the necessity of verification of the motion for dissolution of a writ order of preliminary injunction should be presented.
of preliminary injunction, this Court has ruled that the requirement of
verification is not absolute but is dependent on the circumstances obtaining If according to the above rulings, Section 6 of Rule 60 (now sec. 6, Rule 58)
in a particular case. In the case of Sy Sam Bio, et al. vs. Barrios and Buyson of the Rules of Court did not require any form for the application for the
Lampa,1 the only question raised was whether the respondent Judge dissolution of the writ of preliminary injunction, then respondent Fausto
exceeded his jurisdiction and abused his discretion in setting aside an order Alberto's motion to lift the preliminary injunction in the court below need not
directing the issuance of a writ of preliminary injunction. In maintaining the be verified, and much less must the motion be supported by affidavits, as
affirmative, petitioners in that case alleged that the questioned order was urged by petitioner.
issued in violation of the provisions of Section 169 of Act 190(which is one of
the sources of Sec. 6 of Rule 58 of the revised Rules of Court)inasmuch as
the Judge set aside said order and directed the dissolution of the preliminary

947
However, in Canlas, et al. vs. Aquino, et al.,3 this Court ruled that a motion for 2. The second and third reasons alleged by petitioner in its petition for
the dissolution of a writ of preliminary injunction should be verified. In that certiorari assume that a preliminary injunction issued after hearing and in
case, respondent Tayag filed an unverified motion for the dissolution of a writ accordance with Rule 58 cannot be set aside. This contention is untenable.
of preliminary injunction, alleging that the same "would work great damage to The provision of Section 6 of Rule 58 that "the injunction may be refused, or,
the defendant who had already spend a considerable sum of money" and if granted ex parte, may be dissolved" can not be construed as putting
that petitioners "can be fully compensated for any damages that they may beyond the reach of the court the dissolution of an injunction which was
suffer." The court granted the motion and dissolved the preliminary injunction. granted after hearing. The reason is because a writ of preliminary injunction
In an original action for a writ of certiorari filed with this Court to annual said is an interlocutory order, and as such it is always under the control of the
order, this Court remarked in part: court before final judgment. Thus, in Caluya, et al. vs. Ramos, et al.,4this
Court said:
Petitioners herein are entitled to the writ prayed for. The motion of
respondent Tayag for the dissolution of the writ of preliminary The first contention of the petitioners is that, as said injunction was
injunction issued on October 22, 1959, was unverified.... issued after a hearing, the same cannot be dissolved, specially on
the strength of an unverified motion for dissolution and in the
From the precedents quoted above, as well as from the terminology of absence to support it. Reliance is placed on Section 6 of Rule 60 of
Section 6 of Rule 58 of the new Rules of Court, it is evident that whether the the Rules of Court which provides that "the injunction may be
application for the dissolution of the writ of preliminary injunction must be reduced, or, if granted ex parte, maybe dissolved," thereby arguing
verified or not depends upon the ground upon which such application is that if an injunction is not issued ex parte the same cannot be
based. If the application is based on the insufficiency of the complaint, the dissolved. The contention is clearly erroneous. Although said section
motion need not be verified. If the motion is based on the ground that the prescribes the grounds for objecting to, or for moving the dissolution
injunction would cause great damage to defendant while the plaintiff can be of, a preliminary injunction prior to its issuance or after its granting ex
fully compensated for such damages as he may suffer, the motion should be parte, it does not thereby outlaw a dissolution if the injunction has
verified. been issued after a hearing. This is to be so, because a writ of
preliminary injunction is an interlocutory order which is always under
In the instant case, it is alleged by petitioner that the motion for the the control of the court before final judgment. (Manila Electric
dissolution of the writ of preliminary injunction was not verified. This Company vs. Artiaga and Green, 50 Phil. 144, 147).
allegation was not denied in the answer. But because said motion does not
appear in the record of the case now before this Court, We cannot determine This Court has also ruled that the dissolution of a writ of preliminary
what are the grounds for the dissolution that are alleged therein, and so We injunction issued after hearing, even if the dissolution is ordered without
cannot rule on whether the motion should have been verified or not. This giving the other party an opportunity to be heard, does not constitute an
Court, therefore, has to rely on the order of respondent Judge, dated August abuse of discretion and may be cured not by certiorari but by appeal.
5, 1964, which states that "the filing of the counter-bond is in accordance with In Clarke vs. Philippine Ready Mix Concrete Co., Inc., et al.,5 one of the
law." Consequently, the first ground alleged by petitioner must be brushed issues presented was whether a writ of preliminary injunction granted the
aside. plaintiff by a trial court after hearing, might be dissolved upon an ex parte
application by the defendant, and this Court ruled that:

948
The action of a trial court in dissolving a writ of preliminary injunction The manager shall be elected by the Board of Directors from among
already issued after hearing, without giving petitioner an opportunity its members.... (Record, p. 48)
to be heard, does not constitute lack or excess of jurisdiction or an
abuse of discretion, and any irregularity committed by the trial court If the managing director-elect was not qualified to become managing director,
on this score may be cured not by certiorari but by appeal. respondent Fausto Alberto could not be compelled to vacate his office and
cede the same to the managing director-elect because the by-laws of the
3. The fourth reason alleged by petitioner in support of its stand is that public corporation provides in Article IV, Section 1 that "Directors shall serve until
interest demanded that the writ enjoining respondent Fausto Alberto from the election and qualification of their duly qualified successor."
exercising the functions of managing director be maintained. Petitioner
contended that respondent Alberto had arrogated to himself the power of the 4. The fifth reason alleged by herein petitioner in support of its contention
Board of Directors of the corporation because he refused to vacate the office that respondent Judge gravely abused his discretion when he lifted the
and surrender the same to Jose de la Rosa who had been elected managing preliminary injunction upon the filing of the counter-bond was that said
director by the Board to succeed him. This assertion, however, was disputed counter-bond could not compensate for the irreparable damage that the
by respondent Alberto who stated that Jose de la Rosa could not be elected corporation would suffer by reason of the continuance of respondent Fausto
managing director because he did not own any stock in the corporation. Alberto as managing director of the corporation. Respondent Alberto, on the
contrary, contended that he really was the owner of the controlling interest in
There is in the record no showing that Jose de la Rosa owned a share of the business carried on the name of the petitioner, having invested therein a
stock in the corporation. If he did not own any share of stock, certainly he total of P57,727.29 as against the sum of P4,000 only invested by one other
could not be a director pursuant to the mandatory provision of Section 30 of director, Jose M. Barredo. We find that there was a question as to who own
the Corporation Law, which in part provides: the controlling interest in the corporation. Where ownership is in dispute, the
party in control or possession of the disputed interest is presumed to have
There is in the record no showing that Jose de la Rosa owned a share of the better right until the contrary is adjudged, and hence that party should not
stock in the corporation. If he did not own any share of stock, certainly he be deprived of the control or possession until the court is prepared to
could not be a director pursuant to the mandatory provision of Section 30 of adjudicate the controverted right in favor of the other party.6
the Corporation Law, which in part provides:
Should it be the truth that respondent Alberto is the controlling stockholder,
Sec. 30. Every director must own in his own right at least one share then the damages said respondent would suffer would be the same, if not
of the capital stock of the stock corporation of which he is a director, more, as the damages that the corporation would suffer if the injunction were
which stock shall stand in his name on the books of the maintained. If the bond of P5,000 filed by petitioner for the injunction would
corporations.... be sufficient to answer for the damages that would be suffered by respondent
Alberto by reason of the injunction, there seems to be no reason why the
If he could not be a director, he could also not be a managing director of the same amount would not be sufficient to answer for the damages that might
corporation, pursuant to Article V, Section 3 of the By-Laws of the be suffered by the petitioning corporation by reason of the lifting of the
Corporation which provides that: injunction. The following ruling of this Court has a persuasive application in
this case:

949
The rule that a court should not, by means of a preliminary It being our considered view that respondent Judge had not committed grave
injunction, transfer property in litigation from the possession of one abuse of discretion in issuing the order dated August 5, 1964 lifting the writ of
party to another is more particularly applicable where the legal title is preliminary injunction which had previously been granted in the order dated
in dispute and the party having possession asserts ownership in June 18, 1964, and the herein petition for certiorari having been filed without
himself.7 previously complying with a well settled procedural requirement, there is no
alternative for this Court but to order its dismissal.
Let it be stated, in relation to all the reason given by petitioner, that it is a
settled rule that the issuance of the writ of preliminary injunction as an WHEREFORE, the instant petition for certiorari with preliminary injunction is
ancillary or preventive remedy to secure the rights of a party in a pending dismissed, with costs againsts the petitioner. It is so ordered.
case is entirely within the discretion of the court taking cognizance of the
case the only limitation being that this discretion should be exercised Concepcion, C.J., Reyes, J.B.L., Dizon, Makalintal, Sanchez, Castro,
based upon the grounds and in the manner provided by law,8 and it is equally Fernando and Capistrano, JJ., concur.
well settled that a wide latitude is given under Section 7 of Rule 58 of the
Rules of Court to the trial court to modify or dissolve the injunction as justice
may require. The court which is to exercise that discretion is the trial court,
not the appellate court.9 The exercise of sound judicial discretion by the lower
court in injunctive matters should not be interfered with except in cases of
manifest abuse.10 In the instant case, We find that petitioner failed to show
manifest abuse of discretion by respondent Judge in setting aside the writ of
preliminary injunction.

There is, however, one vital reason why the instant petition for certiorari
should be denied. And it is, that from the order dissolving the writ of
preliminary injunction, the petitioner has gone directly to this Court without
giving the respondent Judge (or trial court) a chance or opportunity to correct
his error, if any, in an appropriate motion for reconsideration. An omission to
comply with this procedural requirement justifies a denial of the writ applied
for.11

The instant case is not one of the exceptions in the application of this rule,
which are: where the questions of jurisdiction has been squarely raised,
argued before, submitted to, and met and decided by the respondent court;
where the questioned order is a patent nullity; and where there is a
deprivation of the petitioner's fundamental right to due process. 12

950
THE COURT OF APPEALS, GRACE VILLAGE ASSOCIATION, INC.,
ALEJANDRO G. BELTRAN, and ERNESTO L. GO, respondents.

MENDOZA, J.:

The question for decision in this case is the right of petitioner's representative
to sit in the board of directors of respondent Grace Village Association, Inc.
as a permanent member thereof. For fifteen years from 1975 until 1989
petitioner's representative had been recognized as a "permanent director" of
the association. But on February 13, 1990, petitioner received notice from the
association's committee on election that the latter was "reexamining"
(actually, reconsidering) the right of petitioner's representative to continue as
an unelected member of the board. As the board denied petitioner's request
to be allowed representation without election, petitioner brought an action
for mandamus in the Home Insurance and Guaranty Corporation. Its action
was dismissed by the hearing officer whose decision was subsequently
affirmed by the appeals board. Petitioner appealed to the Court of Appeals,
which in turn upheld the decision of the HIGC's appeals board. Hence this
petition for review based on the following contentions:

1. The Petitioner herein has already acquired a vested right


to a permanent seat in the Board of Directors of Grace
Village Association;

2. The amended By-laws of the Association drafted and


promulgated by a Committee on December 20, 1975 is valid
and binding; and

3. The Practice of tolerating the automatic inclusion of


petitioner as a permanent member of the Board of Directors
G.R. No. 108905 October 23, 1997
of the Association without the benefit of election is allowed
under the law. 1
GRACE CHRISTIAN HIGH SCHOOL, petitioner,
vs.
Briefly stated, the facts are as follows:

951
Petitioner Grace Christian High School is an educational institution offering qualified. GRACE CHRISTIAN HIGH SCHOOL
preparatory, kindergarten and secondary courses at the Grace Village in representative is a permanent Director of the
Quezon City. Private respondent Grace Village Association, Inc., on the other ASSOCIATION.
hand, is an organization of lot and/or building owners, lessees and residents
at Grace Village, while private respondents Alejandro G. Beltran and Ernesto This draft was never presented to the general membership for approval.
L. Go were its president and chairman of the committee on election, Nevertheless, from 1975, after it was presumably submitted to the board, up
respectively, in 1990, when this suit was brought. to 1990, petitioner was given a permanent seat in the board of directors of
the association. On February 13, 1990, the association's committee on
As adopted in 1968, the by-laws of the association provided in Article IV, as election in a letter informed James Tan, principal of the school, that "it was
follows: the sentiment that all directors should be elected by members of the
association" because "to make a person or entity a permanent Director would
The annual meeting of the members of the Association shall deprive the right of voters to vote for fifteen (15) members of the Board," and
be held on the first Sunday of January in each calendar year "it is undemocratic for a person or entity to hold office in perpetuity." 4For this
at the principal office of the Association at 2:00 P.M. where reason, Tan was told that "the proposal to make the Grace Christian High
they shall elect by plurality vote and by secret balloting, the School representative as a permanent director of the association, although
Board of Directors, composed of eleven (11) members to previously tolerated in the past elections should be reexamined." Following
serve for one (1) year until their successors are duly elected this advice, notices were sent to the members of the association that the
and have qualified. 2 provision on election of directors of the 1968 by-laws of the association
would be observed.
It appears, that on December 20, 1975, a committee of the board of directors
prepared a draft of an amendment to the by-laws, reading as follows: 3 Petitioner requested the chairman of the election committee to change the
notice of election by following the procedure in previous elections, claiming
VI. ANNUAL MEETING that the notice issued for the 1990 elections ran "counter to the practice in
previous years" and was "in violation of the by-laws (of 1975)" and
"unlawfully deprive[d] Grace Christian High School of its vested right [to] a
The Annual Meeting of the members of the Association shall
permanent seat in the board." 5
be held on the second Thursday of January of each year.
Each Charter or Associate Member of the Association is
entitled to vote. He shall be entitled to as many votes as he As the association denied its request, the school brought suit
has acquired thru his monthly membership fees for mandamus in the Home Insurance and Guaranty Corporation to compel
onlycomputed on a ratio of TEN (P10.00) PESOS for one the board of directors of the association to recognize its right to a permanent
vote. seat in the board. Petitioner based its claim on the following portion of the
proposed amendment which, it contended, had become part of the by-laws of
the association as Article VI, paragraph 2, thereof:
The Charter and Associate Members shall elect the Directors
of the Association. The candidates receiving the first
fourteen (14) highest number of votes shall be declared and The Charter and Associate Members shall elect the Directors
proclaimed elected until their successors are elected and of the Association. The candidates receiving the first

952
fourteen (14) highest number of votes shall be declared and laws which, although implemented in the past, had not yet been ratified by
proclaimed elected until their successors are elected and the members of the association nor approved by competent authority"; that,
qualified. GRACE CHRISTIAN HIGH SCHOOL on the contrary, in the meeting held on April 17, 1990, the directors of the
representative is a permanent Director of the association declared "the proposed by-law dated December 20, 1975
ASSOCIATION. prepared by the committee on by-laws . . . null and void" and the by-laws of
December 17, 1968 as the "prevailing by-laws under which the association is
It appears that the opinion of the Securities and Exchange Commission on to operate until such time that the proposed amendments to the by-laws are
the validity of this provision was sought by the association and that in reply to approved and ratified by a majority of the members of the association and
the query, the SEC rendered an opinion to the effect that the practice of duly filed and approved by the pertinent government agency." The hearing
allowing unelected members in the board was contrary to the existing by- officer rejected petitioner's contention that it had acquired a vested right to a
laws of the association and to 92 of the Corporation Code (B.P. Blg. 68). permanent seat in the board of directors. He held that past practice in
election of directors could not give rise to a vested right and that departure
Private respondent association cited the SEC opinion in its answer. from such practice was justified because it deprived members of association
Additionally, the association contended that the basis of the petition of their right to elect or to be voted in office, not to say that "allowing the
for mandamus was merely "a proposed by-laws which has not yet been automatic inclusion of a member representative of petitioner as permanent
approved by competent authority nor registered with the SEC or HIGC." It director [was] contrary to law and the registered by-laws of respondent
argued that "the by-laws which was registered with the SEC on January 16, association." 8
1969 should be the prevailing by-laws of the association and not the
proposed amended by-laws." 6 The appeals board of the HIGC affirmed the decision of the hearing officer in
its resolution dated September 13, 1990. It cited the opinion of the SEC
In reply, petitioner maintained that the "amended by-laws is valid and based on 92 of the Corporation Code which reads:
binding" and that the association was estopped from questioning the by-
laws. 7 92. Election and term of trustees. Unless otherwise
provided in the articles of incorporation or the by-laws, the
A preliminary conference was held on March 29, 1990 but nothing substantial board of trustees of non-stock corporations, which may be
was agreed upon. The parties merely agreed that the board of directors of more than fifteen (15) in number as may be fixed in their
the association should meet on April 17, 1990 and April 24, 1990 for the articles of incorporation or by-laws, shall, as soon as
purpose of discussing the amendment of the by-laws and a possible organized, so classify themselves that the term of office of
amicable settlement of the case. A meeting was held on April 17, 1990, but one-third (1/3) of the number shall expire every year; and
the parties failed to reach an agreement. Instead, the board adopted a subsequent elections of trustees comprising one-third (1/3)
resolution declaring the 1975 provision null and void for lack of approval by of the board of trustees shall be held annually and trustees
members of the association and the 1968 by-laws to be effective. so elected shall have a term of three (3) years. Trustees
thereafter elected to fill vacancies occurring before the
expiration of a particular term shall hold office only for the
On June 20, 1990, the hearing officer of the HIGC rendered a decision
unexpired period.
dismissing petitioner's action. The hearing officer held that the amended by-
laws, upon which petitioner based its claim, "[was] merely a proposed by-

953
The HIGC appeals board denied claims that the school "[was] being vote at a regular or special meeting. And provided, further,
deprived of its right to be a member of the Board of Directors of That the Director of the Bureau of Commerce and Industry
respondent association," because the fact was that "it may nominate shall not hereafter file an amendment to the by-laws of any
as many representatives to the Association's Board as it may deem bank, banking institution or building and loan association,
appropriate." It said that "what is merely being upheld is the act of unless accompanied by certificate of the Bank
the incumbent directors of the Board of correcting a long standing Commissioner to the effect that such amendments are in
practice which is not anchored upon any legal basis." 9 accordance with law.

Petitioner appealed to the Court of Appeals but petitioner again lost as the The proposed amendment to the by-laws was never approved by the
appellate court on February 9, 1993, affirmed the decision of the HIGC. The majority of the members of the association as required by these provisions of
Court of Appeals held that there was no valid amendment of the association's the law and by-laws. But petitioner contends that the members of the
by-laws because of failure to comply with the requirement of its existing by- committee which prepared the proposed amendment were duly authorized to
laws, prescribing the affirmative vote of the majority of the members of the do so and that because the members of the association thereafter
association at a regular or special meeting called for the adoption of implemented the provision for fifteen years, the proposed amendment for all
amendment to the by-laws. Article XIX of the by-laws provides: 10 intents and purposes should be considered to have been ratified by them.
Petitioner contends: 11
The members of the Association by an affirmative vote of the
majority at any regular or special meeting called for the Considering, therefore, that the "agents" or committee were
purpose, may alter, amend, change or adopt any new by- duly authorized to draft the amended by-laws and the acts
laws. done by the "agents" were in accordance with such authority,
the acts of the "agents" from the very beginning were lawful
This provision of the by-laws actually implements 22 of the Corporation Law and binding on the homeowners (the principals) per
(Act No. 1459) which provides: sewithout need of any ratification or adoption. The more has
the amended by-laws become binding on the homeowners
22. The owners of a majority of the subscribed capital when the homeowners followed and implemented the
stock, or a majority of the members if there be no capital provisions of the amended by-laws. This is not merely
stock, may, at a regular or special meeting duly called for the tantamount to tacit ratification of the acts done by duly
purpose, amend or repeal any by-law or adopt new by-laws. authorized "agents" but express approval and confirmation of
The owners of two-thirds of the subscribed capital stock, or what the "agents" did pursuant to the authority granted to
two-thirds of the members if there be no capital stock, may them.
delegate to the board of directors the power to amend or
repeal any by-law or to adopt new by-laws: Provided, Corollarily, petitioner claims that it has acquired a vested right to a permanent
however, That any power delegated to the board of directors seat in the board. Says petitioner:
to amend or repeal any by-law or adopt new by-laws shall be
considered as revoked whenever a majority of the The right of the petitioner to an automatic membership in the
stockholders or of the members of the corporation shall so board of the Association was granted by the members of the

954
Association themselves and this grant has been If fact, the truth is that this is allowed and is being practiced
implemented by members of the board themselves all by some corporations duly organized and existing under the
through the years. Outside the present membership of the laws of the Philippines.
board, not a single member of the Association has registered
any desire to remove the right of herein petitioner to an One example is the Plus XII Catholic Center, Inc. Under the
automatic membership in the board. If there is anybody who by-laws of this corporation, that whoever is the Archbishop of
has the right to take away such right of the petitioner, it Manila is considered a member of the board of trustees
would be the individual members of the Association through without benefit of election. And not only that. He also
a referendum and not the present board some of the automatically sits as the Chairman of the Board of Trustees,
members of which are motivated by personal interest. again without need of any election.

Petitioner disputes the ruling that the provision in question, giving Another concrete example is the Cardinal Santos Memorial
petitioner's representative a permanent seat in the board of the Hospital, Inc. It is also provided in the by-laws of this
association, is contrary to law. Petitioner claims that that is not so corporation that whoever is the Archbishop of Manila is
because there is really no provision of law prohibiting unelected considered a member of the board of trustees year after year
members of boards of directors of corporations. Referring to 92 of without benefit of any election and he also sits automatically
the present Corporation Code, petitioner says: as the Chairman of the Board of Trustees.

It is clear that the above provision of the Corporation Code It is actually 28 and 29 of the Corporation Law not 92 of the present
only provides for the manner of election of the members of law or 29 of the former one which require members of the boards of
the board of trustees of non-stock corporations which may directors of corporations to be elected. These provisions read:
be more than fifteen in number and which manner of election
is even subject to what is provided in the articles of 28. Unless otherwise provided in this Act, the corporate
incorporation or by-laws of the association thus showing that powers of all corporations formed under this Act shall be
the above provisions [are] not even mandatory. exercised, all business conducted and all property of such
corporations controlled and held by a board of not less than
Even a careful perusal of the above provision of the five nor more than eleven directors to be elected from
Corporation Code would not show that it prohibits a non- among the holders of stock or, where there is no stock, from
stock corporation or association from granting one of its the members of the corporation: Provided, however, That in
members a permanent seat in its board of directors or corporations, other than banks, in which the United States
trustees. If there is no such legal prohibition then it is has or may have a vested interest, pursuant to the powers
allowable provided it is so provided in the Articles of granted or delegated by the Trading with the Enemy Act, as
Incorporation or in the by-laws as in the instant case. amended, and similar Acts of Congress of the United States
relating to the same subject, or by Executive Order No. 9095
xxx xxx xxx of the President of the United States, as heretofore or
hereafter amended, or both, the directors need not be

955
elected from among the holders of the stock, or, where there be electedfrom among the holders of stocks, or where there
is no stock from the members of the corporation. (emphasis is no stock, from among the members of the corporation,
added) who shall hold office for one (1) year and until their
successors are elected and qualified. (Emphasis added)
29. At the meeting for the adoption of the original by-laws,
or at such subsequent meeting as may be then determined, These provisions of the former and present corporation law leave no room for
directors shall be elected to hold their offices for one year doubt as to their meaning: the board of directors of corporations must be
and until their successors are elected and qualified. elected from among the stockholders or members. There may be
Thereafter the directors of the corporation shall be elected corporations in which there are unelected members in the board but it is clear
annually by the stockholders if it be a stock corporation or by that in the examples cited by petitioner the unelected members sit as ex
the members if it be a nonstock corporation, and if no officio members, i.e., by virtue of and for as long as they hold a particular
provision is made in the by-laws for the time of election the office. But in the case of petitioner, there is no reason at all for its
same shall be held on the first Tuesday after the first representative to be given a seat in the board. Nor does petitioner claim a
Monday in January. Unless otherwise provided in the by- right to such seat by virtue of an office held. In fact it was not given such seat
laws, two weeks' notice of the election of directors must be in the beginning. It was only in 1975 that a proposed amendment to the by-
given by publication in some newspaper of general laws sought to give it one.
circulation devoted to the publication of general news at the
place where the principal office of the corporation is Since the provision in question is contrary to law, the fact that for fifteen
established or located, and by written notice deposited in the years it has not been questioned or challenged but, on the contrary, appears
post-office, postage pre-paid, addressed to each to have been implemented by the members of the association cannot
stockholder, or, if there be no stockholders, then to each forestall a later challenge to its validity. Neither can it attain validity through
member, at his last known place of residence. If there be no acquiescence because, if it is contrary to law, it is beyond the power of the
newspaper published at the place where the principal office members of the association to waive its invalidity. For that matter the
of the corporation is established or located, a notice of the members of the association may have formally adopted the provision in
election of directors shall be posted for a period of three question, but their action would be of no avail because no provision of the by-
weeks immediately preceding the election in at least three laws can be adopted if it is contrary to law. 13
public places, in the place where the principal office of the
corporation is established or located. (Emphasis added) It is probable that, in allowing petitioner's representative to sit on the board,
the members of the association were not aware that this was contrary to law.
The present Corporation Code (B.P. Blg. 68), which took effect on May 1, It should be noted that they did not actually implement the provision in
1980, 12 similarly provides: question except perhaps insofar as it increased the number of directors from
11 to 15, but certainly not the allowance of petitioner's representative as an
23. The Board of Directors or Trustees. Unless otherwise unelected member of the board of directors. It is more accurate to say that
provided in this Code, the corporate powers of all the members merely tolerated petitioner's representative and tolerance
corporations formed under this Code shall be exercised, all cannot be considered ratification.
business conducted and all property of such corporations
controlled and held by the board of directors or trustees to

956
Nor can petitioner claim a vested right to sit in the board on the basis of
"practice." Practice, no matter how long continued, cannot give rise to any
vested right if it is contrary to law. Even less tenable is petitioner's claim that
its right is "coterminus with the existence of the association." 14

Finally, petitioner questions the authority of the SEC to render an opinion on


the validity of the provision in question. It contends that jurisdiction over this
case is exclusively vested in the HIGC.

But this case was not decided by the SEC but by the HIGC. The HIGC
merely cited as authority for its ruling the opinion of the SEC chairman. The
HIGC could have cited any other authority for the view that under the law
members of the board of directors of a corporation must be elected and it
would be none the worse for doing so.

WHEREFORE, the decision of the Court of Appeals is AFFIRMED.

SO ORDERED.

Puno and Torres, Jr., JJ., concur.

957
Domingo Ponce; and P8,000 spent by the latter in his trip to the United
States, or a total sum of P18,690, plus interest, or such sum as may be found
G.R. No. L-5883 November 28, 1953 after the accounting shall have been rendered to have been misspent,
misapplied, missappropriated and converted by the petitioner Domingo
DOMINGO PONCE AND BUHAY L. PONCE, petitioners, Ponce to his own use and benefit; that on 18 May 1951 the plaintiff in that
vs. case, the respondent Potenciano Gapol in this case, filed a motion praying
DEMETRIO B. ENCARNACION, Judge of the Court of First Instance of that the petitioners be removed as members of the board of directors which
Manila, Branch I, and POTENCIANO GAPOL, respondents. was denied by the court; that on 3 January 1952 respondent Potenciano
Gapol filed a petition (civil No. 15445, Exhibit L), praying for an order
directing him to a call a meeting of the stockholders of the corporation and to
Marcelino Lontok for petitioners.
preside at such meeting in accordance with section 26 of the Corporation
Zavalla, Bautista and Nuevas for respondents.
law; that two days later, without notice to the petitioners and to the other
members of the board of directors and in violation of the Rules of Court
PADILLA, J.: which require that the adverse parties be notified of the hearing of the motion
three days in advance, the respondent court issued the order as prayed for
This is a petition for a writ of certiorari to annul an order of the respondent (Exhibit M); that the petitioners learned only of this order of the court on 27
court granting Potenciano Gapol authority, pursuant to section 26, Act No. February, when the Bank of America refused to recognize the new board of
1459, otherwise known as the Corporation Law, to call a meeting of the directors elected at such meeting and returned the checks drawn upon it by
stockholders of the Dagunoy Enterprises, Inc. and to preside at such meeting the said board of directors; that the election of Juanito R. Tianzon as member
by giving proper notice to the stockholders, as required by law or by laws of of the board of directors of the corporation he must be a member of
the corporation, until after the majority of the stockholders present and the Legionarios del Trabajo, as required and provided for in article 7 of the
qualified to vote shall have chosen one of them to act as presiding officer of by-laws of the corporation; that on 5 March the petitioners filed a petition in
the meeting; another order denying a motion of the petitioners to have the the respondent court to have the order of 5 January set aside but on April,
previous order set aside; and a third order denying a motion to the same the date set for the hearing of the petition, as the respondent judge was on
effect as the one previously filed. leave vacation judge directed its transfer to the branch of the respondent
judge; that without having set the motion for hearing, the respondent court
The petitioners aver that the Daguhoy Enterprises, Inc., was duly registered denied the motion of 5 March in its order of 7 May; that on 14 May the
as such on 24 June 1948; that on 16 April 1951 at a meeting duly called, the petitioners filed another motion inviting the attention of the respondent court
voluntary dissolution of the corporation and the appointment of Potenciano to the irregularity and illegality of its procedure and setting the motion for
Gapol as receiver were agreed upon and to that end a petitioner Domingo hearing on 21 May, but the court denied the motion by its order of 13 June.
Ponce; that instead of filing the petition for voluntary dissolution of the of the
corporation as agreed upon, the respondent Potenciano Gapol, who is the The only question to determine in this case is whether under and pursuant to
largest stockholder, charged his mind and filed a complaint in the Court of section 26 of Act No. 1459, known as the Corporation law, the respondent
First Instance of Manila (civil No. 13753) to compel the petitioners to render court may issue the order complained of. Said section provides:
an accounting of the funds and assets of the corporation, to reimburse it,
jointly and severally, in the sum of P4,500, the purchase price of a parcel of Whenever, from any cause, there is no person authorized to call a
land acquired by the corporation; P6,190 loaned to the wife of petitioner meeting, or when the officer authorized to do so refuses, fails or

958
neglects to call a meeting, any judge of a Court of First Instance on served upon the board of directors. The respondent court was satisfied that
the showing of good cause therefor, may issue an order to any there was a showing of good cause for authorizing the respondent
stockholder or member of a corporation, directing him to call a Potenciano Gapol to call a meeting of the stockholders for the purpose of
meeting of the corporation by giving the proper notice required by electing the board of directors as required and provided for in the by-laws,
this Act or by-laws; and if there be no person legally authorized to because the chairman of the board of directors called upon to do so had
preside at such meeting, the judge of the Court of First Instance may failed, neglected, or refused to perform his duty. It may be likened to a writ of
direct the person calling the meeting to preside at the same until a preliminary injunction or of attachment which may be issued ex-parte upon
majority of the members or stockholders representing a majority of compliance with the requirements of the rules and upon the court being
the stock members or stockholders presenting a majority of the stock satisfied that the same should be issue. Such provisional reliefs have not
present and permitted by law to be voted have chosen one of their been deemed and held as violative of the due process of law clause of the
number to act as presiding officer for the purposes of the meeting. Constitution.

On the showing of good cause therefor, the court may authorize a In several state of the Union1 the remedy which may be availed of our
stockholder to call a meeting and to preside threat until the majority resorted to in a situation such as the one brought about in this case is
stockholders representing a majority strockholders representing a majority of mandamus to compel the officer or incumbent board of directors to perform a
the stock present and permitted to be voted shall have chosen one among duties specifically enjoined by law or by-laws, to wit: to call a meeting of the
them to preside it. And this showing of good cause therefor exists when the stockholders. Dela ware is the estate that has a law similar to ours and there
court is apprised of the fact that the by-laws of the corporation require the the chancellor of a chancery court may summarily issue or enter an order
calling of a general meeting of the stockholders to elect the board of directors authorizing a stockholder to call a meeting of the stockholders of the
but call for such meeting has not been done. corporation and preside thereat.2 It means that the chancellor may issue
such order without notice and hearing.
Article 9 of the by-laws of the Daguhoy Enterprises, Inc., provides:
That the relief granted by the respondent court lies within its jurisdiction is not
The Board of Directors shall compose of five (5) members who shall disputed. Having the authority to grant the relief, the respondent court did not
be elected by the stockholders in a general meeting called for that exceed its jurisdiction; nor did it abuse its discretion in granting it.
purpose which shall be held every even year during the month of
January. With persistency petitioners claim that they have been deprived of their right
without due process of law. They had no right to continue as directors of the
Article 20 of the by-laws in part provides: corporation unless reflected by the stockholders in a meeting called for that
purpose every even year. They had no right to a hold-over brought about by
. . . Regular general meetings are those which shall be called for the failure to perform the duty incumbent upon one of them. If they felt that
every even year, . . . . they were sure to be reelected, why did they fail, neglect, or refuse to call the
meeting to elect the members of the board? Or, why did they not seek their
reelection at the meeting called to elect the directors pursuant to the order of
The requirement that "on the showing of good cause therefor," the court may
the respondent court.
grant to a stockholder the authority to call such meeting and to preside
thereat does not mean that the petition must be set for hearing with notice

959
The alleged illegality of the election of one member of the board of directors It appears that the Binalbagan Estate, Inc., is a corporation having its
at the meeting called by the respondent Potenciano Gapol as authorized by principal plant in Occidental Negros where it is engaged in the manufacture
the court being subsequent to the order complained of cannot affect the of raw sugar from canes grown upon farms accessible to its central. In July,
validity and legality of the order. If it be true that one of the directors elected 1924, the possessors of a majority of the shares of the Binalbagan Estate,
at the meting called by the respondent Potenciano Gapol, as authorized by Inc., formed a voting trust composed of three members, namely, Salvador
the order of the court complained of, was not qualified in accordance with the Laguna, Segunda Monteblanco, and Arthur F. Fisher, as trustee. By the
provisions of the by-laws, the remedy of an aggrieved party would be quo a document constituting this voting trust the trustees were authorized to
warranto. Also, the alleged previous agreement to dissolve the corporation represent and vote the shares pertaining to their constituents, and to this end
does not affect or render illegal the order issued by the respondent court. The the shareholders undertook to assign their shares to the trustees on the
petition is denied, with costs against the petitioners. books of the company. The total number of outstanding shares of the
corporation is somewhat over 5,500, while the number of shares controlled
G.R. No. L-26555 November 16, 1926 by the voting trust is less than 3,000.

BALDOMERO ROXAS, ENRIQUE ECHAUS and ROMAN J. On February 1, 1926, the general annual meeting of the shareholders of the
LACSON, petitioners, Binalbagan Estate, Inc., took place, at which Mr. J. P. Heilbronn appeared as
vs. representative of the voting trust, his authority being recognized by the
Honorable MARIANO DE LA ROSA, Auxiliary Judge of First Instance of holders of all the other shares present at this meeting. Upon said occasion
Occidental Negros, AGUSTIN CORUNA, MAURO LEDESMA and Heilbronn, by virtue of controlling the majority of the shares, was able to
BINALBAGAN ESTATE, INC., respondents. nominate and elect a board of directors to his own liking, without opposition
from the minority. After the board of directors had been thus elected and had
Roman J. Lacson, for petitioners. qualified, they chose a set of officers constituting of Jose M. Yusay,
The respondent judge in his own behalf. president, Timoteo Unson, vice-president, Jose G. Montalvo, secretary-
The respondent corporation in its own behalf. treasurer, and H. W. Corp and Agustin Coruna, as members. Said officials
R. Nolan and Feria and La O for the respondents Coruna and Ledesma. immediately entered upon the discharged of their duties and have continued
in possession of their respective offices until the present time.

Since the creation of the voting trust there have been a number of vacancies
caused by resignation or the absence of members from the Philippine
STREET, J.:
Islands, with the result that various substitutions have been made in the
personnel of the voting trust. At the present time the petitioners Roxas,
This is an original petition for the writ of certiorari whereby the petitioners, Echaus, and Lacson presumably constitute its membership. We say
Baldomeo Roxas, Enrique Echaus, and Roman J. Lacson, seek to procure presumably, because in the present proceedings an issue of fact is made by
the abrogation of an order of the respondent judge granting a preliminary the respondents upon the point whether the three individuals named have
injunction in an action in the Court of First Instance of Occidental Negros, been regularly substituted for their several predecessors. In the view we take
instituted by Agustin Coruna and Mauro Ledesma against the petitioners and of the case it is not necessary to determine this issue; and we shall assume
the Binalbagan Estate, Inc. The cause is now before us upon the issues provisionally that the three petitioners are the lawful components of the
made by the answers filed by the respondents. voting trust.

960
Although the present officers of the Binalbagan Estate, Inc., were elected by stock entitled to vote (Act No. 1459, sec. 34); while vacancies in the board,
the representative of the voting trust, the present trustee are apparently when they exist, can be filled by mere majority vote, (Act No. 1459, sec. 25).
desirous of ousting said officers, without awaiting the termination of their Moreover, the law requires that when action is to be taken at a special
official terms at the expiration of one year from the date of their election. In meeting to remove the directors, such purpose shall be indicated in the call
other to effect this purpose the petitioners in their character as members of (Act No. 1459, sec. 34).
the voting trust, on August 2, 1926, caused the secretary of the Binalbagan
Estate, Inc., to issue to the shareholders a notice calling for a special general Now, upon examining into the number of shares controlled by the voting
meeting of shareholders to be held at 10 a. m., on August 16, 1926, "for the trust, it will be seen that, while the trust controls a majority of the stock, it
election of the board of directors, for the amendment of the By-Laws, and for does not have a clear two-thirds majority. It was therefore impolitic for the
any other business that can be dealt with in said meeting." petitioners, in forcing the call for the meeting of August 16, to come out
frankly and say in the notice that one of the purpose of the meeting was to
Within a few days after said notice was issued Agustin Corua, as member of removed the directors of the corporation from office. Instead, the call was
the existing board, and Mauro Ledesma, as a simple shareholder of the limited to the election of the board of directors, it being the evident intention
corporation, instituted a civil action (No. 3840) in the Court of First Instance of the voting trust to elect a new board as if the directorate had been then
of Occidental Negros against the trustees and the Binalbagan Estate, Inc., vacant.
for the purpose of enjoining the meeting completed in the notice above-
mentioned. But the complaint in civil No. 3840 directly asserts that the members of the
present directorate were regularly elected at the general annual meeting held
In response to a proper for a preliminary injunction, in connection with said in February, 1926; and if that assertion be true, the proposal to elect, another
action, the respondent judge issued the restraining order, or preliminary directorate, as per the call of August 2, if carried into effect, would result in
injunction, which gave rise to the present petition for the writ of certiorari. In the election of a rival set of directors, who would probably need the
the dispositive part of said order the Binalbagan Estate, Inc., its lawyers, assistance of judgment of court in an independent action of quo warrantoto
agents, representatives, and all others who may be assisting or corroborating get them installed into office, even supposing that their title to the office could
with them, are restrained from holding the general shareholders' meeting be maintained. That the trial judge had jurisdiction to forestall that step and
called for the date mentioned and from electing new directors for the enjoin the contemplated election is a matter about which there cannot be the
company in substitution of the present incumbents, said injunction to be slightest doubt. The law contemplates and intends that there will be one of
effective until further order of the court. it is now asserted here by the directors at a time and that new directors shall be elected only as vacancies
petitioners that the making of this order was beyond the legitimate powers of occur in the directorate by death, resignation, removal, or
the respondent judge, and it is accordingly prayed that said order be set otherwise. lawphil.net
aside.
It is instituted that there was some irregularity or another in the election of the
We are of the opinion that this contention is untenable and that the present directorate. We see nothing upon which this suggestion can be
respondent judge acted within his legitimate powers in making the order safely planted; And at any rate the present board of directors are de
against which relief is sought. In order to expose the true inwardness of the factoincumbents of the office whose acts will be valid until they shall be
situation before us it is necessary to take not of the fact that under the law lawfully removed from the office or cease from the discharge of their
the directors of a corporation can only be removed from office by a vote of functions. In this case it is not necessary for us to agitate ourselves over the
the stockholders representing at least two-thirds of the subscribed capital question whether the respondent judge properly exercised his judicial

961
discretion in granting the order complained of. If suffices to know that in The Antecedents
making the order he was acting within the limits of his judicial powers.
Korean Airlines (KAL) is a corporation established and registered in the
It will be noted that the order in question enjoins the defendants from holding Republic of South Korea and licensed to do business in the Philippines. Its
the meeting called for August 16; and said order must not be understood as general manager in the Philippines is Suk Kyoo Kim, while its appointed
constituting any obstacle for the holding of the regular meeting at the time counsel was Atty. Mario Aguinaldo and his law firm.
appointed in the by-laws of the corporation.
On September 6, 1999, KAL, through Atty. Aguinaldo, filed a
For the reasons stated the petition will be denied, and it is so ordered, with Complaint2 against ETI with the Regional Trial Court (RTC) of Manila, for the
costs. collection of the principal amount of P260,150.00, plus attorneys fees and
exemplary damages. The verification and certification against forum
Johnson, Malcom, Ostrand, Johns, Romualdez and Villa-Real, JJ., concur. shopping was signed by Atty. Aguinaldo, who indicated therein that he was
the resident agent and legal counsel of KAL and had caused the preparation
of the complaint.

ETI filed a motion to dismiss the complaint on the ground that Atty. Aguinaldo
was not authorized to execute the verification and certificate of non-forum
shopping as required by Section 5, Rule 7 of the Rules of Court. KAL
opposed the motion, contending that Atty. Aguinaldo was its resident agent
and was registered as such with the Securities and Exchange Commission
(SEC) as required by the Corporation Code of the Philippines. It was further
alleged that Atty. Aguinaldo was also the corporate secretary of KAL.
G.R. No. 152392 May 26, 2005 Appended to the said opposition was the identification card of Atty.
Aguinaldo, showing that he was the lawyer of KAL.
EXPERTRAVEL & TOURS, INC., petitioner,
vs. During the hearing of January 28, 2000, Atty. Aguinaldo claimed that he had
COURT OF APPEALS and KOREAN AIRLINES, respondent. been authorized to file the complaint through a resolution of the KAL Board of
Directors approved during a special meeting held on June 25, 1999. Upon
DECISION his motion, KAL was given a period of 10 days within which to submit a copy
of the said resolution. The trial court granted the motion. Atty. Aguinaldo
CALLEJO, SR., J.: subsequently filed other similar motions, which the trial court granted.

Before us is a petition for review on certiorari of the Decision1 of the Court of Finally, KAL submitted on March 6, 2000 an Affidavit 3 of even date, executed
Appeals (CA) in CA-G.R. SP No. 61000 dismissing the petition by its general manager Suk Kyoo Kim, alleging that the board of directors
for certiorari and mandamus filed by Expertravel and Tours, Inc. (ETI). conducted a special teleconference on June 25, 1999, which he and Atty.
Aguinaldo attended. It was also averred that in that same teleconference, the

962
board of directors approved a resolution authorizing Atty. Aguinaldo to RESOLVED, that Mario A. Aguinaldo and his law firm M.A.
execute the certificate of non-forum shopping and to file the complaint. Suk Aguinaldo & Associates or any of its lawyers are hereby
Kyoo Kim also alleged, however, that the corporation had no written copy of appointed and authorized to take with whatever legal action
the aforesaid resolution. necessary to effect the collection of the unpaid account of
Expert Travel & Tours. They are hereby specifically
On April 12, 2000, the trial court issued an Order 4 denying the motion to authorized to prosecute, litigate, defend, sign and execute
dismiss, giving credence to the claims of Atty. Aguinaldo and Suk Kyoo Kim any document or paper necessary to the filing and
that the KAL Board of Directors indeed conducted a teleconference on June prosecution of said claim in Court, attend the Pre-Trial
25, 1999, during which it approved a resolution as quoted in the submitted Proceedings and enter into a compromise agreement
affidavit. relative to the above-mentioned claim.

ETI filed a motion for the reconsideration of the Order, contending that it was IN WITNESS WHEREOF, I have hereunto affixed my signature this
inappropriate for the court to take judicial notice of the said teleconference 10th day of January, 1999, in the City of Manila, Philippines.
without any prior hearing. The trial court denied the motion in its Order 5dated
August 8, 2000.
(Sgd.)
ETI then filed a petition for certiorari and mandamus, assailing the orders of
the RTC. In its comment on the petition, KAL appended a certificate signed MARIO A. AGUINALDO
by Atty. Aguinaldo dated January 10, 2000, worded as follows: Resident Agent

SECRETARYS/RESIDENT AGENTS CERTIFICATE

SUBSCRIBED AND SWORN to before me this 10 th day of January,


KNOW ALL MEN BY THESE PRESENTS:
1999, Atty. Mario A. Aguinaldo exhibiting to me his Community Tax
Certificate No. 14914545, issued on January 7, 2000 at Manila,
I, Mario A. Aguinaldo, of legal age, Filipino, and duly elected and Philippines.
appointed Corporate Secretary and Resident Agent of KOREAN
AIRLINES, a foreign corporation duly organized and existing under
and by virtue of the laws of the Republic of Korea and also duly
registered and authorized to do business in the Philippines, with Doc. No. 119; (Sgd.)
office address at Ground Floor, LPL Plaza Building, 124 Alfaro St., Page No. 25; ATTY. HENRY D.
Salcedo Village, Makati City, HEREBY CERTIFY that during a Book No. XXIV Notary
special meeting of the Board of Directors of the Corporation held on Series of 2000. Until December 31,
June 25, 1999 at which a quorum was present, the said Board PTR #889583/MLA 1/3/20006
unanimously passed, voted upon and approved the following
resolution which is now in full force and effect, to wit:

963
On December 18, 2001, the CA rendered judgment dismissing the petition, certificate of non-forum shopping required by Section 5, Rule 7 of the Rules
ruling that the verification and certificate of non-forum shopping executed by of Court, on top of the board resolution approved during the teleconference
Atty. Aguinaldo was sufficient compliance with the Rules of Court. According of June 25, 1999. The respondent insists that "technological advances in this
to the appellate court, Atty. Aguinaldo had been duly authorized by the board time and age are as commonplace as daybreak." Hence, the courts may take
resolution approved on June 25, 1999, and was the resident agent of KAL. judicial notice that the Philippine Long Distance Telephone Company, Inc.
As such, the RTC could not be faulted for taking judicial notice of the said had provided a record of corporate conferences and meetings through
teleconference of the KAL Board of Directors. FiberNet using fiber-optic transmission technology, and that such technology
facilitates voice and image transmission with ease; this makes constant
ETI filed a motion for reconsideration of the said decision, which the CA communication between a foreign-based office and its Philippine-based
denied. Thus, ETI, now the petitioner, comes to the Court by way of petition branches faster and easier, allowing for cost-cutting in terms of travel
for review on certiorari and raises the following issue: concerns. It points out that even the E-Commerce Law has recognized this
modern technology. The respondent posits that the courts are aware of this
DID PUBLIC RESPONDENT COURT OF APPEALS DEPART FROM development in technology; hence, may take judicial notice thereof without
THE ACCEPTED AND USUAL COURSE OF JUDICIAL need of hearings. Even if such hearing is required, the requirement is
PROCEEDINGS WHEN IT RENDERED ITS QUESTIONED nevertheless satisfied if a party is allowed to file pleadings by way of
DECISION AND WHEN IT ISSUED ITS QUESTIONED comment or opposition thereto.
RESOLUTION, ANNEXES A AND B OF THE INSTANT PETITION?7
In its reply, the petitioner pointed out that there are no rulings on the matter of
The petitioner asserts that compliance with Section 5, Rule 7, of the Rules of teleconferencing as a means of conducting meetings of board of directors for
Court can be determined only from the contents of the complaint and not by purposes of passing a resolution; until and after teleconferencing is
documents or pleadings outside thereof. Hence, the trial court committed recognized as a legitimate means of gathering a quorum of board of
grave abuse of discretion amounting to excess of jurisdiction, and the CA directors, such cannot be taken judicial notice of by the court. It asserts that
erred in considering the affidavit of the respondents general manager, as safeguards must first be set up to prevent any mischief on the public or to
well as the Secretarys/Resident Agents Certification and the resolution of protect the general public from any possible fraud. It further proposes
the board of directors contained therein, as proof of compliance with the possible amendments to the Corporation Code to give recognition to such
requirements of Section 5, Rule 7 of the Rules of Court. The petitioner also manner of board meetings to transact business for the corporation, or other
maintains that the RTC cannot take judicial notice of the said related corporate matters; until then, the petitioner asserts, teleconferencing
teleconferencewithout prior hearing, nor any motion therefor. The petitioner cannot be the subject of judicial notice.
reiterates its submission that the teleconference and the resolution adverted
to by the respondent was a mere fabrication. The petitioner further avers that the supposed holding of a special meeting
on June 25, 1999 through teleconferencing where Atty. Aguinaldo was
The respondent, for its part, avers that the issue of whether modern supposedly given such an authority is a farce, considering that there was no
technology is used in the field of business is a factual issue; hence, cannot mention of where it was held, whether in this country or elsewhere. It insists
be raised in a petition for review on certiorari under Rule 45 of the Rules of that the Corporation Code requires board resolutions of corporations to be
Court. On the merits of the petition, it insists that Atty. Aguinaldo, as the submitted to the SEC. Even assuming that there was such a teleconference,
resident agent and corporate secretary, is authorized to sign and execute the it would be against the provisions of the Corporation Code not to have any
record thereof.

964
The petitioner insists that the teleconference and resolution adverted to by excused. The certification is a peculiar and personal responsibility of the
the respondent in its pleadings were mere fabrications foisted by the party, an assurance given to the court or other tribunal that there are no other
respondent and its counsel on the RTC, the CA and this Court. pending cases involving basically the same parties, issues and causes of
action. Hence, the certification must be accomplished by the party himself
The petition is meritorious. because he has actual knowledge of whether or not he has initiated similar
actions or proceedings in different courts or tribunals. Even his counsel may
Section 5, Rule 7 of the Rules of Court provides: be unaware of such facts.9 Hence, the requisite certification executed by the
plaintiffs counsel will not suffice.10
SEC. 5. Certification against forum shopping. The plaintiff or
principal party shall certify under oath in the complaint or other In a case where the plaintiff is a private corporation, the certification may be
initiatory pleading asserting a claim for relief, or in a sworn signed, for and on behalf of the said corporation, by a specifically authorized
certification annexed thereto and simultaneously filed therewith: (a) person, including its retained counsel, who has personal knowledge of the
that he has not theretofore commenced any action or filed any claim facts required to be established by the documents. The reason was
involving the same issues in any court, tribunal or quasi-judicial explained by the Court in National Steel Corporation v. Court of Appeals,11 as
agency and, to the best of his knowledge, no such other action or follows:
claim is pending therein; (b) if there is such other pending action or
claim, a complete statement of the present status thereof; and (c) if Unlike natural persons, corporations may perform physical actions
he should thereafter learn that the same or similar action or claim only through properly delegated individuals; namely, its officers
has been filed or is pending, he shall report that fact within five (5) and/or agents.
days therefrom to the court wherein his aforesaid complaint or
initiatory pleading has been filed.

Failure to comply with the foregoing requirements shall not be The corporation, such as the petitioner, has no powers except those
curable by mere amendment of the complaint or other initiatory expressly conferred on it by the Corporation Code and those that are
pleading but shall be cause for the dismissal of the case without implied by or are incidental to its existence. In turn, a corporation
prejudice, unless otherwise provided, upon motion and after hearing. exercises said powers through its board of directors and/or its duly-
The submission of a false certification or non-compliance with any of authorized officers and agents. Physical acts, like the signing of
the undertakings therein shall constitute indirect contempt of court, documents, can be performed only by natural persons duly-
without prejudice to the corresponding administrative and criminal authorized for the purpose by corporate by-laws or by specific act of
actions. If the acts of the party or his counsel clearly constitute willful the board of directors. "All acts within the powers of a corporation
and deliberate forum shopping, the same shall be ground for may be performed by agents of its selection; and except so far as
summary dismissal with prejudice and shall constitute direct limitations or restrictions which may be imposed by special charter,
contempt, as well as a cause for administrative sanctions. by-law, or statutory provisions, the same general principles of law
which govern the relation of agency for a natural person govern the
It is settled that the requirement to file a certificate of non-forum shopping is officer or agent of a corporation, of whatever status or rank, in
mandatory8 and that the failure to comply with this requirement cannot be respect to his power to act for the corporation; and agents once

965
appointed, or members acting in their stead, are subject to the same 1. I am the Resident Agent and Legal Counsel of the plaintiff in the
rules, liabilities and incapacities as are agents of individuals and above entitled case and have caused the preparation of the above
private persons." complaint;

2. I have read the complaint and that all the allegations contained
therein are true and correct based on the records on files;
For who else knows of the circumstances required in the
Certificate but its own retained counsel. Its regular officers, like its 3. I hereby further certify that I have not commenced any other action
board chairman and president, may not even know the details or proceeding involving the same issues in the Supreme Court, the
required therein. Court of Appeals, or different divisions thereof, or any other tribunal
or agency. If I subsequently learned that a similar action or
Indeed, the certificate of non-forum shopping may be incorporated in the proceeding has been filed or is pending before the Supreme Court,
complaint or appended thereto as an integral part of the complaint. The rule the Court of Appeals, or different divisions thereof, or any tribunal or
is that compliance with the rule after the filing of the complaint, or the agency, I will notify the court, tribunal or agency within five (5) days
dismissal of a complaint based on its non-compliance with the rule, is from such notice/knowledge.
impermissible. However, in exceptional circumstances, the court may allow
subsequent compliance with the rule. 12 If the authority of a partys counsel to
execute a certificate of non-forum shopping is disputed by the adverse party, (Sgd.)
the former is required to show proof of such authority or representation.
MARIO A. AGUINALDO
In this case, the petitioner, as the defendant in the RTC, assailed the Affiant
authority of Atty. Aguinaldo to execute the requisite verification and certificate CITY OF MANILA
of non-forum shopping as the resident agent and counsel of the respondent.
It was, thus, incumbent upon the respondent, as the plaintiff, to allege and
establish that Atty. Aguinaldo had such authority to execute the requisite
verification and certification for and in its behalf. The respondent, however, SUBSCRIBED AND SWORN TO before me this 30 th day of August,
failed to do so. 1999, affiant exhibiting to me his Community Tax Certificate No.
00671047 issued on January 7, 1999 at Manila, Philippines.
The verification and certificate of non-forum shopping which was
incorporated in the complaint and signed by Atty. Aguinaldo reads:
Doc. No. 1005; (Sgd.)
I, Mario A. Aguinaldo of legal age, Filipino, with office address at Page No. 198;
Suite 210 Gedisco Centre, 1564 A. Mabini cor. P. Gil Sts., Ermita, Book No. XXI ATTY. HENRY D. ADA
Manila, after having sworn to in accordance with law hereby deposes Series of 1999. Notary Pu
and say: THAT -
Until December 31, 20

966
Under the law, Atty. Aguinaldo was not specifically authorized to execute a
certificate of non-forum shopping as required by Section 5, Rule 7 of the
PTR No. 320501 Mla. 1/4/9913 Rules of Court. This is because while a resident agent may be aware of
actions filed against his principal (a foreign corporation doing business in the
Philippines), such resident may not be aware of actions initiated by its
As gleaned from the aforequoted certification, there was no allegation that principal, whether in the Philippines against a domestic corporation or private
Atty. Aguinaldo had been authorized to execute the certificate of non-forum individual, or in the country where such corporation was organized and
shopping by the respondents Board of Directors; moreover, no such board registered, against a Philippine registered corporation or a Filipino citizen.
resolution was appended thereto or incorporated therein.
The respondent knew that its counsel, Atty. Aguinaldo, as its resident agent,
While Atty. Aguinaldo is the resident agent of the respondent in the was not specifically authorized to execute the said certification. It attempted
Philippines, this does not mean that he is authorized to execute the requisite to show its compliance with the rule subsequent to the filing of its complaint
certification against forum shopping. Under Section 127, in relation to Section by submitting, on March 6, 2000, a resolution purporting to have been
128 of the Corporation Code, the authority of the resident agent of a foreign approved by its Board of Directors during a teleconference held on June 25,
corporation with license to do business in the Philippines is to receive, for 1999, allegedly with Atty. Aguinaldo and Suk Kyoo Kim in attendance.
and in behalf of the foreign corporation, services and other legal processes in However, such attempt of the respondent casts veritable doubt not only on its
all actions and other legal proceedings against such corporation, thus: claim that such a teleconference was held, but also on the approval by the
Board of Directors of the resolution authorizing Atty. Aguinaldo to execute the
certificate of non-forum shopping.
SEC. 127. Who may be a resident agent. A resident agent may
either be an individual residing in the Philippines or a domestic
corporation lawfully transacting business in the Philippines: Provided, In its April 12, 2000 Order, the RTC took judicial notice that because of the
That in the case of an individual, he must be of good moral character onset of modern technology, persons in one location may confer with other
and of sound financial standing. persons in other places, and, based on the said premise, concluded that Suk
Kyoo Kim and Atty. Aguinaldo had a teleconference with the respondents
Board of Directors in South Korea on June 25, 1999. The CA, likewise, gave
SEC. 128. Resident agent; service of process. The Securities and
credence to the respondents claim that such a teleconference took place, as
Exchange Commission shall require as a condition precedent to the
contained in the affidavit of Suk Kyoo Kim, as well as Atty. Aguinaldos
issuance of the license to transact business in the Philippines by any
certification.
foreign corporation that such corporation file with the Securities and
Exchange Commission a written power of attorney designating some
persons who must be a resident of the Philippines, on whom any Generally speaking, matters of judicial notice have three material requisites:
summons and other legal processes may be served in all actions or (1) the matter must be one of common and general knowledge; (2) it must be
other legal proceedings against such corporation, and consenting well and authoritatively settled and not doubtful or uncertain; and (3) it must
that service upon such resident agent shall be admitted and held as be known to be within the limits of the jurisdiction of the court. The principal
valid as if served upon the duly-authorized officers of the foreign guide in determining what facts may be assumed to be judicially known is
corporation as its home office.14 that of notoriety. Hence, it can be said that judicial notice is limited to facts
evidenced by public records and facts of general notoriety.[15] Moreover, a
judicially noticed fact must be one not subject to a reasonable dispute in that

967
it is either: (1) generally known within the territorial jurisdiction of the trial teleconferencing in the course of business and corporate governance,
court; or (2) capable of accurate and ready determination by resorting to because of the money saved, among other advantages include:
sources whose accuracy cannot reasonably be questionable. 16
1. People (including outside guest speakers) who wouldnt normally
Things of "common knowledge," of which courts take judicial matters coming attend a distant FTF meeting can participate.
to the knowledge of men generally in the course of the ordinary experiences
of life, or they may be matters which are generally accepted by mankind as 2. Follow-up to earlier meetings can be done with relative ease and
true and are capable of ready and unquestioned demonstration. Thus, facts little expense.
which are universally known, and which may be found in encyclopedias,
dictionaries or other publications, are judicially noticed, provided, they are of 3. Socializing is minimal compared to an FTF meeting; therefore,
such universal notoriety and so generally understood that they may be meetings are shorter and more oriented to the primary purpose of
regarded as forming part of the common knowledge of every person. As the the meeting.
common knowledge of man ranges far and wide, a wide variety of particular
facts have been judicially noticed as being matters of common
4. Some routine meetings are more effective since one can audio-
knowledge. But a court cannot take judicial notice of any fact which, in part,
conference from any location equipped with a telephone.
is dependent on the existence or non-existence of a fact of which the court
has no constructive knowledge.17
5. Communication between the home office and field staffs is
maximized.
In this age of modern technology, the courts may take judicial notice that
business transactions may be made by individuals through teleconferencing.
Teleconferencing is interactive group communication (three or more people in 6. Severe climate and/or unreliable transportation may necessitate
two or more locations) through an electronic medium. In general terms, teleconferencing.
teleconferencing can bring people together under one roof even though they
are separated by hundreds of miles.18 This type of group communication may 7. Participants are generally better prepared than for FTF meetings.
be used in a number of ways, and have three basic types: (1) video
conferencing - television-like communication augmented with sound; (2) 8. It is particularly satisfactory for simple problem-solving, information
computer conferencing - printed communication through keyboard terminals, exchange, and procedural tasks.
and (3) audio-conferencing-verbal communication via the telephone with
optional capacity for telewriting or telecopying.19 9. Group members participate more equally in well-moderated
teleconferences than an FTF meeting.21
A teleconference represents a unique alternative to face-to-face (FTF)
meetings. It was first introduced in the 1960s with American Telephone and On the other hand, other private corporations opt not to hold teleconferences
Telegraphs Picturephone. At that time, however, no demand existed for the because of the following disadvantages:
new technology. Travel costs were reasonable and consumers were unwilling
to pay the monthly service charge for using the picturephone, which was 1. Technical failures with equipment, including connections that arent
regarded as more of a novelty than as an actual means for everyday made.
communication.20 In time, people found it advantageous to hold

968
2. Unsatisfactory for complex interpersonal communication, such as specifically authorizing Atty. Aguinaldo to file the complaint and execute the
negotiation or bargaining. required certification against forum shopping.

3. Impersonal, less easy to create an atmosphere of group rapport. The records show that the petitioner filed a motion to dismiss the complaint
on the ground that the respondent failed to comply with Section 5, Rule 7 of
4. Lack of participant familiarity with the equipment, the medium the Rules of Court. The respondent opposed the motion on December 1,
itself, and meeting skills. 1999, on its contention that Atty. Aguinaldo, its resident agent, was duly
authorized to sue in its behalf. The respondent, however, failed to establish
5. Acoustical problems within the teleconferencing rooms. its claim that Atty. Aguinaldo was its resident agent in the Philippines. Even
the identification card25 of Atty. Aguinaldo which the respondent appended to
its pleading merely showed that he is the company lawyer of the
6. Difficulty in determining participant speaking order; frequently one
respondents Manila Regional Office.
person monopolizes the meeting.

The respondent, through Atty. Aguinaldo, announced the holding of the


7. Greater participant preparation time needed.
teleconference only during the hearing of January 28, 2000; Atty. Aguinaldo
then prayed for ten days, or until February 8, 2000, within which to submit the
8. Informal, one-to-one, social interaction not possible. 22 board resolution purportedly authorizing him to file the complaint and execute
the required certification against forum shopping. The court granted the
Indeed, teleconferencing can only facilitate the linking of people; it does not motion.26 The respondent, however, failed to comply, and instead prayed for
alter the complexity of group communication. Although it may be easier to 15 more days to submit the said resolution, contending that it was with its
communicate via teleconferencing, it may also be easier to miscommunicate. main office in Korea. The court granted the motion per its Order 27 dated
Teleconferencing cannot satisfy the individual needs of every type of February 11, 2000. The respondent again prayed for an extension within
meeting.23 which to submit the said resolution, until March 6, 2000. 28 It was on the said
date that the respondent submitted an affidavit of its general manager Suk
In the Philippines, teleconferencing and videoconferencing of members of Kyoo Kim, stating, inter alia, that he and Atty. Aguinaldo attended the said
board of directors of private corporations is a reality, in light of Republic Act teleconference on June 25, 1999, where the Board of Directors supposedly
No. 8792. The Securities and Exchange Commission issued SEC approved the following resolution:
Memorandum Circular No. 15, on November 30, 2001, providing the
guidelines to be complied with related to such conferences. 24 Thus, the Court RESOLVED, that Mario A. Aguinaldo and his law firm M.A. Aguinaldo
agrees with the RTC that persons in the Philippines may have a & Associates or any of its lawyers are hereby appointed and
teleconference with a group of persons in South Korea relating to business authorized to take with whatever legal action necessary to effect the
transactions or corporate governance. collection of the unpaid account of Expert Travel & Tours. They are
hereby specifically authorized to prosecute, litigate, defend, sign and
Even given the possibility that Atty. Aguinaldo and Suk Kyoo Kim participated execute any document or paper necessary to the filing and
in a teleconference along with the respondents Board of Directors, the Court prosecution of said claim in Court, attend the Pre-trial Proceedings
is not convinced that one was conducted; even if there had been one, the and enter into a compromise agreement relative to the above-
Court is not inclined to believe that a board resolution was duly passed mentioned claim.29

969
But then, in the same affidavit, Suk Kyoo Kim declared that the respondent Worse still, it appears that as early as January 10, 1999, Atty. Aguinaldo had
"do[es] not keep a written copy of the aforesaid Resolution" because no signed a Secretarys/Resident Agents Certificate alleging that the board of
records of board resolutions approved during teleconferences were kept. directors held a teleconference on June 25, 1999. No such certificate was
This belied the respondents earlier allegation in its February 10, 2000 motion appended to the complaint, which was filed on September 6, 1999. More
for extension of time to submit the questioned resolution that it was in the importantly, the respondent did not explain why the said certificate was
custody of its main office in Korea. The respondent gave the trial court the signed by Atty. Aguinaldo as early as January 9, 1999, and yet was notarized
impression that it needed time to secure a copy of the resolution kept in one year later (on January 10, 2000); it also did not explain its failure to
Korea, only to allege later (via the affidavit of Suk Kyoo Kim) that it had no append the said certificate to the complaint, as well as to its Compliance
such written copy. Moreover, Suk Kyoo Kim stated in his affidavit that the dated March 6, 2000. It was only on January 26, 2001 when the respondent
resolution was embodied in the Secretarys/Resident Agents Certificate filed its comment in the CA that it submitted the Secretarys/Resident Agents
signed by Atty. Aguinaldo. However, no such resolution was appended to the Certificate30 dated January 10, 2000.
said certificate.
The Court is, thus, more inclined to believe that the alleged teleconference
The respondents allegation that its board of directors conducted a on June 25, 1999 never took place, and that the resolution allegedly
teleconference on June 25, 1999 and approved the said resolution (with Atty. approved by the respondents Board of Directors during the said
Aguinaldo in attendance) is incredible, given the additional fact that no such teleconference was a mere concoction purposefully foisted on the RTC, the
allegation was made in the complaint. If the resolution had indeed been CA and this Court, to avert the dismissal of its complaint against the
approved on June 25, 1999, long before the complaint was filed, the petitioner.
respondent should have incorporated it in its complaint, or at least appended
a copy thereof. The respondent failed to do so. It was only on January 28, IN LIGHT OF ALL THE FOREGOING, the petition is GRANTED. The
2000 that the respondent claimed, for the first time, that there was such a Decision of the Court of Appeals in CA-G.R. SP No. 61000 is REVERSED
meeting of the Board of Directors held on June 25, 1999; it even represented and SET ASIDE. The Regional Trial Court of Manila is hereby ORDERED to
to the Court that a copy of its resolution was with its main office in Korea, dismiss, without prejudice, the complaint of the respondent.
only to allege later that no written copy existed. It was only on March 6, 2000
that the respondent alleged, for the first time, that the meeting of the Board of SO ORDERED.
Directors where the resolution was approved was held via teleconference.
Puno, Acting C.J., (Chairman), Austria-Martinez, and Chico-Nazario,
JJ., concur.
Tinga, J., out of the country.

970
[G.R. No. 113032. August 21, 1997] In said meeting, the Board of Trustees passed Resolution No. 48, s.
1986, granting monthly compensation to the private respondents as
WESTERN INSTITUTE OF TECHNOLOGY, INC., HOMERO L. VILLASIS, corporate officers retroactive June 1, 1985, viz.:
DIMAS ENRIQUEZ, PRESTON F. VILLASIS & REGINALD F.
VILLASIS, petitioners, vs. RICARDO T. SALAS, SOLEDAD Resolution No. 48 s. 1986
SALAS-TUBILLEJA, ANTONIO S. SALAS, RICHARD S. SALAS &
HON. JUDGE PORFIRIO PARIAN, respondents. On the motion of Mr. Richard Salas (accused), duly seconded by Mrs.
Soledad Tubilleja (accused), it was unanimously resolved that:
DECISION
The Officers of the Corporation be granted monthly compensation for
HERMOSISIMA, JR., J.: services rendered as follows: Chairman - P9,000.00/month, Vice-Chairman
- P3,500.00/month, Corporate Treasurer - P3,500.00/month and Corporate
Up for review on certiorari are: (1) the Decision September 6, 1993 and Secretary - P3,500.00/month, retroactive June 1, 1985 and the ten
(2) the order dated November 23, 1993 of Branch 33 of the Regional Trial percentum of the net profits shall be distributed equally among the ten
Court of Iloilo City in Criminal Cases Nos. 37097 and 37098 for estafa and members of the Board of Trustees. This shall amend and superceed(sic) any
falsification of a public document, respectively. The judgment acquitted the previous resolution.
private respondents of both charges, but petitioners seek to hold them civilly
liable. There were no other business.

Private respondents Ricardo T. Salas, Salvador T. Salas, Soledad The Chairman declared the meeting adjourned at 5:11 P.M.
Salas-Tubilleja, Antonio S. Salas, and Richard S. Salas, belonging to the
same family, are the majority and controlling members of the Board of This is to certify that the foregoing minutes of the regular meeting of the
Trustees of Western Institute of Technology, Inc. (WIT, for short), a stock Board of Trustees of Western Institute of Technology, Inc. held on March 30,
corporation engaged in the operation, among others, of an educational 1986 is true and correct to the best of my knowledge and belief.
institution. According to petitioners, the minority stockholders of WIT,
sometime on June 1, 1986 in the principal office of WIT at La Paz, Iloilo City, (Sgd) ANTONIO
a Special Board meeting was held. In attendance were other members of the S. SALAS
Board including one of the petitioners Reginald Villasis. Prior to aforesaid
Special Board Meeting, copies of notice thereof, dated May 24, 1986, were
Corporate
distributed to all Board Members. The notice allegedly indicated that the
Secretary[2]
meeting to be held on June 1, 1986 included item No. 6 which states:

A few years later, that is, on March 13, 1991, petitioners Homero
"Possible implementation of Art. III, Sec. 6 of the Amended By-Laws of
Villasis, Preston Villasis, Reginald Villasis and Dimas Enriquez filed an
Western Institute of Technology, Inc. on compensation of all officers of the
affidavit-complaint against private respondents before the Office of the City
corporation." [1]
Prosecutor of Iloilo, as a result of which two (2) separate criminal
informations, one for falsification of a public document under Article 171 of

971
the Revised Penal Code and the other for estafa under Article 315, par. 1(b) fact, as said accused well knew, no such Resolution No. 48, Series of 1986
of the RPC, were filed before Branch 33 of the Regional Trial Court of Iloilo was passed on March 30, 1986.
City. The charge for falsification of public document was anchored on the
private respondents submission of WITs income statement for the fiscal year CONTRARY TO LAW.
1985-1986 with the Securities and Exchange Commission (SEC) reflecting
therein the disbursement of corporate funds for the compensation of private Iloilo City, Philippines, November 22,1991.[3] [Underscoring ours].
respondents based on Resolution No. 4, series of 1986, making it appear
that the same was passed by the board on March 30, 1986, when in truth,
The Information, on the other hand, for estafa reads:
the same was actually passed on June 1, 1986, a date not covered by the
corporations fiscal year 1985-1986 (beginning May 1, 1985 and ending April
30, 1986). The information for falsification of a public document states: The undersigned City Prosecutor accuses RICARDO SALAS, SALVADOR T.
SALAS, SOLEDAD SALAS-TUBILLEJA, ANTONIO S. SALAS, RICHARD S.
SALAS (whose dates and places of birth cannot be ascertained) of the crime
The undersigned City Prosecutor accuses RICARDO T. SALAS, SALVADOR
of ESTAFA, Art. 315, par 1(b) of the Revised Penal Code, committed as
T. SALAS, SOLEDAD SALAS-TUBILLEJA, ANTONIO S. SALAS and
follows:
RICHARD S. SALAS (whose dates and places of birth cannot be
ascertained) of the crime of FALSIFICATION OF A PUBLC DOCUMENT, Art.
171 of the Revised Penal Code, committed as follows: That on or about the 1st day of June, 1986, in the City of Iloilo, Philippines
and within the jurisdiction of this Honorable Court, the above-named
accused, being then the Chairman, Vice-Chairman, Treasurer, Secretary and
That on or about the 10th day of June, 1986, in the City of Iloilo, Philippines
Trustee (who later became the secretary), respectively, of the board of
and within the jurisdiction of this Honorable Court, the above-named
trustees of the Western Institute of Technology, Inc., a corporation duly
accused, being then the Chairman, Vice-Chairman, Treasurer, Secretary and
organized and existing under the laws of the Republic of the Philippines,
Trustee (who later became the secretary), respectively, of the board of
conspiring and confederating together and mutually helping one another, to
trustees of the Western Institute of Technology, Inc., a corporation duly
better realize their purpose, did then and there wilfully, unlawfully and
organized and existing under the laws of the Republic of the Philippines,
feloniously defraud the said corporation (and its stockholders) in the following
conspiring and confederating together and mutually helping one another, to
manner, to wit: herein accused, knowing fully well that they have no
better realized (sic) their purpose, did then and there wilfully, unlawfully and
sufficient, lawful authority to disburse--- let alone violation of applicable laws
criminally prepare and execute and subsequently cause to be submitted to
and jurisprudence, disbursed the funds of the corporation by effecting
the Securities and Exchange Commission an income statement of the
payment of their retroactive salaries in the amount of P186,470.70 and
corporation for the fiscal year 1985-1986, the same being required to be
subsequently paying themselves every 15th and 30th of the month starting
submitted every end of the corporation fiscal year by the aforesaid
June 15, 1986 until the present, in the amount of P19,500.00 per month, as if
Commission and therefore, a public document, including therein the
the same were their own, and when herein accused were informed of the
disbursement of the retroactive compensation of accused corporate officers
illegality of these disbursements by the minority stockholders by way of
in the amount of P186,470.70, by then and there making it appear that the
objections made in an annual stockholders meeting held on June 14, 1986
basis thereof Resolution No. 4, Series of 1986 was passed by the board of
and every year thereafter, they refused, and still refuse, to rectify the same to
trustees on March 30, 1986, a date covered by the corporations fiscal year
the damage and prejudice of the corporation (and its stockholders) in the
1985-1986 (i.e., from May 1, 1985 to April 30, 1986), when in truth and in
total sum of P1,453,970.79 as of November 15, 1991.

972
CONTRARY TO LAW. Petitioners would like us to hold private respondents civilly liable despite
their acquittal in Criminal Cases Nos. 37097 and 37098. They base their
Iloilo City, Philippines, November 22,1991.[4] [Underscoring ours] claim on the alleged illegal issuance by private respondents of Resolution
No. 48, series of 1986 ordering the disbursement of corporate funds in the
Thereafter, trial for the two criminal cases, docketed as Criminal Cases amount of P186,470.70 representing the retroactive compensation as of
Nos. 37097 and 37098, was consolidated. After a full-blown hearing, Judge June 1, 1985 in favor of private respondents, board members of WIT,
Porfirio Parian handed down a verdict of acquittal on both counts [5] dated plus P1,453,970.79 for the subsequent collective salaries of private
September 6, 1993 without imposing any civil liability against the accused respondent every 15th and 30th of the month until the filing of the criminal
therein. complaints against them on March 1991. Petitioners maintain that this grant
of compensation to private respondents is proscribed under Section 30 of the
Corporation Code. Thus, private respondents are obliged to return these
Petitioners filed a Motion for Reconsideration [6] of the civil aspect of the
amounts to the corporation with interest.
RTC Decision which was, however, denied in an Order dated November 23,
1993.[7]
We cannot sustain the petitioners. The pertinent section of the
Corporation Code provides:
Hence, the instant petition.

Sec. 30. Compensation of directors.--- In the absence of any provision in the


Significantly on December 8, 1994, a Motion for Intervention, dated
by-laws fixing their compensation, the directors shall not receive any
December 2, 1994, was filed before this Court by Western Institute of
compensation, as such directors, except for reasonable per diems: Provided,
Technology, Inc., supposedly one of the petitioners herein, disowning its
however, That any such compensation (other than per diems) may be
inclusion in the petition and submitting that Atty. Tranquilino R. Gale, counsel
granted to directors by the vote of the stockholders representing at least a
for the other petitioners, had no authority whatsoever to represent the
majority of the outstanding capital stock at a regular or special stockholders
corporation in filing the petition. Intervenor likewise prayed for the dismissal
meeting. In no case shall the total yearly compensation of directors, as such
of the petition for being utterly without merit. The Motion for Intervention was
directors, exceed ten (10%) percent of the net income before income tax of
granted on January 16, 1995.[8]
the corporation during the preceding year. [Underscoring ours]

There is no argument that directors or trustees, as the case may be, are
not entitled to salary or other compensation when they perform nothing more
than the usual and ordinary duties of their office. This rule is founded upon a
presumption that directors /trustees render service gratuitously and that the
return upon their shares adequately furnishes the motives for service, without
compensation[9] Under the foregoing section, there are only two (2) ways by
which members of the board can be granted compensation apart from
reasonable per diems: (1) when there is a provision in the by-laws fixing their
compensation; and (2) when the stockholders representing a majority of the
outstanding capital stock at a regular or special stockholders meeting agree
to give it to them.

973
This proscription, however, against granting compensation to (Sgd) ANTONI
directors/trustees of a corporation is not a sweeping rule. Worthy of note is O S. SALAS
the clear phraseology of Section 30 which states: xxx [T]he directors shall not
receive any compensation, as such directors, xxx. The phrase as such Corporate
directors is not without significance for it delimits the scope of the prohibition Secretary[11] [Underscoring ours]
to compensation given to them for services performed purely in their capacity
as directors or trustees. The unambiguous implication is that members of the Clearly, therefore , the prohibition with respect to granting compensation to
board may receive compensation, in addition to reasonable per diems, when corporate directors/trustees as such under Section 30 is not violated in this
they render services to the corporation in a capacity other than as particular case. Consequently, the last sentence of Section 30 which
directors/trustees.[10] In the case at bench, Resolution No. 48, s. 1986 granted provides:
monthly compensation to private respondents not in their capacity as
members of the board, but rather as officers of the corporation, more
xxx xxx. In no case shall the total yearly compensation of directors, as such
particularly as Chairman, Vice-Chairman, Treasurer and Secretary of
directors, exceed ten (10%) percent of the net income before income tax of
Western Institute of Technology. We quote once more Resolution No. 48, s.
the corporation during the preceding year. [Underscoring ours]
1986 for easy reference, viz.:

does not likewise find application in this case since the compensation is
Resolution No. 48 s. 1986
being given to private respondents in their capacity as officers of WIT and not
as board members.
On the motion of Mr. Richard Salas (accused), duly seconded by Mrs.
Soledad Tubilleja (accused), it was unanimously resolved that:
Petitioners assert that the instant case is a derivative suit brought by
them as minority shareholders of WIT for and on behalf of the corporation to
The Officers of the Corporation be granted monthly compensation for annul Resolution No. 48, s. 1986 which is prejudicial to the corporation.
services rendered as follows: Chairman - P9,000.00/month, Vice-
Chairman - P3,500.00/month, Corporate Treasurer - P3,500.00/month
We are unpersuaded. A derivative suit is an action brought by minority
and Corporate Secretary - P3,500.00/month, retroactive June 1, 1985 and
shareholders in the name of the corporation to redress wrongs committed
the ten percentum of the net profits shall be distributed equally among the
against it, for which the directors refuse to sue. [12] It is a remedy designed by
ten members of the Board of Trustees. This shall amend and superceed(sic)
equity and has been the principal defense of the minority shareholders
any previous resolution.
against abuses by the majority.[13] Here, however, the case is not a derivative
suit but is merely an appeal on the civil aspect of Criminal Cases Nos. 37097
There were no other business. and 37098 filed with the RTC of Iloilo for estafa and falsification of public
document. Among the basic requirements for a derivative suit to prosper is
The Chairman declared the meeting adjourned at 5:11 P.M. that the minority shareholder who is suing for and on behalf of the
corporation must allege his complaint before the proper forum that he is
This is to certify that the foregoing minutes of the regular meeting of the suing on a derivative cause of action on behalf of the corporation and all
Board of Trustees of Western Institute of Technology, Inc. held on March 30, other shareholders similarly situated who wish to join. [14] This is necessary to
1986 is true and correct to the best of my knowledge and belief. vest jurisdiction upon the tribunal in line with the rule that it is the allegations

974
in the complaint that vests jurisdiction upon the court or quasi-judicial body Once the case is decided by the SEC, the losing party may file a petition for
concerned over the subject matter and nature of the action. [15] This was not review before the Court of Appeals raising questions of fact, of law, or mixed
complied with by the petitioners either in their complaint before the court a questions of fact and law.[17] It is only after the case has ran this course, and
quo nor in the instant petition which, in part, merely states that this is a not earlier, can it be brought to us via a petition for review on certiorari under
petition for review on certiorari on pure questions of law to set aside a portion Rule 45 raising only pure questions of law.[18] Petitioners, in pleading that we
of the RTC decision in Criminal Cases Nos. 37097 and 37098 [16] since the treat the instant petition as a derivative suit, are trying to short-circuit the
trial courts judgment of acquittal failed to impose any civil liability against the entire process which we cannot here sanction.
private respondents. By no amount of equity considerations, if at all
deserved, can a mere appeal on the civil aspect of a criminal case be treated As an appeal on the civil aspect of Criminal Cases Nos. 37097 and
as a derivative suit. 37098 for falsification of public document and estafa, which this petition truly
is, we have to deny the petition just the same. It will be well to quote the
Granting, for purposes of discussion, that this is a derivative suit as respondent courts ratiocinations acquitting the private respondents on both
insisted by petitioners, which it is not, the same is outrightly dismissible for counts:
having been wrongfully filed in the regular court devoid of any jurisdiction to
entertain the complaint. The case should have been filed with the Securities The prosecution wants this Court to believe and agree that there is
and Exchange Commission (SEC) which exercises original and exclusive falsification of public document because, as claimed by the prosecution,
jurisdiction over derivative suits, they being intra-corporate disputes, per Resolution No. 48, Series of 1986 (Exh. 1-E-1) was not taken up and passed
Section 5(b) of P.D. No. 902-A: during the Regular Meeting of the Board of Trustees of the western Institute
of Technology (WIT), Inc. on March 30, 1986, but on June 1, 1986 special
In addition to the regulatory and adjudicative functions of the Securities and meeting of the same board of trustees.
Exchange Commission over corporations, partnerships and other forms of
associations registered with it as expressly granted under existing laws and This Court is reluctant to accept this claim of falsification. The prosecution
decrees, it shall have original and exclusive jurisdiction to hear and decide omitted to submit the complete minutes of the regular meeting of the Board
cases involving: of Trustees on March 30, 1986. It only presented in evidence Exh. C, which
is page 5 or the last page of the said minutes. Had the complete minutes
xxx xxx xxx (Exh. 1 consisting of five (5) pages, been submitted, it can readily be seen
and understood that Resolution No. 48, Series of 1986 (Exh. 1-E-1) giving
b) Controversies arising out of intra-corporate or compensation to corporate officers, was indeed included in Other Business,
partnership relations, between and among stockholders, members, or No. 6 of the Agenda, and was taken up and passed on March 30, 1986. The
associates; between any or all of them and the corporation, partnership or mere fact of existence of Exh. C also proves that it was passed on March 30,
association of which they are stockholders, members or associates, 1986 for Exh,. C is a part and parcel of the whole minutes of the Board of
respectively; and between such corporation, partnership or association and Trustees Regular Meeting on March 30, 1986. No better and more credible
the State insofar as it concerns their individual franchise or right to exist as proof can be considered other than the Minutes (Exh. 1) itself of the Regular
such entity; Meeting of the Board of Trustees on March 30, 1986. The imputation that
said Resolution No.48 was neither taken up nor passed on March 30, 1986
xxx xxx xxx. [Underscoring ours] because the matter regarding compensation was not specifically stated or
written in the Agenda and that the words possible implementation of said

975
Resolution No. 48, was expressly written in the Agenda for the Special xxx xxx xxx.[19] [Underscoring ours]
Meeting of the Board on June 1, 1986, is simply an implication. This
evidence by implication to the mind of the court cannot prevail over the From the foregoing factual findings, which we find to be amply
Minutes (Exh. 1) and cannot ripen into proof beyond reasonable doubt which substantiated by the records, it is evident that there is simply no basis to hold
is demanded in all criminal prosecutions. the accused, private respondents herein, civilly liable. Section 2(b) of Rule
111 on the New Rules on Criminal Procedure provides:
This Court finds that under the Eleventh Article (Exh. 3-D-1) of the Articles of
Incorporation (Exh. 3-B) of the Panay Educational Institution, Inc., now the SEC. 2. Institution of separate civil action.
Western Institute of Technology, Inc., the officers of the corporation shall
receive such compensation as the Board of Directors may provide. These xxx xxx xxx
Articles of Incorporation was adopted on May 17, 1957 (Exh. 3-E). The
Officers of the corporation and their corresponding duties are enumerated
(b) Extinction of the penal action does not carry with it extinction of the
and stated in Sections 1, 2, 3 and 4 of Art. III of the Amended By-Laws of the
civil, unless the extinction proceeds from a declaration in a final judgment
Corporation (Exh. 4-A) which was adopted on May 31, 1957. According to
that the fact from which the civil might arise did not exist.[Underscoring ours]
Sec. 6, Art. III of the same By-Laws, all officers shall receive such
compensation as may be fixed by the Board of Directors.
Likewise, the last paragraph of Section 2, Rule 120 reads:
It is the perception of this Court that the grant of compensation or salary to
the accused in their capacity as officers of the corporation, through SEC. 2. Form and contents of judgment.
Resolution No. 48, enacted on March 30, 1986 by the Board of Trustees, is
authorized by both the Articles of Incorporation and the By-Laws of the xxx xxx xxx
Corporation. To state otherwise is to depart from the clear terms of the said
articles and by-laws. In their defense the accused have properly and rightly In case of acquittal, unless there is a clear showing that the act from which
asserted that the grant of salary is not for directors, but for their being the civil liability might arise did not exist, the judgment shall make a finding
officers of the corporation who oversee the day to day activities and on the civil liability of the accused in favor of the offended party.
operations of the school. [Underscoring ours]

xxx xxx xxx The acquittal in Criminal Cases Nos. 37097 and 37098 is not merely
based on reasonable doubt but rather on a finding that the accused-private
xxx [O]n the question of whether or not the accused can be held liable for respondents did not commit the criminal acts complained of. Thus, pursuant
estafa under Sec. 1 (b) of Art. 315 of the Revised Penal Code, it is perceived to the above rule and settled jurisprudence, any civil action ex delicto cannot
by this Court that the receipt and the holding of the money by the accused as prosper. Acquittal in a criminal action bars the civil action arising therefrom
salary on basis of the authority granted by the Articles and By-Laws of the where the judgment of acquittal holds that the accused did not commit the
corporation are not tainted with abuse of confidence. The money they criminal acts imputed to them.[20]
received belongs to them and cannot be said to have been converted and/or
misappropriated by them.

976
WHEREFORE, the instant petition is hereby DENIED with costs against
petitioners.

SO ORDERED.

Padilla, (Chairman), Bellosillo, Vitug, and Kapunan, JJ., concur.

[G.R. No. 119877. March 31, 1997]

BIENVENIDO ONGKINGCO, as President and GALERIA DE


MAGALLANES CONDOMINIUM ASSOCIATION,
INC., petitioners, vs. NATIONAL LABOR RELATIONS
COMMISSION and FEDERICO B. GUILAS, respondents.

DECISION

KAPUNAN, J.:

At fore, once again, is the jurisdictional tug of war between the National
Labor Relations Commission (NLRC) and the Securities & Exchange
Commission (SEC) in this special civil action for certiorari under Rule 65 of
the Revised Rules of Court. It seeks to set aside the Resolutions of the
NLRC in NLRC NCR Case No. 00-05-02780-92 (NLRC CA No. 004329-93)
dated 9 March 1995 and 4 April 1995 which reversed the decision of Labor
Arbiter Oswald Lorenzo and denied petitioners' motion for reconsideration,
respectively.

Petitioner Galeria de Magallanes Condominium Association, Inc.


(Galeria for brevity) is a non-stock, non-profit corporation formed in
accordance with R.A. No. 4726, otherwise known as the Condominium Act.
"Its primary purpose is to hold title to the common areas of the Galeria de
Magallanes Condominium Project and to manage and administer the same

977
for the use and convenience of the residents and/or owners." [1] Petitioner ONE. The Position of Administrator or Superintendent is a corporate position,
Bienvenido Ongkingco was the president of Galeria at the time private whose appointment depended on the Board of Directors. As such, the
respondent filed his complaint. position of the administrator is a corporate creation.

On 1 September 1990, Galeria's Board of Directors appointed private TWO. Clearly from the respondent corporation's Articles of Incorporation, Art.
respondent Federico B. Guilas as Administrator/Superintendent. He was V, Sec. 6 thereof, the appointment and removal of the administrator is a
given a "monthly salary of P10,000 subject to review after five (5) months prerogative that belongs to the Board, and thereby involves the exercise of
and subsequently thereafter as Galeria's finances improved." [2] deliberate choice and faculty of discriminative selection.

As Administrator, private respondent was tasked with the maintenance THIRD. Thus, we find lacking of merit the argument of complainant that since
of the "performance and elegance of the common areas of the condominium he is not a member of the condominium association where he was formerly
and external appearance of the compound thereof for the convenience and administrator, or is not a unit holder thereof, since a person's relationship to a
comfort of the residents as well as to keep up the quality image, and hence corporation is not determinative of the services performed but by the
the value of the investment for the owners thereof." [3] incidents of the relationship as they exist. (PSBA vs. LEANO, 127 SCRA
778.)
However, on 17 March 1992, through a resolution passed by the Board
of Directors of Galeria, private respondent was not re-appointed as The resolution, therefore, of the other pending incident, which is the MOTION
Administrator. FOR SUBSTITUTION OF PARTIES is hereby deferred for action by the SEC.

As a result, on 15 May 1992, private respondent instituted a complaint WHEREFORE, in view of all the foregoing considerations, this Office hereby
against petitioners for illegal dismissal and non-payment of salaries with the orders the dismissal of the instant action for reason of lack of jurisdiction. The
NLRC. complainant, if he is mindful should file this case with the Securities and
Exchange Commission.
In response, on 22 July 1992, petitioners filed a motion to dismiss
alleging that it is the SEC, and not the labor arbiter, which has jurisdiction SO ORDERED.[4]
over the subject matter of the complaint.
The NLRC, however, reversed the Labor Arbiter's order in its resolution
Labor Arbiter Lorenzo granted the aforestated motion to dismiss in his dated 9 March 1995. It ruled in this wise:
order dated 29 December 1992. He ruled, thus:
We find merit in the appeal. It cannot be gainsaid that the complainant's
A judicious calibration of the position taken by the contending parties cause of action in his complaint is illegal dismissal which issue falls four
preponderate clearly in favor of respondents, that this case is within the square within the jurisdiction of the NLRC. This is so, because while it may
jurisdiction of the Securities and Exchange Commission and not this Office be true that the termination of the complainant was effected allegedly by a
(Labor Arbiter). resolution of the Board of Directors of the respondent association, this did not
make the dispute intracorporate in nature. Moreover, We have taken note of
Our reasons are as follows: the fact that the complainant is neither a member of the association nor an

978
officer thereof. Hence, We are more convinced that he is an employee of the Section 5(c), PD. 902-A since the real question was the invalidity of the board
respondent association occupying the position of administrator who is in (sic) of director's meeting wherein corporate officers involved were not re-elected,
charged with the function of managing and administering the building or resulting in the termination of their services." (Underscoring ours.)
condominium owned by the members. Indeed, there is a whale of difference
between a member of the association who is a part owner of the building and As obtaining in this case, no intracorporate controversy exists, hence, the
a mere employee performing managerial and administrative functions which jurisdiction of the NLRC should be sustained.
are necessary in the usual undertaking of the respondent Association. The
complainant falls under the second category. WHEREFORE, finding merit on the appeal, the same is hereby, given due
course. Accordingly, the Order appealed from is declared Null and Void and
And, to the point of being repetitious, it needs to be stressed that the fact that is hereby, VACATED and SET ASIDE. Accordingly, let the records of the case
the complainant was removed by the Board of Directors did not change the be remanded to the Arbitration Branch of origin for further proceedings. With
issue from an illegal dismissal case to an intracorporate one. For, what the directive that the instant case be given priority in the calendar of the
remains to be resolved here is whether or not the complainant's removal Labor Arbiter for the speedy disposition hereon. Concomitant hereto, the
from his position as Administrator was for a just and valid cause and in respondents are hereby directed to submit their position paper within ten (10)
compliance with due process. And, as the facts now stand, the issue is within days from receipt hereof.
the scope of authority of the National Labor Relations Commission to
resolve. SO ORDERED.[5]

We simply could not agree with the conclusions of law made by the Petitioners filed a motion for reconsideration but the same was denied in
Arbiter a quo on the applicability of the provisions of P.D. 902. Our view finds the NLRC's resolution dated 4 April 1995.[6] Hence, the present recourse.
basis in the case of Gregorio Araneta University Foundation vs. Antonio J.
Teodoro and NLRC (167 SCRA 79) wherein the Supreme Court had the
The petitioners raised a single issue:
occasion to clarify the jurisdiction of the Securities and Exchange
Commission and that of the NLRC. It (Supreme Court) held, thus
THE PRIVATE RESPONDENT ACTED WITHOUT OR IN EXCESS OF
ITS JURISDICTION OR COMMITTED GRAVE ABUSE OF
"x x x Relying on Philippine School of Business Administration, et al., (127
DISCRETION IN TAKING COGNIZANCE OF A SUBJECT MATTER
SCRA 778) and Dy, et al., vs. National Labor Relations Commission, et al.,
THAT FELL WITHIN THE ORIGINAL AND EXCLUSIVE JURISDICTION
(145 SCRA 211), Petitioner theorizes that since private respondent was a
OF THE SEC.
corporate officer, the present controversy is within the jurisdiction of the
Securities and Exchange Commission, pursuant to P.D. 902-A, and not in the
public respondent. The petition is granted.

Without need of applying the rule on estoppel by laches against petitioner, its Specifically delineated in P.D. 902-A are the cases over which the SEC
contention must fail on the ground of misplaced reliance. As explained in Dy, exercises exclusive jurisdiction:
the same is true with Philippine Business Administration, the controversies
therein were intra corporate in nature and squarely within the purview of SECTION 5. In addition to the regulatory and adjudicative functions of the
Securities and Exchange Commission over corporations, partnerships and

979
other forms of associations registered with it as expressly granted under The contentions of public respondent lack merit. That private
existing laws and decrees, it shall have original and exclusive jurisdiction to respondent is an officer of petitioner corporation and not its mere employee
hear and decide cases involving: cannot be questioned. The by-laws of the Galeria de Magallanes
Condominium Association specifically includes the
a) Devices or schemes employed by or any acts of the board of directors, Superintendent/Administrator in its roster of corporate officers:
business associates, its officers or partners, amounting to fraud and
misrepresentation which may be detrimental to the interest of the public ARTICLE IV
and/or of the stockholders, partners, members of associations or
organizations registered with the Commission. OFFICERS

b) Controversies arising out of intra-corporate or partnership relations, Section 1. Executive Officers The Executive officers of the corporation shall
between and among stockholders, members, or associates; between any or be a President, a Vice President, a Treasurer, all of whom shall be elected by
all of them and the corporation, partnership or association of which they are the Board of Directors. They may be removed with or without cause at any
stockholders, members or associates, respectively; and between such meeting by the concurrence of four directors. The Board of Directors may
corporation, partnership or association and the State insofar as it concerns appoint a Superintendent or Administrator and such other officers and
their individual franchise or right to exist as such entity; employees and delineate their powers and duties as the Board shall find
necessary to manage the affairs of the corporation.[8] (Underscoring ours.)
c) Controversies in the election or appointment of directors, trustees, officers,
or managers of such corporations, partnerships or associations. xxx.

d) Petitions of corporations, partnerships or associations to be declared in Section 6. The Superintendent or Administrator The Board of Directors may
the state of suspension of payments in cases where the corporation, appoint a Superintendent or Administrator for the condominium project if the
partnership or association possesses property to cover all of its debts but activities and financial condition of the Association so warrant. If one is so
foresees the impossibility of meeting them when they respectively fall due or appointed, he shall be the principal administrative officer of the Association.
in cases where the corporation, partnership or association has no sufficient He shall attend to routinary and day-to-day business and activities of the
assets to cover its liabilities, but is under the Management Committee Association and shall keep regular officer hours for the purpose. He shall
created pursuant to this Decree. (Underscoring ours.) have such other duties and powers as may be conferred upon him by the
Board of Directors or delegated by the President of the Association.
The Solicitor General contends that the case at bar falls outside the
purview of the aforequoted provision. He insists that private respondent was At the discretion of the Board of Directors, the work and duties of
a mere employee of petitioner corporation being tasked mainly, as Superintendent or Administrator may be entrusted to a juridical entity which is
administrator/superintendent, with the upkeep of the condominium's common qualified and competent to perform such work.[9]
areas. He, thus, maintains that private respondent cannot be deemed a
corporate officer because "it is the nature of one's functions and not the Closely approximating the dispute at bar is the recent case of Tabang v.
nomenclature or title given to one's job which determines one's status in a NLRC.[10] This Court, through Justice Florenz D. Regalado, ruled that:
corporation."[7]

980
Contrary to the contention of petitioner, a medical director and a hospital A corporate officer's dismissal is always a corporate act and/or an intra-
administrator are considered as corporate officers under the by-laws of corporate controversy and that nature is not altered by the reason or wisdom
respondent corporation. Section 2(i), Article I thereof states that one of the which the Board of Directors may have in taking such action.
powers of the Board of Trustees is "(t)o appoint a Medical Director,
Comptroller/Administrator, Chiefs of Services and such other officers as it Based on the foregoing, we must rule that private respondent was
may deem necessary and prescribe their powers and duties." indeed a corporate officer. He was appointed directly by the Board of
Directors not by any managing officer of the corporation and his salary was,
The president, vice-president, secretary and treasurer are commonly likewise, set by the same Board. Having thus determined, his dismissal or
regarded as the principal or executive officers of a corporation, and modern non-appointment is clearly an intra-corporate matter and jurisdiction,
corporation statutes usually designate them as the officers of the corporation. therefore, properly belongs to the SEC and not the NLRC.
However, other offices are sometimes created by the charter or by-laws of a
corporation, or the board of directors may be empowered under the by-laws The respondents also attack the SEC's jurisdiction over the instant case
of a corporation to create additional offices as may be necessary. on grounds that Guilas was not elected by the Board of Directors but was
merely appointed.
It has been held that an "office" is created by the charter of the corporation
and the officer is elected by the directors or stockholders. On the other hand, This particular argument baffles us. P.D. 902-A cannot be any clearer.
an "employee" usually occupies no office and generally is employed not by Sec. 5(c) of said law expressly covers both election and appointment of
action of the directors or stockholders but by the managing officer of the corporate directors, trustees, officers and managers.[14]
corporation who also determines the compensation to be paid to such
employee. It is of no consequence, likewise, that the complaint of private
respondent for illegal dismissal includes money claims, jurisdiction remains
In the case at bar, considering that herein petitioner, unlike an ordinary with the SEC as ruled in the case of Cagayan de Oro Coliseum, Inc. v. Office
employee, was appointed by respondent corporation's Board of Trustees in of the MOLE:[15]
its memorandum of October 30, 1990, she is deemed an officer of the
corporation. Perforce, Section 5(c) of Presidential Decree No. 902-A, which Although the reliefs sought by Chaves appear to fall under the jurisdiction of
provides that the SEC exercises exclusive jurisdiction over controversies in the labor arbiter as they are claims for unpaid salaries and other
the election or appointment of directors, trustees, officers or managers of remunerations for services rendered, a close scrutiny thereof shows that said
corporations, partnerships or associations, applies in the present dispute. claims are actually part of the perquisites of his position in, and therefore
Accordingly, jurisdiction over the same is vested in the SEC, and not in the interlinked with his relations with the corporation. In Dy vs. NLRC, the Court
Labor Arbiter or the NLRC. said: "(t)he question of remuneration involving as it does, a person who is not
a mere employee but a stockholder and officer, an integral part, it might be
Supplementing the afore-quoted ruling, in Lozon v. NLRC[11] and said, of the corporation, is not a simple labor problem but a matter that
Espino v. NLRC,[12] citing Fortune Cement Corp. v. NLRC,[13] we declared comes within the area of corporate affairs and, management, and is in fact a
that: corporate controversy in contemplation of the Corporation Code."

981
WHEREFORE, the petition for certiorari is given DUE COURSE, the
assailed resolutions of the NLRC are hereby REVERSED and the Order of
the Labor Arbiter dated 29 December 1992 REINSTATED.

SO ORDERED.

Bellosillo, Vitug, and Hermosisima, Jr., JJ., concur.

Padilla, (Chairman), no part; member of Magallanes Village Association.

[G.R. No. 121143. January 21, 1997]

PURIFICACION G. TABANG, petitioner, vs. NATIONAL LABOR


RELATIONS COMMISSION and PAMANA GOLDEN CARE
MEDICAL CENTER FOUNDATION, INC., respondents.

DECISION

REGALADO, J.:

This is a petition for certiorari which seeks to annul the resolution of the
National Labor Relations Commission (NLRC), dated June 26, 1995,
affirming in toto the order of the labor arbiter, dated April 26, 1994, which

982
dismissed petitioners complaint for illegal dismissal with money claims for her position as member of the Board of Trustees, hence, her dismissal is an
lack of jurisdiction. intra-corporate controversy which falls within the exclusive jurisdiction of the
Securities and Exchange Commission (SEC).
The records show that petitioner Purificacion Tabang was a founding
member, a member of the Board of Trustees, and the corporate secretary of Petitioner opposed the motion to dismiss, contending that her position
private respondent Pamana Golden Care Medical Center Foundation, Inc., a as Medical Director and Hospital Administrator was separate and distinct
non-stock corporation engaged in extending medical and surgical services. from her position as member of the Board of Trustees. She claimed that
there is no intra-corporate controversy involved since she filed the complaint
On October 30, 1990, the Board of Trustees issued a memorandum in her capacity as Medical Director and Hospital Administrator, or as an
appointing petitioner as Medical Director and Hospital Administrator of private employee of private respondent.
respondents Pamana Golden Care Medical Center in Calamba, Laguna.
On April 26, 1994, the labor arbiter issued an order dismissing the
Although the memorandum was silent as to the amount of remuneration complaint for lack of jurisdiction. He ruled that the case falls within the
for the position, petitioner claims that she received a monthly retainer fee of jurisdiction of the SEC, pursuant to Section 5 of Presidential Decree No. 902-
five thousand pesos (P5,000.00) from private respondent, but the payment A. [1]
thereof was allegedly stopped in November, 1991.
Petitioners motion for reconsideration was treated as an appeal by the
As medical director and hospital administrator, petitioner was tasked to labor arbiter who consequently ordered the elevation of the entire records of
run the affairs of the aforesaid medical center and perform all acts of the case to public respondent NLRC for appellate review. [2]
administration relative to its daily operations.
On appeal, respondent NLRC affirmed the dismissal of the case on the
On May 1, 1993, petitioner was allegedly informed personally by Dr. additional ground that the position of a Medical Director and Hospital
Ernesto Naval that in a special meeting held on April 30, 1993, the Board of Administrator is akin to that of an executive position in a corporate ladder
Trustees passed a resolution relieving her of her position as Medical Director structure, hence, petitioners removal from the said position was an intra-
and Hospital Administrator, and appointing the latter and Dr. Benjamin corporate controversy within the original and exclusive jurisdiction of the
Donasco as acting Medical Director and acting Hospital Administrator, SEC. [3]
respectively. Petitioner averred that she thereafter received a copy of said
board resolution. Aggrieved by the decision, petitioner filed the instant petition which we
find, however, to be without merit.
On June 6, 1993, petitioner filed a complaint for illegal dismissal and
non-payment of wages, allowances and 13th month pay before the labor We agree with the findings of the NLRC that it is the SEC which has
arbiter. jurisdiction over the case at bar. The charges against herein private
respondent partake of the nature of an intra-corporate controversy. Similarly,
Respondent corporation moved for the dismissal of the complaint on the the determination of the rights of petitioner and the concomitant liability of
ground of lack of jurisdiction over the subject matter. It argued that petitioners private respondent arising from her ouster as a medical director and/or
position as Medical Director and Hospital Administrator was interlinked with

983
hospital administrator, which are corporate offices, is an intra-corporate belied by the tenor of the memorandum itself. It states: We hope that you will
controversy subject to the jurisdiction of the SEC. uphold and promote the mission of our foundation, [10] and this cannot be
construed other than in reference to her position or capacity as a corporate
Contrary to the contention of petitioner, a medical director and a hospital trustee.
administrator are considered as corporate officers under the by-laws of
respondent corporation. Section 2(i), Article I thereof states that one of the A corporate officers dismissal is always a corporate act, or an intra-
powers of the Board of Trustees is (t)o appoint a Medical Director, corporate controversy, and the nature is not altered by the reason or wisdom
Comptroller/Administrator, Chiefs of Services and such other officers as it with which the Board of Directors may have in taking such action. [11] Also, an
may deem necessary and prescribe their powers and duties. [4] intra-corporate controversy is one which arises between a stockholder and
the corporation. There is no distinction, qualification, nor any exemption
The president, vice-president, secretary and treasurer are commonly whatsoever. The provision is broad and covers all kinds of controversies
regarded as the principal or executive officers of a corporation, and modern between stockholders and corporations. [12]
corporation statutes usually designate them as the officers of the corporation.
[5]
However, other offices are sometimes created by the charter or by-laws of With regard to the amount of P5,000.00 formerly received by herein
a corporation, or the board of directors may be empowered under the by- petitioner every month, the same cannot be considered as compensation for
laws of a corporation to create additional offices as may be necessary.[6] her services rendered as Medical Director and Hospital Administrator. The
vouchers[13] submitted by petitioner show that the said amount was paid to
It has been held that an office is created by the charter of the her by PAMANA, Inc., a stock corporation which is separate and distinct from
corporation and the officer is elected by the directors or stockholders. [7] On herein private respondent. Although the payments were considered
the other hand, an employee usually occupies no office and generally is advances to Pamana Golden Care, Calamba branch, there is no evidence to
employed not by action of the directors or stockholders but by the managing show that the Pamana Golden Care stated in the vouchers refers to herein
officer of the corporation who also determines the compensation to be paid to respondent Pamana Golden Care Medical Center Foundation, Inc.
such employee.[8]
Pamana Golden Care is a division of Pamana, Inc., while respondent
In the case at bar, considering that herein petitioner, unlike an ordinary Pamana Golden Care Medical Center Foundation, Inc. is a non-stock, non-
employee, was appointed by respondent corporations Board of Trustees in profit corporation. It is stated in the memorandum of petitioner that Pamana,
its memorandum of October 30, 1990, [9] she is deemed an officer of the Inc. is a stock and profit corporation selling pre-need plan for education,
corporation. Perforce, Section 5(c) of Presidential Decree No. 902-A, which pension and health care. The health care plan is called Pamana Golden Care
provides that the SEC exercises exclusive jurisdiction over controversies in Plan and the holders are called Pamana Golden Care Card Holders or,
the election or appointment of directors, trustees, officers or managers of simply, Pamana Members. [14]
corporations, partnerships or associations, applies in the present
dispute. Accordingly, jurisdiction over the same is vested in the SEC, and not It is an admitted fact that herein petitioner is a retained physician of
in the Labor Arbiter or the NLRC. Pamana, Inc., whose patients are holders of the Pamana Golden Care
Card. In fact, in her complaint[15] filed before the Regional Trial Court of
Moreover, the allegation of petitioner that her being a member of the Calamba, herein petitioner is asking, among others, for professional fees
Board of Trustees was not one of the considerations for her appointment is and/or retainer fees earned for her treatment of Pamana Golden Care card

984
holders.[16]Thus, at most, said vouchers can only be considered as proof of
payment of retainer fees made by Pamana, Inc. to herein petitioner as a
retained physician of Pamana Golden Care.

Moreover, even assuming that the monthly payment of P5,000.00 was a


valid claim against respondent corporation, this would not operate to
effectively remove this case from the jurisdiction of the SEC. In the case of
Cagayan de Oro Coliseum, Inc. vs. Office of the Minister of Labor and
Employment, etc., et al.,[17] we ruled that (a)lthough the reliefs sought by
Chavez appear to fall under the jurisdiction of the labor arbiter as they are
claims for unpaid salaries and other remunerations for services rendered, a
close scrutiny thereof shows that said claims are actually part of the
perquisites of his position in, and therefore interlinked with, his relations with
the corporation. In Dy, et al., vs. NLRC, et al., the Court said: (t)he question
of remuneration involving as it does, a person who is not a mere employee
but a stockholder and officer, an integral part, it might be said, of the
corporation, is not a simple labor problem but a matter that comes within the
area of corporate affairs and management and is in fact a corporate
controversy in contemplation of the Corporation Code.

WHEREFORE, the questioned resolution of the NLRC is hereby


AFFIRMED, without prejudice to petitioners taking recourse to and seeking
relief through the appropriate remedy in the proper forum.

SO ORDERED.

Romero, Puno, Mendoza, and Torres, Jr., JJ., concur.

985
In connection with this complaint, plaintiff moved for the issuance of a writ of
preliminary injunction to restrain defendant Jose Manuel Lezama from
managing the corporation pending the determination of this case, but after
hearing where parties presented testimonial and documentary evidence, the
court denied the motion. Thereafter, by agreement of the parties and without
any trial on the merits, the case was submitted for judgment on the sole legal
question of whether plaintiff could be legally removed as manager of the
corporation merely by resolution of the board of directors or whether the
affirmative vote of 2/3 of the paid shares of stocks was necessary for that
purpose. And passing upon this legal point, the trial court held that the
removal of plaintiff was legal and dismissed the complaint without
G.R. No. L-10556 April 30, 1958 pronouncement as to costs. Plaintiff appealed to the Court of Appeals but
finding that the question at issue is one of law, the latter certified the case to
RICARDO GURREA, plaintiff-appellant, us for decision.
vs.
JOSE MANUEL LEZAMA, ET AL., defendants-appellees. Section 33 of the Corporation Law provides: "Immediately after the election,
the directors of a corporation must organize by the election of a president,
Fulgencio Vega and Felix D. Bacabac for appellant. who must be one of their number, a secretary or clerk who shall be a resident
Jose Manuel Lezama for appellees. of the Philippines . . . and such other officers as may be provided for in the
Jose Manuel Lezama and Genivera F. de Lezama. Domingo B. Lauren for by-laws." The by-laws of the instant corporation in turn provide that in the
the other appellees. board of directors there shall be a president, a vice-president, a secretary
and a treasurer. These are the only ones mentioned therein as officers of the
BAUTISTA ANGELO, J.: corporation. The manager is not included although the latter is mentioned as
the person in whom the administration of the corporation is vested, and with
Plaintiff instituted this action in the Court of First Instance of Iloilo to have the exception of the president, the by-laws provide that the officers of the
Resolution No. 65 of the Board of Directors of the La Paz Ice Plant and Cold corporation may be removed or suspended by the affirmative vote of 2/3 of
Storage Co., Inc., removing him from his position of manager of said the corporation (Exhibit A).
corporation declared null and void and to recover damages incident thereto.
The action is predicated on the ground that said resolution was adopted in From the above the following conclusion is clear: that we can only regard as
contravention of the provisions of the by-laws of the corporation, of the officers of a corporation those who are given that character either by the
Corporation Law and of the understanding, intention and agreement reached Corporation Law or by its by-laws. The rest can be considered merely as
among its stockholders. employees or subordinate officials. And considering that plaintiff has been
appointed manager by the board of directors and as such does not have the
Defendant answered the complaint setting up as defense that plaintiff had character of an officer, the conclusion is inescapable that he can be
been removed by virtue of a valid resolution. suspended or removed by said board of directors under such terms as it may
see fit and not as provided for in the by-laws. Evidently, the power to appoint
carries with it the power to remove, and it would be incongruous to hold that

986
having been appointed by the board of directors he could only be removed of any person in the organization. (Grange, Corporation Law for
by the stockholders. Officers and Directors, p. 432; Emphasis supplied.)

The above interpretation finds also support in the American authorities. One distinction between officers and agents of a corporation lies in
Fletcher, in his treatise, states the rule in the following wise: "It is sometimes the manner of their creation. An officer is created by the charter of
important to determine whether a person representing a corporation is to be the corporation, and the officer is elected by the directors or the
classed as an officer of the company or merely as an agent or employee, stockholders. An agency is usually created by the officers, or one or
especially in construing statutes renting only to 'officers' of corporations. more of them, and the agent is appointed by the same authority. It is
Generally the officers of a corporation are enumerated in its charter or by- clear that the two terms officers and agents are by no means
laws, and include a president, vice-president, secretary, treasurer and interchangeable. One, deriving its existence from the other, and
sometimes others. The statutes in most of the states expressly provide for being dependent upon that other for its continuation, is necessarily
the election of a president, secretary and treasurer, and then provide, that restricted in its powers and duties, and such powers and duties, are
there shall be such other officers, agents and factors as the corporation shall not necessarily the same as those pertaining to the authority creating
authorize for that purpose. If the charter expressly enumerates who shall be it. The officers, as such, are the corporation. An agent is an
officers of the company, a person whose position is not enumerated is not an employee. "A mere employment, however liberally compensated,
officer as to members of the corporation, since the charter is conclusive upon does not rise to the dignity of an office."21 Am and Eng. Enc. Law
them" (Fletcher, Cyclopedia of the Law of Private Corporations, Vol. II, p. 19). (2d Ed.) 836. In Wheeler and Wilson Mfg. Co. vs. Lawson, 57 Wis.
It has been likewise held "that the offices pertaining to a private corporation 400, 15 N. W. 398, it was held that under a statute requiring an
are defined in its charter and by-laws, and that no other positions in the affidavit to be made by an officer of a corporation, the general agent
service of the corporation are offices" (Ann. 53 A.L.R., 599). or managing agent, within the state, of a foreign corporation is not an
officer. In Farmers' Loan and Trust Co. vs. Warring, 20 Wis. 305,
Indeed, there are authorities galore that hold that a general manager is not service was made upon the "principal agent" of a corporation holding
an officer of a corporation, even if his powers and influence may be as great in trust a railroad, when the statute required service upon a "principal
as those of any officer in said organization. officer." In answering the question whether or not the agent was a
principal officer the court said: "It is evident he was not, and must be
Officers Distinguished from Mere Employees. As already stated, regarded only as an agent not an officer of any kind, much less a
both officers and employees are agents of the corporation and the principal officer." A ruling that a "general manager" of a corporation
difference between them is largely one of degree; the officers are the was not authorized to verify pleadings, under a statute requiring
most important employees exercising greater authority or power in verification by "an officer" was made in Meton vs. Isham Wagon Co.
the management of the business. Ordinarily, too, the principal offices (Sup.) 4 N. Y. Supp. 215. In Raleigh, etc. R. Co. vs. Pullman Co., 122
are designated by statute, charter or by-law provisions, and specific Ga. 704, 5O S.E. 1008 (4), it was held that the term "general
duties are imposed upon certain officers. Thus the state statute or a manager." as applied to one representing a corporation, and
by-law may provide that stock certificates shall be signed by the especially a railroad corporation, imported an agent of a very
president and countersigned by the secretary or treasurer. The extensive authority; but it was not ruled that even the term "general
general manager of a corporation is not ordinarily classed as an manager" would import that the person holding that position was
officer, but his powers and influence may be quite as great as those necessarily an officer of the company. One distinction between an
officer and an agent suggested in Commonwealth vs. Christian, 9

987
Phila. (Pa.) 558, is that on officer of a corporation, if illegally corporation. This liability flows from the nature of his duties which are
excluded from his office, may by mandamus compel the corporation, delegated to him by the board of directors. He is paid for them. Hence, he
to reinstate him; while an agent may be dismissed without cause, has to answer for them should he use it in violation of law. In the case of
and his only remedy would be compensation in damages. It would Robinson vs. Moark-Nemo Consol Mining Co., et al., 163 S. W. 889, in
not be contended that the "general agent of the defendant at connection with the liability of the manager, the court said:
Columbus," in the event of his discharge, could be reinstated by
mandamus. We do not think the general agent at Columbus was an Common justice and common sense demand that, where those in
officer of the defendant company. Therefore his alleged waiver of a charge and control of the management of a corporation direct it
condition in the policy was not binding upon the company. along paths of wrongdoing, they should be held accountable by law. .
(Vardeman vs. Penn. Mut. Life Ins. Co. 125 Ga. 117, 54 S.E. P. 66; . . This doctrine will prevent many wrongs, and have a salutary
Emphasis supplied.) influence in bringing about the lawful and orderly management of
corporations.
The plaintiff-predicates this action on said contract, and claims that
the same being signed by the defendant through its "general It is claimed that the cases of Meton vs. Isham Wagon, 4 N.Y.S., 215 and
manager" if admitted evidence, would show sufficient authority prima State vs. Bergs, 217 N. W., 736, supporting the theory that a manager is not
facie to do any act which the directors could authorize or ratify. The necessarily an officer, are in illo tempore.1 It is submitted that we do not
instrument in question being signed by James W. Codle, "General adopt a rule just because it is new nor reject another just because it is old.
Manager" and no evidence in the trial being produced showing the We adopt a rule because it is a good and sound rule. The fact however is
duties of said manager or what kind of an office he was general that they are not the only authorities supporting that theory. Additional cases
manager of, the "general manager" without proof as to the nature of are cited by Fletcher in support thereof, such as the cases of
services performed by the persons called "general manager", have Vardeman vs. Penn. Mut. Life Ins. Co., supraStudebaker Bros. Co. vs. R. M.
no meaning in law, excepting that the persons bearing the title is an Rose Co., supra.
employee who has been designated with a title. It does not make
him an officer of the company employing him. (Studerbaker Bros. The dissenting opinion quotes from Thompson and Fletcher to support the
Co. vs. R. M. Rose Co., 119 N.Y.S pp. 970, 97; Emphasis supplied.) theory that the general manager of a corporation may be considered as its
principal officer even though not so mentioned in its charter or bylaws. We
We therefore hold that plaintiff has been properly removed when the board of have examined the cast cited in support of that theory but we have found that
directors of the instant corporation approved its Resolution No. 65 on June 3, they are not in point. Thus, we have found (1) that the parties involved are
1948. mostly outsiders who press their transactions against the corporation; (2) that
the point raised is whether the acts of the manager bind the corporation; (3)
We will now clarify some of the points raised by the distinguished dissenter in that the tendency of the courts is to hold the corporation liable for the acts of
his dissenting opinion. the manager so long as they are within the powers granted, hence, the
courts emphasized the importance of the position of manager; and (4) the
The fact that the "manager" of the corporation in the several statutes enacted position of manager was discussed from the point of view of an outsider and
by Congress is held criminally liable for violation of any of the penal not from the internal organization of the corporation, or in accordance with its
provisions therein prescribed does not make him an "officer" of the charter or by-laws. In the present case, however, the parties are the manager
and the corporation. And the solution of the problem hinges on the internal

988
government of the corporation where the charter and the by-laws are hold the theory that managers are not necessarily officers, then our theory is
necessarily involved in the determination of the rights of the parties. Indeed, supported by the majority view. Indeed, this view is upheld by nine
it has been held: "But it is urged that a corporation may have officers not states,2 whereas only six states adopt the view that managers are considered
recognized by the charter and by-laws. It is possible this may be as to principal officers of the corporation.3
matters arising between strangers and the corporation." [Com. vs. Christian,
9 Phila. (Pa.) 556; emphasis supplied]. The dissenting opinion quotes the provision of the bylaws relative to the
administration of the affairs of the instant corporation. It is there provided that
The cases on all fours with the present are those of State ex rel the affairs of the corporation shall be successively administered by (1) the
Blackwood vs. Brast, et al., 127 S. E. 507 and Denton Milling Co. vs. Blewitt, stockholders; (2) the board of directors; and(3) the manager. From this it
254 S. W. 236, 238, where the parties involved are the manager and the concludes that the manager should be considered an officer.
corporation. The issue raised is the relation of the manager towards the
corporation. The position of the manager is discussed from the point of view The above enumeration only emphasizes the different organs through which
of its internal government. And the holding of the court is that the manager is the affairs of the corporation should be administered and the order in which
the creation of the board of directors and the agent through whom the the powers should be exercised. The stockholders are the entity, composing
corporate duties of the board are performed. Hence, the manager holds his the whole corporation. The board of directors is the entity elected by the
position at the pleasure of the board. This stipulation is well expressed in the stockholders to manage the affairs of the corporation. And the manager is the
following words of Thompson: individual appointed by the board of directors to carry out the powers
delegated to him. In other words, the manager is the creation of the board of
The word "manager" implies agency, control, and presumptively directors. He is an alter egoof the board. As our law provides that only those
sufficient authority to bind a corporation in a case in which the enumerated in the charter or in the by-laws are considered officers, the
corporation was an actual party. It has been said that such agent manager who has not been so enumerated therein, but only incidentally
must have the same general supervision of the corporation as is mentioned in the order of management, cannot be considered an officer of
associated with the office of cashier or secretary. By whatever name the corporation within their purview.
he may be called, such, managing agent is a mere employee of the
board of directors and holds his position subject to the particular The mere fact that the directors are not mentioned in the by-laws as officers
contract of employment; and unless the contract of employment fixes does not deprive them of their category as such for their character as officer
his term of office, it may be terminated at the pleasure of the is secured in the charter. The same is not true with the manager. Customs
board. . . . The manager, like any other appointed agent, is subject to and corporate usages cannot prevail over the express provisions of the
removal when his term expires and on the request of the proper charter and the by-laws.
officer he should turn over his business to the corporation and,
where he refuses to comply, he may be restrained from the further There is no comparison between an appointee of the President, especially
performance of work for the corporation. (Thompson on one in the judiciary, and the appointee of the board of directors of a
Corporations, Vol. III, 3rd., pp. 209-210; Emphasis supplied.) corporation. In the first case, removal is especially provided for by law and in
the second, the appointee holds office at the pleasure of the board. And with
It is not correct to hold that the theory that a manager is not classed as an regard to the powers of the board of director, to remove a manager of the
officer of a corporation is only the minority view. If we consider the states that corporation, Thompson has the following to say:
hold that managers are merely agents or employees as among those that

989
. . . Below the grade of director and such other officers as are elected [G.R. No. L-58468. February 24, 1984.]
by the corporation at large, the general rule is that the officers of
private corporations hold their offices during the will of the directors, PHILIPPINE SCHOOL OF BUSINESS ADMINISTRATION, MANILA,
and are hence removable by the directors without assigning any ANTONIO M. MAGTALAS, JOSE ARANAS, JUAN D. LIM, JOSE F.
cause for the removal, except so far as their power may be PERALTA and BENJAMIN P. PAULINO, Petitioners, v. LABOR ARBITER
restrained by contract with the particular officer, just as any other LACANDOLA S. LEANO of the National Labor Relations Commission
employer may discharge his employee. Speaking generally, it may and RUFINO R. TAN, Respondents.
be said that the power to appoint carries with it the power to
remove. . . . the directors who appoint a ministerial officer may De Santos, Balgos and Perez Law Office, for Petitioners.
undoubtedly remove him at pleasure, and he has no remedy other
than an action for damages against the corporation for a breach of The Solicitor General for respondent Arbiter.
contract. . . . The ordinary ministerial and other lesser officers,
however, hold their offices during the pleasure of the directors and Caparas, Ilagan, Alcantara & Gatmaytan Law Office for Private
may be removed at will, without assigned cause. Of this class of Respondent.
officers and agents are the secretary and treasurer of the
corporation, the general manager, the assistant manager, the field
manager, the attorney of the company, an assistant horticulturist, and
the bookkeepers. (Thompson on Corporations, Vol. III, 521-523.)
SYLLABUS
Wherefore, the decision appealed from is affirmed, with costs against
appellant.

Paras, C.J., Montemayor, Concepcion, Reyes, J.B.L. and Endencia,


JJ., concur.
1. COMMERCIAL LAW; CORPORATION LAW; SECURITIES AND
EXCHANGE COMMISSION; JURISDICTION THEREOF VIS-A-VIS THE
NATIONAL LABOR RELATIONS COMMISSION; CASE AT BAR. The
jurisdiction of the Securities and Exchange Commission (SEC) vis-a-vis the
National Labor Relations Commission (NLRC) is in issue. An intracorporate
controversy would call for SEC jurisdiction. A labor dispute, that of the NLRC.

2. ID.; ID.; INTRA-CORPORATE CONTROVERSIES; LEGALITY OF


ELECTION OF CORPORATE DIRECTORS, IN THE NATURE OF; CASE AT
BAR. Basically, therefore, the question is whether the election of directors
on August 1, 1981 and the election of officers on September 5, 1981, which
resulted in TANs failure to be re-elected, were validly held. This is the crux of
the question that TAN has raised before the SEC. Even in his position paper

990
before the NLRC, TAN alleged that the election on August 1, 1981 of the DECISION
three directors was in contravention of the PSBA By-Laws providing that any
vacancy in the Board shall be filled by a majority vote of the stockholders at a
meeting specially called for the purpose. Thus, he concludes, the Board
meeting on September 5, 1981 was tainted with irregularity on account of the
presence of illegally elected directors without whom the results could have MELENCIO-HERRERA, J.:
been different. TAN invoked the same allegations in his complaint filed with
the SEC. So much so, that on December 17, 1981, the SEC (Case No. 2145)
rendered a Partial Decision annulling the election of the three directors and
ordered the convening of a stockholders meeting for the purpose of electing
new members of the Board. 9 The correctness of said conclusion is not for
us to pass upon in this case. TAN was present at said meeting and again This Petition for Certiorari questions the jurisdiction of respondent Labor
sought the issuance of injunctive relief from the SEC. The foregoing Arbiter over the present controversy (No. NCR-9-20-81) involving private
indubitably show that, fundamentally, the controversy is intra-corporate in respondent-complainant, Rufino R. Tan (TAN), and petitioners, the Philippine
nature. School of Business Administration (PSBA), a domestic corporation, and
majority of its Directors.chanrobles lawlibrary : rednad
3. ID.; ID.; SECURITIES AND EXCHANGE COMMISSION; JURISDICTION;
ORIGINAL AND EXCLUSIVE OVER INTRA-CORPORATE TAN is one of the principal stockholders of PSBA. Before September 5, 1981,
CONTROVERSIES UNDER PRESIDENTIAL DECREE NO. 902-A; CASE AT he was a Director and the Executive Vice President enjoying salaries and
BAR. Presidential Decree No. 902-A vests in the Securities and Exchange allowances.
Commission original and exclusive jurisdiction to hear and decide
controversies involving the election of directors, officers, or managers of On August 1, 1981, at the PSBA Board of Directors regular meeting, three
corporations registered with the Commission, the relation between and members were elected to fill vacancies in the seven-man body.
among its stockholders, and between them and the corporation. The instant
case is not a case of dismissal. The situation is that of a corporate office On September 5, 1981, also during a regular meeting, the Board declared all
having been declared vacant, and of TANs not having been elected corporate positions vacant except those of the Chairman and President, and
thereafter. The matter of whom to elect is a prerogative that belongs to the at the same time elected a new set of officers. TAN was not re-elected as
Board, and involves the exercise of deliberate choice and the faculty of Executive Vice-President. 1
discriminative selection. Generally speaking, the relationship of a person to a
corporation, whether as officer or as agent or employee, is not determined by On September 16, 1981, TAN filed with the National Labor Relations
the nature of the services performed, but by the incidents of the relationship Commission (NLRC) (National Capital Region) a complaint for Illegal
as they actually exist. (Bruce v. Travelers Ins. Co., 266 F2d 781, cited in 19 Dismissal against petitioners alleging that he was "summarily, illegally,
Am. Jur. 2d 526). irregularly and improperly removed from his position as Executive Vice-
President . . . without cause, investigation or notice" (NLRC Case No. NCR-
9-20-81) (the Labor Case, in brief).

On September 21, 1981, TAN also filed a one-million-peso damage suit

991
against petitioners before the then Court of First Instance of Rizal, Quezon declared illegal and that his reinstatement be ordered with full backwages
City, for illegal and oppressive removal (Civil Case No. Q-33444). and without loss of other benefits.chanroblesvirtualawlibrary

And, on September 28, 1981, TAN lodged before the Securities and We issued a Temporary Restraining Order, enjoining respondent Labor
Exchange Commission (SEC) another complaint against petitioners Arbiter from proceeding in any manner with the Labor Case, and
essentially questioning the validity of the PSBA elections of August 1, 1981 subsequently gave due course to the Petition.
and September 5, 1981, and of his "ouster" as Executive Vice-President
(SEC Case No. 2145).chanrobles lawlibrary : rednad The jurisdiction of the SEC vis-a-vis the NLRC is in issue. An intracorporate
controversy would call for SEC jurisdiction. A labor dispute, that of the NLRC.
On October 13, 1981, SEC issued a subpoena duces tecum commanding
the production of corporate documents, books and records. 2 Relevant and pertinent it is to note that the PSBA is a domestic corporation
duly organized and existing under our laws. General management is vested
On October 15, 1981, respondent Labor Arbiter also issued a subpoena in a Board of seven directors elected annually by the stockholders entitled to
duces tecum to submit the same books and documents. 3 vote, who serve until the election and qualification of their successors. Any
vacancy in the Board of Directors is filled by a majority vote of the subscribed
Before the NLRC, petitioners moved for the dismissal of TANs complaint, capital stock entitled to vote at a meeting specially called for the purpose,
invoking the principle against split jurisdiction. and the director or directors so chosen hold office for the unexpired term. 5
Corporate officers are provided for, among them, the Executive Vice-
On October 22, 1981, petitioners availed of this Petition contending mainly President, who is elected by the Board of Directors from their own number. 6
that:jgc:chanrobles.com.ph The officers receive such salaries or compensation as the Board of Directors
may fix. 7 The By-Laws likewise provide that should the position of any
"1. The respondent labor arbiter illegally assumed jurisdiction over the officer of the corporation become vacant by reason of death, resignation,
complaint for Illegal Dismissal because the failure of the private respondent disqualification, or otherwise, the Board of Directors, by a majority vote, may
to be re-elected to the corporate position of Executive Vice-President was an choose a successor or successors who shall hold office for the expired term
intra-corporate question over which the Securities and Exchange of his predecessor. 8
Commission had already assumed jurisdiction.
It was at a board regular monthly meeting held on August 1, 1981, that three
"2. The issuance by the respondent labor arbiter of a subpoena duces tecum directors were elected to fill vacancies. And, it was at the regular Board
was likewise without jurisdiction especially if considered in the light of Meeting of September 5, 1981 that all corporate positions were declared
procedural and substantial requirements therefor such that it is imperative vacant in order to effect a reorganization, and at the ensuing election of
that the supervising authority of this Honorable Court should be exercised to officers, TAN was not re-elected as Executive Vice-President.
prevent a substantial wrong and to do substantial justice." 4
Basically, therefore, the question is whether the election of directors on
TAN counter-argues that his sole and exclusive cause of action is illegal August 1, 1981 and the election of officers on September 5, 1981, which
dismissal, falling within the jurisdiction of the NLRC, for he was dismissed resulted in TANs failure to be re-elected, were validly held. This is the crux of
suddenly and summarily without cause in violation of his constitutional rights the question that TAN has raised before the SEC. Even in his position paper
to due process and security of tenure. He prays that his dismissal be before the NLRC, TAN alleged that the election on August 1, 1981 of the

992
three directors was in contravention of the PSBA By-Laws providing that any all of them and the corporation, partnership or association of which they are
vacancy in the Board shall be filled by a majority vote of the stockholders at a stockholders, members or associates, respectively; and between such
meeting specially called for the purpose. Thus, he concludes, the Board corporation, partnership or association and the state insofar as it concerns
meeting on September 5, 1981 was tainted with irregularity on account of the their individual franchise or right to exist as such entity;
presence of illegally elected directors without whom the results could have
been different. "c) Controversies in the election or appointments of directors, trustees,
officers or managers of such corporations, partnerships or associations. 10
TAN invoked the same allegations in his complaint filed with the SEC. So
much so, that on December 17, 1981, the SEC (Case No. 2145) rendered a This is not a case of dismissal. The situation is that of a corporate office
Partial Decision annulling the election of the three directors and ordered the having been declared vacant, and of TANs not having been elected
convening of a stockholders meeting for the purpose of electing new thereafter. The matter of whom to elect is a prerogative that belongs to the
members of the Board. 9 The correctness of said conclusion is not for us to Board, and involves the exercise of deliberate choice and the faculty of
pass upon in this case. TAN was present at said meeting and again sought discriminative selection. Generally speaking, the relationship of a person to a
the issuance of injunctive relief from the SEC. corporation, whether as officer or as agent or employee, is not determined by
the nature of the services performed, but by the incidents of the relationship
The foregoing indubitably show that, fundamentally, the controversy is intra- as they actually exist. 11
corporate in nature. It revolves around the election of directors, officers or
managers of the PSBA, the relation between and among its stockholders, With the foregoing conclusion, it follows that the issuance of a subpoena
and between them and the corporation. Private respondent also contends duces tecum by the Labor Arbiter will have to be set aside.
that his "ouster" was a scheme to intimidate him into selling his shares and to
deprive him of his just and fair return on his investment as a stockholder WHEREFORE, judgment is hereby rendered (1) ordering respondent Labor
received through his salary and allowances as Executive Vice-President. Vis- Arbiter to dismiss the complaint in NLRC Case No. NCR-9-20-81 for lack of
a-vis the NLRC, these matters fall within the jurisdiction of the SEC. jurisdiction; (2) nullifying the subpoena duces tecum issued by him in said
Presidential Decree No. 902-A vests in the Securities and Exchange case; and (3) declaring the Temporary Restraining Order heretofore issued
Commission:jgc:chanrobles.com.ph permanent.

". . . original and exclusive jurisdiction to hear and decide cases No costs.
involving:jgc:chanrobles.com.ph
SO ORDERED.
"a) Devices or schemes employed by or any acts, of the board of directors,
business associates, its officers or partners, amounting to fraud and Plana and Gutierrez, Jr., JJ., concur.
misrepresentation which may be detrimental to the interest of the public
and/or stockholders, partners, members of associations or organizations Teehankee, and Relova, JJ., concur in the result.
registered with the Commission.

"b) Controversies arising out of intra-corporate or partnership relations,


between and among stockholders, members, or associates; between any or

993
[G.R. No. 113928. February 1, 1996]

PEARSON & GEORGE, (S.E. ASIA), INC. petitioner, vs. NATIONAL


LABOR RELATIONS COMMISSION and LEOPOLDO
LLORENTE, respondents.

DECISION

DAVIDE, JR., J.:

In this special civil action for certiorari under Rule 65 of the Rules of
Court, the petitioner seeks the annulment of the decision of 22 April
1993[1] and order of 25 November 1993 [2] of public respondent National Labor
Relations Commission (NLRC) in NLRC CA No. 0034-07-92 which,
respectively, dismissed the petitioners appeal from the decision of the Labor
Arbiter in NLRC NCR Case No. 00-04-02127-90 and denied the petitioners
motion for reconsideration.

994
The petitioner insists that the Labor Arbiter and the NLRC do not have decision of the Commission; (3) for purposes of showing the timeliness of the
jurisdiction over the private respondents complaint for illegal dismissal arising petition, the petitioner has only to state, as it did, the date the order denying
out of his removal as Managing Director of the petitioner due to his non- the motion for reconsideration was received; and (4) in order to resolve the
reelection and the abolition of the said position. It claims that the matter is main issue raised in this petition, viz., whether the NLRC has jurisdiction over
intra-corporate and thus falls within the exclusive jurisdiction of the Securities this case, it was necessary to state the factual circumstances of the case.
and Exchange Commission (SEC) pursuant to Section 5(c) of P.D. No. 902-
A. After deliberating on the pleadings submitted by the parties, we resolved
to give due course to this petition and to require the parties to submit their
In a Manifestation submitted in lieu of the required comment on the respective memoranda.
petition, the Office of the Solicitor General agrees with the petitioner that the
NLRC has no jurisdiction over the private respondents complaint for illegal The factual antecedents as culled from the pleadings are not in dispute:
dismissal and prays that the NLRC be granted a new period within which to
file its own comment should it desire to do so. Private respondent Leopoldo Llorente (hereinafter Llorente) was a
member of the Board of Directors of the petitioner. In its organizational
The NLRC filed its own comment contending that it has jurisdiction over meeting on 12 January 1989, the Board of Directors elected among
the case because the private respondent was not just an incorporator but themselves the corporate officers. Llorente was elected as Vice-Chairman of
also a Managing Director and a line officer or an employee of the petitioner the Board and as Managing Director for a term of one year and until his
with a salary of P33,000.00 a month; hence, his complaint for illegal successor should have been duly elected pursuant to the petitioners by-laws.
dismissal as such employee is within the jurisdiction of the NLRC.
On 29 January 1990, Llorente was preventively suspended, with pay, by
The private respondent does not meet the substantive issues raised by reason of alleged anomalous transactions entered by him, which were
the petitioner but merely sets up the following defenses: (1) the petition was prejudicial to the interest of the petitioner.
filed long after the lapse often days provided for in Article 223 of the Labor
Code; (2) a special civil action for certiorari under Rule 65 is not the proper In a letter dated 1 February 1990, Llorente demanded from the
remedy because of the aforementioned provision; (3) the petition is defective petitioner access to his room which the latter allegedly sealed; compensation
because it does not allege when the petitioner received the NLRC decision; for his suspension or termination; and delivery of his stock certificates for
and (4) the petition raises factual issues. 9,998 shares.

In its Reply, the petitioner refutes the foregoing arguments of the private On 17 February 1990, the petitioner sent Llorente a letter requiring him
respondent by stating that (1) this Court may take cognizance of petitions to explain the acts enumerated therein which he allegedly committed.
questioning the decisions of the NLRC on the ground of lack or excess of
jurisdiction or grave abuse of discretion inspite of Article 223 of the Labor
On 27 February 1990, Llorente, through his counsel, protested his
Code making final the said decisions after ten calendar days from receipt
suspension and requested an examination of the supporting documents to
thereof; (2) the only way by which a labor case may reach this Court is
enable him to explain the accusations leveled against him, but to no avail.
through a petition for certiorari, which must be filed within a reasonable time
from receipt of the resolution denying the motion for reconsideration of the

995
At the regular stockholders meeting on 5 March 1990, the stockholders 2. To pay complainant attorneys fees equivalent to ten (10%) percent of his
of the petitioner elected a new set of directors. Llorente was not backwages;
reelected. On the same day, the new Board of Directors held a meeting
wherein it elected a new set of officers and abolished the position of 3. This Office is cognizant of the fact that due to the instant case, the
Managing Director. relations between the parties is so strained that the reinstatement may no
longer be feasible. Besides, there may be no equivalent position as the
On 12 March 1990, the petitioners counsel informed Llorente of his non- Office of the Managing Director had been abolished; and
reelection, the abolition of the position of Managing Director, and his
termination for cause. 4. To pay complainant moral damages in the amount of Fifty Thousand
(P50,000.00) Pesos.
On 11 April 1990, Llorente filed with the Labor Arbiter a complaint for
unfair labor practice, illegal dismissal, and illegal suspension alleging therein The petitioner appealed to the NLRC from the said decision.
that he was dismissed without due process of law. The case was docketed
as NLRC-NCR Case No. 00-04-02127-90. Relying on our decision in LEP International Philippines, Inc. vs.
National Labor Relations Commission,[3] the NLRC dismissed the petitioners
Upon receipt of the summons, the petitioner filed a Motion to Dismiss appeal and affirmed the decision of the Labor Arbiter. It likewise denied the
alleging therein that the case falls within the jurisdiction of the SEC and not of petitioners motion for reconsideration.
the NLRC.
Hence, this petition for certiorari in support of which the petitioner
In his order of 1 March 1991, the Labor Arbiter denied the said motion asserts as follows:
on the ground that Llorente was not merely acting as a Director but was
likewise doing the functions of a manager or line officer of the corporation.

The parties thereafter filed their respective position papers. THE NLRC ACTED WITHOUT JURISDICTION AND WITH GRAVE ABUSE
OF DISCRETION IN ASSUMING JURISDICTION OVER THE PRESENT
In a decision dated 18 May 1992, the Labor Arbiter found for Llorente, CONTROVERSY BETWEEN PETITIONER AND PRIVATE RESPONDENT
ruled that he was illegally terminated from employment, and disposed as WHO IS ADMITTEDLY ONE OF ITS INCORPORATORS/STOCKHOLDERS
follows: AND A CORPORATE OFFICER.

WHEREFORE, premises considered, judgment is hereby rendered finding II


the suspension and the eventual dismissal as illegal and ordering respondent
to: THE NLRC COMMITTED SERIOUS ERRORS AND ACTED WITH GRAVE
ABUSE OF DISCRETION IN FINDING THAT THE REMOVAL FROM
1. Pay the complainant his full backwages from January 29, 1990 to date or OFFICE BY NON-REELECTION OF PRIVATE RESPONDENT IS ONE OF
in the amount of Nine Hundred Twelve Thousand Seven Hundred Eighty ILLEGAL DISMISSAL CASE WHEREIN IT HAS JURISDICTION TO TRY
(P912,780.00) Pesos; AND DECIDE.

996
III Any question relating or incident to the election of the new Board of
Directors, the non-reelection of Liorente as a Director, his loss of the position
THE NLRC COMMITTED SERIOUS ERROR AND ACTED WITH GRAVE of Managing Director, or the abolition of the said office are intra-corporate
ABUSE OF DISCRETION, ASSUMING WITHOUT CONCEDING THAT IT matters. Disputes arising therefrom are intra-corporate disputes which, if
HAS JURISDICTION OVER THE PRESENT CONTROVERSY, THAT unresolved within the corporate structure of the petitioner, may be resolved in
PRIVATE RESPONDENT WAS ILLEGALLY DISMISSED FROM SERVICE. an appropriate action only by the SEC pursuant to its authority under
paragraphs (b) and (c), Section 5 of P.D. No. 902-A, [4] which provide as
The pith issue thus raised is whether it is the SEC or the NLRC which follows:
has jurisdiction over the complaint for illegal dismissal which the private
respondent had filed with the NLRC. SEC. 5. In addition to the regulatory and adjudicative functions of the
Securities and Exchange Commission over corporations, partnerships and
We agree with both the petitioner and the Office of the Solicitor General other forms of associations registered with it as expressly granted under
that the removal of Llorente as Managing Director is purely an intra-corporate existing laws and decrees, it shall have original and exclusive jurisdiction to
dispute which falls within the exclusive jurisdiction of the SEC and not of the hear and decide cases involving:
NLRC.
xxx xxx xxx
In reality, Llorente was not dismissed. If he lost the position of Managing
Director, it was primarily because he was not reelected as Director during the (b) Controversies arising out of intra-corporate or partnership relations,
regular stockholders meeting on 5 March 1990. The office of Managing between and among stockholders, members, or associates; between any or
Director presupposes that its occupant is a Director; hence, one who is not a all of them and the corporation, partnership or association of which they are
Director of the petitioner or who has ceased to be a Director cannot be stockholders, members or associates, respectively; and between such
elected or appointed as a Managing Director. Elsewise stated, the holding of corporation, partnership or association and the state insofar as it concerns
the position of Director is a prerequisite for the election, appointment, or their individual franchise or right to exist as such entity;
designation of Managing Director. If a Managing Director should lose his
position because he ceased to be a Director for any reason, such as non- (c) Controversies in the election or appointments of directors, trustees,
reelection as in the case of Liorente, such loss is not dismissal but failure to officers or managers of such corporations, partnership or associations.
qualify or to maintain a prerequisite for that position. Then too, the position of
Managing Director was abolished. Thus, in Philippine School of Business Administration vs. Leano, [5] we
ruled that a complaint for illegal dismissal arising from a Board of Directors
action declaring vacant all corporate positions except that of Chairman and
President, and from the non-reelection of the former Executive Vice-
President during the ensuing election of officers is not cognizable by the
NLRC.Pertinent portions of our opinion therein read as follows:

Basically, therefore, the question is whether the election of directors


on August 1, 1981 and the election of officers on September 5, 1981, which

997
resulted in TANs failure to be re-elected, were validly held. This is the crux of This is not a case of dismissal. The situation is that of a corporate office
the question that TAN has raised before the SEC. Even in his position paper having been declared vacant, and of TANs not having been elected
before the NLRC, TAN alleged that the election on August 1, 1981 of the thereafter. The matter of whom to elect is a prerogative that belongs to the
three directors was in contravention of the PSBA By-Laws providing that any Board, and involves the exercise of deliberate choice and the faculty of
vacancy in the Board shall be filled by a majority vote of the stockholders at a discriminative selection. Generally speaking, the relationship of a person to a
meeting specially called for the purpose. Thus, he concludes, the Board corporation, whether as officer or as agent or employee, is not determined by
meeting on September 5, 1981 was tainted with irregularity on account of the the nature of the services performed, but by the incidents of the relationship
presence of illegally elected directors without whom the results could have as they actually exist.
been different.
We reiterated this rule in Dy vs. National Labor Relation Commission,
[6]
TAN invoked the same allegations in his complaint filed with the SEC. which involved an action for illegal dismissal filed by a bank manager who
So much so, that on December 17, 1981, the SEC (Case No. 2145) rendered was not reelected as such, and in Fortune Cement Corporation vs. National
a Partial Decision annulling the election of the three directors and ordered Labor Relations Commission,[7] which involved a complaint for illegal
the convening of a stockholders meeting for the purpose of electing new dismissal instituted by an Executive Vice-President of the corporation who
members of the Board. The correctness of said conclusion is not for us to lost that position when he was dismissed as such by the Board of Directors
pass upon in this case. TAN was present at said meeting and again sought for loss of trust and confidence.
the issuance of injunctive relief from the SEC.
The reliance by the NLRC on LEP International Philippines, Inc. vs.
The foregoing indubitably show that, fundamentally, the controversy is National Labor Relations Commission is misplaced. What was challenged in
intra-corporate in nature. It revolves around the election of directors, officers that case was not the jurisdiction of the respondent Commission but its act of
or managers of the PSBA, the relation between and among its stockholders, upholding the validity of the dismissal of LEPs Chief Executive, who was not
and between them and the corporation. Private respondent also contends a stockholder, much less a director, of LEP but was merely a managerial
that his ouster was a scheme to intimidate him into selling his shares and to employee of the said company.
deprive him of his just and fair return on his investment as a stockholder
received through his salary and allowances as Executive Vice-President. Vis- WHEREFORE, the instant petition is GRANTED. The challenged
a-vis the NLRC, these matters fall within the jurisdiction of the SEC. decision of 22 April 1993 and order of 25 November 1993 of public
respondent National Labor Relations Commission in NLRC Case No. 0034-
xxx x x x xxx 07-92 and the decision of 18 May 1992 of the Labor Arbiter in NLRC NCR
Case No. 00-04-02 127-90 are hereby ANNULLED and SET ASIDE for
having been rendered without jurisdiction.

No pronouncement as to costs.

SO ORDERED.

998
Narvasa, C. J. (Chairman), Melo, Francisco, and Panganiban,
JJ., concur.

[G.R. No. 117473. April 15, 1997]

REAHS CORPORATION, SEVERO CASTULO, ROMEO PASCUA, and


DANIEL VALENZUELA, petitioners, vs. NATIONAL LABOR
RELATIONS COMMISSION, BONIFACIO RED, VICTORIA
PADILLA, MA. SUSAN R. CALWIT, SONIA DELA CRUZ, SUSAN
DE LA CRUZ, EDNA WAHINGON, NANCY B. CENITA and
BENEDICTO A. TULABING, respondents.

DECISION

PADILLA, J.:

This is a petition for certiorari under Rule 65 of the Rules of Court to


annul and set aside the decision dated 29 April 1994 rendered by the
National Labor Relations Commission (NLRC) in NLRC Case No. 005024-93
entitled "Bonifacio Red, et al., v. Reah's Corporation, et. al.", which affirmed
the decision of the Labor arbiter holding individual petitioners jointly and
severally liable with petitioner Reah's Corporation to pay private respondents'
claims for underpayment of wages, holiday pay, 13th month pay and
separation pay.

The facts, as culled by the labor arbiter from the position papers of both
parties, are as follows:

"Complainant Bonifacio Red alleges that he started working as a supervisor


at the health and sauna parlor of respondents from September 5, 1977 to
November 6, 1990, with a salary of P50.00, that the said establishment was
closed by respondents on November 6, 1990, without any notice and without
paying his wages, separation pay and other benefits under the law; and that
he works a minimum of twelve (12) hours a day without being paid overtime.

999
Complainant Benedicto Tulabing alleges that he started on December 16, On 6 May 1993, the labor arbiter rendered judgment dismissing private
1986 up to November 6, 1990 in the same establishment with a salary respondents' complaints for unfair labor practice and illegal dismissal but
of P26.00 a day; that he works thirteen (13) hours a day without payment of upholding the claims for separation pay, underpayment of wages, holiday pay
overtime pay. and 13th month pay. All eight (8) private respondents were awarded
separation pay. However, only Bonifacio Red and Benedicto Tulabing were
Complainant Nancy Cenita and Susan Calwit alleges [sic] that they were declared entitled to the claimed labor standard benefits as the rest were
hired as waitresses on May 20, 1990 up to November 6, 1990 and paid on found to have been employed on commission basis. The labor arbiter further
commission basis at P0.25 per bottle of beer sold to or consumed by the awarded attorney's fees to private respondents Bonifacio Red and Benedicto
customers and that they work ten (10) hours a day without being paid Tulabing amounting to ten (10%) percent of their adjudged money claims.
overtime.
Petitioners appealed the labor arbiter's decision to the NLRC,
Complainants Edna Wahingon, Susan dela Cruz, Sonia dela Cruz and contending mainly that Article 283 of the Labor Code, "exempts
Victoria Padilla claims [sic] working as attendants and were hired on different establishment(s) from payment of termination pay when the closure of
dates until November 6, 1990. All were paid on commission basis at the rate business is due to serious business losses or financial reverses"; that
of twenty (20%) percent of the service fee paid by the customers, P90.00 petitioners Castulo, Pascua and Valenzuela, while admittedly the acting
and P110.00 respectively, for ordinary and VIP service; that they render(ed) chairman of the board, board member and accountant acting manager
eleven (11) hours of work a day without being paid overtime; and that the respectively of Reah's Corporation, cannot be held jointly and severally liable
closure of the health parlor was illegal as they were not notified. with Reah's "unless there is evidence to show that the cause of the closure of
the business was due to the criminal negligence of the [respondent] officers."
On the other hand, respondents allege that sometime in 1986, a certain Ms.
Soledad Domingo, the sole proprietress and operator of Rainbow Sauna The NLRC dismissed the appeal based on the following dispositions:
located at 316 Araneta Avenue, Quezon City, offered to sell her business to
respondent Reah's Corporation. After the sale, all the assets of Ms. Domingo "Anent the issue on separation pay, Article 283 of the Labor Code provides
were turned over to respondent Reah's, which put a sing-along coffee shop that '[T]he employer may x x x terminate the employment of any employee
and massage clinic; that complainant Red started his employment on the first due to x x x the closing or cessation of operation of the establishment or
week of December 1988 as a roomboy at P50.00/day and was given living undertaking x x x by serving a written notice on the workers and the Ministry
quarters inside the premises as he requested; that sometime in March 1989, of Labor and Employment at least one (1) month before the intended date
complainant Red asked permission to go to Bicol for a period of ten (10) thereof. x x x.' This, respondents failed to comply. Neither did respondents
days, which was granted, and was given an advance money of P1,200.00 to present any evidence to prove that Reah's closure was really due to
bring some girls from the province to work as attendants at the respondent's SERIOUS business losses or financial reverses. We only have respondents'
massage clinic; that it was only on January 1, 1990 that complainant Red mere say-so on the matter.
returned and was re-hired under the same terms and conditions of his
previous employment with the understanding that he will have to refund The Supreme Court held in Basilio Balasbas vs. NLRC, et. al. (G.R. No.
the P1,200.00 cash advance given to him; that due to poor business, 85286, August 24, 1992, 3rd Division, Romero, J.) that -
increase in the rental cost and the failure of Meralco to reconnect the
electrical services in the establishment, it suffered losses leading to its
closure."[1]

1000
'Under Article 283 of the Labor Code, the closure of a business Petitioners argue that since the charges of illegal dismissal and unfair
establishment or reduction of personnel is a ground for the termination of the labor practices were dismissed by the labor arbiter, they cannot be held
services of any employee unless the closing or retrenching is for the purpose solidarily liable with the corporation for the payment of separation pay and
of circumventing the provision of the law. But while business reverses can be labor standard benefits to private respondents, when they used their
a just cause for terminating employees, these must be sufficiently proved by business judgment to close the establishment because of serious business
the employer. (Indino vs. NLRC, G.R. No. 80352, September 29, 1989, 178 losses.They contend that even if they were the top corporate officers of
SCRA 168).' Reah's corporation at the time they closed the business, the corporation has
a personality that is separate and distinct from its officers and
Thus, we cannot but agree that complainants are entitled to the payment of stockholders. Since there was no finding that they violated Sec. 31 of the
separation pay."[2] Corporation Code[3] they cannot be held solidarily liable with the
corporation. Petitioners further maintain that the corporation also cannot be
Petitioners filed a motion for reconsideration but this was denied by the held liable because Article 283 of the Labor Code "orders payment of
NLRC on 30 August 1994. In the present petition, petitioners raise three (3) separation pay only when the closure of the business is due to causes other
issues which, for brevity and clarity, may be simplified as follows: than serious business losses or financial reverses".

I. Petitioners have obviously resorted to a misreading of the last sentence


of Article 283 which provides that -
WHETHER OR NOT PETITIONERS-OFFICERS CAN BE HELD JOINTLY
AND SEVERALLY LIABLE WITH THE CORPORATION IN THE PAYMENT " x x x In case of retrenchment to prevent losses and in cases of closures or
OF SEPARATION PAY TO PRIVATE RESPONDENTS UNDER ARTICLE cessation of operations of establishment or undertaking not due to serious
283 OF THE LABOR CODE. business losses or financial reverses, the separation pay shall be equivalent
to one (1) month pay or at least () month pay for every year of service,
whichever is higher. A fraction of at least six (6) months shall be considered
II.
as one (1) whole year."

WHETHER OR NOT THE OFFICERS OF REAH'S CORPORATION CAN BE


It is not the function of the law nor its intent to supplant the prerogative
HELD JOINTLY AND SEVERALLY LIABLE WITH THE CORPORATION IN
of management in running its business, such as, to compel the latter to
PAYMENT OF THE MONETARY CLAIMS AWARDED PRIVATE
operate at a continuing loss. Thus, Article 283 provides as an authorized
RESPONDENTS IN THE ABSENCE OF ANY FINDING OF UNFAIR LABOR
cause in the termination of employment the "closing or cessation of operation
PRACTICES OR ILLEGAL DISMISSAL.
of the establishment or undertaking". However, the burden of proving that the
termination was for a valid or authorized cause shall rest on the employer. [4] If
III. the business closure is due to serious losses or financial reverses, the
employer must present sufficient proof of its actual or imminent losses; it
WHETHER OR NOT THERE IS LEGAL BASIS FOR THE NLRC TO AFFIRM must show proof that the cessation of or withdrawal from business operations
THE AWARD OF 10% ATTORNEY'S FEES TO PRIVATE RESPONDENTS. was bona fide in character.[5]

1001
The grant of separation pay, as an incidence of termination of The Solicitor General, in behalf of private respondents, argues that the
employment under Article 283, is a statutory obligation on the part of the doctrine laid down in the case of A.C. Ransom Labor Union - CCLU v.
employer and a demandable right on the part of the employee, except only NLRC[8] should be applied to the case at bar. In that case, a judgment against
where the closure or cessation of operations was due to serious business a corporation (A.C. Ransom) to reinstate its dismissed employees with back
losses or financial reverses and there is sufficient proof of this fact or wages was declared to be a continuing solidary liability of the company
condition. In the absence of such proof of serious business losses or president and all who may have thereafter succeeded to said office after the
financial reverses, the employer closing his business is obligated to pay his records failed to identify the officer or agents directly responsible for failure to
employees and workers their separation pay. pay the back wages of its employees. The Court noted Ransom's subterfuge
in organizing another family corporation while the case was on litigation with
The rule, therefore, is that in all cases of business closure or cessation the intent to phase out the existing corporation in case of an adverse
of operation or undertaking of the employer, the affected employee is entitled decision, as what actually happened when it ceased operations a few months
to separation pay. This is consistent with the state policy of treating labor as after the labor arbiter ruled in favor of Ransom's employees.
a primary social economic force, affording full protection to its rights as well
as its welfare.[6] The exception is when the closure of business or cessation The basis, said the Court, is found in Article 212(c) of the Labor Code
of operations is due to serious business losses or financial reverses; duly which provides that "an employer includes any person acting in the interest of
proved, in which case, the right of affected employees to separation pay is an employer, directly or indirectly.""Since Ransom is an artificial person, it
lost for obvious reasons. In the case at bar, the corporation's alleged serious must have an officer who can be presumed to be the employer, x x x. The
business losses and financial reverses were not amply shown or proved. corporation only in the technical sense is the employer."

We now proceed to rule on the corollary issue of whether or not This ruling was eventually applied by the Court in the following
individual petitioners Castulo, Pascua and Valenzuela should be held cases: Maglutac v. NLRC[9] an illegal dismissal case, where the most ranking
liable in solidum with the corporation (REAH's) in the payment to private officer of Commart, petitioner therein, was held solidarily liable with the
respondents of separation pay and labor standard benefits. corporation which thereafter became insolvent and suspended
operations; Chua v. NLRC,[10] also an illegal dismissal case, where the vice-
As a general rule established by legal fiction, the corporation has a president of a corporation was held solidarily liable with the corporation for
personality separate and distinct from its officers, stockholders and the payment of the unpaid salaries of its president; and in Gudez v. NLRC,
[11]
members. Hence, officers of a corporation are not personally liable for their where the president and treasurer were held solidarily liable with the
official acts unless it is shown that they have exceeded their authority. This corporation which had ceased operations but failed to pay the wage and
fictional veil, however, can be pierced by the very same law which created it money claims of its employees.
when "the notion of the legal entity is used as a means to perpetrate fraud,
an illegal act, as a vehicle for the evasion of an existing obligation, and to These cases, however, should be construed still as exceptions to the
confuse legitimate issues". Under the Labor Code, for instance, when a doctrine of separate personality of a corporation which should remain as the
corporation violates a provision declared to be penal in nature, the penalty guiding rule in determining corporate liability to its employees. At the very
shall be imposed upon the guilty officer or officers of the corporation. [7] least, as what we held in Pabalan v. NLRC,[12] to justify solidary liability, "there
must be an allegation or showing that the officers of the corporation
deliberately or maliciously designed to evade the financial obligation of the
corporation to its employees", or a showing that the officers indiscriminately

1002
stopped its business to perpetrate an illegal act, as a vehicle for the evasion of the leased premises [sic] for failure to pay the increased monthly rentals
of existing obligations, in circumvention of statutes, and to confuse legitimate from P8,000 to P20,000."[14] Under the Rules of Evidence, petitioners are
issues. bound by the allegations contained in their pleading. Since petitioners
themselves have admitted that they have dissolved the corporation de facto,
In the case at bar, the thrust of petitioners' arguments was aimed at the Court presumes that Reah's Corporation had become insolvent and
confining liability solely to the corporation, as if the entity were an automaton therefore would be unable to satisfy the judgment in favor of its
designed to perform functions at the push of a button. The issue, however, is employees. Under these circumstances, we cannot allow labor to go home
not limited to payment of separation pay under Article 283 but also payment with an empty victory. Neither would it be oppressive to capital to hold
of labor standard benefits such as underpayment of wages, holiday pay and petitioners Castulo, Pascua and Valenzuela solidarily liable with Reah's
13th month pay to two of the private respondents. While there is no sufficient Corporation because the law presumes that they have acted in the latter's
evidence to conclude that petitioners have indiscriminately stopped the interest when they obstinately refused to grant the labor standard benefits
entity's business, at the same time, petitioners have opted to abstain from and separation pay due private respondent-employees.
presenting sufficient evidence to establish the serious and adverse financial
condition of the company. The last issue raised by petitioners is whether there is legal basis for the
payment of 10% attorney's fees out of the total amount awarded to private
As the NLRC aptly stated: respondents Red and Tulabing. The Court finds this portion of the assailed
decision to have been rendered with grave abuse of discretion as both the
"Neither did respondents (petitioners) present any evidence to prove that labor arbiter and the NLRC failed to make an express finding of fact and cite
Reah's closure was really due to SERIOUS business losses or financial the applicable law to justify the grant of such award. Under Article 111 of the
reverses. We only have respondents mere say-so on the matter." [13] Labor Code, 10% attorneys fees may be assessed only in cases where there
is an unlawful withholding of wages,[15] or under Article 222 those arising from
collective bargaining negotiations that may be charged against union funds in
This uncaring attitude on the part of the officers of Reah's gives
an amount to be agreed upon by the parties. None of these situations exists
credence to the supposition that they simply ignored the side of the workers
in the case at bar.
who, more or less, were only demanding what is due them in accordance
with law. In fine, these officers were conscious that the corporation was
violating labor standard provisions but they did not act to correct these WHEREFORE, the decision of respondent National Labor Relations
violations; instead, they abruptly closed business. Neither did they offer Commission is hereby AFFIRMED in so far as it holds petitioners Castulo,
separation pay to the employees as they conveniently resorted to a lame Pascua, and Valenzuela jointly and severally liable with Reah's Corporation
excuse that they suffered serious business losses, knowing fully well that to pay all private respondents separation pay and private respondents Red
they had no substantial proof in their hands to prove such losses. and Tulabing other monetary benefits but the award of ten percent (10%)
attorneys fees is hereby DELETED for lack of factual and legal basis.
The findings of the NLRC did not indicate whether or not Reah's
Corporation has continued its personality after it had stopped operations SO ORDERED.
when it closed its sing-along, coffee shop, and massage clinic in November
1990. But in its petition, petitioners aver, among others, that the "company Bellosillo, Vitug, and Kapunan, JJ., concur. Hermosisima, Jr., J., on
totally folded for lack of patrons, (disconnection of) light and discontinuance leave.

1003
[...]SMITH [...] 11
v.
[...] VAN GORKOM, [...] [...]

3 12

Supreme Court of Delaware. We hold: (1) that the Board's decision, reached September 20, 1980, to
[...] approve the proposed cash-out merger was not the product of an informed
Decided: January 29, 1985. business judgment; (2) that the Board's subsequent efforts to amend the
[...] Merger Agreement and take other curative action were ineffectual, both
legally and factually; and (3) that the Board did not deal with complete candor
45678 with the stockholders by failing to disclose all material facts, which they knew
or should have known, before securing the stockholders' approval of the
HORSEY, Justice (for the majority): merger.

9 13

This appeal from the Court of Chancery involves a class action brought by [...]
shareholders of the defendant Trans Union Corporation ("Trans Union" or
"the Company"), originally seeking rescission of a cash-out merger of Trans 141516
Union into the defendant New T Company ("New T"), a wholly-owned
subsidiary of the defendant, Marmon Group, Inc. ("Marmon"). Alternate relief Trans Union was a publicly-traded, diversified holding company, the principal
in the form of damages is sought against the defendant members of the earnings of which were generated by its railcar leasing business. During the
Board of Directors of Trans Union, [864] New T, and Jay A. Pritzker and period here involved, the Company had a cash flow of hundreds of millions of
Robert A. Pritzker, owners of Marmon.[1] dollars annually. However, the Company had difficulty in generating sufficient
taxable income to offset increasingly large investment tax credits (ITCs). [...]
10
1718
Following trial, the former Chancellor granted judgment for the defendant
directors by unreported letter opinion dated July 6, 1982. [2] Judgment was Beginning in the late 1960's, and continuing through the 1970's, Trans Union
based on two findings: (1) that the Board of Directors had acted in an pursued a program of acquiring small companies in order to increase
informed manner so as to be entitled to protection of the business judgment available taxable income. In July 1980, Trans Union Management prepared
rule in approving the cash-out merger; and (2) that the shareholder vote the annual revision of the Company's Five Year Forecast. This
approving the merger should not be set aside because the stockholders had report [...] referred to the ITC situation as a "nagging problem" and, given
been "fairly informed" by the Board of Directors before voting thereon. The that problem, the leasing company "would still appear to be constrained to a
plaintiffs appeal. tax breakeven." The report then listed four alternative uses of the projected

1004
1982-1985 equity surplus: (1) stock repurchase; (2) dividend increases; (3) a program. [...] According to Romans: They did not "come up" with a price for
major acquisition program; and (4) combinations of the above. The sale of the Company. They merely "ran the numbers" at $50 a share and at $60 a
Trans Union was not among the alternatives. The report emphasized that, share with the "rough form" of their cash figures at the time. Their "figures
despite the overall surplus, the operation of the Company would consume all indicated that $50 would be very easy to do but $60 would be very difficult to
available equity for the next several years, and concluded: "As a result, we do under those figures." This work did not purport to establish a fair price for
have sufficient time to fully develop our course of action." either the Company or 100% of the stock. It was intended to determine the
cash flow needed to service the debt that would "probably" be incurred in a
19 leveraged buyout, based on "rough calculations" without "any benefit of
experts to identify what the limits were to that, and so forth." [...]
-B-
23
20
At this meeting, Van Gorkom stated that he would be willing to take $55 per
On August 27, 1980, Van Gorkom met with Senior Management of Trans share for his own 75,000 shares. He vetoed the suggestion of a leveraged
Union. Van Gorkom reported on his lobbying efforts in Washington and his buy-out by Management, however, as involving a potential conflict of interest
desire to find a solution to the tax credit problem more permanent than a for Management. Van Gorkom, a certified public accountant and lawyer, had
continued program of acquisitions. Various alternatives were suggested and been an officer of Trans Union [866] for 24 years, its Chief Executive Officer
discussed preliminarily, including the sale of Trans Union to a company with for more than 17 years, and Chairman of its Board for 2 years. It is
a large amount of taxable income. noteworthy in this connection that he was then approaching 65 years of age
and mandatory retirement.
21
24
Donald Romans, Chief Financial Officer of Trans Union, stated that his
department had done a "very brief bit of work on the possibility of a leveraged For several days following the September 5 meeting, Van Gorkom pondered
buy-out." This work had been prompted by a media article which Romans the idea of a sale. [...]
had seen regarding a leveraged buy-out by management. The work
consisted of a "preliminary study" of the cash which could be generated by 25
the Company if it participated in a leveraged buyout. As Romans stated, this
analysis "was very first and rough cut at seeing whether a cash flow would Van Gorkom decided to meet with Jay A. Pritzker, a well-known corporate
support what might be considered a high price for this type of transaction." takeover specialist and a social acquaintance. However, rather than
approaching Pritzker simply to determine his interest in acquiring Trans
22 Union, Van Gorkom assembled a proposed per share price for sale of the
Company and a financing structure by which to accomplish the sale. Van
On September 5, at another Senior Management meeting which Van Gorkom Gorkom did so without consulting either his Board or any members of Senior
attended, Romans again brought up the idea of a leveraged buy-out as a Management except one: Carl Peterson, Trans Union's Controller. Telling
"possible strategic alternative" to the Company's acquisition Peterson that he wanted no other person on his staff to know what he was

1005
doing, but without telling him why, Van Gorkom directed Peterson to calculate that to be sure that $55 was the best price obtainable, Trans Union should be
the feasibility of a leveraged buy-out at an assumed price per share of $55. free to accept any better offer. Pritzker demurred, stating that his
Apart from the Company's historic stock market price, [5] and Van Gorkom's organization would serve as a "stalking horse" for an "auction contest" only if
long association with Trans Union, the record is devoid of any competent Trans Union would permit Pritzker to buy 1,750,000 shares of Trans Union
evidence that $55 represented the per share intrinsic value of the Company. stock at market price which Pritzker could then sell to any higher bidder. After
further discussion on this point, Pritzker told Van Gorkom that he would give
26 him a more definite reaction soon.

Having thus chosen the $55 figure, based solely on the availability of a 29
leveraged buy-out, Van Gorkom multiplied the price per share by the number
of shares outstanding to reach a total value of the Company of $690 million. [867] On Monday, September 15, Pritzker advised Van Gorkom that he was
Van Gorkom told Peterson to use this $690 million figure and to assume a interested in the $55 cash-out merger proposal and requested more
$200 million equity contribution by the buyer. Based on these assumptions, information on Trans Union. Van Gorkom agreed to meet privately with
Van Gorkom directed Peterson to determine whether the debt portion of the Pritzker, accompanied by Peterson, Chelberg, and Michael Carpenter, Trans
purchase price could be paid off in five years or less if financed by Trans Union's consultant from the Boston Consulting Group. The meetings took
Union's cash flow as projected in the Five Year Forecast, and by the sale of place on September 16 and 17. Van Gorkom was "astounded that events
certain weaker divisions identified in a study done for Trans Union by the were moving with such amazing rapidity."
Boston Consulting Group ("BCG study"). Peterson reported that, of the
purchase price, approximately $50-80 million would remain outstanding after 30
five years. Van Gorkom was disappointed, but decided to meet with Pritzker
nevertheless. On Thursday, September 18, Van Gorkom met again with Pritzker. At that
time, Van Gorkom knew that Pritzker intended to make a cash-out merger
27 offer at Van Gorkom's proposed $55 per share. Pritzker instructed his
attorney, a merger and acquisition specialist, to begin drafting merger
Van Gorkom arranged a meeting with Pritzker at the latter's home on documents. There was no further discussion of the $55 price. However, the
Saturday, September 13, 1980. Van Gorkom prefaced his presentation by number of shares of Trans Union's treasury stock to be offered to Pritzker
stating to Pritzker: "Now as far as you are concerned, I can, I think, show was negotiated down to one million shares; the price was set at $38-75 cents
how you can pay a substantial premium over the present stock price and pay above the per share price at the close of the market on September 19. At this
off most of the loan in the first five years. * * * If you could pay $55 for this point, Pritzker insisted that the Trans Union Board act on his merger proposal
Company, here is a way in which I think it can be financed." within the next three days, stating to Van Gorkom: "We have to have a
decision by no later than Sunday [evening, September 21] [...]
28
31
Van Gorkom then reviewed with Pritzker his calculations based upon his
proposed price of $55 per share. Although Pritzker mentioned $50 as a more Van Gorkom retained James Brennan, Esquire, to advise Trans Union on the
attractive figure, no other price was mentioned. However, Van Gorkom stated legal aspects of the merger. Van Gorkom did not consult with William

1006
Browder, a Vice-President and director of Trans Union and former head of its 35
legal department, or with William Moore, then the head of Trans Union's legal
staff. Ten directors served on the Trans Union Board, five inside (defendants
Bonser, O'Boyle, Browder, Chelberg, and Van Gorkom) and five outside
32 (defendants Wallis, Johnson, Lanterman, Morgan and Reneker). All directors
were present at the meeting, except O'Boyle who was ill. [...]
On Friday, September 19, Van Gorkom called a special meeting of the Trans
Union Board for noon the following day. He also called a meeting of the 36
Company's Senior Management to convene at 11:00 a.m., prior to the
meeting of the Board. No one, except Chelberg and Peterson, was told the Van Gorkom began the Special Meeting of the Board with a twenty-minute
purpose of the meetings. Van Gorkom did not invite Trans Union's investment oral presentation. Copies of the proposed Merger Agreement were delivered
banker, Salomon Brothers or its Chicago-based partner, to attend. too late for study before or during the meeting. [7] He reviewed the Company's
ITC and depreciation problems and the efforts theretofore made to solve
33 them. He discussed his initial meeting with Pritzker and his motivation in
arranging that meeting. Van Gorkom did not disclose to the Board, however,
Of those present at the Senior Management meeting on September 20, only the methodology by which he alone had arrived at the $55 figure, or the fact
Chelberg and Peterson had prior knowledge of Pritzker's offer. Van Gorkom that he first proposed the $55 price in his negotiations with Pritzker.
disclosed the offer and described its terms, but he furnished no copies of the
proposed Merger Agreement. Romans announced that his department had 37
done a second study which showed that, for a leveraged buy-out, the price
range for Trans Union stock was between $55 and $65 per share. Van Van Gorkom outlined the terms of the Pritzker offer as follows: Pritzker would
Gorkom neither saw the study nor asked Romans to make it available for the pay $55 in cash for all outstanding shares of Trans Union stock upon
Board meeting. completion of which Trans Union would be merged into New T Company, a
subsidiary wholly-owned by Pritzker and formed to implement the merger; for
34 a period of 90 days, Trans Union could receive, but could not actively solicit,
competing offers; the offer had to be acted on by the next evening, Sunday,
Senior Management's reaction to the Pritzker proposal was completely September 21; [...]
negative. [...] Romans objected to the price as being too low; [6] he was critical
of the timing and suggested that consideration should be given to the 38
adverse tax consequences of an all-cash deal for low-basis shareholders;
and he took the position that the agreement to sell Pritzker one million newly- Van Gorkom took the position that putting Trans Union "up for auction"
issued shares at market price would inhibit other offers, as would the through a 90-day market test would validate a decision by the Board that $55
prohibitions against soliciting bids and furnishing inside information [868] to was a fair price. He told the Board that the "free market will have an
other bidders. Romans argued that the Pritzker proposal was a "lock up" and opportunity to judge whether $55 is a fair price." Van Gorkom framed the
amounted to "an agreed merger as opposed to an offer." Nevertheless, Van decision before the Board not as whether $55 per share was the highest
Gorkom proceeded to the Board meeting as scheduled without further delay.

1007
price that could be obtained, but as whether the $55 price was a fair price The Board meeting of September 20 lasted about two hours. Based solely
that the stockholders should be given the opportunity to accept or reject. [8] upon Van Gorkom's oral presentation, Chelberg's supporting representations,
Romans' oral statement, Brennan's legal advice, and their knowledge of the
39 market history of the Company's stock,[9] the directors approved the proposed
Merger Agreement. [...]
Attorney Brennan advised the members of the Board that they might be sued
if they failed to accept the offer and that a fairness opinion was not required 45
as a matter of law.
The Merger Agreement was executed by Van Gorkom during the evening of
40 September 20 at a formal social event that he hosted for the opening of the
Chicago Lyric Opera. Neither he nor any other director read the agreement
Romans attended the meeting as chief financial officer of the Company. He prior to its signing and delivery to Pritzker.
told the Board that he had not been involved in the negotiations with Pritzker
and knew nothing about the merger proposal until [869] the morning of the 46
meeting; [...] Romans testified:
[...]
41
47
I told the Board that the study ran the numbers at 50 and 60,
and then the subsequent study at 55 and 65, and that was not the On Monday, September 22, the Company issued a press release announcing
same thing as saying that I have a valuation of the company at X that Trans Union had entered into a "definitive" Merger Agreement with an
dollars. But it was a way a first step towards reaching that affiliate of the Marmon Group, Inc., a Pritzker holding company. Within 10
conclusion. days of the public announcement, dissent among Senior Management over
the merger had become widespread. Faced with threatened resignations of
42 key officers, Van Gorkom met with Pritzker who agreed to several
modifications of the Agreement. [...]
Romans told the Board that, in his opinion, $55 was "in the range of a fair
price," but "at the beginning of the range." 48

43 Van Gorkom reconvened the Board on October 8 and secured the directors'
approval of the proposed amendments sight unseen. The Board also
[...] authorized the employment of Salomon Brothers, its investment [870] banker,
to solicit other offers for Trans Union during the proposed "market test"
period.
44

49

1008
The next day, October 9, Trans Union issued a press release announcing: (1) 58
that Pritzker had obtained "the financing commitments necessary to
consummate" the merger with Trans Union; (2) that Pritzker had acquired We turn to the issue of the application of the business judgment rule to the
one million shares of Trans Union common stock at $38 per share; (3) that September 20 meeting of the Board.
Trans Union was now permitted to actively seek other offers and had
retained Salomon Brothers for that purpose; and (4) that if a more favorable 59
offer were not received before February 1, 1981, Trans Union's shareholders
would thereafter meet to vote on the Pritzker proposal.
[...]

50
60616263646566

It was not until the following day, October 10, that the actual amendments to
Under Delaware law, the business judgment rule is the offspring of the
the Merger Agreement were prepared by Pritzker and delivered to Van
fundamental principle, codified in 8 Del.C. 141(a), that the business and
Gorkom for execution. As will be seen, the amendments were considerably at
affairs of a Delaware corporation are managed by or under its board of
variance with Van Gorkom's representations of the amendments to the Board
directors.[...] In carrying out their managerial roles, directors are charged with
on October 8; and the amendments placed serious constraints on Trans
an unyielding fiduciary duty to the corporation and its shareholders. [...] The
Union's ability to negotiate a better deal and withdraw from the Pritzker
business judgment rule exists to protect and promote the full and free
agreement. Nevertheless, Van Gorkom proceeded to execute what became
exercise of the managerial power granted to Delaware directors. [...] The rule
the October 10 amendments to the Merger Agreement without conferring
itself "is a presumption that in making a business decision, the directors of a
further with the Board members and apparently without comprehending the
corporation acted on an informed basis, in good faith and in the honest belief
actual implications of the amendments.
that the action taken was in the best interests of the company." Aronson,
supra at 812. Thus, the party attacking a board decision as uninformed must
51 rebut the presumption that its business judgment was an informed one. [...]

[...] 67

5253545556 The determination of whether a business judgment is an informed one turns


on whether the directors have informed themselves "prior to making a
On February 10, the stockholders of Trans Union approved the Pritzker business decision, of all material information reasonably available to
merger proposal. Of the outstanding shares, 69.9% were voted in favor of the them." Id.[12]
merger; 7.25% were voted against the merger; and 22.85% were not voted.
68
57
Under the business judgment rule there is no protection for directors who
II. have made "an unintelligent or unadvised judgment." [...] A director's duty to
inform himself in preparation for a decision derives from the fiduciary

1009
capacity in which he serves the corporation and its stockholders. [...] Since a 74
director is vested with the responsibility for the management of the affairs of
the corporation, he must execute that duty with the recognition that he acts In the specific context of a proposed merger of domestic corporations, a
on behalf of others. Such obligation does not tolerate faithlessness or self- director has a duty under 8 Del.C.251(b),[14] along with his fellow directors, to
dealing. But fulfillment of the fiduciary function requires more than the mere act in an informed and deliberate manner in determining whether to approve
absence of bad faith or fraud. Representation of the financial interests of an agreement of merger before submitting the proposal to the stockholders.
others imposes on a director an affirmative duty to protect those interests Certainly in the merger context, a director may not abdicate that duty by
and to proceed with a critical eye in assessing information of the type and leaving to the shareholders alone the decision to approve or disapprove the
under the circumstances present here. [...] agreement. [...]

69 75

Thus, a director's duty to exercise an informed business judgment is in [873] It is against those standards that the conduct of the directors of Trans Union
the nature of a duty of care, as distinguished from a duty of loyalty. Here, must be tested, as a matter of law and as a matter of fact, regarding their
there were no allegations of fraud, bad faith, or self-dealing, or proof thereof. exercise of an informed business judgment in voting to approve the Pritzker
Hence, it is presumed that the directors reached their business judgment in merger proposal.
good faith, [...] and considerations of motive are irrelevant to the issue before
us. 76

70 III.

The standard of care applicable to a director's duty of care has also been 77
recently restated by this Court. In Aronson, supra, we stated:
The defendants argue that the determination of whether their decision to
71 accept $55 per share for Trans Union represented an informed business
judgment requires consideration, not only of that which they knew and
While the Delaware cases use a variety of terms to describe learned on September 20, but also of that which they subsequently learned
the applicable standard of care, our analysis satisfies us that under and did over the following four-month [874] period before the shareholders
the business judgment rule director liability is predicated upon met to vote on the proposal in February, 1981. The defendants thereby seek
concepts of gross negligence. [...] to reduce the significance of their action on September 20 and to widen the
time frame for determining whether their decision to accept the Pritzker
7273 proposal was an informed one. Thus, the defendants contend that what the
directors did and learned subsequent to September 20 and through January
We again confirm that view. We think the concept of gross negligence is also 26, 1981, was properly taken into account by the Trial Court in determining
the proper standard for determining whether a business judgment reached by whether the Board's judgment was an informed one. We disagree with
a board of directors was an informed one.[13] this post hoc approach.

1010
78 82

The issue of whether the directors reached an informed decision to "sell" the As has been noted, the Board based its September 20 decision to approve
Company on September 20, 1980 must be determined only upon the basis of the cash-out merger primarily on Van Gorkom's representations. None of the
the information then reasonably available to the directors and relevant to directors, other than Van Gorkom and Chelberg, had any prior knowledge
their decision to accept the Pritzker merger proposal. This is not to say that that the purpose of the meeting was to propose a cash-out merger of Trans
the directors were precluded from altering their original plan of action, had Union. No members of Senior Management were present, other than
they done so in an informed manner. What we do say is that the question of Chelberg, Romans and Peterson; and the latter two had only learned of the
whether the directors reached an informed business judgment in agreeing to proposed sale an hour earlier. Both general counsel Moore and former
sell the Company, pursuant to the terms of the September 20 Agreement general counsel Browder attended the meeting, but were equally uninformed
presents, in reality, two questions: (A) whether the directors reached an as to the purpose of the meeting and the documents to be acted upon.
informed business judgment on September 20, 1980; and (B) if they did not,
whether the directors' actions taken subsequent to September 20 were 83
adequate to cure any infirmity in their action taken on September 20. We first
consider the directors' September 20 action in terms of their reaching an Without any documents before them concerning the proposed transaction,
informed business judgment. the members of the Board were required to rely entirely upon Van Gorkom's
20-minute oral presentation of the proposal. No written summary of the terms
79 of the merger was presented; the directors were given no documentation to
support the adequacy of $55 price per share for sale of the Company; and
-A- the Board had before it nothing more than Van Gorkom's statement of his
understanding of the substance of an agreement which he admittedly had
80 never read, nor which any member of the Board had ever seen.

On the record before us, we must conclude that the Board of Directors did 84
not reach an informed business judgment on September 20, 1980 in voting to
"sell" the Company for $55 per share pursuant to the Pritzker cash-out Under 8 Del.C. 141(e),[15] "directors are fully protected in relying in [875]
merger proposal. Our reasons, in summary, are as follows: good faith on reports made by officers." [...] The term "report" has been
liberally construed to include reports of informal personal investigations by
81 corporate officers, [...]However, there is no evidence that any "report," as
defined under 141(e), concerning the Pritzker proposal, was presented to
The directors (1) did not adequately inform themselves as to Van Gorkom's the Board on September 20.[16] Van Gorkom's oral presentation of his
role in forcing the "sale" of the Company and in establishing the per share understanding of the terms of the proposed Merger Agreement, which he had
purchase price; (2) were uninformed as to the intrinsic value of the Company; not seen, and Romans' brief oral statement of his preliminary study regarding
and (3) given these circumstances, at a minimum, were grossly negligent in the feasibility of a leveraged buy-out of Trans Union do not qualify as
approving the "sale" of the Company upon two hours' consideration, without 141(e) "reports" for these reasons: The former lacked substance because
prior notice, and without the exigency of a crisis or emergency. Van Gorkom was basically uninformed as to the essential provisions of the

1011
very document about which he was talking. Romans' statement was Trans Union market price and the amount of the Pritzker offer. Using market
irrelevant to the issues before the Board since it did not purport to be a price as a basis for concluding that the premium adequately reflected the true
valuation study. At a minimum for a report to enjoy the status conferred by value [876] of the Company was a clearly faulty, indeed fallacious, premise,
141(e), it must be pertinent to the subject matter upon which a board is called as the defendants' own evidence demonstrates.
to act, and otherwise be entitled to good faith, not blind, reliance. Considering
all of the surrounding circumstances hastily calling the meeting without 88
prior notice of its subject matter, the proposed sale of the Company without
any prior consideration of the issue or necessity therefor, the urgent time The record is clear that before September 20, Van Gorkom and other
constraints imposed by Pritzker, and the total absence of any documentation members of Trans Union's Board knew that the market had consistently
whatsoever the directors were duty bound to make reasonable inquiry of undervalued the worth of Trans Union's stock, despite steady increases in
Van Gorkom and Romans, and if they had done so, the inadequacy of that the Company's operating income in the seven years preceding the
upon which they now claim to have relied would have been apparent. merger. [...]

85 89

The defendants rely on the following factors to sustain the Trial Court's In the Proxy Statement, however, the directors reversed their position. There,
finding that the Board's decision was an informed one: (1) the magnitude of they stated that, although the earnings prospects for Trans Union were
the premium or spread between the $55 Pritzker offering price and Trans "excellent," they found no basis for believing that this would be reflected in
Union's current market price of $38 per share; (2) the amendment of the future stock prices. With regard to past trading, the Board stated that the
Agreement as submitted on September 20 to permit the Board to accept any prices at which the Company's common stock had traded in recent years did
better offer during the "market test" period; (3) the collective experience and not reflect the "inherent" value of the Company. But having referred to the
expertise of the Board's "inside" and "outside" directors; [17] and (4) their "inherent" value of Trans Union, the directors ascribed no number to it.
reliance on Brennan's legal advice that the directors might be sued if they Moreover, nowhere did they disclose that they had no basis on which to fix
rejected the Pritzker proposal. [...] "inherent" worth beyond an impressionistic reaction to the premium over
market and an unsubstantiated belief that the value of the assets was
86 "significantly greater" than book value. By their own admission they could not
rely on the stock price as an accurate measure of value. Yet, also by their
(1) own admission, the Board members assumed that Trans Union's market
price was adequate to serve as a basis upon which to assess the adequacy
87 of the premium for purposes of the September 20 meeting.

A substantial premium may provide one reason to recommend a merger, but 90


in the absence of other sound valuation information, the fact of a premium
alone does not provide an adequate basis upon which to assess the fairness [...]
of an offering price. Here, the judgment reached as to the adequacy of the
premium was based on a comparison between the historically depressed 91

1012
Indeed, as of September 20, the Board had no other information on which to Here, the record establishes that the Board did not request its Chief Financial
base a determination of the intrinsic value of Trans Union as a going Officer, Romans, to make any valuation study or review of the proposal to
concern. As of September 20, the Board had made no evaluation of the determine the adequacy of $55 per share for sale of the Company. On the
Company designed to value the entire enterprise, nor had the Board ever record before us: The Board rested on Romans' elicited response that the
previously considered selling the Company or consenting to a buy-out $55 figure was within a "fair price range" within the context of a leveraged
merger. Thus, the adequacy of a premium is indeterminate unless it is buy-out. No director sought any further information from Romans. No director
assessed in terms of other competent and sound valuation information that asked him why he put $55 at the bottom of his range. No director asked
reflects the value of the particular business. Romans for any details as to his study, the reason why it had been
undertaken or its depth. No director asked to see the study; and no director
92 asked Romans whether Trans Union's finance department could do a
fairness study within the remaining 36-hour [18] period available under the
[...] Pritzker offer.

93 95

We do not imply that an outside valuation study is essential to support an [...]


informed business judgment; nor do we state that fairness opinions by
independent investment bankers are required as a matter of law. Often 96979899
insiders familiar with the business of a going concern are in a better position
than are outsiders to gather relevant information; and under appropriate Thus, the record compels the conclusion that on September 20 the Board
circumstances, such directors may be fully protected in relying in good faith lacked valuation information adequate to reach an informed business
upon the valuation reports of their management. [...] judgment as to the fairness of $55 per share for sale of the Company.[20]

94 100

(2)

101

This brings us to the post-September 20 "market test" upon which the


defendants ultimately rely to confirm the reasonableness of their September
20 decision to accept the Pritzker proposal. In this connection, the directors
present a two-part argument: (a) that by making a "market test" of Pritzker's
$55 per share offer a condition of their September 20 decision to accept his
offer, they cannot be found to have acted impulsively or in an uninformed
manner on September 20; and (b) that the adequacy of the $17 premium for

1013
sale of the Company was conclusively established over the following 90 to Several of Trans Union's outside directors resolutely maintained that the
120 days by the most reliable evidence available the marketplace. Thus, Agreement as submitted was approved on the understanding that, "if we got
the defendants impliedly contend that the "market test" eliminated the need a better deal, we had a right to take it." Director Johnson so testified; but he
for the Board to perform any other form of fairness test either on September then added, "And if they didn't put that in the agreement, then the
20, or thereafter. management did not carry out the conclusion of the Board. And I just don't
know whether they did or not." The only clause in the Agreement as finally
102 executed to which the defendants can point as "keeping the door open" is the
following underlined statement found in subparagraph (a) of section 2.03 of
Again, the facts of record do not support the defendants' argument. There is the Merger Agreement as executed:
no evidence: (a) that the Merger Agreement was effectively amended to give
the Board freedom to put Trans Union up for auction sale to the highest 107
bidder; or (b) that a public auction was in fact permitted to occur. T[...]
The Board of Directors shall recommend to the stockholders
103 of Trans Union that they approve and adopt the Merger Agreement
(`the stockholders' approval') and to use its best efforts to obtain the
The Merger Agreement, specifically identified as that originally presented to requisite votes therefor. GL acknowledges that Trans Union directors
the Board on September 20, has never been produced by the defendants, may have a competing fiduciary obligation to the shareholders under
notwithstanding the plaintiffs' several demands for production before as well certain circumstances.
as during trial. No acceptable explanation of this failure to produce
documents has been given to either the Trial Court or this Court. 108
Significantly, neither the defendants nor their counsel have made the
affirmative representation that this critical document has been produced. Clearly, this language on its face cannot be construed as incorporating either
Thus, the Court is deprived of the best evidence on which to judge the merits of the two "conditions" described above: either the right to accept a better
of the defendants' position as to the care and attention which they gave to offer or the right to distribute proprietary information to third parties. The
the terms of the Agreement on September 20. logical witness for the defendants to call to confirm their construction of this
clause of the Agreement would have been Trans Union's outside attorney,
104 James Brennan. The defendants' failure, without explanation, to call this
witness again permits the logical inference that his testimony would not have
[...]Van Gorkom, conceding that he never read the Agreement, stated that he been helpful to them. The further fact that the directors adjourned, rather
was relying upon his understanding that, under corporate law, directors than recessed, the meeting without incorporating in the Agreement these
always have an inherent right, as well as a fiduciary duty, to accept a better important "conditions" further weakens the defendants' position. As has been
offer notwithstanding an existing contractual commitment by the Board. [...] noted, nothing in the Board's Minutes supports these claims. No reference to
either of the so-called "conditions" or of Trans Union's reserved right to test
the market appears in any notes of the Board meeting or in the Board
105106
Resolution accepting the Pritzker offer or in the Minutes of the meeting itself.
That evening, in the midst of a formal party which he hosted for the opening

1014
of the Chicago Lyric Opera, Van Gorkom executed the Merger Agreement defendants also concede that Trans Union never made a subsequent public
without he or any other member of the Board having read the instruments. announcement stating that it had in fact reserved the right to accept alternate
offers, the Agreement notwithstanding.
109
132
[...]
The public announcement of the Pritzker merger resulted in an "en masse"
110111112113 revolt of Trans Union's Senior Management. [...]

(4) 133134

114 Van Gorkom then advised Senior Management that the Agreement would be
amended to give Trans Union the right to solicit competing offers through
Part of the defense is based on a claim that the directors relied on legal January, 1981, if they would agree to remain with Trans Union. Senior
advice rendered at the September 20 meeting by James Brennan, Esquire, Management was temporarily mollified; and Van Gorkom then called a
who was present at Van Gorkom's request. Unfortunately, Brennan did not special meeting of Trans Union's Board for October 8.
appear and testify at trial even though his firm participated in the defense of
this action. [...] 135

115116117118119120 Thus, the primary purpose of the October 8 Board meeting was to amend the
Merger Agreement, in a manner agreeable to Pritzker, to permit Trans Union
-B- to conduct a "market test."[...] Van Gorkom presumably so represented the
amendments to Trans Union's Board members on October 8. In a brief
session, the directors approved Van Gorkom's oral presentation of the
121
substance of the proposed amendments, [883] the terms of which were not
reduced to writing until October 10. But rather than waiting to review the
We now examine the Board's post-September 20 conduct for the purpose of amendments, the Board again approved them sight unseen and adjourned,
determining first, whether it was informed and not grossly negligent; and giving Van Gorkom authority to execute the papers when he received them. [...]
second, if informed, whether it was sufficient to legally rectify and cure the
Board's derelictions of September 20.[...]
136137

122123124125126127128129130131
The next day, October 9, and before the Agreement was amended, Pritzker
moved swiftly to off-set the proposed market test amendment. First, Pritzker
made no reference to provisions allegedly reserving to the Board the rights to informed Trans Union that he had completed arrangements for financing its
perform a "market test" and to withdraw from the Pritzker Agreement if Trans acquisition and that the parties were thereby mutually bound to a firm
Union received a better offer before the shareholder meeting. The purchase and sale arrangement. Second, Pritzker announced the exercise of

1015
his option to purchase one million shares of Trans Union's treasury stock at 141
$38 per share 75 cents above the current market price. [...]
We conclude that the Board acted in a grossly negligent manner on October
138 8; [...]

The next day, October 10, Pritzker delivered to Trans Union the proposed
amendments to the September 20 Merger Agreement. Van Gorkom promptly
proceeded to countersign all the instruments on behalf of Trans Union
without reviewing the instruments to determine if they were consistent with
the authority previously granted him by the Board. The amending documents G.R. No. 6217 December 26, 1911
were apparently not approved by Trans Union's Board until a much later
date, December 2. The record does not affirmatively establish that Trans
CHARLES W. MEAD, plaintiff-appellant,
Union's directors ever read the October 10 amendments. [...]
vs.
E. C. McCULLOUGH, ET AL., and THE PHILIPPINE ENGINEERING AND
139 CONSTRUCTION COMPANY,defendant-appellants.

The October 10 amendments to the Merger Agreement did authorize Trans Haussermann, Cohn & Fisher and A. D. Gibbs for plaintiff.
Union to solicit competing offers, but the amendments had more far-reaching James J. Peterson and O'Brien & DeWitt for defendant McCullough.
effects. [...] Under the October 10 amendments, a better offer was no longer
sufficient to permit Trans Union's withdrawal. Trans Union was now permitted
to terminate the Pritzker Agreement and abandon the merger only if, prior to
February 10, 1981, Trans Union had either consummated a merger (or sale
of assets) with a third party or had entered into a "definitive" merger TRENT, J.:
agreement more favorable than Pritzker's and for a greater consideration
subject only to stockholder approval. Further, the "extension" of the market This action was originally brought by Charles W. Mead against Edwin C.
test period to February 10, 1981 was circumscribed by other amendments McCullough, Thomas L. Hartigan, Frank E. Green, and Frederick H. Hilbert.
which required Trans Union to file its preliminary proxy statement on the Mead has died since the commencement of the action and the case is now
Pritzker merger proposal by December 5, 1980 and use its best efforts to going forward in the name of his administrator as plaintiff.
mail the statement to its shareholders by January 5, 1981. Thus, the market
test period was effectively reduced, not extended. [...] The complaint contains three causes of action, which are substantially as
follows: The first, for salary; the second, for profits; and the third, for the
140 value of the personal effects alleged to have been left Mead and sold by the
defendants.
In our view, the record compels the conclusion that the directors' conduct on
October [884] 8 exhibited the same deficiencies as did their conduct on A joint and several judgment was rendered by default against each and all of
September 20. [...] the defendants for the sum of $3,450.61 gold. The defendant McCullough

1016
alone having made application to have this judgment set aside, the court for the purpose of discussing the condition of the company at that time and
granted this motion, vacating the judgment as to him only, the judgment as to determining what course to pursue. They did on that date enter into the
the other three defendants remaining undisturbed.1awphi1.net following contract with the defendant McCullough, to wit:1awphil.net

At the new trial, which took place some two or three years later and after the For value received, this contract and all the rights and interests of the
death of Mead, the judgment was rendered upon merits, dismissing the case Philippine Engineering and construction Company in the same are
as to the first and second causes of action and for the sum of $1,200 gold in hereby assigned to E. C. McCullough of Manila, P. I.
the plaintiff's favor on the third cause of action. From this judgment both
parties appealed and have presented separate bills of exceptions. No appeal (Sgd.) E. C. McCULLOUGH,
was taken by the defendant McCullough from the ruling of the court denying President, Philippine Engineering and
a recovery on his cross complaint. Construction Company.

On March 15, 1902, the plaintiff (Mead will be referred to as the plaintiff in (Sgd.) F. E. GREEN, Treasurer.
this opinion unless it is otherwise stated) and the defendant organized the (Sgd.) THOMAS L. HARTIGAN, Secretary.
"Philippine Engineering and Construction Company," the incorporators being
the only stockholders and also the directors of said company, with general The contract reffered to in the foregoing document was known as the
ordinary powers. Each of the stockholders paid into the company $2,000 wrecking contract with the naval authorities.
mexican currency in cash, with the exception of Mead, who turned over to
the company personal property in lieu of cash.
On the 28th of the same month, McCullough executed and signed the
following instrumental:
Shortly after the organization, the directors held a meeting and elected the
plaintiff as general manager. The plaintiff held this position with the company
For value received, and having the above assignment from my
for nine months, when he resigned to accept the position of engineer of the
associates in the Philippine Engineering and Construction Company,
Canton and Shanghai Railway Company. Under the organization the
I hereby transfer my right, title, and interest in the within contract,
company began business about April 1, 102.itc-alf
with the exception of one sixth, which I hereby retain, to R. W.
Brown, H. D. C. Jones, John T. Macleod, and T. H. Twentyman.
The contract and work undertaken by the company during the management
of Mead were the wrecking contract with the Navy Department at Cavite for
The assignees of the wrecking contract, including McCullough, formed was
the raising of the Spanish ships sunk by Admiral Dewey; the contract for the
not known as the "Manila Salvage Association." This association paid to
construction of certain warehouses for the quartermaster department; the
McCullough $15,000 Mexican Currency cash for the assignment of said
construction of a wharf at Fort McKinley for the Government; The supervision
contract. In addition to this payment, McCullough retained a one-sixth
of the construction of the Pacific Oriental Trading Company's warehouse; and
interest in the new company or association.
some other odd jobs not specifically set out in the record.

The plaintiff insists that he was received as general manager of the first
Shortly after the plaintiff left the Philippine Islands for China, the other
company a salary which was not to be less than $3,500 gold (which amount
directors, the defendants in this case, held a meeting on December 24, 1903,
he was receiving as city engineer at the time of the corporation of the

1017
company), plus 20 per cent of the net profits which might be derived from the It is admitted that the plaintiff received $1.500 gold for his services, and
business; while McCullough contends that the plaintiff was to receive only his whether he is entitled to receive an additional amount depends upon the
necessary expenses unless the company made a profit, when he could result of the second cause of action.
receive $3,500 per year and 20 per cent of the profits. The contract entered
into between the board of directors and the plaintiffs as to the latter's salary The second cause of action is more difficult to determine. On this point
was a verbal one. The plaintiff testified that this contract was unconditional counsel for the plaintiff has filed a very able and exhaustive brief, dealing
and that his salary, which was fixed at $3,500 gold, was not dependent upon principally with the facts.
the success of the company, but that his share of the profits was to
necessarily depend upon the net income. On the other hand, McCullough, It is urged that the net profits accruing to the company after the completion of
Green and Hilbert testify that the salary of the plaintiff was to be determined all the contracts (except the salvage contract) made before the plaintiff
according to whether or not the company was successful in its operations; resigned as manager and up to the time the salvage contract was transferred
that if the company made gains, he was to receive $3,5000 gold, and a to McCullough and from him to the new company, amounted to $5,628.37
percentage, but that if the company did not make any profits, he was to gold. This conclusion is reached, according to the memorandum of counsel
receive only his necessary living expenses. for the plaintiff which appears on pages 38 and 39 of the record, in the
following manner:
It is strongly urged that the plaintiff would not have accepted the
management of the company upon such conditions, as he was receiving
from the city of Manila a salary of $3,500 gold. This argument is not only
answered by the positive and direct testimony of three of the defendants, but Profits from the construction of warehouses for the $6,962.54
also by the circumstances under which this company was organized and Government
principal object, which was the raising of the Spanish ships. The plaintiff put
no money into the organization, the defendants put but little: just sufficient to
get the work of raising the wrecks under way. This venture was a risky one.
All the members of the company realized that they were undertaking a most Profits from the construction of the wall at Fort 500.00
difficult and expensive project. If they were successful, handsome profits McKinley
would be realized; while if they were unsuccessful, all the expenses for the
hiring of machinery, launches, and labor would be a total loss. The plaintiff
was in complete charge and control of this work and was to receive,
according to the great preponderance of the evidence, in case the company
Profits from the inspection of the construction of the 1,000.00
made no profits, sufficient amount to cover his expenses, which included his
P. O. T. warehouse
room, board, transportation, etc. The defendants were to furnish money out
of their own private funds to meet these expenses, as the original $8,000
Mexican currency was soon exhausted in the work thus undertaken. So the
contract entered into between the directors and the plaintiff as to the latter's
salary was a contingent one. Profits obtained from the projects (according to 1,000.00
Mead's calculations)

1018
It was this exhibit that the lower court based its conclusion when it found that
on January 25, 1903, after making the transfer of the salvage contract to
McCullough, the company was in debt $2,278.30 gold. The balance of
$10,728.44 Mexican currency deducted from the $16,439.40 Mexican
Total 9,462.54 currency (McCullough's losses in the Manila Salvage Association) leaves
$2,278.30 United States currency at the then existing rate of exchange. In
Exhibit K, McCullough charged himself with the $15,000 Mexican currency
which he received from his associates in the new company, but did not credit
In this same memorandum, the expense for the operation of the company himself with the $16,439.40 Mexican currency, losses in said company, for
during Mead's management, consisting of rents, the hire of one muchacho, the reason that on April 1, 1903, said losses had not occurred. It must be
the publication of various notices, the salary of an engineer for four months, borne in mind that Exhibit K is an abstract from a ledger.
and plaintiff's salary for nine months, amounts to $3,834.17 gold. This
amount, deducted from the sum total of profits, leaves $5,628.37 gold. The defendant McCullough, in order to show in detail his transactions with
the old company, presented Exhibits 1 and 2. These accounts read as
Counsel for the plaintiff, in order to show conclusively as they assert that the follows:
company, after paying all expenses and indebtedness, had a considerable
balance to its credit, calls attention to Exhibit K. This balance reads as Detailed account of the receipts and disbursements of E. C.
follows: McCullough and the Philippine Engineering and Construction
Company.
Abstract copy of ledger No. 3, folios 276-277. Philippine Engineering
and Construction Company. Then follow the debits ad credits. These two accounts cover the period from
March 5 1902, to June 9, 1905. According to Exhibit No. 1, the old company
Then follow the debits and credits, with a balance in favor of the company of was indebted to McCullough in the sum of $14,918.75 Mexican currency, and
$10,728.44 Mexican currency. This account purports to cover the period from according to Exhibit No. 2 he indebtedness amounted to $6,358.15 Mexican
July 1, 1902, to April 1, 1903. Ledger No. 3, above mentioned, is that the currency. The debits and credits in these two exhibits are exactly the me with
defendant McCullough and not one of the books of the company. the following exceptions; I Exhibit No. 1, McCullough credits himself with the
$10,000 Mexican currency (the amount borrowed from the bank and
deposited with the admiral as a guarantee for the faithful performance of the
salvage contract); while in Exhibit No. 2 he credits himself with this $10,000
and at he same time charges himself with this amount. In the same exhibit
(No. 2) he credits himself with $16,439.40 Mexican currency, his losses in the
new company, received from said company. Eliminating entirely from these
two exhibits the $10,000 Mexican currency, the $15,000 Mexican currency,
and the $16,39.40 Mexican currency, the balance shown in McCullough's
favor is exactly the same in both exhibits. This balance amounts to $4,918.75
Mexian currency.

1019
According to McCullough's accounts in Exhibits 1 and 2 the profits derived
from the construction of the Government warehouse amounted to $4,005.02
gold, while the plaintiff contends that these profits amounted to $6,962.54 Feb. 2, 1903 McCullough 1,300.00
gold. The plaintiff, during his management of the old company, made a
contract with the Government for the construction of these are house and
commenced work. After he resigned and left for China, McCullough took
charge of and completed the said warehouse. McCullough gives a complete, Feb. 2, 1903 Green 1,027.92
detailed statements of express for the completion of this work, showing the
dates, to whom paid, and for what purpose. He also gives the various
amounts he received from the Government with the amounts of the receipt of
the same. On the first examination, McCullough testified that the total amount Feb. 19, 1905 P. O. T. Co. note 2,236.80
received from the Government for the construction of these warehouse was
$1,123 gold. The case was suspended for the purpose of examination the
records of the Auditor and the quater master, to determine the exact amount
paid for this work. As a result of this examination, the vouchers show an May 23, 1905 Hilbert 1,856.02
additional amount of about $5,000 gold, paid in checks. These checks show
that the same were endorsed by the plaintiff and collected by him from the
Hongkong and Shanghai Banking Corporation. This money was not handled
by McCullough and as it was collected by the plaintiff, it must be presumed,
June 9, 1905 Hartigan 1,225.00
in the absence of proof, that it was disbursed by him. McCullough did not
charge himself with the $2,5000 gold, alleged to have been profits from the
construction of the wall at Fort McKinley, the inspection of the construction of
the P. O. T. warehouse, and other projects. This work was done under the
McCullough says that these amounts represents cash borrowed from the
management of the plaintiff and it is not shown that the profits from these
evidence parties to carry on the operations of the old company while it was
contracts ever reached the ands of McCullough. McCullough was not the
trying to raise the sunken vessels. There is no proof to the contrary, and
treasurer of the company at that time. The other items which the plaintiff
McCullough's testimony on this point is strongly corroborated by the fact that
insist that McCullough had no right to credit himself with are the following:
the work done by the company in attempting to raise theses vessels was it
first undertaking. The company had made no profits while tat work was going
on under the management of the plaintiff, but its expenses greatly exceeded
that of the original $8,000 Mexican currency. It was necessary to borrow
Date To whom paid. Amount (Mex. currency).
money to continue that work. These amounts, having been borrowed, were
outstanding debts when McCullough took charge for the purpose of
completing the warehouses and winding up the business of the old company.
These amounts do not represent payments or refunds of the original capital.
Jan. 30, 1903 Green $2,000.00 McCullough did not credit himself with any amount for his services for
supervising the completion of the warehouses, nor for liquidating or winding
up the company's affairs. We think that the amount of $4,918.75 Mexican

1020
currency, balance in McCullough's favor up to this point, represents a fair, corporation not a general copartnership nor a limited copartnership. (Arts.
equitable, and just settlement. 37, 38,1656 of the Civil Code; Compania Agricola de Ultimar vs. Reyes et al.,
4 Phil. Rep., 2; and Chief Justice Marshall's definition of a corporation, 17 U.
So far we have referred to the Philippine Engineering and Construction S., 518.)
Company as the "company," without any attempt to define its legal status.
The inscribing of its articles of agreement in the commercial register was not
The plaintiff and defendants organized this company with a capital stock of necessary to make it a juridicial person a corporation. Such inscription
$100,000 Mexican currency, each paying in on the organization $2,000 only operated to show that it partook of the form of a commercial corporation.
Mexican currency. The remainder, $9,000, according to the articles of (Compania Agricola de Ultimar vs. Reyes et al., supra.)
agreement, were to be offered to the public in shares of $100 Mexican
currency, each. The names of all the organizers appear in the articles of Did a majority of the stockholders, who were at the same time a majority of
agreement, which articles were duly inscribed in the commercial register. The the directors of this corporation, have the power under the law and its articles
purpose for which this organization was affected were to engage in general of agreement, to sell or transfer to one of its members the assets of said
engineering and construction work, and operating under the name of the corporation?
"Philippine Engineering and Construction Company." during its active
existence, it engaged in the business of attempting to rise the sunken In the first article of the statutes of incorporation it is stated tat by virtue of a
Spanish fleet, constructing under contract warehouses and a wharf for the public document the organizers, whose names are given in full, agreed to
United States Government, supervising the construction of a warehouse for a form a sociedad anonima. Article II provides that the organizers should be
private firm, and some assay work. It was, therefore, an industrial civil the directors an administrators until the second general meeting, and until
partnership, as distinguished from a commercial one; a civil partnership in their successors were duly elected and installed. The third provides that
the mercantile form, an anonymous partnership legally constituted in the city the sociedad should run for ninety-nine years from the date of the execution
of Manila. of its articles of agreement. Article IV sets forth the object or purpose of the
organization. Article V makes the capital $100,000 Mexican currency, divided
The articles of agreement appeared in a public document and were duly into one thousand shares at $100 Mexican currency each. Article VI provides
inscribed in the commercial register. To the extent of this inscription the that each shareholder should be considered as a coowner in the assets of
corporation partook of the form of a mercantile one and as such must e the company and entitled to participate in the profits in proportion to the
governed by articles 151 to 174 of the Code of Commerce, in so far as these amount of his stock. Article VII fixed the time of holding general meetings and
provisions are not in conflict with the Civil Code (art. 1670, Civil Code); but the manner of calling special meetings of the stockholders. Article VIII
the direct and principal law applicable is the Civil Code. Those provisions of provides that the board of directors shall be elected annually. Article IX
the Code of Commerce are applicable subsidiary. provides for the filing of vacancies in the board of directors. Article X provides
that "the board of directors shall elect the officers of the sociedad and have
This partnership or stock company (sociedad anonima) upon the execution of under is charge the administration of the said sociedad." Article XI: "In all the
the public instrument in which is articles of agreement appear, and the questions with reference to the administration of the affairs of the sociedad, it
contribution of funds and personal property, became a juridicial person an shall be necessary to secure the unanimous vote of the board of directors,
artificial being, invisible, intangible and existing only in contemplation of law and at least three of said board must be provides that all of the stock, except
with the power to hold, buy, and ell property, and to use and be sued a that which was divided among the organizers should remain in the treasury
subject to the disposition of the board of directors. Article XIII reads: "In all

1021
the meetings of the stockholders, a majority vote of the stockholders present Articles 1700 to 1708 of the Civil Code deal with the manner of dissolving a
shall be necessary to determine any question discussed." The fourteenth corporation. There is nothing in these articles which expressly or impliedly
articles authorizes the board of directors to adopt such rules and regulations prohibits the sale of corporate property to one of its members, nor a
for the government of the sociedad as it should deem proper, which were not dissolution of a corporation in this manner. Neither is there anything in
in conflict with its statutes. articles 151 to 174 of the Code of Commerce which prohibits the dissolution
of a corporation by such sale or transfer.
When the sale or transfer heretofore mentioned took place, there were
present four directors, all of whom gave their consent to that sale or transfer. The articles of incorporation must include:
The plaintiff was then about and his express consent to make this transfer or
sale was not obtained. He was, before leaving, one of the directors in this xxx xxx xxx
corporation, and although he had resigned as manager, he had not resigned
as a director. He accepted the position of engineer of the Canton and The submission to the vote of the majority of the meeting of
Shanghai Railway Company, knowing that his duties as such engineer would members, duly called and held, of such matters as may properly be
require his whole time and attention and prevent his returning to the brought before the same. (No. 10, art. 151, Code of Commerce.)
Philippine Islands for at least a year or more. The new position which he
accepted in China was incompatible with his position as director in the
Article XIII of the corporation's statutes expressly provides that "in all the
Philippine Engineering and Construction Company, a corporation whose
meetings of the stockholders, a majority vote of the stockholders present
sphere of operations was limited to the Philippine Islands. These facts are
shall be necessary to determine any question discussed."
sufficient to constitute an abandoning or vacating of hid position as director in
said corporation. (10 Cyc., 741.) Consequently, the transfer or sale of the
corporation's assets to one of its members was made by the unanimous The sale or transfer to one of its members was a matter which a majority of
consent of all the directors in the corporation at that time. the stockholders could very properly consider. But it i said that if the acts and
resolutions of a majority of the stockholders in a corporation are binding in
every case upon the minority, the minority would be completely wiped out
There were only five stockholders in this corporation at any time, four of
and their rights would be wholly at the mercy of the abuses of the majority.
whom were the directors who made the sale, and the other the plaintiff, who
was absent in China when the said sale took place. The sale was, therefore,
made by the unanimous consent of four-fifths of all the stockholders. Under Generally speaking, the voice of a majority of the stockholders is the law of
the articles of incorporation, the stockholders and directors had general the corporation, but there are exceptions to this rule. There must necessarily
ordinary powers. There is nothing in said articles which expressly prohibits be a limit upon the power of the majority. Without such a limit the will of the
the sale or transfer of the corporate property to one of the stockholders of majority would be absolute and irresistible and might easily degenerate into
said corporation. an arbitrary tyranny. The reason for these limitations is that in every contract
of partnership (and a corporation can be something fundamental and
unalterable which is beyond the power of the majority of the stockholders,
Is there anything in the law which prohibits such a sale or transfer? To
and which constitutes the rule controlling their actions. this rule which must
determine this question, it is necessary to examine, first, the provisions of the
be observed is to be found in the essential compacts of such partnership,
Civil Code, and second, those provisions (art. 151 to 174) of the Code o ]
which gave served as a basis upon which the members have united, and
Commerce.
without which it is not probable that they would have entered not the

1022
corporation. Notwithstanding these limitations upon the power of the majority Power of private corporation to alienate property. This power of
of the stockholders, their (the majority's) resolutions, when passed in good absolute alienability of corporate property applies especially to
faith and for a just cause, deserve careful consideration and are generally private corporations that are established solely for the purpose of
binding upon the minority. trade or manufacturing and in which he public has no direct interest.
While this power is spoken of as belonging to the corporation it must
Eixala, in his work entitled "Instituciones del Derecho Mercantil de Espaa," be observed that the authorities point out that the trustees or
speaking of sociedades anonimas, says: directors of a corporation do not possess the power to dispose of the
corporate property so as to virtually end the existence of the
The resolutions of the boards passed by a majority vote are valid . . . corporation and prevent it from carrying on the business for which it
and authority for passing such resolutions is unlimited, provided that was incorporated. (Thompson on Corporation, second edition, sec.
the original contract is not broken by them, the partnership funds not 2416, and cases cited thereunder.)
devoted to foreign purposes, or the partnerships transformed, or
changes made which are against public policy or which infringe upon Power to dispose of all property. Where there are no creditors,
the rights of third persons. and no stockholder objects, a corporation, as against all other
persons but the state, may sell and dispose of all its property. The
The supreme court of Spain, in its decision dated June 30, 1888, said: state in its sovereign capacity may question the power of the
corporation to do so, but with these exceptions such as a sale is
void. A rule of general application is that a corporation of a purely
In order to be valid and binding upon dissenting members, it s an
private business character, one which owes no special duty to the
indispensable requisite that resolutions passed by a general meeting
public, and is not given the right of eminent domain, where
of stockholders conform absolutely to the contracts and conditions of
exigencies of its business require it or when the circumstances are
the articles of the association, which are to be strictly construed.
such that it can no longer continue the business with profit, may sell
and dispose of all its property, pay its debts, divide the remaining
That resolutions passed within certain limitations by a majority of the assets and wind up the affairs of the corporation. (Id., sec. 2417.)
stockholders of a corporation are binding upon the minority, is therefore
recognized by the Spanish authorities.
When directors or officers may dispose of all the property. It is
within the dominion of the managing officers and agents of the
corporation to dispose of all the corporate property under certain
circumstances; and this may be done without reference to the assent
or authority of the stockholders. This disposition of the property may
be temporarily by lease, or permanently by absolute conveyance.
But it can only be done in the course of the corporate business and
for the furtherance of the purposes of the incorporation. The board of
directors possess this power when the corporation becomes involved
and by reason of its embarrassed or insolvent condition is unable
either to pay its debts or to secure capital and funds for the further
prosecution of its enterprise, and especially where creditors are

1023
pressing their claims and demands and are threatening to or have A purely private business corporation, like a manufacturing or trading
instituted actions to enforce their claims. This power of the directors company, which is not given the right of eminent domain, and which
to alienate the property is conceded where it is regarded as of owes no special duties to the public, may certainly sell and convey
imperative necessity. (If., sec. 2418, and case cited.) absolutely the whole of its property, when the exigencies of its
business require it to do so, or when the circumstances are such that
When majority stockholder may dispose of all corporate property. it can no longer profitably continue its business, provided the
Another rule that permits a majority of the stockholders to dispose of transaction is not in fraud of the rights of creditors, or in violation of
all the corporate property and wind up the business, is where the charter or statutory restrictions. And, by the weight of authority, this
corporation has became insolvent, and the disposition of the property may be done a majority of the stockholders against the dissent of the
is necessary to pay the debt; or where from any cause the business minority. (Id., sec. 160, and cases cited.)
is a failure, and the best interest of the corporation and all the
stockholders require it, then the majority have clearly the power to The above citations are taken from the works of the most eminent writers on
dispose of all the property even as against the protests of a minority. corporation law. The citation of cases in support of the rules herein
It would be a harsh rule that could permit one stockholder, or any announced are too numerous to insert.
minority of the stockholders, to hold the majority to their investment
where the continuation of the business would be at a loss and where From these authorities it appears to be well settled, first, that a private
there was no prospect or hope that the enterprise could be made corporation, which owes no special duty to the public and which has not been
profitable. The rule as stated by some courts is that the majority given the right of eminent domain, has the absolute right and power as
stockholders may dispose of the property when just cause exists; against the whole world except the state, to sell and dispose of all of its
and this just cause is usually defined to be the unprofitableness of property; second, that the board of directors, has the power, without
the business and where its continuation would be ruinous to the referrence to the assent or authority of the stockholders, when the
corporation and against the interest of stockholders. (Id., sec. 2424, corporation is in failing circumstances or insolvent or when it can no longer
and cases cited.) continue the business with profit, and when it is regarded as an imperative
necessity; third, that a majority of the stockholders or directors, even against
Nothing is better settled in the law of corporations than the doctrine the protest of the minority, have this power where, from any cause, the
that a corporation has the same capacity and power as a natural business is a failure and the best interest of the corporation and all the
person to dispose of the convey its property, real or personal, stockholders require it.
provided it does not do so for a purpose which is foreign to the
objects for which it was created, and provided, further, it violates no May officer or directors of the corporation purchase the corporate property?
charter or statutory restriction, on rule of law based upon public The authorities are not uniform on this question, but on the general
policy. . . .This power need not be expressly conferred upon a proposition whether a director or an officer may deal with the corporation, we
corporation by its charter. It is implied as an incident to its ownership think the weight of authority is that he may. (Merrick vs. Peru Coal Co., 61 Ill.,
of property, unless there is some clear restriction in this charter or in 472; Harts et al. vs. Brown et al., 77 Ill., 226; Twin-Lick Oil Company vs.
some statute. (Clark and Marshall's Private Corporations, sec. 152, Marbury, 91 U.S., 587; Whitwell vs, Warner, 20 Vt., 425; Smith vs. Lansing,
and cases cited.) 22 N.Y., 520; City of St. Loius vs. Alexander, 23 Mo., 483; Beach et al vs.
Miller, 130 Ill., 162.)

1024
While a corporation remains solvent, we can see no reason why a director or In the case of the Twin-Lick Oil Company vs. Marbury, supra, the complaint
officer, by the authority of a majority of the stockholders or board of was a corporation organized under the laws of West Virginia, engaged in the
managers, may not deal with the corporation, loan it money or buy property business of raising and selling petroleum. It became very much embarrased
from it, in like manner as a stranger. So long as a purely private corporation and a note was given secured by a deed of trust, conveying all the property
remains solvent, its directors are agents or trustees for the stockholders. rights, and franchise of the corporation to William Thomas to secure the
They owe no duties or obligations to others. But the moment such a payment of said note, with the usual power of sale in default of payment. The
corporation becomes insolvent, its directors are trustees of all the creditors, property was sold under the deed of trust; was bought in by defendant's
whether they are members of the corporation or not, and must manage its agent for his benefit, and conveyed to him the same year. The defendant was
property and assets with strict regard to their interest; and if they are at the time of these transactions a stockholder and director in the company.
themselves creditors while the insolvent corporation is under their At the time the defendant's money became due there was no apparent
management, they will not be permitted to secure to themselves by possibility of the corporation's paying it at any time. The corporation was then
purchasing the corporate property or otherwise any personal advantage over insolvent. The property was sold by the trustee and bough in by the
the other creditors. Nevertheless, a director or officer may in good faith and defendant at a fair and open sale and at a reasonable price. The sale and
for an adequate consideration purchase from a majority of the directors or purchase was the only mode left to the defendant to make his money. The
stockholders the property even of an insolvent corporation, and a sale thus court said:
made to him is valid and binding upon the minority. (Beach et al. vs.
Miller, supra; Twin-Lick Oil Company vs. Marbury, supra; Drury vs. Cross, 7 That a director of a joint-stock corporation occupies one of those
Wall., 299; Curran vs. State of Arkansas, 15 How., 304; Richards vs. New fiduciary relations where his dealings with the subject-matter of his
Hamphshire Insurance Company, 43 N. H., 263; Morawetz on Corporations trust or agency, and with the beneficiary or party whose interest is
(first edition), sec. 579; Haywood vs. Lincoln Lumber Company et al., 64 confided to his care, is viewed with jealousy by the courts, and may
Wis., 639; Port vs. Russels, 36 Ind., 60; Lippincott vs. Shaw Carriage be set aside on slight grounds, is a doctrine founded on the soundest
Company, 21 Fed. Rep., 577.) morality, and which has received the clearest recognition in this court
and others. (Koehler vs. Iron., 2 Black, 715; Drury vs. Cross, 7 Wall.,
299; R.R. Co. vs. Magnay, 25 Beav., 586; Cumberland Co vs.
Sherman, 30 Barb., 553; Hoffman S. Coal Co. vs. Cumberland Co.,
16 Md., 456.) The general doctrine, however, in regard to contracts
of this class, is, not that they are absolutely void, but that they are
voidable at the election of the party whose interest has been so
represented by the party claiming under it. We say, this is the general
rule; for there may be cases where such contracts would be void ab
initio; as when an agent to sell buys of himself, and by his power of
attorney conveys to himself that which he was authorized to sell. but
even here, acts which amount t a ratification by the principal may
validate the sale.

The present case is not one of that class. While it is true that the
defendant, a s a director of the corporation, was bound by all those

1025
rules of conscientious fairness which courts of equity have imposed stockholders who appointed him their agent. If he should be a sole
as the guides for dealing in such cases, it can not be maintained that director, or one of a smaller number vested with certain powers, this
any rule forbids one director among several from loaning money to obligation would be still stronger, and his acts subject to more severe
the corporation when the money is needed, and the transaction is scrutiny, and their validity determined by more rigid principles of
open, and otherwise free from blame. No adjudged case has gone morality, and freedom from motives of selfishness. All this falls far
so far as this. Such a doctrine, while it would afford little protection to short, however, of holding that no such contract can be made which
the corporation against actual fraud or oppression, would deprive it will be valid; . . . .
of the air of those most interested in giving aid judiciously, and best
qualified to judge of the necessity of that aid, and of the extent to In the case of Hancock vs. Holbrook et al. (40 La. Ann., 53), the court said:
which it may safely be given.
As a strictly legal question, the right of a board of directors of a
There are in such a transaction three distinct parties whose interest corporation to apply it property to the payment of its debts, and the
is affected by it; namely, the lender, the corporation, and the right of the majority of stockholders present at a meeting called for
stockholders of the corporation. the purpose to ratify such action and to dissolve the corporation, can
not be questioned.
The directors are the officers or agents of the corporation, and
represent the interests of the abstract legal entity, and of those who But were such action is taken at the instance, and through the
own the shares of its stock. One of the objects of creating a influence of the president of the corporation, and were the debt to
corporation by law is to enable it to make contracts; and these which the property is applied is one for which he is himself primarily
contracts may be made with its stockholders as well as with others. liable, and specially where he subsequently acquires, in his personal
In some classes of corporations, as in mutual insurance companies, right, the proerty thus disposed of, such circumstances undoubtedly
the main object of the act of the incorporation is to enable the subject his acts to severe scrutiny, and oblige him to establish that he
company to make contracts which its stockholders, or with persons acted with the utmost candor and fair-dealing for the interest of the
who become stockholders by the very act of making the contract of corporation, and without taint of selfish motive.
insurance. It is very true, that as a stockholder, in making a contract
of any kind with the corporation of which he is a member, is in some The sale or transfer of the corporate property in the case at bar was made by
sense dealing with a creature of which he is a part, and holds a three directors who were at the same time a majority of stockholders. If a
common interest with the other stockholders, who, with him, majority of the stockholders have a clear and a better right to sell the
constitute the whole of that artificial entity, he is properly held to a corporate property than a majority of the directors, then it can be said that a
larger measure of candor and good faith than if he were not a majority of the stockholders made this sale or transfer to the defendant
stockholder. So, when the lender is a director, charged, with others, McCullough.
with the control and management of the affairs of the corporation,
representing in this regard the aggregated interest of all the
What were the circumstances under which said sale was made? The
stockholders, his obligation, if he becomes a party to a contract with
corporation had been going from bad to worse. The work of trying to raise the
the company, to candor and fair dealing, is increased in the precise
sunken Spanish fleet had been for several months abandoned. The
degree that his representative character has given him power and
corporation under the management of the plaintiff had entirely failed in this
control derived from the confidence reposed in him by the

1026
undertaking. It had broken its contract with the naval authorities and the company for $15,000 Mexican currency and retained one-sixth as his share
$10,000 Mexican currency deposited had been confiscated. It had no money. of the stock in the new concern. The Manila Salvage company paid to the
It was considerably in debt. It was a losing concern and a financial failure. To bank the $10,000 Mexican currency which had been borrowed to deposit
continue its operation meant more losses. Success was impossible. The with the naval authorities, and began operations. All of the $10,000 Mexican
corporation was civilly dead and had passed into the limbo of utter currency so deposited was refund to the new company except P2,000. The
insolvency. The majority of the stockholders or directors sold the assets of new association failed and McCullough, by reason of this failure, lost over
this corporation, thereby relieving themselves and the plaintiff of all $16,000 Mexican currency. These facts show that McCullough acted in good
responsibility. This was only the wise and sensible thing for them to do. They faith in purchasing the old corporation's assets, and that he certainly paid for
acted in perfectly good faith and for the best interests of all the stockholders. the same a valuable consideration.
"It would be a harsh rule that would permit one stockholder, or any minority of
stockholders to hold a majority to their investment where a continuation of But cancel for the plaintiff say: "The board of directors possessed only
the business would be at a loss and where there was no prospect or hope ordinary powers of administration (Article X of the Articles of incorporation),
that the enterprise would be profitable." which in no manner empowered it either to transfer or to authorize the
transfer of the assets of the company to McCullough (art. 1773, Civil Code;
The above sets forth the condition of this insolvent corporation when the decisions of the supreme court of Spain of April 2, 1862, and July 8, 1903)."
defendant McCullough proposed to the majority of stockholders to take over
the assets and assume all responsibility for the payment of the debts and the Article X of the articles of incorporation above referred to provides that the
completion of the warehouses which had been undertaken. The assets board of directors shall elect the officers of the corporation and "have under
consisted of office furniture of a value of less than P400, the uncompleted its charge the administration of the said corporation." Articles XI reads: "In all
contract for the construction of the Government warehouses, and the the questions with reference to the administration of the affairs of the
wrecking contract. The liabilities amounted to at least $19,645.74 Mexican corporation, it shall be necessary to secure the unanimous vote of the board
currency. $9,645.74 Mexican currency of this amount represented borrowed of directors, and at least three of said board must be present in order to
money, and $10,000 Mexican currency was the deposit with the naval constitute a legal meeting." It will be noted that article X statute a legal
authorities which had been confiscated and which was due the bank. meeting." It will be noted that Article X placed the administration of the affairs
McCullough's profits on the warehouse contract amounted to almost enough of the corporation in the hands of the board of directors. If Article XI had been
to the pay the amounts which the corporation had borrowed from its omitted, it is clear that under the rules which govern business of that
members. The wrecking contract which had been broken was of no value to character, and in view of the fact that before the plaintiff left this country and
the corporation for the reason that the naval authorities absolutely refused to abandoned his office as director, there were only five directors in the
have anything further to do with the Philippine Engineering and Construction corporation, then three would have been sufficient to constitute a quorum
Company. They the naval authorities) had declined to consider the petition of and could perform all the duties and exercise all the powers conferred upon
the corporation for an extension in which to raise the Spanish fleet, and had the board under this article. It would not have been necessary to obtain the
also refused to reconsider their action in confiscating the deposit. They did consent of all three of such members which constituted the quorum in order
agree, however, that if the defendant McCullough would organize a new that a solution affecting the administration of the corporation should be
association, that they would give the new concern an extension of time and binding, as two votes a majority of the quorum would have been
would reconsider the question of forfeiture of the amount deposited. Under sufficient for this purpose. (Buell vs. Buckingham & Co., 16 Iowa, 284; 2
these circumstances and conditions, McCullough organized the Manila Kent. Com., 293; Cahill vs. Kalamazoo Mutual Insurance Company, 2 Doug.
Salvage Company, sold five-sixth of this wrecking contract to the new

1027
(Mich.), 124; Sargent vs. Webster, 13 Met., 497; In re Insurance Company, provision of law is based upon the broad principles of sound reason and
22 Wend., 591; Ex parte Wilcox, 7 Cow., 402; id., 527, note a.) public policy. There is a manifest impropriety in allowing the same person to
act as the agent of the seller and to become himself the buyer. In such
It might appear on first examination that the organizers of this corporation cases, there arises so often a conflict between duty and interest. "The wise
when they asserted the first part of Article XI intended that no resolution policy of the law put the sting of a disability into the temptation, as a
affecting the administration of the affairs should be binding upon the defensive weapon against the strength of the danger which lies in the
corporation unless the unanimous consent of the entire board was first situation."
obtained; but the reading of the last part of this same article shows clearly
that the said organizers had no such intention, for they said: "At least three of Hilbert, Green, and Hartigan were not only all creditors at the time the sale or
said board must be present in order to constitute a legal meeting." Now, if transfer of the assets of the insolvent corporation was made, but they were
three constitute a legal meeting, three were sufficient to transact business, also directors and stockholders. In addition to being a creditor, McCullough
three constituted the quorum, and, under the above-cited authorities, two of sustained the corporation the double relation of a stockholder and president.
the three would be sufficient to pass binding resolutions relating to the The plaintiff was only a stockholder. He would have been a creditor to the
administration of the corporation. extent of his unpaid salary if the corporation had been a profitable instead of
a losing concern.
If the clause "have under in charge and administer the affairs of the
corporation" refers to the ordinary business transactions of the corporation But as we have said when the sale or transfer under consideration took
and does not include the power to sell the corporate property and to dissolve place, there were three directors present, and all voted in favor of making this
the corporation when it becomes insolvent a change we admit organic and sale. It was not necessary for the president, McCullough, to vote. There was
fundamental then the majority of the stockholders in whom the ultimate a quorum without him: a quorum of the directors, and at the same time a
and controlling power lies must surely have the power to do so. majority of the stockholders.

Article 1713 of the Civil Code reads: A corporation is essential a partnership, except in form. "The directors are the
trustees or managing partners, and the stockholders are the cestui que
An agency stated in general terms only includes acts of trust and have a joint interest in all the property and effects of the
administration. corporation." (Per Walworth, Ch., in Robinson vs. Smith, 3 Paige, 222, 232;
5 idem, 607; Slee vs. Bloom, 19 Johns., 479; Hoyt vs. Thompson, 1 Seld.,
In order to compromise, alienate, mortgage, or execute any other act 320.)
of strict ownership an express commission is required.
The Philippine Engineering and Construction Company was an artificial
This article appears in title 9, chapter 1 of the Civil Code, which deals with person, owning its property and necessarily acting by its agents; and these
the character, form, and kind of agency. Now, were the positions of Hilbert, agents were the directors. McCullough was then an agent or a trustee, and
Green, Hartigan, and McCullough that the agents within the meaning of the the stockholders the principal. Or say (as corporation was insolvent) that he
article above quoted when the assets of the corporation were transferred or was an agent or trustee and the creditors were the beneficiaries. This being
sold to McCullough? If so, it would appear from said article that in order to the true relation, then the rules of the law (art. 1713 of the Civil Code)
make the sale valid, an express commission would be required. This applicable to sales and purchases by agents and trustees would not apply to

1028
the purchase in question for the reason that there was a quorum without As to the third cause of action, it is insisted: First, that the court erred in
McCullough, and for the further reason that an officer or director of a holding the defendant McCullough responsible for the personal effects of the
corporation, being an agent of an artificial person and having a joint interest plaintiff; and second, that the court erred in finding that the effects left by the
in the corporate property, is not such an agent as that treated of in article plaintiff were worth P2,400.
1713 of the Civil Code.
As we have said, the plaintiff was the manager of the Philippine Engineering
Again, McCullough did not represent the corporation in this transaction. It Company from April 1, 1902, up to January 1, 1903. Sometimes during the
was represented by a quorum of the board of directors, who were at the previous month of December he resigned to accept a position in China, but
same time a majority of the stockholders. Ordinarily, McCullough's duties as did not leave Manila until about January 20. He remained in Manila about
president were to preside at the meetings, rule on questions of order, vote in twenty days after he severed his connection with the company. He lived in
case of a tie, etc. He could not have voted in this transaction because there rooms in the same building which was rented by the company and were the
was no tie. company had its offices. When he started for China he left his personal
effects in those rooms, having turned the same over to one Paulsen.
The acts of Hilbert, Green, Hartigan, and McCullough in this transaction, in Testifying on this point the plaintiff said:
view of the relations which they bore to the corporation, are subject to the
most severe scrutiny. They are obliged to establish that they acted with the Q. To whom did you turn over these personal effects on leaving
utmost candor and fair dealing for the interest of the corporation, and without here? A. To Mr. Paulsen.
taint motives. We have subjected their conduct to this test, and, under the
evidence, we believe it has safely emerged from the ordeal. Q. Have you demanded payment of this sum [referring to the value
of his personal effects]? A. On leaving for China I gave Mr.
Transaction which only accomplish justice, which are done in good Haussermann power of attorney to represent me in this case and
faith and operate legal injury to no one, lack the characteristics of demand payment.
fraud and are not to be upset because the relations of the parties
give rise to suspicions which are fully cleared away. (Hancock vs. Q. Please state whether or not you have an inventory of these
Holbrook, supra.) effects. A. I had an inventory which was in my possession but it
was lost when the company took all of the books and carried them
We therefore conclude that the sale or transfer made by the quorum of the away from the office.
board of directors a majority of the stockholders is valid and binding
upon the majority-the plaintiff. This conclusion is not in violation of the articles Q. Can you give a list or a partial list of your effect? A. I remember
of incorporation of the Philippine Engineering and Construction Company. some of the items. There was a complete bedroom set, two marble
Nor do we here announce a doctrine contrary to that announced by the tables, one glass bookcase, chairs, all of the household effects I
supreme court of Spain in its decisions dated April 2, 1862, and July 8, 1903. used when I was living in the Botanical Garden as city engineer, one
theodolite, which I bought after commencing work with the company.

Q. How much do you estimate to be the total reasonable value of


these effects? A. The total would not be less than $1,200 gold.

1029
Counsel for the plaintiff, on page 56 of their brief, say: Q. What did you do with this money? A. I took it and considered it
part of the assets of the company. All of the other effects of the office
Mr. McCullough, in his testimony (pp. 39 and 40) admits full were sold at the same time and brought P347.16.
knowledge of and participation in the removal and sale of the effects
and states that he took the proceeds and considered them part of the Q. Did Mr. Mead leave anyone in charge of his effects when he left
assets of the company. He further admits that Mr. Haussermann Manila? A. I think he left Paulsen in charge, but Paulsen did not
made a demand for the proceeds of Mr. Mead's personal effects (p. take these effects, so when we vacated the office we had to move
44). them.

McCullough's testimony, referred by the counsel, is as follows: Q. Did Paulsen continue occupying the living room where these
effects were and did he use these effects? A. I do not know
Q. At the time Mr. Mead left for China, in the building where the office because I was in the office for three months before we vacated.
was and in the office, there were left some of the personal effects of
Mr. Mead. What do you know about these effects, a list of which is Q. Don't you know that it is a fact that Mr. Haussermann, as
Exhibit B? A. Nothing appearing in this Exhibit B was never representative of Mr. Mead, demanded of you and the company the
delivered to the Philippine Engineering and Construction Company, payment of the salary which was due Mr. Mead and the value of his
according to my list. personal effects? A. Yes, sir.

Q. Do you know what became of these effects? A. No, sir. I have As to the value of these personal effects, Hartigan, testifying as witness for
no idea. I never saw them. I never heard these effects talked about. I the defendant, said:
only heard something said about certain effects which Mr. Mead had
in his living room. I think the personal effects were sold for P50. His personal effects
consisted of ordinary articles, such as a person would use who had
Q. Do you know what became of the bed of Mr. Mead? A. I know to be going from one place to another all the time, as Mr. Mead. I
there were effects, such as a bed, washstand, chairs, table, and know that all those effects were sold for less than P100, if I am not
other things, which are used in a living room, and that they were in mistaken.
Mr. Mead's room. These effects were sent to the warehouse of the
Pacific Oriental Trading Company, together with the office furniture. The foregoing is the material testimony with reference to the defendant
We had to vacate the building where the offices were and we had to McCullough's responsibility and the value of the personal effects of the
take out everything therein. These things were deposited in the plaintiff.
warehouse of the Pacific Oriental Trading Company and were finally
sold by that company and the money turned over to me. McCullough was a member of the company and was responsible as such for
the rents where the offices were located. The company had no further use for
Q. How much? A. P49.97. the building after the plaintiff resigned. The vacating of the building was the
proper thing to do. The office furniture was removed and stored in a place
where it cost nothing for rents. When Hilbert, member of the company, went

1030
to the office to remove the company's office furniture, he found no one in The plaintiff gave an estimate of the value of the effects which he left in his
charge of the plaintiff's personal effects. He took them and stored them in the rooms and placed this value at P2,400. He did not give a complete list of the
same place and later sold them, together with the office furniture, and turned effects so left, neither did he give the value of a single item separately. The
the entire amount over to defendant McCullough. plaintiff's testimony is so indefinite and uncertain that i t is impossible to
determine with any degree of certainty just what these personal effects
Paulsen, in whose charge Mead left his effects, apparently took no interest in consisted of and their values, especially when we take into consideration the
caring for them. Was the company to leave Mead's personal effects in that significant fact that these effects were abondoned by Paulsen. On the other
building and take the chances of having to continue to pay rents, solely on hand, w have before us the positive testimony of Hilbert as to the amount
account of the plaintiff's property remaining there? The company had reason received for the plaintiff's personal effects, the testimony of Hartigan that the
to believe that it would have to continue paying these rents, as they had same were sold for less than P100, and the testimony of McCullough as to
rented the building and authorized the plaintiff to occupy rooms therein. the amount turned over to him by Herbert.

The plaintiff knew when he left for China that he would be away a long time. So we conclude that the great preponderance of evidence as to the value of
He had accepted a position of importance, and which he knew would require these effects is in the favor of the contention of the defendant. Their value
his personal attention. He did not gather up his personal effects, but left them therefore be fixed at P49.97.
in the room in charge of Paulsen. Paulsen took no interest in caring for them,
but apparently left these effects to take care of them selves. The plaintiff did For these reasons the judgment appealed from as to the first and second
not even carry with him an inventory of these effects, but attempted on the causes of action is hereby affirmed. Judgment appealed from as to the third
trial to give a list of them and did give a partial list of the things he left in his cause of action is reduced to P49.97, without costs.
room; but it is not shown that all this things were there when Herbert
removed the office furniture and some of the plaintiff's effects. The fact that Arellano, C.J., Torres, Mapa, Carson and Moreland, JJ., concur.
the plaintiff remained in Manila some twenty days after resigning and never
cared for his own effects but left them in the possession of an irresponsible
person, shows extreme negligence on his part. He exhibited a reckless
indifference to the consequences of leaving his effects in the lease premises.
The law imposes on every person the duty of using ordinary care against
injury or damages. What constitutes ordinary care depends upon the
circumstances of each particular case and the danger reasonably to be
apprehended.

McCullough did not have anything personally to do with these effects at any
time. He only accepted the money which Herbert turned over to him. He,
personally, did not contribute in any way whatsoever to the loss of the
property, neither did he as a member of the corporation do so.

1031
HONORABLE INTERMEDIATE APPELLATE COURT and ALEJANDRO
TE, respondents.

De Jesus & Associates for petitioner.

Padlan, Sutton, Mendoza & Associates for private respondent.

CAMPOS, JR., J.:

Before Us is a Petition for Review on Certiorari filed by petitioner Prime


White Cement Corporation seeking the reversal of the decision * of the then
Intermediate Appellate Court, the dispositive portion of which reads as
follows:

WHEREFORE, in view of the foregoing, the judgment


appealed from is hereby affirmed in toto. 1

The facts, as found by the trial court and as adopted by the respondent Court
are hereby quoted, to wit:

On or about the 16th day of July, 1969, plaintiff and


defendant corporation thru its President, Mr. Zosimo Falcon
and Justo C. Trazo, as Chairman of the Board, entered into
a dealership agreement (Exhibit A) whereby said plaintiff was
obligated to act as the exclusive dealer and/or distributor of
the said defendant corporation of its cement products in the
entire Mindanao area for a term of five (5) years and proving
(sic) among others that:

G.R. No. L-68555 March 19, 1993 a. The corporation shall, commencing
September, 1970, sell to and supply the
PRIME WHITE CEMENT CORPORATION, petitioner, plaintiff, as dealer with 20,000 bags (94
vs. lbs/bag) of white cement per month;

1032
b. The plaintiff shall pay the defendant plaintiff was assured by his supposed buyer that his
corporation P9.70, Philippine Currency, per allocation of 20,000 bags of white cement can be disposed
bag of white cement, FOB Davao and of, he informed the defendant corporation in his letter dated
Cagayan de Oro ports; August 18, 1970 that he is making the necessary preparation
for the opening of the requisite letter of credit to cover the
c. The plaintiff shall, every time the price of the due initial delivery for the month of September,
defendant corporation is ready to deliver the 1970 (Exhibit B), looking forward to the defendant
good, open with any bank or banking corporation's duty to comply with the dealership agreement.
institution a confirmed, unconditional, and In reply to the aforesaid letter of the plaintiff, the defendant
irrevocable letter of credit in favor of the corporation thru its corporate secretary, replied that the
corporation and that upon certification by the board of directors of the said defendant decided to impose
boat captain on the bill of lading that the the following conditions:
goods have been loaded on board the
vessel bound for Davao the said bank or a. Delivery of white cement shall commence
banking institution shall release the at the end of November, 1970;
corresponding amount as payment of the
goods so shipped. b. Only 8,000 bags of white cement per
month for only a period of three (3) months
Right after the plaintiff entered into the aforesaid dealership will be delivered;
agreement, he placed an advertisement in a national,
circulating newspaper the fact of his being the exclusive c. The price of white cement was priced at
dealer of the defendant corporation's white cement products P13.30 per bag;
in Mindanao area, more particularly, in the Manila Chronicle
dated August 16, 1969 (Exhibits R and R-1) and was even d. The price of white cement is subject to
congratulated by his business associates, so much so, he readjustment unilaterally on the part of the
was asked by some of his businessmen friends and close defendant;
associates if they can be his
sub-dealer in the Mindanao area.
e. The place of delivery of white cement
shall be Austurias (sic);
Relying heavily on the dealership agreement, plaintiff
sometime in the months of September, October, and
f. The letter of credit may be opened only
December, 1969, entered into a written agreement with
with the Prudential Bank, Makati Branch;
several hardware stores dealing in buying and selling white
cement in the Cities of Davao and Cagayan de Oro which
would thus enable him to sell his allocation of 20,000 bags g. Payment of white cement shall be made
regular supply of the said commodity, by September, 1970 in advance and which payment shall be
(Exhibits O, O-1, O-2, P, P-1, P-2, Q, Q-1 and Q-2). After the used by the defendant as guaranty in the

1033
opening of a foreign letter of credit to cover fact, it appears on the face of the contract itself that both
costs and expenses in the procurement of officers were duly authorized to enter into the said
materials in the manufacture of white agreement and signed the same for and in behalf of the
cement. (Exhibit C). corporation. When they, therefore, entered into the said
transaction they created the impression that they were duly
xxx xxx xxx clothed with the authority to do so. It cannot now be said that
the disputed agreement which possesses all the essential
Several demands to comply with the dealership agreement requisites of a valid contract was never intended to bind the
(Exhibits D, E, G, I, R, L, and N) were made by the plaintiff to corporation as this avoidance is barred by the principle of
the defendant, however, defendant refused to comply with estoppel. 3
the same, and plaintiff by force of circumstances was
constrained to cancel his agreement for the supply of white In this petition for review, petitioner Prime White Cement Corporation made
cement with third parties, which were concluded in the following assignment of errors. 4
anticipation of, and pursuant to the said dealership
agreement. I

Notwithstanding that the dealership agreement between the THE DECISION AND RESOLUTION OF THE
plaintiff and defendant was in force and subsisting, the INTERMEDIATE APPELLATE COURT ARE
defendant corporation, in violation of, and with evident UNPRECEDENTED DEPARTURES FROM THE CODIFIED
intention not to be bound by the terms and conditions PRINCIPLE THAT CORPORATE OFFICERS COULD
thereof, entered into an exclusive dealership agreement with ENTER INTO CONTRACTS IN BEHALF OF THE
a certain Napoleon Co for the marketing of white cement in CORPORATION ONLY WITH PRIOR APPROVAL OF THE
Mindanao (Exhibit T) hence, this suit. (Plaintiff's Record on BOARD OF DIRECTORS.
Appeal, pp. 86-90). 2
II
After trial, the trial court adjudged the corporation liable to Alejandro Te in the
amount of P3,302,400.00 as actual damages, P100,000.00 as moral THE DECISION AND RESOLUTION OF THE
damages, and P10,000.00 as and for attorney's fees and costs. The INTERMEDIATE APPELLATE COURT ARE CONTRARY TO
appellate court affirmed the said decision mainly on the following basis, and THE ESTABLISHED JURISPRUDENCE, PRINCIPLE AND
We quote: RULE ON FIDUCIARY DUTY OF DIRECTORS AND
OFFICERS OF THE CORPORATION.
There is no dispute that when Zosimo R. Falcon and Justo
B. Trazo signed the dealership agreement Exhibit "A", they III
were the President and Chairman of the Board, respectively,
of defendant-appellant corporation. Neither is the THE DECISION AND RESOLUTION OF THE
genuineness of the said agreement contested. As a matter of INTERMEDIATE APPELLATE COURT DISREGARDED THE

1034
PRINCIPLE AND JURISPRUDENCE, PRINCIPLE AND therefrom. 7 Furthermore, even in the absence of express or implied authority
RULE ON UNENFORCEABLE CONTRACTS AS by ratification, the President as such may, as a general rule, bind the
PROVIDED IN ARTICLE 1317 OF THE NEW CIVIL CODE. corporation by a contract in the ordinary course of business, provided the
same is reasonable under the circumstances. 8 These rules are basic, but are
IV all general and thus quite flexible. They apply where the President or other
officer, purportedly acting for the corporation, is dealing with a third person, i.
THE DECISION AND RESOLUTION OF THE e., a person outside the corporation.
INTERMEDIATE APPELLATE COURT DISREGARDED THE
PRINCIPLE AND JURISPRUDENCE AS TO WHEN AWARD The situation is quite different where a director or officer is dealing with his
OF ACTUAL AND MORAL DAMAGES IS PROPER. own corporation. In the instant case respondent Te was not an ordinary
stockholder; he was a member of the Board of Directors and Auditor of the
V corporation as well. He was what is often referred to as a "self-dealing"
director.
IN NOT AWARDING PETITIONER'S CAUSE OF ACTION
AS STATED IN ITS ANSWER WITH SPECIAL AND A director of a corporation holds a position of trust and as such, he owes a
AFFIRMATIVE DEFENSES WITH COUNTERCLAIM THE duty of loyalty to his corporation. 9 In case his interests conflict with those of
INTERMEDIATE APPELLATE COURT HAS CLEARLY the corporation, he cannot sacrifice the latter to his own advantage and
DEPARTED FROM THE ACCEPTED USUAL, COURSE OF benefit. As corporate managers, directors are committed to seek the
JUDICIAL PROCEEDINGS. maximum amount of profits for the corporation. This trust relationship "is not
a matter of statutory or technical law. It springs from the fact that directors
have the control and guidance of corporate affairs and property and hence of
There is only one legal issue to be resolved by this Court: whether or not the
the property interests of the stockholders." 10 In the case of Gokongwei v.
"dealership agreement" referred by the President and Chairman of the Board
Securities and Exchange Commission, this Court quoted with favor
of petitioner corporation is a valid and enforceable contract. We do not agree
from Pepper v. Litton, 11 thus:
with the conclusion of the respondent Court that it is.

. . . He cannot by the intervention of a corporate entity violate


Under the Corporation Law, which was then in force at the time this case
the ancient precept against serving two masters. . . . He
arose, 5 as well as under the present Corporation Code, all corporate powers
cannot utilize his inside information and his strategic position
shall be exercised by the Board of Directors, except as otherwise provided by
for his own preferment. He cannot violate rules of fair play by
law. 6Although it cannot completely abdicate its power and responsibility to
doing indirectly through the corporation what he could not do
act for the juridical entity, the Board may expressly delegate specific powers
directly. He cannot use his power for his personal advantage
to its President or any of its officers. In the absence of such express
and to the detriment of the stockholders and creditors no
delegation, a contract entered into by its President, on behalf of the
matter how absolute in terms that power may be and no
corporation, may still bind the corporation if the board should ratify the same
matter how meticulous he is to satisfy technical
expressly or impliedly. Implied ratification may take various forms like
requirements. For that power is at all times subject to the
silence or acquiescence; by acts showing approval or adoption of the
equitable limitation that it may not be exercised for the
contract; or by acceptance and retention of benefits flowing

1035
aggrandizement, preference, or advantage of the fiduciary to however, That the contract is fair and reasonable under the
the exclusion or detriment of the cestuis. . . . . circumstances.

On the other hand, a director's contract with his corporation is not in all Although the old Corporation Law which governs the instant case did not
instances void or voidable. If the contract is fair and reasonable under the contain a similar provision, yet the cited provision substantially incorporates
circumstances, it may be ratified by the stockholders provided a full well-settled principles in corporate law. 12
disclosure of his adverse interest is made. Section 32 of the Corporation
Code provides, thus: Granting arguendo that the "dealership agreement" involved here would be
valid and enforceable if entered into with a person other than a director or
Sec. 32. Dealings of directors, trustees or officers with the officer of the corporation, the fact that the other party to the contract was a
corporation. A contract of the corporation with one or Director and Auditor of the petitioner corporation changes the whole situation.
more of its directors or trustees or officers is voidable, at the First of all, We believe that the contract was neither fair nor reasonable. The
option of such corporation, unless all the following conditions "dealership agreement" entered into in July, 1969, was to sell and supply to
are present: respondent Te 20,000 bags of white cement per month, for five years starting
September, 1970, at thefixed price of P9.70 per bag. Respondent Te is a
1. That the presence of such director or trustee in the board businessman himself and must have known, or at least must be presumed to
meeting in which the contract was approved was not know, that at that time, prices of commodities in general, and white cement in
necessary to constitute a quorum for such meeting; particular, were not stable and were expected to rise. At the time of the
contract, petitioner corporation had not even commenced the manufacture of
2. That the vote of such director or trustee was not white cement, the reason why delivery was not to begin until 14 months later.
necessary for the approval of the contract; He must have known that within that period of six years, there would be a
considerable rise in the price of white cement. In fact, respondent Te's own
Memorandum shows that in September, 1970, the price per bag was P14.50,
3. That the contract is fair and reasonable under the
and by the middle of 1975, it was already P37.50 per bag. Despite this, no
circumstances; and
provision was made in the "dealership agreement" to allow for an increase in
price mutually acceptable to the parties. Instead, the price was pegged at
4. That in the case of an officer, the contract with the officer P9.70 per bag for the whole five years of the contract. Fairness on his part as
has been previously authorized by the Board of Directors. a director of the corporation from whom he was to buy the cement, would
require such a provision. In fact, this unfairness in the contract is also a basis
Where any of the first two conditions set forth in the which renders a contract entered into by the President, without authority from
preceding paragraph is absent, in the case of a contract with the Board of Directors, void or voidable, although it may have been in the
a director or trustee, such contract may be ratified by the ordinary course of business. We believe that the fixed price of P9.70 per bag
vote of the stockholders representing at least two-thirds (2/3) for a period of five years was not fair and reasonable. Respondent Te,
of the outstanding capital stock or of two-thirds (2/3) of the himself, when he subsequently entered into contracts to resell the cement to
members in a meeting called for the purpose: Provided, That his "new dealers" Henry Wee 13 and Gaudencio Galang 14 stipulated as
full disclosure of the adverse interest of the directors or follows:
trustees involved is made at such meeting: Provided,

1036
The price of white cement shall be mutually determined by Narvasa, C.J., Padilla, Regalado and Nocon, JJ., concur.
us but in no case shall the same be less than P14.00 per
bag (94 lbs).

The contract with Henry Wee was on September 15, 1969, and that with
Gaudencio Galang, on October 13, 1967. A similar contract with Prudencio
Lim was made on December 29, 1969. 15 All of these contracts were entered
into soon after his "dealership agreement" with petitioner corporation, and in
each one of them he protected himself from any increase in the market price
of white cement. Yet, except for the contract with Henry Wee, the contracts
were for only two years from October, 1970. Why did he not protect the
corporation in the same manner when he entered into the "dealership
agreement"? For that matter, why did the President and the Chairman of the
Board not do so either? As director, specially since he was the other party in
interest, respondent Te's bounden duty was to act in such manner as not to
unduly prejudice the corporation. In the light of the circumstances of this
case, it is to Us quite clear that he was guilty of disloyalty to the corporation;
he was attempting in effect, to enrich himself at the expense of the
corporation. There is no showing that the stockholders ratified the "dealership
agreement" or that they were fully aware of its provisions. The contract was
therefore not valid and this Court cannot allow him to reap the fruits of his
disloyalty.

As a result of this action which has been proven to be without legal basis,
petitioner corporation's reputation and goodwill have been prejudiced.
However, there can be no award for moral damages under Article 2217 and
succeeding articles on Section 1 of Chapter 3 of Title XVIII of the Civil Code
in favor of a corporation.

In view of the foregoing, the Decision and Resolution of the Intermediate


Appellate Court dated March 30, 1984 and August 6, 1984, respectively, are
hereby SET ASIDE. Private respondent Alejandro Te is hereby ordered to
pay petitioner corporation the sum of P20,000.00 for attorney's fees, plus the
cost of suit and expenses of litigation.
G.R. No. 142936 April 17, 2002
SO ORDERED.

1037
PHILIPPINE NATIONAL BANK & NATIONAL SUGAR DEVELOPMENT operating under the laws of the Philippines, with office and principal
CORPORATION, petitioners, place of business at Escolta Street, Sta. Cruz, Manila; whereas, the
vs. other defendant, the National Sugar Development Corporation
ANDRADA ELECTRIC & ENGINEERING COMPANY, respondent. (NASUDECO in brief), is also a semi-government corporation and
the sugar arm of the PNB, with office and principal place of business
PANGANIBAN, J.: at the 2nd Floor, Sampaguita Building, Cubao, Quezon City; and the
defendant Pampanga Sugar Mills (PASUMIL in short), is a
Basic is the rule that a corporation has a legal personality distinct and corporation organized, existing and operating under the 1975 laws of
separate from the persons and entities owning it. The corporate veil may be the Philippines, and had its business office before 1975 at Del
lifted only if it has been used to shield fraud, defend crime, justify a wrong, Carmen, Floridablanca, Pampanga; that the plaintiff is engaged in
defeat public convenience, insulate bad faith or perpetuate injustice. Thus, the business of general construction for the repairs and/or
the mere fact that the Philippine National Bank (PNB) acquired ownership or construction of different kinds of machineries and buildings; that on
management of some assets of the Pampanga Sugar Mill (PASUMIL), which August 26, 1975, the defendant PNB acquired the assets of the
had earlier been foreclosed and purchased at the resulting public auction by defendant PASUMIL that were earlier foreclosed by the Development
the Development Bank of the Philippines (DBP), will not make PNB liable for Bank of the Philippines (DBP) under LOI No. 311; that the defendant
the PASUMILs contractual debts to respondent. PNB organized the defendant NASUDECO in September, 1975, to
take ownership and possession of the assets and ultimately to
nationalize and consolidate its interest in other PNB controlled sugar
Statement of the Case
mills; that prior to October 29, 1971, the defendant PASUMIL
engaged the services of plaintiff for electrical rewinding and repair,
Before us is a Petition for Review assailing the April 17, 2000 Decision 1 of the most of which were partially paid by the defendant PASUMIL, leaving
Court of Appeals (CA) in CA-GR CV No. 57610. The decretal portion of the several unpaid accounts with the plaintiff; that finally, on October 29,
challenged Decision reads as follows: 1971, the plaintiff and the defendant PASUMIL entered into a
contract for the plaintiff to perform the following, to wit
"WHEREFORE, the judgment appealed from is hereby AFFIRMED." 2
(a) Construction of one (1) power house building;
The Facts
(b) Construction of three (3) reinforced concrete foundation
The factual antecedents of the case are summarized by the Court of Appeals for three (3) units 350 KW diesel engine generating set[s];
as follows:
(c) Construction of three (3) reinforced concrete foundation
"In its complaint, the plaintiff [herein respondent] alleged that it is a for the 5,000 KW and 1,250 KW turbo generator sets;
partnership duly organized, existing, and operating under the laws of
the Philippines, with office and principal place of business at Nos. (d) Complete overhauling and reconditioning tests sum for
794-812 Del Monte [A]venue, Quezon City, while the defendant three (3) 350 KW diesel engine generating set[s];
[herein petitioner] Philippine National Bank (herein referred to as
PNB), is a semi-government corporation duly organized, existing and

1038
(e) Installation of turbine and diesel generating sets of June 27, 1973, amounting to P527,263.80, as shown in the
including transformer, switchboard, electrical wirings and Certification of the chief accountant of the PNB, a machine copy of
pipe provided those stated units are completely supplied with which is appended as Annex C of the complaint; that out of said
their accessories; unpaid balance of P527,263.80, the defendant PASUMIL made a
partial payment to the plaintiff of P14,000.00, in broken amounts,
(f) Relocating of 2,400 V transmission line, demolition of all covering the period from January 5, 1974 up to May 23, 1974,
existing concrete foundation and drainage canals, leaving an unpaid balance of P513,263.80; that the defendant
excavation, and earth fillings all for the total amount of PASUMIL and the defendant PNB, and now the defendant
P543,500.00 as evidenced by a contract, [a] xerox copy of NASUDECO, failed and refused to pay the plaintiff their just, valid
which is hereto attached as Annex A and made an integral and demandable obligation; that the President of the NASUDECO is
part of this complaint; also the Vice-President of the PNB, and this official holds office at
the 10th Floor of the PNB, Escolta, Manila, and plaintiff besought this
that aside from the work contract mentioned-above, the defendant official to pay the outstanding obligation of the defendant PASUMIL,
PASUMIL required the plaintiff to perform extra work, and provide inasmuch as the defendant PNB and NASUDECO now owned and
electrical equipment and spare parts, such as: possessed the assets of the defendant PASUMIL, and these
defendants all benefited from the works, and the electrical, as well as
the engineering and repairs, performed by the plaintiff; that because
(a) Supply of electrical devices;
of the failure and refusal of the defendants to pay their just, valid,
and demandable obligations, plaintiff suffered actual damages in the
(b) Extra mechanical works; total amount of P513,263.80; and that in order to recover these
sums, the plaintiff was compelled to engage the professional
(c) Extra fabrication works; services of counsel, to whom the plaintiff agreed to pay a sum
equivalent to 25% of the amount of the obligation due by way of
(d) Supply of materials and consumable items; attorneys fees. Accordingly, the plaintiff prayed that judgment be
rendered against the defendants PNB, NASUDECO, and PASUMIL,
(e) Electrical shop repair; jointly and severally to wit:

(f) Supply of parts and related works for turbine generator; (1) Sentencing the defendants to pay the plaintiffs the sum
of P513,263.80, with annual interest of 14% from the time
(g) Supply of electrical equipment for machinery; the obligation falls due and demandable;

(h) Supply of diesel engine parts and other related works (2) Condemning the defendants to pay attorneys fees
including fabrication of parts. amounting to 25% of the amount claim;

that out of the total obligation of P777,263.80, the defendant (3) Ordering the defendants to pay the costs of the suit.
PASUMIL had paid only P250,000.00, leaving an unpaid balance, as

1039
"The defendants PNB and NASUDECO filed a joint motion to dismiss "By way of counterclaim, the NASUDECO averred that by reason of
the complaint chiefly on the ground that the complaint failed to state the filing by the plaintiff of the present suit, which it [labeled] as
sufficient allegations to establish a cause of action against both unfounded or baseless, the defendant NASUDECO was constrained
defendants, inasmuch as there is lack or want of privity of contract to litigate and incur litigation expenses in the amount of P50,000.00,
between the plaintiff and the two defendants, the PNB and which plaintiff should be sentenced to pay. Accordingly, NASUDECO
NASUDECO, said defendants citing Article 1311 of the New Civil prayed that the complaint be dismissed and on its counterclaim, that
Code, and the case law ruling in Salonga v. Warner Barnes & Co., 88 the plaintiff be condemned to pay P50,000.00 in concept of
Phil. 125; and Manila Port Service, et al. v. Court of Appeals, et al., attorneys fees as well as exemplary damages.
20 SCRA 1214.
"In its answer, the defendant PNB likewise reiterated the grounds of
"The motion to dismiss was by the court a quo denied in its Order of its motion to dismiss, namely: (1) the complaint states no cause of
November 27, 1980; in the same order, that court directed the action against the defendant PNB; (2) that PNB is not a party to the
defendants to file their answer to the complaint within 15 days. contract alleged in par. 6 of the complaint and that the alleged
services rendered by the plaintiff to the defendant PASUMIL upon
"In their answer, the defendant NASUDECO reiterated the grounds which plaintiffs suit is erected, was rendered long before PNB took
of its motion to dismiss, to wit: possession of the assets of the defendant PASUMIL under LOI No.
189-A; (3) that the PNB take-over of the assets of the defendant
That the complaint does not state a sufficient cause of PASUMIL under LOI 189-A was solely for the purpose of
action against the defendant NASUDECO because: (a) reconditioning the sugar central so that PASUMIL may resume its
NASUDECO is not x x x privy to the various electrical operations in time for the 1974-75 milling season, and that nothing in
construction jobs being sued upon by the plaintiff under the the said LOI No. 189-A, as well as in LOI No. 311, authorized or
present complaint; (b) the taking over by NASUDECO of the directed PNB to assume the corporate obligation/s of PASUMIL, let
assets of defendant PASUMIL was solely for the purpose of alone that for which the present action is brought; (4) that PNBs
reconditioning the sugar central of defendant PASUMIL management and operation under LOI No. 311 did not refer to any
pursuant to martial law powers of the President under the asset of PASUMIL which the PNB had to acquire and thereafter
Constitution; (c) nothing in the LOI No. 189-A (as well as in [manage], but only to those which were foreclosed by the DBP and
LOI No. 311) authorized or commanded the PNB or its were in turn redeemed by the PNB from the DBP; (5) that
subsidiary corporation, the NASUDECO, to assume the conformably to LOI No. 311, on August 15, 1975, the PNB and the
corporate obligations of PASUMIL as that being involved in Development Bank of the Philippines (DBP) entered into a
the present case; and, (d) all that was mentioned by the said Redemption Agreement whereby DBP sold, transferred and
letter of instruction insofar as the PASUMIL liabilities [were] conveyed in favor of the PNB, by way of redemption, all its (DBP)
concerned [was] for the PNB, or its subsidiary corporation rights and interest in and over the foreclosed real and/or personal
the NASUDECO, to make a study of, and submit [a] properties of PASUMIL, as shown in Annex C which is made an
recommendation on the problems concerning the same. integral part of the answer; (6) that again, conformably with LOI No.
311, PNB pursuant to a Deed of Assignment dated October 21, 1975,
conveyed, transferred, and assigned for valuable consideration, in
favor of NASUDECO, a distinct and independent corporation, all its

1040
(PNB) rights and interest in and under the above Redemption National Bank (PNB) NATIONAL SUGAR DEVELOPMENT
Agreement. This is shown in Annex D which is also made an CORPORATION (NASUDECO) and PAMPANGA SUGAR
integral part of the answer; [7] that as a consequence of the said MILLS (PASUMIL), ordering the latter to pay jointly and
Deed of Assignment, PNB on October 21, 1975 ceased to managed severally the former the following:
and operate the above-mentioned assets of PASUMIL, which
function was now actually transferred to NASUDECO. In other 1. The sum of P513,623.80 plus interest thereon at
words, so asserted PNB, the complaint as to PNB, had become moot the rate of 14% per annum as claimed from
and academic because of the execution of the said Deed of September 25, 1980 until fully paid;
Assignment; [8] that moreover, LOI No. 311 did not authorize or
direct PNB to assume the corporate obligations of PASUMIL, 2. The sum of P102,724.76 as attorneys fees; and,
including the alleged obligation upon which this present suit was
brought; and [9] that, at most, what was granted to PNB in this
3. Costs.
respect was the authority to make a study of and submit
recommendation on the problems concerning the claims of PASUMIL
creditors, under sub-par. 5 LOI No. 311. SO ORDERED.

"In its counterclaim, the PNB averred that it was unnecessarily Manila, Philippines, September 4, 1986.
constrained to litigate and to incur expenses in this case, hence it is
entitled to claim attorneys fees in the amount of at least P50,000.00.
Accordingly, PNB prayed that the complaint be dismissed; and that
'(SGD) ERNESTO S. TENGCO
on its counterclaim, that the plaintiff be sentenced to pay defendant
Judge"3
PNB the sum of P50,000.00 as attorneys fees, aside from
exemplary damages in such amount that the court may seem just
and equitable in the premises.
Ruling of the Court of Appeals
"Summons by publication was made via the Philippines Daily
Express, a newspaper with editorial office at 371 Bonifacio Drive, Affirming the trial court, the CA held that it was offensive to the basic tenets
Port Area, Manila, against the defendant PASUMIL, which was of justice and equity for a corporation to take over and operate the business
thereafter declared in default as shown in the August 7, 1981 Order of another corporation, while disavowing or repudiating any responsibility,
issued by the Trial Court. obligation or liability arising therefrom.4

"After due proceedings, the Trial Court rendered judgment, the Hence, this Petition.5
decretal portion of which reads:
Issues
WHEREFORE, judgment is hereby rendered in favor of
plaintiff and against the defendant Corporation, Philippine

1041
In their Memorandum, petitioners raise the following errors for the Courts As a general rule, questions of fact may not be raised in a petition for review
consideration: under Rule 45 of the Rules of Court. 7 To this rule, however, there are some
exceptions enumerated in Fuentes v. Court of Appeals.8 After a careful
"I scrutiny of the records and the pleadings submitted by the parties, we find
that the lower courts misappreciated the evidence presented. 9 Overlooked by
The Court of Appeals gravely erred in law in holding the herein the CA were certain relevant facts that would justify a conclusion different
petitioners liable for the unpaid corporate debts of PASUMIL, a from that reached in the assailed Decision.10
corporation whose corporate existence has not been legally
extinguished or terminated, simply because of petitioners[] take-over Petitioners posit that they should not be held liable for the corporate debts of
of the management and operation of PASUMIL pursuant to the PASUMIL, because their takeover of the latters foreclosed assets did not
mandates of LOI No. 189-A, as amended by LOI No. 311. make them assignees. On the other hand, respondent asserts that
petitioners and PASUMIL should be treated as one entity and, as such, jointly
"II and severally held liable for PASUMILs unpaid obligation.1wphi1.nt

The Court of Appeals gravely erred in law in not applying [to] the As a rule, a corporation that purchases the assets of another will not be liable
case at bench the ruling enunciated in Edward J. Nell Co. v. Pacific for the debts of the selling corporation, provided the former acted in good
Farms, 15 SCRA 415."6 faith and paid adequate consideration for such assets, except when any of
the following circumstances is present: (1) where the purchaser expressly or
impliedly agrees to assume the debts, (2) where the transaction amounts to a
Succinctly put, the aforesaid errors boil down to the principal issue of
consolidation or merger of the corporations, (3) where the purchasing
whether PNB is liable for the unpaid debts of PASUMIL to respondent.
corporation is merely a continuation of the selling corporation, and (4) where
the transaction is fraudulently entered into in order to escape liability for
This Courts Ruling those debts.11

The Petition is meritorious. Piercing the Corporate

Main Issue: Veil Not Warranted

Liability for Corporate Debts A corporation is an artificial being created by operation of law. It possesses
the right of succession and such powers, attributes, and properties expressly
authorized by law or incident to its existence. 12 It has a personality separate
and distinct from the persons composing it, as well as from any other legal
entity to which it may be related.13 This is basic.

Equally well-settled is the principle that the corporate mask may be removed
or the corporate veil pierced when the corporation is just an alter ego of a

1042
person or of another corporation.14 For reasons of public policy and in the control over it warrants the disregard of corporate personalities. 31 Second,
interest of justice, the corporate veil will justifiably be impaled 15 only when it there is no evidence that their juridical personality was used to commit a
becomes a shield for fraud, illegality or inequity committed against third fraud or to do a wrong; or that the separate corporate entity was farcically
persons.16 used as a mere alter ego, business conduit or instrumentality of another
entity or person.32 Third, respondent was not defrauded or injured when
Hence, any application of the doctrine of piercing the corporate veil should be petitioners acquired the assets of PASUMIL.33
done with caution.17 A court should be mindful of the milieu where it is to be
applied.18 It must be certain that the corporate fiction was misused to such an Being the party that asked for the piercing of the corporate veil, respondent
extent that injustice, fraud, or crime was committed against another, in had the burden of presenting clear and convincing evidence to justify the
disregard of its rights.19 The wrongdoing must be clearly and convincingly setting aside of the separate corporate personality rule. 34 However, it utterly
established; it cannot be presumed. 20 Otherwise, an injustice that was never failed to discharge this burden;35 it failed to establish by competent evidence
unintended may result from an erroneous application.21 that petitioners separate corporate veil had been used to conceal fraud,
illegality or inequity.36
This Court has pierced the corporate veil to ward off a judgment credit, 22 to
avoid inclusion of corporate assets as part of the estate of the decedent, 23 to While we agree with respondents claim that the assets of the National Sugar
escape liability arising from a debt, 24 or to perpetuate fraud and/or confuse Development Corporation (NASUDECO) can be easily traced to
legitimate issues25 either to promote or to shield unfair objectives 26 or to cover PASUMIL,37 we are not convinced that the transfer of the latters assets to
up an otherwise blatant violation of the prohibition against forum- petitioners was fraudulently entered into in order to escape liability for its debt
shopping.27 Only in these and similar instances may the veil be pierced and to respondent.38
disregarded.28
A careful review of the records reveals that DBP foreclosed the mortgage
The question of whether a corporation is a mere alter ego is one of executed by PASUMIL and acquired the assets as the highest bidder at the
fact.29 Piercing the veil of corporate fiction may be allowed only if the public auction conducted.39 The bank was justified in foreclosing the
following elements concur: (1) control -- not mere stock control, but complete mortgage, because the PASUMIL account had incurred arrearages of more
domination -- not only of finances, but of policy and business practice in than 20 percent of the total outstanding obligation. 40 Thus, DBP had not only
respect to the transaction attacked, must have been such that the corporate a right, but also a duty under the law to foreclose the subject properties. 41
entity as to this transaction had at the time no separate mind, will or
existence of its own; (2) such control must have been used by the defendant Pursuant to LOI No. 189-A42 as amended by LOI No. 311,43 PNB acquired
to commit a fraud or a wrong to perpetuate the violation of a statutory or PASUMILs assets that DBP had foreclosed and purchased in the normal
other positive legal duty, or a dishonest and an unjust act in contravention of course. Petitioner bank was likewise tasked to manage temporarily the
plaintiffs legal right; and (3) the said control and breach of duty must have operation of such assets either by itself or through a subsidiary corporation. 44
proximately caused the injury or unjust loss complained of. 30
PNB, as the second mortgagee, redeemed from DBP the foreclosed
We believe that the absence of the foregoing elements in the present case PASUMIL assets pursuant to Section 6 of Act No. 3135. 45 These assets were
precludes the piercing of the corporate veil. First, other than the fact that later conveyed to PNB for a consideration, the terms of which were
petitioners acquired the assets of PASUMIL, there is no showing that their embodied in the Redemption Agreement. 46 PNB, as successor-in-interest,

1043
stepped into the shoes of DBP as PASUMILs creditor. 47 By way of a Deed of fundamental changes in the corporation, as well as in the rights of
Assignment,48 PNB then transferred to NASUDECO all its rights under the stockholders and creditors, there must be an express provision of law
Redemption Agreement. authorizing them.56 For a valid merger or consolidation, the approval by the
Securities and Exchange Commission (SEC) of the articles of merger or
In Development Bank of the Philippines v. Court of Appeals,49 we had the consolidation is required.57 These articles must likewise be duly approved by
occasion to resolve a similar issue. We ruled that PNB, DBP and their a majority of the respective stockholders of the constituent corporations. 58
transferees were not liable for Marinduque Minings unpaid obligations to
Remington Industrial Sales Corporation (Remington) after the two banks had In the case at bar, we hold that there is no merger or consolidation with
foreclosed the assets of Marinduque Mining. We likewise held that respect to PASUMIL and PNB. The procedure prescribed under Title IX of the
Remington failed to discharge its burden of proving bad faith on the part of Corporation Code59 was not followed.
Marinduque Mining to justify the piercing of the corporate veil.
In fact, PASUMILs corporate existence, as correctly found by the CA, had
In the instant case, the CA erred in affirming the trial courts lifting of the not been legally extinguished or terminated. 60 Further, prior to PNBs
corporate mask.50 The CA did not point to any fact evidencing bad faith on acquisition of the foreclosed assets, PASUMIL had previously made partial
the part of PNB and its transferee. 51 The corporate fiction was not used to payments to respondent for the formers obligation in the amount of
defeat public convenience, justify a wrong, protect fraud or defend P777,263.80. As of June 27, 1973, PASUMIL had paid P250,000 to
crime.52 None of the foregoing exceptions was shown to exist in the present respondent and, from January 5, 1974 to May 23, 1974, another P14,000.
case.53 On the contrary, the lifting of the corporate veil would result in
manifest injustice. This we cannot allow. Neither did petitioner expressly or impliedly agree to assume the debt of
PASUMIL to respondent.61 LOI No. 11 explicitly provides that PNB shall study
No Merger or Consolidation and submit recommendations on the claims of PASUMILs creditors. 62Clearly,
the corporate separateness between PASUMIL and PNB remains, despite
Respondent further claims that petitioners should be held liable for the respondents insistence to the contrary.63
unpaid obligations of PASUMIL by virtue of LOI Nos. 189-A and 311, which
expressly authorized PASUMIL and PNB to merge or consolidate. On the WHEREFORE, the Petition is hereby GRANTED and the assailed
other hand, petitioners contend that their takeover of the operations of Decision SET ASIDE. No pronouncement as to costs.
PASUMIL did not involve any corporate merger or consolidation, because the
latter had never lost its separate identity as a corporation. SO ORDERED.

A consolidation is the union of two or more existing entities to form a new Vitug, Sandoval-Gutierrez, and Carpio, JJ., concur.
entity called the consolidated corporation. A merger, on the other hand, is a Melo, J., Abroad, on official leave.
union whereby one or more existing corporations are absorbed by another
corporation that survives and continues the combined business. 54

The merger, however, does not become effective upon the mere agreement
of the constituent corporations.55Since a merger or consolidation involves

1044
G.R. No. 123793 June 29, 1998

ASSOCIATED BANK, petitioner,


vs.
COURT OF APPEALS and LORENZO SARMIENTO JR., respondents.

PANGANIBAN, J.:

In a merger, does the surviving corporation have a right to enforce a contract


entered into by the absorbed company subsequent to the date of the merger
agreement, but prior to the issuance of a certificate of merger by the
Securities and Exchange Commission?

The Case

This is a petition for review under Rule 45 of the Rules of Court, seeking to
set aside the Decision 1 of the Court of Appeals 2 in CA-GR CV No. 26465
promulgated on January 30, 1996, which answered the above question in the
negative. The challenged Decision reversed and set aside the October 17,
1986 Decision 3 in Civil Case No. 85-32243, promulgated by the Regional
Trial Court of Manila, Branch 48, which disposed of the controversy in favor
of herein petitioner as follows: 4

WHEREFORE, judgment is hereby rendered in favor of the


plaintiff Associated Bank. The defendant Lorenzo Sarmiento,
Jr. is ordered to pay plaintiff:

1. The amount of P4,689,413.63 with interest thereon at 14%


per annum until fully paid;

2. The amount of P200,000.00 as and for attorney's fees;


and

1045
3. The costs of suit. . . . [T]he defendant denied all the pertinent allegations in the
complaint and alleged as affirmative and[/]or special
On the other hand, the Court of Appeals resolved the case in this wise: 5 defenses that the complaint states no valid cause of action;
that the plaintiff is not the proper party in interest because
WHEREFORE, premises considered, the decision appealed the promissory note was executed in favor of Citizens Bank
from, dated October 17, 1986 is REVERSED and SET and Trust Company; that the promissory note does not
ASIDE and another judgment rendered DISMISSING accurately reflect the true intention and agreement of the
plaintiff-appellee's complaint, docketed as Civil Case No. 85- parties; that terms and conditions of the promissory note are
32243. There is no pronouncement as to costs. onerous and must be construed against the creditor-payee
bank; that several partial payments made in the promissory
note are not properly applied; that the present action is
The Facts
premature; that as compulsory counterclaim the defendant
prays for attorney's fees, moral damages and expenses of
The undisputed factual antecedents, as narrated by the trial court and litigation.
adopted by public respondent, are as follows: 6
On May 22, 1986, the defendant was declared as if in default
. . . [O]n or about September 16, 1975 Associated Banking for failure to appear at the Pre-Trial Conference despite due
Corporation and Citizens Bank and Trust Company merged notice.
to form just one banking corporation known as Associated
Citizens Bank, the surviving bank. On or about March 10,
A Motion to Lift Order of Default and/or Reconsideration of
1981, the Associated Citizens Bank changed its corporate
Order dated May 22, 1986 was filed by defendant's counsel
name to Associated Bank by virtue of the Amended Articles
which was denied by the Court in [an] order dated
of Incorporation. On September 7, 1977, the defendant
September 16, 1986 and the plaintiff was allowed to present
executed in favor of Associated Bank a promissory note
its evidence before the Court ex-parte on October 16, 1986.
whereby the former undertook to pay the latter the sum of
P2,500,000.00 payable on or before March 6, 1978. As per
said promissory note, the defendant agreed to pay interest at At the hearing before the Court ex-parte, Esteban C.
14% per annum, 3% per annum in the form of liquidated Ocampo testified that . . . he is an accountant of the Loans
damages, compounded interests, and attorney's fees, in and Discount Department of the plaintiff bank; that as such,
case of litigation equivalent to 10% of the amount due. The he supervises the accounting section of the bank, he
defendant, to date, still owes plaintiff bank the amount of counterchecks all the transactions that transpired during the
P2,250,000.00 exclusive of interest and other charges. day and is responsible for all the accounts and records and
Despite repeated demands the defendant failed to pay the other things that may[ ]be assigned to the Loans and
amount due. Discount Department; that he knows the [D]efendant
Lorenzo Sarmiento, Jr. because he has an outstanding loan
with them as per their records; that Lorenzo Sarmiento, Jr.
xxx xxx xxx
executed a promissory note No. TL-2649-77 dated
September 7, 1977 in the amount of P2,500,000.00 (Exhibit

1046
A); that Associated Banking Corporation and the Citizens III The [trial court] erred and gravely abuse[d] its discretion in
Bank and Trust Company merged to form one banking rendering the two as if in default orders dated May 22, 1986
corporation known as the Associated Citizens Bank and is and September 16, 1986 and in not reconsidering the same
now known as Associated Bank by virtue of its Amended upon technical grounds which in effect subvert the best
Articles of Incorporation; that there were partial payments primordial interest of substantial justice and equity.
made but not full; that the defendant has not paid his
obligation as evidenced by the latest statement of account IV The court a quo erred in issuing the orders dated May 22,
(Exh. B); that as per statement of account the outstanding 1986 and September 16, 1986 declaring appellant as if in
obligation of the defendant is P5,689,413.63 less default due to non-appearance of appellant's attending
P1,000,000.00 or P4,689,413.63 (Exh. B, B-1); that a counsel who had resigned from the law firm and while the
demand letter dated June 6, 1985 was sent by the bank thru parties [were] negotiating for settlement of the case and after
its counsel (Exh. C) which was received by the defendant on a one million peso payment had in fact been paid to appellee
November 12, 1985 (Exh. C, C-1, C-2, C-3); that the bank for appellant's account at the start of such negotiation
defendant paid only P1,000,000.00 which is reflected in the on February 18, 1986 as act of earnest desire to settle the
Exhibit C. obligation in good faith by the interested parties.

Based on the evidence presented by petitioner, the trial court ordered V The lower court erred in according credence to appellee
Respondent Sarmiento to pay the bank his remaining balance plus interests bank's Exhibit B statement of account which had been
and attorney's fees. In his appeal, Sarmiento assigned to the trial court merely requested by its counsel during the trial and bearing
several errors, namely: 7 date of September 30, 1986.

I The [trial court] erred in denying appellant's motion to VI The lower court erred in accepting and giving credence to
dismiss appellee bank's complaint on the ground of lack of appellee bank's 27-year-old witness Esteban C. Ocampo as
cause of action and for being barred by prescription and of the date he testified on October 16, 1986, and therefore,
laches. he was merely an eighteen-year-old minor when appellant
supposedly incurred the foisted obligation under the subject
II The same lower court erred in admitting plaintiff-appellee PN No. TL-2649-77 dated September 7, 1977, Exhibit A of
bank's amended complaint while defendant-appellant's appellee bank.
motion to dismiss appelle bank's original complaint and
using/availing [itself of] the new additional allegations as VII The [trial court] erred in adopting appellee bank's Exhibit
bases in denial of said appellant's motion and in the B dated September 30, 1986 in its decision given in open
interpretation and application of the agreement of merger court on October 17, 1986 which exacted eighteen percent
and Section 80 of BP Blg. 68, Corporation Code of the (18%) per annum on the foisted principal amount of P2.5
Philippines. million when the subject PN, Exhibit A, stipulated only
fourteen percent (14%) per annum and which was actually

1047
9
prayed for in appellee bank's original and amended In its petition, petitioner cites the following "reasons":
complaints.
I The Court of Appeals erred in reversing the decision of the
VIII The appealed decision of the lower court erred in not trial court and in declaring that petitioner has no cause of
considering at all appellant's affirmative defenses that (1) the action against respondent over the promissory note.
subject PN No. TL-2649-77 for P2.5 million dated September
7, 1977, is merely an accommodation pour autrui of any II The Court of Appeals also erred in declaring that, since the
actual consideration to appellant himself and (2) the subject promissory note was executed in favor of Citizens Bank and
PN is a contract of adhesion, hence, [it] needs [to] be strictly Trust Company two years after the merger between
construed against appellee bank assuming for granted Associated Banking Corporation and Citizens Bank and
that it has the right to enforce and seek collection thereof. Trust Company, respondent is not liable to petitioner
because there is no privity of contract between respondent
IX The lower court should have at least allowed appellant the and Associated Bank.
opportunity to present countervailing evidence considering
the huge amounts claimed by appellee bank (principal sum III The Court of Appeals erred when it ruled that petitioner,
of P2.5 million which including accrued interests, penalties despite the merger between petitioner and Citizens Bank
and cost of litigation totaled P4,689,413.63) and appellant's and Trust Company, is not a real party in interest insofar as
affirmative defenses pursuant to substantial justice and the promissory note executed in favor of the merger.
equity.
In a nutshell, the main issue is whether Associated Bank, the surviving
The appellate court, however, found no need to tackle all the assigned errors corporation, may enforce the promissory note made by private respondent in
and limited itself to the question of "whether [herein petitioner had] favor of CBTC, the absorbed company, after the merger agreement had been
established or proven a cause of action against [herein private respondent]." signed.
Accordingly, Respondent Court held that the Associated Bank had no cause
of action against Lorenzo Sarmiento Jr., since said bank was not privy to the The Court's Ruling
promissory note executed by Sarmiento in favor of Citizens Bank and Trust
Company (CBTC). The court ruled that the earlier merger between the two
The petition is impressed with merit.
banks could not have vested Associated Bank with any interest arising from
the promissory note executed in favor of CBTC after such merger.
The Main Issue:
Associated Bank Assumed
Thus, as earlier stated, Respondent Court set aside the decision of the trial
All Rights of CBTC
court and dismissed the complaint. Petitioner now comes to us for a reversal
of this ruling. 8
Ordinarily, in the merger of two or more existing corporations, one of the
combining corporations survives and continues the combined business, while
Issues
the rest are dissolved and all their rights, properties and liabilities are
acquired by the surviving corporation. 10 Although there is a dissolution of the

1048
absorbed corporations, there is no winding up of their affairs or liquidation of immunities, franchises and all appointments, designations
their assets, because the surviving corporation automatically acquires all and nominations, and all other rights and interests of [CBTC]
their rights, privileges and powers, as well as their liabilities. 11 as trustee, executor, administrator, registrar of stocks and
bonds, guardian of estates, assignee, receiver, trustee of
The merger, however, does not become effective upon the mere agreement estates of persons mentally ill and in every other fiduciary
of the constituent corporations. The procedure to be followed is prescribed capacity, and all and every other interest of [CBTC] shall
under the Corporation Code. 12 Section 79 of said Code requires the approval thereafter be effectually the property of [ABC] as they were
by the Securities and Exchange Commission (SEC) of the articles of merger of [CBTC], and title to any real estate, whether by deed or
which, in turn, must have been duly approved by a majority of the respective otherwise, vested in [CBTC] shall not revert or be in any way
stockholders of the constituent corporations. The same provision further impaired by reason thereof; provided, however, that all rights
states that the merger shall be effective only upon the issuance by the SEC of creditors and all liens upon any property of [CBTC] shall
of a certificate of merger. The effectivity date of the merger is crucial for be preserved and unimpaired and all debts, liabilities,
determining when the merged or absorbed corporation ceases to exist; and obligations, duties and undertakings of [CBTC], whether
when its rights, privileges, properties as well as liabilities pass on to the contractual or otherwise, expressed or implied, actual or
surviving corporation. contingent, shall henceforth attach to [ABC] which shall be
responsible therefor and may be enforced against [ABC] to
Consistent with the aforementioned Section 79, the September 16, 1975 the same extent as if the same debts liabilities, obligations,
Agreement of Merger, 13 which Associated Banking Corporation (ABC) and duties and undertakings have been originally incurred or
Citizens Bank and Trust Company (CBTC) entered into, provided that its contracted by [ABC], subject, however, to all rights,
effectivity "shall, for all intents and purposes, be the date when the necessary privileges, defenses, set-offs and counterclaims which
papers to carry out this [m]erger shall have been approved by the Securities [CBTC] has or might have and which shall pertain to
and Exchange Commission." 14 As to the transfer of the properties of CBTC [ABC]. 15
to ABC, the agreement provides:
The records do not show when the SEC approved the merger. Private
10. Upon effective date of the Merger, all rights, privileges, respondent's theory is that it took effect on the date of the execution of the
powers, immunities, franchises, assets and property of agreement itself, which was September 16, 1975. Private respondent
[CBTC], whether real, personal or mixed, and including contends that, since he issued the promissory note to CBTC on September
[CBTC's] goodwill and tradename, and all debts due to 7, 1977 two years after the merger agreement had been executed
[CBTC] on whatever act, and all other things in action CBTC could not have conveyed or transferred to petitioner its interest in the
belonging to [CBTC] as of the effective date of the [m]erger said note, which was not yet in existence at the time of the merger.
shall be vested in [ABC], the SURVIVING BANK, without Therefore, petitioner, the surviving bank, has no right to enforce the
need of further act or deed, unless by express requirements promissory note on private respondent; such right properly pertains only to
of law or of a government agency, any separate or specific CBTC.
deed of conveyance to legally effect the transfer or
assignment of any kind of property [or] asset is required, in
which case such document or deed shall be executed
accordingly; and all property, rights, privileges, powers,

1049
Assuming that the effectivity date of the merger was the date of its execution, In light of the foregoing, the Court holds that petitioner has a valid cause of
we still cannot agree that petitioner no longer has any interest in the action against private respondent. Clearly, the failure of private respondent to
promissory note. A closer perusal of the merger agreement leads to a honor his obligation under the promissory note constitutes a violation of
different conclusion. The provision quoted earlier has this other clause: petitioner's right to collect the proceeds of the loan it extended to the former.

Upon the effective date of the [m]erger, all references to Secondary Issues:
[CBTC] in any deed, documents, or other papers of Prescription, Laches, Contract
whatever kind or nature and wherever found shall be Pour Autrui, Lack of Consideration
deemed for all intents and purposes, references to [ABC],
the SURVIVING BANK, as if such references were direct No Prescription
references to [ABC]. . . . 6 (Emphasis supplied) or Laches

Thus, the fact that the promissory note was executed after the effectivity date Private respondent's claim that the action has prescribed, pursuant to Article
of the merger does not militate against petitioner. The agreement itself clearly 1149 of the Civil Code, is legally untenable. Petitioner's suit for collection of a
provides that all contracts irrespective of the date of execution entered sum of money was based on a written contract and prescribes after ten years
into in the name of CBTC shall be understood as pertaining to the surviving from the time its right of action arose. 19 Sarmiento's obligation under the
bank, herein petitioner. Since, in contrast to the earlier aforequoted provision, promissory note became due and demandable on March 6, 1978. Petitioner's
the latter clause no longer specifically refers only to contracts existing at the complaint was instituted on August 22, 1985, before the lapse of the ten-year
time of the merger, no distinction should be made. The clause must have prescriptive period. Definitely, petitioner still had every right to commence suit
been deliberately included in the agreement in order to protect the interests against the payor/obligor, the private respondent herein.
of the combining banks; specifically, to avoid giving the merger agreement a
farcical interpretation aimed at evading fulfillment of a due obligation. Neither is petitioner's action barred by laches. The principle of laches is a
creation of equity, which is applied not to penalize neglect or failure to assert
Thus, although the subject promissory note names CBTC as the payee, the a right within a reasonable time, but rather to avoid recognizing a right when
reference to CBTC in the note shall be construed, under the very provisions to do so would result in a clearly inequitable situation 20 or in an
of the merger agreement, as a reference to petitioner bank, "as if such injustice. 21 To require private respondent to pay the remaining balance of his
reference [was a] direct reference to" the latter "for all intents and purposes." loan is certainly not inequitable or unjust. What would be manifestly unjust
and inequitable is his contention that CBTC is the proper party to proceed
No other construction can be given to the unequivocal stipulation. Being against him despite the fact, which he himself asserts, that CBTC's corporate
clear, plain and free of ambiguity, the provision must be given its literal personality has been dissolved by virtue of its merger with petitioner. To hold
meaning 17 and applied without a convoluted interpretation. Verba lelegis non that no payee/obligee exists and to let private respondent enjoy the fruits of
est recedendum. 18 his loan without liability is surely most unfair and unconscionable, amounting
to unjust enrichment at the expense of petitioner. Besides, this Court has
held that the doctrine of laches is inapplicable where the claim was filed
within the prescriptive period set forth under the law. 22

1050
No Contract Consideration
Pour Autrui
Private respondent also claims that he received no consideration for the
Private respondent, while not denying that he executed the promissory note promissory note and, in support thereof, cites petitioner's failure to submit
in the amount of P2,500,000 in favor of CBTC, offers the alternative defense any proof of his loan application and of his actual receipt of the amount
that said note was a contract pour autrui. loaned. These arguments deserve no merit. Res ipsa loquitur. The
instrument, bearing the signature of private respondent, speaks for itself.
A stipulation pour autrui is one in favor of a third person who may demand its Respondent Sarmiento has not questioned the genuineness and due
fulfillment, provided he communicated his acceptance to the obligor before its execution thereof. No further proof is necessary to show that he undertook to
revocation. An incidental benefit or interest, which another person gains, is pay P2,500,000, plus interest, to petitioner bank on or before March 6, 1978.
not sufficient. The contracting parties must have clearly and deliberately This he failed to do, as testified to by petitioner's accountant. The latter
conferred a favor upon a third person. 23 presented before the trial court private respondent's statement of
account 27 as of September 30, 1986, showing an outstanding balance of
Florentino vs. Encarnacion Sr. 24 enumerates the requisites for such contract: P4,689,413.63 after deducting P1,000,000.00 paid seven months earlier.
(1) the stipulation in favor of a third person must be a part of the contract, Furthermore, such partial payment is equivalent to an express
and not the contract itself; (2) the favorable stipulation should not be acknowledgment of his obligation. Private respondent can no longer
conditioned or compensated by any kind of obligation; and (3) neither of the backtrack and deny his liability to petitioner bank. "A person cannot accept
contracting parties bears the legal representation or authorization of the third and reject the same instrument." 28
party. The "fairest test" in determining whether the third person's interest in a
contract is a stipulation pour autrui or merely an incidental interest is to WHEREFORE, the petition is GRANTED. The assailed Decision is SET
examine the intention of the parties as disclosed by their contract. 25 ASIDE and the Decision of RTC-Manila, Branch 48, in Civil Case No. 26465
is hereby REINSTATED.
We carefully and thoroughly perused the promissory note, but found no
stipulation at all that would even resemble a provision in consideration of a SO ORDERED.
third person. The instrument itself does not disclose the purpose of the loan
contract. It merely lays down the terms of payment and the penalties incurred Davide, Jr., Bellosillo, Vitug and Quisumbing, JJ., concur.
for failure to pay upon maturity. It is patently devoid of any indication that a
benefit or interest was thereby created in favor of a person other than the
contracting parties. In fact, in no part of the instrument is there any mention
of a third party at all. Except for his barefaced statement, no evidence was
proffered by private respondent to support his argument. Accordingly, his
contention cannot be sustained. At any rate, if indeed the loan actually
benefited a third person who undertook to repay the bank, private respondent
could have availed himself of the legal remedy of a third-party
complaint. 26That he made no effort to implead such third person proves the
hollowness of his arguments.

1051
In view of the government policy which ordained that cargo handling
operations should be limited to only one cargo handling operator-contractor
for every port (under Customs Memorandum Order 28075, later on
superseded by General Ports Regulations of the Philippine Ports Authority)
the different stevedoring and arrastre corporations operating in the Port of
Davao were integrated into a single dockhandlers corporation, known as the
Davao Dockhandlers, Inc., which was registered with the Securities and
G.R. No. 97237 August 16, 1991
Exchange Commission on July 13, 1976.

FILIPINAS PORT SERVICES, INC., petitioner,


Due to the late receipt of its permit to operate at the Port of Davao from the
vs.
Bureau of Customs, Davao Dockhandlers, Inc., which was subsequently
NATIONAL LABOR RELATIONS COMMISSION, PATERNO LIBOON,
renamed Filport, actually started its operation on February 16, 1977.
SEGUNDO AQUINO, JOVITO BULAY, DOMINGO NAVOA, DELFIN
BERMEJO, CELEDONIO MANCUBAT, ALBERTO MAHINAY, SR.,
TEODULO SILAYA, SANTOS ARGUIDO, JUANITO LABANON, As a result of the merger, Section 118, Article X of the General Guidelines on
FLORENCIO MIRANTES, LUCIO BARRERA, VICENTE GILDORE, LEON The Integration of Stevedoring/Arrastre Services (PPA Administrative Order
FUENTES, CASIMIRO MAGSAYO, FERNANDO MORIENTE, MATIAS No. 13-77) mandated Filport to draw its personnel complements from the
ORBITA, SR., FRANCISCO PARDILLO, ILDEFONSO JUMILLA AND JOSE merging operators, as follows:
CANTONJOS, respondents.
Sec. 118. Absorption of labor.Subject to the provisions of
Yap, Ocampo & Associates for petitioner. the immediate preceding section, and consistent with the
actual operational requirements of the new management, all
labor force together with its necessary personnel
Porfirio S. Daclan for private respondents.
complement, of the merging operators shall be absorbed by
the merged or integrated organization to constitute its labor
force. (Emphasis supplied)

PARAS, J.:p Thus, Filport's labor force was mostly taken from the integrating corporations,
among them the private respondents.
This is a petition for clarification with prayer for preliminary injunction filed by
Filipinas Port Services, Inc. (hereinafter referred to as Filport) seeking to On February 4,1987, private respondent Paterno Liboon and 18 others filed a
clarify two conflicting decisions rendered by this Court in cases involving complaint with the Department of Labor and Employment Regional Office in
identical or similar parties, facts and issues. Davao City, alleging that they were employees of Filport since 1955 through
1958 up to December 31, 1986 when they retired; that they were paid
The antecedent facts of the case are as follows: retirement benefits computed from February 16,1977 up to December 31,
1986 only; and that taking into consideration their continuous length of
service, they are entitled to be paid retirement benefits differentials from the

1052
time they started working with the predecessors of Filport up to the time they obligation of the merging operators who shall continue to be
were absorbed by the latter in 1977 (p. 15, Rollo). individually liable for their respective liabilities or obligations,
if any. (emphasis supplied) ...
Finding Filport a mere alter ego of the different integrating corporations, the
Labor Arbiter held Filport liable for retirement benefits due private xxx xxx xxx
respondents for services rendered prior to February 16, 1977. Said decision
was affirmed by the NLRC on appeal. The new organization's liability shall be the payment of
salaries, benefits and all other money due the employee as a
Filport filed a petition for certiorari with the Supreme Court docketed as G.R. result of his employment, starting on the date of his service
No. 85704, claiming that it is an entirely new corporation with a separate in the newly integrated organization.
juridical personality from the integrating corporations; and that Filport is not a
successor-employer, liable for the obligations of private respondents' In answer to your query, therefore, the absorption of an
previous employers, as shown clearly in the memorandum dated November employee into a newly integrated organization does not
21,1978 of PPA Assistant General Manager Maximo S. Dumlao, Jr., to wit: include the carry over of his length of service.

21 November 1978 S. DUMLAO, JR.


s/t MAXIMO

MEMORANDUM Asst. General Manager

TO: The Officer-in-Charge While G.R. No. 85704 was still pending decision by this Court, Josefino Silva,
another employee of Filport, instituted a suit against Filport and Damasticor
PMU Davao (one of the defunct stevedoring firms) claiming for retirement benefits for
services rendered prior to February 19, 1977. The labor arbiter found for
FROM: The AGM for Operations Josefino Silva and said decision was affirmed by the NLRC.

SUBJECT: Clarification of Sec. 116 of PPA Administrative Filport filed a petition for certiorari with the Supreme Court docketed as G.R.
No. 86026. On August 31, 1989, this Court, through the First Division,
Order No. 13-77 of New Organization's Liability. rendered a decision, holding that:

In reply to your telegram dated November 16, 1978, Sec. Petitioner (Filport) cannot be held liable for the payment of the retirement pay
116 of PPA Administrative Order #13-77 is hereby quoted for of private respondent (Josefino Silva) while in the employ of
clarification: DAMASTICOR ... who is held responsible for the same as the labor contract
is in personam and cannot be passed on to the petitioner." (Rollo, p. 7)
New Organization's LiabilityThe integrated cargo-handling
organization shall be absolutely free from any liability or In so ruling, the First Division relied heavily on the case of Fernando v. Angat
Labor Union (5 SCRA 248) where it was held that unless expressly assumed,

1053
labor contracts are not enforceable against a transferee of an enterprise The eventual incorporation of the arrastre/stevedoring firms
labor contracts being in personam. and their subsequent registration with the Securities and
Exchange Commission on July 13, 1975 brought to the fore
Per entry of judgment, the aforesaid decision became final and executory on the interlocking ownership of the new corporation.
November 24, 1989 (p. 87, Rollo).
xxx xxx xxx
On September 3, 1990, however, this Court, through the Second Division,
dismissed the petition in G.R. No. 85704 "for failure to sufficiently show that Subsequent amendment of its Articles of Incorporation
the questioned judgment is tainted with grave abuse of discretion." highlighted by the renaming of the Davao Dockhandlers, Inc.
to Filipinas Port Services, Inc. did not diminish the fact that
Per entry of judgment, said resolution became final and executory on the ownership and constituency of the new corporation are
December 4, 1 990 (p. 108, Rollo). basically Identical with the previous owners.

Hence, the instant petition for clarification with prayer for preliminary It is, therefore, the considered view of this Office that
injunction to enjoin the respondents from enforcing the decision in G.R. No. respondent Filport being a mere alter ego of the different
85704 until further orders of this Court. merging companies has at the very least, the obligation not
only to absorb into its employ workers of the dissolved
We see no reason to disturb the findings of fact of the public respondent, companies, but also to absorb the length of service earned
supported as they are by substantial evidence in the light of the well by the absorbed employees from their former employers.
established principle that findings of administrative agencies which have
acquired expertise because their jurisdiction is confined to specific matters xxx xxx xxx
are generally accorded not only respect but at times even finality, and that
judicial review by this Court on labor cases does not go so far as to evaluate We are in full accord with, and hereby sustain, the findings
the sufficiency of the evidence upon which the Labor Arbiter and the NLRC and conclusions of the Labor Arbiter. Under the
based their determinations but are limited to issues of jurisdiction or grave circumstances, respondent-appellant is a successor-
abuse of discretion. (National Federation of Labor Union v. Ople, 143 SCRA employer. As a successor entity, it is answerable to the lawful
129). obligations of the predecessor employers, herein integrees.
This Commission has so held under the principle of
In the case filed by private respondent Paterno Liboon et al against Filport, 'substitution' that the successor firm is liable to (sic) the
the findings of the NLRC in its November 27, 1987 decision are categorical: obligations of the predecessor employer, notwithstanding the
change in management or even personality, of the new
In resolving the issues, the Labor Arbiter concludes as follows: contracting employer." (Lakas Ng Manggagawang Filipino
(LAKAS] v. Tarlac Electrical Cooperative, Inc. et al., NLRC
Case No. RB III-1 57-75, January 28,1978, En Banc). ... The
Supreme Court earlier upheld the "Substitutionary" doctrine
in the case of Benguet Consolidated, Inc. vs. BOI

1054
Employees & Workers Union, (G.R. L-24711, April 30, 1968, and vested in the new corporation formed by the
23 SCRA 465). (pp. 35 & 37, Rollo) consolidation, or in the surviving corporation in case of
merger, without further act or deed; and the title to real
Said findings were reiterated in the case filed by Josefino Silva against estate, either by deed or otherwise, under the laws of the
Filport where the NLRC, in its decision dated January 19, 1988, further ruled Philippine Islands vested in either corporation, shall not be
that: deemed in any way impaired by reason of this Act: Provided,
however, That the rights of creditors and all liens upon the
... As We have ruled in the similar case involving herein property of either of said corporations shall be preserved
appellant, the latter is deemed a survivor entity because it unimpaired; and all debts liabilities, and duties of said
continued in an essentially unchanged manner the business corporations shall thenceforth attach to the new corporation
operators of the predecessor arrastre and port service in case of a consolidation, or to the surviving corporation in
operators, hiring substantially the same workers, including case of a merger, and be enforced against said new
herein appellee, of the integree predecessors, using corporation or surviving corporation as if said debts,
substantially the same facilities, with similar working liabilities, and duties had been incurred or contracted by it.
conditions and line of business, and employing the same
corporate control, although under a new management and As earlier stated, it was mandated that Filport shall absorb all labor force and
corporate personality. (G.R. No. 86026, p. 35, Rollo) necessary personnel complement of the merging operators, thus, clearly
indicating the intention to continue the employer-employee relationships of
Thus, granting that Filport had no contract whatsoever with the private the individual companies with its employees through Filport.
respondents regarding the services rendered by them prior to February 16,
1977, by the fact of the merger, a succession of employment rights and The alleged memorandum of the PPA Assistant General Manager
obligations had occurred between Filport and the private respondents. The exonerating Filport from any liability arising from and as a result of the
law enforced at the time of the merger was Section 3 of Act No. 2772 which merger is contrary to public policy and is violative of the workers' right to
took effect on March 6, 1918. Said law provides: security of tenure. Said memorandum was issued in response to a query of
the PMU Officer-in-Charge and was not even published nor made known to
Sec. 3. Upon the perfecting, as aforesaid, of a consolidation the workers who came to know of its existence only at the hearing before the
made in the manner herein provided, the several NLRC. (G.R. No. 86026, pp. 93-94, Rollo)
corporations parties thereto shall be deemed and taken as
one corporation, upon the terms and conditions set forth in The principle involved in the case cited by the First Division (Fernando v.
said agreement; or, upon the perfecting of a merger, the Angat Labor Union [supra]) applies only when the transferee is an entirely
corporation merged shall be deemed and taken as absorbed new corporation with a distinct personality from the integrating firms and NOT
by the other corporation and incorporated in it; and all and where the transferee was found to be merely an alter ego of the different
singular rights, privileges, and franchises of each of said merging firms, as in this case. Thus, Filport has the obligation not only to
corporations, and all property, real and personal, and all absorb the workers of the dissolved companies but also to include the length
debts due on whatever account, belonging to each of such of service earned by the absorbed employees with their former employees as
corporations, shall be taken and deemed as transferred to well. To rule otherwise would be manifestly less than fair, certainly, less than
just and equitable.

1055
Finally, to deny the private respondents the fruits of their labor corresponding for Assistance in the Liquidation of the Philippine Veterans Bank, the same
to the time they worked with their previous employers would render at naught docketed as Case No. SP-32311. Thereafter, the Philipppine Veterans Bank
the constitutional provisions on labor protection. In interpreting the protection Employees Union-N.U.B.E., herein petitioner, represented by petitioner
to labor and social justice provisions of the Constitution and the labor laws, Perfecto V. Fernandez, filed claims for accrued and unpaid employee wages
and rules and regulations implementing the constitutional mandate, the and benefits with said court in SP-32311.1
Supreme Court has always adopted the liberal approach which favors the
exercise of labor rights. (EuroLinea Phils., Inc. v. NLRC, 156 SCRA 83). After lengthy proceedings, partial payment of the sums due to the employees
were made. However, due to the piecemeal hearings on the benefits, many
WHEREFORE, the Resolution of the Second Division of this Court in G.R. remain unpaid.2
No. 85704 dated September 3, 1990 is hereby REITERATED. SO
ORDERED. On March 8, 1991, petitioners moved to disqualify the respondent judge from
hearing the above case on grounds of bias and hostility towards petitioners. 3
G.R. No. 105364* June 28, 2001
On January 2, 1992, the Congress enacted Republic Act No. 7169 providing
PHILIPPINE VETERANS BANK EMPLOYEES UNION-N.U.B.E. and for the rehabilitation of the Philippine Veterans Bank. 4
PERFECTO V. FERNANDEZ, petitioners,
vs. Thereafter, petitioners filed with the labor tribunals their residual claims for
HONORABLE BENJAMIN VEGA, Presiding Judge of Branch 39 of the benefits and for reinstatement upon reopening of the bank. 5
REGIONAL TRIAL COURT of Manila, the CENTRAL BANK OF THE
PHILIPPINES and THE LIQUIDATOR OF THE PHILIPPINE VETERANS Sometime in May 1992, the Central Bank issued a certificate of authority
BANK,respondents allowing the PVB to reopen.6

KAPUNAN, J.: Despite the legislative mandate for rehabilitation and reopening of PVB,
respondent judge continued with the liquidation proceedings of the bank.
May a liquidation court continue with liquidation proceedings of the Philippine Moreover, petitioners learned that respondents were set to order the
Veterans Bank (PVB) when Congress had mandated its rehabilitation and payment and release of employee benefits upon motion of another lawyer,
reopening? while petitioners claims have been frozen to their prejudice.

This is the sole issue raised in the instant Petition for Prohibition with Petition Hence, the instant petition.
for Preliminary Injunction and application for Ex Parte Temporary Restraining
Order. Petitioners argue that with the passage of R.A. 7169, the liquidation court
became functus officio, and no longer had the authority to continue with
The antecedent facts of the case are as follows: liquidation proceedings.

Sometime in 1985, the Central Bank of the Philippines (Central Bank, for
brevity) filed with Branch 39 of the Regional Trial Court of Manila a Petition

1056
In a Resolution, dated June 8, 1992, the Supreme Court resolved to issue a Mechanisms Therefor, And For Other Purposes", which was signed into law
Temporary Restraining Order enjoining the trial court from further proceeding by President Corazon C. Aquino on January 2, 1992 and which was
with the case. published in the Official Gazette on February 24, 1992, provides in part for
the reopening of the Philippine Veterans Bank together with all its branches
On June 22, 1992, VOP Security & Detective Agency (VOPSDA) and its 162 within the period of three (3) years from the date of the reopening of the head
security guards filed a Motion for Intervention with prayer that they be office.7 The law likewise provides for the creation of a rehabilitation
excluded from the operation of the Temporary Restraining Order issued by committee in order to facilitate the implementation of the provisions of the
the Court. They alleged that they had filed a motion before Branch 39 of the same.8
RTC of Manila, in SP-No. 32311, praying that said court order PVB to pay
their backwages and salary differentials by authority of R.A. No 6727, Wage Pursuant to said R.A. No. 7169, the Rehabilitation Committee submitted the
Orders No. NCR-01 and NCR-01-Ad and Wage Orders No. NCR-02 and proposed Rehabilitation Plan of the PVB to the Monetary Board for its
NCR-02-A; and, that said court, in an Order dated June 5, 1992, approved approval. Meanwhile, PVB filed a Motion to Terminate Liquidation of
therein movants case and directed the bank liquidator or PVB itself to pay Philippine Veterans Bank dated March 13, 1992 with the respondent judge
the backwages and differentials in accordance with the computation praying that the liquidation proceedings be immediately terminated in view of
incorporated in the order. Said intervenors likewise manifested that there was the passage of R.A. No. 7169.
an error in the computation of the monetary benefits due them.
On April 10, 1992, the Monetary Board issued Monetary Board Resolution
On August 18, 1992, petitioners, pursuant to the Resolution of this Court, No. 348 which approved the Rehabilitation Plan submitted by the
dated July 6, 1992, filed their Comment opposing the Motion for Leave to File Rehabilitaion Committee.
Intervention and for exclusion from the operation of the T.R.O. on the
grounds that the movants have no legal interest in the subject matter of the Thereafter, the Monetary Board issued a Certificate of Authority allowing PVB
pending action; that allowing intervention would only cause delay in the to reopen.
proceedings; and that the motion to exclude the movants from the T.R.O. is
without legal basis and would render moot the relief sought in the petition. On June 3, 1992, the liquidator filed A Motion for the Termination of the
Liquidation Proceedings of the Philippine Veterans Bank with the respondent
On September 3, 1992, the PVB filed a Petition-In-Intervention praying for judge.
the issuance of the writs of certiorari and prohibition under Rule 65 of the
Rules of Court in connection with the issuance by respondent judge of As stated above, the Court, in a Resolution dated June 8, 1992, issued a
several orders involving acts of liquidation of PVB even after the effectivity of temporary restraining order in the instant case restraining respondent judge
R.A. No. 7169. PVB further alleges that respondent judge clearly acted in from further proceeding with the liquidation of PVB.
excess of or without jurisdiction when he issued the questioned orders.
On August 3, 1992, the Philippine Veterans Bank opened its doors to the
We find for the petitioners. public and started regular banking operations.

Republic Act No. 7169 entitled "An Act To Rehabilitate The Philippine Clearly, the enactment of Republic Act No. 7169, as well as the subsequent
Veterans Bank Created Under Republic Act No. 3518, Providing The developments has rendered the liquidation court functus officio.

1057
Consequently, respondent judge has been stripped of the authority to issue Hence, it is clear that the legislature intended to make the law
orders involving acts of liquidation. effective immediately upon its approval. It is undisputed that R.A. No. 7169
was signed into law by President Corazon C. Aquino on January 2, 1992.
Liquidation, in corporation law, connotes a winding up or settling with Therefore, said law became effective on said date.
creditors and debtors.9 It is the winding up of a corporation so that assets are
distributed to those entitled to receive them. It is the process of reducing Assuming for the sake of argument that publication is necessary for the
assets to cash, discharging liabilities and dividing surplus or loss. effectivity of R.A. No. 7169, then it became legally effective on February 24,
1992, the date when the same was published in the Official Gazette, and not
On the opposite end of the spectrum is rehabilitation which connotes a on March 10, 1992, as erroneously claimed by respondents Central Bank
reopening or reorganization. Rehabilitation contemplates a continuance of and Liquidator.
corporate life and activities in an effort to restore and reinstate the
corporation to its former position of successful operation and solvency.10 WHEREFORE, in view of the foregoing, the instant petition is hereby GIVEN
DUE COURSE and GRANTED. Respondent Judge is hereby
It is crystal clear that the concept of liquidation is diametrically opposed or PERMANENTLY ENJOINED from further proceeding with Civil Case No. SP-
contrary to the concept of rehabilitation, such that both cannot be undertaken 32311.
at the same time. To allow the liquidation proceedings to continue would
seriously hinder the rehabilitation of the subject bank. SO ORDERED.

Anent the claim of respondents Central Bank and Liquidator of PVB that R.A. Davide, Jr., C.J., (Chairman), Puno, Pardo, and Ynares-Santiago,
No. 7169 became effective only on March 10, 1992 or fifteen (15) days after JJ., concur.
its publication in the Official Gazette; and, the contention of intervenors VOP
Security, et. al. that the effectivity of said law is conditioned on the approval
of a rehabilitation plan by the Monetary Board, among others, the Court is of
the view that both contentions are bereft of merit.
* This case was transferred to the ponente pursuant to the resolution in AM
While as a rule, laws take effect after fifteen (15) days following the No. 00-9-03-SC. Re: Creation of Special Committee on Case Backlog dated
completion of their publication in the Official Gazette or in a newspaper of February 27, 2001.
general circulation in the Philippines, the legislature has the authority to
provide for exceptions, as indicated in the clause "unless otherwise
provided."

In the case at bar, Section 10 of R.A. No. 7169 provides:

Sec. 10. Effectivity. - This Act shall take effect upon its approval.

1058
In April 1997, respondents, all Muslim Filipinos, returned to Manila from
their pilgrimage to the Holy City of Mecca, Saudi Arabia, on board a
Philippines Airlines (PAL) flight. Respondents claimed that they were unable
to retrieve their checked-in luggages. On 05 January 1998, respondents filed
a complaint with the Regional Trial Court (RTC) of Marawi City against PAL
for breach of contract resulting in damages due to negligence in the custody
of the missing luggages.

On 02 March 1998, PAL filed its answer invoking, among its defenses,
the limitations under the Warsaw Convention. On 19 June 1998, before the
case could be heard on pre-trial, PAL, claiming to have suffered serious
business losses due to the Asian economic crisis, followed by a massive
strike by its employees, filed a petition for the approval of a rehabilitation plan
and the appointment of a rehabilitation receiver before the Securities and
Exchange Commission (SEC). On 23 June 1998, the SEC issued an order
granting the prayer for an appointment of a rehabilitation receiver, and it
constituted a three-man panel to oversee PALs rehabilitation. On 25
September 1998, the SEC created a management committee conformably
with Section 6(d) of Presidential Decree (P.D.) 902, as amended, declaring
the suspension of all actions for money claims against PAL pending before
any court, tribunal, board or body. Thereupon, PAL moved for the suspension
of the proceedings before the Marawi City RTC. On 11 January 1999, the trial
court issued an order denying the motion for suspension of the proceedings
on the ground that the claim of respondents was only yet to be established.
PALs motion for reconsideration was denied by the trial court.

[G.R. No. 146698. September 24, 2002] PAL went to the Court of Appeals via a petition for certiorari. On 16 April
1999, the appellate court dismissed the petition for the failure of PAL to serve
PHILIPPINE AIRLINES, petitioner, vs. SPOUSES SADIC AND AISHA a copy of the petition on respondents. PAL moved for a reconsideration. In its
KURANGKING and SPOUSES ABDUL SAMAD T. DIANALAN resolution, dated 08 October 1999, the appellate court denied the motion but
AND MORSHIDA L. DIANALAN, respondents. added that a second motion for reconsideration before the trial court could
still be feasible inasmuch as the assailed orders of the trial court were merely
DECISION interlocutory in nature. Consonantly, PAL filed before the trial court a motion
for leave to file a second motion for reconsideration. The trial court, however,
VITUG, J.: denied leave of court to admit the second motion for reconsideration. Again,
PAL filed a motion for reconsideration which sought reconsideration of the

1059
denial of the prayed leave to file a second motion for reconsideration. In an of Republic Act No. 8799. The rules require trial courts to issue, among other
order, dated 28 December 2000, the trial court denied the motion. things, a stay order in the enforcement of all claims, whether for money or
otherwise, and whether such enforcement is by court action or otherwise,
On the thesis that there was no other plain, speedy and adequate against the corporation under rehabilitation, its guarantors and sureties not
remedy available to it, PAL went to this Court via a petition for review solidarily liable with it. Specifically, Section 6, Rule 4, of the Interim Rules of
on certiorari under Rule 45 of the Rules of Court, raising the question of - Procedure On Corporate Rehabilitation, provides:

"Whether or not the proceedings before the trial court should have been SEC. 6. Stay Order. - If the court finds the petition to be sufficient in form and
suspended after the court was informed that a rehabilitation receiver was substance, it shall, not later than five (5) days from the filing of the petition,
appointed over the petitioner by the Securities and Exchange Commission issue an Order (a) appointing a Rehabilitation Receiver and fixing his bond;
under Section 6(c) of Presidential Decree No. 902-A.[1] (b) staying enforcement of all claims, whether for money or otherwise and
whether such enforcement is by court action or otherwise, against the debtor,
In their comment to the petition, private respondents posited (a) that the its guarantors and sureties not solidarily liable with the debtor; (c) prohibiting
instant petition under Rule 45 would not lie, the assailed orders of the court the debtor from selling, encumbering, transferring, or disposing in any
a quo being merely interlocutory; (b) that PAL was already operational and manner any of its properties except in the ordinary course of business; (d)
thus claims and actions against it should no longer be suspended; (c) that prohibiting the debtor from making any payment of its liabilities outstanding
the SEC, not the RTC, should have the prerogative to determine the as at the date of filing of the petition; (e) prohibiting the debtors suppliers of
necessity of suspending the proceedings; and (d) that the only claims or goods or services from withholding supply of goods and services in the
actions that could be suspended under P.D. 902-A were those pending with ordinary course of business for as long as the debtor makes payments for
the SEC. the services and goods supplied after the issuance of the stay order; (f)
directing the payment in full of all administrative expenses incurred after the
issuance of the stay order; (g) fixing the initial hearing on the petition not
While a petition for review on certiorari under Rule 45 would ordinarily
earlier than forty-five (45) days but not later than sixty (60) days from the
be inappropriate to assail an interlocutory order, in the interest, however, of
filing thereof; (h) directing the petitioner to publish the Order in a newspaper
arresting the perpetuation of an apparent error committed below that could
of general circulation in the Philippines once a week for two (2) consecutive
only serve to unnecessarily burden the parties, the Court has resolved to
weeks; (I) directing all creditors and all interested parties (including the
ignore the technical flaw and, also, to treat the petition, there being no other
Securities and Exchange Commission) to file and serve on the debtor a
plain, speedy and adequate remedy, as a special civil action for certiorari.
verified comment on or opposition to the petition, with supporting affidavits
Not much, after all, can be gained if the Court were to refrain from now
and documents, not later than ten (10) days before the date of the initial
making a pronouncement on an issue so basic as that submitted by the
hearing and putting them on notice that their failure to do so will bar them
parties.
from participating in the proceedings; and (j) directing the creditors and
interested parties to secure from the court copies of the petition and its
On 15 December 2000, the Supreme Court, in A.M. No. 00-8-10-SC, annexes within such time as to enable themselves to file their comment on or
adopted the Interim Rules of Procedure on Corporate Rehabilitation and opposition to the petition and to prepare for the initial hearing of the petition.
directed to be transferred from the SEC to Regional Trial Courts, [2] all
petitions for rehabilitation filed by corporations, partnerships, and
The stay order is effective from the date of its issuance until the dismissal of
associations under P.D. 902-A in accordance with the amendatory provisions
the petition or the termination of the rehabilitation proceedings. [3]

1060
The interim rules must likewise be read and applied along with Section x x x (T)he reason for suspending actions for claims against the corporation
6(c) of P.D. 902-A, as so amended, directing that upon the appointment of a should not be difficult to discover. it is not really to enable the management
management committee, rehabilitation receiver, board or body pursuant to committee or the rehabilitation receiver to substitute the defendant in any
the decree, all actions for claims against the distressed corporation pending pending action against it before any court, tribunal, board or body. Obviously,
before any court, tribunal, board or body shall be suspended accordingly. the real justification is to enable the management committee or rehabilitation
Paragraph (c) of Section 6 of the law reads: receiver to effectively exercise its/his powers free from any judicial or extra-
judicial interference that might unduly hinder or prevent the rescue of the
Section 6. In order to effectively exercise such jurisdiction, the Commission debtor company. To allow such other action to continue would only add to the
shall possess the following powers: burden of the management committee or rehabilitation receiver, whose time,
effort and resources would be wasted in defending claims against the
xxx xxx xxx. corporation instead of being directed toward its restructuring and
rehabilitation.[8]
c) To appoint one or more receivers of the property, real or personal, which is
the subject of the action pending before the Commission in accordance with WHEREFORE, the petition is GRANTED. The assailed orders of the
the pertinent provisions of the Rules of Court in such other cases whenever Regional Trial Court, Branch 9, of Marawi City, are SET ASIDE. No costs.
necessary in order to preserve the rights of the parties-litigants and/or protect
the interest of the investing public and creditors: x x x Provided, finally, That SO ORDERED.
upon appointment of a management committee, the rehabilitation receiver,
board or body, pursuant to this Decree, all actions for claims against Davide, Jr., C.J., (Chairman), Ynares-Santiago, and Carpio, JJ., concur.
corporations, partnerships, or associations under management or
receivership pending before any court, tribunal, board or body shall be
suspended accordingly.

A claim is said to be a right to payment, whether or not It is reduced to


judgment, liquidated or unliquidated, fixed or contingent, matured or
unmatured, disputed or undisputed, legal or equitable, and secured or
unsecured.[4] In Finasia Investments and Finance Corporation [5] this Court
has defined the word claim, contemplated in Section 6(c) of P.D. 902-A, as
referring to debts or demands of a pecuniary nature and the assertion of a
right to have money paid as well.

Verily, the claim of private respondents against petitioner PAL is a


money claim for the missing luggages, a financial demand, that the law
requires to be suspended pending the rehabilitation proceedings. [6] In B.F.
Homes, Inc. vs. Court of Appeals,[7] the Court has ratiocinated:

1061
G.R. No. 74851 December 9, 1999

RIZAL COMMERCIAL BANKING CORPORATION, petitioner,


vs.
INTERMEDIATE APPELLATE COURT AND BF HOMES,
INC., respondents.

RESOLUTION

MELO, J.:

On September 14, 1992, the Court passed upon the case at bar and
rendered its decision, dismissing the petition of Rizal Commercial Banking
Corporation (RCBC), thereby affirming the decision of the Court of Appeals
which canceled the transfer certificate of title issued in favor of RCBC, and
reinstating that of respondent BF Homes.

This will now resolve petitioner's motion for reconsideration which, although
filed in 1992 was not deemed submitted for resolution until in late 1998. The
delay was occasioned by exchange of pleadings, the submission of
supplemental papers, withdrawal and change of lawyers, not to speak of the
case having been passed from one departing to another retiring justice. It
was not until May 3, 1999, when the case was re-raffled to herein ponente,
but the record was given to him only sometime in the late October 1999.

By way of review, the pertinent facts as stated in our decision are reproduced
herein, to wit:

1062
On September 28, 1984, BF Homes filed a "Petition for Because of the proceedings in the SEC, the sheriff withheld
Rehabilitation and for Declaration of Suspension of the delivery to RCBC of a certificate of sale covering the
Payments" (SEC Case No. 002693) with the Securities and auctioned properties.
Exchange Commission (SEC).
On February 13, 1985, the SEC in Case No. 002693
One of the creditors listed in its inventory of creditors and belatedly issued a writ of preliminary injunction stopping the
liabilities was RCBC. auction sale which had been conducted by the sheriff two
weeks earlier.
On October 26, 1984, RCBC requested the Provincial Sheriff
of Rizal to extra-judicially foreclose its real estate mortgage On March 13, 1985, despite SEC Case No. 002693, RCBC
on some properties of BF Homes. A notice of extra-judicial filed with the Regional Trial Court, Br. 140, Rizal (CC 10042)
foreclosure sale was issued by the Sheriff on October 29, an action for mandamus against the provincial sheriff of Rizal
1984, scheduled on November 29, 1984, copies furnished and his deputy to compel them to execute in its favor a
both BF Homes (mortgagor) and RCBC (mortgagee). certificate of sale of the auctioned properties.

On motion of BF Homes, the SEC issued on November 28, In answer, the sheriffs alleged that they proceeded with the
1984 in SEC Case No. 002693 a temporary restraining order auction sale on January 29, 1985 because no writ of
(TRO), effective for 20 days, enjoining RCBC and the sheriff preliminary injunction had been issued by SEC as of that
from proceeding with the public auction sale. The sale was date, but they informed the SEC that they would suspend the
rescheduled to January 29, 1985. issuance of a certificate of sale to RCBC.

On January 25, 1985, the SEC ordered the issuance of a On March 18, 1985, the SEC appointed a Management
writ of preliminary injunction upon petitioner's filing of a bond. Committee for BF Homes.
However, petitioner did not file a bond until January 29,
1985, the very day of the auction sale, so no writ of On RCBC's motion in the mandamus case, the trial court
preliminary injunction was issued by the SEC. Presumably, issued on May 8, 1985 a judgment on the pleadings, the
unaware of the filing of the bond, the sheriffs proceeded with dispositive portion of which states:
the public auction sale on January 29, 1985, in which RCBC
was the highest bidder for the properties auctioned. WHEREFORE, petitioner's Motion for
Judgment on the pleadings is granted and
On February 5, 1985, BF Homes filed in the SEC a judgment is hereby rendered ordering
consolidated motion to annul the auction sale and to cite respondents to execute and deliver to
RCBC and the sheriff for contempt. RCBC opposed the petitioner the Certificate of the Auction Sale
motion of January 29, 1985, involving the properties
sold therein, more particularly those

1063
described in Annex "C" of their Answer." (p. Banking Corporation, of the owner's copies
87, Rollo.) of the new land titles replacing them until the
matter shall have been resolved by the
On June 4, 1985, B.F. Homes filed an original complaint with Securities and Exchange Commission in
the IAC pursuant to Section 9 of B.P. 129 praying for the SEC Case No. 002693.
annulment of the judgment, premised on the following:
(p. 257-26
. . .: (1) even before RCBC asked the sheriff also pp.
to extra-judicially foreclose its mortgage on 213 SCR
petitioner's properties, the SEC had already [1992]; Emp
assumed exclusive jurisdiction over those the original.
assets, and (2) that there was extrinsic fraud
in procuring the judgment because the On June 18, 1986, RCBC appealed the decision of the then Intermediate
petitioner was not impleaded as a party in Appellate Court (now, back to its old revered name, the Court of Appeals) to
the mandamus case, respondent court did this Court, arguing that:
not acquire jurisdiction over it, and it was
deprived of its right to be heard. (CA 1. Petitioner did not commit extrinsic fraud in
Decision, p. 88, Rollo). excluding private respondent as party
defendant in Special Civil Case No. 10042
On April 8, 1986, the IAC rendered a decision, setting aside as private respondent was not indispensable
the decision of the trial court, dismissing party thereto, its participation not being
the mandamus case and suspending issuance to RCBC of necessary for the full resolution of the issues
new land titles, "until the resolution of case by SEC in Case raised in said case.
No. 002693," disposing as follows:
2. SEC Case No. 2693 cannot be invoked to
WHEREFORE, the judgment dated May 8, suspend Special Civil Case No. 10042, and
1985 in Civil Case No. 10042 is hereby for that matter, the extra-judicial foreclosure
annulled and set aside and the case is of the real estate mortgage in petitioner's
hereby dismissed. In view of the admission favor, as these do not constitute actions
of respondent Rizal Commercial Banking against private respondent contemplated
Corporation that the sheriff's certificate of under Section 6(c) of Presidential Decree
sale has been registered on BF Homes' No. 902-A.
TCT's . . . (here the TCTs were enumerated)
the Register of Deeds for Pasay City is 3. Even assuming arguendo that the extra-
hereby ordered to suspend the issuance to judicial sale constitute an action that may be
the mortgagee-purchaser, Rizal Commercial suspended under Section 6(c) of

1064
Presidential Decree No. 902-A, the basis for SO ORDERED.
the suspension thereof did not exist so as to
adversely affect the validity and regularity (p. 143, Rol
thereof.
By virtue of the aforesaid order, the Register of Deeds of Pasay City effected
4. The Regional Trial court had jurisdiction to the transfer of title over subject pieces of property to petitioner RCBC, and
take cognizable of Special Civil Case No. the issuance of new titles in its name. Thereafter, RCBC presented a motion
10042. for the dismissal of the petition, theorizing that the issuance of said new
transfer certificates of title in its name rendered the petition moot and
5. The Regional Trial court had jurisdiction academic.
over Special Civil Case No. 10042.
In the decision sought to be reconsidered, a greatly divided Court (Justices
(p. 5, Gutierrez, Nocon, and Melo concurred with the ponente, Justice Medialdea;
Chief Justice Narvasa, Justices Bidin, Regalado, and Bellosillo concurred
On November 12, 1986, the Court gave due course to the petition. During the only in the result; while Justice Feliciano dissented and was joined by Justice
pendency of the case, RCBC brought to the attention of the Court an order Padilla, then Justice, now Chief Justice Davide, and Justice Romero;
issued by the SEC on October 16, 1986 in Case No. 002693, denying the Justices Grio-Aquino and Campos took no part) denied petitioner's motion
consolidated Motion to Annul the Auction Sale and to cite RCBC and the to dismiss, finding basis for nullifying and setting aside the TCTs in the name
Sheriff for Contempt, and ruling as follows: of RCBC. Ruling on the merits, the Court upheld the decision of the
Intermediate Appellate Court which dismissed the mandamus case filed by
WHEREFORE, the petitioner's RCBC and suspended the issuance of new titles to RCBC. Setting aside
"Consolidated Motion to Cite Sheriff and RCBC's acquisition of title and nullifying the TCTs issued to it, the Court held
Rizal Commercial Banking Corporation for that:
Contempt and to Annul Proceedings and
Sale," dated February 5, 1985, should be as . . . whenever a distressed corporation asks
is, hereby DENIED. the SEC for rehabilitation and suspension of
payments, preferred creditors may no longer
While we cannot direct the Register of assert such preference, but . . . stand on
Deeds to allow the consolidation of the titles equal footing with other creditors.
subject of the Omnibus Motion dated Foreclosure shall be disallowed so as not to
September 18, 1986 filed by the Rizal prejudice other creditors, or cause
Commercial Banking Corporation, and discrimination among them. If foreclosure is
therefore, denies said Motion, neither can undertaken despite the fact that a petition,
this Commission restrain the said bank and for rehabilitation has been filed, the
the Register of Deeds from effecting the said certificate of sale shall not be delivered
consolidation. pending rehabilitation. Likewise, if this has

1065
also been done, no transfer of title shall be The restraining order and the writ of preliminary injunction
effected also, within the period of issued by the Securities and Exchange Commission
rehabilitation. The rationale behind PD 902- enjoining the foreclosure sale of the properties of respondent
A, as amended to effect a feasible and BF Homes were issued without or in excess of its jurisdiction
viable rehabilitation. This cannot be because it was violative of the clear provision of Presidential
achieved if one creditor is preferred over the Decree No. 902-A, and are therefore null and void; and
others.
Petitioner, being a mortgage creditor, is entitled to rely solely
In this connection, the prohibition against on its security and to refrain from joining the unsecured
foreclosure attaches as soon as a petition creditors in SEC Case No. 002693, the petition for
for rehabilitation is filed. Were it otherwise, rehabilitation filed by private respondent.
what is to prevent the petitioner from
delaying the creation of a Management We find the motion for reconsideration meritorious.
Committee and in the meantime dissipate all
its assets. The sooner the SEC takes over The issue of whether or not preferred creditors of distressed corporations
and imposes a freeze on all the assets, the stand on equal footing with all other creditors gains relevance and materiality
better for all concerned. only upon the appointment of a management committee, rehabilitation
receiver, board, or body. Insofar as petitioner RCBC is concerned, the
(pp. 265-266,
provisions of Presidential Decree No. 902-A are not yet applicable and it may
also p. still
838, 213
be allowed to assert its preferred status because it foreclosed on the
SCRA 830 [1992].)prior to the appointment of the management committee on March
mortgage
18, 1985. The Court, therefore, grants the motion for reconsideration on this
Then Justice Feliciano (joined by three other Justices), dissented and voted score.
to grant the petition. He opined that the SEC acted prematurely and without
jurisdiction or legal authority in enjoining RCBC and the sheriff from The law on the matter, Paragraph (c), Section 6 of Presidential Decree 902-
proceeding with the public auction sale. The dissent maintain that Section 6 A, provides:
(c) of Presidential Decree 902-A is clear and unequivocal that, claims against
the corporations, partnerships, or associations shall be suspended only upon Sec. 6. In order to effectively exercise such jurisdiction, the
the appointment of a management committee, rehabilitation receiver, board Commission shall posses the following powers:
or body. Thus, in the case under consideration, only upon the appointment of
the Management Committee for BF Homes on March 18, 1985, should the
c) To appoint one or more receivers of the property, real and
suspension of actions for claims against BF Homes have taken effect and not
personal, which is the subject of the action pending before
earlier.
the Commission in accordance with the pertinent provisions
of the Rules of Court in such other cases whenever
In support of its motion for reconsideration, RCBC contends: necessary to preserve the rights of the parties litigants to
and/or protect the interest of the investing public and

1066
creditors; Provided, however, that the Commission may, in Employees Association vs. Ople, 138 SCRA 273 [1985];
appropriate cases, appoint a rehabilitation receiver of Luzon Surety Co., Inc. vs. De Garcia, 30 SCRA 111 [1969];
corporations, partnerships or other associations not Quijano vs. Development Bank of the Philippines, 35 SCRA
supervised or regulated by other government agencies who 270 [1970]).
shall have, in addition to the powers of a regular receiver
under the provisions of the Rules of Court, such functions Only when the law is ambiguous or of doubtful meaning may the court
and powers as are provided for in the succeeding paragraph interpret or construe its true intent. Ambiguity is a condition of admitting two
(d) hereof: Provided, finally, That upon appointment of a or more meanings, of being understood in more than one way, or of referring
management committee rehabilitation receiver, board or to two or more things at the same time. A statute is ambiguous if it is
body, pursuant to this Decree, all actions for claims against admissible of two or more possible meanings, in which case, the Court is
corporations, partnerships or associations under called upon to exercise one of its judicial functions, which is to interpret the
management or receivership, pending before any court, law according to its true intent.
tribunal, board or body shall be suspended accordingly. (As
amended by PDs No. 1673, 1758 and by PD No. 1799. Furthermore, as relevantly pointed out in the dissenting opinion, a petition for
Emphasis supplied.) rehabilitation does nor always result in the appointment of a receiver or the
creation of a management committee. The SEC has to initially determine
It is thus adequately clear that suspension of claims against a corporation whether such appointment is appropriate and necessary under the
under rehabilitation is counted or figured up only upon the appointment of a circumstances. Under Paragraph (d), Section 6 of Presidential Decree No.
management committee or a rehabilitation receiver. The holding that 902-A, certain situations must be shown to exist before a management
suspension of actions for claims against a corporation under rehabilitation committee may be created or appointed, such as;
takes effect as soon as the application or a petition for rehabilitation is filed
with the SEC may, to some, be more logical and wise but unfortunately, 1. when there is imminent danger of
such is incongruent with the clear language of the law. To insist on such dissipation, loss, wastage or destruction of
ruling, no matter how practical and noble, would be to encroach upon assets or other properties; or
legislative prerogative to define the wisdom of the law plainly judicial
legislation.
2. when there is paralization of business
operations of such corporations or entities
It bears stressing that the first and fundamental duty of the Court is to apply which may be prejudicial to the interest of
the law. When the law is clear and free from any doubt or ambiguity, there is minority stockholders, parties-litigants or to
no room for construction or interpretation. As has been our consistent ruling, the general public.
where the law speaks in clear and categorical language, there is no occasion
for interpretation; there is only room for application (Cebu Portland Cement
On the other hand, receivers may be appointed whenever:
Co. vs. Municipality of Naga, 24 SCRA-708 [1968]).

1. necessary in order to preserve the rights


Where the law is clear and unambiguous, it must be taken to
of the parties-litigants; and/or
mean exactly what it says and the court has no choice but to
see to it that its mandate is obeyed (Chartered Bank

1067
2. protect the interest of the investing public of September 14, 1992, if not clarified, might mislead the Bench and the Bar,
and creditors. (Section 6 (c), P.D. 902-A.) the Court resolved to discuss further.

These situations are rather serious in nature, requiring the appointment of a It may be recalled that in the herein en banc majority opinion (pp. 256-
management committee or a receiver to preserve the existing assets and 275, Rollo, also published as RCBC vs. IAC, 213 SCRA 830 [1992]), we held
property of the corporation in order to protect the interests of its investors and that:
creditors. Thus, in such situations, suspension of actions for claims against a
corporation as provided in Paragraph (c) of Section 6, of Presidential Decree . . . whenever a distressed corporation asks the SEC for
No. 902-A is necessary, and here we borrow the words of the late Justice rehabilitation and suspension of payments, preferred
Medialdea, "so as not to render the SEC management Committee irrelevant creditors may no longer assert such preference,
and inutile and to give it unhampered "rescue efforts" over the distressed but . . . stand on equal footing with other creditors.
firm" (Rollo, p. 265). Foreclosure shall be disallowed so as not to prejudice other
creditors, or cause discrimination among them. If foreclosure
Otherwise, when such circumstances are not obtaining or when the SEC is undertaken despite the fact that a petition for rehabilitation
finds no such imminent danger of losing the corporate assets, a management has been filed, the certificate of sale shall not be delivered
committee or rehabilitation receiver need not be appointed and suspension of pending rehabilitation. Likewise, if this has also, been done,
actions for claims may not be ordered by the SEC. When the SEC does not no transfer of title shall be effected also, within the period of
deem it necessary to appoint a receiver or to create a management rehabilitation. The rationale behind PD 902-A, as amended,
committee, it may be assumed, that there are sufficient assets to sustain the is to effect a feasible and viable rehabilitation. This cannot
rehabilitation plan and, that the creditors and investors are amply protected. be achieved if one creditor is preferred over the others.

Petitioner additionally argues in its motion for reconsideration that, being a In this connection, the prohibition against foreclosure
mortgage creditor, it is entitled to rely on its security and that it need not join attaches as soon as a petition for rehabilitation is filed. Were
the unsecured creditors in filing their claims before the SEC appointed it otherwise, what is to prevent the petitioner from delaying
receiver. To support its position, petitioner cites the Court's ruling in the case the creation of a Management Committee and in the
of Philippine Commercial International Bank vs. Court of Appeals, (172 meantime dissipate all its assets. The sooner the SEC takes
SCRA 436 [1989]) that an order of suspension of payments as well as over and imposes a freeze on all the assets, the better for all
actions for claims applies only to claims of unsecured creditors and cannot concerned.
extend to creditors holding a mortgage, pledge, or any lien on the property.
p. 265-266
Ordinarily, the Court would refrain from discussing additional matters such as 838, 213 S
that presented in RCBC's second ground, and would rather limit itself only to Emphasis s
the relevant issues by which the controversy may be settled with finality.
The foregoing majority opinion relied upon BF Homes, Inc. vs. Court of
In view, however, of the significance of such issue, and the conflicting Appeals (190 SCRA 262 [1990] per Cruz, J.: First Division) where it held
decisions of this Court on the matter, coupled with the fact that our decision that "when a corporation threatened by bankruptcy is taken over by a

1068
receiver, all the creditors should stand on an equal footing. Not anyone of creditors may no longer assert such preference, but shall stand on equal
them should be given preference by paying one or some of them ahead of footing with other creditors . . ." (pp. 227-228).
the others. This is precisely the reason for the suspension of all pending
claims against the corporation under receivership. Instead of creditors vexing It may be stressed, however, that of all the cases cited by Justice Bellosillo in
the courts with suits against the distressed firm, they are directed to file their BPI, which abandoned the Court's ruling in PCIB, only the present case
claims with the receiver who is a duly appointed officer of the SEC (pp. 269- satisfies the constitutional requirement that "no doctrine or principle of law
270; emphasis in the original). This ruling is a reiteration of Alemar's Sibal & laid down by the court in a decision rendered en banc or in division may be
Sons, Inc. vs. Hon. Jesus M. Elbinias (pp. 99-100; 186 SCRA 94 [1991] modified or reversed except by the court sitting en banc" (Sec 4, Article VIII,
per Fernan, C.J.: Third Division). 1987 Constitution). The rest were division decisions.

Taking the lead from Alemar's Sibal & Sons, the Court also applied this same It behooves the Court, therefore, to settle the issue in this present resolution
ruling in Araneta vs. Court of Appeals (211 SCRA 390 [1992] per Nocon, once and for all, and for the guidance of the Bench and the Bar, the following
J.: Second Division). rules of thumb shall are laid down:

All the foregoing cases departed from the ruling of the Court in the much 1. All claims against corporations, partnerships, or associations that are
earlier case of PCIB vs. Court of Appeals(172 SCRA 436 [1989] per pending before any court, tribunal, or board, without distinction as to whether
Medialdea, J.: First Division) where the Court categorically ruled that: or not a creditor is secured or unsecured, shall be suspended effective upon
the appointment of a management committee, rehabilitation receiver, board,
SEC's order for suspension of payments of Philfinance as or body in accordance which the provisions of Presidential Decree No. 902-
well as for all actions of claims against Philfinance could only A.
be applied to claims of unsecured creditors. Such order can
not extend to creditors holding a mortgage, pledge or any 2. Secured creditors retain their preference over unsecured creditors, but
lien on the property unless they give up the property, security enforcement of such preference is equally suspended upon the appointment
or lien in favor of all the creditors of Philfinance . . . of a management committee, rehabilitation receiver, board, or body. In the
event that the assets of the corporation, partnership, or association are finally
(p. 440. liquidated, however, secured and preferred credits under the applicable
Emphasis provisions of the Civil Code will definitely have preference over unsecured
supplied) ones.

Thus, in BPI vs. Court of Appeals (229 SCRA 223 [1994] per Bellosilio, J.: In other words, once a management committee, rehabilitation receiver, board
First Division) the Court explicitly stared that ". . . the doctrine in the PCIB or body is appointed pursuant to P.D. 902-A, all actions for claims against a
Case has since been abrogated. In Alemar's Sibal & Sons v. Elbinias, BF distressed corporation pending before any court, tribunal, board or body shall
Homes, Inc. v. Court of Appeals, Araneta v. Court of Appeals and RCBC v. be suspended accordingly.
Court of Appeals, we already ruled that whenever a distressed corporation
asks SEC for rehabilitation and suspension of payments, preferred This suspension shall not prejudice or render ineffective the status of a
secured creditor as compared totally unsecured creditor P.D. 902-A does not

1069
state anything to this effect. What it merely provides is that all actions for SO ORDERED.
claims against the corporation, partnership or association shall be
suspended. This should give the receiver a chance to rehabilitate the Davide, Jr., C.J., Bellosillo, Puno, Vitug, Kapunan, Mendoza, Quisumbing,
corporation if there should still be a possibility of doing so. (This will be in Purisima, Pardo, Buena, Gonzaga-Reyes, Ynares-Santiago and De Leon,
consonance with Alemar's BF Homes, Araneta, and RCBC insofar as Jr., JJ., concur.
enforcing liens by preferred creditors are concerned.)
Panganiban, J., please see separate (concuring) opinion.
However, in the event that rehabilitation is no longer feasible and claims
against the distressed corporation would eventually have to be settled, the
secured creditors shall enjoy preference over the unsecured creditors (still
maintaining PCIB ruling), subject only to the provisions of the Civil Code on
Concurrence and Preferences of Credit (our ruling in State Investment
House, Inc. vs. Court of Appeals, 277 SCRA 209 [1997]).

The Majority ruling in our 1992 decision that preferred creditors of distressed
corporations shall, in a way, stand an equal footing with all other creditors,
must be read and understood in the light of the foregoing rulings. All claims of
both a secured or unsecured creditors, without distinction on this score, are
suspended once a management committee is appointed. Secured creditors,
in the meantime, shall not be allowed to assert such preference before the
Securities and Exchange Commission. It may be stressed, however, that this
shall only take effect upon the appointment of a management committee,
rehabilitation receiver, board, or body, as opined in the dissent.

In fine, the Court grants the motion for reconsideration for the cogent reason
that suspension of actions for claims commences only from the time a
management committee or receiver is appointed by the SEC. Petitioner
RCBC, therefore, could have rightfully, as it did, move for the extrajudicial
foreclosure of its mortgage on October 26, 1984 because a management
committee was not appointed by the SEC until March 18, 1985.

WHEREFORE, petitioner's motion for reconsideration is hereby GRANTED.


The decision, dated September 14, 1992 is vacated, the decision of
Intermediate Appellate Court in AC-G.R. No. SP-06313 REVERSED and
SET ASIDE, and the judgment of the Regional Trial Court National Capital
Judicial Region, Branch 140, in Civil Case No. 10042 REINSTATED.

1070
This is an appeal from the "Resolution" of the Court of First Instance of
Manila. dated April 3, 1968, Hon. Francisco Geronimo presiding, granting the
petition of the Republic of the Philippines to (1) the for quo warranto it filed
against the respondent Bisaya Land Transportation Company, Inc., and its
Board of consisting of Miguel Cuenco, Manuel Cuenco, Jose P. Velez, Jesus
P. Velez and Federico A. Reyes, and later including the now member thereof,
Antonio V. Cuenco, Carmen Cuenco, Dioscoro B. Lazaro and Manuel V.
Cuenco, Jr., and (2) the cross-claim filed by respondent Miguel Cuenco
against his co-respondent.

The Bisaya Land Transportation Company is a corporation organized on or


G.R. No. L-31490 January 6, 1978 about June 10, 1935 under Act No. 1459, otherwise known as the
Corporation Law, for the principal purpose of engaging in the business of
REPUBLIC OF THE PHILIPPINES, petitioner-appellee, land and water transportation, having its domicile and principal place of
vs. business in Cebu City.
BISAYA LAND TRANSPORTATION CO., INC., MIGUEL CUENCO,
MANUEL CUENCO, LOURDES CUENCO, JOSE P. VELEZ, JESUS P. The instant case came into being on March 21, 1959, when the Republic of
VELEZ and FEDERICO A. REYES (Original Respondents); and ANTONIO the Philippines, through the then Solicitor General Edilberto Barot, filed a
V. CUENCO, CARMEN CUENCO, DIOSCORO B. LAZARO and MANUEL petition for quo warranto in the Court of First Instance of Manila, docketed as
V. CUENCO, JR. (New Directors of respondent corporation), respondent- Special Civil Case No. 39766, for the dissolution of the Bisaya Land
appellees. MIGUEL CUENCO, respondent-crossclaimant-appellant. Transportation Company.

Vicente J. Francisco for respondent-crossclaimant-appellant. The petition alleges that respondent corporation, through its co-respondents
named therein, acting in their offended as officers and controlling stockholder
Norberto J. Quisumbing & Humberto V. Quisumbing for respondents- of the corporation, by conspiring and confabulating together and with the aid
appellees. offended their associates, agents and confederates, had violated and
continues to violate, offended and continues to offend the proceeding of the
Solicitor General Felix Q. Antonio, Acting Assistant Solicitor General Corporation Law and other statutes of the Philippines by having committed
Dominador L. Quiroz and Solicitor Rosario A. de Leon for petitioner- and continuing to commit acts amounting to a forfeiture of the present
appellees. corporation's franchise, rights and private and, through venous means,
misused and continues to and continues to abuse, the terms of its franchise,
palpably in contravention of the law and public policy.

CASTRO, C. J.: The acts allegedly committed by the corporation, through as corespondent,
are embodied in nine causes of action which, in substance, are as follows:

1071
FIRST CAUSE OF ACTION In violation of its charter and articles of incorporation, as well as applicable
statutes concerning its operation, it engaged in mining by organizing the Jose
To conceal its illegal transaction, respondent corporation falsely reconstituted P. Velez Coal Mines, and allowing said corporation to use the facilities and
its articles of incorporation in July 1948 by adding new cattle ranch, assets of respondent corporation;
agriculture, and general merchandise;
SEVENTH CAUSE OF ACTION
SECOND CAUSE OF ACTION
It imported and sold at black market prices to third persons truck spare Parts,
On May 25, 1948, respondent corporation through its Board of Directors, the of which were appropriated by respondent directors;
adopted a resolution authorizing it to acquire 1,024 hectares of public land in
Zamboanga and 10,000 hectares of timber concession in Mindanao in EIGHTH CAUSE OF ACTION
violation of Section 6, Act No. 143);
It paid its laborers and employees wages below the minimum wage law to
THIRD CAUSE OF ACTION the great prejudice of its labor force, and in violation of the laws of the state,
manipulating its books and records so as to make it appear that its laborers
In May, 1949, respondent office constituting themselves as Board of and employees were and have been paid their salaries and wages in
Directors of respondent corporation, passed a resolution authorizing the accordance with the minimum wage law;
corporation to lease a pasture land of 2,000 hectares of cattle ranch on a
public land in Bayawan, Negros Occidental; NINTH CAUSE OF ACTION

FOURTH CAUSE OF ACTION It deliberately failed to maintain accurate and faithful stock and transfer
books since 1945 up to the filing of the petition, enabling it to defraud the
From August 1946 to the end of 1952, respondent corporation operated a state, mislead the general public, its creditors, investors and its stockholders
general merchandise store, a business which is neither for, nor incidental to, by not accurately and faithfully making
the accomplishment of its principal business for which it was organized, i.e.,
the operation of land and water transportation; a. an adequate, accurate and complete record of dividend distribution, and

FIFTH CAUSE OF ACTION b. an adequate, accurate and complete record of transfers of its stocks.

Respondent corporation snowed Mariano Cuenco and Manuel Cuenco to act The petition for quo warranto prayed that during the pendency of the action,
as president in 1945 to 1948 and 1953 to 1954, respectively, when at that a receiver be appointed by the court for the purpose of preserving the assets
time, neither of them owned a single stock; of the respondent corporation pursuant to section I (a) of Rule 61 of the
Rules of Court.
SIXTH CAUSE OF ACTION

1072
Under date of April 17, 1959, respondents (except Miguel Cuenco) filed a special and affirmative defenses, and praying that judgment be rendered
motion to dismiss the petition for quo warranto on the grounds of lack of dismissing the petition. Also, on October 19, 1959, respondents (except
cause of action, prescription, and the failure of the Solicitor General to the Miguel Cuenco) filed their answer to the cross-claim.
court's permission as required in section 4 of Rule 66 of the Rules of Court.
The motion to dismiss was denied on June 27, 1959. On January 29, 1960, respondents (except Miguel Cuenco) filed a petition for
certiorari with the Supreme Court (G.R. No. L-16593, Bisaya Land
On April 25, 1959, respondent Miguel Cuenco filed his answer admitting Transportation Company vs. Judge Bonifacio Isip), questioning the, orders of
certain allegations and denying others, especially participation in the acts the lower court dated June 27, 1959 (denying respondents' [except Miguel
imputed to respondent corporation which were made the basis of the quo Cuenco] motion to dismiss quo warranto and September 19, 1959 (deferring
warranto proceedings. He alleged several other causes, namely: (1) that resolution on the motion to dismiss Miguel Cuenco's cross-claim). However,
stock dividends were issued to respondents Manuel Cuenco and Jose P. on February 11, 1960, the Supreme Court dismiss the petition on the ground
Velez, without giving the same benefit to other stockholders; (2) that, thru that the orders complained of were interlocutory.
respondent directors, Bisaya had destroyed company payrolls, books of
accounts and other records; (3) that Bisaya Land had allowed Manuel On February 23, 1960, the lower court appointed the clerk of court as
Cuenco to use corporate funds, facilities and stocks for his personal benefit; commissioner to receive the evidence of the parties.
and (4) that, to cover up these irregularities, said directors had manipulated
the corporate books and accounts. Moreover, he set up a cross-claim against Thereafter, on February 28, 1962, respondent corporation filed a motion for
respondents-directors to recover from them, for the benefit of Bisaya, under judgment on consent, manifesting its consent to and moving for judgment to
several causes of action, the aggregate sum of P4,336,701.19. Respondent be rendered ordering the dissolution of respondent Bisaya Land
Miguel Cuenco also prayed for the appointment of a receiver without bond for transportation Company, inc. and, in furtherance of that dissolution, ordering
the purpose of preserving the assets of respondent corporation. its board of directors to proceed to the liquidation of its assets in accordance
with the provisions of the corporation law. In said motion for judgment on
On May 6, 1959, respondents (except Miguel Cuenco) filed a motion to consent, respondent corporation did not admit having committed any act
dismiss Miguel Page Cuenco's cross-claim on the ground, among others, requiring its forcible dissolution, but alleged, as reason for the filing of said
that the claims subject of the cross-claim could not be pleaded by respondent motion, that the pendency of the petition of quo warranto had prejudiced the
Miguel Cuenco since they did not arise out of the transactions or occurrences corporation its business. as well as its innocent stockholders, and that its
that were the subject-matter of the original petition for quo warranto which did business interests that late relief be given to the corporation and to its
not assert any claim against respondent directors or any one of them, which thousands of stockholders; and that the majority of the board of directors and
would thus entitle respondent Miguel Cuenco to claim indemnity from the stockholders representing more than two-thirds of its capital stock had
others. indicated their election to voluntarily dissolve the corporation as the most
feasible remedy to the corporation's problems brought about by the
On September 19, 1959, the lower court deferred resolution on the cross- respondent Miguel Cuenco.
claim after trial on the merits.
The petitioner, Republic of the Philippines, filed a manifestation stating that
Under date of September 25, 1959, respondents (except Miguel Cuenco) the motion for judgment on consent being in accordance with the petition
filed their answer, denying as well as admitting certain facts and setting up for quo warranto, the matter of the implementation of the dissolution of

1073
respondent corporation be submitted to the discretion of the lower court. Six errors are imputed by appellant Miguel Cuenco to the lower court in
Respondent Miguel Cuenco, on the other hand, filed his answer agreeing to issuing its resolution of April 3, 1968, namely:
such judgment, but urging that a receiver be appointed, and that judgment be
rendered on his cross-claim. I

On May 27, 1963, respondents (except Miguel Cuenco) filed their motion to THE LOWER COURT ERRED IN NOT ORDERING ENTRY OF JUDGMENT
withdraw motion for judgment on consent on the grand that the conditions to DISSOLVING RESPONDENT-APPELLEE CORPORATION ON THE
which the motion was subject had not been accepted. Miguel Cuenco STRENGTH OF IT'S OWN MOTION CONSENTING TO AND PRAYING FOR
opposed said withdrawal and pressed for the appointment of a receiver, SUCH JUDGMENT, DESPITE APPELLANT MIGUEL CUENCO'S
which was, in turn objected to by respondent corporation. The denial of the INSISTENT PLEA FOR ACTION ON THE SAID MOTION.
motion to withdraw was questioned by respondent corporation in a petition
for prohibition filed with the Supreme Court, docketed as G.R. No. L-22097, II
which was, however, dismiss by this Court.
THE LOWER COURT ERRED IN FAILING TO CONSIDER THAT
Several incidents followed thereafter which, for purposes of the disposition of RESPONDENT-APPELLEE CORPORATION'S MOTION FOR JUDGMENT
the present case, need not be set forth herein. OF DISSOLUTION WAS A CONFESSION OF JUDGMENT OR, AT ANY
RATE, THAT THE ENTRY OF SUCH JUDGMENT HAD BEEN FORMALLY
The motion to withdraw judgment on consent was, on December 3, 1963, AGREED TO BY THE SOLICITOR GENERAL AS WELL AS APPELLANT
denied by the lower court which, however, granted receivership. From this MIGUEL CUENCO.
order, respondent corporation filed a petition for certiorari with the Court of
Appeals (CA-G.R. No. 33266-R), which issued an ex-parte writ of preliminary III
injunction enjoining the lower court from enforcing its order of December 3,
1963. On January 29, 1968, the Supreme Court in G.R. No. L-23012
THE LOWER COURT ERRED IN HOLDING THAT THE EVIDENCE SO FAR
annulment all the proceedings in the Court of Appeals in CA-G.R. No. 33266-
ADDUCED BY PETITIONER WAS INSUFFICIENT TO DISSOLVE THE
R, on the ground that the latter court had no jurisdiction over said case.
CORPORATION, AND IN NOT HOLDING INSTEAD THAT THE SAID
EVIDENCE HAS MORE THAN SUFFICIENTLY ESTABLISHED FACTS
On October 20, 1966, the then Solicitor General Barredo (now Supreme CONSTITUTING PRACTICALLY ALL THE GROUNDS FOR QUO
Court Justice) filed a motion for dismissal of the quo warranto proceedings, WARRANT AGAINST A CORPORATION AND SHOWING THAT
to which motion respondent Miguel Cuenco riled his opposition on December RESPONDENT-APPELLEE CORPORATION WAS AND IS BEING
3, 1966. On April 3, 1968, the court a quo issued a resolution granting OPERATED VIRTUALLY AS A CRIME SYNDICATE AND, HENCE, MUST
petitioner's motion for the dismissal of the action for quo warranto, and BE ORDERED DISSOLVED.
dismissing respondent Miguel Cuenco's cross-claim. Respondent and cross-
claimant Miguel Cuenco has appealed to this Court to question this
IV
resolution.

THE LOWER COURT ERRED IN HOLDING THAT THE SOLICITOR


GENERAL WAS VESTED WITH ABSOLUTE AND UNLIMITED POWER TO

1074
DISCONTINUE THE STATE'S LITIGATION AND ACCORDINGLY TO HAVE exercises a certain amount of supervision over the entry of judgment, as well
THE QUO WARRANT PETITION DISMISSED, IF AND WHEN IN HIS as equitable jurisdiction over their subsequent status (Ibid., pp. 268-269).
OPINION THIS SHOULD BE DONE, AND IN NOT HOLDING INSTEAD
THAT THE SOLICITOR GENERALS MOTION FOR DISMISSAL OF THE The records would show that there was no meeting of the minds among the
PETITION IS DEVOID OF ANY MERIT, AND INDEED MUST BE DENIED. parties hereto with respect to the motion for judgment on consent filed by
appellee corporation and agreed to by petitioner appellee. Whereas, appellee
V corporation conditioned its motion in that its liquidation shall be effected by its
Board of Directors, appellant Miguel Cuenco would agree to such liquidation
THE LOWER COURT ERRED IN HOLDING THAT THE CROSS-CLAIM only if his cross-claim was first summarily adjudged and a receiver appointed
FILED BY APPELLANT MIGUEL CUENCO AGAINST HIS CO- by the court to effect said liquidation. On the other hand, the petitioner-
RESPONDENTS DOES NOT PRECLUDE THE DISMISSAL OF THE appellee would have the matter regarding the implementation of the
PETITION WITHOUT HIS CONSENT AND IN NOT HOLDING INSTEAD dissolution of appellee corporation submitted to the discretion of the lower
THAT BY REASON OF THE SAID CROSS-CLAIM, THE SOLICITOR court. And, before the parties could come to an unqualified agreement as to
GENERAL COULD NOT MOVE FOR THE DISMISSAL OF THE PETITION the judgment requested to be entered by appellee corporation, the latter
OVER APPELLANT MIGUEL CUENCO'S OBJECTION. decided to withdraw its motion for judgment on consent. Clearly therefore, in
view of the non-agreement of the parties as to the terms, and considering the
VI nature of a judgment by consent as explained above, it can not be said that
the lower court erred in not rendering judgment dissolving respondent
corporation on the basis of the motion for judgment by consent filed by
THE LOWER COURT ERRED IN DISMISSING THE CROSS-CLAIM.
appellee corporation.

Appellant argues, on his first and second assignments of error, that the lower
On the third assignment of error, appellant Miguel Cuenco would attribute
court should have rendered judgment dissolving appellee corporation on the
error to the court a quo in not holding that the evidence which petitioner had
strength of its own motion consenting to and praying for its dissolution, as
presented in the hearings established facts constituting practically all the
such Motion amounted to a confession of judgment, besides the fact that it
grounds for quo warranto against a corporation.
had been formally agreed to by the Solicitor General as well as by appellant
Miguel Cuenco. The claim is without merit. As pointed out in the brief for
Petitioner-appellee Republic of the Philippines, a motion for judgment on It will be recalled that at the before the Clerk of Court who was appointed
consent is not to be equated with a judgment by confession. The former is commissioner to remove the evidence of the petitioner, the latter had
one the provisions and terms of which are settled and a agreed upon by the presented three wit , namely, Juan P. Mata, Clemente Vasquez and Silverio
parties to the action, and which is entered in the record by the consent and Mata, and several exhibits which were Identified by said witnesses. After a
sanction of the court, Hence, there must be an unqualified agreement among very careful and deliberate consideration of the evidence adduced by
the parties to be bound by the judgment on consent before said judgment petitioner, the lower court came to the conclusion that the same did not really
may be entered. The court does not have the Power to supply terms, warrant a quo warranto by the State that could truly justify to decapitate
provisions, or essential details not previously agreed to by the parties (49 corporate life, and that the corporate acts or missions complained of had not
C.J.S. 308). On the other hand, a judgment by confession is not a Plea but resulted in substantial injury to the public, nor were they wilful and clearly
an affirmative and voluntary act of the defendant himself, Here, the court obdurate. The court found that the several acts of misuse and misapplication
of the funds and/or assets of the Bisaya Land Transportation Co., Inc. were

1075
committed new particularly by the respondent Dr. Manuel Cuenco with the After a very careful and conscientious study of the records of
cooperation of Jose P. Velez, for the commission of which they may be this case, this court is not prepared to say that sufficient
personally held liable. There appears to be no reason for us to disregard the competent evidence has been adduced to impeach the
findings of the trial court, which, applying well settled doctrines, ought to be motive of Solicitor General Barredo in filing his motion for
given due weight and credit (De la Rama vs. Ma-ao Sugar Central, L-17504 dismissal, since honesty rather than dishonesty, good faith
& L-17506, Feb. 28, 1969). Besides, the court a quo found that the rather than bad faith should always be presumed in the
controversy between the parties was more personal than anything else and absence of clear contrary evidence. The Solicitor General
did not at all affect public interest. Thus, the Court held: explained that after having been briefed on the evidence by
Solicitor Rosete before the negotiation of an amicable
A careful perusal of the above-quoted letters patently reveals settlement between the parties involved in this case started,
that rather than public interest the personal interests of both he had already in mind asking for the dismissal of the quo
Dr. Manuel Cuenco and Mr. Miguel Cuenco are principally warranto proceeding for he believed in all sincerity that the
involved in this controversy. The allegations, therefore, of evidence so far presented did not justify the dissolution of
Solicitor Barredo in his motion for dismissal of action filed on the corporation through a quo warranto proceeding. He
October 20, 1966 admitted that even after he had filed the motion for dismissal
he continued the negotiation for the settlement of the case,
xxx xxx xxx but he explained that it was because of the request of the
respondent and cross-claimant Miguel Cuenco and his wife
that he continue to use his good offices to effect an amicable
6. That in a large sense, this case involves
settlement between the parties.
personal controversies among the Cuenco's,
and their relatives, by consanguinity and
affinity, involving their respective interests as The Solicitor General himself asserts that the only purpose of his ration for
stockholders in the Bisaya Land the of this quo warranto is to take the State out of an unnecessary court
Transportation Company, Inc., more than litigation, so that the dismissal of the case would result in the disposition
anything else; solely of the quo warranto by and between petitioner Republic of the
Philippines and the respondents named therein. Other interested parties who
might feel aggrieved, therefore, would not be without their remedies since
7. That, such private controversies can be
they can still maintain whatever claims they may have against each other. It
ventilated in appropriate stockholders' suits
has been held that relief by dissolution will be awarded only where no other
which do not have to occupy the time and
adequate remedy is available, and is not available where the rights of the
attention of government officials which can
stockholders can be, or are, protected in some other way (16 Fletcher Cyc.
be better devoted to matters of more direct
Corporations, 1942 Ed., pp. 812-813, citing "Thwing vs. McDonald", 134
public interest.
Minn. 148,156 N.W. 780,158 N.W. 820, 159 N.W. 564, Ann. Cas. 1918 E
420; Mitchell vs. Bank of St. Paul, 7 Minn. 252, cited in De la Rama vs. Ma-
are well founded. ao Sugar Central, supra). The third assignment of error, therefore, should be
disregarded.

1076
The pivotal question in this case is case in the fourth assignment of error, i.e., which he commenced was brought for purposes of enforcing
whether or not the lower court erred in holding that the Solicitor General was a right or a benefit, the advisability or necessity of which he
vested with full power to manage and control the State's litigation, which later discovers no longer exists, or that the result of the
includes the power to continue such litigation, if and when in his opinion this action would be different from what he had intended, then he
should be done. should be permitted to withdraw his action, subject to the
approval of the court. The plaintiff should not be required to
Appellant would maintain the negative of the above proposition, giving three continue the action, subject to some well-defined exceptions,
main reasons therefor, namely: when it is not to his advantage so to do. Litigation should be
discouraged and not encouraged. Courts should not with
1. The evidence so far adduced was in fact sufficient to dissolve the require parties to litigate when they no longer desire so to
respondent corporation; do. Courts, in granting permission to dismiss an action, of
course, should always take into consideration the effect
which said dismissal would have upon the rights of the
2. There was a pending motion of respondents for judgment on consent, by
defendant.
virtue of which instead of dismissal of the petition, the corporation should be
considered dissolved;
... in the case of a municipality, where the agents of the
public are spending public money, we are of the opinion that
3. A cross-claim has been interposed by Miguel Cuenco which precluded the
such agent should not be required to continue an action
dismissal of the petition for quo warranto.
when (a) it clearly appears that there is no longer a necessity
therefor, or (b) when it clearly appears that to continue the
It will be noted that the first two reasons, given in support of the fourth action, the result would be prejudicial to the interests of the
assigned error, have already been discussed in connection with appellant's public. We think that this conclusion is more in harmony with
first three assignments of error, and shown to be unworthy of serious the rational conduct of public affairs than the opposite rule.
consideration. We shall dwell on the third reason in our discussion of the fifth (City of Manila vs. Ruymann, 37 Phil. 421, 424-425, 427,
assignment of error. cited in Metropolitan Water District vs. De los Angeles, 55
Phil. 776, 790.)
Meeting squarely the issue of whether or not the Solicitor General is vested
with absolute and unlimited power to discontinue the State's litigation and, American authorities likewise uphold the power and authority of the state
accordingly, to have the quo warranto petition dismissed, if and when in his attorney to control and manage all litigation in behalf of the State, which
opinion this should be done, the general rule seems to be that the plaintiff power involves the power to discontinue the same if and when, in his opinion,
may do so with the approval of the court, subject to be defined exceptions this should be done. (7 Am. Jur. 2d 18- 19).
(such as, for example, where the answer sets up a counterclaim which
cannot stand independently of the main action).
Thus, in State vs. Finch, 280 Pac. Rep. 910, 912, 915, the Attorney General
moved to dismiss a liquor prosecution brought against one of the defendants
The right of the plaintiff to dismiss an action with the consent by the county attorney, which motion was overruled. On appeal, the Kansas
of the court is universally recognized with certain well- Supreme Court held:
defined exceptions. If the plaintiff discovers that the action

1077
And, as a rule, the attorney-general has power, both under xxx xxx xxx
the common law and by statute, to make any disposition of
the state's litigation that the deems for its best interest; for ... The power effectively to control a prosecution involves the
instance, he may abandon, discontinue, dismiss, or power to discontinue if, and when, in the opinion of the
compromise it. But he cannot enter into any agreement with prosecutor in charge, this should be done. We are of the
respect to the conduct of litigation which will bind his opinion the trial court should have sustained the Attorney-
successor in office, nor can he empower any other person to General's motion to dismiss in the instant case.
do so. ... The attorney-general may dismiss any suit or
proceeding, prosecuted solely in the public interest, In a much later case, State vs. City of Kansas, 350 P2d 37, the authority of
regardless of the relator's wishes. ... Where the attorney- the Attorney-General to dismiss any time before submission, a quo
general is empowered, either generally or specifically, to warranto proceeding commenced by the State on the relation of the county
conduct a criminal prosecution, he may do any act which the attorney questioning certain ordinances of the city of Kansas making part of
prosecuting attorney might do in the premises; that is, he the city a portion of the Fairfax industrial district, was declared by the Kansas
can do each and every thing essential to prosecute in Supreme court to be absolute, and a duty that could not be questioned,
accordance e with the law of the land, and this includes notwithstanding 18 months had passed subsequent to the commencement of
appearing in proceedings before the grand jury. So an the action and after the commissioner had fixed the date the hearing was to
attorney-general, even at common law. had the right to enter begin. Said the court:
a nolle prosequi; although he could not do so during the trial
without leave of court.
Applying the above reasoning to our present case we
conclude the attorney general by his motion to intervene and
xxx xxx xxx supersede the county attorney exercised his powers and
duties under the constitution and appropriate statutes; this
... At common law the duties of the attorney-general, as chief was as far as he could go as an executive officer and as an
law officer of the realm, were very numerous and varied. He attorney and officer of this court. Since he is an officer of the
was the chief legal adviser of the crown, and was instructed judicial branch, under the separation of powers of the three
with the management of all legal affairs and the prosecution branches of government, he was limited and restricted in his
of all suits, civil and criminal, in which the crown was conduct before this court by the code of professional ethics
interested. He alone could discontinue a criminal prosecution to the same extent any other lawyer would be. If, therefore,
by entering a nolle prosequi; therein. ... It is generally the attorney-general considered the action unmeritorious, he
acknowledged that the attorney-general is the proper party not only had the authority but he also had a duty to move for
to determine the necessity and advisability of undertaking or dismissal. ...
prosecuting actions on the part of the state. Thus it has
been' held that the discretion of the attorney-general in xxx xxx xxx
determining what the public interests require as to bringing
an action against a domestic business corporation or its
Any plaintiff has an absolute right to dismiss his action any
officers is absolute, and cannot be made the subject of
time before submission. (G.S. 1949, 603105; Bavuso vs.
inquiry by the courts.

1078
Angwin, 166 Kan. 469, 201 P. 2d 1057) and in Kinsch vs. a counterclaim which cannot remain pending for respondent adjudication by
Missouri Kansas-Texas Railroad Co., 183 Kan. 224, 326 P. the court has been pleaded by a defendant prior to the service upon him of
2d 327, when plaintiff moved to dismiss before final plaintiffs motion to dismiss. It advisedly omits reference to a cross-claim as a
submission, this court held the trial court committed factor to be considered by the court in determining whether, considering the
reversible error in overruling such motion. circumstances set forth therein, an action should be dismissed. The reason
could be that the framers of the rules realized that the policy against
xxx xxx xxx dismissal of an action behind the provision involved, while applicable to a
counterclaim, is not so with respect to a cross-claim, expense in preparing for
... As previously stated, the attorney-general, plaintiff herein, his defense, only to see the case dismissed without prejudice at the instance
had an absolute right to dismiss this action and the of plaintiff, thereby putting the defendant "to expense literally for nothing."
allegations in Quindaro's pleadings must fall because they (Francisco, Revised Rules of Court, Vol. I, p. 980, citing McCann vs. Bently
clearly show that Quindaro, one municipality, is questioning Stores Corp., 34 F. Supp. 234, 3 Fed. Rules of service 41a., Case 3 [W.D.
the organization, or reorganization, of the city, another Mo. 1940]). On the other hand, the dismissal of the main action would only
municipality, and this can be done only by the estate through benefit the cross-claim and not result in any prejudice or disadvantage to
its proper officers. him.

Well might the State vs. city of Kansas case, supra, be applied to the present Neither do the provisions of Section 4 of Rule 17 help the position taken by
case, since in our jurisdiction it is the Solicitor General Who must commence appellant Miguel Cuenco. For although this section makes the provisions of
the action involved herein (Rule 66, Sec. 3) and he, therefore, also has, Rule 17 also applicable to the dismissal of a claim, what it actually
before submission, the right to terminate of the same where he deems it best contemplates is the allowance or allowance of a motion to dismiss a cross-
for the interest of the State. (See also: Lyle vs. Luna, 338 P2d 1060, 1065) claim as an "independent" action against which a counterclaim has been
interposed. It does not intend to prohibit the dismissal of an action just
because a cross-claim which cannot be the subject to independent
In his fifth assignment of error, appellant Miguel Cuenco claims that the lower
adjudication, has been pleaded. Section 4, Rule 17, should taken in
court erred in holding that the cross-claim filed by him did not proceeding the
correlation with Section 2 of the same Rule which would have included also
dismissal of the petition without his consent, and in not holding that by
"cross-claim," instead of merely mentioning "counterclaim, as a bar to the
reason of the said cross-claim, the Solicitor General could not move for the
dismissal of an action where the same cannot remain pending for
dismissal of the petition over appellant Miguel Cuenco's objection. Citing the
independent adjudication, if it had really been its intention to do so.
proceeding of 2 and 4 of Rule 17 of the Rules of Court, appellant opines that
since his cross-claim cannot remain pending for independent adjudication, it
being allegedly interwoven with the petition for quo warranto, the lower court A cross-claim is allowed to be interposed by a party against a co-party to
should not have dismissed said petition. enable the former to recover from the latter whatever he might be made
liable to pay to the plaintiff. Considering then the nature of a cross-claim, we
fail to grasp the logic in appellant Miguel Cuenco's objection to the dismissal
The position taken by appellant Miguel Cuenco is legally untenable. The
of the main action the quo warrantoproceedings. The withdrawal or
Provision of Section 2 of Rule 17, invoked by him in support of his stand, is
dismissal of said action would effectively prevent him from suffering any
inapplicable. It governs dismissal of an action by order of the court,
prejudice exonerate him from any liability, to evade or mitigate rules grant
prohibiting such dismissal against defendants objection where
him the right to file a cross-claim dismissal of the main action would not wipe

1079
out hi cross-claim, since, if minded to do he may subsequently bring the
corresponding action based thereon (Watts vs. Watts, 15 S.W. 2d 998;
Severance vs. Heyl and Patterson, 174 A. 789).

In view of our conclusion that the court a quo committed no error in


dismissing the quo warranto proceedings, it also stands to reason that it
acted correctly in dismissing appellant Miguel Cuenco's cross- claim. A
cross-claim is proper only where the cross-claimant stands to be prejudiced
by the filing of an action against him. Hence, where such action has been
dismissed, his cross-claim would have no leg to stand on (Sm. 7, Rule 6, 4
Moore's Federal Practice Under the New Rules p. 693). This disposes of
the sixth assignment of error.

ACCORDINGLY, without prejudice to the rights of the private parties herein to


take proper steps to enforce whatever causes of action they may have
against each other, the order of the lower court embodied in its "Resolution"
dated April 3, 1968, granting the Solicitor General's motion to dismiss
the quo warranto proceedings and dismissing appellant Miguel Cuenco's
cross-claim, is hereby upheld; the receivership on the respondent
corporation's property and assets is hereby ordered terminated, effective
upon the lapse of thirty (30) days from the date of promulgation of this
decision; and the receiver is directed to render, within three (3) months from
the finality of this decision, a full and complete accounting to the Board of
Directors of the respondent corporation. No costs.

Muoz Palma, Martin, Fernandez and Guerrero, JJ., concur.

Makasiar, J., took no part.

G.R. No. L-11789 April 2, 1918

1080
THE GOVERNMENT OF THE PHILIPPINE ISLANDS, plaintiff-appellant, admitted having entered into the contract set out in paragraph III and alleged
vs. that The Tayabas Land Company was an ordinary partnership and not a
THE PHILIPPINE SUGAR ESTATES DEVELOPMENT CO. corporation as alleged in paragraph III. The allegations of paragraph IV were
(LTD.) defendant-appellant. denied generally. The defendant prayed that the complaint be dismissed with
costs.
Acting Attorney-General Paredes for plaintiff.
Gilbert, Cohn & Fisher and L. M. Southworth for defendant. On August 24, 1915, the parties filed a stipulation of facts which appears at
page 16 of bill of exceptions. By said stipulation it was agreed that the
JOHNSON, J.: defendant was a corporation duly organized; that The Tayabas Land
Company was a partnership; that a contract was entered into between the
This is an action in the nature of quo warranto. It was brought by the defendant and The Tayabas Land Company by virtue of which the defendant
Attorney-General for and on the behalf of the Government of the Philippine delivered to the said The Tayabas Land Company P304,459.42 which the
Islands for the purpose of having the charter of the defendant corporation plaintiff contended amounted to a contribution by the defendant to the capital
declared forfeited. The complaint alleged that the defendant was a of The Tayabas Land Company but which the defendant contended
corporation duly organized under the laws of the Philippine Islands; that for a amounted to a loan to said concern; that the money thus received was
period of eighteen months previous to the filing of the complaint (Nov. 21, devoted to the purchase of the real estate in the Province of Tayabas along
1914), it had continuously offended against the laws of the Philippine Islands the proposed right of way of the Manila Railroad Company in that province;
and had misused its corporate authority, franchises, and privileges and had that the purpose of these purchases was for resale to the Manila Railroad
assumed privileges and franchises not granted; that it had engaged in the Company or any other person offering an acceptable price. Attached to this
business of buying and selling real estate; that on the 31st of May, 1913, it stipulation are three exhibits which are (a) copies of the charter of the
entered into a contract with the Tayabas Land Company for the purpose of defendant; (b) the partnership agreement of The Tayabas Land Company;
engaging in the business of purchasing lands along the right of way of the and (c) the contract entered into between the defendant and The Tayabas
Manila Railroad Company through the Province of Tayabas with a view to Land Company. On October 14, 1915, it was further stipulated and agreed
reselling the same to the Manila Railroad Company at a profit. A copy of the that all of the deeds of land executed to The Tayabas Land Company, which
contract was made part of the complaint. The plaintiff alleged, that by the had been in the possession of the defendant, had been delivered to the
acts and omissions of the defendant, it had forfeited its corporate rights, Board of Public Utility Commissioners in obedience to a subpoena
privileges, powers, and franchises, dissolving it as a corporation, and to grant duces tecum.
such other and further relief as might seem just and equitable to the court,
and for costs. On February 21, 1916, the court rendered judgment ordering the defendant
to abstain in the future from engaging in the business of buying and selling
The defendant appeared on January 18, 1915, and demurred to the lands and to pay the costs of the action. Both parties excepted to this
complaint upon the ground that it failed to state a cause of action. On April judgment and moved for a new trial which was denied by the court. They
28, 1915, the court overruled said demurrer and the defendant duly appealed and filed one bill of exceptions.
excepted. On May 5, 1915, the defendant answered the complaint. The
defendant admitted the first paragraph of the complaint and denied generally The lower court reduced the issue in this case to two queries: (1) Did the
the allegations of the second and third paragraphs with the exception that it defendant engage in the business of buying and selling land or was this
transaction merely a loan to a partnership, which was engaged in the

1081
business of buying and selling land? (2) Assuming that the defendant was Second. These powers are necessarily limited by section 75 of the Act of
engaged in the business of buying and selling land, does the law require that Congress of July 1, 1902, and by the section 13 Act of 1459, the latter being
it be dissolved or is the prohibition of future acts of this nature sufficient? a reproduction of the former, which is as follows:

The decision upon the demurrer was to the effect that the contract entered That no corporation shall be authorized to conduct the business of
into between the defendant and The Tayabas Land Company constituted an buying and selling real estate or be permitted to hold or own real
agreement to enter into a partnership. After studying the proof and the estate except such as may be reasonably necessary to enable it to
evidence the court decided that the contract was not one of partnership but carry out the purposes for which it is created, . . . . Corporations,
was a contract of cuentas en participacion. The court found that the however, may loan funds upon real estate, security, and purchase
defendant had interested itself in The Tayabas Land Company to such an real estate when necessary for the collection of loans, but they shall
extent that it was in effect carrying on the business of buying and selling dispose of real estate so obtained within five years after receiving the
land. The court found that the law did not require that the charter of the title . . . .
defendant be forfeited and it further found that the Government could not be
benefited by such forfeiture. The court ordered the defendant to abstain from Third. The provisions of the contract between the defendant and The
the further prosecution of this class of business. Tayabas Land Company which are pertinent to this case are as follows:

Certain facts are important in order to make clear the argument of the First. That the Philippine Sugar Estates Development Co., (Ltd.)
appellants. which hereinafter shall be denominated "The Philippine Sugar," shall
take part in the business which The Tayabas Land Company has
First. The defendant corporation by its charter was authorized, among other established, and, for this purpose, brings in the sum of four hundred
things: thousand pesos (P400,000), Philippine currency, which it places at
the disposal of the latter concern, through the opening of a credit for
( j) To buy shares of the Compaia de Navegacion, Ferrocarriles, said amount, of which credit The Tayabas Land Company may
Diques, y Almacenes de Depositos, and, in this manner or otherwise, dispose in accordance with the requirements of the business it at
to engage in any mercantile or industrial enterprise. present conducts or in the future may conduct.

(k) With no other restrictions than those provided by law, place funds Second. That The Tayabas Land Company agrees that the Philippine
of the corporation in hypothecary or pignorative loans, in public Sugar take part in the business of the former, in the amount and
securities of the United States, in stocks or shares issued by firms, manner stated in the preceding clause.
corporations, or companies that are legally organized and operated,
and in rural and urban property. It may also contract and guarantee Third. That the allotments by The Philippine Sugar, up to the amount
all kinds of obligations, in conformity with existing laws. (Bill of of the four thousand pesos above specified, shall be the following 1.
Exceptions, p.21) (P100,000 already received. 2. (P30,000 already received). 3. Two
hundred and seventy thousand pesos (P270,000) which The
Philippine Sugar shall deliver from time to time in accordance with
the needs of the business of The Tayabas Land Company.

1082
Fourth. . . . Ninth. The Philippine Sugar shall be the treasurer and depository of
the aforementioned credit, shall make all disbursements against it as
Fifth. The Philippine Sugar shall have a share of twenty-five per cent required from time to time, and shall receive all sums derived from
(25% per cent) of the net profits derived from any and all the the sale of the lands. The deeds of purchase of the said lands shall
business of The Tayabas Land Company . The Tayabas Land be filed in its office.
Company shall keep its accounts in proper manner, and shall exhibit
to the Philippine Sugar all books, balances, and accounts such as Tenth. The proceeds from the first sales of lands and improvements
the latter company may require to be shown in order to have of the line from Pagbilao to Lopez shall be integrity set aside for the
complete proof of the correctness of any and all the proper total amortization of the capital.
settlements.
Eleventh. The Tayabas Land Company binds itself to make use the
Sixth. The general, actual, and necessary expenses which The credit in its entirety. (Bill of Exceptions, p. 41.)
Tayabas Land Company may have to defray in the transaction of its
business, shall be paid by it and by The Philippine Sugar in the same Fourth. The foregoing contract was made in pursuance of a meeting
proportion as they each share in the profits in accordance with the of the board of directors of the defendant corporation held on May
preceding clause. Exception is made of the loss referred to in the 30, 1913. The minutes, after stating the history of the relations
fourth clause of this contract, which as therein set forth, shall be between the defendant and The Tayabas Land Company, set out the
borne solely by The Tayabas Land Company. following authorization:

Seventh. All lands bought or which may be bought with the credit, In view of the foregoing facts and considerations, unanimously
which The Philippine Sugar brings to The Tayabas Land Company recognized, and for the stability of the business, the board resolved
and which lie within and without the railway line from Pagbilao to by a majority vote: 1. To grant to The Tayabas Land Company the
Lopez, shall be held as security for such credit, at their respective credit applied for, amounting to four hundred thousand (P400,000)
cost price, until their alienation, except the part thereof which pesos, the one hundred thousand (P100,000) pesos of the original
pertains to D. Mariano Lim in The Tayabas Land Company. loan being included in this sum; 2. To change the form of the
mortgage, in view of the impossibility of the borrowers to find
Eighth. When The Tayabas Land Company is to sell lands and sufficient and substantial securities wherewith to cover said new
improvements at a price lower than P0.50 per meter, it shall first credit under the new form of contract, the bases and conditions
obtain the consent of The Philippine Sugar Estates Development Co. thereof being follows:
(Ltd.) Such consent, however, shall not be necessary when the price
is more than that fixed above. CREDIT WITH SECURITY AND IN CO-PARTNERSHIP.

Credit which The Philippine Sugar . . . opens in favor of The Tayabas


Land Co. up to P400,000. . . .

1083
With security because all the lands bought with funds from said 8. The Philippine Sugar shall be the depositary-treasurer of said
credit are held as security. . . . credit, and shall deliver the sums needed from time to time and, in
exchange for the deposit in its office, of the deeds of purchase of the
And in co-partnership because the loan applied for shares in the lands.
gross profits to the extent of 25 per cent, instead of interest, as being
enormously more advantageous. . . . 9. The proceeds from the first and successive sales shall be devoted
to the total amortization of the capital . . .
CONDITIONS.
10. The Tayabas Land Company, binds itself to make use of this loan
1. Share of 25 per cent in the profits. in its entirety, or, in a contrary case, it shall be obliged to reimburse
the totality of the loss corresponding to the three hundred thousand
2. Reimbursement of all loses which The Philippine Sugar may suffer (P300,000) pesos which The Philippine Sugar bind itself to place in
in the realization of its securities. the market, . . .

3. Charge against The Tayabas Land Company for all the general The purchase price agreed upon with the manager of The Tayabas
expenses, enumerated under letter "i", and other especial expenses Land Company is at the rate of 95 percent . . . . (Record pp. 105-
of whatever kind which it may incur. This condition is inserted in view 108.)
of the disproportional share of the capital employed in the business,
and as a compensation . . . Said sum of money was turned over to The Tayabas Land Company and land
was purchased by it. It must be presumed that the relation between the
4. To revert this 25 per cent share to the present liabilities of The defendant corporation and The Tayabas Land Company was governed by the
Tayabas Land Company. . . . contract which has been quoted above.

5. Settlement of the interest, at 12 per cent of the first loan. . . . There are a number of features of this contract which should be noted. (1)
there was no period fixed in the contract for the repayment of the money,
except that the first returns from the sale of the land was to be devoted to the
6. All the lands bought or which may be bought with funds from that
payment of the capital. There was no date fixed for this payment. (2) The
credit, within and without line from Pagbilao to Lopez, shall be held
entire amount of the "credit" was not to be turned over at once but was to be
as security, except the share which Mr. Lim has in The Tayabas Land
used by The Tayabas Land Company as it was needed. (3) The return on the
Company.
capital was not by a fixed rate of interest but 25 per cent of the profits earned
by The Tayabas Land Company in "todos los negocios" was to be paid to the
7. The Philippine Sugar shall give its approval; ... to the sale of lands defendant. (4) The defendant corporation agreed to pay 25 per cent of "los
to the Railroad Company, whether there settlement or litigation, gastos generales, reales y necesarios que The Tayabas Land Company
whether such sale be made by compromise, or whether suit must be tenga que efectuar para el desenvolvimiento de los asuntos" (all general
brought. . . . expenditures true and necessary that The Tayabas Land Company must
make for the development of its business.) (Articulo sexto of the contract.) (5)

1084
The consent of the defendant was necessary when The Tayabas Land but the usual method is to pay the interest first. In the present instance after
Company desired to sell the land at a price under P0.50 per square meter sufficient land had been sold to repay the capital the remaining land unsold
but was not required if the selling price was over that amount. (6) The represented the profit between the defendant and The Tayabas Land
defendant acted as the treasurer of the enterprise. It paid out the money as it Company in the proportion of 25 to 75. The remaining land, under the
was needed for the purchase of land and received the proceeds of the sale agreement, must be sold at a profit and the result must be therefore a profit
of land as well as acting as the depositary of the deeds, covering the land. upon the profit. The remaining land was not necessary to guarantee the
repayment of the original loan because the original loan had already been
The lower court found that the contract entered into was that of "cuentas en paid. Under the contract, even though the 25 per cent of the profit were
participacion" and that the proper judgment was an order prohibiting a turned over to the defendant, if The Tayabas Land Company reinvested its
continuance of this relationship. share of the profits and continued the business, the defendant would still be
entitled to a profit on that investment. The contract provides that the
The defendant-appellant contends that the contract was within its powers; defendant shall be entitled to 25 percent upon "todos los negocios" of The
that the contract was in reality merely a loan. It is argued that the board of Tayabas Land Company.
directors did not authorize Suarez to enter into a partnership agreement or a
"cuentas en participacion" but only to negotiate a loan and 25 per cent of the After the capital is returned the land remaining, if any, is profit. It can be
profits to be paid in lieu of interest. Any contract which is not authorized by nothing else and belongs to both parties in the proportion of 25 to 75. The
the board of directors (meeting of May 30, 1913) would, it is argued, be ultra defendant at least has an equitable interest in the land itself; it has in fact
vires on the part of the officer executing it and would not bind the corporation. an equitable title to 25 percent of all the remaining land. If these lands were
A comparison of the contract actually entered into with the minutes of the to be registered, the defendant could demand that its interest be noted to be
board of directors will show that they are practically identical. register. This being so, can it be denied that the defendant is, at least
Clause decima of the "condiciones" in the minutes of the board of directors is indirectly, conducting the business of buying and selling real estate? When
explained as follows: The Philippine Sugar Estates Development Co., in an individual or a corporation becomes the owner of land by purchase he or it
order to raise part of the capital which they loaned to The Tayabas Land must be a purchaser. The defendant-appellant argues that it never secured
Company sold P300,000 worth of Japanese bonds. The loss occasioned by and title to any of the real estate purchased and therefore it could not sell it.
this sale was to be paid by The Tayabas Land Company as provided in the Does not an equitable title in the real estate come at least within the spirit of
4th section of the contract. (See pp. 14-15, Bill of Exceptions.) the prohibition?

It is difficult to understand how this contract can be considered a loan. There The lower court found that the relation between the defendant and The
was no date fixed for the return of the money and there was no fixed return to Tayabas Land Company was that of "cuentas en participacion." Whether the
be made for the use of the money. The return was dependent solely upon the relation between the defendant and The Tayabas Land Company was that of
profits of the business. It is possible for the defendant to receive a return a co-partnership or one of "cuentas en participacion" is of little importance if
from the business even after all of the "capital" has been returned. The under such relation the defendant, as a party to such relation actually
"capital" was to be returned as soon as the land was sold and apparently, engaged in the business of "holding and owing" real estate which was
from clause "decima," there were to be no profits until this "capital" was "unnecessary to carry out the purposes for which it was created."
returned. The defendant was not to receive anything for the use of said sum
until after the capital had been fully repaid, which is not consistent with the The plaintiff has appealed upon the ground that the lower court erred in not
idea of a loan. It is not impossible to provide that the capital be repaid first declaring that the defendant has forfeited its charter. Section 198 of Act No.

1085
190 provides that an action may be maintained by the Government against In the case of State of Minnesota vs. Minnesota Thresher Manufacturing Co.
the corporation: (a) when it has offended against the provision of an act for (3 L.R.A. 510) the court said (p. 518):
its creation or renewal or any act altering or amending such act; (b) when it
has forfeited its privileges and franchise by nonuser; (c) when it has The scope of the remedy furnished by its (quo warranto) is to forfeit
committed or omitted an act which amounts to a surrender of its corporate the franchises of a corporation for misuser or nonuser. It is therefore
rights, privileges, or franchises; (d) when it misused a franchise, privilege, or necessary in order to secure a judicial forfeiture of respondent's
right conferred upon it by law, or when it has exercised a franchise, privilege, charter to show a misuser of its franchises justifying such a forfeiture.
or right in contravention of law. And as already remarked the object being to protect the public, and
not to redress private grievances, the misuser must be such as to
Section 212 of Act No. 190 provides a judgment which may be rendered in work or threaten a substantial injury to the public, or such as to
said case: amount to a violation of the fundamental condition of the contract by
which the franchise was granted and thus defeat the purpose of the
When in any such action, it is found and adjudged that the grant; and ordinarily the wrong or evil must be one remediable in no
corporation has, by any act done or omitted surrendered, or forfeited other form of judicial proceeding.
its corporate rights, privileges, and franchise, or has not used the
same during the term of five years, judgment shall be entered that it Courts always proceed with great caution in declaring a forfeiture of
be ousted and excluded therefrom and that it be dissolved; but when franchises, and require the prosecutor seeking the forfeiture to bring
it is found and adjudged that a corporation has offended in any the case clearly within the rules of law entitling him to exact so
matter or manner which does not by law work as a surrender or severe a penalty. (People vs. North River Sugar Refining Co., 9
forfeiture, or has misused a franchise or exercised a power not L.R.A., 33, 39; State vs. Portland Natural Gas Co., 153, Ind., 483.)
conferred by law, but not of such a character as to work a surrender
or forfeiture of its franchise, judgment shall be rendered that it be While it is true that the courts are given a wide discretion in ordering the
ousted from the continuance of such offense or the exercise of such dissolution of corporations for violations of its franchises, etc., yet
power. nevertheless, when such abuses and violations constitute or threaten a
substantial injury to the public or such as to amount to a violation of the
It will be seen that said section (212) gives the court a wide discretion in its fundamental conditions of the contract (charter) by which the franchises were
judgment in depriving corporations of their franchise. High, in his work on granted and thus defeat the purpose of the grant, then the power of the
Extraordinary Legal Remedies, says at page 606: courts should be exercised for the protection of the people. Under the law the
people of the Philippine Islands have guaranteed the payment of the interest
It is to be observed in the outset that the courts proceed with upon cost of the construction of the railroad which occupied or occupies at
extreme caution in the proceeding which have for their object the least some of the lands purchased by the defendant. Every additional dollar
forfeiture of corporate franchises, and a forfeiture will not be allowed, of increase in the price of the land purchased by the railroad company added
except under express limitation, or for a plain abuse of power by that much to the costs of construction and thereby increased the burden
which the corporation fails to fulfill the design and purpose of its imposed upon the people. The very and sole purpose of the intervention of
organization. the defendants in the purchase of the land from the original owners was for
the purpose of selling the same to the Railroad Company at profit at an
increased price, thereby directly increasing the burden of the people by way

1086
of additional taxation. The purpose of the intervention of the defendant in the sec. 15 [5]; Code of the Civil Procedure, secs. 198, 212; New Orleans Water
transactions in question, was to enrich itself at the expense of the taxpayers Works vs. Lousiana [1902], 185 U.S., 336; State vs. New Orleans Gas Light
of the Philippine Islands, who had, by a franchise granted, permitted the & Banking Co. [1842], 2 Rob. [La.], 529, Commonwealth vs. Commercial
defendant to exist and do business as a corporation. The defendant was not Bank of Pennsylvania [1857], 28 Pa., St., 383., etc.)
willing to allow the Railroad Company to purchase the land of the original
owners. Its intervention with The Tayabas Land Company was to obtain an G.R. No. L-20583 January 23, 1967
increase in the price of the land in a resale of the same to the railroad
company. The conduct of the defendant in the premises merits the severest REPUBLIC OF THE PHILIPPINES, petitioner,
condemnation of the law. vs.
SECURITY CREDIT AND ACCEPTANCE CORPORATION, ROSENDO T.
The judgment of the lower court should be modified. It is hereby ordered and RESUELLO, PABLO TANJUTCO, ARTURO SORIANO, RUBEN BELTRAN,
decreed that the franchise heretofore granted to the defendant by which it BIENVENIDO V. ZAPA, PILAR G. RESUELLO, RICARDO D. BALATBAT,
was permitted to exist and do business as a corporation in the Philippine JOSE SEBASTIAN and VITO TANJUTCO JR., respondents.
Islands, be withdrawn and annulled and that it be disallowed to do and to
continue doing business in the Philippine Islands, unless it shall within a Office of the Solicitor General Arturo A. Alafriz and Solicitor E. M. Salva for
period of six months after final decision, liquidate, dissolve and separate petitioner.
absolutely in every respect and in all of its relations, complained of in the Sycip, Salazar, Luna, Manalo & Feliciano for respondents.
petition, with The Tayabas Land Company, without any findings to costs. And Natalio M. Balboa and F. E. Evangelista for the receiver.
it is hereby further ordered and decreed that the record be returned to the
lower court with direction that a judgment be entered in accordance herewith.
And be it further ordered and decreed, upon the presentation of positive
proof that the defendant corporation has complied in full with the foregoing
order and decree that then and in that case the complaint shall be dismissed, CONCEPCION, C.J.:
otherwise this decree shall remain in full force and effect. So ordered.
This is an original quo warranto proceeding, initiated by the Solicitor General,
Torres, Araullo and Street, JJ., concur. to dissolve the Security and Acceptance Corporation for allegedly engaging
Carson, J., concurs in result. in banking operations without the authority required therefor by the General
Banking Act (Republic Act No. 337). Named as respondents in the petition
are, in addition to said corporation, the following, as alleged members of its
Separate Opinions MALCOLM, J., concurring:
Board of Directors and/or Executive Officers, namely:

I concur in the result. The defendant corporation has violated the plain
provision of the statute law and organic law the law of its creation. It has
performed acts hostile to the policy of the state and detrimental to the public. NAME POSITION
Consequently, the charter of the Philippine Sugar Estates Development Co.
(Ltd.), should be declared forfeited for willful misuser and the corporation
should be immediately dissolved. (Philippine Bill, secs. 75; Corporation Law,

1087
The record shows that the Articles of Incorporation of defendant
corporation1 were registered with the Securities and Exchange Commission
Rosendo T. Resuello President & Chairman of the Board
on March 27, 1961; that the next day, the Board of Directors of the
corporation adopted a set of by-laws,2 which were filed with said Commission
on April 5, 1961; that on September 19, 1961, the Superintendent of Banks of
the Central Bank of the Philippines asked its legal counsel an opinion on
Pablo Tanjutco Director
whether or not said corporation is a banking institution, within the purview of
Republic Act No. 337; that, acting upon this request, on October 11, 1961,
said legal counsel rendered an opinion resolving the query in the affirmative;
Arturo Soriano Director that in a letter, dated January 15, 1962, addressed to said Superintendent of
Banks, the corporation through its president, Rosendo T. Resuello, one of
defendants herein, sought a reconsideration of the aforementioned opinion,
which reconsideration was denied on March 16, 1962; that, prior thereto, or
Ruben Beltran Director on March 9, 1961, the corporation had applied with the Securities and
Exchange Commission for the registration and licensing of its securities
under the Securities Act; that, before acting on this application, the
Commission referred it to the Central Bank, which, in turn, gave the former a
Bienvenido V. Zapa Director & Vice-President copy of the above-mentioned opinion, in line with which, the Commission
advised the corporation on December 5, 1961, to comply with the
requirements of the General Banking Act; that, upon application of members
of the Manila Police Department and an agent of the Central Bank, on May
Pilar G. Resuello Director & Secretary-Treasurer 18, 1962, the Municipal Court of Manila issued Search Warrant No. A-1019;
that, pursuant thereto, members of the intelligence division of the Central
Bank and of the Manila Police Department searched the premises of the
corporation and seized documents and records thereof relative to its
Ricardo D. Balatbat Director & Auditor business operations; that, upon the return of said warrant, the seized
documents and records were, with the authority of the court, placed under
the custody of the Central Bank of the Philippines; that, upon examination
and evaluation of said documents and records, the intelligence division of the
Jose R. Sebastian Director & Legal Counsel Central Bank submitted, to the Acting Deputy Governor thereof, a
memorandum dated September 10, 1962, finding that the corporation is:

1. Performing banking functions, without requisite certificate of


Vito Tanjutco Jr. Director & Personnel Manager authority from the Monetary Board of the Central Bank, in violation of
Secs. 2 and 6 of Republic Act 337, in that it is soliciting and
accepting deposit from the public and lending out the funds so
received;

1088
2. Soliciting and accepting savings deposits from the general Corporation's financial statements as conditional
public when the company's articles of incorporation authorize it only subscription to capital stock. Accumulated deposits of
to engage primarily in financing agricultural, commercial and P5,000 of an individual depositor may be converted into
industrial projects, and secondarily, in buying and selling stocks and stock subscription to the capital stock of the Security Credit
bonds of any corporation, thereby exceeding the scope of its powers and Acceptance Corporation at the option of the depositor.
and authority as granted under its charter; consequently such acts Sale of its shares of stock or subscriptions to its capital stock
are ultra-vires: are offered to the public as part of its regular operations.

3. Soliciting subscriptions to the corporate shares of stock and b. That out of the funds obtained from the public through the
accepting deposits on account thereof, without prior registration receipt of deposits and/or the sale of securities, loans are
and/or licensing of such shares or securing exemption therefor, in made regularly to any person by the Security Credit and
violation of the Securities Act; and Acceptance Corporation.

4. That being a private credit and financial institution, it should come A copy of the Memorandum Report dated July 30, 1962 of the
under the supervision of the Monetary Board of the Central Bank, by examination made by Examiners of this Department of the seized
virtue of the transfer of the authority, power, duties and functions of books and records of the Corporation is attached hereto.
the Secretary of Finance, Bank Commissioner and the defunct
Bureau of Banking, to the said Board, pursuant to Secs. 139 and 140 12. Section 2 of Republic Act No. 337, otherwise known as the
of Republic Act 265 and Secs. 88 and 89 of Republic Act 337." General Banking Act, defines the term, "banking institution" as
(Emphasis Supplied.) that upon examination and evaluation of the follows:
same records of the corporation, as well as of other documents and
pertinent pipers obtained elsewhere, the Superintendent of Banks, Sec. 2. Only duly authorized persons and entities may
submitted to the Monetary Board of the Central Bank a engage in the lending of funds obtained from the public
memorandum dated August 28, 1962, stating inter alia. through the receipts of deposits or the sale of bonds,
securities, or obligations of any kind and all entities regularly
11. Pursuant to the request for assistance by the Chief, Intelligence conducting operations shall be considered as banking
Division, contained in his Memorandum to the Governor dated May institutions and shall be subject to the provisions of this Act,
23, 1962 and in accordance with the written instructions of Governor of the Central Bank Act, and of other pertinent laws. ...
Castillo dated May 31, 1962, an examination of the books and
records of the Security Credit and Loans Organizations, Inc. seized 13. Premises considered, the examination disclosed that the Security
by the combined MPD-CB team was conducted by this Department. Credit and Acceptance Corporation is regularly lending funds
The examination disclosed the following findings: obtained from the receipt of deposits and/or the sale of securities.
The Corporation therefore is performing 'banking functions' as
a. Considering the extent of its operations, the Security contemplated in Republic Act No. 337, without having first complied
Credit and Acceptance Corporation, Inc., receives deposits with the provisions of said Act.
from the public regularly. Such deposits are treated in the

1089
Recommendations: 130,000 founders' shares and 470,000 preferred shares, both with a
par value of P5.00 each; and that, according to its statement of
In view of all the foregoing, it is recommended that the Monetary assets and liabilities, as of December 31, 1961, the corporation had
Board decide and declare: a capital stock aggregating P1,273,265.98 and suffered, during the
year 1961, a loss of P96,685.29. Accordingly, on December 6, 1962,
1. That the Security Credit and Acceptance Corporation is performing the Solicitor General commenced this quo warranto proceedings for
banking functions without having first complied with the provisions of the dissolution of the corporation, with a prayer that, meanwhile, a
Republic Act No. 337, otherwise known as the General Banking Act, writ of preliminary injunction be issued ex parte, enjoining the
in violation of Sections 2 and 6 thereof; and corporation and its branches, as well as its officers and agents, from
performing the banking operations complained of, and that a receiver
be appointed pendente lite.
2. That this case be referred to the Special Assistant to the Governor
(Legal Counsel) for whatever legal actions are warranted, including,
if warranted criminal action against the Persons criminally liable Upon joint motion of both parties, on August 20, 1963, the Superintendent of
and/or quo warranto proceedings with preliminary injunction against Banks of the Central Bank of the Philippines was appointed by this Court
the Corporation for its dissolution. (Emphasis supplied.) receiver pendente lite of defendant corporation, and upon the filing of the
requisite bond, said officer assumed his functions as such receiver on
September 16, 1963.
that, acting upon said memorandum of the Superintendent of Banks,
on September 14, 1962, the Monetary Board promulgated its
Resolution No. 1095, declaring that the corporation is performing In their answer, defendants admitted practically all of the allegations of fact
banking operations, without having first complied with the provisions made in the petition. They, however, denied that defendants Tanjutco (Pablo
of Sections 2 and 6 of Republic Act No. 337; 3that on September 25, and Vito, Jr.), Soriano, Beltran, Zapa, Balatbat and Sebastian, are directors
1962, the corporation was advised of the aforementioned resolution, of the corporation, as well as the validity of the opinion, ruling, evaluation and
but, this notwithstanding, the corporation, as well as the members of conclusions, rendered, made and/or reached by the legal counsel and the
its Board of Directors and the officers of the corporation, have been intelligence division of the Central Bank, the Securities and Exchange
and still are performing the functions and activities which had been Commission, and the Superintendent of Banks of the Philippines, or in
declared to constitute illegal banking operations; that during the Resolution No. 1095 of the Monetary Board, or of Search Warrant No. A-
period from March 27, 1961 to May 18, 1962, the corporation had 1019 of the Municipal Court of Manila, and of the search and seizure made
established 74 branches in principal cities and towns throughout the thereunder. By way of affirmative allegations, defendants averred that, as of
Philippines; that through a systematic and vigorous campaign July 7, 1961, the Board of Directors of the corporation was composed of
undertaken by the corporation, the same had managed to induce the defendants Rosendo T. Resuello, Aquilino L. Illera and Pilar G. Resuello; that
public to open 59,463 savings deposit accounts with an aggregate on July 11, 1962, the corporation had filed with the Superintendent of Banks
deposit of P1,689,136.74; that, in consequence of the foregoing an application for conversion into a Security Savings and Mortgage Bank,
deposits with the corporation, its original capital stock of P500,000, with defendants Zapa, Balatbat, Tanjutco (Pablo and Vito, Jr.), Soriano,
divided into 20,000 founders' shares of stock and 80,000 preferred Beltran and Sebastian as proposed directors, in addition to the defendants
shares of stock, both of which had a par value of P5.00 each, was first named above, with defendants Rosendo T. Resullo, Zapa, Pilar G.
increased, in less than one (1) year, to P3,000,000 divided into Resuello, Balatbat and Sebastian as proposed president, vice-president,
secretary-treasurer, auditor and legal counsel, respectively; that said

1090
additional officers had never assumed their respective offices because of the meeting of stockholders of the corporation, held on September 27, 1962,
pendency of the approval of said application for conversion; that defendants showing that said defendants had been elected officers thereof; that the
Soriano, Beltran, Sebastian, Vito Tanjutco Jr. and Pablo Tanjutco had views of the legal counsel of the Central Bank, of the Securities and
subsequently withdrawn from the proposed mortgage and savings bank; that Exchange Commission, the Intelligence Division, the Superintendent of
on November 29, 1962 or before the commencement of the present Banks and the Monetary Board above referred to have been expressed in
proceedings the corporation and defendants Rosendo T. Resuello and the lawful performance of their respective duties and have not been assailed
Pilar G. Resuello had instituted Civil Case No. 52342 of the Court of First or impugned in accordance with law; that neither has the validity of Search
Instance of Manila against Purificacion Santos and other members of the Warrant No. A-1019 been contested as provided by law; that the only assets
savings plan of the corporation and the City Fiscal for a declaratory relief and of the corporation now consist of accounts receivable amounting
an injunction; that on December 3, 1962, Judge Gaudencio Cloribel of said approximately to P500,000, and its office equipment and appliances, despite
court issued a writ directing the defendants in said case No. 52342 and their its increased capitalization of P3,000,000 and its deposits amounting to not
representatives or agents to refrain from prosecuting the plaintiff spouses less than P1,689,136.74; and that the aforementioned petition of the
and other officers of the corporation by reason of or in connection with the corporation, in Civil Case No. 52342 of the Court of First Instance of Manila,
acceptance by the same of deposits under its savings plan; that acting upon for a declaratory relief is now highly improper, the defendants having already
a petition filed by plaintiffs in said case No. 52342, on December 6, 1962, the committed infractions and violations of the law justifying the dissolution of the
Court of First Instance of Manila had appointed Jose Ma. Ramirez as corporation.
receiver of the corporation; that, on December 12, 1962, said Ramirez
qualified as such receiver, after filing the requisite bond; that, except as to Although, admittedly, defendant corporation has not secured the requisite
one of the defendants in said case No. 52342, the issues therein have authority to engage in banking, defendants deny that its transactions partake
already been joined; that the failure of the corporation to honor the demands of the nature of banking operations. It is conceded, however, that, in
for withdrawal of its depositors or members of its savings plan and its former consequence of a propaganda campaign therefor, a total of 59,463 savings
employees was due, not to mismanagement or misappropriation of corporate account deposits have been made by the public with the corporation and its
funds, but to an abnormal situation created by the mass demand for 74 branches, with an aggregate deposit of P1,689,136.74, which has been
withdrawal of deposits, by the attachment of property of the corporation by its lent out to such persons as the corporation deemed suitable therefor. It is
creditors, by the suspension by debtors of the corporation of the payment of clear that these transactions partake of the nature of banking, as the term is
their debts thereto and by an order of the Securities and Exchange used in Section 2 of the General Banking Act. Indeed, a bank has been
Commission dated September 26, 1962, to the corporation to stop soliciting defined as:
and receiving deposits; and that the withdrawal of deposits of members of
the savings plan of the corporation was understood to be subject, as to time ... a moneyed institute [Talmage vs. Pell 7 N.Y. (3 Seld. ) 328, 347,
and amounts, to the financial condition of the corporation as an investment 348] founded to facilitate the borrowing, lending and safe-keeping of
firm. money (Smith vs. Kansas City Title & Trust Co., 41 S. Ct. 243, 255
U.S. 180, 210, 65 L. Ed. 577) and to deal, in notes, bills of exchange,
In its reply, plaintiff alleged that a photostat copy, attached to said pleading, and credits (State vs. Cornings Sav. Bank, 115 N.W. 937, 139 Iowa
of the anniversary publication of defendant corporation showed that 338). (Banks & Banking, by Zellmann Vol. 1, p. 46).
defendants Pablo Tanjutco, Arturo Soriano, Ruben Beltran, Bienvenido V.
Zapa, Ricardo D. Balatbat, Jose R. Sebastian and Vito Tanjutco Jr. are Moreover, it has been held that:
officers and/or directors thereof; that this is confirmed by the minutes of a

1091
An investment company which loans out the money of its customers, It is urged, however, that this case should be remanded to the Court of First
collects the interest and charges a commission to both lender and Instance of Manila upon the authority of Veraguth vs. Isabela Sugar Co. (57
borrower, is a bank. (Western Investment Banking Co. vs. Murray, 56 Phil. 266). In this connection, it should be noted that this Court is vested with
P. 728, 730, 731; 6 Ariz 215.) original jurisdiction, concurrently with courts of first instance, to hear and
decide quo warranto cases and, that, consequently, it is discretionary for us
... any person engaged in the business carried on by banks of to entertain the present case or to require that the issues therein be taken up
deposit, of discount, or of circulation is doing a banking business, in said Civil Case No. 52342. The Veraguth case cited by herein defendants,
although but one of these functions is exercised. (MacLaren vs. in support of the second alternative, is not in point, because in said case
State, 124 N.W. 667, 141 Wis. 577, 135 Am. S.R. 55, 18 Ann. Cas. there were issues of fact which required the presentation of evidence, and
826; 9 C.J.S. 30.) courts of first instance are, in general, better equipped than appellate courts
for the taking of testimony and the determination of questions of fact. In the
Accordingly, defendant corporation has violated the law by engaging case at bar, there is, however, no dispute as to the principal facts or acts
in banking without securing the administrative authority required in performed by the corporation in the conduct of its business. The main issue
Republic Act No. 337. here is one of law, namely, the legal nature of said facts or of the
aforementioned acts of the corporation. For this reason, and because public
interest demands an early disposition of the case, we have deemed it best to
That the illegal transactions thus undertaken by defendant corporation
determine the merits thereof.
warrant its dissolution is apparent from the fact that the foregoing misuser of
the corporate funds and franchise affects the essence of its business, that it
is willful and has been repeated 59,463 times, and that its continuance inflicts Wherefore, the writ prayed for should be, as it is hereby granted and
injury upon the public, owing to the number of persons affected thereby. defendant corporation is, accordingly, ordered dissolved. The appointment of
receiver herein issued pendente lite is hereby made permanent, and the
receiver is, accordingly, directed to administer the properties, deposits, and
other assets of defendant corporation and wind up the affairs thereof
conformably to Rules 59 and 66 of the Rules of Court. It is so ordered.

Reyes, J.B.L., Dizon, Regala, Makalintal, Bengzon, J.P., Zaldivar, Sanchez


and Castro, JJ., concur.

1092
JAIME T. BUENAFLOR, petitioner,
vs.
CAMARINES SUR INDUSTRY CORPORATION, respondent.

Manuel O. Chan and Vicente Ampil for petitioner.


Evaristo R. Sandoval and Claro T. Almeda for respondent.

BENGZON, J.:

Jaime T. Buenaflor has appealed the decision of the Public Service


Commission which rejected his application to install and operate a 5-ton ice
plant in Sabang (Calabanga, Camarines Sur) even as it permitted Camarines
Sur Industry Corporation to build in that barrio, a factory with the same
output.

On June 25, 1957, Buenaflor filed his said application (P. S. Case 107548)
together with another application to establish a cold storage and refrigeration
service of about 6,000 cubic feet capacity (P. S. Case 107549). The
Commission, by order of September 12, 1957, set the applications for
hearing on October 9, 1957, requiring applicant to publish them in two
newspapers, and to serve copy thereof to Iigo Daza and Camarines Sur
Industry Corporation (hereinafter called Camarines Corporation). These
owned ice plants in neighboring municipalities and had been apparently
selling ice to Sabang's inhabitants.

After receiving copy of Buenaflor's applications, the Camarines Corporation


submitted to the Commission on October 1, 1957, its own two applications:
one for authority to construct and manage a 5-ton ice plant, and another for a
cold storage and refrigeration system, both in Sabang too (P. S. Cases
109874 and 109875). It likewise registered opposition to Buenaflor's
proposed ice business, on the ground that it was the pioneer distributor of the
commodity in that particular locality.

When the petitions of Buenaflor were called for hearing on October 9, 1957,
G.R. Nos. L-14991-94 May 30, 1960 the attorney for Camarines Corporation voicing its application, invited

1093
attention to his client's applications moved for postponement, and agreed to Evidence was presented in support of the applications and oppositions.
a joint hearing of the four applications of both parties on October 25, 1957.
The Commission, in its decision of December 12, 1958, after settling forth the
On the last mentioned date, Buenaflor's attorneys presented a motion to gist of the proofs submitted to it, made the following considerations and
dismiss the Camarines Corporation's applications, challenging its personality, conclusions.
inasmuch as its corporate life had expired in November 1953, in accordance
with its own articles of incorporation. Surprised by the move, counsel of There is a clear need for an ice plant and a cold storage service in
Camarines Corporation asked, and was granted, time to answer. Immediately the barrio of Sabang and question to decide is who of the applicant
thereafter, the corporators of Camarines Corporation got busy and executed should be granted the necessary authority inasmuch as we do not
on October 30, 1957, and registered October 31, 1957, new articles of believe from the evidence that we should authorize two ice plants of
incorporation of Camarines Sur Industry Corporation, and at the same time, 5 tons each and two cold storage chambers with a total capacity of
notarized a deed of conveyance assigning to the new corporation, all the 14,000 cubic feet. As to the ice plant service, we find that Buenaflor
assets of the expired (old) corporation, together with its existing certificates of filed his application ahead of the Camarines Corporation but the
public convenience to operate ice factories in Naga and Magarao. evidence and our records show that the Camarines Corporation is
really the pioneer ice plant in Magarao since 1945 which now has a
Without loss of time, the corporators of the defunct (old) corporation and the capacity of 10 tons, and another ice plant established that the
newly organized corporation petitioned the Public Service Commission for Camarines Corporation has been rendering ice service thru delivery
the approval of the conveyance, and on November 7, 1957, the Commission in Sabang but we doubt whether its service has been adequate
provisionally approved the transfer of assets, plus the certificates of public because there is satisfactory proof that ice also comes from other
convenience. places. The fact, however, is that the Camarines Corporation cannot
be said to have neglected its duty to serve Sabang and we believe
On November 8, 1957, the Camarines Corporation (new) answered the that the shortage in its service has been due to the fact that the
motion to dismiss, by alleging to the amazement of Buenaflor its recent produce of its Naga and Magarao plants are needed for its other
incorporation, plus its acquisition of the assets and certificates of the old territories with not much to spare for Sabang. We think that as the
Camarines Corporation with the Commission's approval as above described. pioneer ice plant operator in Naga and Magarao with authority to
serve Sabang the Camarines Corporation, which has not abandoned
Reiterating his application, while resisting the Camarines Corporation, its service in Sabang, is entitled to the protection of its investments
Buenaflor argued: (a) he was first to apply; (b) although the old Camarines and to put up an ice plant in Sabang, and that the Camarines
Corporation had been operating an ice plant in Magarao town, only six Corporation has been rendering service therein although in a limited
kilometers away, it neglected to take trouble of applying until Buenaflor had manner. We believe, therefore, that applicant Camarines Corporation
made his application; (c) the preference which the new Camarines has a better right than Buenaflor to the certificate for a 5-ton ice plant
Corporation claims by virtue of the old corporation's having distributed ice in in Sabang. As to the cold storage service, we think that Buenaflor
Sabang for the years previous to Buenaflor's application, should not be has a better right to the certificate.... By virtue of Buenaflor's right of
granted, because since 1953 such old corporation had ceased to be juridical priority in the filing of his application and the fact that he is as
entity, and could not lawfully continue in business nor invoke any protection financially capable as the Camarines Corporation to install the
or preference. service, we believe that the certificate for the cold storage service in
Sabang should be granted to Buenaflor, but insamuch as we take

1094
notice of the fact that a cold storage operator also needs ice for the such serve and distribution of ice applying the "prior operator" rule. 1 In
preservation of fish and other perishable foodstuffs when these other words, the new Camarines Corporation is rewarded, precisely because
cannot be immediately deposited in the refrigerating chambers and the old corporation, its predecessor, had violated the lawduring that period
also to provide its customers with ice they need after the goods are (1953-1957). We can not, and should not countenance such anomalous
removed from the chambers, we believe that applicant Buenaflor result.
may also be granted a certificate for a one (1) ton ice plant in
Sabang together with a certificate of 5,000 cubic feet cold storage On the other hand, when the old Camarines Corporation docketed its
service. application October 1, 1957, it had no juridical personality, it had ceased to
exist as a corporation and could not sue 2 nor apply for certificate, for it was
The Camarines Corporation did not appeal. Buenaflor appealed in so far as incapable of receiving a grant3 . It was not even a corporation de facto4 . And
he was denied authority to erect a 5-ton ice plant. then, there is no application subscribed by the new Camarines Corporation.
Far from being mere technicality, these point support a conclusion which
Therefore, the question of cold storage service is not here in issue, since appears to be just and equitable, not only for the reasons already indicated,
Buenaflor got it, and Camarines Corporation did not appeal. but also to compensate Buenaflor's diligence and courage in exposing the
irregular practice5 of a "ghost" corporation foisting its services upon the
As to the ice plant, Buenaflor insists he should be given authority to establish unsuspecting public of Sabang and neighboring territory enjoying a
a 5-ton ice plant not the new Camarines Corporation. His line of argument franchise without paying, perhaps, the corporate income tax 6 and other
centers around the expiration of the old Corporation's charter in 1953; and burdens attached to corporate existence.
we think he touches the vital spot.
Remembering the Camarines Corporation's automatic cessation in
It is admitted and the Commission found--that the needs of Sabang Barrio November 1956 (three years after November 1953) we must decline to
will be conveniently served with the establishment of a 5-ton ice plant. But it regard the new Camarines Corporation (formed October 30, 1957) as
elected to deny Buenaflor's application, even as it awarded the privilege to a continuation of the old.7 At most, it is the transferee of the properties of the
the new Camarines Corporation on the ground that it (the old old corporation (or more properly, the assets of the stockholders) plus the
corporation) had been serving ice in Sabang up to the time of Buenaflor's certificate of public convenience to operate the ice plant in Naga and
application, and was, consequently, the pioneer operator there. Magarao.8 And yet, as stated, the new corporation has not filed any
application for certificate of public convenience in Sabang, and has not
published such application.
The fact, however, is that since 1953, the old Corporation had been illegally
plying its business of selling ice in Sabang because, under the Corporation
Law, Sec. 77, after November 1953, it could not lawfully continue the On these grounds, we think it was error to grant preferential treatment to the
business for which it had been established (operate ice plant, sell ice, etc). new Camarines Corporation over Jaime T. Buenaflor who, besides being
After November 1953, it could only continue to exist for three years for the qualified, in the eyes of the Commission, had applied for the privilege months
purpose of prosecuting and defending suits by or against it, and of enabling it in advance of the old Camarines Corporation, and of the incorporation of the
gradually to settle and close its affairs, to dispose and convey its property new Camarines Corporation.
and to divide its capital stock. It could not, without violating the law, continue
to sell ice. And yet, the Commission awarded the certificate on the basis of

1095
Wherefore, revoking the appealed decision in so far as it awarded the
certificate to said Corporation, we hereby approve Buenaflor's application for
five tons, instead of one ton, subject to the usual conditions imposed by the
Public Service Commission on ice plant establishments.

Costs against Camarines Corporation.

Paras, C. J., Montemayor, Bautista Angelo, Labrador, Concepcion, Barrera,


and Gutierrez David, JJ., concur.

G.R. No. L-16779 August 16, 1961

NATIONAL ABACA AND OTHER FIBERS CORPORATION, plaintiff-


appellant,
vs.
APOLONIA PORE, defendant-appellee.

A. LLamas & Arsenio P. Roman for plaintiff-appellant.


Serafin Ramento for defendant-appellee.

CONCEPCION, J.:

Appeal by plaintiff National Abaca and other Fibers Corporation, from two (2)
orders of the Court of First Instance of Leyte.

On November 14, 1953, plaintiff filed with the Municipal Court of Tacloban,
Leyte, a complaint, against defendant Apolonia Pore, for the recovery of
P1,213.34, allegedly advanced to her for the purchase of hemp for the
account of the former and for which she had allegedly failed to account. In
her answer, defendant alleged that she had accounted for all cash advances
received by her for the aforementioned purpose from the plaintiff. In due
course, said court rendering judgment on April 11, 1956, finding that the
defendant had not accounted for cash advances in the sum of P272.49,
which she was, accordingly, sentenced to pay to the plaintiff, with legal
interest from November 18, 1953, in addition to the costs.

1096
Said court having subsequently denied a reconsideration of this decision, as record book of plaintiff's outgoing correspondence on August 24, 1956, only
well a new trial prayed for the plaintiff, the latter appealed to the Court of First the copy addressed to defendant's counsel had actually been mailed (as
Instance of Leyte, in which defendant moved to dismiss the complaint upon evidenced by registry receipt No. 57209 dated August 25, 1956); that the
the ground that plaintiff has no legal capacity to sue, it having abolished by original copy of the amended complaint, addressed to the clerk of court,
Executive Order No. 372 of the President of the Philippines, dated November could not be located, despite diligent efforts made to find the same; that
24,1950. Plaintiff objected thereto upon the ground that pursuant to said plaintiff's failure to file in court the original of said amended complaint is
executive order, plaintiff "shall nevertheless be continued as a body imputable to the excusable negligence of the aforementioned Mrs. Ocampo,
corporate for a period of three (3) years from the effective date" of said whose affidavit was annexed, also, to the motion for reconsideration; and
executive order, which was November 30, 1950, "for the purpose of that, plaintiff has a just and valid claim against the defendant. Plaintiff prayed,
prosecuting and defending suits by or against it and of enabling the Board of therefore, that said order of September 1, 1956 be reconsidered and set
Liquidators" thereby created "gradually to settle and close its affairs", . . aside and that its aforementioned amended complaint be admitted.
. and that this case was begun on November 14, 1953, or before the
expiration of the period aforementioned. After due hearing, the court of first Said motion for reconsideration was denied by an order dated October 2,
instance issued an order dated August 1, 1956, directing plaintiff to amend 1956, whereupon plaintiff brought the case for review, by Record on Appeal,
the complaint, within ten (10) days from notice, by including the Board of to the Court of Appeals which, however, forwarded the records to us, the
Liquidators as co-party plaintiff, with the admonition that otherwise the case issues raised in the appeal being purely of law, namely;(1) whether an action,
would be dismissed. commenced within three (3) years after the abolition of plaintiff, as a
corporation, may be continued by the same after the expiration of said
On September 1, 1956, said court issued another order dismissing the case, period; and (2) whether, under the facts set forth above, the lower court
without pronouncement as to costs, it appearing that the aforementioned should have granted plaintiff's motion for reconsideration of its order of
amended had not been made, despite the fact that copy of said order of September 1, 1956.
August 1, 1956 had been sent, by registered mail, to plaintiff's counsel on
August 6, 1956. Copy of the last order was delivered, on September 13, With respect to the first question, the rule appears to be well settled that, in
1956, to counsel for the plaintiff, which filed, on September 21, 1956, a the absence of statutory provision to the contrary, pending actions by or
motion alleging that, copy of the order of August 1, 1956 was received by the against a corporation are abated upon expiration of the period allowed by law
plaintiff on August 17, 1956; that thereupon said counsel prepared an for the liquidation of its affairs.
amended complaint copy of which was annexed to the motion as
directed by the court; that on August 24, 1956, said counsel handed two It is generally held, that where a statute continues the existence of a
copies of said amended complaint to Mrs. Receda Vda. de Ocampo, the corporation for a certain period after its dissolution for the purpose of
employee of the aforesaid Board of Liquidators in charge of plaintiff's prosecuting and defending suits, etc., the corporation becomes
incoming and outgoing correspondence, with instructions to them mail said defunct upon the expiration of such period, at least in the absence of
copies to the Court of First Instance of Leyte and to counsel for defendant a provision to the contrary, so that no action can afterwards be
herein; that on September 13, 1956, plaintiff's counsel received copy of the brought by or against it, and must be dismissed. Actions pending by
order of September 1, 1956; that thereupon he inquired from plaintiff's or against the corporation when the period allowed by the statute
mailing clerk whether or not his instructions, concerning the mailing of copies expires, ordinarily abate.
of said amended complaint, had been complied with; that he then found out
that, although said copies of the amended complaint were entered in the

1097
. . . This time limit does not apply unless the circumstances are such been conveyed pursuant to the authority of section 78 may used and
as to bring the corporation within the provision of the statute. be sued as such in all matters connected with the liquidation. By the
However, the wording of the statutes, in some jurisdictions authorize terms of the statute the effect of the conveyance is to make the
suits after the expiration of the time limit, where the statute provides trustees the legal owners of the property conveyed, subject to the
that for the purpose of any suit brought by or against the corporation beneficial interest therein of creditors and stockholders. (pp. 389-
shall continue beyond such period for a further named period after 390; see also Sumera v. Valencia [67 Phil. 721, 726-727).
final judgment. (Fletcher's Cyclopedia on Corporations, Vol. 16, pp.
892-893.). Obviously, the complete loss of plaintiff's corporate existence after the
expiration of the period of three (3) years for the settlement of its affairs is
Our Corporation Law contains no provision authorizing a corporation, after what impelled the President to create a Board of Liquidators, to continue the
three (3) years from the expiration of its lifetime, to continue in its corporate management of such matters as may then be pending. The first question
name actions instituted by it within said period of three (3) years. in fact, must, therefore, be answered in the negative.
section 77 of said law provides that the corporation shall "be continued as a
body corporate for three (3) years after the time when it would have been . . . With respect, however, to the second question, we hold that the lower court
dissolved, for the purposed of prosecuting and defending suits by or against erred in not granting plaintiff's motion for reconsideration of September 21,
it . . .", so that, thereafter, it shall no longer enjoy corporate existence for such 1956. To begin with, the judgment of the municipal court of Tacloban against
purpose. For this reason, section 78 of the same law authorizes the the defendant is a strong indication of the validity and justice of plaintiff's
corporation, "at any time during said three years . . . to convey all of its claim against her. Moreover, the record satisfactorily shows that plaintiff had
property to trustees for the benefit of members, stockholders, creditors and prepared an amended complaint, as directed in the order of August 1, 1956,
other interested", evidently for the purpose, among others, of enabling said upon receipt thereof; that copy of said amended complaint had actually been
trustees to prosecute and defend suits by or against the corporation begun sent by registered mail to defendant's counsel; that plaintiff's counsel had
before the expiration of said period. Hence, commenting on said sections, given to its mailing clerk the proper instructions for the filing of the original of
Judge Fisher, in his work entitled Philippines Law on Stock Corporations said amended complaint with the office of the Court of First Instance of
(1929 ed.), has the following to say: Leyte; that said mailing clerk had endeavored to comply with the
aforementioned instructions, as evidenced by the corresponding entry in the
It is to be noted that the time during which the corporation, through record book of plaintiff's outgoing correspondence; and that the failure to file
its own officers, may conduct the liquidation of its assets and sue in court said original of the amended complaint must have been due,
and be sued as a corporation is limited to three years from the time therefore, either to accident or to excusable negligence on the part of said
the period of dissolution commences; but that there is no time limited mailing clerk.
within the trustees must complete a liquidation placed in their hands.
It is provided only (Corp. Law, Sec. 78) that the conveyance to the WHEREFORE, the orders appealed from, dated September 1 and October 3,
trustees must be made within the three-year period. It may be found 1956 are reversed, plaintiff's amended complaint is hereby admitted, and the
impossible to complete the work of liquidation within the three-year record remanded to the lower court for further proceedings, with the costs of
period or to reduce disputed claims to judgment. The authorities are this instance against defendant-appellee, Apolonio Pore.
to the effect that suits by or against a corporation abate when it
ceased to be an entity capable of suing or being sued (7 R.C.L. It is so ordered.
Corps., Par. 750); but trustees to whom the corporate assets have

1098
Bengzon, C.J., Padilla, Labrador, Reyes, J.B.L., Barrera, Paredes, Dizon, De
Leon and Natividad, JJ., concur.
Bautista Angelo, J., is on leave.

G.R. No. L-15778 April 23, 1962

TAN TIONG BIO, ET AL., petitioners,


vs.
COMMISSIONER OF INTERNAL REVENUE, respondent.

Sycip, Salazar and Associates for petitioners.


Office of the Solicitor General for respondent.

BAUTISTA ANGELO, J.:

1099
On October 19, 1946, the Central Syndicate, a corporation organized under On September 8, 1954, the Central Syndicate elevated the case to the Court
the laws of the Philippines, thru its General Manager, David Sycip, sent a of Tax Appeals questioning the ruling of the Collector which denies its claim
letter to the Collector of Internal Revenue advising the latter that it purchased for refund as well as the assessment made against it of the sum of
from Dee Hong Lue the entire stock of surplus properties which the said Dee P33,797.88, plus the sum of P300.00 as compromise penalty, as stated
Hong Lue had bought from the Foreign Liquidation Commission and that as it above. The Collector filed his answer thereto wherein he reiterated his ruling
assumed Dee Hong Lue's obligation to pay the 3-1/2% sales tax on said and prayed that the Central Syndicate be ordered to pay the deficiency sales
surplus goods, it was remitting the sum of P43,750.00 in his behalf as tax and surcharge as demanded in his letters dated January 4, 1952 and
deposit to answer for the payment of said sales tax with the understanding August 5, 1954. On October 28, 1954, the syndicate filed a motion
that it would later be adjusted after the determination of the exact requesting that the issue of prescription it has raised against the collection of
consideration of the sale. the tax be first determined as a preliminary question, but action thereon was
deferred by the Court of Tax Appeals until after the trial of the case on the
On January 31, 1948, the syndicate again wrote the Collector requesting the merits.
refund of P1,103.28 representing alleged excess payment of sales tax due to
the adjustment and reduction of the purchase price in the amount of On November 5, 1954, the Collector filed a motion requiring the syndicate to
P31,522.18. Said letter was referred to an agent for verification and report. file a bond to guarantee the payment of the tax assessed against it which
On September 18, 1951, after a thorough investigation of the facts and motion was denied by the Court of Tax Appeals on the ground that cannot be
circumstances surrounding the transaction, the agent reported (1) that Dee legally done it appearing that the syndicate is already a non-existing entity
Hong Lue purchased the surplus goods as trustee for the Central Syndicate due to the expiration of its corporate existence. In view of this development,
which was in the process of organization at the time of the bidding; (2) that it the Collector filed a motion to dismiss the appeal on the ground of lack of
was the representatives of the Central Syndicate that removed the surplus personality on the part of the syndicate, which met an opposition on the part
goods from their base at Leyte on February 21, 1947; (3) that the syndicate of the latter, but on January 25, 1955, the Court of Tax Appeals issued a
must have realized a gross profit of 18.8% from its sales thereof; and (4) that resolution dismissing the appeal primarily on the ground that the Central
if the sales tax were to be assessed on its gross sales it would still be liable Syndicate has no personality to maintain the action then pending before it.
for the amount of P33,797.88 as deficiency sales tax and surcharge in From this order the syndicate appealed to the Supreme Court wherein it
addition to the amount of P43,750.00 which the corporation had deposited in intimated that the appeal should not be dismissed because it could be
the name of Dee Hong Lue as estimated sales tax due from the latter. substituted by its successors-in-interest, to wit: Tan Tiong Bio, Yu Khe Thai,
Alfonso Sycip, Dee Hong Lue, Lim Shui Ty, Sy Seng Tong, Sy En, Co Giap
Based on the above findings of the agent in charge of the investigation, the and David Sycip. And taking cue from this suggestion, this Court ruled
Collector decided that the Central Syndicate was the importer and original against the dismissal and held: "The resolution appealed from is set aside
seller of the surplus goods in question and, therefore, the one liable to pay and the respondent court is ordered to permit the substitution of the officers
the sales tax. Accordingly, on January 4, 1952, the Collector assessed and directors of the defunct Central Syndicate as appellants, and to proceed
against the syndicate the amount of P33,797.88 and P300.00 as deficiency with the hearing of the appeal upon its merits." In permitting the substitution,
sales tax, inclusive of the 25% surcharge and compromise penalty, this Court labored under the premise that said officers and directors "may be
respectively, and on the same date, in a separate letter, he denied the held personally liable for the unpaid deficiency assessments made by the
request of the syndicate for the refund of the sum of P1,103.28. Collector of Internal Revenue against the defunct syndicate."

1100
After trial, the Court of Tax Appeals rendered decision the dispositive part of 1. Petitioners contend that the Central Syndicate cannot be held liable for the
which reads as follows: deficiency sales tax in question because it is not the importer of the surplus
goods purchased from the Foreign Liquidation Commission for the reason
WHEREFORE, in view of the foregoing considerations, the decision that said surplus goods were purchased by Dee Hong Lue as shown by the
of the Collector of Internal Revenue appealed from is hereby contract executed between him and the Foreign Liquidation Commission and
affirmed, except with regard to the imposition of the compromise the fact that the Central Syndicate only purchased the same from Dee Hong
penalty of P300.00 the collection of which is unauthorized and illegal Lue and not from the Foreign Liquidation Commission as shown by Exhibit
in the absence of a compromise agreement between the parties. 13.
(Collector of Internal Revenue vs. University of Sto. Tomas, G. R. No.
L-11274, November 28, 1958; Collector of Internal Revenue vs. This contention cannot be sustained. As correctly observed by the Court of
Bautista & Tan, G.R. No. L-12250, May 27, 1959.) . Tax Appeals, the overwhelming evidence presented by the Collector points to
the conclusion that Dee Hong Lue purchased the surplus goods in question
The petitioners Tan Tiong Bio, Yu Khe Thai, Lim Shui Ty, Alfonso not for himself but for the Central Syndicate which was then in the process of
Sycip, Sy En alias Sy Seng Sui, Dee Hong Lue, and Sy Seng Tong, incorporation such that the deed of sale Exhibit 13 which purports to show
who appear in the Articles of Incorporation of the Central Syndicate that Dee Hong Lue sold said goods to the syndicate for a consideration of
Annex A (pp. 60-66, CTA rec.) as incorporators and directors of the P1,250,000.00 (the same amount paid by Dee Hong Lue to the Foreign
corporation, the second named being in addition its President and Liquidation Commission) "is but a ruse to evade payment of a greater
the seventh its Treasurer, are hereby ordered to pay jointly and amount of percentage tax." The aforesaid conclusion of the lower court was
severally, to the Collector of Internal Revenue, the sum of arrived at after a thorough analysis of the evidence on record, pertinent
P33,797.88 as deficiency sales tax and surcharge on the surplus portion of which we quote hereunder with approval:
goods purchased by them from the Foreign Liquidation Commission
on July 5, 1946, from which they realized an estimated gross sales of Exhibit "38-A" for the respondent (p. 178, BIR rec.) shows that as
P1,447,551.65, with costs. .. early as July 23, 1946, or before the organization and incorporation
of Central Syndicate, Mr. David Sycip, who was subsequently
Petitioners interposed the present appeal. appointed General Manager of the corporation, together with Messrs.
Sy En alias Sy Seng Sui (one of the incorporators of Central
The important issues to be determined in this appeal are: (1) whether the Syndicate), Serge Gordeof and Chin Siu Bun (an employee of the
importer of the surplus goods in question the sale of which is subject to the same corporation), for and in the name of Central Syndicate then in
present tax liability is Dee Hong Lue or the Central Syndicate who has been the process of organization, went to Leyte to take over the surplus
substituted by the present petitioners; (2) whether the deficiency sales tax properties sold by the FLC to Dee Hong Lue, which the latter held in
which is now sought to be collected has already prescribed; and (3) the trust for the corporation. Exhibit 38-A, which is a certificate issued by
Central Syndicate having already been dissolved because of the expiration of no less than David Sycip himself who was subsequently appointed
its corporate existence, whether the sales tax in question can be enforced General Manager of the corporation admits in express terms the
against its successors-in-interest who are the present petitioners. following "... the surplus property sold by the Foreign Liquidation
Commission to Dee Hong Lue (and held in trust by the latter for the
Syndicate ...." (Emphasis ours.) We give full weight and credence to
the adverse admissions made by David Sycip against the petitioners

1101
as appearing in his certificate Exhibit 38-A (p. 178, BIR rec.) (28) days later viz., on August 20, 1946? Is this not another clear and
considering that at the time he made them, he was a person jointly unmistakable indication that from the very start, as is the theory of
interested with the petitioners in the transaction over which there was the respondent, the real purchasers of the 'Mystery Pile' from the
yet no controversy over any sales tax liability. (Secs. 11 and 33, Rule FLC and as such the 'importers' of the goods, were the Central
123, Rules of Court; Clem vs. Forbeso, Tex. Cir App. 10 S.W. 2d 223; Syndicate and/or the group of big financiers composing it before said
Street vs. Masterson, Tex. Cir. App. 277 S.W. 407.) . corporation was incorporated on August 15, 1946; and, that Dee
Hong Lue acted merely as agent of these persons when he
Exhibit '39' for the respondent (pp. 184-187, BIR rec.) which is a purchased the pile from the FLC? As a general rule, one does not
letter of Mr. Yu Khe Thai President, Director and biggest stockholder exercise all the acts of ownership over a property especially if it
of Central Syndicate (Exhibit A, pp. 60-65, CTA rec.) dated involves a big amount until after the documents evidencing such
September 17, 1946 and addressed to the Commanding General ownership are fully accomplished.
AFWESPAC, Manila, contains the following categorical admissions
which corroborate the admissions made by David Sycip; that the so- Moreover, it appears that on October 3, 1946, Dee Hong Lue was
called Leyte 'Mystery Pile' surplus properties were owned by Central investigated by Major Primitivo San Agustin, Jr., G-2 of the Philippine
Syndicate by virtue of a purchase from the FLC, effected in the Army, because of the discovery of some gun parts found in his
name of Dee Hong Lue on July 5, 1946, inasmuch as Central shipment of surplus material from Palo, Leyte.
Syndicate was then still in the process of organization; that Dee
Hong Lue held the said surplus properties in trust until the mere In his sworn statement, Exhibit 16 (pp. 133-139, BIR rec.) before
formal turnover to the corporation on August 20, 1946, when the said officer, Dee Hong Lue admitted the following: That he paid the
corporation had already been organized and incorporated under the FLC the amount of P1,250,000.00 "with the checks of Yu Khe Thai,
laws of the Philippines; and that on July 23, 1946 viz., twenty-two maybe also Alfonso Sycip and my checks with many others"; that "at
(22) days before the incorporation of Central Syndicate on August the beginning I was trying to buy the pile for myself without telling
15, 1946 'our General Manager, Mr. David Sycip accompanied by other people and other friends of mine." "Watkins came to me and he
one of our directors, Mr. Sy En, arrived in Leyte to take over the bid for me for P600,000 or P700,000, but later on when the price
properties.' went up to P1,250,000, I talked to my friends who said I could get
money." "So, I bought it with their checks and mine" (Exhibit 16-B, p.
Before passing on to the rest of the evidence supporting the finding 138, BIR rec.) and, that after buying the "Mystery Pile", he (Dee
of respondent, we would like to call attention to this significant detail. Hong Lue) never inspected the same personally. (p. 141, BIR rec.)
It is stated in the letter, Exhibit 39 (pp. 184-187, BIR rec.) of Mr. Yu
Khe Thai that 'on July 23, 1946, our General Manager, Mr. David In his affidavit, Exhibit 15 (p. 144, BIR rec.) Dee Hong Lue admitted
Sycip, accompanied by one of our directors, Mr. Sy En, arrived in that of the amount of P1,250,000.00 which he paid in two
Leyte to take over the properties,' We ask: Why was there such a installments sometime in July, 1946, to the FLC, P1,181,250.00
hurry on the part of the promoters of Central Syndicate in taking over (should be P1,181,000.00) of the amount came from the following:
the surplus properties when the formal agreement, Exhibit 13 (p. 66, Yu Khe Thai who advanced to him P250,000.00; Sy Seng Tong
BIR rec.), purporting to be a contract of sale of the 'Mystery Pile' P375,000.00; Alfonso Z. Sycip - P375,000.00; Tan Tiong Bio -
between Dee Hong Lue as vendor, and the Central Syndicate, as P125,000.00; Robert Dee Se Wee P25,000.00; and, Jose S. Lim
vendee, for the amount of P1,250,000.00, was effected twenty-eight P31,000.00 that his understanding with these persons was that

1102
should they eventually join him in Central Syndicate, such advances capitalists when he could very well have disposed of the surplus
would be adjusted to constitute their investments; and, that soon goods to the public in his individual capacity and keep all the profits
after the "Mystery Pile" was purchased from the FLC, all the above- to himself without sharing 9/10th of it to the other six incorporators
named persons with the exception of Robert Dee Se Wee and Jose and stockholders of the newly incorporated Syndicate.
S. Lim, formed the Central Syndicate and a re-allocation of shares
was made corresponding to the amounts advanced by them. It appears that Dee Hong Lue "sold" the pile to the Central Syndicate
for exactly the same price barely forty-six (46) days after acquiring it
Added to these, we have before us other documentary evidence for from FLC and exactly five (5) days after the Syndicate was
the respondent consisting of Exhibits 18, 19, 20, 21, 23, 24, 25, 26, registered with the Securities and Exchange Commission on August
27, 28 and 29 (pp. 85, 88, 92-96, 99-103, 117-128, 119-120, 121- 19, 1946. This is indeed most unusual for a businessman like Dee
128, BIR rec.) all tending to prove the same thing - that the Central Hong Lue who, it is to be presumed, was out to make a killing when
Syndicate and/or the group of big financiers composing it and not he acquired the surplus goods from the FLC for the staggering
Dee Hong Lue was the real purchaser (importer) of the "Mystery amount of P1,750,000.00 in cash.
Pile" from the FLC; that in the contract of sale between Dee Hong
Lue and the FLC the former acted principally as agent (Article 1930, Again, why did Dee Hong Lue waste all his time and effort not to say
New Civil Code) of the petitioners Yu Khe Thai, Sy Seng Tong, his good connections with the FLC by acquiring the goods from that
Alfonso Z. Sycip and Tan Tiong Bio who advanced the purchased agency only to sell it for the same amount to the Central Syndicate?
price of P1,125,000.00 out of the P1,250,000.00 paid to the FLC, This would have been understandable if Dee Hong Lue were the
Dee Hong Lue being the purchaser in his own right only with respect biggest and controlling stockholder of the Syndicate. He could
to the amount of P69,000.00; and, that the deed, Exhibit 13 (p. 77, perhaps reason out to himself, "the profits which I am sacrificing now
BIR rec.) purporting to show that Dee Hong Lue sold the "Mystery in this sale to the Syndicate, I will get it anyway in the form of
Pile" to the Central Syndicate for consideration of P1,250.000.00 is dividends from it after it shall have disposed of all the "Mystery Pile"
but a ruse to evade payment of a greater amount of percentage to the public.' But then, how could this be possible when Dee Hong
tax. 1wph1.t Lue was the smallest subscriber to the capital stock of the
Syndicate? It appears from the Articles of Incorporation that of the
To our mind, the deed of sale, Exhibit 13 (p. 66, BIR rec.) as well as authorized capital stock of the corporation in the amount of
the circumstances surrounding the incorporation of the Central P500,000.00, Dee Hong Lue subscribes to only P20,000.00 or 1/25th
Syndicate, are shrouded with as much mystery as the so-called of the capital stock authorized and of this amount only P5,000.00
"Mystery Pile" subject of the transaction. But, as oil is to water, the was paid by him at the time of incorporation. So here is an
truth and underlying motives behind these transactions have to experienced businessman like Dee Hong Lue who, following the
surface in the end. Petitioners would want us to believe that Dee theory of petitioners' counsel, bought the "'Mystery Pile" for himself
Hong Lue bought in his own right and for himself the surplus goods for P1,250,000.00 in cash, and after a few days sold the same at
in question for P1,250,000.00 from the FLC and then, by virtue of a cost to a corporation wherein he owned only 1/25th of the authorized
valid contract of sale, Exhibit 13 (p. 66, BIR rec.) transferred and capital stock and wherein he was not even an officer, thus doling out
conveyed the same to the Central Syndicate at cost. If this be so, to the other six incorporators and stockholders net profits in the sum
what need was there for Dee Hong Lue to agree in the immediate conservatively estimated by the respondent to be P206,116.45 out of
organization and incorporation of the Central Syndicate with six other a total of P229,073.83 which normally could all go to him. We take

1103
judicial notice of the fact that as a result of our immense losses in incorporator wanted to limit whatever civil liability that might arise in
property throughout the archipelago the during the Japanese favor of third persons, as the present tax liability has now arisen, up
occupation, either through destruction or systematic commandering to the amount of their subscriptions, although the surplus deal they
by the enemy and our forces, surplus properties commanded a very transacted and which we believe was the only purpose in the
good price in the open market after the liberation and that quite a incorporation of the Central Syndicate, was very much over and
number of surplus dealers made immense fortunes out of it. We above their authorized capital. Moreover, by limiting its capital, the
believe the respondent was quite charitable if not more than fair to corporation was also able to save on incidental expenses, such as
the Central Syndicate in computing the profits realized by it in the attorney's fee and the filing fee paid to the Securities and Exchange
resale of the "Mystery Pile" to the public at only 18.8% of the Commission, which were based on the amount of the authorized
acquisition price. capital stock.

Now, from the side of the Central Syndicate. This corporation, as its Another mystery worth unravelling is what happened to the
articles of incorporation, Exhibit A (pp. 60-66, CTA rec.) will show, P1,181,240.00 (should be P1,181,000.00) which Dee Hong Lue in
was incorporated on August 15, 1946 with an authorized capital his affidavit, Exhibit 15 (p. 144, BIR rec.) claims to have received
stock of P500,000.00 of which P200,000.00 worth was subscribed by from Messrs. Uy Khe Thai, Sy Seng Tong, Alfonso Z. Sycip, Tan
seven (7) persons and P50,000.00 paid-up in cash at the time of Tiong Bio (all incorporators of the Syndicate) and two others as
incorporation. Five (5) days after its incorporation, as the Deed of 'advances' with which to pay the FLC. There is no evidence on
Sale, Exhibit 13 (p. 66, BIR rec.) purports to show, the said record to show that Dee Hong Lue ever returned this amount to
corporation bought from Dee Hong Lue the "Mystery Pile" for those six (6) persons after he supposedly received P1,250,000.00
P1,250,000.00 in cash. This is indeed quite phenomenal and from the newly incorporated Syndicate by virtue of the Deed of Sale,
fantastic not to say the utmost degree of finance considering that the Exhibit 13. This is the explanation that Dee Hong Lue gave in this
corporation had a subscribed capital stock of only P200,000.00 of regard as appearing in his affidavit, Exhibit 15: "That soon after the
which only P50,000.00 was paid-up at the time of incorporation and above-mentioned property was purchased, the above parties, with
with not the least proof showing that it never borrowed money in its the exception of Robert Dee Se Wee and Jose S. Lim decided to join
own name from outside source to raise the enormous amount the proposed Central Syndicate and a re-allocation of shares was
allegedly paid to Dee Hong Lue nor evidence to show that it had by made for the reason that some of the above parties in turn had to get
then in so short a time is five (5) days accumulated a substantial advances from third parties." If this were true, why was it that
reserve to meet Dee Hong Lue's selling price. Messrs. Yu Khe Thai, Sy Seng Tong, Alfonso Z. Sycip and Tan Tiong
Bio who advanced P250,000.00; P375,000.00 and P125,000.00 to
Furthermore, at first blush it would seem quite difficult to understand Dee Hong Lue were made to appear in the Articles of incorporation
why the seven (7) incorporators and stockholders of the Central of the Central Syndicate as having subscribed to shares worth only
Syndicate formed a corporation with a subscribed capital stock of P40,000.00; P30,000.00; P30,000.00 and P20,000.00 and of having
only P200,000.00, and with cash on hand of only P50,000.00 paid only P10,000.00, P7,500.00, P7,500.00, and P5,000.00 on their
knowing fully well that there was a transaction awaiting the newly subscriptions, respectively? Would it not be more in keeping with
registered corporation involving an outlay of P1,250,000.00 in cash. corporate practice, following the explanation of Dee Hong Lue, to just
We believe this was done after mature deliberation and for some credit those four (4) persons in the corporation with shares worth the
ulterior motive. As we see it, the only logical answer is that the amount advanced by them to Dee Hong Lue?

1104
On the basis of the above figures, the re-allocation of shares in favor much more than their investments including those who advanced
of the four (4) incorporators who advanced enormous sums for the P1,181,000.00 to the FLC for the corporation.
Syndicate seems at first glance to be totally disproportionate and
unfair to them. However, in the final analysis it is not so as we will Petitioners would dispute the finding that Dee Hong Lue merely acted as a
now show. Immediately after the incorporation of the Syndicate, as trustee of the Central Syndicate when he purchased the surplus goods in
the evidence shows, Dee Hong Lue was made to execute a deed of question from the Foreign Liquidation Commission on July 5, 1946
transfer under the guise of a contract of sale, conveying full and considering that on that date the syndicate has not yet been incorporated on
complete ownership of the "Mystery Pile" to the newly organized the theory that no legal relation may exist between parties one of whom has
corporation. So we have, on the face of the Articles of Incorporation yet no legal existence. Technically this may be true, but the fact remains that
and Exhibit 13, a corporation with assets worth only P50,000.00 cash it cannot be denied that Dee Hong Lue purchased the goods on behalf of
owning properties worth over a million pesos. Obviously, the those who advanced the money for the purchase thereof who later became
incorporators of the Syndicate, particularly those four who advanced the incorporators and only stockholders of the syndicate with the
enormous sums to Dee Hong Lue, are not ordinary businessmen understanding that the amounts they had respectively advanced would be
who could easily be taken for a ride. With the precipitated execution their investment and would represent their interest in the corporation. And
of the "Deed of Sale" by Dee Hong Lue in favor of the Syndicate, this is further evidenced by the fact that this purchase made by Dee Hong
transferring and conveying ownership over the entire pile to the Lue was later approved and adopted as the act of the Central Syndicate itself
latter, the recoupment of their advances from the newly acquired as can be gleaned from the certificate executed by David Sycip, general
assets of the corporation was sufficiently secured, and at the same manager of said syndicate, on September 16, 1946, wherein he emphasized
time, by making the document appear to be a deed of sale instead of that the persons named therein (from whom Dee Hong Lue obtained the
a deed of transfer as it should be under Article 1891 of the New Civil money) merely acted on behalf of the syndicate and in fact were the ones
Code, they have reduced (at least attempted to) their sales tax who went to Leyte to take over the aforesaid surplus goods. In any event,
liability with the argument that Dee Hong Lue was the original even if Dee Hong Lue may be deemed as the purchaser of the surplus goods
"purchaser" or "importer" of the goods and therefore the taxable sale in his own right, nevertheless, the corporation still may be regarded as the
was that one made by him to the Syndicate and not the sales made importer of the same goods for the reason that Dee Hong Lue transferred to
by the latter to the public. After going over the Articles of it all his rights and interests in the contract with the Foreign Liquidation
Incorporation of the Central Syndicate and the other circumstances Commission, and it was said corporation that took delivery thereof from the
of this case, we draw the conclusion that it was organized just for this place where they were stored in Leyte as may be seen from the letter of Dee
particular transaction that its life span was expressly limited to two Hong Lue to the Foreign Liquidation Commission dated September 2, 1946
(2) years from and after the date of incorporation just to give it time and the letter of the Central Syndicate to the said Commission bearing the
to dispose of the "Mystery Pile" to the public and then liquidate all its same date. Under these facts, it is clear that the Central Syndicate is the
assets among the seven incorporators-stockholders as in fact it was importer of the surplus goods as correctly observed by Judge Umali in his
done on August 15, 1948; that from the very start, the seven (7) concurring opinion, from which we quote: .
incorporators had intended it to be a closed corporation without the
least intention of ever selling to other persons the remaining It is now well settled that a person who bought surplus goods from
authorized capital stock of P300,000.00 still unsubscribed; and, that the Foreign Liquidation Commission and who removed the goods
upon its liquidation, the seven (7) incorporators composing it got bought from the U.S. military bases in the Philippines is considered
an importer of such goods and is subject to the sales tax or

1105
compensating tax, as the case may be. (Go Cheng Tee v. Meer, 47 considered as a return filed by the syndicate and so cannot serve as basis
O.G. 269; Saura Import and Export v. Meer, G.R. No. L-2927, Jan. for the computation of the prescriptive period of five years prescribed by law.
26, 1951; P.M.P. Navigation v. Meer, G.R. No. L-4621, March 24,
1953; Soriano y Cia v. Coll. of Int. Rev., 51 O.G. 4548.) In this case it Nor can the fact that the Collector did not include in the assessment a
appearing that the Central Syndicate was the owner of the 'Mystery surcharge of 50% serve as an argument that a return had already been filed,
Pile' before its removal from Base K and that it was the one which for such failure can only mean that an oversight had been committed in the
actually took delivery thereof and removed the same from the U.S. non-inclusion of said surcharge. The syndicate having failed to file its
military base, it is the importer within the meaning of Section 186 of quarterly returns as required by Section 183 of the Tax Code, the period that
the Revenue Code, as it stood before the enactment of Republic Act has to be reckoned with is that embodied in Section 332 of the same Code
No. 594, and its sales of the surplus goods are the original sales which provides that in case of failure to file the return the tax may be
taxable under said section and not the sale to it by Dee Hong Lue. assessed within 10 years after discovery of the falsity, fraud or omission of
the payment of the proper tax. Since it appears that the Collector discovered
2. Since the Central Syndicate, as we have already pointed out, was the the failure of the syndicate to file the return only on September 12, 1951 he
importer of the surplus goods in question, it was its duty under Section 183 of has therefore up to September 18, 1961 within which to assess or collect the
the Internal Revenue Code to file a return of its gross sales within 20 days deficiency tax in question. Consequently the assessment made on January
after the end of each quarter in order that the office of the internal revenue 4, 1952 was made within the prescribed period.
may assess the sales tax that may be due thereon, but, as the record shows,
the Central Syndicate failed to file any return of its quarterly sales on the 3. Petitioners argue (1) that the Court of Tax Appeals acted in excess of its
pretext that it was Dee Hong Lue who imported the surplus goods and it jurisdiction in holding them liable as officers or directors of the defunct
merely purchased them from said importer. This is in fact what the syndicate Central Syndicate for the tax liability of the latter; (2) that petitioners cannot
intended to impress upon the Collector when it wrote to him its letter of be held liable for said tax liability there being no statutory provision in this
October 19, 1946 informing him that it purchased from Dee Hong Lue the jurisdiction authorizing the government to proceed against the stockholders
entire stock of the surplus goods which the latter had bought from the of a defunct corporation as transferees of the corporate assets upon
Foreign Liquidation Commission and was therefore depositing in his name liquidation; (3) that assuming that the stockholders can be held so liable, they
the sum of P43,750.00 to answer for his sales tax liability, but this letter are only liable to the extent of the benefits derived by them from the
certainly cannot be considered as a return that may set in operation the corporation and there is no evidence showing that petitioners had been the
application of the prescriptive period provided for in Section 331 of the Tax beneficiaries of the defunct syndicate; (4) that considering that the Collector
Code, for, evidently, said letter if at all could only be considered as such in instituted the present action on September 23, 1954 when he filed his answer
behalf of Dee Hong Lue and not in behalf of the Central Syndicate because to the appeal of petitioners, said action was already barred by prescription
such is the only nature and import of the letter. Besides, how can such letter pursuant to Sections 77 and 78 of the Corporation Law which allows
be considered as a return of the sales of the Central Syndicate when it was corporations to continue as a body corporate only for three years from its
only on February 21, 1947 when it removed the surplus goods in question dissolution; and (5) that assuming that petitioners are liable to pay the tax,
from their base at Leyte? How can such return inure to the benefit of the their liability is not solidary, but only limited to the benefits derived by them
syndicate when the same surplus goods which were removed on said date from the corporation.
could not have been sold by the corporation earlier than the aforesaid date?
It is obvious that the letter of October 19, 1946 cannot possibly be It should be stated at the outset that it was petitioners themselves who
caused their substitution as parties in the present case, being the

1106
successors-in-interest of the defunct syndicate, when they appealed this In the second place, there is good authority to the effect that the creditor of a
case to the Supreme Court for which reason the latter Court declared that dissolved corporation may follow its assets once they passed into the hands
"the respondent Court of Tax Appeals should have allowed the substitution of of the stockholders. Thus, recognized are the following rules in American
its former officers and directors is parties-appellants, since they are proper jurisprudence: The dissolution of a corporation does not extinguish the debts
parties in interest insofar as they may be (and in fact are) held personally due or owing to it (Bacon v. Robertson, 18 How. 480, 15 L. Ed., 406; Curron
liable for the unpaid deficiency assessments made by the Collector of v. State, 16 How. 304, 14 L. Ed., 705). A creditor of a dissolve corporation
Internal Revenue against the defunct Syndicate." In fact, because of this may follow its assets, as in the nature of a trust fund, into the hands of its
directive their substitution was effected. They cannot, therefore, be now stockholders (MacWilliams v. Excelsier Coal Co. [1924] 298 Fed. 384). An
heard to complain if they are made responsible for the tax liability of the indebtedness of a corporation to the federal government for income and
defunct syndicate whose representation they assumed and whose assets excess profit taxes is not extinguished by the dissolution of the corporation
were distributed among them. (Quinn v. McLeudon, 152 Ark. 271, 238 S.W., 32). And it has been stated,
with reference to the effect of dissolution upon taxes due from a corporation,
"that the hands of the government cannot, of course, collect taxes from a
defunct corporation, it loses thereby none of its rights to assess taxes which
had been due from the corporation, and to collect them from persons, who by
reason of transactions with the corporation, hold property against which the
tax can be enforced and that the legal death of the corporation no more
prevents such action than would the physical death of an individual prevent
the government from assessing taxes against him and collecting them from
his administrator, who holds the property which the decedent had formerly
possessed" (Wonder Bakeries Co. v. U.S. [1934] Ct. Cl. 6 F. Supp. 288).
Bearing in mind that our corporation law is of American origin, the foregoing
authorities have persuasive effect in considering similar cases in this
jurisdiction. This must have been taken into account when in G.R. No. L-
8800 this Court said that petitioners could be held personally liable for the
taxes in question as successors-in-interest of the defunct corporation.

Considering that the Central Syndicate realized from the sale of the surplus
goods a net profit of P229,073.83, and that the sale of said goods was the
only transaction undertaken by said syndicate, there being no evidence to
the contrary, the conclusion is that said net profit remained intact and was
distributed among the stockholders when the corporation liquidated and
distributed its assets on August 15, 1948, immediately after the sale of the
said surplus goods. Petitioners are therefore the beneficiaries of the defunct
corporation and as such should be held liable to pay the taxes in question.
However, there being no express provision requiring the stockholders of the
corporation to be solidarily liable for its debts which liability must be express

1107
and cannot be presumed, petitioners should be held to be liable for the tax in
question only in proportion to their shares in the distribution of the assets of
the defunct corporation. The decision of the trial court should be modified
accordingly.

WHEREFORE, with the above modification, we hereby affirm the decision


appealed from, with costs against petitioners.

Bengzon, C.J., Padilla, Labrador, Concepcion, Reyes. J.B.L., Paredes and


Dizon, JJ., concur.
Barrera, J., took no part.

G.R. No. L-39050 February 24, 1981

CARLOS GELANO and GUILLERMINA MENDOZA DE


GELANO, petitioners,
vs.
THE HONORABLE COURT OF APPEALS and INSULAR SAWMILL,
INC., respondents.

1108
DE CASTRO, J.: On July 14, 1952, in order to accommodate and help petitioners renew
previous loans obtained by them from the China Banking Corporation, private
Private respondent Insular Sawmill, Inc. is a corporation organized on respondent, through Joseph Tan Yoc Su, executed a joint and several
September 17, 1945 with a corporate life of fifty (50) years, or up to promissory note with Carlos Gelano in favor of said bank in the amount of
September 17, 1995, with the primary purpose of carrying on a general P8,000.00 payable in sixty (60) days. For failure of Carlos Gelano to pay the
lumber and sawmill business. To carry on this business, private respondent promissory note upon maturity, the bank collected from the respondent
leased the paraphernal property of petitioner-wife Guillermina M. Gelano at corporation the amount of P9,106.00 including interests, by debiting it from
the corner of Canonigo and Otis, Paco, Manila for P1,200.00 a month. It was the corporation's current account with the bank. Petitioner Carlos Gelano was
while private respondent was leasing the aforesaid property that its officers able to pay private respondent the amount of P5,000.00 but the balance of
and directors had come to know petitioner-husband Carlos Gelano who P4,106.00 remained unsettled. Guillermina M. Gelano refused to pay on the
received from the corporation cash advances on account of rentals to be paid ground that she had no knowledge about the accommodation made by the
by the corporation on the land. corporation in favor of her husband.

Between November 19, 1947 to December 26, 1950 petitioner Carlos Gelano On May 29, 1959 the corporation, thru Atty. German Lee, filed a complaint for
obtained from private respondent cash advances of P25,950.00. The said collection against herein petitioners before the Court of First Instance of
sum was taken and received by petitioner Carlos Gelano on the agreement Manila. Trial was held and when the case was at the stage of submitting
that private respondent could deduct the same from the monthly rentals of memorandum, Atty. Lee retired from active law practice and Atty. Eduardo F.
the leased premises until said cash advances are fully paid. Out of the Elizalde took over and prepared the memorandum.
aforementioned cash advances in the total sum of P25,950.00, petitioner
Carlos Gelano was able to pay only P5,950.00 thereby leaving an unpaid In the meantime, private respondent amended its Articles of Incorporation to
balance of P20,000.00 which he refused to pay despite repeated demands shorten its term of existence up to December 31, 1960 only. The amended
by private respondent. Petitioner Guillermina M. Gelano refused to pay on Articles of Incorporation was filed with, and approved by the Securities and
the ground that said amount was for the personal account of her husband Exchange Commission, but the trial court was not notified of the amendment
asked for by, and given to him, without her knowledge and consent and did shortening the corporate existence and no substitution of party was ever
not benefit the family. made. On November 20, 1964 and almost four (4) years after the dissolution
of the corporation, the trial court rendered a decision in favor of private
On various occasions from May 4, 1948 to September 11, 1949 petitioners respondent the dispositive portion of which reads as follows:
husband and wife also made credit purchases of lumber materials from
private respondent with a total price of P1,120.46 in connection with the WHEREFORE, judgment is rendered, ordering:
repair and improvement of petitioners' residence. On November 9, 1949
partial payment was made by petitioners in the amount of P91.00 and in view 1. Defendant Carlos Gelano to pay plaintiff the sum of:
of the cash discount in favor of petitioners in the amount of P83.00, the
amount due private respondent on account of credit purchases of lumber (a) P19,650.00 with interest thereon at the
materials is P946.46 which petitioners failed to pay. legal rate from the date of the filing of the
complaint on May 29, 1959, until said sum is
fully paid;

1109
(b) P4,106.00, with interest thereon at the the clarification that the conjugal partnership of the spouses
legal rate from the date of the filing of the is jointly and severally liable for the obligations adjudged
complaint until said sum is fully paid; against defendant Carlos Gelano, the judgment appealed
from is affirmed in all other respects. 2
2. Defendants Carlos Gelano and Guillermina Mendoza to
pay jointly and severally the sum of: After petitioners received a copy of the decision on August 24, 1973, they
came to know that the Insular Sawmill Inc. was dissolved way back on
(a) P946.46, with interest thereon, at the December 31, 1960. Hence, petitioners filed a motion to dismiss the case
agreed rate of 12% per annum from October and/or reconsideration of the decision of the Court of Appeals on grounds
6, 1946, until said sum is fully paid; that the case was prosecuted even after dissolution of private respondent as
a corporation and that a defunct corporation cannot maintain any suit for or
(b) P550.00, with interest thereon at the against it without first complying with the requirements of the winding up of
legal rate from the date of the filing of the the affairs of the corporation and the assignment of its property rights within
complaint until the said sum is fully paid; the required period.

(c) Costs of the suit; and Incidentally, after receipt of petitioners' motion to dismiss and/or
reconsideration or on October 28, 1973, private respondent thru its former
directors filed a Petition for Receivership before the Court of First Instance of
3. Defendant Carlos Gelano to pay the plaintiff the sum of
Manila, docketed as Special Proceedings No. 92303, 3 which petition is still
P2,000.00 attorney's fees.
pending before said court.

The Countered of defendants are dismissed.


On November 5, 1973, private respondent filed comment on the motion to
dismiss and or reconsideration and after the parties have filed reply and
SO ORDERED. 1 rejoinder, the Court of Appeals on July 5, 1974 issued a resolution 4 denying
the aforesaid motion.
Both parties appealed to the Court of Appeals, private respondent also
appealing because it insisted that both Carlos Gelano and Guillermina Hence, the present petition for review, petitioners assigning the following
Gelano should be held liable for the substantial portion of the claim. errors:

On August 23, 1973, the Court of Appeals rendered a decision modifying the I
judgment of the trial court by holding petitioner spouses jointly and severally
liable on private respondent's claim and increasing the award of P4,106.00.
THE "RESPONDENT COURT" ERRED IN DENYING
The dispositive portion of the decision reads as follows:
PETlTIONERS MOTION TO DISMISS THIS CASE DESPITE
THE CLEAR FINDING THAT "RESPONDENT" HAD
WHEREFORE, modified in the sense that the amount of ALREADY CEASED TO EXIST AS A CORPORATION
P4,160.00 under paragraph 1 (b) is raised to P8,160.00 and SINCE DECEMBER 31, 1960 YET.

1110
II THE "RESPONDENT COURT" ERRED IN THE
APPLICATION TO THIS CASE OF ITS RULING IN PASAY
THE "RESPONDENT COURT" ERRED IN NOT HOLDING CREDIT AND FINANCE CORPORATION, VERSUS
THAT ACTIONS PENDING FOR OR AGAINST A DEFUNCT LAZARO, ET AL., 46 O.G. (11) 5528, AND IN
CORPORATION ARE DEEMED ABATED. OVERLOOKING THE DISTINCTION LAID DOWN BY THIS
HONORABLE COURT IN NUMEROUS DECIDED CASES
III THAT ONLY CASES FILED IN THE NAME OF ASSIGNEES,
TRUSTEES OR RECEIVERS (FOR A DEFUNCT
CORPORATION), AI)POINTED WITHIN THREE YEARS
THE "RESPONDENT COURT" ERRED IN HOLDING
FROM ITS DISSOLUTION, MAY BE PROSECUTED
INSTEAD THAT EVEN IF THERE WAS NO COMPLIANCE
BEYOND THE SAID THREE YEAR PERIOD, AND THAT,
WITH SECTIONS 77 AND 78 OF THE CORPORATION
ALL OTHERS ARE DEEMED ABATED.
LAW FOR THE WINDING UP OF THE AFFAIRS OF THE
CORPORATION BY THE CONVEYANCE OF CORPORATE
PROPERTY AND PROPERTY RIGHTS TO AN ASSIGNEE, V
OR TRUSTEE OR THE APPOINTMENT OF A RECEIVER
WITHIN THREE YEARS FROM THE DISSOLUTION OF THE "RESPONDENT COURT" ERRED IN HOLDING THAT
SUCH CORPORATION, ANY LITIGATION FILED BY OR WITH THE FILING OF SPECIAL PROCEEDINGS NO.
AGAINST THE DISSOLVED CORPORATION, INSTITUTED 92303 IN THE COURT OF FIRST INSTANCE OF MANILA
WITHIN THREE YEARS AFTER SUCH DISSOLUTION BUT BY FORMER DIRECTORS OF "PRIVATE RESPONDENT"
WHICH COULD NOT BE TERMINATED WITHIN SAID ON OCTOBER 23,1973, OR, THIRTEEN YEARS AFTER
PERIOD, MAY STILL BE CONTINUED AS IT IS NOT ITS DISSOLUTION, A LEGAL, PERSONALITY WILL BE
DEEMED ABATED. APPOINTED TO REPRESENT THE CORPORATION.

IV VI

THE "RESPONDENT COURT" ERRED IN PRACTICALLY


RULING THAT THE THREE-YEAR PERIOD PROVIDED
FOR BY THE CORPORATION LAW WITHIN WHICH
ASSIGNEES, TRUSTEES FOR RECEIVERS MAY BE
APPOINTED MAY BE EXTENDED.

VII

THE "RESPONDENT COURT" ERRED IN NOT HOLDING


THAT THE FAILURE OF "PRIVATE RESPONDENT" OR ITS
AUTHORIZED COUNSEL TO NOTIFY THE TRIAL COURT

1111
OF ITS DISSOLUTION OR OF ITS "CIVIL DEATH" MAY BE The complaint in this case was filed on May 29, 1959 when private
CONSIDERED AS AN ABANDONMENT OF ITS CAUSE OF respondent Insular Sawmill, Inc. was still existing. While the case was being
ACTION AMOUNTING TO A FAILURE TO PROSECUTE tried, the stockholders amended its Articles of Incorporation by shortening the
AND RESULTING IN THE ABATEMENT OF THE SUIT. term of its existence from December 31, 1995 to December 31, 1960, which
was approved by the Securities and Exchange Commission.
VIII
In American corporate law, upon which our Corporation Law was patterned, it
THE "RESPONDENT COURT" ERRED IN RECOGNIZING is well settled that, unless the statutes otherwise provide, all pending suits
THE PERSONALITY OF COUNSEL APPEARING FOR and actions by and against a corporation are abated by a dissolution of the
PRIVATE RESPONDENT' DESPITE HIS ADMISSION THAT corporation. 5 Section 77 of the Corporation Law provides that the
HE DOES NOT KNOW THE "PRIVATE RESPONDENT" corporation shall "be continued as a body corporate for three (3) years after
NOR HAS HE MET ANY OF ITS DIRECTORS AND the time when it would have been ... dissolved, for the purpose of
OFFICERS. prosecuting and defending suits By or against it ...," so that, thereafter, it shall
no longer enjoy corporate existence for such purpose. For this reason,
IX Section 78 of the same law authorizes the corporation, "at any time during
said three years ... to convey all of its property to trustees for the benefit of
members, Stockholders, creditors and other interested," evidently for the
THE "RESPONDENT COURT" ERRED IN AFFIRMING THE
purpose, among others, of enabling said trustees to prosecute and defend
DECISION OF THE TRIAL COURT HOLDING IN FAVOR OF
suits by or against the corporation begun before the expiration of said
"PRIVATE RESPONDENT".
period. 6 Commenting on said sections, Justice Fisher said:

X
It is to be noted that the time during which the corporation,
through its own officers, may conduct the liquidation of its
THE "RESPONDENT COURT" ERRED IN MODIFYING THE assets and sue and be sued as a corporation is limited to
TRIAL COURT'S DECISION AND HOLDING EVEN THE three years from the time the period of dissolution
CONJUGAL PARTNERSHIP OF PETITIONERS JOINTLY commences; but that there is no time limited within which the
AND SEVERALLY LIABLE FOR THE OBLIGATION trustees must complete a liquidation placed in their hands. It
ADJUDGED AGAINST PETITIONER-HUSBAND, CARLOS is provided only (Corp. Law, Sec. 78) that the conveyance to
GELANO. the trustees must be made within the three-year period. It
may be found impossible to complete the work of liquidation
The main issue raised by petitioner is whether a corporation, whose within the three-year period or to reduce disputed claims to
corporate life had ceased by the expiration of its term of existence, could still judgment. The authorities are to the effect that suits by or
continue prosecuting and defending suits after its dissolution and beyond the against a corporation abate when it ceased to be an entity
period of three years provided for under Act No. 1459, otherwise known as capable of suing or being sued (7 R.C.L. Corps., Par. 750);
the Corporation law, to wind up its affairs, without having undertaken any but trustees to whom the corporate assets have been
step to transfer its assets to a trustee or assignee. conveyed pursuant to the authority of Section 78 may sue
and be sued as such in all matters connected with the

1112
liquidation. By the terms of the statute the effect of the The word "trustee" as sued in the corporation statute must be understood in
conveyance is to make the trustees the legal owners of the its general concept which could include the counsel to whom was entrusted
property conveyed, subject to the beneficial interest therein in the instant case, the prosecution of the suit filed by the corporation. The
of creditors and stockholders. 7 purpose in the transfer of the assets of the corporation to a trustee upon its
dissolution is more for the protection of its creditor and stockholders. Debtors
When Insular Sawmill, Inc. was dissolved on December 31, 1960, under like the petitioners herein may not take advantage of the failure of the
Section 77 of the Corporation Law, it stin has the right until December 31, corporation to transfer its assets to a trustee, assuming it has any to transfer
1963 to prosecute in its name the present case. After the expiration of said which petitioner has failed to show, in the first place. To sustain petitioners'
period, the corporation ceased to exist for all purposes and it can no longer contention would be to allow them to enrich themselves at the expense of
sue or be sued. 8 another, which all enlightened legal systems condemn.

However, a corporation that has a pending action and which cannot be The observation of the Court of Appeals on the issue now before Us that:
terminated within the three-year period after its dissolution is authorized
under Section 78 to convey all its property to trustees to enable it to Under Section 77 of the Corporation Law, when the
prosecute and defend suits by or against the corporation beyond the Three- corporate existence is terminated in any legal manner, the
year period although private respondent (did not appoint any trustee, yet the corporation shall nevertheless continue as a body corporate
counsel who prosecuted and defended the interest of the corporation in the for three (3) years after the time when it would have been
instant case and who in fact appeared in behalf of the corporation may be dissolved, for the purpose of prosecuting and defending suits
considered a trustee of the corporation at least with respect to the matter in by or against it. According to authorities, the corporation
litigation only. Said counsel had been handling the case when the same was "becomes incapable of making contracts or receiving a
pending before the trial court until it was appealed before the Court of grant. It does not, however, cease to be a body corporate for
Appeals and finally to this Court. We therefore hold that there was a all purposes." In the case of Pasay Credit and Finance Corp.
substantial compliance with Section 78 of the Corporation Law and as such, vs. Isidro Lazaro and others, 46 OG (11) 5528, this Court
private respondent Insular Sawmill, Inc. could still continue prosecuting the held that "a corporation may continue a pending 'litigation
present case even beyond the period of three (3) years from the time of its even after the lapse of the 3-year period granted by Section
dissolution. 77 of Act 1459 to corporation subsequent to their dissolution
to continue its corporate existence for the purpose of winding
From the above quoted commentary of Justice Fisher, the trustee may up their affairs and settling all the claims by and against
commence a suit which can proceed to final judgment even beyond the same." We note that the plaintiff Insular Sawmill, Inc. ceased
three-year period. No reason can be conceived why a suit already as a corporation on December 30, 1960 but the case at bar
commenced By the corporation itself during its existence, not by a mere was instituted on May 29, 1959, during the time when the
trustee who, by fiction, merely continues the legal personality of the dissolved corporation was still very much alive. Accordingly, it is our
corporation should not be accorded similar treatment allowed to proceed view that "any litigation filed by or against it instituted within
to final judgment and execution thereof. the period, but which could not be terminated, must
necessarily prolong that period until the final termination of
said litigation as otherwise corporations in liquidation would
lose what should justly belong to them or would be exempt

1113
from the payment of just obligations through a mere [G.R. No. 36930. June 30, 1933.]
technicality, something that courts should prevent"
(Philippine Commercial Laws by Martin, 1962 Ed., Vol. 2, p. In the matter of the Voluntary Dissolution of George, OFarrell & Cie.,
1716). Inc. CHINA BANKING CORPORATION and LEOPOLDO KAHN,
claimants-appellants, v. M. MICHELIN & CIE., claimants-appellee.
merits the approval of this Court.
Feria & La O and Felipe Canillas, for Appellants.
The last two assigned errors refer to the disposition of the main case.
Petitioners contend that the obligations contracted by petitioner Carlos Gutierrez Repide & Monzon and Ramon L. Sunico, for Appellee.
Gelano from November 19, 1947 until August 18, 1950 (before the effectivity
of the New Civil Code) and from December 26, 1950 until July 14, 1952 SYLLABUS
(during the effectivity of the New Civil Code) were his personal obligations,
hence, petitioners should not be held jointly and severally liable. As regards 1. CORPORATIONS; RECEIVERSHIP; PROCEDURE BY COURT IN
the said issues, suffice it to say that with the findings of the Court of Appeals APPROVING CLAIMS. Claims against a corporation in the hands of a
that the obligation contracted by petitioner-husband Carlos Gelano receiver should not be approved and paid without some formal and regular
redounded to the benefit of the family, the inevitable conclusion is that the proceeding whereby their justice and correctness may be inquired into after a
conjugal property is liable for his debt pursuant to paragraph 1, Article 1408, reasonable opportunity has been given to all the parties in interest to present
Civil Code of 1889 9 which provision incidentally can still be found in objections and submit evidence in support of such objections. (Whalen v.
paragraph 1, Article 161 of the New Civil Code. 10 Only the conjugal Pasig Iron Works, 13 Phil., 417.)
partnership is liable, not joint and several as erroneously described by the
Court of Appeals, the conjugal partnership being only a single entity. 2. ID.; VOLUNTARY DISSOLUTION; APPOINTMENT OF RECEIVER.
Section 176 of the Code of Civil Procedure dealing with the appointment of
WHEREFORE, with the modification that only the conjugal partnership is receiver upon decree of a corporation provides that the court "may . . .
liable, the appealed decision is hereby affirmed in all other respects. Without appoint a receiver to take charge" of the estate and effects of the corporation,
pronouncement as to costs. "and to pay the outstanding debts thereof, and to divide the money and other
properties that shall remain over among the stockholders or members," and
consistent with respect to decrees of dissolution rendered upon voluntary
SO ORDERED.
application that the court "may appoint receivers to collect and take charge of
the assets of the corporation." Such language found in both statutes on the
Makasiar, Fernandez, and Guerrero, JJ., concur. subject is permissive rather than mandatory and tends to recognize that in
cases of voluntary dissolution there is no occasion for the appointment of a
Teehankee, J., concur in the result. receiver except under special circumstances and upon proper showing.
There can be no doubt that when enacting the Corporation Law the
Mr. Justice de Castro was designated to sit with the First Division under Legislature intended to let the shareholders have the control of the assets of
Special Order No. 225. the corporation upon dissolution in winding up its affairs. The normal method
of procedure is for the directors and executive officers to have charge of the
winding up operations, though there is the alternative method of assigning

1114
the property of the corporation to trustees for the benefit of its creditors and
shareholders.

3. ID.; ID.; ID. Statutes authorizing voluntary dissolutions are generally This is a joint appeal of the China Banking Corporation and Leopoldo Kahn
held to apply only to a dissolution brought about by the stockholders from the order of the Court of First Instance of Manila dated November 7,
themselves, and while the appointment of a receiver rests within the sound 1931, denying the appellant banks motion for reconsideration of the order of
judicial discretion of the court, such discretion must, however, always be the said court of November 8, 1930, allowing the claim of the appellee, M.
exercised with caution and governed by legal and equitable principles, the Michelin & Cie., as a preferred claim against the corporation in dissolution,
violation of which will amount to its abuse, and in making such appointment George, OFarrell & Cie., Inc., to which reference will be made as the
the court should take into consideration all the facts and weigh the relative "corporation."
advantages and disadvantages of appointing a receiver to wind up the
corporate business. The court should only act on facts which have been The appellant, China Banking Corporation, is a claimant against the
proved by competent legal evidence. (8 Thompson on Corp. [2d ed. ], pages corporation as the holder of a note for P8,5000 signed jointly and severally
693, 701, 727, and 738.) by the corporation and the other appellant, Leopoldo Kahn, who has joined
the appeal to protect his interest.
4. ID.; ID.; WHEN CLAIMS CAN BE REVIEWED BY COURTS. The
decree of dissolution in the case at bar having been entered on August 22, George OFarrell & Cie., Inc., is a domestic corporation organized in 1925
1930, and the motion of the appellant, China Banking Corporation, appearing and registered in the same year in the mercantile register of the Bureau of
to have been filed on September 30, 1931, or about thirteen months later, it Commerce and Industry, one of its purposes being that of acting as the agent
follows that the motion was filed on time to have the appellees claim and representative of foreign firms for the sale and distribution of their
reviewed by the court under the provisions of the Corporation Law, and the products in the Philippines.
trial court, therefore, erred in finding that the order of November 8, 1930,
allowing appellees claim was final and unappealable under the provisions of For a number of years prior to its dissolution the corporation had been acting
section 113 of the Code of Civil Procedure. as the representative of the appellee, M. Michelin & Cie., in the Philippine
Islands for the sale and distribution of the rubber tires for motor cars
produced by the appellee and broadly known as "Michelin tires." These
business relations between the appellee decided to discontinue them, and
upon settlement of accounts between both concerns it was found that the
DECISION corporation failed to account for the sum of P23,268.83, the sale price of a
number of rubber tires sold by the corporation. This amount according to
appellees claim and taking appellees own words "was disposed of by the
corporation for its own use and benefit and without the authority or consent"
of the appellee. A few days later, however, the corporation, Gaston OFarrell,
personally, and one Rosario Sanchez, represented by Gaston OFarrell as
OSTRAND, J.:
her attorney-in-fact, executed a mortgage in favor of the appellee of a house
belonging to Gaston OFarrell and of a number of shares of stock of the
corporation owned by OFarrell and Rosario Sanchez to guarantee payment

1115
of the said amount to the appellee in five monthly installments, the first one to likewise served on said Jesus O. Serrano as attorney for the corporation. No
be made on June 1, 1930, and prior to the filing of the petition for dissolution notice appears to have been given to anybody else neither of the claim nor of
the corporation made a partial payment of P1,300 leaving an unpaid balance the hearing thereof, and on the same date set for the hearing, that is,
of P21,968.83, which is the amount claimed by the appellee and allowed by November 8, 1930, the court rendered a judgment allowing the claim as a
the court below as a preferred claim. (Record of Appeal, pages 16, 17, 18, preferred claim against the corporation and directing the receiver to pay the
73-79.) amount thereof out of any funds in his possession. Nobody except the
claimant and the attorney for the receiver was notified of such order. Under
On July 9, 1930, the board of directors filed the petition for its dissolution and date of November 26, 1930, the appellee filed an "ex parte petition", praying
for the appointment of its president and general manager, Gaston OFarrell, for an order directing the liquidator, Gaston OFarrell, to pay appellees claim
as receiver and liquidator to wind up the affairs of the corporation which, within three days, and acting on said petition the court granted the same and
according to the petition, had a balance of P57,601.24 over and above its directed the liquidator to pay the claim within three days with preference to all
just debts and liabilities, and upon publication of the notices required by law other claims. Again nobody was served with notice of this order, and
and hearing of the petition the trial court decreed the dissolution of the pursuant thereto, the receiver paid the appellee on December 9, 1930, the
corporation on August 22, 1930, and appointed the said Gaston OFarrell as sum of P5,000 on account which was receipted for by appellees attorney.
receiver and liquidator to wind up the affairs of the corporation, pay all its (Record of Appeal, pages 16-21; Main Record, pages 233, 569, 576, 572,
liabilities, collect all debts and obligations and dispose of all the remaining and 579.)
assets and property of the corporation subject to the order of the court and
as the law may permit and justice may require. On September 30, 1931, the appellant, China Banking Corporation, filed a
motion praying that the orders of November 8 and November 26, 1930, be
The appellee, M. Michelin & Cie., is a foreign "sociedad annima" organized set aside as null and void, that appellees claim be allowed as an ordinary
under the laws of France and domiciled in said country, and on November 4, claim and that the sum of P5,000 paid by the receiver to the appellee on
1930, filed its claim against the corporation for the aforesaid balance of account of the latters claim be refunded to the funds of the corporation in
P21,968.83 with a prayer that the claim be allowed as a preferred one liquidation for the benefit of the rest of the creditors. In support of said motion
against the corporation on the ground that the said amount represented the and with the permission of the court the appellant, Leopoldo Kahn, submitted
proceeds from the sale of a number of rubber tires which were on deposit a memorandum, arguing on the nullity of the said orders on the ground of
with and sold by the corporation. The attorney for the corporation, Jesus O. want of notice and on the proposition that under the provisions of the
Serrano, gave his conformity to the petition by signing at the foot thereof Insolvency Law appellees claim could not and should not have been allowed
under the words "estoy conforme en que la presente reclamacin sea as a preferred claim under the allegations contained therein. Upon
considerada como credito preferente." Notice setting the hearing of the claim consideration of the motion, the memorandum, appellees opposition and the
for Saturday, November 8, 1930, was likewise served on said Jesus O. arguments of counsel, the trial court denied the motion under order of
Serrano as attorney for the corporation. No notice appears to have been November 7, 1931, as far as it referred to the order of November 8, 1930, on
given to anybody else neither of the claim nor of the hearing thereof, and on the ground that said order, "whether erroneous or not", had become final and
the same date set for the hearing, that is, November 8, 1930, the court unappealable and ordered that the order of November 26, 1930, be given no
rendered a judgment allowing the claim as a preferred claim against the effect as being manifestly contrary to law. (Record of Appeal, pages 27-32;
corporation and directing the receiver to pay the amount thereof out of any page 1266 of the second "Pieza" of the main record.)
funds in his possession. Nobody except the claimant and the attorney for the
receiver was notified of such order. Under date of November 26, 1930, was The assignment of error made by the appellants contends that the lower

1116
court erred in not finding that the order of November 8, 1930, is null and void in its petition for dissolution and did not desire to continue doing business
ab initio for lack of jurisdiction on the part of the court when the said order because of failing conditions or of any other reason, we are unable to
was issued and in not finding the said order just as contrary to the law as the understand the necessity of its seeking judicial intervention in the winding up
court found the order of November 26, 1930, to be, and arguing on this point, of its affairs coupled with the appointment for a receiver to deal with its
they raise the question as to whether appellees claim can legally be allowed creditors as though they were the creditors of an insolvent corporation.
on its face as a preferred claim.
Section 176 of the Code of Civil Procedure dealing with the appointment of
The appellee, on the other hand, argues at length on the theory that the receiver upon decree of dissolution of a corporation provides that the court
appellants have had sufficient constructive notice of the claim and of its "may . . . appoint a receiver to take charge" of the estate and effects of the
allowance by the court and that under the provisions of section 113 of the corporation, "and to pay the outstanding debts thereof, and to divide the
Code of Civil Procedure appellants motion for reconsideration is untenable. money and other properties that shall remain over among the stockholders or
members", and consistent with said provision section 66 of the Corporation
In so far as the service of notice is concerned, we adhere to the rule laid Law provides with respect to decrees of dissolution rendered upon voluntary
down in Whalen v. Pasig Iron Works (13 Phil., 417), where this court held application that the court "may appoint receivers to collect and take charge of
that." . . claims against a corporation in the hands of a receiver should not be the assets of the corporation." Such language found in both statutes on the
approved and paid without some formal and regular proceeding whereby subject is permissive rather than mandatory and tends to recognize that in
their justice and correctness may be inquired into after a reasonable cases of voluntary dissolution there is no occasion for the appointment of a
opportunity has been given to all the parties in interest to present objections receiver except under special circumstances and upon proper showing.
and submit evidence in support of such objections." The said case is a There can be no doubt that when enacting the Corporation Law the
parallel of the case at bar in that the receiver in that case, together with the Legislature intended to let the shareholders have the control of the assets of
claimant, appeared in open court and without previous notice to any of the the corporation upon dissolution in winding up its affairs. The normal method
other parties in interest, the claim was submitted upon the favorable of procedure is for the directors and executive officers to have charge of the
recommendation of the receiver and allowed by the court, and upon appeal winding up operations, though there is the alternative method of assigning
to this court it was held that the trial court erred in rendering judgment in such the property of the corporation to trustees for the benefit of its creditors and
a summary manner. shareholders.

After a careful consideration of the arguments of the attorney for the appellee Section 77 and 78 of the Corporation Law make the general purpose of the
to show that under the provisions of section 113 of the Code of Civil law manifest. Section 77 provides that every corporation whose charter
Procedure appellants motion for reconsideration cannot be favorably expires by its own limitation or whose corporate existence terminates in "any
considered and upon due consideration having been given also to the other manner", shall nevertheless be continued as a body corporate for three
peculiar circumstances surrounding this case, we have reached the years "after the time when it would have been so dissolved" for winding up
conclusion that the issue should be governed by the provisions of the operations; and section 78 provides that "said corporations at any time during
Corporation Law or Act No. 1459. the three years term may convey its property to trustees for the benefit of
creditors, stockholders and others concerned."
A close examination of the record in this case fails to disclose the reasons
which led the corporation to resort to the court for a decree of voluntary Statutes authorizing voluntary dissolutions are generally held to apply only to
dissolution. If the corporation was under such a financial condition as alleged a dissolution brought about by the stockholders themselves, and while the

1117
appointment of a receiver rests within the sound judicial discretion of the appellees claim reviewed by the court under the provisions of the said
court, such discretion must, however, always be exercised with caution and sections of the Corporation Law, and the trial court, therefore, erred in finding
governed by legal and equitable principles, the violation of which will amount that the order of November 8, 1930, allowing appellees claim was final and
to its abuse, and in making such appointment the court should take into unappealable under the provisions of section 113 of the Code of Civil
consideration all the facts and weigh the relative advantages and Procedure.
disadvantages of appointing a receiver to wind up the corporate business.
The court should only act on facts which have been proved by competent The record in this case shows that Gaston OFarrell, the receiver herein,
legal evidence. (8 Thompson on Corp. [2d ed. ], pages 693, 701, 727, and besides being the principal promoter of the corporation and the holder of the
738.) largest number of shares was elected president and general manager and
that he held the said offices ever since the organization of the corporation
The appointment of a receiver by the court to wind up the affairs of the and his conduct in executing a mortgage on his own house and giving a
corporation upon petition of voluntary dissolution does not empower the court pledge on his shares of stock and on those of Rosario Sanchez represented
to hear and pass on the claims of the creditors of the corporation at first by him as attorney in fact, in favor of the appellee to guarantee the latters
hand. In such cases the receiver does not act as a receiver of an insolvent claim, lends itself to a serious suspicion. The facts appearing of record leave
corporation. Since "liquidation" as applied to the settlement of the affairs of a no room for doubt that his administration of the business of the corporation
corporation consists of adjusting the debts and claims, that is, of collecting all left much to be desired and that he alone ought to be blamed for the
that is due the corporation, the settlement and adjustment of claims against it shortage claimed by the appellee, but to save himself from personal liability
and the payment of its just debts, all claims must be presented for allowance he made the corporation shoulder the burden of the obligation in exchange
to the receiver or trustee or other proper persons during the winding up for a simulated conveyance of his house to the corporation. No sooner had
proceedings which in this jurisdiction would be within the three years the corporation become delinquent in the payment of the obligation under the
provided by sections 77 and 78 of the Corporation Law as the term for the terms of the written agreement than he resorted to a judicial proceeding of
corporate existence of the corporation, and if a claim is disputed or voluntary dissolution in an attempt to settle appellees claim and to free
unliquidated so that the receiver cannot safely allow the same, it should be himself from all harm, but fearing that the alleged preference of appellees
transferred to the proper court for trial and allowance, and the amount so claim might be defeated, in collusion with the appellee they had the claim
allowed then presented to the receiver or trustee for payment. The rulings of allowed summarily as a preferred claim ignoring the rest of the world.
the receiver on the validity of claims submitted are subject to review by the
court appointing such receiver though no appeal is taken to the latters ruling Appellants contention that appellees claim cannot be allowed as a preferred
(8 Thompson on Corp., 718), and during the winding up proceedings after claim is well taken for even admitting for the sake of argument that the
dissolution, no creditor will be permitted by legal process or otherwise to merchandise which sale price is the subject of appellees claim was shipped
acquire priority, or to enforce his claim against the property held for to the corporation under a commission agreement or any other agreement
distribution as against the rights of other creditors. (5 Thompson on Corp. [2d carrying the obligation to return either the goods or its price, the fact is that
ed. ], pages 1389, 1391, 1402, and 1403.) the merchandise in the case at bar was no longer in the corporations
possession nor could the appellee trace the proceeds from its sale, and this
The decree of dissolution in the case at bar having been entered on August is made manifest by the very fact of the written agreement entered into
22, 1930, and the motion of the appellant, China Banking Corporation, between the appellee and the corporation whereby the appellee accepted
appearing to have been filed on September 30, 1931, or about thirteen payment of the obligation by installments duly secured with a mortgage of
months later, it follows that the motion was filed on time to have the property to guarantee its payment. But such is not the case, however, for the

1118
very agreement of May 31, 1930, mentioned in paragraph 5 of appellees
claim, shows that the rubber tires consigned to the corporation were to be
sold by the latter "por orden, cuenta y riesgo de los Sres. M. Michelin & Cie."
and that the customers accounts were opened "por orden, cuenta y riesgo
de M. Michelin & Cie.", and so much is this true that the uncollected accounts
were turned over to and received by the appellee, M. Michelin & Cie. Under
such circumstances the amount of appellees claim appears to be in the
nature of a balance of a current account between the two firms more than
anything else. (Record of Appeal, page 68, together with the 4th and 5th
paragraphs of the agreement.)

The order appealed from is reversed, and the appellees claim is hereby
declared to be an ordinary claim. The appellee is ordered to refund to the
corporation the sum of P5,000 erroneously paid by the receiver, with costs
against the appellee. So ordered.

Avancea, C.J., Street, Malcolm, Villa-Real, Abad Santos and Imperial, JJ.,
concur.

G.R. No. L-18956 April 27, 1972

REPUBLIC OF THE PHILIPPINES, plaintiff-appellee,


vs.
MARSMAN DEVELOPMENT COMPANY and/or F.H. BURGESS, in his
capacity as Liquidator of the Marsman Development
Company, defendants-appellants.

1119
BARREDO, J.:p The-contention of the defendant that the assessment in
question have not yet become final and executory is not
Appeal from the decision of the Court of First Instance of Manila, the borne out by the record. The Bureau of Internal Revenue
Honorable Conrado, M. Vasquez, presiding, sentencing defendants- made its first demand for the payment of P13,136.00 as
appellants to pay the amounts of P44,134.35, P6,603.20 and P456.12, plus forest charges and surcharges in the letter dated October
legal interest from August 26, 1959, on the first item, and, from September 5, 15, 1953 (Exh. M). After further investigation, a second
1958, on the later two, representing sales taxes and forest charges, together assessment in the total amount of P45,541.66 was
with surcharges and penalties. demanded from the defendant corporation representing
sales tax and surcharges, and is contained in the letter dated
As found by His Honor, the factual setting of the decision is as follows: September 13, 1954 (Exh. A). The third assessment for the
payment of P456.12 representing 25% surcharge for
discharging lumber without permit was made on November
Defendant corporation was a timber licensee holding Timber
8, 1954 (Exh. B).
Licensee Agreement No. 37-A, with concessions in the
Municipality of Basud and Mondazo, Camarines Norte.
Sometime before October 15, 1953 an investigation was The first acknowledgment by the defendant corporation of its
conducted on the business operation and activities of the receipt of assessment contained in the letter of September
corporation leading to the discovery that certain taxes were 13, 1954, Exh. A, was the letter of the defendant corporation
due (from) it on logs produced from its concession. On under the signature of its counsel, Atty. Pedro L. Moya dated
October 15, 1953, the Deputy Collector of Internal Revenue December 28, 1954, wherein it is requested that said
demanded the payment of P13,136.00 representing forest defendant be furnished with an itemized statement of the
charges due from May 18, 1950 to September 30, 1953, and said taxes and wherein notice is served of its intention to
a surcharge of 25% (Exh. M). On September 13, 1954, after question the validity and the legality of the assessments and
further investigation another assessment was sent to the to appear before the Conference Staff of the Bureau of
defendant corporation by the Bureau of Internal Revenue Internal Revenue in connection with the said tax (Exhibit B).
demanding from it the total sum of P45,541.66 representing In reply to the letter, Exhibit B, the Bureau of Internal
deficiency sales tax, forest charges, surcharges and Revenue wrote Atty. Moya a letter dated February 11, 1955
penalties (Exh. A). On November 8, 1954 another informing him that before the case may be acted upon by the
assessment was addressed to the defendant corporation for Conference Staff, it was necessary that the defendant
the payment of P456.12 as 25% surcharge for discharging corporation comply within 10 days from date of said letter,
lumber without permit (Exh. P). The three assessments with the provisions of Dept. Order No. 213 dated November
totalling P59,133.78 are the subject matter of the instant 2, 1954 which required, among others, that requests for
case for collection. reinvestigation or reexamination of tax assessments shall be
made in writing under oath of the taxpayer concerned,
specifying the ground or grounds relied upon for the revision
xxx xxx xxx
of the assessment and accompanied by such documents
and other documents relied upon in support of the request;
and that, as a general rule, the revision will be granted only

1120
upon payment of one-half of the total assessments and upon evidence, the defendants admit having received the same,
filing of a bond to guarantee the payment of the balance of as shown by the contents of defendant corporation's letters
the tax (Exhibit C). Acknowledgment of Exhibit C was made dated May 10, 1956, Exhibit H, and August 7, 1956, Exhibit
by Atty. Moya in the latter's letter of February 23, 1955 J. In said Exhibit H defendant corporation again protested
wherein, for the reasons therein stated, he requested the assessment of P45,541.66 and reiterated its request for
exemption from the requirements contained in the letter specification of the items disputing the assessment in
Exhibit C (Exhibit D). In Reply to Exhibit D, the Collector of question. It further requests for a period of 30 days from the
Internal Revenue wrote Atty. Moya on May 3, 1955 informing receipt of the specifications within which to consider its tax
him that his request to exempt his client from the liability, further reserving its right to contest the legality or
requirements contained in the letter dated February 11, validity of the assessment or any particular items thereof
1955, cannot be favorably considered and that in order that within the said period of 30 days. Defendant corporation also
the Conference Staff may be directed to hear the case on protested the sending of final notices and requested that
the merits, the said requirements must be complied with they be countermanded or withheld. Finding no merit in the
within five days from receipt of said letter; otherwise, the protests of the defendant corporation, a warrant of distraint
"assessment will be considered final" (Exhibit E). A follow-up and levy was issued against it by the Bureau of Internal
letter dated June 4, 1955, was addressed to Atty. Moya after Revenue on July 3, 1956 (Exhibit O).
discovering that the requirements mentioned in the letters
dated February 11, 1955 and March 3, 1955 have not been On August 3, 1956, defendant corporation again wrote the
complied with inspite of the considerable length of time that Collector of Internal Revenue acknowledging the receipt of
had already elapsed (Exhibit F). In the last paragraph of the the warrant of distraint and levy served upon it and
said letter, Exhibit F, the defendant corporation was warned reiterating its request for a specification of the different items
that unless the aforementioned requirements are complied of the assessment, subject to the right to contest the legality
with within five (5) days from receipt, the "case will be and validity of the same within 30 days after receipt of said
considered abandoned and appropriate action will be taken specifications (Exh. J). The record does not show what
in accordance with law". Again on November 14, 1955, after action was taken on the request contained in said letter on
discovering that the letters dated February 11, 1955, March August 3, 1956. The next communication appearing in the
3, 1955 and June 4, 1955 have remained unheeded by the record is that of the Commissioner of Internal Revenue
defendant corporation, the latter was given another chance dated July 30, 1959, addressed to the defendant corporation
of complying with the requirements mentioned within five demanding on the letter the payment of the assessment of
days from receipt of said letter otherwise, the Bureau of P45,541.66 which has remained unpaid, and informing the
Internal Revenue "will be constrained to enforce the said corporation that if they do not settle said tax obligation
immediate collection of the deficiency percentage tax and within five days from receipt thereof, the Bureau of Internal
forest charges due" (Exhibit G). Revenue will be constrained to file an action in Court for the
collection thereof without further notice (Exhibit I). Defendant
On April 27, 1956, the Bureau of Internal Revenue issued corporation replied to Exhibit I in a letter dated August 17,
"final tax notices" to the defendant corporation. Although the 1959 stating that it needed more time to go over the records
letters containing the "final tax notices" were not presented in and vouchers, and requesting for an extension of 10 days

1121
(Exhibit E). In another letter of same date, the defendant IV
corporation reiterated its exception to the validity and legality
of the assessment against it in the sum of P45,541.66 and THE LOWER COURT FURTHER ERRED IN NOT
its request for a detailed statement of the transactions DECLARING THAT SUIT AGAINST F.H. BURGESS IN HIS
involved (Exhibit L). [Record on Appeal pp. 188-189, 190- CAPACITY AS LIQUIDATOR OF MARSMAN
195.] DEVELOPMENT COMPANY HAS PRESCRIBED AND IN
ORDERING HIM TO PAY THE SUMS CONTAINED IN ITS
According to the Record on Appeal, and as additionally stated also by the DECISION.
trial court, the original complaint filed on September 5, 1958 prayed for the
payment of only P13,695.96, and it was only in an amended complaint filed The Court does not agree.
on August 26, 1959 and admitted on September 23, 1959 that, for the first
time, the amount of P59,133.78 was judicially demanded to be paid. Anent the first assignment of error, it may be stated that regardless of what
might have been alleged in appellee's pleadings and memoranda, the facts
Upon these facts, appellants now complain that proven by evidence, which are not alleged to have been objected to as
varying supposed judicial admissions, unmistakably show that when Atty.
I Pedro L. Moya acknowledged receipt on December 28, 1954, on behalf of
appellant corporation, of the Bureau of Internal Revenue's assessments of
THE LOWER COURT ERRED IN DECLARING THAT THE September 13, 1954 and November 8, 1954, requesting at the same time for
NOTICES OF THE COMMISSIONER OF INTERNAL a reinvestigation before the Conference Staff, he was informed that his
REVENUE DATED APRIL 27, 1956 WERE THE request for investigation would not be given due course unless his client
"ASSESSMENTS" THAT BECAME FINAL AND priorly complied within ten (10) days from Februaxy 11, 1955, the date of the
EXECUTORY. letter of the Bureau, with the provisions of Department Order No. 213, dated
November 2, 1954, which required inter alia, that requests for reinvestigation
II or reexamination of tax assessments should be made in writing and under
oath of the taxpayer concerned, specifying the ground or grounds relied upon
for the requested revision and accompanied by the documents relied upon, in
THE LOWER COURT ERRED IN DECLARING THAT THE
support of the request, as well as by the payment of one-half of the total
GOVERNMENT'S RIGHT TO ASSESS AND COLLECT THE
assessments, plus a bond to guarantee payment of the balance, but
TAXES FOR THE YEARS 1947 TO SEPTEMBER 23, 1949
appellants failed to comply with said conditions: that in reply to Atty. Moya's
HAS NOT PRESCRIBED.
request for exemption from the Department order, on March 3, 1955 (not
May), the attorney was advised that his request was denied and that if the
III corporation failed to comply therewith within five (5) days from receipt of the
letter, "the assessment (would) be considered final"; that on June 4, 1955,
THE LOWER COURT LIKEWISE ERRED IN DECLARING said Atty. Moya was reminded in writing that the previous demands had not
THAT THE GOVERNMENT'S RIGHT TO COLLECT THE been properly attended to, with the warning that should appellants further fail
SUM OF P45,541.66 HAS NOT PRESCRIBED. to comply with the requirements in the letter of February 11, 1955, within five
(5) days from receipt thereof, the "case (would) be considered abandoned

1122
and appropriate action (would) be taken in accordance with law"; that even Under these circumstances, it is plain that His Honor committed no error in
as late as November 14, 1955, the corporation was again advised to comply holding that the period to question the tax assessments herein involved had
with the earlier communications of February 11, 1955, March 3, 1955 and already expired when the Commissioner of Internal Revenue initiated this
June 4, 1955, within five days, otherwise, the Bureau of Internal Revenue suit against defendants. Defendant corporation aknowledged receipt of the
would "be constrained to enforce immediate collection of the deficiency said assessments way back on December 28, 1954, and, in fact, it requested
percentage tax and forest charges due"; that as nothing was done by it to for a reinvestigation before the Conference Staff, but when the Bureau
comply with this last letter, the Bureau of Internal Revenue issued, on April demanded compliance with the prerequisites aforementioned of such
27, 1956, "final tax notices" to it, and all that the latter did after receipt thereof reinvestigation, the corporation failed to comply. The corporation did ask for
was to reiterate, by its letters of May 19, 1956 and August 7, 1956, its exemption, but when this request was denied, again there was no
request for specification of the items involved in the assessment and for compliance. In view of such non-compliance, in its letter of March 3, 1955,
another period of 30 days within which to consider its tax liabilities, reserving the Bureau unequivocally warned the corporation that should it fail further to
once more its right to contest the legality or validity of the assessment and to comply, within five days from receipt thereof, the "assessments (would) be
protest the issuance of the "final tax notices"; that evidently tired of awaiting considered final". still no compliance came. Subsequent follow-up letters
compliance by the said appellant, the Bureau of Internal Revenue issued on brought no better results.
July 3, 1956 a warrant of distraint and levy against it, which it acknowledged
on August 3, 1956, only to reiterate again its position previously stated of As it appears, therefore, appellant corporation, by its own omission, made it
asking for specification and reserving its right to contest the validity of the impossible for the Bureau of Internal Revenue to act on its motion for
assessment; that, finally, on July 30, 1959, after three years, the reconsideration. Not that it would have otherwise mattered, for it has been
Commissioner of Internal Revenue made extrajudicial demand for payment held that the mere filing of such a motion does not suspend the running of
of the amounts in question within five (5) days, and since no payment came, the period for the collection of the tax, 1which implies that any assessment
and instead, defendants asked for more time to go over the records and, made by the Bureau is supposed to be final and executory, insofar as the
under separate cover, questioned for the nth time, the validity of the taxpayer is concerned, unless revised by the Bureau in accordance with law
assessment, the present action was filed. and regulations, but it is to be emphasized that a taxpayer cannot delay the
collection of taxes by the simple expedient of barely asking for clarification or
reconsideration, very often unnecessary and unwarranted, without doing
anything to comply with the statutory and reglementary requirements for the
reconsideration of the assessment made against him. In any event, since
appellant corporation did nothing from December, 1954 when it
acknowledged receipt of the assessment now impugned to appeal the same,
if such an appeal was possible, to the Court of Tax Appeals, even after it was
warned by the Bureau of Internal Revenue that its failure to comply with the
requirements for reconsideration within five (5) days would result in its being
"considered" final, We find no merit in appellants' posture that the
assessments here in question has not yet become final and executory.
Consequently, overruling of appellants' first assignment of error is clearly in
order.

1123
In their second assignment of error, appellants raise the issue of prescription. of the bigger amount of P45,541.66 was not filed within five (5) years from
They point out that the Collector of Internal Revenue had only five years September 13, 1954, the date of the earliest assessment, has neither factual
within which to assess the percentage and forest charges herein involved. nor legal basis. As aptly explained by his Honor, such argument proceeds
Since it does not appear, however, that appellant corporation had filed any from the erroneous premises that because the amended complaint in which
return in relation to the taxes herein involved, and it was incumbent upon the said amount was first alleged and demanded was formally admitted by
appellants to show that such a return had been submitted, 2 We find the the court only on September 23, 1959 and that the filing of said amended
following holding of His Honor to be fully in accordance with law: complaint on August 26, 1959 is immaterial. While in the procedural sense,
especially in relation to the possible necessity of and time for the filing of
Defendants' contention that the right to assess the responsive and other corresponding pleadings, an amended complaint is
percentage and forest charges for the period from 1947 to deemed filed only as of the date of its admission, nothing in Breslin v. Luzon,
September 23, 1949 had already prescribed is based on the 84 Phil. 625, relied upon by appellant, was intended to modify the self-
provision of Section 231 of the Revenue Code which evident proposition that for practical reasons and to avoid the complications
requires the Collector of Internal Revenue to assess the tax that may rise from undue delays in the admission thereof, such an amended
within the period of five years. The Court agrees with the complaint must be considered as filed, for the purposes of such a substantive
plaintiff that said Section 231 is not applicable in this case matter as prescription, on the date it is actually filed with the court, regardless
inasmuch as defendant corporation did not file returns for the of when it is ultimately formally admitted by the court. After all, the only
taxes in question. The pertinent provision applicable herein purpose of requiring leave of and formal admission by the court of an
is Section 332 (a) which provides that "in case of a false or amended pleading after issues have already been joined as to the original
fraudulent return or of a failure to file a return, the tax may be ones is to prevent the injection of other issues which might either to be
assessed ... at anytime within ten years after the discovery considered as barred already or made the subject of another proceeding, if
of the falsity, fraud or omission." The assessments made on they are not anyway indispensable for the resolution of the original ones and
October 15, 1953, September 13, 1954, and November 3, no unnecessary multiplicity of suits would result; so, when the court
1954 were all within the aforecited 10-year period for the ultimately admits the amendment, the legal effect, for substantive purposes,
assessment of the tax. of such admission retroacts as a rule to the date of its actual filing.

Even if the Court were to consider, as appellants suggest, the fact brought Appellants' last assignment of error was disposed of by the trial court this
out in their brief but not found by the trial court that what are being sought to wise:
be collected are deficiency taxes, thereby implying a return must have been
filed, nothing can he gained by appellants, for in order that the filing of a The defendants further contend that the present action is
return may serve as the starting point of the period for the making of an already barred under section 77 of the Corporation Law, Act
assessment, the return must be as substantive complete as to include the No. 1459, as amended, which allows the corporate existence
needed details on which the full assessment may be made, and appellants of a corporation to continue only for three years after its
have not shown that such was the nature of the return they would infer had dissolution, for the purpose of presenting or defending suits
been filed by the corporation. 3 by or against it, and to settle and close its affairs. They point
out that inasmuch as the Marsman Development Co. was
Appellants' third assignment of error does not require any extended extra-judicially dissolved on April 23, 1954, a fact admitted in
discussion. The argument thereunder that the judicial action for the recovery the amended complaint, the filing of both the original

1124
complaint on September 8, 1958 and the amended purpose of continuing the business for which it was
complaint on August 26, 1956 was beyond the aforesaid established.
three-year period.
the next provision, Section 78, adds for clarification:
The record shows that the filing of the amended complaint
was intended, among others, to include as a party At any time during said three years said corporation is
defendant, in an alternative capacity, Mr. F.H. Burgess, who authorized and empowered to convey all of its property to
is the liquidator of the Marsman Development Co. Although it trustees for the benefit of members, stock-holders, creditors,
is an admitted fact that the defendant corporation was and others interested. From and after any such conveyance
extrajudicially dissolved on April 23, 1954, there is no claim by the corporation of its property in trust for the benefit of its
that the affairs of said corporation had already been finally members, stockholders, creditors, and others in interest, all
liquidated or settled. Evidently, Mr. F.H. Burgess is still interest which the corporation had in the property terminates,
continuing in his aforesaid capacity as liquidator of the the legal interest vests in the trustee, and the beneficial
Marsman Development Co. While section 77 of the interest in the members, stockholders, creditors, or other
Corporation Law provides for a three-year period for the persons in interest.
continuation of the corporate existence of the corporation for
purposes of liquidation, there is nothing in said provision It is to be recalled that the assessments against appellant corporation for
which bars an action for the recovery of the debts of the deficiency taxes due for its operations since 1947 were made by the Bureau
corporation against the liquidator thereof, after the lapse of of Internal Revenue on October 15, 1953, September 13, 1954 and
the said three-year period. November 8, 1954, such that the first was before its dissolution and the last
two not later than six months after such dissolution. Thus, in whatever way
We agree with His Honor. The stress given by appellants to the extinction of the matter may be viewed, the Government became the creditor of the
the corporate and juridical personality as such of appellant corporation by corporation before the completion of its dissolution by the liquidation of its
virtue of its extra-judicial dissolution which admittedly took place on April 23, assets. Appellant F.H. Burgess, whom it chose as liquidator, became in law
1954 is misdirected. While Section 77 of the Corporation Law does provide the trustee of all its assets for the benefit of all persons enumerated in
that: Section 78, including its creditors, among whom is the Government, for the
taxes herein involved. To assume otherwise would render the extra-judicial
Every corporation whose charter expires by its own limitation dissolution illegal and void, since, according to Section 62 of the Corporation
or is annulled by forfeiture or otherwise, or whose corporate Law, such kind of dissolution is permitted only when it "does not affect the
existence for other purposes is terminated in any other rights of any creditor having a claim against the corporation." It is immaterial
manner, shall nevertheless be continued as a body that the present action was filed after the expiration of three years after April
corporate for three years after the time when it would have 23, 1954, for at the very least, and assuming that judicial enforcement of
been so dissolved, for the purpose of prosecuting and taxes may not be initiated after said three years despite the fact that the
defending suits by or against it and of enabling it gradually to actual liquidation has not been terminated and the one in charge thereof is
settle and close its affairs, to dispose of and convey its still holding the assets of the corporation, obviously for the benefit of all the
property and to divide its capital stock, but not for the creditors thereof, the assessment aforementioned, made within the three
years, definitely established the Government as a creditor of the corporation

1125
for whom the liquidator is supposed to hold assets of the corporation. And
since the suit at bar is only for the collection of taxes finally assessed against
the corporation within the three years invoked by appellants, their fourth
assignment of error cannot be sustained. As to the allegation that appellant
Burgess has not in fact received any property or asset of the corporation, that
is a matter that can well be taken care of in the execution of the judgment
which may be rendered herein, albeit it seems some kind of fraud would be
perceptible, if the corporation had been dissolved without leaving any assets
whatsoever with the liquidator.

ACCORDINGLY, the judgment of the trial court is affirmed with costs against
the appellants. [G.R. No. 97642. August 29, 1997]

Reyes, J.B.L., Makalintal, Zaldivar, Castro, Fernando, Teehankee, Makasiar AVON INSURANCE PLC, BRITISH RESERVE INSURANCE. CO. LTD.,
and Antonio, JJ., concur. CORNHILL INSURANCE PLC, IMPERIO REINSURANCE CO. (UK)
LTD., INSTITUTE DE RESEGURROS DO BRAZIL, INSURANCE
CORPORATION OF IRELAND PLC, LEGAL AND GENERAL
Concepcion,C.J., is on leave.
ASSURANCE SOCIETY LTD., PROVINCIAL INSURANCE PLC,
QBL INSURANCE (UK) LTD., ROYAL INSURANCE CO. LTD.,
TRINITY INSURANCE CO. LTD., GENERAL ACCIDENT FIRE AND
LIFE ASSURANCE CORP. LTD., COOPERATIVE INSURANCE
SOCIETY and PEARL ASSURANCE CO. LTD., petitioners,
vs.COURT OF APPEALS, REGIONAL TRIAL COURT OF MANILA,
BRANCH 51, YUPANGCO COTTON MILLS, WORLDWIDE
SURETY & INSURANCE CO., INC., respondents.

DECISION

TORRES, JR., J.:

Just how far can our court assert jurisdiction over the persons of foreign
entities being charged with contractual liabilities by residents of the
Philippines?

Appealing from the Court of Appeals October 11, 1990 Decision [1] in CA-
G.R. No. 22005, petitioners claim that the trial courts jurisdiction does not

1126
extend to them, since they are foreign reinsurance companies that are not It will be remembered that in the plaintiffs complaint, [4] it was contended
doing business in the Philippines. Having entered into reinsurance contracts that on July 6, 1979 and on October 1, 1980, Yupangco Cotton Mills engaged
abroad, petitioners are beyond the jurisdictional ambit of our courts and to secure with Worldwide Security and Insurance Co. Inc., several of its
cannot be rendered summons through extraterritorial service, as under properties for the periods July 6, 1979 to July 6, 1980 as under Policy No.
Section 17, Rule 14 of the Rules of Court, nor through the Insurance 20719 for a coverage of P100,000,000.00 and from October 1, 1980 to
Commissioner, under Section 14. Private respondent Yupangco Cotton Mills October 1, 1981, under Policy No. 25896, also for P100,000,000.00. Both
contend on the other hand that petitioners are within our courts cognitive contracts were covered by reinsurance treaties between Worldwide Surety
powers, having submitted voluntarily to their jurisdiction by filing motions to and Insurance and several foreign reinsurance companies, including the
dismiss[2] the private respondents suit below. petitioners. The reinsurance arrangements had been made through
international broker C.J. Boatright and Co. Ltd., acting as agent of Worldwide
The antecedent facts, as found by the appellate court, are as follows: Surety and Insurance.

Respondent Yupangco Cotton Mills filed a complaint against several foreign As fate would have it, on December 16, 1979 and May 2, 1981, with in
reinsurance companies (among which are petitioners) to collect their alleged the respective effectivity periods of Policies 20719 and 25896, the properties
percentage liability under contract treaties between the foreign insurance therein insured were razed by fire , thereby giving rise to the obligation of the
companies and the international insurance broker C.J. Boatright, acting as insurer to indemnify the Yupangco Cotton Mills. Partial payments were made
agent for respondent Worldwide Surety and Insurance Company. Inasmuch by Worldwide Surety and Insurance and some of the reinsurance companies.
as petitioners are not engaged in business in the Philippines with no offices,
places of business or agents in the Philippines, the reinsurance treaties On May 2, 1983, Worldwide Surety and Insurance, in a deed of
having been rendered abroad, service of summons upon motion of Assignment, acknowledge a remaining balance of P19,444,447.75 still due
respondent Yupangco, was made upon petitioners through the office of the Yupangco Cotton Mills, and assigned to the latter all reinsurance proceeds
Insurance Commissioner. Petitioners, by counsel on special appearance, still collectible from all the foreign reinsurance companies. Thus, in its
seasonably filed motions to dismiss disputing the jurisdiction of respondent interest as assignee and original insured, Yupangco Cotton Mills instituted
Court and the extra-territorial service of summons. Respondent Yupangco this collection suit against the petitioners.
filed its opposition to the motion to dismiss, petitioners filed their reply, and
respondent Yupangco filed its rejoinder. In an order dated April 30, 1990 Service of summons upon the petitioners was made by notification to
respondent Court denied the motions to dismiss and directed petitioners to the Insurance Commissioner, pursuant to Section 14, Rule 14 of the Rules of
file their answer. On May 29, 1990, petitioners filed their notice of appeal. In Court.[5]
an order dated June 4, 1990, respondent court denied due course to the
appeal.[3] In a Petition for Certiorari filed with the Court of Appeals, petitioners
submitted that respondent Court has no jurisdiction over them, being all
To this day, trial on the merits of the collection suit has not proceeded as foreign corporations not doing business in the Philippines with no office,
in the present petition, petitioners continue vigorously to dispute the trial place of business or agents in the Philippines. The remedy of Certiorari was
courts assumption of jurisdiction over them. resorted to by petitioners on the premise that if petitioners had filed an
answer to the complaint as ordered by the respondent court, they would risk
abandoning the issue of jurisdiction. Moreover, extra-territorial service of

1127
summons on petitioners is null and void because the complaint for collection 4. The issue of whether or not petitioners are doing business in the
is not one affecting plaintiffs status and not relating to property within the country is a matter best reffered to a trial on the merits of the
Philippines. case and so should be addressed there.

The Court of Appeals found the petition devoid of merit, stating that: Maintaining its submission that they are beyond the jurisdiction of the
Philippine Courts, petitioners are now before us, stating:
1. Petitioners were properly served with summons and whatever
defect, if any, in the service of summons were cured by their Petitioners, being foreign corporations, as found by the trial court, not doing
voluntary appearance in court, via motion to dismiss. business in the Philippines with no office, place of business or agents in the
Philippines, are not subject to the jurisdiction of the Philippine courts.
2. Even assuming that petitioners have not yet voluntarily appeared
as co-defendants in the case below even after having filed the The complaint for sum of money being a personal action not affecting status
motion to dismiss adverted to, still the situation does not or relating to property, extraterritorial service of summons on petitioners all
deserve dismissal of the complaint as far as they are not doing business in the Philippines is null and void.
concerned, since as held by this Court in Linger Fisher
GMBH vs. IAC, 125 SCRA 253. The appearance of counsel for petitioners being explicitly by special
appearance without waiving objections to the jurisdiction over their persons
A case should not be dismissed simply because an original summons was or the subject matter and the motions do dismiss having excluded non-
wrongfully served. It should be difficult to conceive for example, that when a jurisdictional grounds, there is no voluntary submission to the jurisdiction of
defendant personally appears before a court complaining that he had not the trial court.[6]
been validly summoned, that the case filed against him should be
dismissed. An alias summons can be actually served on said defendant. For its part, private respondent Yupangco counter-submits:

3. Being reinsurers of respondent Worlwide Surety and Insurance 1. Foreign corporations, such as petitioners, not doing business in the
of the risk which the latter assumed when it issued the fire Philippines, can be sued in the Philippine Courts, not withstanding petitioners
insurance policies in dispute in favor of respondent Yupangco, claim to the contrary.
petitioners cannot now validly argue that they do not do
business in this country. At the very least, petitioners must be 2. While the complaint before the Honorable Trial Court is for a sum of
deemed to have engaged in business in the Philippines no money, not affecting status or relating to property, petitioners (then
matter how isolated or singular such business might be, even defendants) can submit themselves voluntarily to the jurisdiction of Philippine
on the assumption that among the local domestic insurance Courts, even if there is no extra-judicial (sic) service of summons upon them.
corporations of this country, it is only in favor of Worldwide
Surety and Insurance that they have ever reinsured any risk
3. The voluntary appearance of the petitioners (then defendants) before the
arising from reinsurance within the territory.
Honorable Trial Court amounted, in effect, to voluntary submission to its
jurisdiction over their persons.[7]

1128
In the decisions of the courts below, there is much left to speculation A single act or transaction made in the Philippines, however, could not
and conjecture as to whether or not the petitioners were determined to be qualify a foreign corporation to be doing business in the Philippines, if such
doing business in the Philippines or not. singular act is not merely incidental or casual, but indicates the foreign
corporations intention to do business in the Philippines. [10]
To qualify the petitioners business of reinsurance within the Philippine
forum, resort must be made to established principles in determining what is There is no sufficient basis in the records which would merit the
meant by doing business in the Philippines. In Communication Materials and institution of this collection suit in the Philippines. More specifically, there is
Design, Inc. et. al vs. Court of Appeals,[8] it was observed that: nothing to substantiate the private respondents submission that the
petitioners had engaged in business activities in this country. This is not an
There is no exact rule of governing principle as to what constitutes doing or instance where the erroneous service of summons upon the defendant can
engaging in or transacting business. Indeed, such case must be judged in be cured by the issuance and service of alias summons, as in the absence of
the light of its peculiar circumstances, upon its peculiarfacts and upon the showing that petitioners had been doing business in the country, they cannot
language of the statute applicable. The true test, however, seems to be be summoned to answer for the charges leveled against them.
whether the foreign corporation is continuing the body or substance of the
business or enterprise for which it was organized. The Court is cognizant of the doctrine is Signetics Corp. vs. Court of
Appeals[11] that for the purpose of acquiring jurisdiction by way of summons
Article 44 of the Omnibus Investments Code of 1987 defines the phrase to on a defendant foreign corporation, there is no need to prove first the fact
include: that defendant is doing business in the Philippines. The plaintiff only has to
allege in the complaint that the defendant has an agent in the Philippines for
'soliciting orders, purchases, service contracts opening offices, whether summons to be validly served thereto, even without prior evidence advancing
called liaison offices of branches; appointing representatives or distributors such factual allegation.
who are domiciled in the Philippines or who in any calendar year stay in the
Philippines for a period or periods totaling one hundred eighty (180) days or As it is, private respondent has made no allegation or demonstration of
more; participating in the management, supervision or control of any the existence of petitioners domestic agent, but avers simply that they are
domestic business firm, entity or corporation in the Philippines, and any other doing business not only abroad but in the Philippines as well. It does not
act or acts that imply a continuity or commercial dealings or arrangements appear at all that the petitioners had performed any act which would give the
and contemplate to that extent the performance of acts or works, or the general public the impression that it had been engaging, or intends to
exercise of some of the functions normally incident to and in progressive engage in its ordinary and usual business undertakings in the country. The
prosecution of, commercial gain or of purpose and object of the business reinsurance treaties between the petitioners and Worldwide Surety and
organization. Insurance were made through an international insurance brokers, and not
through any entity of means remotely connected with the
The term ordinarily implies a continuity of commercial dealings and Philippines. Moreover there is authority to the effect that a reinsurance
arrangements, and contemplates, to that extent, the performance of acts or company is not doing business in a certain state merely because the
works or the exercise of the functions normally incident to and in progressive property of lives which are insured by the original insurer company are
prosecution of the purpose and object of its organization. [9] located in that state.[12] The reason for this is that a contract or reinsurance is
generally a separate and distinct arrangement from the original contract of
insurance, whose contracted risk is insured in the reinsurance agreement.

1129
[13]
Hence, the original insured has generally no interest in the contract of The same danger does not exist among foreign corporations that are
reinsurance.[14] indubitably not doing business in the Philippines. Indeed, if a foreign
corporation does not do business here, there would be no reason for it to be
A foreign corporation, is one which owes its existence to the laws of subject to the States regulation. As we observed, in so far as State is
another state,[15] and generally has no legal existence within the state in concerned, such foreign corporation has no legal existence. Therefore, to
which it is foreign. In Marshall Wells Co. vs. Elser,[16] it was held that subject such corporation to the courts jurisdiction would violate the essence
corporations have no legal status beyond the bounds of sovereignty by which of sovereignty.
they are created. Nevertheless, it is widely accepted that foreign corporations
are, by reason of state comity, allowed to transact business in other states In the alternative, private respondent submits that foreign corporations
and to sue in the courts of such fora. In the Philippines foreign corporations not doing business in the Philippines are not exempt from suits leveled
are allowed such privileges, subject to certain restrictions, arising from the against them in courts, citing the case of Facilities Management
states sovereign right of regulation. Corporation vs. Leonardo Dela Osa, et. al.[20] where we ruled that indeed, if a
foreign corporation, not engaged in business in the Philippines, is not barred
Before a foreign corporation can transact business in the country, it from seeking redress from Courts in the Philippines, a fortiori, that same
must first obtain a license to transact business here [17] and secure the proper corporation cannot claim exemption from being sued in the Philippines
authorizations under existing law. Courts for acts done against a person or persons in the Philippines.

If a foreign corporation engages in business activities without the We are not persuaded by the position taken by the private
necessary requirements, it opens itself to court actions against it, but it shall respondent. In Facilities Management case, the principal issue presented
not be allowed maintain or intervene in an action, suit or proceeding for its was whether the petitioner had been doing business in the Philippines, so
own account in any court or tribunal or agency in the Philippines. [18] that service of summons upon its agent as under Section 14, Rule 14 of the
Rules of Court can be made in order that the Court of First Instance could
The purpose of the law in requiring that foreign corporations doing assume jurisdiction over it. The court ruled that the petitioner was doing
business in the country be licensed to do so, is to subject the foreign business in the Philippines, and that by serving summons upon its resident
corporations doing business in the Philippines to the jurisdiction of the courts, agent, the trial court had effectively acquired jurisdiction. In that case, the
[19]
otherwise, a foreign corporation illegally doing business here because of court made no prescription as the absolute suability of foreign corporations
its refusal or neglect to obtain the required license and authority to do not doing business in the country, but merely discounts the absolute
business may successfully though unfairly plead such neglect or illegal act so exemption of such foreign corporations from liabilities particularly arising from
as to avoid service and thereby impugn the jurisdiction of the local courts. acts done against a person or persons in the Philippines.

As we have found, there is no showing that petitioners had performed


any act in the country that would place it within the sphere of the courts
jurisdiction. A general allegation standing alone, that a party is doing
business in the Philippines does not make it so. A conclusion of fact or law
cannot be derived from the unsubstantiated assertions of parties
notwithstanding the demands of convenience or dispatch in legal actions,
otherwise, the Court would be guilty of sorcery; extracting substance out of

1130
nothingness. In addition, the assertion that a resident of the Philippines will by local courts. In Time, Inc. vs. Reyes,[29] it was held that the action of a
be inconvenienced by an out-of-town suit against a foreign entity, is irrelevant court in refusing to rule of deferring its ruling on a motion to dismiss for lack
and unavailing to sustain the continuance of a local action, for jurisdiction is or excess of jurisdiction is correctable by a writ of prohibition
not dependent upon the convenience or inconvenience of a party.[21] or certiorari sued out in the appellate court even before trial on the merits is
had. The same remedy is available should the motion to dismiss be denied,
It is also argued that having filed a motion to dismiss in the proceedings and the court, over the foreign corporations objections, theratens to impose
before the trial court, petitioners have thus acquiesced to the courts its jurisdiction upon the same.
jurisdiction, and they cannot maintain the contrary at this juncture.
If the defendant, besides setting up in a motion to dismiss his objections
This argument is at the most, flimsy. to the jurisdiction of the court, alleges at the same time any other ground for
dismissing the action, or seeks an affirmative refief in the motion, [30] he is
In civil cases, jurisdiction over the person of the defendant is acquired deemed to have submitted himself to the jurisdiction of the court.
either by his voluntary appearance in court and his submission to its authority
or by service of summons.[22] In this instance, however, the petitioners from the time they filed their
motions to dismiss, their submission have been consistently and unfailingly
Fundamentally, the service of summons is intended to give official notice to object to the trial courts assumption of jurisdiction, anchored on the fact
to the defendant or respondent that an action had been commenced against that they are all foreign corporations not doing business in the Philippines.
it. The defendant or respondent is thus put on guard as to the demands of
the plaintiff as stated in the complaint. [23] The service of summons, upon the As we have consistently held, if the appearance of a party in a suit is
defendant becomes an important element in the operation of a courts precisely to question the jurisdiction of the said tribunal over the person of
jurisdiction upon a party to a suit, as service of summons upon the defendant the defendant, then this appearance is not equivalent to service of summons,
is the means by which the court acquires jurisdiction over his person. nor does is constitute an acquiescence to the courts jurisdiction. [31] Thus it
[24]
Without service of summons, or when summons are improperly made, cannot be argued that the petitioners had abandoned their objections to the
both the trial and the judgment, being in violation of due process, are null and jurisdiction of the court, as their motions to dismiss in the trial court, and all
void,[25] unless the defendant waives the service of summons by voluntarily their subsequent posturings, were all in protest of the private respondent's
appearing and answering the suit.[26] insistence on holding them so answer a charge in a forum where they believe
they are not subject to. Clearly, to continue the proceedings in a case such
When a defendant voluntarily appears, he is deemed to have submitted as those before Us would just be useless and a waste of time. [32]
himself to the jurisdiction of the court. [27] This is not, however, always the
case. Admittedly, and without subjecting himself to the courts jurisdiction, the ACCORDINGLY, the decision appealed from dated October 11, 1990, is
defendant in an action can, by special appearance object to the courts SET ASIDE and the instant petition is hereby GRANTED. The respondent
assumption on the ground of lack of jurisdiction. If he so wishes to assert this Regional Trial Court of Manila, Branch 51 is declared without jurisdiction to
defense, he must do so seasonably by motion for the purpose of objecting to take cognizance of Civil Case No. 86-37932, and all its orders and issuances
the jurisdiction of the court, otherwise, he shall be deemed to have submitted in connection therewith are hereby ANNULLED and SET ASIDE.The
himself to that jurisdiction.[28] In the case of foreign corporations, it has been respondent court is hereby ORDERED to DESIST from maintaining further
held that they may seek relief against the wrongful assumption of jurisdiction proceeding in the case aforestated.

1131
SO ORDERED. REGALADO, J.:

Romero, Puno and Mendoza, JJ., concur. Before us is a petition for review on certiorari of the decision of the Court
of Appeals[1] promulgated on July 22, 1992 and its resolution [2] of May 10,
Regalado, J. (Chairman), on leave. 1993 denying petitioners motion for reconsideration, both of which sustained
the order[3] of the Regional Trial Court, Branch 133, Makati, Metro Manila,
dated November 22, 1988 for the quashal of Search Warrant No. 87-053
earlier issued per its own order [4] on September 5, 1988 for violation of
Section 56 of Presidential Decree No. 49, as amended, otherwise known as
the Decree on the Protection of Intellectual Property.

The material facts found by respondent appellate court are as follows:

Complainants thru counsel lodged a formal complaint with the National


Bureau of Investigation for violation of PD No. 49, as amended, and sought
its assistance in their anti-film piracy drive. Agents of the NBI and private
researchers made discreet surveillance on various video establishments in
Metro Manila including Sunshine Home Video Inc. (Sunshine for brevity),
owned and operated by Danilo A. Pelindario with address at No. 6 Mayfair
Center, Magallanes, Makati, Metro Manila.

On November 14, 1987, NBI Senior Agent Lauro C. Reyes applied for a
search warrant with the court a quo against Sunshine seeking the seizure,
among others, of pirated video tapes of copyrighted films all of which were
enumerated in a list attached to the application; and, television sets, video
[G.R. No. 110318. August 28, 1996] cassettes and/or laser disc recordings equipment and other machines and
paraphernalia used or intended to be used in the unlawful exhibition,
showing, reproduction, sale, lease or disposition of videograms tapes in the
COLUMBIA PICTURES, INC., ORION PICTURES CORPORATION,
premises above described.In the hearing of the application, NBI Senior Agent
PARAMOUNT PICTURES CORPORATION, TWENTIETH
Lauro C. Reyes, upon questions by the court a quo, reiterated in substance
CENTURY FOX FILM CORPORATION, UNITED ARTISTS
his averments in his affidavit. His testimony was corroborated by another
CORPORATION, UNIVERSAL CITY STUDIOS, INC., THE WALT
witness, Mr. Rene C. Baltazar. Atty. Rico V. Domingos deposition was also
DISNEY COMPANY, and WARNER BROTHERS, INC., petitioners,
taken. On the basis of the affidavits and depositions of NBI Senior Agent
vs. COURT OF APPEALS, SUNSHINE HOME VIDEO, INC. and
Lauro C. Reyes, Rene C. Baltazar and Atty. Rico V. Domingo, Search
DANILO A. PELINDARIO, respondents.
Warrant No 87-053 for violation of Section 56 of PD No. 49, as amended,
was issued by the court a quo.
DECISION

1132
The search warrant was served at about 1:45 p.m. on December 14, I
1987 to Sunshine and/or their representatives. In the course of the search of
the premises indicated in the search warrant, the NBI Agents found and Inceptively, we shall settle the procedural considerations on the matter
seized various video tapes of duly copyrighted motion pictures/films owned of and the challenge to petitioners legal standing in our courts, they being
or exclusively distributed by private complainants, and machines, equipment, foreign corporations not licensed to do business in the Philippines.
television sets, paraphernalia, materials, accessories all of which were
included in the receipt for properties accomplished by the raiding team. Copy Private respondents aver that being foreign corporations, petitioners
of the receipt was furnished and/or tendered to Mr. Danilo A. Pelindario, should have such license to be able to maintain an action in Philippine
registered owner-proprietor of Sunshine Home Video. courts. In so challenging petitioners personality to sue, private respondents
point to the fact that petitioners are the copyright owners or owners of
On December 16, 1987, a Return of Search Warrant was filed with the exclusive rights of distribution in the Philippines of copyrighted motion
Court. pictures or films, and also to the appointment of Atty. Rico V. Domingo as
their attorney-in-fact, as being constitutive of doing business in the
A Motion To Lift the Order of Search Warrant was filed but was later Philippines under Section 1(f) (1) and (2), Rule 1 of the Rules of the Board of
denied for lack of merit (p. 280, Records). Investments. As foreign corporations doing business in the Philippines,
Section 133 of Batas Pambansa Blg. 68, or the Corporation Code of the
A Motion for reconsideration of the Order of denial was filed. The Philippines, denies them the right to maintain a suit in Philippine courts in the
court a quo granted the said motion for reconsideration and justified it in this absence of a license to do business. Consequently, they have no right to ask
manner: for the issuance of a search warrant.[7]

It is undisputed that the master tapes of the copyrighted films from which the In refutation, petitioners flatly deny that they are doing business in the
pirated films were allegedly copies (sic), were never presented in the Philippines,[8] and contend that private respondents have not adduced
proceedings for the issuance of the search warrants in question. The orders evidence to prove that petitioners are doing such business here, as would
of the Court granting the search warrants and denying the urgent motion to require them to be licensed by the Securities and Exchange Commission,
lift order of search warrants were, therefore, issued in error. Consequently, other than averments in the quoted portions of petitioners Opposition to
they must be set aside. (p. 13, Appellants Brief)[5] Urgent Motion to Lift Order of Search Warrant dated April 28, 1988 and Atty.
Rico V. Domingos affidavit of December 14, 1987. Moreover, an exclusive
Petitioners thereafter appealed the order of the trial court granting right to distribute a product or the ownership of such exclusive right does not
private respondents motion for reconsideration, thus lifting the search conclusively prove the act of doing business nor establish the presumption of
warrant which it had therefore issued, to the Court of Appeals. As stated at doing business.[9]
the outset, said appeal was dismissed and the motion for reconsideration
thereof was denied. Hence, this petition was brought to this Court particularly The Corporation Code provides:
challenging the validity of respondent courts retroactive application of the
ruling in 20th Century Fox Film Corporation vs. Court of Appeals, et al.,[6] in Sec. 133. Doing business without a license. No foreign corporation
dismissing petitioners appeal and upholding the quashal of the search transacting business in the Philippines without a license, or its successors or
warrant by the trial court. assigns, shall be permitted to maintain or intervene in any action, suit or

1133
proceeding in any court or administrative agency of the Philippines; but such organized or whether it has substantially retired from it and turned it over to
corporation may be sued or proceeded against before Philippine courts or another.[14]
administrative tribunals on any valid cause of action recognized under
Philippine laws. As a general proposition upon which many authorities agree in principle,
subject to such modifications as may be necessary in view of the particular
The obtainment of a license prescribed by Section 125 of the issue or of the terms of the statute involved, it is recognized that a foreign
Corporation Code is not a condition precedent to the maintenance of any corporation is doing, transacting, engaging in, or carrying on business in the
kind of action in Philippine courts by a foreign corporation. However, under State when, and ordinarily only when, it has entered the State by its agents
the aforequoted provision, no foreign corporation shall be permitted to and is there engaged in carrying on and transacting through them some
transact business in the Philippines, as this phrase is understood under the substantial part of its ordinary or customary business, usually continuous in
Corporation Code, unless it shall have the license required by law, and until it the sense that it may be distinguished from merely casual, sporadic, or
complies with the law in transacting business here, it shall not be permitted to occasional transactions and isolated acts.[15]
maintain any suit in local courts. [10] As thus interpreted, any foreign
corporation not doing business in the Philippines may maintain an action in The Corporation Code does not itself define or categorize what acts
our courts upon any cause of action, provided that the subject matter and the constitute doing or transacting business in the Philippines. Jurisprudence
defendant are within the jurisdiction of the court. It is not the absence of the has, however, held that the term implies a continuity of commercial dealings
prescribed license but doing business in the Philippines without such license and arrangements, and contemplates, to that extent, the performance of acts
which debars the foreign corporation from access to our courts. In other or works or the exercise of some of the functions normally incident to or in
words, although a foreign corporation is without license to transact business progressive prosecution of the purpose and subject of its organization. [16]
in the Philippines, it does not follow that it has no capacity to bring an
action.Such license is not necessary if it is not engaged in business in the This traditional case law definition has evolved into a statutory definition,
Philippines.[11] having been adopted with some qualifications in various pieces of legislation
in our jurisdiction.
Statutory provisions in many jurisdictions are determinative of what
constitutes doing business or transacting business within that forum, in which For instance, Republic Act No. 5455[17] provides:
case said provisions are controlling there. In others where no such definition
or qualification is laid down regarding acts or transactions falling within its
SECTION 1. Definitions and scope of this Act. (1) x x x; and the phrase doing
purview, the question rests primarily on facts and intent. It is thus held that all
business shall include soliciting orders, purchases, service contracts,
the combined acts of a foreign corporation in the State must be considered,
opening offices, whether called liaison offices or branches; appointing
and every circumstance is material which indicates a purpose on the part of
representatives or distributors who are domiciled in the Philippines or who in
the corporation to engage in some part of its regular business in the State. [12]
any calendar year stay in the Philippines for a period or periods totalling one
hundred eighty days or more; participating in the management, supervision
No general rule or governing principles can be laid down as to what or control of any domestic business firm, entity or corporation in the
constitutes doing or engaging in or transacting business. Each case must be Philippines; and any other act or acts that imply a continuity of commercial
judged in the light of its own peculiar environmental circumstances. [13] The dealings or arrangements, and contemplate to that extent the performance of
true tests, however, seem to be whether the foreign corporation is continuing acts or works, or the exercise of some of the functions normally incident to,
the body or substance of the business or enterprise for which it was

1134
and in-progressive prosecution of, commercial gain or of the purpose and (d) the phrase doing business shall include soliciting orders, service
object of the business organization. contracts, opening offices, whether called liaison offices or branches;
appointing representatives or distributors domiciled in the Philippines or who
Presidential Decree No. 1789,[18] in Article 65 thereof, defines doing in any calendar year stay in the country for a period or periods totalling one
business to include soliciting orders, purchases, service contracts, opening hundred eight(y) (180) days or more; participating in the management,
offices, whether called liaison offices or branches; appointing representatives supervision or control of any domestic business, firm, entity or corporation in
or distributors who are domiciled in the Philippines or who in any calendar the Philippines; and any other act or acts that imply a continuity of
year stay in the Philippines for a period or periods totalling one hundred commercial dealings or arrangements, and contemplate to that extent the
eighty days or more; participating in the management, supervision or control performance of acts or works, or the exercise of some of the functions
of any domestic business firm, entity or corporation in the Philippines, and normally incident to, and in progressive prosecution of, commercial gain or of
any other act or acts that imply a continuity of commercial dealings or the purpose and object of the business organization: Provided, however,
arrangements and contemplate to that extent the performance of acts or That the phrase doing business shall not be deemed to include mere
works, or the exercise of some of the functions normally incident to, and in investment as a shareholder by a foreign entity in domestic corporations duly
progressive prosecution of, commercial gain or of the purpose and object of registered to do business, and/or the exercise of rights as such investors; nor
the business organization. having a nominee director or officer to represent its interests in such
corporation; nor appointing a representative or distributor domiciled in the
The implementing rules and regulations of said presidential decree Philippines which transacts business in its own name and for its own
conclude the enumeration of acts constituting doing business with a catch-all account.
definition, thus:
Based on Article 133 of the Corporation Code and gauged by such
Sec. 1(g). Doing Business shall be any act or combination of acts statutory standards, petitioners are not barred from maintaining the present
enumerated in Article 65 of the Code. In particular doing business includes: action. There is no showing that, under our statutory or case law, petitioners
are doing, transacting, engaging in or carrying on business in the Philippines
as would require obtention of a license before they can seek redress from
xxx xxx xxx
our courts. No evidence has been offered to show that petitioners have
performed any of the enumerated acts or any other specific act indicative of
(10) Any other act or acts which imply a continuity of commercial dealings or an intention to conduct or transact business in the Philippines.
arrangements, and contemplate to that extent the performance of acts or
works, or the exercise of some of the functions normally incident to, or in the
Accordingly, the certification issued by the Securities and Exchange
progressive prosecution of, commercial gain or of the purpose and object of
Commission[20] stating that its records do not show the registration of
the business organization.
petitioner film companies either as corporations or partnerships or that they
have been licensed to transact business in the Philippines, while undeniably
Finally, Republic Act No. 7042[19] embodies such concept in this wise: true, is of no consequence to petitioners right to bring action in the
Philippines. Verily, no record of such registration by petitioners can be
SEC. 3. Definitions. As used in this Act: expected to be found for, as aforestated, said foreign film corporations do not
transact or do business in the Philippines and, therefore, do not need to be
xxx xxx xxx licensed in order to take recourse to our courts.

1135
Although Section 1(g) of the Implementing Rules and Regulations of the To lay criminal complaints with the appropriate authorities and to provide
Omnibus Investments Code lists, among others evidence in support of both civil and criminal proceedings against any person
or persons involved in the criminal infringement of copyright, or concerning
(1) Soliciting orders, purchases (sales) or service contracts. Concrete and the unauthorized importation, duplication, exhibition or distribution of any
specific solicitations by a foreign firm, or by an agent of such foreign firm, not cinematographic work(s) films or video cassettes of which x x x is the owner
acting independently of the foreign firm amounting to negotiations or fixing of of copyright or the owner of exclusive rights of distribution in the Philippines
the terms and conditions of sales or service contracts, regardless of where pursuant to any agreement(s) between x x x and the respective owners of
the contracts are actually reduced to writing, shall constitute doing business copyright in such cinematographic work(s), to initiate and prosecute on
even if the enterprise has no office or fixed place of business in the behalf of x x x criminal or civil actions in the Philippines against any person or
Philippines. The arrangements agreed upon as to manner, time and terms of persons unlawfully distributing, exhibiting, selling or offering for sale any films
delivery of the goods or the transfer of title thereto is immaterial. A foreign or video cassettes of which x x x is the owner of copyright or the owner of
firm which does business through the middlemen acting in their own names, exclusive rights of distribution in the Philippines pursuant to any
such as indentors, commercial brokers or commission merchants, shall not agreement(s) between x x x and the respective owners of copyright in such
be deemed doing business in the Philippines. But such indentors, works.[21]
commercial brokers or commission merchants shall be the ones deemed to
be doing business in the Philippines. tantamount to doing business in the Philippines. We fail to see how
exercising ones legal and property rights and taking steps for the vigilant
(2) Appointing a representative or distributor who is domiciled in the protection of said rights, particularly the appointment of an attorney-in-fact,
Philippines, unless said representative or distributor has an independent can be deemed by and of themselves to be doing business here.
status, i.e., it transacts business in its name and for its own account, and not
in the name or for the account of a principal. Thus, where a foreign firm is As a general rule, a foreign corporation will not be regarded as doing
represented in the Philippines by a person or local company which does not business in the State simply because it enters into contracts with residents of
act in its name but in the name of the foreign firm, the latter is doing business the State, where such contracts are consummated outside the State. [22] In
in the Philippines. fact, a view is taken that a foreign corporation is not doing business in the
state merely because sales of its product are made there or other business
as acts constitutive of doing business, the fact that petitioners are admittedly furthering its interests is transacted there by an alleged agent, whether a
copyright owners or owners of exclusive distribution rights in the Philippines corporation or a natural person, where such activities are not under the
of motion pictures or films does not convert such ownership into an indicium direction and control of the foreign corporation but are engaged in by the
of doing business which would require them to obtain a license before they alleged agent as an independent business.[23]
can sue upon a cause of action in local courts.
It is generally held that sales made to customers in the State by an
Neither is the appointment of Atty. Rico V. Domingo as attorney-in-fact of independent dealer who has purchased and obtained title from the
petitioners, with express authority pursuant to a special power of corporation to the products sold are not a doing of business by the
attorney, inter alia corporation.[24] Likewise, a foreign corporation which sells its products to
persons styled distributing agents in the State, for distribution by them, is not
doing business in the State so as to render it subject to service of process
therein, where the contract with these purchasers is that they shall buy

1136
exclusively from the foreign corporation such goods as it manufactures and sue upon proof that the plaintiff is not the real party-in-interest, hence
shall sell them at trade prices established by it.[25] grounded on failure to state a cause of action. [32] The term lack of capacity to
sue should not be confused with the term lack of personality to sue. While
It has moreover been held that the act of a foreign corporation in the former refers to a plaintiffs general disability to sue, such as on account
engaging an attorney to represent it in a Federal court sitting in a particular of minority, insanity, incompetence, lack of juridical personality or any other
State is not doing business within the scope of the minimum contact test. general disqualifications of a party, the latter refers to the fact that the plaintiff
[26]
With much more reason should this doctrine apply to the mere retainer of is not the real party- in-interest.Correspondingly, the first can be a ground for
Atty. Domingo for legal protection against contingent acts of intellectual a motion to dismiss based on the ground of lack of legal capacity to sue;
[33]
piracy. whereas the second can be used as a ground for a motion to dismiss
based on the fact that the complaint, on the face thereof, evidently states no
In accordance with the rule that doing business imports only acts in cause of action.[34]
furtherance of the purposes for which a foreign corporation was organized, it
is held that the mere institution and prosecution or defense of a suit, Applying the above discussion to the instant petition, the ground
particularly if the transaction which is the basis of the suit took place out of available for barring recourse to our courts by an unlicensed foreign
the State, do not amount to the doing of business in the State. The institution corporation doing or transacting business in the Philippines should properly
of a suit or the removal thereof is neither the making of a contract nor the be lack of capacity to sue, not lack of personality to sue. Certainly, a
doing of business within a constitutional provision placing foreign corporation whose legal rights have been violated is undeniably such, if not
corporations licensed to do business in the State under the same regulations, the only, real party-in-interest to bring suit thereon although, for failure to
limitations and liabilities with respect to such acts as domestic comply with the licensing requirement, it is not capacitated to maintain any
corporations. Merely engaging in litigation has been considered as not a suit before our courts.
sufficient minimum contact to warrant the exercise of jurisdiction over a
foreign corporation.[27] Lastly, on this point, we reiterate this Courts rejection of the common
procedural tactics of erring local companies which, when sued by unlicensed
As a consideration aside, we have perforce to comment on private foreign corporations not engaged in business in the Philippines, invoke the
respondents basis for arguing that petitioners are barred from maintaining latters supposed lack of capacity to sue. The doctrine of lack of capacity to
suit in the Philippines. For allegedly being foreign corporations doing sue based on failure to first acquire a local license is based on
business in the Philippines without a license, private respondents repeatedly considerations of public policy. It was never intended to favor nor insulate
maintain in all their pleadings that petitioners have thereby no legal from suit unscrupulous establishments or nationals in case of breach of valid
personality to bring an action before Philippine courts.[28] obligations or violations of legal rights of unsuspecting foreign firms or
entities simply because they are not licensed to do business in the country.[35]
Among the grounds for a motion to dismiss under the Rules of Court are
lack of legal capacity to sue [29] and that the complaint states no cause of II
action.[30] Lack of legal capacity to sue means that the plaintiff is not in the
exercise of his civil rights, or does not have the necessary qualification to We now proceed to the main issue of the retroactive application to the
appear in the case, or does not have the character or representation he present controversy of the ruling in 20th Century Fox Film Corporation vs.
claims.[31] On the other hand, a case is dismissible for lack of personality to Court of Appeals, et al., promulgated on August 19, 1988, [36] that for the

1137
determination of probable cause to support the issuance of a search warrant governing law on the matter. The ruling in 20th Century Fox was merely an
in copyright infringement cases involving videograms, the production of the application of the law on probable cause. Hence, they posit that there was no
master tape for comparison with the allegedly pirated copies is necessary. law that was retrospectively applied, since the law had been there all
along. To refrain from applying the 20th Century Fox ruling, which had
Petitioners assert that the issuance of a search warrant is addressed to supervened as a doctrine promulgated at the time of the resolution of private
the discretion of the court subject to the determination of probable cause in respondents motion for reconsideration seeking the quashal of the search
accordance with the procedure prescribed therefor under Sections 3 and 4 of warrant for failure of the trial court to require presentation of the master tapes
Rule 126. As of the time of the application for the search warrant in question, prior to the issuance of the search warrant, would have constituted grave
the controlling criterion for the finding of probable cause was that enunciated abuse of discretion.[38]
in Burgos vs. Chief of Staff[37] stating that:
Respondent court upheld the retroactive application of the 20th Century
Probable cause for a search warrant is defined as such facts and Fox ruling by the trial court in resolving petitioners motion for reconsideration
circumstances which would lead a reasonably discrete and prudent man to in favor of the quashal of the search warrant, on this renovated thesis:
believe that an offense has been committed and that the objects sought in
connection with the offense are in the place sought to be searched. And whether this doctrine should apply retroactively, it must be noted that in
the 20th Century Fox case, the lower court quashed the earlier search
According to petitioners, after complying with what the law then warrant it issued. On certiorari, the Supreme Court affirmed the quashal on
required, the lower court determined that there was probable cause for the the ground among others that the master tapes or copyrighted films were not
issuance of a search warrant, and which determination in fact led to the presented for comparison with the purchased evidence of the video tapes to
issuance and service on December 14, 1987 of Search Warrant No. 87- determine whether the latter is an unauthorized reproduction of the former.
053. It is further argued that any search warrant so issued in accordance with
all applicable legal requirements is valid, for the lower court could not If the lower court in the Century Fox case did not quash the warrant, it is Our
possibly have been expected to apply, as the basis for a finding of probable view that the Supreme Court would have invalidated the warrant just the
cause for the issuance of a search warrant in copyright infringement cases same considering the very strict requirement set by the Supreme Court for
involving videograms, a pronouncement which was not existent at the time of the determination of probable cause in copyright infringement cases as
such determination, on December 14, 1987, that is, the doctrine in the 20th enunciated in this 20th Century Fox case. This is so because, as was stated
Century Fox case that was promulgated only on August 19, 1988, or over by the Supreme Court in the said case, the master tapes and the pirated
eight months later. tapes must be presented for comparison to satisfy the requirement of
probable cause. So it goes back to the very existence of probable cause. x x
Private respondents predictably argue in support of the ruling of the x[39]
Court of Appeals sustaining the quashal of the search warrant by the lower
court on the strength of that 20th Century Fox ruling which, they claim, goes Mindful as we are of the ramifications of the doctrine of stare decisis and
into the very essence of probable cause. At the time of the issuance of the the rudiments of fair play, it is our considered view that the 20th Century
search warrant involved here, although the 20th Century Fox case had not Fox ruling cannot be retroactively applied to the instant case to justify the
yet been decided, Section 2, Article III of the Constitution and Section 3, Rule quashal of Search Warrant No. 87-053. Herein petitioners consistent position
126 of the 1985 Rules on Criminal Procedure embodied the prevailing and that the order of the lower court of September 5, 1988 denying therein

1138
defendants motion to lift the order of search warrant was properly issued, effectuate. The settled rule supported by numerous authorities is a
there having been satisfactory compliance with the then prevailing standards restatement of the legal maxim legis interpretation legis vim obtinet the
under the law for determination of probable cause, is indeed well taken. The interpretation placed upon the written law by a competent court has the force
lower court could not possibly have expected more evidence from petitioners of law. x x x, but when a doctrine of this Court is overruled and a different
in their application for a search warrant other than what the law and view is adopted, the new doctrine should be applied prospectively, and
jurisprudence, then existing and judicially accepted, required with respect to should not apply to parties who had relied on the old doctrine and acted on
the finding of probable cause. the faith thereof. x x x. (Stress supplied).

Article 4 of the Civil Code provides that (l)aws shall have no retroactive This was forcefully reiterated in Spouses Benzonan vs. Court of
effect, unless the contrary is provided. Correlatively, Article 8 of the same Appeals, et al.,[45] where the Court expounded:
Code declares that (j)udicial decisions applying the laws or the Constitution
shall form part of the legal system of the Philippines. x x x. But while our decisions form part of the law of the land, they are also
subject to Article 4 of the Civil Code which provides that laws shall have no
Jurisprudence, in our system of government, cannot be considered as retroactive effect unless the contrary is provided. This is expressed in the
an independent source of law; it cannot create law.[40] While it is true that familiar legal maximum lex prospicit, non respicit, the law looks forward not
judicial decisions which apply or interpret the Constitution or the laws are part backward. The rationale against retroactivity is easy to perceive. The
of the legal system of the Philippines, still they are not laws. Judicial retroactive application of a law usually divests rights that have already
decisions, though not laws, are nonetheless evidence of what the laws mean, become vested or impairs the obligations of contract and hence, is
and it is for this reason that they are part of the legal system of the unconstitutional (Francisco v. Certeza, 3 SCRA 565 [1961]). The same
Philippines.[41] Judicial decisions of the Supreme Court assume the same consideration underlies our rulings giving only prospective effect to decisions
authority as the statute itself.[42] enunciating new doctrines. x x x.

Interpreting the aforequoted correlated provisions of the Civil Code and The reasoning behind Senarillos vs. Hermosisima[46] that judicial
in light of the above disquisition, this Court emphatically declared in Co vs. interpretation of a statute constitutes part of the law as of the date it was
Court of Appeals, et al.[43] that the principle of prospectivity applies not only to originally passed, since the Courts construction merely establishes the
original amendatory statutes and administrative rulings and circulars, but contemporaneous legislative intent that the interpreted law carried into effect,
also, and properly so, to judicial decisions. Our holding in the earlier case is all too familiar. Such judicial doctrine does not amount to the passage of a
of People vs. Jubinal[44] echoes the rationale for this judicial declaration, viz.: new law but consists merely of a construction or interpretation of a pre-
existing one, and that is precisely the situation obtaining in this case.
Decisions of this Court, although in themselves not laws, are nevertheless
evidence of what the laws mean, and this is the reason why under Article 8 of It is consequently clear that a judicial interpretation becomes a part of
the New Civil Code, Judicial decisions applying or interpreting the laws or the the law as of the date that law was originally passed, subject only to the
Constitution shall form part of the legal system. The interpretation upon a law qualification that when a doctrine of this Court is overruled and a different
by this Court constitutes, in a way, a part of the law as of the date that the view is adopted, and more so when there is a reversal thereof, the new
law was originally passed, since this Courts construction merely establishes doctrine should be applied prospectively and should not apply to parties who
the contemporaneous legislative intent that the law thus construed intends to relied on the old doctrine and acted in good faith. [47] To hold otherwise would

1139
be to deprive the law of its quality of fairness and justice then, if there is no xxx xxx xxx
recognition of what had transpired prior to such adjudication. [48]
x x x. If the lower courts reversal will be sustained, what encouragement can
There is merit in petitioners impassioned and well-founded be given to courts and litigants to respect the law and rules if they can expect
argumentation: with reasonable certainty that upon the passage of a new rule, their conduct
can still be open to question? This certainly breeds instability in our system of
The case of 20th Century Fox Film Corporation vs. Court of Appeals, et al., dispensing justice. For Petitioners who took special effort to redress their
164 SCRA 655 (August 19, 1988) (hereinafter 20th Century Fox) was grievances and to protect their property rights by resorting to the remedies
inexistent in December of 1987 when Search Warrant 87-053 was issued by provided by the law, it is most unfair that fealty to the rules and procedures
the lower court. Hence, it boggles the imagination how the lower court could then obtaining would bear but fruits of injustice.[49]
be expected to apply the formulation of 20th Century Fox in finding probable
cause when the formulation was yet non-existent. Withal, even the proposition that the prospectivity of judicial decisions
imports application thereof not only to future cases but also to cases still
xxx xxx xxx ongoing or not yet final when the decision was promulgated, should not be
countenanced in the jural sphere on account of its inevitably unsettling
In short, the lower court was convinced at that time after conducting repercussions. More to the point, it is felt that the reasonableness of the
searching examination questions of the applicant and his witnesses that an added requirement in 20th Century Fox calling for the production of the
offense had been committed and that the objects sought in connection with master tapes of the copyrighted films for determination of probable cause in
the offense (were) in the place sought to be searched (Burgos v. Chief of copyright infringement cases needs revisiting and clarification.
Staff, et al., 133 SCRA 800). It is indisputable, therefore, that at the time of
the application, or on December 14, 1987, the lower court did not commit any It will be recalled that the 20th Century Fox case arose from search
error nor did it fail to comply with any legal requirement for the valid issuance warrant proceedings in anticipation of the filing of a case for the unauthorized
of search warrant. sale or renting out of copyrighted films in videotape format in violation of
Presidential Decree No. 49. It revolved around the meaning of probable
x x x. (W)e believe that the lower court should be considered as having cause within the context of the constitutional provision against illegal
followed the requirements of the law in issuing Search Warrant No. 87- searches and seizures, as applied to copyright infringement cases involving
053. The search warrant is therefore valid and binding. It must be noted that videotapes.
nowhere is it found in the allegations of the Respondents that the lower court
failed to apply the law as then interpreted in 1987. Hence, we find it absurd Therein it was ruled that
that it is (sic) should be seen otherwise, because it is simply impossible to
have required the lower court to apply a formulation which will only be The presentation of master tapes of the copyrighted films from which the
defined six months later. pirated films were allegedly copied, was necessary for the validity of search
warrants against those who have in their possession the pirated films. The
Furthermore, it is unjust and unfair to require compliance with legal and/or petitioners argument to the effect that the presentation of the master tapes at
doctrinal requirements which are inexistent at the time they were supposed the time of application may not be necessary as these would be merely
to have been complied with. evidentiary in nature and not determinative of whether or not a probable

1140
cause exists to justify the issuance of the search warrants is not cassettes were pirated without stating the manner it was pirated and that it
meritorious. The court cannot presume that duplicate or copied tapes were was Atty. Domingo that has knowledge of that fact.
necessarily reproduced from master tapes that it owns.
On the part of Atty. Domingo, he said that the re-taping of the allegedly
The application for search warrants was directed against video tape outlets pirated tapes was from master tapes allegedly belonging to the Twentieth
which allegedly were engaged in the unauthorized sale and renting out of Century Fox, because, according to him it is of his personal knowledge.
copyrighted films belonging to the petitioner pursuant to P.D. 49.
At the hearing of the Motion for Reconsideration, Senior NBI Agent Atty.
The essence of a copyright infringement is the similarity or at least Albino Reyes testified that when the complaint for infringement was brought
substantial similarity of the purported pirated works to the copyrighted to the NBI, the master tapes of the allegedly pirated tapes were shown to
work. Hence, the applicant must present to the court the copyrighted films to him and he made comparisons of the tapes with those purchased by their
compare them with the purchased evidence of the video tapes allegedly man Bacani. Why the master tapes or at least the film reels of the allegedly
pirated to determine whether the latter is an unauthorized reproduction of the pirated tapes were not shown to the Court during the application gives some
former. This linkage of the copyrighted films to the pirated films must be misgivings as to the truth of that bare statement of the NBI agent on the
established to satisfy the requirements of probable cause. Mere allegations witness stand.
as to the existence of the copyrighted films cannot serve as basis for the
issuance of a search warrant. Again as the application and search proceedings is a prelude to the filing of
criminal cases under P.D. 49, the copyright infringement law, and although
For a closer and more perspicuous appreciation of the factual what is required for the issuance thereof is merely the presence of probable
antecedents of 20th Century Fox, the pertinent portions of the decision cause, that probable cause must be satisfactory to the Court, for it is a time-
therein are quoted hereunder, to wit: honored precept that proceedings to put a man to task as an offender under
our laws should be interpreted in strictissimi juris against the government and
In the instant case, the lower court lifted the three questioned search liberally in favor of the alleged offender.
warrants against the private respondents on the ground that it acted on the
application for the issuance of the said search warrants and granted it on the xxx xxx xxx
misrepresentations of applicant NBI and its witnesses that infringement of
copyright or a piracy of a particular film have been committed. Thus the lower This doctrine has never been overturned, and as a matter of fact it had been
court stated in its questioned order dated January 2, 1986: enshrined in the Bill of Rights in our 1973 Constitution.

According to the movant, all three witnesses during the proceedings in the So that lacking in persuasive effect, the allegation that master tapes
application for the three search warrants testified of their own personal were viewed by the NBI and were compared to the purchased and seized
knowledge. Yet, Atty. Albino Reyes of the NBI stated that the counsel or video tapes from the respondents establishments, it should be dismissed as
representative of the Twentieth Century Fox Corporation will testify on the not supported by competent evidence and for that matter the probable cause
video cassettes that were pirated, so that he did not have personal hovers in that grey debatable twilight zone between black and white
knowledge of the alleged piracy. The witness Bacani also said that the video resolvable in favor of respondents herein.

1141
But the glaring fact is that Cocoon, the first video tape mentioned in the In fine, the supposed pronunciamento in said case regarding the
search warrant, was not even duly registered or copyrighted in the necessity for the presentation of the master tapes of the copyrighted films for
Philippines. (Annex C of Opposition, p. 152, record.) So that lacking in the the validity of search warrants should at most be understood to merely serve
requisite presentation to the Court of an alleged master tape for purposes of as a guidepost in determining the existence of probable cause in copyright
comparison with the purchased evidence of the video tapes allegedly pirated infringement cases where there is doubt as to the true nexus between the
and those seized from respondents, there was no way to determine whether master tape and the pirated copies. An objective and careful reading of the
there really was piracy, or copying of the film of the complainant Twentieth decision in said case could lead to no other conclusion than that said
Century Fox. directive was hardly intended to be a sweeping and inflexible requirement in
all or similar copyright infringement cases. Judicial dicta should always be
xxx xxx xxx construed within the factual matrix of their parturition, otherwise a careless
interpretation thereof could unfairly fault the writer with the vice of
The lower court, therefore, lifted the three (3) questioned search warrants in overstatement and the reader with the fallacy of undue generalization.
the absence of probable cause that the private respondents violated P.D.
49. As found by the court, the NBI agents who acted as witnesses did not In the case at bar, NBI Senior Agent Lauro C. Reyes who filed the
have personal knowledge of the subject matter of their testimony which was application for search warrant with the lower court following a formal
the alleged commission of the offense by the private respondents. Only the complaint lodged by petitioners, judging from his affidavit [51] and his
petitioners counsel who was also a witness during the application for the deposition,[52] did testify on matters within his personal knowledge based on
issuance of the search warrants stated that he had personal knowledge that said complaint of petitioners as well as his own investigation and surveillance
the confiscated tapes owned by the private respondents were pirated tapes of the private respondents video rental shop. Likewise, Atty. Rico V.
taken from master tapes belonging to the petitioner. However, the lower court Domingo, in his capacity as attorney-in-fact, stated in his affidavit [53] and
did not give much credence to his testimony in view of the fact that the further expounded in his deposition[54] that he personally knew of the fact that
master tapes of the allegedly pirated tapes were not shown to the court private respondents had never been authorized by his clients to reproduce,
during the application (Italics ours). lease and possess for the purpose of selling any of the copyrighted films.

The italicized passages readily expose the reason why the trial court Both testimonies of Agent Reyes and Atty. Domingo were corroborated
therein required the presentation of the master tapes of the allegedly pirated by Rene C. Baltazar, a private researcher retained by Motion Pictures
films in order to convince itself of the existence of probable cause under the Association of America, Inc. (MPAA, Inc.), who was likewise presented as a
factual milieu peculiar to that case. In the case at bar, respondent appellate witness during the search warrant proceedings. [55] The records clearly reflect
court itself observed: that the testimonies of the abovenamed witnesses were straightforward and
stemmed from matters within their personal knowledge. They displayed none
We feel that the rationale behind the aforequoted doctrine is that the pirated of the ambivalence and uncertainty that the witnesses in the 20th Century
copies as well as the master tapes, unlike the other types of personal Fox case exhibited. This categorical forthrightness in their statements,
properties which may be seized, were available for presentation to the court among others, was what initially and correctly convinced the trial court to
at the time of the application for a search warrant to determine the existence make a finding of the existence of probable cause.
of the linkage of the copyrighted films with the pirated ones. Thus, there is no
reason not to present them (Italics supplied for emphasis). [50] There is no originality in the argument of private respondents against
the validity of the search warrant, obviously borrowed from 20th Century Fox,

1142
that petitioners witnesses NBI Agent Lauro C. Reyes, Atty. Rico V. Domingo warrant shall issue except upon probable cause to be determined personally
and Rene C. Baltazar did not have personal knowledge of the subject matter by the judge after examination under oath or affirmation of the complainant
of their respective testimonies and that said witnesses claim that the video and the witnesses he may produce, and particularly describing the place to
tapes were pirated, without stating the manner by which these were pirated, be searched and the things to be seized; and Sec. 3 thereof provides that
is a conclusion of fact without basis.[56] The difference, it must be pointed out, any evidence obtained in violation of the preceding section shall be
is that the records in the present case reveal that (1) there is no allegation of inadmissible for any purpose in any proceeding.
misrepresentation, much less a finding thereof by the lower court, on the part
of petitioners witnesses; (2) there is no denial on the part of private These constitutional strictures are implemented by the following
respondents that the tapes seized were illegitimate copies of the copyrighted provisions of Rule 126 of the Rules of Court:
ones nor have they shown that they were given any authority by petitioners
to copy, sell, lease, distribute or circulate, or at least, to offer for sale, lease, Sec. 3. Requisites for issuing search warrant. A search warrant shall not
distribution or circulation the said video tapes; and (3) a discreet but issue but upon probable cause in connection with one specific offense to be
extensive surveillance of the suspected area was undertaken by petitioners determined personally by the judge after examination under oath or
witnesses sufficient to enable them to execute trustworthy affidavits and affirmation of the complainant and the witnesses he may produce, and
depositions regarding matters discovered in the course thereof and of which particularly describing the place to be searched and the things to be seized.
they have personal knowledge.
Sec. 4. Examination of complainant; record. The judge must, before issuing
It is evidently incorrect to suggest, as the ruling in 20th Century Fox may the warrant, personally examine in the form of searching questions and
appear to do, that in copyright infringement cases, the presentation of master answers, in writing and under oath the complainant and any witnesses he
tapes of the copyrighted films is always necessary to meet the requirement of may produce on facts personally known to them and attach to the record
probable cause and that, in the absence thereof, there can be no finding of their sworn statements together with any affidavits submitted.
probable cause for the issuance of a search warrant. It is true that such
master tapes are object evidence, with the merit that in this class of evidence
Sec. 5. Issuance and form of search warrant. If the judge is thereupon
the ascertainment of the controverted fact is made through demonstrations
satisfied of the existence of facts upon which the application is based, or that
involving the direct use of the senses of the presiding magistrate. [57] Such
there is probable cause to believe that they exist, he must issue the warrant,
auxiliary procedure, however, does not rule out the use of testimonial or
which must be substantially in the form prescribed by these Rules.
documentary evidence, depositions, admissions or other classes of evidence
tending to prove the factum probandum,[58] especially where the production in
court of object evidence would result in delay, inconvenience or expenses out The constitutional and statutory provisions of various jurisdictions
of proportion to its evidentiary value.[59] requiring a showing of probable cause before a search warrant can be issued
are mandatory and must be complied with, and such a showing has been
held to be an unqualified condition precedent to the issuance of a warrant. A
Of course, as a general rule, constitutional and statutory provisions
search warrant not based on probable cause is a nullity, or is void, and the
relating to search warrants prohibit their issuance except on a showing of
issuance thereof is, in legal contemplation, arbitrary.[61] It behooves us, then,
probable cause, supported by oath or affirmation. These provisions prevent
to review the concept of probable cause, firstly, from representative holdings
the issuance of warrants on loose, vague, or doubtful bases of fact, and
in the American jurisdiction from which we patterned our doctrines on the
emphasize the purpose to protect against all general searches. [60] Indeed,
matter.
Article III of our Constitution mandates in Sec. 2 thereof that no search

1143
Although the term probable cause has been said to have a well-defined As already stated, the definition of probable cause enunciated
meaning in the law, the term is exceedingly difficult to define, in this case, in Burgos, Sr. vs. Chief of Staff, et al., supra, vis-a-vis the provisions of
with any degree of precision; indeed, no definition of it which would justify the Sections 3 and 4 of Rule 126, were the prevailing and controlling legal
issuance of a search warrant can be formulated which would cover every standards, as they continue to be, by which a finding of probable cause is
state of facts which might arise, and no formula or standard, or hard and fast tested. Since the proprietary of the issuance of a search warrant is to be
rule, may be laid down which may be applied to the facts of every situation. determined at the time of the application therefor, which in turn must not be
[62]
As to what acts constitute probable cause seem incapable of definition. too remote in time from the occurrence of the offense alleged to have been
[63]
There is, of necessity, no exact test.[64] committed, the issuing judge, in determining the existence of probable cause,
can and should logically look to the touchstones in the laws therefore
At best, the term probable cause has been understood to mean a enacted and the decisions already promulgated at the time, and not to those
reasonable ground of suspicion, supported by circumstances sufficiently which had not yet even been conceived or formulated.
strong in themselves to warrant a cautious man in the belief that the person
accused is guilty of the offense with which he is charged; [65] or the existence It is worth noting that neither the Constitution nor the Rules of Court
of such facts and circumstances as would excite an honest belief in a attempt to define probable cause, obviously for the purpose of leaving such
reasonable mind acting on all the facts and circumstances within the matter to the courts discretion within the particular facts of each
knowledge of the magistrate that the charge made by the applicant for the case. Although the Constitution prohibits the issuance of a search warrant in
warrant is true.[66] the absence of probable cause, such constitutional inhibition does not
command the legislature to establish a definition or formula for determining
Probable cause does not mean actual and positive cause, nor does it what shall constitute probable cause. [71] Thus, Congress, despite its broad
import absolute certainty. The determination of the existence of probable authority to fashion standards of reasonableness for searches and seizures,
[72]
cause is not concerned with the question of whether the offense charged has does not venture to make such a definition or standard formulation of
been or is being committed in fact, or whether the accused is guilty or probable cause, nor categorize what facts and circumstances make up the
innocent, but only whether the affiant has reasonable grounds for his belief. same, much less limit the determination thereof to and within the
[67]
The requirement is less than certainty or proof, but more than suspicion or circumscription of a particular class of evidence, all in deference to judicial
possibility.[68] discretion and probity.[73]

In Philippine jurisprudence, probable cause has been uniformly defined Accordingly, to restrict the exercise of discretion by a judge by adding a
as such facts and circumstances which would lead a reasonable, discreet particular requirement (the presentation of master tapes, as intimated
and prudent man to believe that an offense has been committed, and that the by 20th Century Fox) not provided nor implied in the law for a finding of
objects sought in connection with the offense are in the place sought to be probable cause is beyond the realm of judicial competence or
searched.[69] It being the duty of the issuing officer to issue, or refuse to issue, statemanship. It serves no purpose but to stultify and constrict the judicious
the warrant as soon as practicable after the application therefor is filed, [70] the exercise of a court's prerogatives and to denigrate the judicial duty of
facts warranting the conclusion of probable cause must be assessed at the determining the existence of probable cause to a mere ministerial or
time of such judicial determination by necessarily using legal standards then mechanical function. There is, to repeat, no law or rule which requires that
set forth in law and jurisprudence, and not those that have yet to be crafted the existence of probable cause is or should be determined solely by a
thereafter. specific kind of evidence. Surely, this could not have been contemplated by
the framers of the Constitution, and we do not believe that the Court intended

1144
the statement in 20th Century Fox regarding master tapes as the dictum for the illegal sale, rental, distribution, circulation or public exhibition of
all seasons and reasons in infringement cases. copyrighted films of MPAA without its written authority or its
members. Knowing that defendant Sunshine Home Video and its proprietor,
Turning now to the case at bar, it can be gleaned from the records that Mr. Danilo Pelindario, were not authorized by MPAA to reproduce, lease, and
the lower court followed the prescribed procedure for the issuances of a possess for the purpose of selling any of its copyrighted motion pictures, he
search warrant: (1) the examination under oath or affirmation of the instructed his researcher, Mr. Rene Baltazar to rent two video cassettes from
complainant and his witnesses, with them particularly describing the place to said defendants on October 21, 1987. Rene C. Baltazar proceeded to
be searched and the things to be seized; (2) an examination personally Sunshine Home Video and rented tapes containing Little Shop of Horror. He
conducted by the judge in the form of searching questions and answers, in was issued rental slip No. 26362 dated October 21, 1987 for P10.00 with a
writing and under oath of the complainant and witnesses on facts personally deposit of P100.00. Again, on December 11, 1987, he returned to Sunshine
known to them; and, (3) the taking of sworn statements, together with the Home Video and rented Robocop with a rental slip No. 25271 also
affidavits submitted, which were duly attached to the records. for P10.00. On the basis of the complaint of MPAA thru counsel, Atty. Lauro
C. Reyes personally went to Sunshine Home Video at No. 6 Mayfair Center,
Thereafter, the court a quo made the following factual findings leading to Magallanes Commercial Center, Makati. His last visit was on December 7,
the issuance of the search warrant now subject to this controversy: 1987. There, he found the video outlet renting, leasing, distributing video
cassette tapes whose titles were copyrighted and without the authority of
MPAA.
In the instant case, the following facts have been established: (1) copyrighted
video tapes bearing titles enumerated in Search Warrant No. 87-053 were
being sold, leased, distributed or circulated, or offered for sale, lease, Given these facts, a probable cause exists. x x x. [74]
distribution, or transferred or caused to be transferred by defendants at their
video outlets, without the written consent of the private complainants or their The lower court subsequently executed a volte-face, despite its prior
assignee; (2) recovered or confiscated from defendants' possession were detailed and substantiated findings, by stating in its order of November 22,
video tapes containing copyrighted motion picture films without the authority 1988 denying petitioners motion for reconsideration and quashing the search
of the complainant; (3) the video tapes originated from spurious or warrant that
unauthorized persons; and (4) said video tapes were exact reproductions of
the films listed in the search warrant whose copyrights or distribution rights x x x. The two (2) cases have a common factual milieu; both involve alleged
were owned by complainants. pirated copyrighted films of private complainants which were found in the
possession or control of the defendants. Hence, the necessity of the
The basis of these facts are the affidavits and depositions of NBI Senior presentation of the master tapes from which the pirated films were allegedly
Agent Lauro C. Reyes, Atty. Rico V. Domingo, and Rene C. Baltazar. Motion copied is necessary in the instant case, to establish the existence of probable
Pictures Association of America, Inc. (MPAA) thru their counsel, Atty. Rico V. cause.[75]
Domingo, filed a complaint with the National Bureau of Investigation against
certain video establishments one of which is defendant, for violation of PD Being based solely on an unjustifiable and improper retroactive
No. 49 as amended by PD No, 1988. Atty. Lauro C. Reyes led a team to application of the master tape requirement generated by 20th Century
conduct discreet surveillance operations on said video establishments. Per Fox upon a factual situation completely different from that in the case at bar,
information earlier gathered by Atty. Domingo, defendants were engaged in and without anything more, this later order clearly defies elemental fair play

1145
and is a gross reversible error. In fact, this observation of the Court in La III
ChemiseLacoste, S.A. vs. Fernandez, et al., supra, may just as easily apply
to the present case:

Potrebbero piacerti anche